Download as pdf or txt
Download as pdf or txt
You are on page 1of 871

Spectrum’s

QUANTITATIVE
APTITUDE

TS
for Bank PO/MT and
Other Competitive Examinations

H
G
U
O
C. Rashmi
Usha Govind
Sandesh Sharma
H
V. Rajeshwari
_T

Editor
Kalpana Rajaram
C
PS

Assistant Editors
R. Vidya
Munib Ahmad
U

Isha Kapoor
@

2019

SPECTRUM BOOKS (P) LTD.


A1 291, First Floor, Janakpuri,
New Delhi 110058
A Note to our Readers

TS
Quantitative aptitude or numerical ability tests are a part of examinations held
for the recruitment of officers to banks as well as certain other sectors. These
tests examine the candidates’ knowledge of basic mathematics and also demand

H
expert time management from them. This book has basically been designed
to help candidates understand the basic concepts of math—or refresh their

G
minds about them—and learn ways to solve problems fast. It is divided into

U
two sections—Learning Curve 1 and Learning Curve 2.
O
LEARNING CURVE 1 consists of several units progressing from the very basic
to more advanced topics of mathematics. Each unit deals with a few topics,
H
presenting the concepts and principles and then going on to illustrate their
use. Woven into these information units are fast-track formulae (or shortcuts)
_T

that are practical and rather essential if the problems are to be solved fast.
Their usage is also amply illustrated through worked examples. Each unit also
C

has a number of practice exercises with questions based on the topics covered.
These multiple choice questions are typical of the kind that are set in the
PS

examinations. Every practice exercise is accompanied by answers and detailed


solutions to the questions. Alongwith conventional solutions, there are shortcuts
suggested for most of the questions. It is important to remember that the
U

multiple choice format allows you to locate the answer without having to
@

solve the sum fully. This book guides the candidates on how to do that—
another time management skill. At times, shortcuts are specific to problems;
the ability to find the correct one where required is a skill that comes with
practice—and that practice is ensured by the large variety and number of
questions in this book and the suggested solutions.
LEARNING CURVE 2 consists of several test papers set in actual examinations
of recent years—bank PO, CDS, NDA-NA, SSC, etc. (Some papers are based
on the questions recalled by candidates.) These papers have been fully solved.
This section is aimed at giving readers a feel of the examination and the
opportunity to assess themselves in terms of both knowledge and time
(iii)
management. Knowing where one falls short is a great help in improving
oneself. And knowing one can answer the questions accurately within the
stipulated time leads to the building up of confidence.
Though mainly designed for candidates of bank probationary officers’
examinations, this book will also prove quite useful for candidates of other
examinations that test numerical aptitude/ability. It could even be useful as

TS
a refresher for high school and college students.
Suggestions for improving the book are welcome.

H
Kalpana Rajaram

G
U
O
H
C _T
PS
U
@

(iv)
CONTENTS
Ratio and Proportion 115
Learning Curve 1 Partnership 122

TS
Profit and Loss 127
UNIT: ONE 3–97 Alligation or Mixture 135
Problems on Age 140
FUNDAMENTALS AND FAST-TRACK

H
FORMULAE
PRACTICE SESSION
Numbers, Numerals and Digits 3

G
Practice Exercise 8 144
Addition and Subtraction of Integers 6
Practice Exercise 9 151
Multiplication of Integers 8
Division of Integers 11
U Practice Exercise
Practice Exercise
10
11
158
165
O
H.C.F. and L.C.M. 14
Practice Exercise 12 175
Fraction 16
Practice Exercise 13 181
H
Decimal Fraction 22
Practice Exercise 14 187
Exponents and Roots 26
Practice Exercise 15 194
_T

Test for a Prime Number 33


Practice Exercise 16 199
Algebraic Expressions and Equations 34
Surds 43
C

UNIT: THREE 204–286


Simplification 44
FUNDAMENTALS AND FAST-TRACK
PS

PRACTICE SESSION FORMULAE


Practice Exercise 1 46 Unitary Method 204
Chain Rule 204
U

Practice Exercise 2 53
Practice Exercise 3 60 Time and Work 208
@

Practice Exercise 4 66 Pipes and Cisterns 214


Practice Exercise 5 73 Time and Distance and Moving 218
Practice Exercise 6 80 Vehicles
Practice Exercise 7 91 Boats and Streams 226
Races and Games 229

UNIT: TWO 98–203


PRACTICE SESSION
FUNDAMENTALS AND FAST-TRACK Practice Exercise 17 233
FORMULAE Practice Exercise 18 242
Percentage 98 Practice Exercise 19 253
Average 108 Practice Exercise 20 261

(v)
Contents

Practice Exercise 21 270 Practice Exercise 30 393


Practice Exercise 22 277 Practice Exercise 31 401
Practice Exercise 23 282 Practice Exercise 32 411
Practice Exercise 33 416
UNIT: FOUR 287–344 Practice Exercise 34 423
FUNDAMENTALS AND FAST-TRACK Practice Exercise 35 429
FORMULAE Practice Exercise 36 434
Practice Exercise 37 438

TS
Interest 287
Simple Interest 287
Compound Interest 295 UNIT: SIX 441–489

H
True Discount 302 FUNDAMENTALS AND FAST-TRACK
Banker’s Discount 306 FORMULAE

G
Stocks, Shares and Debentures 309 Series 441
Permutation and Combination 445
PRACTICE SESSION
Practice Exercise 24 316
U Probability 452
O
Practice Exercise 25 322 PRACTICE SESSION
Practice Exercise 26 330
Practice Exercise 38 460
H
Practice Exercise 27 339
Practice Exercise 39 464
Practice Exercise 40 469
_T

UNIT: FIVE 345–440


Practice Exercise 41 474
FUNDAMENTALS AND FAST-TRACK Practice Exercise 42 481
FORMULAE
C

Mensuration 345 UNIT: SEVEN 490–510


I. Area and Perimeter of Two-
PS

FUNDAMENTALS AND WORKED


Dimensional Figures 345
II. Volume and Surface Area of EXAMPLES
Three-Dimensional Figures 357 Logarithm 490
U

1. Rectangular Solid (Cuboid) 357 Basic Trigonometry 493


2. Cube 358 Heights and Distances 494
@

3. Prism 358
4. Cylinder 359 PRACTICE SESSION
5. Cone 360 Practice Exercise 43 499
6. Pyramid 362 Practice Exercise 44 504
7. Sphere 362
Clocks 368 UNIT: EIGHT 511–620
Calendars 371
FUNDAMENTALS AND FAST-TRACK
FORMULAE
PRACTICE SESSION Data Analysis: Tables, Graphs and 511
Practice Exercise 28 375 Diagrams
Practice Exercise 29 383 Basic Concepts and Tools 511

(vi)
Contents

Ratio 512 State Bank of India Probationary 682


Percentages 512 Officers Examination, April 2013
Answers and Explanatory Notes 687
Rates 513
Interpretation of Tables 515 Reserve Bank of India Grade B 695
Interpretation of Graphs and 522 Officers Examination, August 2013
Diagrams Answers and Explanatory Notes 698
Passage-based Questions 538
Reserve Bank of India Assistants 702

TS
PRACTICE SESSION Examination, July 2013
Answers and Explanatory Notes 704
Practice Exercise 45 543
Practice Exercise 46 557 Management Aptitude Test, 709

H
Practice Exercise 47 567 February 2013
Practice Exercise 48 580 Answers and Explanatory Notes 713

G
Practice Exercise 49 591
Practice Exercise 50 600 Reserve Bank of India (RBI) Officers, 725
Practice Exercise 51 611
U Grade ‘B’ Examination, November 2015
Answers and Explanatory Notes 728
O
UNIT: NINE 621–655 Life Insursance Corporation (LIC) 737
AAO Examination, March 2016
H
FUNDAMENTALS AND FAST-TRACK
FORMULAE Answers and Explanatory Notes 739
_T

Data Sufficiency:
National Insurance Company Ltd. 744
Type and Strategies 621 Administrative Officers Examination,
April 2015
C

APPENDIX Answers and Explanatory Notes 748


Tables of Metric Units of 656
PS

Measurement Institute of Banking Personnel Selection 756


(IBPS) Specialist Officers (I.T.)
Examination, February 2016
Learning Curve 2 Answers and Explanatory Notes 760
U

Institute of Banking Personnel 768


@

SOLVED TEST PAPERS OF Selection (IBPS) PO (PT) Examination,


VARIOUS EXAMINATIONS October 2016
Answers and Explanatory Notes 770

Institute of Banking Personnel Selection 659 Institute of Banking Personnel Selection 776
(IBPS) Specialist Officers Common (IBPS) Specialist Officers (Marketing)
written Examination, March 2013 Examination, February 2016
Answers and Explanatory Notes 662 Answers and Explanatory Notes 780

Institute of Banking Personnel Selection 669 State Bank of India (SBI) 788
(IBPS) Probationary Officers Probationary Officers (PT) Examination,
Examination, October 2013 July 2016
Answers and Explanatory Notes 673 Answers and Explanatory Notes 790

(vii)
Contents

IBPS-RRBs Officers (PT) 796 IBPS PO (PT) 834


Examination, 2017 Examination, 2018
Answers and Explanatory Notes 799 Answers and Explanatory Notes 837

IBPS Specialist Officer (PT) 807 IBPS PO (PT) Examination, 2018 845
Examination, 2017 Answers and Explanatory Notes 848
Answers and Explanatory Notes 811
State Bank of India (SBI) 856
IBPS Specialist Officers Marketing 821 PO (PT) Examination,

TS
(PT) Examination, 2017 July 2018
Answers and Explanatory Notes 825 Answers and Explanatory Notes 859

H
G
U
O
H
C _T
PS
U
@

(viii)
Learning Curve 1

TS
H
This section is divided into units, each of which

G
covers a set of specific topics of quantitative
U
aptitude, giving (i) step-by-step guidance to the
O
student on the basic concepts and use of shortcuts
to solve problems related to those topics, and (ii)
H
a number of practice exercises on those topics. The
_T

questions range over different aspects of each


subject and are based on the kind of questions set
C

in competitive examinations. The units progress


PS

from the basic level to the more complex problem-


oriented higher levels. The problems in the practice
U

exercises are fully solved, so that students may first


attempt the exercises and then check the answers
@

to know how well they have understood concepts


and how aptly they can apply them.
@
U
PS
C_T
H
O
U
G
H
TS
UNIT ONE

TS
✫ NUMBERS ✫ ADDITION AND SUBTRACTION ✫ MULTIPLICATION
✫ DIVISION ✫ H.C.F. AND L.C.M. ✫ FRACTION ✫ DECIMAL FRACTION

H
✫ EXPONENTS AND ROOTS ✫ PRIME NUMBER ✫ ALGEBRAIC
EXPRESSIONS AND EQUATIONS ✫ SURDS ✫ SIMPLIFICATION

G
FUNDAMENTALS AND U
O
FAST-TRACK FORMULAE
H

Numbers, Numerals and Digits


_T

The number 680957 can be expressed as


Numbers can be expressed in words and in follows:
numerals. The number ‘three hundred and
fifty-seven’ is the word form and ‘357’ is the
C

numeral. Expressing a number in words is


called numeration and representing a number
PS

in numerals is called notation.


Just as all words are made up of alphabets,
all numerals are built up of digits. In our
U

modern number system, we use ten digits—0,


1, 2, 3, 4, 5, 6, 7, 8 and 9—either individually
@

or in combination with each other to build a


numeral.
Numbers are named in terms of the place
value system with base 10, in which the value
of a digit depends on the place in which it is
located in the numeral. The rightmost digit
represents the unit’s or one’s place. Its place Thus, 680957 represents 7 units, 5 tens, 9
value is calculated by multiplying its face hundreds, 0 thousands, 8 ten thousands and
value (i.e., the digit itself) with 1. The second 6 lakhs, or six lakh eighty thousand nine
digit from the right represents the ten’s place. Its hundred and fifty-seven.
place value is calculated by multiplying its face The following table will be useful for
value with 10. The third digit from the right translating any number in numerical form to
represents the hundred’s place, and so on. word form.

3
4 ✫ Quantitative Aptitude

Place value Integers that cannot be divided fully by 2


are odd numbers; –7, –5, –3, –1, 1, 3, 5, 7 are

an nds)
) examples of odd numbers. Any integer, no
res

(m n La s) akh
Te illi on00 L )
s

ho o usa
n m s ( ion

matter how large, is odd if its last digit is not

ds
Te ns) Cro

(te ore mill

s
Cr red s

n s

Te ed th

nd

ds
d

d re

fully divisible by 2.
illio kh

us
1
il li re

L s)
un ro

re
nd s

sa
(b und

(hu akh
r
(h n C

nd

its
ou

ns
nt
o

Absolute value is the number of units a


Hu

Un
H

Th

Te
number is away from 0 on a number line.
9 8 7 6 5 4
10 10 10 10 10 10 10 10 10 10
3 2 1 0 Each integer has a location on a real number

TS
line, where the sign of the number determines
to which side of 0 the number is located.
Digits from the right

Incidentally, in British usage, billion means

H
–5 –4 –3 –2 –1 0 +1 +2 +3 +4 +5
a million million (i.e., 1012), while the American
equivalent is a thousand million (i.e., 109). The number 0 is neither positive nor

G
[The power to which 10 is raised indicates negative.
the number of zeros in the numeral. So 100, Since absolute value is a measure of
which means there is no zero, is 1; 101 means
there is one zero, so it is 10; 102, i.e., two zeros
is 100.] U distance, it is always a positive value. The
symbol for absolute value is two bars on either
O
side of a numerical value or expression: the
All counting numbers are natural numbers. absolute value of 5 is written as |5|; the
Thus, a set of natural numbers N = {1, 2, 3, 4, absolute value of –5 is written as |–5|, and
H
5...} since –5 is 5 units away from 0 on a number
Since each natural number is followed by line, |–5|= 5.
_T

another natural number, the set of natural Any number that can be expressed in the
numbers is infinite. a
Whole numbers include natural numbers form of a fraction, i.e., in the form of , where
b
and zero. Thus, the set of whole numbers W = a and b are integers, and b is not 0, is a rational
C

{0, 1, 2, 3, 4, 5 . . .} number. All integers can be expressed as


So, while all natural numbers are whole rational numbers.
PS

numbers, zero is a whole number which is not 4 –6 0


a natural number. 4= , –6= , 0=
1 1 1
Integers There are infinite rational numbers between
U

Integers, denoted by I, include whole numbers 1 and 10.


and negative numbers. Positive integers are 1,
Decimal numbers are also composed of
@

2, 3, 4, 5, and so on. Negative integers are


–1, –2, –3, –4, –5, and so on. However, 0 is digits, each of which has a value depending on
neither positive nor negative. the place in which it is located. The place value
So, we may say, 0, 1, 2 … are non-negative system for the digits before the decimal is the
integers, and 0, –1, –2, –3 ... are non-positive same as for the digits in integers. Thus, in the
integers. decimal number 5436.2178,
A number with decimal places is not an 6 is the unit’s digit
integer. 3 is the ten’s digit
Integers are either even or odd. Those which 4 is the hundred’s digit
can be divided fully by 2 are even numbers: 5 is the thousand’s digit
– 6, – 4, – 2, 0, 2, 4, 6 are examples of even After the decimal, the first digit is called the
numbers. Any integer, no matter how large, is tenth’s digit, the second one is called the
even if its last digit is fully divisible by 2. hundredth’s digit, the third, the thousandth’s
Unit One : Fundamentals / Numbers ✫ 5

digit, and so on. Thus, in the number 5436.2178, Prime numbers are counting numbers that
2 is the tenth’s digit, are divisible by either 1 or themselves. Thus 2,
1 is the hundredth’s digit, 3, 5, 7, 11 are prime numbers. The following are
a few facts about prime numbers:
7 is the thousandth’s digit, and
(i) All prime numbers are positive.
8 is the ten thousandth’s digit. (ii) Zero is not a prime number. Zero can be
All decimal numbers whose decimals either divided by all counting numbers, even
terminate or repeat infinitely can be expressed though the answer is always zero.
as rational numbers, such as 2.14 and 1.3. 0 0 0
= 0, = 0, =0

TS
214 107 1 10 421
2.14 = =
100 50 (iii) 1 is not a prime number. 1 has only one
4 factor—itself.
1.3 = 1.3333 . . . = (iv) 2 is the only even prime. All other prime
3

H
A number that cannot be expressed in the numbers are odd. That is because all even
a
form of , where a and b are integers, is an numbers except 2 are divisible at least by
b

G
1, 2 and themselves. Of course, all odd
irrational number. Numbers whose decimals
numbers are not prime.
do not terminate and do not repeat cannot be (v) All counting numbers, except 0, 1, and
expressed as rational numbers.
For example, the value of 2 is
U prime numbers, can be expressed in the
form of multiples of prime numbers. For
O
1.414213562 . . . The decimals neither terminate instance, 92 can be expressed as 23 × 2 × 2,
nor repeat. 2 , therefore, is an irrational and 105 can be expressed as 2 × 3 × 17.
number.
H
The product of prime numbers is called
Real numbers include natural numbers, complete factorisation of the given
whole numbers, integers, rational numbers and counting number. There is only one
_T

irrational numbers. The relationship between complete factorisation of any counting


all these numbers is shown by the following number.
Venn diagram. (vi) Pairs of prime numbers which have a
C

difference of two are known as twin primes.


For instance, 3, 5; 5, 7; 11, 13; 71, 73 are
Rational Real Numbers
PS

twin primes.
Integer (vii) The set of three consecutive twin primes
Whole is called a prime triplet. There is only one
such set: 3, 5, 7.
U

Natural Irrational (viii) Pairs of numbers whose highest common


factor is 1 are co-prime to each other. For
@

instance, 7, 15; 8, 21; 6, 19; 9, 16 are co-


primes. The numbers in a pair may or
may not be prime in themselves.
Composite numbers are counting numbers
that are divisible by more than just 1 and
Numbers for which there exists no solution themselves. For instance, 4, 6, 8, 9, 10 are
in the set of real numbers are imaginary composite numbers. Note the following facts
numbers; −1 , for instance, is an imaginary about composite numbers:
number. (Such numbers are also known as (i) All composite numbers are positive.
undefined or complex numbers.) The solution (ii) Composite numbers can be even or odd.
of −1 is i (or iota). (iii) Zero is not a composite number. (Zero is
Thus, − 4 = 4 × −1 = 2i neither prime nor composite.)
(iv) 1 is not a composite number. (1 is neither
−25 = 25 × −1 = 5i prime nor composite.)
2
Also, i = –1 (v) 2 is not a composite number.
6 ✫ Quantitative Aptitude

Addition and Subtraction The following expressions say that B is


of Integers being subtracted from A
A – B
The number obtained after addition is called
the difference between A and B;
the sum and the numbers to be added are
subtract B from A
called addends.
A decreased or diminished by B
Each of the following expressions indicates
B less than A.
that the numbers A and B are to be added:
A + B
the sum of A and B

TS
Fast-track Ways to Add and Subtract
A added to B
A increased by B ● Speed in adding numbers can come
A more than B only with practice. Practise till you
are comfortable with adding numbers

H
total of A and B
mentally. Unless the calculations are
In the conventional method of addition, the
too long, you should not require a

G
numbers to be added are arranged vertically in
paper to put down numbers vertically
columns. In the case of whole numbers, the
and then add.
right-hand-side digits (units), and in the case
of decimals, the decimal points are aligned.
21 3.06 U ● To avoid wastage of time while adding
numbers, leave out all superfluous
words. For instance, while solving
O
232 52.64 2 + 9 + 6 + 4 + 5, do not say 2 plus
7164 0.761 9 is 11, 11 plus 6 is 17, 17 plus 4 is
H
529 239.50 21, etc. Just say 11, 17, 21, 26.
4 62.009 ● Addition of numbers in a single row
_T

Before beginning the addition, it is better to Start adding the numbers from the
estimate mentally what the answer will be. For unit’s digit and strike off the digits
this, the numbers are rounded off to nearest which have been dealt with.
tens, hundreds, thousands and so on. Let’s suppose we have to find the value of
C

For example, 72901 + 82405 + 903 + 1142 + 31


As the first step, add up the digits at
PS

4291  4000
the unit’s place of all the numbers and
792  800
strike them off.
5143  5000
273  300 72901 + 82405 + 903 + 1142 + 31 = 12;
U

(actual) 10499 10100 (estimated) write 2


(1 is to be carried over to the ten’s place)
@

Alternatively, the digits at the left most (Striking off the digits already added
places, in this case the thousands’ digits, can
will help in avoiding duplication.)
be added to get an estimate of the sum. In the
After this, take up the digits at the tens
above example, 4 + 5 = 9 which suggests that
place and add carried over number, if
the total will be somewhere around 9000. As
any:
is obvious, the estimate by rounding off values
is more accurate. A better estimate is possible 72901 + 82405 + 903 + 1142 + 31 = 8 2
by adding up two columns at the left.
(4 + 3 + 1 carried over = 8)
The operation of finding the remaining
number when a number is taken from another In the hundred’s place—
number is called subtraction. The number 72901 + 82405 + 903 + 1142 + 31 = 3 8 2
which is taken from another number is called
subtrahend and the number from which it is 9 + 4 + 9 + 1 = 23; write 3 and carry
subtracted is called minuend. over 2 to thousand’s place
Unit One : Fundamentals / Addition and Subtraction ✫ 7

Next, 72901 + 82405 + 903 + 1142 + 31 and continue. 0 + 2 = 2; write 2 in the


totals.
is 2 + 2 + 1 = 5 + 2 (carried over) Step V: There is one slash mark in this last
= 7382 column. Write 1 at the end in the totals.
Finally, add 7 and 8 to get 15. ● The sum can be solved without putting
The sum will be 157382. the figures in a column. You have to
● You can reach the answer faster by not be careful in counting the correct slash
counting higher than 10. As the total marks in carrying them over.
Suppose you have to add

TS
becomes greater than 10, go on adding
8713 + 142 + 2637 + 4562
with the figures in the unit’s place,
Step I: Starting from the right, 2 + 7 = 9;
making a small mark (a dot or a slash
9 + 2 = 11; put slash mark over 2 and
mark) over the number that led to a continue adding with 1; 1 + 3 = 4. Put

H
total higher than 10 to indicate one to 4 in unit’s total.
be carried over.
8713 + 142’ + 2637 + 4562 = 4

G
Suppose you have to add
7692 + 363 + 1459 + 2563 Step II: Take the next digit, 6. Add 1 to it
(indicated by one slash mark); 6 + 1 =
You may put the figures in a column
7692 ’
’363 U 7; 7 + 3 = 10; slash over 3, carry on with
0; 0 + 4 = 4; 4 + 1 = 5. So five is the
O
ten’s total.
’ ’’
1459 ’ + 4562 = 54
8713 + 142 + 2637

H
2563 Step III: Now the next digit, 5. Add the
12077 carried over 1 represented by the slash
_T

mark to get 6; 6 + 6 = 12, so put a slash


Step I: Start with the unit’s place, either
over 6 and carry on with 2; 2 + 1 = 3;
from top or from bottom. Starting from
3 + 7 = 10. Write 0 in the hundred’s
top we have 2 + 3 = 5; 5 + 9 = 14, which
total, and put a slash over 7.
C

is more than 10; so make a mark (’)


’ + 142 + 2637
8713 ’ + 4562 = 054
against 9 and continue to add with 4;
PS

4 + 3 = 7; write 7 in the unit’s total. Step IV: There are two slash marks in the
Step II: One slash mark means carry over hundred’s place, so add 2 carried over
1 to ten’s column. Now 9 + 1 = 10. to the next digit, 4. 4 + 2 = 6. Now carry
on adding the thousand’s digits. 6 + 2
U

Make a slash mark against 9 and


= 8; 8 + 8 = 16. Write 6 in the totals;
continue. 0 + 6 = 6; 6 + 5 = 11; make
and as there is one slash mark and no
a slash mark against 5 and continue
@

further place value, write 1 at the left


adding with 1. 1 + 6 = 7. Put 7 in the end of the totals.
ten’s total. ’
8713 + 142 + 2637 + 4562 = 16054
Step III: There are two slash marks in the
The method has to be mastered by
ten’s column, so carry over 2 to the
practice.
hundred’s column and add 6 + 2 = 8;
8 + 3 = 11; slash against 3, continue ● When subtracting integers, rewrite the
with 1; 1 + 4 = 5, 5 + 5 = 10; slash problem as an addition problem by
against 5, and 0 in the totals. changing the subtraction sign to an
addition sign and switching the sign
Step IV: Carry over 2 (there are two slash
of the number being subtracted. Then
marks in the hundred’s column) and
simply follow the rules for addition.
add to 7 in the thousand’s column. 7
To find the value of –25 – 5, state the
+ 2 = 9; 9 + 1 = 10; slash against 1, problem as –25 + (–5). Since the signs
8 ✫ Quantitative Aptitude

are both negative, add the absolute Multiplication of Integers


values to get 25 + 5 = 30, but remember The operation of finding the sum of a given
that the solution must be kept negative; number repeated as many times as there are
so, units in the other given number is called
–25 – 5 = –30 multiplication. The number to be repeated is
● Addition and Subtraction Suppose you called multiplicand and the other number is
have to find the value of called multiplier.
7568 – 2248 + 732 – 945 The following expressions are used to
In such questions where there are denote multiplication:

TS
numbers to be added as well as A × B
subtracted the following steps are useful. A multiplied by B
Step 1: First add the numbers with (A) (B)
+ve sign A B

H
Step 2: Add the numbers with –ve AB.
sign The result of multiplying given numbers is

G
Step 3: Subtract the result of step 2 called the product.
from that of step 1.
Multiplication by a two-digit number
In the given question,
Step 1: 7568 + 732
Step 2: –(2248 + 945)
= 8300
= –3193 U Multiply the unit’s digits of multiplicand and
multiplier. Now, cross multiply. Add the two
O
Step 3: Difference 8300 – 3193 = 5107 products and write down to the left of first
result.
H
Checking Addition Illustration 17 × 28
The simplest way to check addition is to check Step 1: 1 7 (7 × 8 = 56, write down 6
_T

in reverse direction. If you have added vertically  and carry over 5)


downwards, check by adding vertically 2 8
upwards. 6
C

Another way to check the calculations is to


tally the sum of the digits of the numbers to be Step 2: 1 7 (1 × 8 + 2 × 7 + 5 = 27, write
PS

added and the sum of the digits of the total down 7 and carry over 2)
obtained. For instance, we found that 294 + 307 2 8
+ 462 totalled 1063. Now let’s check through 76
the sum of the digits method to see if our Step 3: 1 7
(1 × 2 + 2 = 4, write
U

calculation is correct.  down 4)


Step I: Total the digits of the numbers to be 2 8
@

added: 4 7 6
2 + 9 + 4 + 3 + 0 + 7 + 4 + 6 + 2 = 37 The answer is 476.
Step II: Reduce the number obtained to a
single digit number—add the digits Illustration 178 × 23
again and again till you get a single
Step 1: 1 7 8
digit:
 (8 × 3 = 24, write down 4
3 + 7 = 10; 1 + 0 = 1
2 3 and carry over 2)
Step III: Now add the digits of the total
4
obtained.
1 + 6 + 3 = 10 Step 2: 1 7 8
1 + 0 = 1 (7 × 3 + 8 × 2 + 2 = 39, write
If the total of the digits of the numbers in 2 3 down 9 and carry over 3)
both cases match the calculation is correct. 9 4
Unit One : Fundamentals / Multiplication ✫ 9

Step 3: 1 7 8 (ii) 1435 × 620


(1 × 3 + 7 × 2 + 3 = 20, write 1 4 3 5
× 6 2 0
2 3 down 0 and carry over 2)
0 9 4 1×6 1×2+ 1×0+4×2 4×0+3×2 3×0+5×2 5×0
Step 4: 1 7 8 =6+2 4×6= +3×6 +5×6 =0+10 = 0
(1 × 2 + 2 = 4, write down 4) (carried) 2+24 =0+8+18 =0+6+30 1
0
2 3 = 8 +2 +3 +1(carried)
40 9 4 (carried) (carried) 3
7
The answer is 4094.

TS
2
8 2
9

Multiplication by a three-digit number Answer is 889700.


Illustration 243 × 561 Checking up Multiplication

H
Step 1: 2 4 3 (i) 73 × 89 = 6497
 (3 × 1 = 3, write down 3) Add the digits of the multiplicand and

G
5 6 1 the multiplier on the one hand and those
3 of the product on the other. The two
Step 2: 2 4 3
(3 × 6 + 4 × 1 = 22, write
5 6 1 down 2 and carry over 2) U sides should be equal.
Digit sum: 7+ 3 × 8 + 9 = 6 + 4 + 9 + 7
O
10 × 17 = 26
2 3 170 = 2+6
Step 3: 2 4 3
1+7+0 = 8
H
(2 × 1 + 4 × 6 + 5 × 3 + 2 = 43, 8 = 8
5 6 1 write down 3 and carry Thus, our calculation is correct.
_T

over 4) (ii) 78436 × 917 = 71925812


3 2 3 Digit sum:
Step 4: 2 4 3 7+8+4+3+6×9+1+7 = 7+1+9+2+5+8+1+2
C

(2 × 6 + 5 × 4 + 4 = 36, write 28 × 17 = 35
5 6 1 down 6 and carry over 3) 10 × 8 = 8
PS

6323 1×8 = 8
Step 5: 2 4 3
8 = 8
 (2 × 5 + 3 = 13, write down
Thus, our calculation is correct.
5 6 1 13)
U

136323 Fast-track Ways to Multiply with


The answer is 136323. Certain Numbers
@

Gross multiplication and addition may be done ● Multiplication by 5, 50, 500, 5000, .... :
as shown in the examples below. To multiply a number by 5 multiply
(i) Multiply 214 and 635 by 10 and divide the product by 2.
2 1 4 You just have to add a zero to the end
× 6 3 5 of the given number, and divide by 2.
It is an easier process.
2×6 2×3+1×6 2×5+1×3+4×6 1×5+4×3 4×5
10 100
12+1 6+6+3 10+3+24+1 5+12+ =20 = 5, = 50 ...
carried carried carried 2 carried (to 2 2
over over over over be (i) 879 × 5 = 8790 ÷ 2 = 4395
= 15 = 38 = 19 carried) 1000 879000
(ii) 879 × 500 = 879 × =
13 1
5 3
8 1
9 2
0 2 2
Answer is 135890. = 439500
10 ✫ Quantitative Aptitude

Also note: 879 × 25 = 879 × 52


10
2 Step 2: Each successive figure of 432
= 879 × is added to its right-hand
2 neighbour.
879 × 100 432 × 11
=
4 52
87900 (3 + 2 = 5, put 5 below the line)
= = 21975
4 432 × 11
● Multiplication by 25: If you have to 752

TS
multiply with 25, you may put two (4 + 3 = 7, put 7 below the line)
zeros to the end of the number—i.e., 432 × 11
multiply it by 100—and divide it by
4, as in the above example.
4752

H
(first figure is 4, put it below
You can see that if any big number the line)
has to be multiplied or divided by any

G
Answer is 4752.
multiple or power of 5, it can be done
10 (ii) 6547 × 11 = ?
by expressing 5 as equal to
2
.

Rather than full scale multiplication, U 6547 × 11


7
(Put 7 below the line)
O
this short cut is useful for multiplying 6547 × 11
numbers by the powers of 5. (4 + 7 = 11, put 1 below the
17
H
For example, 56328 × 125 = 56328 × 53 line and carry over 1)
3 6547 × 11
_T

10 56328000 (5 + 4 + 1 = 10, put 0 below


= 56328 × = 017
2 8 the line and carry over 1)
= 7041000 6547 × 11
(6 + 5 + 1 = 12, put 2 below
C

● Multiplication by 9, 99, . . . : Place 2017


the line and carry over 1)
same number of zeros to the right of 6547 × 11
PS

the multiplicand as is the number of (6 + 1 = 7, put 7 below the


72017
nines in the multiplier and from this line)
number subtract the real number to Answer is 72017.
U

get the result. ● Multiplication by 12: Double each digit


(i) 3856 × 99 = 3856 (100 – 1) in turn and add to its neighbour. Same
@

= 385600 – 3856 = 381744 procedure as done in multiplication by


(ii) 43516 × 999 = 43516 (1000 – 1) 11 except that before addition, double
= 43516000 – 43516 = 43472484 the number.
846 × 12 = ?
● Multiplication by 11: Put down the 846 × 12
last digit of the multiplicand as the Step 1: (6 × 2 = 12, put 2 below the
unit’s digit. Each successive digit of 2
line and carry over 1)
the multiplicand is added to its
neighbour at the right. 846 × 12
Step 2: (4 × 2 + 6 + 1 = 15, put 5
52
(i) 432 × 11 = ? below the line and carry over 1)
Step 1: Put down the last figure of 432
as the unit’s digit of the answer: 846 × 12
Step 3: (8 × 2 + 4 + 1 = 21, put 1
432 × 11 152
below the line and carry over 2)
2
Unit One : Fundamentals / Division ✫ 11

846 × 12 (ii) If there is a remainder,


Step 4: (8 + 2 = 10, put 10 below the Dividend = (Divisor × Quotient)
10152 + Remainder
line)
or
Answer is 10152. Dividend – Remainder
Divisor =
● Multiplication by 13: Triple each digit in
Quotient
These formulae are useful to keep in
turn and add to its right neighbour. mind.
348 × 13 = ?
(iii) If a number when divided by another
348 × 13

TS
Step 1: (8 × 3 = 24, put 4 below the leaves a remainder, and we need to find the
4 closest number exactly divisible by the given
line and carry over 2) divisor, we can use two methods:
348 × 13 (a) Subtract the remainder from the number

H
Step 2: (4 × 3 + 8 + 2 = 22, put 2 (dividend).
24 So, here the remainder is the least
below the line and carry over 2)
number to be subtracted from any

G
348 × 13 number to make that number exactly
Step 3: (3 × 3 + 4 + 2 = 15, put 5
524 divisible.
below the line and carry over 1)
348 × 13 U (b) Add to the dividend (divisor –
remainder).
O
Step 4: (3 + 1 = 4, put 4 below the So, the least number to be added to a
4524 number to make it exactly divisible is
line) then divisor minus remainder.
H
Answer is 4524.
Example 1 A number when divided by 17
_T

● Multiplication by 14 and 15: The same leaves a remainder of 14. If the quotient
procedure as for multiplication by 12 is 18, what is the number?
and 13 is used, except that instead of Number or Dividend
doubling or tripling, the digit is = Quotient × Divisor + Remainder.
C

multiplied by 4/5, and so on before Now, Number = 18 × 17 + 14 = 320


adding to the right neighbour.
Example 2 Find the greatest three-digit
PS

number divisible by 17.


Division of Integers Now, the greatest three-digit number is
The operation of finding how many times one 999 (as, after that we have 1000, a four-
U

number can be taken from another number is digit number).


called division. The number to be taken is Divide 999 by 17:
@

called the dividend, and that which tells us 17 999 58


how many times it can be subtracted is called 85
the quotient. The number left behind from 149
which the divisor cannot be taken any more is
136
called the remainder.
Dividend = Divisor × Quotient + Remainder. 13
Some rules to remember are: So 13 is the remainder.
If we subtract 13 from 999 we get 986,
(i) When we divide the dividend by the which is the greatest three-digit number
divisor and no remainder is left, then the exactly divisible by 17.
division is called exact division. Example 3 Find the least three-digit number
In case of exact division, exactly divisible by 17.
Dividend = Divisor × Quotient The least three-digit number is 100.
12 ✫ Quantitative Aptitude

So divide 100 by 17 ● Divisibility by 13 To test for divisibility by


13, multiply the unit’s digit by 4 and add
17 100 5 the preceding digit. Continue the process till
85 the end. If you get a number that is divisible
by 13, the entire number is divisible by 13.
15
Now, if we simply minus 15 (remainder) To see if 8567 is divisible by 13, follow the
from 100, we get 85. This may be process:
divisible by 17, but it is not a three-digit 8567
number. So we use method (ii) and add

TS
(divisor – remainder) to the dividend.
7 × 4 = 28
We have
28 + 6 (from 8567 = 34
100 + (17 – 15) = 102
So the required three-digit number is ↓

H
102. 4 × 4 = 16
16 + 3 (from 34) + 5 (from 8567) = 24

G
(iv) When the same number is divided by
two different divisors, and one divisor is a ↓
multiple of the other divisor, and when the 4 × 4 = 16
remainder for the greater divisor is given
( r1 )
π U
, 16 + 2 (from 24) + 8 (from 8567) = 26
Now, 26 is divisible by 13; you may conclude
O
the remainder (r2) for the smaller divisor (d) can that 8567 is divisible by 13.
be found by dividing r1 by d.
Take another number: 63609. To test for its
H
Example A number divided by 512 leaves divisibility by 13, your working should be
a remainder of 23. What will be the remainder
somewhat like this:
_T

if the same number is divided by 16? 6 36 09


Here the greater divisor is 512. Let us see 39/18/33/36
if the smaller divisor is a multiple: 512 ÷ 16 = Since 39 is divisible by 13, you may conclude
32, so 16 is a multiple of 512. We can find the 63609 is divisible by 13.
C

second remainder as we know r1 is 23. [Full working


23 ÷ 16 leaves a remainder of 7. So the same
63609
PS

number if divided by 16 will leave a remainder


of 7.

9 × 4 = 36
(v) If two numbers on being divided by the 36 + 0 (from 63609) = 36
U

same divisor leave the same remainder, the ↓


difference between the two numbers will be 6 × 4 = 24
24 + 3 (from 36) + 6 (from 63609) = 33
@

exactly divisible by the same number.



Example Two numbers 303 and 225, after 3 × 4 = 12
being divided by a two-digit number, leave the 12 + 3 (from 33) + 3 (from 63609) = 18
same remainder. What could be the divisor? ↓
Now, 303 – 225 = 78 8 × 4 = 32
We need to find a two-digit number 32 + 1 (from 18) + 6 (from 63609) = 39]
Factorising 78 we get 78 = 13 × 2 × 3 ● Divisibility by 17 In this case, you need to
We may ignore one-digit numbers 2 and 3. multiply the unit’s digit by 5 and deduct the
Now 13 is, however, not the only possible result from the rest of the digits of the
two-digit number, as multiples of 13 may also number taken together (similar to the case
be an exact divisor of 78. The possible two-digit of testing divisibility by 7). The result has
divisors are 13; 13 × 2 = 26; 13 × 3 = 39; and to be divisible by 17. If the number is too
13 × 2 × 3 = 78. large, continue the process till you get a
Unit One : Fundamentals / Division ✫ 13

smaller number. As soon as you get a result ● Divisibility by 3 A number is divisible


that is divisible by 17, you may stop your by 3 if the sum of the individual digits
calculations and conclude that the given in the number is divisible by 3. The
number is divisible by 17. number 18,036 is divisible by 3 because
To test if 58293 is divisible by 17, follow the 1 + 8 + 0 + 3 + 6 = 18, which is divisible
process: by 3.
58293 ● Divisibility by 4 A number is divisible
↓ by 4 if the last two digits taken together
3 × 5 = 15 is divisible by 4 or if the last two digits

TS
5829 – 15 = 5814 are 0s. Thus 2,11,736 and 4,01,700 are
↓ both divisible by 4, but 2,11,722 is not.
4 × 5 = 20
581 – 20 = 561 ● Divisibility by 5 A number is divisible

H
↓ by 5 if the unit’s digit is either 5 or 0.
1 × 5 = 5 ● Divisibility by 7 A number is divisible

G
56 – 5 = 51 by 7 if on multiplying the unit’s digit by
Now 51 is divisible by 17, so 58293 is 2 and deducting the sum from the


divisible by 17.
Divisibility by 19 Multiply the unit’s digit U remaining number the result is divisible
by 7. Take 6,412. Double the unit’s digit
O
by 2 and add the preceding digits (as in the 2 to get 4. Deduct 4 from 641 to get 637,
case of testing for divisibility by 13). which is divisible by 7.
H
To test if 120137 is divisible by 19, follow Now, if 637 appears too big a number,
the process: repeat the process.
_T

120137 6412
↓ ↓
7 × 2 = 14
2 × 2 = 4
14 + 3 = 17
C

↓ 641 – 4 = 637
7 × 2 = 14 ↓
PS

14 + 1 + 1 = 16 7 × 2 = 14

6 × 2 = 12 63 – 14 = 49, which is divisible by 7.
U

12 + 1 + 0 = 13 So 6412 is divisible by 7.
↓ ● Divisibility by 8 A number is divisible
3 × 2 = 6
@

by 8 if its last three digits are 0, or a


6 + 1 + 2 = 9
multiple of 8. Thus the number 1,79,128

9 × 2 = 18 is divisible by 8 because the last three
18 + 1 = 19 which is divisible by 19. digits taken together, 128, is divisible by
So 120137 is divisible by 19. 8. Again, the number 1,79,000 is divisible
by 8.
Fast-track Tests for Divisibility
● Divisibility by 9 A number is divisible
Divisibility rules for whole numbers are by 9 if the sum of the individual digits
helpful in finding factors. Some of these in the number is divisible by 9. Take the
rules are given here. number 38,322. If we add the digits 3 +
● Divisibility by 2 A number is divisible 8 + 3 + 2 + 2, we get 18, which is
by 2 if the number is an even number, divisible by 9. So the given number is
or if the unit’s digit is 0. divisible by 9.
14 ✫ Quantitative Aptitude

● Divisibility by 10 Any number ending in H.C.F. and L.C.M.


0 is divisible by 10.
Factors and Multiples
● Divisibility by 11 If the difference A factor of a number x is a whole number that
between the sum of digits in the even divides into x evenly without a remainder.
places and the sum of digits in the odd The factors of 50, for example, are 1, 2, 5, 10,
places is either 0 or is a multiple of 11, 25, 50.
the number is divisible by 11. Take 4059. All the factors of a number can be reduced
to prime numbers. Every number has a unique
Now, 9 + 0 is 9, and 4 + 5 is also 9, and
set of prime factors. The prime factors for 50

TS
the difference is 0. The number is divisible are 2 and 5. (2 × 5 × 5 = 50).
by 11. Take 291918. The difference
between (2+1+1) and (9+9+8) is 22, which Highest Common Factor (H.C.F.)
The greatest common factor (GCF), also called

H
is divisible by 11. So 291819 is divisible
highest common factor (H.C.F.), is the largest
by 11. common factor of two or more numbers.

G
● Divisibility by 25 A number is divisible H.C.F. by Factorisation Prime factorisation
by 25 if the last two digits of the number is a convenient way to find the H.C.F. of two
are 0, or taken together they are divisible
by 25. The numbers divisible by 25 end in
U or more numbers. After finding the prime factors
of the numbers given, you have to find the
common factors in pairs. Multiply these common
O
00, 25, 50, 70.
factors to get the H.C.F.
● Divisibility by 99 Starting from the unit’s To find H.C.F. of 24 and 96, break up each
H
place make pairs of the digits. Add the into prime factors:
pairs and any odd number left. If the 24 = 2 × 2 × 2 × 3
_T

sum is divisible by 99, the given number


is also divisible by 99. Take 95337. 96 = 2 × 2 × 2 × 2 × 2 × 3
Making pairs as told, we get 37 + 53 + Common factors are 2 × 2 × 2 × 3 = 24
C

9 = 99, which is divisible by 99. So 95337 Now let’s find the H.C.F. of 1560 and 1755
is divisible by 99.
1560 = 2 × 2 × 2 × 3 × 5 × 13
PS

The methods given above can be used


to find whether a number is divisible 1755 = 3 × 3 × 3 × 5 × 13
by a divisor which can be written as a
H.C.F. = 3 × 5 × 13 = 195
U

product of two or more co-prime


numbers. H.C.F. by Division Method
We can use the division method to find the
@

● To test for divisibility by 6, test if the H.C.F. too. Let’s take the case of two numbers.
number is divisible by 2 as well as 3 (2 Step 1: Divide the bigger number by the
and 3 being co-primes) smaller number and find the
● For divisibility by 12, test for divisibility remainder.
Step 2: Divide the first divisor by this
by co-primes 3 and 4.
remainder and find the second
● For divisibility by 14, test for divisibility remainder.
by co-primes 2 and 7. Step 3: The second divisor is to be divided
● For divisibility by 15, test for divisibility by the second remainder to find the
third remainder and so on till no
by 3 and 5.
remainder is left. The last divisor
● For divisibility by 18, test for divisibility will be the H.C.F. of the given
by 2 and 9. numbers.
Unit One : Fundamentals / H.C.F. and L.C.M. ✫ 15

If the H.C.F. is to be found for 15 and 475, which are already included in the L.C.M. of the
we follow the steps: first two numbers. Multiply the remaining
15) 475 (31 factors to get L.C.M. of the three numbers.
... Step I The process may be repeated for the other
45
numbers if there are more.
25 To find the L.C.M. of 12, 15 and 18, first
15 factorise 12 and 15.
10 ) 15 (1 ... Step II (i) 12 = 2 × 2 × 3
10 (ii) 15 = 3 × 5
5) 10 (2

TS
... Step III Ignore 3 in (ii) as it is included in (i).
10 L.C.M. of 12 and 15 is 2 × 2 × 3 × 5
× Now,
The H.C.F. of 15 and 475 is 5. (iii) 18 = 2 × 3 × 3

H
As 2 × 3 is already there in the L.C.M.
If there are more than two numbers, (say,
of 12 and 15, ignore it, retaining only

G
three numbers), find the H.C.F. of any two
numbers, then find the H.C.F. of the third the 3 left.
number and the H.C.F. of the first two L.C.M. of 12, 15 and 18 is
numbers. That will be the final H.C.F..
Suppose we have to find the H.C.F. of 48, U 2 × 2 × 3 × 5 × 3 = 180
O
72 and 96. L.C.M. by Division We can also find the
First find the H.C.F. of 48 and 72. L.C.M. by division method. Arrange the given
numbers in a row. Divide by a number which
H
divides exactly at least two of the given numbers.
Write the quotients and undivided numbers in
_T

the next line. Repeat the process until you get


a line of numbers which are prime to one
another. The product of the divisors and the
C

∴ H.C.F. is 24. undivided number is the required L.C.M..


Now find the H.C.F. of 24 and 96 Suppose we have to find the L.C.M. of
PS

26, 48, 88, 110


We divide as shown:
2 26, 48, 88, 110
U

24 is the required H.C.F.


11 13, 24, 44, 55
4 13, 24, 4, 5
@

Least Common Multiple 13, 6, 1, 5


The least common multiple (L.C.M.) is the L.C.M. = 2 × 11 × 4 × 13 × 6 × 5 = 34320
smallest number that two or more numbers will
divide into evenly. Fast-Track Formulae and Facts
L.C.M. by Factorisation Prime factorisation
of numbers is one way to find the L.C.M. of ● If H.C.F. of two numbers is 1, then they
given numbers. are co-primes.
Write the factors of two numbers. Strike out H.C.F. of Numerator
the factors in the second number, which are ● H.C.F. of a fraction =
L.C.M. of Denominator
already there in the first number. Multiply the L.C.M. of Numerator
remaining factors of the second number with ● L.C.M. of a fraction =
the factors of the first number to get the L.C.M. H.C.F. of Denominator
of the two numbers. If there is a third number, ● Product of two numbers = Product of
write down its factors. Strike out the factors their H.C.F. and L.C.M..
16 ✫ Quantitative Aptitude

● H.C.F. (being a factor or a divisor) will (ii) If in the above case, there isa
be a smaller entity than L.C.M. (being a remainder r in each case then the
multiple or dividend.) required smallest n-digit number =
● To find the greatest number that will the above answer in 7(i) + r.
exactly divide x, y and z, the required The smallest 2, 3, 4, 5, . . . digit
number = H.C.F. (the greatest divisor) of numbers are 10, 100, 1000, 10000,
x, y and z. . . . respectively.
● To find the greatest number that will
divide x, y and z leaving remainder a, b ● (i) To find a relation between product

TS
and c respectively, the required number of 3 numbers p, q, r and their L.C.M.
= H.C.F. of (x – a), (y – b) and (z – c). and H.C.F.
● To find the least number which is exactly
divisible by x, y and z, the required L.C.M. (p, q, r)

H
number = L.C.M. of x, y and z.
● To find the least number (smallest number) p. q. r. H.C.F. (p, q, r)
=
which when divided by x, y and z leaves H.C.F. (p, q) . H.C.F. (q, r) . H.C.F. (p, r)

G
the remainders a, b and c respectively,
the required number (ii) H.C.F. (p, q, r)
= (L.C.M. of x, y and z) – k
where k = x – a = y – b = z – c.
U =
p. q. r. L.C.M. (p, q, r)
L.C.M. (p, q) . L.C.M. (q, r) . L.C.M. (p, r)
O
● To find the least number which when
divided by x, y and z leaves the same Fraction
remainder k in all three cases, then the
H
x
required number A fraction is a number, written in the form ,
= L.C.M. of x, y and z + k. y
_T

● (i) To find the greatest n-digit number


which represents a ratio or division of two
which is exactly divisible by x, y whole numbers. Here x is the numerator and y
and z (that means, remainder in is the denominator. The denominator tells how
each case is zero). Here, first find many equal parts are there, and the numerator
C

the L.C.M. of x, y and z, then the tells how many of these equal parts are taken.
greatest n-digit number ÷ L.C.M. For example, the given circle
PS

gives a remainder, say, k; (the is divided into 4 equal parts. The


quotient is immaterial here) then 1
shaded part = of the circle. The
the required greatest n-digit number 4
exactly divisible by x, y and z = the 3
U

unshaded part = of the circle.


greatest n-digit number–k. 4
1 3
(ii) If in the above case, it left a Here and are fractions.
@

remainder r in each case, then the 4 4


required number = the above answer This type of fraction is known as common
in (i) + r. The greatest 2, 3, 4, 5. . . fraction or vulgar fraction.
digit numbers are 99, 999, 9999, If the numerator is less than the
99999. . . respectively. denominator, the fraction is called a proper
● (i) To find the smallest n-digit number, 1 19 101
fraction; examples being , , .
which is exactly divisible by x, y 5 21 115
and z, find the L.C.M. of x, y and If the numerator is equal to or greater than
z; then the smallest n-digit number the denominator, the fraction is called an
÷ L.C.M. gives a remainder say k. 21
∴ The required smallest n-digit improper fraction, an example being which
19
number exactly divisible by x, y
and z = smallest n-digit number – 2
can be expressed as the mixed number 1 .
k + L.C.M. 19
Unit One : Fundamentals / Fraction ✫ 17

A mixed number consists of a whole (iii) Between any two given fractions, we can
1 1 find an unlimited number of fractions.
number and a fraction, for example, 2 , 3 , 2 3
5 8 Between and , for example,
2 1 5 13
7 , 9 . These numbers can be easily converted 5 7 9 11
3 2 , , , , ...
into fractions. we have
18 23 28 33
Multiply the whole number by the (iv) If the numerator and denominator of a
denominator of the fraction and add the fraction are equal then value of the
numerator to the product, and place the result fraction is 1.

TS
over the denominator. So, we have 5 8 23
= 1, = 1, =1
1 2×5+1 11 5 8 23
2 = =
5 5 5 (v) If the numerator of a fraction is zero and
1 3×8+1 25 the denominator is a non-zero number,

H
3 = =
8 8 8 then the value of the fraction is 0.
2 7×3+ 2 23 0 0
= 0; =0

G
7 = =
3 3 3 3 12
1 9×2+1 19 (vi) A fraction with zero as denominator is not
9
2
=
2
=
2
Fractions whose denominators are same
U defined because any number divided by
zero is infinity.
7
O
are called like fractions, e.g., 2 , 3 , 6 . 0
= ∞
7 7 7
Fractions whose denominators are different Operations on Fractions
H
are called unlike fractions. Addition and Subtraction To add or
A fraction whose denominator is 10 or any subtract fractions, the denominators must be
_T

power of 10 is known as a decimal fraction, the same or, if they are different, they should
7 17 31 be converted to become the same.
e.g., , , . When denominators are same Add/subtract
10 100 1000
C

all the numerators and place their sum/


Two fractions may be equivalent or equal
difference over the common denominator.
if they represent the same ratio or number
PS

2 4 6 20 6 18 23 6 + 18 + 23 47
= = = + + = =
5 10 15 50 100 100 100 100 100
A fraction whose numerator and 38 27 38 − 27 11
denominator are themselves fractions is called − = =
U

100 100 100 100


a complex fraction.
2 3 When denominators are not same First take
@

9 , 4 the L.C.M. of denominators. Now, the common


5 2 denominator is the L.C.M. itself. Divide the
7 6 L.C.M. by the denominator of each fraction.
Properties of Fractions Then multiply the result with the numerator of
the respective fraction. Lastly add/subtract the
(i) If we multiply both numerator and
numerators as the case may be and place it over
denominator of a fraction with the same
the common denominator.
number (that number is not 0), then the
value of the fraction remains the same. 7 3
Suppose you have to add and
3 2 6 3 9 5
× = = 7 3
5 2 10 5 +
(ii) All fractions are rational numbers. 9 5
2
is a fraction and a rational number. LCM of 9 and 5 is 45
3
18 ✫ Quantitative Aptitude

7 3 (7 × 5) + (3 × 9) 2 1 7 1
∴ + = Now take 3 − 4 + 6 −2
9 5 45 3 4 20 5
3 5 + 27 Write the whole numbers and fractions
=
45 separately with their respective signs, but with
62 a (+) sign between the two sets.
=
45 2 1 7 1
(3 – 4 + 6 – 2) + − + −
As the denominator is larger than the 3 4 20 5
numerator, convert it to a mixed number: 40 − 15 + 21 − 12
62 17 = 3 +

TS
=1 60
45 45 34 17
(See how many times the denominator = 3 = 3
divides into the numerator. Write that number 60 30
(Reduce the fraction to the lowest terms so

H
as the whole number and the remainder as the
numerator.) that the numerator and denominator have no
7 3 common factor.)

G
Now take − If the operation on the set of fractions
9 5 results in a negative sign, subtract the fraction
L.C.M. being 45, we have
(7 × 5) − (3 × 9)
U from the whole number, and that will be the
answer.
O
45 1 1 9 1
Take 5 −4 −3 +5
35 − 27 8 3 4 20 15
= =
H
45 45 1 1 9 1
The same method applies if there are more (5 – 4 – 3 + 5) + − − +
3 4 20 15
than two fractions to add or subtract.
_T

When mixed numbers are involved, the 20 − 15 − 27 + 4


= 3 +
conventional method is to first change the 60
mixed number into an improper fraction and −18
= 3 +
C

then use the correct procedure for fractions. 60


x 18 3
Change A to an improper fraction by = 3 – = 3−
PS

y 60 10
multiplying y and A, then adding the product (fraction reduced to lowest terms)
( A × y) + x Now, you may convert 3 to improper
to x. Put this number over y . .
y fraction and subtract.
U

However, a faster method is to add the 3 × 10 30


whole numbers separately and the fractions 3 = =
10 10
@

separately.
30 3 27 7
1 1 1 − = =2
Take 3 + 4 + 5 10 10 10 10
3 4 5
7
1 1 1 Answer: 2
3 + 4 + 5 + + + 10
3 4 5 Multiplication of Fractions Multiply the
20 + 15 + 12 numerators and divide it by the product of their
= 12 + denominators. Then cancel the common
60
[L.C.M. of 3, 4 and 5 = 60] factors, if any, from the numerator and
denominator.
47
= 12 + 33 12 33 × 12
60 × =
14 9 14 × 9
47 396 22
= 12 = =
60 126 7
Unit One : Fundamentals / Fraction ✫ 19

If there are mixed numbers, change them to How to compare fractions with different
improper fractions and then multiply all numerators and different denominators (i) To
numerators and place this product over the compare two fractions having different
product of denominators. numerators and different denominators, change
2 4 5
Take × 2 × = ? the fractions to equivalent fractions having
3 7 9 common denominators, and compare.
Change mixed numbers into improper
fractions. To find an equivalent fraction, multiply or
4 18 divide the fraction by the number one, in the
2 = 3 1

TS
7 7 a
form of . For example, is equivalent to
2 18 5 180 20 a 9 3
Now, × × = = 3 3 1
3 7 9 189
21 because ÷ = . To find an equivalent
9 3 3
Division of Fractions To divide one fraction

H
1
by another, invert the divisor and multiply; i.e., fraction to , multiply by some fractional form
3
first fraction × reciprocal of second 1 4 4

G
33 13 33 4 of one: × = .
÷ = × 3 4 12
4 4 4 13 (ii) You can find the L.C.M. of the
=
33
13
= 2
7
13 U
denominators—thus getting the same
denominator for all the fractions—and convert
the numerators accordingly to compare.
O
Suppose you have to divide
33 Suppose you have to compare
by 7 2 4
H
4 1 and
The reciprocal of 7 will , so we have 13 15
7 The L.C.M. of 13 and 15 is 13 × 15 = 195
_T

33 33 1 33
÷7= × = So the given fractions are equivalent to
7 4 7 28
2 × 15 4 × 13
Change mixed numbers to improper and
13 × 15 15 × 13
C

fractions. Invert the second fraction (or divisor)


and multiply. 30 52
2 1 and
PS

Suppose we have to divide by 2 195 195


3 4 4 2
1 9 So >
2 = 15 13
4 4 This method is convenient even if a number
U

2 9 2 4 8
÷ = × = of fractions are to be compared.
3 4 3 9 27 2 6 9 4
Comparison of Fractions Take , , ,
@

5 7 11 5
(i) If two fractions have the same denominator, In such a case, it is easier to change the
the one having the larger numerator is the series in decimal form or equating the series by
3 2 taking L.C.M.
greater fraction: is greater than .
7 7 Change in decimal form 0.4, 0.85, 0.81, 0.8
(ii) If two fractions have the same numerator, 6 9 4 2
the one having the larger denominator is Hence, > > >
5 5 7 11 5 5 a c
the smaller fraction: is smaller than . (iii) If there are two fractions and , then
12 11 b d
(iii) When fractions are to be compared, the a c
(a) if ad = bc, then =
fraction with lower numerator and higher b d
denominator is lower in value than the a c
(b) if ad > bc, then >
215 217 b d
other fraction. Of and , the first a c
341 334 (c) if ad < bc, then <
is smaller in value. b d
20 ✫ Quantitative Aptitude

If we have to compare In the case where ratio between the increase


in numerator and increase in denominator is
4 6 less than the first fraction, then last fraction is
and
15 23 considered the least.
We cross multiply: 1 3 5
Take , ,
4 × 23 = 92 3 11 19
6 × 15 = 90 increase in numerator 2 1
Here = < .
The fraction whose numerator after cross increase in denominator 8 3
multiplication gives the greatest value is greater.

TS
So, the last fraction is the least, and if
4 6 increase in numerator
Hence, > = first fraction, then all
15 23 increase in denominator
values are equal.

H
3 7
Now take and 1 2 3
9 12 Take , ,
3 6 9

G
Cross multiply:
numerator 1
3 × 12 = 36 The increases by every
and 9 × 7 = 63
3 7 U denominator
step which is equal to the first fraction.
3
O
Hence, < Here, all the values are equal.
9 12
H
(iv) To find the ascending or descending (v) From the given fractions, if the maximum
orders in given fractions, when the numerator number of digits in the numerator is n, the
maximum number of digits in the denominator
_T

and denominator both increase by a constant


value then the last fraction is the biggest. is m, then find m – n.
If m – n = 0 or 1, then multiply the fraction
1 2 3 4 5 by 10.
Take , , , ,
C

2 3 4 5 6 m–n =2 then multiply the fraction


In this series, the numerators and by 10 2
PS

denominators of the fractions increase by 1, so m–n=3 then multiply the fraction


5 by 103 and so on.
the last fraction, i.e.,
is the greatest fraction.
6 Then find the whole number value (i.e.,
U

2 5 8 11 integer value) of the fraction. In case, if two


Now take , , , fractions have same integer values, then find
7 11 15 19
@

Here the numerator increases by 3 and the one more decimal value for each and then
11 proceed. Then arrange the given fractions
denominator by 4 so is the greatest fraction. according to the integer value, either in
19
But this rule is applicable only when : ascending or decending order as required.

increase in numerator Example Compare and write the following


> first fraction fractions in ascending order.
increase in denominator
98 4 10 1 7 5
In the above example, , , , , ,
1421 7 12 10 135 112
increase in numerator 3 2 Solution The given fractions are
= > ,
increase in denominator 4 7
98 4 10 1 7 5
the last fraction is greater. , , , , ,
1421 7 12 10 135 112
Unit One : Fundamentals / Fraction ✫ 21

98 4 10 1 7 5 (ii) Any fractional number divided by its


1421 7 12 10 135 112 related fraction will give the original number.
Suppose we have to find the total distance
1
m–n = m–n = m–n = m–n = m–n = m–n = travelled by a boy who goes part by bicycle,
3 4
4–2 = 1–1 = 2–2 = 2–1 = 3–1 = 3–1 =
by foot and the rest 48 km by bus.
2 0 0 1 2 2
8
Here the ‘original number’ to be found is
the total distance. The related fraction to total
98 × 100 4 × 10 10 × 10 1 × 10 7 × 100 5 × 100 distance is 1 (being the whole). So fraction

TS
1421 7 12 10 135 112 standing for 48 km
1 3
98 00 40 100 10 70 0 500
= 1− +
4 8

H
1 421 7 12 10 13 5 112 As fractional number ÷ its related fraction
is the original number, here

G
≅ 6.8... 5.7... 8.3... 1 5.1... 4.4... Rest distance
= total distance
Related fraction
∴ 1 < 4.4 < 5.1 < 5.7 < 6.8 < 8.3
1
<
5
<
7
<
4
<
98
<
10 U 48
O
i.e., 1 3
10 112 135 7 1421 12 1− +
4 8
H
Inserting a Fraction in ascending/ 48 48
= =
descending Order 2+3 5
1− 1−
8
_T

a c 8
If and are two fractions, then the
b d
a+c a c 48 48
fraction will lie between and , i.e., = =
b+d b d 8−5 3
C

corresponding numerators are added and the 8 8


8
PS

corresponding denominators are added. 48 × = 128 km


3
a c a a+c c The total distance is 128 km.
Suppose < , then < < .
b d b b+d d
U

(iii) When the fractions are independent,


Now if you want to insert one fraction the fraction for the balance part is
a a+c 1 – sum of the independent fractions
@

between and and similarly one fraction


b b+d 1
a+c c Suppose a family spent of the total
between and , then it will be 3 6
b+d d income on food, of the income on house rent,
8
a 2a + c a+c a + 2c c 5
< < < < and of the income on clothing, and was left
b 2b + d b+d b + 2d d 12
with Rs 450. What is the family’s total income?
Finding a Fraction in Relation to the
Here the amounts spent on the different
Balance
things are independent fractions, each being
(i) The fractional part of any number is indicated to be out of the total income (which
that number × its related fraction. is the original number).
2 2 So, the fraction for balance part
Thus of 30 = × 30 = 20 = 1 – Sum of all independent fractions
3 3
22 ✫ Quantitative Aptitude

1 3 5 the place in which it is located. The place value


1 – + + system for the digits before the decimal (point)
6 8 12
is the same as for the digits in integers (as
4 + 9 + 10 explained earlier). Thus, in the decimal number
= 1−
24 5436.2178, taking the digits before the decimal,
6 is the unit’s digit (value = 6 × 100 = 6),
23 1
= 1− = 3 is the ten’s digit (value 3 × 101 = 30),
24 24 4 is the hundred’s digit (value 4 × 102 =
Now, total income 400), and
Balance (rest) amount 5 is the thousand’s digit (value = 5 × 103

TS
=
Fraction related to = 5000).
balance (rest) amount After the decimal, come the ‘decimal places’,
four in the present case; the first digit is called
450

H
= 450 × 24 = Rs 10800 the tenth’s digit, the second one is called the
1 hundredth’s digit, the third, the thousandth’s digit
24

G
and so on. Thus, in the number 5436.2178,
(iv) In the case of fractions that are
2
dependent on the earlier fractions, the fraction 2 is the tenth’s digit ,

× (1 – second fraction) × . . . × (1 – nth fraction)U


for the balance (or rest) part = (1 – first fraction)
1 is the hundredth’s digit
10
1
100
,
O
1
Suppose the family spends of the total 7
3 6 7 is the thousandth’s digit , and
H
income on food, of the rest on house rent, 1000
5 8
and of what is left on clothing and is then 8
8 is the ten thousandth’s digit .
_T

12 10000
left with Rs 350. What is the total income?
A decimal is actually a fraction, the
Here, after the first item, the next spending denominator of which is understood to be 10
fraction is out of what is left at every stage, so
C

or a power of 10. The number of digits, or


the fractions are dependent. places after a decimal point, determines which
The fraction for the balance part power of 10 the denominator is:
PS

1 3 5 3 56 641
= 1− × 1− × 1− 0.3 = ; 0.56 = ; 0.641 = ;
6 8 12 10 100 1000
5 5 7 175 7121
U

= × × = 0.7121 =
6 8 12 576 10000
The addition of zeros at the end of the
@

Rest amount
Now total income = digits coming after the decimal point does not
Related fraction change the value of the decimal;
350 576 .7 = .70 = .700 . . .
= 350 × 0.98 = 0.980 = 0.9800 = 0.980000
175 175
576 An integer can be represented in decimal
form by adding zero after the decimal point.
= Rs 1152 Any number of such zeros do not change the
value of the whole number.
Decimal Fraction 4 8 = 48.0 (decimal form)
The decimal number system is based on 4 8 = 48.0 = 48.00 = 48.000
representing numbers using powers of 10. Decimals are often approximated and
Decimal numbers are also composed of contracted (to a certain number of decimal
digits, each of which has a value depending on places). The rule is to increase the previous
Unit One : Fundamentals / Decimal Fraction ✫ 23

figure by 1 if the succeeding figure is 5 or more digit after decimal. So we now have 3 6
than 5 after the decimal. Thus, the approximate Carry on:
value of 1.4886 upto three places of decimal is 401.250 + 392.045 + 6.010 + 204.100 + 50.631
1.489 (as 6 is more than 5, increase the previous
figure, 8, by 1 to make it 9). The following The addition of the tenth’s digits after the
examples will make it clearer. decimal gives 9. And 1 carried over to get 10.
The approximate values of the following Write 0, carry over 1 to the unit’s place; so you
(upto the decimal points mentioned) are: have after the decimal .0 3 6.
(i) 0.158 upto 1 decimal place is 0.2 Now carry on with the addition, without

TS
(ii) 1.448 upto 2 decimal places is 1.45. forgetting to add the carried over digits.
(iii) 0.3269 upto 3 decimal places is 0.327.
The total is 1054.036
(iv) 1.44889 upto 4 decimal places is 1.4489.
Multiplication of Decimals
Addition and Subtraction of Decimals

H
Firstly, arrange the decimal numbers in such a Multiplication by a power of 10 When a
way that the decimal points lie in one column. decimal is multiplied by 10, 100, 1000, etc., the

G
Then add or subtract these numbers in the decimal point gets shifted to the right side by
usual way, with the decimal point appearing as many places as is the power of 10 (or zeros).
in the answer at the same place.
Take 6.023 + 37.21 + 8.5707 + 2.9 + 7
U 3.47 × 10 = 34.7
0.347 × 100 = 34.7
O
Place in column: 0.0347 × 1000 = 34.7
6.0230 Multiplication by a whole number Multiply
H
37.2100 the decimal by the whole number ignoring the
8.5707 decimal point. Put the decimal point in the
2.9000
_T

product so that the product has the same


+ 7.0000 number of decimal places as there was in the
61.7037 original decimal.
13.47 × 4 = 53.88
C

Now take 8.0021 – 7.093 1.347× 4 = 5.388


Place in column: 134.7 × 4 = 538.8
PS

To multiply a decimal by another, multiply


8.0021
the two numbers ignoring the decimal points.
– 7.0930 In the product, mark off as many decimal
U

0.9091 places from the right as the sum of the decimal


places in the multiplicand and the multiplier,
For adding up decimal numbers, perplexing zeros if necessary.
@

401.25 + 392.045 + 6.01 + 204.1 + 50.631 For example, when we are to multiply
Put zeros to the end of last digit after 21.255 by 13.4, we ignore the decimal point and
decimal to make the number of digits after the multiply 21255 by 134, and get the product
decimal equal in each number. 2848170. Now count: the multiplicand contains
three places of decimal and the multiplier
The above numbers can be written as
contains one place of decimal. So mark off
401.250 + 392.045 + 6.010 + 204.100 + 50.631 decimal point in the above product taking 4
Now, add in the same way. places from the right.
401.250 + 392.045 + 6.010 + 204.100 + 50.631 = 1 Thus 21.255 × 13.4 = 284.8170 = 284.817

401.250 + 392.045 + 6.010 + 204.100 + 50.631 Division of Decimals


When the divisor is 10 or multiple of 10
The addition of the hundredth digits after Move the decimal point in the dividend to the
decimal gives 13. Write 3, carry 1 to the tenth’s left.
24 ✫ Quantitative Aptitude

93.2 ÷ 10 = 9.32 To find the H.C.F. of 16.50, 0.45 and 15 do


as follows:
8.93
= .0893 Step I 16.50, 0.45 and 15.00
100 (Note: that 15 is to be seen as 15.00, as the other
To divide a decimal by a whole number numbers have two places after the decimal.)
Place the decimal point in the quotient directly Step II Consider these numbers
above the decimal point in the dividend. without decimals: 1650, 45,
Take 18.48 ÷ 24 = 0.77 1500
Step III Now calculate their H.C.F.,

TS
which comes to be 15.
Step IV Now place the decimal in the
result to get 0.15

H
L.C.M. of Decimals The same procedure is
applied as in ‘H.C.F. of decimals’. Make the
To divide a decimal by a decimal Both

G
same number of decimal places in the given
dividend and divisor are multiplied by 10 or numbers, then find their L.C.M. as if they are
a power of 10 so that the divisor becomes a
whole number, i.e., the decimal points in both
the dividend and divisor are shifted by as U integers; in the end, mark decimal places in the
result.
If you have to find the L.C.M. of 0.6, 9.6 and
O
many places to the right as will make the
0.36, you do as follows.
divisor a whole number. For example:
Step I 0.60, 9.60, 0.36
H
12.6061 12.6061 × 102 1260.61 Step II Consider the numbers without
= =
11.02 11.02 × 102 1102 decimals: 60, 960, 36
_T

Now you may go ahead with the division Step III Calculate their L.C.M., which
proper. comes to be 2880.
Just place the decimal in the quotient when Step IV Put decimal places (two places from
bringing down the digit after the decimal in the the right) in the result to get 28.80
C

dividend.
Recurring Decimals
PS

1102 1260.61 1.14 A decimal in which a figure or set of figures is


1102 repeated continually is called a recurring or
circulating or periodic decimal.
1586
U

7
1102 = 1.16666...
6
4841 8
@

4408 = 0.727272...
11
441 13
You may now add zeros to the remainders = 0.295454...
44
and go on dividing till you get no remainder
A decimal fraction in which all the figures
or you get just a repetition of the number in the
quotient, in which case you have a recurring after the decimal point are repeated, is called a
1
decimal, and you may stop. pure recurring decimal, for example, = 0.33....
3
H.C.F. and L.C.M. of Decimals A decimal fraction in which some figures do
H.C.F. for Decimals First, make the same
not recur, is called a mixed recurring decimal,
number of places of decimals in the given 13
numbers. Then consider these numbers without for example, = 0.295454...
decimals. Now calculate their H.C.F. Lastly, 44
there must be as many decimal places in the If it is a single figure repeated, we put a
result as there are in each of the numbers. dot over the figure: 1.3 ; if it is a set of figures
Unit One : Fundamentals / Decimal Fraction ✫ 25

repeated, we put a line (or bar) over the figures: 143


(ii) 1.43 =
3.142857 . 99
71
In the case of mixed recurring decimals, (iii) 0.071 =
e.g., 0.18444..., this can be expressed as 0.184 . 999
Operations on Recurring Decimals The In case of mixed recurring decimals, for the
recurring decimals are converted into vulgar numerator, take the difference between the
fractions and then added, subtracted, multiplied number formed by all the digits after the decimal
or divided as required. point (taking repeated digits only once) and the

TS
Such conversion is not required if the number formed by the non-repeated digits.
multiplier/divisor is 10 or a power of 10. For the denominator, put as many 9s as the
Multiplying by 10 or by a power of 10 does not number of repeating digits followed by as many

H
change the set of repeating digits. You have to zeros as the number of non-repeating digits.
just shift the decimal to include in the integral Take 0.49 . Here the required numerator is
value as many digits as there are zeros in the

G
(49-4), 9 being the repeated digit, and 4 being
multiplier. See the illustrations below.
the non-repeated digit. The denominator is 90.
3.76 × 10 = 3.767676 × 10
= 37.67676 = 37.6 76
U So we have

0.49 =
49 − 4 45 1
= =
O
3.76 × 100 = 3.767676 × 100 90 90 2
= 376. 76 Similarly,
H
3.76 × 1000 = 3.767676 × 1000
2271 − 22 2249
= 3767.6 76 0.2271 = =
9900 9900
_T

In case of division,
and 2.071 = 2 + 0.071
3.76 ÷ 10 = 0.37676 = 0.3 76
3.76 ÷ 100 = 0.037676 = 0.03 76 71 − 0 71 71
= 2+ = 2+ =2
C

990 990 990


3.76 ÷ 1000 = 0.0037676 = 0.003 76
Converting Vulgar Fractions into
PS

Converting Decimals into Vulgar Decimals


Fractions The decimal form of numbers makes
If you have to convert a decimal into a vulgar comparisons easy. To order or compare vulgar
fraction, put 1 in the denominator under the fractions, convert the fractions to decimals by
U

decimal and annex to it as many zeros as there dividing, and then order the decimal
are digits after the decimal point. Remove the equivalents.
@

decimal point. So, 11 16


If you have to compare and , divide
3 5
5 1 the numerators by the respective denomi-
(i) 0.5 = =
10 2 nators.
225 9 1 11 16
(ii) 2.25 = = = 2 = 3.7 ; = 3.2
100 4 4 3 5
11 16
If a pure recurring decimal is to be converted Now, 3.7 > 3.2, so >
3 5
to a vulgar fraction, write the repeated digits Another method can be used if we have to
only once in the numerator and place as many compare a number of fractions with large
9s in the denominator as the number of denominators and numerators, and arrange
repeating digits. So, them in order.
. 4 (i) Let the maximum number of digits
(i) 0.4 =
9 along all the numerators be n, and the
26 ✫ Quantitative Aptitude

maximum number of digits among all Now you need to compare the other three
the denominators be d. as the integers are same.
Find d – n. Divide further to first decimal place.
(ii) If d – n = 0 or 1, multiply each fraction 4940
by 10. = 5.0
971
If d – n = 2, 3, 4, … multiply each 870
fraction by 10 to an equivalent power: = 5.2
102, 103, 104, ... respectively. 165
(iii) After multiplication, find only the 1230
= 5.2

TS
integer value of the resultant fraction 235
at first stage. Now you have the smallest fraction,
494 .
(iv) If, in the above step, two or more 971
We need to go further to second decimal

H
fractions have the same integer value,
find the decimal values one after digit for the other two fractions.
another till you get differences in value 870

G
for all fractions. = 5.27
165
(v) Now you can compare the fractions or
arrange them as required.
Suppose you have to arrange these fractions U 1230
235
= 5.23
O
Now you can arrange the fractions in
in ascending order:
ascending order:
8 494 87 123
H
, , , 494 123 87 8
13 971 165 235 , , ,
You note that the maximum number of 971 235 165 13
_T

digits among the numerators,


Exponents and Roots
n = 3 [494, 123]
The repeated multiplication of a number by
C

The maximum number of digits among the itself can be shown by the use of exponents; for
denominators, d = 3 [971, 235] example 2 × 2 × 2 × 2 = 24 = 16. In the
PS

d – n = 3 – 3 = 0, so multiply each fraction expression 24, 2 is called the base and 4 is the
by 10. exponent or index (plural indices), and we
read the expression as 2 raised to the power
8 80 of 4, or the fourth power of 2.
× 10
U

=
13 13 Squaring
494 4940 When the exponent is 2, the process is called
@

× 10 = squaring. Thus a square is a number or


971 971
expression obtained by multiplying a number
87 870 or expression by itself. For example, the square
× 10 =
165 165 of 4 is 4 × 4 (written as 42), the square of 12
is 12 × 12 (written as 122) and so on.
123 1230
× 10 = There are some short-cut methods for
235 235 squaring, which can be used for the squaring of
Find the integer value; you have big numbers.
Squaring is similar to multiplication. The
80 4940 870 1230 method is to square and double the cross
= 6; = 5; = 5; and =5
13 971 165 235 product to get the answer.
8 Point to Note: If a number has n digits, its
So you have the greatest fraction, square will have either 2n or 2n – 1 digits.
13
Unit One : Fundamentals / Exponents and Roots ✫ 27

Squaring Two-Digit Number Suppose you (1234)2 = 12 2 (1×2) 2 (1×3) + 22 2 (1×4)+2(2+3)


have to square 43.
Step I: Square the right-hand digit: 2 (2×4) + 32 2 (3×4) 42
43 1 4 10 20 25 24 16
(3 × 3 = 9) 2 2 7 5
9 1 5 1 2 2 2 16
Step II: Multiply the two digits of the 1522756
number together and double: 4 × (1234)2 = 1522756
3 is 12; doubled (12 × 2) = 24
43 Fast-Track Ways to find the Square of

TS
49 Some Numbers
Write 24 as 4 and 22 below to
show it is to be carried over. ● For squaring numbers from 51 to 59, use

H
Step III: Square the left-hand figure of the the formula (5X)2 = 52 + X X2
number and add the carried over
number. For example,

G
43 51 2 = (25+1) 12 = 26 01 = 2601
(4 × 4 + carried over 2 = 18)
1849 52 2 = (25+2) 22 = 27 04 = 2704
The answer is 1849.
U 53 2 = (25+3) 32 = 28 09 = 2809
O
2
Squaring Three-Digit Number Suppose 56 = (25+6) 62 = 31 36 = 3136
you have to square a three-digit number, 897. 57 2 = (25+7) 72 = 32 49 = 3249
The method is once again a combination of
H
squaring the digits and doubling the cross- 582 = (25+8) 82 = 33 64 = 3364
products of the various digits and adding them 59 2 = (25+9) 92 = 34 81 = 3481
_T

in specific ways.
(897)2 = 82 2 (8 × 9) 2 (8×7) + 92 2(9×7) 7 2 ● For squaring a number that has 5 in the
= 64 144 193 126 49 unit digit, such as X5, use the formula
C

4 6 0 9 X (X + 1) 25
= 80 16 22 13 4
(The result will always have 25 at the
PS

(Writing down only the unit digits in each case,


carrying over the others and adding to the next end.)
number.) For example,
= 804609 15 2 = 1(1+1) 25 = 2 25 = 225
U

(897)2 = 804609 25 2 = 2(3) 25 = 6 25 = 625


Take another example: 1722 35 2
= 3(4) 25 = 12 25 = 1225
@

(172)2 = 1 2 2 (1×7) 2 (1×2) + 72 2 (7×2) 2 2 2


125 = 12(13) 25 = 156 25 = 15625
=2 19 55 28 4 2352 = 23(24) 25 = 552 25 = 55225
(adding the carried over figures at every stage)
= 29584 ● When a number is near to a power of 10,
(172)2 = 29584 i.e. any 10x, we use the formula,
[When you double the cross product of the end x2 = (x + y) (x – y) + y2
digits, you also add to it the square of the where x is the number that is to be squared
middle digit.] and y is the difference of x from the
nearest power of 10, i.e., 10x – x = y.
Squaring 4-Digit Number Suppose you have
to square a four-digit number. A slightly Suppose you have to find the square of
different combination of the squaring and 996. 996 is nearest to 1000 (1000 is 996
doubling the cross-product is to be followed. + 4). So,
28 ✫ Quantitative Aptitude

(996)2 = (996 + 4) (996 – 4) + 42 (ii) 51 2 = 2601


= (1000) (992) + 42 Digit sum (5+1)2 = 2 + 6 + 0 + 1
= 992000 + 16 (6)2 = 9
= 992016 36 = 9
Suppose you have to square 103. Now 103 3+6 =9
is nearest 100. So, 9 =9
(103)2 = (103 – 3) (103 + 3) + 32 Thus, the square value is correct.
= (100) (106) + 32
Cubing
= 10600 + 9

TS
The cube of a number is the third power of a
= 10609 number. The cube of 2 is 2 × 2 × 2 (written as
● When a number has every digit as one, the 23), cube of 3 is 3 × 3 × 3 (written as 33) and so
method is to count the digits in the given on.

H
number and then write numbers in It would be best to commit to memory the
ascending order from one to the number cubes of 1 to 25 (as well as the squares)—for

G
of digits and then in descending order up facilitating which the table in this section will
to 1. be useful. However, there is a short cut method
to cube two-digit numbers which may be used
Take (111)2
There are three digits, so
U when in doubt or in the case of numbers of
higher dimensions.
O
ascending order
Take an example. If you have to cube 26.
1 2 3 2 1 Step I: Write down the cube of the digit
H
at ten’s place. (2 × 2 × 2 = 8)
descending order The next three digits should be in
and 12321 is (111)2. geometrical ratio of the two digits
_T

Now if it is (4444)2 in the number. Ratio between the


This is 42 × (1111)2 digits = 2 : 6 or 1 : 3
= 16 × (1234321) So the next digits after 23 will be
C

= 19749136 3 3 3 3 3 3
2 3× 23 × × 23× × ×
1 1 1 1 1 1
PS

Checking a Result of Squaring


(i) Say we have the result (897)2 = 804609 = 24 = 72 = 216
Add the digits of both sides separately, till Step II: Under the second and third
you get single digits. If both sides are the number, write down double these
U

same, the result is correct. numbers. Add the two rows.


(897) 2 804609 8 24 72 216
@

48 (24×2) 144 (72×2)


↓ ↓
(8 + 9 + 7)2 8+0+4+6+0+9 17 95 23 7 21 6
↓ ↓ (26)3 = 17576
(2 + 4)2 27 Let’s now take 923
Ratio between the two digits is 9 : 2
↓ ↓
(6)2 2+7 Step I: 93 = 729
Next 3 digits
↓ ↓ 2 2 2 2 2 2
36 9 = 93× 93× × 93× × ×
9 9 9 9 9 9
↓ ↓ So,
3+6 9 729 162 36 8
↓ ↓ Step II: 324 72
9 = 9 778 496 10 8 8
Thus, the square value is correct. (92)3 = 778688
Unit One : Fundamentals / Exponents and Roots ✫ 29

Now, if we take a three-digit number to So,


cube, we must treat the first two digits as a 4096 512 64 8
single unit and proceed as in the cases Step II: 1024 128
illustrated above. 4251 155 5 19 2 8
Let’s find the cube of 162. 3
Ratio is 16 : 2 or 8 : 1 (162) = 4251528

Step I: 163 = 4096 Another example—


Next three digits: (104)3 = ?
Ratio of 10 and 4 is 5 : 2

TS
1 1 1 1 1 1
163× 163× × 163× × × Step I: 1000 400 160 64
8 8 8 8 8 8
Step II: 800 320
= 512 = 64 = 8
1124 124 8 48 6 64

H
3
Squares and cubes of natural numbers (104) = 1124864
(1 to 25) Checking the Accuracy of Cubing

G
(i) 28 3 = 21952
Square Cube Digit sum (2 + 8)3 = 2 + 1 + 9 + 5 + 2

1 1 1
U (10) 3 = 1 9
1000 = 1 + 9
O
2 4 8 1 = 10
3 9 27 1 =1
4 16 64
H
Thus, the cube value is correct.
5 25 125
6 36 216 (ii) 92 3 = 778688
_T

7 49 343 Digit sum (9 + 2)3 = 7 + 7 + 8 + 6 + 8 + 8


8 64 512 (1 + 1)3 = 4 4
9 81 729 2 3 = 4+4
C

10 100 1000 8 =8
11 121 1331 Thus, the cube value is correct.
PS

12 144 1728
13 169 2197 Roots
14 196 2744 If you raise a number a to the nth power and
15 225 3375
U

the result is b, then a is called the nth root of


16 256 4096 b, which is usually written as n b = a.
17 289 4913
The second root is called the square root
@

18 324 5832
and is written as , the third root is called
19 361 6859
the cube root, 3 .
20 400 8000
21 441 9261 There are two possibilities for the square
22 484 10648 root of positive numbers; the positive one is
23 529 12167 called the square root. Thus we say 9 = 3
24 576 13824 although (–3) × (–3) is also 9. Since the square
25 625 15625 of any non-zero number is positive, the square
root of a negative number is not defined as a
Note: (i) The square of a number, other than 1, is
real number. Thus −2 is not a real number.
either a multiple of 4 or greater than a
multiple of 4 by 1. There are cube roots of negative numbers.
(ii) No perfect square ends in 2, 3, 7, 8. 3
−8 = –2, because (–2) × (–2) × (–2) = –8.
30 ✫ Quantitative Aptitude

Finding the Square Root The square root Step III We get the remainder (here, 2) and
of a given number can be found by means of bring down the next pair. Double
(i) prime factorisation method, and the quotient (here, 3) and write to
(ii) long division method the left.
Prime factorisation method In this method,
3 11 97 16 3
the given number is expressed in the form of
9
its prime factors. Then we make pairs of the
6 297
similar factors and choose one out of every pair.
Now the product of these chosen numbers is Step IV At this stage, the divisor will be

TS
the square root of the given number. equal to double the above quotient
(here, 6) with the new quotient
For example, suppose you have to find the (here, 4).

H
square root of 676. Find the prime factors.
2 6 7 6

G
2 3 3 8
13 1 6 9
13 1
1
3
U Step V We get the remainder (here, 41)
O
and bring down the next pair.
676 = 2 × 2 × 13 × 13
Now we take 34 × 2 = 68 for new
H
= 2×13 divisor with the quotient of this
= 26 stage (here, 6)
_T

Long division method Make the pairs of


the digits of the given number from right to left
and then divide the number. The quotient is the
C

answer.
It can be better understood with the help
of the following example.
PS

Suppose we want to find the square root


of 119716.
Step I We make pairs of the digits of the We repeat these steps till all the pairs get
U

number from right to left. exhausted.


11 97 16 Here, square root of 119716 is 346.
@

The last digit may be either single


Let’s take another example: find the square
or a pair.
root of 15625.
Step II Now we start dividing the
number. Firstly, we have to find a 1 15625 125
number whose square may be 1
equal to or less than the first pair 22 056
(here, 11). 44
In our case the number is 3. 245 1225
1225
×
∴ 15625 = 125
Unit One : Fundamentals / Exponents and Roots ✫ 31

Another example: Square root of 1734489 is Finding the unit digit in a number raised
to some powers
The method is to multiply the index of the given
number by 4 and check the remainder.
(i) If the remainder is 1, the last digit of the
end product will be the same as the unit
digit of the given base number.
(ii) If the remainder is 2, the last digit will
be the last digit of the square of the unit

TS
digit of the given base number.
(iii) If the remainder is 3, the last digit will
∴ 1734489 = 1317 be the last digit of the cube of the unit
Finding the Cube Root The cube root of a digit of the given base number.

H
given number can be found by means of prime (iv) If the remainder is zero, and the unit
factorisation method. In cube root, grouping of digit of the given base number is odd,

G
three similar factors is done. the last digit of the end product will be
1.
For example, the cube root of 551368 is (v) If the remainder is zero, and the unit
2 551368
2 275684 U digit of the given base number is even,
the last digit of the end product will be
O
6.
2 137842 [The above (i) to (v) are applicable to base
H
41 68921 numbers that do not end in 5; if the base
41 1681 number ends in 5, the last digit of that number
raised to any power will be 5.]
_T

41 41
(vi) We may conclude that, in a given
1 number (abc)n, where abc is the base
3 number with c in the unit digit and n
551368
C

is the index, and x is the last (unit) digit


= 2 × 2 × 2 × 41 × 41 × 41 of (abc)n,
PS

= 2×41 (a) if c is 1, 5 or 6, then x will be 1, 5


= 82 or 6 respectively, whatever the value
of n.
Take another example: Cube root of 13824 (b) if c is 4, and n is odd, x will be 4.
U

2 1 3824 (c) if c is 4, and n is even, x will be 6.


2 6 912 (d) if c is 9, and n is odd, x will be 9.
2 3 456
@

(e) if c is 9, and n is even, x will be 1.


2 1 728
2 8 64 Suppose we have to find the unit digits in
2 4 32 (1963)130, (2368)199, (2119)173, (586)1876 and 7105
2 2 16
2 1 08 ● (1963)130: 130 ÷ 4 remainder 2; the
2 54 last digit in the given base number is
3 27 3; so by (ii) above, the unit digit in
39 (1963)130 will be 32 = 9.
33
1 ● (2368)199: 199 ÷ 4 remainder 3; the
last digit in the given base number is
3
13824 = 2 × 2 × 2 × 2 × 2 × 2 × 2 × 2 × 2 × 3 × 3 × 3
8; so by (iii) above, the unit digit in
= 2×2×2×3
= 24 (2368)199 will be 83 = 51 2 , i.e., 2.
32 ✫ Quantitative Aptitude

● (2119)173: Here the last digit of the base When the bases are different but exponents are
number is 9 and the index 173 is odd; the same.
a n × b n = ( a b)
n
so the last digit of (2119)173 will be 9 ●

2 2 × 3 2 = ( 2 × 3) = 6 2 = 36
2
[by vi (d)].
m
● (586)1877: The last digit in the base am a
● =
number is 6; so, whatever the index, the bm b
last digit of (586)1877 will be 6. 2
22 2
=

TS
● 7 : 105 ÷ 4
105
remainder 1. Here the
32 3
unit digit in the given base number is m
a m .b m ab
7. So going by (i) above, the last digit ● =
of 7105 will be 7. cm c

H
6
26 . 8 6 2×8
Rules of Exponents (or Indices) 6
= = 46
4 4

G
There are some rules which, if kept in mind,
will be useful in solving problems involving Remember also . . .
exponents.
When the base is the same:
a m × a n = am+ n U 0
● a = 1 (always where a is a rational
number)
( a − n )0 = 1
O


22 × 2 3 = 2 2 + 3 = 2 5 = 32
n ● ( a n )0 = 1
● a m is read as a raised to the power m
H
power n. ● If am = bm, then a = b.
3 ● If am = an, then m = n.
22 = 2 = 8 (when m > n)
8
_T

● The expression ( a m ) is read as ‘a raised If an = 1, then n is 0.


n

to the power m, the whole raised to n’. 1
● an = n
a , i.e., the nth root of a.
( a m )n = a m × n
C

m
( 22 )3 = 22 × 3 = 26 = 64n ● an = n a m , i.e., the nth root of a to the
[It will be seen that ( a m ) is not the power m.
PS

same as am × an; nor is it to be confused ● If two numbers are to be compared,


n
with a m .] either the base or the exponent should
be made the same for both numbers.
am
U

● = am− n Worked Examples


an 2
● Suppose you have to evaluate (.008) 3 .
@

23
= 23 − 2 = 21 = 2 Focus on 8 and see how you can break
22
it down into a simplest factor. 8 = 23.
1 Now do not forget the two zeros after
= am
a− m the decimal.
1 10
am . ak m+ k − n− + 3 .008 = 23 × = 23 × 10–3
1
= a 2
1000
2
an . a . a −3
(2 )
2 2
(.008) 3 =
3
× 10 −3 3

am 1 2 2
= n− m (if n is greater than m) 3. −3.
a n
a = 2 3
× 10 3

= 22 × 10–2
37 1 1 1
= = 2 = 22 4
39 39 − 7 3 9 = = = .04
102 100
Unit One : Fundamentals / Prime Number ✫ 33

● Suppose you have to find which of the (ii) The nearest value of 191 is 14.
two is greater, 2300 or 3200. 142 = 196 > 191
Here it is necessary to make either the The prime numbers less than 14 are 2,
exponents or the base same for both 3, 5, 7, 11 and 13
numbers. It is best to make the exponents 191 is not divisible by 2 and 5.
the same. Both numbers can be made to Sum of the digits = 1 + 9 + 1 = 11, not
have exponents of 100. divisible by 3 either. It is not divisible
2300 = (23)100 = 8100 by 7, 11 or 13.
∴ 191 is a prime number.

TS
3200 = (32)100 = 9100
It becomes clear that 9100 > 8100; so 3200 Some Rules and Concepts for Numbers
is greater than 2300.
● Sum of all the first n natural numbers

H
● Suppose you have to find the value of n (n + 1)
x if 2x+1 . 5x = 200. =
2

G
Now 2x+1 . 5x = 200 ● Sum of first n odd numbers = n2
(21 × 2x) × 5x = 200 ● Sum of first n even numbers
2 × 2x × 5x = 200
2x × 5x = 100 = 102
U ●
= n (n +1)
Sum of squares of first n natural
n (n + 1) (2n + 1)
O
10x = 102 numbers =
6
∴ x = 2 ● Sum of cubes of first n natural numbers
H
2
n (n + 1)
=
2
Test for a Prime Number
_T

● Addition or multiplication of any type


Prime numbers less than 100 are: of numbers (natural, whole, integers, or
2, 3, 5, 7, 11, 13, 17, 19, 23, 29, 31, 37, 41, 43, rational) gives the same type of number
47, 53, 59, 61, 67, 71, 73, 79, 83, 89, 97 as a result.
C

Finding out a prime number > 100 For example,


Let x be the given number. Find a number 2 + 3 = 5 (all natural numbers)
PS

bigger than the approximate square root of the 2 × 3 = 6 (all natural numbers)
number x. Let it be k such that k2 > x. 2 + (–3) = –1 (all integers)
Check the divisibility of x by all the prime 2 × (–3) = –6 (all integers)
U

numbers less than k. If x is not divisible, then ● Addition or multiplication of numbers


x is a prime number. does not depend on the order of the
Suppose you have to find out if (i) 287
@

addends or multiplicands:
and (ii) 191 are prime numbers. a + b = b + a
(i) 17 is nearly the root of 287 a × b = b × a
a c
172 = 289 > 287 For rational numbers, and ,
The prime numbers less than 17 are 2, b d
a c c a
3, 5, 7, 11, 13. + = +
b d d b
Sum of the digits of the given number, a c c a
287 = 2 + 8 + 7 = 17, which is not × = ×
b d d b
divisible by 3. (Refer to Fast-track tests ● Addition or multiplication of numbers
of divisibility.) Obviously 287 is not
does not depend on how the numbers are
divisible by 2 or by 5. But 287 is
grouped. For numbers a, b and c:
divisible by 7.
a + (b + c) = (a + b) + c
∴ It is not a prime number. a × (b × c) = (a × b) × c
34 ✫ Quantitative Aptitude

a c e ● If a positive number is added to a


For rational numbers, , and negative number, then the difference
b d f
between the numbers is found out
a c e a c e
+ + = + + and then the sign of the bigger number
b d f b d f among them is given. For example:
a c e a c e +10 – 12 = –2
× × = × × 7 – 18 = –11
b d f b d f
100 – 125 = –25
● The property of identity is applicable 125 – 100 = +25

TS
in case of whole numbers, integers 18 – 7 = 11
and rational numbers. In addition of ● If both the numbers are negative and
numbers, the additive identity is 0. then the sum is found out, the negative
a + 0 = a (–ve) sign is given.

H
a 0 a
+ = For example: –10 – 12 = –22
b c b –7 – 8 = –15

G
In multiplication of numbers, the –100 – 125 = –225
multiplication identity is 1. ● In the case of multiplication, when a
a × 1 = a
a 1 a
× = U positive number is multiplied by a
positive number, then the product
O
b 1 b will be a +ve number. Thus,
● Additive inverse is applicable for 2 × 4 = 8 (all positive)
integers and rational numbers. This When a negative number is multiplied
H
property is based on the fact that by a negative number, then the product
every number has an inverse. If the will be a positive number. Thus,
_T

operation is addition, the number –2 × –4 = 8 (positive)


‘–a’ is said to be an additive inverse When a positive number is multiplied
of the number ‘a’. Thus by a negative number, the product is
C

a + (–a) = 0 always negative. Thus,


a −a 2 × –4 = –8
+ =0 –2 × 4 = –8
PS

b b
b
● The number is said to be the
a Algebraic Expressions
a
U

multiplicative inverse of . and Equations


b
a b Elementary algebra can be viewed as an
× =1
@

b a extension of arithmetic. An algebraic expression


● a × (b + c) = (a × b) + (a × c) usually involves letters besides numbers. The
letters are called variables and represent
However,
a + (b × c) ≠ (a + b) × (a + c) quantities whose values are unknown.
Numbers, in algebra, are called constants.
● A number is considered positive if no
A number that comes in front of a variable (that
sign is given before the number.
is connected by multiplication) is known as a
● If a positive number is added to coefficient. Algebraic expressions containing
another positive number, then the terms created by the product of constants and
sum will definitely be positive (+ve). variables are called polynomials. A single term
For example is called a monomial. Examples of monomials:
(+) 10 + (+) 12 = (+) 22 2, 3x, –4bc, and 33x2y. Examples of polynomials:
(+) 7 + (+) 8 = (+) 15 a–b, 4x–3y, a2+2a–4. The term like terms are
Unit One : Fundamentals / Algebraic Expressions and Equations ✫ 35

polynomial terms containing exactly the same we should factorise the numerator and
variable(s) and exponent(s); e.g., 3x and 4x; 7xy denominator. We get
and –6xy; 8x2y and –10x2y.
3x (x + 2)
Operation with Algebraic Expressions
4x2 (x + 2)
Operations with algebraic expression are subject
The factors common to both numerator and
to the same rules that govern operations with
denominator are x and (x + 2). Dividing these
numbers. Like terms can be combined by simply 3
adding their coefficients as shown: out, we get x.
4

TS
3x + 6x = 9x One must be careful not to cancel terms
y2x + 5y3 – 2y3 + 7 = y2x + 3y3 + 7 where such cancelling is not warranted. Take
When multiplying two algebraic the term
6 + t

H
expressions, each term of the first expression is
multiplied by each term of the second expression 8 + t
There is no scope of cancelling here as

G
and the results added as shown:
(a + 2) (3a – 5) = a (3a) + a (–5) + 2 (3a) + 2 (–5) there is no factor that divides the entire
= 3a2 – 5a + 6a – 10 numerator as well as the entire denominator.
= 3a2 + a – 10
[Signs must be carefully noted and correctly U Suppose we have to find the sum of
1
a
1
and .
b
O
placed when multiplied. So (+) × (+) = (+); Cross multiply and put the sum over the
(+) × (–) = (–); (–) × (–) = (+).] denominator product and add the cross product.
H
When dividing, you can get the result by 1 1 b+a
dividing each term by the divisor and adding + =
_T

a b ab
all the quotients. For example, a3 b3
Now, what is the product of and .
9a + 4ab + b 2
9a 4 ab b 2
2
b a2
+ + =
a a a a Divide a2 into the first numerator and
C

b2 second denominator; then divide b2 into the


= 9 + 4b +
PS

a first denominator and second numerator; the


Or you can cancel common factors, for a b
example expression is now × or ab.
1 1
a 2 − 2a a ( a − 2) Some Important Algebraic Identities
=
U

= a
a−2 a−2 A statement of equality between two algebraic
Again, you have to keep signs in mind expressions that is true for all possible values
@

while writing the quotients. of the variables involved is known as an


If an algebraic expression is the product of identity. A statement of equality between two
other algebraic expressions, the expressions are algebraic expressions that is true for only some
called factors of the original expression. We values of the variables involved is known as
need to be able to factor algebraic expressions an equation.
in order to solve quadratic equations. It is also Basic to using algebra in problems is
helpful in dividing algebraic expressions. conceptualising an equation. The first step is
In 3x + 12, 3 is a common factor and can to determine what quantity or letter you wish
be factored out to give 3 (x + 4). to isolate. Solving an equation for x means
If we have to reduce to its lowest factor the getting x on one side of the ‘=’ sign and
term everything else on the other side.
3x 2 + 6 x Some standard identities must be kept in
,
4x 3 + 8x2 mind as they come in useful not just in algebra
36 ✫ Quantitative Aptitude

problems but also in arithmetic problems of You have to learn to look for and find pairs
simplification and in rationalising surds. of numbers that can be in the form of one of
Remember the following: these identities.
2
(i) (a + b)2 = a2 + b2 + 2ab ● Suppose you have to factor a + 4a + 3.
(ii) (a – b)2 = a2 + b2 – 2ab You need numbers whose product is 3
(iii) (a + b)2 = (a – b)2 + 4ab and which add up to 4. Look at the
or possible factors of 3 and check whether
(a + b)2 – (a – b)2 = 4ab they add up to 4. As 3 = 3 × 1 and 3 + 1
(iv) (a + b)2 = 2(a2 + b2) – (a – b)2 = 4, the factors are (x + 3) and (x + 1).
2

TS
● Suppose it is a + a – 6 that you have
or
to factorise. The number –6 is negative,
(a + b)2 + (a – b)2 = 2(a2 + b2)
so one of the factors must be negative.
(v) a2 + b2 = (a + b)2 – 2ab
Possible pairs of factors for –6 are 1 and
or

H
–6, –1 and 6, 3 and –2, and –3 and 2.
a2 + b2 = (a – b)2 + 2ab As the coefficient of a is one, the factors
(vi) a2 – b2 = (a + b) (a – b)

G
of –6 must be 3 and –2 [as 3 + (–2) =
(vii) a3 + b3 = (a + b) (a2 – ab + b2) 1]. So the factors of the given expression
or are (a + 3) and (a – 2).
a3 + b3 = (a + b)3 – 3ab (a + b)
(viii) a3 – b3 = (a – b) (a2 + ab + b2)
U Equations
An equation indicates that two algebraic
O
or
a3 – b3 = (a – b)3 + 3ab (a – b) expressions are equal. Solving an equation
(ix) (a + b)3 = a3 + 3a2b + 3ab2+b3 means finding the values of the variables that
H
make the equation true; i.e., the values satisfy
or
the equation. Generally, one refers to the ‘left
(a + b)3 = a3 + b3 + 3ab (a + b)
_T

hand side’ (L.H.S.) and ‘right hand side’ (R.H.S.)


(x) (a – b)3 = a3 – 3a2b + 3ab2– b3
of the equation depending on which side of the
or equals sign one refers to. The general method
(a – b)3 = a3 – b3 – 3ab (a – b) for solving an equation is to find successively
C

(xi) a4 – b4 = (a2 + b2) (a + b) (a – b) simpler equivalent equations till the simplest


(xii) (a+b+c)2 = a2 + b2 + c2 + 2ab + 2bc + 2ca equivalent equation makes the solutions clear.
PS

(xiii) (a + b + c)3 = a3 + b3 + c3 + 3(b + c) (c + a) One equation is equivalent to another, if


(a + b) they have exactly the same solutions. The
(xiv) a3 + b3 + c3 – 3abc equations 2a + 2 = 4 and a + 1 = 2 are
= (a + b + c) (a2 + b2 + c2–ab–bc–ca)
U

equivalent; both are true when a = 1.


or
a3 + b3 + c3 – 3abc Important Rules for Producing
@

1 Equivalent Equations
= (a + b + c) [(a – b)2 +(b–c)2 +(c–a)2]
2
(xv) If a + b + c = 0, then a3 + b3 + c3 = 3abc ● If the same algebraic expression or
a + b + c − 3 abc
3 3 3 constant is added to or subtracted from
(xvi) a + b + c = each side of an equation, the equality is
a2 + b 2 + c 2 − ab − bc − ca
preserved, and the new equation is
(xvii) (a + b) (b + c) (c + a) = ab (a + b) + bc (c + b) equivalent to the original equation.
+ ca (c + a) + 2abc ● If both sides of an equation are
(xviii) (a – b) (b – c) (c – a) = –[a2 (b – c) + b2 (c – a) multiplied or divided by the same non-
+ c2 (a – b)] zero algebraic expression or constant,
(xix) (a + b + c) (ab + bc + ca) = a2 (b + c) + b2 the equality is preserved, and the new
(c + a) + c2 (a + b) +3abc equation is equivalent to the original
(xx) (x + a) (x + b) = x2 +x (a + b) + ab equation.
Unit One : Fundamentals / Algebraic Expressions and Equations ✫ 37

(i) Linear Equations ● Solve for x : 3x + 15 = 3 – 4x


A linear equation may involve one or more You could just group the variables to the
variable but no variables is multiplied together
L.H.S. and the constants to the R.H.S.
or raised to a power greater than 1.
3x + 4x = 3 – 15
Linear Equations of One Variable A linear
7x = –12
equation in one variable is an equation of the
type Divide each side by 7
ax + b = 0 or ax = c, −12
x =
where a, b and c are constants, a ≠ 0, and x 7

TS
is an unknown variable. In the following illustrations you see how the
To solve a linear equation of one variable, algebraic equation is used to solve problems on
simplify each side of the equation. numbers.
(i) Group all the terms that involve the ● The numerator of a fraction is 2 less

H
unknown terms on one side of the than the denominator. If 1 is subtracted
equation and all the terms that are from the numerator and 3 added to the

G
purely numerical on the other side of the 1
denominator, the fraction becomes .
equation. This is called isolating the 4
What is the fraction?
unknown (variable). Generally, the
unknown quantity (variable) is placed
U Let the numerator of the fraction be x.
O
on the L.H.S. and the constant terms on Then its denominator is x + 2.
the R.H.S. x
Thus, the fraction is
(ii) Combine the terms on each side. x+2
H
(iii) Divide each side by the coefficient of the Given that,
unknown (variable). x −1 1
=
_T

Suppose you have to get the value of x, ( x + 2) + 3 4


given 4 (x – 1) = (x + 2) + 3
6x + 2 = 3 4x – 4 = x + 5
C

Group the like terms 4x – x = 5 + 4


6x = 3 – 2 3x = 9
9
PS

6x = 1 x =
1 3
x = [Dividing each side by 6] x =3
6
x 3
=
U

Worked Examples Hence, the fraction is


x+2 5
x −1 1
● Solve for x : = 3x − 7 ● A number consists of two digits with 4
@

2 2
in the unit’s place. If the two digits are
Simplify both sides 4
x−1 6 x − 15 interchanged the new number is times
7
= the original number. What is the
2 2
x – 1 = 6x – 15 number?
Group the variables on R.H.S. A two-digit number has 4 in its unit’s
x – 6x = –15 + 1 place. Let x be in its ten’s place.
[When the terms move from one side to the ∴ Original number is 10x + 4
other, the signs change; so 6x becomes –6x, By interchanging the digits, the number
and –1 becomes 1.] becomes 10 (4) + x = 40 + x
–5x = –14 Given that,
4
−14 14 4 40 + x = (10x + 4)
x = = =2 7
−5 5 5 7 (40 + x) = 4 (10x + 4)
38 ✫ Quantitative Aptitude

280 + 7x = 40x + 16 x − 6 12 − 6
Thus, the fraction is =
7x – 40x = 16 – 280 x 12
–33x = –264 6 1
= =
−264 12 2
x = [Many other problems relating to age of
−33
persons, profit and loss, interest, ratio, time and
x =8
work, etc. are easy to solve using algebraic
Thus, the original number is 10x + 4
equations, as will be seen in the coming units.]
= 10 × 8 + 4 = 84
Linear Equations in Two Variables A

TS
● The sum of digits of a two-digit number
linear equation in two variables is an equation
is 9. The new number formed by
of the type
interchanging the digits is 27 less than
ax + by + c = 0
the original number. What is the
or ax + by = d,

H
number?
where a, b, c and d are constants, and a ≠ 0,
Let the digit at unit’s place be x, then the b ≠ 0; the unknown variables are x and y.

G
digit at ten’s place is 9 – x.
For example, 2x + 7y + 4 = 0,
∴ The number is 10 (9 – x) + x
7 x − 5y + 12 = 0,
By interchanging the digits, the number
we get is 10 (x) + (9 – x)
Given that U x – y = 43
Any pair of real values of x and y that
O
10 (9 – x) + x – 27 = 10x + (9 – x) satisfies the equation is known as a solution
90 – 10x + x – 27 = 10x + 9 – x of the linear equation. A pair of linear equations
H
63 – 9x = 9x + 9 in two variables is said to form a system of
–9x – 9x = 9 – 63 simultaneous linear equations.
_T

–18x = –54 For example, x + y = 17 and 2x + 9y = 12;


− 54 7y = 32
x = 8x – 3y = 71 and x +
− 18
x = 3
C

Thus, the number is 10 (9 – x) + x Conditions of Solvability


= 10 (9 – 3) + 3
PS

The equations a1x + b1y + c1 = 0 and


= 63
a2x + b2y +c2 = 0 have
● The numerator of a fraction is 6 less a1 b1 c1
than the denominator. If the denominator ● no solution when a = b ≠ c
U

2 2 2
is increased by 8 and the numerator is ● an infinite number of solutions when
3 a1 b1 c1
doubled, we get . What is the fraction?
@

5 = =
a2 b2 c2
Let the denominator be x, then the a b
1 1
numerator is x – 6 ● a unique solution when a ≠ b
2 2
Given that,
2( x − 6) 3 ● only one solution x = 0, y = 0 when
= a1 b
x+8 5 ≠ 1
a2 b2
2 x − 12 3
=
x+8 5 The methods to solve these equations are
5 (2x – 12) = 3 (x + 8) as follows.
10x – 60 = 3x + 24 Substitution In the substitution method,
10x – 3x = 24 + 60 one of the equations for one of the variables,
7x = 84 say y, is solved, and then this value for y is
x = 12 substituted in the second equation for y.
Unit One : Fundamentals / Algebraic Expressions and Equations ✫ 39

The second equation then becomes an Substitute the value of x in any equation
equation with one variable, in this case x, as in the earlier example to get the value of y.
which can be solved. Now, substituting this Comparison In this method one variable is
value of x, thus obtained, in the first equation, expressed in terms of the other variable from
the value of y can be found. both the equations. Then the expressions are
If we have to solve 3x + y = –2, 7x – 3y equated to the value of one variable.
= –26, we solve the first equation for y; Suppose we have to solve the equations
y = –3x – 2 x + 4y = 2 and
Substituting this value of y in the second x

TS
equation, we get + 8y = 7
2
7x – 3 (–3x – 2) = –26 Here, x + 4y = 2
Now the equation contains only one x = 2 – 4y … (i)
variable, so applying the distributive property,

H
x
we have and + 8y = 7
2
7x + 9x + 6 = –26 x

G
16x = –26 – 6 = 7 – 8y
2
16x = –32 x = 2(7 – 8y) … (ii)

x =
−32
16
= –2
U ∴ From (i) and (ii)
2 – 4y = 2 (7 – 8y)
O
Taking any of the given equations, we can 2 – 4y = 14 – 16y
substitute the value of x : 12y = 1 2
H
3x + y = –2 y =1
3 × (–2) + y = –2 Put y = 1 in equation (i)
_T

–6 + y = –2 x = 2 – 4 (1)
y = –2 + 6 = 4 x = –2
So, x = –2, y = 4 Hence, x = –2 and y = 11
C

Elimination In the elimination method, we Cross-Multiplication Suppose we have two


multiply both equations by such numbers so as equations
PS

to make the coefficients of one of the unknown a1x + b1y + c1 = 0 and


variables numerically the same. a2x + b2y +c2 = 0
Then we combine the two equations; one Then,
of the variables will be eliminated. Then follow x y 1
U

the steps of substituting the value of the variable b1 c1 a1 b1


as in the other method.
@

Suppose we have to solve b2 c2 a2 b2


3x + y = –2
7x – 3y = –26 x y 1
= =
Take the variable y. In the first equation, it b1c2 − b 2 c1 c1 a2 − a2c 2 a1b 2 − a2 b1
has a coefficient of 1, in the second it has a The solution is
coefficient of –3. Multiply the first equation by
3 (positive) we have b1c 2 − b2 c1
x =
9x + 3y = –6 a1b 2 − a2b1
7x – 3y = –26 c1 a2 − a1c 2
Now combine the like terms of the two y =
equations. The ‘y’ terms get eliminated and we a1b 2 − a2b1
have Let’s find the solution of the equations
16x = –32 4x – 9y = 0, and
x = –2 3x + 2y = 35
40 ✫ Quantitative Aptitude

We would proceed thus: ● Find the solution of the equations.


4x – 9y = 0 … (i) x+4 y+3
+ =2
3x + 2y = 35 … (ii) 4 5
Here, a1 = 4, b1 = –9, c1 = 0 and
a2 = 3, b2 = 2 and c2 = –35 and 2x + 1 − y + 5 = 2 is
3 2
x y 1
x+4 y+3
–9 0 4 –9 + =2
4 5
5 (x + 4) + 4 (y + 3) = 2 × 4 × 5

TS
2 3 2
–35 5x + 20 + 4y + 12 = 40
−9( −35) − 2(0) 315 5x + 4y = 8 … (i)
x= = =9 2x + 1 y + 5
4(2) − 3( −9) 35 Also, given − =2
3 2

H
0(3) − ( −35) (4) 140 2 (2x + 1) – 3 (y + 5) = 2 × 3 × 2
y= = =4 4x + 2 – 3y – 15 = 12
4(2) − 3( −9) 35

G
4x – 3y = 25 … (ii)
Hence, x = 9 and y = 4 Multiply equation (i) by 4 and equation (ii)

Worked Examples
U by 5, then subtract them
20x + 16y = 3 2
O
● Find the values of x, y and z respectively 20x – 15y = 125
– + –
in the equations 2x + 3y = 0, 3y + 4z = 14,
31y = –93
and 2x + 4z = 26.
H
∴ y = –3
2x + 3y = 0 … (i) Put the value of y in equation (i)
3y + 4z = 1 4 … (ii)
_T

5x + 4 (–3) = 8
2x + 4z = 2 6 … (iii) 5 x = 8 + 12
Put the value of y from equation (i) in 5x = 20
equation (ii) x =4
C

2x 4
From equation (i) : y = – ● A fraction becomes when 5 is added to
3 5
−2x
PS

its numerator and 1 is subtracted from its


∴ 3 + 4 z = 14 1
3 denominator. It becomes when 3 and 5
2
–2x + 4z = 14 … (iv) are subtracted from its numerator and
U

Now add equations (iv) and (iii)


denominator respectively. What is the
–2x + 4z = 1 4
numerator of the fraction?
2x + 4z = 2 6
@

8z = 4 0 Let the numerator be x and denominator


∴ z = 5 be y
Then,
Now put the value of z in equation (ii) x+5 4
3y + 4 (5) = 1 4 y −1 =
5
3 y = 14 – 20 5 (x + 5) = 4 (y – 1)
3y = –6
5x + 25 = 4y – 4
y = –2
Now put the value of y in equation (i) 5x – 4y = –29 … (i)
2x + 3 (–2) = 0 x−3 1
Also y−5 =
2x = 6 2
x = 3 2x – 6 = y – 5
∴ x = 3, y = –2 and z = 5 2x – y = 1 … (ii)
Unit One : Fundamentals / Algebraic Expressions and Equations ✫ 41

Multiply equation (ii) by 4 and subtract it x – 4y = 3 … (i)


from equation (i) x−1
5x – 4y = –29 and y −1 = 7
8x – 4y = 4 x – 7y = – 6 … (ii)
– + –
–3x = –33 Subtract equation (ii) from equation (i)
∴ x = 11 x – 4y = 3
Hence, the numerator is 11. x – 7y = – 6
– + +
● If the numerator of a certain fraction is 3y = 9
increased by 2 and the denominator is ∴ y = 3

TS
increased by 1, then the resulting fraction Put y = 3 in equation (i)
1 x – 4 (3) = 3
equals . If, however, the numerator is
2 x = 3 + 12
increased by 1 and the denominator x = 15

H
decreased by 2, then the resulting fraction Hence, the numerator of the fraction is 15.
3

G
equals . What is the fraction? ● A lady has only 10 paise and 25 paise
5 coins in her purse. If in all she has 60 coins
Let the numerator be x and denominator be totalling Rs 8.25, then what are the number
y. Then
x+2
=
1
U of 10 paise and 25 paise coins possessed
by the lady respectively?
O
y+1 2
Let the number of 10 paise coins be x and
2 (x + 2) = y + 1
that of 25 paise coins be y.
H
2x + 4 = y + 1 ∴ x + y = 60 … (i)
2x – y = – 3 … (i)
10 x 25
_T

x +1 3 Now + y = 8.25
and = 100 100
y−2 5 10x + 25y = 825
5 (x + 1) = 3 (y – 2)
2x + 5y = 165 … (ii)
C

5x + 5 = 3y – 6 Multiply equation (i) by 2 and subtract


5x – 3y = –11 … (ii) equation (ii) from it
PS

Multiply equation (i) by 3 and then subtract 2x + 2y = 120


equation (ii) from it. 2x + 5y = 165
6x – 3y = – 9 – – –
5x – 3y = –11 –3y = –45
∴ y = 15
U

– + +
x = 2
Now put x = 2 in equation (i) Putting the value of y in equation (i), we get
@

2 (2) – y = – 3 x + 15 = 6 0
– y = –3 – 4 x = 45
y = 7 The lady has 45 ten paise coins and 15
2 twenty five paise coins.
∴ The fraction is .
7 (ii) Quadratic Equations
● A fraction becomes 4 when 1 is added to If we equate a quadratic polynomial to zero, it
both the numerator and denominator and is called a quadratic equation. The most general
it becomes 7 when 1 is subtracted from both form of a quadratic equation is
the numerator and the denominator. What
ax2 + bx + c = 0,
is the numerator of the fraction?
x where a, b, c are real numbers, and a ≠ 0
Let the fraction be y . Then If we simply write ax2 + bx + c, then the
x+1 expression is not a quadratic equation—it is
y+1 = 4 only a quadratic polynomial.
42 ✫ Quantitative Aptitude

A quadratic equation will usually have 2 Linear Inequalities


different solutions, but a quadratic may also Inequalities can involve variables and are
have only one solution or elude a solution similar to equations, except that the sign of
altogether. inequality replaces the sign of equality used in
an equation.
Solving a Quadratic Equation by
It is an axiom that if a and b are two
Factorisation The method depends on the fact
rational numbers then either
that if the product of expressions is zero then
a = b
at least one of the expressions must be zero.
or a > b
Take an example: solve x2 + 5x = –4

TS
or a < b
x2 + 5x + 4 = –0 The procedure to solve a linear inequality
x2 + 5x + 4 = (x + 4) (x + 1) = 0 is similar to that used to solve a linear equation:
So x = –4 and x = –1 simplify by isolating the variable on one side

H
of the inequality using the rules:
Quadratic Formula The solution to a (i) When the some constant is added to or
quadratic equation can also be found using the

G
subtracted from both sides of an
formula inequality, the direction of the inequality
b ± b 2 − 4 ac is preserved and the new inequality is
x =
2a
where the notation ± indicates that there are U equivalent to the original.
(ii) When both sides of an inequality are
O
two solutions—one that uses the plus sign and multiplied or divided by the same
the other that uses the minus sign. constant (which is not zero), the direction
H
of the inequality is preserved if the
Worked Examples constant is positive, but reversed if the
constant is negative. However, either
_T

● Solve 6x2 + x – 15 = 0. way, the new inequality is equivalent to


6x2 + x – 15 = 0 the original.
2
6x + 10x – 9x – 15 = 0 ● Solving the inequality
2x (3x + 5) – 3 (3x + 5) = 0
C

(3x + 5) (2x – 3) = 0 –3x – 6 ≤ 21


−5 3 –3x ≤ 15
PS

x= or
3 2 (subtracting 6 from both sides)
● Find the value of x if 2x2 – 5x – 3 = 0. −3 15
2x2 – 5x – 3 = 0 x ≥
−3 −3
U

−b ± b2 − 4 ac (both sides divided by –3, which


x = reverses the direction of the inequality)
@

2a
x ≥ –5
− (−5) ± (−5)2 − (4) (2) ( −3)
=
2 (2) Principles of Inequalities to
5 ± 25 + 24 Remember
=
4 (i) If a < b and b < c, then a < c
(ii) If a < b then a + c < b + c
5± 49
= (iii) If a < b and c > 0, then ac < bc
4 (iv) If a < b and c < 0 then ac > bc
5±7
= (v) The product of xa and yb is maximum
4 (provided x + y = constant) when
12 −2
= or x a
4 4 = .
−1 y b
3 or
2
Unit One : Fundamentals / Surds ✫ 43

Surds of the two surds is known as a rationalising


factor of the other.
If a is a positive rational number, and n is a
1 If we multiply 3 8 by 8 , we get 3 8 × 8
positive integer such that n a or a n is irrational,
= 3 × 8 = 24, a rational number. So 8 is the
then n a is called a surd of order n.
n
rationalising factor of 3 8 .
a is called the radical sign.
In n a , a is the radicand, and n is the order If the surd is of the type, a + b , its
of the surd, also known as radical index. rationalising factor is a – b .

TS
Surds are those quantities whose roots Rationalising factor of
cannot be exactly obtained.
A pure surd is a surd with unity as its 1
(i) is a
rational factor, the other factor being irrational, a

H
e.g., 7 , 3 5, 4 7 . 1
(ii) is a  b
A mixed surd is a surd in which there is a± b

G
a rational factor other than unity and the 1
(iii) is a b
a± b
irrational factor, e.g., 2 7 , 3 5, 3 3 18 .

Rules to remember U Comparing Surds


O
1 Surds can be compared only if they are of the
(i)
n
a= a n
same order. The radicals can then be compared.
H
(ii) n
ab = n
a× n
b Comparing 3 4 and 3
● 8 , we may say
a n
a that since 8 > 4,
_T

(iii) n =
8 >
n
b b 3 3
4
(iv) ( n a )n = a ● If the surds are of different order and
different base, we need to reduce them
C

mn
(v) m n
a = a
to the same order.
(vi) ( n a )m = n a m
PS

3 5
Take 6 and 4
1 1
(vii) If x = n (n + 1), then
These terms are 6 3 and 4 5
n = x − x − x − .......∞
U

We need to find the LCM of 3 and 5.


LCM of 3 and 5 is 15.
n+1 = x + x + x + .......∞
@

Now,
Rationalising Surds 1 5 1 1
= (6 5 )
3 15 15
= (7776 )
15
Rationalisation is the process of removing the 6 = 6
radicals, i.e., roots ( ) from an expression or 1 3 1 1
= ( 43 )
and 5 15 15
= (64 )
a part of it without changing the value of the 15
4 = 4
whole expression.
In a given fraction in the form of roots, we We may conclude 3 6 > 5 4
have to try and change the root form of the [We need not even calculate 65, as
denominator by multiplying the denominator obviously 65 > 43.]
with a suitable quantity such that it becomes ● You may be asked to arrange a set of
an integer. surds in ascending/descending order.
If n
a× n
b is a rational number, then each Take
4
6, 2 and 3
4
44 ✫ Quantitative Aptitude

Mathematical Signs and Symbols Find the LCM of the order of the surds.
LCM of 4, 2 and 3 is 12.
+ Plus, the sign of addition, e.g., 5 + 3. It
1 3 1 1
also denotes a positive quantity, e.g., +3.
= (6 )
12 12
= ( 216 )
4 12
– Minus, the sign of subtraction. It also
4
6 = 6 = 6 3

denotes a negative quantity. 1 6 1 1


= ( 26 )
2 12 12
= (64 )
12
× Sign of multiplication. 2 = 2 = 2
÷ Sign of division. 1 4 1 1
= (44 )
. Dot at the centre of the two numbers is 3 12 12
= ( 256 )
12
3
4 = 4 = 4
the sign of multiplication.

TS
In descending order, we have
. Dot at the base of the two numbers is
the sign of decimal. 1 1 1
= The sign of equality, read as equal to. ( 256 )12 , (216)12 , (64)12

H
≠ The sign of not equal to.
> Sign of greater than. Or 3
4 > 4
6 > 2

G
Sign of greater than or equal to. (The ascending order will be
< Sign of less than.
≤ Sign of less than or equal to. 2 < 4
6 < 34)

3
Sign of square root or under root.
Sign of cube root. U In simplifying surds, some of the standard
algebraic identities come in useful.
O
n Sign of ‘n’th root.
a2 The square or the second power of a; Simplification
H
a2 = a × a. To simplify a given expression accurately, the
a3 The cube or the third power of a; magic word is VBODMAS. In a lengthy and
_T

a3 = a × a × a. complicated algebraic expression, start from the


an The ‘n’th power of a. left and apply VBODMAS in the same order as
|| Two vertical bars denote the absolute it appears in the term, where
value of a number or mode of a number, V stands for vinculum or bar : ——
C

e.g., | – 4 | = 4. B stands for brackets : [{( )}]


∝ Sign of infinity.
PS

O stands for of : of
( ) Sign of parenthesis.
D stands for division : ÷
[ ] Sign of bracket.
M stands for multiplication : ×
{ } Figured bracket.
A stands for addition : +
U

~ Sign of similarity.
S stands for subtraction : –
≅ Sign of congruency.
We first remove the bar and then brackets
 Sign of since or because.
@

∴ Sign of therefore or hence. in the order parenthesis ( ), then curly brackets


∪ Sign of union. { }, then the square bracket [ ]. Then we proceed
∩ Sign of intersection. with the arithmetic operations of division,
⊆ Sign of subset. multiplication, addition and subtraction in that
∈ Sign of ‘belongs to’ or ‘is a member of’. order.
∑ Sign of summation. Another point to remember is that you can
∫d Sign of integration. apply the algebraic identities to solve many
Sign of differentiation. problems easily are below.
dn
≡ Sign of identical to.
! Sign of factorial. Worked Examples
i Sign of −1 .
1 1 1 1 1 1
? Question mark. ● 7− of 1 + 2 − + −
3 4 2 2 2 4
Unit One : Fundamentals / Simplification ✫ 45

(1.3)3 + (1.2)3
= 7 − 1 of 5 5 1 1
+ − + ● =?
3 4 2 2 4 (1.3)2 − 1.3 × 1.2 + (1.2)2
The given expression is in the form
= 7 − 1 of  5 +  5 − 3  a3 + b3
3  4 2 4  , where a = 1.3 and b = 1.2
a − ab + b 2
2

= 7 − 1 of  5 + 7  ( a + b) ( a 2 − ab + b 2 )
4 4 =
3  
( a 2 − ab + b 2 )

TS
= 7 − 1 of 3 = a + b
3 1.3 + 1.2 = 2.5
1 ● Find the value of
= 7− × 3

H
3 0.538 × 0.538 − 0.462 × 0.462
= 7 – 1 1 − 0.924

G
= 6 The given expression is in the form
a2 − b2

1
3 −
2
1
3
of
1 1
3 − 2 −
2 2
1
2 U a− b
, where a = 0.538 and b = 0.462
O
( a + b) ( a − b)
1 1 1 5 1 =
( a − b)
= 32 − of 3 − −
H
3 2 2 2 = a + b
_T

  0.538 + 0.462 = 1.00


= 7 −  1 of  7 − 2 
2 3 2  2x
● If = 1, what is x?
1
1+
C

= −  of 
7 1 3 x
2 3 2 1+
1− x
PS

7 1 3 The given expression


= − × 
2 3 2 2x
U

1
7 1 6 1+
= − = (1 − x ) + x
2 2 2
1− x
@

= 3 2x
1
● Find the value of 0.87 × 0.87 + 0.13 × 0.13 1+
1
+ 2 × 0.87 × 0.13 1− x
The given expression is in the form a2 + b2 2x
+ 2ab, where a = 0.87 and b = 0.13 1 + (1 − x )
= (a + b)2 2x = 2 – x
= (0.87 + 0.13)2 3x = 2
= (1.00)2
x = 2
= 1 3
46 ✫ Quantitative Aptitude

PRACTICE SESSION
Practice Exercise 1
1. What is the difference between the place 10. Which of the following numbers is exactly
value and the face value of 7 in the divisible by 24?
numeral 2 4 3 6 7 5 2 4 9 ? (a) 35718 (b) 63810
(a) 7 9 9 3 (b) 6 9 9 9 3 (c) 537804 (d) 3125736

TS
(c) 6 9 9 3 (d) 7 0 0 0 0
11. In a sum involving division, the divisor is
2. What would be the unit digit in 964 × 12 times the quotient and 5 times the
8218 × 617 × 2223 ? remainder. If the remainder is 48, then

H
(a) 7 (b) 4 what would be the dividend?
(c) 2 (d) 6 (a) 2147 (b) 3100

G
3. The sum of prime numbers lying between (c) 3694 (d) 4848
60 and 75 is 12. A student was asked to divide a number
(a) 199
(c) 211
(b) 201
(d) 272
U by 6 and add 12 to the quotient. He,
however, first added 12 to the number and
then divided it by 6, getting 112 as the
O
4. There are four prime numbers written in
ascending order. The product of the first answer. The correct answer should have
three is 385 and that of the last three is been
H
1001. The last number is (a) 143 (b) 138
(a) 11 (b) 1 3 (c) 122 (d) 136
_T

(c) 1 7 (d) 1 9 13. A number when divided by the sum of 555


5. If we write all the whole numbers from and 445 gives two times their difference
200 to 400, then how many of these contain as quotient and 30 as the remainder. The
C

the digit 7 once and only once? number is


(a) 3 2 (b) 3 4 (a) 1220 (b) 1250
(c) 3 5 (d) 3 6
PS

(c) 22030 (d) 220030


6. If n is a natural number, then n is 14. 998 × 1002 = ?
(a) always a rational number (a) 99996 (b) 999996
(b) always a natural number (c) 999994 (d) 2000
U

(c) always an irrational number


15. 212 × 188 is equal to
(d) sometimes a natural number and
(a) 38956 (b) 35986
@

sometimes an irrational number


(c) 39856 (d) 39866
7. The sum of all prime numbers between 61
16. The difference between the squares of two
and 89 is
(a) 610 (b) 523 consecutive numbers is 35. The numbers
(c) 460 (d) 373 are
(a) 14, 15 (b) 15, 16
8. The sum of three consecutive odd numbers (c) 17, 18 (d) 18, 19
is always divisible by
(a) 2 (b) 3 17. The sum of two numbers is twice their
(c) 5 (d) 7 difference. If one of the numbers is 10, the
other number is
9. The sum of the first 45 natural numbers
1
is (a) 3 (b) 3 0
(a) 1035 (b) 1280 3
1 1
(c) 2070 (d) 2140 (c) 30 or − 3 (d) 30 or 3
3 3
46
Unit One : Practice Session ✫ 47

18. The largest number of five digits exactly (a) 0, 6 (b) 5, 6


divisible by 77 is (c) 5, 4 (d) None of these
(a) 99946 (b) 99956 27. The smallest number that must be added
(c) 99964 (d) 99965 to 803642 in order to obtain a multiple of
19. The number 23*7 is divisible by 3. The 11 is
missing digit (*) is (a) 1 (b) 4
(a) 1 (b) 2 (c) 7 (d) 9
(c) 3 (d) 4
28. The number which is nearest to 457 and
20. The difference between the largest 4 digit is exactly divisible by 11 is

TS
number and the smallest 3 digit number (a) 450 (b) 451
is (c) 460 (d) 462
(a) 9899 (b) 8999
29. The owner of a local jewellery store hired

H
(c) 9989 (d) 9889
3 watchmen to guard his diamonds, but
21. On dividing 59761 by a certain number, a thief got in and stole some diamonds.

G
the quotient is 189 and the remainder is On the way out, the thief met each
37. The divisor is watchman, one at a time. To each he gave
(a) 517 (b) 316
(c) 711 (d) 418
22. A number when divided by 114 leaves the U half of the diamonds he had taken, besides
2 more. He escaped only with one
diamond. How many did the thief
O
remainder 21. If the same number is originally steal ?
divided by 19, then the remainder will be (a) 1 6 (b) 2 6
H
(a) 1 (b) 2 (c) 3 6 (d) 4 6
(c) 7 (d) 2 1 30. The smallest five digit number exactly
_T

23. The difference between two numbers is divisible by 476 is


1365. When the larger number is divided (a) 10000 (b) 10472
by the smaller one, the quotient is 6 and (c) 10476 (d) 47600
C

the remainder is 15. The smaller number


31. 997 × 997 is equal to
is
(a) 9994009 (b) 994000
(a) 240 (b) 270
PS

(c) 994009 (d) 9949


(c) 295 (d) 360
32. The least number of five digits exactly
24. While solving a question involving
divisible by 567 is
division with zero remainder, a candidate
U

(a) 10206 (b) 10702


took 12 as divisor instead of 21. The
(c) 10361 (d) 10567
quotient obtained by him was 35. The
@

correct quotient should have been 33. What least number must be subtracted
(a) 0 (b) 1 2 from 1000 to get a number exactly divisible
(c) 1 3 (d) 2 0 by 17?
25. The sum of three consecutive odd numbers (a) 2 1 (b) 1 7
is always divisible by (c) 1 4 (d) 9
I. 2 II. 3 34. The number which is nearest to 3006 and
III. 5 IV. 6 exactly divisible by 29 is
(a) Only I (b) Only II (a) 3042 (b) 3031
(c) I and III (d) II and IV (c) 3024 (d) 3016
26. If the number 357*25* is divisible by both 35. The largest number of four digits exactly
3 and 5, then the missing digits in the divisible by 88 is
unit’s place and the thousandth place (a) 9768 (b) 8888
respectively are (c) 9988 (d) 9944
48 ✫ Quantitative Aptitude

36. 128 players start in the men’s singles at (a) 1, 2 (b) 2, 3


a tennis tournament, where this number (c) 3, 2 (d) 4, 1
reduces to half on every succeeding round. 40. The unit digit in 7105 will be
How many matches are played totally in (a) 9 (b) 7
the event? (c) 1 (d) None of these
(a) 6 0 (b) 8 3
(c) 127 (d) 153 41. The unit digit in the product of 365 × 659
× 771 will be
37. A boy was asked to multiply a certain (a) 2 (b) 4
number by 53. He multiplied it by 35 and (c) 6 (d) 7

TS
got his answer less than the correct one
by 1206. Find the number to be multiplied. 42. (1000)9 ÷ 1024 = ?
(a) 6 8 (b) 6 7 (a) 1 0 (b) 100
(c) 7 7 (d) 8 8 (c) 1000 (d) 10000

H
38. A number is divided successively in order 43. (112 × 54) = ?
by 4, 5 and 6. The remainders are (a) 76000 (b) 7000

G
respectively 2, 3 and 4. The number is (c) 67050 (d) 70000
(a) 214 (b) 476 44. 217 × 217 + 183 × 183 = ?
(c) 954 (d) 1908
39. A number when divided successively U (a) 80578
(c) 70568
(b) 80698
(d) 81268
O
by 4 and 5 leaves remainders 1 and 4 45. What will be the remainder when 17200 is
respectively. When it is successively divided by 18?
H
divided by 5 and 4, then the respective (a) 1 (b) 2
remainders will be (c) 1 7 (d) 7
_T

Answers with Solutions


1. Ans. (b) 4. Ans. (b)
C

Sol. The face value of 7 is 7. The place Sol. Let the four prime numbers be p, q, r
value of 7 in the numeral is 70000. The and s. Given pqr = 385 and qrs = 1001
PS

difference between the two values is qrs 1 00 1


∴ =
70000 – 7 = 69993. pqr 3 85
s 13
U

2. Ans. (c) ⇒ =
Sol. Multiplying the unit digits of the given p 5
numerals will give the required unit Thus, the last number is 13.
@

digit. So, [Alternative Method: You are given that


4 × 8 × 7 × 3 p, q, r, s are in ascending order. Now
= 3 2 × 2 1 factorise 385; 385 = 5×7×11. Clearly the
last number must be greater than 11.
Again multiplying the unit digits, we
Factorise 1001. You may safely assume
have 2.
7 and 11 to be two factors in 1001. The
∴ 2 must be the unit digit of the product
factors in 1001 are 7×11×13. So the last
of the given numerals.
number is 13.]
3. Ans. (d)
5. Ans. (d)
Sol. Prime numbers lying between 60 and
Sol. The whole numbers between 200 and
75 are 61, 67, 71 and 73.
300, which contain the digit 7 once and
Their sum is 61+67+71+73 = 272
only once, are 207, 217, 227, 237, 247,
Unit One : Practice Session ✫ 49

257, 267, 270, 271, 272, 273, 274, 275, 9. Ans. (a) n(n + 1)
276, 278, 279, 287, and 297. Sol. We know that 1 + 2 + 3 + ... n =
2
Similarly, the total number of whole 45 (45 + 1) 45 × 46
numbers between 300 and 400 which ∴ =
2 2
contain the digit 7 once and only once = 1035
are also 18.
∴ Total of such numbers from 200 to 400 10. Ans. (d)
is 18 + 18 = 36. Sol. Any number divisible by 24 should
[Alternative Method: Take the first 100 also be divisible by 3 and 8.

TS
numbers. Whole numbers with 7 as 35718 is not divisible by 8.
last digit occur once in every 10 63810 is not divisible by 8.
numbers. However, once it occurs 537804 is not divisible by 8.

H
double (in 77) which goes against the 3125736 is divisible by both 3 and 8.
given condition. So we have only 7 11. Ans. (d)

G
occurring as last digit for 9 times. Now, Sol. Divisor = 12 × Quotient
as the first digit it also occurs 10 times Divisor = 5 × Remainder
(70 to 79) but rejecting 77 for the same
reason as given above, we have it 9
U Remainder = 48
∴ Divisor = 5 × 48 = 240
O
times. So, a total of 18 times. This Divisor 240
would be true for every hundred. From ∴ Quotient = = = 20
12 12
200 to 400, we have two sets of 100s
H
Dividend = Divisor × Quotient
so the whole numbers containing 7 + Remainder
once and only are 18 + 18 = 36.]
_T

= 240 × 20 + 48
6. Ans. (d) = 4848
Sol.Take n = 2; n = 2 which is an 12. Ans. (c)
Sol. Let the number be x.
C

irrational number, so n is not always


rational. ∴ (x + 12) ÷ 6 = 112
x + 12
PS

If n = 4, 4 = 2 which is a natural = 112


number. 6
x = 660
Thus, n may be a natural number or x 660
∴ Correct answer is + 12 = + 12
U

an irrational number. 6 6
7. Ans. (d) = 110+12
@

Sol. Prime numbers between 61 and 89 are = 122


13. Ans. (d)
67, 71, 73, 79 and 83.
Sol. Dividend = Divisor × Quotient
∴ The sum of prime numbers between 61
+ Remainder
and 89 is 67 + 71 + 73 + 79 + 83 = 373
∴ Number = (555 + 445) × 2 × 110 + 30
8. Ans. (b) = 220000 + 30
Sol. Three consecutive odd numbers are = 220030
(2x + 1), (2x + 3) and (2x + 5) 14. Ans. (b)
Their sum= 2x + 1 + 2x + 3 + 2x + 5 Sol. 998 × 1002
= 6x + 9 = (1000 – 2) (1000 + 2)
= 3(2x+3) = 10002 – 22 [ (a–b) (a+b) = a2–b2]
which is always divisible by 3. = 1000000 – 4
= 999996
50 ✫ Quantitative Aptitude

15. Ans. (c) 20. Ans. (a)


Sol.  212×188 = (200 + 12)(200 – 12) Sol. Largest four digit number = 9999
= (200)2 – (12)2 Smallest three digit number = 100
= 40000–144 ∴ 9999 – 100 = 9899
= 39856 21. Ans. (b)
16. Ans. (c) Dividend − Remainder
Sol. Divisor =
Sol. Let the consecutive numbers be x and Quotient
x+1 59761 − 37
= = 316

TS
(x + 1) 2 – x2 = 35 189
2 2
x + 2x + 1 – x = 3 5
22. Ans. (b)
2x + 1 = 3 5 Sol. 114 x ∴ x = 114×1+21 = 135

H
2x = 35 – 1 = 34
34 1 → 21 (Taking quotient = 1)
x = = 17 When 135 is divided by 19, then the

G
2
remainder will be 2.
x + 1 = 17 + 1 19 135
= 18
∴ The next number is 18. U 23. Ans. (b)
7 → 2
O
Sol. Let the smaller number be x. Then
17. Ans. (b) larger number is 1365 + x
Sol. Let the required number be x. ∴ 1365+x = 6x + 15
H
Then, x + 10 = 2(x – 10) 5x = 1350
x = 270
= 2x – 20
_T

24. Ans. (d)


10 + 20 = 2x – x
Sol. Dividend=
∴ x = 30 Divisor × Quotient + Remainder
= 12×35+0
C

18. Ans. (a)


Sol. Largest number of 5 digits = 99999 = 420
Now, dividend = 420 and divisor = 21
PS

∴ 77)99999(1298 ∴ 99999 420


77 ∴ Correct quotient = = 20
− 53 21
229 25. Ans. (b)
154 99946
U

Sol. Let the three consecutive odd numbers


759 be (2x + 1), (2x + 3) and (2x + 5)
693 Their sum = (6x + 9) = 3 (2x + 3), which is
@

669 always divisible by 3.


616
53 26. Ans. (b)
Sol. Given that 357*25* is divisible by both
19. Ans. (c)
3 and 5. For divisibility by 5, we must
Sol. Divisiblity by 3 : If the sum of the have 0 or 5 at the unit place of the
digits of a number is divisible by 3, given number.
then the number is divisible by 3. For divisibility by 3, sum of the digits
23 * 7 = 2 + 3 + * + 7 of the given number should be divisible
= 12 + * by 3.
Let the unit place of the given number
Substitute the options in place marked*
have 5, then 3 + 5 + 7 +* + 2 + 5 + 5 =
and add digits. When we put * = 3, we 27 + *. When we put * = 6, the number
get 15 which is divisible by 3. is divisible by 3.
Unit One : Practice Session ✫ 51

∴ The missing digits in the unit’s place 33. Ans. (c)


and the thousandth place respectively Sol. On dividing 1000 by 17, the remainder
are 5 and 6. obtained is 14.
27. Ans. (c) 58
Sol. On dividing 803642 by 11, we get 17 1000
remainder 4. 986
∴ required number to be added is 11 – 4
14
=7 ∴ The required number is 14.
28. Ans. (d)

TS
34. Ans. (d)
Sol. On dividing 457 by 11, remainder Sol. On dividing 3006 by 29, the remainder
is 6. is 19
∴ required number is either 451 or 462. 103

H
So, 462 is the number which is nearest 29 3006
to 457 and exactly divisible by 11.
2987

G
29. Ans. (c) 19
Sol. The thief escaped with one diamond. So, we have to add 29 – 19 = 10 to the
Before he met the 3rd watchman, the
thief had (1 + 2) × 2 = 6 diamonds.
U number.
∴ Required number = 3006 + 10 = 3016
O
Before he met the 2nd watchman, the
35. Ans. (d)
thief had (6 + 2) × 2 = 16 diamonds.
Sol. 88)9999(113
Before he met the 1st watchman, the
H
88
thief had (16 + 2) × 2 = 36 diamonds.
119
_T

30. Ans. (b) 88


Sol. Smallest number of 5 digits is 10000. 319
On dividing 10000 by 476, we get 264
remainder 4. 55
C

∴ required number is 10000+(476 – 4) ∴ Required number = 9999–55


= 10472 = 9944
PS

36. Ans. (c)


31. Ans. (c) Sol. At first, matches played = 64
Sol.  997×997 = 9972 (i.e., 2 players in one men’s singles
= (1000 – 3)2
U

match)
= 1000000 – 6000 + 9 Total number of matches played
[ (a – b)2 = a2 – 2ab + b2] = 64 + 32 + 16 + 8 + 4 + 2 + 1
@

= 994009 = 127
32. Ans. (a) 37. Ans. (b)
Sol.The least number of five digits is 10000. Sol. By the given condition 53x – 35x =
1206
∴ 567 )10000(17 ⇒ 18x = 1206
567 1206
4330 ⇒ x = = 67
18
3969
361 38. Ans. (a)
∴ The least 5 digit number divisible by Sol. 4 x ∴z = 6×1+4 = 10
567 5 y → 2 y = 5z+3 = 5×10+3 = 53
= 10000 + (567–361) 6 z → 3 x = 4y+2 = 4×53+2 = 214
= 10000 + 206
1 4
= 10206
52 ✫ Quantitative Aptitude

39. Ans. (b) In the case of 7, we know 74 will give


Sol. A number when divided successively 1 in the unit’s digit.
by 4 and 5 leaves remainders 1 and 4 771 = (74)17 × 73 ⇒ 1 × 3
respectively. ( 73 will have 3 in the unit digit)
4 x The unit digit in the given product will
5 y → 1 be 4.
1 → 4 (3 × 6 × 3) ⇒ 54
∴ y = 5×1+4 = 9 42. Ans. (c)
∴ x = 4y + 1 = 4 × 9 + 1 = 37

TS
Sol. (1000)9 ÷ 1024
Now, 37 when divided by 5 and 4,
leaves remainders 2 and 3 respectively. (10 3 )9 10 27
= 24 = 10(27 – 24)
= 103 = 1000
40. Ans. (b) 10 24 10

H
Sol. We know any number with 1 in unit 43. Ans. (d)
digit raised to any power will result in Sol. (112 × 54)

G
4
1 in the unit digit. So let’s see which 10 104 112 × 10 4
= 112 × = 112 × 4 =
power of 7 will lead to 1 in the unit 2 2 16
digit.
72 = 49; 492 gives 1 in the unit digitU = 7 × 104 = 70000
44. Ans. (a)
O
492 = 72 × 72 = 74
So 74 will give 1 in unit digit. Sol. 217 = 200 + 17; 183 = 200 – 17
The given expression
H
Now, 7105 = (74)26 × 71
⇒ (1)26 × 7 = 1 × 7 = 7 = (217)2 + (183)2
So unit digit of 7105 will be 7. = (200 + 17)2 + (200 – 17)2 = 2 [(200)2 + (17)2]
_T

[ (a + b)2 + (a – b)2 = 2 (a2 + b2)]


41. Ans. (b) = 2 (40000 + 289)
Sol. Let’s take 365 and see which power of = 2 × 40289 = 80578
C

3 leads to 1 in unit digit.


32 = 9; 92 = 81 45. Ans. (a)
Sol. (xn – an) is completely divisible by (x + a)
PS

So 32 × 32 or 34 ⇒ 1 in the unit digit.


Now unit digit for 365 if n is even.
= (34)16 × 3 ⇒ (1)16 × 3 = 3 ∴ (17200 – 1200) is completely divisible by
In the case of 6, you will find that 6 17 + 1, i.e., 18
U

raised to any power will give 6 in If 17200 – 1200 is completely divisible by


unit’s digit. So 659 will give 6 in the 18, dividing 17200 by 18 will give the
@

unit’s digit. remainder 1.


Unit One : Practice Session ✫ 53

Practice Exercise 2
1. The H.C.F. and L.C.M. of two numbers are 9. The smallest sum of money which contains
18 and 3780 respectively. If one of them is Rs 2.50, Rs 20, Rs 1.20 and Rs 7.50 is
540, then the other number is (a) Rs 100 (b) Rs 80
(a) 142 (b) 126 (c) Rs 60 (d) Rs 40
(c) 118 (d) 112 10. Three men start together to travel the same
2. Three measuring rods are 64 cm, 80 cm, and way around a circular track of 11 km. Their

TS
96 cm in length. The least length of cloth speeds are 4, 5.5 and 8 km per hour
that can be measured exact number of times respectively. They will meet after
using anyone of the above rods is (a) 10 hours (b) 20 hours
(a) 0.96 m (b) 19.20 m (c) 22 hours (d) 28 hours

H
(c) 9.60 m (d) 96.00 m 11. There are three consecutive road crossings
3. Greatest number which when subtracted at which traffic lights change after every 48

G
from 10,000 so that the remainder may be seconds, 72 seconds and 108 seconds
divisible by 32, 36, 48 and 54 is respectively. If the lights change
(a) 9,136
(c) 8,478
(b) 9,250
(d) 8,784 U simultaneously at 8 : 20 : 00 hours, then at
what time will they again change
O
simultaneously?
4. In a school, 391 boys and 323 girls have
(a) 8 : 27 : 12 hours
been divided into the largest possible
H
(b) 8 : 27 : 24 hours
classes, so that each class of boys numbers
(c) 8 : 27 : 36 hours
the same as each class of girls. The number
(d) 8 : 27 : 48 hours
_T

of classes is
(a) 5 (b) 1 7 12. A, B and C start at the same time in the same
(c) 3 7 (d) 5 3 direction to run around a circular stadium.
A completes a round in 252 seconds, B in
C

5. The H.C.F. and L.C.M. of two numbers are


27 and 2079 respectively. If the first number 308 seconds and C in 198 seconds, all
starting from the same point. After what time
PS

is 189, the other number is


(a) 287 (b) 297 will they meet again at the starting point?
(c) 278 (d) 279 (a) 46 minutes 5 seconds
(b) 46 minutes 9 seconds
U

6. If the L.C.M. and H.C.F. of two numbers are (c) 46 mintues 12 seconds
2400 and 16 and if one number is 80, the (d) 46 minutes 16 seconds
other number is
@

(a) 460 (b) 470 13. A rectangular courtyard 3.78 metres long
(c) 480 (d) 490 and 5.25 metres wide is to be paved exactly
with square tiles, all of the same size. What
4 10 20 is the largest size of the tile which could be
7. The H.C.F. of , and is
9 21 63 used for the purpose?
2 2 (a) 24 cm (b) 23 cm
(a) (b)
63 9 (c) 22 cm (d) 21 cm
10 5 14. Find the maximum number of students
(c) (d)
63 63 among whom 456 bottles of soft drinks and
8. If the H.C.F. of 456 and 874 is 38, then the 874 pieces of chocolates can be equally
L.C.M. is distributed?
(a) 665 (b) 10844 (a) 3 8 (b) 5 1
(c) 10488 (d) 10456 (c) 5 7 (d) 6 2
54 ✫ Quantitative Aptitude

15. The greatest and the smallest four-digit 23. Six bells commence tolling together and toll
numbers which are exactly divisible by 3, 4 at intervals 2, 4, 6, 8, 10 and 12 seconds
and 5 are respectively. In 30 minutes, how many times
(a) 9999 and 1000 do they toll together?
(b) 9980 and 1010 (a) 1 4 (b) 1 5
(c) 9970 and 1010 (c) 1 6 (d) None of these
(d) 9960 and 1020 24. The smallest number which when
16. Five bells beginning together toll at intervals diminished by 3, is divisible by 21, 28, 36
4, 5, 7, 8 and 10 seconds respectively. After and 45 is

TS
what interval of time will they toll together? (a) 420 (b) 1257
(a) 4 minutes 40 seconds (c) 1260 (d) 1263
(b) 4 minutes 30 seconds 25. The greatest number less than 900, which is

H
(c) 4 minutes 20 seconds divisible by 8, 12 and 28 is
(d) 4 minutes 10 seconds (a) 542 (b) 670

G
17. The least number of square tiles required (c) 766 (d) 840
for a terrace 23 m 80 cm long and 8 m 16 cm 26. The product of two co-prime numbers is 117.
wide is
(a) 420
(c) 340
(b) 410
(d) 310 U Their L.C.M. should be
(a) 1 (b) 117
O
(c) 121 (d) None of these
18. Three different containers contain milk of 27. Find the greatest number that will divide
different quality like Mother Dairy, Milma
H
43, 91 and 183 so as to leave the same
and Parag respectively whose measure- remainder in each case.
ments are 280 litres, 315 litres and 385 litres (a) 4 (b) 7
_T

respectively. What biggest measure should (c) 9 (d) 1 3


be used to measure all different qualities an
28. The greatest and smallest five-digit
exact number of times?
numbers which when divided by 6, 8 and 9
C

(a) 1 5 (b) 2 0
leave a remainder 5 in each case are
(c) 3 0 (d) 3 5
(a) 99999 and 10000
PS

19. The L.C.M. of 2 3 × 3 2 ×5, 2 2 ×3 3 ×5 2 and (b) 99983 and 10010


22 ×34×52 is (c) 99941 and 10013
(a) 2 3×3 3×5 2 (b) 2 2 ×3 2×5 (d) 99925 and 10025
3 4
(d) 2 3×3 4×5 2
U

(c) 2 ×3 ×5
29. Find the smallest number which when
20. The sum of two numbers is 1000 and their decreased by 8 is exactly divisible by 15, 21
@

L.C.M. is 8919. The two numbers are and 30.


(a) 993 and 7 (b) 989 and 11 (a) 142 (b) 193
(c) 991 and 9 (d) 987 and 13 (c) 218 (d) 421
21. The H.C.F. and L.C.M. of two numbers are 30. Find the numbers between 100 and 200 such
12 and 72 respectively. If the sum of the two that when they are divided by 4, 5 and 6,
numbers is 60, then one of the two numbers they leave no remainder.
will be (a) 120 and 180 (b) 130 and 180
(a) 1 2 (b) 2 4 (c) 120 and 160 (d) 130 and 160
(c) 6 0 (d) 7 2
31. Which would be the greatest number to
22. The greatest 4-digit number exactly divisible divide 148, 246 and 623 leaving remainders
by 66 is 4, 6, and 11 respectively?
(a) 9966 (b) 9933 (a) 1 2 (b) 1 4
(c) 6666 (d) 9999 (c) 2 4 (d) None of the above
Unit One : Practice Session ✫ 55

32. What is the H.C.F. of 11, 0.121 and 0.1331? 37. Two numbers, both greater than 29, have
(a) 11 (b) 0.11 H.C.F. of 29 and L.C.M. of 4147. What would
(c) 0.011 (d) 0.0011 be the sum of the numbers?
1095 (a) 686 (b) 696
33. What is the simplest expression of ?
1168 (c) 756 (d) 796
25 17
(a) (b) 38. What would be the least number which,
26 26
15 13 when divided by 48, 60, 72, 108 and 140
(c) (d) leaves 38, 50, 62, 98 and 130 as remainders,
16 16

TS
respectively?
34. What is the H.C.F. of 4 × 7 × 3125;
(a) 15110 (b) 15100
8 × 9 × 25 × 7; and 16 × 81 × 5 × 11 × 49?
(a) 120 (b) 1860 (c) 16110 (d) None of these

H
(c) 180 (d) 540 39. The L.C.M. of two numbers is 45 times their
35. Three numbers are co-prime to each other. H.C.F. If one of the numbers is 125, and the

G
The product of the first two is 551, and that sum of the H.C.F. and L.C.M. is 1150, what
of the second and third is 1073. What is the is the other number?
sum of the three numbers? (a) 225 (b) 235
(a) 8 9
(c) 7 9
(b) 8 5
(d) 8 1 U (c) 125 (d) 2215
O
40. What is the greatest number of three digits
36. Two numbers add up to 528 and their H.C.F. which, when divided by 6, 9 and 12, leaves
is 33. How many pairs of numbers satisfy a remainder of 3 in each case?
H
the given conditions?
(a) 975 (b) 996
(a) Two (b) Three
(c) 939 (d) 903
_T

(c) Four (d) None of these

Answers with Solutions


C

1. Ans. (b) 5. Ans. (b) L.C.M. × H.C.F.


Sol. Product of two numbers Sol. Second number =
PS

First number
= their H.C.F. × their L.C.M.
∴ 540 × second number = 18×3780 2079 × 27
= = 297
18 × 3780 189
U

The second number = 6. Ans. (c)


540
Sol. Product of two numbers
= 126
@

= their H.C.F. × their L.C.M.


2. Ans. (c) 2400 ×16
Sol. Find the L.C.M. of the given lengths ∴ Second number =
80
∴ L.C.M. of 64, 80 and 96 is 960 cm = 480
= 9.60m 7. Ans. (a)
3. Ans. (a) Sol. H.C.F. of fractions
Sol. L.C.M. of 32, 36, 48, 54 is 864. H.C.F. of Numerators
∴ The required number =
L.C.M. of Denominators
= 10000 – 864
= 9136 H.C.F. of 4, 10 and 20
= L.C.M. of 9, 21 and 63
4. Ans. (b)
Sol. Number of classes H.C.F. of 4, 10, 20
= H.C.F. of 391 and 323 = H.C.F. of [22; 2 × 5; 22 × 5]
= 17 = 2
56 ✫ Quantitative Aptitude

L.C.M. of 9, 21 and 63 = 63 14. Ans. (a)


3 9, 21, 63 Sol. The given items have to be divided
3 3, 7, 21 among the students equally.
7 1, 7, 7 ∴ H.C.F. of 456 and 874 is to be found.
1, 1, 1 2
∴ H.C.F. of fraction = 456) 874 ( 1
63
456
8. Ans. (c)
Product of the numbers 418) 456 (1
Sol. L.C.M. =

TS
H.C.F. 418
456 × 874
= 38) 418(11
38
418
= 10488

H
X
9. Ans. (c) H.C.F. = 38
Sol. L.C.M. of 2.5, 20, 1.2, 7.5

G
15. Ans. (d)
= (L.C.M. of 25, 200, 12 and 75) × 0.1
Sol. The L.C.M. of 3, 4 and 5 = 60
= 600 × 0.1 The greatest 4-digit number
= Rs 60
10. Ans. (c) U = 9999
O
Sol. Time taken by them to complete one
revolution
H
11 11 11
= , and hours
45.5 8
_T

112 11
= , and
4 1 8
∴ The required number
11 2 11
C

L.C.M. of , and = 9999 – 39 = 9960


4 1 8 The smallest 4-digit number = 1000
L.C.M. of 11, 2, 11
PS

= H.C.F. of 4, 1, 8
22
= = 22 hours
1
U

∴ They will meet after 22 hours.


11. Ans. (a) ⇒ The required number
@

Sol. L.C.M. of 48, 72, 108 is 432. = 1000 – 40 + 60 = 1020


∴ Light will change after every 432 16. Ans. (a)
seconds, i.e., after every 7 minutes Sol. L.C.M. of 4, 5, 7, 8
and 12 seconds. and 10
= 2×2×5×7×2
12. Ans. (c) = 280 seconds
Sol. L.C.M. of 252, 308 and 198 is 2772. = 4 minutes 40 seconds
So, A, B and C will again meet at the 17. Ans. (a)
starting point in 2772 seconds, i.e., Sol. Number of tiles will be the least when
46 minutes 12 seconds.
the size of one tile is maximum. Hence
13. Ans. (d) H.C.F. of 2380 and 816 (the length has
Sol. Largest size of the tile = H.C.F. of 378 to be converted into cm) will be the
and 525 = 21 cm maximum size of the square tile.
Unit One : Practice Session ✫ 57

∴ H.C.F. of 2380 and 816 = 68cm 22. Ans. (a)


816 × 2380 Sol.
∴ The number of tiles = 9999 is the greatest
68 × 68
4-digit number
= 420 Divide 9999 by 66
18. Ans. (d) Remainder is 33
9999–33 = 9966 which is the
Sol. The required answer greatest number divisible
= H.C.F. of 280, 315 and 385 by 66

TS
= 35
23. Ans. (c)
315) 385( 1
Sol. L.C.M. of 2, 4, 6, 8, 10 and 12
315
= 2×2×3×2×5

H
70) 315(4
= 120
280

G
35) 280(8
280
X
(Note: First find the H.C.F. of any U Bells will toll simultaneously after
O
two numbers, say 280 and 315. That every 120 seconds.
is 35. Then find the H.C.F. of 35 and ∴ In 30 minutes, they will toll together
H
385. Thus, we get the H.C.F. of the 30 × 60
three numbers.) = = 15 times
120
_T

19. Ans. (d) In 30 minutes, they would have


Sol. L.C.M. is the product of terms with tolled together 15 + 1 (the very first
the highest powers of all the factors time they tolled together) = 16 times.
C

of the number. 24. Ans. (d)


20. Ans. (c) Sol. L.C.M. of (21, 28, 36, 45) + 3
PS

Sol. Given that L.C.M. of two numbers is = 1260+3


8919. = 1263
Here L.C.M. of 991 and 9 25. Ans. (d)
Sol. L.C.M. of 8, 12 and 28 is 168.
U

= 991×9 = 8919
Any multiple of 168 will be exactly
21. Ans. (b)
divisible by each of the numbers 8,
@

Sol. Sum of two numbers = 60


12 and 28. But since the required
Let one number be 24, then
the other number is 60–24 = 36 number is less than 900, it is 168×5
H.C.F. of 24 and 36 = 840.
24 = 2×2×2×3 26. Ans. (b)
36 = 2×2×9 = 2×2×3×3 Sol. H.C.F. of co-prime numbers is 1.
H.C.F. is 22 × 3 = 12 117
L.C.M. of 24 and 36 So, L.C.M. = = 117
1
27. Ans. (a)
Sol. Required number is the H.C.F. of
(91 – 43), (183 – 91) and (183 – 43)
= H.C.F. of 48, 92 and 140
L.C.M. is 2×2×3×2×3 = 72 = 4
58 ✫ Quantitative Aptitude

28. Ans. (c) 31. Ans. (a)


Sol. L.C.M. of 6, 8 and 9 Sol. The required number will be the
= 2 × 3× 4 ×3 = 7 2 H.C.F. of the numbers minus their
Dividing 99999 respective remainders.
(the greatest 5-digit no.) So the greatest number to divide the
by 72, we get a remainder of 63. given numbers leaving the given
The greatest 5-digit number remainders is
divisible by 6, 8 and 9 H.C.F. of (148 – 4), (246 – 6) and
623 – 11)

TS
H.C.F. of 144, 240 and 612
2 144 2 240 2 612
= 99999 – 63 2 72 2 120 2 306
= 99936

H
2 36 2 60 3 153
As the required number leaves a
2 18 2 30 3 51
remainder 5, the number

G
3 9 3 15 17
3 5

= 99936+5 U Here 144 = 2 × 2 × 2 × 2 × 3 × 3


240 = 2 × 2 × 2 × 2 × 3 × 5
O
= 99941
The smallest 5-digit number divisible 612 = 2 × 2 × 3 × 3 × 17
H
by6, 8 and 9 So, H.C.F. is the product of the
= 10000 – 64 + 72 = 10008 maximum common prime factors
∴ The required number (encircled) = 2 × 2 × 3 = 12
_T

= 10008 + 5 = 10013 The greatest number to divide 148,


29. Ans. (c) 246 and 623 leaving the given
Sol. The smallest number divisible by 15, remainders is 12.
C

21 and 30 32. Ans. (d)


= L.C.M. of 15, 21 and 30 Sol. Find the H.C.F. of the numbers
PS

= 3×5×7×2 without decimal.


= 210
H.C.F. of 11, 121 and 1331 is 11.
∴ The required number
Now the decimal places have to be
U

= 210 + 8
= 218 marked off. The maximum decimal
[When 218 is decreased by 8, it places = 4 (in 0.1331).
@

becomes 210. Only then it is the So the H.C.F. = 0.0011.


smallest common multiple of 15, 21 33. Ans. (c)
and 30]. Sol. We need to find the highest common
30. Ans. (a) factor.
1095) 1168 (1
Sol. The numbers leave no remainder.
1095
⇒ They are exactly divisible by 4, 5
73)1095(15
and 6
73
∴ L.C.M. of 4, 5 and 6
= 2×2×3×5 365
365
= 60
The multiples of 60 between 100 X
and 200 are 120 and 180. H.C.F. of 1095 and 1168 is 73.
Unit One : Practice Session ✫ 59

1095 As the H.C.F. is 29, let’s assume the


Now the simplest form of is two numbers to be 29a and 29b
1168
1095 ÷ 73 15 ∴ 29a × 29b = 29 × 4147
=
1168 ÷ 73 16 29 × 4147
⇒ ab = = 143
34. Ans. (c) 29 × 29
Sol. Convert the multiples into numbers Co-primes of 143 are (1, 143) and
with exponents. (11, 13), and the numbers could be
4 × 27 × 3125 = 22 × 33 × 55 (29 × 1, 29 × 143) or (29 × 11, 29 × 13)
8 × 9 × 25 × 7 = 23 × 32 × 52 × 7

TS
As we are told both numbers are
16 × 81 × 5 × 11 × 49 = 24 × 34 × 5 × 11 × 72 ‘greater’ than 29, we may ignore the
H.C.F. = 22 × 32 × 5 = 180 set with (29 × 1). The required sum
35. Ans. (b) is (29 × 11) + (29 × 13)

H
Sol. As the numbers are co-prime, only = 319 + 377 = 696.
1 can be the common factor. Also, 38. Ans. (a)

G
the two given products have one Sol. A look at the remainders tells you that
number in common. the difference in each case of divisor
Factorising 551 we get 19 × 29.
One of these terms is common.
As 19 does not fully divide into the U and remainder is the same, i.e.,
(48 – 38) = 10, (60 – 50) = 10,
(72 – 62) = 10, (108 – 98) = 10, and
O
other number, 1073, but 29 does, 29 (140 – 130) = 10.
is the middle term. So the required number is the (L.C.M.
H
551 of 48, 60, 72, 108 and 140) – 10
∴ The first number is = 19 = 15120 – 10 = 15110.
29
_T

1073 39. Ans. (a)


and the second number is = 37
29 Sol. Let the unknown number be x.
The sum of the three numbers is Given L.C.M = 45 × H.C.F. and
19 + 37 + 29 = 85
C

L.C.M. + H.C.F. = 1150


36. Ans. (c) ∴ H.C.F. + 45 × H.C.F. = 1150
⇒ 46 H.C.F. = 1150
PS

Sol. Let the two numbers be 33a and 33b.


33a + 33b = 528 1150
⇒ H.C.F. =
⇒ 33 (a + b) = 528 46
U

528 Also, 125x = H.C.F. × L.C.M.


⇒ a + b = = 16
33 1150 1150 1
Now the co-primes of 16 are (1, 15), ⇒ x = × 45 × ×
@

(3, 13), (5, 11) and (7, 9). 46 46 125


∴ The two numbers could be any of 1
the four sets: (33 × 1, 33 × 15), ⇒ x = 25 × 45 × 25 ×
125
(33 × 3, 33 × 13), (33 × 5, 33 × 11) or ⇒ x = 225
(33 × 7, 33 × 9). The other number is 225.
So there could be four possible pairs
of numbers to satisfy the given 40. Ans. (a)
conditions. Sol. The greatest three-digit number is
999.
37. Ans. (b) Find the L.C.M. of the given divisors.
Sol. Product of two numbers = Product L.C.M. of 6, 9 and 12 = 36
of their H.C.F. and L.C.M. 999 ÷ 36 = 27 and remainder is 27.
Here the product of the two numbers The required number is
= 29 × 4147 999 – 27 + 3 = 975.
60 ✫ Quantitative Aptitude

Practice Exercise 3
1. Which one of the following sets of fractions 7. Which of the following is correct?
is in the correct sequence of ascending order 5 3 2 8 5 2 3 8
of values? (a) > > > (b) > > >
7 9 3 11 7 3 9 11
1 5 4 3 5 3
(a) − , , − (b) − , − , 8 5 2 3 8 2 3 5
2 6 9 7 6 5 (c) > > > (d) > > >
1 4 5 4 5 1 11 7 3 9 11 3 9 7
(c) − , − , (d) − , ,

TS
2 9 6 9 6 6 3 7 8
8. The value of + + is
2. Among the following the largest fraction is 1 10 100
10 (a) 3.078 (b) 3.78
(a) (b) 0.91 (c) 3.87 (d) 3.087

H
11
7.3 8 .3 9. Value of 2.5 ÷ 0.0005 is
(a) 5 (b) 5 0

G
(c) (d)
8 9
(c) 500 (d) 5000
3. A decimal number has 16 decimal places.
The number of decimal places in the square
root of this number will be U 10. The value of (2 + 0.2 + 0.02 + 0.002) is
(a) 2.002
(c) 2.006
(b) 2.022
(d) 2.222
O
(a) 2 (b) 4
11. The value of
(c) 8 (d) 1 6
 0.1× 0.1× 0.1 + 0.02 × 0.02× 0.02 
H
4. Which of the following is the smallest   is
 0.2× 0.2 × 0.2 + 0.04 × 0.04 × 0.04 
fraction?
(a) 1.25 (b) 0.125
_T

1 1 9 500
, , , (c) 0.0125 (d) 0.00125
10 100 1000 10, 000
12. Given 168 × 32 = 5376, then 5.376 ÷ 16.8 is
1 1 equal to
C

(a) (b)
10 100 (a) 0.32 (b) 320
9 500 (c) 3 2 (d) 3.2
PS

(c) (d) 13. The sum of 2.75 and 3.78 is


1000 10000
(a) 1.03 (b) 1.53
5. What is the difference between the biggest
(c) 4.53 (d) 5.53
2 3 4
U

and the smallest fraction among , , 14. Write the ascending order of the following
5 3 4 5 rational numbers.
@

and ?
6 −7 5 2
, ,
1 1 10 −8 −3
(a) (b)
30 20 2 5 −7 5 −7 2
1 1 (a) < < (b) < <
(c) (d) −3 −8 10 −8 10 −3
12 6 −7 2 5 −7 5 2
6. Which of the following is correct? (c) < < (d) < <
10 −3 −8 10 −8 −3
5 9 11 14
I. < II. > 15. The fraction for the recurring decimal
7 13 13 17 0.535353 is
2 5
III. < (a)
26
(b)
28
9 6 53 53
(a) Only I (b) Only II 53 27
(c) (d)
(c) I and III (d) II and III 99 53
Unit One : Practice Session ✫ 61

16. What is the least fraction which when added 24. What fraction must be subtracted from the
29 15 1 1 1
to or subtracted from + will make the sum of and to have an average of of
4 6 12
12 16
result a whole number? all three fractions?
1 1
17 31 (a) (b)
(a) (b) 2 3
48 48 1
1
31 21 (c) (d)
(c) (d) 4 6
38 38
1 1 1

TS
25. If + added to of a certain sum is 7000,
10 12 ? 6 9 2
17. × × = 16 then the sum is
3 5 4
(a) 6 (b) 2 (a) 7000 (b) 9000

H
(c) 8 (d) 4 (c) 10,000 (d) 11,000
(25× 7) + 19× 8
1 7  3 2  2 

G
18. +  − + −  = ? 26. = ........
7 9  9 9  9  243÷ (10× 9 − 9)
(a) 109 (b) 103
(a)
1
9
2
(b)
3
7
1 U (c) 107
27. Square of 679 is
(d) 2.5
O
(c) (d)
9 7 (a) 429721 (b) 493651
1 (c) 440781 (d) 461041
H
÷ 4 + 20
19. 2 =? 28. Square of 3972 is
1
× 4 + 20 (a) 15776784 (b) 24327882
_T

2 (c) 17932154 (d) 26734032


81 3
(a) (b) 2 29. Square of the number 3658 is
88 11
C

161 (a) 13380944 (b) 12280944


(c) (d) 1
176 (c) 13380964 (d) 12280964
PS

20. 0.07 × 0.008 × 0.2 =? 30 . Square of the number 58 is


(a) 0.000112 (b) 0.00056
(a) 2704 (b) 3254
(c) 0.00112 (d) 0.0112
(c) 3364 (d) 4264
U

21. (0.6 + 0.7+ 0.8 + 0.3) gives


31. 433 is equal to
3 33
@

(a) 2 (b) 2 (a) 79507 (b) 83267


10 100 (c) 34277 (d) 93567
2
(c) 2 (d) 2.35 32. 973 is equal to
3
22. 80.40 ÷ 20 – (– 4.2) =? (a) 432164 (b) 546213
(a) 497.8 (b) 5.786 (c) 912673 (d) 739429
(c) 947.00 (d) 8.22 33. 423 is equal to
23. It being given that 15 = 3.88 , the best (a) 64398 (b) 74088
(c) 73118 (d) 92468
5
approximation to is 34. 642 + 182 – 152 is equal to
3
(a) 0.43 (b) 1.89 (a) 4195 (b) 6192
(c) 1.29 (d) 1.63 (c) 7320 (d) 8032
62 ✫ Quantitative Aptitude

35. 722 – (452+(202 + 92)) is equal to 16 1


38. The value of + is
(a) 1340 (b) 2678 36 4
2 1
(c) 3926 (d) 4813 (a) (b)
5 3
36. Square root of 117649 is 5 7
(c) (d)
(a) 243 (b) 343 6 6
(c) 247 (d) 347 39. The square root of 0.4 is
9 (a) 0.6 (b) 0.7
37. Square root of 4 is

TS
25 (c) 0.8 (d) 0.9
1 2
(a) 2 (b) 2 40. Cube of 93 is
5 5
3 (a) 804357 (b) 704927
(c) 2

H
(d) None of these (c) 643217 (d) 913267
5

G
Answers with Solutions
1. Ans. (b)
2. Ans. (d)
U 5. Ans. (d)

Sol.
2
= 0.666... ,
3
= 0.75
O
3 4
10
Sol. = 0.9090 4 5
11 = 0.8 , = 0.8333...
H
0.91 = 0.91 5 6
7.3 ∴ 0.833 > 0.8 > 0.75 > 0.66
_T

= 0.9125 5 4 3 2
8 > > >
so
8.3 6 5 4 3
= 0.922 5 2 1
9 ∴ Required difference is – =
6 3 6
C

3. Ans. (c) 6. Ans. (d)


Sol. For every pair of decimal places, 5 9
PS

there is one decimal place in the Sol. and = 5 × 13 > 9 × 7


7 13
square root; for example,
11 14
and = 11 × 17 > 14 × 13
0.04 = .2 13 17
U

0.0001 = .01 2 5
and =2 × 6 < 5 × 9
1.21 = 1.1 9 6
@

7. Ans. (c)
So for 16 decimal places, there will 5 3 2
Sol. The given fractions are , ,
be 8 decimal places in the square 7 9 3
root. 8
and
11
4. Ans. (c)
L.C.M. of 7, 9, 3 and 11 is 693.
1 1 9 500
Sol. , , ,
10 100 1000 10000 5 5 × 99 495
∴ = =
Make the denominators the same: 7 7 × 99 693
1000 100 90 500 3 3 × 77 231
, , , = =
10000 10000 10000 10000 9 9 × 77 693
90 9
Clearly, or is the 2 2 × 231 462
10000 1000 = =
smallest fraction. 3 3 × 231 693
Unit One : Practice Session ✫ 63

8 8 × 63 504 14. Ans. (c)


= = –7
11 11× 63 693
Sol. = –0.7
8 5 2 3 10
∴ > > > 5
11 7 3 9
= –0.625
When denominators of fractions are –8
same, the fraction with the largest 2
numerator is considered the largest. and = –0.666
–3
8. Ans. (b) ∴ –0.7 < –0.666 < –0.625

TS
3 7 8 –7 2 5
Sol. + + ⇒ < <
1 10 100 10 –3 –8
= 3 + 0.7 + 0.08 15. Ans. (c) 53
Sol. 0.5353 .... = 0.53 =

H
= 3.78 99
16. Ans. (a)
9. Ans. (d)

G
2.5 29 15 29 × 4 + 15 × 3
Sol. + =
Sol. 2.5 ÷ 0.0005 = 12 16 48
0.0005

=
25000
5
= 5000
U =
116 + 45

161
48
17
O
10. Ans. (d) = = 3
48 48
Sol. 2 + 0.2 + 0.02 + 0.002 = 2.222 17
H
So will have to be subtracted.
11. Ans. (b) 48
0.1× 0.1× 0.1 + 0.02 × 0.02 × 0.02 17. Ans. (c)
_T

Sol. 10 12 x
0.2 × 0.2 × 0.2 + 0.04 × 0.04 × 0.04 Sol. × × = 16
3 5 4
0.001 + 0.000008
= 16
C

0.008 + 0.000064  x= =8
2
0.001008 1008
PS

= = 18. Ans. (d)


0.008064 8064
1 1 7 3 2 2
= = 0.125 Sol. + – + –
8 7 9 9 9 9
U

12. Ans. (a) 1 7 5 2


= + – –
Sol. 168 × 32 = 5376 7 9 9 9
@

⇒ 5376 ÷ 168 = 32 1 1
= +0 =
5.376 7 7
Now, 5.376 ÷ 16.8 = 19. Ans. (c)
16.8
5376 32 Sol. Given expression
= = 1 1 1
168 × 100 100 × + 20 + 20
= 0.32 = 2 4 = 8
13. Ans. (c) 1 22
× 4 + 20
Sol. 2.75 + 3.78 = (2 + 0.75) + (3 + 0.78) 2
= 5 + 1.53 1 + 160 161
= =
= 5 + 1 + 0.53 8 × 22 176
= 4 + 0.53 20. Ans. (a)
Sol. 0.07 × 0.008 × 0.2
= 4.53 = 0.000112
64 ✫ Quantitative Aptitude

21. Ans. (c) 28. Ans. (a)


6 7 8 3 Sol. (3972)2 = 3 2/2(3×9)/2(3×7)+92/
Sol. Given expression = + + +
9 9 9 9 2(3×2)+2(9×7)/2(9×2)+72/
24 2 2(7×2)/22
= = 2
9 3 = 9/54/123/138/85/28/4
22. Ans. (d) 80.40
+ 4.2 = 15/ 67/ 13 7/14 6/8 7/2 8/4
Sol. Given expression =
20 = 15776784 146/ 8 7/ 28/4
= 8.22

TS
= 15776784
23. Ans. (c)
29. Ans. (c)
5 5×3 Sol. (3658)2 = ?
Sol. =
3 3×3

H
Using shortcut method
15 3.88
= = I → (8)2 = 64 ⇒ 64
3

G
9
II → 2 × (5 × 8) + 6 = 86 ⇒ 86
= 1.29
III → 2 × (6 × 8) + 52 + 8 = 129 ⇒ 129
24. Ans. (d)
1 1 1 U IV → 2 × (3×8) + 2 (6×5) + 12 = 120 ⇒ 120
V → 2 × (3 × 5) + 62 + 12 = 78 ⇒ 78
O
Sol. + – x = 3×
4 6 12 VI → 2 × (3 × 6) + 7 = 43 ⇒ 43
1 1 1 VII → 32 + 4 = 13
H
∴ x = + –
4 6 4 Hence, (3658)2 = 13380964
1
= 30. Ans. (c)
_T

6
25. Ans. (b) Sol. (58)2 = ?
1 1 1 3+2+9 = (5)2 + 8/(8)2
Sol. + + =
6 9 2 18 = 25 + 8/64
C

14 7 = 33/64
= =
PS

18 9 = 3364
7 31. Ans. (a)
of x = 7000
9 Sol. 433 = 64 48 36 27
9
x = 7000 ×
U

= 9000 [Ratio between 96 72


7
26. Ans. (a) 4 and 3 = 4 : 3] 7 9 1 5 5 1 1 0 2
7
@

25 × 7 + 19 × 8 175 + 152 = 79507


Sol. 243 ÷ (10 × 9 – 9) = 243 ÷ (90 – 9) 32. Ans. (c)
175 + 152 Sol. 973 = 729 567 441 343
= [Ratio between 1134 882
243 ÷ 81
327 9 and 7 = 9 : 7] 912 183 6 135
7 34
3
= = 109
3 = 912673
27. Ans. (d) 33. Ans. (b)
Sol. (679)2 = 6 2/2(6×7)/2(6×9)+72 Sol. 423 = 64 32 16 8
/2(7×9)/92 [Ratio between 64 32
= 36/84/157/126/81
= 46/ 101 / 170/ 134/8 1 4 and 2 = 2 : 1] 74 10
0 4
8 8
= 461041 = 74088
Unit One : Practice Session ✫ 65

34. Ans. (a) 37. Ans. (d)


Sol. 642+182–152 9 109
Sol. 4 =
= [62/2(6×4) / 4 2]+ [12 / 2(1×8)/ 25 25
82] – [1(1+1) / 25] 109 109
⇒ =
= [36/48/16]+[1/16/64]–[2/25] 25 5
= [40/49/ 16]+[3/ 22/ 6 4]–225
38. Ans. (c)
= 4096 + 324 – 225
= 4195 16 1 16 + 9 25 5
Sol. + = = =

TS
35. Ans. (b) 36 4 36 36 6
Sol. 722 – [452 + (202 + 92)]
39. Ans. (a)
=72/2(7×2)/22–[4(5)/25+(400+81]

H
= 49/28/4–(2025 + 481) 4 2
Sol. 0.4 = = = 0.666...
= 51/ 28/4 – 2506 9 3

G
= 5184 – 2506 = 0.6
= 2678
40.Ans. (a)
36. Ans. (b)
Sol. U Sol. (9)3 9 3 ×
3
9
3 3
93 × ×
9 9
3 3 3
93 × × ×
9 9 9
O
3 117649 343
9  729 243 81 27
64 276 486 162
H
256
804 3 5 7
683 2049 75 24 2
∴ 117649 = 343
_T

2049 3
(93) = 804357
×
C
PS
U
@
66 ✫ Quantitative Aptitude

Practice Exercise 4
1. 48.95 – 32.006 is equal to (a) 11 (b) 1.1
(a) 16.089 (b) 16.35
(c) 11 (d) 0.11
(c) 16.89 (d) 16.944
2. 892.7 – 573.07 – 95.007 is equal to 9.5 × 0.085
11. is equal to
(a) 224.623 (b) 224.777 0.0017 × 0.19
(c) 233.523 (d) 414.637 (a) 2 5 (b) 250
(c) 2500 (d) 25000

TS
3. 3 × 0.3 × 0.03 × 0.003 × 30 is equal to
5
(a) 0.243 (b) 0.0243 12. If 15 = 3.88, then the value of is
(c) 0.00243 (d) 0.000243 3
(a) 1.2933 (b) 1.2929

H
2 3 2 (c) 1.2393 (d) 1.299
4. 2 + 1 – 2 is equal to
7 5 5
17 17 0.324 × 0.081 × 4.624

G
(a) 1 (b) 1 13. is equal to
175 35 1.5625 × 0.0289 × 72.9 × 64
17 17
(c) 3 3 (a) 0.024 (b) 0.24
3
175
2 5
(d)

5. 7 + 2 – 6 is equal to
35
U (c) 2.4 (d) 2 4
O
14. Which of the following is the correct
5 3 6
arrangement of the given fractions in
11 7
(a) 3 (b) 2 ascending order?
H
15 45
13 1 5 10 15 20 25
, , , ,
(c) 3 (d) 1 7 11 17 21 27
_T

30 90
20 25 10 15 5
6. 0.777777 ÷ 0.011 is equal to (a) < < < <
(a) 70.707 (b) 77.07 21 27 11 17 7
(c) 0.70707 (d) None of these 25 20 15 10 5
C

(b) < < < <


27 21 17 11 7
7 7
×
PS

20 10 25 15 5
7. What is the value of 9 9 = ? (c) < < < <
7 7 7 21 11 27 17 7
+ +
9 9 9 5 15 10 25 20
7 7 (d) < < < <
7 17 11 27 21
U

(a) (b)
27 18 9 2 8 5
49 7 15. If the fractions , , , are arranged
(c) (d) 13 3 11 7
@

81 81 in ascending order, then the correct sequence


8. 832.58 – 242.31 = 779.84 – ? is
(a) 295.05 (b) 179.57 5 8 2 9 2 8 5 9
(c) 199.57 (d) None of these (a) , , , (b) , , ,
7 11 3 13 3 11 7 13
(0.01) + (0.22)2 + (0.333)2
2
2 9 5 8 9 2 8 5
9. The value of is (c) , , , (d) , , ,
(0.001)2 + (0.022)2 + (0.0333)2 3 13 7 11 13 3 11 7
1
(a) (b) 1 0 1 2 3 1
10 16. How much more is of than of ?
2 3 4 3
(c) 100 (d) 1000
1 1
10. The expression (a) (b)
4 3
0.85 ×(0.105 + 0.024 – 0.008) 1 7
simplifies to (c) (d)
0.022 × 0.25 ×1.7 12 12
Unit One : Practice Session ✫ 67

17. (0.6 + 0.7 + 0.8 + 0.3) × 9000 is equal to 28. If 12276 ÷155 = 79.2 the value of
(a) 21600 (b) 23500 122.76 ÷ 15.5 = ?
(c) 24000 (d) 21000 (a) 7.092 (b) 7.92
1 1 (c) 79.02 (d) 79.2
18. If = 0.0276 , then is equal to
36.18 0.0003618 5.7 × 5.7 × 5.7 + 2.3 × 2.3 × 2.3
29. =?
(a) 2763.95 (b) 2760 5.7 × 5.7 + 2.3 × 2.3 − 5.7 × 2.3
(c) 276.39 (d) 27.603 (a) 2.3 (b) 3.4
(c) 5.7 (d) 8.0

TS
19. 3.87 − 2.59 is equal to
30. The simplification of
(a) 1.28 (b) 1.27
(c) 1.2 (d) 1.20 (0.87)3 + (0.13)3
(0.87)2 + (0.13)2 − (0.87) × (0.13)

H
20. The value of 2.136 is
11 11 yields the result
(a) 4 (b) 2 (a) 0.13 (b) 0.74

G
90 99
3 (c) 0.87 (d) 1
(c) 2 (d) None of these
1.04× 1.04 + 1.04× 0.04 + 0.04× 0.04
22
21. If a * b =
ab
a+b
, find 3 * (3* – 1). U 31.
1.04× 1.04× 1.04 − 0.04× 0.04× 0.04
is
O
(a) – 2 (b) 2 (a) 0.001 (b) 0.1
(c) – 3 (d) 3 (c) 1 (d) 0.01
× − ×
H
32. 0.778 0.778 0.222 0.222 = ?
? 54 0.556
22. = (a) 3 (b) 1
_T

169 39
(c) 2 (d) 4
(a) 108 (b) 324
(c) 2916 (d) 4800 (0.82)2 + (0.284)2 + (0.795)2
33. =?
(8.2)2 + (2.84)2 + (7.95)2
C

1.21× 0.9
23. The value of is (a) 100 (b) 0.01
1.1× 0.11
PS

(a) 2 (b) 3 (c) 1 0 (d) 0.1


(c) 9 (d) 11 34. Find the value of
1
24. The value of ( 8) 3 is 0.05 × 0.05× 0.05 + 0.04 × 0.04 × 0.04
U

(a) 2 (b) 4 0.05 × 0.05 − 0.05× 0.04 + 0.04 × 0.04


(c) 2 (d) 8 (a) 0.0002 (b) 0.09
@

(c) 0.009 (d) 0.41


x 21
25. If = , find the value of x. 1 1 1
144 36 35. 2 + 3 + 4 = ?
2 3 4
(a) 1296 (b) 441 1 5
(c) 196 (d) 4 9 (a) 11 (b) 5
12 6
? 1 1
26. = 0.02 (c) 9 (d) 10
200 3 12
(a) 0.4 (b) 4
(c) 1 6 (d) 1.6 36. If 15625 = 125, then

27. 0.0009 ÷ 0.01 = ? 15625 + 156.25 + 1.5625 = ?


(a) 3 (b) 0.3 (a) 1.3875 (b) 13.875
1
(c) (d) None of these (c) 138.75 (d) 156.25
3
68 ✫ Quantitative Aptitude

37. A number of men went to a hotel and each 42. The largest number in the sequence 1, 21/2,
spent as many rupees as there were men. If 31/3, 41/4 is
the money spent was Rs 15,625, find the (a) 1 (b) 2 1/2
number of men. (c) 3 1/3 (d) 4 1/4
(a) 115 (b) 125
(c) 135 (d) 130 2 +1
43. =?
38. The value of 3
0.000064 is 2 −1
(a) 0.1 (b) 0.2 (a) 0.732 (b) 1.3142
(c) 0.4 (d) 0.6 (c) 2.4142 (d) 0.3652

TS
3
 −8 4 44. The greatest number among the following
39. The value of  3 × 16  is equal to is
 625× 9−4 
  (a) 6 3 5 (b) 8 3 2

H
4 6
(a) (b)
35 75 (c) 2 3 130 (d) 3
900
8 9

G
(c) (d) x y ab
125 145 45. If a = and b = then is equal
40. The largest 3-digit number which is a perfect x+y x−y a+b
square is
(a) 999 (b) 998 U to
x
2
O
y
(c) 990 (d) 961 (a) x + y (b)
x+y
41. The least number to be subtracted from 1298
H
to make it a perfect square is xy x2 + y2
(a) 2 (b) 8 (c) (d)
x + y2
2
xy
_T

(c) 1 4 (d) 2 0

Answers with Solutions


C

1. Ans. (d) 16 8 12 16 × 5 + 8 × 7 – 12 × 7
= + – =
Sol. 48.95 – 32.006 7 5 5 35
PS

= 48.950 – 32.006 80 + 56 – 84 52
= 16.944 = =
35 35
2. Ans. (a) 17
= 1
U

Sol. 892.7 – 573.07 – 95.007 35


= 892.700 – 573.070 – 95.007
= 224.623 Alternative Method
@

Evaluate the whole numbers and fractions


3. Ans. (c) separately and then together
Sol. 3 × 0.3 × 0.03 × 0.003 × 30 So 2 + 1 – 2 = 1 . . . (i)
3 3 3
= 3× × × × 30 2 3 2 10 + 21 – 14 17
10 100 1000 + – = . . . (ii)
3 5 5 35 35
3× 3× 3×3×3 17
= Bringing (i) and (ii) together, we have 1
100000 35
243 5. Ans. (c)
= = 0.00243
100000 3 2 5
Sol. 7 +2 –6
4. Ans. (b) 5 3 6
2 3 2 38 8 41
Sol. 2 +1 – 2 = + –
7 5 5 5 3 6
Unit One : Practice Session ✫ 69

0.85 × 0.121
= 38 × 6 + 8 × 5 × 2 – 41× 5 =
30 0.00935
228 + 80 – 205 103 0.10285
= = =
30 30 0.00935
13
= 3
30 = 11
Alternative Method 11. Ans. (c)
Taking the whole numbers first, we have 9.5 × 0.085 95 × 85 ×100
Sol. =
0.0017 × 0.19 17 ×19

TS
7 + 2 – 6 = 3 . . . (i)
= 5×5×100 = 2500
Taking the fractions, we have
3 2 5 18 + 20 − 25 12. Ans. (a)
+ − = 5 5 3 15

H
5 3 6 30 Sol. = × =
13 3 3 3 3
= . . . (ii)
30 3.88

G
13 = = 1.2933
Adding (i) and (ii) we have 3 3
30
6. Ans. (a)
0.777777 U 13. Ans. (a)

Sol.
0.324 × 0.081× 4.624
O
Sol. 1.5625 × 0.0289 × 72.9 × 64
0.011
777.777 324 × 81× 4624 ×10 –9
= = 70.707
H
11 =
15625 × 289 × 729 × 64 ×10 –9
7. Ans. (a)
7 7 49 18 × 9 × 68
_T

× = = 0.024
125 × 17 × 27 × 8
Sol. 9 9 = 81
7 7 7 21
+ + 14. Ans. (d)
9 9 9 9
Sol. Given fractions are
C

49 9 7 5 10 15 20 25
= × = , , , ,
81 21 27 7 11 17 21 27
PS

8. Ans. (d) L.C.M. of 5, 10, 15, 20 and 25 is 300.


Sol. 832.58 – 242.31 = 779.84 – ? 5 5 × 60 300
Let the missing number be x. ∴ = =
7 7 × 60 420
U

832.58 – 242.31 = 779.84 – x


⇒ x = 779.84 – 832.58 + 242.31 10 10 × 30 300
⇒ x = 189.57 = =
11 11× 30 330
@

9. Ans. (c) 15 15 × 20 300


= =
17 17 × 20 340
(0.01)2 + (0.22)2 + (0.333)2 20 ×15 300
Sol. 20
(0.001)2 + (0.022)2 + (0.0333)2 = =
21 21×15 315
0.0001 + 0.0484 + 0.110889 25 25 × 12 300
= = =
0.000001 + 0.000484 + 0.00110889 27 27 ×12 324
0.159389 5 15 10 25 20
= = 100 ∴ < < < <
0.00159389 7 17 11 27 21
10. Ans. (a) We know that when numerators of
0.85 ×(0.105 + 0.024 – 0.008) fractions are same, the fraction with
Sol. the smallest denominator is the
0.022 × 0.25 ×1.7
largest.
70 ✫ Quantitative Aptitude

15. Ans. (c) 21. Ans. (c)


9 2 3 × (−1) −3
Sol. = 0.692 = 0.666 Sol. (3* – 1) = =
13 3 3 + (−1) 2
8 5
= 0.727 = 0.714 −3
11 7 So, 3* (3* – 1) = 3*
2
16. Ans. (c)
1 2 3 1 −3 −9
Sol. of – of 3×
2 3 4 3 2
= = 2

TS
1 1 1 −3 3
= − = 3+
3 4 12 2 2
17. Ans. (c)
9 2
Sol. ? = (0.6 + 0.7 + 0.8 + 0.3) × 9000 = − × = −3

H
2 3
22. Ans. (b)
6 7 8 3
+ + +

G
? = × 9000 x2 54
9 9 9 9 Sol. =
169 39
24
? =
9
× 9000
? = 24000 U x
=
13 39
54
O
54
18. Ans. (b) x= × 13 = 18
39
1
H
Sol. x2 = (18)2
0.0003618 = 324
_T

1 23. Ans. (b)


=
36.18 × (10)−5
1.21 × 0.9 121× 9
(10)5 Sol. =
= 1.1 × 0.11 11 ×11
C

36.18
= (0.276) × 105 = 3 [ (11)2 = 121]
PS

24. Ans. (c)


= 2760 1 1

19. Ans. (a) Sol. ( 8) 3 = ( 23 ) 3


3× 1
= 22 3
Sol. ? = 3.87 − 2.59
U

1
87 59 = 22
? = 3+ − 2+
@

99 99 = 2
87 59
= 3+ −2− 25. Ans. (d)
99 99
28 x 21
= 1+ Sol. =
99 144 36
= 1.28 x 21
20. Ans. (c) = [ (12)2 = 144]
12 36
Sol. 2.136 = 2 + 0.136
21× 12
136 − 1 = = 7
= 2+ x 36
990
135 3 x = (7)2
= 2+ =2
990 22  x = 49
Unit One : Practice Session ✫ 71

26. Ans. (c) 1


=
x a−b
Sol. = 0.02 where a = 1.04 and b = 0.04
200
1
x = 200 × 0.02 = =1
1.04 − 0.04
= 4 32. Ans. (b)
x = 16 Sol. Given expression
27. Ans. (b)
a2 − b2
0.0009 9 × 100 = = a+b

TS
Sol. = a−b
0.01 10000 × 1
where a = 0.778 and b = 0.222
9 ∴ a + b = 1.000
= = 1

H
100
3 33. Ans. (b)
= = 0.3 Sol. Given expression

G
10
28. Ans. (b) a 2 + b 2 + c2
=
122.76 12276 ×10 (10 a)2 + (10b)2 + (10c)2
Sol.
15.5
=
155 ×100
79.2 U where a = 0.82, b = 0.284, c = 0.795
a2 + b2 + c 2
O
= = 7.92 =
10 100 a + 100 b 2 + 100 c 2
2
29. Ans. (d)
H
Sol. Given expression is in the form ( a2 + b2 + c 2 ) 1
= =
3
a +b 3
100 ( a 2 + b 2 + c 2 ) 100
_T

a + b 2 – ab
2 = 0.01
34. Ans. (b)
( a + b) ( a 2 − ab + b 2 )
= Sol. Given expression
a 2 + b 2 − ab
C

a3 + b3
where a = 5.7 and b = 2.3
= a + b a − ab + b 2
2
PS

= 5.7 + 2.3 ( a + b)( a 2 − ab + b 2 )


=
= 8.0 a 2 + b 2 + bb
30. Ans. (d) = a + b
U

Sol. Given expression is in the form = 0.05 + 0.04


a3 + b 3 = 0.09
@

35. Ans. (d)


a + b 2 – ab
2
1 1 1
( a + b)( a2 − ab + b 2 ) Sol. 2 + 3 + 4
= 2 3 4
a2 + b2 − ab 1 1 1
=a + b = 2+ 3+ 4+ + +
2 3 4
= 0.87 + 0.13 6+4+3
=1 = 9+
12
31. Ans. (c) 13
Sol. Given expression = 9+
12
a 2 + ab + b 2 1
a3 – b 3 = 9+1
12
a2 + ab + b 2 1
= = 10
( a − b )( a 2 + ab + b 2 ) 12
72 ✫ Quantitative Aptitude

36. Ans. (c) 41. Ans. (a)


Sol. Given expression Sol. 3 1298 36
= 125 + 12.5 + 1.25 9
= 138.75 66 398
37. Ans. (b) 296
Sol. Let there be x number of men and 2
each spent Rs x, then ∴ 1298 – 2 = 1296 is a perfect square
x × x = 15625 of 36.
x 2 = 15625 42. Ans. (c)

TS
1
∴ x = 15625 Sol. 112 = 1, (2 2 )12 = 26 = 64
1 1
= 125 (3 3 )12 = 34 = 81, (4 4 )12 = 43 = 64
38. Ans. (b) 1

H
33 is the largest number among the
Sol. 3 0.000064
given options.
8

G
64 43. Ans. (c)
= 3 = 3
1000000 1000
2 +1 ( 2 + 1) ( 2 + 1)
= 3
2
10
3

U Sol.
2 −1
=
( 2 − 1) ( 2 + 1)
O
[Multiplying numerator and
= 3
(0.2)3 = 0.2 denominator by ( 2 + 1) ]
H
39. Ans. (c) ( 2 + 1)2
3
−8
3 × 16 4 =
1
Sol.
_T

625 × 9−4 = 2 +1
3 3
3−8 × 24 4
3−8 × 24 4
= 1.4142 + 1
= = 4
5 × (32 )−4
4
5 × 3 −8 = 2.4142
C

4× 3 3
44. Ans. (a)
2 4
23 2 8
= = = Sol. (6 3 5)3 = 216 × 5 = 1080
PS

= 4× 3
5 4 53 5 125
(8 3 2)3 = 512 × 2 = 1024
40. Ans. (d)
(2 3 130)3 = 8 × 130 = 1040
Sol. The largest three-digit number is 999
U

( 3 900)3 = 900
3 999 31 45. Ans. (c)
9
@

61 99 x y
Sol. a = x + y and b = x − y
61
38 xy x2 + y2
∴ 999 – 38 = 961 is the largest three- ∴ ab = 2 , a+b = 2
x −y 2
x − y2
digit perfect square, the root of which ab xy
is 31. ∴ = 2
a + b x + y2
Unit One : Practice Session ✫ 73

Practice Exercise 5
1. The value of [(10)150 ÷ (10)146] is
(a) 1000 (b) 10000
14. ( ) (
28 × 24 ÷ )
42 × 8 is equal to

(c) 100000 (d) 10 6 (a) 2 (b) 2 2

2. (17)3.5 × (17)? = 178 (c) 3 (d) 3 3


(a) 2.29 (b) 2.75 15. Taking 2 = 1.414 , 3 = 1.732 ,
5 = 2.236
(c) 4.25 (d) 4.5
9+ 2
1 4 and 6 = 2.449 , the value of

TS
− −
3. (64) 2 − (−32) 5 = ? 5+ 3
6− 2
+ to three places of decimals is
1 3 5− 3
(a) (b)
8 8 (a) 9.231 (b) 13.716

H
1 3 (c) 11.723 (d) 15.892
(c) (d)
16 16 4

G
4. The cube root of 3375 is 16. If 3 = 1.732 , value of is
3 +1
(a) 2 3 (b) 1 5
(a) 1.281 (b) 1.362
(c) 2 5 (d) 3 5
5. The cube root of 140.608 is
U (c) 1.464 (d) 1.547
17. If 2n = 256 , then the value of n is
O
(a) 14.2 (b) 12.2
(a) 8 (b) 1 0
(c) 8.2 (d) 5.2
(c) 1 2 (d) 1 6
H
6. The cube root of 274625 is
18. The smallest number of six digits which is a
(a) 4 5 (b) 5 5
perfect square is
(c) 6 5 (d) 7 5
_T

(a) 100489 (b) 100000


7. Cube root of 9261 is (c) 101124 (d) 101761
(a) 2 1 (b) 121
19. By what least number should 6728 be
(c) 3 1 (d) 131
C

divided so that the resulting number is a


8. Cube root of 1906.624 is perfect square?
(a) 12.2 (b) 12.4 (a) 2 (b) 3
PS

(c) 13.2 (d) 13.4 (c) 4 (d) 6


9. 1.52 × 0.0225 is equal to 20. (1000)7 ÷ 1018 = ?
(a) 0.2275 (b) 0.3375 (a) 1 0 (b) 100
U

(c) 0.2375 (d) 0.3275 (c) 1000 (d) 10000


3 3
10. a3 is equal to 2 3
@

21. If 6 = 2.55 , then the value of +3 is


(a) a 1 (b) 1 3 2
(c) a 3 (d) a 3 (a) 4.128 (b) 4.439
(c) 4.675 (d) 4.826
11. 3 3 3 3 is equal to 22. The value of
(a) 1 (b) 3
(c) 3
15
16 (d) 3
9
2 [(12.1)2 − (8.1)2 ] ÷ [(0.25)2 + (0.25)(19.95)
(a) 1 (b) 2
12. The square root of (2722 – 1282) is
(c) 3 (d) 4
(a) 144 (b) 200
(c) 240 (d) 256  1 1 
23.  2 + 2 + +  simplifies to
13. If 18 × 14 × x = 84 , then x equals  2+ 2 2 −2
(a) 2 2 (b) 2 4 (a) 2 (b) 4 2
(c) 2 6 (d) 2 8 (c) 4 − 2 (d) 4 + 2
74 ✫ Quantitative Aptitude

5+2 3 2 2 2 2
24. If = a + b 3 , then a and b are equal to 32. + + + =?
7+4 3 5 3 5 3
(a) –11, –6 (b) –11, 6 2
(c) 11, –6 (d) 6, 11 (a) 1 (b) 2
15
1 1 1 1
25. The value of 1+ 1+ (c) 2 (d) 1
x+1 x+2 15 15
1 1
1+ 1+ is 0.361
x+3 x+4 33. =?
0.00169

TS
1 1
(a) 1 + (b)
x+5 x+5 19 1.9
(a) (b)
1 x+5 130 13
(c) x + (d)
x+5 x +1

H
19 190
26. The difference between the squares of two (c) (d)
13 13
consecutive odd integers is always divisible

G
34. 1012 × 988 = ?
by
(a) 3 (b) 5 (a) 988866 (b) 98996
(c) 8 (d) 11
U (c) 999856 (d) 992786
O
25 x 196 ?
27. If 1+ = 1 + , then x = ? 35. =
144 12 22 484
H
(a) 1 (b) 2 (a) 1 4 (b) 196
(c) 5 (d) 7
(c) 1016 (d) None of these
_T

5 +1 5 −1
28. If a = and b = , then the value
5 −1 5 +1 36. 0.00059049 = ?
a 2 + ab + b 2 (a) 0.243 (b) 0.0243
of 2
C

a − ab + b 2 (c) 0.000243 (d) 0.00243


1
(a) 1 (b)
PS

4 37. 3
0.000027 = ?
1 4
(c) (d) (a) 0.3 (b) 0.03
2 3
2 (c) 0.003 (d) None of these
U

1 1
29. If a + = 3, then the value of a +
3
is
a a3 324 ?
38. =
@

(a) 0 (b) 3 3 1.5 256


10 3 (a) 192 (b) 432
(c) 6 3 (d)
3 (c) 288 (d) 122
30. The value of 2 2 2 2 is ? 130
39. If = 10, then the value of x is
(a) 0 (b) 2 x
(c) 2 15/16 (d) 2 31/32 (a) 1 3 (b) 1300
3420 ? (c) 1690 (d) 169
31. = ×7
19 0.01 7 x
35 63 40. If = 2, then x is?
(a) (b) 49
9 5
18 (a) 1 4 (b) 196
(c) (d) None of these (c) 214 (d) 8 4
7
Unit One : Practice Session ✫ 75

0.289 (a) 2 (b) 2


41. =? 1
0.00121 (c) (d) 1
2
170 17 x −1 x−3
(a) (b) a b
11 110 44. If = , then x is equal to
b a
0.17 17
(c) (d) 1 7
11 11 (a) (b)
2 2
42. If 64009 = 253, then 64009 + 640.09 + (c) 1 (d) 2

TS
6.4009 + 0.064009 is 24 + 216
45. =?
(a) 80.3 (b) 2810.83 96
(c) 281.083 (d) 333.33 (a) 2 6 (b) 6 2

H
392 2
43. =? (c) 2 (d)
6

G
1568

U
Answers with Solutions
O
1. Ans. (b) 5. Ans. (d)
Sol. (10)150 ÷ (10)146 3 140608
Sol. 140.608 = 3
H
1000
= (10)(150–146)
= 10 4 = 10000
_T

2. Ans. (d)
Sol. Let (17)3.5 × (17)x = 178
then, (17)3.5 + x = (17)8
C

∴ 3.5 + x = 8
PS

x = 8 – 3.5 = 4.5
3. Ans. (c)
1 –4 5
3
140608 = 3 2 × 2 × 2 × 2 × 2 × 2 × 13 × 13 × 13
Sol. (64)– 2 – (–32)
U

= 2×2×13
− 12 −4
= (82 ) − {(−2)5 } 5 = 52
@

= 8−1 − ( −2)−4 and 3


1000 = 10
1 1 1 1 1 140608 52
= − = − = ∴ 3 = = 5.2
8 (−2)4 8 16 16 1000 10

4. Ans. (b) 6. Ans. (c)


Sol. Sol.
5 3375 3
3375 = 3 5 274625 3
274625 = 3 5 × 5 × 5 × 13 × 13 × 13
5× 5×5×3×3×3
5 675 5 54925
= 5×3 = 5×13
5 135 5 10985
= 15 = 65
3 27 13 2197
3 9 13 169
3 3 13 13
1 1
76 ✫ Quantitative Aptitude

7. Ans. (a) 12. Ans. (c)


Sol. Sol. (272 2 – 1282 ) = (272 + 128)(272 − 128)
3 9261 3
9261 = 3 3 × 3 × 3 × 7 × 7 × 7 = 400 × 144
3 3087 = 3×7 = 57600
3 1029
= 21 = 240
7 343
7 49 13. Ans. (d)
7 7 Sol. 18 × 14 × x = 84

TS
1
18 × 14 × x = (84)2
8. Ans. (b) 84 × 84
1906624 x = = 28
18 × 14
Sol. 1906.624 =
1000

H
14. Ans. (a)
1906624 2 × 2 × 31
( ) ( )
2 1906624
2 953312
∴ 3 = Sol. 28 × 24 ÷ 42 × 8
1000 10

G
2 476656 = 12.4 28 × 24 7 × 4 × 6× 4
2 238328 = =
42 × 8 7× 6 × 4× 2
2 119164
2 59582
31 29791 U =
7 × 2× 2 × 2 × 3
=
2
=
2
×
2
O
7 × 2 × 3 × 2× 2 2 2 2
31 961
31 31 = 2
H
× 15. Ans. (c)
9. Ans. (b) 9+ 2 6− 2
+
_T

Sol.
Sol. 5+ 3 5− 3
2 225 9 5 − 9 3 + 10 − 6 + 6 5 + 6 3 − 10 − 6
1.52 × 0.0225 = 1.5 × =
10000 ( 5 + 3) ( 5 − 3)
C

15 15 5 − 3 3 − 2 6
= 2.25 × =
100 2
PS

= 2.25×0.15 = 0.3375 15 × 2.236 − 3 × 1.732 − 2 × 2.449


=
10. Ans. (c) 2
1 1 33.54 − 5.196 − 4.898
Sol. 3 3
a3 =
3
(a3 )3 =
3
a = a3 = = 11.723
U

2
11. Ans. (c) 16. Ans. (c)
4 4 3 −1
×
@

Sol. =
Sol. 3 3 3 3 = 3 3
1
3 +1 3 +1 3 −1
3(3)2
4( 3 − 1)
1 1 =
= 3 3(3)2 .34 3−1
4(1.732 − 1)
=
=
1 1
+ 2
3 3.3 2 4
= 2(0.732)
1 3 = 1.464
= 3 32.(3)4 17. Ans. (d)
8
1 3
+
Sol. 2nn = 256 =2
= 3.3 2 4
⇒ 22 = 28
7 n
= 3.38 so, = 8
1 7 15 2
= 3 2 + 8 = 316 n = 16
Unit One : Practice Session ✫ 77

18. Ans. (a) 23. Ans. (a)


1 1
Sol. Smallest number of six digits is 100000 Sol. 2 + 2 + +
2+ 2 2 −2
3 100000 316 1 2− 2 1 2 +2
9 = 2+ 2 + × + ×
2+ 2 2− 2 2 −2 2 +2
61 100
61 2− 2 2+2
= 2+ 2 + +
626 3900 4 −2 2− 4
3756
2− 2 2 +2
144 = 2+ 2 + −

TS
∴ Required number is (317)2 = 100489 2 2
19. Ans. (a) 2− 2 − 2 −2
= 2+ 2 +
Sol. 2

H
2 6728 ∴ 6728 = 2 × 2 × 2 × 29 × 29 −2 2
= 2+ 2 +
2 3364 = 2 2 × 29 2 × 2 2
= 2+ 2 − 2

G
2 1682
6728
29 841 Thus, = 2 2 × 29 2 =2
29 29 2
1
Therefore, 6728 should be divided by 2, so U24. Ans. (c)
5+2 3 7−4 3
×
O
that the result is a perfect square. Sol. a + b 3 =
7+4 3 7−4 3
20. Ans. (c)
35 − 20 3 + 14 3 − 24
H
(1000)7 =
7
Sol. (1000) ÷ 10 =
18
(7)2 − (4 3)2
1018
11 − 6 3
_T

(103 )7 1021 =
= = 18
1018 10 49 − 48
21−18
= 10 = 103 = 1000 = 11 − 6 3
C

21. Ans. (c) ∴ a = 11 and b = –6


2 3 2 3 3 2 25. Ans. (d)
PS

Sol. + 3× = × + 3× ×
3 2 3 3 2 2 Sol.
1 1 1 1
=
6 3 6
+ 1+ 1+ 1+ 1+
3 2 x+1 x+2 x+3 x+4
U

2.55 3 × 2.55 x + 1+ 1 x+ 2+1 x +3+1 x + 4 +1


= + =
3 2 x+1 x+2 x+3 x+4
@

2.55 7.65 5.10 + 22.95


= + = x+2 x+3 x+4 x+5
3 2 6 = × × ×
28.05 x +1 x + 2 x + 3 x + 4
= = 4.675
6 x+5
22. Ans. (d) =
x +1
Sol. 26. Ans. (c)
(12.1)2 − (8.1)2  ÷ (0.25)2 + (0.25)(19.95) Sol. Let the two consecutive odd numbers
   
be 2x + 3 and 2x + 1
(12.1 + 8.1)(12.1 − 8.1) 20.2 × 4 According to the given condition,
= =
0.25(0.25 + 19.95) 0.25 × 20.2 (2 x + 3)2 − (2 x + 1)2
4 400 = 4 x 2 + 12 x + 9 − (4 x 2 + 4 x + 1)
= = = 16 = 8x + 8
0.25 25
= 4 = 8(x + 1)
78 ✫ Quantitative Aptitude

27. Ans. (a) 30. Ans. (c)


25 x 1
Sol. 1+ = 1+ Sol. 2 2 2 2 = 2 2 2 × 22
144 12
144 + 25 169 13 x 3
m+n
= 2 2 22 [a × a = a
m n
= = = 1+ ]
144 144 12 12
1
1 x 3 ×1
[ a = a 2 ]
= 1+ = 1+ = 2 2× 2 2 2
12 12

TS
∴ x =1 3
= 2 2 × 24
28. Ans. (d)
1+ 3 [ am × an = am+n ]
4
5 +1 = 2 2

H
Sol. Given that a =
5 −1 7
= 2 24

G
5 +1 5+1
a = × 1

5 −1 5+1 2(2 4 ) 2
7 1
[ a = a 2 ]
=

=
5+1+ 2 5 6+ 2 5
= U = 2 × 28
7
O
4 4
1+ 7
5 −1 = 2 8

and b =
H
5 +1 15
= 28
5 −1 5 −1 15 × 1 15
_T

 b = × = 28 2
= 216
5+1 5 −1
31. Ans. (d)
5+1−2 5 6−2 5
= = 3420 x
C

4 4 Sol. = ×7
19 0.01
a 2 + ab + b 2 ( a + b)2 − ab
PS

3420 0.01
 2 =
( a + b)2 − 3ab x = ×
a − ab + b 2 19 7
9−1 8 4 180 × 0.01 1.8
= = = = =
U

9−3 6 3 7 7
32. Ans. (b)
29. Ans. (a) 2 2 2 2  2 × 3 + 5× 2 
@

1
2
Sol.  +  +  +  =  ×2
Sol. a+ =3  5 3  5 3  15 
a  6 + 10 
=   ×2
1  15 
 a+ = 3
a 16 × 2 32
= =
3 15 15
1 1 1 2
a+ = a + +3 a+
3
∴ a3 a = 2
a 15
1 33. Ans. (d)
3 3 = a + a3 + 3 3
3
 0.361 361× 100000
Sol. =
1 0.00169 1000 × 169
 a3 + 3 = 0 19 × 10 190
a = =
13 13
Unit One : Practice Session ✫ 79

34. Ans. (c) 39. Ans. (d)


Sol. 1012 × 988 = (1000 + 12) (1000 – 12) 130
Sol. = 10
= (1000)2 – (12)2 x
= 1000000 – 144 130
x= = 13
= 999856 10
x = 169
35. Ans. (d) 40. Ans. (b)
196 x 196 x 7 x
Sol. = (Let ? = x2) Sol. = 2
22 484 22 22

TS
49 2× 49
196 × 22 ∴ x =
∴ x= 7
22 = 14
x = 196
x = 196 196 2

H
x 38416
41. Ans. (a)
36. Ans. (b) 0.289 289 × 100000

G
Sol. =
Sol. 0.00121 121 × 1000
17 × 10
0.00059049 = ?
3 59049
U =
11
[ We know that 289 = 17, 121 = 11]
O
3 19683 170
3 6561 =
11
3 2187 42. Ans. (c)
H
3 729 Sol. Given 64009 = 253
3 243 Then the given expression
_T

3 81
3 27 64009 + 640.09 + 6.4009 + 0.064009
3 9 = 253.00 + 25.30 + 2.53 + 0.253
3 = 281.083
C

59049 = 3 × 3 × 3 × 3 × 3 × 3 × 3 × 3 × 3 × 3 43. Ans. (c)


2 × 196
PS

392 1 1
∴ 59049 = 3 × 3 × 3 × 3 × 3 Sol. = = =
1568 2 × 784 4 2
= 243 44. Ans. (d)
x −1 x −3

U

0.00059049 = 0.0243 a b
Sol. =
b a
37. Ans. (b) − x +3
a
@

=
Sol. 3
0.000027 b
∴ x – 1 = –x + 3
27 3
3 = = 0.03 ( 3 27 = 3)  2x = 4
1000000 100 x = 2
38. Ans. (a) 45. Ans. (c)
324 x 24 + 216 24 + 24 × 9
Sol. = Sol. =
1.5 256 96 24 × 4
We know that (16)2 = 256 24 + 3 24
=
(18)2 = 324 2 24
18 × 16 × 10 Taking 24 common in the numerator
∴ x=
15 24 × (1 + 3) 4
= 192 = = =2
2 24 2
80 ✫ Quantitative Aptitude

Practice Exercise 6
Directions (Qs. 1-8): What will come in place 10. The difference between a two-digit number
of the question-mark (?) in the following and the number after interchanging the
questions? position of the digits is 36. What is the
1. 49 × 49 × 49 × 49 = 7? difference between the two digits of the
1. 4 2. 7 number ?
3. 8 4. 16 1. 4
5. None of these 2. 6

TS
3. 3
2. (500 + 200) × 4 × (3 + 2) = ?
4. Cannot be determined
1. 2902 2. 8402
5. None of these
3. 14000 4. 16800
5. None of these 11. If the digits of a two-digit number are

H
3. 16.02 × 0.001 = ? interchanged, the newly formed number is
1. 0.01602 2. 0.001602 more than the original number by 18, and

G
3. 1.6021 4. 0.1602 sum of the digits is 8, then what was the
5. None of these original number?

4.
?
50
=
60.5
? U 1. 5 3
2. 2 6
3. 3 5
O
1. 55 2. 1512.5 4. Can’t be determined
3. 3025 4. 57.5 5. None of these
H
5. None of these
12. What should come in place of the question
5. 5400 ÷ 9 ÷ 3 = ?
mark (?) in the following equation?
_T

1. 300 2. 1800
138,009 + 341.981 – 146.305 = 123.6 + ?
3. 900 4. 450
1. 210.85 2. 120.85
5. None of these
3. 220.085 4. 120.085
1 1 2
C

6. + + =? 5. None of these
5 7 3 5
1 104 13. One-fifth of a number is equal to th of
PS

1. 1 2. 8
105 105 another number. If 35 is added to the first
1 2 number, it becomes four times the second
3. 4. 1
105 105 number. Find the second number.
U

5. None of these 1. 125 2. 7 0


7. 10150 ÷ 10146 = ? 3. 3 0 4. 2 5
1. 10 6 5. None of these
@

2. 100000
3. 10000 4. 1000
14. Five-eighth of three-tenth of four-ninth of a
5. None of these
number is 45. What is the number?
8. 200 + 5 × 4 = ? 1. 450 2. 550
1. 420 2. 202 3. 560 4. 540
3. 820 4. 209 5. None of these
5. None of these
9. Which of the following is the highest 15. In a two digit number the digit in the unit’s
fraction? place is more than twice the digit in ten’s
5 3 place by 1. If the digits in the unit’s place
1. 2. and the ten’s place are interchanged, the
7 4
2 6 difference between the newly-formed
3. 4. number and the original number is less than
3 7
7 the original number by 1. What is the
5. original number?
8
Unit One : Practice Session ✫ 81

1. 38 2. 73 22. What will come in place of both the question


3. 37 4. 49 marks (?) in the following equation?
5. 25 72 ?
=
16. What should come in place of both the ? 24
question marks (?) in the following 1. 1 2 2. 144
equation? 3. 3 6 4. 2 4
28
5. None of these
?
= 23. What should come in place of the question
? 112
1. 74 2. 56 mark (?) in the following equation?

TS
3. 44 4. 3136 18265 + 2736 + 41328 = ?
5. None of these 1. 61329 2. 62239
17. If A and B are whole numbers such that 3. 62329 4. 62319
9A2 = 12A + 96 and B2 = 2B + 3, then which 5. None of these

H
x2 – 1
of the following is the value of 5A + 7B? 24. If x + 1 = 4, x = ?
1. 4 5 2. 3 7

G
3. 3 1 4. 4 3 1. 1
2. 0
5. 4 1
3. 5
18. A two-digit number is seven times the sum
of its digits. If each digit is increased by 2,U 4. Cannot be determined
5. None of these
O
the number thus obtained is more than six 25. What should come in place of the question
times the sum of its digits by 4. Find the mark (?) in the following equation?
H
number.
(7 × ?)2
1. 4 2 2. 2 4 = 16
3. 4 8 4. Data inadequate 49
_T

1. 7 2. 3
5. None of these 3. 2 4. 4
19. What should come in place of the question 5. None of these
mark (?) in the following equation? 26. What least number would be subtracted from
C

( )
3 ? 427398 so that the remaining number is
477.5 ÷ 472 × 47–3 = 47 divisible by 15?
1
PS

1. 4 2. 2 1. 6 2. 1 6
2 3. 3 4. 1 5
3. 6 4. 5 5. None of these
5. None of these 27. If a number is decreased by 4 and divided
U

20. What will come in place of the question mark by 6 the result is 8. What would be the result
(?) in the following equation? if 2 is subtracted from the number and then
@

3 it is divided by 5?
of 168 × 15 ÷ 5 + ? = 549 ÷ 9 + 235 2 1
8 1. 9 2. 10
1. 184 2. 107 3 5
3. 174 4. 197 2 2
3. 11 4. 10
5. None of these 5 5
21. In a fraction, if numerator is increased by 5. None of these
40%, and denominator is increased by 80%, 28. Which of the following will come in place
then what fraction of the old fraction is the of both the question marks (?) in the
new fraction? following equation?
1 7 128 ÷ 16 × ? − 7 × 2
1. 2. = 1
2 9 72 − 8 × 6 + ?2
7 1. 17 2. 16
3. 4. Data inadequate
18 3. 18 4. 3
5. None of these 5. 14
82 ✫ Quantitative Aptitude

29. What approximate value should come in


place of the question mark (?) in the 37. 5 × 2 ÷ 4 ÷ 6 = ?
6 9 9 7
following equation? (1) 0.44 (2) 0.32
39.05 × 14.95 – 27.99 × 10.12 (3) 0.49 (4) 0.36
= (36 + ?) × 5 (5) 0.40
1. 2 5 2. 2 9 38. 754.002 + 3.00024 × 88.976 = ?
3. 3 4 4. 3 2 (1) 66616 (2) 1021
5. 2 2 (3) 1000 (4) 66600
4 4 2 (5) 1035
×2 ÷ =?

TS
30.
7 5 5 39. Which number should replace both the
6 question marks in the following equation?
(1) (2) 8 ? 48
7
2 =
432 ?

H
(3) (4) 4 (1) 141 (2) 143
5
(5) None of these (3) 145 (4) 147

G
31. – 56 × 61 + 89 = ? (5) None of these
(1) – 3237 (2) 3327 40. The sum of three consecutive even numbers
(3) – 3327
(5) None of these
(4) 3416
U is 44 more than the average of these numbers.
Which of the following is the third (largest)
O
32. 3625 × ? = 1450 of these numbers?
(1) 1 6
1 2
(1) (2) (2) 1 8
H
3 5 (3) 2 4
1 4 (4) Cannot be determined
(3) (4)
_T

5 5 (5) None of these


(5) None of these
Directions (Qs. 41-45): What should come in
33. 916 × ? × 3 = 214344 place of the question mark (?) in the following
(1) 78 (2) 6 8 questions?
C

(3) 84 (4) 6 6 41. ? = (756 × 67) ÷ 804


3
PS

(5) None of these (1) 195112 (2) 250047


(3) 226981 (4) 274625
34. (54679 + 5982 + 32614) – (312 × 69) = ?
(5) None of these
(1) 71528 (2) 77147
42. 0.3 + 3 + 3.33 + 3.3 + 3.03 + 333 = ?
U

(3) 71747 (4) 61757


(1) 375.66 (2) 345.99
(5) None of these
(3) 375.93 (4) 355.96
@

35. ? = (88 × 42) ÷ 16 (5) None of these


(1) 3696 (2) 39660 43. (73425 – 33267 – 22418 – 17650) × 11025 =?
(3) 43163 (4) 53361 (1) 10165 (2) 9785
(5) None of these (3) 8370 (4) 9450
36. 41 + 4.11 + 0.41 + 411 + 41.11 = ? (5) None of these
(1) 456.63 (2) 487.63 44. –76 × 33 + 221 = ?
(3) 456.52 (4) 479.63 (1) –2287 (2) –19304
(5) None of these (3) 2287 (4) 19304
(5) None of these
Directions (Qs. 37-38): What approximate value 45. (34.12)2 – 7396 = ?
should come in place of the question mark (?) (1) 1080.1744 (2) 1078.1474
in the following questions? (You are not (3) 1078.1744 (4) 1080.1474
expected to calculate the exact value.) (5) None of these
Unit One : Practice Session ✫ 83

Answers with Solutions


1. Ans. (3) 8. Ans. (5)
Sol. Given 49 × 49 × 49 × 49 = 7? Sol. 200 + 5 × 4 = ?
We know that Using BODMAS rule, we get
7 2 = 49 200 + 20 = ?
∴ LHS ? = 220
= 72 × 72 × 72 × 72 9. Ans. (5)
= 72+2+2+2 [ ax × ay = ax + y] Sol. In such type of questions, since the
= 78 = 7?

TS
denominators of the fractions are not
∴ ? = 8 (  If ax = ay ⇒ x = y) equal, we have to find the respective
2. Ans. (3) values of the fractions.
Sol. (500 + 200) × 4 × (3 + 2) = ?

H
5
Using BODMAS rule, we get 1. = 0.714
7
700 × 4 × 5 = ?
3

G
? = 14000 2. = 0.75
3. Ans. (1) 4
2
Sol. 16.02 × 0.001
= 0.01602
[Remove decimals and get 1602 × 1 = U 3.

4.
3
6
= 0.67

= 0.86
O
1602 and then put the decimal point 7
7
counting 5 from the right.] 5. = 0.88
H
8
4. Ans. (1) 7
∴ is the highest fraction
? 60.5 8
_T

Sol. =
50 ? 10. Ans. (1)
⇒ (?)2 = 60.5x50
Sol. Let x be the digit at unit’s place and
⇒ (?)2 = 3025
let y be the digit at ten’s place.
C

⇒ ?= 3025 =55 ∴ The two digit number will be 10y + x


After interchanging the position of the
PS

5. Ans. (2)
Sol. 5400 ÷ 9 ÷ 3 = ? two digits, the new number will be
⇒ 5400 ÷ 3 = ? [ 9 ÷ 3 =3] 10x + y
⇒ ? = 1800 Given that the difference between the
U

number and the number after


6. Ans. (1)
interchanging the position of the two
1 1 2
@

Sol. + + =? digits is 36.


5 7 3 ⇒ (10y + x) – (10x + y) = 36
Taking L.C.M., we get ⇒ 10y + x – 10x – y = 36
21 + 15 + 70 106
= ⇒ 9y – 9x = 36
105 105 ⇒ 9 (y – x) = 36
1 36
=1 ⇒ y–x = =4
105 9
7. Ans. (3) ∴ The difference between the two digits
Sol. 10150 ÷ 10146 = ? =4
We know that
ax ÷ ay= ax–y 11. Ans. (3)
So, 10150 ÷ 10 146 Sol. Method I (Short-cut method)
Consider Option (1)
⇒ 10 150–146 = ?
If 53 is the original no. the new number
⇒ 10 4 = ?
will be 35
⇒ ? = 10000
84 ✫ Quantitative Aptitude

Sum of digits = 3 + 5 = 8 12. Ans. (5)


The new number is more than original Sol. 138.009 + 341.981 – 146.305
number (given) = 123.6 + ?
But 35 < 53 Using BODMAS rule, we get
⇒ 479.99 – 146.305 = 123.6 + ?
∴ Option (1) is not the answer. ⇒ 333.685 = 123.6 + ?
Consider option 2 ⇒ ? = 333.685 – 123.6
Sum of digits of 26 = 2 + 6 = 8 ⇒ ? = 210.085
New number will be 62 which is more 13. Ans. (5)

TS
than the original number. Sol. Let a be the first number
The difference between new and and let b be the second number
original number should be 18. Given that

H
But 62 – 26 = 36 1 5
∴ Option (2) is not the answer. of a = of b
5 8
5×5

G
Now consider option (3) ⇒ a= ×b
8
Sum of digits of number 35 = 3 + 5 = 8
25
Newly formed number = 53 which is
more than original number
U ⇒ a=
Also given that
8
b ... (1)
O
Now difference between 53 and 35 = 18 a + 35 = 4b
and this criteria is also fulfilled ⇒ a = 4b – 35 ... (2)
H
∴ The answer is option (3). We can see that equation (1) = equation (2)
Method II 25
⇒ b = 4b – 35
_T

Let the digit at the unit’s place be x and 8


let the digit at the ten’s place be y 25
⇒ 4b – b = 35
Sum of digits = 8 i.e. y + x = 8 ... (1) 8
32 b − 25 b
C

∴ The two digit number will be 10y + x ⇒ = 35


8
When the digits are interchanged, new 7
⇒ b = 35
PS

number is 10x + y 8
Given that newly formed number is 8
⇒ b = 35 ×
more than the original number by 18 7
⇒ ⇒ b= 5×8
U

(10x + y) – (10y + x) = 18 ... (2)


⇒ 10x + y – 10y – x = 1 8 ⇒ b = 40
∴ The second number is 40
⇒ 9x – 9y = 1 8
@

⇒ x–y= 2 ... (3) 14. Ans. (4)


From equation (1) x + y = 8 Sol. Let x be the required number
⇒ x=8–y ... (4) Given that
Putting the value of x in equation (3) 5 3 4
of of of x = 45
(8 – y) – y = 2 8 10 9
8 – 2y = 2 5 3 4
⇒ × × × x = 45
2y = 8 – 2 = 6 8 10 9
y= 3 Cancel as many common terms as
Putting the value of y in equation (4) possible.
1
x= 8–3=5 ⇒ × x = 45
4×3
∴ The original number is 10(3) + 5 ⇒ x = 45 × 12
= 30 + 5 = 3 5 ⇒ x = 540
Unit One : Practice Session ✫ 85

15. Ans. (3) Consider option (2) : 73


Sol. Let x be the digit at unit’s place and Condition I :
let y be the digit at ten’s place LHS =3 – 2 × 7 = 3 – 14 = –11 ≠ 1
∴ The number is (10y + x) ∴ Condition I is not satisfied.
Given that So option (2) is not the answer
x – 2y = 1 ... (1)
Consider option (3) : 37
When we interchange the digits the new
Condition I :
number is 10x + y
LHS =7 – 2 × 3 = 7 – 6 = 1 = RHS
Given that

TS
Condition I is satisfied
10x + y – (10y + x) =10y + x – 1 Now consider condition II:
⇒ 10x + y – 10y – x = 10y + x – 1 LHS = 73 – 37 = 36
⇒ 9x – 9y = 10y + x – 1 RHS = 37 – 1 = 36

H
⇒ 9x – x = 10y + 9y – 1 LHS = RHS
⇒ 8x = 19y – 1 ∴ Condition II is also satisfied

G
⇒ 19y = 8x + 1 ... (2) ⇒ The required answer is Option (3).
Consider equation (1) 16. Ans. (2)


x – 2y = 1
x = 1 + 2y U Sol. Given
28
=
?
O
Putting the value of x = 1 + 2y in ? 112
equation (2) we get ⇒ 28 × 112 = ? × ?
H
19y = 8 (1 + 2y) + 1 ⇒ (?)2 = 3136
⇒ 19y = 8 + 16y + 1 ⇒ ? = 3136

_T

⇒ 19y – 16y = 9 ? = 56
⇒ 3y = 9 17. Ans. (5)
9
⇒ y= =3 Sol. Given
3
C

9A2 = 12A + 96 ... (1)


Putting the value y = 3 in equation (1)
B2 = 2B + 3 ... (2)
x = 1 + 2y, we get
PS

Solving equation (1)


x = 1 + 2(3)
9A2 – 12A – 96 = 0
x= 1+6
⇒ 3A2 – 4A – 32 = 0
x= 7
⇒ 2
U

3A – 12A + 8A – 32 = 0
∴ The original number is 37
[ Splitting the middle term]
Alternative Method ⇒ 3A (A – 4) + 8 (A – 4) = 0
@

The required two-digit number = 10y +x ⇒ (A – 4) (3A + 8) = 0


The number obtained after inter-changing ⇒ (A – 4) = 0; (3A + 8) = 0
digits = 10x + y 8
Condition I : x – 2y = 1 ⇒ A = 4; A = –
3
Condition II : (10x + y) – (10y + x) Given that A is a whole no., so
= (10y + x) – 1 8
A= – is not possible
Consider option (1) : 38 3
∴ A=4
The original number = 38 Solving equation (2)
Condition I: LHS = 8 – 2 × 3 = 8 – 6 = 2 B2 – 2B – 3 = 0
Condition 1 is not satisfied because ⇒ 2
B + B – 3B – 3 = 0
LHS ≠ RHS [ Splitting the middle term]
∴ Option I is not the answer.
86 ✫ Quantitative Aptitude

⇒ B(B + 1) – 3(B + 1) = 0 Given that


⇒ (B + 1) (B – 3) = 0 7(x + y) = 10y + x ... (1)
Also given that if each digit is increased
⇒ (B + 1) = 0; (B – 3) = 0
by 2, the no. thus obtained is more than
⇒ B = – 1; B = 3 six times the sum of its digits by 4.
Again, B is also a whole no. So B = –1 is ⇒ 10(y + 2) + (x + 2) – 4
not possible = 6(x + 2 + y + 2) ... (2)
∴ B= 3 Solving equation (1) we get,
To find 5A + 7B 7x + 7y = 10y + x

TS
7x – x = 10y – 7y
= 5(4) + 7(3)
⇒ 6x = 3 y
= 20 + 21 = 41 1
⇒ x= y ... (3)
Alternative Method 2

H
We know that roots of a quadratic Solving equation (2) we get,
equation ax2 + bx + c = 0 10y + 20 + x + 2 – 4 = 6(x + y + 4)

G
10y + x + 18 = 6x + 6y + 24
– b ± (b)2 – 4 ac ⇒ 10y – 6y = 6x – x + 24 – 18
is x =
2a ⇒ 4y = 5x + 6


9A2 – 12A – 96 = 0
3A2 – 4A – 32 = 0
... (1)
U ⇒ y=
1
4
(5x + 6) ... (4)
O
1
4 ± ( −4)2 − 4(3)(−32) Putting y = (5x + 6) in equation (3) we get
A= 4
2× 3
H
1 1
4 ± 16 + 384 x= (5x + 6)
A= 2 4
_T

6
1
4 ± 400 ⇒ x= (5x + 6)
A= 8
6 ⇒ 8x = 5x + 6

C

4 + 20 4 − 20 8x – 5x = 6
A= ; ⇒ 3x = 6
6 6
PS

−8 6
A = 4; ⇒ x= =2
3 3
But A is a whole no. 1
∴ y= [5(2) + 6]
∴ A=4 4
U

B2 – 2B – 3 = 0 ... (2) 1
⇒ y= × 16 = 4
2 ± 4 − 4(1)(−3) 4
@

B= ∴ The required number is 42


2× 1
2± 4
B= Alternative Method
2 Let the two-digit no. be 10y + x
2+ 4 2 − 4
B= ; Condition I: 7(x + y) = 10y + x
2 2 Condition II: 10(y + 2) + (x + 2) – 4 = 6
B = 3; – 1
(x + 2 + y + 2)
B is also a whole no.
∴ B=3 Now check whether any of the given
∴ 5A + 7B = 5(4) + 7(3) = 20 + 21 = 41 numbers in the options fulfils both the
conditions.
18. Ans. (1)
Option (1): 42
Sol. Let x be the digit at unit’ place and let
y be the digit at ten’s place. Condition I: LHS = 7 (4+2) = 7×6 = 42
∴ The two digit number is 10y + x = RHS
Unit One : Practice Session ✫ 87

1 ×?
∴ Condition I is fulfilled ⇒ 47
7.5– 3 2
× 47–3 = 47 2
1 ×?
Condition II: LHS = 10 (4+2) + (2 + 2) – 4 ⇒ 476–3 = 47 2
= 10 × 6 + 4 – 4 1
⇒ 3= ×?
= 64 – 4 = 60 2
RHS = 6(4 + 2 + 2 +2) ⇒ ?= 6
= 6 × 10 = 60 20. Ans. (2)
LHS = RHS Sol. Given
∴ Condition II is fulfilled 3
of 168 × 15 ÷ 5 + ? = 549 ÷ 9 + 235
⇒ Option (1) is the answer 8

TS
Cancelling all common terms and using
19. Ans. (3)
BODMAS rule, we solve
( )
3 ?
Sol. 477.5 ÷ 47 2
× 47–3 = 47 ⇒ 63 × 3 + ? = 61 + 235

H
3
⇒ 477.5 ÷ 47 2
× 47–3 = (47)
1 ×x
2 ⇒ 189 + ? = 296
⇒ ? = 296 – 189
(Let the unknown number be x) ⇒

G
? = 107
( a) = a
n

1 n
2 21. Ans. (2)
x


477.5
47
3
2
× 47–3 = (47)
1 ×x
2
U Sol. Let y be the unknown fraction where x
is the numerator and y is the
O
7.5 −
3 1 ×x denominator.
⇒ 47 2 × 47–3 = (47) 2
When numerator is increased by 40%
H
an and denominator is increased by 80%,
 = an – r
ar the new fraction is
_T

1 ×x 40
⇒ 476 × 47–3 = (47) 2 x + 40%of x x+ x
100
1 1 ×x 1 y + 80%of y = 80
⇒ 476 × = (47) 2 a –n = y+ y
an 100
C

473
47 6 1 ×x
100 x + 40 x
⇒ = (47) 2 100
PS

47 3 =
100 y + 80 y
1 ×x
⇒ 47 6–3
= (47) 2
100
a n n–r 140 x
 =a
U

ar 100
=
1 ×x 180 y
⇒ 473 = (47) 2
@

1 100
⇒ 3= ×x Cancelling the common denominators,
2

we get
If an = ar n= r 140 x
⇒ ?= 3×2=6 180 y
Alternative Method 7x
= 9y
( )
3 ?
47 ÷ 47 × 47 = 47
7.5 2 –3

We know that 7 x
an ÷ ar = a n–r ⇒ New fraction is of old fraction y
9
an × ar = an + r
( a)
n 7x 7 x x
= a 12 n [∴ 9 y = 9 × y where y is the old fraction]
88 ✫ Quantitative Aptitude

22. Ans. (2) 24. Ans. (3)


Sol. Given
72 ?
Sol. = x2 – 1
? 24 x +1 =4
⇒ ? × ? = 72 × 24
⇒ 2
x – 1 = 4 (x + 1)
⇒ ? × ? = 1728
1
⇒ x2 – 1 = 4x + 4
⇒ ? × (?) 2 = 1728 ⇒ x2 – 4x – 1 – 4 = 0
Squaring both sides we get ⇒ x2 – 4x – 5 = 0
2
1
⇒ 2
x + x – 5x – 5 = 0

TS
⇒ ? × (?) 2
= [1728]2
[∴ Splitting the middle term]
⇒ ?2 × ? = (1728)2
⇒ ?3 = (1728)2 ⇒ x (x + 1) – 5 (x + 1) =0
⇒ ?3 = 1728 × 1728 (x + 1) (x – 5) = 0

H
⇒ ?3 = 72 × 24 × 72 × 24 (x + 1) = 0; (x – 5) = 0
⇒ ?= 72 × 24 × 72 × 24 ⇒ x=–1;x =5

G
3

⇒ ?= 3 (24 × 3) × 24 × (24 × 3) × 24 Alternative Method


x2 – 1

⇒ ?= 3
[  72 = 24 × 3]
24 × 24 × 24 × 3 × (3 × 8) U x+1 = 4
x – 12
2
O
[  3 × 8 = 24] x +1 = 4 [ 12 = 1]
⇒ ?= 3 (24 × 24 × 24) × (3 × 3 × 3) × (2 × 2 × 2) ( x +1)( x – 1)
H
=4
[ 8 = 2 × 2 × 2] ( x +1)
[∴ a2 – b2 = (a + b) (a – b)]
_T

⇒ ?= 3
(24)3 × (3)3 × (2)3
⇒ x–1 =4
⇒ ? = 24 × 3 × 2 ⇒ x=4+1 =5
⇒ ? = 144
25. Ans. (3)
C

Alternative Method Sol. Given


72 ? (7 × ?)2
=
PS

? 24 = 16
49
Consider option (1) : ? = 12
Let ? be x
12 1
RHS = = (7 x )2
U

24 2 ⇒ = 16
49
72 72 1
LHS = = ≠ 49 x 2

@

12 2 3 2 =4
49
∴ Option (1) is not the answer ⇒ x2 =4
Consider option (2): ? = 144 ⇒ x = ±2
144 ∴ x =2
RHS = =6
24 26. Ans. (3)
72 72 Sol. To find the least number that should be
LHS = = =6
144 12 subtracted from 427398 so that remaining
LHS = RHS number is divisible by 15, we shall first
∴ Option (2) is the answer divide 427398 by 15
3
23. Ans. (3) ⇒ 427398 ÷ 15= 28493
15
Sol. 18265 + 2736 + 41328 We can see that when 423798 is divided
= 62329 by 15, we get the remainder = 3
Unit One : Practice Session ✫ 89

∴ 3 is the least number that should be ⇒ x= 3 ; x = 5


subtracted from 427398 ⇒ ?=3 ; ?=5
[427398 – 3 = 427395 which is fully ∴ In the given options the value 3 only is
divisible by 15.] given.
∴ ?=3
27. Ans. (5)
Sol. Let x be the unknown number. 29. Ans. (1)
Given that when x is decreased by 4 and Sol. Given
divided by 6, the result is 8 39.05 × 14.95 – 27.99 × 10.12

TS
x–4 = (36 + ?) × 5
⇒ =8 Note: In such questions where
6
⇒ x – 4 = 48 approximate value has to be determined,
⇒ x = 48 + 4 first convert the given decimal numbers

H
⇒ x = 52 to whole numbers and then calculate
When 2 is subtracted from x and then it ⇒ 39 × 15 – 28 × 10 = (36 + ?) × 5

G
is divided by 5, we get Using BODMAS rule, we get,
52 − 2 50 585 – 280 = 36 × 5 + 5 × ?
=
5 5
= 10
U ⇒

305 = 180 + 5 × ?
5 × ? = 305 – 180
O
28. Ans. (4) ⇒ 5 × ? = 125
Sol. Given 125

H
128 ÷ 16 × ? – 7 × 2 ? =
5
7 2 – 8 × 6 + ?2 ⇒ ? = 25
_T

128 ∴ The approximate value of ? = 25


×? – 7 ×2
⇒ 16 =1
49 – 8 × 6 + ? 2 30. Ans. (4)
Cancel out as many common terms as 4 4 2
Sol. ×2 ÷ =?
C

possible 7 5 5
8 ×? – 7 ×2 4 14 5 4
PS

⇒ × = ×7=4
49 – 8 × 6 + ? 2 = 1 7 5 2 7
Using BODMAS rule, we get 31. Ans. (3)
8 × ? – 14 Sol. – 56 × 61 + 89 = ?
U

⇒ =1
49 – 48 + ? 2 – 3416 + 89 = –3327
8 × ? – 14
@

⇒ =1 32. Ans. (2)


1 + ?2 Sol. 3625 × ? = 1450
⇒ 8 × ? – 14 = 1 + ?2 1450 2
?= =
⇒ ?2 – 8? = –14 – 1 3625 5
Let ? be x 33. Ans. (1)
⇒ x2 – 8x = – 15 Sol. 916 × ? × 3 = 214344
⇒ 2
x – 8x + 15 = 0 214344
?=
⇒ x2 – 3x – 5x + 15 = 0 916 × 3
[∴ Splitting the middle term] ? =7 8
⇒ x (x – 3) – 5 (x – 3)= 0 34. Ans. (3)
⇒ (x – 3) (x – 5) = 0 Sol. (54679 + 5982 + 32614) – (312 × 69) = ?
⇒ (x – 3) = 0 ; (x – 5) = 0 93275 – 21528 = 71747
90 ✫ Quantitative Aptitude

35. Ans. (4) 40. Ans. (5)


Sol. Let the even numbers be x, x+2 and x + 4
Sol. ? = (88 × 42) ÷ 16
x+x+2+x+4
88 × 42 Sol. x + x + 2 + x + 4 = 44 +
= = 231 3
16 3(3x + 6) = 132 + 3x + 6
∴ ? = (231)2 = 53361 6 x = 120
36. Ans. (5) x = 20
Sol. 41 + 4.11+ 0.41 + 411 + 41.11 = ? The third largest number is 20

? = 497.63 41. Ans. (2)

TS
Sol. 3 = (756 × 67) ÷ 804
?
37. Ans. (3)
756 × 67
5 2 4 6
× ÷ ÷ =?
= = 63
Sol. 804

H
6 9 9 7
5 2 9 7 35
3
? = 63
× × × = ∴ ? = (63)3 = 250047
6 9 4 6 72

G
1 42. Ans. (5)
This is nearly or .50
2 Sol. 0.3 + 3 + 3.33 + 3.3 + 3.03 + 333 = ?
So option (3) is correct.
(The actual value is 0.486 which is
U 345.96 = ?
43. Ans. (4)
O
approximately 0.49)
Sol. (73425 – 33267 – 22418 – 17650) × 11025
38. Ans. (2) ? = 90 × 105
H
Sol. 754.002 + 3.00024 × 88.976 = ? ? = 9450
? = 754.002 + 266.949 = 1020.951 44. Ans. (1)
_T

Sol. –76 × 33 + 221


= 1021 (approximate)
= –2508 + 221
39. Ans. (5) = –2287
? 48
C

Sol. = 45. Ans. (3)


432 ?
⇒ (?)2 = 48 × 432 = 12 × 4 × 12 × 36 Sol. (34.12)2 – 7396
PS

= 1164.1744 – 86
∴ ? = 12 2 × 4 × 36 = 12 × 2 × 6 = 144 = 1078.1744
U
@
Unit One : Practice Session ✫ 91

Practice Exercise 7
1. Which one of the following is a factor of (a) 6 (b) 5
the sum of the first twenty-five natural (c) 3 (d) 2
numbers? 12. Which smallest number must be added to
(a) 2 6 (b) 2 4 2203 so that we get a perfect square?
(c) 1 3 (d) 1 2 (a) 1 (b) 3
2. Find the average of the squares of the first (c) 6 (d) 8
ten natural numbers.

TS
13. The value of:
(a) 35.5 (b) 3 6
(c) 37.5 (d) 38.5 1 1 2
- - is
3. What is the sum of all even numbers between (12 - 140) (8 - 60) (10 + 84)

H
21 and 51? (a) 0 (b) 1
(a) 518 (b) 540 (c) 2 (d) 3

G
(c) 580 (d) 598 14. Which smallest number must be added to
4. Find the average of odd numbers upto 100. 710 so that the sum is a perfect cube?
(a) 4 9
(c) 5 0
(b) 49.5
(d) 50.5
U (a) 2 9
(c) 11
(b) 1 9
(d) 2 1
O
5. What is the value of 12 + 22 + 32 + . . . . +102? 15. 8.31 + 0.6 + 0.002 is equal to
(a) 390 (b) 380
(c) 385 (d) 395 (a) 8.912 (b) 8.912
H
6. The sum of a positive integer and its square (c) 8.979 (d) 8.979
_T

is 2450. What is the positive integer? 2 3


(a) 4 8 (b) 4 9 16. The value of ´ is
3 5 ¸ 2 of 1 1
(c) 5 0 (d) 4 5 6 3 4
é (0.73)3 + (0.27)3 ù (a) 2 (b) 1
C

7. êê ú simplifie s
ú 1 2
ë (0.73) 2
+ (0.27) 2
- (0.73) ´ (0.27) û (c) (d)
2 3
PS

to:
(a) 1 (b) 0.4087 17. The value of 3
24 2-5 2 6 is
(c) 0.73 (d) 0.27 5
(a) 2 (b) 2 3
U

8. If the difference of two numbers is 3 and the


difference of their squares is 39, then the (c) 2 5 (d) 1
larger number is: 18. Given: 13 + 23 + 33 + … + 103 = 3025 then
@

(a) 8 (b) 9
23 + 43 + 63 + … + 203 is equal to
(c) 1 2 (d) 1 3
(a) 6050 (b) 9075
9. éê 3 2 ´ 2 ´ 3 3 ´ 3 ùú is equal to: (c) 12100 (d) 24200
ë û 5
(a) 6 5 (b) 6 6 5+1 5-1
19. If a = and b = , then the value
(c) 6 (d) None of these 5 -1 5 +1
10. The sum of three different even numbers, a2 + ab + b 2
each greater than 15, is 58. Find one of them. of is
a2 - ab + b 2
(a) 1 6 (b) 1 8 3 4
(c) 2 0 (d) 2 2 (a) (b)
4 3
11. The smaller positive integer, when
3 5
multiplied by 392 (the product is a perfect (c) (d)
square) is: 5 3
92 ✫ Quantitative Aptitude

20. The value of: 29. A divisor is 25 times the quotient and 5 times
0.2 ´ 0.2 ´ 0.2 + 0.04 ´ 0.04 ´ 0.04 the remainder. If the quotient is 16, the
3
is dividend is
0.4 ´ 0.4 ´ 0.4 + 0.08 ´ 0.08 ´ 0.08
(a) 6400 (b) 6480
(a) 0.5 (b) 0.25
(c) 400 (d) 480
(c) 0.75 (d) 0.125
30. Out of three numbers, the first is twice the
21. The greatest of 2 , 6 3 , 3 4 , 4 5 is second and is half of the third. If the average
(a) 2 (b) 6 3 of the three numbers is 56, then difference of
(c) 3 4 (d) 4 5 the first and third numbers is

TS
(a) 1 2 (b) 2 0
22. A number divided by 13 leaves a remainder (c) 2 4 (d) 4 8
1 and if the quotient, thus obtained, is
divided by 5, we get a remainder of 3. What 31. The smallest natural number, by which 3000

H
will be the remainder if the number is must be divided to make the quotient a
divided by 65? perfect cube, is
(a) 3 (b) 4

G
(a) 2 8 (b) 1 6
(c) 1 8 (d) 4 0 (c) 5 (d) 6
32. Given that 5 = 2.236 and 3 = 1.732, the
23. What is the greatest number that will divide
307 and 330 leaving remainders 3 and 7
respectively? U value of
1
5+ 3
is
O
(a) 0.504 (b) 0.252
(a) 1 9 (b) 1 6
(c) 0.362 (d) 0.372
(c) 1 7 (d) 2 3
H
7 -2
24. If a = 4.36, b = 2.39 and c = 1.97, then the 33. If = a 7 + b , then the value of a is
7 +2
value of a3 – b3 – c3 – 3 abc is 11 4
_T

(a) 3.94 (b) 2.39 (a) (b) -


3 3
(c) 0 (d) 1 4 4
(c) (d) - 7
25. What is the least number which when 3 3
C

divided by the numbers 3, 5, 6, 8, 10 and 12 34. 8 - 2 15 is equal to


leaves in each case a remainder 2 but when
(a) 5+ 3 (b) 5 - 3
PS

divided by 13 leaves no remainder?


(a) 312 (b) 962 (c) 5- 3 (d) 3 - 5
(c) 1562 (d) 1586 æ 2.75 ´ 2.75 ´ 2.75 - 2.25 ´ 2.25 ´ 2.25 ö÷
35. ççç ÷ is equal
è 2.75 ´ 2.75 + 2.75 ´ 2.25 + 2.25 ´ 2.25 ø÷
U

26. Which of the following numbers is NOT


divisible by 18? to
(a) 54036 (b) 50436 (a) – 5 (b) 0.5
@

(c) 34056 (d) 65043 (c) –0.5 (d) 5


(6.25)1/2 ´ (0.0144)1/2 + 1 36. If the sum of the squares of three consecutive
27. is simplified to natural numbers is 110, then the smallest of
(0.027)1/3 ´ (81)1/4
these natural numbers is
(a) 0.14 (b) 1.4 (a) 8 (b) 6
(c) 1 (d) 1.4 (c) 7 (d) 5
14 é 3 2 4 3 6 ùú
28. The product of two fractions is and their 37. êê - + is simplified
35
15
ëê 6 + 3 6+ 2 3 + 2 úúû
quotient is . The greater of the fractions to
24
is (a) 0 (b) 1
7 7 (c) 3 (d) 6
(a) (b)
4 6 1 1 1
7 4 38. If x 3 + y 3 = z 3 , then {(x + y – z)3 + 27 xyz}
(c) (d) equals
3 5
Unit One : Practice Session ✫ 93

(a) – 1 (b) 1 40. The number of trees in each row of a garden


(c) 0 (d) 2 7 is equal to the total number of rows in the
æ 999 ö garden. After 111 trees have been uprooted
39. çç999 ´ 7 ÷÷÷ is equal to
çè 1000 ø in a storm, there remain 10914 trees in the
7 7 garden. The number of rows of trees in the
(a) 6993 (b) 7000 garden is
1000 1000
(a) 100 (b) 105
7 993
(c) 6633 (d) 6999 (c) 115 (d) 125
1000 1000

TS
Answers with Solutions
1. Ans. (c)
n (n + 1) Alternative Method
Sol.  1 + 2 + 3 + . . . . + n =

H
2 22 + 24 + 26 + . . . . + 50
∴ 1 + 2 + 3 + . . . . + 25 = 2 (11 + 12 + 13 + . . . . + 25)
25 (25 + 1) = 2 [1 + 2 + 3 + . . . . + 25)

G
=
2 – (1 + 2 + 3 + . . . + 10)]
25 (26) 25 × 26 10 × 11
=
2
= 25 × 13 U =2
2

2
O
Hence, 13 is a factor of the required = 2 (325 – 55)
sum. = 2 × 270 = 540
H
2. Ans. (d) 4. Ans. (c)
Sol. Average of the first n natural odd
12 + 2 2 + 32 + . . . . + n 2
_T

numbers = n
Sol.
n Number of odd numbers upto 100
(n + 1) (2n + 1) 100
= = = 50
2
C

6 5. Ans. (c)
12 + 23 + 33 + . . . . + 102 Sol. 12 + 22 + 32 + . . . . + n2

PS

10 n (n + 1) (2n + 1)
=
(10 + 1) (2 × 10 + 1) 6
= ∴ 12 + 22 + 32 + . . . . + 102
6
11 × 21
U

= 10 (10 + 1) (2 × 10 + 1)
6 =
6
@

=
231
= 38.5 10 × 11 × 21
=
6 6
3. Ans. (b) 2310
Sol. Sum of first n even numbers = = 385
6
= n (n + 1)
6. Ans. (b)
∴ Required sum = Sum of 25 even
Sol. Let the positive integer be x.
numbers from 1 to 50 – sum of 10 even
Then,
numbers from 1 to 20 x2 + x = 2450
= 25 × (25 + 1) – 10 (10 + 1) x (x + 1) = 2450
= 25 × 26 – 10 × 11 x (x + 1) = 49 × 50
= 650 – 110 x = 49
= 540 Hence, the positive integer is 49.
94 ✫ Quantitative Aptitude

7. Ans. (a) 13. Ans. (a)


(0.73)3 + (0.27)3
Sol. (0.73)2 + (0.27)2 - (0.73) ´ (0.27) 1 1
Sol. -
(12 - 140) (8 - 60)
æ a 3 + b 3 = ( a + b) ( a 2 - ab + b 2 )ö÷
çç ÷ 2
çç Here a = 0.73 and b = 0.27 ÷÷ -
è ø ...(i)
(10 + 84 )
a +b 3 3
1
a + b 2 − ab
2 Consider and rationalising, we
12 - 140
get

TS
( a + b ) ( a 2 + b 2 − ab)
= 1 12 + 2 35
a 2 + b 2 − ab ´
= a + b = 0.73 + 0.27 = 1 12 2 35 12 + 2 35
-
( 140 = 2 ´ 5 ´ 2´ 7 = 2 35 )

H
8. Ans. (a)
Sol. Let one number be x and the other be 12 + 2 35 12 + 2 35
=

G
x+3 =
144 - 140 4
(x + 3)2 – x2 = 3 9 2
( 7 )2 + ( 5)2 + 2 7 . 5 æç 7 + 5 ö÷
x + 6x + 9 – x2 = 39
2

6x = 30
U =
22
= çç
çè 2
÷÷
÷ø
O
x =5 æ 7 + 5 ÷ö
2
7+ 5
= ççç
1
∴ The larger number is 5 + 3 = 8 ∴ ÷÷ =
ç
è 2 ÷
ø 2
12 - 140
H
9. Ans. (b)
Similarly rationalising the second and third
1 1 1 1
2 ´ 2 ´ 3 ´ 3 = 2 ´ 2 ´3 ´ 3 terms, we get
_T

Sol. 3 3 3 2 3 2

1 5+ 3 é 2ù
( 5 + 3) ú
1 1 1 1 5 5
= 23 + 2 ´ 3 3 + 2 = (2´3) 6 = 66 = ê 8 + 60 =
8 - 60 2 ë û
10. Ans. (a)
C

1 7- 3
Sol. The even numbers greater than 15 are and =
10 + 84 4
PS

16, 18, 20, 22, 24, …


é 2ù
Three different even numbers whose ê 10 - 84 = 10 - 2 21 = ( 7 - 3 ) ú
ë û
sum is 58 can be 16, 20, 22 or 16, 18, 24
7+ 5 5+ 3 æ 7 - 3 ö÷
∴ One of them must be 16. - 2 ççç
U

∴ Eqn. (i)  - ÷÷
2 2 çè 4 ÷ø
11. Ans. (d)
@

7 5 5 3 7 3
Sol. From the given options, 392 × 2 = 784 + - - - + =0
2 2 2 2 2 2
which is a perfect square.
14. Ans. (b)
12. Ans. (c)
46 Sol. The number nearest to 710 which is a
Sol. perfect cube is 729.
4 2203
16 Then, Required number = (9)3 – 710
86 603 = 729 – 710 = 19
516 ∴ The number to be added is 19.
87 15. Ans. (c)
462 < 2203 < 472 Sol. 8.31 + 0.6 + 0.002
∴ Required number = 472 – 2203 31 - 3 28
= 8.31 = 8 + .31 = 8 + =8
= 2209 – 2203 = 6 90 90
Unit One : Practice Session ✫ 95

6 2 Similarly,
0.6 = and .002 = 4 5
9 900 a–b = = 5
4
ab = 1
∴ 8 28 + 6 + 2 = 8 280 + 600 + 2
90 9 900 900 a 2 + ab + b 2 ( a + b )2 - ab
∴ =
882 979 - 97 a 2 - ab + b 2 ( a - b)2 + ab
= 8 = 8+ = 8.979
900 900 9-1 8 4
= = =
16. Ans. (a) 5 +1 6 3
2 3 2 3

TS
Sol. ´ = ´ 20. Ans. (a)
3 5 2 1 3 5 2 5
¸ of 1 ¸ ´
6 3 4 6 3 4 0.2 ´ 0.2 ´ 0.2 + 0.04 ´ 0.04 ´ 0.04
Sol. 3 … (i)
2 3 0.4 ´ 0.4 ´ 0.4 + 0.08 ´ 0.08 ´ 0.08

H
= ´
3 5 6 Let 0.2 = x then 0.4 = 2x and 0.04 = y
´
6 5 then 0.08 = 2y

G
2
= × 3 = 2 ∴ From equation (i)
3
17. Ans. (a)

Sol. 3 2 4 2-5 26 U  3
x3 + y3
(2 x )3 + (2 y )3
= 3
1 (x 3 + y3 )
8 (x + y ) 2
3 3
1
= = 0.5
O
3 6 21. Ans. (c)
= 2 4 2- 5 ´ 2 2
= 3
2 4 2-2
H
6 3 4
Sol. 2 , 3, 4, 5
4
1 2
4 = 2 = 2 =2
4 3 3
= 3 2 3
L.C.M. of 2, 6, 3, 4 is 12
_T

2 = 12 26 ; 6
3 = 12 3 2 ; 3
4 = 12 4 4 ; 4
5 = 12 53
18. Ans. (d)
12 12 12 12
Sol. 13 + 23 + … + 103 = 3025 … (i) 64 ; 9; 256 ; 125
C

23 + 43 + 63 + … + 203 The greatest value is 12


256 = 3
4
= 23 (13 + 23 + 33 + … + 103)
PS

22. Ans. (d)


= 8 × 3025 [from (i)] Sol. Let the number be x
= 24200 Dividend = Divisor × Quotient
+ Remainder
U

19. Ans. (b)


x = 13 × Q + 1
5 +1 5 -1 = 13 (5x + 3) + 1
Sol. a = , b=
@

5 -1 5 +1 = 65x + 39 + 1 = 65x + 40
∴ When the number is divided by 65 the
5 +1 5 −1
a+b = + remainder is 40
5 −1 5 +1 23. Ans. (a)
Sol. The greatest number that will divide
( ) ( 5 − 1)
2 2
5 +1 + 307 and 330 leaving remainders 3 and
=
( 5 − 1) ( 5 + 1) 7 is H.C.F. of (307 – 3) and (330 – 7)
(i.e.) H.C.F. of 304 and 323 which is 19.
( 5 + 1)2 + ( 5 - 1)2 24. Ans. (c)
=
5 -1 Sol. a = 4.36; b = 2.39; c = 1.97
5 + 1 + 2 5 + 5 + 1- 2 5 If a + b + c = 0, then a3 + b3 + c3 – 3abc = 0
= =3 If a = 4.36; –b = –2.39; –c = –1.97 then
4
96 ✫ Quantitative Aptitude

a + (–b) + (–c) = 4.36 – 2.39 – 1.97 = 0 30. Ans. (d)


∴ a3 +(–b)3 + (–c3) – 3(a) (–b) (–c) = 0 Sol. Let the first number be x, then the
x
second number is and the third number
25. Ans. (b) 2
is 2x
Sol. L.C.M. of 3, 5, 6, 8, 10, 12 is 120 x
x + + 2x
120 × 1 + 2 = 122 is not divisible by 13 ∴ 2 = 56
120 × 2 + 2 = 242 is not divisible by 13 3
2x + x + 4x = 336
120 × 8 + 2 = 962 is divisible by 13 x = 48

TS
26. Ans. (d) Difference of the first and third number
Sol. If a number is divisible by 18, it is is 2x – x = x = 48
divisible by 2 and 9. From the given 31. Ans. (a)

H
options, 65043 is odd and not divisible Sol. 3000 = 2 × 2 × 2 × 3 × 5 × 5 × 5
by 2. = 23 × 53 × 3

G
27. Ans. (d) 3000
Thus, = 1000 = 103
(6.25)1/2 ´ (0.0144)1/2 + 1 3
Sol. (0.027)1/3 ´ (81)1/4
2.5 ´ 0.12 + 1 0.3 + 1 U Therefore, 3000 should be divided by
3, so that the result is a perfect cube.
O
= = 32. Ans. (b)
0.3 ´ 3 0.9
1.3 1 1 ( 5 - 3)
H
= = 1.4 Sol. = ´
0.9 5 + 3 ( 5 + 3) ( 5 - 3)
28. Ans. (b)
_T

5 - 3 2.236 - 1.732
Sol. Let the two fractions be x and y, then =
5-3 2
14 x 35
xy = … (i) and = … (ii)
15 y 24
C

0.504
x 14 35 = = 0.252
(i) × (ii)  xy × = ´ 2
PS

y 15 24
49 7 33. Ans. (b)
x2 =  x=
36 6 7 -2
= a 7 +b
U

14 6 4 7 +2
∴ y = ´ =
15 7 5
( 7 - 2) ( 7 - 2)
@

7 4 ( 7 + 2) ( 7 - 2)
= a 7 +b
∴ >
6 5
29. Ans. (b) 7 +2-4 7
= a 7 +b
Sol. Dividend = Divisor × Quotient 7-4
+ Remainder 9-4 7 4
= a 7 +b ⇒ a = -
Dividend = 25 × 16 × 16 + Remainder 3 3
… (i) 34. Ans. (c)
Also Divisor = 5 × Remainder Sol. Let 8 - 2 15 = a - b
25 ´16 Squaring both sides we get
= Remainder
5 8 - 2 15 = a + b - 2 ab
80 = R ⇒ a+b=8 … (i) and
∴ (i)  Dividend = 6400 + 80 = 6480
ab = 15
Unit One : Practice Session ✫ 97

⇒ ab = 15 … (ii) 3 2 ( 6 - 3) 4 3 ( 6 - 2)
-
Solving (i) and (ii) 3 4
15 6 ( 3 - 2)
a+ -8 = 0 +
a (rationalising)
1
⇒ a2 – 8a + 15 = 0 1 é
24 3 - 12 6 - 36 2 + 12 6 + 36 2 - 24 3 ùú
(a – 5) (a – 3) = 0 12 ëê û

⇒ a = 5, 3 and b = 3, 5 = 0

TS
But a > b, so a = 5, b = 3 38. Ans. (c)
1 1 1
∴ 8 - 2 15 = 5 - 3 Sol. x 3 + y 3 = z 3
35. Ans. (b) Cubing both sides,

H
1 1 1 1
2.75 ´ 2.75 ´ 2.75 - 2.25 ´ 2.25 ´ 2.25 x + y + 3x 3 y 3 (x 3 + y 3 ) = z
Sol.
2.75 ´ 2.75 + 2.75 ´ 2.25 + 2.25 ´ 2.25

G
1 1 1

ì
ï ïü x + y + 3x 3 y 3 z 3 = z
ï a -b
3 3

= 2.75 – 2.25 í 2 = a -bïý


ï + + 2
ïþï 1 1 1

= 0.5
ï
î a ab b

U x + y – z = -3x 3 y 3 z 3
Cubing both sides,
O
36. Ans. (d)
(x + y – z)3 = –27xyz
Sol. Let the numbers be x, x + 1, x + 2
(x + y – z)3 + 27xyz = 0
H
x2 + (x + 1)2 + (x + 2)2 = 110
39. Ans. (d)
x2 + x2 + 2x + 1 + x2 + 4x + 4 = 110
_T

999 999999 ´ 7
3x2 + 6x + 5 = 110 Sol. 999 ´7 =
1000 1000
3x2 + 6x – 105 = 0
6999993 993
C

⇒ x2 + 2x – 35 = 0 = = 6999
1000 1000
⇒ (x + 7) (x – 5) = 0 40. Ans. (b)
PS

⇒ x=5 Sol. Total number of trees


37. Ans. (a) = 10914 + 111 = 11025
Since number of rows = number of
U

3 2 4 3 6
Sol. - + trees, the required answer is 11025
6+ 3 6+ 2 3+ 2
= 105 rows of trees
@
98 ✫ Quantitative Aptitude

UNIT TWO

✫ PERCENTAGE ✫ AVERAGE ✫ RATIO AND PROPORTION

TS
✫ PARTNERSHIP ✫ PROFIT AND LOSS ✫ ALLIGATION
✫ PROBLEMS ON AGE

FUNDAMENTALS AND

H
FAST-TRACK FORMULAE

G
Percentage
The term per cent means ‘for every hundred’ U ● If 8% of a number is 32, what is the
number?
Here we have to find the base which is
O
or ‘out of hundred’. A fraction whose
denominator is 100 is called a percentage, and Percentage value
Rate %
H
the numerator of the fraction is called the rate
per cent. The term ‘per cent’ is denoted by the 32
= × 100 = 400.
symbol ‘%’. 8
_T

1
1 per cent means 1 part out of 100 or . Changing Rate Per cent to a Decimal
100
25 1 and a Decimal to a Per cent
25 per cent means 25 parts out of 100 or = . (i) To change a per cent to a decimal, remove
100 4
C

So per cent is really a fraction whose denominator the per cent symbol (%) and divide by 100.
is 100, and the numerator of this fraction is the Fast-track Method
PS

‘rate per cent’.


Percentage is important in interpreting the You just need to move the decimal point
data given in graphs and tables. It is necessary two places (for the two zeros in 100) to the
to understand clearly how to work it out. left, instead of actually dividing by 100. So
U

24
Percentage (value) = Rate % × Base number 24% = = 0.24
100
@

Percentage (value) (24 is actually 24.0, so shifting the decimal


Base number = left gives 0.24)
Rate % If it is a single digit, you need to add a
Percentage value zero to the left of that digit (so that it
Rate % =
Base number becomes a two-digit number) and place the
decimal point.
● What per cent is 16 of 28? 9
Here, base number = 28 9% = = 0.09
and percentage = 16 100
Now what about 351%?
The rate % is to be found.
351% = 3.51
Percentage and 1% = 0.01
Rate = [Do not confuse 0.01 with 0.01%. The per
Base number
cent symbol is significant.
16 1 0.01 is 1%; 0.01% = 0.0001.]
= = 57 %.
28 7
98
Unit Two : Fundamentals / Percentage ✫ 99

(ii) To change a decimal to a per cent,


Remember these fractions and their
multiply by 100.
equivalent per cents . . .
● 0.03 = ?% 1 1
1 = 100% = 50% = 33.33%
Let x% = 0.03 2 3
1 1 1
x = 25% = 20% = 16.67%
Then, = 0.03 4 5 6
100 1 1 1
x = 0.03 × 100 = 12.5% = 11.11% = 10%
8 9 10

TS
x = 3 1 1 1
= 9.09% = 8.33% = 6.25%
or 0.03 is 3% 11 12 16
1 1 1
= 5% = 4% = 2%

H
Fast-track Method 20 25 50
2 2 3
Here the shortcut is to move the decimal = 66.67% = 40% = 60%

G
3 5 5
point two places to the right.
So 0.03 becomes 3%
Expressing a Quantity as Percentage of
0.734 becomes 73.4%
0.7 becomes 70% U another Quantity
O
(As two places of decimal are required (v) If x and y are two quantities, and you
are asked to find what percentage of y is x,
when 100 is the multiplier, 0.7 becomes 70.)
you use the formula,
H
x
Required percentage = × 100%.
y
_T

Changing a Fraction to Rate Per cent (The units of x and y need to be the same;
and Rate Per cent to a Fraction if not, they should be converted to the same
a units if possible, otherwise comparison is not
(iii) To change a fraction to per cent,
C

b to be made.)
multiply it with 100 and put a % sign.
If you are asked what per cent of 150 is
3
PS

For example, = ? per cent 30, you would have x = 30 and y = 150.
12
30
3 So, × 100 = 20% is your answer.
It is × 100 = 300 = 25% 150
12 12
U

Or you could convert the fraction to a (vi) If two values are respectively x% and
decimal and then multiply by 100. y% more than a third value, then the first is
@

3 100 + x
= 0.25 × 100 % of the second and the second
12 100 + y
0.25 × 100 = 25%.
100 + y
is × 100 % of the first.
(iv) To convert a per cent into a fraction, 100 + x
put the given per cent over 100, removing the
● Two numbers are respectively 32% and
symbol %, and simplify if required.
65% more than a third number. What
per cent is the first of the second?
10 1
10% = = Here, x = 32 and y = 65
100 10
∴ First number
14 7 100 + x
14% = =
100 50 = 100 + y × 100% of the second
100 ✫ Quantitative Aptitude

100 + 32 (ix) If A is x% more than B, then B is less


= × 100% than A by
100 + 65 x
× 100 %
132 × 100 100 + x
= %
165
= 80% ● Gopal’s salary is 16% more than
Hence, the first number is 80% of the Mohan’s. By how much per cent is
second. Mohan’s salary less than that of Gopal?

TS
(vii) If two values are respectively x% and
Here, x = 16
y% less than a third value, then the first is
Let Mohan’s salary be Rs 100/month.
100 − x Then, using the formula
× 100 % of the second and the second
100 − y x

H
× 100 %
100 + x
100 − y
is × 100 % of the first.

G
100 − x 16
× 100 %
We have
100 + 16
● Two numbers are respectively 36% and
22% less than a third number. What per
cent is the first of the second? U =
16 × 100
116
%
O
Here x = 36 and y = 22
= 13.79%
∴ First number
H
Mohan’s salary is 13.79% less than
100 − x Gopal’s.
= × 100% of the second
100 − y
_T

100 − 36 (x) If A is x% less than B then B is more


= × 100% than A by
100 − 22 x
× 100 %
C

64 × 100 100 − x
= %
78 ● If Sudha’s salary is 36% less that of
PS

= 82.05% Rekha’s then how much per cent is


Hence, the first number is 82.05% of the Rekha’s salary more than that of Sudha?
second.
U

Here, x = 36
(viii) If A is x% of C and B is y% of C,
Let Rekha’s salary be Rs 100 a month.
x
@

then A is × 100 % of B. Then, using the formula


y
x
● If A is 28% of C and B is 35% of C, × 100 %
100 − x
then what percentage is A of B?
Here, x = 28 and y = 35 36
We have × 100 %
100 − 36
x
A = × 100% of B
y 36 × 100
= %
28 64
= × 100% of B
35
= 56.25%
2800
= % of B Rekha’s salary is 56.25% more than
35
Sudha’s.
= 80% of B.
Unit Two : Fundamentals / Percentage ✫ 101

Percentage Change and its Effect xy


x−y− % increase or decrease, according
(xi) If a quantity or number is increased 100
by x%, then the to the positive and negative sign respectively.
New quantity (number)
● The salary of Sashi is first increased by
100 + x 10 per cent and then decreased by 8 per
= Original quantity ×
100 cent. What was the net change in Sashi’s
[or = original quantity × 1 + decimal equivalent salary?
of x%] If the first change is x, and the second
If a quantity or number is decreased by x%, the

TS
change is y, using the formula
100 − x xy
New quantity = original quantity × . x−y− %
100 100

H
[or = original quantity × 1 – decimal equivalent 10 × 8
of x%] We have 10 − 8 − %
100

G
● The salary of Rasi is Rs 8000 at present. 80
It will increase by 8% in the next year. = 2− %
100
What will Rasi’s new salary be?
If salary increase is by 8%, its decimal U =
120
100
% = 1.2%
O
equivalent is .08. The net change in Sashi’s salary is an
100 + 8 increase by 1.2 per cent.
∴ The new salary = 8000 ×
H
100 [Here, the negative and positive signs
Or 8000 × (1 + 0.08) must be carefully noted. If, for example,
_T

= 8000 × 1.08 the decrease had been 12 per cent, we


would have had
= Rs 8640
[If the salary had been decreased by 8%, 10 × 12
10 − 12 − %
C

the new amount would be 8000 × (1 – 0.08) 100


= 8000 × 0.92 = Rs 7360.] = (10 – 12 – 1.2)%
= –3.2%
PS

(xii) If the value of a number is first So there would be a net decrease of 3.2
increased by x%, and later decreased by x%, the per cent in Sashi’s salary.]
net change is always a decrease which is equal
● Tax on a commodity is increased by
U

x2
to x% of x or . 18% and consumption decreases by
100 12%. Find the effect on revenue.
@

● The salary of a worker was first


increased by 10% and then it was Here, x = 18 and y = 12
reduced by 10%. What was the net Tax × consumption = Revenue
change in his salary? ∴ Net % change in revenue
xy
Here, increase and decrease per cent = x−y− %
x = 10 100
x2 18 × 12
Using the formula % = 18 − 12 − %
100 100
2
(10)
We have = 216
100 = 6− %
100
= 1% decrease.
(xiii) If the value is first increased by x% 600 − 216
= %
and then decreased by y% then there is 100
102 ✫ Quantitative Aptitude

384 (xv) A specific formula for finding the


= % = 3.84% percentage area increase if the sides of a two-
100 dimensional figure are increased by x% is
Hence, the revenue increases by 3.84%.
x + 200
(xiv) If the value is increased successively Increase in area = x %
by x% and y% then the final increase is given 100
x2
xy Or 2x + %
by x + y + %. 100
100
● If the sides of a rectangle are increased

TS
● The salary of Sam is increased first by by 20%, what is the percentage increase
8 per cent and then by 10 per cent. If in its area?
Sam’s original salary was Rs 8000 what
is his salary after the successive Here, x = 20

H
increase? Then, using the formula
x2
If x = 8% and y = 10%, % increase area = 2x + %

G
100
8 × 10
the final increase = 8 + 10 + % (20)2
100 = 2 × 20 + 100 %

= 8 + 10 +
80
% U 400
O
100 40 + %
=
= (8 + 10 + 0.8)% 100
H
= (40 + 4)%
= 18.8%
= 44%.
Now 18.8% of Rs 8000 = Rs 1504
_T

The salary after the increases is (xvi) If the product of two variable quantities
Rs 8000 + Rs 1504 = Rs 9504 is to be kept fixed, variable must decrease while
the other increases. The net percentage change
The formula is used also to get the on the product is zero.
C

percentage effect on the area of a square/ So if one quantity A increases by x%, the
rectangle/triangle/circle, when the sides/ x
PS

radius are/is increased or decreased, as other quantity B decreases by × 100%.


100 + x
side one × side two = area If A decreases by x%, then B increases by
or radius × radius = area
x
U

× 100%.
● If the side of a square is increased by 100 − x
25%, find the percentage increase in These formulae can be used to find out
@

area. (a) percentage change in either rate or


Area of square = side × side consumption when expenditure is to
Here, x = 25, y = 25 be kept unchanged because expenditure
Using the formula = rate × consumption;
xy (b) percentage change in either length or
Net % change in area = x + y + % breadth of a rectangle whose area is
100
to be kept unchanged, as area = length
25 × 25 × breadth;
We have 25 + 25 + %
100 (c) percentage change in either speed or
25 time, when distance is unchanged, as
= 50 + % distance = speed × time.
4
= (50 + 6.25)% ● If the price of honey increases by 32%,
= 56.25%. find by how much per cent its
Unit Two : Fundamentals / Percentage ✫ 103

consumption should be reduced so as Using equation (1) we have


not to increase the expenditure. Expenditure
± change in quantity
Here, x = 32 x
Reduction in consumption Expenditure
=
New rate
x (+ indicates increase; – indicates decrease
= × 100 %
100 + x in quantity due to rate change; x is original rate
per unit quantity.)
32
= × 100 % ● A reduction of 20% in the price of sugar

TS
100 + 32
enables a purchaser to obtain 3 kg more
32 × 100 for Rs 120. What the original price of
= %
132 sugar per kg?

H
= 24.24%. Let the original rate = Rs x per kg. As
there is a rate reduction of 20%, the new

G
● If the price of almonds decreases by (or reduced) rate = (1 – 0.20) x = 0.80x
20%, find by how much per cent its Expenditure = Rs 120
consumption can be increased so as not
to decrease the expenditure.
U Using the formula
Expenditure
+ change in quantity
O
Here, x = 20 x
Expenditure
Increase in consumption =
New rate
H
x
= × 100 % 120 120
100 − x +3 =
x 0.80x
_T

20 (+ 3 as quantity available increases on


= × 100 %
100 − 20 rate reduction)
20 × 100 120
C

= % + 3 × 0.80x= 120
80 x
96 + 2.4x = 120
PS

= 25%
2.4x = 120 – 96 = 24
(xvii) If the original rate of an item is x 24
x = = 10
per unit quantity, and the expenditure is fixed, 2.4
U

The original price of sugar was Rs 10 per kg.


Expenditure
Quantity of the item = . . . (1)
@

x Alternative Method
If x changes to a new rate, but expenditure Here, let’s have reduction in price (x) = 20%,
is fixed, with the change in rate the quantity amount (y) = Rs 120,
in equation (1) will also change accordingly. So, more sugar purchased (a) = 3 kg
due to change in rate Then, using the formula,
Original price of sugar
Expenditure xy
New quantity of item = = ,
New rate (100 − x ) a
Original quantity ± change in quantity 20 × 120
we have
(100 − 20) × 3
Expenditure
= 20 × 120
New rate = = Rs 10 per kg.
80 × 3
104 ✫ Quantitative Aptitude

(xviii) Per cent change = Percentage increase


New number of visitors
(New Amount) − (Original Amount)
× 100 =
– Original number of visitors
× 100
Original Amount Original visitors
Where a new amount is less than the 160 − 100
= × 100
original amount, the number on top will be a 100
negative number and the result will be a per 60 × 100
cent decrease. However, the negative sign is =
100

TS
omitted and per cent decrease is given as such.
= 60%.
If the new amount is greater, the per cent
change is positive and is called per cent Effect of Increase/Decrease in Stages

H
increase. As in the case of addition of fractions to a
● The price of tea being increased by whole or their subtraction from a whole,
percentage addition and subtraction in stages

G
20%, a man reduces his consumption by
cause changes in the balance according to a
20%. By how much per cent will his
pattern.
expenses for tea be decreased?
Let original consumption = 100 units U (xix) If x% of a quantity is spent on
first, y% of the remaining is spent on
the
the
O
and original price = Rs 100 per unit second and z% of the remaining is spent on
Original expenditure = Rs (100 × 100) the third person or item, and B is left in the
H
= Rs 10000 fund, then the original amount was
After increase in price, tea costs (100 +
B × 100 × 100 × 100
_T

20) = Rs 120 and consumption is


(100 − x ) (100 − y ) (100 − z )
reduced to (100 – 20) = 80 units.
New expenditure = Rs (120 × 80)
● Suresh spends 10% of his salary as
= Rs 9600
C

house rent, 15% of the rest on children’s


Expenditure %
(New expenditure) education and 10% of the balance on
PS

– (Original expenditure) clothes. After this expenditure, he is left


= × 100 with Rs 1377. What is Suresh’s salary?
Original expenditure
Here, amount left (B) = Rs 1377, x = 10,
(9600 − 10000)
U

= × 100 y = 15, z = 10
10000
Using the formula
@

400
= – × 100 B × 100 × 100 × 100
10000 Income =
= –4% or 4% decrease. (100 − x ) (100 − y ) (100 − z )

● The entry fee in an exhibition was 1377 × 100 × 100 × 100


=
Re 1. Later, this was reduced by 25% (100 − 10) (100 − 15) (100 − 10)
which increased the sales by 20%. Find
the percentage increase in the number 1377 × 100 × 100 × 100
of visitors. =
90 × 85 × 90
Let the total original sale be Rs 100. = Rs 2000.
Then, original number of visitors = 100
(xx) If x% of a quantity is added to an item,
120 y% of the increased quantity is added again,
New number of visitors = = 160
0.75 z% of the again increased quantity is added
Unit Two : Fundamentals / Percentage ✫ 105

and the amount becomes A, then the initial [For (xix) and (xx), if there are more than
amount is given by three items, say a, b, c, . . . n items on which
some of the balance was spent or to which
A × 100 × 100 × 100 some amount was added, the original amount
(100 + x ) (100 + y ) (100 + z ) would be
Balance (or New Amount) × 100n
● The manufacturer of an article makes
a profit of 25%, the wholesale dealer (100  a) (100  b) (100  c ).... (100  nth amount)
makes a profit of 20%, and the retailer with  indicating whether amount was
makes a profit of 28%. Find the subtracted or added at every stage.]

TS
manufacturing price of the article if the
Percentages and Scores/Marks
retailer sold it for Rs 48.
(xxi) If the pass marks in an examination
Here, A = Rs 48, x = 25, y = 20 and is x% and if a candidate who scores y marks

H
z = 28. fails by z marks, then the maximum marks,
Using the formula 100 ( y + z)
M=

G
A × 100 × 100 × 100 x
=
(100 + x ) (100 + y ) (100 + z ) ● In an examination, the pass marks is

Cost of manufacturing
48 × 100 × 100 × 100 U 40%. If a student scores 225 marks but
fails by 125 marks, find the maximum
marks.
O
= (100 + 25) (100 + 20) (100 + 28)
Here, x = 40, y = 225 and z = 125
48 × 100 × 100 × 100 Then, using the formula
H
= (125) (120) (128) 100 ( y + z)
Maximum marks (M) =
_T

= Rs 25. x
● A man divided his 35-hour work 100 (225 + 125)
1 =
40
schedule as follows: of the time—
5 100 (350)
C

1 =
sorting mail; of the time—filing 40
2
1 = 875.
PS

letters; of the time—answering


7
enquiries. The rest of the time he spent (xxii) A candidate scoring x% in an
in running errands. What is the examination fails by a marks while another
candidate who scores y% marks gets b marks
U

percentage of time spent on running


errands? more than the minimum required pass marks.
Then the maximum marks for that examination
@

The total time spent on sorting mail, filing are


letters, and answering enquiries is 100 ( a + b)
1 1 1 1 M=
+ + = 29 hours. The time left for y−x
5 2 7 2 ● In an examination, a candidate scores
1 1
running errands is 35 − 29 = 5 hours 20% and fails by 30 marks, while
2 2
Per cent of time spent on running another candidate who scores 32% gets
1 42 marks more than the minimum
5
errands = 2 required pass marks. Find the pass
35
mark.
11 1 11
= × = Here, x = 20, y = 32, a = 30 and b = 42
2 35 70
Using the formula
5 100 ( a + b )
= 15 % or approximately 16%. M =
7 y−x
106 ✫ Quantitative Aptitude

100 (30 + 42) Or, Population n years ago


= Original Population
32 − 20 = n
100 × 72 Annual Decrease
1–
= 100
12 P
= 600 = n
Hence, the pass mark is 600. r
1–
100
(xxiii) In an examination, x% failed in one ● The population of a town increases 10

TS
subject and y% failed in another. If z% students per cent annually. If its present
failed in both the subjects, the percentage of population is 1676000, what will it be
students who passed in both the subjects is in 2 years’ time?
100 – (x + y – z)

H
Here, P = 1676000, r = 10, n = 2
● In an examination, 32% students failed Using the formula
in Hindi and 52% failed in English. If n
r

G
18% failed in both the subjects, find the P 1+
percentage of those who passed in both 100
Population after 2 years
the subjects.
Here, x = 32, y = 52, z = 18 U = 1676000 1 +
10
2
O
∴ Percentage of students who passed 100
2
in both the subjects is 100 – (x + y – z) 110
= 100 – (32 + 52 – 18) = 1676000
H
100
= 100 – (84 – 18) 110 110
= 100 – 66 = 1676000 × ×
_T

= 34%. 100 100


= 2027960.
Per cent Change in Population
● The population of a town increases at
C

(xxiv) If the present population of a town the rate of 5 per cent annually. If the
be P and the population changes at r% per present population is 176400, what was
PS

annum, then it 2 years ago?


(a) Population in n years
Annual increase
n Here, P = 176400, r = 5 and n = 2
= Original population 1 + Using the formula
100
U

n
r
= P 1+ P
100 = n
r
@

Or, Population in n years 1+


n 100
Annual decrease
= Original population 1 – Population 2 years ago
n
100
r 176400
= P 1– = 2
100 5
1+
(b) Population n years ago 100
176400
Original Population =
= 105
2
n
Annual Increase
1+ 100
100
P 176400
= n = 2
r 21
1+
100 20
Unit Two : Fundamentals / Percentage ✫ 107

176400 Using the formula


=
21 21
× P × (100 + x) (100 − y) (100 + z )
20 20 ,
100 × 100 × 100
20 20
= 176400 × × population after 3 years
21 21
= 160000. 85000 × (100 + 4) (100 − 10) (100 + 15)
=
(xxv) If the population of a town is P which 100 × 100 × 100
increases by x% during the first year, by y%
85000 × 104 × 90 × 115
during the second year and again increases by =

TS
z% during the third year then the population 100 × 100 × 100
after 3 years will be = 91494.
P × (100 + x ) (100 + y ) (100 + z ) Error Percentage

H
100 ×100 × 100 (xxvii) For any given quantity, if a particular
● The population of a town is 150000. It value given is wrong then,

G
increases by 4% during the first year, Error percentage
5% during the second year and 8%
(False value – Actual value)
during the third year. What is the
population after 3 years?
U =
Actual value
× 100
O
Here, P = 150000, x = 4, y = 5 and ● In measuring the sides of a rectangle
z = 8 errors of 5% and 3% in excess are made.
H
Using the formula Find the error per cent in the calculated
P × (100 + x) (100 + y) (100 + z ) area.
,
_T

100 × 100 × 100 Let length = x units and breadth = y units


population after 3 years Then, actual area = xy sq. units
100 + 5
150000 × (100 + 4) (100 + 5) (100 + 8) Length shown = × x units
C

= 100
100 × 100 × 100
21 x
PS

150000 × 104 × 105 × 108 = units


= 20
100 × 100 × 100
100 + 3
= 176904. Breadth shown = × y units
100
U

(xxvi) If the population decreases by y%


during the second year and increases in first 103 y
= units
@

and third year by x% and z%, then the 100


population after 3 years will be
21x 103 y
Calculated area = ×
P (100 + x ) (100 – y ) (100 + z) 20 100
100 ×100 ×100
2163
= xy sq. units
● The population of a town is 85000. It 2000
increases by 4% during the first year, Error percentage
decreases by 10% during the second
(False value – Actual value)
year and increases by 15% during the = × 100
third year. What is the population after Actual value
3 years? 2163
xy − xy
Here, P = 85000, x = 4, y = –10 and 2000
= × 100
z = 15 xy
108 ✫ Quantitative Aptitude

2163 xy − 2000 xy subjects have the following weights: English


2000 4, Maths 3, Hindi 3 and Economics 1?
= × 100
xy English 90 × 4 = 360
163 xy Maths 84 × 3 = 252
Hindi 75 × 3 = 225
2000
= × 100 Economics 76 × 1 = 76
xy
913
163 xy 1 Weight = 4 + 3 + 3 + 1 = 11
= × × 100
2000 xy

TS
Average = 913 ÷ 11 = 83
163
= % = 8.15% excess. Finding the Average of Different Groups
20
If the average of different groups are known, the

H
average of a group made up of all the different
Average groups can be found.
xa + yb
New Average =

G
An average is the sum of ‘n’ different data x+y
divided by n. For example, if three students where x and y are the number of items in two
score 98, 74 and 80 marks in English
respectively, then their average marks in English
is equal to: U groups and a and b are their respective averages.
The formula is applicable to more than two
O
groups; only the numerator will be formed by
98 + 74 + 80 252 adding the products of the items in each group
= = 84 marks
3 3 and their averages and the denominator by
H
In statistics, average is also known as arithmetic adding all the items in the groups.
mean.
_T

There are other ‘averages’, mainly used in ● The average weight of 15 students of Section
statistics, such as ‘median’ and ‘mode’, which A is 20 kg, that of 28 students of Section
we will discuss in a later unit. B is 22 kg and that of 20 students of Section
C

C is 21 kg. What is the average weight of


Here, we may say
the three Sections?
Total of data
PS

Average = Here, there are three groups and three


Number of data
averages.
Total of data = Average × Number of data Average weight of three Sections
U

(15 × 20) + (23 × 22) + (20 × 21)


Total of data =
Number of data = 15 + 23 + 20
Average
@

300 + 506 + 420


Weighted Average When some numbers =
among terms to be averaged occur more than 58
once, they must be given the appropriate 1226
= = 21.14
weight. To obtain the average of quantities that 58
are weighted: (i) multiply the value of each
quantity by its respective weight; (ii) add the Fast-track Formulae and Worked
products; (iii) add up the weights; and (iv)
Examples
divide the sum of products by the sum of the
weights. Change of Average on Addition or
Removal of Items
● What are the average marks of a student
who received 90 in English, 84 in Maths, (i) If every observation is increased/
75 in Hindi, and 76 in Economics, if the decreased by a constant, the mean of the
Unit Two : Fundamentals / Average ✫ 109

observations so obtained also increases/ The average of new set of mumbers


decreases by the same constant. = previous average × number
New mean = Previous mean + Number multiplied to each term
added to each term = 24 × 7
New mean = Previous mean – Number = 168.
subtracted from each term
(iii) When one or more items are added
● The average value of six numbers 7, 12, to or removed from a group of items, the
17, 24, 26 and 28 is 19. If 8 is added to average changes.
each number, what will be the new

TS
(a) If A is the average of X items and Y
average? new items are added and the average
Here, previous mean = 19 changes (increases or decreases) by x,
number added = 8 then

H
Then, using the formula Average of the new items added
New mean = Previous mean + Number X
= A ± 1+ x

G
added to each term Y
New average = 19 + 8 (+ for average increasing; – for average
= 27.
● The average of x numbers is 6x. If x – 2
U decreasing)
(b) If A is the average of X items and Y
items are removed, and the average
O
is subtracted from each given number,
what will be the new average? changes by x, then
Average of items removed
H
Here, previous average = 6x
X
number subtracted = x – 2 = A ± 1− x
Y
_T

Then, using the formula


New average = Previous average (+ for average increasing; – for average
– Number subtracted decreasing)
from each term
C

● The average weight of 20 girls in a class


New average = 6x – (x – 2) is 30 kg. When 10 new girls are
= 6x – x + 2 admitted, the average increased by 5
PS

= 5x + 2. kg. What is the average weight of the


(ii) If each observation is multiplied/divided new students?
by a constant, the mean of the observations can Here the average weight has increased.
U

be obtained by multiplying/dividing the mean So,


by the same constant. Average weight of the new girls
@

New mean = Previous mean × Constant X


by which each term is = A + 1+ x
Y
multiplied
20
Previous mean = 30 + 1 + 5
New mean = 10
Number by which each 30
mean is divided = 30 + 5
10
● The average of 7 numbers is 24. If each = 30 + (3 × 5)
of the numbers is multiplied by 7, find = 45 kg.
the average of the new set of numbers.
● The average salary of 15 teachers is Rs
Here, previous average = 24 and 6000 per month. Three teachers left the
number multiplied to each term = 7 school and the average salary of the
Then, using the formula remaining teachers fell by Rs 175. What
110 ✫ Quantitative Aptitude

was the total salary of the three teachers Here,


who left? Weight of new man
– weight of removed man
Here the average has decreased. So,
= –Xx (as average decreases)
Average salary of the teachers who left
Weight of new man – 80 = – (5 × 4)
X
= A – 1− x Weight of new man = 80 – 20
Y = 60 kg.
15
= 6000 – 1 − 175 ● The average age of A and B is 20 years.
3 If C were to replace A, the average

TS
= 6000 – (–4) (175) would be 19, and if C were to replace
= 6000 – (–700) B, the average would be 21. What is
= 6000 + 700 = Rs 6700 the age of A, B and C?
This is the average salary of the three

H
teachers who left. The total salary of the Let the ages of A, B and C be a, b and c.
three teachers is 3 × 6700 = Rs 20,100. As per the given conditions,

G
(iv) The average of n quantities is equal to a+b
= 20
x. When a quantity is added or removed, the 2
average becomes y. The value of the new
quantity is n(y – x) + y or n(x – y) + y. U a + b = 40 . . . (i)
Now, when C replaces A, average age
O
● The average of 15 students in a class is becomes 19 – 20 = –1.
4 years. If the teacher’s age is included, As per the formula (v),
H
the average age becomes 7 years. Find c – a = – 2 × 1 . . . (ii)
the teacher’s age. Again, when C replaces B, average age
becomes 21 – 20 = +1.
_T

Here, n = 15 students, x = 4 years, c – b = 2 × 1 . . . (iii)


y = 7 years Adding (i), (ii) and (iii) we get
Then, using the formula a + b + c – a + c – b = 40 – 2 + 2
C

The teacher’s age = n (y – x) + y 2c = 40


= 15 (7 – 4) + 7 c = 20 years
= 15 (3) + 7
PS

Now, subtracting c in (ii)


= 45 + 7 = 52 years. c – a = –2
Change of Average on Items being 20 – a = –2
Replaced a = 22 years
U

From (iii) we get


(v) When a number of items are replaced
c–b =2
@

by an equal number of items the average of


20 – b = 2
the group may increase or decrease. If the
b = 20 – 2 = 18 years
group has X number of items, the change in
average is x. So A is 22 years, B is 18 years and C
Sum of new items added – Sum of removed is 20 years.
items Average Related to Speed and Distance
= ± Xx (vi) (a) If a person travels a distance at a
(+ meaning average increase; – meaning average speed of x kmph and the same distance at a
decrease.) speed of y kmph, then the average speed during
● A man weighing 80 kg was replaced by
2xy
another man in a group of five persons. the whole journey is given by kmph.
As a result the average weight of the x+y
group came down by 4 kg. How much ● A man runs 1 km at 16 km per hour and
does the new man weigh? another 1 km he walks at 6 km per hour.
Unit Two : Fundamentals / Average ✫ 111

Find his average speed for covering the Here, x = 42 km/hr and y = 30 km/hr
2 km. Then, using the formula
Here, x = 16 km/hr and y = 6 km/hr 2xy
The average speed =
Then, using the formula x+y
Average speed for the whole distance 2 × 42 × 30
2xy =
= 42 + 30
x+y
520
2 × 16 × 6 =
= 72

TS
16 + 6 = 35 km/hr.
192
= (vii) If a person travels three equal distances
22 at a speed of x kmph, y kmph and z kmph

H
= 8.72 km/hr. respectively, then the average speed during the
(b) If half of the journey is travelled at a 3xyz
whole journey is kmph.

G
speed of x kmph and the next half at a speed xy + yz + xz
of y kmph, then the average speed during the ● John moves over three equal distances
2 xy
whole journey is
x+y
kmph.
U at a speed of 12 kmph, 16 kmph and 10
kmph respectively. What was the
O
● If half of a journey is travelled at a average speed during his whole journey?
speed of 18 km per hour and the next Here, x = 12 kmph, y = 16 kmph and
half at a speed of 14 km per hour, find
H
z = 10 kmph
the average speed during the entire Then, using the formula,
journey. Average speed
_T

Here, speed of the first half journey (x) 3xyz


= 18 km/h and speed of the next half =
xy + yz + xz
journey (y) = 14 km/h.
C

Then, using the formula 3 × 12 × 16 × 10


=
2xy 12 × 16 + 16 × 10 + 12 × 10
PS

Average speed =
x+y 5760
=
2 × 18 × 14 192 + 160 + 120
=
18 + 14
U

5760
504 =
= 472
@

32
720 12
= 15.75 km/hour. = = 12 kmph.
59 59
(c) If a man goes to a certain place at a
speed of x kmph and returns to the original (viii) If a person covers A km at a speed of
place at a speed of y kmph, then the average x kmph, B km at a speed of y kmph and C km
speed during up and down journey is at a speed of z kmph, the average speed during
2 xy A+B+C
kmph. the entire journey is kmph.
x+y A B C
+ +
● Rakesh goes to a certain place at a x y z
speed of 42 km/hr and returns to the
original place at a speed of 30 km per ● Sushila covers 8 km at a speed of 2 kmph,
hour. Find the average speed during up 20 km at a speed of 4 kmph and 24 km
and down journey. at a speed of 6 kmph. What is her
112 ✫ Quantitative Aptitude

average speed during the entire journey? 1


Here, A= of the journey,
Here, Distance (A) = 8 km, Distance (B) 6
= 20 km, Distance (C) = 24 km, speed (x) 3
B= of the journey and
= 2 kmph, speed (y) = 4 kmph and speed 8
(z) = 6 kmph 1 3
C=1– +
Then, using the formula 6 8
Average speed 26 22 11
= 1− = = of the journey
A+B+C 48 48 24
=

TS
A B C Speed (x) = 42 kmph, speed (y) = 35
+ +
x y z kmph and speed (z) = 25 kmph
8 + 20 + 24 Then, using the formula
= Average speed

H
8 20 24
+ + 1
2 4 6 =
A B C

G
52 + +
= x y z
4+5+ 4 1
=
=
52
13
= 4 kmph.
U 1 3 11
6 + 8 + 24
O
42 35 25
● Mr. Sharma drove for 6 hours at an 1
average rate of 50 km per hour and for =
1 3 11
H
2 hours at an average rate of 60 km per + +
252 280 600
hour. What was his average rate for the
_T

entire trip? 1
=
50 + 135 + 231
His average rate for the entire trip
12600
6 (50) + 2 (60)
C

= 1 12600
8 = =
416 416
PS

300 + 120 420 1 12600


= = = 52
8 8 2 3150 15
= = 30 kmph.
(Do not make the mistake of simply 104 52
U

adding the two rates and dividing


by 2). Average and Numbers
@

(ix) If a person covers Ath part of the (x) (a) The average of first n natural
n+1
distance at x kmph, Bth part of the distance at numbers is .
y kmph and the remaining Cth part at z kmph, 2
then the average speed during the entire journey ● Find the average of the first 71 natural

1 numbers.
is kmph.
A B C Here n = 71
+ +
x y z Using the formula
1
● A train covers th of the journey at 42 n+1
6 Average = ,
3 2
kmph, the next th of the journey at 35 71 + 1 72
8 we have = = 36
kmph and the remaining distance at 25 2 2
kmph. Find the average speed of the The average of the first 71 natural
train during the entire journey. numbers is 36.
Unit Two : Fundamentals / Average ✫ 113

(b) The average of even numbers from 1 to ● Calculate the average of cubes of natural
last even number + 2 numbers from 1 to 40.
n is .
2 Here, n = 40
● Find the average of even numbers from Using the formula
1 to 40. n (n + 1)2
Average=
Here, last even number = 40 4
Using the formula 40 (40 + 1)2
last even number + 2 =
4

TS
Average=
2 40 × 1681
40 + 2 42 =
= 4
= = 21.
2 2 = 16810.

H
(c) The average of odd numbers from 1 to
(f) The average of first n consecutive even
last odd number + 1
n is . numbers is (n + 1).

G
2
● Find the average of odd numbers from
● Find the average of first 60 consecutive
even numbers.
1 to 60.
Here, last odd number = 59 U Here, n = 60
Then, using the formula
O
Using the formula
Average = (n + 1)
last odd number + 1 = (60 + 1)
Average=
H
2 = 61.
59 + 1
=
_T

(g) The average of first n consecutive odd


2
numbers is n.
60
= = 30. ● Find the average of first 40 consecutive
2
C

odd numbers.
(d) The average of squares of natural
( n + 1)(2n + 1) Here, n = 40
PS

numbers till n is . The required average = n


6 = 40.
● Calculate the average of squares of the (h) The average of squares of first n
natural numbers from 1 to 40. 2 (n + 1) (2n + 1)
U

consecutive even numbers is .


Here, n = 40 3
Using the formula ● Calculate the average of squares of first
@

(n + 1) (2n + 1) 40 consecutive even numbers.


Average=
6 Here, n = 40
(40 + 1) (2 × 40 + 1) Then, using the formula
=
6 2 (n + 1) (2n + 1)
41 × 81 Average =
= 3
6 2 (40 + 1) (2 × 40 + 1)
=
3321 3
= = 553.5. 2 (41) (81)
6 =
3
(e) The average of cubes of natural numbers = 2214.
n (n+ 1)2 (i) The average of squares of consecutive
till n is . (n + 1) (n + 2)
4 even numbers till n is .
3
114 ✫ Quantitative Aptitude

● Find the average of squares of M + T + W = (38 × 3) = 114 . . . (i)


consecutive even numbers from 1 to 70. T + W + Th = (40 × 3) = 120 . . . (ii)
4
Here, n = 70 Th = M
Then, using the formula 5
5
(n + 1) ( n + 2) ⇒ M = Th
Average = 4
3 Equations (ii) – (i), we get
(70 + 1) (70 + 2) M – Th = 120 – 114
= M – Th = 6
3

TS
71 × 72 5 5
= Th – Th = 6 M = Th
3 4 4
= 1704.
5th – 4th

H
(j) The average of squares of consecutive = 6
n (n+ 2) 4
odd numbers till n is . Th = 6 × 4

G
3 Th = 24°C.
● Find the average of squares of Hence, the temperature on Thursday was
consecutive odd numbers from 1 to 70.
Here, n = 69
U ●
24°C.
The average marks in Mathematics for five
O
Then, using the formula students was found to be 60. Later on, it
n ( n + 2) was discovered that in case of one student,
Average =
H
3 the marks 58 were misread as 85. Find the
correct average.
69 (69 + 2)
= Total recorded marks
_T

3 = (60 × 5) = 300
69 × 71 Correct total = (300 + 58 – 85)
=
3 = 385 – 85 = 273
C

= 1633. 273
Correct average =
5
PS

Miscellaneous Worked Examples


= 54.6.
● A census shows that in a certain block of
● In an examination, a pupil’s average marks
flats the number of children in each family
were 53 per paper. If he had obtained 22
U

is 3, 4, 4, 0, 1, 2, 0, 2 and 2 respectively.
more marks in his Geography paper and
Find the average number of children. 2 more marks in his History paper, his
@

average per paper would have been 55.


Average = 3 + 4 + 4 + 0 + 1 + 2 + 0 + 2 + 2 How many papers were there in the
9
18 examination?
= = 2.
9 Let there be n papers
● The mean temperature of Monday to Total marks of all papers = 53n
Wednesday was 38°C and that of Tuesday 55n – 53n = (22 + 2)
to Thursday was 40°C. If the temperature 2n = 24
4 n = 12
on Thursday was that of Monday, what
5 So, there were 12 papers in the examination.
was the temperature on Thursday?
● A batsman makes a score of 88 runs in the
Here, 19th innings and thus increases his average
Monday = M, Tuesday = T, Wednesday = W, by 4. What is his average of runs after the
and Thursday = Th 19th innings?
Unit Two : Fundamentals / Ratio and Proportion ✫ 115

For a batsman, average is ● A ratio a : b is said to be in the simplest


total runs scored form if the H.C.F. of a and b is 1 (i.e.,
total number of innings a and b are prime to each other).
Let the average run after the 19th innings If A is the given quantity to be divided
be x. Then, in the ratio a : b, then
average run after 18th innings = (x – 3) a
19x – 18 (x – 3) = 88 First part = ×A
a+b
19x – 18x + 54 = 88 b
x = 88 – 54 Second part = ×A
a+b

TS
x = 34 If the ratio is given in the form of
∴ Average runs after the 19th innings = 34. 1 1
[Note: Though the problems are easy to fraction, say : , then the L.C.M. of
m n
solve by these formulae, it is necessary to read the denominators of the fractions has to

H
each problem carefully and understand what be found and then the fractions should
you are asked to find—the new average, the be multiplied by the L.C.M. so that the

G
number of new items or the total, and so on, ratio can be expressed in the form of
and then use the correct formula.] integers.
Ratio and Proportion
Basics of Ratio U Comparing Ratios
(i) In the two ratios a : b and c : d,
O
A ratio expresses a comparison of two (or more) (a) a : b > c : d if ad > bc
quantities in terms of numbers. The mark used a c
H
to indicate ratio is (:) and is read ‘is to’. or > if ad > bc
b d
Any ratio of two terms may be written as (b) a : b = c : d if ad = bc
_T

a fraction and any fraction, as a ratio. (c) a : b < c : d if ad < bc


The ratio of two numbers, say x and y, is
x
● If we have to compare the ratios 3 : 2
written as and is denoted by x : y. Here x and 5 : 7, we multiply the terms
C

y
3 × 7 and 2 × 5
is called antecedent and y is called consequent.
⇒ 21 and 10 3 : 2 and 5 : 7
PS

● The ratio of two quantities shows how


Here ad > bc, so a : b > c : d
many times one quantity is contained in or 3 : 2 > 5 : 7
the other.
● Two terms are being compared in a a
(ii) If > 1, a > b; i.e. the first term of the
U

ratio, so the two terms must be of the b


same kind, or expressed in the same ratio is greater than the second term, and if
units. a
@

● The order of the terms in a ratio is


< 1, then a < b.
b
significant; a : b is not the same as b : a, a a+x a
for 3 : 4 is not the same as 4 : 3. (iii) If = 1, then = =1
b b+x b
● A ratio is not affected on each of its
(Adding the same positive integer to
terms being multiplied or divided by the
same non-zero number. But the addition both terms of a ratio does not affect a
or subtraction of the same number can ratio that is equal to 1.)
change the ratio. a a+ x a
(iv) If > 1, then <
a ma b b+x b
= such that a : b = ma : mb
b mb (Adding the same positive integer to both
a terms decreases a ratio greater than 1.)
a a b
= m such that a : b = : a a+ x a
b b m m (v) If < 1, then >
m
b b+x b
116 ✫ Quantitative Aptitude

(Adding the same positive integer to c the third term and d the fourth term. The first and
both terms increases a ratio less than 1.) the fourth terms are called extremes (end terms),
● Suppose you add the positive quantity and the second and the third terms are called
means (middle terms). (The symbol : stands for ‘is
7
3 to the ratio . to’ and the symbol : : stands for ‘as’.)
5
7 Extremes
Now > 1
5
7 + 3 10 7 a : b :: c : d
= <
5+ 3 8 5

TS
5 means
● Suppose you add 3 to the ratio .
7 The product of the extremes must be equal
5
Now < 1 to the product of the means (or middles) if a,
7 b, c and d are in proportion.

H
5+ 3 8 5 There are different types of proportion:
= >
7 + 3 10 7 Continued Proportion If a : b = b : c, then

G
a a− x a a, b and c are said to be in continued proportion.
(vi) If = 1, then = = 1. Here c is called the third proportional to a and
b b−x b
(Subtracting the same positive integer
from both terms of a ratio does not affect U b and b is the mean proportional to the extremes.
Direct Proportion Two quantities ‘a’ and
‘b’ are said to be directly proportional to each
O
a ratio that equals 1.)
other when one increases or decreases, and the
a a−x a
(vii) If > 1, then > other also increases or decreases accordingly to
H
b b−x b the same extent. It is written as a ∝ b.
(Subtracting the same positive integer a
from both terms of a ratio increases the i.e., a = k × b or = k, where k is a constant.
_T

ratio if it is greater than 1.) b


Inverse Proportion Two quantities ‘a’ and
a a−x a ‘b’ are said to be inversely proportional to each
(viii) If < 1, then <
b b−x other when one increases and the other
C

b
(Subtracting the same positive integer decreases and vice-versa to the same extent. It
from both terms of a ratio decreases the 1
PS

is written as a ∝ .
ratio if it is less than 1.) b
1
7 i.e., a = k×
● Suppose you subtract 3 from the ratio . b
5
U

7 a.b = k
> 1
5 where k is a constant.
@

7−3 4 7 Points to Remember


= > .
5−3 2 5
5 The following points are to be kept in mind.
● Suppose you subtract 3 from the ratio . ● If a : b : : c : d are in proportion then,
7
5 product of extremes = product of means
< 1 ad = bc
7
● If a, b and c are in continued proportion
5−3 2 5
= < . (a : b :: b : c) then b 2 = ac
7−3 4 7
● Mean proportional between a and b is
Basics of Proportion
Proportion is the equality of two ratios. If a, b, ab .
c, d are in proportion then we can write ● If a : b :: c : d then,
a : b :: c : d a+b c+d
where a is the first term, b the second term, (i) =
b d
Unit Two : Fundamentals / Ratio and Proportion ✫ 117

a−b c−d
(ii) = =
36
× 6300
b d 24 + 36 + 45 + 105
a+b c+d
(iii) = 36
a−b c−d = × 6300
a b 210
(iv) = = Rs 1,080.
c d
b d Many problems in which three terms are
(v) =
a c given and one term is unknown can be solved
a c by using proportions. However, the proportion

TS
(vi) =
a−b c−d should be formulated carefully according to
a+c what is given—if any term is misplaced, the
(vii) Each ratio = solution will be incorrect.
b+d

H
When there are more than two Fast-track Formulae and Worked
quantities Examples

G
(i) Three Quantities Suppose we have three
items a, b and c such that a : b = x : y; b : c (i) To simplify a complicated ratio of two
= p : q. How are a, b and c related?
a : b : c = xp : py : yq U terms containing fractions, decimals or
percentages, divide the first term by the
O
and a : c = xp : yq. second, write as a fraction in lowest
● If the savings of Jack and Jane are in the
terms, and write the fraction as a ratio.
5 7
H
ratio of 3 : 5 and that of Jane and Jill Suppose you have to simplify : ,
6 8
are in the ratio of 7 : 4, in what ratio
5 7 5 8 40 20
÷ = × = =
_T

are the savings of all three? = 20 : 21


6 8 6 7 42 21
Here, a is Jack; b is Jane and c is Jill (ii) To solve problems in which the ratio is
x : y is 3 : 5; p : q = 7 : 4 given, add the terms in the ratio, divide
C

So the ratio of the savings of the three the total to be put into a ratio by this
= 3 × 7 : 5 × 7 : 5 × 4 sum, and multiply each term in the ratio
PS

= 21 : 35 : 20 by this quotient.

(ii) Four Quantities If there are four items a, b, The sum of Rs 360 is to be divided
c and d such that a : b = x : y; b : c = p : q; and c : d among three people in the ratio 3:4:5.
U

= m : n, then a : b : c : d = xpm : ypm : yqm : yqn and How much does each one receive?
a : d = xpm : yqn. Add up the ratios
@

3 + 4 + 5 = 12
● Arun, Balu, Charu and Dima collected Divide the total by the sum of ratios
Rs 6,300. If their collections were in the Rs 360 ÷ 12 = Rs 30
ratio— Arun : Balu = 2 : 3, Balu : Charu Now each one receives as follows:
= 4 : 5, and Charu : Dima = 3 : 7, what Rs 30 × 3 = Rs 90
was Balu’s collection in Rs 6,300? Rs 30 × 4 = Rs 120
Arun : Balu : Charu : Dima Rs 30 × 5 = Rs 150
The money is thus divided as Rs 90,
=2×4×3:3×4×3:3×5×3:3×5×7
Rs 120, Rs 150.
= 24 : 36 : 45 : 105
(iii) (a) A number which, when subtracted
Balu’s collection
from the terms of the ratio a : b, makes
Ratio term for Balu
= × Total amount it equal to the ratio c : d is bc − ad .
Total ratio terms c−d
118 ✫ Quantitative Aptitude

● Find the number that must be subtracted A+B:A–B =a+b: a–b


from the terms 5 : 7 to make it equal = 5 + 2 : 5 – 2
to 1 : 3. = 7 : 3.
(vi) If two numbers are in the ratio of a : b
We have a : b = 5 : 7 and c : d = 1 : 3
and the sum of these numbers is x, then
bc − ad ax bx
Required number = these numbers will be and
c−d a+b a+b
7 ×1− 5 × 3 respectively.
= Or
1− 3

TS
If in a mixture of x litres, two liquids A
7 − 15 −8 and B are in the ratio a : b, then the
= = = 4.
−2 −2 quantities of liquids A and B in the
(b) A number which, when added to the ax bx

H
mixture will be litres and
terms of the ratio a : b, makes it equal a+b a+b
litres, respectively.
ad − bc

G
to the ratio c : d is . Or
c−d
● Find the number that must be added If three numbers are in the ratio of a : b : c
to the terms 1 : 7 to make it equal to
1 : 4. U and the sum of these numbers is x, then
these numbers will be
ax
,
bx
O
a+b+c a+b+c
We have a : b = 1 : 7 and c : d = 1 : 4 cx
and , respectively.
ad − bc a+b+c
H
Required number = ● Two numbers are in the ratio 7 : 3 and
c−d
the sum of these numbers is 30. Find
1× 4 − 7 ×1
_T

the two numbers.


=
1− 4
We have a = 7, b = 3 and x = 30
4 − 7 −3 ax
= = = 1. The required numbers are and
−3 −3 a+b
C

bx
(iv) If the sum of two numbers is S and their
a+b
PS

difference is d, then the ratio of the


numbers is given by S + d : S – d. ax 7 × 30
∴ =
a+b 7+3
● If the sum of two numbers is 7 and
U

their difference is 3, find the ratio of 210


= = 21
these numbers. 10
3 × 30
@

We have S = 7 and d = 3 bx
∴ =
Required ratio = S + d : S – d a+b 7+3
= 7 + 3 : 7 – 3 90
= 10 : 4 = = 9.
10
= 5 : 2.
● Three numbers are in the ratio 2 : 3 :
(v) If two quantities A and B are in the ratio 4 and the sum of these numbers is 27.
a : b, then Find the numbers.
A+B : A–B = a+b : a–b
We have a = 2, b = 3, c = 4 and x = 27
● If two quantities A and B are in the ax
ratio 5 : 2, then what is A + B : A – B The required numbers are ,
equal to? a+b+c
bx cx
and
We have a = 5, b = 2 a+b+c a+b+c
Unit Two : Fundamentals / Ratio and Proportion ✫ 119

numbers, the ratio becomes c : d.


ax 2 × 27 54
∴ = = = 6 The two numbers are given as:
a+b+c 2+ 3+ 4 9 ax ( c − d) bx (c − d)
and
bx 3 × 27 81 ad − bc ad − bc
= = = 9
a+b+c 2+ 3+ 4 9 ● The ratio of two numbers is 3 : 5. If
9 is added to each of them, their ratio
cx 4 × 27 108 is changed to 13 : 21. Find the
= = = 12.
a+b+c 2+3+ 4 9 numbers.
We have a = 3, b = 5, c = 13, d = 21

TS
(vii) If two numbers are in the ratio of a : and x = 9
b and the difference between these ax (c − d)
numbers is x, then these numbers will be The required numbers are
ad − bc
bx (c − d)
and .

H
ax bx
(a) and , where a > b ad − bc
a−b a−b ax (c − d) 3 × 9 (13 − 21)
∴ =

G
● Two numbers are in the ratio of 7 : ad − bc 3 × 21 − 5 × 13
5. If the difference between these 27 × (−8)
numbers is 12, find the numbers. = = 108
We have a = 7, b = 5 and x = 12
U ∴
bx (c − d)
63 − 65
5 × 9 (13 − 21)
O
Here, a > b =
ax ad − bc 3 × 21 − 5 × 13
The required numbers are and
a−b 45 × (−8)
H
bx = = 180.
. 63 − 65
a−b (b) The ratio between two numbers is
_T

ax 7 × 12 84 a : b. If x is subtracted from each of


∴ = = = 42
a−b 7−5 2 these numbers, the ratio becomes c : d.
5 × 12 60 The two numbers are given as:
bx =
C

= = 30.
a−b 7−5 2 ax ( d − c) bx ( d − c )
and
ad − bc ad − bc
PS

ax bx ● The ratio of two numbers is 1 : 5.


(b) and , where a < b
b−a b−a If 4 is subtracted from each number,
● Two numbers are in the ratio of 3 : their ratio is changed to 1 : 7. Find
U

5. If the difference between these the numbers.


numbers is 12, find the numbers. We have a = 1, b = 5, c = 1, d = 7 and
@

We have a = 3, b = 5 and x = 12 x = 4
Here a < b ax (d − c)
ax The required numbers are
The required numbers are and ad − bc
b−a bx (d − c)
and .
bx ad − bc
.
b−a ax ( d − c) 1 × 4 (7 − 1)
∴ =
ax 3 × 12 36 ad − bc 1× 7 − 5 ×1
∴ = = = 18
b−a 5−3 2 4×6
= = 12
bx 5 × 12 60 7−5
= = = 30.
b−a 5−3 2 bx (d − c) 5 × 4 (7 − 1)
∴ =
(viii) (a) The ratio between two numbers is ad − bc 1× 7 − 5 ×1
a : b. If x is added to each of these 20 × 6
= = 60.
7−5
120 ✫ Quantitative Aptitude

(ix) The incomes of two persons are in ∴ Gurmeet’s expenditure is


the ratio a : b and their expenditure
Rc (b − a)
are in the ratio c : d. If each of them Rs
saves Rs R, then their incomes are ad − bc
given by 4000 × 3 × (3 − 5)
=
Ra ( d − c) Rb (d − c) 5 ×1− 3 × 3
Rs and Rs
ad − bc ad − bc 4000 × 3 × (−2)
=
and their expenditures are given by 5−9
Rc (b − a) Rd (b − a) = Rs 6000

TS
Rs and Rs and Ruchi’s expenditure is
ad − bc ad − bc
Rd (b − a)
The annual income of Gurmeet and Rs
● ad − bc
Ruchi is in the ratio 5 : 3 and their

H
4000 × 1 × (3 − 5)
annual expenses bear a ratio of 3 : 1. =
If each of them saves Rs 4000 at the 5 ×1− 3 × 3

G
end of the year, then find their annual 4000 × 1 × (−2)
=
incomes and annual expenditures. 5−9
We have a = 5, b = 3, c = 3, d = 1 and
R = Rs 4000 U =
4000 × 2
4
O
Ra (d − c) = Rs 2000.
Annual income = Rs and
ad − bc
Rb (d − c) (x) (a) If a : b = n1 : d1 and b : c = n2 : d2, then
H
Rs a : b : c = (n1 × n2) : (d1 × n2) : (d1 × d2)
ad − bc
Ra (d − c)
_T

● If A : B = 5 : 7 and B : C = 6 : 11, find


∴ Gurmeet’s income is A : B : C.
ad − bc
4000 × 5 × (1 − 3) Here, n1 = 5, n2 = 6, d1 = 7 and d2 = 11
=
C

5 ×1− 3× 3 ∴ A : B : C = (n1 × n2) : (d1 × n2) : (d1 × d2)


4000 × 5 × (−2) = (5 × 6) : (7 × 6) : (7 × 11)
PS

=
5−9 = 30 : 42 : 77.
4000 × 5 × 2 (b) If a : b = n1 : d1, b : c = n2 : d2 and
=
4 c : d = n3 : d3, then
U

= Rs 10000
a : b : c : d = (n1 × n2 × n3) : (d1 × n2 × n3)
Rb (d − c) : (d1 × d2 × n3) : (d1 × d2 × d3)
@

and Ruchi’s income is


ad − bc
● If A : B = 3 : 4, B : C = 4 : 5 and C : D =
4000 × 3 × (1 − 3) 5 : 6, find A : D.
=
5 ×1− 3× 3
Here, n1 = 3, n2 = 4, n3 = 5, d1 = 4, d2 = 5
4000 × 3 × (−2)
= and d3 = 6
5−9 ∴ A : B : C : D = (n1 × n2 × n3) : (d1 × n2 × n3)
4000 × 3 × 2 : (d1 × d2 × n3) : (d1 × d2 × d3)
=
4 = (3 × 4 × 5) : (4 × 4 × 5) : (4 × 5 × 5)
= Rs 6000 : (4 × 5 × 6)
= 60 : 80 : 100 : 120
∴ Annual expenditure =3:4:5:6
Rc (b − a) Rd (b − a) Thus, A : D = 3 : 6
= Rs and Rs .
ad − bc ad − bc = 1 : 2.
Unit Two : Fundamentals / Ratio and Proportion ✫ 121

(xi) (a) In any two two-dimensional figures, ● In a mixture of two types of alcohol
if the corresponding sides are in the A and B, the ratio A : B is 5 : 3. If the
ratio a : b, then their areas would be cost of alcohol A is Rs 10 per litre
in the ratio a2 : b2. and that of B is Rs 15 per litre, find
the cost per litre of the resulting
● In two squares, the corresponding
mixture.
sides are in the ratio 3 : 5. Find the
ratio of these two squares. We have, q1 = 5, q2 = 3, x1 = 10, x2 = 15
The cost of resulting mixture
We have, a = 3 and b = 5
The ratio of their areas = a2 : b2 x1q1 + x 2 × q2

TS
= 32 : 5 2 =
q1 + q2
= 9 : 25.
10 × 5 + 15 × 3
(b) In any two 3-dimensional figures, if =
5+ 3

H
the corresponding sides or other
50 + 45 95
measuring lengths are in the ratio = =

G
a : b, then their volumes would be 8 8
in the ratio a3 : b3. = Rs 11.88.
● In two cubes, the corresponding sides
U
are in the ratio 1 : 3. Find the ratio of
(xiii) If a mixture contains two ingredients A
and B in the ratio x : y,
O
their volumes. then percentage of A in the mixture
We have a = 1 and b = 3 x
= × 100%
H
The ratio of their volumes = a3 : b3 x+y
= 13 : 33 and percentage of B in the mixture
_T

= 1 : 27. y
= × 100%.
x+y
(xii) When two ingredients A and B of
quantities q1 and q2 and cost price/unit ● A mixture contains water and alcohol
C

x1 and x2, are mixed to get a mixture in the ratio 3 : 4. What is the
x having cost price/unit xm, then percentage quantity of water in the
mixture?
PS

q1 x 2 − x m
(a) = and Here, x = 3 and y = 4
q2 x m − x1
Percentage quantity of water in the
x1q1 + x 2 × q2 x
U

(b) xm = mixture = × 100%


q1 + q2 x+y
3
@

● In what ratio must the two kinds of = × 100%


3+ 4
tea be mixed together, one at Rs 15
per kg and another at Rs 21 per kg, 3
= × 100%
so that the mixture may cost Rs 17 per 7
kg? = 42.86%
Here, x1 = 15, x2 = 21 and xm = 17 ● If an alloy contains copper and silver
in the ratio 3 : 7, find the percentage
q1 x 2 − x m
Now, = of silver in the alloy.
q2 x m − x1
We have x = 3 and y = 3
21 − 17 4 2
= = = Percentage quantity of silver in the
17 − 15 2 1 alloy
Thus, the two kinds of tea are mixed y
in the ratio 2 : 1. = × 100%
x+y
122 ✫ Quantitative Aptitude

Simple Partnership
7 × 100
= % In simple partnership the capitals of all the
3+7 partners are invested for the same time period,
= 70% and the profit/loss is distributed proportionally
● A mixture contains alcohol and water according to the investment of the partners.
in the ratio 5 : 4. If 9 litres of water ● Atul, Amit and Sohail started a business
is added to the mixture, the ratio of in partnership. Atul invested Rs
alcohol and water becomes 4 : 5. 1,20,000, Amit invested Rs 1,50,000 and
Find the quantity of alcohol in the Sohail invested Rs 1,80,000 respectively

TS
mixture. in business. How should they divide a
Let the quantity of alcohol and profit of Rs 50,00,000?
water be 5x litres and 4x litres
respectively. Profit should be divided in the ratios of

H
Then, the capitals, i.e., in the ratio
5x 120000 : 150000 : 180000 = 12 : 15 : 18.

G
4
= Now, 12 + 15 + 18 = 45
4x + 9 5
12
⇒ 25x = 16x + 36
⇒ 25x – 16x = 36
9x = 36 U Atul’s share =
=
45
of Rs 5000000
Rs 1333333.30
O
x = 4 15
Quantity of alcohol in the mixture Amit’s share = of Rs 5000000
45
H
= 5 × 4 = 20 litres. = Rs 1666666.60
● Two numbers are in the ratio of
18
_T

3 : 4. If the difference between these Sohail’s share = of Rs 5000000


numbers is 23, then find the numbers. 45
Here, a = 3, b = 4 and x = 23 = Rs 2000000
C

ax ● A, B and C are three partners. A invested


∴ The first number = Rs 2000, B invested Rs 10000 and C
b−a
invested Rs 6000. If C got a profit of
PS

3 × 23 Rs 450 at the end of the year, how much


= = 69
4−3 did A and B get?
and the second number Here, Ratio of investments
U

bx = 2000 : 10000 : 6000


=
b−a = 1 : 5 : 3
@

4 × 23 3
= = 92. C’s share of profit = × x = 450
4−3 9
where x is the total profit.
450 × 9
Partnership Total profit is x = = Rs 1350
3
An association of two or more persons who
put their money together in order to carry on 1
∴ A’s share of profit = × 1350 = Rs 150
a certain business is called partnership and 9
these persons are called partners. 5
B’s share of profit = × 1350 = Rs 750
Partners may be active or sleeping. An 9
active partner is a partner who devotes his time Compound Partnership
for the business in addition to his investment. In compound partnership capitals are invested
A sleeping partner is a partner who only for different time periods, and the capital of
invests money in the business. each partner is multiplied by the number of
Unit Two : Fundamentals / Partnership ✫ 123

months the money was invested in the business, 22500 × 50000


and the profit/loss is proportionately =
distributed. 50000 + 40000

● Reena and Meena are partners in a 22500 × 50000


=
business in which Reena has contri- 90000
buted Rs 1,470 and Meena Rs 1,050. = Rs 12500
After 4 months, Venu joined them with = Rs 22500 – 12500
a capital of Rs 1,890. Find the ratio in = Rs 10000
which the profit should be divided at [If only Kavita’s share in the profits is

TS
the end of the year.
to be found, you can use the
We calculate each one’s total investment p×y
formula .]
in the year. x+y

H
Reena’s investment (ii) If the capitals of three partners be x, y,
= 1470 × 12 months and z for the same period and the total

G
= Rs 17640 profit be P, then the shares of the
partners in the profits are
Meena’s investment = 1050 × 12
= Rs 12600
Venu’s investment = 1890 × 8 U P
x+ y+z
× x,
P
x+ y+z
× y and
O
= Rs 15120 P
Ratio of their investment ×z
x+ y+z
H
= 17640 : 12600 : 15120
(H.C.F. = 2520 and then dividing each ● Mohit, Ankur and Ajay started a
amount by H.C.F.) = 7 : 5 : 6 business in partnership. Mohit
_T

invested Rs 1,20,000, Ankur invested


Rs 1,50,000 and Ajay invested Rs
Fast-track Formulae and Worked 1,80,000 in the business. How should
C

they divide a profit of Rs 50,00,000?


Examples
We have x = 120000, y = 150000,
PS

(i) If the capitals of two partners be x and z = 180000 and p = 5000000


y for the same period and the total profit p×x
be P, then the shares of the partners in So, Mohit’s share = x + y + z
the profits are
U

5000000 × 120000
P P =
×x and ×y 120000 + 150000 + 180000
x+y x+y
@

5000000 × 120000
=
● Nikita and Kavita enter into a 450000
partnership investing Rs 50000 and 5000000 × 12
Rs 40000, respectively. They agree to =
45
share profits in the ratio of their
= Rs 1333333.33
capitals. Find the share of Nikita
and Kavita in a profit of Rs 22500 p×y
after one year. Ankur’s share =
x+y+z
We have x = 50000, y = 40000 and 5000000 × 150000
p = 22500 =
120000 + 150000 + 180000
So, p× x
Nikita’s share = 5000000 × 150000
x+y =
450000
124 ✫ Quantitative Aptitude

5000000 6000 × 4 × 45000


= =
3 72000
= Rs 1666666.66 = Rs 15000.]
p× z
Ajay’s share = (iv) If the capitals of three partners be x, y
x+y+z and z for the periods t1, t2 and t3
5000000 × 180000 respectively and the total profit be P,
= then the shares of the partners in the
120000 + 150000 + 180000

TS
profits are
5000000 × 180000
=
450000 x × t1 × P y × t2 × P
= Rs 20,00,000 x t1 + y t2 + z t3 , x t1 + y t2 + z t3

H
(iii) If the capitals of two partners be x and z × t3 × P
and
y for the periods t1 and t2 respectively x t1 + y t2 + z t3

G
and the total profit be P, the shares of
the partners in the profits are: ● Akhil, Vinod and Rajesh entered

x × t1 × P y × t2 × P
x t1 + y t2 and x t1 + y t2 U into a partnership. Akhil invested Rs
16000 for 9 months, Vinod invested
Rs 12000 for 6 months and Rajesh
O
invested Rs 8000 for 12 months. At
● A and B enter into a partnership. A the end of a year there was a profit
H
contributes Rs 8000 for 6 months and of Rs 26000. Find their respective
B, Rs 6000 for 4 months. If the total shares.
profit is Rs 45000, find their respective
_T

shares. We have x = 16000, y = 12000,


z = 8000, t1 = 9, t2 = 6, t3 = 12 and
We have x = 8000, y = 6000, t1 = 6,
P = 26000
C

t2 = 4 and p = 45000
x × t1 × p So, profit share of Akhil
Profit share of A = x × t1 × P
PS

xt1 + yt2 =
8000 × 6 × 45000 xt1 + yt2 + zt3
=
8000 × 6 + 6000 × 4 16000 × 9 × 26000
=
U

8000 × 6 × 45000 16000 × 9 + 12000 × 6 + 8000 × 12


=
48000 + 24000 16000 × 9 × 26000
@

=
8000 × 6 × 45000 144000 + 72000 + 96000
=
72000 16000 × 9 × 26000
= 2 × 15000 = Rs 30000 =
312000
B’s share of profit would be
16000 × 9
Rs 45000 – Rs 30000 = Rs 15000 =
12
y × t2 × p
[Profit share of B = = Rs 12000.
xt1 + yt2
6000 × 4 × 45000 y × t2 × P
= Profit share of Vinod =
8000 × 6 + 6000 × 4 xt1 + yt2 + zt3
12000 × 6 × 26000
6000 × 4 × 45000 =
= 16000 × 9 + 12000 × 6 + 8000 × 12
48000 + 24000
Unit Two : Fundamentals / Partnership ✫ 125

12000 × 6 × 26000 ⇒ y = 3150 × 3


= ⇒ y = Rs 9450.
144000 + 72000 + 96000
12000 × 6 × 26000 (vi) If more than two persons invest money
= in a business, say A, B and C invest x,
312000
y and z for t1, t2 and t3 time respectively,
12000 × 6
= Amount of investment of A × No. of
12
months invested by A :
= Rs 6000.
Amount of investment of B × No. of

TS
z × t3 × P months invested by B :
Profit share of Rajesh =
xt1 + yt2 + zt3 Amount of investment of C × No. of
8000 × 12 × 26000 months invested by C
=

H
16000 × 9 + 12000 × 6 + 8000 × 12 x t1 : y t2 : z t3
8000 × 12 × 26000 = Profit of A : Profit of B : Profit of C

G
=
144000 + 72000 + 96000 (Or Loss of A : Loss of B : Loss of C)
8000 × 12 × 26000
=
312000
U ● There are three partners A, B and
C in a certain business. A puts in Rs
6000 for 6 months, B Rs 1000 for 7
O
8000 × 12
= months and C Rs 12000 for 4 months.
12 Find the ratio of their shares in the
H
= Rs 8000. profit.
(v) If two partners A and B invest x and
We have x = 6000, y = 10000,
_T

y amounts in a business for t1 and t2


respectively, z = 12000, t1 = 6, t2 = 7
and t3 = 4
x × t1 Profit of A Loss of A
= Profit A : Profit B : Profit C
C

or
y × t2 Profit of B Loss of B
= xt1 : yt2 : zt3
PS

● A starts business with Rs 4200, and 4 = 6000 × 6 : 10000 × 7 : 12000 × 4


months later B joins A as his partner.
= 36000 : 70000 : 48000
After one year the profits are divided
in the ratio of 2 : 3. How much did or 36 : 70 : 48.
U

B contribute?
(vii) If the time period of investments of
We have x = 4200, y = ?, t1 = 12, three partners are in the ratio t1 : t2 : t3,
@

t2 = 12 – 4 = 8 and and their profits are in the ratio P1 : P2 : P3,


then their capitals x, y, z are in the ratio
profit A : profit B = 2 : 3
Now,
P1 P P
: 2 : 3
t1 t2 t3
Profit for A x × t1
= ● Anil, Rajat and Ramesh start a
Profit for B y × t2 business. If the ratio of their periods
of investments are 1 : 2 : 5 and their
2 4200 × 12 profits are in the ratio of 3 : 4 : 5,
⇒ =
3 y×8 find the ratio of capitals of Anil,
⇒ 2 × 8 × y = 3 × 4200 × 12 Rajat and Ramesh.
Here, P1 : P2 : P3 = 3 : 4 : 5,
3 × 4200 × 12
⇒ y = t1 : t2 : t3 = 1 : 2 : 5
2× 8
126 ✫ Quantitative Aptitude

P1 P2 P3 1 2 3
The required ratio = : : =: :
t1 t 2 t 3 3 4 6
3 4 5 1 1 1
∴ The required ratio = : : = : :
1 2 5 3 2 2
or 2 : 3 : 3.
= 3 : 2 : 1
Thus, Anil, Rajat and Ramesh
invested their capitals in the ratio = Miscellaneous Examples
3 : 2 : 1. ● X, Y and Z enter into a partnership and their

TS
● The ratio of investments of two 1 1 1
investments are in the ratio : : . X
partners is 5 : 7 and the ratio of their 3 2 5
profits is 20 : 21. If A invested the withdraws half the capital after 4 months. At

H
money for 8 months, find for how the end of the year, a profit of Rs 20,003 is
long did B invest the money? divided among them. Find the share of each.

G
Let us assume that B invested the Ratio of investment
money for x months. 1 1 1
X : Y : Z = : :
Then, 5 × 8 : 7 × x = 20 : 21
40
=
20
U 3 2 5
= 10 : 15 : 6
O
7x 21 Ratio of profit of X : Y : Z.
40 × 21 = 5 × 12 + 5 × 4 : 15 × 12 : 6 × 12
7x =
H
20 = 80 : 180 : 72
2 × 21 = 20 : 45 : 18
x =
_T

7 Sum of ratios = 83
= 6 months
(viii) If the investments of three partners are 20
∴ X’s share = × 20003 = Rs 4,820
in the ratio x : y : z, and their profits 83
C

are in the ratio P1 : P2 : P3, the time 45


periods of their investment are in the Y’s share = × 20003 = Rs 10,845
PS

83
ratio
18
P1 P P Z’s share = × 20003 = Rs 4,338.
: 2 : 3 83
x y z ● A and B together invested Rs 1,800 in a
U

business. At the end of the year, out of a


● Ram, Shyam and Anuj started a total profit of Rs 1,500, A’s share was
@

business by investing their capitals in Rs 500. What was the investment of A?


the ratio 3 : 4 : 6. At the end of the
business term, they received the A’s investment Profit share of A
profits in the ratio 1 : 2 : 3. Find the =
B’s investment Profit share of B
ratio of time for which they invested
500 500 1
their capitals. ∴ = =
1500 − 500 1000 2
We have x : y : z = 3 : 4 : 6
p1 : p2 : p3 = 1 : 2 : 3 1
∴ Investment of A = × 1800
Therefore, the ratio of time for which 1+ 2
Ram, Shyam and Anuj invested their 1
capitals = × 1800
3
p1 p2 p = Rs 600.
= : : 3
x y z
Unit Two : Fundamentals / Profit and Loss ✫ 127

Profit and Loss ● If the article is sold at its cost price


Profit is an amount of money that is gained itself, then there is neither profit nor
in business or trade. When the cost of making loss. This is because in such cases S.P.
something is less than the amount it is sold is equal to C.P. and thus there is no
for, there is considered to be a profit or gain. loss and no profit.
And, if a business makes a loss, it earns less ● Profit or loss is expressed in terms of
money than it spends and is therefore in debt. rupees or in terms of per cent.
In other words, when a business or shop sells ● There are certain expenses like
something at a loss, it sells it for a price which transportation cost, rent, salaries,

TS
is less than what it cost to produce or less than packing costs, etc. Such costs are added
what it cost to buy it. to the cost price before ascertaining
The terms profit and loss are largely used profit/loss.
in trade problems. Some other terms used in Thus actual cost price

H
the context are cost price, selling price, marked = cost price + other costs.
price, and discount which are normally used in

G
problems relating to profit and loss. Gain/Loss and Per cent Gain/
Cost price (C.P.) is the price at which an Per cent Loss
article is purchased.
Selling price (S.P.) is the price at which
an article is sold. U (i) Gain or Profit = S.P. – C.P.
(ii) Loss = C.P. – S.P.
O
When selling price is greater than cost price
there is always some profit or gain. (S.P. > C.P. (iii) Percentage of Gain or Loss
Profit). Actual gain or loss
H
When the selling price is less than the cost = × 100
Cost Price
price, there is always a loss (S.P. < C.P. Loss). S.P. – C.P.
_T

Marked price is the list price of the goods. (iv) Gain % = × 100%
C.P.
Discount is a certain amount of rebate
S.P.
given to attract customers. = − 1 × 100%
Other terms related to the purchase of an C.P.
C

article include: C.P. – S.P.


MP = Marked Price (v) Loss % = × 100%
C.P.
PS

AP = Advertised Price S.P.


PP = Printed Price = 1− × 100%
LP = Listed Price C.P.
NP = Net Price
U

S.P.
(Net price = Marked price – discount) [Just the formula − 1 × 100% will do
C.P.
All these prices are the prices at which the to calculate gain or loss; if you get a negative
@

article is listed or marked to be sold. In other (–) sign in the answer, a loss is indicated, just
words, they are the price that are labelled on as a loss is indicated if S.P. is more than C.P.]
the article and the price at which the article
shall be sold. Relation between Gain/Loss Per cent
and Cost Price and Selling Price
Points to Remember
When an article is sold at a profit,
● Profit or loss is always calculated as a 100 + Gain percentage
percentage of cost price. Even if a (vi) S.P. = C.P. ×
100
problem gives a gain or loss amount/
S.P. % Gain
percentage in relation to the sale price, or = 1+
the gain/loss has to be recalculated on C.P. 100
cost price—which is known as real 100
gain/loss. (vii) C.P. = S.P. × 100 + Profit percentage
128 ✫ Quantitative Aptitude

When an article is sold at a loss, [The same problem may be also solved
100 − Loss percentage in the following way:
(viii) S.P. = C.P. × Loss % × C.P.
100 Loss =
S.P. % Loss 100
or = 1− Loss × 100
C.P. 100 ∴ C.P. =
Loss %
100
(ix) C.P. = S.P. × 100 − Loss percentage 24.50 × 100
=
5

TS
We may derive the following from the 2450
above equations. = = Rs 490
5
Gain/Loss C.P. ∴ S.P. = 490 – 24.50
(x) = and = Rs 465.50.]

H
% Gain / % Loss 100
S.P. C.P. Selling an Article at Different Prices
(xi) = Even if an article is sold at different prices,

G
100 + % Gain 100
the cost price remains the same. So, if S.P.1 and
Therefore, S.P.2 are two different sale prices for the article,

(xii)
Gain
Gain %
=
Cost Price
100
=
Sale Price
100 + Gain % U we may say
C.P.
=
S.P.1
=
S.P.2
O
[If loss is incurred, ‘gain’ is to be replaced 100 100 + % Gain 1 100 + % Gain 2
by ‘loss’, and % loss should carry a negative [For loss, use negative sign (–).]
H
(–) sign.]
From the above formula, an unknown ● A man sold an article for Rs 750 and
_T

quantity may be found as needed by equating suffered a loss of 25%. At what price
any two parts. should he sell it to gain 50%?
● What is the cost price of an article if As C.P. is the same, we know
C

an 8% gain gives the seller a profit of S.P.1 S.P.2


Rs 60? =
100 − % Loss 100 + % Gain
PS

Gain C.P.
We know = (We may ignore the cost price here
Gain % 100 altogether.)
60 C.P.
= 750 S.P.2
=
U

8 100 So,
100 − 25 100 + 50
600
C.P. = = 75 750
@

8 S.P.2 = × 150
The cost price is Rs 75. 75

● Find the selling price of an article if the = Rs 1500


loss made by the seller is Rs 24.50 and The man should sell at Rs 1500 to gain
50 per cent.
loss per cent is 5.
S.P. Loss When Selling Price is the same but
= Gain %/ Loss % is Different
100 − Loss % Loss %
S.P. 24.50 In case the articles are different though selling
= price is the same, the cost price must be
100 − 5 5
different.
24.50
S.P. = × 95 = 465.50 (a) The selling price being the same, let the
5
The selling price of the article is two gains/losses be x1 and x2 respectively.
Rs 465.50. Then,
Unit Two : Fundamentals / Profit and Loss ✫ 129

Overall gain % or loss % We have, x1 = 15 and x2 = –20


Then, using the formula
2 (100 + x1 ) (100 + x2 ) Overall gain/loss %
= 100 −
(100 + x1 ) + (100 + x2 )
100 (x1 + x2 ) + 2 x1x2
[A negative (–) sign will indicate loss.] = %
(100 + x1 ) + (100 + x2 )
● Amir sold two items for Rs 2400 each.
On one he gained 15% and on the other 100 (15 − 20) + 2 × 15( −20)
%
he lost 10%. How much did he gain (100 + 15) + (100 − 20)

TS
or lose on the whole transaction? Here,
the gain % (x1) on the first item is 15% −500 − 600
and the loss % (x2) on the second item =
115 + 80
is 10%.

H
−1100
Overall gain %/loss % =
195
2 (100 + x1 ) (100 + x2 )

G
= 100 − = –5.64%
(100 + x1 ) + (100 + x2 )
∴ Overall loss = 5.64%.
100 −
2 (100 + 15) (100 − 10)
(100 + 15) + (100 − 10) U (b) The selling price being the same, if the
gain % on one article is equal to the loss %
O
2 × 115 × 90 on the other article, there is always an overall
= 100 – loss.
115 + 90
H
If gain % = loss % = x,
2 × 115 × 90 2
= 100 – x
_T

205 Overall loss = %


10
2 × 23 × 90
= 100 – ● If the selling price of two chairs is
41
Rs 600 each and 10% loss on one chair
C

4100 − 4140 −40 is equal to 10% gain on the other, then


= = = –0.98%
41 41 find the total loss %.
PS

As the sign is negative there is an


overall loss of 0.98% on the transaction. If x is the gain/loss %,
2
x
Alternative Formula overall loss % = %
U

10
There is another formula for the same situation: 2
10
= = 1%
@

(a) If two different articles are sold at the 10


same selling price, getting gain or loss of x1%
on the first and gain or loss of x2% on the Discount
second, then A merchant buys an article at a price called
Overall % gain or % loss in the transaction
cost price. He labels the article at a price
100 (x1 + x2 ) + 2 x1x2 (usually higher than the cost price). This is the
= %
(100 + x1 ) + (100 + x2 ) marked price. He may allow a discount on the
(–ve sign would indicate overall loss %) marked price before selling it to a customer.
Generally even after the discount, the merchant
● Amay sold two articles, each for Rs
makes a profit. It is possible in some cases that
1800. If he makes 15% profit on the first
he gives a heavy discount and the selling price
and 20% loss on the second, what is
his overall profit or loss per cent in the falls below the cost price and there is a loss
transaction? on the transaction.
130 ✫ Quantitative Aptitude

Now, if d% is the discount on the marked Series of Discounts A trader may allow
price (M), more than one discount on the marked price of
an item to be sold. These are known as
d
(i) Discount = ×M ‘successive discounts’ or ‘series of discounts’.
100 After the successive discounts the customer
(ii) Selling price or customer price
pays a net selling price.
=M–d
d (i) If marked price is M, and x and y are
=M– M the two successive discounts, and N the net
100
d selling price,

TS
= M 1− or M (1 – d%)
100 100 − x 100 − y
N = M
or M (100 – d)% 100 100
(iii) Selling Price = Cost Price (1 + gain%)

H
x y
(iv) From (ii) and (iii) we get Or N = 1− × 1− ×M
M (1 – d%) = S.P. = C.P. (1 + gain %) 100 100

G
● A machine cost a dealer Rs 516. He This rule is applicable for more successive
wishes to mark it so that he may deduct discounts as well, only then that many
20% from the marked price, and still 100 − discounts
make a profit of 25% of the cost. What
is the marked price? U 100
together.
would have to be multiplied
O
Here the C.P. = Rs 516, gain = 25% If there are three successive discounts of
d = 20% x, y and z.
H
So, M (1 – d%) = C.P. (1 + gain %)
x y z
25 N = M 1− 1− 1−
M (1 – 20%) = 516 1 + 100 100 100
_T

100
20 125 x y
M 1− = 516 As the calculation of 1− × 1−
100 100 100 100
C

M (80%) = 516 + 129 = 645 z


× 1− , etc. may be lengthy, a single
645 100 100
M= = 645 × = Rs 806.25
PS

80% 80 equivalent discount for the series of discounts


The marked price must be Rs 806.25. can be worked out.
● A shopkeeper offers customers a 10% (ii) Single equivalent discount (D) for two
discount on a vase and still makes a successive discounts x and y
U

profit of 26 per cent. If the vase was xy


= x + y –
marked Rs 140, what did it cost the 100
@

shopkeeper? Now,
Here M = Rs 140, d = 10%, gain = 26% D
N = M 1−
using the formula, 100
M (1 – d%) = C.P. (1 + gain %)
● The marked price of a microwave oven
140 (1 – 10%) = C.P. (1 + 26%)
100 − 10 100 + 26 is Rs 10,000. What would I need to pay
= 140 = C.P. if two successive discounts of 20% and
100 100 20% are available on the oven?
90 126 x y
= 140
100
= C.P.
100 N = M 1− 1−
100 100
90 100
C.P. = 140 × × Here M = Rs 10000, x = 20 and y = 20
100 126
= Rs 100 20 20
The cost price is Rs 100. N = 10000 1− 1−
100 100
Unit Two : Fundamentals / Profit and Loss ✫ 131

80 80 True measure
= 10000 100 + g = × (100 + x )
100 100 False measure
= 80 × 80 = Rs 6400.
(ii) If the shopkeeper sells his goods at cost
● A microwave oven priced at Rs 15,000 price, then x = 0
is available at two successive discounts True measure
of 10% and 20%. What is the single 100 + g = × 100
False measure
equivalent discount?
Here the single equivalent discount (D) ● A cloth merchant says that he sells cloth

TS
for two successive discounts x and y at 10% loss, but he uses a false metre
xy scale and actually makes a gain of 15%.
= x + y – What is the length of his false measuring
100
scale?

H
10 × 20
D = 10 + 20 – 100 + g True Scale
100 =

G
= 30 – 2 = 28% 100 + x False Scale
Here g = 15%, x = –10% (loss) and true
● What would be the equivalent discount
of a series of discounts of 10%, 20% and
25%? U scale = 100 cm. So, we have
100 + 15
=
100
O
100 − 10 False Scale
Here the equivalent discounts have to 100 − 10
False scale = 100 ×
H
be worked out two at a time. So, if the 100 + 15
discounts are 10%, 20% and 25%, work 90
out the equivalent discount for 10% and = 100 × = 78.26 cm.
_T

115
20% first and then the discount
The length of the false measuring scale
equivalent of the result and 25%. is 78.26 cm instead of 1 metre.
Discount equivalent to 10% and 20%
C

● A dealer professing to sell at cost price


10 × 20
= 10 + 20 – uses a 900 gm weight for a kilogram.
100
PS

Find his gain per cent.


= 10 + 20 – 2 = 28% We have true weight = 1000 gm,
false weight = 900 gm.
Discount equivalent to 28% and 25%
U

If g is the Gain %
25 × 28
= 25 + 28 – True weight
100 100 + g = × 100
@

= 25 + 28 – 7 = 46% False weight


So the equivalent discount of the series 1000
100 + g = × 100
of three discounts is 46%. 900
1000
100 + g =
Use of False Scales/Weights in Buying 9
and Selling 1000
Gain % = – 100
When a shopkeeper uses false scale or false 9
weights or measures and announces that he 1000 − 900
=
sells his goods at loss/gain of x%, then his 9
overall gain % (g) is given as 100
=
100 + g True weight True measure 9
(i) = 1
100 + x False weight False measure Gain % = 11 %.
9
132 ✫ Quantitative Aptitude

(iii) A shopkeeper uses false scales to weigh x × (100 + b)


and deceives a seller by x% when =
(100 − a ) + (100 + b)
purchasing an article. Later he deceives C.P. of the item sold at gain
his customer by y% when selling that x × (100 − a )
article. That means he has gained at two =
(100 − a ) + (100 + b)
levels. His overall gain
● Ankur bought two cycles for Rs 980.
(100 + x ) (100 + y ) On one he gains 20% and on the
G = − 100 %
100 other he loses 20%. What is the cost
price of each cycle?

TS
Some More Fast-track Formulae and We have x = 980, a = 20 and b = 20
C.P. of the cycle sold at loss
Worked Examples
x × (100 + b)

H
(i) If the cost price of x articles is equal =
to the selling price of y articles, then (100 − a) + (100 + b )

G
x−y 980 × (100 + 20)
% gain or loss = × 100 =
y (100 − 20) + (100 + 20)
[gain in case of x > y and loss in case
of x < y]
U =
980 × 120
80 + 120
O
● The selling price of 48 pencils is equal
to the cost price of 64 pencils. Find 980 × 120
= = Rs 588
the profit percentage. 200
H
C.P. of the cycle sold at gain
If x = 64 and y = 48
x−y x × (100 − a)
_T

Proft % = × 100 =
y (100 − a) + (100 + b )
64 − 48
= × 100 980 × (100 − 20)
48 =
C

(100 − 20) + (100 + 20)


16
= × 100 980 × 80
PS

48 =
100 1 80 + 120
= = 33 % .
3 3 980 × 80
= = Rs 392.
U

● The selling price of 32 pencils is equal 200


to the cost price of 24 pencils. Find The cost of one cycle is Rs 588 and the
the loss percentage.
@

cost of the other cycle is Rs 392.


If x = 24 and y = 32 (iii) A man purchases a certain number of
y−x articles at Rs x and the same number at
Loss % = × 100 Rs y. He mixes them together and sells
y
[  Loss in case of x < y) them at Rs z. Then his gain or loss %
2 xy
32 − 24 = − 1 × 100 according as the sign
So we have = × 100 z (x + y )
32
8 × 100 is ‘+ve’ or ‘–ve’.
= = 25%.
32 ● A shopkeeper buys two types of
(ii) If cost price of two items is Rs x, and sugar, one for Rs 1600 and the other
if one is sold at a loss of a% and the for Rs 1000. He mixes them together
other at a gain of b% then, and sells at Rs 1200. Find his gain per
C.P. of the item sold at loss cent or loss per cent.
Unit Two : Fundamentals / Profit and Loss ✫ 133

We have x = 1600, y = 1000 and z = 1200 ● A manufacturer sells an article to a


Then, wholesale dealer at a profit of 20%
Gain % or loss % and the wholesale dealer sells it to
a retail merchant at a loss of 5%. Find
2 xy the resultant profit or loss.
= − 1 × 100
z (x + y) Here, x = 20 and y = –5
Resultant profit or loss %
2 × 1600 × 1000
− 1 × 100 xy
1200 (1600 + 1000) = x+y+ %

TS
100
3200000
= − 1 × 100 20 × (−5)
1200 × (2600) 20 − 5 + %
100

H
3200000 100
= − 1 × 100 = 15 − %
3120000 100

G
= (15 – 1)%
3200000 − 3120000 = 14%
= × 100

=
3120000
80000
× 100 U Which represents profit as the sign is
+ve.
(v) If a man buys x items for Rs p and
O
3120000 sells y items for Rs q then the gain or
= 2.56% loss per cent made by him is equal to
H
∴ The shopkeeper’s gain is 2.56%. x q
−1 × 100%
(iv) If A sells an article to B at a gain/loss y p
_T

of x% and B sells it to C at a gain/loss (In case of loss per cent, the result
of y%, then obtained bears –ve sign)
(a) If x% and y% are profits,
xy ● Rajesh bought 6 pencils for Rs 5 and
C

Resultant profit % = x + y + %
sold 3 of them for Rs 3. Find his gain
100
(b) If x% is profit and y% is loss, per cent.
PS

xy We have x = 6, y = 5, p = 3 and q = 3
Resultant loss % = x − y − %
100 xq
according to the –ve and +ve Gain % = y p − 1 × 100%
U

signs respectively.
6×3
● A sells a scooter to B at a profit of − 1 × 100%
@

10% and B sells it to C at a profit 5×3


of 20%. Find the resultant profit. 18 − 15
= × 100%
Here, x = 10 and y = 20 15
Resultant profit % 3
xy = × 100% = 20%
15
= x+y+ %
100 Gain is 20%.
10 × 20 (vi) A sells an article to B at a gain/loss
10 + 20 + % of x% and B sells to C at a gain/loss
100
of y%. If C pays Rs p for it to B then
200
= 30 + % the cost price for A is equal to
100 100 2 p
= (30 + 2)% (100 + x ) (100 + y )
= 32%. (–ve value of x or y indicates loss)
134 ✫ Quantitative Aptitude

● Amit sells a cellphone to Gaurav at


a loss of 10% and Gaurav sells it to ( xy + xz + yz) xyz
x+y+z− +
Mohan at a loss of 20%. If Mohan 100 (100)2 %
pays Rs 1800 for it, at what price did
Amit buy it? ● Find a single discount equivalent to
three successive discounts of 20%,
We have x = –10, y = –20 and p = Rs 1800
30% and 40%.
1002 p We have x = 20, y = 30 and z = 40
Cost price =
(100 + x) (100 + y) Single equivalent discount

TS
100 2 × 1800 ( xy + xz + yz ) xyz
= x+y+x− − %
(100 − 10) (100 − 20) 100 100 2
100 × 100 × 1800 ∴ Single equivalent discount

H
=
90 × 80 (20 × 30 + 20 × 40 + 30 × 40)
= 20 + 30 + 40 −
= Rs 2500. 100

G
(vii) A shopkeeper sells an item for Rs p 20 × 30 × 40
after giving a discount of x% on the − %
1002
labelled price. Had he not given the
discount, he would have earned a U = 90 −
600 + 800 + 1200 24000
+ %
O
profit of y% on the cost price. The cost 100 100 2
price of the item is given by
2600 24000
90 − +
H
(100)2 p = %
100 10000
C.P. =
(100 − x ) (100 + y )
_T

24
= 90 − 26 + %
● A shopkeeper sold shirts at Rs 354 10
each after giving 8% discount on 900 − 260 + 24
labelled price. Had he not given the = %
C

10
discount, he would have earned a 664
profit of 18% on the cost price. What
PS

= %= 66.4%.
was the cost price of each shirt? 10

We have discount (x) = 8%,


profit (y) = 18% and labelled price = Rs Miscellaneous Examples
U

354 ● A dealer marks his goods 25% above cost


price. He then allows some discount on it
(100)2 p
@

and makes a profit of 10%. Find the rate


Cost price =
(100 − x ) (100 + y ) of discount.
Let C.P. be Rs 100. Then,
100 × 100 × 354 Marked Price (M.P.) = Rs 125 and S.P. = Rs 110
(100 − 8) (100 + 18)
M.P. – S.P.
Rate of discount = × 100 %
M.P.
100 × 100 × 354
= 125 – 110
92 × 118 × 100 %
125
= Rs 326.
(viii) If three successive discounts on an article 15
× 100 %
=
are x%, y% and z% respectively, then a 125
single discount equivalent to the three = 12%
successive discounts will be Rate of discount is 12%.
Unit Two : Fundamentals / Alligation or Mixture ✫ 135

● A chair is listed at Rs 160 with a discount If two ingredients are mixed, then
of 25%. What additional discount must be Quantity of cheaper item
offered to the customer to bring the net Quantity of superior item
price to Rs 108?
Cost of superior item − Mean cost
Here, List price = Rs 160, Discount = 25% =
Mean item − Cost of cheaper item
Selling Price (S.P.) = 75% of Rs 160
Here, the cost price (C.P.) of a unit quantity
75 of the mixture is called the mean price.
= Rs 160 × = Rs 120
100 We can also write the above formula as

TS
Net price = Rs 108 C.P. of a unit of C.P. of a unit of
Additional discount cheaper quantity (a) superior quantity (b)

H
Selling price – Net price
= × 100 %
Selling price Mean price
(m)

G
Additional discount
120 − 108 (b – m) (m – a)
× 100 %
=
120
12 U ∴
Quantity of cheaper item b − m
=
Quantity of superior item m − a
O
= × 100 %
120
= 10%. Ratio – Quantity Rule
H
If a mixture of two ingredients x1 and x2
contains Q amount in total, the quantity of
_T

Alligation or Mixture x1
x1 = × Q
Alligation literally means ‘linking’. x1 + x 2
Also called the Rule of Mixture, the and quantity of
C

Alligation Rule is useful for finding x2


x2 = × Q
(i) the combined average of a mixture of x1 + x 2
PS

items which are divided into two groups, each


● In what proportion must a merchant
group with its own average; and mix rice priced at Rs 10 per kg and rice
(ii) the ratio in which items from two or priced at Rs 10.44 per kg so as to make
U

more groups should be taken to get a combined the mixture worth Rs 10.24 per kg?
average for all the items.
As per the rule of alligation
@

First average (a) Second average ( b) Quantity of cheaper rice


Quantity of superior rice
10.44 − 10.24
Combined average (m) =
10.24 − 10
(the mean price being Rs 10.24)
(b – m ) (m – a ) 0.20 5
|(m – b)| |(m – a )| = =
0.24 6
The required ratio is (b – m) : (m – a) The proportion of the two kinds of rice
[or |(m – b)| : |m – a|] must be 5 : 6.
[|(m – b)| and |(m – a)| being absolute ● Two mixtures of milk and water contain
values, they do not carry a negative sign.] 10% and 20% of water. In what ratio
136 ✫ Quantitative Aptitude

should the two mixtures be mixed so x2


that the new mixture has 16% water? x2 = × Amount of mixture
x1 + x2 + x3
Mixture 1 Mixture 2 x3
10% 20% x3 = × Amount of mixture
x1 + x2 + x3
● In what proportions must nuts of
16%
different quality worth Rs 333, Rs 335
and Rs 338 per kg be mixed to produce
|(16 – 20)| = 4 |(16 – 10)| = 6 a mixture worth Rs 336 per kg.

TS
The ratio required is 4 : 6 or 2 : 3.
Here a1 = Rs 333
● In an examination, a correct answer a2 = Rs 335
gets the candidate 3 marks, but there a3 = Rs 338

H
is negative marking of 1 for every m = Rs 336
wrong answer. Marup gets 0 in the x 1 : x2 : x3
paper of 100 questions. How many

G
= (m – a2) (m – a3)
answers did he get correct?
: (m – a1) (m – a3)
Correct Wrong
3 –1

U =
: (m – a2) (m – a1)
(336 – 335) (336 – 338)
O
: (336 – 333) (336 – 338)
0
: (336 – 335) (336 – 333)
H
1
= (1 × 2) : (3 × 2) : (1 × 3)
3
Here, = 2:6:3
_T

The proportion of the nuts of different


Correct Answers |1 − 0| 1
= = quality will be in the ratio 2 : 6 : 3.
Wrong Answers |(0 − 3)| 3
Mean Value of a Mixture
The ratio of correct to wrong answers
C

If the quantities of n number of ingredients are


is 1 : 3
So, out of a total of 4 answers, 1 is x1, x2, x3 . . . xn, and a1, a2, a3 . . . an are the respective
PS

correct. cost prices of the ingredients, then


1 Mean value or cost (m)
∴ Correct answers = × 100 = 25.
4 ( a1x1 + a2 x2 + a3 x3 + . . . + anxn )
=
U

Rule of Alligation for Mixture of ( x1 + x2 + x3 + . . . xn )


Three Ingredients
Adding the Contents of Several Vessels
@

If ingredients x1, x2 and x3 with cost prices of


a1, a2 and a3 respectively are mixed to produce (a) Suppose we have n number of vessels
a mixture of mean price m, then of equal size filled with mixtures of liquids A
and B in the ratio a1 : b1, a2 : b2, .... an : bn
Proportion of x1 = (m – a2) (m – a3) [Leave out cost of x1] respectively. If the contents of all the vessels
Proportion of x2 = (m – a1) (m – a3) [Leave out cost of x2] are poured into a single large vessel, then
Proportion of x3 = (m – a2) (m – a1) [Leave out cost of x3] Quantity of liquid A
We get x1 : x2 : x3 Quantity of liquid B
= (m – a2) (m – a3) : (m – a1) (m – a3) : (m – a2) (m – a1) a1 a2 an
+ + ... +
The required quantities can now be found. a1 + b1 a2 + b2 an + bn
=
x1 b1 b2 bn
x1 = × Amount of mixture + + ... +
x1 + x2 + x3 a1 + b1 a2 + b2 an + bn
Unit Two : Fundamentals / Alligation or Mixture ✫ 137

● We have four vessels of equal size


filled with mixtures of milk and water a1c1 a c a c
+ 2 2 + ... + n n
in the ratio 3 : 2, 4 : 3, 5 : 4 and 7 : 6 a1 + b1 a2 + b2 an + bn
respectively. If all the contents are =
poured into a single large vessel, then b1c1 b c b c
+ 2 2 + ... + n n
what is the new proportion of milk and a1 + b1 a2 + b2 an + bn
water in it?
We have a1 = 3, a2 = 4, a3 = 5, a4 = 7 and ● We have three vessels of capacities 2
b1 = 2, b2 = 3, b3 = 4, b4 = 6 litres, 3 litres and 4 litres filled with

TS
mixtures of liquids, milk and water in
Quantity of milk the ratio 2 : 3, 3 : 4 and 4 : 5
Quantity of water respectively. If the contents of the
vessels are poured into a single large

H
a1 a2 a3 a4 vessel, then find the ratio of milk and
+ + +
a1 + b1 a2 + b2 a3 + b3 a4 + b4 water in the new mixture?

G
=
b1 b2 b3 b4 We have a1 = 2, a2 = 3, a3 = 4, b1 = 3, b2 = 4,
+ + +
a1 + b1 a2 + b2 a3 + b3 a4 + b4
3
+
4
+
5
+
7
U b3 = 5 and c1 = 2, c2 = 3, c3 = 4
Quantity of milk
O
3+ 2 4 + 3 5+ 4 7 + 6 Quantity of water
2 3 4 6
+ + +
H
3+ 2 4 + 3 5+ 4 7 + 6 a1 c1 a c a c
+ 2 2 + 3 3
a1 + b1 a2 + b2 a3 + b3
3 4 5 7 =
+ + +
_T

b1 c1 b c b c
5 7 9 13 + 2 2 + 3 3
= a1 + b1 a2 + b2 a3 + b3
2 3 4 6
+ + +
5 7 9 13
2×2 3×3 4×4
C

3 × 819 + 4 × 585 + 5 × 455 + 7 × 315 + +


2+ 3 3+ 4 4+ 5
3×2 4×3 5× 4
PS

= 4095
2 × 819 + 3 × 585 + 4 × 455 + 6 × 315 + +
2+ 3 3+ 4 4+ 5
4095
4 9 16
U

2457 + 2340 + 2275 + 2205 + +


= 5 7 9
1638 + 1755 + 1820 + 1890 =
6 12 20
@

+ +
9277 5 7 9
= litres
7103
or 1.31 litres (approximate) 4 × 63 + 9 × 45 + 16 × 35
= 315
(b) Suppose we have n number of vessels of 6 × 63 + 12 × 45 + 20 × 35
size c1, c2, ..., cn filled with mixtures of liquids A 315
and B in the ratio a 1 : b1, a2 : b2, ..., an : bn,
respectively. If the contents of all the vessels are 252 + 405 + 560
=
poured into a single large vessel, then 378 + 540 + 700
Quantity of liquid A 1217
= litres or 0.75 litres.
Quantity of liquid B 1618
138 ✫ Quantitative Aptitude

Removal and Replacement by a 3


60
Certain Amount 180 × 1 −
180
(a) When a certain quantity (x) of A is
3
removed from a Mixture (M) containing 1
ingredients A and B and replaced by x quantity = 180 × 1 −
3
of B, and this is done n times, after n
3
operations 2
= 180 ×
Amount of A removed 3
=

TS
Amount of A originally present 8
x
n = 180 ×
= 1− 27
M 160 1
= = 53 litres.

H
Amount of B removed 3 3
= M – Amount of A removed
● 7 litres are drawn from a cask full of
[Amount of A removed is first found as

G
wine and water replaces the wine
its replacement is by an equal amount of B.]
removed. This operation is performed
three more times. The ratio of the
(b) When there is only one ingredient A
and a certain amount x is removed and replaced
in equal amount by another ingredient B and U quantity of wine now left in the cask
to that of water is 16 : 81. How much
O
this removal-replacement operation is done n wine did the cask hold originally?
times, then Let the quantity of wine in the cask
H
n
Amount of A remaining x originally be x litres.
= = 1−
Amount of A initially present a Then, quantity of wine left in the cask
_T

after 4 operations
[where a is the amount of A originally present.]
n 4
x 7
∴ Amount of A remaining = a 1 − = x 1− litres
a x
C

Also
n
x 4
PS

1− 7 16
Amount of A remaining a ∴ x 1− =
= x 81
Amount of B remaining n
x
1− 1− 4 4
a 7 2
1−
U

=
x 3
● A container contains 180 litres of vinegar.
@

7 2
60 litres of vinegar was taken out of 1− =
the vessel and replaced by water. Then x 3
60 litres of mixture was again x −7 2
withdrawn and replaced by water. The =
x 3
operation was again repeated for the 3x – 21 = 2x
third time. How much vinegar is now 3x – 2x = 21
left in the vessel? x = 21 litres.
We have a = 180, x = 60 and n = 3
[The problems on alligation may be solved
Amount of vinegar left in the vessel easily if the above basic formulae are understood
n and applied correctly. The principles of ratio
x
= a × 1− and proportion and percentage must also be
a kept in mind.]
Unit Two : Fundamentals / Alligation or Mixture ✫ 139

Miscellaneous Worked Examples Water = 2k = 2 × 9


= 18 litres
● The average age of children going in a bus
Now suppose x litres of water is added
is 11 years. If the average age of boys is
to the mixture such that
11.2 years and that of girls is 10.9 years,
how many children are in the bus given 27 9
=
that the boys total 50? 18 + x 11
162 + 9x = 297
Boys Girls 9x = 297 – 162
11.2 10.9 9x = 135

TS
x = 15 litres.
11
● How many kg of apple costing Rs 8 per
kg must be mixed with 20 kg of oranges

H
11 – 10.9 = 0.1 11.2 – 11 = 0.2 costing Rs 4 per kg so that 20% gain may
Ratio of boys to girls = 0.1 : 0.2 or 1 : 2 be obtained by selling the mixture at Rs

G
If there are 50 boys in the bus, there are 7.20 per kg?
50 × 2 = 100 girls in the bus. S.P. of 1 kg mixture = Rs 7.20 per kg,


Total number of children is 150.
A mixture of chemical and water contains U gain 20%
∴ C.P. of 1 kg mixture = Rs
100
120
× 7.20
O
10% water. How many litres of water = Rs 6
must be added to make it 37% of the new By alligation rule, we have
H
mixture?
10 C.P. of 1 kg apple C.P. of 1 kg of orange
10% of 70 l = 100 × 70 = 7 l of water
_T

800 paise 400 paise


So chemical is 70 – 7 = 63 l
In the new mixture, water is 37%, and Mean price
chemical will be 100 – 37 = 63% 600 paise
C

Now, if chemical is 63 l, water is 37 l.


∴ Water to be added is 37 – 7 = 30 l. (d – m ) (m – c )
PS

= 200 = 200
● How much water must be added to 100 cc
of 80% solution of boric acid to reduce it to Quantity of apple 200 1
a 50% solution? ∴ = =
Quantity of orange 200 1
U

In 100 cc solution, quantity of boric acid Let x kg of apple be mixed with 20 kg


is 80 cc and quantity of water is 20 cc. of orange. Then, 1 : 1 = x : 20
@

Suppose x cc of water is added to make x = 20 kg.


the solution 50%.
∴ 50% (100 + x) = 80 x = 60 cc ● A person has a chemical of Rs 40 per litre.
In what ratio should water be mixed in
● In a mixture of 45 litres, the ratio of milk that chemical so that after selling the
and water is 3 : 2. How much water must mixture at Rs 30/litre he may get a profit
be added to make the ratio 9 : 11? of 30%?
Let milk and water be 3k and 2k respectively.
3k + 2k = 45 S.P. of mixture = Rs 30 per litre,
5k = 45 profit 30%
k = 9 100
∴ Average price = × 30
Milk = 3k = 3 × 9 130
= 27 litres = Rs 23.07 per litre
140 ✫ Quantitative Aptitude

Chemical Water ● The present age of a man is six times


40 0 the age of his son. Five years ago, the
age of the man was 11 times the age
of his son at that time. How old is the
Mean price
23.07
man at present?

( d – m) (m – c ) We have, x1 = 11, x2 = 6 and t = 5 years


23.07 16.93 The man’s present age
x1 − 1
∴ Chemical : Water = 23.07 : 16.93. = x2 t
x1 − x 2

TS
11 − 1
Problems on Age × 6 × 5
11 − 6

H
Age-related problems often come in quantitative
aptitude tests. While most of these problems 10
can be solved using algebraic expressions and = × 6 × 5

G
5
equations, there are some formulae that could
= 60 years.
be helpful.
(i) The present age of A is x1 times the
present age of B. If t years hence, the age of
U (iii) If the ratio of the present age of A
and of B is p : q and t years hence, it will
O
A would be x2 times that of B, then be r : s, then
x2 − 1 pt ( r − s)
A’s present age = x2 t years
H
x1 − x 2 A’s present age =
ps − qr
x2 − 1 qt ( r − s)
_T

and B’s present age = t years and B’s present age =


x1 − x2 ps − qr
● A man’s age is now three times that of (If, with the ratio of their present age, the
his son. In 15 years, it will be double ratio of their age t years ago is given, then
C

that of his son. Find the present age replace t by (–t) in the above formula.)
of the son. pt ( r − s)
PS

A’s present age = –


We have A, the man and B, the son; ps − qr
x1 = 3, x2 = 2 and t = 15 ● The ratio of the age of Ram and Shyam
Present age of the son at present is 6 : 1. After 4 years, the
U

x2 − 1 ratio will become 7 : 2. Find the present


= x − x t years age of Shyam.
@

1 2

2−1 Here, p = 6, q = 1, r = 7, s = 2 and t = 4


× 15 years The present age of Shyam
3−2
= 1 × 15 years qt ( r − s)
=
= 15 years. ps − qr
1 × 4 (7 − 2)
(ii) If the age of A, t years ago, was x1
times the age of B and at present A’s age is 6 × 2 −1×7
x2 times that of B, then 4 (5)
=
x1 − 1 12 − 7
A’s present age = x t years
x1 − x 2 2 20
= = 4 years
x1 − 1 5
and B’s present age = t years Hence, the present age of Shyam is 4 years.
x1 − x 2
Unit Two : Fundamentals / Problems on Ages ✫ 141

(iv) The sum of the present age of A and years, the age of the father will be 5
that of B is T years and a years ago, the age times that of the son. Find their
of A was x times the age of B. respective ages.
Tx − a (x − 1) We have T = 60, x = 5, a = 3
A’s present age = years,
x+1 Tx + a (x − 1)
Father’s age =
T + a (x − 1) x+1
and B’s present age = years
x +1 60 × 5 + 3 (5 − 1)
● The sum of the age of Shyam and that 5+1

TS
of Gopal is 45 years. 5 years back, the 300 + 12
age of Shyam was 4 times the age of =
6
Gopal. Find the difference between the
312
present age of Shyam and that of = = 52 years

H
Gopal. 6
T − a ( x − 1)
We have, T = 45, x = 4, a = 5 The son’s age =

G
Shyam’s present age x+1
60 − 3 (5 − 1)
Tx − a (x − 1)
=
x+1
U 5+1
60 − 12
O
45 × 4 − 5 (4 − 1) =
6
4+1 48
H
180 − 15 = = 8 years.
6
= (vi) The age of A, a1 years ago, was x1
5
_T

times the age of B. If a2 years hence A’s age


165 would be x2 times that of B, then
= = 33 years
5 Present age of A
and Gopal’s present age
x1 ( a1 + a2 ) ( x 2 − 1)
C

T + a (x − 1) = + a1 years
= x1 − x 2
x +1
PS

and present age of B


45 + 5 (4 − 1)
a2 ( x 2 − 1) + a1 ( x1 − 1)
4+1 = years
x1 − x 2
45 + 15
U

● Eight years ago Sonu’s mother was 5


=
5 times older than her son. After 8 years,
60 the mother will be three times older
@

= = 12 years than her son. Find the present age of


5
∴ Difference between the present age of Sonu.
Shyam and the present age of Gopal We have, a1 = 8, a2 = 8, x1 = 5, and x2 = 3
= 33 – 12 = 21 years. Present age of Sonu
(v) The sum of the present age of A and
a2 ( x 2 − 1) + a1 ( x1 − 1)
that of B is T years. If a years hence, the age =
of A would be x times the age of B, x1 − x 2
Tx + a (x − 1) 8 (3 − 1) + 8 (5 − 1)
A’s present age = years 5−3
x+1
T − a ( x − 1) 16 + 32
and B’s present age = years =
x+1 2
● The sum of the age of a son and the 48
age of his father is 60 years. After 3 = = 24 years.
2
142 ✫ Quantitative Aptitude

Formulate Equations for the Problems ∴ The present age of the three persons are
Even if you cannot remember all the formulae, (4 × 4), (7 × 4) and (9 × 4)
you should be able to formulate algebraic Or 16 years, 28 years and 36 years.
equations from the problem before you and
● The age of two persons differ by 16
reach the correct answer by solving the
years. If 6 years ago, the older one was
equations.
3 times as old as the younger one, find
See the following worked examples:
their present age.
● Tipu’s grandfather was 8 times older Let the younger person’s age be x
to her 16 years ago. He would be 3 years.

TS
times of her age 8 years from now. The older person will then be (x + 16)
Eight years ago, what was the ratio of years.
Tipu’s age to that of her grandfather? By the given conditions,

H
Let Tipu’s (T’s) age 16 years ago be x 3 (x – 6) = (x + 16 – 6)
years. Or 3x – 18 = x + 10
Or 2x = 28

G
Her grandfather’s (G’s) age was 8x years.
Or x = 14
8 years from now, T will be (x + 16 + 8)
The younger person is at present 14
years = (x + 24) years
and G will be (8x + 16 + 8) years
U years old, while the older person is
(14 + 16 =) 30 years old.
O
= (8x + 24) years
● One year ago, the ratio of Gaurav’s
We are given that G will be 3 times and Sachin’s age was 6 : 7 respectively.
H
T’s age 8 years from now. Four years hence, this ratio would
∴ (8x + 24) = 3 (x + 24) become 7 : 8. How old is Sachin?
_T

Or 8x – 3x = 72 – 24
Ratio merely means that if Gaurav’s
48 age was 6x, Sachin’s age was 7x one
Or 5x = 48, or x =
5 year ago.
C

We have to find the ratio of their ages 8 Gaurav’s age 4 years from now
years ago.
48 = (6x + 1) + 4
+8
PS

T x+8 5 = (6x + 5) years


= =
G 8x + 8 48 (Remember, it was 6x one year ago)
8× +8
5 Sachin’s age 4 years from now
U

88 11 = (7x + 1) + 4 = 7x + 5 years
= =
424 53 After 4 years the ratio of their age will
@

become 7 : 8.
The ratio of Tipu’s age to her grand-
father’s age 8 years ago was 11 : 53. Gaurav’s age 4 years hence 7
∴ :
Sachin’s age 4 years hence 8
● The present age of three persons are
in proportions 4 : 7 : 9. Eight years ago, 6x + 5 7
the sum of their age was 56. Find their Or =
7x + 5 8
present age (in years).
Or 8 (6x + 5) = 7 (7x + 5)
Let their present age be 4x, 7x and 9x. Or 48x + 40 = 49x + 35
By the given conditions, Or x = 5
(4x – 8) + (7x – 8) + (9x – 8) = 56 Sachin’s present age is
Or 20x = 56 + 24 (7 × 5 + 1) = 36 years.
80 (Don’t forget to add the ‘1’, as 7x
Or x= =4 refers to Sachin’s age one year ago.)
20
Unit Two : Fundamentals / Problems on Ages ✫ 143

● The ratio between the present age of ● Sumit is as much younger to Ankit as
P and Q is 6 : 7. If Q is 4 years older he is older to Gopal. If the sum of the
than P, what will be the ratio of the age of Ankit and that of Gopal is 52
age of P and Q after 4 years? years, how old is Sumit?
Let P’s age and Q’s age be 6x years A is Amit, S is Sumit and G is Gopal
and 7x years respectively. A – S = S – G
Then, 7x – 6x = 4 A + G = 2S
Or x = 4 2S = 52
∴ S = 26

TS
Required ratio = (6x + 4) : (7x + 4)
= (6 × 4 + 4) : (7 × 4 + 4) Hence, Sumit is 26 years old.
= 28 : 32 ● The sum of the age of four children,
= 7 : 8. each one born at the interval of 3

H
● A is two years older than B who is years, is 42 years. What is the age of
the youngest child?
twice as old as C. If the total of the

G
age of A, B and C, be 27, then how Let the age of the youngest child be
old is B? x years. Then,
Let C’s age be x
Then, B’s age
years
= 2x years U x + (x + 3) + (x + 6) + (x + 9) = 42
4x + 18 = 42
4x = 42 – 18
O
A’s age = (2x + 2) years 4x = 24
∴ (2x + 2) + 2x + x = 27 x = 6
H
Or 5x = 25 Hence, the youngest child age is 6
Or x = 5. years.
C _T
PS
U
@
144 ✫ Quantitative Aptitude

PRACTICE SESSION
Practice Exercise 8
1. What per cent of 49 is 84? 12. 45% of 750 – 25% of 480 = ?
(a) 58.66% (b) 171.43% (a) 217 (b) 218
(c) 58.33% (d) None of these (c) 236.50 (d) 217.50

TS
2. 77% of 11100 is 13. If 20% of a number is 120, then 120% of
(a) 777 (b) 7777 that number will be
(c) 8557 (d) 8547 (a) 620 (b) 520
(c) 720 (d) 360

H
3. ? % of 130 = 11.7
(a) 90 (b) 9 14. If 75 is added to 75% of a number, the
(c) 0.9 (d) 0.09 result is that number itself. What is the

G
number?
4. 40% of 70 = 4 × ? (a) 300 (b) 400
(a) 28 (b) 280
(c) 7 (d) 70
5. 25% of 25% is equal to U (c) 60 (d) Cannot be determined
15. If x% of x is 36, then x is
x2
O
(a) 6.25 (b) 0.625 (a) (b) 360x2
100
(c) 0.0625 (d) 0.00625
(c) 360 (d) 60
H
6. 30% of 80 = ? × 24
16. An ice-cream seller sells 60% of the ice-
3 cream cups he has and still has 300 ice-
_T

(a) (b) 1
10 cream cups left with him. The total number
3 of cups he initially had was
(c) (d) 2 (a) 750 (b) 600
17
C

(c) 540 (d) 450


7. 20% of 26 × 60 = 6% of 200 + x% of 1500.
The value of x is 17. If 150% of a certain number is 80, what
PS

(a) 30 (b) 20 will be 60% of that number?


(c) 60 (d) None of these (a) 80 (b) 66
(c) 49 (d) 32
8. If 42% of a given number is 12.6, then the
U

number is 18. Gayatri’s salary is increased from Rs 5000


(a) 30 (b) 40 to Rs 6300. Find the percentage of increase
(c) 35 (d) 45 in her salary.
@

(a) 11% (b) 20%


9. Two numbers are less than a third number (c) 26% (d) 30%
by 30% and 37% respectively. How much
per cent is the second number less than 19. A candidate who gets 30% of the marks
fails by 50 marks. Another candidate who
the first?
gets 320 marks fails by 30 marks. The
(a) 3% (b) 4%
maximum mark is
(c) 7% (d) 10%
(a) 1000 (b) 900
10. The number which exceeds 20% of it by (c) 800 (d) 700
38 is 20. Anand donated 5% of his income to a
(a) 47.5 (b) 46 charitable organisation and deposited 20%
(c) 7.6 (d) 5 of the remainder in a bank. Now, if he
11. 860% of 50 + 50% of 860 = ? has Rs 1919 left, his total income was
(a) 430 (b) 860 (a) Rs 2500 (b) Rs 2525
(c) 50 (d) 950 (c) Rs 2558.60 (d) Rs 2570
144
Unit Two : Practice Session ✫ 145

21. In an examination, 85% students passed 27. The population of a village is 7986. During
in Mathematics, 15 students failed in the the last three years, the population
paper and 15 students were given a increased at the rate of 10% per year. The
chance for re-test. Find the total number population before the three years was
of students. (a) 5000 (b) 5500
(a) 115 (b) 200 (c) 6000 (d) 6600
(c) 170 (d) 225
28. In a college election, a candidate secured
22. A owns a house worth Rs 10,000. He sells 62% of the votes and was elected by a
it to B at a profit of 10% based on the majority of 144 votes. The total number

TS
worth of the house. B sells the house back of votes polled was
to A at a loss of 10%. In this transaction, (a) 600 (b) 800
A gets (c) 925 (d) 1200
(a) a profit of Rs 2000

H
(b) a profit of Rs 1100 29. In a city, 40% of the people are illiterate
(c) a profit of Rs 1000 and 60% are poor. Among the rich 10%
are illiterate. What percentage of the poor

G
(d) no profit no loss
population is illiterate?
23. The price of an article is cut by 10%. To
(a) 36% (b) 40%
restore it to the former value, the new
price must be increased by
1 U (c) 60% (d) 75%
30. The population of a town increases 4%
O
(a) 10% (b) 9 %
11 annually but it also decreases annually
1 owing to emigration to the extent of 0.5%.
(c) 11 % (d) 11%
H
9 What will be percentage of increase in
24. The salary of a salesperson for two months population in three years?
is in the ratio 2 : 3. By what % was the
_T

(a) 9.8 (b) 10


salary (i) of the second month more than (c) 10.5 (d) 10.8
the first month? (ii) of the first month less
31. On decreasing the price of a car by 30%,
than the second month respectively?
its sale is increased by 20%. What is the
C

1
(a) 50% and 33 % effect on the revenue received by the
3
1 shopkeeper?
PS

(b) 33 % and 50% (a) 10% increase (b) 10% decrease


3
(c) 50% and 25% (c) 16% increase (d) 16% decrease
(d) None of these 32. Water tax is increased by 20% but its
U

25. Two positive numbers x and y are related consumption is decreased by 10%. Then
in such a manner that a% of x plus b% the increase or decrease in the expenditure
@

of y is equal to c% of x plus d% of y. If of the money is


b ≠ d, then the percentage of y to that of (a) 10% decrease (b) 10% increase
x will be (c) 8% decrease (d) 8% increase
100 ( a + c) 100 (c − a) 33. In an examination, 52% candidates failed
(a) (b) in English, 42% failed in Mathematics and
d−b d−b
17% failed in both. The number of students
100 ( a − c) 100 ( a + c)
(c) (d) who passed in both the subjects were
d−b b−d (a) 23% (b) 35%
26. Passing marks in an examination is 36%.
(c) 25% (d) 40%
If a student gets 72 marks and fails by
18 marks, then the maximum mark in the 34. In an examination there were 1500
examination would be candidates, out of which 920 were girls
(a) 100 (b) 200 and the rest were boys. If 50% of the girls
(c) 250 (d) 400 and 35% of the boys passed the exam,
146 ✫ Quantitative Aptitude

then the total percentage of candidates (a) 50% (b) 55%


who failed is 5 6
(a) 58.5% (b) 15% (c) 45 % (d) 54 %
11 11
(c) 10% (d) 55.8% 38. Of the 1000 inhabitants of a village, 60%
35. If the side of a square is increased by 25%, are males of whom 20% are literate. Of
then by how much per cent would its area all the inhabitants, 25% are literate. What
increase? per cent of the females in the village are
(a) 56.25 (b) 50 literate?
(c) 125 (d) 156.25 (a) 32.5% (b) 37%

TS
36. In an election between two candidates, (c) 22.5% (d) Cannot be determined.
10% of voters did not cast votes, and 10% 39. If the price of wheat falls by 16%, by what
of votes polled were invalid. The winning percentage can a person increase

H
candidate got 54% of the valid votes and consumption so that his overall budget
won by a majority of 1620 votes. How does not change?
many voters were enrolled on the voters’ (a) 16% (b) 18%

G
list? (c) 19% (d) 20%
(a) 2500 (b) 25000 40. Two workers A and B are paid a total of
(c) 35000 (d) 3500
37. A batsman scored 110 runs which included U Rs 550 by their employer. If A is paid 120
per cent of what is paid to B, how much
O
3 boundaries and 8 sixes. What per cent is B paid?
of his total score was made through runs (a) Rs 200 (b) Rs 250
H
between the wickets? (c) Rs 300 (d) Rs 350
Answers with Solutions
_T

1. Ans. (b) 5. Ans. (c)


Sol.Let x% of 49 = 84 25 25 625
x Sol. × =
100 100 10000
C

× 49 = 84
100 = 0.0625.
84 × 100
 x = 6. Ans. (b)
PS

49
= 171.43%. Sol. If x is the unknown quantity
80 × 30
2. Ans. (d) = x × 24
77 100
× 11100 = 77 × 111  x = 1.
U

Sol.
100 = 8547. 7. Ans. (b)
3. Ans. (b)
@

20 6 x
x Sol. × 26 × 60 = × 200 + × 1500
Sol. × 130 = 11.7 100 100 100
100 5.2 × 60 = 12 + 15x
11.7 × 100
x = 312 – 12 = 15x
130
300
117 × 100 x = = 20.
= 15
130 × 10 8. Ans. (a)
x = 9. Sol. 42% of a number = 12.6
4. Ans. (c) Let the number be x, then
40 42
Sol. × 70 = 4 × x × x = 12.6
100 100
40 × 70 700 100
x = =  x = 12.6 ×
4 × 100 100 42
 x = 7.  x = 30.
Unit Two : Practice Session ✫ 147

9. Ans. (d) = (0.45 × 750) – (0.25 × 480)


Sol. Let the third number be = x = 337.50 – 120 = 217.50.
Then, first number = 70% of x 13. Ans. (c)
70 x Sol. Let the number be x.
=
100 20
It is given that × x = 120
and second number = 63% of x 100
120 × 100
63x  x= = 600
= 20
100

TS
120
70 x 63x 7x ∴ 120% of x = × 600 = 720.
Difference = − = 100
100 100 100
14. Ans. (a)
∴ Required percentage Sol. Let the number be x. Then,

H
7 x 100 75
= × × 100 % x = 75
100 70 x 100

G
= 10%.  x – 0.75x = 75
Shortcut  0.25x = 75
Second number is
100 – y
100 – x
× 100 % of First
U  x =
75
0.25
= 300.
O
100 – 37
× 100 % 15. Ans. (d)
100 – 30 Sol. x% of x = 36
H
63 x
= × 100 % = 90%  × x = 36
70 100

_T

x2 = 3600
Second number is less than the first by
(100 – 90)% = 10%.  x = 3600 = 60.
16. Ans. (a)
10. Ans. (a)
Sol. Ice-cream seller has (100 – 60) = 40% of
C

Sol. Let the number be x. Then, ice-cream cups.


x – 20% of x = 38 40% of x = 300
PS

20 40
 x− x = 38
 x = 300
100 100
1 300 × 100
 x − x = 38 = 750 .
 x =
U

5 40
4 17. Ans. (d)
 x = 38
5
@

Sol. 150% of x = 80
5
 x = 38 × = 47.5.
4 150 160
11. Ans. (b) x = 80 x
Sol. The given expression 100 3
60 160
860 50 ∴ 60% of x = × = 32 .
= × 50 + × 860 100 3
100 100
18. Ans. (c)
= 430 + 430 = 860.
12. Ans. (d) Sol. Increase in salary = Rs (6300 – 5000)
Sol. The given expression = Rs 1300
1300
45
× 750 −
25
× 480 Percentage increase = × 100
= 5000
100 100
= 26%.
148 ✫ Quantitative Aptitude

19. Ans. (a) A’s profit in first transaction


Sol. Suppose x is the maximum mark = Rs 11000 – Rs 10000 = Rs 1000
In the second transaction,
Then, the pass marks for the first
A pays (Rs 10000 – Rs 9900 =) Rs 100
candidate =30% of x + 50 Less than the house’s worth.
and the pass marks for the second ∴ A’s profit in both the transactions
candidate =320 + 30 = 1000 + 100 = Rs 1100.
23. Ans. (c)
Therefore, 30% of x + 50 = 320 + 30 Sol. Let the original price be Rs 100
3x Decreased price = (100 – 10) = Rs 90

TS
or = 300
10 Now to raise it to former value of Rs 100
300 × 10 10 1
∴ x= = 1000 . × 100 % = 11 % .
3 Difference % =
90 9

H
20. Ans. (b) 24. Ans. (a)
Sol. Let Anand’s income be Rs x. Sol. Let the salaries be 2x and 3x

G
5 20 5x
x= x+ of x − + 1919 % excess of second month’s salary
100 100 100 (3x − 2x )
 x−
x 20 × 95 × x

20 100 × 100
= 1919
U =
2x
x
× 100 %

× 100 = 50%
O
x 19x =
x− − 2x
 = 1919
20 100 % by which first month’s salary was
H
 100x – 5x – 19x = 191900 lesser than the second month’s.
191900 (3x − 2x)
∴ x = × 100
_T

76 =
3x
= 2525 x 1
= × 100 = 33 % .
Thus, Anand’s total income was Rs 2525. 3x 3
25. Ans. (c)
C

Shortcut
Sol. According to the question
In this problem, the original amount
PS

ax by cx dy
Balance × 100 × 100 + = +
= 100 100 100 100
(100 − 5) (100 − 20) 100 ( a − c)
∴ Percentage of y to that of x = .
1919 × 100 × 100 d−b
U

= 26. Ans. (c)


95 × 80
Sol. Total marks needed to pass the exam
1919 × 25
@

= = Rs 2525. = 72 + 18 = 90
19 Passing marks in exam = 36%
21. Ans. (b) Maximum marks in the examination
Sol. Let x be the total number of students 100
= 90 × = 250.
15 36
× x = 30 27. Ans. (c)
100
30 × 100 Sol. Population three years ago
 x = = 200. P
15
22. Ans. (b) = 3
R
1+
110 100
Sol. Price paid by B = 10,000 × = Rs 11000
100 7986
90 = 3
Price paid by A = 11,000 × = Rs 9900 10
100 1+
100
Unit Two : Practice Session ✫ 149

7986 New sale value = Rs (70 × 120)


=
11 11 11
× × = Rs 8400
10 10 10
7986000 1600
= Decrease % = × 100 %
11 × 11 × 11 10000
= 6000. = 16%
28. Ans. (a) Shortcut
Sol. (62% of x – 38% of x) = 144 Increase % – Decrease %
 24% of x = 144 Increase % × Decrease %

TS
144 × 100 –
100
∴ x =
24 20 × 30
= 600.  20 – 30 –
100

H
29. Ans. (c) = –10 – 6
Sol. Let the total population be x. Then = –16%

G
60 3 Thus, there is a total loss of 16%.
Poor population = x = x
100 5 32. Ans. (d)
Illiterate population =
40
100
x = x
2
5 U Sol. Increase % – Decrease %


Increase % × Decrease %
O
Rich illiterates = 10% of (100 – 60)% 100
of x 20 × 10
= 20 – 10 –
H
10 40 x 100
= × ×x =
100 100 25 = 10 – 2
2 x 9x = 8%
_T

Poor illiterates = x− =
5 25 25 Thus, 8% increase.
∴ Required percentage
33. Ans. (a)
9x 5
C

= × × 100 % Sol. % of students who failed in both subjects


25 3x = 17%
PS

= 60%. % of students who failed only in English


30. Ans. (d) = 52 – 17 = 35%
Sol. Let original population = 100 % of students who failed only in Maths
Population after 3 years
U

= 42 – 17 = 25%
3
1 Total % of students who failed
3
= 17 + 35 + 25 = 77%
@

= 100 × 1 + 2
100 ∴ % of students who passed
= 100 – 77 = 23%.
207 207 207 Shortcut
= 100 × × ×
200 200 200 100 – (52 + 42 – 17)
= 110.87 = 23.
∴ Increase = [110.87 – 100]% 34. Ans. (d)
= 10.87%
Sol. No. of boys = 1500 – 920 = 580
= 10.8%.
Total students who passed
31. Ans. (d)
Sol. Let price = Rs 100, and sale = 100 50 35
= × 920 + × 580
Then, sale value = Rs (100 × 100) 100 100
= Rs 10000 = 460 + 203 = 663
150 ✫ Quantitative Aptitude

∴ % of candidates who failed  8% of [90% of (90% of x)] = 1620


(1500 − 663) 8 90 90
= × 100  × × × x = 1620
1500 100 100 100
837 1620 × 100 × 100 × 100
= × 100  x = = 25000.
1500 8 × 90 × 90
= 55.8%.
35. Ans. (a) 37. Ans. (c)
Sol. The batsman’s number of runs made
Sol. If Side of the square = a, Area = a2

TS
2
by running between the wickets
25a = 110 – (3 × 4 + 8 × 6) = 110 – 60 = 50
New Area = a +
100 50 5
2 2 The percentage = × 100 = 45 %.
a 5a 110 11

H
= a+ = 38. Ans. (a)
4 4
Sol. Males number = 60% of 1000 = 600
25 2

G
= a Females number = 1000 – 600 = 400
16 Total literates = 25% of 1000 = 250
25 a 2 Literate males = 20% of 600 = 120
Increase in area =
16
− a2
U Literate females = 250 – 120 = 130
Percentage of literate females
O
9a2 130
= = × 100 = 32.5%.
16 400
H
39. Ans. (c)
9a2 1
Increase % = × × 100 % Sol. Increase in consumption is given by
16 a2
_T

R
225 × 100 %
= % = 56.25% . (100 + R
Shortcut
4 (R being decrease %)
C

If x is the increase in the side of a square, 16


We have (100 − 16 × 100 %
x2
increase in area = 2x +
PS

100 16
= × 100 %
84
(25)2
 2 × 25 + 400
100
U

= % = 19%.
21
625 40. Ans. (b)
= 50 +
@

100 Sol. Let B be paid Rs x.


Then x + 120% x = 550
5625
= = 56.25 . 120
100
 x + x = 550
36. Ans. (b) 100
Sol. Let the total number of voters be x. 100 x + 120 x
Valid votes = 90% of (90% of x); as  = 550
10% of voters did not cast votes, 100
only 90% of the votes would be 220 x
considered for the non-valid vote  = 550
100
base.
From the given conditions, 100
 x = 550 ×
54% of [90% of (90% of x)] 220
– 46% of [90% of (90% of x)] = 1620 x = 5 × 50 = Rs 250.
Unit Two : Practice Session ✫ 151

Practice Exercise 9
1. After deducting a commission of 5%, a TV 10. The price of apples fell by 10%, enabling
set costs Rs 9595. Its marked price is a man to buy 50 more apples for Rs 225.
(a) Rs 10,500 (b) Rs 10,100 The reduced price of 100 apples is
(c) Rs 10,075 (d) Rs 10,000 (a) 25 (b) 35
2. If 20% of 60 is greater than 30% of a (c) 30 (d) 45
number by 2, then the number is 11. A salesman’s terms were changed from a
(a) 40.5 (b) 47

TS
flat commission of 5% on all his sales to
(c) 38 (d) 33.3
a fixed salary of Rs 1000 plus 2.5%
3. In a solution, 10 out of 350 parts is water. commission on all sales exceeding Rs 4000.
What is the percentage of water in the If his remuneration as per the new

H
solution? scheme was Rs 600 more than the first
(a) 35% (b) 30% scheme, what were his sales worth?
20 15

G
(a) Rs 13,500 (b) Rs 13,900
(c) % (d) %
7 2 (c) Rs 12,000 (d) Rs 11,900
4. The annual income of a man is Rs 5000.
If he spends 8% of it every month, then
his yearly saving is
U 12. Rajeev started a garment factory by
investing 65% in machinery, 20% in raw
materials, and 12% in land. He has Rs 7500
O
(a) Rs 200 (b) Rs 4600
(c) Rs 600 (d) Rs 400 left with him to set up the office. Calculate
his total investment.
H
5. An inspector rejects 0.08% of the meters (a) Rs 2,50,000 (b) Rs 2,40,000
as defective. How many would have been (c) Rs 23,000 (d) Rs 22,000
examined if he rejects 2?
_T

(a) 200 (b) 250 13. A solution of salt and water contains 15%
(c) 2500 (d) 3000 salt by weight. Of it, 30 kg water
evaporates and the solution now contains
6. The price of petrol went up by 25%. In
C

20% of salt. The original quantity of


order that expenses on petrol should not
increase, one must reduce travel by solution is
PS

(a) 25% (b) 20% (a) 35 kg (b) 73 kg


(c) 18% (d) 15% (c) 90 kg (d) 120 kg
7. In a Maths examination, the average for 14. In measuring the sides of a rectangle, one
side is taken 5% in excess and the other
U

the entire class was 80 marks. If 10% of


the students scored 95 marks and 20% 4% in deficit. Find the error percentage
scored 90 marks, what was the average in area calculated from the measurement?
@

marks of the remaining students? 5 5


(a) % less (b) % more
(a) 65.5 (b) 72.5 7 7
(c) 75 (d) 85 4 4
(c) % less (d) % more
8. A number is reduced to its 80 per cent 5 5
when 35 is subtracted from it. What is 15. Namrata went to the stationers and bought
four-fifth of that number? things worth Rs 25, out of which 30 paise
(a) 70 (b) 90 went on sales tax on taxable purchases.
(c) 120 (d) 140 If the tax rate was 6%, then what was the
9. 300 litres of oil was poured into a tank cost of the tax-free items?
and it was still 40% empty. Find the (a) Rs 15 (b) Rs 15.70
capacity of the tank. (c) Rs 19.70 (d) Rs 20
(a) 300 litres (b) 500 litres 16. The tax on a commodity is diminished by
(c) 200 litres (d) None of these 15% and its consumption is increased by

151
152 ✫ Quantitative Aptitude

15%. The percentage reduction in the (a) 12,000 (b) 10,000


revenue is (c) 9,950 (d) 600
(a) 2.25% (b) 1.5% 24. A woman had a certain sum of money. If
(c) 0% (d) 3% she gives 10% to her husband, 20% of
17. A mixture of 50 litres of milk and water the rest to her daughter and 20% of the
contains 8% water. How much water rest to her son, she has Rs 1,32,480 left
should be added to the mixture so that with her. Find the total amount of money
water may be 20% in the new mixture? she had.
(a) 7 litres (b) 5 litres (a) Rs 2,20,000 (b) Rs 2,30,000

TS
(c) 4 litres (d) 7.5 litres (c) Rs 2,40,000 (d) Rs 2,50,000
18. If the price of a commodity is increased 25. A man used to save 25% of his earnings.
by 9%, then by how much should a family Then his income was increased by 20%

H
reduce its consumption to have no extra and he increased his expenditure by 10%.
expenditure? Find the % hike in his savings.
(a) 8.26% (b) 8.59% (a) 45% (b) 49%

G
(c) 8.73% (d) 8.88% (c) 50% (d) 55%
19. A candidate scoring 25% marks in an 26. A trader dealing in pressure cookers
examination fails by 30 marks while another
candidate who scores 50% marks gets 20 U reduced the price by 20% as a result of
which his sale went up by 80%. What was
O
marks more than the minimum marks the net effect on his sales income?
required for passing. Find the minimum (a) 33% increase (b) 33% decrease
H
pass percentage. (c) 44% increase (d) 44% decrease
(a) 40% (b) 38% 27. In a certain examination, 40% of the
(c) 36% (d) 35%
_T

students failed in Mathematics, 30% failed


20. Mr X’s salary is increased by 20%. On the in English and 10% failed in both. The
increase the tax rate is 10% higher. Find percentage of students who passed in
the percentage of increase in the tax both the subjects is
C

liability. (a) 20 (b) 40


(a) 14% (b) 22% (c) 30 (d) 10
PS

(c) 28% (d) 34% 28. Sohan’s wages were decreased by 40%.
21. If the diameter of a sphere is doubled, Again the reduced wages were increased
then the surface area is increased by by 40%. He has a
U

(a) 100% (b) 200% (a) loss of 16% (b) gain of 16%
(c) 300% (d) 400% (c) loss of 10% (d) gain of 10%
@

22. In an examination, 70% candidates passed 29. The price of cooking oil has increased by
in English and 65% in Mathematics. If 27% 25%. The percentage of reduction that a
of the candidates failed in both the family should effect in the use of cooking
subjects, and 248 passed the examination, oil so as not to increase the expenditure
the total number of candidates who on this account is
appeared in the examination were (a) 15% (b) 20%
(a) 400 (b) 348 (c) 25% (d) 30%
(c) 420 (d) 484 30. Two candidates contested an election. One
23. In an election, a candidate who got 40% secured 63% of the votes and won the
of total votes polled was defeated by the election by a margin of 32500 votes. Find
only other candidate by 200 votes. How the number of total votes polled.
many votes were polled by the successful (a) 12750 (b) 125000
candidate? (c) 128200 (d) 127950
Unit Two : Practice Session ✫ 153

31. Of those eligible to vote, 8% are between October it further reduced by 10%. What
18 and 21 years of age. In an election, 85% was the number of pickpockets caught in
of those eligible to vote, who were October?
between 18 and 21 years of age, actually (a) 3591 (b) 3571
voted. What percentage of those eligible (c) 3121 (d) 3255
to vote was formed by the number of 34. From a container of pure milk, 20% is
people between 18 and 21 years of age replaced by water and the process
who actually voted in the election? repeated. At the end of the third such
(a) 6.5% (b) 6.8% operation, how pure is the milk?

TS
(c) 7 % (d) 8 % (a) 50% (b) 40%
32. Depreciation applicable to some equipment (c) 50.8% (d) 51.2%
is 20%. The value of the equipment 3 years 35. At a party, 60% of guests had the vegetarian
from now will be less by

H
food while 30% had the non-vegetarian
(a) 51.20% (b) 50% food and 15% had both types of food. If
(c) 48.8% (d) 45.8% 96 people were present, how many did

G
33. In July the police caught 4000 pickpockets. not eat either kind of food?
In August that number rose by 5%. Next (a) 24 (b) 20
month the number reduced by 5% and in
U (c) 28 (d) 30
O
Answers with Solutions
1. Ans. (b) ∴ Yearly saving = 4% of 5000
H
Sol. Let the marked price be x 4
= × 5000
Then, x – 5% of x= 9595 100
_T

 95% of x = 9595 = 200.


9595 × 100 5. Ans. (c)
 x = Sol. Let the number of meters examined be x.
C

95
= 10100. 0.08% of x = 2
0.08
PS

2. Ans. (d) ⇒ x =2
100
Sol. Let the number be x
200
Then, 20% of 60 – 30% of x = 2 ⇒ x = = 2500.
0.08
U

20 30
× 60 − x=2 6. Ans. (b)
100 100
Sol. Here R = 25
@

3 R
12 − x = 2 Using formula × 100 %
10 R + 100
3
− x = − 10 25
10 ⇒ × 100 % = 20%.
100 25 + 100
 x= = 33.3.
3 7. Ans. (c)
3. Ans. (c) 10
× 100 % Sol. Let the number of students in the class
Sol. Required percentage =
350 be 100 and let the required average be x
20 Then, (10 × 95) + (20 × 90) + (70 × x)
= %
7 = 100 × 80
4. Ans. (a) 70x = 8000 – (950 + 1800)
Yearly expenses = 12 × 8% 5250
x = = 75.
= 96% 70
154 ✫ Quantitative Aptitude

8. Ans. (d) 3% of x = 7500


Sol. Let the number be x. 3
× x = 7500
80 100
Then, x – 35 = x
100 7500 × 100
80 x =
x = 35 3
x– = 250000
100
35 × 100 Hence, total investment is Rs 2,50,000.
 x= = 175
20 13. Ans. (d)

TS
4 4
 x = × 175 = 140 . Sol. Original quantity of solution
5 5 = Quantity of water evaporated
9. Ans. (b)
Final % of salt

H
Sol. Let x be the total capacity of the tank. ×
40 Difference % of salt
300 litres + × x = x
100

G
2 20
300 = x − x = 30
5 20 − 15

300 =
3x
5
5 × 300 U 14. Ans. (d)
= 120 kg.

Sol. Here x = 5 and y = –4


O
x = = 500 litres.
3 xy
10. Ans. (d) Error (%) = x + y +
H
10 100
Sol. Reduction = 225 × = 22.5
100 5×4
Error (%) = 5 – 4 –
 Cost of 50 apples = 22.5
_T

100
22.5
∴ Cost of 100 apples = × 100 20
50 = 1−
= Rs 45. 100
4
C

= % more because the sign


11. Ans. (c) 5
is +ve.
Sol. Let the total sales be for Rs x.
PS

15. Ans. (c)


5 Sol. Let the amount of taxable purchases be
⇒ [1000 + 2.5% (x – 4000)] – × x = 600 Rs x.
100
30
U

2.5 2.5 5 Then, 6% of x =


1000 + ×x− × 4000 − × x = 600 100
100 100 100
30 100
@

5x 2.5x x= × =5
1000 – 100 – 600 = − 100 6
100 100
2.5x ∴ Cost of tax free items
300 = = Rs [25 – (5 + 0.30)]
100
300 × 100 × 10 = Rs 19.70.
⇒ x =
25 16. Ans. (a)
x = 12000 Sol. Increase % – Decrease %
Hence, total sales is Rs 12,000.
Increase % value × Decrease % value
12. Ans. (a) –
100
Sol. Let x be the total investment.
Total percentage spent on preliminary 15 × 15
= 15 − 15 −
needs of the factory 100
= (65 + 20 + 12)% = 97%  –2.25% = 2.25% reduction.
Unit Two : Practice Session ✫ 155

17. Ans. (d) 10


Sol. Let x litres be the water added to it y+ y
100 11y
4+x (Rs 20) will be × 20 =
∴ % water = × 100 = 20 100 50
50 + x 11y
(4 + x) × 100 = 20 (50 + x) ∴ Increase in tax liability = 50 y × 100
400 + 100x= 1000 + 20x = 22%.
100x – 20x = 1000 – 400
21. Ans. (c)
600
x = Sol. Let diameter be 2r (radius = r)

TS
80
Surface area of a sphere = 4πr2
= 7.5 litres.
If the diameter doubles,
18. Ans. (a)
then d = 4r
Sol. % change in quantity of consumption

H
 New radius = 2r
% change in rate New surface area= 4π × (2r)2
= × 100
(100 + % change in rate)

G
= 4π × 4r2
9 900 = 16πr2
= ×100 = %
100 + 9 109
= 8.26%.
U Increase % in surface area
(16π r 2 − 4π r 2 )
× 100 %
O
19. Ans. (a) = 4π r 2
Sol. Let the maximum marks be x. 12π r 2
H
25x 50 x = 4π r 2 × 100 %
+ 30 = − 20
100 100
_T

= 300%.
50 x 25x
30 + 20 = − 22. Ans. (a)
100 100
Sol. Let x be the total number of candidates
x x
= − % of candidates who failed in both
C

2 4 subjects = 27
x % who failed only in English
PS

50 =
4 = 30 – 27 = 3
% who failed only in Maths
∴ x = 50 × 4 = 200 marks
= 35 – 27 = 8
25 ∴ % of those who failed in all
U

∴ Pass marks = × 200 + 30


100 = 27 + 3 + 8
= 50 + 30 = 38%
@

= 80 62
 % who passed = × x = 248
Minimum % of pass marks 100
248 × 100
80
× 100 %  x = = 400.
= 62
200
Shortcut
= 40%.
% of those who failed
20. Ans. (b) = 100 – (30 + 35 – 27)
Sol. Let Mr X’s salary be Rs 100. = 62
∴ Salary after increase = Rs 120, i.e., 62 × x
Rs 20 is the increase on Rs 100 % who passed  = 248
100
Let the tax on original salary be y% 24800
x = = 400.
Hence the tax on the increased salary 62
156 ✫ Quantitative Aptitude

23. Ans. (d) ( −20) × (80)


⇒ −20 + 80 +
Sol. Let the total votes be x. 100
% of votes for the defeated candidate = 60 – 16 = 44% increase.
= 40%
∴ % of votes for the winning candidate 27. Ans. (b)
= 60% Sol. Here, x = 40, y = 30 and z = –10
Difference % = (60 – 40)% = 20% ∴ Percentage of students passing both the
20 subjects = 100 – (x + y – z)%
× x = 200
100 =100 – (40 + 30 – 10)%

TS
200
x = × 100 = 1000 = 40%.
20
28. Ans. (a)
Successful candidate’s votes Sol. Let Sohan’s wage be Rs 100

H
60 Reduced wage = Rs 40
= 1000 × = 600.
100 Increased wage = 140% of Rs 40

G
24. Ans. (b) 140
= × 40 = Rs 56
Sol. Let x be the total sum 100

Then 1−
10
1−
20
1−
20
U ∴ Change in wage = Rs 40 – Rs 56
= –16%
O
100 100 100
= Loss of 16%.
× total sum = 132,480 Shortcut
H
90 80 80 Increase % – Decrease %
× × × x = 132480
100 100 100 Increase % value × Decrease % value
_T


9 4 4 100
× × × x = 132480
10 5 5 40 × 40
= 40 − 40 − = –16%
132480 × 250 100
C

x =
144 = Loss of 16%.
Total sum = Rs 2,30,000. 29. Ans. (b)
PS

25. Ans. (c) Sol. Here x = 25


Sol. Let the man’s earnings be Rs 100 % change in consumption
Savings = Rs 25 x
U

⇒ Expenditure = Rs 75 = × 100 %
100 + x
After the increase, his income = Rs 120
@

Expenditure after the increase 25


⇒ × 100 %
110 100 + 25
= 75 ×
100 25
= 82.5 = × 100 %
125
∴ Present saving = 120 – 82.5 = 37.5
12.5 = 20%.
∴ % hike in savings = × 100 % 30. Ans. (b)
25
Sol. % of votes secured by the winning
= 50%.
26. Ans. (c) candidate = 63%
Sol. Here, x = –20 and y = 80 ∴ % of the votes secured by the defeated
The net % effect in sales income candidate = 100 – 63 = 37%
xy
= x+ y+ Difference in percentage
100
= (63–37)% = 26%
Unit Two : Practice Session ✫ 157

Let x be the number of votes polled Reduction in value after 3 years


26 = (100 – 51.20) = Rs 48.80
∴ × x = 32500 or 48.8%
100
32500 × 100 (as the base is 100).
∴ x =
26 33. Ans. (a)
i.e., total votes polled = 1,25,000. Sol. Number of pickpockets caught in
October
31. Ans. (b) 5 5 10
Sol. If the number of people eligible to = 4000 × 1 + 1− 1−
100 100 100

TS
vote is x, those eligible to vote between
18 and 21 years of age = 8% of x. 21 19 9
= 4000 × × × = 3591.
Number of persons between 18 and 21 20 20 10
years who voted 34. Ans. (d)

H
= 85% of (8% of x) Sol. Let the quantity of milk in the first
instance be 1 l or 1000 ml.
85 8
×

G
= x After the first operation, milk
100 100 = 80% of 1000 = 800 ml.
17 After the second operation, milk
=
250
Required percentage
x

U = 80% of 800 = 640 ml


After the third operation, milk
O
17 = 80% of 640 ml = 512 ml
= x x × 100 As a percentage the purity of milk
250
H
512
17 1 = × 100 = 51.2%
x × × 100 1000
=
_T

250 x
35. Ans. (a)
34 Sol. Those who had vegetarian food
= = 6.8%
2 60 288
= × 96 =
C

32. Ans. (c) 100 5


Those who had non-vegetarian food
Sol. Let the present value of the equipment
PS

30 144
be Rs 100. = × 96 =
100 5
Value after 3 years Those who had both types of food
U

20
3 15 72
100 × 1 − = × 96 =
= Rs 100 100 5
Those who had food of either type
@

4 4 4 288 144 72 360


100 × × × = + − = = 72
= Rs 5 5 5 5
5 5 5
∴ Persons who had neither type of food
= Rs 51.20 = 96 – 72 = 24.
158 ✫ Quantitative Aptitude

Practice Exercise 10
1. Three numbers are in the ratio 1 : 2 : 3, in the school, the number of boys in the
so that the sum of their squares is equal school is
to 504. The three numbers are (a) 500 (b) 480
(a) 3, 6 and 9 (b) 4, 8 and 12 (c) 370 (d) 350
(c) 6, 12 and 18 (d) 8, 16 and 24 10. What must be added to each term of the
2. If 12 men can reap 40 hectares, then how ratio 6 : 11 so that it may become equal
many hectares can 36 men reap? to 2 : 3?

TS
(a) 120 hectares (b) 60 hectares (a) 1 (b) 2
(c) 13.34 hectares (d) 10.8 hectares (c) 3 (d) 4

3. The proportion of iron and tin in a steel 11. Two numbers are in the ratio 2 : 3 and

H
the sum of the numbers is 120. The
piece is 13 : 7. How much iron will be
numbers are
there in 100 kg of such a piece?
(a) 110 and 10 (b) 96 and 24

G
(a) 140 kg (b) 110 kg
(c) 48 and 72 (d) 100 and 20
(c) 65 kg (d) 20 kg
12. In what ratio should Kavita and Kumud
4. If A =
1
4
(a) 1 : 4 : 8
1
B and B = C , then A : B : C is
2
(b) 1 : 8 : 4 U divide a profit of Rs 312 if their investments
are Rs 500 and Rs 300 respectively?
O
(c) 8 : 4 : 1 (d) 8 : 1 : 4 (a) 3 : 5 (b) 5 : 3
(c) 4 : 4 (d) 1 : 3
5. In what ratio should Hema and Kavita
H
divide a profit of Rs 6000 if their 13. A, B and C enter into a partnership. A
1
investments are Rs 15000 and Rs 10000 contributes of the capital while B
_T

3
respectively? contributes as much as A and C together
(a) 3 : 2 (b) 2 : 3 contribute. If the profit at the end of the year
(c) 1 : 3 (d) None of these amounts to Rs 840, what would A receive?
C

6. Niju, Satish and Sunil started a business. (a) 280 (b) 420
Niju invested Rs 5000 for 2 years, Satish (c) 120 (d) 600
PS

invested Rs 4000 for 15 months and Sunil 14. The ratio between the ages of Gayatri and
invested Rs 3000 for 10 months. If there Savitri is 6 : 5, and the sum of their ages
is a profit of Rs 1925, find out the share is 44 years. The ratio of their ages after
of Niju. 8 years will be
U

(a) Rs 550 (b) Rs 1100 (a) 5 : 6 (b) 7 : 8


(c) Rs 275 (d) Nothing (c) 8 : 7 (d) 14 : 13
@

7. The price of a scooter and a television set 15. The ratio between the speed of walking
are in the ratio 3 : 2. If the scooter costs of A and B is 3 : 2. If the time taken by
B to cover a certain distance is 24 minutes,
Rs 6000 more than the television set, the
how much time will be taken by A to
price of the television set is
cover the same distance?
(a) Rs 18,000 (b) Rs 12,000
(a) 12 minutes (b) 16 minutes
(c) Rs 10,000 (d) Rs 6,000
(c) 15 minutes (d) 18 minutes
8. The last three terms of a proportion are 16. Arshad and Prahlad start a business with
10, 27 and 30. What is the first term? Rs 1500 and Rs 1200 respectively. Prahlad
(a) 2 (b) 4 1
(c) 6 (d) 9 gets 12 % of the profit for managing the
2
9. The ratio of the number of boys and girls business. If the total profit gained in a
in a school is 4 : 3. If there be 360 girls year is Rs 2160, find the share of Prahlad.

158
Unit Two : Practice Session ✫ 159

(a) Rs 1050 (b) Rs 1080 24. 50 coins of Re 1 and 50 paise. are in the
(c) Rs 1110 (d) Rs 1200 ratio 2 : 3. The value of the coins is
(a) Rs 20 (b) Rs 25
17. Rs 5625 is to be divided among A, B and
1 (c) Rs 30 (d) Rs 35
C so that A may receive much as B and
2 1 25. H.C.F. of two numbers is 43. If their ratio
C together receive, and B receives of is 5 : 2 then the numbers are
4
what A and C together receive. The share (a) 215 and 86 (b) 200 and 72
of A is more than that of B by (c) 149 and 63 (d) 143 and 43
(a) Rs 750 (b) Rs 775

TS
26. Two numbers are in the ratio 5 : 9. With
(c) Rs 1500 (d) Rs 1600
the addition of nine, their ratio becomes
18. A man divides his property so that his 16 : 27. Then, one of the two numbers
wife’s share to his daughter and his would be

H
daughter’s share to his son are both in (a) 66 (b) 77
the ratio 3 : 2. If the son gets Rs 8000 (c) 88 (d) 99
less than the daughter, find the worth of

G
the total property? 27. In a mixture of 60 litres, the ratio of milk
(a) Rs 36000 (b) Rs 24000 and water is 2 : 1. If the ratio of milk
(c) Rs 76000 (d) Rs 16000
19. A bag contains fifty paise, twenty five U and water is to be 1 : 2, then the amount
of water further added would be
O
paise, ten paise and five paise coins in the (a) 20 litres (b) 30 litres
ratio 2 : 3 : 5 : 7. If the total amount is (c) 40 litres (d) 60 litres
H
Rs 52, the number of 50 paise coins is 28. Ratio of Vijay’s age to Vivek’s age is equal
(a) 50 (b) 40 to 10 : 9. If Vijay will be 26 years after
_T

(c) 8 0 (d) 2 5 6 years, what is the present age of Vivek?


20. A man has a few hens and cows. If the (a) 18 years (b) 20 years
number of heads equals 48 and the number 1
of feet equals 140, the number of hens will (c) 27 years (d) 22 years
C

2
be 29. In a bag, there are coins of 25 paise, 10
(a) 26 (b) 30
PS

paise and 5 paise in the ratio 1 : 2 : 3.


(c) 3 3 (d) 4 0 If there is Rs 30 in the bag, how many
21. 11 chairs cost as much as 3 tables. If a coins of 5 paise are there?
chair and a table cost Rs 140, how much (a) 50 (b) 100
U

does a chair cost alone? (c) 125 (d) 150


(a) Rs 110 (b) Rs 30
30. The ratio of two numbers is 7 : 8 and their
@

(c) Rs 70 (d) Nothing


L.C.M. is 280. The numbers are
22. Ratio between annual incomes of Mahavir (a) 10 and 28 (b) 35 and 40
and Sarosh is 5 : 4 and between their (c) 20 and 14 (d) 35 and 8
expenditure is 3 : 2. If at the end of the
year, both save Rs 800 each, find the 31. In a 500-metre race, Q starts 45 metres
income of Mahavir. ahead of P. But P wins the race while Q
(a) Rs 1600 (b) Rs 1400 is still 36 metres behind. The ratio of their
(c) Rs 2000 (d) Rs 2400 speeds, assuming that both start at the
same time, is
23. Three persons are walking from a place (a) 5 : 7 (b) 5 : 3
P to another place Q. If their speed is in
(c) 5 : 6 (d) 25 : 21
the ratio 5 : 7 : 6, then the ratio of their
time taken to reach Q would be 32. Radhey, Manohar and Nandlal rent a
(a) 42 : 30 : 35 (b) 42 : 36 : 35 pasture for one year for Rs 799. Radhey
(c) 36 : 30 : 35 (d) 30 : 24 : 20 puts 60 cows for 10 months, Manohar puts
160 ✫ Quantitative Aptitude

40 cows and 40 sheep for one year and (a) 5 : 9 (b) 5 : 7


Nandlal puts in 80 cows and 20 sheep for (c) 7 : 5 (d) 9 : 5
eight months. If one cow eats as much as a b c
2 sheep, what should Radhey pay as his 34. If = = and a + b + c ≠ 0 then
b+c c+a a+b
share of rent?
(a) a + b = c (b) a + c = b
(a) Rs 260 (b) Rs 282
(c) b + c = a (d) a = b = c
(c) Rs 267 (d) Rs 235
33. A and B are two alloys of gold and copper a2 b2 c2 1 1
35. If = = = 1 , then + +
prepared by mixing metals in proportions b+c c+a a+b a+1 b+1

TS
7 : 2 and 7 : 11 respectively. If equal 1
is equal to
quantities of alloys are melted to form a c+1
third alloy C, the proportion of gold and (a) 1 (b) a2 + b2 + c2
copper in C will be (c) abc (d) abc (a2 + b2 + c2)

H
Answers with Solutions

G
1. Ans. (c) 6. Ans. (b)
Sol. Let the numbers be
Then, x2 + (2x)2 + (3x)2
x,
=
2x and 3x
504
U Sol. Ratio of investment
= 5000 : 4000 : 3000
O
⇒ x2 + 4x2 + 9x 2 = 504 =5 : 4 : 3
Ratio of period of investment
⇒ 14x2 = 504 = 24 months : 15 months : 10 months
H
504 = 24 : 15 : 10
x2 = = 36
14 ∴ The share of profit according to the time
_T

⇒ x =6 = 5 × 24 : 4 × 15 : 3 × 10
The numbers are 6, 12 and 18. = 120 : 60 : 30
2. Ans. (a) =4:2:1
4
C

Sol. 12 men : 36 men :: 40 hectares : Required ∴ Niju’s share = × 1925 = Rs 1100.


7
number of hectares
∴ Required number of hectares 7. Ans. (b)
PS

36 × 40 Sol. Let x be the cost of a TV


= Scooter price : TV price = 3 : 2
12
x + 6000 3
= 120 hectares. =
U

3. Ans. (c) x 2
Sol. 20 kg of steel contains = 13 kg iron 2(x + 6000) = 3x
@

∴ 100 kg of steel will contain 12000 = x.


13 8. Ans. (d)
= ×100
20 Sol. Let x be the first term of the proportion
= 65 kg iron. Then, x : 10 = 27 : 30
4. Ans. (a)
Sol. Let A = x x 27
∴ =
Then, B = 4x, C = 2B = 8x 10 30
∴ A : B : C= x : 4x : 8x ∴ 30x = 27×10
= 1 : 4 : 8. 27 ×10
5. Ans. (a) ∴ x =
Sol. Ratio of share in profits = Ratio of 30
Investments = 9.
Ratio of investments = 15000 : 10000 9. Ans. (b)
= 3 : 2 Sol. Let x be the number of boys in the
∴ The required ratio for profit = 3 : 2. school
Unit Two : Practice Session ✫ 161

x 4 44
Then, = x = = 4
360 3 11
∴ 3 x = 360×4 ∴ Gayatri’s age = 24
360 × 4 Savitri’s age = 20
∴ x = = 480.
3 After 8 years, Gayatri : Savitri
10. Ans. (d) = 32 : 28 = 8 : 7.
Sol. Let the required number to be added to 15. Ans. (b)
each term of the given ratio be x. Sol. Ratio of the speeds of A and B = 3 : 2

TS
6+x 2 Ratio of the time taken by A and B
Then =
11+ x 3
1 1
18 + 3x = 22 + 2x = : = 2 : 3
3 2

H
∴ x = 4.
∴ 2 : 3 = x : 24
11. Ans. (c)
2 x

G
Sol. Let the numbers be 2x and 3x. =
3 24
2x + 3x = 120
24 × 2
x =
120
5
= 24
∴ The numbers are 48 and 72. U ∴

16. Ans. (c)


x =
3
= 16 minutes.
O
12. Ans. (b) Sol. Ratio of investment = 1500 : 1200
= 30 : 24
H
Sol. Ratio of investment = 500 : 300
= 5 : 3 Prahlad’s share for managing the
(  Ratio of share in profit = Ratio of business 25
_T

= × 2160
investment). 2 × 100
= Rs 270
13. Ans. (a)
Remaining profit = 2160 – 270 = 1890
C

Sol. Let B’s contribution 24


Prahlad’s share of profit = × 1890
= (A + C)’s contribution 54
PS

1
= of the investment = Rs 840
2 ∴ Total share of Prahlad = 840+270
1 1 1 1
A : B : C = : : 1– + = Rs 1110.
U

3 2 2 3
1 1 5 17. Ans. (a)
= : : 1– Sol. A + B + C’s share = Rs 5625
@

3 2 6
1 1 1 1
= : : Given A = (B + C)
3 2 6 2
1
1 1 1 i.e., (B + C) + (B + C) = 5625
= 6× : 6× : 6× 2
3 2 6
3
= 2 : 3 : 1 (B + C) = 5625
2 2
A’s share of profit = × 840 2
6 B + C = 5625 × = 3750 ...(i)
= 280. 3
1
14. Ans. (c) ∴ A’s share = (B + C) = 1875
2
1
Sol. Let Gayatri’s and Savitri’s age be 6x Also given B = (A + C) ...(ii)
and 5x. 4
4B = (A + C)
6x + 5x = 44
162 ✫ Quantitative Aptitude

∴ A + B + C = 5625 ...(iii) 20. Ans. (a)


Putting (ii) in (iii) Sol. Let the number of hens be = x
⇒ B + 4B = 5625 and number of cows be = y
Then x + y = 48 . . . (i)
5625
B = = 1125 and 2x + 4y = 140 . . . (ii)
5
By solving these equations, we get
∴ C = 3750 – 1125 2y = 44 or y = 22
= 2625 from (i) so, x = (48 – 22) = 26
A’s share = Rs 1875,

TS
∴ Number of hens = 26.
B’s share = Rs 1125
and C’s share = Rs 2625 21. Ans. (b)
A – B = 1875 – 1125 = Rs 750. Sol. Cost of 11 chairs (C) = Cost of 3 tables (T)
11 C = 3 T

H
Shortcut
A’s share : B and C’s share = 1 : 2 11
i.e., T = C
Sum of ratio = 1 + 2 = 3 3

G
11C
5625 × 1 + C = 140
A’s share = = Rs 1875 3
3
B’s share : A and C’s share = 1 : 4
Sum of ratio = 1 + 4 = 5 U 11C+3C = 140×3
14C = 140×3
O
140 × 3
C = = Rs 30
5625 × 1 14
B’s share = = Rs 1125 Hence, cost of one chair is Rs 30.
H
5
A – B = 1875 – 1125 = Rs 750. 22. Ans. (c)
18. Ans. (c) Sol. If 5x and 4x are their annual incomes,
_T

Sol. Let Wife : Daughter = 3x : 2x 5 x – 800 3


and Daughter : Son = 3x : 2x then =
4 x – 800 2
Wife : Daughter : Son = 9x : 6x : 4x 10x – 1600 = 12x – 2400
If the son gets Rs 8000 less than the
C

2400 – 1600 = 12x – 10x


daughter, then
2x = 800
6x – 4x = 8000
PS

x = 400
2x = 8000
x = 4000 ∴ Mahavir’s income = 5 × 400 = Rs 2000.
∴ Total property = (9 + 6 + 4 ) × x Shortcut Ra ( d – c)
= 19 × 4000 Mahavir’s income =
U

ad – bc
= Rs 76000.
[where R = savings of each; a, b, c and
19. Ans. (b) d are the ratios]
@

Sol. Let the no. of coins be 2x, 3x, 5x and 800 × 5 (2 – 3)


7x respectively. =
5×2 – 4× 3
Total value of the coins = Rs 2000.
= No. of coins × Value of Coins 23. Ans. (a)
2x × 50 + 3x × 25 + 5x × 10 + 7x × 5 Sol. Let PQ = x unit
= 5200 PQ PQ PQ
∴ : :
100x + 75x + 50x + 35x = 5200 t1 t2 t3 = 5 : 7 : 6
260x = 5200 (t1, t2 and t3 being the time taken by the
5200 persons.)
x = = 20 ⇒ 5t1 = 7t2 = 6t3 = k (say)
260
∴ The numbers of 50 paise coins = 2x 1 1 1
∴ t1 : t2 : t3 = : :
= 2×20 5 7 6
= 40. = 42 : 30 : 35.
Unit Two : Practice Session ✫ 163

24. Ans. (d) The ratio of their value


2 = 25 × 1 : 10 × 2 : 5 × 3
Sol. 1 Re coins = × 50 = 20 coins = Rs 20 = 25 : 20 : 15
5
3 = 5:4:3
50 paise coins = × 50 = 30 coins = Rs 15 ∴ The share of 5 p coins in Rs 30
5
Total value = Rs 20 + Rs 15 = Rs 35. 3
= × 30 = Rs 7.50
12
25. Ans. (a) 750
Sol. The numbers are 5 × 43 and 2 × 43 ∴ Number of coins = = 150.
5
= 215 and 86.

TS
30. Ans. (b)
26. Ans. (d) Sol. Let the numbers be 7x and 8x
Sol. Suppose x : y :: 5 : 9 respectively.
x + 9 : y + 9 :: 16 : 27 L.C.M. of the numbers = 7 × 8 × x

H
x x+9 = 56x
5 16
i.e., = and Given that 56x = 280
y 9 y + 9 = 27

G
x = 5
i.e., x = 55
Hence, the numbers are
and y = 99.
7 × 5 = 35 and 8 × 5 = 40.
Shortcut
We have a = 5, b = 9, c = 16, d = 27, x = 9 U 31. Ans. (d)
Sol. Suppose speeds of P and Q are
O
ax (c – d) 5 × 9 (16 – 27) respectively x m/min and y m/min
Ist number = = = 55 Distance covered by P = 500 m
ad – bc 5 × 27 – 9 ×16
H
Distance covered by Q in same time
bx (c – d) 9 × 9 (16 – 27)
IInd number = = = 99. = 500 – (45 + 36)
ad – bc 5× 27 – 9 ×16
= 500 – 81 = 419 m
_T

27. Ans. (d)


500 419
Sol. Ratio of milk and water in the mixture Thus, =
x y
of 60 litres = 2 : 1.
or x : y = 500 : 419
C

∴ Quantity of milk = 40 litres


and quantity of water = 20 litres = 25 : 20.95
= 25 : 21 (approximate)
PS

If the ratio of milk and water is to be 1 :


2, then in 40 litres of milk, water should 32. Ans. (d)
be 80 litres. Sol. Given one cow eats as much as 2 sheep.
∴ Amount of water to be added further All the cows are converted in terms of
U

= 60 litres. sheep.
28. Ans. (a) ⇒ Radhey’s ⇒ 60 × 2
@

Sol. Vijay’s present age = 20 i.e., 120 sheep for 10 months.


Ratio of Vijay’s age and Vivek’s age ⇒ Manohar ’s ⇒ 40 × 2 + 40
= 10 : 9 120 sheep for 12 months.
∴ Vivek’s age x is given as ⇒ Nandlal’s ⇒ 80 × 2 + 20
26 − 6 10 180 sheep for 8 months.
=
x 9 i.e., Share of grass eaten by Radhey’s :
20 10 Manohar’s : Nandlal’s sheep
⇒ =
x 9 = 120 × 10 : 120 × 12 : 180 × 8
20 × 9
x= = 18. = 1200 : 1440 : 1440
10 = 100 : 120 : 120
29. Ans. (d)
Sol. The ratio of 25 p coins : 10 p coins : 5 p = 5:6:6
5
coins ∴ Required answer = × 799 = Rs 235.
= 1:2:3 17
164 ✫ Quantitative Aptitude

33. Ans. (c) 1 1 1


∴ = =
Sol. For the preparation of alloy C, a b c
if one unit of A is mixed with 1 unit of B, (dividing each part by
a + b + c as a + b + c ≠ 0)
7 7 14 + 7 21 ∴ a = b = c (by invertendo)
then gold content = + = =
9 18 18 18
2 11 4 + 11 15 35. Ans. (a)
and copper content = + = =
9 18 18 18 a2 b2 c2
21 15 Sol. = = = 1
Gold : Copper = : = 21 : 15 = 7 : 5. b+c c+a a+b

TS
18 18
∴ a2 = b + c, b2 = c + a and c2 = a + b
34. Ans. (d)
1 1 1
Sol.
a
=
b
=
c ∴ + +
a +1 b +1 c +1
b+c c+a a+b

H
b+c c+a a+b a b c
∴ = = (by invertendo) = 2
+ 2 + 2
a b c a +a b +b c +c

G
b+c c+a a+b a b c
∴ +1 = +1 = +1 = + +
a b c b+c+a c+ a+b a+b+c
∴ b+c+a
a
=
c+ a+b
b
=
a+b+c
c
U =
a+b+c
a+b+c
= 1
O
H
C _T
PS
U
@
Unit Two : Practice Session ✫ 165

Practice Exercise 11
1. The average of 13 results is 40, that of (a) 3 (b) 7
the first 6 is 35 and that of the last 6 is (c) 17 (d) 31
38. The value of the 7th number is 9. A cricketer scored 180 runs in the first test
(a) 82 (b) 56.5 and 258 runs in the second test. How
(c) 4 1 (d) 36.5 many runs should he score in the third
2. The average of ten results is 50 and that test so that his average score in the three
of first nine is 45. The tenth result is tests would be 230 runs?

TS
(a) 95 (b) 73.54 (a) 213 (b) 220
(c) 56.9 (d) 47.5 (c) 235 (d) 252
3. The average of 20 numbers is 8. If two 10. The average age of A, B, C and D five

H
numbers 12 and 10 are discarded, the years ago was 45 years. By including X,
average of the remaining set of numbers the present average age of all the five

G
will be becomes 49 years. The present age of X
1 2 is
(a) 9 (b) 7 (a) 45 years (b) 40 years
(c) 7
3
1
3
(d) 9
3
2
3 U (c) 35 years (d) 30 years
11. The average age of 30 students in a class
O
4. The average weight of 33 students of a is 15 years. If 6 students of this class have
class is 52 kg. The average weight of 18 the average age of 16 years, then the
H
of them is 48 kg. The average weight of average age of the remaining 24 students
the remaining students is would be
_T

(a) 50.8 kg (b) 56.8 kg (a) 14 years


(c) 48.8 kg (d) 40.8 kg (b) 14 years and 6 months
5. Class X has 50 students and the average (c) 14 years and 9 months
marks obtained by this class is 50. Class (d) 15 years and 3 months
C

XI has 50 students and the average marks 12. The average weight of a group of 150
obtained by this class is 40. The average students in a class is 60 kg. If the mean
PS

marks of both the classes as a whole is of the weights of all the 50 boys in the
(a) 40 (b) 45 class is 70 kg, then the average weight
(c) 50 (d) None of these of the 100 girls in the class is
U

6. The average score of a cricketer in 2 (a) 65 kg (b) 50 kg


matches is 27 and that in 3 others is 32. (c) 60 kg (d) 55 kg
@

His average score in 5 matches is 13. The average of first 35 numbers is


(a) 11.8 (b) 25 (a) 24.2 (b) 21
(c) 29.5 (d) 30 (c) 18 (d) 17.5
7. The average temperature from Monday to
14. The average of 9 results is 40. If the
Thursday is 30° and from Tuesday to
average of first five results is 39 and that
Friday is 48°. If the temperature on
of last five results is 42, then the 5th result
Monday is 24°, what was it on Friday?
is
(a) 96° (b) 83°
(c) 51° (d) 29° (a) 32 (b) 38
(c) 40 (d) 45
8. A student was asked to find the average
of numbers 3, 11, 7, 9, 15, 13, 8, 19, 17, 15. A student bought 10 pens of Rs 15 each,
21, 14 and x. He found the average to be 12 notebooks of Rs 22 each and 4 books
12. What should be the number in place of Rs 250 each. The average value of the
of x? stationary is

165
166 ✫ Quantitative Aptitude

(a) Rs 50 (b) Rs 54.38 average age of the surviving children four


(c) Rs 57.46 (d) Rs 59.53 years after the death of the child?
16. Seven men agree with the eighth man to 1 3
(a) 15 years (b) 14 years
subscribe a sum of money for a chit fund. 4 4
(c) 15 years (d) 14 years
The first seven are to subscribe Rs 50 each
and the eighth man Rs 70 more than the 23. What is the average of n, n + 1, n + 2,
average of all the eight. Find how much n + 3, n + 4 and n + 5?
did the eighth man subscribe? (a) 6n + 15 (b) n + 2
(a) 130 (b) 122 1 1

TS
(c) n + 3 (d) n + 2
(c) 119 (d) 114 2 2
17. The average temperature of first three 24. Three years ago, the average age of A and
days is 27° and of the next three days is B was 18 years. With C joining them, the

H
29°. If the average of the whole week is average age becomes 22 years. How old
28.5 °C, the temperature on the last day is C now?
(a) 20 years (b) 24 years

G
was
(a) 31.5° (b) 10.5° (c) 10 years (d) 15 years
(c) 21° (d) 42°
18. In a primary school, there are 60 boys of
age 12 each, 40 of age 13 each, 50 of age U 25. The average height of 40 students is
163 cm. On a particular day, three students
A, B and C were absent and the average
O
14 each and another 50 of age 15 each. of the remaining 37 students was found
The average age of the boys of the school to be 162 cm. If A and B have equal height
H
is and the height of C is 2 cm less than
(a) 13.5 (b) 13 that of A, then find the height of A, B
_T

(c) 13.45 (d) 14 and C.


19. The average age of a class was 16 years. (a) 142 cm, 142 cm, 140 cm
But 4 boys whose average age was 13 (b) 176 cm, 176 cm, 174 cm
(c) 157 cm, 157 cm, 155 cm
C

years 6 months were admitted. The class


average became 15 years 7 months. How (d) 180 cm, 180 cm, 178 cm
PS

many boys were there after the new 26. A motorist completes the journey between
admission? A and B at a constant speed of 20 kmph
(a) 24 (b) 16 and covers the return journey from B to
(c) 21 (d) 29 A at a constant speed of 30 kmph. What
U

20. A batsman has a certain average runs for was his average speed?
10 innings. In the 11th innings, he scored (a) 24 kmph (b) 21 kmph
@

50 runs thereby increasing his average by (c) 25 kmph (d) 20 kmph


4 runs. What was his average after the 27. The average weight of A and B is 42 kg
11th innings? while the average weight of A, B and C
(a) 11 (b) 8 is 44 kg. If the average weight of B and
(c) 10 (d) 9 C is 43 kg, find the weight of B.
21. If a car travels from A to B at 60 km/hr (a) 36 kg (b) 38 kg
and returns to A from B at a speed of (c) 39 kg (d) 42 kg
40 km/hr, the average speed of the car is 28. The average monthly expenditure of
(a) 45 km/hr (b) 50 km/hr Vijayan is Rs 2300 for the first three
(c) 48 km/hr (d) 55 km/hr months, Rs 2100 for the next 5 months and
22. A man had 5 children. When their average Rs 2000 for the last 4 months in a year.
age was 10 years, the child who was 5 If he managed to save Rs 9400 during the
years of age died. What will be the year what is his average monthly income?
Unit Two : Practice Session ✫ 167

(a) Rs 2900 (b) Rs 2800 (a) 30° (b) 35°


(c) Rs 2700 (d) Rs 2600 (c) 40° (d) 45°
29. Out of three numbers, the first is twice 36. The average of runs made by Shyam, Hari
the second and is half of the third. If the and Raju is 7 less than that made by
average of the three numbers is 56, then Shyam, Hari and Kishore. If Kishore made
the three numbers in order are 35 runs, how many runs did Raju score?
(a) 48, 96, 24 (b) 48, 24, 96 (a) 7 (b) 14
(c) 96, 24, 48 (d) 96, 48, 24 (c) 21 (d) 35
30. The average age of three boys is 15 years. 37. Average age of students in a class is 16

TS
If their ages are in ratio 3 : 5 : 7, the age years. If the class teacher aged 40 years
of the youngest boy is old is also included, the average age rises
(a) 21 years (b) 18 years to 17 years. Then the number of students
in the class are

H
(c) 15 years (d) 9 years
(a) 23 (b) 33
31. For ten hours, a train travels at a constant (c) 44 (d) 16

G
speed of 20 miles per hour and during the
next 15 hours, it travels 240 miles. What 38. Average of 100 values is 25. If one value
is added to this data set, the average
is the average speed of the train for the
whole journey?
(a) 8.4 miles (b) 11.12 miles U remains the same. The value is:
(a) 0
(c) 50
(b) 25
(d) 100
O
(c) 14.8 miles (d) 17.6 miles
39. A bus travels from Amritsar to Chandigarh
32. What is the average speed of a train
at the speed of 80 kmph and from
H
running at the rate of 30 kmph during the Chandigarh to Amritsar at the speed of
first 100 km; at 40 kmph during the 90 kmph. The average speed of the whole
_T

second 100 km; and at 50 kmph during journey was


the third 100 km? (a) 85 kmph (b) 84.70 kmph
(a) 40 kmph (b) 42.5 kmph (c) 84 kmph (d) 83.80 kmph
(c) 38.3 kmph (d) 36.8 kmph
C

40. The average marks obtained by 100


33. The average age of husband and wife was candidates in an examination is 50. If the
23 years when they were married 5 years
PS

average marks of candidates who passed


ago. The average age of the husband, the is 65 and that of the candidates who failed
wife and a child, who was born during is 15, then the number of candidates who
the interval, is 20 years now. The present passed the examination is
U

age of the child is (a) 70 (b) 64


(a) 1 year (b) 2 years (c) 55 (d) 40
(c) 3 years (d) 4 years
@

41. In a class, there are 20 boys whose


34. The average age of 10 men is increased average age is decreased by 2 months,
by 2 years when two of them who are when one boy aged 18 years is replaced
20 years and 22 years of age are replaced by a new boy. The age of the new boy
by two new men. The average age of the is
two new men is (a) 14 years
(a) 31 years (b) 33 years (b) 14 years 2 months
(c) 35 years (d) 40 years (c) 14 years 6 months
(d) 14 years 8 months
35. The average temperature of Monday to
Wednesday was 37 °C and that of Tuesday 42. In an examination, the average score of
to Thursday was 34 °C. If the temperature 10 students is 60. If the score of one
5 student is omitted and the average score
on Monday was th of that on Thursday
4 becomes 55, then the score of that student
find the temperature of Monday. must have been
168 ✫ Quantitative Aptitude

(a) 55 (b) 115 (a) 1 (b) 3


(c) 105 (d) 60 (c) 5 (d) 10
43. Average monthly income of a family of 50. The captain of a cricket team of 11 players
four earning members was Rs 735. One is 25 years old and the wicketkeeper of
of the earning members died and therefore the same team is 3 years older than the
the average income came down to Rs 650. captain. If the age of these two players
The income of the deceased was are replaced by that of another two
(a) Rs 1500 (b) Rs 1000 players, the average age of the cricket
(c) Rs 990 (d) Rs 820 team drops by 2 years. The average age

TS
44. In the first 10 overs of a cricket match, of these two players is
the run rate was 4.2. What should be the (a) 17.3 years (b) 17 years
run rate in the remaining 40 overs to reach (c) 15.5 years (d) 13.2 years
the target of 346 runs?

H
51. The average daily wages of A, B and C
(a) 4.3 (b) 5.1 is Rs 100. If B earns Rs 20 more than C
(c) 6.4 (d) 7.6 per day and A earns double of what C

G
45. A library has an average of 600 visitors earns per day, the wage of A per day
on Sundays and 250 on other days. The is
average number of visitors per day in a
month of 30 days beginning with a Sunday
U (a) Rs 120
(c) Rs 140
(b) Rs 130
(d) Rs 150
O
is
52. Three years ago, the average age of a
(a) 340 (b) 308
family of 5 members was 17 years. A baby
(c) 290 (d) 275
H
having been born, the average age of the
46. The average of six numbers is 3.95. The family is the same today. The present age
average of two of them is 3.4, while the of the baby is
_T

average of the other two is 3.85. What is 1


(a) 1 year (b) 1 years
the average of the remaining two numbers? 2
(a) 4.5 (b) 4.6 (c) 2 years (d) 3 years
(c) 4.7 (d) 4.8
C

53. The average of the scores of a group of


47. The average monthly income of P and Q students in a test was 52. The brightest
PS

is Rs 5050. The average monthly income 20% of them secure a mean score of 80
of Q and R is Rs 6250 and the average and the dullest 25% a mean score of 31.
monthly income of P and R is Rs 5200. The mean score of the remaining 55% is
The monthly income of P is (a) 54.6 (approximate)
U

(a) Rs 1500 (b) Rs 2000 (b) 51.4 (approximate)


(c) Rs 2500 (d) Rs 4000 (c) 50
@

48. The average expenditure of a man for the (d) 45


first five months is Rs 120 and for the next 54. If a, b, c, d and e are five consecutive odd
seven months is Rs 130. Find his monthly numbers, their average would be
average income if he saves Rs 290 in that (a) 5 (a + 4)
year. abcde
(a) Rs 100 (b) Rs 150 (b)
5
(c) Rs 180 (d) Rs 195 (c) 5 (a + b + c + d + e )
49. A man was engaged for work for 40 days (d) None of these
on the condition that he would be paid 55. If the average of a, b and c is d, then the
Rs 20 for each day he works and Rs 10 average of a, b, c and d is
be taken from him for each absence. At d
the end of 40 days, the man was paid Rs (a) (b) d
2
500. Find out how many days was the
(c) 2d (d) None of these
man absent?
Unit Two : Practice Session ✫ 169

Answers with Solutions


1. Ans. (a) 7. Ans. (a)
Sol. Seventh number Sol. Temperature on Monday + Tuesday +
= Total of 13 results – Wednesday + Thursday = 4 × 30° = 120°
(Total of first 6 + Total of Temperature on Monday = 24°
last 6 results) ∴ Tuesday + Wednesday + Thursday
= 13 × 40 – (6 × 35 + 6 × 38) = 120°–24° =96°
Temperature on Tuesday + Wednesday

TS
= 520 – (210 + 228)
+ Thursday + Friday = 4 × 48° = 192°
= 520 – 438 ∴ Temperature on Friday = 192° – 96°
= 82. = 96°.

H
2. Ans. (a) 8. Ans. (b) 3 + 11 + 7 + 9 + 15 + 13 + 8
Sol. Tenth result = (10 × 50 – 9 × 45)
+ 19 + 17 + 21 + 14 + x
= 500 – 405 Sol. Average =

G
12
= 95. 137 + x
12 =
3. Ans. (b)
Sol. Average of the remaining set of numbers
U ⇒

x
x
12
= 144 – 137
= 7.
O
20 × 8 – (12 + 10) 160 – 22
= = 9. Ans. (d)
20 – 2 18
H
138 2 Sol. 180 + 258 + x = 3 × 230
= = 7 . 438 + x = 690
18 3
⇒ x = 252.
_T

4. Ans. (b)
10. Ans. (a)
Sol. Average weight of remaining students
Sol. Total age of A, B, C, D
33 × 52 – 48 ×18 = 4 × 45 = 180 years
C

=
33 – 18 Present age of A, B, C, D
1716 – 864 = 180 + 4 × 5 = 200 years
PS

=
15 Total age of A, B, C, D and X
852 = 5 × 49 = 245 years
= = 56.8 kg.
15 ∴ X’s age = 245 – 200 = 45 years.
U

5. Ans. (b)
11. Ans. (c)
50 × 50 + 50 × 40
Sol. Total average = Sol. According to condition (i)
@

50 + 50
2500 + 2000 Total of data
= = 15
100 30
4500 ∴ Total of data = 15 × 30 = 450 ...(i)
= = 45.
100 According to condition (ii)
6. Ans. (d) Total of data of 6 students
2 × 27 + 3 × 32 = 16
6
Sol. Average =
5 ∴ Total of data of 6 students = 96 ...(ii)
54 + 96 Thus, using equation (i) and (ii), we get
=
5 Average of remaining 24 students
150 450 – 96
= = 30. =
5 24
170 ✫ Quantitative Aptitude

354 The total of first 5 results


= = 14.75 = 5 × 39 = 195
24
The total of last 5 results
= 14 years and 9 months. = 5 × 42 = 210
Shortcut The fifth result is common to both.
30 students × average = 6 students × ∴ Fifth result = 195+210–360
average + 24 students × average = 45.
15. Ans. (b)
30 × 15 = 6 × 16 + 24 × x
10 ×15 + 12 × 22 + 4 × 250
Sol. Average =

TS
30 ×15 – 6 ×16 354 59 10 + 12 + 4
x = = =
24 24 4 150 + 264 + 1000
=
= 14 years and 9 months. 26

H
12. Ans. (d) 1414
=
Sol. According to condition (i) 26

G
Total weight = Rs 54.38.
= 60
150 16. Ans. (a)
∴ Total weight = 150 × 60
= 9000 kg ...(i)
U Sol. Let the average subscription of all 8
men be x.
350 + x + 70
O
According to condition (ii) Average subscription is =x
8
Total weight of 50 male students 420 + x =8x
= 70
H
50 ⇒ x = 60
∴ Total weight of 50 boys
∴ Eighth man’s subscription
= 70 × 50 = 3500 kg ...(ii)
_T

= 60 + 70 = 130.
Thus using equations (i) and (ii), we get
average of remaining 100 girls 17. Ans. (a)
9000 – 3500 Sol. Sum of the temperature for the full week
C

= = 7 × 28.5
100
5500 = 199.5°
= = 55 kg. ∴ The temperature on the last day
PS

100
= 199.5 – [3 × 27 + 3 × 29]
Shortcut = 199.5 – [81 + 87]
150 students × average = 50 male × = 31.5°.
U

average + 100 females × average 18. Ans. (c)


150 × 60 = 50 × 70 + 100 × x Sol. Required answer
@

150 × 60 – 50 × 70 60 ×12 + 40 ×13 + 50 × 14 + 50 ×15


x = =
100 60 + 40 + 50 + 50
= 55 kg.
13. Ans. (c) 720 + 520 + 700 + 750
= = 13.45.
n+1 200
Sol. Average of first n numbers =
2 19. Ans. (a)
35 + 1
Average of first 35 numbers = Sol. Let the number of students before the
2 new admissions be x.
36 ∴ Total age of the class = 16x
= = 18.
2 Total age of the four new boys
14. Ans. (d) = 4 × 13.5 = 54 years
Sol. The total of 9 results 16 x + 54 7
= 40 × 9 = 360 ⇒ = 15 .
x+4 12
Unit Two : Practice Session ✫ 171

16 x + 54 187 25. Ans. (b)


⇒ =
x+4 12 Sol. Let the height of A, B and C be x,
12 (16x + 54) = 187 (x + 4) x and x – 2 respectively.
⇒ 192x – 187x = 748 – 648 ∴ The total heights of A, B and C are
5x = 100 given by x + x + x – 2
x = 20 = 40 × 163 – 37 × 162
∴ x + 4 = 20 + 4 = 24. 3x – 2 = 6520 – 5994
20. Ans. (c) 3x – 2 = 526
Sol. Let the average runs for 10 innings be x 3x = 528

TS
⇒ Total runs for 10 innings = 10x 528
Average for the 11th innings is given by x =
3
10 x + 50 = 176
=x+4
∴ A’s height = 176 cm

H
11
⇒ 10x + 50 = 11x + 44 B’s height = 176 cm
50 – 44 = 11x – 10x

G
C’s height = 176 – 2 = 174 cm.
x =6
∴ Average after 11 innings 26. Ans. (a) 2xy

21. Ans. (c)


= 6 + 4 = 10.

Sol. Average Speed U Sol. Average speed = x + y

=
2 × 20 × 30
O
20 + 30
2xy
= x + y km/hr 1200
=
H
2 × 60 × 40 50
= = 24 kmph.
60 + 40
27. Ans. (b)
_T

2 × 60 × 40 A+B
= = 48 km/hr. Sol. Average weight of A and B = = 42
100 2
∴ A + B = 2 × 42 = 84 kg ...(i)
22. Ans. (a)
Similarly, A + B + C= 3 × 44
Sol. Total age of the surviving children
C

= 132 kg ...(ii)
= 5 × 10 – 5 = 45 B + C = 2 × 43 = 86 kg ...(iii)
After 4 years, average age of those
PS

Subtracting (ii) from (iii),


children we get A = 46 kg
45 Now putting this value of A in (i) we get
= +4
4 B = 84 – 46 = 38 kg.
U

1 1 28. Ans. (a)


= 11 + 4 = 15 years. Sol. Total yearly income
4 4
23. Ans. (d) = Total expenditure + Saving
@

Sol. Average =3 × 2300 + 5 × 2100 + 4 × 2000 + 9400


= 6900 + 10500 + 8000 + 9400
= n + n +1+ n + 2 + n + 3 + n + 4 + n + 5 = 34800
6 ∴ Average monthly income
6 n + 15 3(2n + 5)
= = 34800
6 6 = = Rs 2900.
(2 n + 5) 5 1 12
= = n+ = n+2 . 29. Ans. (b)
2 2 2
Sol. Let the numbers be 2x, x and 4x
24. Ans. (b)
2x + x + 4x = 56×3
Sol. (A – 3) + (B – 3) = 36 (given average is 18)
7x = 168
A + B = 42
168
Also, A + B + C = 66 x =
42 + C = 66 7
= 24
⇒ C = 24 years.
∴ The numbers are 48, 24, 96.
172 ✫ Quantitative Aptitude

30. Ans. (d) 1


∴ of temperature on Thursday = 9°
Sol. 3x + 5x + 7x = 15 × 3 4
15x = 45 ∴ Temperature on Thursday = 36°
x= 3 5
∴ Temperature on Monday = × 36°
∴ The age of the boys = 9, 15, 21 4
∴ Required answer = 9 years. = 45°.
31. Ans. (d)
36. Ans. (b)
Sol. Average speed Sol. Let the average runs made by Shyam,
Hari and Kishore be x.

TS
20 ×10 + 240 200 + 240
= = ∴ The average runs made by Shyam, Hari
25 25 and Raju= x – 7
440 88 ⇒ Total runs made by Shyam, Hari and
= = 17.6 miles.
25 5 Kishore = 3x

H
32. Ans. (c)
Total runs made by Shyam, Hari and
300 Raju = 3x – 21
Sol. Average speed =
∴ Difference of runs made by Kishore

G
100 100 100
+ +
30 40 50 and Raju = 3x – (3x – 21) = 21
300 ∴ Runs made by Raju = 35 – 21 = 14.
=
10 5 2
+ +
3 2 1 U 37. Ans. (a)
Sol. Let the number of students be x, then
O
300 × 6 according to condition (i)
= = 38.3 kmph.
47 Total of data
H
= 16
33. Ans. (d) x
Sol. Age of the child ∴ Total of data = 16x ...(i)
_T

= [(20 × 3) – (23 × 2 + 5 × 2)] According to condition (ii)


= [60 – (46 + 10)] Total of data + 40
= [60 – 56] = 4 years. = 17
x+1
34. Ans. (a) ∴ Total of data = 17(x + 1 ) – 40 ...(ii)
C

Sol. Total age increased = 10 × 2 By equalising equation (i) and (ii), we


= 20 years get
PS

Sum of the age of two new men 16x = 17(x + 1)–40


= (20 + 22 + 20) = 62 years 16x = 17x + 17 – 40
∴ Average age of the two new men ∴ x = 23
62
U

= = 31 years. Thus, the number of students in the


2 class is 23.
35. Ans. (d)
@

Sol. Temperature on Monday + Tuesday Shortcut


Using the formula:
+ Wednesday = 3×37° = 111°
Teacher’s age = n (y – x) + y
Temperature on Tuesday + Wednesday
where y is the new average age, x is the
+ Thurday = 3 × 34 = 102° ...(i)
original average age and n is number of
Temperature on Monday
students
5
= × Thursday Here, Teacher’s age
4 = 40 years, y = 17, x = 16, n = ?
5
∴ × Thursday + Tuesday + Wednesday ∴ 40 = n (17–16) + 17
4 n = 23.
= 111 ...(ii)
38. Ans. (b)
Subtracting (i) from (ii), we get
Sol. Let the new value be x, then
5
Thursday – Thursday 25 ×100 + x
4 = 25
= 111 – 102 = 9° 101
Unit Two : Practice Session ✫ 173

2500 + x = 2525 ∴ Total of 10 students = 600 ...(i)


x = 25 According to condition (ii)
Thus, the value is 25. Total of 10 students – One student score
= 55
Alternative Method 9
Value = n (y – x) + y 600 – One student score
∴ = 55
= 100 (25 – 25) + 25 9
= 25. Let the score omitted be x then,
39. Ans. (b) 600 – x
= 55
2 × 80 × 90 9

TS
Sol. Average speed = ∴ x = 105.
80 + 90
14400 Shortcut
= Let the score omitted be x
170

H
= 84.70 kmph. 10 students × 60 = 9 students × 55
40. Ans. (a) + 1 student × x

G
Sol. Let the number of candidates who x = 10 × 60 – 9 × 55
passed be x = 105.
Then total marks


100 × 50 = 65x + (100 – x)15
5000 = 65x + 1500 – 15x U 43. Ans. (c)
Sol. Income of the deceased
O
⇒ 5000 – 1500 = 65x – 15x = Rs (735 × 4 – 650 × 3)
⇒ 3500 = 50x = Rs (2940 – 1950)
H
∴ x = 70. = Rs 990.
Shortcut 44. Ans. (d) 346 – (4.2 ×10)
Number of candidates who passed
_T

Sol. Required run rate =


= Total candidates 40
304
(Total average – Failed average) = = 7.6.
× 40
(Passed average – Failed average) 45. Ans. (b)
C

100(50 – 15)
= Sol. Since the month begins with a Sunday,
65 – 15
PS

there will be five Sundays in the month.


3500
= = 70. 600 × 5 + 250 × 25
50 ∴ Required average =
41. Ans. (d) 30
U

Sol. Age of new boy = Age of the boy 3000 + 6250


removed – No. of persons × Decrease =
30
@

in average age
9250
2 = = 308.3
= 18 – 20 × 30
12
10 = 308 (approximate)
= 18 –
3
44 2 46. Ans. (b)
= = 14 years
3 3 Sol. Sum of the remaining two numbers
2 =(3.95 × 6) – [(3.4 × 2)+(3.85 × 2)]
= 14 years and × 12 months
3
=23.70 – (6.8+7.7)
= 14 years and 8 months.
=23.70 – 14.5
42. Ans. (c)
=9.20
Sol. According to condition (i)
9.2
Total of 10 students ∴ Required average = = 4.6.
= 60 2
10
174 ✫ Quantitative Aptitude

47. Ans. (d) 51. Ans. (c)


P+Q
Sol. Average of P and Q = = 5050 Sol. Let the daily wage of C be x, then the
2
P + Q = 2 × 5050 = 10100 ...(i) wage of A becomes 2x and B becomes
x + 20
Similarly Q + R = 2 × 6250 = 12500 ...(ii)
Hence, the average wage
P + R= 2 × 5200 = 10400 ...(iii) x + 2 x + x + 20
By adding (i), (ii) and (iii), we get =
3
2 (P + Q + R) = 33000 4 x + 20
or P + Q + R = 16500 ...(iv) =
3

TS
Subtracting (ii) from (iv), we get 4 x + 20
∴ = 100 ⇒ 4x + 20 = 300
P = 4000 3
∴ P’s monthly income = Rs 4000. ⇒ 4x = 280 ⇒ x = 70
∴ Wage of A per day = 2 × 70 = Rs 140.

H
48. Ans. (b)
Sol. Average monthly income 52. Ans. (c)
Sol. Present age of five members

G
5 × 120 + 7 ×130 + 290
= = 5 × 17 + 3 × 5
12
= 100 years
=
600 + 910 + 290

= Rs 150.
12
U Present age of five members and a
baby = 17 × 6 = 102 years
O
∴ Age of the baby = 102 – 100
49. Ans. (d) = 2 years.
53. Ans. (b)
H
Sol. Let x be the number of days the man
was absent. Sol. Let the average score be x
Then, 20 × 80 +25 × 31 + 55 × x
_T

(40 – x) × 20 + x × (–10) = 500


800 – 20x – 10x = 500 = 52 × 100
800 – 500 = 30x ⇒ 1600 + 775 + 55x = 5200
300 ⇒ 55x = 2825
C

x = = 10.
30 565
50. Ans. (c) ⇒ x =
11
PS

Sol. Let x be the average age of the cricket


= 51.4 (approximate)
team and y be the sum of age of the two
new players. 54. Ans. (d)
∴ Sum of the ages of the nine players Sol. Being consecutive odd numbers,
U

(who are not affected by the change) their average


a+ a+2+a+ 4+ a+6+ a+8
@

=
= 11x – (25 + 28) 5
5 a + 20
=
= 11x – 53 5
11x – 53 + y = a + 4.
∴ = x–2
11 55. Ans. (b)
11x – 53 + y = 11x – 22
a+b+c
⇒ y = 31 Sol. = d
3
31
∴ Average age of two players = ⇒ a + b + c = 3d
2
a+b +c+ d 3d + d 4d
= 15.5 years. ∴ = = = d.
4 4 4
Unit Two : Practice Session ✫ 175

Practice Exercise 12
1. Three partners A, B and C invest Rs 5,000, months he adds another Rs 10,000 to the
Rs 7,000 and Rs 10,000 respectively in a capital. B withdraws Rs 4,000 at the end
business. If at the end of the year they of 4 months. At the end of the year, A
earned Rs 17,600 as profit, the share of and B’s ratio of profit is equal. Find B’s
partner C in this profit amounts to capital in the beginning.
(a) Rs 5,000 (b) Rs 6,000 (a) Rs 26,000 (b) Rs 30,000
(c) Rs 7,000 (d) Rs 8,000 (c) Rs 35,000 (d) Rs 40,000

TS
2. A starts a business with Rs 5,000. B joins 8. Three partners A, B and C invested Rs
him after 3 months with Rs 8,000. C puts 8,000, Rs 5,000 and Rs 7,000 respectively.
a sum of Rs 12,000 in the business for 2 They decide to distribute 60% of the
profit equally and the remaining 40% as

H
months only. At the end of the year, the
business earns a profit of Rs 9,100. The the interest on their respective capitals. If
share of C in the profit is A received Rs 840 less than the sum of

G
(a) Rs 1,400 (b) Rs 2,000 the other two, find the total profit.
(c) Rs 2,200 (d) Rs 2,500 (a) Rs 2,500 (b) Rs 3,000
3. A and B enter into a partnership with
their capitals in the ratio 7 : 9. At the endU (c) Rs 4,800 (d) Rs 9,000
9. A sum of money is to be divided among
O
of 8 months, A withdraws his capital. A, B and C in the ratio 2 : 3 : 7. If the
If they receive the profits in the ratio total share of A and B together is Rs 1,500
less than C, what is A’s share in it?
H
8 : 9, find for how long B’s capital was
used. (a) Rs 1,500 (b) Rs 1,700
(a) 4 months (b) 7 months (c) Rs 2,000 (d) Rs 2,200
_T

(c) 9 months (d) 1 year 10. A, B and C enter into a partnership and
4. A and B entered into a partnership. A 1 1 1
their shares are in the ratio : : .
invested Rs 8000. If the shares of profits 2 3 4
C

of A and B be Rs 360 and Rs 450, the After 2 months, A withdraws half of his
investment of B is capital and after 10 months, a profit of
PS

(a) Rs 4,000 (b) Rs 5,000 Rs 378 is divided among them. What is


(c) Rs 8,000 (d) Rs 10,000 B’s share?
5. A started a business with Rs 76,000. After (a) Rs 129 (b) Rs 144
U

few months, B joined him with Rs 57,000. (c) Rs 156 (d) Rs 168
At the end of the year, the total profit 11. A, B and C start a business. A invests 3
@

was divided between them in the ratio times as much as B who in turn invests
2 : 1. After how many months did B join? two-third of what C invests. Then, the
(a) 4 months (b) 6 months ratio of capitals of A, B and C is
(c) 8 months (d) 11 months (a) 2 : 3 : 6 (b) 2 : 6 : 3
6. Rs 56,250 is to be divided among A, B (c) 6 : 3 : 2 (d) 6 : 2 : 3
and C so that A may receive half as much 12. A, B and C enter into a partnership. A
as B and C together and B receives one- invests some money at the beginning; B
fourth of what A and C together receive. invests double the amount after 6 months
The share of A is more than that of B by and C invests thrice the amount after 8
(a) Rs 7,500 (b) Rs 7,750 months. If the annual profit is Rs 18,000,
(c) Rs 15,000 (d) Rs 16,000 C’s share in the profit amounts to
7. Two merchants A and B are in partnership (a) Rs 6,000 (b) Rs 7,000
A puts in Rs 20,000 and then after eight (c) Rs 8,000 (d) Rs 9,000
176 ✫ Quantitative Aptitude

13. A and B entered into a partnership If Vinod had joined the business only six
investing Rs 12,000 and Rs 20,000. A is a months later, how much must be the share
working partner whereas B is a sleeping of Vinod in a profit of Rs 5,200?
partner. A receives 10% of the profits for (a) Rs 1,200 (b) Rs 1,300
managing, the rest of the profit is divided (c) Rs 1,400 (d) Rs 1,500
in proportion of their capitals. Out of a
21. Krishna and Gargi started a joint firm.
total profit of Rs, 9,600, the money received
by A would amount to Krishna’s investment was thrice the
(a) Rs 4,200 (b) Rs 5,340 investment of Gargi’s and the period of
(c) Rs 5,800 (d) Rs 6,270 her investment was two times the period

TS
of investment of Gargi. If Gargi got
14. A, B and C rented a video cassette for Rs 4,000 as her share of profit according
one week at a rent of Rs 350. If they use to her share of contribution, what is the
it for 6 hours, 10 hours and 12 hours total profit earned?

H
respectively, the rent to be paid by C is
(a) Rs 27,000 (b) Rs 28,000
(a) Rs 200 (b) Rs 170
(c) Rs 29,000 (d) Rs 30,000
(c) Rs 150 (d) Rs 120

G
15. A, B and C invested capitals in the ratio 22. A, B and C enter into a partnership by
2 : 3 : 5, the timing of their investments making investments in the ratio 3 : 5 : 7.
being in the ratio 4 : 5 : 6. In what ratio
would their profits be distributed? U After a year, C invests another Rs 3,37,600
while A withdraws Rs 45,600. The ratio of
O
(a) 2 : 1 : 7 (b) 4 : 12 : 27 investments then changes to 24 : 59 : 167.
(c) 8 : 15 : 30 (d) 7 : 13 : 20 How much did A invest initially?
(a) Rs 1,41,600 (b) Rs 1,40,050
H
16. Divide Rs 7,500 among A, B and C such
(c) Rs 1,38,200 (d) Rs 1,30,040
that A’s share to B’s share is in the ratio
23. Anju, Shooja and Jessy start a garments
_T

5 : 2 and B’s share to C’s share is in the


ratio 7 : 13. How much will B receive? exports business together. Anju invests a
(a) Rs 1,400 (b) Rs 3,500 certain sum at the beginning, Shooja invests
(c) Rs 2,600 (d) Rs 7,000 double that amount after 4 months and
C

17. Rs 120 is divided between X, Y and Z so Jessy invests thrice that amount after 6
that X’s share is Rs 20 more than Y’s and months. If the annual profit is 23,000,
PS

Rs 20 less than Z’s. How much is Y’s share? what is Jessy’s share?
(a) Rs 40 (b) Rs 30 (a) Rs, 9,000 (b) Rs 10,000
(c) Rs 25 (d) Rs 20 (c) Rs 11,000 (d) Rs 12,000
U

18. A and B invested in trade. They earned 24. Ram, Rahim and Robert rent a piece of
some profit which they divided in the pasture for a month. Ram puts in 20 cattle
ratio 2 : 3 of their investment. If A for 25 days, Rahim puts in 18 cattle for
@

invested Rs 40, then the amount invested 26 days and Robert puts 22 cattle for a
by B is 23 days. Of the following, which would
(a) Rs 30 (b) Rs 60 not be the rent paid by any of them if
(c) Rs 90 (d) Rs 100 the rent of the pasture is Rs 368.50?
19. Four persons A, B, C D agree with a fifth (a) Rs 126.50 (b) Rs 117.00
person E to collect a certain sum. They (c) Rs 125.20 (d) Rs 138.00
subscribe Rs 15 each and E is to pay Rs 25. John, Sandeep and Uzair invest in a
4 more than the average of all the five. business. If 5 times John’s capital is equal
The whole sum collected amounts to to 8 times Uzair’s capital which is equal
(a) Rs 60 (b) Rs 70 to 2 times Sandeep’s capital, find Sandeep’s
(c) Rs 80 (d) Rs 90 share in a profit of Rs 66,000.
20. Praveen and Vinod started a business (a) Rs 70,000 (b) Rs 60,000
investing Rs 6,000 and Rs 4,000 respectively. (c) Rs 50,000 (d) Rs 40,000
Unit Two : Practice Session ✫ 177

26. A sum of money is divided among A, B (a) Rs 4 (b) Rs 10


and C so that for each rupee A gets, B (c) Rs 22 (d) Rs 26
gets 76 paise and C gets 38 paise. If C’s 29. Two friends P and Q started a business
share is Rs 57, what is the total sum? investing in the ratio of 5 : 6. R joined
(a) Rs 221 (b) Rs 321 them after 6 months investing an amount
(c) Rs 421 (d) Rs 521 equal to that of Q’s. At the end of the
27. A, B and C are partners in business. A’s year 20% profit was earned which was
capital is double that of B’s capital and equal to Rs 98,000. What was the amount
1 invested by R?
C’s capital is th of the total capital. What

TS
5 (a) Rs 1,00,000
is the share of A out of an annual profit (b) Rs 1,05,000
of Rs 1,800? (c) Rs 1,75,000
(a) Rs 960 (b) Rs 1,000 (d) Rs 2,10,000

H
(c) Rs 1,100 (d) Rs 1,300
30. A started a software business by investing
28. Three hikers A, B and C start on a trip Rs 50,000. After 6 months, B joined him

G
with Rs 50 each and agree to share the with a capital of Rs 80,000. After 3 years,
expenses equally. If at the end of the trip, they earned a profit of Rs 24,500. What
A has Rs 20 left with him, B Rs 30 and
C Rs 40, how must they settle their
U was A’s share in profit?
(a) Rs 10,250 (b) 12,500
O
accounts? (c) Rs 14,000 (d) None of these
H
Answers with Solutions
1. Ans. (d) 4. Ans. (d)
_T

Sol. The profit should be divided in the Sol. Let B’s investment be Rs x.
ratios of the capitals, i.e., in the ratio Then, 8000 : x : : 360 : 450
5 : 7 : 10. 8000 360
C

Sum of ratio = 5 + 7 + 10 = 22 ⇒ =
x 450
10
C’s share = ×17600 = Rs 8000. 8000 × 450
PS

22 ⇒ x =
360
2. Ans. (a) ⇒ = Rs 10000.
Sol. Ratio of capitals 5. Ans. (a)
U

= 5000 × 12 : 8,000 × 9 : 12000 × 2


= 5 × 12 : 8 × 9 : 12 × 2 Sol. Suppose B joined after x months.
@

=5 : 6 : 2 Then, B’s money was invested for


Sum of ratio = 5 + 6 + 2 = 13 (12 – x) months.
2
C’s share = × 9100 = Rs 1400. 76000 ×12 2
13 ∴ 57000 ×(12 – x )
=
3. Ans. (b) 1
Sol. Let B’s capital be used for x months. ⇒ 912000 = 114000 (12 – x)
The ratio of A’s and B’s capital 114(12 – x) = 912
= 7×8 : 9× x ⇒ 12 – x = 8
7 ×8 8 ⇒ x =4
So, we have = Hence, B joined after 4 months.
9×x 9
7×8×9 6. Ans. (a)
⇒ x = 1
8×9 Sol. A = (B + C) ⇒ 2A = B + C
⇒ x = 7 2
1
∴ B’s capital is used for 7 months. B = (A + C) ⇒ 4B = A + C
4
178 ✫ Quantitative Aptitude

A + B + C = 56250 9x 32x
⇒ A + 2A = 56250 ⇒ + 840 =
25 50
⇒ A = 18750 32 x 9 x
Similarly, B = 11250 and C = 26250. ⇒ 840 = –
50 25
∴ Share of A is more than that of B by
32 x – 18 x
Rs 7500. =
7. Ans. (a) 50
Sol. Let ‘x’ be B’s initial capital 14 x 7x
840 = =
Ratio of profit of A and B 50 25

TS
= 20000 × 12 + 10000 × 4 840 × 25
⇒ x = = Rs 3000
: x × 4 + (x – 4000) × 8 7
= 240000 + 40000 : 4x + 8x – 32000 ∴ Total profit is Rs 3,000.
= 280000 : 12x – 32000 are equal (given) 9. Ans. (a)

H
i.e., 12x – 32000 = 280000 Sol. A : B : C = 2 : 3 : 7
12x = 280000 + 32000 i.e.,these shares are 2x, 3x and 7x
respectively.

G
= 312000
312000 i.e., 2x + 3x = 7x–1500
x = = Rs 26000 i.e., 1500 = 7x – 5x = 2x

8. Ans. (b)
12
Hence, B’s initial capital is Rs 26,000.
U x =
1500
2
= 750
O
Sol. Ratio of investment ∴ A’s share = 2 × 750 = Rs 1,500.
= 8000 : 5000 : 7000 10. Ans. (b)
Sol. Ratio of initial investments
H
Sum of the ratios = 8 : 5 : 7
Let the total profit be x. 1 1 1
60 = : :
×x 2 3 4
_T

Then 60% of the profit =


100 =6:4:3
3x Let their investments be 6x, 2x and 3x
=
5 respectively.
Let A, B and C be the partners.
C

A:B:C
3x =(6x × 2 + 3x × 10) : (4x × 12) : (3x × 12)
3x x
Then share of each = 5 = = 42 : 48 : 36
PS

=
3 3×5 5 =7:8:6
∴ A’s share of the remaining profit 8
∴ B’s share =Rs 378 × = Rs 144.
8 40 4x 21
= × ×x = 11. Ans. (d)
U

20 100 25
Sol. Suppose C invests Rs x.
5 40 x 2x
B’s share of profit = × ×x =
Then, B invests Rs and A invests Rs 2x.
@

20 100 10 3
7 40 x 7x 2
C’s share = × = ∴ Ratio of investments = 2 x : x : x
20 100 50 3
x 4x 5x + 4x = 6 : 2 : 3.
A’s total share = + = 12. Ans. (a)
5 25 25
9x Sol. Suppose A invested Rs x.
= Then, A : B : C = 12x : 6 × (2x) : 4 × (3x)
25
x x 3x = 1:1:1
B’s total share = + =
5 10 10 1
x 7x 17 x ∴ C’s share=Rs 18000 × = Rs 6000
3
C’s total share = + =
5 50 50 13. Ans. (a)
9x 3 x 17 x
+ 840 = + Sol. Ratio of investments of A and B
25 10 50
15 x + 17 x 32 x = 12000 : 20000
= = =3 : 5
50 50
Unit Two : Practice Session ✫ 179

As for management, A receives ⇒ 5x = 80 + x


= Rs 960 ⇒ 4x = 80
∴ Balance = Rs 9600 – 960 ⇒ x = 20
= Rs 8640 ∴ E subscribes Rs 20.
3 Whole sum = 60 + x = 60 + 20 = Rs 80.
∴ A’s share = Rs 8640 ×
8
20. Ans. (b)
= Rs 3240
So, A receives = Rs (3240 + 960) Sol. Ratio of their investments
= Rs 4,200. = 6000 : 4000 = 3 : 2

TS
14. Ans. (c) Ratio of their profit
Sol. Ratio of rents = 6 : 10 : 12
=3 : 5 : 6 = 6000 × 12 : 4000 × 6
6 = 72000 : 24000
∴ C has to pay = Rs 350 ×

H
14 =3 : 1
= Rs 150. 1
15. Ans. (c) Vinod’s share = × 5200 = Rs 1300.

G
4
Sol. The required ratio =2 × 4 : 3 × 5 : 5 × 6
=8 : 15 : 30. 21. Ans. (b)
16. Ans. (a)
Sol. A : B = 5 : 2
B : C = 7 : 13 U Sol. Ratio of investment of Krishna and
Gargi = 3 : 1
O
Ratio of time = 2 : 1
∴ A : B : C = 35 : 14 : 26
⇒ ratio of profit = 6 : 1
14
H
∴ B receives = × 7500 If Gargi got Rs 4000
35 + 14 + 26
= Rs 1,400. ⇒ Total profit = 7 × 4000
_T

17. Ans. (d) = Rs 28,000.


Sol. Let X’s share = x, Y’s share = y and 22. Ans. (a)
Z’s share = z.
By hypothesis Sol. Initial investments of A : B : C = 3 : 5 : 7
C

x = y + 20 and x = z – 20 After one year, the investment of A : B : C


∴ y + 20 = z – 20 = 3x – 45600 : 5x : 7x + 337600
PS

∴ z = y + 40
Now, y + 20 + y + y + 40 = 120 = 24 : 59 : 167
or 3y + 60 = 120 i.e., 3 x – 45600 : 5x = 24 : 59
or 3y = 120 – 60 3 x – 45600 24
U

60 =
5x 59
∴ y = = Rs 20.
3 59 × (3x – 45600) = 24 × 5x
@

Hence, Y’s share is Rs 20. 177x – 120x = 59 × 45600


18. Ans. (b)
57x = 59 × 45600
Sol. 2 : 3 = 40 : x
⇒ 2 × x = 3 × 40 45600
⇒ x = 59 ×
3 × 40 57
⇒ x = = Rs 60. = 47200
19. Ans. (c) 2
∴ A’s initial investment = 3 × 47200
Sol. Suppose E subscribes Rs x = Rs 1,41,600.
15 × 4 + x 60 + x
∴ Average = = 23. Ans. (a)
5 5 Sol. Let the sum invested by Anju be x for
According to the given situation 12 months.
60 + x Then Shooja’s investment
x = +4
5 = 2x for 8 months
60 + x + 20 and Jessy’s investment
⇒ x =
5 = 3x for 6 months
180 ✫ Quantitative Aptitude

Profit of Anju : Shooja : Jessey 20


Sandeep’s share = × 66000
= 12x : 16x : 18x 33
= 6 : 8 : 9 = Rs 40000.
Sum of the ratios = 6 + 8 + 9 = 23 26. Ans. (b)
∴ Anju’s share of profit Sol. A : B : C = 100 paise : 76 paise : 38 paise
6 = 50 : 38 : 19
= × 23000 = Rs 6000
23 Total ratio = 107
∴ Shooja’s share of profit ∴ If x is the total sum,
8 19
= × 23000 = Rs 8000 × x = 57

TS
23 107
and Jessy’s share 57 ×107
9 x= = Rs 321.
× 23000 = Rs 9000. 19
= 27. Ans. (a)
23

H
24. Ans. (d) 1
Sol. C’s capital is th of the total capital.
5
Sol. Converting into equivalent number of 1
⇒ C’s ratio is th of the total capital

G
cattle for 1 day. 5
1 4
Ram’s cattle = 20 × 25 = 500 ⇒ A + B’s share = 1 − =
Rahim’s cattle = 18 × 26 = 468
Robert’s cattle = 22 × 23 = 506
U 5 5
A’s capital = 2 × B’s capital
4
O
Ratio of cattle of Ram : Rahim : Robert i.e., 2B + B = 3B=
= 500 : 468 : 506 5
= 250 : 234 : 253 4
H
4
Sum of the ratio = 250 + 234 + 253 B = 5 =
3 15
= 737 4
_T

8
∴ Ram’s share of payment A = 2×
15
=
15
250 8
= × 368.50 = Rs 125
737 ∴ A’s share = ×1800 = Rs 960.
15
C

Rahim’s share of payment 28. Ans. (b)


234 Sol. Total money = 50 × 3 = 150
= × 368.50 = Rs 117
PS

737 Money spent = 30 + 20 + 10 = 60


Robert’s share Share of each = Rs 20
253 ⇒ A has spent Rs 10 more and C has spent
= × 368.50 = Rs 126.50. Rs 10 less.
737
∴ C should give Rs 10 to A.
U

25. Ans. (d)


29. Ans. (d)
Sol. 5J = 8U = 2S = x Sol. Let the total profit be Rs x.
@

(J is John’s Capital, U is Uzair’s capital Then 20% of x = 98000


and S is Sandeep’s capital) 100
x = 98000 × = 490000
x x x 20
∴ J = , U = , S = Let the capitals of P, Q and R be Rs 5x,
5 8 2
x x x 1 1 1 Rs 6x and Rs 6x respectively. Then,
i.e., J : U : S = : : = : :
5 8 2 5 8 2 (5x + 12)+(6x + 12) + (6x × 6)
= 8 : 5 : 20 490000 × 12
= = 35000
8 168
∴ John’s share of profit = × 66000 ∴ R’s investment = (6x) = Rs (6 × 35000)
33
= Rs 16000 = Rs 2,10,000.
5 30. Ans. (d)
Uzair’s share = × 66000 Sol. A : B = 50000 × 36 : 80000 × 30 = 3 : 4
33
3
= Rs 10000 ∴ A’s share = Rs 24500 × = Rs 10,500.
7
Unit Two : Practice Session ✫ 181

Practice Exercise 13
1. A man buys an article for Rs 25 and sells 10. A man purchases an electric heater whose
it for Rs 30. His profit is printed price is Rs 2000. If he received two
(a) 16.67% (b) 20% successive discounts of 20% and 10%, he
(c) 25.5% (d) 25.67% paid
2. 200 mangoes are bought at the rate of Rs (a) Rs 1440 (b) Rs 1560
700 and sold at the rate of Rs 96 for 12. (c) Rs 1670 (d) Rs 1790
The percentage of profit or loss is 11. In a certain shop, the profit is 320% of

TS
5 3 the cost. If the cost increases by 25% but
(a) 14 (b) 58 the selling price remains the same, what
9 7
4 4 percentage of the selling price is the
(c) 102 (d) 128 profit?

H
5 7
3. By selling an article for Rs 200, a man (a) 250% (b) 100%
(c) 70% (d) 30%

G
gains Rs 30. His gain per cent is
(a) 17.64% (b) 18.93% 12. A shopkeeper sells 17 bats at Rs 720
(c) 19.02% (d) 20% making a loss equal to the cost price of
4. If a man reduces the selling price of a fan
from Rs 400 to Rs 380, his loss increases U 5 bats. What is the cost price of a bat?
(a) Rs 45 (b) Rs 50
O
(c) Rs 57 (d) Rs 60
by 2%. The cost price of the fan is
13. The cost of 20 articles is the same as the
(a) Rs 1000 (b) Rs 1500
H
selling price of x articles. If a profit of 25%
(c) Rs 1750 (d) Rs 2000
is made, what is the value of x?
5. If a shopkeeper sells an item for Rs 141
_T

(a) 25 (b) 18
his loss is 6%. To earn a profit of 10% (c) 16 (d) 15
he should sell it for 14. If the cost price of 12 pencils is equal to
(a) Rs 155 (b) Rs 160 the selling price of 8 pencils, what is the
C

(c) Rs 165 (d) Rs 170 gain per cent?


6. At a clearance sale, the prices had been (a) 50% (b) 40%
PS

reduced by 20%. If a transistor was being 1


sold at Rs 150 before reduction, the price (c) 33 % (d) 60%
3
at the clearance sale was 15. Ravi bought a cycle for a certain sum of
U

(a) Rs 130 (b) Rs 125 money. He spent 10% of the cost on


(c) Rs 120 (d) Rs 115 repairing it. Then he sold the cycle for a
@

7. If an article is sold at a loss of 50%, the profit of Rs 1100. If he made a profit of


cost price in terms of selling price is 20%, how much did Ravi spend on the
(a) 0.5 (b) 2 repairs?
(a) Rs 560 (b) Rs 500
(c) 2.5 (d) 3
(c) Rs 450 (d) Rs 400
8. A shirt marked at Rs 400 is sold for Rs
16. A property dealer sells a house for Rs
325. The rate of discount is 6,30,000 and in the bargain makes a profit
(a) 15% (b) 18.75% of 5%. Had he sold it for Rs. 5,00,000,
(c) 19.35% (d) 20% what percentage of loss or gain he would
9. By selling 80 metres of cloth, a person gains have made?
the cost of 12 metres. The gain % is (a) 15% loss (b) 15 % gain
(a) 13.35% (b) 14.57% 2 2
(c) 16 % loss (d) 16 % gain
(c) 15% (d) 17.65% 3 3

181
182 ✫ Quantitative Aptitude

17. If cost price is Rs 80, overhead is Rs 20 26. A trader marked the selling price of an
and selling price is Rs 120, then the profit article at 10% above the cost price. At the
per cent is time of selling, he allows a certain discount
(a) 20% (b) 50% and suffers a loss of 1%. He allowed a
(c) 40% (d) 30% discount of
18. The cost price of an article which is sold (a) 10% (b) 12%
at a loss of 25% for Rs 150 is (c) 13% (d) 15%
(a) Rs 125 (b) Rs 175
27. A trader marked the price of his
(c) Rs 200 (d) Rs 225
commodity so as to include a profit of

TS
19. By selling an article for Rs 150, the profit 25%. He allowed discount of 16% on the
is 25%. Find the cost price. marked price. His actual profit was
(a) Rs 120 (b) Rs 125
(a) 55% (b) 48%
(c) Rs 75 (d) None of these

H
(c) 35% (d) 5%
20. On selling an article for Rs 240, a trader
loses 4%. In order to gain 10%, he must 28. The list price of an article is Rs. 90. If it

G
sell that article for is sold at 10% discount, then its selling
(a) Rs 264 (b) Rs 273.20 price is
(c) Rs 275.00
21. A man purchased
(d) Rs 280
a watch for Rs 400 and U (a) Rs 43
(c) Rs 81
(b) Rs 57
(d) Rs 103
O
sold it at a gain of 20% of the selling price. 29. By selling a book for Rs 31 a person loses
The selling price of the watch is 7% of his outlay. If he sells the same book
(a) Rs 300 (b) Rs 320
H
for Rs 35, the bargain is
(c) Rs 440 (d) Rs 500 (a) 4% loss (b) 4% profit
22. A man gains 10% by selling a certain (c) 5% loss (d) 5% profit
_T

article for a certain price. If he sells it at 30. A tradesman allows a discount of 30% on
double the price, the profit made is the written price. How much above the
(a) 120% (b) 60%
C.P. must he mark his goods to gain 20%?
C

(c) 100% (d) 80%


(a) 82% (b) 71.42%
23. A trader purchased an old bicycle for Rs (c) 69.3% (d) 60%
PS

480. He spent 20% of the cost on its repair.


If he wants to earn Rs 144 as net profit 31. What profit per cent is made by selling
on it, how much percentage should he add an article at a certain price, if by selling
2
to the purchase price of the bicycle? at of that price there would be a loss
U

(a) 50% (b) 48% 3


of 20%?
(c) 96% (d) 100%
@

(a) 10% (b) 20%


24. A vendor sells a part of oranges at a gain (c) 25% (d) 30%
of 20% and the balance at a loss of 5%.
If the vendor has 24 kg oranges and on 32. An article passes through three hands and
the whole he earns a profit of 10%, the each gains 25%. If the third sells it for
amount of oranges sold at a loss is Rs. 250, what did the first pay for it?
(a) 9.3 kg (b) 9.6 kg (a) Rs 500 (b) Rs 460
(c) 9.8 kg (d) 9.9 kg (c) Rs 340 (d) Rs 128
25. Suchitra buys an article at a discount of
33. If an article is sold for Rs 178 at a loss
25%. At what percentage above the cost
of 11%, what should be its selling price
price should she sell it to make a profit
of 25% over the original list price? in order to earn a profit of 11%?
(a) 25 (b) 30 (a) Rs 222.50 (b) Rs 267
(c) 40 (d) 66.67 (c) Rs 222 (d) Rs 220
Unit Two : Practice Session ✫ 183

34. By selling a table for Rs. 350 instead of 35. The loss incurred on selling an article for
Rs 400, loss per cent increases by 5%. The Rs. 270 is as much as profit made after
selling it at 10% profit. The C.P. of the
cost price of the table is
article is
(a) Rs 1,050 (b) Rs 417.50 (a) Rs 90 (b) Rs 110
(c) Rs 435 (d) Rs 1,000 (c) Rs 363 (d) Rs 300

Answers with Solutions

TS
1. Ans. (b) 6. Ans. (c)
Sol. Profit = Rs (30 – 25) = Rs 5 Sol. S.P. before reduction = Rs 150
5 S.P. after reduction = 100 – 20
Profit % = × 100 = 20%.
25 = 80% of Rs 150

H
80
2. Ans. (d) = ×150
100

G
700 = Rs 120.
Sol. C.P. of 1 mango = Rs = Rs 3.50
200 7. Ans. (b)
Sol. Let C.P. = 100, Loss = 50%
S.P. of 1 mango = Rs
96
12
= Rs 8
U S.P.
or S.P.
=
=
100 – 50 = 50
50% of C.P.
O
8 – 3.50 or C.P. = 200% of S.P.
∴ Gain % = × 100 % or C.P. = 2 × S.P.
3.50
H
8. Ans. (b) 400 – 325
4.50 4 ×100
= ×100 % = 128 % . Sol. Discount =
3.50 7 400
_T

75
3. Ans. (a) = ×100 = 18.75%.
400
Sol. S.P. = Rs 200, gain = Rs 30 9. Ans. (d)
∴ C.P. = Rs (200 – 30) = Rs 170 Sol. Let S.P. of 80 m of cloth = Rs 80
C

30
×100 % = 17.64%. ∴ Gain = Rs 12
Gain % =
170 C.P. = S.P. – Gain = Rs 68
PS

4. Ans. (a) 12
∴ Gain % = × 100
Sol. Let C.P. be Rs x 68
Then 2% of x = (400 – 380) = 20 = 17.65%.
U

x 10. Ans. (a)


= 20 Sol. Price after 1st discount
50
= 80% of Rs 2000
@

x = Rs 1000.
= Rs 1600
5. Ans. (c) Price after 2nd discount
Sol. S.P. of article = Rs 141 = 90% of Rs 1600
Loss = 6% = Rs 1440.
100 11. Ans. (b)
∴ C.P. of article = Rs 141× Sol. Let C.P. be Rs 100. Then profit = Rs 320
100 – 6 and S.P. is Rs (100 + 320 =) Rs 420.
100 New C.P. = 125% of Rs 100 = Rs 125
= 141×
94 New S.P. is same as before = Rs 420
= Rs 150 New profit = (Rs 420 – 125 =) Rs 295
Now, in order to have a profit = 10% As a percentage of the S.P., the profit is
100 + 10 295 1475
S.P. should be = Rs 150 × × 100 % =
100 420 21
= Rs 165. = 70% approximately.
184 ✫ Quantitative Aptitude

12. Ans. (d) [Alternately, the actual cost can be


Sol. Loss being C.P. – S.P., under the given 100
conditions, worked out as × 5500 = Rs 5000
110
C.P. of 5 bats Expenditure on repairs
= C.P. of 17 bats – S.P. of 17 bats = Rs 5500 – Rs 5000 = Rs 500.]
⇒ Rs 720 = C.P. of 12 bats = S.P. of 17 bats
16. Ans. (c)
720 Sol. S.P. of house = Rs 630000
⇒ Cost of 1 bat = = Rs 60.
12 Profit = 5% 100
13. Ans. (c) ∴ C.P. of house = Rs 630000 ×

TS
105
Sol. Let’s take the C.P. of each article to be = Rs 600000
Re 1. So C.P. of x articles is Rs x, and When S.P. is Rs 500000
S.P. of x articles will be Rs 20.
600000 – 500000

H
Profit will be Rs 20 – x. Loss % = × 100
As it is given that a profit of 25% is 600000
made, we have 100000

G
= × 100
20 − x 600000
× 100 = 25 100 50 2
x


2000 − 100x
= 25 U 17. Ans. (a)
=
6
=
3
= 16 % .
3
O
x
Sol. Actual cost is C.P. + overhead
⇒ 2000 – 100x = 25x
⇒ 125x = 2000 = 80 + 20 = Rs 100
H
2000 S.P. = Rs 120
⇒ x = = 16. Profit = 120 – 100 = Rs 20 on Rs 100
125

_T

Profit % = 20%.
14. Ans. (a)
18. Ans. (c)
Sol. Suppose the C.P. of one pencil is Re 1.
Then C.P. of 8 pencils = Rs 8. 150 × 100
Sol. C.P. =
C

S.P. is C.P. of 12 pencils or Rs 12. 100 – 25%


Gain = 12 – 8 = Rs 4. 150 ×100
= = Rs 200.
PS

4 75
∴ Gain % = × 100 % = 50%. 19. Ans. (a)
8
150
15. Ans. (b) Sol. C.P. = ×100 = Rs 120.
125
U

Sol. Suppose C.P. to be x.


Then gain is 20% of x = Rs 1100 20. Ans. (c)
20 240 ×100
@

⇒ x = 1100 Sol. C.P. =


96
= 250
100
100 250 ×110
⇒ x = 1100 × = Rs 5500 S.P. = = Rs 275.
20 100
C.P. is Rs 5500. 21. Ans. (d)
But Ravi spends 10% on repairs so his Sol. Let the S.P. be x
cost is 10% more. S.P. – Profit = C.P.
So if the original cost is Rs 100, the cost 20
x– × x = 400
after repairs, which is the actual cost, is 100
Rs 110. 5x – x
Now if the cost is Rs 110, money spent = 400
5
on repairs is Rs 10. 4x
If cost is Rs 5500, money spent on = 400
5
10 400 × 5
repairs is × 5500 = Rs 500. ∴ S.P. = = Rs 500.
110 4
Unit Two : Practice Session ✫ 185

22. Ans. (a) 26. Ans. (a)


Sol. Suppose C.P. of article = Rs x Sol. Let C.P. = Rs 100
100 + 10 11 Then, Marked price = Rs 110
∴ S.P. of article = Rs × x = Rs x
100 10 S.P. = Rs 99
11 11
When S.P. = 2 × x = Rs x 11
10 5 ∴ Discount % = × 100 % = 10%.
11 110
x–x 27. Ans. (d)
Then, profit = 5 ×100
x Sol. Let C.P. be Rs 100

TS
6 Then, marked price = Rs 125
= × 100 = 120%.
5 S.P. = 84% of Rs 125
23. Ans. (a)
84

H
Sol. C.P. of old bicycle = Rs 480 = ×125
100
Expenditure on its repair = 20% of 480 = Rs 105

G
20
= × 480 = Rs 96 ∴ Profit (%) = (105 – 100)% = 5%.
100
His net profit = Rs 144 28. Ans. (c)
∴ Required % =
144 + 96
480
×100
U Sol. List price = Rs 90,
O
Discount = 10% of Rs 90
240
= ×100 = 50%. 10 × 90
480 = = Rs 9
H
24. Ans. (b) 100
Sol. Let the quantity sold at a loss be x kg ∴ Selling price = Rs (90 – 9)
_T

and let C.P. per kg be Re 1 = Rs 81.


Total C.P. = Rs 24 29. Ans. (d)
Total S.P. = Rs 120% of (24 – x) + 95% Sol. S.P. = Rs 31, loss 7 %
C

of x 31×100 100
6 19 x ∴ C.P. = = Rs
(24 – x ) + 93 3
= Rs
PS

5 20 IInd S.P. = Rs 35
576 – 5 x 100 5
= Rs Profit = Rs 35 – Rs = Rs
20 3 3
5 3
U

576 – 5 x % profit = × ×100 = 5%.


∴ = 110% of 24 3 100
20
@

576 – 5 x 264 30. Ans. (b)


⇒ =
20 10 Sol. Let C.P. = Rs 100
⇒ 576 – 5x = 528 Gain required = 20%
⇒ 5x = 48 ∴ S.P. must be Rs 120
⇒ x = 9.6 kg. Now 70% of marked price = Rs 120
25. Ans. (d) 70
∴ × x = 120
Sol. Let original list price = Rs 100 100
Then, C.P. = Rs 75 120 × 100
⇒ x = = 171.42
Desired S.P. = Rs 125 70
50 ∴ Marked price = Rs 171.42
∴ Required percentage = × 100 %
75 So, the trader must mark his goods
= 66.67%. 71.42% above cost price.
186 ✫ Quantitative Aptitude

31. Ans. (b) 178 × 100 100 + 11


Required S.P. = ×
2 100 – 11 100
Sol. of the selling price
3 = Rs 222.
= (100 – 20)% of cost price
34. Ans. (d)
80 × 3 Sol. Let initial loss be x%
S.P. = % of C.P.
2 350 ×100 400 ×100
= 120% of the C.P. Here, 100 – ( x + 5) =
100 – x
∴ Profit = 20%. ⇒ x = 60

TS
400 ×100
32. Ans. (d) ∴ C.P. =
100 – 60
Sol. First purchased for = Rs 1,000.

H
100 100 100 35. Ans. (d)
Rs 250 × × ×
125 125 125 Sol. Let the cost price be x.

G
= Rs 128. 10
x – 270 = ×x
33. Ans. (c) 100
S.P. × 100
Sol. C.P. = 100 – Loss % U ⇒ x–
x
10
= 270
9x = 10 × 270
O
C.P. × (100 + gain %) 10 × 270
S.P. = x = = Rs 300.
100 9
H
C _T
PS
U
@
Unit Two : Practice Session ✫ 187

Practice Exercise 14
1. Ravi purchased 120 reams of paper at Rs 8. The sale price of an article including the
80 per ream. He spent Rs. 280 on sales tax is Rs. 616. The rate of sales tax
transportation, paid octroi at the rate of is 10%. If the shopkeeper has made a
40 paise per ream and paid Rs. 72 to the profit of 12%, then the cost price of the
coolie. If he wants to have a gain of 8%, article is
the selling price per ream must be (a) Rs 500 (b) Rs 515
(a) Rs 86 (b) Rs 89 (c) Rs 550 (d) Rs 600

TS
(c) Rs 90 (d) Rs 87.48 9. The ratio between profit and sale price of
2. A man sells a car to his friend at 10% loss. an article is 1 : 6. What is the ratio
If the friend sells it for Rs 54000 and gains between the sale price and cost price of
20%, the original C.P. of the car was

H
the article?
(a) Rs 25000 (b) Rs 37500 (a) 5 : 6 (b) 6 : 5
(c) Rs 50000 (d) Rs 60000 (c) 3 : 2 (d) 4 : 2

G
3. While selling an article for Rs. 450, a man 10. Somesh bought a microwave oven and
loses 25%. At what price should he sell paid 10% less than its original price. He
in order to gain 25%?
(a) Rs 550 (b) Rs 600 U sold it with 30% profit on the price he
had paid. What percentage of profit did
O
(c) Rs 650 (d) Rs 750 Somesh earn on the original price?
4. An electric heater is sold at a gain of 16%. (a) 32% (b) 117%
H
If it were sold for Rs. 20 more, there (c) 20% (d) 17%
would have been a gain of 20%. The C.P. 11. The marked price of a certain commodity
of the heater (in Rs) is
_T

is 30% higher than its cost price. A


(a) 450 (b) 550 discount of 20% on the marked price
(c) 850 (d) 500 makes the selling price Rs. 208. What is
5. By selling an article for Rs. 24, a man gains the net profit as a per cent of cost price?
C

as many per cent as the cost price. Then (a) 10% (b) 4%
the cost price is (c) 8% (d) 12%
PS

(a) Rs 10 (b) Rs 20 12. A man bought a gas cooker and microwave


(c) Rs 30 (d) None of these for Rs 15000. He sold the gas cooker at
6. Sajita sold an article to Rekha which costs a gain of 20% and the microwave at a loss
U

Sajita Rs 80,000 at a loss of 20%. Rekha of 10%, thereby gaining 4% on the whole.
sells it to Manisha making a profit of 15% Find the cost of the gas cooker?
on the price she paid to Sajita. What is (a) Rs 7000 (b) Rs 6500
@

the difference in original C.P. and final (c) Rs 6000 (d) Rs 5500
S.P.? 13. When the price of a product was increased
(a) 4600 (b) 6400 by 15%, the number sold was decreased
(c) 5600 (d) None of these by 20%. What was the net effect?
7. Deena buys two cycles for a total cost of (a) 8% gain (b) 5% loss
4 (c) 8% loss (d) None of these
Rs 4000. By selling one cycle for th of
5 5 14. If 7% of the sale price of an article is
its cost and the other for th of its cost, equivalent to 8% of its cost price and 9%
4
she makes a profit of Rs 190 on the whole of its sale price exceeds 10% of its cost
transaction. Find the cost price of each price by Re 1, then what is the cost price
cycle. of the article?
(a) 2000 each (b) 3000 and 1000 (a) Rs 400 (b) Rs 350
(c) 1200 and 2800 (d) 1800 and 2200 (c) Rs 300 (d) Rs 280

187
188 ✫ Quantitative Aptitude

15. A cycle agent buys 30 bicycles for Rs 3150, 21. A man sells 320 mangoes at the cost price
of which eight are first grade and the rest of 400 mangoes. His gain % is
are second grade. Find at what price he (a) 10% (b) 25%
must sell the first grade bicycles so that (c) 15% (d) 20%
if he sells the second grade bicycles at
22. If I purchased 11 books for Rs 10 and sold
three quarters of the price, he may make
all the books at the rate of 10 books for
a profit of 40% on his outlay?
Rs 11, the profit % is
(a) Rs 200 (b) Rs 240
(a) 10% (b) 11%
(c) Rs 180 (d) Rs 210
(c) 21% (d) 100%

TS
16. A man sells two articles at Rs 99 each. In
23. A person sells 36 toffees per rupee and
one he gains 10% and on the other he loses
suffers a loss of 4%. Find how many
10%. What is his gain or loss per cent on
toffees per rupee should he sell if he

H
the whole transaction?
(a) Loss, 1% (b) Loss, 1.5% wants to have a gain of 8%?
(c) Profit, 1% (d) Profit, 1.5% (a) 32 (b) 34

G
(c) 38 (d) 40
17. A manufacturer of a certain item can sell
all he can produce at the selling price of 24. A cloth merchant professes to sell cloth
Rs 60 each. It costs him Rs 40 in materials
and labour to produce each item and he U at 10% loss, but uses a false metre scale
and actually gains 15%. Find the actual
O
has overhead expenses of Rs 3000 per length of the scale.
week in order to operate that plant. The (a) 78 cm (b) 87 cm
H
number of units he should produce and (c) 78.26 cm (d) 80 cm
sell in order to make a profit of at least 25. A sells his goods 20% cheaper than B and
_T

Rs 1000 per week is 20% dearer than C. A man buys goods


(a) 400 (b) 300 worth Rs 9600 from A. Would it have been
(c) 250 (d) 200 more profitable or less profitable and by
how much to have bought half the goods
C

18. A tradesman by means of false balance


defrauds to the extent of 8% in buying from B and half from C?
goods and also defrauds to the extent of (a) More profitable, by Rs 760
PS

8% in selling. His gain per cent is (b) Less profitable, by Rs 540


(a) 16% (b) 15.48% (c) Less profitable, by Rs 400
(c) 16.64% (d) 36% (d) Less profitable, by Rs 310
U

19. A sells an item to B at 15% profit, B sells 26. A manufacturer undertakes to supply 2000
it to C at 20% profit and C sells it to D pieces of a particular component of a
@

at 25% profit. If D pays Rs 34.50 for it, machine at Rs 25 per piece. He estimates
then the item cost to A will be a 25% profit even if 5% of the consignment
(a) Rs 18 (b) Rs 20 fails the quality tests. However, 50% of
(c) Rs 22 (d) Rs 24 the components failed the test and were
20. Two motor cars were sold for Rs. 9,900 rejected. What was the loss to the
each gaining 10% on one and losing 10% manufacturer?
on the other. The gain or loss per cent (a) Rs 13000 (b) Rs 14000
in the whole transaction is (c) Rs 15000 (d) None of these
(a) neither loss nor gain 27. A pair of articles was bought for Rs 37.40
(b) 1% profit at a discount of 15%. What must be the
100 marked price of each article?
(c) % profit
99 (a) Rs 40 (b) Rs 44
(d) 1% loss (c) Rs 22 (d) Rs 11
Unit Two : Practice Session ✫ 189

28. If the selling price doubles, the profit 32. A trader marked the selling price of an
trebles. What is the profit per cent? article at 10% above cost price. When
(a) 105% (b) 100% selling the article, he allows a discount
2 that leads to a loss of 1% overall for him.
(c) 66 % (d) 120%
3 What discount did he allow?
29. A trader mixes three varieties of grain (a) 11% (b) 12%
costing Rs 50, Rs 20 and Rs 30 per kg in (c) 9% (d) 10%
the ratio 2 : 4 : 3 in terms of weight. He 33. Sahil bought a car at 20% discount on its
sells this mixture at Rs 33 per kg. What original price. He sold it with a 40%

TS
percentage of profit does he make? increase on the price at which he bought
(a) 10% (b) 9% it. By what per cent is the new sale price
(c) 8% (d) 7.5% more than the original price?

H
30. Some articles were bought at 6 for Rs 5 (a) 8% (b) 10%
and sold at 5 for Rs 6. What was the gain (c) 12% (d) 12.5%
per cent?

G
34. A shopkeeper claims to sell at cost price
(a) 40% (b) 30% but uses a weight of 800 gm for a
(c) 35% (d) 44% kilogram weight. What profit does he
31. Arun bought 30 kg of wheat at Rs 17.50
per kg and another 30 kg of wheat at a U make?
(a) 20% (b) 25%
O
different rate. He sold the entire mixture (c) 27% (d) Cannot be determined
of the two varieties at the rate of Rs 18.60 35. The marked price of a ring was Rs 720.
H
per kg. If he made an overall profit of Sumi bought the ring for Rs 550.80 after
20%, at what price per kg did Arun buy two successive discounts. The first discount
_T

the second variety of wheat? being 10%, what is the second discount?
(a) Rs 12.50 (b) Rs 13.50 (a) 15% (b) 18%
(c) Rs 14.50 (d) Rs 15.50 (c) 12% (d) 12.5%
C

Answers with Solutions


PS

1. Ans. (c) 3. Ans. (d)


Sol. Total C.P. of 120 reams of paper S.P. × 100
Sol. C.P. =
= 80 × 120 + 280 + 0.40 × 120 + 72 100 – Loss %
U

450 ×100
= 9600 + 280 + 48.00 + 72 = = Rs 600
75
= Rs 10000
@

600 × 125
10000 × 108 S.P. = = Rs 750.
S.P. per ream = 100
100 × 120 4. Ans. (d)
= Rs 90 per ream. Sol. Let the C.P. be x.
2. Ans. (c) 116 120
x + 20 = x
Sol. Let the original C.P. be x. 100 100
90 116x + 2000 = 120x
S.P. = x 2000 = 4x
100
90 120 x = 500.
x× = 54000
100 100 5. Ans. (b)
54000 ×100 Sol. Let the C.P. be x.
x =
12 × 9 100 + x
Then, x× = 24
Original C.P. = Rs 50000. 100
190 ✫ Quantitative Aptitude

100x + x2 = 2400 10. Ans. (d)


2
x + 100x – 2400 = 0 Sol. Let the original price of the microwave
(x + 120)(x – 20) = 0 be Rs x.
x + 120 = 0 or x – 20 = 0 90 9x
⇒ x = –120 or x = 20 ∴ C.P. for Somesh = × x = Rs
100 10
∴ x = 20 9x
Hence, the cost price is Rs 20. Somesh’s S.P. = 130% of = 1.17x
10
∴ Profit = 1.17x – x = 0.17x
6. Ans. (b) 0.17 x
∴ Required % profit = × 100 = 17%.

TS
80
Sol. Sajita’s S.P. = 80000 × x
100 11. Ans. (b)
= Rs 64000
115 Sol. Let C.P. be Rs x.
Rekha’s S.P. = 64000 ×

H
100 130 13 x
∴ Marked price = × x = Rs
= Rs 73600 100 10
13 x (100 – 20)

G
Difference between original C.P. and ×
S.P. = = Rs 208
final S.P. 10 100
= 80000 – 73600 = Rs 6400. ⇒ x = Rs 200
7. Ans. (d)
U ∴ Net % profit =
208 – 200
200
×100
O
Sol. Let x be the cost of one cycle.
= 4%.
⇒ 4000 – x is the cost of the other cycle. 12. Ans. (a)
H
4 5 Sol. Let the C.P. of the gas cooker be Rs x.
× x + × (4000 – x ) = 4190
5 4 Then, C.P. of microwave = Rs (15000 – x)
∴ 20% of x – 10% of (15000 – x)
_T

16 x + 25(4000 – x )
= 4190
20 = 4% of 15000
16x + 25 × 4000 – 25x = 4190 × 20 x (15000 – x )
100000 – 83800 = 9x ⇒ – = 600
5 10
C

16200 ⇒ 2x – (15000 – x) = 6000


x = = 1800. ⇒ 2x – 15000 + x = 6000
9
PS

∴ Cost of one cycle = Rs 1800. ⇒ 3x = 21000


∴ C.P. of the other cycle ⇒ x = 7000
= 4000 – 1800 = Rs 2200. Hence, C.P. of the gas cooker is Rs 7000.
U

8. Ans. (a) 13. Ans. (c)


Sol. Let the C.P. be Rs x. xy
Sol. Net effect = x – y –
∴ S.P. = x + 12% of x
@

100
112 x 28 x where, x = 15 and y = 20
= = 15 × 20
100 25 = 15 – 20 –
28 x 28 x 100
∴ + 10% of = 616 = –8
25 25
∴ Loss = 8%.
⇒ x = 500
Since the sign is –ve, there is always a
Hence, the cost price of the article is
Rs 500. loss.
9. Ans. (b) 14. Ans. (b)
Sol. Let sale price = Rs 6 Sol. Suppose C.P. = x and S.P. = y
Then, profit = Re 1 ⇒ 7 % of y = 8% of x
∴ Cost Price = Rs (6 – 1) = Rs 5
Hence, ratio between S.P. and C.P. and 9% of y = 10% of x + 1
= 6 : 5. ⇒ 7y = 8x and 9y = 10x +100
Unit Two : Practice Session ✫ 191

8x 19. Ans. (b)


⇒ 9× = 10x+100
7 Sol. Cost to A
72 x
⇒ = 10x+100 100 100 100
7 = 34.50 × × ×
100 + 25 100 + 20 100 + 15
72 x
⇒ – 10 x = 100 3450 100 100 100
7 = × × ×
2x 100 125 120 115
⇒ = 100
7 = 20.
7
⇒ x = 100 × = 350 Hence, the item cost to A is Rs 20.

TS
2
Hence, cost price of the article is Rs 350. 20. Ans. (d)
15. Ans. (c) Sol. If S.P. in two cases is same, there is
always a loss
Sol. Suppose the price of a first grade cycle

H
= x% of x
is Rs x and that of a second grade cycle = 10% of 10 = 1%.

G
is Rs y.
21. Ans. (b)
∴ 8x + 22y = 3150 Sol. Let C.P. of 400 mangoes = Rs x.
Suppose he sells the first grade bicycles
at Rs z per bicycle.
U C.P. of 1 mango =
x
400
and S.P. =
x
320
O
3z x x
∴ 8 z + 22 × = 3150 + 40% of 3150 –
4 ∴ Gain % = 320 400
×100
x
H
⇒ 98z = 17640
400
⇒ z = 180
(400 x – 320 x ) 400
_T

∴ S.P. of the first grade bicycle = Rs 180. = × ×100


400 × 320 x
16. Ans. (a) 80 x 400
= × ×100 = 25%.
Sol. When S.P. of two articles is same and 400 × 320 x
C

x% is the gain on one while x% is the


loss on the other article, there is always 22. Ans. (c)
10
PS

a loss of x% of x. Sol. C.P. of 1 book = and


11
Here, x = 10 11
10 S.P. of 1 book = .
∴ Loss = × 10 = 1%. 10
100
U

11 10
17. Ans. (d) –
10 11
Sol. Let the required number of units Profit % = ×100
10
@

produced and sold be x. 11


∴ 60x – 40x – 3000 = 1000 121 – 100
×100
⇒ 20x = 4000 110
⇒ x = Rs 200. =
10
18. Ans. (c) 11
(100 + x )2 21 11
Sol. Gain % = – 100 = × ×100 = 21%.
100 110 10
Here x = 8%
23. Ans. (a)
(100 + 8)2 1
= – 100 Sol. S.P. of 1 toffee =
100 36
11664 – 10000 1
= ×100
100 C.P. = 36
= 16.64%. 100 – 4
192 ✫ Quantitative Aptitude

1×100 25 Total cost incurred


= = 100
36 × 96 864 × 25 × (95% of 2000)
= Rs
25 108 125
Now, S.P. = ×
864 100 (gain % being 25 and S.P. of each
1 component supplied being Rs 25)
=
32 100
= × 25 × 1900
⇒ 32 toffees should be sold for Re 1. 125
= Rs 38000

TS
24. Ans. (c) As half the components fail the test, the
True scale manufacturer sells only 1000 components
Sol. 100 + gain (g) = ×(100 + x )
False scale S.P. = 1000 × 25 = 25000

H
True scale Loss = Rs (38000 – 25000) = Rs 13000.
∴ False scale = ×(100 + x )
100 + g 27. Ans. (c)

G
100 × (100 – 10) 37.40
= Sol. S.P. of one article = = Rs 18.70
(100 + 15) 2

=
100 × 90
115
=
1800
23 U Suppose marked price is x
Then with a 15% discount,
85% of x = Rs 18.70
O
= 78.26 cm. 100
25. Ans. (c) ⇒ x = 18.70 × = Rs 22.
85
H
Sol. Let the S.P. of 1 article for C and B be 28. Ans. (b)
Rs x and Rs y respectively. Sol. Let C.P. be x and S.P. be y.
_T

120 x 80 y Under the given terms,


∴ S.P. of 1 article for A = or
100 100 3 (y – x) = 2y – x
120 x 80 y ⇒ 3y – 3x = 2y – x
C

∴ =
100 100 ⇒ 3y – 2y = 3x – x
3 ⇒ y = 2x
PS

⇒ y = x
2 Profit = y – x = 2x – x = Rs x
The man buys goods from A for Rs 9600. x
Profit % = × 100 = 100%.
100 8000 x
∴ Number of goods = 9600 × =
U

120x x 29. Ans. (a)


4000 4000 3 Sol. Considering the ratio, let us suppose
@

Total cost = ×x + × x
the trader bought 2 kg, 4 kg and 3 kg
x x 2
3 of the three varieties of grain.
= Rs 10000  y = x
2 C.P. of (2 + 4 + 3 =) 9 kg
∴ It is less profitable by Rs (10000–9600) = Rs (2 × 50 + 4 × 20 + 3 × 30)
= Rs 400. = Rs (100 + 80 + 90) = Rs 270
S.P. of 9 kg of grain mixture
26. Ans. (a)
= (9 × 33 =) Rs 297
100 Profit Rs (297 – 270) = Rs 27
Sol. C.P. = × S.P.
(100 + Gain %) 27
Profit % = × 100 = 10%.
Under the given terms, if his estimates 270
are correct, items accepted would come 30. Ans. (d)
to 100 – 5 (estimated failure) = 95% of Sol. In such cases, the L.C.M. of the number
2000. of items bought and sold at different
Unit Two : Practice Session ✫ 193

rates may be taken as the total number Discount on marked price of Rs 110 is
of items. Rs (10 + 1) = Rs 11
So, let the total number of items bought 11
= L.C.M. of 6 and 5, which is 30. % Discount = × 100 = 10%.
110
5 33. Ans. (c)
C.P. of 30 articles = × 30 = Rs 25
6 Sol. Let the original price be Rs 100.
6 Then Sahil buys the car for (i.e. C.P. is)
S.P. of 30 articles = × 30 = Rs 36 (Rs 100 – discount of 20 =) Rs 80.
5
Gain = Rs 36 – Rs 25 = Rs 11 S.P. is 140% of Rs 80

TS
11 140
Gain % = × 100 = 44%. = × 80 = Rs 112.
25 100
Gain on 100 = 112 – 100 = 12 or 12%.
31. Ans. (b)

H
Sol. Let’s suppose Arun bought the second 34. Ans. (b)
variety at Rs x per kg. Then
Error

G
C.P. of 60 kg of wheat Sol. Gain % = × 100
= 30 × 17.50 + 30x True value – error
= Rs 525 + 30x 1000 − 800
S.P. of 60 kg of wheat
= 60 × 18.60 = Rs 1116 U Here Gain % =
1000 − Error
× 100
O
200
1116 − (525 + 30 x ) = × 100
Profit % = × 100 = 20 1000 − 200
525 + 30 x
H
(given) 200
591 + 30 x 20 1 = × 100 = 25%.
⇒ = = 800
525 + 30 x 100 5
_T

⇒ 5 (591 + 30x) = 525 + 30x 35. Ans. (a)


⇒ 2955 – 150x = 525 + 30x Sol. Let the second discount be x%.
⇒ 180x = 2955 – 525 = 2430 Then (100 – x)% of 90% of 720 = 550.80
C

(given first discount is 10%)


2430 27
⇒ x = = = 13.50 100 − x 90
180 2 ⇒ ×
PS

× 720 = 550.80
So the cost price of the second variety 100 100
of wheat is Rs 13.50 per kg. 55080
⇒ 100 – x = = 85
32. Ans. (d) 9 × 72
U

Sol. If the C.P. is Rs 100, the marked price ⇒ x = 100 – 85 = 15


is Rs 110, and S.P. is Rs 99. The second discount is 15%.
@
194 ✫ Quantitative Aptitude

Practice Exercise 15
1. How many litres of pure acid is there in 9. The milk sold by a milkman contains 5%
8 litres of a 20% solution? water. What quantity of pure milk should
(a) 1.2 litres (b) 1.4 litres be added to 20 litres so that the water
(c) 1.6 litres (d) 1.8 litres content comes down to 2%?
2. Milk and water are in the ratio of 3 : 2 (a) 50 litres (b) 40 litres
in a mixture of 80 litres. How much water (c) 44 litres (d) 30 litres
should be added so that the ratio of milk 10. Find the ratio in which rice at Rs 7.20 a

TS
and water becomes 2 : 3? kg should be mixed with rice at Rs 5.70
(a) 40 litres (b) 37 litres a kg to produce a mixture worth Rs 6.30
(c) 32 litres (d) 30 litres a kg.
(a) 1 : 2 (b) 2 : 3

H
3. In a mixture of 45 litres, the ratio of milk
and water is 4 : 1. How much water (c) 3 : 4 (d) 4 : 5
must be added to make the mixture ratio

G
11. A dishonest milkman professes to sell his
3 : 2?
milk at cost price but he mixes it with
(a) 10 litres (b) 15 litres
water and thereby gains 25%. The
(c) 20 litres (d) 25 litres
4. The cost of type I rice is Rs 15 per kg U percentage of water in the mixture is
(a) 5% (b) 10%
O
and type II is Rs 20 per kg. If both types (c) 15% (d) 20%
are mixed in the ratio of 2 : 3, then the
12. Two liquids are mixed in the proportion
H
price per kg of the mixed variety of rice
of 3 : 2 and the mixture is sold at Rs 11
is
per litre at a 10% profit. If the Ist liquid
(a) Rs 15.75 (b) Rs 18
_T

costs Rs 2 more per litre than the 2nd


(c) Rs 19.05 (d) Rs 25
liquid, what does it cost per litre?
5. In what ratio must a grocer mix two (a) Rs 10.20 (b) Rs 10.80
varieties of pulses costing Rs 15 and Rs (c) Rs 11 (d) None of these
C

20 per kg respectively so as to get a


13. Pure milk costs Rs 3.60 per litre. A
mixture worth Rs 16.50 per kg?
milkman adds water to 25 litres of pure
PS

(a) 7 : 3 (b) 9 : 1
milk and sells the mixture at Rs 3 per litre.
(c) 12 : 5 (d) 15 : 4
How many litres of water does he add?
6. A merchant has 1000 kg of sugar, part of (a) 4 litres (b) 5 litres
which he sells at 4% profit and the rest,
U

(c) 6 litres (d) 10 litres


at 9% profit. He gains 7% on the whole.
14. How many kg of salt at 42 paise per kg
The quantity sold at 9% profit is
@

must a man mix with 25 kg of salt at 24


(a) 600 kg (b) 500 kg
paise per kg so that he may, on selling
(c) 400 kg (d) 300 kg
the mixture at 40 paise per kg, gain 25%
7. In what ratio must a grocer mix two on the outlay?
varieties of tea worth Rs 60 per kg and (a) 15 kg (b) 20 kg
Rs 66 per kg so that by selling the mixture (c) 25 kg (d) 27 kg
at Rs. 68.20 kg he may gain 10%?
15. One variety of sugar is sold for Rs 3.20
(a) 2 : 1 (b) 3 : 2
a kg at a loss of 20% and another variety
(c) 4 : 3 (d) 5 : 4
is sold for Rs 6 per kg at a gain of 20%.
8. How many kg of pure salt must be added If equal quantities of the two are mixed
to 30 kg of 2% solution of salt and water together and the mixture is sold at Rs 5.40
to increase it to a 10% solution? per kg, what is the loss or gain per cent?
(a) 3 kg (b) 7 kg (a) Gain 15% (b) Loss 20%
(c) 12 kg (d) 15 kg (c) Gain 20% (d) Loss 15%
Unit Two : Practice Session ✫ 195

16. A sum of Rs 39 was divided among 45 of pure copper are melted together. An
boys and girls. Each girl gets 50 paise, alloy was obtained in which the ratio of
whereas a boy gets one rupee. Find the copper to zinc was 3 : 2. Find the weight
number of boys. of the new alloy.
(a) 12 (b) 32 (a) 32 kg (b) 33 kg
(c) 33 (d) 37 (c) 35 kg (d) 42 kg
17. Several litres of acid were drawn off a 22. Two brass alloys are composed as follows:
54 litre vessel full of acid and an equal the first contains 3 parts tin, 20 parts
amount of water added. Again the same copper, and 3 parts zinc. The second alloy

TS
volume of the mixture was drawn off and contains 7 parts tin, 56 parts copper, and
replaced by water. As a result, the vessel 2 parts zinc. These alloys are fused together
contained 24 litres of pure acid. How in equal quantities (by weight). Find the
much of the acid was drawn off initially? ratio of the three ingredients in the

H
(a) 2 litres (b) 9 litres resulting alloy.
(c) 18 litres (d) 90 litres (a) 15 : 117 : 19 (b) 29 : 212 : 19

G
18. If 50% of the 2 : 3 solution of milk and (c) 25 : 27 : 28 (d) 23 : 118 : 27
water is replaced with water, then the 23. As a result of processing, 38 tons of a
concentration of the solution is reduced
by
U second grade material containing 25% of
impurity yields 30 tons of first grade
O
(a) 24% (b) 27% material. What is the percentage of
1 1 impurity in the first grade material?
(c) 33 % (d) 42 %
3 3 (a) 3% (b) 4%
H
19. A dairyman pays Rs 64 per litre of milk. (c) 5% (d) 6%
He adds water and sells the mixture at 24. An ore contains 40% mass impurity, while
_T

Rs 8 per litre, thereby making a profit of the metal extracted from this ore contains
37.5%. The proportion of water to milk 4% impurity. How much metal will 24 tons
received by the customers is in the ratio of ore yield?
(a) 1 : 10 (b) 1 : 12
C

(a) 12 (b) 13
(c) 1 : 16 (d) 1 : 19 (c) 15 (d) 20
PS

20. Tea worth Rs 126 per kg and Rs 135 per 25. In two alloys, copper and zinc are related
kg are mixed with a third variety in the in the ratios 3 : 1 and 2 : 3 respectively.
ratio 1 : 1 : 2. If the mixture is worth After alloying together 12 kg of the first
Rs 153 per kg, the price of the third alloy, 10 kg of the second and a certain
U

variety per kg will be amount of pure zinc, an alloy is obtained


(a) Rs 199.50 (b) Rs 192.50 in which copper and zinc are in equal
@

(c) Rs 185.50 (d) Rs 175.50 proportions. Find the weight of pure zinc
21. In two alloys, copper and zinc are related added.
in the ratios of 4 : 1 and 1 : 3. Ten kg (a) 3 kg (b) 4 kg
of Ist alloy, 16 kg of IInd alloy and some (c) 5 kg (d) 9 kg

Answers with Solutions


1. Ans. (c) Let x litres of water be added in the
20
Sol. 20% of 8 litres, i.e., × 8 = 1.6 litres. mixture so that
100
2. Ans. (a) 48 2
=
Sol. Quantity of milk in the mixture 32 + x 3
= 48 litres ⇒ 2x + 64 = 144
Quantity of water in the mixture
= 32 litres ⇒ x = 40.
196 ✫ Quantitative Aptitude

3. Ans. (b) 6. Ans. (a)


Sol. Milk = 36 litres Sol. By alligation rule,
Water = 9 litres
Profit on Ist part Profit on 2nd part
To make the ratio of milk : water 3 : 2
Let x litres of water be added in the
mixture so that
36 3
=
9+x 2

TS
27 + 3x = 72 ∴ Ratio of 1st and 2nd parts = 2 : 3
x = 15. 3
∴ Quantity of 2nd kind = ×1000 kg
4. Ans. (b) 5
= 600 kg.

H
Sol. Let the mean price be Rs x.
7. Ans. (a)
By alligation rule,

G
Sol. C.P. of 1 kg of mixture
Cost of Type I Cost of Type II 100
=Rs × 68.20 =Rs 62
Rs 15

Mean price
Rs 20

U 110
By alligation rule,
O
Rs x
(d – m ) (m – c )
H
= 20 – x = x – 15
20 – x 2
_T

∴ =
x – 15 3

60 – 3x = 2x – 30 ∴ Required ratio = 4 : 2 or 2 : 1.
C

⇒ 5x = 90 8. Ans. (a)
Sol. Let x kg of salt be added to the solution
⇒ x = 18
PS

2 10
Then × 30 + x = × (30 + x )
The price per kg of the mixed variety 100 100
of rice = Rs 18.
⇒ 0.6 + x = 0.1 × (30 + x)
U

5. Ans. (a) ⇒ 6 + 10x = 30 + x


Sol. By alligation rule, ⇒ 9x = 24

@

x = 2.66 kg
= 3 kg (approximate).
9. Ans. (d)
Sol. Let the quantity of pure milk added to
the mixture be x.
We have one litre water and 19 litres
milk then
∴ 1 = 2% of 20 + x
2
1 = (20 + x )
100
3.50 35 7
∴ Required ratio = = = 100 = 40 + 2x
1.50 15 3
⇒ 2x = 60
= 7 : 3.
⇒ x = 30.
Unit Two : Practice Session ✫ 197

10. Ans. (b) adds 1 litre of water, and for every 25


Sol. By alligation rule, litres of milk, he adds 5 litres of water.
14. Ans. (b) 100
Sol. Cost price of mixture = 40 × (100 + 25)
= 32 paise
32 – 24 8 4
⇒ = = [By allegation rule]
42 – 32 10 5
4
∴ Required quantity = × 25 = 20 kg.

TS
5
0.60 60 2 15. Ans. (c)
∴ Required ratio = = = 3.2
+
6.0
0.90 90 3
Sol. Cost price of mixture = 0.8 1.2
= 2 : 3. 2

H
= Rs 4.50 per kg
11. Ans. (d)
5.4 – 4.5

G
100 Gain % = × 100 = 20%.
Sol. C.P. of 1 litre mixture = Re ×1 4.5
125 16. Ans. (c)
= Re
4
5
U Sol. Average amount of money received by
each
O
By alligation rule, 39 13
= = Rs
45 15
Amount received by each girl
H
1
= 50 paise = Rs
2
_T

Amount received by each boy = Re 1


13 1
4 1 Number of boys – 11
∴ Ratio of milk to water = : =4:1 = 15 2 =
C

5 5 Number of girls 13 4
1–
Hence, percentage of water in the mixture 15
PS

1 11
= ×100 % = 20%. ∴ Number of boys = × 45 = 33 .
5 11 + 4
12. Ans. (b) 17. Ans. (c)
U

Sol. S.P. of the mixture = Rs 11 Sol. We have x = 54, y = l and n = 2


Since the mixture is sold at 10% profit, n
y
@

C.P. of the mixture = Rs 10 x 1–


∴ Using the formula x
Let the C.P. of the 1st liquid be Rs x per
litre l
2

Then the C.P. of the 2nd liquid 54 1 – = 24


54
= Rs (x – 2) per litre
3 10 – ( x – 2) 54 ×(54 – l)2
⇒ = = 24
2 x – 10 54 × 54
x = Rs 10.80 per litre. 2916 – 108l + l2= 1296
13. Ans. (b) ⇒ l2 – 108l + 1620 = 0
Sol. Cost of water = 0 ⇒ (l – 90)(l – 18) = 0
Pure milk 3.00 – 0 3 5 ⇒ l = 90 or 18
So, = = =
Water 3.6 – 3.0 0.6 1 Since 90 > 54,
∴ For every 5 litres of milk, the milkman l = 18 is considered.
198 ✫ Quantitative Aptitude

18. Ans. (c) x – 153


Sol. Let the quantity of milk in the solution ∴ =1
22.50
be 20 litres ⇒ x – 153 = 22.50
Let the quantity of water in the solution
⇒ x = Rs 175.50 per kg.
be 30 litres
50% of the solution (10 litres milk + 15 21. Ans. (c)
litre water) is replaced with water Sol. Let the amount of pure copper be x kg
Now the quantity of milk becomes 10 ∴ Pure copper + copper in Ist alloy
litres and water becomes 40 litres + copper in IInd alloy

TS
(15 + 25) = copper in IIIrd alloy
The concentration of solution is
4 1 3
reduced by 50%, i.e., from 20 to 10. The ⇒ x+ ×10 + × 16 = (10 + 16 + x )
dilution of the solution increased from 5 4 5

H
1 x = 9 kg
30 to 40, i.e., increased by 33 % . ∴ The weight of new alloy
3

G
19. Ans. (a) = 10 + 16 + 9 = 35 kg.
Sol. Let the quantity of milk purchased be
= x litres and the quantity of water mixed 22. Ans. (b)
= y litres.
The ratio of water and milk in the U Sol. Tin : Copper : Zinc

=
3
+
7
:
20
+
56
:
3
+
2
O
mixture = y : x. C.P. of milk is Rs. 64 per 26 65 26 65 26 65
litre and S.P. is Rs 8 per litre. = 29 : 212 : 19.
H
S.P. ×100
 C.P. = 23. Ans. (c)
100 + Gain%
Sol. Let the percentage of purity of first
_T

8( x + y) ×100
64x = grade be x
100 + 37.5
⇒ 8800x = (8x + 8y) × 100 ∴ % purity of first grade × Amount of first
grade

C

8800x = 800x + 800y


⇒ 8000x = 800y = % purity in second grade material
x 800 1 × Amount of second grade material
PS

⇒ y = = ⇒ x × 30 = 75% × 38
8000 10
⇒ x : y = 1 : 10. ⇒ x = 95%
20. Ans. (d) ∴ % impurity in first grade material
U

Sol. Average price of the first and the second = 100 – 95 = 5%.
variety of tea
@

126 + 135 24. Ans. (c)


= Sol. Consider the pure portion
2
= Rs 130.50 per kg ∴ 60% × 24 = 96% × x
Let the cost of the third variety be x. ⇒ x = 15.
By alligation rule,
25. Ans. (b)
Cost of 1 kg tea of Sol. Let the amount of pure zinc added be
mixture of first and Cost of 1 kg tea of x kg.
second variety Rs 130.50 the third variety Rs x
3 2
× 12 + × 10
Copper 4 5 1
∴ = =
Zinc 1 3 1
×12 + ×10 + x
4 5
x – 153 22.50
x = 4 kg.
Unit Two : Practice Session ✫ 199

Practice Exercise 16
1. The ratio between the present age of P 4 : 5 : 7, then the age of the youngest
and Q is 6 : 7. If Q is 4 years older than boy is
P, what will be the ratio of their age after (a) 1 year (b) 12 years
4 years? (c) 16 years (d) 20 years
(a) 7 : 8 (b) 6 : 7 9. Ten years ago, A was half of B in age.
(c) 5 : 6 (d) 4 : 5 If the ratio of their present age is 3 : 4,
what is the sum of their present age?

TS
2. The sum of the age of five children each
(a) 70 years (b) 64 years
born at 3 years’ interval is 50 years. What
(c) 44 years (d) 35 years
is the age of the youngest child?
(a) 10 years (b) 8 years 10. Q is as much younger than R as he is older

H
(c) 4 years (d) None of these than T. If the sum of the age of R and
T is 50 years, how much age difference
3. A is two years older than B who is twice
is there between R and Q ?

G
as old as C. If the total of the age of A,
(a) 3 years (b) 7 years
B and C is 27, then how old is B?
(c) 12 years (d) Data inadequate
(a) 7
(c) 9
(b) 8
(d) 10
U 11. The ratio of Prerna’s age to the age of
her mother is 4 : 9. The difference in their
O
4. A person was asked to state his age in
age is 20 years. The ratio of their age after
years. His reply was, “Take my age three
4 years will be
years hence, multiply it by 3 and then
H
(a) 2 : 1 (b) 3 : 2
subtract three times my age three years
(c) 1 : 2 (d) 2 : 3
ago and you will know how old I am.”
12. Three years ago, X’s age was double Y’s
_T

What was the age of the person?


age. Seven years hence, the sum of their
(a) 12 years (b) 18 years united age will be 83 years. How old is
(c) 16 years (d) 14 years X at present?
C

5. Deepika’s father was 38 years when she (a) 40 years (b) 41 years
was born, while her mother was 36 years (c) 43 years (d) 45 years
PS

old when Deepika’s brother, four years 13. A person’s present age is two-fifth the age
younger to her, was born. What is the of his mother. After 8 years, he will be
difference between the age of her parents? one-half the age of his mother. How old
(a) 1 year (b) 3 years is his mother at present?
U

(c) 6 years (d) 8 years (a) 40 years (b) 35 years


6. Eighteen years ago, a father was three (c) 30 years (d) 25 years
@

times as old as his son. Now the father 14. The sum of the age of a son and a father
is twice as old as his son. The sum of the is 56 years. After four years, the age of
present age of the son and the father is the father will be three times that of the
(a) 108 (b) 100 son. Find the father’s age.
(c) 91 (d) 84 (a) 37 years (b) 40 years
7. The average age of the mother and her (c) 44 years (d) 46 years
6 children is 12 years which is reduced 15. A mother after 4 years will be twice the
by 5 years if the age of the mother is age of her daughter. The sum of their age
excluded. How old is the mother? is 46. What is the daughter’s present age?
(a) 42 years (b) 39 years (a) 10 years (b) 11 years
(c) 61 years (d) 54 years (c) 14 years (d) 19 years
8. The average age of three boys is 16 years. 16. A demographic survey of 100 families in
If the age of the children is in the ratio which two parents were present revealed
200 ✫ Quantitative Aptitude

that the average age A, of the oldest child, 21. Z’s age is twice the average age of X, Y
1 and Z. X’s age is one half of the average
is 20 years less than the sum of the
2 of X, Y and Z. If Y is 5 years old, what
age of one parent F and the age of the is the average age of X, Y and Z?
other parent, M. Which of the following (a) 12 years (b) 11 years
is equivalent to A? (c) 10 years (d) 9 years
F+M F+M 22. A father said to his son, “I was as old
(a) + 20 (b) – 20
2 2 as you are at present at the time of your
F + M + 20 F + M – 20 birth.” If the father’s present age is 38

TS
(c) (d) years, then the son’s age five years back
2 2
must have been
17. The ratio of the present age of two (a) 27 years (b) 25 years
brothers is 1 : 2 and the ratio 5 years back (c) 19 years (d) 14 years

H
was 1 : 3. What will be the ratio of their
age after 5 years? 23. The sum of the age of a mother and a
(a) 1 : 4 (b) 2 : 3 daughter is 50 years. 5 years ago, the

G
(c) 3 : 5 (d) 5 : 6 mother ’s age was 7 times the age of the
daughter. What is the present age of the
18. Shaurya’s mother was four times as old
as Shaurya ten years ago. After 10 years,
she will be twice as old as Shaurya. How
U mother and daughter, respectively?
(a) 48 and 2 years
(b) 45 and 5 years
O
old is the mother today?
(c) 43 and 7 years
(a) 32 years (b) 50 years
(d) 40 and 10 years
(c) 43 years (d) 71 years
H
24. Mihir is as much younger than Monali as
19. The sum of the present age of a father
he is older than Meenal. If the sum of the
and his son is 60 years. Six years ago, the
_T

father’s age was five times the age of the age of Monali and Meenal is 48 years, how
son. After 6 years, the son’s age will be old is Mihir?
(a) 20 years (b) 18 years (a) 24 years (b) 12 years
(c) 14 years (d) 12 years (c) 27 years (d) 32 years
C

20. Present age of X and Y are in the ratio 25. Tanya’s grandfather was 8 times older
than her 16 years ago. He would be 3
PS

5 : 6 respectively. Seven years hence this


ratio will be 6 : 7 respectively. What is times her age 8 years from now. Eight
X’s present age in years? years ago, what was the ratio of their age?
(a) 22 (b) 28 (a) 1 : 2 (b) 1 : 5
U

(c) 35 (d) 43 (c) 3 : 8 (d) None of these

Answers with Solutions


@

1. Ans. (a) 2. Ans. c)


Sol. Let P’s age be 6x, then Q’s age = 7x. Sol. Let the age of the children be x, x + 3,
Given that 7x = 6x + 4 x + 6, x + 9 and x + 12
⇒ x =4 By the given condition, we get
∴ P’s present age = 6×4 = 24 and x + x + 3 + x + 6 + x + 9 + x + 12 = 50
Q’s present age = 7×4 = 28 ⇒ 5x + 30 = 50
After 4 years P’s age would be
⇒ 5x = 20
24 + 4 = 28 years
⇒ x =4
and Q’s age would be 28 + 4 = 32 years
∴ The age of youngest child is 4 years.
P 28 7
∴ = = 3. Ans. (d)
Q 32 8
Sol. Let C’s age be x years, B’s age be 2x
P : Q = 7 : 8. years and A’s age be 2x + 2 years
Unit Two : Practice Session ✫ 201

A+B+C = 27 ⇒ 6x – 20 = 4x – 10
2x + 2 + 2x + x = 27 ⇒ 2x = 10
⇒ x = 5
⇒ 5x = 25
∴ Sum of their present age would be
⇒ x = 5 3 × 5 + 4 × 5 = 15 + 20 = 35 years.
∴ B’s age is 2 × 5 = 10 years. 10. Ans. (d)
4. Ans. (b) Sol. R–Q = Q–T
Sol. Let the present age of the person be ⇒ 2Q = T + R
x years. Then, Also, R + T = 50
3 (x + 3) – 3 (x – 3) = x ⇒ Q = 50

TS
⇒ (3x + 9) – (3x – 9) = x We cannot solve this problem, because
⇒ x = 18 years. of inadequate data.

5. Ans. (c) 11. Ans. (c)

H
Sol. Mother’s age when Deepika’s brother Sol. Let the age of Prerna and her mother
be 4x and 9x, respectively
was born = 36 years.

G
Also, given that
Father ’s age when Deepika’s brother 9x – 4x = 20
was born = 38 + 4 = 42 years ⇒ 5x = 20
∴ The required difference
= 42 – 36 = 6 years.
U ⇒ x =4
∴ Prerna’s age = 4 × 4 = 16 years and her
mother’s age = 9 × 4 = 36 years
O
6. Ans. (a)
Sol. Let the present age of father be 2x ∴ After 4 years, the ratio of their ages
years. Then the present age of son is x
H
16 + 4 20 1
years. = = = .
36 + 4 40 2
∴ 2x – 18 = 3(x – 18)
_T

12. Ans. (d)


⇒ x = 36
Sol. X – 3 = 2 (Y – 3) ...(i)
∴ Required sum = 2x + x and (X + 7) + (Y + 7) = 83 ...(ii)
= 2 × 36 + 36 By simplifying and solving equations (i)
C

= 72 + 36 and (ii), we get


= 108. X = 45 years and Y = 24 years.
PS

7. Ans. (a) 13. Ans. (a)


Sol. Total age of mother and 6 children Sol. Let the mother’s age be x years. Then,
= 12 × 7 = 84 years 2
the person’s present age = x years
Total age of children 5
U

2 1
= 7 × 6 = 42 years ∴ x + 8 = ( x + 8)
5 2
Mother’s age = 84 – 42 = 42 years
@

∴ Mother’s age is 42 years. ⇒ 2(2x + 40) = 5(x + 8)


8. Ans. (b) ⇒ 4x + 80 = 5x + 40
Sol. Let the age of the three boys be 4x, 5x ⇒ x = 40.
Shortcut
and 7x, respectively. 2 1
4x + 5x + 7x = 48 Here, n1 = , t = 8 and n2 =
5 2
∴ 16x = 48
n2 – 1
⇒ x =3 Mother’s age = × t years
∴ Age of the youngest boy n1 – n2
= 4 × 3 = 12 years. 1
–1
9. Ans. (d)
= 2 × 8
Sol. Let their present age be 3x and 4x. 2 1

Then, 5 2
2 (3x – 10) = 4x – 10 = 40 years.
202 ✫ Quantitative Aptitude

14. Ans. (c) 19. Ans. (a)


Sol. We have S = 56, n = 3 and t = 4 Sol. Let the son’s age be x and the father’s
Sn + t ( n – 1) age be y
Using the formula From the given conditions, we get
n +1
x + y = 60 ...(i)
56 × 3 + 4(3 – 1)
Father’s age = and y – 6 = 5(x – 6)
3 +1
176 ⇒ y – 6 = 5x – 30
=
4
= 44 years. ⇒ 5x – y = 24 ...(ii)
15. Ans. (c) Adding equations (i) and (ii), we get

TS
Sol. Let the mother ’s age be x years and the x + y + 5x – y = 60 + 24
daughter’s age be y years. 6x = 84
x = 14
(x + y) = 46 ...(i)
∴ After 6 years, the son’s age will be 14 + 6

H
and x + 4 = 2(y + 4) ...(ii)
= 20 years.
By solving equations (i) and (ii), we get

G
x = 32 and y = 14. Alternative Method
Shortcut S + t ( n – 1)
Son’s present age = years
S = 46, t = 4, n = 2
Daughter’s present age U n+1
[Here, S = 60, t = 6, n = 5]
O
S – t (n – 1) 60 + 6 (5 – 1)
= = = 14 years
n+1 5 +1
H
46 – 4 (2 – 1) After 6 years, son’s age
= = 14 + 6 = 20 years.
2+1
_T

46 – 4 20. Ans. (c)


= = 14 years. Sol. Let the present ages of X and Y be
3
16. Ans. (b) 5x and 6x years respectively. Then,
C

F+M 5x + 7 6
Sol. A = – 20 . =
2 6x + 7 7
PS

17. Ans. (c) 7 (5x + 7) = 6 (6x + 7)


Sol. Let the present age of the two brothers 35x + 49 = 36x +42
be x and 2x years, respectively. Then, ⇒ x =7
x–5 1 ∴ Present age of X is 5 × 7 = 35 years.
U

=
2x – 5 3 Shortcut
⇒ a = 5, b = 6, t = 7, c = 6, d = 7
@

3(x – 5) = (2x – 5)
at (c – d)
⇒ x = 10 X’s present age =
ad – bc
After 5 years, required ratio 5 × 7 (6 – 7)
=
x+5 15 3 5 ×7 – 6 × 6
= = = or 3 : 5. = 35 years.
2x + 5 25 5
18. Ans. (b) 21. Ans. (c)
Sol. Let Shaurya’s age be x years and her X+Y+Z
mother’s age be y years Sol. Z = 2 ...(i)
3
(y – 10) = 4(x – 10) ...(i) ⇒ 3Z = 2X + 2Y + 2Z ...(ii)
and (y + 10) = 2(x + 10) ...(ii) Z = 2X + 2Y
By solving (i) and (ii), we get
1 X+Y+Z
x = 20 years and y = 50 years. and X =
2 3
Unit Two : Practice Session ✫ 203

⇒ 6X = X + Y + Z (Putting value of Z) ∴ Daughter’s age = 10 years


6X = X + Y + 2X + 2Y and mother’s age = 50 – 10 = 40 years.
⇒ 6X = 3X + 3Y (Given Y = 5) 24. Ans. (a)
⇒ 3X =3×5 Sol. Monali – Mihir = Mihir – Meenal
⇒ X =5 ⇒ 2 Mihir = Monali + Meenal
∴ Z = 2X + 2Y ⇒ 2 Mihir = 48 years
⇒ Mihir = 24 years.
=2×5+2×5
25. Ans. (d)
= 20
Sol. 16 years ago, let Tanya’s age be x years
∴ Average age of X, Y And Z

TS
5 + 5 + 20 and that of the grandfather be 8x years.
= After 8 years, Tanya becomes x + 16 + 8
3
30 years and the grandfather would be
= = 10 years. 8x + 16 + 8 years.

H
3
22. Ans. (d) ∴ 8x + 24 = 3(x + 24)
Sol. Let the son’s age be x years. Then, ⇒ 5 x = 48

G
38 – x = x x+8
⇒ 2x = 38 Eight years ago, their age =
8x + 8
⇒ x = 19
∴ Son’s age 5 years back = 19 – 5 years
= 14 years.U 48
5
+8
O
23. Ans. (d) = 48
8× +8
Sol. Let the daughter’s age be x years and 5
H
mother ’s age be 50 – x years 88 11
∴ 7 (x – 5) = (50 – x – 5) = =
⇒ 7x – 35 = 45 – x 424 53
_T

⇒ 8x = 80 ∴ The ratio of their age eight years ago


⇒ x = 10 was 11 : 53.
C
PS
U
@
204 ✫ Quantitative Aptitude

UNIT THREE

TS
✫ UNITARY METHOD ✫ CHAIN RULE ✫ TIME AND WORK
✫ PIPES AND CISTERNS ✫ TIME AND DISTANCE AND
MOVING VEHICLES ✫ BOATS AND STREAMS ✫ RACES AND GAMES

H
FUNDAMENTALS AND

G
FAST-TRACK FORMULAE
U
O
Unitary Method Most of the time we skip the ‘reduction to
unit’ step and go straight to the last step. So,
The method in which the value of a unit is first
H
for example, if asked how much loss will be
found is called unitary method. Unitary method
made on a consignment of 20 boxes if the loss
is applied for solving many problems, such as
_T

on 5 boxes is Rs 250, one would straightaway


percentages problems, ratio and proportion calculate
problems, profit and loss problems, time and
250
work problems, simple interest and compound × 20 = Rs 1000.
5
C

interest problems. This method is used in many problems to


Unitary method is also called method of reach the answer quickly.
PS

reduction to the unit. This we can explain more


easily with the help of an example.
If you are given the cost of 20 pencils as Chain Rule
U

Rs 25 and you have to find the cost of 5 pencils, A series of quantities may be connected to one
you would first find the cost of 1 pencil. another and we may want to know how much
@

20 pencils cost Rs 25 of one quantity is equivalent to a certain


25 quantity of another and so on. Chain rule helps
1 pencil cost Rs us to find how much of the last kind in the
20
25 series is equivalent to a certain quantity of the
5 pencils cost × 5 = Rs 6.25. first kind. Chain rule defines the repeated use
20
Suppose a woman earns Rs 480 in 2 weeks. of ‘rule of three’ in which the fourth proportional
How long will she take to earn Rs 1440? is found out when the other three are given.
In solving problems by chain rule, we compare
The woman earns Rs 480 in 2 weeks. every quantity with the quantity or term that
2
She will earn Re 1 in weeks is to be found out.
480
2 Variation and Proportion
She will earn Rs 1440 in × 1440 Direct Proportion If two quantities (or
480
variables) x and y are such that their ratio is
= 6 weeks.

204
Unit Three : Fundamentals / Chain Rule ✫ 205

always constant, then y is said to vary directly ∴ 10 : 16 : : 35 : x


as x, or to be in direct proportion to x. This 10 35
is expressed as =
16 x
y ∝ x or y = xk 16 × 35
x = = Rs 56
(where k is a constant) 10
In other words, two quantities are said to The cost of 16 books is Rs 56.
be directly proportional when on one quantity Indirect Proportion If two quantities x and
increasing or decreasing, the other quantity y are such that y is proportional to the

TS
also increases or decreases correspondingly. reciprocal of x, then y is said to vary inversely
Distance covered is directly proportional to as x or to be in inverse or indirect proportion
the time taken to cover the distance, i.e., the to x. This is expressed as
more distance to be covered, more is the time 1 k

H
required to cover the distance (given speed y or y =
x x
remains the same). (where k is a constant)

G
Money spent on purchasing is directly Two quantities are said to be indirectly
proportional to the number of articles purchased. proportional if one quantity increases, the other
Work done is directly proportional to the
number of workers, provided their rate of
work is the same. U quantity decreases and vice versa.
Speed of a vehicle is indirectly (inversely)
proportional to the time taken to cover a
O
distance.
● If 12 oranges cost Rs 36, what will 40 The faster the pace of work, less the time
oranges cost?
H
required to complete the work.
Here, the unitary method can be applied Most problems on time and work, time and
as more items mean more cost. So, distance, and pipes and cisterns are based on
_T

12 oranges cost Rs 36 these principles.


36
∴ 40 oranges cost × 40 = Rs 120 ● If 18 men can do a piece of work in 25
12 days, in how many days will 5 men do
C

Applying the method for direct it?


proportion, we have
PS

Here we have men and work and days.


Oranges Cost
More men means faster work and fewer
12 36
Direct proportion days; fewer men means slower work
40 x and more days. So there is an indirect
U

(x being the cost of 40 oranges) proportion between men and days here.
12 : 40 : : 36 : x Men Days
@

12 36 18 25
= 5 x
40 x
40 × 36 (x being the days 5 men would take)
x = = Rs 120 18 : 5 : : x : 25
12
18 x
● If 10 books cost Rs 35, what would be =
5 25
the cost of 16 books?
18
Items and cost are directly proportional, x = × 25 = 90 days.
so, both arrows point in the same 5
So, 5 men would take 90 days to do
direction. If one increases, the other
the given work.
item also increases.
Books Rupees ● If 27 kg of wheat feeds 42 men for 21
10 35 days, how many days would 36 kg of
16 x wheat feed 21 men?
206 ✫ Quantitative Aptitude

Here we have a mixed proportion. Then,


More wheat, more days (direct) (a) In case of direct proportion
Fewer men, more days (indirect) N1 × T1 × R1 × E1 ∝ W1
If x is the number of days to be found, and N2 × T2 × R2 × E2 ∝ W2
Wheat 27 : 36
Men 21 : 42 } : : 21 : x N 1 × T1 × R 1
W1
N × T2 × R 2
= 2
W2
27 21 21 If W1 = W2, we have
: ::
36 42 x N1 × T1 = N2 × T2

TS
27 × 21 × x = 36 × 42 × 21 N1 × T1 × R1 = N2 × T2 × R2
36 × 42 × 21 N1 × T1 × R1 × E1 = N2 × T2 × R2 × E2
x = = 56 days
27 × 21 (b) In case of indirect proportion

H
Hence 36 kg of wheat would feed 21
1
men for 56 days. N1 ∝

G
T1
Points to Remember and N1 × T1 = 1 (or the work done)
Fast-Track Formulae
U Worked Examples
O
There are some basic formulae that are
● If 12 men do a piece of work in 45 days,
applicable whether the problem is about
in how many days will 27 men do it.
men and work, money or food and
H
consumption, or engines and fuel Here we have more men, less days (indirect
consumption rate. variation)
_T

(i) The unitary method is applicable in Men Days


direct proportion problems. 12 45
(ii) (a) If increase or decrease in amount 27 x
A1 leads to increase or decrease in ∴ 12 : 27 : : x : 45
C

another amount A2, 12 × 45


A1 ∝ A2 (direct proportion). ∴ x = = 20 days
PS

27
(b) If increase in amount A1 leads to It will take 20 days for 27 men to complete
decrease in amount A2, the work.
1
A1 ∝
U

(indirect proportion) An alternative method would be to apply


A2 the formula N1 × T1 = N2 × T2, as work
(iii) Given, in the cases to be compared, is the same in both cases.
@

Number of men/machines/items
= N1 and N2 We have N1 = 12, T1 = 45, N2 = 27, T2 = ?
Time for work to be completed or So,
something to be consumed 12 × 45 = 27 × T2
= T1 and T2 12 × 45
Rate of work/wages/consumption = T2 = 20 days.
27
= R1 and R2
(Rate being amount/person, time or item) ● If a piece of work could be completed in
24 days by 98 men, how many men would
Efficiency (taken to be 1, unless be required to complete the work in 21
otherwise stated) days?
= E1 and E2
Work (of the same nature) N1 × T1 = N2 × T2
= W1 and W2 Here, N1 = 98, T1 = 24, N2 = ? T2 = 21
Unit Three : Fundamentals / Chain Rule ✫ 207

∴ 98 × 24 = N2 × 21 Here the proportion is inverse: more


98 × 24 workers, fewer days; fewer workers, more
= N2 = 112 men. days.
21
Women Days
● If the wages of 5 men for 12 days are 12 25
Rs 600, what would be the wages of 6 men 20 x
for 20 days? ∴ 12 : 20 :: x : 25
Here, more men and more days mean more 12 × 25
wages. So it is a matter of direct proportion. x = = 15 days
20

TS
Men Days Wages 20 women or 6 men and 11 women can
5 12 600 do the work in 15 days.
6 20 x Alternatively, once we have the fact that

H
we have to find out how many days 20
Men 5 : 6
Days 12 : 20 } : : 600 : x women will take to do the same job, we

G
can apply the formula
5 12 600
× = N1 × T1 = N2 × T2
6 20 x
5 × 12 × x = 600 × 6 × 20
U 12 × 25 = 20 × x
(x being the time to be found)
O
600 × 6 × 20
x = = Rs 1200 12 × 25
5 × 12 x = = 15 days
20
H
The wages of 6 men will be Rs 1200 for
20 days. So, 20 women can do the job in 15 days.
_T

Alternatively, we can apply the formula ● Some men promised to do a job in 18 days,
N1 × T1 N 2 × T2 but 6 of them were absent from work. The
= remaining men completed the job in 20
W1 W2
C

days. What was the original number of


Now N1 = 5 men, T1 = 12 days, W1 = Rs 600, men?
N2 = 6 men, T2 = 20 days, W2 = ?
PS

So we have, Here, the data should be broken down to


such figures where the formula
5 × 12 6 × 20
= = N1 × T2 = N2 × T2 becomes applicable.
600 W2
U

We have to find N1:


6 × 20 × 600 N 2 = N1 – 6
W2 = = Rs 1200.
5 × 12 T 1 = 18 days
@

● If 8 men or 12 women can do a piece of T 2 = 20 days


work in 25 days, in how many days can W1 = W2
the work be done by 6 men and 11 women So, we have
working together? N1 × 18 = (N1 – 6) × 20
Given that, 8 men = 12 women 18N1 = 20N1 – 120
2 men = 3 women
2N1 = 120
or 6 men = 9 women
∴ 6 men + 11 women = 9 women + 11 women 120
= 20 women N1 = = 60
2
Now we can put the question in the form, The original number of men were 60.
that if 12 women can do a job in 25 days, [If you are asked how many men did the
then 20 women can do the same job in how work in 20 days, the number would be
many days?’ 60 – 6 = 54.]
208 ✫ Quantitative Aptitude

● If Rs 1600 maintains a family of 8 persons men arrives and it is found that the
for 4 days, for how long would Rs 2100 provisions will last for 10 days more if
maintain a family of 12 persons? rations are provided at 840 g per head.
What is the strength of the reinforcement?
Here the money consumption will be
indirectly proportional to the number of Let the number of men in the reinforcement
persons and days. be x. Provisions left are for (25 – 11 =) 14
Using the formula days as x men come after 11 days.
N1 × T1 N 2 × T2 Now,
= N 1 = 3000

TS
W1 W2
We have N1 = 8 persons, N2 = 12 persons T 1 = 14 days
W1 = Rs 1600, W2 = Rs 2100 R1 = 900 g/head
(whether it is wages on money consumed N 2 = 3000 + x

H
or work, we use W1, W2 etc.) T 2 = 10 days (given)
T1 = 4 days R2 = 840 g/head

G
T2 = ? So we use the formula
So we have N1 × T1 × R1 = N2 × T2 × R2
8×4
1600
=
12 × T2
2100 U 3000 × 14 × 900 = (3000 + x) × 10 × 840
3000 × 14 × 900
O
3000 + x =
8 × 4 2100 10 × 840
= × = T2
1600 12 3000 × 14 × 900
H
T2 = 3.5 days x = – 3000
10 × 840
Rs 2100 will feed 12 persons for 3.5 days. = 4500 – 3000 = 1500
_T

● Ten men working 6 hours a day can The reinforcement comprised 1500 men.
complete a piece of work in 18 days. How
many hours a day must 15 men work to
C

complete the same work in 12 days? Time and Work


Something that one does as part of one’s
Here we are required to find the rate of
PS

regular work may consist of many small tasks


work in the second case.
or a large one. But the work is always to be
N1 = 10, N2 = 15 completed in a given period of time. To
T1 = 18, T2 = 12 complete the given task earlier one has to
U

R1 = 6, R2 = ? increase the number of persons engaged in


We may ignore W as work is the same in doing that task or vice versa. While dealing
@

both cases. So, with problems on wages, it may be kept in


N1 × T1 × R1 = N2 × T2 × R2 mind that the money obtained is always divided
in the ratio of the work done by each person
10 × 18 × 6 = 15 × 12 × R2 and for how much time.
10 × 18 × 6 In work problems there are three items
= R2 involved: the number of people working, the
15 × 12
R2 = 6 hours a day. time, and the amount of work done.
So, if 15 men are to complete the same work ● The number of people working is
in 12 days, they too have to work for 6 directly proportional to the amount of
hours a day. work done, i.e., the more people on
the job, the more the work that will
● A garrison of 3000 men has provisions for be done, and vice versa.
25 days when rations are at 900 g per head. ● The number of people working is
At the end of 11 days, a reinforcement of inversely proportional to the time, i.e.,
Unit Three : Fundamentals / Time and Work ✫ 209

the more people on the job, the less 12 2


Therefore, it will take them or 2
time it will take to finish the job, and 5 5
vice versa. days to complete the job working
● The time spent on the work is directly together.
proportional to the amount of work (vi) Part of the work done at any time t by
done; i.e., more time spent, more one or more persons = t × 1 day’s work.
work done, and vice versa.
● Ravi and Uday can do a work alone
Points to Remember in 8 days and 12 days, respectively.
How much of the work can be done

TS
(i) Total work assigned is always taken to be
1, unless specifically mentioned otherwise. by them in 4 days?
(ii) If A can do a piece of work in n days, Part of the work done by Ravi and
1 Uday together in 4 days
then A’s 1 day’s work = .

H
n = 4 × (Ravi and Uday)’s 1 day work
1 1 1
(iii) If A’s 1 day’s work = , then A can finish =4× + of the work

G
n 8 12
the work in n days.
5 5
1 =4× = of the work
(iv) 1 day’s work for any person is
n
part of
the total work, n being the time taken by
U 24 6
Ravi and Uday would be able to do
5
O
the person to finish the work alone. [The 6
reciprocal of 1 day’s work gives the time of the work in 4 days.
a person would take to complete the work
H
(vii) If more than one person are working for
by himself/herself.]
different time periods to complete a job,
2
_T

● Seema can do of a job in a day. then take the total time for completion of
3
3 1 the job to be T and find out days worked
She would take or 1 days to do by each person with reference to T (if it
2 2
the complete work by herself. is not mentioned in the problem). Then
C

find out the part of work done by each


● Suresh can paint a house in 8 days.
person; the sum of these parts will be
PS

1
In one day he can paint of the house. equal to the work = 1 (as the job is
8
(v) If more than one person is working on the complete). By now you will have the
same piece of work, their combined work known quantities and can find out the
U

in one day will be equal to the sum of unknown.


each one’s one day work.
● Rashid and Suresh can do a painting job
@

● If A takes 4 days to dig a ditch whereas in 20 days and 30 days respectively.


B takes 8 days and C, 24 days for the They start working together, but Rashid
same work, how long would it take for leaves 5 days before the work is
all three to do the job?
completed. Suresh works alone to
A can do the work in 4 days or he can complete the job. How long will the
1 entire painting job take?
do of the work in 1 day.
4
1 Let’s take the total time for the work
Similarly, in one day B can do and
8 to be done as T days.
1 As Rashid leaves 5 days before the work
C, of the work. Together in one day
24 is completed, he works for T – 5 days.
1 1 1 10 5
A, B and C can do + + = = Suresh works for T days
4 8 24 24 12
of the work. After work is finished,
210 ✫ Quantitative Aptitude

Rashid’s work and Suresh’s work = 1 ● Three workers can do a job in 16 days.
Two of them work twice as fast as the
T −5 T
+ = 1 third. How long would it take for one
20 30 of the faster workers to do the job
3T − 15 + 2T himself?
= 1
60
If there are 2 fast workers and one slow
5T = 60 + 15 = 75 worker, by the terms given, there are
75 actually 5 slow workers working at
T = = 15 days equal rates.
5

TS
So, the entire painting job will take 15 1 slow worker will take 16 × 5 or 80 days.
days to complete. 1 fast worker = 2 times slow workers :
80
● Four men working together can dig a therefore, he will take or 40 days to
2

H
ditch in 56 days. They begin but one finish the job.
man works only half-days. How long

G
will it take to complete the job? Fast-Track Formulae with Worked
It would take 1 man 56 × 4 = 224 days Examples
to complete the work alone.
As one man works half-days and 3
1 U (i) If A can do a piece of work in x
days and B can do it in y days then A and
O
work full days, we may say 3 men are
2 B working together will do the same work
working. xy
H
1
in days.
x+y
For 3 men the job would take
2 ● If A can do a piece of work in 12 days
_T

1
224 ÷ 3 = 64 days. and B can do it in 15 days, then in how
2
many days can A and B working
● A stenographer has been asked to fill together do it?
600 forms. She fills 30 forms by the end
C

of half-an-hour, when she is joined by We have x = 12 and y = 15


another steno who fills forms at the xy
PS

Then, using the formula


rate of 54 an hour. In how much time x+y
will the entire assignment be carried A and B can together finish the work in
out?
12 × 15
U

=
At the end of the first half hour, there 12 + 15
are 600 – 30 = 570 forms remaining.
12 × 15
@

The first steno’s rate per hour is 30 × 2 =


= 60 forms. 27
The combined rate is 60 + 54 = 114 20 2
forms per hour. = = 6 days.
3 3
The remaining forms can be completed
(ii) If A and B together can do a piece
570 of work in x days and A alone can do it in
in = 5 hours. Adding the first half-
114 xy
1 y days then B alone can do the work in
hour the entire job requires 5 hours.
2 y−x
days.
(viii) If A is twice as good as a workman as
B, then the ratio of work done by A and ● If A and B together can do a piece of
B is 2 : 1 and the ratio of time taken by work in 8 days and A alone can do it
A and B to do the same piece of work in 16 days, then in how many days can
is 1 : 2. B alone do this work?
Unit Three : Fundamentals / Time and Work ✫ 211

We have x = 8 and y = 16 6×5


xy x =
Then, using the formula 9
y−x 10 1
B alone can finish the work in
= = 3 days.
3 3
8 × 16 (v) If p men and q women can do a piece
days
16 − 8 of work in N days then x men and y women
8 × 16 can do the same work in
= days
8 1 pqN

TS
= 16 days. days or days.
x
+
y qx + py
(iii) If A, B and C can do a work in x, p×N q×N
y and z days respectively then all of them ● If 10 men and 12 women can do a piece

H
working together can finish the work in of work in 18 days, then in how many
xyz days can 6 men and 8 women do the
days. same work?
xy + yz + xz

G
● If A, B and C can do a work in 4, 6 We have p = 10, q = 12, N = 18,
and 12 days respectively, then in how x = 6 and y = 8
many days can all of them working
together finish the work? U Then, using the formula
1
O
x y
We have, x = 4, y = 6 and z = 12 +
Then, using the formula p×N q×N
H
xyz 6 men and 8 women can do the work in
xy + yz + xz
_T

1
A, B and C can finish the work in =
6 8
+
4 × 6 × 12 10 × 18 12 × 18
=
4 × 6 + 6 × 12 + 4 × 12
C

1
=
4 × 6 × 12 6 8
= +
PS

24 + 72 + 48 180 216
1
4 × 6 × 12 =
= 1 1
144 +
U

30 27
= 2 days.
1
=
@

(iv) If A number of persons doing a piece 9 + 10


of work is increased (or decreased) in a certain 270
ratio, the time needed to do the same work
1
will be decreased (or increased) in the same = 19
ratio.
270
● If 6 persons can complete a piece of 270
work in 5 days, then 9 persons will be =
19
able to do the same piece of work in
4
how many days? = 14 days.
19
Persons Days
6 5 (vi) If A and B working together can finish
9 x a piece of work in x days, B and C in y days,
∴ 6 : 9 :: x : 5 C and A in z days, then
212 ✫ Quantitative Aptitude

(a) A, B, C working together can finish 2 × 18 × 24 × 36


2xyz =
the work in days. 864 + 648 − 432
xy + yz + zx
31104
(b) A alone will finish the work in =
1512 − 432
2xyz
days. 31104
xy + yz − zx =
1080
(c) B alone will finish the work in
144 4
2xyz = = 28 days.

TS
days. 5 5 2xyz
yz + zx − xy
C alone can do the work in zx + xy − yz
(d) C alone will finish the work in days
2xyz

H
days. 31104
zx + xy − yz =
648 + 432 − 864
● If A and B, working together can finish

G
a piece of work in 18 days, B and C in 31104
24 days, C and A in 36 days, how much =
216
time will they take to complete the
work if A, B and C work together? Also
how long would each take separately to U = 144 days.
O
(vii) If A working alone takes x days more
do the same work? than A and B working together and B alone
takes y days more than A and B working
H
We have x = 18, y = 24 and z = 36
∴ A, B and C working together can finish together, then the number of days taken by
A and B, working together, to finish a work
2xyz
_T

the work in Days is xy days.


xy + yz + zx
2 × 18 × 24 × 36 ● A working alone takes 9 days more than
= A and B working together; B working
18 × 24 + 24 × 36 + 36 × 18
C

alone takes 16 days more than A and B


2 × 18 × 24 × 36 together. How many days did A and B
=
PS

432 + 864 + 648 together require to finish the work?


36 × 24 × 36 We have, x = 9 and y = 16
= = 16 days.
1944 Then, using the formula xy ,
U

2xyz A and B can complete the work in


A alone can do the work in
days xy + yz − zx 9 × 16
@

2 × 18 × 24 × 36 = 144 = 12 days
=
18 × 24 + 24 × 36 − 36 × 18 (viii) If A is n times more efficient than
B and is therefore able to finish a work in d
31104
= days less than B, then
432 + 864 − 648 (a) A and B, working together, can finish
31104 nd
= the work in 2 days.
1296 − 648 n −1
(b) A, working alone, can finish the work
31104 d
= = 48 days. in days.
648 n −1
2xyz (c) B, working alone, can finish the work in
B alone can do the work in nd
yz + zx − xy days.
days n−1
Unit Three : Fundamentals / Time and Work ✫ 213

● A is thrice as good a workman as B Then, using the formula


and therefore is able to finish a job in Time taken by A, working alone to
60 days less than B. How much time complete the work
will they take to do it working together?
n+1
We have n = 3 and d = 60 = T
n
nd
Then, using the formula 2 3+1
n −1 = ×9
A and B, working together, can finish 3
the work in = 12 days.

TS
3 × 60 3 × 60 ● A and B together can do a work in 12
(3)2 − 1 = 9 − 1 days. B and C together do it in 15 days.
3 × 60 If A’s efficiency is twice that of C, then

H
= the days required for B alone to finish
8
the work is
45 1

G
= = 22 days 1
2 2 (A + B)’s 1 day’s work = . . . (i)
12
(ix) If A can finish a work in T days and
B is n times efficient than A,
U (B + C)’s 1 day’s work =
1
15
. . . (ii)
O
(a) The time taken by both A and B Difference between A and C’s 1 day’s
working together, to complete the work work
H
T 1 1
is days. = –
1+ n 12 15
_T

(b) A, working alone, will complete the 5− 4 1


work in (n + 1) T days. = =
60 60
(c) B, working alone, will complete the If A alone completes the work in x
n+1
C

days, C will do the same in 2x days


work in T days.
n 1 1 1
∴ − =
PS

● Aman can do a piece of work in 36 days. x 2x 60


If Rajesh works twice as fast as Aman, 2−1 1
how long would they take to finish the =
2x 60
U

work working together?


1 1
We have, T = 36 and n = 2 ∴ =
2x 60
@

T 2x = 60
Then, using the formula days, x = 30
1+ n
1
Time taken by Aman and Rajesh, working ∴ A’s 1 day’s work =
together, to complete the work 30
∴ B’s 1 day’s work
36 = (A + B)’s 1 day’s work
days
1+ 2 – A’s 1 day’s work
= 12 days. 1 1
= −
● A and B together can do a piece of work 12 30
in 9 days. If A does thrice as much work 5−2 3 1
as B in a given time, find how long A == =
60 60 20
alone would take do the work? Hence, B alone will complete the work
We have T = 9 and n = 3 in 20 days.
214 ✫ Quantitative Aptitude

● A is 50% an efficient as B. C does half ● If 50 men working 6 hours a day can


of the work done by A and B together. write 2140 pages in 5 days, then 70 men
If C alone does the work in 20 days, working 5 hours a day can write how
then A, B and C together can do the many pages in 3 days?
work in how many days?
We have N1 = 50, W1 = 2140, T1 = 6,
If B alone completes the work in x days,
A will do the same in 2x days. D1 = 5, N2 = 70, W2 = x,
∴ (A + B)’s 1 day’s work T2 = 5, D2 = 3
1 1 Then, using the formula
+

TS
=
2x x N1W2T1D1 = N2W1D2T2
1+ 2 3 50 × x × 6 × 5 = 70 × 2140 × 3 × 5
= =
2x 2x
70 × 2140 × 3 × 5

H
∴ C does half of the work done by A x =
and B together. Then, 50 × 6 × 5

G
1 3 3 x = 1498.
C’s 1 day’s work= × = 1
2 2x 4x ● If 12 men can do th of the work in 6

3
4x
=
1
20 U 5 1
days, then how many men can do rd
3
O
4x = 3 × 20 of the work in 8 days?
x = 15
1
H
(A + B + C)’s 1 day’s work We have N1 = 12, W1 = , D1 = 6,
5
1 1 3 1
= + +
_T

2x x 4x N2 = x, W2 = , D2 = 8
3
1 1 3 Then, using the formula
= + + [ x = 15] N1W2D1 = N2W1D2
30 15 60
C

1 1
2+ 4+3 12 × × 6 = x× × 8
= 3 5
60
PS

24 × 5 = 8 × x
9 3
= = 24 × 5
60 20 8x =
Hence, all three together will complete 8
U

the work in x = 15
20 2 ∴ N2 = 15 men.
= =6 days.
@

3 3
(x) Just to recall, the formulae in chain rule Pipes and Cisterns
problems are also applicable to time and work Pipe and cistern or tank problems are similar
problems. to time and work problems. Completely filling
(a) If N1 men can do W1 work in D1 days (or emptying) a tank may be thought of as
in T1 time and N2 men can do W2 completing a job.
If a pipe can fill a tank in x hours, then
work in D2 days in T2 time, then the
1
relationship between them is the part filled in 1 hour is .
N1W2D1T1 = N2W1D2T2 x
If a pipe can empty a tank in y hours, then
(b) In the above problem, if time is not
the part of the full tank emptied in 1 hour
given, then the formula becomes 1
N1W2D1 = N2W1D2 is .
y
Unit Three : Fundamentals / Pipes and Cisterns ✫ 215

● Pipe A can fill a tank in 4 minutes and


Fast-Track Methods with
pipe B can fill it in 5 minutes. How long
would it take both pipes, working Worked Examples
together, to fill it?
1 (i) If a pipe can fill a tank in x hours
Pipe A can fill of the tank in 1 minute. and another pipe can fill the same tank in
4
1 y hours, then the net part filled in 1 hour,
Pipe B can fill of the tank in 1 minute. 1 1
5 when both the pipes are opened, is + .
Pipe A and Pipe B can together fill in one x y

TS
1 1 9 xy
minute + = of the tank. ∴ Time taken to fill the tank is .
4 5 20 x+y
Therefore, they can fill the tank together ● If a pipe can fill a tank in 20 hours and
another pipe on fill the same tank in 16

H
20 2
in minutes or 2 and minutes. hours, how long will it take to fill the
9 9
tank if both the pipes are opened

G
● A tank is three-fourths full. Pipe A can together?
fill the tank in 16 minutes. Pipe B can Time taken to fill the tank
empty it in 12 minutes. If both pipes are
open, how long will it take to empty the
tank? U =
xy
x+y
O
1 20 × 16
Pipe A can fill of the tank in =
16 20 + 16
1 minute.
H
1
Pipe B can empty of the tank in (as x = 20 and y = 16)
12
1 minute. 320
_T

1 1 1 =
In 1 minute − = of the tank is 36
12 16 48
emptied. 80
It would take 48 minutes to empty the =
9
C

whole tank. 3 8
However, the tank is only full. or 8 hours.
PS

4 9
3
So, it will take × 48 = 36 minutes to Both pipes together will fill the tank in
4 8
empty the tank. 8 hours.
9
U

● A certain tank can be filled by pipe A in


8 minutes. Pipe B can empty the tank in (ii) (a) If a pipe can fill a tank in x hours
12 minutes. If both pipes are open, how and another pipe can empty the full tank in
@

long will it take to fill or empty the y hours, then the net part filled in 1 hour,
tank? 1 1
1 when both the pipes are opened, is − .
Pipe A fills of the tank in 1 minute. x y
8 1 xy
Pipe B empties of the tank in 1 ∴ Time taken to fill the tank is
12 y−x
minute.
1 1 ● If a pipe can fill a tank in 6 hours and
Since is greater than , the tank will another pipe can empty the tank in 10
8 12 hours, then how long will it take to
ultimately be filled.
1 1 1 fill the tank?
In one minute − = of the tank is Time taken to fill the tank
8 12 24
actually filled. xy
Therefore, the tank will be completely =
y−x
filled in 24 minutes.
216 ✫ Quantitative Aptitude

6 × 10 8 × 10 × 12
= =
10 − 6 296
(as x = 6 and y = 10) 120 9
= = 3 hours.
60 37 37
= = 15 hours.
4 (b) If a pipe fills a tank in x hours and
(b) A pipe can fill a tank in x hours. Due another pipe fills the same tank in y hours,
to a leak in the bottom it is filled in y hours. but a third pipe empties the full tank in z
If the tank is full, the time taken to empty hours, and all of them are opened together,

TS
xy 1 1 1
the tank is
y−x
hours. the net part filled in 1 hour is x + y − z .
● A pipe can fill a tank in 6 hours. Due to ∴ Time taken to fill the tank is

H
a leak in the bottom, it is filled in 10 xyz
hours. If the tank is full, how long will hours.
yz + xz − xy
it take to become empty because of the

G
● Pipe A fills a tank in 4 hours and pipe
leak.
B fills the same tank in 6 hours, but pipe
We have x = 6 and y = 10
Time taken by the leak to empty the full
tank U C empties the full tank in 8 hours. If all
of them are opened together, how much
time will it take to fill the tank?
O
xy We have x = 4, y = 6 and z = 8
= hours
y−x Time taken to fill the tank
H
6 × 10 xyz
= =
10 − 6
_T

yz + xz − xy
60
= = 15 hours. 4×6×8
4 =
6×8+8× 4− 4×6
C

(iii) (a) Three pipes A, B and C,


4×6×8
individually, can fill a cistern in x, y and z =
hours respectively. If all the three pipes are 48 + 32 − 24
PS

opened together, the time taken to fill the 4×6×8


=
xyz 56
cistern is given by . 24 3
xy + yz + zx
U

= = 3 hours.
7 7
● Three pipes A, B and C, individually can
(iv) A cistern has a leak from which the
@

fill a cistern in 8, 10 and 12 hours


cistern could get empty in x hours. A pipe
respectively. If all the three pipes are
which admits y litres of water per hour into
opened together, then how much time
the cistern is turned on and now the cistern
is required to fill the cistern.
gets emptied in z hours. The capacity of the
We have x = 8, y = 10, and z = 12 xyz
∴ Time taken to fill the tank cistern is litres.
z−x
xyz
= ● A leak in the bottom of a tank can empty
xy + yz + zx the full tank in 5 hours. An inlet pipe fills
8 × 10 × 12 water at the rate of 3 litres per minute.
=
8 × 10 + 10 × 12 + 12 × 8 When the tank is full, the inlet is opened
and due to the leak, the tank gets
8 × 10 × 12 emptied in 7 hours. Find the capacity of
=
80 + 120 + 96 the tank.
Unit Three : Fundamentals / Pipes and Cisterns ✫ 217

We have x = 5, y = 3 × 60 = 180 and with both the fill pipes opened together,
z = 7 n
is S hours.
xyz n+1
Capacity of the tank = litres
z−x ● One fill pipe A is 8 times faster than the
5 × 180 × 7 second fill pipe B. If B can fill a cistern
= in 45 minutes, then find the time when
7−5
the cistern will be full if both fill pipes
5 × 180 × 7 are opened together.
=
2

TS
= 3150 litres. We have n = 8 and R = 45
R
(v) If one fill pipe A is n times faster and Cistern will be full in minutes
n+1
takes t minutes less time than the other fill pipe
45

H
B, then the time taken to fill a cistern, if both =
nt 8+1
the pipes are opened together is ( n 1)2 45

G
= = 5 minutes.
minutes. 9
t One fill pipe A is 6 times faster than a
(a) A will fill the cistern in
n−1
nt
minutes.
U ●
second fill pipe B. If A can fill a cistern
in 21 minutes, then find the time when
O
(b) B will fill the cistern in minutes. the cistern will be full if both fill pipes
n−1
are opened together.
H
● One fill pipe A is 4 times faster than the
We have n = 6, S = 21
second fill pipe B and takes 36 minutes
less than the fill pipe B. When will the n
_T

Cistern will be full in S minutes


cistern be full if both fill pipes are n+1
opened together? 6
= 21
6+1
We have n = 4 and t = 36
C

6
nt = × 21
Cistern will be full in minutes 7
( n − 1)2
PS

= 6 × 3 = 18 minutes.
4 × 36
=
(4 − 1)2 Miscellaneous Worked Examples
U

4 × 36 ● A pipe can empty a tank in 30 minutes. A


=
(3)2 second pipe with diameter twice as much
@

as that of the first is also attached with


= 4 × 36 = 16 minutes. the tank to empty it. The two together can
9 empty the tank in:
(vi) Given that fill pipe A is n times A pipe with double diameter will take half-
faster than the fill pipe B, time.
(a) If B can fill a cistern in R hours, then the So, the second pipe can empty the full tank
time in which the cistern will be full, in 15 minutes.
with both the fill pipes opened together, Part emptied by both in 1 minute
1 1
R = +
is hours. 30 15
n+1
(b) If A can fill a cistern in S hours, than the 1+ 2
=
time in which the cistern will be full, 30
218 ✫ Quantitative Aptitude

3 1 1 1 1
= = − =
30 10 x 2x 36
Time taken to empty the full tank 2−1 1
10 =
= = 10 minutes. 2x 36
1
1 1
● Two pipes can fill a tank with water in 15 =
2x 36
and 12 hours respectively, and a third pipe
can empty it in 4 hours. If the pipes be 36
x = = 18 hours.

TS
opened in order, at 8, 9 and 11 a.m. 2
respectively, the tank will be emptied at:
Time and Distance and
Part filled by pipe A from 8 a.m. to 11 a.m.
Moving Vehicles

H
3 1 In time and distance problems there are usually
= =
15 5 three quantities involved: the distance, the

G
Part filled by pipe B from 9 a.m. to 11 a.m. time, and the rate or speed (distance per unit
time).
2 1
=
=
12 6
Part filled till 11 a.m. U Distance = Speed × Time
Distance
O
Speed =
1 1 Time
= +
5 6 Distance
H
6 + 5 11 Time =
Speed
= =
30 30 When two people or objects are travelling
_T

At 11 a.m. pipe C is opened to empty it. towards each other, the rate at which they
∴ Part of tank emptied in 1 hour approach each other is the sum of their respective
1 1 1 rates (or speeds).
= − −
C

When two people or objects are travelling


4 15 12
in directly opposite directions, the rate at
15 − 4 − 5 which they are separating is the sum of their
PS

=
60 respective speeds.
Speed of an object is measured in kilometres
6 1
= = per hour (kmph) or metres per second (m/s).
60 10
U

To convert the unit, we use the following


11 11 10 formulae
∴ part will be emptied in ×
30 30 1 1000 m
@

5
1 kmph = = m/s
11 60 × 60 s 18
= hours
3 5
∴ x kmph = x × m/s
i.e. in 3 hours 40 minutes 18
i.e., at 11.40 a.m. 18
and x m/s = x × kmph
● A tap takes 36 hours extra to fill a tank 5
∴ It follows that 5 m/s = 18 kmph,
due to a leakage equivalent to half of its
10 m/s = 36 kmph, 15 m/s = 54 kmph
inflow. The inflow can fill the tank in how
20 m/s = 72 kmph, 25 m/s = 90 kmph
many hours?
Let the inflow fill the tank in x hours Average Speed
1 A moving body travels distances D 1, D 2,
The inflow can fill of the tank in 1 hour
x D3, . . . Dn metres at different speeds, S1, S2,
Time taken to fill the tank S3, . . . Sn in T1, T2, T3, . . . Tn seconds
Unit Three : Fundamentals / Time and Distance and Moving Vehicles ✫ 219

respectively. If the average speed of the moving


body is Sa,
Fast-Track Formula

Total distance Distance D


(i) Sa =
Total time Product of time T1 × T2
D1 + D2 + D3 + . . . + Dn S1 × S2
= Product of Speeds
T1 + T2 + T3 + . . . + Tn = D
Distance
if distance and time values are known.
S1 − S2

TS
Difference in Speeds
S T + S2 T2 + . . . + Sn Tn =
T1 − T2
(ii) Sa = 1 1 Difference in Time
T1 + T2 + . . . + Tn
if the speed and time values are

H
When Distance Travelled is Different
known. If the body moves over distances D1 and D2
D1 + D2 + D3 + . . . + Dn

G
at S1 and S2 speeds and T1 and T2 time,
(iii) Sa =
D1 D2 D3 D
+ + + ... + n (i) D1 = S1 T1 ; D2 = S2 T2
S1 S2 S3 Sn
if the distance and speed values are
U (ii) T1 =
D1
S1
; T2 = 2
D
S2
O
known. D1 D2
(iii) S1 = ; S2 =
● A car goes 150 km at a speed of 50 km T1 T2
H
per hour and then it increases its speed These in combination with the earlier
to 60 km per hour to travel another 300 relations worked out have to be used with care
_T

km. What is the average speed of the in the light of what is asked in a problem.
car?
Fast-Track Formulae and
We know the distance and speed, but
Worked Examples
C

not the time. So we use the formula,


D1 + D2 (i) If a body covers distance D1 and D2
PS

Sa =
D1 D2 at a speed of S1 and S2, then the total time
+
S1 S2 taken is
D1 D2
150 + 300 450 T= S +S
U

Sa = = 1 2
150 300
+ 3 +5 ● Mohan and Sumit are walking towards
50 60
@

each other over a road 150 km long,


450 Mohan at the rate of 9 kmph and Sumit
= = 56.25 kmph at the rate of 6 kmph. How soon will
8
they meet?
Average speed is 56.25 kmph.
D
Time =
When Distance Travelled is the Same S1 + S2
Suppose a body moves to cover the same We have one Distance (D) = 150 km,
distance D at two different speeds S1 and S2 S1 = 9 kmph and S2 = 6 kmph
in time T1 and T2 respectively, 150
∴Time =
(i) D = S1 × T1 = S2 T2 9+6
From this we may derive further 150
= = 10 hours
15
(ii) D2 = S1 S2 T1 T2 Mohan and Sumit will meet in 10 hours.
220 ✫ Quantitative Aptitude

(ii) (a) If a distance D from A to B is covered 22



at a kmph in time t and from B to A at b kmph, = 3 7
2 ab 22
then the average speed is given by kmph. 4+
a+b 7
● Abhay walks home from school at the 88
speed of 4 kmph and from school to = 3 7
3 28 + 22
home at the speed of 3 kmph. Find the 7
6
average speed of the boy.

TS
88 7
2 ab = 3 ×
Average speed Sa = 7 50
a+b
3 21 88
= 3× = 5.28 km.

H
Here a = 4 and b = 3 = 50
6 6
21 (iii) If the speed of a body is changed in the
2×4×

G
∴ Sa = 6 ratio a : b, then the ratio of the time taken changes
21 in the ratio b : a.
4+

=
28
24 + 21
6
U a
If the new speed is of the original speed,
b
then the change in time taken to cover the same
O
distance is given by,
6 b
change in time = – 1 × original time
H
28 a
=
45
6 3
_T

● By walking at of his usual speed,


28 × 6 4
= = 3.73 kmph. Mahesh is 7 minutes late to his office.
45
Find his usual time to cover the distance.
C

(b) If the distance from A to B is covered Original time (here, usual time)
at a kmph and from B to A is covered at b kmph
Change in time
PS

and total T hours is taken in the whole journey, =


ab b
then the distance between A and B is T −1
a+b a a 3
Here change in time = 7 and =
U

km. b 4
b 4
or = .
● Raju walks from home to school at the a 3
@

speed of 4 kmph and from school to 7


1 Mahesh’s usual time =
home at the speed of 3 kmph and he 4
7 −1
takes 3 hours for the whole journey. 3
Calculate the distance between home 7
and school. =
4−3
ab 3
Distance (D) = T
a+b 1 7
Here a = 4 and b = 3 , T = 3 =
7 1
1 3
4×3
∴ D = 3 7 7×3
1 = = 21 minutes.
4+ 3 1
7
Unit Three : Fundamentals / Time and Distance and Moving Vehicles ✫ 221

(iv) If A and B start moving at the same time Shashikant’s speed = 40 kmph
at a and b speeds respectively, Then,
1
(a) A and B will be 1 km apart in Shashikant's speed T2
a−b =
hour, if they walk in the same direction, and Nikhil's speed T1
1
(b) A and B will be 1 km apart in
a+b 40 4
hour, if they walk in opposite directions. ⇒ =
Nikhil's speed 9
1
● Amit and Sumit start running from a 16

TS
1
gate at 2 and 2 km an hour 4
2 =
1 25
respectively. At the end of 3 hours
2 16

H
how far apart will they be if
(a) they walk in the same direction 2 2× 4
= =

G
(b) they walk in opposite directions. 5 5
4
1
(a) In 1 hour, A and B will be 2 − 2

km apart or
1
2
km apart. At the endU =
8
5
O
2 40 × 5
1 ∴ Nikhil’s speed = = 25 kmph.
of 3 hours, they will be 8
2
H
1 7 7 3 (vi) If a person takes one kind of transport
× = = 1 km apart
2 2 4 4 to travel between A and B and takes another
_T

1 mode to return from B to A, then


(b) In 1 hour, A and B will be 2 + 2
2 Time taken for going both ways in one mode
5 9
km apart or 2 + = km apart. At – Time taken by mixed mode
C

2 2 = Time gained or lost.


1
the end of 3 hours, they will be
PS

2 ● By walking to his office from home and


cycling back from office using the same
9 7 63 3
× = = 15 km apart way, a man takes a total of 4 hours. He
2 2 4 4
would have gained 1 hour 30 minutes
U

(v) If A and B start at the same time from by cycling both ways. How long would
two points P and Q towards each other and it take for him to walk both ways?
@

after crossing each other they take T1 and T2


hours in reaching Q and P respectively, then, Time to walk both ways – time to cycle
A's speed T and walk = time gained
= 2 Time to walk both ways
B's speed T1
= time gained + time to cycle and walk
● Shashikant and Nikhil start their journey 1
from Ranchi and Delhi. After crossing = 1 + 4
2
each other they finish their remaining
9 1
journey in 1 hours and 4 hours = 5 hours
16 2
respectively. Find Nikhil’s speed, if 1
The man would take 5 hours to
Shashikant’s speed is 40 kmph. 2
walk both ways.
9
We have T1 = 1 , T2 = 4,
16
222 ✫ Quantitative Aptitude

Moving Trains/Vehicles 18
 1 m/sec = kmph
Where moving trains are seen to cross an object, 5
the entire length of the train has to be considered. = 9 × 18 kmph
The object needs to be crossed entirely. So, = 162 kmph.
distance covered by the train
= Length of the train (ii) If a moving body overtakes a bridge
+ Length of the object crossed. or a tunnel or a platform (all of which have
lengths of their own), then
Time required for the train to cross the object
Distance covered = Sum of two lengths.

TS
Length of train + Length of object ∴ Speed of moving body
=
Speed of train – Speed of object
Length of moving body + Length of bridge
If the object to be crossed is of negligible =
Time taken
length, its length can be considered zero.

H
If the object crossed is stationary, its speed ● A train 320 m long passes a bridge in 24
is zero. seconds, moving at a speed of 120

G
kmph. Find the length of the bridge.
Points to Remember
In all these problems, it is necessary to Speed of the train
convert length into the same unit. So remember
1 km/hr =
5
m/s. U =
Length of train + Length of bridge
Time taken to cross the bridge
O
18
Length of train = 320 m,
Length of bridge = x
H
Fast-Track Formulae and
Time = 24 seconds
Worked Examples
5 320 + x
_T

⇒ 120 × =
(i) If a moving body, such as a train, 18 24
overtakes a man or a pole or a milestone 5
 1 kmph = m/sec
C

(whose length is negligible and, hence, 0), 18


then the distance covered in overtaking 5
320 + x = 120 × × 24
PS

= length of the moving body 18


320 + x = 800
∴ Speed of moving body x = 800 – 320
Length of moving body = 480 m
=
U

Time taken The length of the bridge is 480 metres.

● A train 270 m long crosses a man in 6 (iii) If a moving body such as a train
@

seconds. What is the speed of the train crosses a moving body of negligible length
in kmph? (whose length can be taken as zero), such as
a running person, then the
Length of a train (a) Time taken to cross
Speed of a train =
Time Length of the train
Length of train = 270 m and =
Speed of (train – object)
time = 6 seconds
270 (if both are moving in the same direction)
Speed of a train = m/sec.
6 (b) Time taken to cross
Length of the train
= 45 m/sec. =
Speed of (train + object)
18 (if the objects are moving in the opposite
= 45 × kmph
5 directions)
Unit Three : Fundamentals / Time and Distance and Moving Vehicles ✫ 223

● A train 110 metres long is travelling at 84 (a) Time taken to cross


kmph. How long will it take to cross a Length of (train + object)
man running at 6 kmph in the opposite =
direction? Speed of (train – object)
(if the objects are moving in the same
Here the objects are moving in opposite direction)
directions.
(b) Time taken to cross
110
Time taken to cross = Length of (train + object)
5 =
(84 + 6) × Speed of (train + object)
18

TS
5 (if the objects are moving in opposite
 1 kmph = m/s directions)
18
110 ● A train 320 m long is running at a speed

H
=
5 of 70 kmph. In what time will it pass a
90 ×
18 train of length 280 m running at a speed

G
of 80 kmph in the opposite direction?
110
== 4.4 seconds Time taken
25
Crossing time is 4.4 seconds.
U =
Length of train + Length of object
Speed of (train + object)
O
● A train of length 480 m is running at a
speed of 60 kmph and Mohan walking 320 + 280
=
in the same direction of the train walks 5
(70 + 80) ×
H
at a speed of 6 kmph. In what time will 18
they pass each other? 5
_T

as 1 kmph = m/s
Here the objects are moving in the 18
same direction and one has negligible 600
length. So, = 5
150 ×
C

Time taken to cross 18


Length of the train 600 × 18
PS

= =
Speed of (train – object) 150 × 5
480 72 2
= = = 14 seconds
5 5 5
U

(60 − 6) × 2
18 Crossing time will be 14 seconds.
5
5
@

 1 kmph = m/sec (v) If a train of length L1 crosses another


18
train of length L2 in T1 seconds if they are
480
= moving in opposite directions and T2 seconds
5
54 × if they are moving in the same direction, then
18
The speed of the faster train
480
= = 32 seconds L +L 1 1
15 = 1 2 +
2 T1 T2
They will pass one another in 32 and,
seconds.
The speed of the slower train
(iv) If a moving body such as a train
crosses another moving body which also has L1 + L2 1 1
some length (for example, two trains passing = −
2 T1 T2
one another), then
224 ✫ Quantitative Aptitude

● A train 40 km long crosses another train (b) Also, the time after which the two
of length 48 km in 32 seconds when they trains will meet is given by
are moving in opposite directions and in xT
48 seconds when they are moving in the y − x hours.
same direction. What is the speed of
● A train leaves the station at 5 a.m. at 60
both the trains? km/hr. Another train leaves the same
Here L1 = 40, L2 = 48, T1 = 32 and station at 6.30 a.m. at 75 km/hr and
travels in the direction of the first
T2 = 48

TS
station. At what time and at what
Speed of the faster train distance from the station will they meet?
L1 + L2 1 1 Distance of meeting from station
= +
2 T1 T2 x × y ×T

H
=
5 5 y−x
40 × + 48 × 1 1

G
18 18 + Here x = 60, y = 75, Time from 5 a.m.
2 32 48 1 3
to 6.30 a.m. (T) = 1 hours = hours
200 240
= 18
+
18 3+ 2
U 60 × 75 ×
75 − 60
3
2
2 2
O
2 96
440 30 × 75 × 3
=
H
5
= 18 15
2 96 = 450 km
_T

440 5 Also, time of their meeting


= ×
18 × 2 96 xT
=
2200 y−x
C

= = 0.63 m/s
3456 3
60 ×
2
PS

Speed of the slower train 75 − 60


L1 + L2 1 1 30 × 3
= − =
2 T1 T2 15
U

5 5 = 6 hours after 6.30 a.m.


40 × + 48 × 1 1
18 18 That is, 12.30 p.m.
@

= −
2 32 48
(vii) The distance between two stations A
440 3−2 and B is P km. A train starts from A to B at
=
18 × 2 96 x km/hr. T hours later another train starts from
= 0.12 m/s B to A at y kmph.
(a) Then the distance from A at which
(vi) A train starts from a place at x both the trains will meet is given by
km/hr and another fast train starts from the
P + yT
same place after T hours at y km/hr in the same x km.
direction. x+y
(a) Then, the distance from the starting (b) Also, the time after which the two
place at which both the trains will P + yT
trains will meet is given by
x× y×T x+y
meet is given by y−x km. hours.
Unit Three : Fundamentals / Time and Distance and Moving Vehicles ✫ 225

● A train starts from station A at 8 a.m., We are told that the first car starts out 6
travels at 50 kmph towards station B, hours ahead of the second at the rate of 30
220 km away. Another train starts from kmph.
station B at 10 a.m. and travels at 70 So it has travelled 6 × 30 or 180 km by the
kmph towards station A. At what time the second one starts.
distance from A, will they meet? The second car travels at the rate of 50
kmph.
Distance of meeting point from station A Therefore it gains 50 – 30 or 20 kmph.
P + yT The second car has to cover 180 km.
= x

TS
180
x+y It will take or 9 hours to catch up with
20
Here x = 50, y = 70, P = 220 and Time the first car.
from 8 a.m. to 10 a.m. (T) = 2 hours
● The time taken by three cars to cover the

H
220 + 70 × 2 same distance is in the ratio 3 : 5 : 7. Find
50
50 + 70 the ratio of their speed.

G
220 + 140 Ratio of the time taken = 3 : 5 : 7
= 50
1 1 1

= 50 ×
120
360
= 150 km. U ∴ Ratio of speed = : :
3 5 7
[  L.C.M. of 3, 5, 7 = 105]
O
120
1 1 1
● Two stations A and B are 100 km apart = 105 × : 105 × : 105 ×
3 5 7
H
on a straight line. One train starts from = 35 : 21 : 15.
A at 8 a.m. and travels towards B at 20
kmph speed. Another train starts from ● The speeds of A and B are in the ratio
_T

B at 9 a.m. and travels towards A at 25 3 : 4. A takes 20 minutes more than B to


kmph speed. At what time will they reach a destination. In what time does A
meet? reach the destination?
C

Time of their meeting Let the time taken by A be x hours.


Then, 20
P + yT
PS

= Time taken by B = x − hours


x+y 60
Here x = 20, y = 25, P = 100, Time from 1
= x− hours
3
U

8 a.m. to 9 a.m. (T) = 1 hour


100 + 25 × 1 Ratio of speeds = Inverse ratio of time taken
1
@

20 + 25 3 : 4 = x− : x
3
125 1
=
45 ⇒ 3 × x = 4 x−
3
= 2 hours 47 minutes after 8 a.m.
That is, 10.47 a.m. 3x − 1
⇒ 3x = 4
3
⇒ 3x × 3 = 12x – 4
Miscellaneous Worked Examples
⇒ 9x = 12x – 4
● Two cars are travelling along the same road.
⇒ 12x – 9x = 4
The first one, which travels at the rate of 30
3x = 4
kmph, starts 6 hours ahead of the second
one, which travels at the rate of 50 kmph. 4 1
x = = 1 hours
How long will it take the second car to catch 3 3
1
up with the first one? A takes 1 hours to reach the destination.
3
226 ✫ Quantitative Aptitude

● A train passes two persons walking in the


same direction in which the train is going. 100
=
3 3
These persons are walking at the rate of 3 + +1
km/hr and 4 km/hr respectively and the 2 2
train passes them completely in 10 seconds 100
=
and 11 seconds respectively. Find the speed 3+ 3+ 2
of the train. 2
Let the length of the train be x km and 100 × 2
= = 25 km/hr.
its speed be y km/hr. 8

TS
Speed of the train relative to first man ● Two cars are travelling along the same
= (y – 3) km/hr road. The first one, which travels at the
Speed of the train relative to second man rate of 40 kmph, starts 7 hours ahead of
= (y – 5) km/hr

H
the second one, which travels at the rate
Time taken to cross the first man of 60 kmph. How long will it take the
second car to catch up with the first one?

G
Length of train
=
Speed of the train relative to first man The first car starts out 7 hours ahead of
the second at the rate of 40 kmph.
=
10
=
x
60 × 60 ( y − 3) U So it has travelled 7 × 40 or 280 km by the
time second one starts.
O
10y – 30 = 3600x ... (i) The second car travels at the rate of 60
Time taken to cross the second man kmph. Therefore it gains 60 – 40 = 20 kmph.
H
The second car has to cover 280 km.
Length of train 280
=
Speed of the train relative of second man It will take = 14 hours to catch up
_T

20
11 x with the first car.
=
60 × 60 ( y − 5)
11y – 55 = 3600x ... (ii)
C

Subtracting equation (i) from equation (ii), Boats and Streams


we get Water may be still or running. Movement (a
PS

11y – 55 = 3600x person swimming or rowing a boat) in the two


10y – 30 = 3600x types of waters is different. A pond or lake
– + –
has still water while a stream or river has
y – 25 = 0
U

running water.
y = 25 The two terms that we frequently come
Hence, speed of the train is 25 km/hr. across while solving questions on boats and
@

● A man completed a certain journey by a streams are ‘upstream’ and ‘downstream’. If


car. If he covered 30% of the distance at the boat or a swimmer moves against the
the speed of 20 km/hr, 60% of the distance current of the stream then it is called upstream
at 40 km/hr and the remaining distance at and if the boat or the swimmer moves with
10 km/hr, what was his average speed for the current of the stream, it is called
the whole journey? downstream. Upstream reduces the speed while
downstream increases speed.
Let the total distance be 100 km.
If a person’s rate of rowing (which is the
Total distance covered same as the speed of the boat) in still water
Average speed =
Time taken is x kmph, the rate of rowing or speed of the
100 boat in a stream will depend on the current
= y kmph and the direction in which the person
30 60 10
+ + is going.
20 40 10
Unit Three : Fundamentals / Boats and Streams ✫ 227

(i) Speed of the boat (or a person’s rate Fast-Track Methods and
of rowing) upstream = x – y kmph
Worked Examples
(ii) Speed of the boat (or a person’s rate
of rowing), downstream = x + y kmph (i) A man can row x kmph in still water.
distance If in a stream which is flowing at y kmph,
(iii) As rate = , the person’s (or
time it takes him z hours to row to a place and
boat’s) upstream rate x – y back then the distance between the two
downstream distance
= z (x2 − y 2 )
time to cover it places is .

TS
2x
The person’s (or boat’s) downstream
● A man can row 12 kmph in still water.
rate x + y
In a stream flowing with the speed of
upstream distance
= 8 kmph he takes 3 hours to move with

H
time to cover it the stream and come back. Find the
(iv) A person’s rate in still water is half the distance he rowed the boat.

G
sum of his rates with and against the
z (x2 − y2 )
current (or downstream and upstream). Distance travelled =
2x
Person’s rate in still water
=
1
(x + y) + (x – y) U Here z = 3, x = 12 and y = 8
3 (12)2 − (8)2
O
2 ⇒
(v) The rate of current or the stream speed 2 × 12
3 [144 − 64]
H
is half the difference between the rates
of the man with and against the current. =
24
_T

The speed of the stream


1 3 × 80
= (x + y) – (x – y) = = 10 km.
2 24
C

(vi) When the distances downstream and (ii) A man rows a certain distance
upstream are the same, then downstream in x hours and returns the same
distance in y hours. If the stream flows at the
PS

Person’s rate in still water


(a) rate of z kmph then the speed of the man in
Speed of stream
z (x + y )
still water is given by kmph.
time upstream + time downstream y−x
U

= ● A man rows downstream in 10 hours


time upstream – time downstream
and returns in 14 hours. If the stream is
@

(b) Average speed for total journey up


flowing with the speed of 15 kmph, then
and down
what is the speed of the man in still
Upstream rate × Downstream rate water?
=
Person’s rate in still water z ( x + y)
(c) Total journey time up and down Speed of man =
y−x
Person’s rate in still water Here z = 15, x = 10, y = 14
× Total distance
= 15 (10 + 14)
Upstream rate × Downstream rate ⇒
14 − 10
(Total distance = 2 × any one side
distance, as upstream and down- 15 × 24
=
stream distances are the same) 4
= 90 kmph.
228 ✫ Quantitative Aptitude

(iii) A man can row a boat in still water at ● Aman rows at the rate of 16 kmph
x kmph. In a stream flowing at y kmph, if it downstream and 13 kmph upstream.
takes z hours more to go upstream than to go Find the speed of the stream and the
downstream for the same distance, then the speed of the boat in still water.
z (x2 − y 2 ) Speed of the boat in still water
distance is given by km.
2y 1
= (x + y)
● A man rows a boat at 8 kmph in still 2
water. In a stream flowing at 2 kmph, Here x = 16 and y = 13
he takes 6 hours more to go upstream 1

TS
(16 + 13)
than to go downstream for the same 2
distance. Find the distance travelled by 1
= (29)
him. 2
z (x2 − y2 ) = 14.5 kmph

H
Distance travelled = 1
2y Speed of the stream= (x – y)
Here z = 6, x = 8, y = 2 2

G
1
6 (8)2 − (2)2 ⇒ (16 – 13)
⇒ 2
2×2
=
6 [64 − 4]
U =
3
2
= 1.5 kmph.
The speed of the stream is 1.5 kmph
O
4
and the speed of the boat in still water
6 × 60 is 14.5 kmph.
= = 90 kmph.
H
4
(iv) If a man capable of rowing at the speed (vi) A man takes x times as long to row
of x kmph in still water, rows the same distance upstream as to row downstream. If the speed
_T

upstream and downstream and if the stream of the man is a kmph and the speed of the
stream is b kmph, then
flows at a rate of y kmph, then the man’s
average speed throughout the journey is given x +1
a =b
x−1
C

by ( x − y ) ( x + y ) kmph. ● A man can row at the rate of 5 km/hr


x in still water. If the time taken to row
PS

● A man can row 8 kmph in still water. If a certain distance upstream is 4 times as
the stream is flowing at 4 kmph, and if much as to row the same distance
the man rows from a point A to B and downstream, find the speed of the
current.
U

from B to A, find the average speed of


the total journey. Speed of the man (a)
( x − y) ( x + y) x +1
@

Average speed = = speed of the current (b)


x x −1
Here x = 8 and y = 4 Here x = 4, speed of the man (a) = 5,
(8 − 4) (8 + 4) and speed of the current (b) = 7

8 4+1
5 = speed of the current
4 × 12 4 −1
= = 6 kmph.
8 5
(v) If a man rows a boat downstream at 5 = speed of the current
3
x kmph and upstream at y kmph, then the
speed of the boat in still water is given by 5 × speed of the current = 5 × 3
1 5×3
(x + y ) kmph and speed of the stream is Speed of the current =
2 5
1 = 3 km/hr.
given by ( x − y ) kmph.
2
Unit Three : Fundamentals / Races and Games ✫ 229

Miscellaneous Worked Examples ● A man can row three-quarters of a kilometre


1
● Two boats A and B start towards each other against the stream in 11 minutes and
1 4
from two places, 108 km apart. Speed of the
return in 7 minutes. What is the speed
boats A and B in still water is 12 km/hr 2
and 15 km/hr respectively. If A proceeds of the man in still water?
down and B up the stream, after how much
time will they meet? Distance covered upstream in
Let the speed of the stream be x kmph and 45 3
minutes = km
4

TS
both the boats meet after t hours 4
According to the question, Distance covered upstream in 1 hour
(12 + x) t + (15 – x) t = 108 3 4
= × × 60
12t + xt + 15t – xt = 108

H
4 45
27t = 108 = 4 kmph

G
108 Distance covered downstream in
t = = 4 hours
27 15 3
minutes = km
● The speed of a motorboat in still water is
45 kmph. If the motorboat travels 80 km
U 2 4
Distance covered downstream in 1 hour
O
along the stream in 1 hour 20 minutes, then
3 2
what will be the time taken by it to cover = × × 60
the same distance against the stream? 4 15
H
= 6 kmph
Let the speed of the current We have, x = 6 km/hr and y = 4 km/hr
= x kmph
_T

We know that,
Rate downstream = (x + 45) kmph x+y
Distance = 80 and Time Speed of the man in still water =
= 1 hour 20 minutes 2
6+4
C

Then, using the formula



Distance 2
PS

= Time = 5 kmph.
Rate downstream
80 Races and Games
⇒ = 1 hour 20 minutes
x + 45
U

A race is any contest of speed in running,


80 80 driving, riding or rowing.
⇒ =
x + 45 60 The starting point is the point from where
@

80 4 the race begins.


⇒ = The finishing point (winning post) is the
x + 45 3
point where the race finishes.
4x + 180 = 240
The winner is the contestant who first
4x = 240 – 180
reaches the finishing point (destination point).
4x = 60
If all the contestants reach the finishing
x = 15 kmph
point exactly at the same time, then the race
Rate upstream = (45 – 15) kmph is said to end in a deadlock or the race is a dead
= 30 kmph heat race. The path or the ground where the
80
∴ Required time = race is conducted is called the race course or
30 the track.
8
= If A and B are two contestants, and A is
3
= 2 hours 40 minutes. the winner, and B the loser, A could have
230 ✫ Quantitative Aptitude

beaten B either by d length units or t time. In ● Ramesh and Hari run 360 metres in 11
other words, when A reaches the finishing point, seconds and 12 seconds respectively. If
B is d metres behind A. We can also say that they started together, find how far Hari
A reached the destination t time (usually seconds) was from the finishing line when Ramesh
ahead of B. Or that B lost by d length or by t completed his 360 metres.
time. Winner’s distance = 360 m, Winner’s
In games of skill, such as billiards, the time = 11 seconds and Loser’s time = 12
winner beats the loser by ‘points’. seconds
Game of 100 means, a game in which the Then, using the formula

TS
participant scoring 100 points first wins the Winner’s time Loser’s time
game. =
Loser’s distance Winner’s distance
If in a game of 100, ‘A can give B 20 points’,
it implies that while A scores 100 points, B 11 12

H
=
scores 80 points (100 – 20). Loser’s distance 360

G
Points to remember 360 × 11
Loser’s distance =
● A beats B by x seconds means, A finishes the 12
race x seconds before B finishes.
● A gives B a start of x seconds means, A starts
x seconds after B starts from the same point. U = 30 × 11
= 330 m
So Hari was (360 – 330 =) 30 metres away
O
● A gives B a start of x metres or t seconds from the finishing line when Ramesh
means, if A starts at the starting point, B completed 360 metres.
H
starts x metres ahead from the starting point
● A can run 2 km in 45 seconds less time
at the same time or A starts t seconds later
than B. In a 2-km race, B gets a start of
_T

than B. 200 metres and loses by 25 metres. Find


● In a race of L length that A wins and B the time A and B take to run 2 km.
loses by d length,
Winner A’s distance = L Let the time taken by A be x seconds.
C

Loser B’s distance = L – d Then the time taken by B = (x + 45)


● If A gives B a start of d, seconds
PS

Loser B’s race length = L – d Total distance = winner’s (A’s) distance


● If A wins against B by t time, = 2 km
Winner A’s time = Loser B’s time – t Initial start (x) = 200 m, beat distance
U

● Summing up, = 25 m
Then, using the formula
(i) Winner’s distance = Length of race
Winner’s time Loser’s time
@

(ii) Loser’s distance = Winner’s distance – =


(beat distance + start distance, if any) Loser’s distance Winner’ distance
(iii) Winner’s time = Loser’s time – (beat x x + 45
=
time + start time, if any) 2000 − (200 + 25) 2000
(iv) In a deadlock, beat time = 0, and beat x x + 45
distance = 0 =
1775 2000
2000x = 1775x + 45 × 1775
Fast-Track Formulae
2000x – 1775x = 45 × 1775
Winner’s time Loser’s time 225x = 45 × 1775
(i) =
Loser’s distance Winner’s distance 45 × 1775
x =
Start time + Beat time 225
=
Start distance + Beat distance = 5 minutes 55 seconds
Unit Three : Fundamentals / Races and Games ✫ 231

∴ Time taken by A = 5 minutes 55 seconds (iv) In the case of three participants A, B and
and time taken by B = x + 45 C in a race of the same length L, if the winner
= 5.55 + 45 seconds beats the IInd position by x distance and the
= 6 minutes 40 seconds.
IIIrd position by y distance, and the IInd beats
(ii) If A is x times as fast as B and A gives the IIIrd by z distance, then
B a start of a m, then the length of the race (L – x) z = L (y – x)
course, so that both A and B reach the winning ● In a hundred-metre race, A beats B by
x 25 metres and B beats C by 4 metres. By
post at the same time, is given by a m.
x −1 how many metres does A beat C?

TS
1
● Raman is 1 times as fast as Pawan. If Here we know the distance (x) by
5
Raman gives Pawan a start of 30 m, how which the winner beats the IInd position
long is the race course when both of and the distance (z) by which the IInd

H
them reach the winning post at the same position beats the IIIrd position. We
time? have to find y.

G
1 6 Using the formula
We have, a = 30, x = 1 = (L – x) z = L (y – x)
5 5
Length of race course = a
x
x−1 U (100 – 25) 4 = 100 (y – 25)
75 × 4 = 100y – 2500
O
6 –100y = –2500 – 300
5 100y = 2500 + 300
30
H
6 2800
−1 y = = 28 metres.
5 100
_T

6 So, A beats C by 28 metres.


= 30 5 Alternative Method
1
5 The same problem may be solved with
C

6 5 the help of ratio.


= 30 × × A : B = 100 : (100 – 25) or 100 : 75
PS

5 1
= 180 metres. B : C = 100 : (100 – 4) or 100 : 96
(iii) If A can run a m race in x seconds and A B 100 100 100
B in y seconds, where x < y, then A beats B ∴ A:C = × = × =
U

a B C 75 96 72
by a distance × (y – x) m.
y = 100 : 72
@

● Rohan can run 400 m in 50 seconds and


Ajay in 64 seconds. By what distance So A beats C by (100 – 72 =) 28 metres.
Rohan beats Ajay?
We have, a = 400, x = 50, y = 64
Miscellaneous Worked Examples
Distance between A and B
a ● A can run 350 metres in 46 seconds and B
= (y – x) metres in 50 seconds. By how many seconds will B
y
Distance between Rohan and Ajay win if he has a 35-metre start?
400 Time taken by A to cover 350 m = 46 seconds
= (64 – 50)
64 Time taken by B to cover 350 m = 50 seconds
400 B gets a start of 35 m
= × 14 Distance to cover = 350 – 35
64
= 87.5 metres. = 315 m
232 ✫ Quantitative Aptitude

∴ Time taken by B to cover 350 m ● In a race A runs at 10 km/hr and B at 13


315 km/hr. A has a start of 100 m and also
= × 50 A sets off 3 minutes before B. How soon
350 will B overtake A?
= 45 seconds
That is, B will win by (46 – 45 =) 1 second. Distance covered by A in 3 minutes
3
● In a 400-metre race, the ratio of the speed = 10 ×
of two contestants A and B is 2 : 3. If A has 60
a start of 140 metres, by how much distance 1
= km = 500 m

TS
will A win? 2
Hence, A has a start of total
A gets a start of 140 m
Distance covered by (100 m + 500 m) = 600 m
B’s speed – A’s speed

H
A = 400 – 140 = 260 m
∴ We get the relation, 2 : 3 : : 260 : x = (13 – 10) km/hr
260 × 3

G
Distance covered by B = = 3 km/hr
2 3000
= 390 m = = 50 m/minute
400 – 390 = 10 metres behind the final point
A wins by 10 metres. U 60
B gains 50 m over A in 1 minute
O
● A and B run a 5-km race on a round course ∴ B will cover 600 m lead and overtake
of 400 m. If their speeds be in the ratio
H
5 : 4, how often does the winner pass the 600
A in = 12 minutes.
other? 50
_T

When A makes 5 rounds, B makes 4 ● In a game of billiards, A can give B 6 points


rounds. in 50 and A gives C 13 in 65. How many can
In order to pass each other, the difference B give C in a game of 55?
in number of rounds made by each must
C

be one. A scores 50, B scores 50 – 6 = 44


Here, A passes B each time, when A makes In a game of 65,
PS

5 rounds. while A scores 65, C scores 65 – 13 = 52


Distance covered by A in 5 rounds
52
5 × 400 ∴ A scores 50, C scores × 50 = 40
= 65
U

1000 ∴ B scores 44, C’s score = 40


= 2 km
@

40
In covering 2 km A passes B once B scores 55, C’s score = × 55 = 50
∴ In covering 5 km, A passes B 44
In a game of 55,
5 1
= = 2 times. B can give C = 55 – 50 = 5 points.
2 2
PRACTICE SESSION
Practice Exercise 17
1. If Madhu can walk a distance of 3 km in 8. If 9 oranges cost as much as 3 mangoes,
30 minutes, how far can she walk in 90 4 mangoes cost as much as 9 lemons and
minutes? 3 lemons cost 36 paise, what will be the

TS
(a) 9 km (b) 10 km cost of an orange?
(c) 11 km (d) 12 km (a) 10 p (b) 3 p
(c) 9 p (d) None of these
2. Sixteen men can reap a field in 30 days.
In how many days will 20 men reap the 9. A contractor undertakes to do a piece of

H
field? work in 40 days. He engages 150 men
(a) 13 days (b) 17 days after 30 days in addition to the 100 men

G
(c) 21 days (d) 24 days working from the beginning. He just
manages to complete the work in the
3. If eighty lamps can be lighted 5 hours per
day for 10 days for Rs 21.25, then the
number of lamps which can be lighted 4 U stipulated time. If he had not engaged the
additional men, how many days behind
schedule would the work be finished?
O
hours daily for 30 days for Rs 76.50 is (a) 5 (b) 6
(a) 100 (b) 120 (c) 3 (d) 9
H
(c) 150 (d) 160
10. A certain number of men can complete a
4. If 12 carpenters working 6 hours a day job in 30 days. If there were 5 men more,
_T

can make 460 chairs in 24 days, how many it could be completed in 10 days less.
chairs will 18 carpenters make in 36 days How many men were there in the
each working 8 hours a day? beginning?
(a) 1380 (b) 1260
C

(a) 10 (b) 15
(c) 920 (d) 1320 (c) 20 (d) 25
PS

5. Some persons can do a piece of work in 11. If the wages of 6 men for 15 days is
12 days. Two times the number of such Rs 700, the wages for 9 men for 12 days
persons will do half of that work in will be
(a) 3 days (b) 4 days (a) Rs 700 (b) Rs 840
U

(c) 6 days (d) 12 days (c) Rs 1050 (d) Rs 900


6. A certain number of men can finish a piece 12. If 48 men working 10 hours per day can
@

of work in 100 days. If, however, there complete a piece of work in 7 days, how
are 10 men less, it would take 10 more many men working 8 hours per day
days for the work to be finished. How should be able to complete the work in
many men were there originally? 21 days?
(a) 75 (b) 82 (a) 16 men (b) 20 men
(c) 100 (d) 110 (c) 36 men (d) 30 men
7. If 4 examiners have to examine a certain 13. If 15 men can build a wall 50 m long in
number of answer books in 15 days by 18 days, how long a wall can be built by
working 9 hours a day, for how many 36 men in 21 days?
hours a day should 9 examiners work in (a) 120 m (b) 130 m
12 days? (c) 140 m (d) 150 m
(a) 8 (b) 9 14. Twenty eight men are able to complete a
(c) 10 (d) 5 piece of work in 12 days working 8 hours

233
234 ✫ Quantitative Aptitude

a day. How long should 16 men work in (a) 11 (b) 10


a day if they have to complete the work (c) 9 (d) 8
in 12 days? 21. 8 children and 12 men complete a certain
(a) 14 hr (b) 12 hr piece of work in 9 days. Each child takes
(c) 13 hr (d) 11 hr twice as much time as that taken by a man
15. Three coal loading machines—each of them to finish the work. In how many days will
working 16 hours/day for 6 days at an 12 men finish the same work?
efficiency of 80%—handle 12000 tonnes of (a) 10 days (b) 12 days
coal. Find how many hours per day (c) 14 days (d) 16 days

TS
should each of two other coal loading
22. Ten men begin to work together on a job;
machines work if they work at an efficiency
but after some days, 4 of them leave. As
of 90% for 8 days handling 9000 tonnes
a result, the job which could have been
of coal?

H
completed in 40 days is completed in 50
(a) 10 hrs/day (b) 14 hrs/day
days. How many days after the
(c) 15 hrs/day (d) 12 hrs/day

G
commencement of the work did the 4 men
16. A rope makes 70 rounds of the leave?
circumference of a cylinder whose base (a) 20 days (b) 22 days
radius is 14 cm. How many times will it
go around a cylinder of radius 20 cm? U (c) 25 days (d) 26 days
O
23. 20 teachers, each working for 15 periods
(a) 98 (b) 17
a week, are required for a certain school.
(c) 49 (d) 5
How many teachers would be needed if
H
17. A contractor undertakes to build a house each teacher took 12 periods a week?
in 8 months and he plans to complete the (a) 40 (b) 32
_T

work in time with 210 men working 20 (c) 25 (d) 23


days a month at 6 hours a day. However,
he manages to get only 140 men and is 24. If 9 engines (type 1) use 24 litres of diesel
allowed 12 months to complete the work. when each works 8 hours a day, how
C

If his men work for 24 days a month, how much diesel will be required for 8 engines
many hours a day should they work, to (type 2), each working 13 hours a day,
PS

complete the work? given that 3 engines of the first type use
(a) 6 hours (b) 7 hours as much diesel as 4 engines of the second
(c) 5 hours (d) 8 hours type?
(a) 20 l (b) 25 l
U

18. 12 men take 20 days of 8 hours each to (c) 26 l (d) 30 l


do a piece of work. How many days 6
25. For completing a contract in 56 days, 104
@

hours each would 20 women take if 3


women do as much work as 2 men? men were set to work, each working 8
2
(a) 29 days (b) 30 days hours a day. After 30 days only of the
(c) 31 days (d) 32 days 5
work was finished. How many more men
19. If the rent of grazing 30 cows for 15 days need to be employed so that the work
is Rs 250, how many cows can graze for may be completed in time, each man
30 days on Rs 200. working 9 hours a day?
(a) 9 cows (b) 10 cows (a) 54 men (b) 56 men
(c) 11 cows (d) 12 cows (c) 57 men (d) 60 men
20. If 20 pumps can raise 2500 tonnes of water 26. If 12 men and 16 boys can do a piece of
in 10 days, working 5 hours a day, in how work in 5 days, and 13 men and 24 boys
many days will 15 pumps raise 1750 can do it in 4 days, then the ratio of daily
tonnes of water working 6 hours a day? work done by a man to that of a boy is:
Unit Three : Practice Session ✫ 235

(a) 2 : 1 (b) 3 : 1 found that now the provision will last for
(c) 3 : 2 (d) 5 : 4 20 days more. Find the number of men
27. If 4 men and 6 boys earn Rs 1600 in 5 in the reinforcement.
days and 3 men and 7 boys earn Rs 1740 (a) 4000 men (b) 1500 men
(c) 2800 men (d) 3000 men
in 6 days, in what time will 7 men and
6 boys earn Rs 3760? 34. If 7 spiders make 7 webs in 7 days, then
(a) 7 days (b) 8 days 1 spider will make 1 web in how many
(c) 10 days (d) 12 days days?
(a) 1 (b) 4
28. One thousand men in a fortress have

TS
(c) 7 (d) 14
provisions for 12 days. How long will the
provisions last if 200 more men join them? 35. In a camp, there is a meal for 120 men
(a) 8 days (b) 10 days or 200 children. If 50 children have taken
the meal, how many men will be catered

H
(c) 12 days (d) 14 days
to with the remaining meal?
29. In a mess, there is a consumption of 1350 (a) 50 (b) 40

G
litres of milk for 70 students for 30 days. (c) 30 (d) 20
How many students will consume 1710
litres of milk in 28 days? 36. A fort had provision of food for 150 men
(a) 77
(c) 95
(b) 83
(d) 101 U for 45 days. After 10 days, 25 men left
the fort. The number of days for which
O
the remaining food will last is
30. A garrison of 3300 men had provisions for (a) 34 (b) 37
32 days, when the provisions are given (c) 40 (d) 42
H
at the rate of 850 gms per head. At the
37. 20 men complete one-third of a piece of
end of 7 days a reinforcement arrives and work in 20 days. How many more men
_T

it was found that now the provisions will should be employed to finish the rest of
last 17 days more, when given at the rate the work in 25 more days?
of 825 gm per head. What is the strength (a) 15 (b) 14
of the reinforcement? (c) 13 (d) 12
C

(a) 1300 (b) 1700


(c) 1900 (d) 2400 38. 10 pumps of equal capacity can fill a tank
PS

in 6 days. If the tank has to be filled in


31. A garrison has provision for x days. After 3 days, the number of extra pumps needed
1
10 days, of the men leave and it is is
5 (a) 10 (b) 15
U

found that the provision will now last just


(c) 20 (d) 30
as long as before. Find the value of x. 1
(a) 15 days (b) 50 days 39. 12 men and 18 boys working 7 hours
@

2
(c) 20 days (d) 40 days a day can do a work in 60 days. If one
32. A certain number of artisans can complete man works equal to 2 boys, the number
a shoe fabrication consignment in 16 days. of boys required to help 21 men to do
8 additional artisans had to be employed twice the work in 50 days, working 9
for the same consignment and together hours a day, will be
they completed it in 4 days less than the (a) 30 (b) 42
original estimate. The number of artisans (c) 48 (d) 90
initially employed was 40. If 16 men working 7 hours a day can
(a) 18 (b) 20 plough a field in 48 days, in how many
(c) 24 (d) none of these days will 14 men working 12 hours a day
33. A garrison of 3000 men has provisions for plough the same field?
45 days. At the end of 15 days, a (a) 30 (b) 32
reinforcement of men arrives and it is (c) 35 (d) 46
236 ✫ Quantitative Aptitude

Answers with Solutions


1. Ans. (a) So lamps = L, days = D and hours = H,
Sol. More time, more distance is covered and we have,
(direct proportion) L1 × D1 × H1 L2 × D2 × H 2
∴ 30 : 90 :: 3 : x ⇒ =
W1 W2
90 × 3
or x = Here L1 = 80, D1 = 10, H1 = 5, W1 = 21.25,
30 D2 = 30, H2 = 4, W2 = 76.50 and L2 = ?
= 9 km.
80 × 10 × 5 L2 × 30 × 4

TS
Alternative Method ⇒ =
The unitary method will also help solve 21.25 76.50
this problem. 80 × 10 × 5 × 76.50
⇒ L2 =
In 30 minutes Madhu walks 3 km 21.25 × 30 × 4

H
3 = 4×5×6
In 1 minute she walks km = 120.
30

G
3 4. Ans. (a)
In 90 minutes she walks × 90 = 9 km. Sol. Here we have N1 and N2, T1 and T2,
30 and W1
2. Ans. (d)
Sol. More men, fewer days (inverse
proportion) U Time = Days × Hours or D × H
So we have
O
N1D1H1 N 2D2 H 2
20 : 16 : : 30 : x W1 = W2
H
20 30 12 × 24 × 6 18 × 36 × 8
⇒ = ⇒ =
16 x 460 W2

_T

20x = 16 × 30 460 × 18 × 36 × 8
16 × 30 ⇒ W2 =
∴ x = 12 × 24 × 6
20 = 460 × 3
= 1380
C

= 24 days
So 18 carpenters will make 1380 chairs
So, 16 men will take 24 days to reap the under the given conditions.
PS

field. 5. Ans. (a)


Alternative Method Sol. N2 = 2 times N1
Use the formula,
N1 T1 = N2 T2 2 N1 × T2
U

N1 × 12 =
Here N1 = 16 men 1
T1 = 30 days 2
@

N2 = 20 men N1 × 12 = 2N 1 × T2 × 2
T2 = ? N1 × 12
So, ∴ T2 = 2 N × 2 = 3 days.
1
16 × 30 = 20 × T2 6. Ans. (d)
16 × 30 Sol. Let the number of men who finish the
⇒ = T2 work in 100 days be M1. Then (M1 – 10)
20
⇒ T2 = 24 days men will finish the work in (100 + 10 =)
110 days. We have found M1. Work
3. Ans. (b) being the same,
Sol. Here, the number of men is replaced by M1 × 100 = (M1 – 10) × 110
number of lamps. Still, the same formula 100M1 = 110M1 – 1100
can be applied. Also, work done will 1100 = 110M1 – 100M1
correspond to the wage/money paid 1100
for it. M1 = = 110.
10
Unit Three : Practice Session ✫ 237

7. Ans. (d) ⇒ W2 × 6 × 15 = 9 × 12 × 700


Sol. N1 T1 = N2 T2 9 × 12 × 700
Here N1 is 4 examiners, N2 = 9 examiners ⇒ W2 =
6 × 15
T1 = 15 days × 9 hours,
= 840
T2 = 12 days × x hours
Wages of 9 men working for 12 days
Work being the same, will be Rs 840.
N1 T1 = N2 T2
⇒ 4 × 15 × 9 = 9 × 12 × x 12. Ans. (b)
Sol. If Men = M, Days = D and Hour = H
4 × 15 × 9

TS
⇒ x = = 5 hours. M 1 D1 H1 = M 2 D2 H 2
9 × 12
⇒ 48 × 10 × 7 = M2 × 8 × 21
8. Ans. (c)
Sol. Let the cost of an orange be x. 48 × 10 × 7

H
M2 =
Then x paise = 1 orange 8 × 21
9 oranges = 3 mangoes = 20 men.

G
4 mangoes = 9 lemons 13. Ans. (c)
3 lemons = 36 paise
N1 T1 N 2 T2
⇒ x × 9 × 4 × 3 = 1 × 3 × 9 × 36
∴ x =
1 × 3 × 9 × 36
9×4× 3
= 9 paise. U Sol. W1 = W2
Here N1 = 15, T1 = 18, W1 = 50, N2 = 36 and
O
9. Ans. (a) T2 = 21
Sol. [100 × 30 + 150 × 10] men working for 1 15 × 18 36 × 21
H
⇒ = W
day can finish the work. 50 2
⇒ 3000 + 1500 = 4500 men can finish it in 50 × 36 × 21
_T

1 day. W2 =
15 × 18
⇒ 100 men can finish the work in = 140 metres.
4500 14. Ans. (a)
= 45 days
C

100 Sol. Work being the same,


Stipulated time = 40 days. N1 T1 = N2 T2
PS

So, work would have been 5 days behind Here T = days into hours
schedule.
N1 = 28, N2 = 16
10. Ans. (a)
T1 = 12 × 8, T2 = 12 × H2
Sol. Let x be the number of men in the
U

beginning. Then, We have to find hours of work done by


using the formula 16 men in a day.
@

N1 T1 = N2 T2 So we have
⇒ x × 30 = (x + 5) × 20 28 × 12 × 8 = 16 × 12 × H2
30x = 20x + 100 28 × 12 × 8
10x = 100 ⇒ = H2
16 × 12
x = 10. ⇒ H2 = 14 hours.
11. Ans. (b) 15. Ans. (d)
Sol. Here instead of work, it will be wages
Sol. Here number of men becomes number
(W). of engines. Total work is the total
N1 T1 N 2 T2 output (coal handled) by the machine.
W1 = W2 Efficiency (E) has to be included.
∴ The formula becomes
6 × 15 9 × 12 N1 D1 H1 E1 N 2 D2 H 2 E2
⇒ = =
700 W2 W1 W2
238 ✫ Quantitative Aptitude

3 × 6 × 16 × 80 2 × 8 × H 2 × 90 19. Ans. (d)


⇒ = Sol. More days, less cows (indirect)
12000 × 100 9000 × 100 Less rent, less cows (direct)
100 × 9000 × 3 × 6 × 16 × 80
∴ H2 =
12000 × 100 × 2 × 8 × 90
= 3 × 4 = 12 hours/day.
Days 30 : 15
}
Rent 250 : 200 : : 30 : x
15 × 200 × 30
16. Ans. (c) ∴ x =
Sol. Greater radius means larger circum- 30 × 250
ference and fewer rounds by the rope. = 12 cows.

TS
Here the number of rounds are N1 and Alternative Method
N2 and the time can be equated with the You can reduce the problem to the
radii, R1 and R2. We have to find N2. N1 T1 = N2 T2 formula

H
So, N1 R1 = N2 R2 Here N1 = 30 cows, N2 is to be found
⇒ 70 × 14 = N2 × 20 T1 = 15 days, T2 = 30 days

G
70 × 14 W1 = Rs 250, W2 = Rs 200
⇒ = N2 (Instead of work/wages, it is rent)
20
⇒ N2 = 49 rounds.
U Wages or rent will be divided by number
and time, so we have
W1 W2
O
17. Ans. (c) =
Sol. N1 T1 = N2 T2 N1 T1 N2 T2
⇒ W1 × N2 × T2 = W2 × N1 × T1
H
N1 = 210, N2 = 140
T1 and T2 are Months × Days × Hours or ⇒ 250 × N2 × 30 = 200 × 30 × 15
200 × 30 × 15
_T

M1 × D1 × H1 and M2 × D2 × H2 ⇒ N2 = = 12 cows.
M1 = 8, D1 = 20, H1 = 6; 250 × 30
20. Ans. (d)
M2 = 12, D2 = 24, H2 = ? Sol. Less pumps, more days (indirect)
C

So, Less water, less days (direct)


210 × 8 × 20 × 6 = 140 × 12 × 24 × H2 More working hours, less days (indirect)
PS

210 × 8 × 20 × 6 Pumps 15 : 20
⇒ = H2 Water 2500 : 1750 : : 10 : x
140 × 12 × 24 Working hours 6 : 5

U

H2 = 5 hours.
We have
18. Ans. (a) 15 × 2500 × 6 × x = 20 × 1750 × 5 × 10
@

Sol. 3 women = 2 men, so 20 women = 11 men 20 × 1750 × 5 × 10


⇒ x =
Now, less men, more days (indirect) 15 × 2500 × 6
less hours, more days (indirect) = 7.77 =8 days (approximate).
N1 T1 = N2 T2 21. Ans. (b)
Sol. 1 man’s work = 2 children’s work
Here N1 = 12,
⇒ 8 children and 12 men = 16 men
N2 = 11 (11 men = 20 women); Less men implies more days.
T1 = D1 × H1 = 20 × 8, T2 = D2 × H2 = D2 × 6 ⇒ 16 : 12 :: x : 9
⇒ 12 × 20 × 8 = 11 × D2 × 6 16 x
=
12 9
12 × 20 × 8 16 × 9
D2 =
11 × 6 ⇒ x =
12
= 29.09 days or 29 days approximately. ⇒ x = 12 days.
Unit Three : Practice Session ✫ 239

22. Ans. (c) 25. Ans. (b)


Sol. Total volume of the job = 10 × 40 Sol. Here you need to locate N1, R1, R2, T1,
= 400 ‘mandays’. T2, W1 and W2 correctly so as to get N2
Let x be the number of days 4 men work. and then calculate the number of
⇒ 10 × 40 = (4 × x) + (6 × 50) additional men required.
400 = 4x + 300
N1 = 104, T1 = 30 days (not 56)
4x = 400 – 300
4x = 100 2
100 R1 = 8, W1 =
∴ x = = 25 days. 5

TS
4 N2 = 104 + x
23. Ans. (c)
Sol. Let x be the number of teachers required. (x being the additional men)
T2 = 56 – 30 = 26 (work needs to be
Teachers Periods

H
completed within the original time)
20 15
x 12 R2 = 9

G
Indirect proportion 2 3
W2 = 1 – = (remaining work)
20 : x :: 12 : 15 5 5
N 1 × T1 × R1 N 2 × T2 × R2
⇒ x =
15 × 20
12
= 25.
U So
W1
104 × 30 × 8
=
W2
(104 + x ) × 26 × 9
O
24. Ans. (c)
⇒ =
Sol. You have to find diesel consumption for 2 3
1 unit in 1 hour, given 3 engines of type 5 5
H
1 are equivalent to 4 engines of type 2. 104 × 30 × 8 5 3
In case of type 1 engine, 3 engines use ⇒ × × = 104 + x
_T

26 × 9 2 5
1 unit in 1 hour ⇒ 160 – 104 = x
1 ⇒ x = 56 men.
∴ 1 engine uses unit in an hour.
3
C

If 4 engines of type 2 use 1 unit of diesel 26. Ans. (a)


in 1 hour, Sol. If Men is M and Boys is B
PS

1 12 M + 16 B can finish a work in 5 days.


1 engine uses unit in 1 hour ⇒ 12 × 5 M + 16 × 5 B can finish the work
4
We now have the rates (R) of diesel in 1 day.
consumption ⇒ 60 M + 80 B can finish the work in 1 day.
U

N1 × T1 × R1 N 2 × T2 × R2 ...(i)
= Similarly 13 × 4 M + 24 × 4 B can finish
@

W1 W2 the work in 1 day.


(W1 and W2 being consumption of diesel ⇒ 52 M + 96 B can finish in 1 day.
in the two cases) ...(ii)
So, 1 1 From (i) and (ii), we get
9× 8× 8 × 13 × 60 M + 80 B = 52 M + 96 B
3 = 4
24 60 M – 52 M = 96 B – 80 B
W2
⇒ 8 M =16 B
1 1 1 1
⇒ 9 × 8 × × = 8 × 13 × × M =2 B
3 24 4 W2 ⇒ The ratio M : B = 2 : 1.
26
⇒ 1 =
W2 27. Ans. (b)
⇒ W2 = 26 N1 D 1 N 2 D 2 N 3 D 3
Sol. = =
So, in the second case, 26 litres of diesel W1 W2 W3
is required. Here, N = Men (M) + Boys (B)
240 ✫ Quantitative Aptitude

So we have
(4 M + 6 B) × 5 (3 M + 7 B) × 6 (7 M + 6 B) × D3
= =
Litres 1350 : 1710
Days 28 : 30 } : : 70 : x

1600 1740 3760 ∴ 1350 × 28 × x = 1710 × 30 × 70


4 M + 6B 3 M + 7B (7 M + 6 B) × D3 1710 × 30 × 70
⇒ = = or x =
320 290 3760 1350 × 28
Taking just the first two relations, = 95 students.
29 (4 M + 6 B) = 32 (3 M + 7 B) Alternative Method
116 M + 174 B = 96 M + 224 B N1 D1 N 2 D2
⇒ 116 M – 96 M = 224 B – 174 B =
W1 W2

TS
⇒ 20 M = 50 B 70 × 30 N 2 × 28
50 5 ⇒ =
M = B = B. 1350 1710
20 2 70 × 30 × 1710
Now, ⇒ = N2 = 95.

H
1350 × 28
In 5 days, 4 M + 6 B earn Rs 1600 30. Ans. (b)
1600 Sol. 3300 men taking 850 gm/head have

G
In 1 day, 4 M + 6 B earn = Rs 320 provisions for (32 – 7) or 25 days. How
5
∴ Substituting value of 1 man in boys, many men taking 825 gms/head have
5
4 × B + 6 B earn Rs 320
2 U provisions for 17 days is another form
of the question. Now,
O
16 B earn Rs 320 Less ratio per head, more men (indirect)
Less days, more men (indirect)

}
320
1 B earns = Rs 20 a day. Ratio 825 : 850
H
16 Days 17 : 25 : : 3300 : x
Now we can solve for the third set to
850 × 25 × 3300

_T

get D3: x = = 5000


5 825 × 17
⇒ 7 M + 6B = 7 × B + 6 B So, the strength of new reinforcement
2 = (5000 – 3300) = 1700.
35 + 12
B = 47 B
C

= 31. Ans. (b)


2 2 Sol. Let M be the number of men and x be
W3 the number of days.
PS

= Rs 20 4
N 3 D3
M (x – 10) = M × x
3760 5
⇒ = Rs 20 × D3 ⇒ 5M × (x – 10) = 4M x
47
U

5x – 50 = 4x
2 3760 5x – 4x = 50
∴ No. of days D3 =
47 ⇒ x = 50 days.
@

× 20
2 Shortcut
3760 As the number of days will be same
= = 8 days. after the men leave, the
470 1
28. Ans. (b) Number of days = 10 ÷
Sol. Here amount of work, i.e., amount of 5
provisions is same = 10 × 5 = 50 days.
∴ 1000 × 12 = (1000 + 200) × T2 32. Ans. (c)
12000 = 1200 × T2 Sol. Let x be the original number of artisans
12000 employed.
T2 = x × 16 = (x + 8) × 12
1200
16x = 12x + 96
T2 = 10 days. 16x – 12x = 96
29. Ans. (c) 4x = 96
Sol. More litres, more students (direct)
96
Less days, more students (indirect) x = = 24.
4
Unit Three : Practice Session ✫ 241

33. Ans. (b) Days 25 : 20


Sol. Let x be the number of men in 1 2 : : 20 : x
⇒ Work :
reinforcement 3 3
∴ 3000 × (45 – 15) = (3000 + x) × 20 2
× 20 × 20
3000 × 30
3000 + x = ⇒ x = 3
20 1
× 25
3000 + x = 4500 3
⇒ x = 1500. 2 × 20 × 20
34. Ans. (c) =

TS
25
Sol. As we can see in the question, all the
= 32 men.
ratios are 7 : 1 or 1 : 7 so answer must
More men to be employed
be 1 or 7.
= 32 – 20 = 12 men.

H
Spiders Webs Days 38. Ans. (a)
7 7 7
Sol. Fewer days, more pumps

G
1 1 x
If N2 is the number of pumps required

Spiders 1 : 7
}::7 : x for 3 days’ work,


Webs 7 : 1
7 × 1× 7 U N1 × D 1 = N2 × D2
(Work being same)
O
x = ⇒ 10 × 6 = N2 × 3
1× 7
=7 10 × 6
⇒ N2 = = 20 pumps
H
One Spider will make a web in 7 days. 3
35. Ans. (c) So the extra pumps required are
_T

Sol. Given that 120 men= 200 children 20 – 10 = 10 pumps.


120 39. Ans. (b)
∴ 50 children = × 50 men
200 Sol. 1 man = 2 boys
C

= 30 men. ⇒ 12 M, 18 Boys
36. Ans. (d) = 12 M + 9 M = 21 M
PS

Sol. After 10 days, 150 men had food for 35 M1 × D1 × R1 M 2 × D2 × R 2


days. =
W1 W2
Suppose 125 left men had food for x
15
U

days, then 21 × × 60 M2 × 50 × 9
Men Days 2 =
150 35 1 2
@

125 x 21 × 15 × 60 × 2
⇒ M2 =
2 × 50 × 9
∴ 125 : 150 :: 35 : x
= 7 × 3 × 2 = 42 Men
150 × 35 ⇒ Boys required = 42 – 21 = 21 Men
⇒ x = = 42 boys.
125
= 42. 40. Ans. (b)
Sol. N1 T1 = N2 T2
37. Ans. (d) Here T = days × hour
Sol. Men Days Work So, we have
1 16 × 7 × 48 = 14 × 12 × D2
20 20
3 16 × 7 × 48
1 2 D2 =
x 25 1− = 14 × 12
3 3 = 32 days.
242 ✫ Quantitative Aptitude

Practice Exercise 18
1. 24 men can assemble a machine in 12 days. (a) Rs 90 (b) Rs 85
In how many days will 36 persons (c) Rs 80 (d) Rs 75
assemble the same machine? 8. A can do a work in 6 days, while B and
(a) 8 days (b) 12 days
C can do the same work in 12 and 18 days
(c) 16 days (d) 20 days
respectively. If they start together to do
2. Aditya and Abhinav together can do a the work on the same day and C stops
certain piece of work in 10 days. Aditya working after 3 days, then the time

TS
alone can do the same work in 15 days. required by A and B together to finish the
Abhinav alone will be able to do the same rest of the work will be
job in (a) 1 day and 12 hours
(a) 15 days (b) 20 days

H
(b) 16 hours
(c) 25 days (d) 30 days (c) 8 hours
3. If 9 girls can prepare 135 garlands in 3 (d) 1 day and 6 hours

G
hours, then the number of girls needed to 9. A and B did a piece of work together and
prepare 270 garlands in one hour will be received Rs 300. If A alone can do a piece
(a) 2 7
(c) 3 6
(b) 1 8
(d) 5 4 U of work in 2 weeks and B alone in 3
weeks, how should the money be divided
O
4. 16 men or 24 women can do a piece of between them?
work in 20 days. The number of days (a) A = Rs 180 and B = Rs 120
H
needed to complete the job, if 20 men and (b) A = Rs 120 and B = Rs 180
30 women are employed to do the same (c) A = Rs 150 and B = Rs 150
_T

piece of work, is (d) A = Rs 200 and B = Rs 100


(a) 1 6 (b) 1 2 10. A piece of work can be done by 6 men
(c) 1 0 (d) 8 and 5 women in 6 days or 3 men and 4
5. Ram and Suresh are able to dig a trench
women in 10 days. How many days will
C

in 8 days. If Ram alone can manage to dig


it take by 9 men and 15 women to do
it in 20 days, how long will Suresh alone
the same work?
PS

take to dig it?


(a) 1 day (b) 2 days
1 1 (c) 3 days (d) 4 days
(a) 14 days (b) 13 days
2 3
2 1 11. A can do a work in 7 days and B in 8
U

(c) 12 days (d) 11 days days. A and B undertake to do this work


3 4
for Rs 56. If with the assistance of a boy
6. A group of 10 students, working one hour
@

they finish the work in 3 days then the


per day, complete a piece of work in 12
boy will get
days. If there are 12 students in the group
(a) Rs 5 (b) Rs 11
and all of them work one hour per day,
(c) Rs 19.50 (d) Rs 25
then they will be able to complete the
work in 12. A takes as much time as B and C together
(a) 8 days (b) 9 days to finish a job. A and B working together
(c) 10 days (d) 11 days finish the job in 10 days. C alone can do
7. A can do a piece of work in 8 days and the same job in 15 days. In how many
B can do the same piece of work in 12 days can B alone do the same work?
days. A and B together complete the same (a) 30 days (b) 60 days
piece of work and get Rs 200 as the (c) 90 days (d) 120 days
combined wage. B’s share of the wage 13. 3 men or 5 women or 8 boys can do a
was work in 30 days. Then in how many days
Unit Three : Practice Session ✫ 243

will 1 man, 1 woman and 1 boy do the 1 1


work? (a) 4 days (b) 5 days
2 2
(a) 45.5 days (b) 40 days 1 1
(c) 38 days (d) 35.5 days (c) 7 days (d) 9 days
2 2
14. Rajat can do a piece of work in 8 days and
Raman can do the same work in 12 days. 21. A can do a piece of work in 10 days, B
How long will they take, if they work in 15 days. They work for 5 days. The
together? rest of the work was finished by C in 2
days. If they get Rs 1500 for the whole
4
(a) 4 days (b) 3 days work, the daily wages of B and C were

TS
5 (a) Rs 225 (b) Rs 300
7 5 (c) Rs 354 (d) Rs 397
(c) 2 days (d) 2 days
8 7
22. A, B and C together can finish a work

H
15. 45 women get Rs 15525 for 48 days of in 10 days. All the three started working
work. How many men must work 16 days at it together and after 4 days A left. Then
to receive Rs 5750, the daily wages of a

G
B and C together completed the work in
man being double those of a woman? 10 more days. A alone could complete the
(a) 20 men (b) 25 men work in
(c) 30 men (d) 35 men
16. A, B and C can do a work in 10 days. U (a) 23 days
(c) 25 days
(b) 24 days
(d) 26 days
O
The three together work for 4 days. Then 23. A certain industrial loom weaves 0.128
A left the work. B and C finished the metres of cloth every second.
H
remaining work in 10 days. In how many Approximately, how many seconds will it
days will A alone do the work? take for the loom to weave 25 metres of
_T

(a) 10 days (b) 14 days the cloth?


(c) 18 days (d) 25 days
(a) 200 seconds (b) 195 seconds
17. A and B can do a job in 10 days. A is (c) 190 seconds (d) 185 seconds
1
C

1 times as efficient as B. The same job 24. 30 men can produce 1500 units in 24 days
2
can be done by A alone in working 6 hours a day. In how many days
PS

(a) 10 days (b) 14 days can 18 men produce 1800 units working
1 2 8 hours a day?
(c) 15 days (d) 16 days (a) 30 days (b) 34 days
4 3
(c) 36 days (d) 38 days
U

18. A does half as much work as B in three-


fourths of the time. If together they take 25. Two candles of the same height are
18 days to complete the work, how much lighted at the same time. The first is
@

time shall B take to do it? consumed in 4 hours and the second in


(a) 21 days (b) 30 days 3 hours. Assuming that each candle burns
(c) 34 days (d) 38 days at its own constant rate, find the number
19. 6 men and 8 women can complete a work of hours, after being lighted, after which
in 9 days, while 5 men and 9 women can the first candle will be twice the height
complete it in 10 days. In how many days of the second candle.
will 15 women complete it? (a) 1.5 hours (b) 1.9 hours
(a) 40 days (b) 21 days (c) 2.4 hours (d) 2.7 hours
(c) 38 days (d) 15 days 26. A certain number of artisans can complete
20. A can do a certain job in 12 days. B is a shoe fabrication consignment in 16 days.
60% more efficient than A. How many 8 additional artisans had to be deployed
days does B alone take to do the same for the same consignment and together
job? they completed it in 4 days less than the
244 ✫ Quantitative Aptitude

earlier estimate. The number of artisans 1


33. Raman can do of a piece of work in
initially employed was 3 1
15 days, while Ashmit can do of the
(a) 24 (b) 20 4
same work in 15 days. In how many days
(c) 18 (d) none of these
can both do it together?
27. 5 men can do a piece of work in 12 days 5 5
and 10 women can do it in 10 days. In (a) 25 days (b) 26 days
7 8
how many days can 3 men and 5 women 5 5
(c) 27 days (d) 28 days
do the same piece of work? 6 9
(a) 12 (b) 6 34. A working alone would take 14 hours

TS
(c) 10 (d) 8 more to complete the job than if A and
28. A and B can weave a carpet in 10 days B worked together. If B worked alone, he
1
and 15 days respectively. They begin to will take 3 hours more to complete the

H
work together but B leaves after 2 days. 2
job than what A and B working together
In what time will A complete the remaining
will take. How much time would they

G
work?
1 2 take if both A and B worked together?
(a) 6 days (b) 6 days (a) 5 days (b) 6 days
3
(c) 7 days
3
(d) 8 days
U (c) 7 days (d) 8 days
35. A computer can perform 30 identical tasks
O
29. A garrison of 2000 men had enough food in six hours. At that rate, what is the
to last for 30 days. After 10 days 500 more minimum number of computers that
H
men joined. How long did the food last should be assigned to complete 80 of the
then? tasks within 3 hours?
_T

(a) 20 days (b) 18 days (a) 9 (b) 8


(c) 17 days (d) 16 days (c) 7 (d) 6
30. To finish a certain job X takes twice as 36. Pooja and Meenu can complete a work in
long as Y and Z together and Z three
C

10 and 15 days respectively. Pooja starts


times as long as X and Y together. If X, the work and after 5 days, Meenu also
Y and Z working together complete the joins her. Together, they would be able to
PS

job in 6 days, how long would X take to complete the work in


complete the work alone? (a) 8 days (b) 9 days
(a) 16 days (b) 18 days (c) 11 days (d) 12 days
U

(c) 22 days (d) 26 days 37. A and B together can finish a work in 30
31. A piece of work can be done by 12 men days. They worked at it for 20 days and
@

and 10 women in 12 days or 6 men and then B left. The remaining work was
8 women in 20 days. In how many days done by A alone in 20 more days. A alone
can it be done by 18 men and 30 women? could finish the work in
(a) 10 days (b) 8 days (a) 50 days (b) 48 days
(c) 6 days (d) 4 days (c) 54 days (d) 60 days
32. X, Y and Z can do a certain job in 8, 10 38. A does half as much work as B in three-
and 8 days respectively. How long would fourths of the time. If together they take
they take to complete the same job when 18 days to complete a work, how much
all work together? time shall B take to do it?
6 (a) 40 days (b) 35 days
(a) 2 days (b) 2 days (c) 30 days (d) None of these
7
8 39. 10 men can finish a piece of work in 10
(c) 1 days (d) 1 day
9 days whereas it takes 12 women to finish
Unit Three : Practice Session ✫ 245

it in 10 days. If 15 men and 6 women (a) 12 (b) 13


undertake to complete the work, how (c) 14 (d) 15
many days will they take to complete it?
43. Ashok takes twice as much time as Arun
(a) 11 (b) 5
and thrice as much time as Alok to finish
(c) 4 (d) 2
a piece of work. They together finish the
40. 3 men or 7 women can do a piece of work work in one day. In how much time can
in 32 days. The number of days required Ashok finish the work?
by 7 men and 5 women to do a piece of (a) 6 days (b) 18 days
work twice as large is (c) 2 days (d) 12 days

TS
(a) 21 (b) 19
44. A can do a piece of work in 6 days, B
(c) 27 (d) 36
in 8 days and C in 12 days. B and C
41. A and B can do a piece of work in 12 together do it for 2 days, and then C is

H
days. B and C can together do it in 15 replaced by A. Find when the work will
days. If A is twice as good as C in the be finished?
work, find how much time B alone will

G
(a) 2 days (b) 4 days
take to do it. (c) 6 days (d) 8 days
(a) 20 days (b) 16 days
(c) 24 days (d) 25 days
42. 12 men can complete a piece of work in U 45. A can do a piece of work in 120 days;
B can do it in 150 days. They worked
together for 20 days, then B leaves and
O
4 days, while 15 women can complete the
A continues the work alone. 12 days after
same work in 4 days. 6 men start working
that, C joins A and the work is completed
H
on the job and after working for two
days, all of them stopped working. How in 48 days more. In how many days can
many women should be put on the job C do it if he works alone?
_T

to complete the remaining work, if it is (a) 180 days (b) 80 days


to be completed in 3 days? (c) 120 days (d) 240 days

Answers with Solutions


C

1. Ans. (a) Shortcut


PS

Sol. 24 men can assemble a machine in 12 Abhinav alone can finish the work in
days xy
1 man can assemble a machine in 12 × 24 y – x days
days (where x = the number of days in which
U

∴ 36 men can assemble a machine in they both do the work, and


12 × 24 y = the number of days in which Aditya
@

= = 8 days.
36 alone does the work)
10 ×15
2. Ans. (d) = = 30 days.
15 – 10
Sol. Work done by Aditya and Abhinav in
1 3. Ans. (d)
one day = Sol. Using formula N1W2D1 = N2W1D2
10
1
Work done by Aditya in one day = ⇒ 9 × 270 × 3 = N2 × 135 × 1
15
9 × 270 × 3
Work done by Abhinav in one day ∴ N2 =
1 1 1 135
= – = = 54
10 15 30
4. Ans. (d)
∴ Abhinav alone can finish the work in 30 Sol. 16 men = 24 women
days. or 4 men = 6 women
246 ✫ Quantitative Aptitude

∴ 20 men + 30 women 9. Ans. (a)


= 30 women + 30 women Sol. Ratio of working capacity of A and B is
= 60 women 3:2
Now 24 women can do a piece of work 3
in 20 days. ∴ A’s share = × 300
5
∴ 60 women can do a piece of work in = Rs 180
20 × 24 2
= = 8 days. B’s share = × 300
60 5
5. Ans. (b) = Rs 120

TS
Sol. We have x = 8 and y = 20 10. Ans. (c)
xy Sol. 6 men + 5 women = 6 days
Using the formula y – x days
or 3 men + 4 women = 10 days
Suresh alone can do the work in

H
Work done by each group in one day
8 × 20 1
20 – 8 ⇒ 6 men + 5 women = ...(i)
6

G
160 40 1
= = or 3 men + 4 women = ...(ii)
12 3 10
= 13
1
3
days.
U Now, by solving equations (i) and (ii)
we get
1 1 1
O
6. Ans. (c) 3 women = – =
5 6 30
Sol. Use formula N1W2D1 = N2W1D2 1 woman can do the work in
H
Here work is the same, W2 = W1 1 1
Now we have the formula = days
3 × 30 90
1
_T

N 1D 1 = N 2 D 2
and 1 man can do the work in days.
⇒ 10 × 12 = 12 × D2 54
9 15
10 ×12 ∴ 9 men + 15 women = +
54 90
D2 =
C

12 1 1
= +
= 10 days. 6 6
PS

7. Ans. (c) 2 1
= =
Sol. Ratio of A : B = 8 : 12 = 2 : 3 6 3
2 Therefore 9 men and 15 women can
∴ B’s share = 200 × finish the work in 3 days.
U

2+3
2
= 200 × = Rs 80. 11. Ans. (b)
5
@

Sol. A’s 3 days’ work + B’s 3 days’ work


8. Ans. (c)
+ Boy’s 3 days’ work = 1
Sol. Work done by A, B and C in 3 days
3 3
1 1 1 ∴ Boy’s 3 days’ work = 1 – +
= + + ×3 7 8
6 12 18
11 11
= of the work =
56
12
11 1 3 3 11
∴ Remaining work = 1 – = Ratio of shares = : :
12 12 7 8 56
Work done by A and B in one day 3 × 56 3 × 56 11× 56
1 1 1 = : :
= + = 7 8 56
6 12 4 = 24 : 21 : 11
∴ A and B complete the work in
56
1 1 1 ∴ Boy’s share = ×11 = Rs 11.
4× = day i.e., × 24 = 8 hours. 24 + 21 + 11
12 3 3
Unit Three : Practice Session ✫ 247

12. Ans. (b) 3


Now, work is done by B and C in 10
Sol. (A + B) + C can do work in 5
days
15 ×10
= 6 days Whole work will be done by B and C in
15 + 10 5 50
Since A’s days = (B + C)’s days = 10 × = days
3 3 1
B + C can do in 6 × 2 = 12 days (A + B + C )’s one day work =
10
15 ×12 3
∴ B [B = (B + C) – C] can do it in (B + C )’s one day work =
15 – 12 50
= 60 days. 1 3 2 1

TS
A’s one day work = – = =
13. Ans. (a) 1 10 50 50 25
1
Sol. = ∴ A alone could complete the work in 25
1 1 1 1 1 1 1
+ + + + days.
30 × 3 30 × 5 30 × 8 30 3 5 8

H
17. Ans. (d)
=
1 Sol. If B is 1, then A is 1 1 ,
1 40 + 24 + 15 2

G
1 1
30 120 So 1 + 1 time = 2 times efficient.
2 2
1
=
1 79
30 120 U ∴
A and B complete the work in 10 days.
B will do the work in 10 × 2 days
1
O
2
30 ×120 = 25 days
= 2
79 ∴ A can finish the work in 25 × days
H
= 45.5 days 3
2
14. Ans. (a) = 16 days
3
_T

Sol. Here x = 8 and y = 12 18. Ans. (b)


∴ Working together, Rajat and Raman will Sol. Suppose B takes x days to do the work.
xy
complete the work in 33x
x + y days ∴ A takes 2 × x = days to do it
C

8 ×12 96 4 4 2
= = 4 days 1
8 + 12 20 5
(A+B)’s 1 day work =
PS

15. Ans. (b) 18


15525
Sol. Wage of a woman for a day = 1 2 1
45 × 48 ∴ + =
115 x 3x 18
= Rs ⇒ x = 30 days.
U

16
Thus, wage of a man for a day 19. Ans. (b)
115 115 Sol. Let 1 man’s 1 day’s work = x
@

= 2× = Rs and 1 woman’s 1 day’s work = y


16 8
1
Number of men Then, 6x + 8y =
Total wages 9
= 1
Number of days × 1 man’s 1 day’s wage and 5x + 9y =
10
By solving the above two equations, we
5750 × 8 get
= = 25 men. 1
16 ×115 y =
315 1
16. Ans. (d) ∴ 1 woman’s 1 day work =
315
Sol. Work done by A, B and C in 4 days ⇒ 15 women’s 1 day’s work
1 2 1 1
= ×4 = = × 15 =
10 5 315 21
2 3 Hence, 15 women will complete the
Remaining work = 1 – =
5 5 work in 21 days.
248 ✫ Quantitative Aptitude

20. Ans. (c) 3


Sol. Ratio of times taken by A and B (B + C)’s 1 day work =
50
= 160 : 100 1 3 2 1
A’s 1 day work = – = =
= 8 : 5 10 50 50 25
Suppose B alone takes x days to do the ∴ A alone could complete the work in 25
job. days.
Then, 8 : 5 :: 12 : x
⇒ 8 x = 5 × 12 23. Ans. (b)
15 1 25
⇒ x = = 7 days. Sol. = 195 seconds.

TS
2 2 0.128
24. Ans. (c)
21. Ans. (a)
Sol. 30 men can produce 1500 units in 24 days
1 1 working 6 hours a day, i.e., 24 × 6 = 144
×5 =

H
Sol. Part of work done by A =
10 2 hours.
1 1 30 men can produce 1800 units in
Part of work done by B = ×5 =

G
15 3 144 864
×1800 = hours
1 1 1 1500 5
Part of work done by C = 1 – + =
2 3 6
So, A’s share : B’s share : C’s share U 18 men can produce 1800 units in
864 30
× hours
O
1 1 1 5 18
= : : =3:2:1 = 288 hours
2 3 6
= 36 days of 8 hours each.
H
3
∴ A’s share = Rs × 1500 = Rs 750 Shortcut
6
_T

2 Using N, W2 D1 T1 = N2 W1 D2 T2
B’s share = Rs × 1500 = Rs 500
6 30 × 1800 × 24 × 6 = 18 × 1500 × D2 × 8
1
C’s share = Rs ×1500 = Rs 250 30×1800×24×6
6 = D2
C

18×1500×8
750
A’s daily wage = Rs = Rs 150 D2 = 36 days.
PS

5
500 25. Ans. (c)
B’s daily wage = Rs = Rs 100 Sol. Quantity of the Ist candle burnt in one
5 1
250 hour =
U

C’s daily wage = Rs = Rs 125 4


2 Quantity of IInd candle burnt in one
∴ Daily wages of B and C 1
@

hour =
= Rs (100 + 125) 3
Let t be the required number of hours.
= Rs 225
At t hours, remaining Ist candle
22. Ans. (c)
Sol. Work done by A, B and C in 4 days t 4–t
= 1– =
1 2 4 4
= ×4 = At t hours, remaining IInd candle
10 5
2 3 t 3–t
Remaining work = 1 – = = 1– =
5 5 3 3
Whole work will be done by B and C in 4–t 3–t
Given = 2×
5 50 4 3
10 × = days 3(4 – t) = 8(3 – t)
3 3
1 12 – 3t = 24 – 8t
(A + B + C)’s 1 day work = 8t – 3t = 24 – 12
10
Unit Three : Practice Session ✫ 249

⇒ 5t = 12 30. Ans. (b)


12 Sol. 2 (X’s) daily work = (Y + Z)’s daily work
⇒ t =
5 Adding X’s daily work on both sides, we
= 2.4 hours.
get 3 times X’s daily work
26. Ans. (a)
1
Sol. Let M1 = the number of artisans initially = (X + Y + Z)’s daily work =
employed. 6
1 1 1
M1 × 16 = (M1 + 8) × 12 ∴ X’s daily work = × =
6 3 18
= 12M1 + 96 ∴ X alone can finish the work in 18 days.

TS
16M1 – 12M1= 96
31. Ans. (c)
96
M1 = = 24 men. Sol. 12 men + 10 women = 12 days
4
27. Ans. (c) or 6 men + 8 women = 20 days

H
Sol. Work done by 5 × 12 men Work done by each group in 1 day:
= work done by 10 × 10 women 1

G
12 men + 10 women = ...(i)
60 men = 100 women 12
3 men = 5 women 1
or 6 men + 8 women = ...(ii)
∴ Work done by 3 men + 5 women
= 5 women + 5 women
= 10 women U 20
Now, by solving (i) and (ii), we get
O
1 1 1
3 men + 5 women will take 10 days to do 6 women = – =
10 12 60
the work.
H
1
1 women can do the work in
28. Ans. (b) 6 × 60
Sol. (A and B)’s 2 days’ work 1
_T

= days
360
1 1 1
= 2× +2× and 1 man can do the work in days
10 15 216
3+2 5 1 18 30
C

=
15
=
15
=
3 ∴ 18 men + 30 women = +
216 360
PS

1 2 1 1 1
Remaining work = 1 − = = + =
3 3 12 12 6
Time taken by A to complete the ∴ 18 men and 30 women can finish the
work in 6 days.
remaining work
U

2
2 20 2 32. Ans. (a)
= 3 = × 10 = = 6 days.
@

1 3 3 3 Sol. We have x = 8, y = 10 and z = 8


10 xyz
Using the formula xy + yz + xz
29. Ans. (d)
Sol. At the end of 10 days, the food for 2000 X, Y and Z together finish the job in
men is left for 30 – 10 = 20 days. At this 8 ×10 × 8
stage number of men are 2500 days
8 ×10 + 10 × 8 + 8 × 8
Men Days 640 640
= =
↓ ↓
2500 20 80 + 80 + 64 224
2000 x 20 6
= = 2 days.
⇒ 2500 : 2000 :: 20 : x 7 7
33. Ans. (a)
2000 × 20
⇒ x = = 16 days. Sol. Raman alone can do the whole work in
2500 3 × 15 = 45 days
250 ✫ Quantitative Aptitude

1
∴ Work done by Raman in one day = Now, work done by Pooja alone in 5
45
days
Ashmit alone can do the whole work in 1 1
4 × 15 = 60 days = 5× =
10 2
1 1 1
∴ Work done by Ashmit in one day = Remaining work = 1 – =
60 2 2
Work done by Raman and Ashmit in Number of days to complete the work
one day
1 1 7 1
= + = 1
45 60 180 = 2 = × 6 = 3 days

TS
1 2
∴ Raman and Ashmit will finish the work
180 5 6
in = 25 days. ∴ Altogether, the work was completed in
7 7
5 + 3 = 8 days.

H
34. Ans. (c)
1 7 37. Ans. (d)
Sol. We have a = 14 and b = 3 = hours.
2 2

G
Sol. Amount of work done by A and B
∴ Time taken by A and B working together 1 20 2
to complete the job is ab days = 20 × = =
30 30 3
= 14 ×
7
2 U Work left =
1
3
O
Number of days A has taken = 20
= 49 = 7 days. 1
35. Ans. (d) 1 1
H
∴ A’s 1 day’s work = 3 = =
20 3 × 20 60
Sol. Computer Tasks/hour
30 ⇒ A can finish the work in 60 days.
_T

1 =5
6
80 38. Ans. (c)
x
3 Sol. Let the time taken by B be x days.
C

80
∴ 1 : x :: 5 : Then time taken by A
3
3 3x
PS

80 1 16 1
⇒ x = × = = 5 = 2× x = days
3 5 3 3 4 2
1
∴ 6 computers are required to complete (A + B)’s 1 day’s work =
18
80 of the tasks within 3 hours.
U

1 2 1
+ =
36. Ans. (a) x 3x 18
1
@

Sol. Pooja’s 1 day’s work = 3+2 1


10 =
1 3x 18
Meenu’s 1 day’s work =
15 18 × 5
Pooja and Meenu’s 1 day’s work x = = 30 days.
3
1 1 39. Ans. (b)
= +
10 15 1
Sol. 10 men’s 1 day’s work =
3+ 2 10
= 1 1
30
1 man’s 1 day’s work = =
10 ×10 100
5 1
= = Similarly, 1 woman’s 1 day’s work
30 6
⇒ Pooja and Meenu will take 6 days to 1 1
= =
complete the work. 12 ×10 120
Unit Three : Practice Session ✫ 251

∴ (15 men + 6 women)’s 1 day’s work 42. Ans. (d)


1 1 Sol. 12M = 15W
= 15 × +6×
100 120 One woman’s 1 day’s work
3 1 4 1 1 1
= + = = = =
20 20 20 5 4 ×15 60
⇒ 15 men and 6 women will complete the One man’s one day’s work =
1
work in 5 days. 12×4
40. Ans. (a) 6×2 1
6 men’s 2 day’s work = =

TS
Sol. 3 men = 7 women 12×4 4
3 1 3
⇒ 1 women = men Remaining work = 1 − =
7
⇒ 7 men + 5 women 4 4

H
3 60 women do one work in one day.
= 7 Men + 5 × men
7 3
∴ x women do work in 3 days.

G
49 + 15 4
= men M1D1 M2D2
7
W1 = W2
=
64
7
men
U 60 × 1
=
x×3
O
M1D1 M2D2 1 3
So, = .
W1 W2 4
3 × 32 64 × D2
H
x × 3× 4
= =
1 7×2 3
7 × 3 × 32 60
_T

D2= x = = 15 women.
32 4
= 21 days 43. Ans. (a)
Sol. Let x be the number of days taken by
41. Ans. (a)
C

1 Ashok to finish the work.


Sol. (A + B)’s one day’s work =
12 x
Then Arun finishes in days
PS

1 2
(B + C)’s one day’s work = x
15
and Alok finishes in days
3
Ratio of work of A and C = 2 : 1
1 2 3
U

∴ (A + B)’s one day’s work Their 1 day’s work = + + = 1


1 x x x
∴ = A’s one day’s work + B’s one 1+ 2 + 3
@

12 = 1
day’s work x
= C ’s two day’s work + B’s one day’s ⇒ x = 6 days.
work 1 44. Ans. (b)
= C ’s one day’s work + 1 1
15 Sol. (B + C)’s 2 day’s work = 2 × + 2 ×
8 12
∴ C ’s one day’s work
3+2 5
1 1 5–4 1 = =
= – = = 12 12
12 15 60 60
1 1 4–1 5 7
∴ B ’s one day’s work = – = Remaining work = 1 − =
15 60 60 12 12
3 1 1 1
= = (B + A)’s one day’s work = +
60 20 6 8
⇒ B alone can complete the work in 20 4+3 7
= =
days. 24 24
252 ✫ Quantitative Aptitude

∴ No. of days taken by A and B to finish 3 1


the work Remaining work = 1 – +
10 10
7 4
7 24 = 1–
12 × 10
= = = 2 days
7 12 7 6 3
24 = =
10 5
∴ Work will be finished after (C + A)’s one day’s work
2 + 2 = 4 days. Work Left
=

TS
Number of days worked
45. Ans. (d) 3
Sol. (A + B)’s 20 days’ work 3 1
= 5 = =
1 1 48 5 × 48 80
= 20 × + 20 ×

H
120 150 1 1
∴ C’s one day’s work = –
10 + 8 18 3 80 120
= = =

G
60 60 10 3–2 1
1 1
= =
240 240
A’s 12 days’ work = 12 × =
120 10

U ⇒ C can complete the work alone in 240 days.


O
H
C _T
PS
U
@
Unit Three : Practice Session ✫ 253

Practice Exercise 19
1. Pipes A and B can fill a tank in 3 hours time in which A and B will fill the cistern
and 6 hours respectively. Both the pipes separately will be respectively
together can fill the tank in (a) 15 minutes, 20 minutes
(a) 9 hours (b) 3 hours (b) 25 minutes, 20 minutes
(c) 2 hours (d) 1 hour (c) 15 minutes, 10 minutes
2. One pipe can fill a cistern in 4 hours and (d) 10 minutes, 15 minutes
another can empty the cistern in 6 hours. 8. Two pipes A and B fill a cistern in 5

TS
How long will they take to fill the cistern minutes and 6 minutes, respectively. If
if both the pipes are opened? these pipes are turned on alternately for
(a) 14 hours (b) 12 hours 1 minute each, how long will it take for
(c) 8 hours (d) 2.5 hours the cistern to fill?

H
3. A tank is filled by pipe A in 12 hours and 1 5
(a) 5 minutes (b) 5 minutes
3 6

G
pipe B in 15 hours. When full, it can be
emptied by pipe C in 9 hours. If all the 1 5
(c) 6 minutes (d) 6 minutes
three pipes are opened simultaneously, 3 6
half of the tank will get filled in
5 U 9. A cistern is normally filled in 10 hours but
it takes one hour longer to fill because of
O
(a) 18 hours (b) 25 hours a leak in its bottom. If the cistern is full,
7
2 the leak will empty it in
(c) 30 hours (d) 32 hours
H
5
(a) 80 hours (b) 90 hours
(c) 100 hours (d) 110 hours
4. Two pipes A and B fill a tank in 24 minutes
_T

and 32 minutes respectively. If both the 10. A tap can fill a tank in 16 minutes and
pipes are opened simultaneously, after another can empty it in 8 minutes. If the
1
how much time should pipe B be closed tank is already full and both the taps
so that the tank is full in 18 minutes? 2
C

are opened together, will the tank be filled


(a) 8 minutes (b) 20 minutes
or emptied? How long will it take before
(c) 35 minutes (d) 40 minutes
PS

the tank is either filled or emptied


5. One pipe can fill a tank three times as fast completely as the case may be?
as another pipe. If together the two pipes (a) Filled, 8 minutes
can fill the tank in 36 minutes, then the (b) Filled, 12 minutes
U

slower pipe alone will be able to fill the (c) Emptied, 8 minutes
tank in (d) Emptied, 12 minutes
@

(a) 81 minutes (b) 108 minutes 11. Two pipes A and B can fill a tank in 7
(c) 144 minutes (d) 192 minutes hours and 4 hours respectively. If they are
6. Three pipes A, B and C can fill a tank opened in alternate hours and if pipe A
in 6 hours. After working at it together is opened first, in how many hours, will
for 2 hours, C is closed and A and B fill the tank be full?
the remaining part in 7 hours. The number 2
of hours taken by C alone to fill the tank (a) 5 hours (b) 5 hours
7
is 1
(c) 4 hours (d) 4 hours
(a) 14 hours (b) 13 hours 2
(c) 12 hours (d) 11 hours 12. Taps A, B and C are connected to a water
7. Pipes A and B running together can fill tank and the rate of flow of water is 42
a cistern in 6 minutes. If B takes 5 minutes litres/hr, 56 litres/hr and 48 litres/hr
more than A to fill the cistern, then the respectively. Taps A and B fill the tank
254 ✫ Quantitative Aptitude

while tap C empties the tank. If all the 18. Pipe A can fill a tank in 3 hours, pipe B
three taps are opened simultaneously, the in 6 hours and pipe C in 9 hours. If all
tank gets completely filled up in 16 hours. the pipes are opened together, in how
What is the capacity of the tank? many hours will the tank be filled?
(a) 960 litres (b) 2346 litres 11
(c) 1600 litres (d) 800 litres (a) 12 hours (b) hours
18
13. An electric pump can fill a tank in 3 hours. 7
(c) 15 hours (d) 1 hours
Because of a leakage in the tank it took 11
1 19. Two pipes A and B can fill a tank in 36

TS
3 hours to fill the tank. The leakage can minutes and 45 minutes respectively.
2
drain out all the water of the tank in Another pipe C can empty the tank in 30
1 minutes. First A and B are opened. After
(a) 10 hours (b) 12 hours 7 minutes, C is also opened. In how much

H
2
time will the tank be filled?
(c) 21 hours (d) 24 hours
(a) 27 minutes (b) 29 minutes

G
14. A leak in the bottom of the tank can (c) 39 minutes (d) 46 minutes
empty the full tank in 12 hours. An inlet
20. A bath has two inlet taps A, B and an
pipe fills water at the rate of 6 litres a
minute and the tank is full when the inlet
is opened for 8 hours. How many litres U outlet C. With A turned on, it fills in 15
minutes; with A and B, it fills in 10
minutes; with B turned on and C opened,
O
does the cistern hold? the water level remains stationary. How
(a) 7580 (b) 7960 long will it take to fill the bath with A
H
(c) 8210 (d) 8640 turned on and C opened?
15. A pipe can fill a bath in 20 minutes, and (a) 10 minutes (b) 20 minutes
_T

another can fill it in 30 minutes. A person (c) 30 minutes (d) 40 minutes


opens both the pipes simultaneously. When 21. A pipe can fill a tank in 6 hours. After
the bath should have been full, he finds half the tank is filled, three more similar
that the waste pipe was open. He then taps are opened. What is the total time
C

closes the waste pipe and in 3 minutes the taken to fill the tank completely?
bath is full. In what time will the waste (a) 3 hours 45 minutes
PS

pipe empty it? (b) 2 hours 32 minutes


(a) 20 minutes (b) 48 minutes (c) 29 minutes
(c) 30 minutes (d) 45 minutes (d) 21 minutes
U

16. A cistern when full is emptied by a waste 22. One large tanker can be filled by two
pipe in 30 minutes; but if a tap is opened pipes A and B in 60 minutes and 40
it takes 40 minutes to empty the cistern.
@

minutes respectively. How many minutes


How long will the cistern take to become will it take to fill the tanker from the
full if both the pipes are working? empty state if B is used for half the time
(a) 60 minutes (b) 120 minutes and A and B fill it together for the other
(c) 140 minutes (d) None of these half?
17. Two pipes A and B individually can fill (a) 15 minutes (b) 20 minutes
a cistern in 20 minutes and 30 minutes (c) 30 minutes (d) 40 minutes
respectively. A third waste pipe C can 23. Two pipes A and B would fill a cistern
empty the cistern in 1 hour. If both the in 20 and 24 minutes respectively. Both
pipes A and B and the waste pipe C are pipes A and B being opened, find when
opened simultaneously when the cistern is the first pipe must be turned off so that
empty, the cistern will be filled up in the cistern may be filled in 12 minutes.
(a) 15 minutes (b) 14 minutes (a) 5 minutes (b) 10 minutes
(c) 13 minutes (d) 12 minutes (c) 12 minutes (d) 15 minutes
Unit Three : Practice Session ✫ 255

24. A cistern is filled by two pipes in 10 and 30. Bucket P has thrice the capacity as bucket
15 hours respectively and then emptied by Q. It takes 60 turns for bucket P to fill
a third pipe in 8 hours. If all the three the drum. How many turns it will take
taps are opened, in what time will the for both the buckets P and Q having each
cistern be filled? turning together to fill the empty drum?
(a) It can never be filled (a) 30 (b) 40
(b) It is filled in 13 hours (c) 45 (d) 90
(c) It is filled in 24 hours 31. Two pipes A and B can fill a tank in 20
(d) It is filled in 33 hours hours and 15 hours respectively; C can

TS
25. Three-fourth of a tank is filled with water. empty it in 12 hours. If these are opened
If 5 litres are added to it, then four-fifth at 8 a.m., 9 a.m. and 10 a.m. respectively,
of the tank becomes full. What is the when will the tank be filled?
capacity of the tank? (a) 8 a.m., next day

H
(a) 130 litres (b) 120 litres (b) 11 a.m., next day
(c) 110 litres (d) 100 litres (c) 12 a.m., next day

G
(d) None of these
26. A pipe can empty a cistern in 10 hours.
Find the part of the cistern emptied in 6 32. Two taps can fill a cistern separately in
hours.
(a)
3
(b)
5
U 60 and 80 minutes respectively. To fill the
empty cistern, both the taps are kept open
for 20 minutes and then the second tap
O
5 3
4 2 is turned off. Find the required time to
(c) (d) fill the cistern.
7 5
H
27. A pipe can fill a tank in 30 minutes. Find (a) 48 minutes (b) 30 minutes
1 (c) 45 minutes (d) 70 minutes
the time in which th of the tank will
_T

4 33. First fill pipe A is 3 times faster than


be filled.
second fill pipe B and takes 10 minutes
1
(a) 10 minutes (b) 9 minutes less time to fill a cistern than B takes. Find
2 when will the cistern be full if fill pipe
C

1
(c) 8 minutes (d) 7 minutes B is opened alone.
2
(a) 15 minutes (b) 5 minutes
PS

28. Eight taps are fitted to a water tank. Some (c) 10 minutes (d) 20 minutes
of them are water taps to fill in the tank,
and the remaining are outlet taps used to 34. Two taps A and B can separately fill a
empty the tank. Each water tap can fill cistern in 45 and 40 minutes respectively.
U

the tank in 12 hours and each outlet tap They started to fill a cistern together, but
can empty it in 36 hours. On opening all tap A was turned off after a few minutes
@

the taps, the tank is filled in 3 hours. Find and tap B filled the rest of the cistern in
the number of inlet water taps. 23 minutes. After how many minutes was
(a) 5 (b) 6 tap A turned off?
(c) 4 (d) 3 (a) 8 minutes (b) 10 minutes
(c) 9 minutes (d) 12 minutes
29. Two pipes A and B can fill a cistern in
12 minutes and 15 minutes respectively, 35. Two inlet pipes of filling rate 10 minutes
and 15 minutes respectively per cistern
but a third pipe C can empty the full tank
and one outlet pipe with emptying rate
in 6 minutes. A and B are kept open for
5 minutes per cistern are all fitted to a
5 minutes in the beginning and then C is
cistern and are opened together. Find
also opened. In what time is the cistern
when will this cistern be full/emptied, as
emptied?
the case may be?
(a) 30 minutes (b) 33 minutes
(a) 30 minutes (b) 15 minutes
1
(c) 37 minutes (d) 45 minutes (c) 25 minutes (d) None of these
2
256 ✫ Quantitative Aptitude

Answers with Solutions

1. Ans. (c) 6. Ans. (a) 2 1


Sol. Part filled in 2 hours = =
Sol. A + B together fill the tank in 6 3
3×6 1 2
hours = 2 hours Remaining part = 1 – =
3+6 3 3
2
2. Ans. (b) ∴ A + B’s 7 hours’ work =
3
Sol. The pipes together fill the tank in 2 1 2

TS
4×6 A + B’s 1 hour work = × =
3 7 21
hours = 12 hours ∴ C’s 1 hour work
6–4
= (A + B + C)’s 1 hour work
3. Ans. (b)
– (A + B)’s 1 hour work

H
Sol. A + B + C fill the tank in
1 2 1
12 ×15 × 9 = – =

G
6 21 14
15 × 9 + 12 × 9 – 15 ×12
∴ C alone can fill the tank in 14 hours.
12 ×15 × 9
=

=
180
63
5
= 25 hours U 7. Ans. (d)
Sol. Let pipe A fill the cistern in x minutes
∴ Pipe B will fill the cistern in (x + 5)
O
7 7
4. Ans. (a) minutes
∴ In one minute, pipes A and B together
H
Sol. Let B be closed after x minutes.
1 1
Then part filled by (A + B) in x minutes can fill + of the cistern.
x x+5
_T

+ Part filled by A in (18 – x) minutes = 1


1 1 1
1 1 1 ⇒ + =
x x+5 6
∴ x + + (18 – x )× =1
24 32 24 ⇒
C

x = 10
7 x 18 – x ∴ Pipes A and B can fill the cistern in 10
⇒ + =1
PS

96 24 minutes and 15 minutes respectively.


⇒ 7x + 4(18 – x) = 96
8. Ans. (b)
⇒ x =8 Sol. As the pipes are operating alternately,
U

Hence, B must be closed after 8 minutes. 1 1 11


they can fill + = part of the
5 6 30
5. Ans. (c)
@

cistern in 2 minutes.
Sol. Let the slower pipe alone fill the tank In the next two minutes, the pipes can
in x minutes. Then, faster pipe will fill 11
x fill another part of cistern.
it in minutes. 30
3 ∴ In 4 minutes, the two pipes which are
1 3 1 operating alternately will fill
∴ + =
x x 36 11 11 22 11
+ = = part
4 1 30 30 30 15
=
x 36 The part of the cistern left unfilled
∴ x = 36 × 4 11 4
= 1– =
= 144 minutes 15 15
Thus, the slower pipe fills the tank in 1
Pipe A can fill of the cistern in one
144 minutes. 5
minute.
Unit Three : Practice Session ✫ 257

1 12. Ans. (d)


Pipe B can fill of the cistern in Sol. Quantity of water flowing into the
6
1 5 water tank in 1 hour
5× = minutes
6 6 = 42 + 56 – 48
Total time taken to fill the cistern = 98 – 48
5 5
5+ = 5 minutes. = 50 litres
6 6
9. Ans. (d) ∴ Capacity of the tank = 16 × 50
Sol. We have x = 10 and y = 11 = 800 litres.

TS
13. Ans. (c)
∴ The leak will empty the cistern in
Sol. Part filled 1 in hour (without the leak)
10 ×11 1
= 110 hours. =
11 – 10 3

H
10. Ans. (c)
Part filled in 1 hour, because of the
Sol. Half the tank is filled by one tap in 2

G
8 minutes. Half the tank is emptied by leak =
7
another tap in 4 minutes.
1 1 1 Part emptied by the leak in one hour
In 1 minute –
8 4
= – is filled when
8
both taps are open i.e.,
1
of the tank U ⇒
=
1 2

3 7
=
7–6
21
=
1
21
O
8 Time taken by the leak to empty the
is emptied.
tank = 21 hours.
∴ It takes 8 minutes to empty the tank.
H
14. Ans. (d)
11. Ans. (a) Sol. Work done by the inlet pipe and the
1 leak
_T

Sol. A’s 1 hour work =


7 1 1 3–2 1
1 = – = =
B’s 1 hour work = 8 12 24 24
4 ⇒ Tank can be full in 24 hours.
C

(A + B)’s 4 hour work when opened ⇒ Tank can be full in 24 × 60


1 1 11 = 1440 minutes
alternatively = + =
PS

7 4 28
∴ Volume of tank = 1440 × 6 litres/minute
(A + B)’s 4 hour work when opened
22 = 8640 litres.
alternately = 15. Ans. (b)
28
U

22 6 Sol. Let x be the time in which the waste


Remaining part= 1 – = pipe can empty the bath. Part filled by
28 28
both pipes in 1 minute
@

3
= 1 1 3+2 5 1
14 = + = = =
1 20 30 60 60 12
Now, it is A’s turn and part is filled ⇒ Bath can be filled in 12 minutes.
7
by A in 1 hour. But the bath was full in 12 + 3 = 15
∴ Remaining part to be filled by B minutes, due to the waste pipe. For
3 1 1 the first 12 minutes, the waste pipe
= – =
14 7 14 was open.
1 4 2 ∴ The whole work done (bath filled) is
B fills in = hours
14 14 7 given as
∴ Total time taken to fill the tank is 1 1 1 1 1
2 12 + – +3 + =1
= 4 + 1 + 20 30 x 20 30
7 1 1 1
2 12 × – 12 × + 3 ×
= 5 hours = 1
7 12 x 12
258 ✫ Quantitative Aptitude

12 1 1
1– + = 1 Now, part is filled in
x 4 60
13
12 1 60 × = 39 minutes
∴ = 20
x 4 ∴ Total time taken to fill the tank is
⇒ x = 48 minutes. 39 + 7 = 46 minutes
16. Ans. (b) 20. Ans. (c)
Sol. Part emptied in one minute (without 1 1
Sol. = ...(i)
1 A 15

TS
open tap) = A = Time taken by A to fill the tank
30
Part emptied in one minute, when the ⇒ A = 15
1 1 1 1
tap is opened = + = ...(ii)

H
40 A B 10
Work done by filling tap B = Time taken by B to fill the tank
1 1

G
– 1 1
= – =0 ... (iii)
30 40 B C
4–3 1 C = Time taken by C to empty the tank.


=
120
=
120
Cistern will be full in 120 minutes. U From equations (i) and (ii) we get,
1 1 1
O
+ =
15 B 10
17. Ans. (a) ∴ B = 30 minutes
H
Sol. Let the cistern be filled up in t minutes. From equation (iii),
If the volume of the cistern is V, then 1 1
_T

– = 0
V V V 30 C
×t + ×t – ×t = V
20 30 60
3+ 2 –1
∴ C = 30
⇒ t = 1
C

60 Time taken to fill the tank with pipes A


60 and C
PS

⇒ t = = 15 minutes. 1 1 1 1 1
4 = – ⇒ – =
18. Ans. (d) A C 15 30 30
1
Sol. Part filled by (A + B + C) in 1 hour ∴ part of tank is filled in 1 minute.
30
U

1 1 1
+ +
11 ∴ Tank is filled in 30 minutes.
= =
3 6 9 18
@

∴ All the three pipes together will fill the 21. Ans. (a)
18 7 Sol. Time taken by one tap to fill the tank
tank in hours = 1 hours
11 11 half = 3 hours.
19. Ans. (d) Part filled by the four taps in 1 hour
Sol. Part filled in 7 minutes 1 2
1 1 7 = 4× =
= 7 + = 6 3
36 45 20 1 1
7 13 Remaining part = 1 – =
Remaining part = 1 – = 2 2
20 20 Part of tank filled Time taken (hour)
Net part filled in 1 minute, when A, B 2
and C are opened 1
3
1 1 1 1 1
= + – = x
36 45 30 60 2
Unit Three : Practice Session ✫ 259

2 1 26. Ans. (a)


∴ : :: 1 : x Sol. Part of the cistern emptied in 1 hour
3 2
1 3 3 1
⇒ x= ×1× = hours =
2 2 4 10
= 45 minutes Part of the cistern emptied in 6 hours
1 3
Total time taken = 3 hours 45 minutes. = ×6 =
10 5
22. Ans. (c) 27. Ans. (d)
Sol. Part of tank filled by A and B in 1 1
Sol. part of the tank is filled in 1 minute

TS
minute 30
1 30 15
1
+
1 40 + 60 1 ∴ part of the tank is filled in =
= = = 4 4 2
60 40 60 × 40 24 1
= 7 minutes.

H
Suppose the tank is filled in x minutes,
2
then 28. Ans. (a)
x 1 1

G
+ Sol. Let x be the number of inlet water taps.
=1
2 24 40 ⇒ Outlet taps = 8 – x
x Part filled by x water taps in one hour


30
= 1
x = 30 minutes. U = x×
1
12
O
23. Ans. (b) Part emptied by (8 – x) outlet taps in 1
Sol. Little thinking can help us to solve this hour
H
problem quickly. Since the cistern is to 1
= (8 – x )×
be filled in 12 minutes, second pipe 36
1 (8 – x ) 1
_T

shall fill half of it. Remaining half of the x× –


i.e., =
cistern is to be filled by the first pipe 12 36 3
A which takes 10 minutes. So, the first 3x – 8 + x 1
=
pipe must be opened for 10 minutes, 36 3
C

i.e., it must be turned off after 10 4x = 12 + 8 = 20


minutes. ⇒ x = 5
PS

24. Ans. (c) The number of inlet water taps is 5.


29. Ans. (d)
Sol. We have x = 10, y = 15 and z = 8
Sol. Work done by A and B for 5 minutes
xyz
Using formula yz + zx – xy , we get 1 1
U

= 5× +
12 15
time required to fill the tank, i.e.,
5+4 5×9 3
@

10 ×15 × 8 = 5× = =
60 60 4
15 × 8 + 8 ×10 – 10 × 15 1
1200 Part unfilled =
= = 24 hours 4
50 Work done by A, B and C in 1 minute
25. Ans. (d) 1 1 1
= + –
Sol. Let the capacity of the tank be x litres 12 15 6
3 4 5 + 4 – 10 –1
∴ x+5 = x = =
4 5 60 60
4 3 (–ve sign shows the tank is getting emptied).
⇒ x– x = 5
5 4 ⇒ One full tank is emptied in 60 minutes
16 x – 15 x 3
⇒ = 5 ∴ Time taken to empty part of the tank
20 4
3
⇒ x = 5 × 20 = 100 litres. = × 60 = 45 minutes
4
260 ✫ Quantitative Aptitude

30. Ans. (c) 32. Ans. (c)


Sol. P takes 60 turns to fill the bucket Sol. Part filled in 20 minutes by both taps
⇒ Q takes 180 turns to fill the bucket
1 1 4+3 7
Part filled in one turn by both P and Q = 20 × + = 20 × =
60 80 240 12
together
1 1 5
+ Part unfilled =
= 12
60 180
3+1 4 1 Time taken to fill the remaining part by
= = = first tap only
180 180 45

TS
⇒ No. of turns required by both P and Q 5
12 5 60
= 45. = = × = 25 minutes
1 12 1
31. Ans. (b) 60

H
Sol. From 8 a.m. to 9 a.m., A worked alone ∴ Total time required to fill the cistern
1 = 20+25 = 45 minutes.
∴ A’s 1 hour work = 33. Ans. (a)

G
20
From 9 a.m. to 10 a.m., both A and B Sol. Let x be the time taken by pipe A to fill
the cistern
worked together.
Part filled by A and B together in 1
hour. U ⇒ 3x is the time taken by pipe B.
∴ 3x – x = 10
O
i.e., 2x = 10 ⇒ x = 5
1 1
= + ∴ Time taken by B = 3 × 5 = 15 minutes
20 15
H
3+4 7 34. Ans. (c)
= = Sol. Let us suppose that the tap was turned
60 60
_T

∴ Work done by A and B from 8 a.m. to off, after x minutes.


10 a.m. ∴ Work done by both the pipes is given as
1 7 3+7 1 1 1
x× + + 23 ×
C

= + = =1
20 60 60 45 40 40
10 1 8+9 23
PS

= = x× + =1
60 6 360 40
1 5 17 x 23 17
Remaining work = 1 − = = 1– =
6 6 360 40 40
U

Work done by A, B and C together in 17 360


1 hour x = ×
40 17
@

7 1 7–5
= – = x = 9 minutes
60 12 60
2 1 35. Ans. (a)
= = Sol. Work done by all the three pipes in one
60 30
5 minute
Time taken to complete part of work 1 1 1
6 + –
=
5 10 15 5
5
= 6 = × 30 = 25 hours 3+ 2 – 6 –1
1 6 = =
30 30
30 (–ve) sign shows that the cistern is
Total time taken = 25 + 2 = 27 hours getting emptied.
∴ The tank will be full on the next day at ∴ The cistern will be emptied in 30
11 a.m. minutes.
Unit Three : Practice Session ✫ 261

Practice Exercise 20
1. Shiela’s house is 10 km away from the 1
by 1 hours at the start then in order
school. She takes 30 minutes to reach the 2
to reach his destination in time, he should
school by bus. If Ram travels from his
ride at a speed of
house at the same speed as Shiela and
(a) 15 kmph (b) 16 kmph
takes only 12 minutes to reach the school.
(c) 18 kmph (d) 20 kmph
The distance between Ram’s house and his
school (in km) is 8. Two cyclists A and B start from the same

TS
(a) 4 (b) 5 place at the same time, one going towards
(c) 6 (d) 7 north at 18 kmph and the other towards
south at 20 kmph. What time will they
2. A boy takes 20 minutes to reach the school
take to be 95 km apart?

H
at an average speed of 12 kmph. If he
1 3
wants to reach the school in 16 minutes, (a) 4 hours (b) 4 hours
his average speed (in kmph) should be 2 4

G
4 1
(a) 15 (b) 16 (c) 5 hours (d) 2 hours
(c) 18 (d) 20 15 2

3. A carriage driving in fog passed a man


who was walking at the rate of 3 km an U 9. A train leaves Meerut at 6 a.m. and
reaches Delhi at 10 a.m. Another train
O
leaves Delhi at 8 a.m. and reaches Meerut
hour in the same direction. He could see
at 11.30 a.m. At what time do the two
the carriage for 4 minutes and it was
trains cross one another?
H
visible to him upto a distance of 100 m.
9 12
The speed of the carriage was (a) 5 hours (b) 4 hours
17 13
_T

(a) 4.5 kmph (b) 5 kmph


19 14
(c) 5.5 kmph (d) 6 kmph (c) 3 hours (d) 2 hours
20 15
4. Aditya is twice as fast as Rohan and
10. A thief steals a car at 2.30 pm and drives
Rohan is thrice as fast as Kaushal. Kaushal
C

it at 60 kmph. The theft is discovered at


covers a journey in 54 minutes. Aditya will
3 pm and the owner sets off in another
cover the same journey in
PS

car at 75 kmph. When will the owner


(a) 14 minutes (b) 12 minutes overtake the thief?
(c) 10 minutes (d) 9 minutes (a) 7 pm (b) 6 pm
5. In a 500 metres race, Q starts 45 metres (c) 5 pm (d) 4 pm
U

ahead of P. But P wins the race while Q 11. Two men start together to walk to a
is still 35 metres behind. The ratio of their certain destination, one at 3 kmph and
@

speeds, assuming that both start at the another at 3.75 kmph. The latter arrives
same time, is half an hour before the former. The
(a) 5 : 7 (b) 5 : 3 distance is
(c) 5 : 6 (d) 25 : 21 (a) 7.5 km (b) 8 km
(c) 9.5 km (d) 10 km
6. A truck completes a journey in 20 hours,
the first half at 80 kmph, and the rest at 12. The ratio between the speeds of two
84 kmph. The distance covered by the trains is 7 : 8. If the second train runs
truck is 400 kms in 4 hours, then the speed of the
(a) 1639 km (b) 1639.02 km first train is
(c) 1639.12 km (d) 1639.32 km (a) 87.5 kmph (b) 84 kmph
(c) 75 kmph (d) 70 kmph
7. A man riding a cycle at 12 kmph can reach
1 13. A scooterist completes a journey in 10
a village in 4 hours. If he is delayed hours, the first half at the rate of 21 kmph
2
262 ✫ Quantitative Aptitude

and the second half at the rate of 24 20. In covering a distance of 20 km, Kunal
kmph. The total distance travelled is takes 2 hours more than Utkarsh. If Kunal
doubles his speed, then he would take 1
(a) 192 km (b) 204 km
hour less than Utkarsh. Kunal’s speed is
(c) 224 km (d) 256 km 3
(a) 1 kmph (b) 1 kmph
14. A monkey climbs a slippery pole 10 m 4
1
high in such a way that every minute it (c) 3 kmph (d) 4 kmph
3
rises 50 cm and every next minute it slips
21. If a person walks at 14 kmph instead of
25 cm. How soon will he reach the top?
10 kmph, he would have walked 20 km

TS
(a) 43 minutes (b) 51 minutes
more. The actual distance travelled by him
(c) 68 minutes (d) 77 minutes is
15. A distance is covered in 3 hours at 5 (a) 30 km (b) 40 km
(c) 50 km (d) 60 km

H
kmph. How much time will be taken to
cover it at 20 kmph. 22. A truck covers a distance of 1000 metres

G
(a) 40 minutes (b) 45 minutes in 2 minutes whereas a bus covers a
(c) 50 minutes (d) 55 minutes distance of 40 kms in 50 minutes. The ratio
of their speeds is
16. A man travelled from a village to the post
office at the rate of 25 kmph and walked
U (a) 1 : 7
(c) 5 : 8
(b) 2 : 9
(d) 6 : 13
O
back at the rate of 4 kmph. If the whole
journey took 5 hours 48 minutes, find the 23. An express train travels at an average
distance of the post-office from the village. speed of 120 kmph, stopping for 2 minutes
H
after every 60 km. How long must it have
(a) 50 km (b) 40 km
taken to reach its destination 1000 km
(c) 30 km (d) 20 km
_T

away from its starting point?


17. A cyclist covers a distance of 750 m in 2 (a) 8 hours
minutes 30 seconds. What is the speed in (b) 8 hours 43 minutes
kmph of the cyclist? (c) 9 hour 5 minutes
C

(a) 21 kmph (b) 18 kmph (d) 10 hours


(c) 15 kmph (d) 13 kmph
PS

24. If a man walks at a speed of 5 km/hour


18. A motor car starts at a speed of 70 kmph from his house to the railway station, he
with its speed increasing every two hours misses the train by 5 minutes. However,
if he walks at a speed of 6 km/hour, he
U

by 10 kmph. In how many hours will it


reaches the railway station 6 minutes
cover 345 kms?
before the scheduled departure time of
1
@

(a) 2 hours the train. The distance between his house


2
and the railway station is equal to
(b) 4 hours 10 minutes
(a) 5 km (b) 5.5 km
1 (c) 6 km (d) 6.5 km
(c) 4 hours
2
(d) 5 hours 7 minutes 25. Two men set out at the same time to walk
towards each other from points A and B,
19. In a flight of 900 km, an aircraft was 72 km apart. The first man walks at the
slowed down due to bad weather. Its constant rate of 4 km/hr. The second man
average speed for the trip was reduced 1
walks 2 km the first hour, 2 km the
by 200 kmph and the time of flight 2
second hour, 3 km the third hour and so
increased by 50 minutes. The duration of
the flight was on. Then the men will meet
(a) 1.43 hours (b) 1.56 hours (a) in 7 hours
(c) 2.19 hours (d) 2.43 hours (b) nearer A than B
Unit Three : Practice Session ✫ 263

(c) nearer B than A punctured if the total time taken for the
(d) midway between A and B 1
journey was 8 hours?
26. The distance between two towns is 800 km. 2
(a) 18 km (b) 54 km
A car starts from the first at 30 kmph. At (c) 36 km (d) 48 km
the same time, another car starts from the
other town. If the distance of the point 32. A man is walking at a speed of 10 km
where these two cars meet is 500 kms from per hour. After every kilometre, he takes
one of the towns, then at what speed did rest for 5 minutes. How much time will
the second car travel? it take for him to cover a distance of 5

TS
(a) 40 kmph (b) 30 kmph kilometres?
(c) 50 kmph (d) None of these (a) 50 minutes (b) 30 minutes
(c) 40 minutes (d) 55 minutes
27. Two men together start on a journey in
33. A man walks one-third of the journey at

H
the same direction. They travel 9 km and
15 km respectively daily. After travelling 4 km/hr, cycles one half of the journey
for 6 days, the man travelling at 9 km/ at 12 km/hr and the remaining journey

G
hr doubles his speed and both of them at 9 km/hr. The total journey was
finish the distance in the same time. Find completed in 5 hours and 10 minutes. Find
the time taken by them to reach their
destination.
U the total distance of the journey.
(a) 30 km (b) 38 km
O
(a) 17 days (b) 18 days (c) 36 km (d) 45 km
(c) 19 days (d) 22 days 34. Two trains 450 m and 350 m long
H
28. Starting from a common point, two cars respectively, travel with uniform speeds on
run in opposite directions. The speed of parallel lines from opposite directions. If
one car is 15 km/hr more than that of the they cross each other in 1 minute, and the
_T

other car. If the distance between them speed of one train is 30 km/hr, then the
is 100 km and they cover it in 8 hours, speed of the second train is
what is the speed in km/hr of the faster (a) 20 kmph (b) 18 kmph
C

car? (c) 25 kmph (d) 30 kmph


(a) 55 (b) 70 35. The average speed of a train, including
PS

(c) 75 (d) 125 stoppages, was 27 km/hr and, excluding


29. The radius of a circular wheel is 35 cm. stoppages, it was 30 km/hr. How many
It moves at the rate of 500 revolutions per minutes per hour did the train stop?
minute. The speed of the wheel is (a) 6 minutes (b) 10 minutes
U

(a) 50 km/hr (b) 60 km/hr (c) 15 minutes (d) 16 minutes


(c) 66 km/hr (d) 63 km/hr 36. A motorist can complete a journey in 6
@

30. The diameters of the front wheel and hind hours, travelling at a speed of 24 km/hr.
wheel of a van are 70 cm and 98 cm If he is delayed by two hours at the start
respectively. Find how many more of the journey, then to make up for the
revolutions the front wheel will make lost time, he should drive at a speed of
than the hind wheel in covering a distance (a) 36 km/hr. (b) 20 km/hr.
of 154 km. (c) 16 km/hr. (d) 12 km/hr.
(a) 10,000 (b) 15,000 37. Two cars start at the same time from A
(c) 20,000 (d) 25,000 and B which are 120 km apart. If the two
31. A person has to make a journey of 72 km. cars travel in opposite direction they meet
He rides a cycle at 12 km/hr. After going after one hour and if they travel in the
a certain distance the cycle gets punctured same direction (from A towards B) they
and he walks the remaining distance at meet after 6 hours. What is the speed of
1 the car starting from A?
4 km/hr. Find where the cycle got
2
264 ✫ Quantitative Aptitude

(a) 70 km/hr the same distance that C covers in 42


(b) 60 km/hr minutes?
(c) Data inadequate (a) 5 minutes (b) 6 minutes
(d) None of these (c) 7 minutes (d) 8 minutes
38. The speeds of three cars are in the ratio 40. Neera leaves home at 11 a.m. and rides
of 2 : 3 : 4. The ratio between the time to Asha’s house to return her bicycle. She
taken by these cars to travel the same travels at 12 km per hour and arrives at
distance is 11.30 a.m. She turns right around and
(a) 2 : 3 : 4 (b) 3 : 2 : 4 walks home. How fast does she walk if

TS
(c) 6 : 4 : 3 (d) 4 : 3 : 2 she returns home at 1 p.m.?
39. A is twice as fast as B and B is thrice as (a) 5 kmph (b) 4 kmph
fast as C. What time will A take to cover (c) 7 kmph (d) 6 kmph

H
Answers with Solutions

G
1. Ans. (a) If Kaushal takes 6 minutes, Aditya takes
Sol. Distance covered by Shiela in 30 minutes 1 minute
= 10 km
Distance covered by Shiela in 1 hour U If Kaushal takes 54 minutes, Aditya will
take
O
10 1
= × 60 = 20 km × 54 = 9 minutes.
30 6
5. Ans. (d)
H
∴ Speed of Ram = 20 kmph Sol. Suppose the speeds of P and Q are
∴ Distance covered by Ram in 12 minutes respectively x m/min and y m/min.
_T

20 Distance covered by P = 500 m


= ×12 = 4 km.
60 Distance covered by Q in the same time
2. Ans. (a) = 500 – (45 + 35)
Sol. Distance travelled in 20 minutes at the = 500 – 80
C

rate of 12 kmph. = 420 m


20 500 420
PS

= 12 × km = 4 km Thus, = y
60 x
4 or x : y = 500 : 420 = 25 : 21.
Average speed = × 60 = 15 kmph.
16 6. Ans. (b)
3. Ans. (a)
U

Sol. Distance
Sol. The distance travelled by the man in 4
2 × Total time × Product of speeds
minutes =
@

Sum of speeds
3 × 1000
= × 4 = 200 metres 2 × 20 × 80 × 84
60 =
∴ Distance travelled by the carriage in 4 80 + 84
minutes 268800
= = 1639.02 km.
= 200 + 100 = 300 metres 164
300 60 7. Ans. (c)
∴ Speed of the carriage = × 9
4 1000 Sol. Distance covered by the man in hours
9 2
= 4.5 kmph. = × 12 = 54 km
4. Ans. (d) 2 1
Sol. Ratio between the time taken by Aditya To cover 54 km in 1 hours less time,
2
and Rohan = 1 : 2 i.e., in 3 hours, he should ride at the speed
Ratio between the time taken by Rohan 54
and Kaushal = 1 : 3 = = 18 kmph.
3
Unit Three : Practice Session ✫ 265

8. Ans. (d) 3 × 3.75


Sol. They are 38 km apart in 1 hour x =
1
0.75 × 2
∴ They will be 95 km apart in × 95 15
38 ⇒ x =
hours 2
= 2.5 hours ⇒ x = 7.5 km.
1 12. Ans. (a)
= 2 hours.
2 Sol. Let the speed of the two trains be
9. Ans. (d)
Sol. Let the distance between Meerut and 7x and 8x kmph

TS
Delhi be y km 400
Then,8x = (given)
Average speed of the train leaving 4
x = 12.5
Meerut
∴ Speed of the first train = 7 × 12.5
y

H
= kmph = 87.5 kmph.
4 13. Ans. (c)
Average speed of the train leaving Delhi
Sol. Let the distance be = x km

G
2y
= kmph x
7 Time taken in travelling km at the rate
2
Suppose they meet x hours after 6 am
Then,
xy 2y (x – 2 )
+ =y U of 21 kmph =
1 x
× =
21 2
x
42
hour
O
4 7 x
x 2x – 4 Time taken in travelling km at the rate
+ =1 2
H
4 7 1 x x
∴ 15x = 44 of 24 kmph = × = hour
24 2 48
44 14 x x
_T

x= =2 hours. But, + = 10
15 15 42 48
10. Ans. (c) 8x + 7 x
Sol. Suppose the thief is overtaken x hours = 10
336
C

after 2.30 pm. 15x = 3360


Then, distance covered by the thief in ⇒ x = 224 km.
PS

x hours = distance covered by the 14. Ans. (d)


1 Sol. Rise in two minutes = 50 – 25 = 25 cm
owner in x – hours Pole length = 10 m = 1000 cm
2
Part of pole climbed in two minutes
U

1
∴ 60x = 75 x – 1000 – 50 950
2 = = = 38
25 25
@

75 ∴ Total time = 38 × 2 + 1
⇒ 15x =
2 = 77 minutes.
5
⇒ x = hours 15. Ans. (b)
2 Sol. Distance = 5 × 3 = 15 km
So, the thief is overtaken at 5 p.m. Time taken to cover it at 20 kmph
11. Ans. (a) 15
Sol. Let the total journey be x km = × 60 = 45 minutes.
20
x x 1 16. Ans. (d)
Then, – =
3 3.75 2 2xy
Sol. Average speed = x + y kmph
3.75x − 3 x 1
= 2 × 25 × 4
3 × 3.75 2 = kmph
3 × 3.75 25 + 4
⇒ 0.75x = 200
2 = kmph
29
266 ✫ Quantitative Aptitude

Distance travelled in 5 hours 48 minutes, –10 ± (10)2 – 4(12)(–45)


i.e., ⇒ x =
2(12)
4 200 29
5 hours = × = 40 km
5 29 5 –10 ± 100 + 2160
⇒ x =
∴ Distance of the post-office from the 24
village –10 ± 2260
40
⇒ x =
24
= = 20 km.
2 –10 ± 47.53
17. Ans. (b) ⇒ x =
24

TS
750 ⇒ x = 1.56 hours
Sol. Speed = m/sec = 5 m/sec
150
(neglecting ‘–’ve value).
18
= 5× kmph

H
5 20. Ans. (c)
= 18 kmph. Sol. Let Kunal’s speed be x kmph
18. Ans. (c)

G
20 20
Sol. Distance covered in the first two hours Then, – =3
x 2x
= 70 × 2 = 140 km 40 – 20
Distance covered in the next two hours
= 80 × 2 = 160 km
U ⇒

2x
=3
20 = 6x
O
Remaining distance 20 10 1
=345 – (140 + 160) ⇒ x = = = 3 kmph.
6 3 3
= 45 km
H
21. Ans. (c)
Speed in the fifth hour = 90 kmph
Time taken to cover 45 km Sol. Let the actual distance travelled be x
_T

45 1 km.
= = hour x x + 20
90 2
1 1 Then, =
10 14
∴ Total time taken = 2 + 2 + = 4 hours. ⇒ 14x = 10x + 200
C

2 2
19. Ans. (b) ⇒ 4x = 200
⇒ x = 50 km.
PS

Sol. Let the duration of the flight be x


22. Ans. (c)
hours. Then,
Sol. Ratio of speeds
900 900
– = 200 1000 18 40
U

x 5 = × : × 60
x+ 2 × 60 5 50
6
= 30 : 48
@

900 5400
⇒ – = 200 = 5 : 8.
x 6x + 5
23. Ans. (c)
900(6 x + 5) – 5400 x Sol. Time taken to cover 1000 km
⇒ = 200 1000
x(6 x + 5)
= = 8.34 hours
120
5400 x + 4500 – 5400x Number of stoppages
⇒ = 200
x(6 x + 5) 1000
= – 1 = 15.67
4500 60
⇒ = 200 Total stoppage time
x(6x + 5)
= 2 × 15.67 = 31.34 minutes
4500 45
⇒ x(6x + 5) = = Hence, total time taken
200 2 = 8.34 hours + 31.34 minutes
⇒ 12x 2 + 10x – 45 = 0 = 9 hours 5 minutes (approximate).
Unit Three : Practice Session ✫ 267

24. Ans. (b) ∴ If 30 × t = 300 km,


Distance then t = 10 hours.
Sol. Time = Speed 500
⇒ Speed of the second car =
Let x be the required distance. 10
x = 50 km/hr.
∴ Time taken = 27. Ans. (b)
5
x Sol. Distance travelled in 6 days by the man
Time taken =
6 travelling 9 km daily
x x 5 6 11

TS
⇒ – = + = = 9 × 6 = 54 km
5 6 60 60 60
6 x – 5x 11 Distance covered by the second man
= = 6 × 15 = 90 km
30 60

H
11 Let the total distance be x km.
x = × 30 = 5.5 km.
60 Time taken by the two men to cover the

G
25. Ans. (d) remaining distances are given as
Sol. Let them meet after t hours. Distance x – 54 x – 90
Speed of first man = 4 km/hr
Distance covered = 4t km
U Speed =
18
15(x – 54) = 18(x – 90)
=
15
O
18x – 15x = –15 × 54 + 18 × 90
Note : The second man’s speed is
comparable with that of an arithmetic 3x = –810 + 1620
H
progression; with the speed changing by 810
1 x = = 270 kms
3
_T

km every one hour.


2
∴ Sum of the distance covered by the x – 54 270 – 54
∴ Time taken = =
second man 18 18
C

t 1 t 8 + (t – 1) 216
= 2 × 2 + (t – 1)× = = = 12
2 2 2 2 18
PS

t
∴ Total time taken = 12 + 6 = 18 days
∴ 4t + [8 + (t – 1)] = 72
Shortcut
4
t Distance travelled by the first man in 6
i.e., 4t + [t + 7] = 72
U

4 days = 9 × 6 = 54 km
16t + t2 + 7t = 288 Distance travelled by the second man in
@

2
t + 23t – 288 = 0 6 days = 6 × 15 = 90 km
Difference in distance
(t + 32)(t – 9) = 0
t = –32 or t = 9 = 90 – 54 = 36 km
Difference in speeds after 6 days
‘–ve’ value is not admissible
∴ t = 9 hours = 9 × 2 – 15 = 3 km/hr.
36
The first man covers 9 × 4 = 36 km Time taken by 3 km/day = = 12
3
⇒ The second man also covers 36 km Total time = 6 + 12 = 18 days.
⇒ They meet midway between A and B.
26. Ans. (c) 28. Ans. (b)
Sol. Let the speed of one car be x km/hr
Sol. Let t be the time after which they meet. If ⇒ Speed of the slower car
one distance is 500, the remaining = (x – 15) km/hr
distance is 300 km. Since the cars are moving in opposite
268 ✫ Quantitative Aptitude

directions, their relative speed = sum of ∴ Total time taken to cover 5 km


their speeds = 30 + 20 = 50 minutes.
= x + x – 15 33. Ans. (c)
Sol. Let the length of this journey be x km.
1000 So the time taken to cover x kms is
i.e., x + x – 15 = = 125
8 given as
2x = 125 + 15 = 140 x x x 10
x = 70 km/hr. + + = 5+
3 × 4 2 × 12 6 × 9 60
29. Ans. (c)
x x x 31

TS
Sol. Distance covered by the wheel = 2πr + + =
12 24 54 6
D Multiplying by 6 throughout
∴ Speed = km/hr
T
x x x
22 500 35 18 ⇒ + +

H
= 31
= 2× × × × 2 4 9
7 60 100 5
= 22 × 3 = 66 km/hr. 18 x + 9 x + 4 x

G
= 31
36
30. Ans. (c)
Sol. D = 2πnr = πn × d when n = number of 31x = 31 × 36
revolutions
∴ Number of revolutions of the front wheel U ⇒
34. Ans. (b)
x = 36 km.
O
D 450 + 350
n = Sol. 60 seconds =
πd 5
x + 30 ×
H
154 ×1000 × 100 × 7 18
= = 70000 where x is the speed of the second train
22 × 70
in m/sec.
_T

Number of revolutions of the hind wheel


800 × 3
154 ×1000 × 100 × 7 ∴ 60 =
= = 50000 3 x + 25
22 × 98
800 × 3
C

∴ More number of revolutions of the 3x + 25 = = 40


60
front wheel than the hind wheel 3x = 40 – 25
PS

= 70000 – 50000 = 20,000. 3x = 15


31. Ans. (b) 18
Distance x = 5 × = 18 km/hr.
5
Sol. Time = Speed
U

35. Ans. (a)


Let x be the distance at which cycle got Sol. Let us assume that distance travelled
punctured.
@

by the train is 270 km, i.e., the L.C.M.


x 2(72 – x ) 17 of 27 and 30.
+ =
12 9 2 ∴ Including stoppages, time taken
6 x + 16 × 72 – 16 x 17
= 270
72 2 = = 10 hrs.
27
17
1152 – 10x = ×72 Excluding stoppages, time taken
2
1152 – 10x = 612 270
= = 9 hrs.
10x = 540 30
x = 54 km. i.e., within 10 hours travel, stoppage time
32. Ans. (a) = 1 hr.
Sol. Rest time = 4 × 5 = 20 min. ⇒ Stoppage time in one hour
Speed 10 km/hr 1
⇒ 5 km in 30 minutes = × 60 minutes = 6 minutes.
10
Unit Three : Practice Session ✫ 269

36. Ans. (a) 39. Ans. (c)


Sol. Distance covered = 24 × 6 = 144 km Sol. Let C’s speed = x km/hr
Required speed to make up for lost time B’s speed = 3x km/hr
144 and A’s speed = 6x km/hr
= = 36 km/hr. ∴ Ratio of speeds of A, B and C
4
37. Ans. (a) = 6x : 3x : x
Sol. When they travel in opposite directions, = 6 : 3 : 1
1 1
Relative speed Ratio of time taken = : : 1
6 3

TS
= Sum of speeds and time = 1 hr. = 1 : 2 : 6
120 If C takes 6 minutes, A takes 1 minute
i.e., = 120 = Sum of speeds ...(i)
1 If C takes 42 minutes, A takes
When they travel in the same direction, 1

H
speed = difference of speed; and time × 42 min = 7 minutes.
6
= 6 hrs. 40. Ans. (b)

G
120
i.e., = 20 = difference of speed ...(ii) Sol. It may be solved algebraically
6
Rate × Time = Distance


From (i) and (ii) it’s clear that the
speeds are 70 and 50 kmph.
The required answer = 70 km/hr. U Going 12 ×
1
= 6
O
2
38. Ans. (c) 1 3
Coming x × 1 = x
2 2
H
Sol. Ratio of speeds of the cars = 2 : 3 : 4
1 1 1 (where x = the speed at which she
⇒ Ratio of time= : : walks if she returns home at 1 p.m.)
2 3 4
_T

1 1 1 The distances are equal.


= 12 × : 12 × : 12 × 3
2 3 4 6 = x or 12 = 3x or x = 4 kmph.
= 6 : 4 : 3. 2
C
PS
U
@
270 ✫ Quantitative Aptitude

Practice Exercise 21
1. A train 500 m long enters a tunnel 1.5 km hours after a goods train leaves, and
long at 7.15 a.m. and is completely out overtakes it in 4 hours. What is the speed
of it at 7.19 a.m. Find the speed of train. of the goods train?
(a) 50 kmph (b) 40 kmph (a) 32 kmph (b) 40 kmph
(c) 30 kmph (d) 60 kmph (c) 45 kmph (d) 50 kmph
2. A train overtakes a man who is walking 8. A person standing on a railway bridge
along the rails in the same direction at 5 which is 50 m long finds that a train

TS
km/hr and completely overtakes him in 1
crosses the bridge in 4 seconds, but
10 seconds. If the length of the train is 2
100 m, then its speed is himself in 2 seconds. The length of the
train is

H
(a) 31 kmph (b) 41 kmph
(c) 36 kmph (d) 40 kmph (a) 50 m (b) 100 m
(c) 200 m (d) 40 m

G
3. A man standing on a platform notes that
a train going in one direction takes 3 9. Kerala Express train consists of 15 bogies,
seconds to pass him; a train of the same each 15 metres long. The train crosses a
length going in the opposite direction
takes 4 seconds. How long did they take U telegraph post in 18 seconds. Due to some
problems, 3 boggies were detached. Find
O
to pass each other? the time taken by the train to cross the
telegraph post now.
3 4
(a) 3 seconds (b) 3 seconds (a) 14.4 seconds (b) 13 seconds
H
7 7 (c) 12.8 seconds (d) 11 seconds
1 1
(c) 3 seconds (d) 3 seconds 10. The length of the bridge which a train,
_T

3 8
130 metres long and travelling at 45 km
4. Two trains of equal length are running on
per hour, can cross in 30 seconds, is
parallel lines in the same direction at 46
(a) 200 m (b) 225 m
and 36 kmph. The faster train passes the
C

(c) 245 m (d) 250 m


slower train in 36 sec. The length of each
train is 11. A train overtakes two persons who are
PS

(a) 47 m (b) 50 m walking in the same direction at the rate


(c) 53 m (d) 56 m of 2 and 4 kmph and passes them in 9
and 10 seconds respectively. The length of
5. Two guns were fired from the same place
the train is
U

at an interval of 10 minutes and 30


(a) 35 m (b) 40 m
seconds, but a person in a train
(c) 45 m (d) 50 m
approaching the place hears the second
@

shot 10 minutes after the first. The speed 12. A toy train crosses two tunnels 210 metres
of the train, supposing that sound travels and 122 metres long in 25 and 17 seconds
at 330 metres per second, is respectively. Find the speed of the train.
(a) 59.4 kmph (b) 57 kmph (a) 40 kmph (b) 39.6 kmph
(c) 55.03 kmph (d) 51.2 kmph (c) 38.9 kmph (d) 37 kmph
6. A train 100 m long meets a man going 13. A 150-metre-long train crosses a bridge of
in the opposite direction at the rate of 5 length 250 metres in 30 seconds. Find the
km per hour and passes him in 7.2 time for the train to cross a platform of
seconds. At what rate is the train going? 130 metres.
(a) 50 kmph (b) 45 kmph (a) 18 seconds (b) 19 seconds
(c) 42 kmph (d) 40 kmph (c) 20 seconds (d) 21 seconds
7. A passenger train running at the speed of 14. A train running at a speed of 48 km/hr
80 km/hr leaves the railway station 6 takes 10 seconds to pass a man running
Unit Three : Practice Session ✫ 271

at 6 km/hr in the opposite direction. Find distance from the station A to the point
the length of the train. where both the trains meet is
(a) 120 m (b) 150 m (a) 80 km (b) 120 km
(c) 180 m (d) 210 m (c) 100 km (d) None of these
15. A train running at a speed of 48 km/hr 21. Two trains 121 metres and 99 metres in
takes 15 seconds to cross a telegraph post. length respectively are running in opposite
Find the time taken to cross a bridge 240 directions, one at the rate of 40 km/hr
metres long. and the other at the rate of 32 km/hr. In
(a) 15 seconds (b) 30 seconds what time will they completely cross each

TS
(c) 33 seconds (d) 48 seconds other?
(a) 10 seconds (b) 10.5 seconds
16. The length of a train and that of a
(c) 10.45 seconds (d) 11 seconds
platform are equal. If with a speed of 90
22. A train 100 m long passes a platform 100

H
km/hr the train crosses the platform in
one minute, then the length of the train m long in 10 seconds. The speed of the
(in metres) is train is

G
(a) 500 (b) 600 (a) 36 kmph (b) 45 kmph
(c) 750 (d) 900 (c) 63 kmph (d) 72 kmph
17. A train 110 m long takes three seconds
to pass a standing man. How long is the U 23. Two trains run in the same direction at
60 kmph and 96 kmph respectively. If the
O
platform if the train passes through it in faster train passes a man sitting in the
15 seconds moving with the same speed? slower train in 20 seconds, then the length
of the faster train is
H
(a) 440 m (b) 400 m
(c) 550 m (d) 450 m (a) 100 m (b) 125 m
(c) 150 m (d) 200 m
_T

18. If a train overtakes 2 persons walking at


the rate of 2 km/hr and 4 km/hr (all 24. Two trains start running at the same time
travelling in the same direction) in 9 from two stations, 210 km apart and going
seconds and 10 seconds respectively, find in opposite directions they cross each
C

the length of the train and its speed in other at a distance of 100 km from one
km/hr. of the stations. The ratio of their speed
PS

(a) 0.05 km and 22 km/hr is


(b) 0.25 km and 11 km/hr (a) 9 : 5 (b) 10 : 7
(c) 200 m and 22 km/hr (c) 11 : 10 (d) 12 : 11
(d) None of these
U

25. Two trains 120 m and 100 m long run at


19. A man standing on a platform notes that the rate of 35 kmph and 55 kmph
a train going in one direction takes 3 respectively on parallel lines in same
@

seconds to pass him; a train of the same directions. To pass each other, they will
length in opposite direction takes 4 seconds. take
How long did they take to pass each (a) 39.6 seconds (b) 40 seconds
other? (c) 18.4 seconds (d) 10.5 seconds
1 1 26. A train covers a distance in 40 minutes,
(a) 2 seconds (b) 3 seconds if it runs at a speed of 50 kmph on an
7 7
2 3 average. The speed at which the train
(c) 3 seconds (d) 3 seconds must run to reduce the time of journey
7 7
20. The distance between two stations A and to 30 minutes will be
B is 220 km. A train leaves the station A (a) 67.43 kmph (b) 66.68 kmph
with a speed of 80 km per hour. After half (c) 65 kmph (d) 64.59 kmph
an hour, another train departs from the 27. Two stations A and B are 110 km apart
station B with a speed of 100 km/hr. The on a straight line. One train starts from
272 ✫ Quantitative Aptitude

A at 7 am and travels towards B at 20 29. A train covers a distance in 70 minutes


kmph. Another train starts from B at 8 if it runs at a speed of 50 km/hr on an
am and travels towards A at a speed of average. At what speed the train must run
25 kmph. At what time will they meet? to reduce the time of journey to 50
(a) 10 am (b) 9 am minutes?
(c) 8 am (d) 7 am (a) 40 kmph (b) 50 kmph
28. A man takes 8 hours to walk to a certain (c) 60 kmph (d) 70 kmph
place and ride back. However, he could 30. Including stoppages, the speed of a bus
have gained 2 hours, if he had covered is 40 km/hr and excluding stoppages, it

TS
both ways by riding. How long would he is 52 km/hr. For how many minutes does
take to walk both ways? the bus stop per hour?
(a) 9 hours (b) 10 hours (a) 10 minutes (b) 14 minutes
(c) 11 hours (d) 12 hours (c) 18 minutes (d) 21 minutes

H
Answers with Solutions

G
1. Ans. (c) Distance covered in 36 seconds at this
speed
Sol. Speed of the train
=
1500 + 500
4 × 60
=
2000
4 × 60 U ∴
= 36 ×
25
= 100 m
9
Length of each train is 2x = 100
O
50 18 ⇒ x = 50 m.
= × = 30 km/hr.
H
6 5 5. Ans. (a)
2. Ans. (b) Sol. Let the speed of the train be x m/sec.
D
_T

Then, distance travelled by the train


Sol. Speed =
t in 10 minutes = distance travelled by
Let x be the speed of train in m/s. sound in 30 seconds
5 100 ⇒ x × 10 × 60 = 330 × 30
i.e., x – 5 ×
C

= = 10 m/s
18 10 ⇒ x = 16.5
25 205 18
x = 10 + = m/s ∴ Speed of the train = 16.5 ×
PS

18 18 5
205 18 = 59.4 kmph.
= × = 41 km/hr 6. Ans. (b)
18 5
Hence, the speed of train is 41 km/hr. Sol. Let x m/s be the speed of the train
U

5 100
3. Ans. (a) x +5× =
18 7.2
Sol. Time taken by trains to pass each
@

18 x + 25 100 × 10
other =
18 72
2xy 2×3×4
= x+y = 100 ×10 ×18
3+4 18x = – 25
72
24 3 225
= = 3 seconds. x =
7 7 18
4. Ans. (b)
25 18
Sol. Let the length of both the trains be = × km/hr
x metres 2 5
Difference in their speeds is 46 – 36 = 45 km/hr.
7. Ans. (a)
= 10 kmph
Sol. Distance covered by the passenger train
5 25 in 4 hrs.
= 10 × = m/s
18 9 = 80 × 4 = 320 kms
Unit Three : Practice Session ✫ 273

= distance covered by goods trains in 5 10


10 hrs. and 4 kmph = 4 × = m/s
320 18 9
∴ Speed of the goods train = Let the speed of the train be x m/s.
10
= 32 km/hr. Then the relative speeds are
8. Ans. (d) 5 10
Sol. Let the length of the train be x metres. x– m/s and x– m/s
9 9
∴ Speed of the train (while passing the
man) Now, length of train
x = Relative speed × time taken to
= m/s

TS
2 pass a man
The speed of train while passing the 5
bridge is the same ∴ Length of train = x– ×9
9
x x + 50 10

H
i.e., = = x– ×10
2 9 9
2

G
100 45 55
x 2 x + 100 ∴ x = – = m/s
i.e., = 9 9 9
2 9
9x = 4x + 200
5x = 200 U ∴ Speed of train =
55 18
9
×
5
= 22 kmph
O
x = 40 m. 5
and length of train = x– ×9
9
9. Ans. (a)
H
55 5
= – × 9 = 50 m
15 × 15 25 9 9
Sol. Speed of the train = = m/s.
_T

18 2 Shortcut
Time taken after the detachment of 3 Length of the train
boggies
Difference in speed × T1 × T2 of two men
C

15 × 12 15 ×12 × 2 =
= = T2 – T1
25 25
PS

2 10 5
– × 9 × 10
72 9 9
= = 14.4 seconds. = = 50 m.
5 10 – 9
10. Ans. (c) 12. Ans. (b)
U

Sol. Let the bridge be x metres long. Sol. Let the length of the train be x m and
130 + x the speed of the train be y m/s.
@

i.e., = 30
5 x + 210
45 × ∴ y = 25 ...(i)
18
x + 122
(130 + x) × 18 = 30 × 45 × 5 = 17 ...(ii)
y
30 × 45 × 5 Divide equation (i) by equation (ii),
x = – 130
18 we get
x = 375 – 130 x + 210 25
⇒ =
= 245 m. x + 122 17
⇒ 17x + 17 × 210 = 25x + 25 × 122
11. Ans. (d)
Sol. Speeds of two men are 2 kmph i.e., 17x + 3570= 25x + 3050
5 5 520 = 8x
= 2× = m/s 520
18 9
x = = 65 m
8
274 ✫ Quantitative Aptitude

65 + 210 3x = 15 × 110 – 3 × 110


From (i), speed (y) = 12 ×110
25 x =
275 3
= = 11 m/s = 440 m.
25
18. Ans. (a)
11×18
= = 39.6 km/hr. Sol. Let the length of the train be x m
5 Let the speed of the train be y metres/
13. Ans. (d) sec.
150 + 250 In passing the first person, the time
Sol. Speed =

TS
30 taken will be
400 40
= = m/s x
30 3 9 =
∴ Time taken to cross the platform 5
y – 2×

H
150 + 130 280 18
= = ×3 x
40 40 9 =
5

G
3 y–
= 21 seconds. 5 9
9y – 9 × = x
9
14. Ans. (b)
Sol. 10 =
Length of train
5 U 9y – x = 5 ...(i)
In passing the second person, the time
O
(48 + 6)× taken will be
18
5 x x
∴ Length of train = 54 × ×10
H
18 10 = =
5 10
x – 4× y–
= 150 m. 18 9
_T

15. Ans. (c) 100


Sol. Length of the train = 48 × 5 × 15 10 y –
9
= x
18
= 200 m 100
10y – x = ...(ii)
C

Time taken to cross the bridge 9


200 + 240 440 ×18 Solving (i) and (ii), we get
PS

= = 100
5 48 × 5
48 × –y = 5 –
18 9
= 33 seconds. –55
16. Ans. (c) =
U

9
Sol. Let the length of train be x m. 55
D 2x ⇒ y = m/s.
@

T = ⇒ 60 = 9
S 5
90 × 55 18
2 x ×18 18 = ×
60 = 9 5
90 × 5
= 22 km/hr.
60 × 25
x = By putting the value of y in equation (i),
2
= 750 m. we get
9y – x = 5
17. Ans. (a) 110
Sol. Train’s speed = m/s 55
3 9× –5 = x
110 + x 9
110 = 15 x = 50 m
3 50
= = 0.05 km.
3 × (110 + x) = 15 × 110 1000
Unit Three : Practice Session ✫ 275

19. Ans. (d) x


Then, 10 =
Sol. Let the length of the trains be x m each. 20
x ⇒ x = 200 m.
Speed of train I =
3
x 24. Ans. (c)
Speed of train II = Sol. Let the speed of two trains be x kmph
4
and y kmph.
x+x 2x
Time taken = = Time taken by first train to cover 110
x x 4 x + 3x km = time taken by second train to
+
3 4 12 cover 100 km

TS
2x ×12 x y
= Thus, =
7x 110 100
24 3 x 110 11
= = 3 seconds. ⇒

H
7 7 y = 100 = 10
20. Ans. (b) Thus, the ratio is 11 : 10.

G
Sol. The distance covered by the train from 25. Ans. (a)
A in 30 minutes = 40 km
Sol. Time taken to cross each other
Time taken by the trains to cross each
other
220 – 40 180 U =
120 + 100
O
= = = 1 hour 5
100 + 80 180 (55 – 35)×
18
∴ The distance covered from A 220 × 18
H
=
= 40 + 1 × 80 20 × 5
= 120 km. = 39.6 seconds.
_T

21. Ans. (d) 26. Ans. (b)


121 + 99 Sol. Distance travelled in one hour = 50 km
Sol. Time taken =
5
(40 + 32) × ∴ Distance travelled in 40 minutes
C

18
220 × 18 50
= = × 40
PS

72 × 5 60
= 33.34 km
220
= = 11 seconds. Time to be reduced to 30 minutes
20
30
U

22. Ans. (d) =hour


60
Sol. Distance covered by the train in 10 sec
33.34
@

= 100+100 = 200 m Required speed =


30
Distance covered by the train in 1 hour 60
(= 3600 sec) 33.34 × 60
200 3600 = = 66.68
× 30
= km
10 1000 = 66.68 kmph.
= 72 kmph. 27. Ans. (a)
Sol. Till 8 am, the train from A covers a
23. Ans. (d) distance of 20 km.
Sol. Let the length of the faster train be x m. Now, the remaining distance
∴ The relative velocity of the train = 110 – 20 km
= 96 – 60 = 36 kmph = 90 km
90 km is covered by the trains with
5 relative speed
= 36 × = 10 m/sec
18 = 20 + 25 kmph = 45 kmph
276 ✫ Quantitative Aptitude

90 29. Ans. (d)


∴ They meet after = = 2 hours Sol. Distance = Speed × Time
45
10
That is, at 8 + 2 = 10 am. = 50 × 1+ km/hr
60
28. Ans. (b) 7 175
Sol. Walking time + Riding time = 50 × = km/hr
6 3
= 8 hours ...(i) Distance
2 × riding time Speed =
Time

TS
= 8 – 2 = 6 hours ...(ii) 175
175 60
Multiplying by 2 in equation (i) and = 3 = × = 70 km/hr.
subtracting equation (ii) from that, we 50 3 50
get 60

H
2 × walking time = 2 × 8 – 6 30. Ans. (b)
2 × walking time = 10 hours. Sol. Due to stoppages, it covers

G
(52 – 40) = 12 km less
Shortcut Now, time taken to cover 12 km
Both ways walking
= one way walking and one way
U =
12
52
× 60 minutes
O
riding time + gain in time. = 13.84 minutes
= 8 + 2 = 10 hours. = 14 minutes (approximate).
H
C _T
PS
U
@
Unit Three : Practice Session ✫ 277

Practice Exercise 22
1. If a man can row upstream at 10 kmph 3 km/hr, the distance between A and B
and downstream at 14 kmph then the (in km) is
man’s rate in still waters is (a) 12 (b) 6
(a) 2 kmph (b) 10 kmph (c) 4 (d) none of the above
(c) 12 kmph (d) 16 kmph 9. A man can row a certain distance
2. The speed of a boat in still waters is 15 downstream in 6 hours and returns the
kmph and the rate of the current is 3 same distance in 9 hours. If the stream

TS
kmph. The distance travelled downstream flows at the rate of 2 km/hr, what will be
in 12 minutes is the man’s speed in still waters?
(a) 1.2 km (b) 1.8 km (a) 10 km/hr (b) 12 km/hr
(c) 8 km/hr (d) 7.6 km/hr

H
(c) 2.4 km (d) 3.6 km
3. The speed of a boat downstream is 20 10. A person rows one kilometre down the

G
kmph and the speed of the stream is 2 stream in 10 minutes and then up the
kmph. The speed of the boat upstream is stream in 30 minutes. Find the velocity of
(a) 12.7 kmph (b) 13 kmph the stream.
(c) 14.9 kmph (d) 16 kmph
U (a) 1.5 km/hr
(c) 2.5 km/hr
(b) 2 km/hr
(d) 3 km/hr
O
4. A boat goes 80 km upstream in 16 hours
and 72 km downstream in 12 hours. The 11. A boat goes in the direction of the current
speed of the boat in standing waters is at 8 km/hr and against the current at 4
H
(a) 5.5 kmph (b) 5.9 kmph km/hr, for going to a place B from A and
(c) 6.2 kmph (d) 6.4 kmph back to A from B in 45 minutes. Find the
_T

distance between A and B.


5. The current of a stream runs at 4 km an (a) 2 km (b) 2.25 km
hour. A boat goes 6 km and comes back to (c) 2.5 km (d) 3 km
the starting point in 2 hours. The speed of
12. A boat goes downstream in half the time
C

the boat in still waters is


it takes to go upstream; what is the ratio
(a) 6 kmph (b) 7 kmph
of the speed in still waters to that of the
PS

(c) 8 kmph (d) 9 kmph


stream?
6. A man rows a certain distance down a (a) 1 : 2 (b) 3 : 1
stream in 6 hours and returns the same (c) 1 : 3 (d) 2 : 1
U

distance in 9 hours. If the stream flows at


13. A boat travels 10 km in 1 hour downstream
the rate of 2.25 kmph, find how far he can
and 14 km in 2 hours upstream. Find how
row in an hour in still waters.
@

much this boat will take to travel 17 km


(a) 14 kmph (b) 12.98 kmph in still waters.
(c) 12.05 kmph (d) 11.25 kmph 1
7. A man can row at 5 kmph in still waters. (a) 1 hour (b) 1 hours
2
If the speed of the current is 1 kmph and 1
it takes him 1 hour to row to a place and (c) 2 hours (d) 2 hours
2
come back, how far is the place? 14. A boat goes upstream a certain distance
(a) 2.2 km (b) 2.4 km in 3 hours and returns the same distance
(c) 2.6 km (d) 2.8 km downstream in 2 hours. If the speed of the
8. A boat travels upstream from B to A and boat in still waters is 25 km/hr, find the
downstream from A to B in 3 hours. If speed of the stream.
the speed of the boat in still waters is (a) 5 km/hr (b) 10 km/hr
9 km/hr and the speed of the current is (c) 15 km/hr (d) 20 km/hr
278 ✫ Quantitative Aptitude

15. A motor boat, whose speed is 15 km/hr (a) 1 km (b) 1.5 km


in still waters, goes 30 km downstream (c) 2 km (d) 2.88 km
and comes back in a total of 4 hrs 30 21. A man swims downstream 20 km and
minutes. The speed of the stream in km/ upstream 8 km, taking 4 hours each time.
hr is Then the velocity of stream is
(a) 5 (b) 4 (a) 1.05 kmph (b) 1.25 kmph
(c) 10 (d) 6 (c) 1.5 kmph (d) 1.75 kmph
16. Speed of a boat in still waters is 9 km/hr 22. A boat, while going downstream in a river
and the speed of the stream is 1.5 km/hr. A covered a distance of 50 miles at an

TS
man rows to a place at a distance of 10.5 km average speed of 60 miles per hour. While
and comes back to the starting point. The returning, because of the water resistance,
total time taken by the man was it took 1 hour 15 minutes to cover the
(a) 1.6 hrs (b) 2 hrs

H
same distance. What was the average
(c) 2.2 hrs (d) 2.4 hrs speed during the whole journey?
17. The current of stream runs at 1 km/hr. (a) 40 mph (b) 48 mph

G
A motorboat goes 35 km upstream and (c) 50 mph (d) 55 mph
back again to the starting point in 12 23. A boat goes 20 km upstream in 2 hours
hours. The speed of the boat in still
waters is
U and downstream in 1 hour. Find how
much time this boat will take to travel 30
O
(a) 6 kmph (b) 5 kmph km in still waters.
(c) 6.5 kmph (d) 7 kmph
1
(a) 1 hr (b) 1 hr
H
18. A man can row 20 kmph in still waters. 2
It takes him thrice as long to row up as 1
to row down the river. Find the rate of (c) 2 hrs (d) 2 hrs
_T

2
the stream.
24. A swimmer swims from a point A against
(a) 10 kmph (b) 12 kmph
the current for 5 minutes and then swims
(c) 30 kmph (d) 100 kmph
along with the current for the next 5
C

19. If a boat goes 10 km upstream in 50 minutes and comes to the point B. If AB


minutes and speed of the stream is 2 = 100 m, find the speed of the current.
PS

kmph. Then the speed of boat in still (a) 0.6 km/hr (b) 1 km/hr
waters is (c) 2 km/hr (d) None of these
(a) 5 kmph (b) 14 kmph
25. A man’s speed of rowing in still waters
U

(c) 26 kmph (d) 30 kmph


is 5 km/hr. If the speed of the stream is
20. A man rows 6 kmph in still waters. When 1 km/hr, it takes him 75 minutes to row
@

the river is running at 1.2 kmph, it takes to a place and back. How far is the place?
him 1 hour to row to a place and back. (a) 3 km (b) 4 km
How far is the place? (c) 3.5 km (d) 4.5 km
Answers with Solutions
1. Ans. (c) 1 3. Ans. (d)
Sol. Rate in still waters = (10 + 14) Sol. Let the speed of boat in still waters be
2
= 12 kmph. x kmph.
Then, x + 2 = 2 0
2. Ans. (d) ⇒ x = 18
Sol. Speed downstream = (15 + 3) ∴ Speed upstream = (18 – 2)
= 18 kmph = 16 kmph
Distance travelled = Speed × Time 4. Ans. (a)
12 80
= 18 × = 3.6 km. Sol. Speed upstream = = 5 kmph
60 16
Unit Three : Practice Session ✫ 279

Speed downstream Shortcut


72 Downstream rate = 9 + 3 = 12
= = 6 kmph
12 Upstream rate = 6
Speed of boat in still waters Let x be distance in km.
1 downstream distance
= (5 + 6) =5.5 kmph. Time taken =
2 downstream rate
5. Ans. (c) upstream distance
+
Sol. Let the speed in still waters be x kmph upstream rate
6 6 x x

TS
Then, + = 2 ∴ + =3
x+4 x–4 12 6
6[x – 4 + x + 4] = 2 (x 2 – 16) x + 2x
=3
⇒ x 2 –16 = 6x 12
3x = 3 × 12

H
⇒ 2
x – 6x – 16 = 0
or (x – 8)(x + 2) = 0 x = 12 km.
so, x = 8 kmph.

G
9. Ans. (a)
6. Ans. (d) Sol. Let the man’s speed in still waters be
Sol. Speed in still waters
Rate of stream (sum of upstream U i.e.,
x km/hr.
Downstream rate
and upstream rate
=
=
(x + 2) km/hr
(x – 2) km/hr
O
and downstream time) ∴ Distance covered = Speed × time
=
Difference of upstream and i.e., 6 (x + 2) = 9 (x – 2)
H
downstream time 6x + 12 = 9x – 18
2.25 (6 + 9) 2.25 ×15 3x = 30
= = = 11.25 kmph. ⇒
_T

9–6 3 x = 10 km/hr.
7. Ans. (b) 10. Ans. (b)
Sol. Speed downstream
Sol. Let the speed of the boat be x kmph and
= 5 + 1 kmph = 6 kmph
C

Speed upstream the speed of the stream be y kmph


= 5 – 1 kmph = 4 kmph 1
PS

x+y = × 60 = 6 km/hr ...(i)


Let the required distance be x km 10
x x 1
Then, + =1 x – y= × 60 = 2 km/hr ...(ii)
6 4 30
U

⇒ 2x + 3x = 1 2 From (i) and (ii), we get


⇒ 5x = 12 ⇒ 2y = 4
⇒ x = 2.4 km. ⇒ y = 2 km/hr.
@

8. Ans. (a) 11. Ans. (a)


Sol. Total time taken = 3 hours Sol. Let the distance between A and B be x
x = 9 km/hr and y = 3 km/hr
kms
Total time taken × upstream rate ×
downstream rate = man’s rate in still Distance
waters × Total distance Time = Speed
∴ Distance from A to B x x 45
1 i.e., + =
= × total distance 8 4 60
2
x + 2x 45
1 (time taken × down rate × up rate) =
= × 8 60
2 man's rate in still water
45 8
1 3 ×(9 + 3) ×(9 – 3)
× x = × = 2 km.
= = 12 km. 60 3
2 9
280 ✫ Quantitative Aptitude

12. Ans. (b) 3 (25 – y) = 2 (25 + y)


Sol. Let the speed of the boat in still waters 75 – 3y = 50 + 2y
be x kmph and speed of the stream be 25 = 5y
y kmph. If d is the distance between the ⇒ y = 5 km/hr.
starting and end points, then time taken 15. Ans. (a)
is given as Sol. Let the speed of the stream be y km/hr.
d 30 30 9
+
x+ y
=
1 15 + y 15 − y = 2
d

TS
2
30 (15 − y ) + 30 (15 + y ) 9
x−y =
225 − y 2 2
x−y 1
i.e., x+y = 2 2 (450 – 30y + 450 + 30y) = 9 × (225 – y2)

H
9y 2 = 2025 – 2 × 900
⇒ 2x – 2y =x+y
225
2x – x = y + 2y y =

G
x = 3y 9
x 3 15

13. Ans. (c)


y
= .
1
U 16. Ans. (d) 10.5
=

10.5
3
= 5 km/hr.
O
Sol. If x is the speed of the boat in still Sol. Total time = +
waters and y is the speed of the stream, 9 + 1.5 9 − 1.5
7
= 1+
H
then, = 2.4 hours.
10 5
x+y = = 10 km/hr ...(i) 17. Ans. (a)
1
_T

14 Sol. Let x kmph be the speed of the motor


x–y = = 7 km/hr ...(ii)
2 boat in still waters.
From (i) and (ii), we get 35 35
⇒ +
C

2x = 17 = 12
x +1 x −1
x = 8.5 km/hr
⇒ 70x = 12x 2 – 12
PS

and y = 1.5 km/hr


⇒ 2
12x – 70x – 12 = 0
17
Required time = = 2 hrs. ⇒ 6x 2 – 35x – 6 = 0
8.5 ⇒ (x – 6)(6x + 1) = 0
14. Ans. (a) 1
U

Sol. Let the distance be d kms and the speed ⇒ x = 6 or x = − which is inadmissible.
6
of stream be y km/hr. ∴ The speed of the motorboat in still
@

Distance waters is 6 kmph.


Time = Speed
18. Ans. (a)
d
∴ =2 ...(i) Sol. Let the man’s rate upstream be x kmph
25 + y
d Then, rate downstream is 3x kmph
and =3 ...(ii) 1
25 − y ∴ Rate in still waters = (3x + x )
From equations (i) and (ii), we get 2
d = 2x kmph
25 + y 2 So, 2x = 20 ⇒ x = 10
d
= ∴ Rate upstream is 10 kmph; rate
3
downstream is 30 kmph.
25 − y Hence, rate of the stream
25 − y 2 1
i.e., 25 + y = 3 = (30 − 10) = 10 kmph.
2
Unit Three : Practice Session ✫ 281

19. Ans. (b) 23. Ans. (c)


10 20
Sol. Rate upstream = × 60 kmph Sol. x + y = downstream rate =
50 1
= 12 kmph = 20 km/hr ...(i)
20
Speed of the stream = 2 kmph x – y = upstream rate =
Let the speed in still waters be x kmph. 2
Then, the speed upstream = 10 km/hr ...(ii)
= (x – 2) kmph where x is speed of boat in still waters
∴ x – 2 = 12 and y is the speed of the stream.
⇒ x = 14 kmph. From equations (i) and (ii), we get

TS
20. Ans. (d) 2x = 30
z (x 2 − y 2 )
Sol. Using formula = ⇒ x = 15 km/hr
2x
Here z = 1, x = 6 and y = 1.2 and y= 5 km/hr

H
1 (62 − 1.2 2 ) 36 − 1.44 30
⇒ = = Time = = 2 hours.
2×6 12 15

G
34.56 24. Ans. (a)
= = 2.88 km.
12 Sol. Let the speed of the swimmer be x
21. Ans. (c)
Sol. Speed =
Distance U km/hr and the speed of the stream be
y km/hr.
O
Time ∴ Swimmer’s downstream rate
Man’s speed downstream
20 = (x + y) km/hr
H
= = 5 kmph Swimmer’s upstream rate
4
Man’s speed upstream = (x – y) km/hr
_T

8
= = 2 kmph Distance travelled in 5 minutes is given
4
∴ Velocity of the stream as
(5 − 2)
C

5( x + y) 5(x − y) 100
= = 1.5 kmph. − = km
2 60 60 1000
22. Ans. (b)
PS

( x + y) ( x − y) 1
Sol. Time taken during downstream journey − =
12 12 10
50 5
= = hours 12
60 6
U

(x + y) – (x – y) =
Time taken during upstream journey 10
= 1 hour 15 minutes 1.2
@

15 5 y = = 0.6 km/hr.
= 1+ = hours 2
60 4 25. Ans. (a)
Total time taken during the whole journey
5 5 25 Sol. Let d be the distance one way
= + = hours d d 75
6 4 12 + =
Total distance covered 5+1 5−1 60
= 50 + 50 = 100 miles d d 75
+ =
∴ Average speed 6 4 60
Total distance covered 2d + 3d 75
= =
Total time taken 12 60
100
= = 48 mph. 75 × 12
25 d = = 3 km.
60 × 5
12
282 ✫ Quantitative Aptitude

Practice Exercise 23
1. A can run one kilometre in 4 minute 50 run over the same course and B wins by
seconds and B in 5 minutes. By what 4 m. C and D run over it and D wins
distance would A beat B? by 16 m. If A and D run over it, then
1 2 who would win and by how much?
(a) 33 m (b) 966 m (a) A, by 8.4 m (b) D, by 8.4 m
3 3
2 (c) A, by 7.2 m (d) D, by 7.2 m
(c) 66 m (d) 100 m
3 9. At a game of billiards, A can give B 5

TS
1 points in 30 and he can give C 7 points
2. A runs 2 times as fast as B. If A gives
3 in 30. How many can B give C in a game
B a start of 60 metres, how far must be of 40?
the destination point if the race ends in

H
(a) 4 (b) 3
a deadlock? (c) 2 (d) 1
(a) 100 m (b) 200 m

G
(c) 105 m (d) 150 m 10. Raj, Aryan and Rohit walk around a circle
1760 metres in circumference at the rate
3. A can run 100 metres in 22.5 seconds and of 160 m, 120 m and 105 m per minute
B can run in 25 seconds. A will beat B
by
U respectively. If they all start together and
walk in the same direction, when will they
O
(a) 9 metres (b) 8 metres be first together again?
(c) 10 metres (d) 5 metres (a) 3 hours 38 minutes
H
4. In a 100-metre race A can beat B by 25 (b) 4 hours 30 minutes
metres and B can beat C by 4 metres. In (c) 5 hours 52 minutes
the same race, A can beat C by (d) 6 hours 21 minutes
_T

(a) 21 m (b) 26 m
11. Raman and Aditya run a 10-km race on
(c) 28 m (d) 29 m
a course 300 metres round. If their speed
5. A and B ran a race which lasted a minute is in the ratio of 5 : 4, how often does the
C

and a half. A gave B a start of 9 metres winner pass the other?


and beat him by 1 metre. A ran 40 metres (a) 11 times (b) 9 times
PS

while B ran 39 metres. Find the length of (c) 8 times (d) 5 times
the race.
(a) 200 m (b) 100 m 12. In a 1000-metre race, the ratio of the
(c) 400 m (d) 800 m speeds of Jyoti and Nandini is 5 : 6. Jyoti
U

3 has a start of 200 metres. Calculate the


6. S is 1 times as fast as J. If S gives J a distance by which Jyoti wins.
8
(a) 70 m (b) 60 m
@

start of 150 metres, how far must the


(c) 50 m (d) 40 m
winning post be placed so that the race
ends in a dead heat? 13. In a 500-metre race Dheeraj beats Sanjeev
(a) 100 m (b) 200 m by 20 metres or 10 seconds. What time
(c) 440 m (d) 550 m will be taken by Dheeraj to cover the
7. A can give B and C a start of 20 metres course?
and 25 metres respectively in a 100 metres (a) 4 minutes (b) 7 minutes
race; B can give C one second over the (c) 10 minutes (d) 13 minutes
course. How long does A take to run 100 14. In a 500-metre race Kinjal takes 2 minutes
metres? and 5 seconds while Nupur take 2 minutes
(a) 10 sec (b) 12 sec and 30 seconds to cover the course. By
(c) 15 sec (d) 16 sec what distance Kinjal will beat Nupur?
8. The racecourse is 400 m long. A and B (a) 83.33 m (b) 80.14 m
run a race and A wins by 5 m. B and C (c) 79.30 m (d) 71.98 m
Unit Three : Practice Session ✫ 283

15. In a 200-metre race, A runs at 12 km per 20. Three runners A, B and C run a race, with
hour if A gives B a start of 8 metres and runner A finishing 12 m ahead of runner
still beats him by 24 seconds. What is the B and 18 m ahead of runner C, while
speed of B? runner B finishes 8 m ahead of runner C.
(a) 7.3 kmph (b) 8 kmph Each runner travels the entire distance at
(c) 8.2 kmph (d) 8.7 kmph a constant speed. What was the length of
the race?
16. In a three-kilometre race, A beat B by 60 (a) 36 m (b) 48 m
metres or 10 seconds. Find A’s time over (c) 60 m (d) 72 m
the course?

TS
(a) 480 seconds (b) 490 seconds 21. A takes 4 minutes 50 seconds while B
takes 5 minutes to complete the race. A
(c) 500 seconds (d) 510 seconds 1
beat B by 33 m. Find the length of the
17. In a 100-metre race, Arjun runs 6 km/hr, 3

H
if Arjun gives Karan a start of 4 metres and course.
still beats him by 12 seconds. What is the (a) 200 m (b) 400 m
(c) 800 m (d) 1000 m

G
speed of Karan?
(a) 3.9 kmph (b) 4.8 kmph 22. A can beat B by 31 metres and C by 18
(c) 5 kmph (d) 5.6 kmph
18. Karan and Arjun run a 100-m race, where U metres in a race of 200 metres. In a race
of 350 metres, C will beat B by
(a) 23 m (b) 24 m
O
Karan beats Arjun by 10 m. Karan starts (c) 25 m (d) 28 m
10 m behind the starting line in a second
23. In a 500-metres race, B gives A a start of
H
100-m race. They both run at their earlier
speeds. Which of the following is true in 160 m. The ratio of the speeds of A and
B is 2 : 3. Who wins and by how much?
connection with the second race?
_T

(a) 6 m (b) 8 m
(a) Karan and Arjun reach the finishing 20
line simultaneously (c) m (d) 20 m
3
(b) Arjun beats Karan by 1 m
24. In a game of 100 points, A can give B 20
C

(c) Arjun beats Karan by 11 m points and C 28 points. Then B can give C
(d) Karan beats Arjun by 1 m (a) 8 points (b) 10 points
PS

19. In the first 10 overs of a cricket game, (c) 14 points (d) 40 points
the run rate was only 3.2. What should 25. A can give B 15 points, A can give C 22
be the run rate in the remaining 40 overs points, and B can give C 10 points. How
U

to reach the target of 282 runs? many points make the game?
(a) 6.25 (b) 6.50 (a) 50 (b) 100
@

(c) 6.75 (d) 7.00 (c) 150 (d) None of these

Answers with Solutions


1. Ans. (a) 2
i.e., A beat B by 1000 – 966
Sol. Let loser’s distance be x m 3
1
Winner’s time Loser’s time = 33 metres.
= 3
Loser’s distance Winner’s distance 2. Ans. (c)
290 sec 300 sec Sol. A : B = 7 : 3
i.e., = In a race of 7 metres, A gains
x 1000
290 × 1000 7 – 3 = 4 m over B
∴ x = ∴ The distance covered by A if A gains 60
300 metres
2900 2 60
= = 966 m = × 7 = 105 metres.
3 3 4
284 ✫ Quantitative Aptitude

3. Ans. (c) 7. Ans. (b)


Sol. When A runs 100 metres, B runs 80
22.5 25
Sol. = and C runs 75 metres
x 100 When B runs 100 metres, the distance
where x is the distance covered by B covered by C
22.5 × 100 75 375
 x = = × 100 = = 93.75 m
25 80 4
= 90 m ⇒ B beats C by 100 – 93.75
= 6.25 m or 1 sec.
 A can beat B by 10 metres.

TS
i.e., Distance covered by C in 1 sec
4. Ans. (c) = 6.25 m
Sol. A : B = 100 : 75 ∴ Time taken by C to cover 75 m
B : C = 100 : 96 75

H
A B 100 100 = = 12 sec.
× × 6.25
A : C = =
B C 75 96 ⇒ Time taken by A to run 100 m

G
= 100 : 72 = 12 seconds.
A beats C by (100 – 72) = 28 metres. 8. Ans. (d)
5. Ans. (c)
Sol. Let k be the length of the race
⇒ Distance covered by A = k metres U Sol. When A covers 400 m, B covers 395 m
When B covers 400 m, C covers 396 m
O
When B covers 395 m, C covers
Distance covered by B = k – (9+1) 396
= k – 10 metres × 395 ≈ 391.05 m
H
400
The rate of speed of A and B is given ∴ When A covers 400 m, C covers 391.05 m
40 : 39 :: k : k – 10 When D covers 400 m, C covers 384 m
_T

i.e., 39k = 40 (k – 10) When C covers 391.05 m, D covers


400 = 40k – 39k = k 407.3 m
i.e., The length of the race course ∴ When A covers 400 m, D covers 407.3 m
= 400 m. When D covers 400 m, A covers 392.8 m
C

6. Ans. (d) ∴ D wins by 7.2 m.


Sol. The race ends in a dead heat 9. Ans. (b)
PS

⇒ time taken by S and J are the same. Sol. A scores 30 points, then B scores 25
Distance points; if A scores 30 points, then C
Time = Speed scores 23 points
U

Now, when B scores 25 points, C scores


J’s distance S’s distance 23 points
i.e., =
∴ When B scores 40 points
@

J’s speed S’s speed


23
If k is the distance, and d is the speed C scores × 40 = 36.8
25
then = 37 points
8k Hence, B can give C, 3 points in a game
k − 150 k
= i.e., k – 150 = of 40.
d 11 11
d 10. Ans. (c)
8 Sol. Raj takes 1 minute to gain 40 m over
11k – 150 × 11 = 8k Aryan. To gain 1760 m over Aryan,
11k – 8k = 150 × 11 Raj takes
1
150 × 11 =
40
× 1760 = 44 minutes
k=
3 Raj takes 1 minute to gain 55 m over
= 550 metres. Rohit
Unit Three : Practice Session ✫ 285

∴ To gain 1760 m over Rohit, 14. Ans. (a)


1 Sol. Distance covered by Nupur in
Raj takes = × 1760 = 32 minutes
55 (30–5) seconds i.e. 25 seconds
Thus Raj and Aryan will be together
500
after 44 minutes, while Raj and Rohit = × 25 = 83.33 metres
150
are together again after 32 minutes. ∴ Kinjal beats Nupur by 83.33 metres.
Now taking L.C.M. of 44 and 32, we get
15. Ans. (c)
352
So, Raj, Aryan and Rohit are first Sol. Time taken by A to cover 200 metres

TS
together again after 352 minutes or 60 × 60
= × 200 sec = 60 sec
5 hours, 52 minutes. 12000
∴ B covers (200 – 8) or 192 metres in
11. Ans. (c) (60 + 24) or 84 sec.

H
Sol. When Raman makes 5 rounds, then Hence, speed of B
Aditya makes 4 rounds
192 × 60 × 60

G
Distance covered by Raman in 5 rounds = km/hr
is 84 × 1000
5 × 300 3
1000
km =
2
km
Distance covered by Aditya in 4 rounds U 16. Ans. (b)
= 8.2 km/hr.
O
Winner’s time
is Sol.
4 × 300 Loser’s distance
6
beat time + start time
H
km = km =
1000 5
beat distance + start distance
So in covering 10 km Raman passes
A’s time 10 + 0
_T

Aditya ⇒ =
3000 − 60 60 + 0
6 2
× × 10 = 8 times. 10
5 3 A’s time = × 2940 = 490
60
C

12. Ans. (d) A’s time over the course is 490 seconds.
Sol. To win the race Jyoti will have to cover 17. Ans. (b)
PS

a distance of (1000 – 200) = 800 metres. Sol. Time taken by Arjun to cover 100 metres
While Jyoti covers 5 metres, Nandini 60 × 60
= × 100 sec = 60 sec
covers 6 metres while Jyoti covers 800 6000
U

metres, Nandini covers ∴ Karan covers (100 – 4) or 96 metres in


6 (60 + 12) or 72 seconds
× 800 = 960 metres
@

5
When Jyoti reaches the winning line, Hence, speed of Karan
Nandini remains (1000 – 960) = 40 metres 96 × 60 × 60
= km/hr
behind. 72 × 1000
∴ Jyoti wins by 40 metres. = 4.8 km/hr.
13. Ans. (a) 18. Ans. (d)
Sol. 20 metres are covered by Sanjeev in Sol. Karan travels 100 m, Arjun travels only
10 seconds
90 m in the same time.
500 metres are covered by Sanjeev in
Hence ratio of their speeds is 10 : 9
10
× 500 = 250 seconds Now Karan runs for 100 + 10 = 110 m
20 Time in which Karan covers 110 m,
∴ Time taken by Dheeraj Arjun will cover 99 m
= (250 – 10) = 240 seconds Since the track is 100 m, Arjun is beaten
= 4 minutes. by 100 – 99 = 1 m.
286 ✫ Quantitative Aptitude

19. Ans. (a) 23. Ans. (c)


Sol. Runs scored in the first 10 overs Sol. A gets a start of 160 m
= 3.2 × 10 = 32 ⇒ Distance covered by A
Target is of 282 runs = 500 – 160 = 340 m
∴ Remaining runs 282 – 32 = 250 in 40 2 : 3 :: 340 : x
overs 3 × 340
∴ Run rate × 40 = 250 ∴ x = = 510 which is not the case
250 2
Run rate = = 6.25 per over. 2 : 3 :: x : 500
40

TS
20. Ans. (b) That implies, if B covers a distance of
Sol. Let the length of the race be x m. 500 m, then the distance covered by A
Distance travelled would be
2 × 500

H
= A : x, B : x – 12, C : x – 18 1000
When B goes x, C goes = =
3 3
( x − 18)

G
×x=x–8 1
x − 12 = 333
⇒ x (x – 18) = (x – 8) (x – 12) 3
1



x2 – 18x = x2 – 12x – 8x + 96
2x = 96
U ∴ Then B beats A by 340 – 333
2 20
3
O
x = 48 m. = 6 metres or metres.
3 3
21. Ans. (d)
24. Ans. (b)
H
Sol. Let k be the length of the race
Sol. A : B : C = 100 : 80 : 72
Winner’s time Loser’s time
=
_T

If B’s score = 100, then let C’s score be x


Loser’s distance Winner’s distance
80 : 72 :: 100 : x
290 300 72 × 100
i.e., = i.e., x =
80
C

1 k
k − 33
3 ∴ C’s score = 90 points
PS

290 × 3 300 i.e., B can give C 10 points.


=
3k − 100 k 25. Ans. (a)
290 × 3k = 300 × (3k – 100) Sol. Let x be the maximum points in the
U

300 × 100 = 300 × 3k – 290 × 3k game.


300 × 100 = 10 × 3k ∴ If A scores x points, B scores x – 15
@

300 × 100 points, and C’s score = (x – 22) points


k = If B’s score is x, then C’s score = x – 10
30
k = 1000 metres. ∴ C’s score when B’s score is x – 15
22. Ans. (c)
( x − 10)
Sol. A : B : C = 200 : (200 – 31) : (200 – 18) x – 22 = × ( x − 15)
= 200 : 169 : 182 x
169 : 182 :: x : 350, if x is the distance i.e., x (x – 22) = x2 – 25x + 150
covered by B. x 2 – 22x – x 2 + 25x = 150
169 × 350
x= = 325 3x = 150
182 x = 50 points.
∴ C will beat A by 350 – 325 = 25 m.
UNIT FOUR

TS
✫ SIMPLE INTEREST ✫ COMPOUND INTEREST
✫ TRUE DISCOUNT AND BANKER’S DISCOUNT
✫ STOCKS, SHARES AND DEBENTURES

H
FUNDAMENTALS AND

G
FAST-TRACK FORMULAE
U
O
Interest years, the basic formulae for calculation are as
Interest is the sum which is paid for the use follows.
H
of other’s money. In other words, interest is P× R×T
(i) S.I. =
the payment made for the money borrowed 100
_T

from banks or moneylenders. The money 100 × S.I.


(ii) P =
borrowed is called principal and the sum of R×T
the principal and interest is called amount. The 100 × S.I.
(iii) R =
C

interest is usually paid yearly, half-yearly or P×T


quarterly as agreed upon. If it is payable 100 × S.I.
(iv) T =
PS

yearly, it is called rate per cent per annum. P×R


For most problems involving interest, one RT
(v) A = P + S.I. = P 1 +
year is taken as 365 days. One month is taken 100
as 30 days. When counting the number of days, PRT
U

(vi) S.I. = A – P =
the day of deposit is not counted, but the day 100
of withdrawal is included. 100 × S.I. 100 × S.I.
@

P = ; R = ;
There are two types of interest: Simple and RT PT
Compound. 100 × S.I.
T =
RT
Simple Interest (vii) Two different cases may be compared
When the interest on a certain sum borrowed A1 − P1 P R T
with the formula = 1 1 1
for a certain period is reckoned uniformly, then A2 − P2 P2 R2 T2
it is called simple interest. Throughout the loan ● What principal will amount to Rs 1200
period, interest is charged on the original sum at 12% per annum in 5 years?
(principal) borrowed. We have, A = Rs 1200, R = 12% per
Basic Formulae for Simple Interest annum, T = 5 years
Where Simple interest = S.I., Principal = P, 100 × A
Principal (P)=
Rate = R% per annum, Amount = A and Time = T 100 + R × T

287
288 ✫ Quantitative Aptitude

● What principal will amount to Rs 708


100 × 1200
= at 9% per annum in 2 years?
100 + 12 × 5
We have, A = Rs 708, R = 9% per
100 × 1200 annum, T = 2 years.
= 100 × A 100 × 708
100 + 60
P = 100 + R × T = 100 + 9 × 2
100 × 1200
= 100 × 708
160 =
118
= Rs 750.

TS
= Rs 600.
● In how many years will a sum of Rs
12000 become Rs 24000 if the rate of If the value of a parameter—principal, rate
or time—changes, then the value of simple
interest is 12%?

H
interest is bound to change. Any or all of the
We have, Amount (A) = Rs 24000 parameters may, by implication, change. In case
of any such change, the generalised formula is:

G
Principal (P) = Rs 12000
S.I. = 24000 – 12000 Change in S.I.
= Rs 12000, R = 12%
Then, using the formula
U =
Product of fixed parameters
100
O
PRT × Difference of the product of variable
S.I. = parameters
100
H
100 × S.I. ● If the simple interest on a certain sum at
∴ T = 2% per annum for 5 years is Rs 50 more
P×R
than the interest on the same sum for 3
_T

100 × 12000 years at 3% per annum, find the sum.


=
12000 × 12
We have change in S.I. = 50, R1 = 2,
100 25
=
C

= R2 = 3, T1 = 5, and T2 = 3
12 3
1 We have to find P which is the fixed
PS

T = 8 years. parameter.
3 2
● Find the amount if P = Rs 2460, R = 6 % P
3 Change in S.I. = (R1 T1 − R2 T2 )
1 100
and T = 3 years.
U

2 P
50 = (2 × 5 − 3 × 3)
20 7 100
RT ×
@

A = P 1+ = 2460 × 1+ 3 2 50 =
P
100 100 100
P = Rs 5000.
20 × 7
= 2460 1+ Fast-Track Formulae and
3 × 2 × 100
Worked Examples
7
= 2460 1+
30 (i) If a certain principal amounts to A
in T years at R % per annum then the
37 principal
= 2460 ×
30 100 × A
P =
= Rs 3034. 100 + R × T
Unit Four : Fundamentals / Simple Interest ✫ 289

(iii) If a sum of money doubles itself in n


● What principal will amount to Rs 1150 at years at simple interest, then the rate of interest
1 is given by
6% per annum in 2 years?
2 100 × (Multiple number of principal − 1)
100 × A R =
P= n
100 + R × T
● A sum of money doubles itself in 12
Here A = Rs 1150, R = 6% per annum years at simple interest. What is the rate
5 of interest?

TS
and T = years
2 100 × (Multiple number of principal − 1)
100 × 1150 R =
⇒ P= n
5 The multiple number of principal = 2,
100 + 6 ×

H
2 n = 12
100 × 1150 100 (2 − 1)
= R =

G
100 + 15 12
100 × 1150 = 8.33%.
=
115
= Rs 1000.
U (iv) The annual payment that will discharge
a debt of Rs A due in T years at the rate of
O
Thus, Rs 1000 will amount to Rs 1150 at interest R% per annum is
1 100 A
H
6% per annum in 2 years. =
2 RT (T − 1)
100 T +
(ii) The time T in which a sum of money 2
_T

becomes x times at S.I. rate of R % is


● Find the annual instalment that will
100 × (x − 1) discharge a debt of Rs 15900 due in 3
T = years.
R
C

years at 6% per annum simple interest.


● In how many years will a sum of money 100 × A
Annual instalment =
PS

treble itself at 10% per annum simple


RT (T − 1)
interest? 100 × T +
2
100 × ( x − 1)
T= years Here A = Rs 15900, T = 3 years,
U

R
Here x = 3 and R = 10% per annum R = 6% per annum
100 × (3 − 1) ⇒ Annual instalment
@

⇒ T=
10 100 × 15900
100 × 2 =
6 × 3 (3 − 1)
= = 20 years. 100 × 3 +
10 2
● A certain sum of money trebles itself in 100 × 15900
=
4 years at simple interest. Find the rate 18 × 2
300 +
per cent per annum. 2
100 × ( x − 1) 100 × 15900
R= =
T 300 + 18
Here x = 3 and T = 4 years
100 × (3 − 1) 100 × 15900
=
⇒ R= 318
4
100 × 2 = Rs 5000.
= = 50%.
4
290 ✫ Quantitative Aptitude

(v) Two equal amounts of money are


416 × 100
deposited at R1% and R2% for T1 and T2 years ⇒ The original sum =
7 8 10
respectively. If the difference between their + +
3 6 2
interest is Id then principal
416 × 100
=
Id × 100 14 + 8 + 30
P =
R1 T1 − R2 T2 6
416 × 100
● The difference between the simple =
interest received from two different 52

TS
sources on Rs 1500 for 3 years is 6
Rs 13.50. Find the difference between 416 × 100 × 6
=
their rates of interest. 52

H
If R1 and R2 are the rates of interest, = Rs 4800.
I d × 100 (vii) If P1 changes to P2 and R, and T are fixed

G
(R1 – R2) = then
sum × t
RT
Here Id = Rs 13.50, sum = Rs 1500 and Change in S.I. = × ( P1 − P2 )
t = 3 years
13.50 × 100 U ●
100
If simple interest on Rs 2500 is more
O
⇒ (R1 – R2) = than the interest on Rs 2000 by Rs 150
1500 × 3 in 5 years, find the rate per cent per
30 annum.
H
= = 0.3%.
100 RT
1 Change in S.I. = (P – P2)
100 1
_T

(vi) Out of a certain sum P, if part is


1 a
invested at R1%, part at R2% and remaining Here Change in S.I. = 150,
1 b P1 = Rs 2500, P2 = Rs 2000, T = 5 years
part at R3%, and the annual income from
R× 5
C

c
all these investments is A, then the original sum ⇒ 150 = (2500 – 2000)
100
PS

is given by
5R
A × 100 150 =
100
× 500
R R R
P = 1
+ 2 + 3 150 = 25R
U

a b c R = 6%.
1
● Out of a certain sum, is invested at
@

3 (viii) If P is fixed, but R1 changes to R2


1 and T1 changes to T2, then
7%, at 8% and the rest at 10%. If the
6
annual income from these is Rs 416, find P
Change in S.I. = ( R1 T1 − R2 T2 )
the original sum. 100
A × 100 ● If the simple interest on a certain sum
The original sum = at 3% per annum for 6 years is Rs 80
R1 R2 R3
+ + more than the interest on the same for
a b c
4 years at 4% per annum, find the sum.
1 1 1 1 1 1 1 1
Here = , = , = 1− + = Change in S.I.=
P
(R T – R2T2)
a 3 b 6 C 3 6 2 100 1 1
R1 = 7%, R2 = 8% and R3 = 10%, Here Change in S.I. = Rs 80,
A = Rs 416 R1 = 3, R2 = 4, T1 = 6, T2 = 4
Unit Four : Fundamentals / Simple Interest ✫ 291

P (b) If P is lent out for a certain time T at


⇒ 80 = (3 × 6 – 4 × 4) S.I. and amounts to A1 at R1% per
100
annum and to A2 at R2% per annum,
P
80 = (18 – 16) A2 R1 − A1 R2
100 P =
R2 − R1
2P
80 = A1 − A2
100 and T = × 100 years
2P = 80 × 100 A2 R1 − A1 R2
● A sum of money is invested for a certain

TS
80 × 100
P = =Rs 4000. time. It amounts to Rs 800 at 5% per
2 annum. But when invested at 4% per
(ix) (a) If a sum of money P lent out at annum, it amounts to Rs 720. Find the
S.I. amounts to A1 in T1 years, and sum and time.

H
to A2 in T2 years,
A2 R1 − A1 R2
A1 T2 − A2 T1 Principal =

G
P = R1 − R2
T2 − T1
Here A1 = Rs 800, A2 = Rs 720,

and R =
A1 − A2
A1 T2 − A2 T1
× 100%
U R1 = 5 and R2 = 4

⇒ Principal =
720 × 5 − 800 × 4
O
● If a sum of money at simple interest 5−4
amounts to Rs 7000 in 3 years and to Rs 3600 − 3200
H
8200 in 4 years, what is the sum and the =
rate of interest? 1
= Rs 400.
_T

A1T2 − A2T1
Principal = A1 − A2
T2 − T1 Time = × 100
A2 R1 − A1 R2
Here A1 = Rs 7000, A2 = Rs 8200, 800 − 720
C

= × 100
T1 = 3 years, T2 = 4 years 720 × 5 − 800 × 4
7000 × 4 − 8200 × 3 80 × 100
PS

⇒ Principal = =
4−3 3600 − 3200
28000 − 24600 8000
= =
U

1 400
= Rs 3400. = 20 years.
@

( A2 − A1 ) × 100 (x) If a debt of Rs M is paid in n number


Rate =
T1 A2 − T2 A 1 of instalments and if the value of each instalment
is Rs a, r = rate of interest per annum, and
(8200 − 7000) × 100 y is the number of instalments per year, then
=
8200 × 3 − 4 × 7000 the borrowed (debt) amount is given by
1200 × 100 ra n (n − 1)
M = na + ×
= 24600 − 28000 100 × y 2
Where,
1200 × 100
= y = 1, when instalments are paid yearly
3400 y = 2, when instalments are paid half-yearly
5 y = 4, when instalments are paid quarterly
= 35 %. y = 12, when instalments are paid monthly.
17
292 ✫ Quantitative Aptitude

● A sum of Rs 220 is lent to be paid back Here, n = 2 and R = 12


in 4 equal half-yearly instalments of Rs
Required Time (T)
40. Find the rate per cent.
(n − 1) × 100
ra n (n − 1) = years
M= na + × R
100 × y 2 (2 − 1) × 100
We have, M = 220, a = 40, n = 4, y = 2 = years
12
r × 40 4 (4 − 1) 100
=8
1
⇒ 220 = 4 × 40 + × = years.
100 × 2 2 12 3

TS
(b) If a certain sum of money becomes
r 4 (3) n times itself in T years at simple
220 = 160 + ×
5 2 interest, then the time T’ in which
6r it will become m times itself is

H
220 = 160 + given by
5 m−1
6r T’ = × T years

G
220 – 160 = n−1
5
6r ● A sum of money put out on simple
= 60 interest triples itself in 12 years. In
5
r =
60 × 5
U how many years would it become
6 times itself?
O
6
r = 50%. If T is the time to be found out,
● What annual instalment will discharge a m −1
H
T’ = × T years
debt of Rs 4200 due in 5 years at 10% n−1
simple interest? Here T = 12, n = 3, m = 6
_T

Here we have to find the yearly 6 −1


⇒ T’ = × 12
instalment (a) 3−1
ra n (n − 1) 5
M = na + × = × 12
C

100 y 2 2
M = Rs 4200, n = 5, y = 1, r = 10%, a = ? = 30 years.
PS

So, (xii) If the rate (R) and the time (T) for
10 × a 5× 4 which a principal (P) has been borrowed at a
4200 = 5a + ×
100 × 1 2 certain simple interest (S.I.) are numerically the
same, then
U

a 20
⇒ 4200 = 5a + × 100 × S.I.
10 2 R or T =
⇒ P
@

4200 = 5a + a = 6a ● Simple interest on a certain sum is 16


4200 over 25 of the sum. Find the rate per
⇒ a = = Rs 700 cent and time if both are equal.
6
The annual instalment will be Rs 700. P× R×T
S.I. =
100
(xi) (a) If a certain sum of money becomes
16 P× R×T
n times itself at R% per annum ⇒ ×P=
simple interest in T years, then 25 100
(as R = T, given)
n−1
T = × 100 years 16 × 100 × P 1600
R ⇒ R2 = = = 64
25 × P 25
● In what time will a sum of money
double itself at a rate of simple R = 64 = 8
interest of 12% per annum? Here R is 8% and time is 8 years.
Unit Four : Fundamentals / Simple Interest ✫ 293

Miscellaneous Worked Examples 205


= 3800 ×
● At what rate will the interest on Rs 4200 be 100
3 = Rs 7,790
of itself in 6 years? How much will it
5
become in 16 years? ● If a sum of money at simple interest amounts
to Rs 6,000 in 2 years and to Rs 6,800 in 3
Here P = 4200, T = 6 years years, what is the sum and the rate of
3 interest?
S.I. = × 4200
5

TS
We have A1 = 6000, A2 = 6800,
= Rs 2520 T1 = 2, T2 = 3
S.I.×100 A1T2 − A2T1
∴ R = Principal =
PT T2 − T1

H
2520 × 100 6000 × 3 − 6800 × 2
= =
3−2

G
4200 × 6
= 18000 – 13600
= 10%
= Rs 4400
In 16 years, if A = P 1 +
RT
100 U Rate =
( A2 − A1 ) × 100
T1 A2 − T2 A1
O
10 × 16
⇒ A = 4200 1+ (6800 − 6000) × 100
100 =
H
2 × 6800 − 3 × 6000
16
= 4200 1+
10 800 × 100
_T

=
26 4400
= 4200 × 800 2
10 = = 18 %
= Rs 10920. 44 11
C

● At what rate will the interest on Rs 3800 be ● A sum of money is invested for a certain
3 time. It amounts to Rs 700 at 4% per annum.
PS

th of itself in 8 years? How much will it


5 But when invested at 3% per annum, it
become in 14 years? amounts to Rs 610. Find the sum and time.
P = 3800; T = 8 years We have A1 = 700, A2 = 610, R1 = 4
U

3 R2 = 3
S.I. = × 3800 = 2280 A2 R1 − A1R2
5
@

S.I. × 100 Principal = R1 − R2


∴ R =
PT 610 × 4 − 700 × 3
2280 × 100 =
= 4−3
3800 × 8 = 2440 – 2100
15 = Rs 340.
= = 7.5% A1 − A2
2
Time = × 100
RT A2R1 − A1R2
In 14 years, A = P 1 +
100
7.5 × 14 700 − 610
= × 100
= 3800 1 + 610 × 4 − 700 × 3
100
90 8
105 = × 100 = 26 years.
= 3800 1 + 340 17
100
294 ✫ Quantitative Aptitude

● A domestic helper borrowed Rs 2,500 from ● If simple interest for 7 years be equal to 35%
two of her employers. For one loan she of the principal, after how many years will
paid 5% per annum and for the other she it be equal to the principal?
paid 7% per annum. The total interest she Let the principal be Rs x and the rate be
paid for two years was Rs 265. How much R% p.a. Then,
did she borrow at each rate?
R 35
Here, S.I. for the total amount = Rs 265 x × × 7 = × x
100 100
Rt , the rate of interest on the total amount = ?
x × R × 7 = 35 × x

TS
100 × S.I . 100 × 265 11 35 × x
Rt = = = % R =
T×P 2 × 2500 2 7×x
Using the Rule of Alligation, R = 5%

H
Let the required time be t years
Sum borrowed at 5%
5
Sum borrowed at 7% x × × t = x

G
100
11 14 − 11 x × 100
7− t =
2 = 2 x×5
=
11
2
−5
11 − 10
2 U ●
t = 20 years.
The difference between the simple interest
O
3 received from two different sources on
3 Rs 1600 for 3 years is Rs 13.50. What is
= 2 =
H
1 1 the difference between their rates of
2 interest?
_T

∴ Sum at 5% per annum


Let the rates of interest be x% p.a. and y%
3 p.a. Then,
× 2500 = Rs 1875
1+ 3
x y 27
1600 × × 3 − 1600 × ×3 =
C

Sum at 7% per annum 100 100 2


1
= × 2500 = Rs 625. 27
PS

1+ 3 48 (x – y) =
2
● If Rs 85 amounts to Rs 95 in 3 years, what
will Rs 102 amount to in 5 years at the same 27 1
x – y = ×
U

interest rate? 2 48
A1 − P1 PR T 9
= 1 1 1 x – y = = 0.28%
@

A2 − P2 P2 R2 T2 32
∴ The difference between the rates of interest
Here A1 = 95, P1 = 85, P2 = 102, T1= 3 years is 0.28%.
and T2 = 5 years. R1 and R2 are same and ● Rs 3000 amounts to be 3600 in 5 years at
A2 has to be found. simple interest. If the interest rate were
So, increased by 3%, it would amount to how
95 − 85 85 × R × 3 much?
=
A2 − 102 102 × R × 5 Sum = Rs 3000, Time = 5 years
S.I. = Amount – Sum
(as R1 and R2 are same)
= Rs (3600 – 3000) = Rs 600
10 × 510 Then, using the formula
A2 – 102 = = 20
85 × 3 100 × S.I.
A2 = 20 + 102 = Rs 122. Rate = Sum × Time
Unit Four : Fundamentals / Compound Interest ✫ 295

100 × 600 annum. The more the number of conversions,


= the greater the value of C.I., other parameters
3000 × 5
remaining unchanged.
= 4% p.a.
Basic Formulae for Compound Interest
Now, sum = Rs 3000, Time = 5 years If compound interest is C.I., Amount is A,
Rate = (3 + 4)% = 7% Principal is P, Rate is R% per annum, and Time
is T years, the basic formulae are as follows.
3000 × 7 × 5
S.I. = Rs T
100 R
(i) C.I. = A – P = P −1 . 1+

TS
= Rs 1050. 100
When interest is compounded annually:
T
Compound Interest R
(ii) A = P 1 +

H
.
The interest charged every year on the amount 100
of the previous year is called compound 1

G
interest. Money is said to be lent at compound A T
(iii) R = − 1 % per annum.
interest when at the end of a fixed period, the P
interest that has become due and is not paid
to the lender, but is added to the sum lent,
and the amount thus obtained becomes the U ● Find the compound interest on Rs 3000
for 3 years at 20% per annum.
O
principal for the next period. The process is R
T

repeated until the amount for the last period C.I. = P 1+


−1
H
has been found. The difference between the 100
original principal and the final amount is called Here P = Rs 3000, R = 20%, T = 3 years
_T

compound interest. 3
20
In simple interest borrowings, the principal ⇒ C.I. = 3000 1+ −1
remains constant through the period for which 100
the sum is borrowed. In borrowings at 3
C

120
compound interest, the principal adds on the = 3000 −1
simple interest at the end of a year and 100
PS

becomes a new principal for the next year. The = 3000 [(1.2)3 – 1]
amount of interest accrued will also become = 3000 [1.728 – 1]
different every year.
As the principal increases after every = 3000 × 0.728
U

reckoning period, the amount of interest in = Rs 2184.


Compound Interest will always be more than ● A sum becomes Rs 2916 in 2 years at 8%
@

Simple Interest. per annum compound interest. What is


The period at the end of which interest is the sum?
compounded is called conversion period. In T
case of interest being compounded yearly, the R
A = P 1+
number of conversions x is 1. Interest may also 100
be compounded and added to the principal Here A = Rs 2916, T = 2 years, R = 8%
half-yearly, i.e., twice in a year. In this case 2
number of conversions per year is 2. If interest 8
⇒ 2916 = P 1 +
is compounded every three months, or 100
quarterly, the number of conversions in a year 108
2

is 4. If the interest is compounded every month, 2916 = P


100
conversions number 12.
2
Unless the problem specifies, it is assumed 54
that interest is compounded yearly or per 2916 = P
50
296 ✫ Quantitative Aptitude

When the interest is compounded n times


50 × 50
P = 2916 × a year, then
54 × 54
P = 50 × 50 n×T
R
= Rs 2500. (iv) A = P 1 +
100 × n
● In what time will Rs 1064.80 amount to
1
Rs 800 at 10% per annum compounded A nT
annually? (v) R = n × 100 −1
T
P
R

TS
A = P 1+
100 n×T
R
(vi) C.I. = P 1+ −1
Here A = Rs 1064.80, P = Rs 800, 100 × n
R = 10%

H
T ● What will be the compound interest on
10
⇒ 1064.80 = 800 1 + Rs 1200 for 2 years at 10% per annum
100 interest being charged half-yearly?

G
T
1064.80 110 n ×T
⇒ = R



800
1.331
(1.1)3
100
= (1.1)T
= (1.1)T
U A = 1+ 2
100
O
⇒ T = 3 years. Here P = Rs 1200, T = 2 years, R = 10%
2×2
H
● At what rate of interest will Rs 20000 10
become Rs 24200 after 2 years when
⇒ A = 1200 1+ 2
_T

interest is compounded annually? 100


T
R
A = P 1+ 10
4
100 = 1200 1+
2 × 100
C

Here A = Rs 24200, P = Rs 20000, 4


T = 2 years 210
= 1200
PS

R
2 200
⇒ 24200 = 20000 1 +
100 21 21 21 21
2
= 1200 × × × ×
24200 R 20 20 20 20
U

⇒ = 1+
20000 100 = Rs 1458.60
∴C.I. = A – P
@

2
R
⇒ 1.21 = 1 + = 1458.60 – 1200
100
= Rs 258.60.
R
⇒ 1+ = 1.21
100
Fast-Track Formulae with
R
⇒ 1+ = 1.1 Worked Examples
100
R (i) When the rate of interests are
⇒ = 1.1 – 1
100 different for different years, then
R R1 R2 R3
⇒ = 0.1 A = P 1+ 1+ 1+ . . . . upto
100 100 100 100
⇒ R = 0.1 × 100
⇒ R = 10%. T times
Unit Four : Fundamentals / Compound Interest ✫ 297

● Manoj invests Rs 12,000 in a bond which


106 106 102
gives interest at 6% per annum during = 6000 × × ×
the first year, 8% during the second 100 100 100
year and 12% during the third year. 53 53 51
How much does he get at the end of the = 6000 × × ×
50 50 50
third year? = Rs 6876.43.
Here P = Rs 12000, R1 = 6, ● What will be the compound interest on
R2 = 8 and R3 = 12 3
Rs 5,000 for 2 years at 8% per annum?
4

TS
Amount (A) 3
We have P = 5000, R = 8% and T = 2
R1 R2 R3 4
= P 1+ 1+ 1+ 3
100 100 100 2 R
R
1+ 4

H
A = P 1+
6 8 12 100 100
= 12000 1 + 1+ 1+
100 100 100

G
3
106 108 112 8
2 ×8
= 12000 = 5000 1 + 1+ 4
100
53
×
54
×
100
56
100

U 100 100
O
= 12000 × = 108 108 6
50 50 50 5000 × × × 1+
= Rs 15386.11. 100 100 100
H
= Rs 15386 approximately. 108 108 106
= 5000 × × ×
100 100 100
(ii) When the rate of interest is compounded
_T

1 = 5000 × 27 × 27 × 53
annually and the time is in fraction, say 4 25 25 50
2
years then, = Rs 6181.92.
C

1 (iii) If a sum of money becomes x times in T


R
4 R years at compound interest and amounts to X,
A = P 1+ 1+ 2
PS

100 100 then the same amount of money becomes xy times


in T × y years.
● A sum of money becomes 4 times in 18
● What will be the compound interest on
U

1 years at compound interest. In how


Rs 6000 for 2 years at 6% per annum? many years will the same amount of
3 1 money become 16 times itself?
@

We have, P = Rs 6000, R = 6%, T = 2


3 Here x = 4, T = 18, X = 16
years X = xy
16 = 4y
1 (4)2 = 4y
2 R y =2
R
A = P 1+ 1+ 3 ∴ The given sum will become 16 times
100 100
itself in T × y years,
1 i.e. 18 × 2 = 36 years.
2 ×6
6 (iv) If a certain sum becomes x times in t
= 6000 1 + 1+ 3
100 100 years, the rate of compound interest is given by
1
106
2
2 R = 100 ( x ) t − 1
= 6000 1+
100 100
298 ✫ Quantitative Aptitude

● At what rate per cent compound interest


1
does a sum of money become 8 times in y q− p
3 years? R = −1 × 100%
x
We have, x = 8, T = 3
● A sum of money at C.I. amounts to
1
Rs 1700 in 3 years and to Rs 3332 in 5
Rate (R) = 100 ( x ) − 1 T
years. Find the rate of interest per
1 annum.
= 100 (8) 3 − 1 1

TS
y q− p

1
R= − 1 × 100%
x
= 100 (2 ) − 1
3 3

Here x = Rs 1700, y = Rs 3332,

H
= 100 [2 – 1] p = 3 years, q = 5 years
= 100%. 1

G
3332 5− 3
∴R = − 1 × 100%
(v) Present worth of Rs x due t years hence 1700
at R% per annum compounded yearly is given
by
A U =
1
(1.96) 2 − 1 × 100%
O
t
R
1+ 1
100
= (1.4 )
2 2
− 1 × 100%
H
● Find the present worth of Rs 17576 due
in 3 years hence at 4% per annum = [1.4 – 1] × 100%
_T

compounded yearly. = 0.4 × 100%


A = 40%.
Present worth= T
R (vii) If a loan of Rs A at R% compound
1+
C

100 interest per annum is to be repaid in n equal


yearly instalments, and each instalment is T,
Here A = Rs 17576, R = 4%, T = 3 years
PS

then
17576 A
⇒ Present worth = 3 I =
4 100 100
2
100
n
1+ + +. . . +
U

100 100 + R 100 + R 100 + R


17576 ● A sum of Rs 5300 was taken as a loan.
=
@

3
104 This is to be repaid in two equal
100 instalments. If the rate of interest be
12% compounded annually, find the
17576 value of each instalment.
= 3
26
We have, A = Rs 5300, R = 12%
25 Each instalment
25 × 25 × 25 A
= 17576 × =
26 × 26 × 26 100 100
2
100
n

+ +. . . +
= Rs 15625. 100 + R 100 + R 100 + R

(vi) When a certain sum of money at C.I. 5300


= 2
amounts to Rs x in p years and Rs y in q years, 100 100
+
then the rate of interest per annum is 100 + 12 100 + 12
Unit Four : Fundamentals / Compound Interest ✫ 299

2
5
5300 ⇒ 6 = P
= 2 100
100 100
+
112 112 25
⇒ 6 = P
5300 10000
=
100 100 6 × 10000
1+
112 112 ⇒ P =
25
5300
= ⇒ P = Rs 2400.

TS
100 212
112 112 Case II: When T = 3 years and n = 1 per year
5300 × 112 × 112
= R
3
R
2
100 × 212

H
C.I. – S.I. = P +3
= Rs 3136. 100 100

G
2
Difference between C.I. and S.I. S.I. R R
= +3
PRT 3 100 100
S.I. =
100
,
Tn U ● The difference between compound
interest and simple interest on a sum of
O
R money for 3 years at 5% per annum is
C.I. = P 1+ −1
100 × n Rs 60. Find the sum.
H
C.I. > S.I. Here C.I. – S.I. = Rs 60, R = 5
n×T > 1 3 2
R R
_T

C.I. – S.I. = P +3
Tn 100 100
R RT
C.I. – S.I. = P 1+ − −1
100 × n 100 5
3
5
2

⇒ 60 = P +3
C

100 100
3 2
PS

1 1
Fast-Track Method ⇒ 60 = P +3
20 20
Case I: When T = 2 years and n = 1 per year 1 3
⇒ 60 = P +
U

C.I. – S.I. 8000 400


2
R 1 + 60
= P ⇒
@

(in terms of P and R) 60 = P


100 8000
R × S.I. 61
= (in terms of SI and R) ⇒ 60 = P
2 × 100 8000
● If the difference between simple interest 60 × 8000
and compound interest for 2 years on P =
61
a sum of money lent at 5% is Rs 6, find
⇒ P = Rs 7868.85.
the sum.
Here difference between S.I. and C.I.
= Rs 6, r = 5% and T = 2 years Miscellaneous Worked Examples
Then, using the formula,
Difference between S.I. and C.I. ● A sum of Rs 12,000 deposited at compound
r
T interest becomes double after 5 years.
= P What will it be after 20 years?
100
300 ✫ Quantitative Aptitude

Here A = 2P and T = 5 years ● What will be the compound interest on Rs


R
T 800 for 2 years at 10% p.a., interest being
A = P charged (a) annually, (b) half yearly?
100
R
5 (a) Here n = 1
⇒ 2P = P 1+ n×T
100 R
A = P 1+
2P R
5
n × 100
⇒ = 1+ 2
P 100 10
= 800 1 +

TS
5
R 100
⇒ 2 = 1+
100 2
110
= 800 ×
1 R 100
= 1+

H
25 100 = 800 × (1.1)2
After 20 years, = 800 × 1.21 = 968

G
1 R
20 ∴ C.I., reckoned annually = 968 – 800
× 20 = 1+ = Rs 168.
25 100

⇒ 24 = 1+
R
20
U (b) Here n = 2
10
2×2
O
100 A = 800 × 1 +
2 × 100
20
R
1+
H
⇒ 16 = 21 21 21 21
100 = 800 × × × ×
20 20 20 20
Hence, the principal will become 16 times,
_T

= 972.405
i.e., Rs (16 × 12000)
= Rs 192000 after 20 years. ∴ C.I. = 972.405 – 800
= Rs 172.405
● Two friends A and B jointly lent out = Rs 172.41
C

Rs 81,600 at 4% per annum compound (correct to two decimal places).


interest. After 2 years A gets the same
PS

amount as B gets after 3 years. Find the ● Sarika invests Rs 10,000 in a bond which
investment made by B. gives interest at 5% per annum during the
first year, 6% during the second year and
Here PB = Rs x, PA = Rs (81600 – x),
10% during the third year. How much does
U

r = 4%, TA = 2 years, TB = 3 years she get at the end of the third year.
TB TA
R R We have P = 10,000, R1 = 5,
@

PB 1 + = PA 1 + R2 = 6, R3 = 10
100 100
2 R1 R2 R3
Amount = P 1 + 1+ 1+
3
4 4 . .. .
⇒ x 1+ = (81600 – x) 1 + 100 100 100
100 100
3 2
104 104 5 6 10
⇒ x = (81600 – x) = 10000 1 + 1+ 1+
100 100 100 100 100

104 105 106 110


⇒ x = (81600 – x) = 10000 × × ×
100 100 100 100
⇒ 104x = 8160000 – 100x 21 53 11
= 10000 × × ×
⇒ 204x = 8160000 20 50 10
⇒ x = Rs 40000. = Rs 12,243.
Unit Four : Fundamentals / Compound Interest ✫ 301

● A sum of money at C.I. amounts to Rs 1,500 T


R
in 2 years and to Rs 2,535 in 4 years. Find Population after T years = P 1 −
100
the rate of interest per annum.
(where P is the initial population).
Here x = 1500, y = 2535,
p = 2, q = 4 ● The bacteria in a culture grows by 5%
1
in the first two hours, decreases by 5%
2535 4 −2 in the next one hour and then increases
∴ R = −1 × 100%
1500 by 5% in the next two hours. The
original number of bacteria in the sample

TS
1
= (1.69) 2 − 1 × 100% was 30,000. What is the count of bacteria
after five hours?
= [1.3 – 1] × 100% Population after (T1 + T2 + T3) years

H
= 0.3 × 100% T1 T2 T3
R1 R2 R3
= 30%. = P 1+ × 1+ × 1+

G
100 100 100
Growth and Depreciation
Here, P = 30000, R1 = 5
Using C.I. Formulae to Estimate Growth
The formula for calculating population on the
basis of the rate of growth has been discussed U R1 = 5, R2 = –5
(decrease, so – sign),
O
R3 = 5
in the section on Percentage in Unit Two. The
formulae are the same as those for finding A T1 = 2, T2 = 1, T3 = 2
Bacteria count after 5 hours
H
and C.I. after T years, given R, in a problem
of compound interest. The formulae can also 2 1 2
5 5 5
be used to find the growth of a tree. = 30000 1 + × 1− × 1+
_T

100 100 100


(i) To reinforce memory:
2 1 2
(a) Population after T years 1 1 1
T = 30000 1+ × 1− × 1+
R 20 20 20
= P 1+
C

100
21 21 19 21 21
(where P is the initial population = 30000 × × × × ×
PS

20 20 20 20 20
and R is the constant rate of growth
in per cent per annum and T is the = 34642.
time) Bacteria count after 5 hours is 34642.
U

(b) Net increase in population in T years ● A tree was planted three years ago. Its
T rate of growth is 30% per annum. At
R present the tree is 670 cm tall. How tall
@

= P 1+ −1
100 was the tree when it was planted?
(ii) The rate of growth varying from year We take the height as equivalent to
to year – R1% per annum in T1 years population.
and R2% per annum in T2 years, then Here, the height of the tree now (670
Population after (T1 + T2) years cm) is its height after 3 years; so T = 3.
R is given as 30%
T T
R 1 R 2 Let H be the height when it was planted.
= P 1+ × 1+ Height of the tree after T years
100 100 T
R
where P is the initial population. = H 1+
100
(iii) In the matter of population the rate 3
30
could be one of a decrease too. If the 670 = H 1 +
rate of decrease is R% per annum, then 100
302 ✫ Quantitative Aptitude

3 True Discount
13
670 = H
10 True discount is the difference between the
670 × 1000 amount or sum due at the end of a given time
= H and its present worth.
2197
= 305 cm (approximate) The money which amounts to the sum due
The original height of the tree was 305 cm. or amount in the given time at the already
agreed upon rate of interest is the present
Depreciation Continuous use of an article,
worth (P.W.) of the amount.
especially a machine, causes wear and tear; as

TS
a consequence its value decreases over time. True discount = Amount – Present Worth
The relative decrease in the value of the article In effect, true discount is the simple interest
over a period of time is known as depreciation.
calculated on the present worth of the amount,
Rate of depreciation is the depreciation per
though discount is seen as a discount on

H
unit time.
Depreciation value is the value of a given amount, i.e.,
article at any time. Present worth = Amount – True Discount

G
If V is the value of a given article at a
certain time— Assume that, I have borrowed some money,
(i) At constant rate of depreciation R% per
year,
R
T U say Rs 1,000, from X and I have to give this
money back after 5 years. The given time has
been agreed upon by X and myself. The rate
O
Value after T years = V 1 − of interest agreed upon is 10% per annum on
100
(As the value decreases, a (–) sign is S.I. basis. So after 5 years, at a given rate of
H
used.) 10% p.a., the amount to be paid back to X (i.e.
the sum due after 5 years) amounts to Rs 1,500.
(ii) At variable rates of depreciation, say R1
_T

But I would like to clear off my debt now,


in T1 years, R2 in T2 years, and so on,
after paying some money. Clearly, I do not
Value at the end of (T1 + T2 + … + Tn) years
want to pay the entire amount in advance
C

T1 T2 Tn either. So, the present worth or value of the


= V 1 − R1 × 1−
R2
× .... × 1 −
Rn
sum which amounts to Rs 1,500 in 5 years at
100 100 100
10% p.a. is calculated.
PS

● A truck cost Rs 6,30,000. If this value Amount (A)


depreciates at the rate of 20% per annum, = Principal (P) + Simple Interest (S.I.)
what will be its value after 3 years?
U

What will be the total depreciation? PRT


= P+
100
Value of the article after T years
@

T P (100 + RT )
R A=
= V 1− 100
100 A × 100
Value of truck after 3 years ∴ P =
3 100 + RT
20
= 630000 1− Here, we have A = 1500, R = 10 and T = 5
100
3 1500 × 100
4 ∴ P = = Rs 1000
= 630000 × 100 + 10 × 5
5
64 The present worth or present value is Rs
= 630000 ×
125 1,000. If I pay Rs 1,000 now, I would be able
= Rs 322560 to clear off my debt. So, in this example,
Total deprecation = Rs 630000 – Rs 322560
= Rs 307440. Present Worth = Rs 1000,
Unit Four : Fundamentals / True Discount ✫ 303

Amount or sum due = Rs 1500, R% = 10% per annum compound interest. Calculate
and T = 5 years. the true discount also.
True discount = Amount – Present Worth Amount
= 1500 – 1000 P.W. = T
= Rs 500 R
1+
100
Formulae to Remember for True Discount
5512.50 5512.50
Let Amount = A, Present Worth = P.W., = =
5
2 1.05 × 1.05
True discount = T.D., Rate = R, Time = T, 1+
100

TS
Simple interest = S.I.
(i) A = P.W. + T.D. 5512.50
= = Rs 5000.00
A× R×T A× R×T A 1.1025
(ii) T.D. = = =

H
100 100 + RT 1+
100 T.D. = A – P.W.
RT = 5512.50 – 5000 = Rs 512.50.

G
● The true discount on bill due 6 years ● Find the present worth of Rs 9280 due 2
hence at 9% per annum simple interest years hence at 8% p.a. on simple interest.
is Rs 810. Find the amount of the bill and
its present worth.
P.W. × RT U Find the true discount also.

A × 100
O
T.D. = P.W. =
100 100 + RT
T.D. × 100 9280 × 100
H
P.W. = =
RT 100 + 2 × 8
810 × 100
_T

= = 1500 9280 × 100


9×6 =
116
∴ Amount = 1500 + 810 = Rs 2310. ∴ P.W. = Rs 8000
C

● Find the true discount reckoning 4% per T.D. = 9280 – 8000 = Rs 1280.
annum simple interest on Rs 2240 due in ART
PS

3 years’ time. (iv) S.I. =


100
We have, A = Rs 2240, R = 4%, T = 3 years
● Find the simple interest (S.I.) on Rs 4200
A× R×T
U

at 4% per annum for 3 years.


True discount (T.D.) =
100 + RT We have, A = Rs 4200, R = 4%,
@

2240 × 4 × 3 T = 3 years
= Then,
100 + 4 × 3
2240 × 4 × 3 ART
S.I. =
= 100
100 + 12
4200 × 4 × 3
2240 × 12 =
= 100
112 = Rs 504.
= 20 × 12 = Rs 240. S.I. × T.D.
A A × 100 (v) P =
(iii) P.W. = S.I. – T.D.
RT = 100 + RT
1+ ● The true discount on a certain sum of
100
money due 2 years hence is Rs 80 and
● Find the present worth of a bill of
the simple interest on the same sum is
Rs 5,512.50 due in 2 years hence at 5%
Rs 85. Find the sum.
304 ✫ Quantitative Aptitude

Here, S.I. = Rs 85, T.D. = Rs 80. annum compound interest. Calculate the
We have to find P. true discount also.
S.I. × T.D. We have, Amount or A = Rs 5832, R = 8%
P =
S.I. – T.D. and T = 2 years
Amount
85 × 80 P.W. =
= R
T
85 − 80 1+
85 × 80 100
=
5832

TS
5 =
= Rs 1360. 8
2

1+
T.D. × R × T A × (RT )2 100
(vi) S.I. – T.D. = =
100 (100 + RT )

H
100 5832
= 2
● The difference between the simple 108

G
interest and the true discount on a 100
certain sum of money for 4 years at 8%
per annum is Rs 243.20 Find the sum.
= 58322
We have, S.I. – T.D. = Rs 243.20,
R = 8% and T = 4 years U 27
O
25
Let T.D. be x.
5832 × 25 × 25
Then, =
H
27 × 27
T.D. × RT
S.I. – T.D. = P.W. = Rs 5000.
100
_T

x ×8×4 ∴ T.D. = A – P.W.


⇒ 243.20 =
100 = 5832 – 5000
⇒ x × 8 × 4 = 243.20 × 100 = Rs 832.
C

243.20 × 100 P.W .1


⇒ x = 100 × RT2
8× 4 (viii) =
PS

P.W .2 100 + RT1


= Rs 760
∴ T.D. = 760 ● The present worth of a bill due 4 years
A× R×T hence is Rs 487.50, and the present
U

T.D. = worth of a bill due 6 years hence is Rs


100 + R × T
450. Find the rate % and sum of the bill.
T.D. × (100 + R × T )
@

∴ A = 100 + RT2
R×T P.W.1
760 × (100 + 8 × 4) = 100 + RT
P.W.2 1
=
8×4
760 × 132 487.50 100 + R × 6
= ⇒ =
32 450 100 + R × 4
A = Rs 3135.
100 + 6 R
(vii) Where the sum is put at compound ⇒ =
interest, 100 + 4 R
A 487.50 (100 + 4R) = 450 (100 + 6R)
P.W. = T
R 48750 + 487.50 × 4R= 45000 + 450 × 6R
1+
100 48750 – 45000 = 2700R – 1950R
● Find the present worth of a bill of
Rs 5832 due in 2 years hence at 8% per 3750 = 750R
Unit Four : Fundamentals / True Discount ✫ 305

3750 1
certain time at 3 % p.a. is Rs 175. Find the
∴ R = = 5% 2
750 time after which it is due.
A × 100
P.W. = P.W. = A – T.D. = 925 – 175 = 750
100 + RT
P.W. × R × T
P.W. (100 + RT ) If T.D. =
∴ A = 100
100 T.D. × 100
Then T =
450 (100 + 5 × 6) P.W. × R

TS
= 175 × 100 × 2
100 =
750 × 7
450 × 130
= 100
100 =

H
15
= Rs 585
20 2
∴ Amount = Rs 585 and Rate % = 5%. = 6 years.

G
=
3 3
T.D.1 P.W.1 × R × T1
(ix) T.D. = P.W. × R × T
2 2 2
(x) Where, A is the amount due after T U ● A shopkeeper purchased a mobile phone
handset for Rs 3000, and sold it for Rs 3960,
on credit for some time, thereby gaining
O
years hence 10%. If the rate of interest is 5%, find for
P is the money of each how long was the credit allowed?
H
instalment
C.P. = 3000; A = (S.P.) = 3960
R is the rate per cent per annum R = 5%; T = ?
_T

on simple interest. C.P. + Gain = Present Worth of the amount


n is the number of equal Gain % × C.P.
instalments to be paid to clear P.W. = 3000 +
100
C

the debt.
10 × 3000
k is the number of instalments = 3000 +
PS

in a year. 100
= 3300
P × R n (n − 1)
A = n×P + × A × 100
100 2k P.W. =
U

(xi) Where, n is the number of equal 100 + RT


instalments 3960 × 100
@

3300 =
T 1 is the time period for the first 100 + 5T
instalment. 3960 × 100
T 2 is the time period for the 100 + 5T =
3300
second instalment. = 120
100 × A × 1 1 5T = 20
P.W. = +
n (100 + RT1 ) (100 + RT2 ) T = 4 years.
1
+.... + ● The true discount on Rs 1125 due after a
100 + RTn
1
certain time at 5 per annum is
2
Miscellaneous Worked Examples Rs 275. Find the time after which it is
due.
● The true discount on Rs 925 due after a
306 ✫ Quantitative Aptitude

We have, A = Rs 1125, T.D. = Rs 275, (paid by the banker/broker) and the legally
due date. (If the date of the bill is not given,
1 11
R = 5 %= % the days of grace are not added.) This simple
2 2 interest amount charged by the banker is
P.W. = A – T.D. known as the banker’s discount.
= 1125 – 275 Banker’s discount is slightly more than the
= Rs 850 True Discount. The difference between the
P.W. × R × T banker’s discount (B.D.) and the true discount
T.D. = (T.D.) is called the banker’s gain (B.G.)
100

TS
T.D. × 100 Formulae to Remember for
∴ T = Banker’s Discount
P.W. × R
(i) Banker’s discount = S.I. on bill for its
275 × 100 unexpired time

H
=
11 Bill amount (A) × Rate (R ) × Unexpired time (T )
850 ×
2 =

G
100
T.D. (100 + RT )
275 × 100 × 2 =
= 100

=
850 × 11
100
= 5
15
years. U =
B.D. (100 + RT )
RT
=
A × B.G.
T.D.
=
A × T.D.
A − T.D.
O
17 17
● A bill is drawn for Rs 5100 on June 14,
2012 at 5 months’ credit. It is discounted
H
Banker’s Discount on September 5 at 10% per annum. Find
the banker’s discount.
_T

Let’s suppose a person A buys goods worth


a sum of Rs 10,000 from a merchant B on credit We have, Amount or A = Rs 5100
for a period of 4 months. B prepares a bill Date of drawing = June 14, 2012
called the bill of exchange. This bill is an (for 5 months)
C

agreement duly signed by the debtor stating Date of maturing = November 17, 2012
that he has accepted the bill and the money (including 3 days’ grace)
PS

can be withdrawn from his bank by the Date of discounting = September 5, 2012
creditor on the due date. Number of days from September 5 to
The sum due or the amount of the bill is November 17 =
called the face value of the bill. Sept. Oct. Nov.
U

The due date—the date on which the face 1


25 + 31 + 17 = 73 days = year
value is due from the debtor—is known as the 5
Then, using the formula
@

nominally due date. Usually a grace period of


Banker’s discount
3 days or so is given; the date so arrived at
is called the legally due date. Amount × Rate × Time
=
The debtor (the borrower) is bound by 100
law, through the Bill of Exchange, to pay the
5100 × 10 × 1
money at the mutually agreed upon terms, to ⇒ B.D. =
the creditor at the latest by the legally due 100 × 5
date. = 51 × 2
If the creditor, B, wants his money before = Rs 102.
the legally due date, he can encash the bill of (ii) T.D.= Bill Amount (A)
exchange from a bank or a broker. The bank – Present Worth (P.W.)
(or broker) will charge simple interest on the
face value for the unexpired time, i.e., the P.W. × R × T B.G. × 100
= S.I. on P.W.= =
period from the date the bill was ‘discounted’ 100 RT
Unit Four : Fundamentals / Banker’s Discount ✫ 307

(iii) Banker’s Gain (B.G.)


ART A × B.D.
= = = Banker’s Discount (B.D.)
100 + RT A + B.D. –True discount (T.D.)
= P.W. × B.G. T.D. × R × T
= S.I. on T.D. =
= ( A − T.D.)(B.D. − T.D.) 100
2
RT
● The banker’s gain of a bill due 1 year = Present Worth
100
hence at 10% per annum is Rs 520. Find
the true discount. (T.D.)2 A ( RT )2

TS
= =
P.W. 100 (100 + RT )
We have, B.G. = Rs 520, R = 10%,
T = 1 year ● The present worth of a certain sum due
Then, using the formula

H
sometime hence is Rs 2400 and the true
B.G. × 100 discount is Rs 120. Find the Banker’s
T.D. = gain.
R×T

G
520 × 100 We have, T.D. = 120, P.W. = 2400
⇒ T.D. = = Rs 5200. Then, using the formula


10 × 1
The banker’s discount on a bill due 4 U B.G. =
(T.D.)2
P.W.
O
months hence at 12% per annum is Rs
624. Find the true discount. (120)2
=
H
2400
We have, B.D. = Rs 624, R = 12%,
1 120 × 120
T = 4 months = years =
_T

3 2400
Then, using the formula
= Rs 6.
B.D. × 100
T.D. = ● If the true discount on a certain sum due
100 + (R × T )
C

4 months hence at 12% is Rs 400, what


is the banker’s discount on the same
624 × 100
PS

⇒ T.D. = sum for the same time and at the same


1 rate?
100 + 12 ×
3
We have, T.D. = Rs 400, R = 12%,
624 × 100 1
U

= T = 4 months = years
100 + 4 3
Then, using the formula
624 × 100
@

= T.D. × R × T
104 B.G. = S.I. on T.D.=
= Rs 600. 100
1
400 × 12 ×
● The present worth of a sum due sometime 3
⇒ B.G. =
hence is Rs 729 and the banker’s gain is 100
Rs 25. Find the true discount. 1600
=
We have, P.W. = 729, B.G. = 25 100
Then, using the formula = Rs 16
T.D. = P.W. × B.G. ∴ B.D. – T.D. = B.G.
B.D. – T.D. = Rs 16
⇒ T.D. = 729 × 25 B.D. – 400 = 16
= 27 × 5 B.D. = 400 + 16
= Rs 135. B.D. = Rs 416.
308 ✫ Quantitative Aptitude

● The banker’s gain on a certain sum of 95 × 100


money for 5 years at 12% is Rs 324. Find ⇒ R =
4
the present worth and sum due. 1710 ×
12
We have, B.G. = Rs 324, R = 12%,
95 × 100 × 12
T = 5 years =
Then, using the formula 1710 × 4
= 16.66%.
T.D. × R × T
B.G. =
100 A × 100 (T.D.)2
(v) Present Worth (P.W.)= =

TS
100 + RT B.G.
B.G. × 100
⇒ T.D. =
RT ● Find the present worth of Rs 8848 due
3 years hence at 4% per annum on
324 × 100

H
= simple interest.
12 × 5
= Rs 540 We have, A = Rs 8848, R = 4%,

G
T = 3 years
(T.D.)2
⇒ P.W. = A × 100
B.G.
540 × 540 U P.W. =
100 + RT
8848 × 100
O
=
324 ⇒ P.W. =
100 + 4 × 3
= Rs 900
H
∴ Sum due = P.W. + T.D. 8848 × 100
=
= 900 + 540 112
_T

= Rs 7900.
= Rs 1440.
B.D. × T.D. B.D. × T.D.
(iv) Amount = = Miscellaneous Worked Examples
C

B.D. – T.D. B.G.


● Calculate the B.D. on a bill due 6 months
● The banker’s discount and true discount
PS

hence at 12% per annum if the present


on a certain sum of money due 4 months worth of the bill is Rs 4200.
hence are Rs 95 and 90 respectively. Find
We have, P.W. = Rs 4200, R = 12%,
the sum due and the rate per cent. 1
U

T = 6 months = year
We have, B.D. = Rs 95, T.D. = Rs 90, 2
Then, using the formula
T = 4 months
@

A × 100
B.D. × T.D. P.W. =
Amount = 100 + R × T
B.D. – T.D.
P.W. (100 + R × T )
95 × 90 ∴ A =
= 100
95 − 90
1
95 × 90 4200 (100 + × 12)
= = Rs 1710 = 2
5 100
ART
B.D. = 4200 (100 + 6)
100 =
100
B.D. × 100 = 42 × 106
R =
A×T A = Rs 4452
Unit Four : Fundamentals / Stocks, Shares and Debentures ✫ 309

A× R×T 20
∴ B.D. =
100
= 23
4452 × 12 × 1 3
=
100 × 2 23
B.D. = Rs 267.12.
20 23
= ×
● The true discount on a certain bill due nine 23 3
months hence at 4% simple interest is 20
Rs 150. Find the amount of the bill. = Rs

TS
3
We have, 20
True discount (T.D.) = Rs 150,  S.I. on Rs for 5 years is Re 1
3
9 3 100 × 3
=

H
R = 4%, T = years
12 4 ∴ Rate =
Then, using the formula 20 × 5

G
100 = 3%.
Present Worth (P.W.) = T.D. ×
R×T
Stocks, Shares and Debentures
=
150 × 100

3 U When a group of individuals come together to
start a business venture, involving a large
O
4 amount of money, a company or a corporation
150 × 100 is said to be formed.
H
= The capital of the company is called the
3
= Rs 5000 stock. Stock is a unit of ownership of a
_T

company. It consists of a group of shares. The


∴ Amount of the bill = Rs 5000 + Rs 150
total investments required by the company or
= Rs 5150. the corporation is known as capital stock.
● Banker’s gain on a certain sum due 5 years Shares can be explained as “a fixed and
C

3 indivisible section of the capital of a company”.


hence is of Banker’s discount. What is Shares are generally worth Rs 10 and Rs 100
23
PS

the rate per cent? each. Money derived from the sale of shares
in a business and used for carrying it on is
We have, 3 termed as share capital. Company issues a
Banker’s gain (B.G.) = Rs , T = 5 years share certificate to the person (shareholder)
U

23 who owns a share or shares to certify the


Let Banker’s discount (B.D.) = Re 1 ownership of its shares.
@

3 The government, at times, raises money


⇒ True Discount (T.D.) = Rs 1 − from the public at a certain fixed rate of
23
interest, to meet the plan expenditure. Bonds
23 − 3 or promissory notes each having a fixed value
= Rs
23 are then issued to the public for sale.
20 If a bond is purchased by a person for Rs
= Rs . 100 at which, say, 7% interest is fixed by the
23
B.D. × T.D. government, we say that the person (holder of
∴ Sum = the bond) has a Rs 100 stock at 7%. Here Rs
B.D. – T.D. 100 is the face value of the stock. The holder
20 20 of the bond gets an annual interest of 7% of the
1× face value, i.e., if the face value of the stock is
= 23 = 23
20 23 − 20 Rs 100, then the annual interest is Rs 7.
1− Though a fixed period is set for the bonds
23 23
310 ✫ Quantitative Aptitude

to mature, stocks are bought and sold in the Ordinary or Equity shares: After paying the
open market through brokers at stock exchanges. preference shareholders, the dividends are paid
The broker charges a commission, known as to the holders of equity shares. These dividends
brokerage. Brokerage is calculated on the vary depending upon the profits of the company.
market value of shares or debentures, and Debenture is the written acknowledgement
brokers charge commission from purchasers as of a debt. Debenture is a security issued by
well as sellers. a company for money borrowed on the
When stock is purchased, brokerage is company’s property, having a fixed rate of
added to the cost price. When stock is sold, interest to the shareholders or public for a
brokerage is subtracted from the selling price.

TS
fixed period. A debenture-holder gets interest
In other words, a purchaser of shares pays calculated on the face value of the debenture,
market value + brokerage, while a seller of at a fixed rate of interest of the company and
shares will get market value – brokerage. So hence the interest does not vary.

H
every time a share is purchased through a Per cent rate of interest is the rate % of
broker, brokerage has to be paid. Buying interest received per annum per Rs 100 stock.
shares directly from the company does not

G
But if the stock is at a premium or at a discount
involve brokerage. on the total investment, actual rate % obtained
1 1 will change accordingly.
Brokerage
5
means Rs
5
for Rs 100 stock.
The selling price of a Rs 100 stock is said U For example, if the stock is at a premium
of 10, then M.V. of the stock = 100 + 10 = 110
O
to be below par, at par or above par, depending If 5% is the rate of interest, then actual rate
upon whether the selling price of the stock is 5
less than Rs 100 or exactly Rs 100 or more than % obtained = × 100 = 4.55%
H
110
Rs 100, respectively. If the stock is at a discount of 10, then for
The original value of the share (the value 5% rate %, actual rate % obtained
_T

at which a company originally issues the share) 5


is called its nominal value or face value or = × 100 = 5.56%
90
par value. Face value or the par value of a
In a discount stock, actual rate % obtained
C

share is the value stated in the stock certificate.


increases while in a premium stock, the actual
Face value always remains the same.
rate % obtained actually decreases.
The market value (or the cash value) of
PS

a share is the price of the share in the market. Formulae for Problems on Stocks
If the market value (cash value) is equal to
its face value/nominal value, then the share is (i) Changes from time to time in the market
value of a share
U

said to be at par. If the market value is greater


than the nominal value, the share is at a (a) If the share is at a premium, then
premium or above par and if the market value Market value (M.V.) = Face value (F.V.)
@

is less than the nominal value, then the share + Premium


is at a discount or below par. (b) If the share is at a discount, then
Market value (M.V.) = Face value (FV)
Dividend is the profit of the company – Discount
which is divided among shareholders in (face value may be Rs 10 or Rs 100)
proportion to their holdings. Dividend is
always paid on the face value of a share. A ● Find the cost of Rs 3000, 8% stock at 10
company may declare the dividend as a certain premium.
percentage of par value of a specified number Here premium = 10,
of rupees or paise per share. face value = 100
A 12% dividend means that, on a share of Market value = Face value + Premium
Rs 100, a shareholder gets Rs 12. = 100 + 10
Preference shares A fixed rate of dividend = Rs 110
is paid on these shares to the shareholder. ∴ Total investment = Total purchase cost
Unit Four : Fundamentals / Stocks, Shares and Debentures ✫ 311

3000 × 110 M.V. – brokerage


× Amount of stock
= =
100 100
419
= Rs 3300. Total cash realised = × 3200
4 × 100
(ii) If the stock is purchased, then the purchase
cost = Rs 3,352.

M.V. + brokerage ● How much stock should be sold to realise


= × Amount of stock 1
100 Rs 3660 (brokerage %) from 6% stock

TS
2
at 8% discount?
● Find the cost of Rs 4000, 8% stock at 10%
discount, brokerage ½%. We have sale value of Rs 100 stock or
face value = 100, discount = 8%,
Here face value = 100, discount = 10%,

H
amount of stock = Rs 4000, realisation = 3660, sale and brokerage =
1 1
%.

G
brokerage = % 2 M.V. = Face value – Discount
2
Market value (M.V.) = 100 – 8 = 92
= Face value – Discount
= 100 – 10 U ∴ Sale realisation
O
= 90 M.V. − brokerage
∴ Purchase cost =
100
× Amount of stock
H
M.V. + brokerage 1
100
× amount of stock 92 −
3660 = 2 × Amount of stock
_T

1 100
90 +
= 2 × 4000 183
100 3660 = × Amount of stock
2 × 100
C

181 × 4000 3660 × 2 × 100


= Amount of stock =
PS

2 × 100 183
= Rs 3620. = Rs 4000.
(iv) If the rate % of stock is given, annual
U

(iii) If the stock is sold, then the sale realisation


income on the total stock can be found.
M.V. – brokerage Annual income
@

= × Amount of stock
100 Amount of stock
= % rate of stock ×
100
● Find the cash realised by selling Rs 3,200,
Since total investment = total purchase
6% stock at 5% premium, while
1 cost, Annual income =
brokerage is %. Investment
4 % rate of interest on stock ×
M.V.+ brokerage
Here, Sale price of Rs 100 stock
1 ● Find the annual income on an investment
= 100 + 5 −
4 of Rs 2900 in 6% stock at 116.
3 419
= 104 = Here total investment = Rs 2900,
4 4
brokerage + MV = 116, rate % = 6%.
Sale realisation or cash realisation
Annual income
312 ✫ Quantitative Aptitude

rate % × investment then M.V. = 100 – discount


= (iii) Brokerage is calculated on market value–
(M.V. + brokerage)
as a per cent of market value.
6 × 2900 % Brokerage is added to market value, in
=
116 a purchase.
= Rs 150. Purchase Value = M.V. + % brokerage on
(v) Number of shares M.V.

Total investment (iv) (a) For one share,


=

TS
Purchase value of 1 share Purchase Cost = M.V. + (1 + % bokerage)
(b) For the stock,
Total investment Purchase cost
=
M.V. (1 + % brokerage)
M.V.(1+% brokerage) × Amount of Stock

H
=
● How many shares of market value 100
Rs 25 each can be purchased for

G
Number of shares
Rs 12750, brokerage being 2%?
Total purchase cost
Here M.V.= 25, total investment = 12750 =
and brokerage = 2%
Then, using the formula U M.V. (1 + % brokerage)

During sale of stock, brokerage is to be


O
Number of shares deducted from M.V.
Total investment
H
= (v) (a) For one share,
M.V. (1 + % brokerage) Sale Value = M.V. (1 – %brokerage)
12750
_T

= (b) For the stock, sale realisation


25 (1 + 2%)
M.V. (1 − % brokerage) × Amount of stock
12750 =
= 100
C

2 Also, number of shares


25 + 25 ×
100 Sale realisation
PS

12750 =
= M.V. (1 − % brokerage)
25 + 0.50
12750 (vi) Amount of stock
= = 500. 100
U

25.50
Purchase Cost
(vi) Number of shares = M.V. (1 + % bokerage)
@

Total amount realised on sale Investment


=
Sale value of one share = M.V. (1 + % brokerage)
Total Sale realisation Sale realisation
= M.V. (1 − % brokerage) = M.V. (1 − % brokerage)
● Investment is the same as total purchase cost.
Formulae for Problems on Shares
and Debentures (vii) Annual income
All the formulae for shares are also applicable % rate of stock × amount of stock
for solving questions on debentures. =
100
(i) If the stock is at premium, % rate of stock × Investment
Market value = 100 + premium = M.V. (1 + % brokerage)
(ii) If the stock is at a discount,
Unit Four : Fundamentals / Stocks, Shares and Debentures ✫ 313

(viii) Actual rate % obtained x 2 (7200 − x)


% rate of stock × 100 + = 500
= M.V. (1 + % brokerage) 24 24
x + 14400 − 2x
= 500
(ix) Dividend (expressed usually as a % rate of 24
dividend) is calculated on face value, not 14400 – x = 500 × 24
on market value.
Income earned from dividend x = 14400 – 12000

% rate of dividend × F.V. × No. of shares x = 2400.

TS
=
100 So, Rs 2400 is invested in 4% stock at 96 and
Actual rate % on investment Rs (7200 – 2400 =) 4800 is invested in 12%
stock at 144.
Dividend % × F.V.

H
= ● Ashok invested Rs 8400 in shares of face
M.V. (1 + % brokerage)
value of Rs 100 and 14% dividend. His
Annual Income

G
income from dividend was Rs 1200. Find the
Investment × Dividend % × F.V. market value of share if the brokerage is nil.
=
100 × M.V. (1 + % brokerage)

Miscellaneous Worked Examples U We have, F.V. = 100, % dividend = 12,


investment = 8400, income from dividend =
O
1200
● A man sells Rs 6000, 6% stock at 120 and Let the (M.V.) of the share be x.
Income earned from dividend
H
invests the proceeds partly in 4% stock at 96
and partly in 12% stock at 144, thereby % rate of dividend × F.V.
increasing his income by Rs 140. How much
_T

× Number of shares
of the sale proceeds were invested in each =
stock? 100
% dividend × F.V. × investment
Here M.V. = 120, amount of stock = 6000, =
C

100 + M.V. (1 + % brokerage)


brokerage = 0
Sale realisation % dividend × F.V. × investment
PS

MV =
(M.V. − brokerage) × Amount of stock Income from dividend × 100
=
100 14 × 100 × 8400
=
1200 × 100
U

120 × 6000
Sale realisation =
100 = 14 × 7 = Rs 98.
@

= Rs 7200. ● Saurav buys 800 debentures of face value of


6 Rs 100 each at 94 and sells the same when the
Annual income of 6% = × 6000
100 price rises to Rs 98. If the brokerage is 2%,
= Rs 360 find his gain or loss.
Income after re-investment = 360 + 140 We have, M.V. = 94, brokerage = 2%,
= Rs 500 number of debentures = 800
Let Rs x from the proceeds of sale be re- Then, using the formula
invested in 4% stock at 96 and Rs (7200 – x) Investment while buying 800 debentures
be invested in 12% stock at Rs 144. Total = MV (1 + % brokerage)
annual income is given. × number of debentures
4 × x 12 (7200 − x) 2
∴ + = 500 (given) = 94 1+ × 800
96 144 100
314 ✫ Quantitative Aptitude

4
94 × 102 × 800 Income = × 105 × 95
= 95
100
= Rs 420.
= 76704. 1
2 (b) 4 % at 105
Total sale amount = 98 1− × 800 2
100 9
98 × 98 × 800 On investing Rs 105, income is Rs
2
= On investing Rs (105 × 95),
100
= 76832 9 1
Income = × × 105 × 95

TS
∴ Gain = 76832 – 76704 2 105
= Rs 427.50
= Rs 128. 1
Clearly (b) 4 at 105 is a better
Find the annual income from an investment 2

H

investment.
of Rs 2800 in 4% stock at 112.

G
Here, the total investment is Rs 2800 but Shortcut
the amount of stock purchased is not given. Cross multiply stock rate and price.
∴ Annual income So, we test whether 4 × 105

=
Investment
×
% rate of stock
U or 4
1
2
× 95 is greater
O
(M.V. + brokerage) 100
4 × 105 = 420
2800 1
= × 4 = Rs 100.
H
4 × 95 = 427.5 (greater)
112 2
So (b) is the better investment.
● What rate per cent is obtained by investing
_T

1 1 ● What would a purchaser have to pay for


in 9 % stock at 90 if brokerage is %?
2 4 250 shares of Rs 20 each quoted at Rs 74?
Rate % obtained If he had purchased the shares at par what
C

would his gain have been?


% rate of stock
=
M.V. + brokerage Here, F.V. of each share = Rs 20
PS

M.V. of each share = Rs 74


1
9 Amount paid by purchaser = 250 × 74
= 2 × 100
1 = Rs 18500
90 +
U

4 Cost of purchase for shareholder


= 250 × 20 = Rs 5000
@

19 3
= × × 100 = 10.5% (approximate). ∴ Gain for shareholder = 18500 – 5000
2 361 = Rs 13500.
● Which of the following is the better
investment? ● Arun buys Rs 50 shares in a company which
(a) 4% stock at 95 or pays a dividend of 10%. What did Arun pay
when buying a share if he gets 12.5% on
1
(b) 4% stock at 105 his investment?
2
Let the investment in each case be The dividend on 1 share = 10% of 50
Rs (105 × 95) = Rs 5
(a) 4% at 95 Rs 12.50 is the income on an investment of
On investing Rs 95, income is Rs 4 Rs 100 (given)
On investing Rs (105 × 95), Rs 5 will be an income on an investment
Unit Four : Fundamentals / Stocks, Shares and Debentures ✫ 315

100 a company at 20% premium. If the company


of Rs × 5 = Rs 40 declared a dividend of 5% at the year-end,
12.50
So, Arun pays Rs 40 per share. how much does Janaki get?

● The market value of a certain share is Investment


Number of shares = F.V. + premium
Rs 25. How many shares would a man
purchase for Rs 12750, brokerage being 2%.
14400 14400
The cost of each share = (M.V. + brokerage) = = = 120
100 + 20 120
= (Rs 25 + 2% of 25)
Face value of shares = Rs 100 × 120

TS
= Rs 25.50
= Rs 12000
∴ Number of shares bought
12750 5
= Rs = 500. Annual income = Rs × 12000
100

H
25.50
● Janaki invested Rs 14400 in Rs 100 shares of = Rs 600.

G
U
O
H
C _T
PS
U
@
316 ✫ Quantitative Aptitude

PRACTICE SESSION
Practice Exercise 24
1. A sum invested at 5% simple interest grows (a) 7 : 4 (b) 4 : 3
to Rs 504 in 4 years. The same amount at (c) 7 : 3 (d) 6 : 4
1
10% simple interest in 2 years will grow 9. The difference between the simple interest
2

TS
to received from two different sources on Rs
1500 for 3 years is Rs 13.50. The difference
(a) Rs 420 (b) Rs 525
between their rates of interest is
(c) Rs 450 (d) Rs 550
(a) 0.1% (b) 0.2%

H
2. A moneylender charged Rs 120 as simple (c) 0.3% (d) 0.4%
interest on a sum of Rs 400 for four months. 10. Three years back, a sum of money was
The rate of interest per annum will be

G
remitted in a bank at 12% per annum S.I.
(a) 30% (b) 15% The accounts are now cleared, the bank
(c) 180% (d) 90% paying a sum of Rs 6800. The sum originally
3. The simple interest on Rs 10 for 4 months at
the rate of 3 paise per rupee per month is U invested was
(a) Rs 20,000 (b) Rs 12,000
O
(a) Rs 1.05 (b) Rs 1.20 (c) Rs 7,500 (d) Rs 5,000
(c) Rs 2.10 (d) Rs 2.50 11. A sum of money put at simple interest trebles
H
4. If the rate of interest is 2 paise per rupee itself in 15 years. The rate per cent per
per month, then the interest on Rs 200 in annum is
(a) 13.33% (b) 10%
_T

one year will be


(c) 8.5% (d) 5%
(a) Rs 4 (b) Rs 24
(c) Rs 48 (d) Rs 50 12. An amount of Rs 1,00,000 is invested in two
types of shares. The first yields an interest
C

5. What sum of money will produce an interest of 9% per annum and the second 11% per
of Rs 80 in 5 years at the rate of 5% per annum. If the total interest at the end of one
annum? 3
PS

(a) Rs 320 (b) Rs 380 year is 9 %, then the amount invested in


4
(c) Rs 420 (d) Rs 500 each share was
6. A sum fetched total simple interest of (a) Rs 72,500, Rs 27,500
U

Rs 4,016.25 at the rate of 9 per cent per (b) Rs 62,500, Rs 37,500


annum in 5 years. What is the sum? (c) Rs 52,500, Rs 47,500
@

(a) Rs 8,925 (b) Rs 8,032.50 (d) Rs 82,500, Rs 17,500


(c) Rs 4,462.50 (d) Rs 8,900 13. What is the rate of annual simple interest at
7. How much interest will Rs 10,000 earn in 9 which a sum of money would double itself
months at an annual rate of 6 per cent? in 25 years?
(a) Rs 450 (b) Rs 500 (a) 4% (b) 5%
(c) Rs 475 (d) Rs 600 (c) 6% (d) 8%
1 1
8. A sum of Rs 2000 is divided into two parts 14. A man invested of his capital at 6%; at
2 4
such that one part is invested at 10% per 8% and the remainder at 10%. If his annual
annum simple interest and the second part
income is Rs 600, the capital is
at 15% per annum simple interest. After 2
(a) Rs 5,793.42
years, if Rs 460 is received as interest, then
(b) Rs 6,545.45
the ratio of investment of first and second
(c) Rs 7,993.20
part is
(d) Rs 8,000
316
Unit Four : Practice Session ✫ 317

15. A certain sum of money at S.I. amounts to A was Rs 20,000, what was the total amount
1
Rs 1,012 in 2 years and to Rs 1,067.20 in 4 received on maturity from both the schemes?
2
(a) Rs 58,200 (b) Rs 50,800
years. The rate of interest per annum is
(c) Rs 51,200 (d) Rs 52,800
(a) 2.5% (b) 3%
(c) 4% (d) 5% 22. A person invested in all Rs 2600 at 4%, 6%
and 8% per annum simple interest. At the
16. A certain sum of money at S.I. amounts to
end of the year, he got the same interest in
Rs 1,260 in 2 years and to Rs 1,350 in 5 years.
The rate per cent per annum is all the three cases. The money invested at

TS
(a) 2.5% (b) 3.75% 4% was
(c) 5% (d) 7.5% (a) Rs 200 (b) Rs 600
(c) Rs 800 (d) Rs 1,200
17. The rates of simple interest in two banks A

H
and B are in the ratio 5 : 4. A person wants 23. If a sum of money at simple interest doubles
to deposit his total savings in the two banks in 8 years, it will become 4 times in

G
in such a way that he receives equal half (a) 16 years (b) 18 years
yearly interests from both. He should (c) 20 years (d) 24 years
deposit the saving in banks A and B in the
ratio
(a) 5 : 2 (b) 2 : 5 U 24. A sum of money at simple interest amounts
to Rs 9,440 in 3 years. If the rate of interest
O
is increased by 25%, the same sum of money
(c) 4 : 5 (d) 5 : 4
amounts to Rs 9,800 in the same time. Find
18. Two equal sums of money were invested, the sum and the rate of interest.
H
1
one at 4% and the other at 4 %. At the end (a) Rs 8,000; 6% per annum
2
(b) Rs 8,700; 4% per annum
_T

of 7 years, the simple interest received from


(c) Rs 10,000; 5% per annum
the latter exceeded that received from the (d) Rs 11,200; 8% per annum
former by Rs 31.50. What was each sum
25. A woman derives her income from an
C

equal to?
(a) Rs 1,000 (b) Rs 500 investment of Rs 2,400 at a certain rate of
interest and Rs 2,800 at 4% higher rate. If
PS

(c) Rs 750 (d) Rs 900


the total interest in 3 years is Rs 1,116, find
19. The sum of money that will give Re 1 as
the rate of interest.
interest per day at 5% per annum S.I. is
(a) 7 % (b) 3 %
U

(a) Rs 7,300 (b) Rs 730


(c) Rs 3,650 (d) Rs 36,500 (c) 8 % (d) 5 %
26. Some money out of 6,500 was lent at 6% per
@

20. x, y and z are three sums of money such that


y is the simple interest on x and z is the annum and the remaining was lent at 9%
simple interest on y for the same time and per annum. If the S.I. from both the parts is
rate. Which one of the following is correct? equal to Rs 1,422 at the end of 3 years, find
(a) x2 = yz (b) y2 = zx the sum lent at 6%?
2 (a) Rs 2,800 (b) Rs 3,700
(c) z = xy (d) xyz = 1
21. Ramakant invested some amount in two (c) Rs 6,500 (d) None of these
1
different schemes A and B for five years in 27. A man invested rd of his capital at 7.5%,
the ratio of 5 : 4 respectively. Scheme A offers 1 3
8% simple interest and bonus equal to 20% at 8% and remainder at 10%. If the annual
4
of the amount of interest earned in 5 years income is Rs 1,300, the capital should be
on maturity. Scheme B offers 9% simple (a) Rs 20,000 (b) Rs 15,000
interest. If the amount invested in scheme (c) Rs 5,800 (d) Rs 10,000
318 ✫ Quantitative Aptitude

3
28. A person lent Rs 5,000 partly at the rate of 4 invested in scheme C is th of the amount
per cent and partly at the rate of 5 per cent 4
invested in scheme A. If the total amount of
per annum simple interest. The total interest
interest accrued in one year through all three
after 2 years is Rs 440. To find the sum of
schemes is Rs 17,670, what was the amount
money lent at each of the above rates, invested in scheme C?
Rs 5,000 is to be divided in the ratio (a) Rs 38,000
(a) 4 : 5 (b) 3 : 2 (b) Rs 28,500
(c) 5 : 4 (d) 2 : 3 (c) Rs 57,000
(d) Cannot be determined

TS
29. Ravi invested some amount in three different
schemes A, B and C at interest 12 per cent 30. The simple interest on a sum of money will
per annum, 14 per cent per annum and 18 be Rs 300 after 5 years. In the next 5 years if
per cent per annum respectively. The amount principal is trebled, what will be the total

H
2 interest at the end of the tenth year?
invested in scheme A is rd of the amount
3 (a) Rs 1,300 (b) Rs 1,200
invested in scheme B and the amount (c) Rs 2,800 (d) Rs 3,100

G
Answers with Solutions
1. Ans. (b)
U 3. Ans. (b)
3
O
Sol. Suppose, sum (principal) = Rs 100, Sol. S.I. = Rs 10 × ×4
100
time = 4 years, rate = 5% and
= Rs 1.20.
H
interest = Rs 504
4. Ans. (c)
P× R×T 100 × 5 × 4
S.I. = =
_T

= Rs 20 Sol. Rate of interest = 2 paise per rupee


100 100
A = S.I. + P = 20 + 100 = Rs 120 = 2% per month
If amount is Rs 120, sum = Rs 100 = 24% per year
C

If amount is Rs 504, sum 200 × 24 × 1


Rs 100 ∴ Interest =
= × 504 = Rs 420 100
PS

120
Now, if = 4800 paise or Rs 48.
1 5 5. Ans. (a)
time = 2 years = years, rate = 10%
U

2 2 S.I.×100
5 Sol. Principal =
420 × 10 × R ×T
S.I. = 2 = Rs 105
80 × 100
@

100 =
Hence, amount = Rs 420 + Rs 105. 5× 5
= Rs 525 = Rs 320.
2. Ans. (d)
Sol. Let the rate of interest per annum be x 6. Ans. (a)
P × R×T I × 100 4016.25 × 100
∴ S.I. = Sol. P = = = Rs 8,925.
100 R×T 9×5
4
 T = 4 months = year 7. Ans. (a)
12
10000 9
400 × x × 1 Sol. I = ×6× = Rs 450.
120 = 100 12
100 × 3
8. Ans. (c)
120 × 3 × 100
x = Sol. Let one part be Rs x
400
= 90%. ∴ Other part will be Rs (2000 – x)
Unit Four : Practice Session ✫ 319

Case I: 9x 11 (100000 − x) 39 × 100000


x × 10 × 2 ∴ + =
S.I.1 = 100 100 4 × 100
100 −x
Case II: ⇒ = 9750 – 11000
50
(2000 − x) × 15 × 2 ⇒ x = 50 × 1250 = Rs 62500
S.I.2 =
100 The amount invested in the first type of
Total S.I. = S.I.1 + S.I.2 = 460 share = Rs 62,500
x × 10 × 2 (2,000 − x ) × 15 × 2 The amount invested in the second type
∴ 460 = + of share
100 100

TS
= (100000 – 62500 =) Rs 37,500.
∴ x = 1,400 13. Ans. (a)
Other part = 2000 – x Sol. Let sum = x then
= 2000 – 1400 A = 2P = 2x

H
= 600 1400 ∴ SI = 2x – x = x
∴ Ratio of investment = = 7 : 3.
600 100 × S.I.

G
9. Ans. (c) Rate =
PT
1, 500 × R1 × 3 1, 500 × R2 × 3 100 × x

Sol.
100
⇒ 4500 (R1 – R2) = 1350
100
= 13.50
U 14. Ans. (d)
=
x × 25
= 4%.
O
1350 Sol. Let the capital be Rs x
⇒ R 1 – R2 = x 6 x 8 x 10
4500 ∴ × + × + ×
H
= 600
= 0.3%. 2 100 4 100 4 100
6x 4x 5x
10. Ans. (d) + + = 600
_T

200 200 200


Sol. Interest for 3 years = 3 × 12 15x = 120000
= 36% of the principal x = Rs 8000.
The amount in 3 years = 100 + 36 15. Ans. (c)
C

= 136% of the principal = Rs 6800 Sol. Use formula


6800 × 100 100 [ A2 − A1 ]
PS

∴ Original sum invested = Rate %=


136 A1 t2 − A2 t1
= Rs 5,000.
100 [1067.20 − 1012]
11. Ans. (a) =
1012 × 4 − 1067.20 × 2.5
U

Sol. Let the sum be Rs 100 5520


After 15 years it becomes Rs 300 = = 4%.
@

1380
∴ Interest = 300 – 100 16. Ans. (a)
= Rs 200 RT
Sol. A = P 1+
S.I. × 100 100
Then, rate = 2R
P×T
1260 = P 1 + ... (i)
200 × 100 100
= 5R
100 × 15 1350 = P 1 + ... (ii)
= 13.33%. 100
12. Ans. (b) Divide equation (i) by (ii),
Sol. Suppose amount invested in the first 100 + 2R
type of share = Rs x 1260 100
⇒ =
∴ Amount invested in the second type of 1350 100 + 5R
share = Rs (100000 – x) 100
320 ✫ Quantitative Aptitude

1260 100 + 2R 100 21. Ans. (d)


= × Sol. Amount invested in Scheme A
1350 100 (100 + 5R)
= Rs 20,000 (given)
1260 × (100 + 5R) Amount invested in Scheme B
= (100 + 2R) × 1350 4
1260 × 5R–1350 × 2R = × 20,000 = 16000
5
= 135000–126000 Total amount required
6300R – 2700R = 9000
8×5 20 20000 × 8 × 5
9000 = 20000 1 + +
R= 100 100 100

TS
3600
= 2.5%. 9× 5
+16000 1 +
Shortcut Method 100
100 [ A2 − A1 ] 140 145

H
Use formula A t − A t = 20000 + 1600 + 16000
1 2 2 1 100 100
100 [1350 − 1260] = 28000 + 1600 + 23200

G
⇒ = 2.5%
1260 × 5 − 1350 × 2 = Rs 52800.
17. Ans. (c)
Sol. Let P1 and P2 be the principals.
The rates are 5R and 4R respectively U 22. Ans. (d)
Sol. Let the parts be x, y and 2600 – (x + y)
O
x × 4 ×1 y × 6 ×1 2600 – ( x + y)× 8 ×1
P1 × 5R × 1 P2 × 4R × 1 Then, = =
= 100 100 100
100 × 2 100 × 2
H
P1 4R × 100 × 2 y 4 2 2
∴ = = or y = x
P2 = 100 × 2 × 5R x 6 3 3
_T

5
P1 : P2 = 4 : 5. 2600 – x × 8
So, x × 4 ×1 3
=
18. Ans. (d) 100 100
C

1 1 (7800 – 5 x ) × 8
Sol. 4 % – 4% = % ⇒ 4x =
3
2 2

PS

Let sum be Rs x 52x = 7800 × 8


x 1 7800 × 8
∴ × × 7 = Rs 31.50 ⇒ x =
100 2 52
⇒ x = Rs 900 ⇒ x = 1,200
U

1
[ difference in interest is due to %]. ∴ Money invested at 4% = Rs 1,200.
2
@

19. Ans. (a) 23. Ans. (d)


Sol. S.I. = Re 1 per day Sol. Let principal be x, then S.I. = x
= Rs 365 per year 100 × x
100 × 365 ∴ Rate =
∴ P = x×8
5×1 25
= Rs 7300. = %
2
20. Ans. (b) Now, Principal = x, S.I. = 3x,
xrt 100y 25
Sol. y = ⇒ rt = Rate = %
100 x 2
yrt 100z 100 × 3 x
z = ⇒ rt = y ∴ Time =
100 25

100 y 100 z 2
∴ = ⇒ y2 = zx. = 24 years.
x y
Unit Four : Practice Session ✫ 321

24. Ans. (a) x x x


+ + = 1300
Sol. Let P = Rs x and Rate be r% per annum 40 50 24
x×r×3 15x + 12x + 25x
∴ x+ = 9,440 ... (i) = 1300
100 600
52x = 1300 × 600
125 1300 × 600
x× r×3
100 =
and x + = 9,800 ... (ii) 52
100
∴ Capital = Rs 15,000.

TS
Subtracting (i) from (ii), we get
28. Ans. (b)
3xr
= 360 Sol. According to the given problem, we
400 can make the equation
or xr = 48,000

H
Putting the value of xr in (i) x × 4 × 2 (5000 − x ) × 5 × 2
+ = 440
48000 × 3 100 100

G
x + = 9,440
100 ⇒ x = 3000 (where x is the money lent)
⇒ x = Rs 8000
∴ Investment at 5% = Rs 5000 – 3000


xr = 48000

r =
48000
=
48000
= 6% U Required ratio
= Rs 2000
= 3000 : 2000
O
x 8000 = 3 : 2.
∴ Sum = Rs 8,000 and Rate
29. Ans. (b)
H
= 6% per annum.
Sol. Total interests accrued through A, B
25. Ans. (d) and C = Rs 17670
_T

Sol. Let r be the rate of interest Let amount A invested in scheme B be


Rs x
2400 × r × 3 2800 × ( r + 4)× 3
1,116 = + 2 12 14 3 2 18
100 100 ⇒ x× + x× + × x× = 17670
C

= 72r + 84 (r + 4) 3 100 100 4 3 100


8 x 14x 9x
1,116 = 72r + 84r + 336 ⇒ + + = 17670
PS

∴ 780 = 156r 100 100 100


780 31x
∴ r= = 5%. ⇒ = 17670
156 100

U

26. Ans. (b) x = 57000


Sol. Let the two parts be x and 6500 – x Amount invested in scheme C
x × 6 × 3 (6500 − x) × 9 × 3 3 2 x
@

1422 = + = × x=
100 100 4 3 2
1422 × 100= 18x + 6500 × 27 – 27x 57000
= = Rs 28,500.
9 x = 175500 – 142200 2
33300 30. Ans. (b)
x = = Rs 3,700. Sol. Simple interest for 5 years = Rs 300
9
27. Ans. (b) When the principal is trebled, the
Sol. Let the capital be x, then the amount simple interest for 5 years will also be
invested at 10% is treble the simple interest on original
x x 4 x + 3x 5x principal for the same period. Thus, S.I.
x− + = x− =
3 4 12 12 for last 5 years when principal is trebled
x × 75 × 1 x × 8 × 1 5x × 10 = 3 × 300 = Rs 900
∴ + + = 1300 ∴ Total S.I. for 10 years = 300 + 900
3 × 10 × 100 4 × 100 12 × 100 = Rs 1200.
322 ✫ Quantitative Aptitude

Practice Exercise 25
1. The amount of Rs 6,400 in 1 year 6 months 9. A man borrows Rs 200 at 4% compound
at 5% p.a. compound interest, interest being interest. If he pays back Rs 58 at the end of
calculated half-yearly, will become each year, then how much does he owe at
(a) Rs 8,692 (b) Rs 9,268 the end of 2 years?
(c) Rs 6,892 (d) Rs 2,968 (a) Rs 156 (b) Rs 164
2. The compound interest on Rs 1500 for 2 (c) Rs 176 (d) Rs 184
years at 5% is

TS
10. Rakesh invested Rs 15,000 at simple interest
(a) Rs 158.25 (b) Rs 153.75 rate of 9 per cent per annum for two years
(c) Rs 143.75 (d) Rs 140.25 and invested Rs 12000 at compound interest
3. In what time will Rs 3,90,625 amount to rate of 8 per cent per annum for two years.

H
Rs 4,56,976 at 4% compound interest? What is the total amount of interest accrued
(a) 4 years (b) 7 years by Rakesh in two years?

G
(c) 2 years (d) 1 years (a) Rs 4096.60 (b) Rs 4696.80
4. At what rate per cent compounded yearly (c) Rs 4896.60 (d) Rs 4698.80
will Rs 80,000 amount to Rs 88,200 in 2
years? U 11. The compound interest on a certain sum for
2 years at 10% per annum is Rs 525. The
O
(a) 3% (b) 5% simple interest on the same sum for double
(c) 6% (d) 8% the time at half the rate per cent per annum
H
5. A merchant commences with a certain capital is
and gains annually at the rate of 25 p.a. At (a) Rs 400 (b) Rs 500
_T

the end of 3 years he has Rs 10,000. The (c) Rs 600 (d) Rs 800
original capital of the merchant was
12. If the compound interest for two successive
(a) Rs 4,300 (b) Rs 4,859
years is Rs 110 and Rs 121 respectively, then
C

(c) Rs 5,120 (d) Rs 5,278


the rate of interest is
6. What would be the simple interest, if C.I. (a) 10% (b) 8%
PS

on a sum for 2 years at 4% is Rs 816? (c) 6% (d) 4%


(a) Rs 800 (b) Rs 480
13. If the total compound interest earned after
(c) Rs 420 (d) Rs 780
2 years at the rate of 12% per annum is
U

7. Ashutosh set up a factory by investing Rs 134.40, then the principal amount is


Rs 20,000. During the first three successive (a) Rs 1,200 (b) Rs 1,100
@

years, his profits were 5, 10 and 15% (c) Rs 528.30 (d) Rs 900
respectively. If each year the profit was on
previous year’s capital, his total profit is 14. If the compound interest on a certain sum
(a) Rs 656.50 (b) Rs 13130 of money for 2 years at 10% per annum be
(c) Rs 7000 (d) Rs 6565.00 Rs 1,000, what would be the simple interest?
(a) Rs 700 (b) Rs 952.40
8. A man invested Rs 5,000 for 3 years at 5%
(c) Rs 990 (d) Rs 1,010.50
per annum compound interest reckoned
yearly. Income tax at the rate of 20% on the 15. A sum is invested at compound interest
interest earned is deducted at the end of payable annually. The interest in two
each year. Find the amount at the end of successive years was Rs 500 and Rs 540. The
the third year. sum is
(a) Rs 5624.32 (b) Rs 5630.50 (a) Rs 3,750 (b) Rs 5,000
(c) Rs 5788.125 (d) Rs 5627.20 (c) Rs 5,600 (d) Rs 6,250
Unit Four : Practice Session ✫ 323

16. A loan was repaid in two annual instalments to Rs 1,500 in 3 years. The rate of interest
of Rs 112 each. If the rate of interest be 10% per annum is
per annum compounded annually, the sum (a) 20% (b) 35%
borrowed was (c) 50% (d) 65%
(a) Rs 200 (b) Rs 210 23. If a sum on compound interest becomes
(c) Rs 217.80 (d) Rs 280 three times in 4 years, then with the same
17. The simple interest on a certain sum for 2 interest rate, the sum will become 27 times
years at 8% per annum amounts to Rs 2,128. in
(a) 8 years (b) 12 years

TS
The corresponding compound interest on it
is (c) 24 years (d) 36 years
(a) Rs 15,513.12 (b) Rs 2213.12 24.The cost of a property depreciates every year
(c) Rs 2213 (d) Rs 2128.12 at the rate of 10% on its value at the

H
18. Sanjay borrowed a certain sum from Anil at beginning of that year. If the present value
a certain rate of simple interest for 2 years. of the property is Rs 65,61,000, its worth 3

G
He lent this sum to Ram at the same rate of years ago was
interest compounded annually for the same (a) Rs 1 crore (b) Rs 90 lakh
period. At the end of two years, he received
Rs 4200 as compound interest but paid U (c) Rs 82 lakh (d) Rs 75 lakh
25. The compound interest on a certain sum of
O
Rs 4000 only as simple interest. Find the money at a certain rate for 2 years is Rs 40.80
rate of interest. and the simple interest on the same sum for
H
(a) 15% (b) 20% the same time is Rs 40 at the same rate. The
(c) 35% (d) 10% rate of interest is
(a) 2% per annum
_T

19. A person wants to divide a sum of Rs


3,90,300 between his two sons who are 13 (b) 3% per annum
and 15 years of age respectively in such a (c) 4% per annum
(d) 5% per annum
way that their shares, if invested at 4% per
C

annum of compound interest, should 26. The difference between compound interest
produce the same amount when they become and simple interest for 2 years on a sum of
PS

18 years of age. Find the share of the money is Rs 160. If the simple interest for 2
younger son. years be Rs 2,880, the rate per cent is
(a) Rs 1,43,215 (b) Rs 1,59,000 5 1
(a) 5 % (b) 12 %
U

(c) Rs 1,87,500 (d) Rs 1,96,450 9 2


1
20. The least number of complete years in which (c) 11 % (d) 9%
@

9
a sum of money put out at 20% compound
27. The compound interest on a certain amount
interest will be more than doubled is
for 2 years at the rate of 3% per annum is Rs
(a) 3 years (b) 4 years
1218. The simple interest on the same
(c) 5 years (d) 6 years
amount for the same period is
21. The difference between simple and (a) Rs 1,200 (b) Rs 1,205
compound interests compounded annually (c) Rs 1,210 (d) Rs 1,215
on a certain sum of money for 2 years at 4%
28. If the difference between the simple interest
per annum is Re 1. The sum is
and compound interest on the same principal
(a) Rs 625 (b) Rs 630
amounts at the rate of 20% for 3 years is Rs
(c) Rs 640 (d) Rs 650
48, then the principal amount is
22. A sum of money invested at compound (a) Rs 648 (b) Rs 600
interest amounts to Rs 1,000 in 2 years and (c) Rs 375 (d) Rs 350
324 ✫ Quantitative Aptitude

29. The compound interest on a certain sum is (a) 12 years (b) 5 years
Rs 104 for 2 years and simple interest is Rs (c) 4 years (d) 7 years
100. The rate per cent is 33. What sum of money at compound interest
(a) 2% (b) 5% will amount to Rs 2249.50 in 3 years, if the
(c) 7% (d) 8% rate of interest is 3% for the first year, 4%
30. A sum borrowed under compound interest for the second year and 5% for the third
year?
doubled itself in 10 years. When will it
(a) Rs 2000 (b) Rs 2400
become four-fold of itself at the same rate
(c) Rs 2785 (d) Rs 2819

TS
of interest?
(a) 15 years (b) 20 years 34. An amount of money grows to Rs 5080 in 2
(c) 24 years (d) 40 years years and to Rs 7500 in 3 years on compound
interest. Find the rate per cent.
31. A sum of money invested at compound

H
(a) 48.91% (b) 47.63%
interest doubles itself in 5 years. It will (c) 46.41% (d) 40.98%
amount to eight times itself in

G
35. The difference between the simple interest
(a) 15 years (b) 12 years
and the compound interest compounded
(c) 10 years (d) 20 years annually at the rate of 12% per annum on
32. Find the least number of complete years in
which the sum will become double when put U Rs 5,000 for two years will be
(a) Rs 17.50 (b) Rs 36
O
on C.I. at 18% p.a.? (c) Rs 45 (d) Rs 72
H
Answers with Solutions
1. Ans. (c) R
T
41
_T

Sol. A = P 1 + = 1500 ×
100 400
3
2.5 = Rs 153.75.
= 6400 1 + 3. Ans. (a)
100
C

3 T
41 R
= 6400 Sol. A = P 1+
40 100
PS

T
6400 × 41× 41× 41 4
= 456976 = 390625 1 +
40 × 40 × 40 100
T
= 6892.1 456976 4
= 1+
U

= Rs 6,892. 390625 100


2. Ans. (b) 26
4
26
T
@

Sol. Principal = Rs 1500, Time = 2 years, =


25 25
Rate = 5%
∴ T = 4 years.
Compound interest
T
R 4. Ans. (b)
= P 1+ –1 2
100 r
Sol. 88200 = 80000 1 +
2
100
5 88200 2
=  1 +
= 1500 1+ –1 r 
100 80000 
100 
2
21
2 441 r
= 1500 –1 = 1+
20 400 100
2 2
21 r
441 = 1+
= 1500 –1 20 100
400
Unit Four : Practice Session ✫ 325

21 r Income tax for second year


∴ = 1+
20 100
260 × 20
21 = = Rs 52
⇒ r = – 1 ×100 = 5%. 100
20 ∴ Principal for third year
5. Ans. (c) = 5200 + 260 – 52
Sol. Let the capital be Rs x = Rs 5408
25
3 Interest for third year
10000 = x 1 + 5
100 = 5408 × = Rs 270.40

TS
100
10000 × 4 × 4 × 4 20
∴ x = = Rs 5120. Income tax = 270.40 × = Rs 54.08
5×5×5 100
Amount at the end of the third year
6. Ans. (a)

H
= 5408 + 270.40 – 54.08
R
T = Rs 5624.32.
Sol. C.I. = P 1+ –1

G
100 9. Ans. (a)
200 × 4 ×1
Sol. S.I. for Ist year = = 8
4
2 100
816 = P 1+
100
= P [(1.04)2 – 1]
–1

U A = Rs 208
He paid Rs 58, and so remaining amount
O
= Rs 150
= P × (1.0816 – 1) 150 × 4 ×1
= 0.0816P ∴ S.I. for IInd year = = 6
100
H
816 A = 150 + 6
∴ P = ×10000 = Rs 10000
816 = Rs 156.
_T

4 10. Ans. (b)


S.I. = 10000 × × 2 = Rs 800.
100 15000 × 9 × 2 8
2

7. Ans. (d) Sol. A = + 12000 1+ –1


100 100
C

T
R
Sol. Amount = R 1+
100 270000 11664
= + 12000 –1
PS

5 10 15 100 10000
A = 20000 × 1 + 1+ 1+
100 100 100 270000 1664
105 110 115 = + 12000
= 20000 × × × 100 10000
U

100 100 100 270000 199680


= Rs 26565 = +
100 100
∴ C.I. = 26565 – 20000
@

= Rs 6565. 469680
= = Rs 4696.80.
100
8. Ans. (a)
11. Ans. (b)
Sol. Interest for first year
5 Sol. Let the sum be Rs P, then
= 5000 × × 1 = Rs 250 2
100 10
20 P 1+ –P = 525
∴ Income tax = 250 × = Rs 50 100
100
∴ Principal for second year 2
11
= 5000 + 250 – 50 P –1 = 525
10
= Rs 5200
525 ×100
Interest for second year P =
5 ×1 21
= 5200 × = Rs 260 ∴ Sum = Rs 2500
100
326 ✫ Quantitative Aptitude

2500 × 5 × 4 16. Ans. (b)


So, S.I. = Rs
100 Sol. Principal = (Present worth of Rs 121
= Rs 500.
12. Ans. (a) due 1 year hence) + (Present worth of
Sol. Rate of interest Rs 121 due 2 years hence)
(Difference of C.I.) × 100
= C.I. of first year
11×100 121 121
Rate of interest = = Rs + 2
= Rs 210.
110 10 10
1+ 1+

TS
= 10%. 100 100
17. Ans. (b)
13. Ans. (c)
100 × 2128
Sol. Let the principal be Rs x Sol. P =
2 8×2

H
12
∴ 134.40 +x = x 1 + = Rs 13300
100 T
28
2 R

G
or x – x = 134.40 A on C.I. basis = P 1 +
25 100
2
x [(28)2 – (25)2] = 134.40 × 25 × 25 8

x =
134.40 × 25 × 25
159 U =
= 13300 × 1 +
100
13300 × (1.08)2
O
= Rs 528.30 . = Rs 15513.12
14. Ans. (b) C.I. = 15513.12 – 13300
H
Sol. Let the principal be Rs x, then = Rs 2213.12.
18. Ans. (d)
_T

T
R
x 1+ –1 =A
100 Sol. Suppose the sum borrowed is Rs x
2 Rate of interest = R%
10
x 1+ –1
C

= 1000 Time = 2 years


100
2 x ×R×2
11 ∴
PS

x –1 4000 =
= 1000 100
10
⇒ Rx = 2,00,000 ...(i)
21 Now,
x = 1000
U

2
100 R
1000 ×100 x 1+ = x + 4200
x = 100
21
@

x = Rs 4761.90 xR2 2 Rx
⇒ + = 4200
10000 100
4761.90 × 2 ×10 ⇒ 20R + 4000 = 4200 [from (i)]
∴ Simple interest= Rs
100
⇒ R = 10%.
= Rs 952.38
= Rs 952.40. 19. Ans. (c)
15. Ans. (d) Sol. Let the share of the son who is 13 years
Sol. Simple interest on Rs 500 for 1 year old be Rs x.
= Rs 40 ∴ share of the other son
100 × 40 =Rs (390300 – x)
∴ Rate = = 8%
500 × 1 4
5
4
3
100 × 500 ∴ x 1+ = (390300 – x) 1 +
Sum = Rs = Rs 6250. 100 100
8 ×1
Unit Four : Practice Session ✫ 327

2 T 4 3
26 R R
⇒ x = 390300 – x 1+ = 1+
25 100 100
⇒ x = Rs 1,87,500. R
T
R
12

20. Ans. (b) ⇒ 1+ = 1+


100 100
T T
20 6 ⇒ T = 12 years.
Sol. P 1 + > 2P or >2
100 5 Shortcut Method
6 6 6 6 Here x = 3, t = 4, and y = 3
Now × × × > 2
5 5 5 5 (i.e. xy = 27 = 33)

TS
[Approach: x becomes 2x in 4 years) ∴ The given sum will become 27 times
So, T = 4 years. itself in yt, i.e., 3 × 4 = 12 years.
21. Ans. (a)
Sol. Let the sum be Rs x. Then, 24. Ans. (b)

H
3
2 10
4 Sol. P 1– = 65,61,000
C.I. = x 1 + 100 –x 100

G
6561000 ×10 ×10 × 10
676 51 ∴ P = Rs
= x–x = x 9×9×9

S.I. =
625
x×4×2
=
2x
625

U
25. Ans. (c)
= Rs 90,00,000.
O
100 25
Sol. P [(1 + R)2 – 1] = 40.80
51x 2 x ⇒ PR (R + 2) = 40.80
∴ – = 1
H
625 25 But PR × 2 = 40 (  S.I. Rs 40)
or x = Rs 625. ∴ PR = 20
_T

Shortcut Method 40.80


R+2 = = 2.04
2 20
R
C.I. – S.I. = P 4
100 R = 0.04 = = 4% p.a.
100
C

2
4
1 = P 26. Ans. (c)
100 Sol. Simple interest for 1 year = Rs 1440
PS

100 × 100
P = = Rs 625. ∴ Simple interest on Rs 1440 for 1 year
16
= Rs 160
22. Ans. (c)
100 ×160
U

Sol. Simple interest on Rs 1000 for 1 year Hence, rate per unit =
= Rs (1500 – 1000) = Rs 500 1440 ×1
@

100 × 500 1
∴ Rate= = 50%. = 11 % .
1000 ×1 9
23. Ans. (b) 27. Ans. (a)
4 Sol. Let the sum be Rs 100
R
Sol. P 1+ = 3P Simple interest for 2 years on Rs 100
100
4 at 3% per annum = Rs 6
R Compound interest for 2 years on Rs
1+ = 3 ...(i)
100 100 at 3% per annum = Rs 6.09
T
R If compound interest is Rs 6.09, simple
Let P 1+ = 27P interest is Rs 6
100
T But compound interest is Rs 1218
R 6
1+ = 27 = (3)3 So, simple interest = ×1218
100 6.09
Now, using (i), we get = Rs 1,200.
328 ✫ Quantitative Aptitude

28. Ans. (c) R


n

Sol. Let the sum be Rs 100 4P = P 1 +


100
Simple interest for 3 years at 20%
n
= Rs 60 R 1

Compound interest ⇒ 4 = 1+ = (2 10 )n
100
3 n
20 ∴ 22 =
= Rs 100 1+ –1 210
100 n
∴ = 2
12 12 12 10
= Rs 100 × × –1 ⇒ n = 20 years.
10 10 10

TS
728
= Rs 100 31. Ans. (a)
1000 Sol. Let P be the sum invested
= Rs 72.8 A = P + I = P + P = 2P

H
5
Difference between C.I. and S.I. R
= Rs 72.8 – Rs 60 ∴ 2P = P 1 +
100

G
= Rs 12.80 5
2P R
If the difference is Rs 12.80, the sum is = 1+
P 100
Rs 100.
But difference is Rs 48, so sum is
U ∴ 1+
R
5

= 2
O
1 100
Rs 100 × × 48 T
12.80 R
= Rs 375. 8P = P 1 +
H
100
Shortcut Method 8P R
T

= 1+
_T

3 2
R R P 100
C.I. – S.I.= P 100
+3
100 5 3 15
R R
3 2 8 = (2)3 = 1+ = 1+
20 20 100 100
=P +3
C

48 100 100
⇒ T = 15 years.
1 3 Shortcut Method
PS

=P +
125 25 Here x = 2, t = 5, xy = 8 = 23
16 ∴ y = 3
48 =P ∴ The given sum of money will become
125
U

eight times itself in yt, i.e., 3 × 5 = 15


125 × 48 years.
P = = Rs 375.
@

16 32. Ans. (b)


29. Ans. (d) Sol. C.I. = P, principal itself.
Sol. Difference between C.I. and S.I. T
18
= 104 – 100 = Rs 4 ∴ 2P = P 1 +
2 × 4 × 100 100
Therefore, Rate = = 8%.
100 2P 100 + 18
T
30. Ans. (b)
=
R
10 P 100
Sol. 2P = P 1 +
100 118
T

R
10 2 = = (1.18)T
100
∴ 2 = 1+
100 (1.18)4 = 1.9387 and (1.18)5 = 2.2877
n
R 1 R ∴ The least number of completed years
∴ 1+ = A = P 1+
100 210 100 = 5.
Unit Four : Practice Session ✫ 329

33. Ans. (a) Method II


Sol. By the rule of fraction : 1
y q–p
Principal R = – 1 × 100%
x
100 100 100
= 2249.52 Here x = 5080, y = 7500
103 104 105
p = 2, q = 3
= Rs 2000. 1
3–2
7500
∴ R = – 1 × 100%
5080

TS
34. Ans. (b) 7500
= – 1 × 100%
Sol. Using direct formula 5080
Difference in amount in n years = 47.63%.

H
and (n + 1) years × 100 35. Ans. (d)
Rate =
Amount after n years n
R P ×R×T
Sol. Difference= P 1 + 100 –1 –

G
Here n = 2 100
7500 – 5080 12
2
5000 ×12 × 2
∴ Rate =

Other Methods
5080
×100 = 47.63%.

U = 5000
= Rs 72.
1+
100
–1 –
100
O
Method I Another Method
P + C.I. of 3 years = Rs 7500 ...(i) Here P = 5,000 and R = 12
H
2
P + C.I. of 2 years = Rs 5080 ...(ii) R
∴ C.I. – S.I. = P
Subtracting (ii) from (i) 100
_T

2
C.I. of 3rd year 12
= 5000
= 7500 – 5080 = Rs 2420 100
5000 × 12 × 12
C

2420 ×100 = = Rs 72.


r = = 47.63%. 100 × 100
5080 ×1
PS
U
@
330 ✫ Quantitative Aptitude

Practice Exercise 26
1. The true discount on a bill due 10 months (a) Rs 1200 (b) Rs 1680
hence at 10% per annum is Rs 80. Find the (c) Rs 720 (d) Rs 1600
amount of the bill? 10. If the true discount on Rs 3475 due after a
(a) Rs 1009 (b) Rs 1071 certain time in Rs 975, what will be the true
(c) Rs 1238 (d) Rs 1040 discount approximately on the same amount
2. Find the present worth of the bill of Rs 8500 after half of this particular given time for
due 2 years hence at 10% compound interest. the same of rate of interest and simple

TS
Also find the true discount? interest?
(a) Rs 1248.25 (b) Rs 1475.20 (a) Rs 487.50 (b) Rs 1462.50
(c) Rs 1826.50 (d) Rs 2025 (c) Rs 1950 (d) Rs 567.05

H
3. The difference between the simple interest 11. If the true discount available on an amount
and the true discount on a certain sum for 6 1 3
months at 4% is Rs 30. Find the sum. due in 2 years hence at the rate of 3 % is
4

G
3
(a) Rs 51803 (b) Rs 62742 the same as the simple interest on a sum of
(c) Rs 76500 (d) Rs 89460 Rs 2400 for the same period and rate of
4. Find the present worth and true discount
reckoning 5% per annum simple interest of U interest, the amount due is Rs
(a) 2610 (b) 6210
O
Rs 150 due in 10 months’ time? (c) 2160 (d) 3230
(a) Rs 2 (b) Rs 4 12. The true discount on Rs 4180 due after a
(c) Rs 6 (d) Rs 8
H
certain time at 6% per annum is Rs 180. Find
5. If the simple interest on Rs 540 for 5 years the time after which it is due.
be equal to the true discount on Rs 945 for (a) 3 months (b) 4 months
_T

the same period and for the same rate of (c) 8 months (d) 9 months
interest, find the rate % per annum.
13. The simple interest and true discount on a
(a) 10% (b) 12%
certain amount for a given period at a given
C

(c) 15% (d) 20%


rate of interest are Rs 672 and Rs 480
6. The present worth of a bill due 8 months respectively. The sum due is
PS

hence is Rs 6300. If the bill was due in 26 (a) Rs 1200 (b) Rs 1680
months, its present worth would be 5400. (c) Rs 1380 (d) Rs 1500
Find the sum of the bill.
(a) Rs 8604 (b) Rs 6804 14. If the true discount on Rs 1750 for a certain
U

(c) Rs 6408 (d) Rs 6480 period of time is Rs 350, what will be the
true discount for the same time if the rate
7. The present worth of a bill of Rs 6480 due 2
@

of interest is double that of the rate in the


years hence at compound interest of 8% is first case?
5 1 1
(a) Rs 5000 (b) Rs 5555 (a) Rs 583 (b) Rs 588
9 3 3
(c) Rs 5555 (d) None of these
(c) Rs 700 (d) Rs 850
8. Calculate the true discount, if the present
15. If the true discount and present worth of a
worth of the bill, which is due 2 years hence
certain sum due in 3 years is Rs 117 and Rs
at 8% compound interest, is Rs 2500.
600 respectively. Find the rate per cent.
(a) Rs 614 (b) Rs 416
(c) Rs 461 (d) Rs 516 1
(a) 6 % (b) 6%
2
9. If the difference between simple interest (c) 5.5% (d) 7%
and the true discount on a certain sum of
money for 4 years at 10% is Rs 192, the 16. If the true discount on a certain sum of
sum is money due 5 years hence is Rs 300 and the
Unit Four : Practice Session ✫ 331

simple interest on the same sum at the same 23. The banker’s discount on a sum of money
rate of interest for the same period is Rs 450, 1
for 3 years is Rs 651 and the true discount
find the rate per cent. 2
(a) 6% (b) 7% on the same sum for 4 years in Rs 600. The
(c) 8% (d) 10% rate per cent is
17. Find the present worth of Rs 5459 due in (a) 4% (b) 5%
one year at 6% simple interest if the amount (c) 6% (d) None of these
1
is to be paid in 2 equal half yearly 24. The banker’s gain on a certain sum due 1
3 2
instalments? years hence is th of the banker’s discount

TS
(a) Rs 5000 (b) Rs 5225 28
(c) Rs 5300 (d) Rs 4900 on it for the same rate per cent and the same
18. A trader owes a merchant Rs 901 due 1 year period. Find the rate per cent.
hence. However, the trader wants to settle (a) 5% (b) 6%

H
the account after 3 months. How much cash (c) 8% (d) 12%
should he pay, if the rate of interest is 8% 25. The true discount on a certain sum due 3

G
per annum. 5
years hence is th of the banker ’s discount
(a) Rs 850 (b) Rs 830 6
on the same amount for the same rate per
(c) Rs 800 (d) Rs 780
19. Find the banker’s discount on a certain sum
U cent and the same period. The rate % is
(a) 6% (b) 6.5%
O
if the present worth of the sum is Rs 8000 at
1 2
3 % and the time left for the amount to be (c) 6 % (d) None of these
2 3
H
1
due to is 2 years. 26. A bill is discounted at 10% p.a. The banker’s
2 discount is allowed. The proceeds are
_T

(a) Rs 700 (b) Rs 720 invested in such a manner that nothing is


(c) Rs 780.97 (d) Rs 761.25 lost despite the bill discounted. At what rate
20. If the true discount on Rs 3475 due after a per cent was the investment done?
(a) 10% (b) 11%
C

2
certain time is Rs 975 at 8 % , what is the
unexpired time?
3 (c) 11 1 % (d) 11.19%
PS

9
1
(a) 4 years (b) 4 years 27. A bill is discounted at 4% per annum. If
2 banker ’s discount be allowed, what should
1
(c) 5 years (d) 5 years be the rate per cent for the proceeds to be
U

2 invested so that nothing may be lost?


21. The present worth of a bill due 7 months 1 2
@

hence is Rs 1200 and the rate of interest is (a) 4 % (b) 4 %


6 5
10%. The banker’s discount is 1
(a) Rs 70 (b) Rs 74 (c) 6 % (d) None of these
4
1 1
(c) Rs 74 (d) None of these 28. The banker ’s discount on a sum due at 8 %
12 4 3
22. Sally owes Sheebu Rs 30,000 payable 4 years is times the true discount. Then the
3
hence at 10% C.I. After 2 years, Sheebu unexpired time is
needs money urgently. So he goes to the
(a) 4 years (b) 3 years
bank and gets the bill of exchange drawn
(c) 2 years (d) 1 year
between them encashed at the same terms—
but on simple interest basis. How much 29. The banker’s gain of a certain sum of money
banker’s discount Sheebu has to pay? is Rs 40 and the true discount on the same
(a) Rs 5138.40 (b) Rs 4392.30 sum of the same time and at the same rate
(c) Rs 13923 (d) Rs 8784.60 is Rs 30. Calculate the sum.
332 ✫ Quantitative Aptitude

(a) Rs 120 (b) Rs 140 (a) Rs 444.44 (b) Rs 333.33


(c) Rs 160 (d) Rs 180 (c) Rs 222.22 (d) Rs 111.11
30. The present worth and the bankers gain on 36. What rate per cent does a man get for his
a bill is Rs 10,000 and Rs 25 respectively. Find money when in discounting a bill due 14
the true discount of the bill. months hence, he deducts 7% of the amount
(a) Rs 700 (b) Rs 600 of the bill?
(c) Rs 500 (d) Rs 400
(a) 6.45% (b) 7.83%
31. If the present worth of an amount due in 10 (c) 5.19% (d) 5.80%
months at 20% rate on SI is Rs 360 and

TS
banker’s gain is Rs 10, then find the amount. 37. Calculate the true discount if the banker’s
(a) Rs 420 (b) Rs 480 gain on a bill due 2 years hence at 10% is
(c) Rs 540 (d) None of these Rs 2.
(a) Rs 15 (b) Rs 13

H
32. The banker’s discount on a bill of Rs 7560.00
(c) Rs 10 (d) Rs 8
is Rs 378. The true discount is
(a) Rs 360 (b) Rs 380 38. Calculate the banker ’s discount when

G
(c) Rs 420 (d) Rs 345 banker ’s gain on a sum due 5 years hence at
33. The true discount on Rs 2442 at 3% is the 5% is Rs 150?
same as the banker’s discount on Rs 2400
for the same period and the same rate of U (a) Rs 500
(c) Rs 700
(b) Rs 600
(d) Rs 800
O
interest. Find the time left for the bill to be 39. Find the present worth when banker’s gain
due. of a certain sum due 3 years hence at 5%
H
(a) 7 months (b) 8 months per annum is Rs 12.
(c) 1 year (d) None of these (a) Rs 320.46 (b) Rs 533.33
_T

34. A bill for Rs 9,236 is drawn on August 19 (c) Rs 612.70 (d) Rs 741.50
for 3 months and 2 weeks. But after 2 weeks
40. A bill was drawn on May 14 at 12 months
on September 5th it is discounted at 6%.
How much money did the holder of the bill after date and was discounted on July 2 at
C

receive? 1
8 % per annum simple interest. If the
(a) Rs 9,000 (b) Rs 9,100 9
banker ’s gain be Re 1, for what sum was
PS

(c) Rs 9,097.46 (d) None of these


the bill drawn?
35. Banker’s gain on a bill due 6 years hence at
(a) Rs 90,300 (b) Rs 80,700
5% is Rs 40. Find the present worth of the
(c) Rs 70,300 (d) Rs 60,700
U

bill?
Answers with Solutions
@

1. Ans.(d) 100 True discount (T.D.) = P – P.W.


Sol. Amount P = T.D. × 1 + = 8500 – 7024.79
RT
= Rs 1475.20.
3. Ans. (c)
100
80 × 1 + P × (RT )2
= 10 Sol. S.I. – T.D. =
10 × 100 (100 + RT )
12 2
= 80 × 13 = Rs 1040 1
P× 4×
∴ Amount of the bill is Rs 1040. 30 =
2
2. Ans. (b) 1
100 100 + 4 ×
P 8500 2
Sol. P.W. = T
= 2 P×4
R 10
1+ 1+ 30 = 100 (102)
100 100
30 ×100 × 102
= Rs 7024.79 P = = Rs 76500.
4
Unit Four : Practice Session ✫ 333

4. Ans. (c)
6480
P 150 =
Sol. P.W. = = 108
2
RT 5 10
1+ 1+ × 100
100 100 12
= Rs 144 6480 ×100 × 100
=
T.D. = P – P.W. = 150 – 144 = Rs 6. 108 × 108
5. Ans. (c) 50000 5
Sol. S.I. on 540 = T.D. on 945; T = 5 years = = Rs 5555 .
9 9

TS
540 × R × 5 945 × R × 5 8. Ans. (b)
=
100 100 + R × 5
A
945 × 5R Sol. P.W. = T
27R = R

H
100 + 5 R 1+
100
945 × 5 R
100 + 5R =

G
27 R T
R
100 + 5R = 35 × 5 = 175 ∴ A = P.W. × 1 +
100

6. Ans. (b)
5R = 75
R = 15%.
U = 2500 × 1 +
8
100
T
O
Sol. (P.W.1) Present worth of bill due 8 108 108
months hence = Rs 6300 (given) = 2500 × ×
H
100 100
(P.W.2) Present worth of bill due 26
= Rs 2916
months hence = Rs 5400 (given)
_T

⇒ True Discount = 2916 – 2500 = Rs 416.


P.W.1 100 + RT2
= 100 + RT 9. Ans. (b)
P.W.2 1

26 T.D. × RT
C

100 + R × Sol. S.I. – T.D. =


6300 12 100
i.e., = 8
5400
PS

100 + R × T.D. × 10 × 4
12
i.e., 192 =
1200 + 26 R 12 100
= ×
12 1200 + 8 R 192 ×100

U

T.D. = = 480
63 (1200 + 8R) = 54(1200 + 26R) 10 × 4
63 × 1200 – 54 × 1200 = 54 × 26R – 63 × 8R T.D. × 100

@

P.W. =
9 × 1200 = 1404R – 504R RT
9 ×1200 480 ×100
R = = 12% = = 1200
900 10 × 4
PW(100 + RT) ∴ Amount (P) = P.W. + T.D. = Rs 1680.
A =
100
10. Ans. (d)
6300 ×108 Sol. Let R be the rate of interested and T be
=
100 the time
= Rs 6804.
7. Ans. (b) T.D.1 P.W.1 × R1T1
=
T.D.2 P.W.2 × R2T2
Amount 6480
Sol. P.W. = T =
R 8
2 (A – T.D.1 ) × R1 × T1
1+ 1+ =
100 100 (A – T.D.2 ) × R2 × T2
334 ✫ Quantitative Aptitude

Here, 14. Ans. (a)


975 (3475 – 975)× R × T Sol. P.W. = 1750 – 350 = 1400
=
T.D.2 T P.W. × RT
(3475 – T.D.2 )× R × T.D. =
2 100
2500 × RT × 2 Let R be the rate of interest and T be
=
RT × (3475 – T.D.2 ) the time in years.
1400 × R × T
975 5000 350 =
= 100

TS
T.D.2 3475 – T.D.2
350 = 14RT
975 × 3475 – 975 T.D.2 = 5000 T.D.2 350
975 × 3475
⇒ RT = = 25

H
14
T.D.2 =
5975 A × RT
= 567.0502 T.D. =

G
100 + RT
= Rs 567.05 approximate.
1750 × 2R × T
=
11. Ans. (a)

Sol.
PRT ART
= (100 + RT ) U 100 + 2R × T

1750 × RT × 2
O
100 =
100 + 2 × RT
2400 ×15 × 7 A ×15 × 7
=
H
15 7 1750 × 25 × 2
4 × 3 ×100 4 × 3 × 100 + × =
4 3 100 + 2 × 25
_T

35 A 1750 × 2 × 25
210 = =
35 150
4 100 +
4 1
= Rs 583 .
C

35 × A 35 A 3
= 435 = 435 15. Ans. (a)

PS

4 P.W. × R × T
210 × 435 Sol. T.D. =
∴ A = = Rs 2610. 100
35
U

T.D. × 100
12. Ans. (d) ∴ R =
P.W. × T
Sol. P.W. = 4180 – 180 = 4000
@

117 × 100
T.D. × 100 180 × 100 =
T = = 600 × 3
P.W. × R 4000 × 6
13 1
3 = = 6 %.
= years = 9 months. 2 2
4
13. Ans. (b) 16. Ans. (d)
S.I. × T.D. S.I. × T.D.
Sol. Amount = Sol. A =
S.I. – T.D. S.I. – T.D.
672 × 480
= 450 × 300
672 – 480 =
450 – 300
672 × 480 450 × 300
= = Rs 1680. = = 900
192 150
Unit Four : Practice Session ✫ 335

A × RT 20. Ans. (b)


S.I. =
100 Sol. P.W. = A – T.D. = 3475 – 975 = 2500
450 ×100
∴ R = = 10%. 3475 ×100
900 × 5 ∴ 2500 =
26
100 + × T
17. Ans. (b) 26 3
Sol. A = 5459; R = 6%; n = 2, ⇒ T = 39
3
6 1 39 × 3 9
T1 = = ; T = 1
12 2 2 ⇒ T = =
26

TS
2
= 4.5 years.
P.W. = 100 × 5459 1 1 21. Ans. (c)
× +
2 6 100 + 6 ×1
100 + 6 × P.W. × R × T

H
12 Sol. T.D. =
100
100 × 5459 1 1
= × +

G
2 103 106 1200 ×10 × 7
= = Rs 70
12 ×100

=
50 × 5459 ×(106 + 103)
103 ×106
U ∴ B.G. =
(T.D.)2
P.W.
O
50 × 5459 × 209 70 × 70
= = Rs 5225. =
103 ×106 1200
H
18. Ans. (a) 49 1
= = 4
Sol. P.W. of Rs 901 due 9 months hence at 12 12
_T

8%. B.D. = T.D. + B.G.


1
100 × 901 ∴ B.D. = 70 + 4
= Rs 12
C

3
100 + 8 × 1
4 ∴ B.D. = Rs 74 .
PS

2
100 × 901×1 22. Ans. (d)
= Rs = Rs 850.
106 Sol. Amount due by Sally
T
19. Ans. (d) R
U

= P 1+
100
P.W. (100 + RT ) 4
Sol. A =
@

100 10
= 30000 × 1 +
100
7 5
100 + × 4
2 2 11
= 8000 × = 30000 ×
100 10
8000 × 435 = 30000 × (1.1)4
=
4 ×100 = Rs 43923
= 8700
A = 43923, unexpired time = 2 years,
8700 × 7 × 5 R = 10%
B.D. =
2 ×100 × 2
43923 ×10 × 2
3045 ∴ B.D. =
100
= = Rs 761.25.
4 = Rs 8784.60.
336 ✫ Quantitative Aptitude

23. Ans. (c) x × 5x × 6


=
6×x
1
Sol. B.D. for 3 years = 651 = 5x
2
651× 2 × 4 B.D. × 100
∴ B.D. for 4 years = ∴ R =
7 A× T
= 93 × 8
x + 100
= Rs 744 =
5x + 3
20 2

TS
B.D. × T.D. = = 6 %.
∴ Amount = 3 3
B.D. – T.D. 26. Ans. (c)
744 × 600 Sol. Proceeds after the bill discount
=

H
= 100 – 10 = 90
144
If Rs 10 has to be the simple interest on
= Rs 3100 Rs 90.

G
(So that nothing is lost) for 1 year, then
B.D. × 100 100 × 10
∴ Rate =
Amount × T

744 ×100 U rate % =

=
90 × 1
100 1
= 11 % .
O
= = 6%. 9 9
3100 × 4
27. Ans. (a)
24. Ans. (c)
H
Sol. Let the amount of the bill be Rs 100.
Sol. Let the banker ’s discount be x. It is discounted at 4%.
3x ⇒ Proceeds of the bill = Rs 96
_T

∴ Banker’s gain =
8 Nothing has to be lost
3x
T.D. = x – ⇒ S.I. Rs 4 to be got on Rs 96 for 1 year.
8
100 × 4 25
C

25 x ∴ Rate % = =
= 96 ×1 6
8 1
PS

= 4 %.
B.D. × T.D. 6
A = 28. Ans. (a)
B.G.
Sol. Let T.D. be x.
x × 25 x × 28
U

= 4
28 × 3 x ∴ B.D. = x
3
25 x
@

= 4 x
3 ⇒ B.G. = x–x =
3 3
B.D. × 100 B.G. × 100
∴ Rate % = T =
A× T T.D. × R
x × 100 × 3 × 2 x ×100 × 3
= = 8%. = = 4 years.
25 x × 3 3 × x × 25
25. Ans. (c)
29. Ans. (a)
Sol. Let the B.D. be x.
5 B.D. × T.D.
⇒ T.D. = x Sol. Sum =
6 B.D. – T.D.
5
x× x 40 × 30
∴ A = 6 = Rs
5 40 – 30
x– x = Rs 120.
6
Unit Four : Practice Session ✫ 337

30. Ans. (c) 1.75


Sol. T.D.= ⇒ T =
P.W. × B.G. 3
= 10000 × 25 1.75
= ×12
3
= 250000
= 7 months.
= Rs 500.
34. Ans. (c)
31. Ans. (a)
Sol. Date on which the bill was drawn
P.W. × RT
Sol. T.D. = = August 19

TS
100
Nominal due date = December 2nd
360 × 20 × 10
= = 60 Legally due date = December 5th
100 ×12
Date of encashment of bill

H
⇒ B.D. = 60 + 10 = 70
= September 5th
70 × 100 ×12
∴ A = Unexpired period = 3 months

G
20 × 10
A × R ×T
= 7 × 6 = 420 ∴ B.D. =
100
or
P.W. =
A × 100
U =
9236 × 6 × 3
100 ×12
O
100 + RT
20 ×10 = Rs 138.54
360 × 100 +
∴ Money received by the holder of the
H
A = 12
100 bill
= 9236 – 138.54
_T

360 × 350
= = Rs 9,097.46.
3 ×100
35. Ans. (c)
= Rs 420.
2
C

32. Ans. (a) RT


Sol. B.G. = P.W.
100
B.D. × T.D.
PS

Sol. A =
B.D. – T.D. 5×6
2

20 = P.W.
378 × T.D. 100
7560 =
378 – T.D.
U

2
30
7560 × 378 – 7560 T.D. = 378 T.D. ⇒ 20 = P.W.
100
@

7560 × 378 2
∴ T.D. = (7560 + 378) 100
⇒ P.W. = 20 ×
30
7560 × 378 ⇒ = Rs 222.22.
= = Rs 360.
7938
33. Ans. (a) 36. Ans. (a)
Sol. B.D. on 240 = T.D. on 2442. Sol. Suppose the amount of bill is Rs 100
Money deducted = Rs 7
2400 × 3 × T 2400 × 3 × T
= Money received by holder of the bill
100 100 + 3T
= Rs (100 – 7) = Rs 93
2442 × 3T
100 + 3T = = 101.75 S.I. on Rs 93 for 14 minutes = Rs 7
24 × 3T
100 × 7 × 6
⇒ 3T= 1.75 Rate = % = 6.45%.
93 × 7
338 ✫ Quantitative Aptitude

37. Ans. (c) 15 3


∴ Time = years or years
B.G. × 100 2 ×100 365 73
Sol. T.D. = =
R×T 10 × 2 Let the sum be Rs 100, then
= Rs 10. A× R×T
38. Ans. (b) T.D. = (100 + RT )
B.G. × 100
Sol. T.D. = 73 3
R×T 100 ××
9 73

TS
150 ×100 73 3
= Rs T.D. = Rs
5×5 100 + ×
9 73
= Rs 600.
100
39. Ans. (b) = Rs

H
301
B.G. × 100
Sol. T.D. = 3 73 1

G
R×T and B.D. = Rs 100 × × ×
73 9 100
12 × 100
= Rs 1
= Rs 80
5×3

U = Rs
1 100
3
1
O
(T.D.)2 ∴ B.G. = Rs – = Rs
P.W. = 3 301 903
B.G.
1
H
(80)2 Now, if B.G. is Rs , then sum
= = Rs 533.33. 903
12
= Rs 100
_T

40. Ans. (a) ∴ If B.G. is Re 1, then sum


Sol. No. of days for which the bill has yet = Rs (100 × 903)
to run = 15 days. = Rs 90,300.
C
PS
U
@
Unit Four : Practice Session ✫ 339

Practice Exercise 27
1. Find the cost of Rs 1,000, 10% stock at par. (a) 2.8% (b) 2%
(a) Rs 1,000 (b) Rs 1,100 (c) 3% (d) None of these
(c) Rs 990 (d) Rs 1,010 9. A broker sold Rs 4,000 stock at 94. If the
1
2. Find the cost of Rs 2,500, 9.5% stock at 5 brokerage is % , find the sale proceeds of
1 2
discount, brokerage % . the broker.
2 (a) Rs 3,700 (b) Rs 3,760

TS
(a) Rs 2,387.50 (b) Rs 2,388 (c) Rs 3,940 (d) Rs 7,340
(c) Rs 2,375 (d) Rs 2,625.00 10. A company has a capital of Rs 10 lakhs made
1 up of shares of Rs 100. A profit of Rs 85,000
3. Which is a better investment: 5 % stock at
was declared at the end of the financial year.

H
3 2
102 or 4 % stock at 98? Find the dividend received by a man
4
holding 500 shares.

G
3
(a) 4 % stock at 98 (a) Rs 3,520 (b) Rs 5,240
4
1 (c) Rs 4,520 (d) Rs 4,250
(b) 5 % stock at 102
2
(c) Both are equally good U 11. What is the price of the stock which pays
15% dividend to Pooja, if by investing in it
O
(d) Neither of them is good she gets 18% on her money?
4. A man invested Rs 8,800 when he bought 2
H
(a) Rs 83 (b) Rs 77
Rs 100 shares at Rs 110. When a 12% 3
dividend is declared, his annual income is 1 1
(c) Rs 83 (d) Rs 77
_T

(a) Rs 900 (b) Rs 960 3 3


(c) Rs 920 (d) None of these 12. Vineet invests Rs 5,400 partly in 7% stock at
5. A man sold 300 shares with face value Rs 100 147 and partly in 6% stock at 144. If his
C

and the market value Rs 680. If he had income is the same, find his two investments.
purchased the shares at 65% premium, what (a) Rs 2,500 and 2,900
PS

was his gain? (b) Rs 2,600 and Rs 2,800


(a) Rs 1,50,000 (b) Rs 1,54,000 (c) Rs 2,520 and Rs 2,880
(c) Rs 1,54,500 (d) Rs 1,60,000 (d) Rs 2,580 and Rs 2,820
U

6. A man bought 250 shares with face value Rs 13. A man invests Rs 12,100 partly in 7% stock
100 and the market value Rs 150. If he sold at 80 and partly in 9% stock at 117. If his
the shares at 150% premium, what was his total income is Rs 990, how much did he
@

gain? invest in each kind of stock?


(a) Rs 15,000 (b) Rs 20,000 (a) Rs 5,500 and Rs 16,600
(c) Rs 25,800 (d) Rs 25,000 (b) Rs 5,000 and Rs 7,100
7. Find the cash realised by selling Rs 2,400, (c) Rs 5,600 and Rs 6,500
9 1 (d) Rs 6,000 and Rs 6100
% stock at 5 premium (brokerage % ).
2 4 14. Arvind has 1,000 Rs 8 shares in a company,
(a) Rs 2,500 (b) Rs 2,514
out of which he sells 300 at Rs 10 each. He
(c) Rs 1,524 (d) None of these
reinvests the proceeds in a stock at 75. Find
8. A man purchased 300 shares of the face value Arvind’s total income in one year from the
of Rs 100 each from the market at Rs 500
shares and the stock if a dividend of 8% per
per share. If a dividend of 14% is declared,
what will be his earning per cent on his annum is paid on the shares and 11% is paid
investment? on the stock.
340 ✫ Quantitative Aptitude

(a) Rs 888 (b) Rs 448 Rs 75 and 6% stock at 80. If the income is


(c) Rs 440 (d) None of these same, then the investment in 5% stock is
15. Jessy invested Rs 3,000 in 13% stock at 75 (a) Rs 3,400 (b) Rs 1,600
and sold it a certain market value. She (c) Rs 1,500 (d) Rs 1,800
invested the proceeds in 15% ‘Mavelikara 22. A man invests some money partly in 3%
rice’ at 90 and increased her income by Rs stock at 96 and partly in 4% stock at 120. To
56.00. What was the market value when she
get equal income from both, he must invest
sold the first stock?
the money in the ratio
(a) Rs 86 (b) Rs 84

TS
(c) Rs 86.60 (d) Rs 86.40 (a) 4 : 3 (b) 3 : 4
(c) 15 : 17 (d) 16 : 15
16. The market value of Rs 10 share of a
company is Rs 16. Mahavir buys 250 shares 23. If Vijayan wants to purchase a 6% stock
which yields 5% on his capital, what price

H
of the company. What will be his annual
income from these shares if the company should he pay to buy the stock?
1 (a) Rs 110 (b) Rs 120

G
declares 6 % dividend.
4 (c) Rs 60 (d) None of these
(a) Rs 200 (b) Rs 225 24. By investing Rs 2,350 in a 7% stock, one
(c) Rs 250 (d) Rs 275
17. What is the sum due whose present worth U earns Rs 100. The market value of the stock
was
O
at 6% per annum in 3 years hence is Rs 1,000? (a) Rs 110.75 (b) Rs 128.25
(a) Rs 1,150 (b) Rs 1,160 (c) Rs 164.50 (d) Rs 146.50
H
(c) Rs 1,170 (d) Rs 1,180 25. Find the cash required to purchase Rs 2,700,
1
_T

18. How much 4% stock at 95 can be purchased 9% stock at 110 (brokerage % ).


1 2
by investing Rs 2667 (brokerage % )? (a) Rs 2,540.35 (b) Rs 2983.50
4
(c) Rs 30,500 (d) Rs 3,782.50
(a) Rs 3,400 (b) Rs 3,200
C

(c) Rs 3,000 (d) Rs 2,800 26. Find the annual income derived by investing
1 1
Rs 770 in 4 % stock at 96 (brokerage % ).
PS

19. An amount of Rs 1,00,000 is invested in two 2 4


types of shares. The first yields an interest 1 5
(a) 4 % (b) 4 %
of 9% per annum and the second 11% per 3 7
annum. If the total interest at the end of one 1 5
(c) 5 % (d) 5 %
U

3 3 7
year is 9 % , then the amount invested in
4 1
each share was 27. Which is the better stock, 4% at 340 or 2 %
@

at 90? 2
(a) Rs 72,500, Rs 27,500 1
(b) Rs 62,500, Rs 37,500 (a) 4% at 340 (b) 2 % at 90
2
(c) Rs 52,500, Rs 47,500 (c) Both are same (d) Data inadequate
(d) Rs 82,500, Rs 17,500 1
28. Find the market value of a 3 % stock, in
20. Rahul purchased stock for Rs 1500 and sold 4
2 which an income of Rs 544 is derived by
of it after its value doubled. He sold the 1
3 investing Rs 12,975, brokerage being %.
remaining stock at 5 times its purchase price. 2
What was his total profit on the stock? (a) Rs 99 (b) Rs 88
(a) Rs 1,500 (b) Rs 2,000 (c) Rs 77 (d) Rs 66
(c) Rs 2,500 (d) Rs 3,000 29. A man’s income increases by Rs 21 by
21. Rs 3,400 was partly invested in 5% stock at 1 1
converting a 3 % stock at 63 % into 5%
2 8
Unit Four : Practice Session ✫ 341

7 1
stock at 74 . Find the stock he has, the th of the stock at 80 and invests the
8 5
1 proceeds in a dotcom venture at 120. What
brokerage being %.
8 dividend must the dotcom stock pay so that
(a) Rs 2,000 (b) Rs 3,000 he may thereby increase his income by
(c) Rs 4,000 (d) Rs 5,000 Rs 50? (Assume the par value of stock as
Rs 100).
30. Jayesh owns 2.5 per cent stock of company (a) 8% (b) 7.5%
LIH, that yields Rs 250 a year. He sells out (c) 9% (d) 8.5%

TS
Answers with Solutions
1. Ans. (a) ⇒ Gain = 300 × (680 – 165)
= 300 × 515

H
Sol. Purchase Cost
= Rs 1,54,500.
Market value × Amount of Stock 6. Ans. (d)
=

G
100 Sol. Sale value of 1 share = 100 + 150 = 250
100 × 1000
= = Rs 1000. Purchase value = 150
2. Ans. (a)
100

Sol. Purchase Cost U Gain = 250 (250 – 150)


= Rs 25000.
O
(M.V. + brokerage)× Amount of stock 7. Ans. (b)
=
100 Sol. Cash realised on sale
H
1 (Market value – brokerage)
100 – 5 + × 2500
= 2 × Amount of Stock
=
_T

100 100
= 95.5 × 25 = Rs 2387.50. 1
100 + 5 – × 2400
4
3. Ans. (b) =
C

100
Sol. Let the amount of investment in each = Rs 2,514.
case be 102 × 98.
PS

8. Ans. (a)
1 Sol. Income earned
∴ Income received from 5 % stock at 102
2
11×102 × 98 Dividend % × Face Value × No. of shares
=
U

= = 539 100
2 × 102
1 14 ×100 × 300
Income received from 4 % stock at 98 = = Rs 4200
@

2 100
9 ×102 × 98 4200 × 100
= = 459 % income = = 2.8%.
2 × 98 1500
By investing the same amount, the
1 9. Ans. (c) 1
income received from 5 % stock at 4000 × 94 –
2 2
Sol. Sale realisation =
102 is better than the other one. 100
4. Ans. (b) 4000 × 93.5
8800 ×12 ×100 =
Sol. Annual income = 100
110 ×100
= Rs 3,740.
= Rs 960.
5. Ans. (c) 10. Ans. (d) 1000000
Sol. Sale value of 1 share = 680 Sol. Total number of shares =
100
Purchase value of 1 share = 165 = 10000 shares
342 ✫ Quantitative Aptitude

85000 11x = 80 × 110(9 × 13 – 110)


∴ Dividend per share = = 8.5
10000 80 ×110 × 7
∴ Dividend received by holder of 500 shares x =
11
= 500 × 8.50
= Rs 4,250. Investment at 7% stock = Rs 5,600
11. Ans. (c) ⇒ The other investment at 9% stock
Sol. Pooja gets 18% on her money = 12100 – 5600
⇒ For an investment of Rs 100, her income = Rs 6,500.
is Rs 18. 14. Ans. (a)

TS
∴ Market value of the stock + brokerage Sol. Face value of 700 shares= 700 × 8
Rate % stock × Investment = Rs 5600
= Dividend from these 700 shares
annual income
5600 × 8

H
15 ×100 250 1 = = Rs 448 ...(i)
= = = 83 100
18 3 3
Cash realised on sale of 300 shares

G
There is no brokerage. = 300 × 10 = 3000
Income from this stock
∴ Market value = Rs 83 1 .
12. Ans. (c)
3
U % rate of stock × Investment
=
Market value
O
Sol. Annual Income 11× 3000
% rate of stock × Investment
= = Rs 440 ...(ii)
75
H
= Market value + brokerage ∴ Arvind’s total income
= 448 + 440 = Rs 888.
_T

Let the investments be x and 5400 – x. 15. Ans. (d)


7×x 6 ×(5400 – x ) Sol. Amount of stock
∴ = 3000 ×100
147 144
= = Rs 4000
75
C

24x = 21(5400 – x)
Income from this stock
24x + 21x = 21 × 5400 13 × 3000
PS

= = Rs 520
21× 5400 75
x = Income from ‘Mavelikara rice’
45
i.e., one investment @ 7% stock = 520 + 56
U

= Rs 576
= Rs 2,520
Cash realised from the sale of first stock
∴ The second investment
= Investment for ‘Mavelikara rice’
@

= 5400 – 2520
= Rs 2,880. Income × Market value
13. Ans. (c) =
% rate of stock
Sol. If x be the amount invested in 7% 576 × 90
stock and 12,100 – x be the amount in = = 3456
15
9% stock, So, in the case of first stock
7 × x 9 × (12100 – x ) cash realised on sale = Rs 3456
+ = 990
80 117 Amount of stock = Rs 4000
7 x 12100 – x ∴ Market value at which the stock is sold
+ = 990 Sale realised × 100
80 13 =
Amount of stock
13 × 7x + 80(12,100 – x)
= 990 × 80 × 13 3456 ×100 3456
= = = Rs 86.40.
91x – 80x = 990 × 80 × 13 – 80 × 12,100 4000 40
Unit Four : Practice Session ✫ 343

16. Ans. (c) 5×x 6 × (3400 – x )


Sol. Cash value of 250 shares = 250 ×16 =
75 80
1 40x = 45 (3400 – x)
∴ Annual income at 6 %
4 40x + 45x = 45 × 3400
25 1 85x = 45 × 3400
= 4000 × ×
4 100 45 × 3400
= Rs 250. x =
85
17. Ans. (d) = 1800
Sol. Simple interest on Rs 1,000 at 6% for ∴ The investment in 5% stock

TS
3 years = Rs 1,800.
1000 × 6 × 3 22. Ans. (d)
= = Rs 180
100 Sol. Let x1 and x2 be the amounts invested
∴ Sum due = 1000 + 180 = Rs 1,180. in 3% stock at 96 and 4% stock at 120

H
18. Ans. (d) respectively. In the case of equal income
1
Sol. ∴ By investing Rs 95 + , it implies

G
4
stock purchased = Rs 100 3 × x1 4 × x2
=
96 120
By investing Rs 2667, the stock
purchased
100 × 4 × 2667 U x1
x2 =
32
30
O
= Rs = Rs 2,800. x1 : x2 = 16 : 15.
381
19. Ans. (b) 23. Ans. (b)
H
Sol. Suppose amount invested in the first Sol. Actual rate % obtained
type of shares = Rs x. % rate of stock × 100
= M.V. + brokerage
_T

∴ The amount invested in the second


type of shares = Rs (100000 – x) 6 ×100
∴ Market value =
9 x 11(100000 – x ) 39 ×100000 5
∴ + = = Rs 120.
C

100 100 4 ×100


–x 24. Ans. (c)
⇒ = 9750 – 1100
PS

50 % rate of stock × Investment


Sol. Income =
⇒ x = 50 × 1250 Market value
⇒ x = Rs 62,500 7 × 2350
=
100000 – x = 100000 – 62500 100
U

= Rs 37,500. = Rs 164.50.
20. Ans. (d) 25. Ans. (b)
@

2 Sol. Cash required for purchasing Rs 100


Sol. CP of of the stock = Rs 11000 stock
3
2 1
SP of
3
of the stock = Rs 2,000 = Rs 110 +
2
1 221
CP of of the stock = Rs 500 = Rs
3 2
1 Cash required for purchasing Rs 2700
SP of of the stock = Rs 2500 stock
3
221× 2700
∴ Total profit = SP – CP = Rs = Rs 2983.50.
= 4500 – 1500 2 × 100
= Rs 3,000. 26. Ans. (d)
1
21. Ans. (d) Sol. On investing Rs 96 + , income
4
Sol. Let Rs x be the investment in 5% stock 11
⇒ 3400 – x is the investment in 6% stock = Rs
2
344 ✫ Quantitative Aptitude

7 1
On investing Rs 100, income Cost price of new stock = 74 + = Rs 75
8 8
11× 4 ×100 63
= Rs ∴ New stock = 100 × = Rs 84
2 × 385 75
40 5 Income from new stock
= Rs = Rs 5 5 × 84 21
7 7 = = Rs
5 100 5
∴ Rate = 5 %. ∴ Difference of income
7
21 1 7
27. Ans. (b) = – 3 = Rs

TS
5 2 10
Sol. Let the investment in each case be Rs 7
(340 × 90) If Rs is the increase in income, then
10
Income from 4% stock value of first stock = Rs 100

H
4 As 21 is the increase in income, value
= Rs × 340 × 90
340 100 × 10
of first stock = × 21 = Rs 3,000.

G
= Rs 360 7
1 30. Ans. (b)
Income from 2 % stock 100

= Rs
5
2 × 90
2
× 340 × 90
U Sol. Total stock = 250 ×
1
2.5
= Rs 10000
10000
O
of the total stock = = Rs 2000
5 5
= Rs 850 1 80
1 SP of of stock = × 2000 = Rs 1600
H
∴ 2 % stock at 90 is a better option. 5 100
2
By investing Rs 120, stock bought worth
28. Ans. (c)
_T

Rs 100
Sol. For an income of Rs 544, investment By investing Rs 1,600, stock in bought
= Rs 12975 worth
13 100 4000
C

For an income of Rs , investment = ×1600 = Rs


4 120 3
12975 13 1
PS

= Rs × = Rs 77.51 Income from of original stock


544 4 5
∴ For a Rs 100 stock, investment 250
= = Rs 50
= Rs 77.51 5
Hence, market value Increase in income = Rs 50
U

1 Let required dividend be x%


= Rs 77.51 – = Rs 77.
2 x 4000
@

∴ + = 50 + 50
29. Ans. (b) 100 3
Sol. Suppose the man has stock of Rs 100 40
⇒ x× = 100
1 3
∴ First income = Rs 3 100 × 3
2 ⇒ x =
1 1 40
First stock sold for Rs 63 – = Rs 63 ⇒ x = 7.5%.
8 8
UNIT FIVE
✫ MENSURATION: AREA AND VOLUME
✫ CLOCKS ✫ CALENDARS

TS
FUNDAMENTALS AND
FAST-TRACK FORMULAE

H
Mensuration B is denoted by b and the side opposite angle

G
Mensuration is the study of the measurements C is denoted by c. The altitude or height h is the
or calculation of the dimensions of geometric perpendicular dropped from a vertex to the
figures in order to find their areas and volumes.
U base. AD is the perpendicular from A to the base
BC in the given triangle. (The base can be any
O
I. Area and Perimeter of side, provided height is calculated as the length
of a perpendicular to that side from the opposite
Two-Dimensional Figures vertex.)
H
Two-dimensional (or plane) figures are those Any Triangle For any triangle ABC with
figures that have length and breadth but not lengths of the sides a, b, and c, and the altitude
_T

depth. or height h,
Perimeter The perimeter of a two- ● Perimeter (P) = a + b + c
dimensional (plane) figure is the distance around 1
the figure, or the total length of the sides ● Area = base × corresponding height h
C

2
enclosing the figure. Or
Area The area of any figure is the amount
PS

1
of surface within its bounding lines. Area is Area = × any side × length of
2
expressed in square units. perpendicular dropped on that
Triangle side from the opposite vertex
U

● Hero’s Formula This formula for the area


A triangle is a plane closed figure bounded by
three straight lines. of a triangle says
@

A Area = s ( s − a ) ( s − b) ( s − c )
where a, b and c are the lengths of the
sides and s is the semiperimeter, i.e.,
b 1
c h s = (a + b + c)
2
[It follows that the perimeter is 2s.]
● The radius of the incircle of a triangle of

B C A
area A and semiperimeter s =
D a s
ABC is a triangle with sides AB, BC and AC, all Equilateral Triangle An equilateral triangle
unequal in length. The side opposite angle A is has all its sides equal and it has all its angles
BC and is denoted by a, the side opposite angle equal, each one being 60°.

345
346 ✫ Quantitative Aptitude

● Hypotenuse c2 = a2 + b2
(Pythagoras Theorem)
c=a b=a a 2 + b2
c =
h or hypotenuse = (base)2 + (height)2
● If the base is b, the height is the
perpendicular (side a)
a 1 1
In ABC, an equilateral triangle, a = b = c (or Area = ba or the product of the
2 2

TS
all equal to a) and ∠A = ∠B = ∠C = 60°. sides containing the right angle.
For an equilateral triangle,
● Perimeter (P) = 3a

H
3 3 c
● Altitude (h) = side = a
2 2 a

G
3 3 2
● Area = side2 = a

● Area =
4
h2
4

U b
O
3 ● Perimeter = a + b + c.
Isosceles Triangle An isosceles triangle has Isosceles Right-triangle In this case, the
two of its sides equal. The altitude from the
H
two sides enclosing the right angles are equal.
vertex bisects the base.
_T

a c=a c
a
C

h
PS

b (base) b (= a)
b/2 b/2 ● Hypotenuse c = a 2
U

1 2 2
For an isosceles triangle,
● Area = a or a
● Perimeter (P) = (2a + b) 2 2
@

1 ● Perimeter = 2a + c = 2a + a 2
● Height (h) = 4 a2 − b 2
2 = a 2 ( 2 + 1)
b = hypotenuse c ( 2 + 1)
● Area = 4 a2 − b 2
4
● If P is the perimeter of an isosceles
Right Triangle A triangle in which one triangle with base b, and a is one of the
angle is a right angle (90°) is called a right- equal sides,
angled triangle or right triangle. The side opposite P−b
to the right angle is called the hypotenuse which a=
2
is also the longest of the three sides. ● If a is one of the equal sides, and b the
If the right-angled triangle has c as the base, and P the perimeter,
hypotenuse, then b = P – 2a
Unit Five : Fundamentals / Mensuration ✫ 347

Quadrilaterals
Points to Note A quadrilateral is a closed figure bounded by
(i) The sum of the angles of a triangle is 180°. four straight lines. It has four angles within it
(ii) The sum of any two sides of a triangle is and their sum is 360°. The diagonal is the
greater than the third side. straight line joining two opposite vertices. A
(iii) The line joining the vertex of a triangle to quadrilateral has two diagonals.
the mid-point of its opposite side is called Let ABCD be any quadrilateral and p1 and
the median. The three medians of a p2 perpendiculars to one of the diagonals, AC
triangle meet at the centroid; the centroid or d, from the opposite vertices. (These

TS
divides each median in the ratio 2 : 1. perpendiculars are called offsets.)
(iv) The median of a triangle divides the
triangle into two triangles having the D C
same area. p1

H
(v) The triangle formed by joining the mid- p2
points of a given triangle will have an

G
area equal to one-fourth the area of the d
given triangle.
A B
A

U ● Perimeter = Sum of the 4 sides


O
1
D F ● Area = d (p1 + p2)
2
H
● If the lengths of the four sides and one
B C diagonal are known,
E 1 Area = Area of Δ ABC + Area of Δ ADC.
_T

Area of Δ DEF = Area of Δ ABC


4
(vi) For any triangle, if sides are a, b and c, Point to Note
and radius of inscribed circle is r and If the diagonal falls outside the figure as
C

radius of circumcircle is R, then shown below, for example, in the concave


2 × area of Δ quadrilateral:
PS

r= C
perimeter of Δ
ab c
R=
4 × area of Δ p2
U

a c p1
a c
@

d
b D
b A B
inscribed circle circumscribed circle
Here the diagonal AC or d lies outside the
or incircle or circumcircle
figure. 1
(vii) For an equilateral triangle of sides a,
inscribed radius r and circumscribed Area of ABCD = d (p1 – p2).
2
radius R,
For a cyclic quadrilateral,
a
(a) r =
2 3 ● Area = s ( s − a) ( s − b) ( s − c) ( s − d )
a
(b) R = a+b+c+d
3 where s = and a, b, c and
(c) R = 2r 2
d are the adjacent sides.
348 ✫ Quantitative Aptitude

[Incidentally, this area formula is known and each of the four angles is 90°. The diagonals
as Brahmagupta’s formula after the bisect each other and are equal.
Indian mathematician of the seventh ABCD is a rectangle with the length AB =
century, who gave the formula.] DC = l and the breadth BC = AD = b. The length
of the diagonal AC = BD = d and the two
● The sum of the angles = 2π. diagonals bisect each other.
(i) Parallelogram Also all angles A, B, C and D are equal to
A parallelogram is a quadrilateral in which 90° each.
l
the opposite sides are equal and parallel. The D C
opposite angles of a parallelogram are equal.

TS
The diagonals of a parallelogram bisect each d
b b
other. Each diagonal divides the parallelogram
into two triangles having the same area.

H
In the parallelogram ABCD shown,
AB = DC = base b A B
l
AD = BC = a

G
● Diagonal = l 2 + b2
Diagonal is AC = d
D ● Perimeter = 2(l + b)
C

a
P
d U If one side (l) and diagonal (d) are given,
Perimeter = 2(l + d 2 − l2 )
O
h ● Area = l × b
Or
P If one side and diagonal are given,
H
A B Area = l × d2 − l 2 or b × d 2 − b 2
base (b)
(as d2 = l2 + b2)
_T

● Perimeter = 2 (a + b) b2 = d2 – l2, and l2 = d2 – b2


where a and b are adjacent sides of the Or
parallelogram. If perimeter (P) and diagonal (d) are given,
C

● Area = base × height = (b × h)


P2 d2
Or Area = −
Area = 2 s(s − a) (s − b) (s − d) 8 2
PS

● If the area (A) and perimeter (P) are


where a and b are adjacent sides, d is known, then the two values of x in the
a+b+d following correlation will give the length
the diagonal and s =
2 and breadth of the rectangle:
U

Or
P
Area = any diagonal × its offset A= x – x2
2
@

D C
Points to Note
a p1 d2 (i) If a parallelogram and rectangle be on the
d1 same base and between the same parallels,
p2
they have equal areas.
A B
A B E
b
Here, area = d1 × p1 or d2 × p2
● The sum of the squares of the diagonals
D C
d12 + d22 = 2(a2 + b2).
Area of rectangle ABCD
(ii) Rectangle = area of parallelograms BDCE
A rectangle is a quadrilateral whose opposite (ii) Among parallelograms of given sides,
sides are equal and parallel (like in a parallelogram) the rectangle has the greatest area.
Unit Five : Fundamentals / Mensuration ✫ 349

(iii) Rhombus ● Length of d = 2 × a


A rhombus is a quadrilateral with all sides or 2 × area
equal and diagonals bisecting each other at 90°. P
The diagonals are not equal. or
2 2
In the figure of the rhombus ABCD below,
AB = BC = CD = DA = a; diagonals AC and BD Points to Note
bisect each other at 90°. AC = d1 and BD = d2 (AC
≠ BD); the perpendicular distance between one a a
side and the opposite vertex is the height h. r

TS
R
D a C

d1
For any square of side a, if r is the radius

H
a of the inscribed circle and R is the radius
h
a of the circumcircle.

G
d2 a
r =
2

A a B
U R =
diagonal
2
=
a
2
O
● Perimeter = 4a
or 2 d12 + d2 2 (v) Trapezium
H
A trapezium or trapezoid is a quadrilateral
1 2 2 which has two opposite sides parallel. The
● Side = d1 + d2
2
_T

height of the trapezium is the distance between


● Area = a × h
1 the parallel sides.
or (d1 × d2) In the trapezium ABCD, the lengths of the
2
parallel sides are a and b and h is the height
C

(iv) Square (perpendicular distance between the two parallel


A square is a quadrilateral with all sides sides) and the lengths of the non-parallel sides
PS

equal and all four angles equal to 90° each. Its


(slant sides) are c and d.
diagonals are equal and bisect each other at 90°.
In the figure of a square ABCD, AB = BC = CD a C
D
= DA = a; diagonals AC = BD = d and they bisect
U

d
each other at 90°. The height = side. c h
a C
@

D
A B
b
1
a d d a ● Area = h (a + b)
2
If all four sides are known but h is not
known, h may be calculated as
A B
a 2
● Perimeter (P) = 4 a h = s (s − c ) ( s − d ) ( s − x )
x
or 2d 2
● Area = side2 = a2 (where x = difference in length between
d2 the parallel sides: i.e., x = a – b if a > b
or and x = b – a if b is greater than a; and
2
c+d+x
Perimeter (P )2 s = .)
or 2
16
350 ✫ Quantitative Aptitude

Regular Polygon Points to Note


A polygon is a figure bounded by four or more
straight lines. A regular polygon is a polygon in Area of a regular polygon r
(i) =
which all the sides and angles are equal. perimeter of a regular polygon 2
D (ii) Area of a regular hexagon (six-sided
a
θ C 3 3
a regular polygon) = × (side)2
θ 2
E θ a (iii) Area of a regular octagon (eight-sided

TS
regular polygon) = 2( 2 + 1) (side)2
θ
a
θ B
a Circle
ext.

H
angle A A circle is a plane figure bounded by one line
such that the distance of this line from a fixed
If a is the length of a side, and θ the measure point within it remains constant throughout.

G
of an angle, O is the centre of the circle and r (radius) is the
● Sum of exterior angles = 2π constant distance from O to the circumference,
● Sum of interior angles = (n–2)π

(n being the number of sides) U and d is the diameter.


O
n (n − 3) r
● Number of diagonals =
2
O
H
● Perimeter (P) = n × a
n−2 d
● Each interior angle θ = ×π
_T

n

● Each exterior angle =
n
The perimeter is known as the circumference
1 1
● Area =
C

P r or n a r in the case of a circle.


2 2 ● Circumference (or perimeter of circle),
(where r is the radius of the inscribed
c = 2π r or π d
PS

circle or circle drawn inside the polygon


touching its sides) (as d = 2r)
● Area = πr
2
2
na a
Area = R2 − πd 2
U

2 2 or
4
(where R is the length of the radius of the
c2
@

circumcircle of the polygon or circle on or


which all the vertices of the polygon lie— 4π
i.e., the polygon is inscribed in a circle of 1
or c × r
radius R.) 2
D Semicircle A figure enclosed by the diameter
a and the part of the circumference cut off by it
a is a semicircle.
C π r2
E ● Area of a semicircle =
2
● Perimeter of a semicircle
R a
a r 2π r
= + 2r = πr + 2r
2
a B = πr + diameter
A
Unit Five : Fundamentals / Mensuration ✫ 351

● Let r be the radius and l the length of


the arc and θ the measure of ∠AOB at
the centre. Then
O θ
r r l= × 2πr
360
area c (The angle at the centre of a circle is 360°.)
● Radius r = or
π 2π 1
(where c is the circumference of the circle) Area of the sector AOB = lr
2
θ 2
× area of circle = πr θ

TS
Points to Note or
360 360
Angles can be measured in radians. If an arc Segments A segment of a circle is bounded
AB of length l subtends an angle at the by a chord and the corresponding arc. A straight

H
centre O of a circle with radius r, then the line joining any two points on the circumference
l of a circle is a chord. (The diameter is the
angle is radians. greatest chord of a circle and divides the circle

G
r
B into two equal halves, i.e., semicircles. All other
l chords are shorter than the diameter.)

A
U Y
O
r O
O r r
H
A B
X
_T

The angle measures 1 radian when the AXB is the minor segment.
length of the arc AB in the above circle AYB is the major segment.
equals the length of OA. This happens when Area of circle = Area of (AXB + AYB)
∠AOB is about 57°. Since the length of the
C

Area of segment = Area of sector


circumference of a circle with radius r is 2πr, – Area of triangle AOB
one complete revolution (360°) is the
PS

circumference divided by the radius, i.e.,


2 πr Fast-Track Formulae and
, or 2π radians, 180° = π radians.
r Worked Examples
Consequently, an angle x subtended by an
U

πx (i) If the length of a rectangle is increased


arc at the centre = radians.
180 by x% and the same area were to be
@

So to convert an angle in degrees into maintained, the breadth would have to be


π
radians, multiply by . 100 x
180 decreased by %.
100 + x
Sectors A sector is the figure enclosed by
two radii and an arc between those radii. (An ● The length of a rectangle is increased by
arc is any part of the circumference; the smaller 60%. By what per cent should its breadth
part is the ‘minor arc’ and the greater part is the be decreased so as to maintain the same
‘major arc’.) area?
A If x is the percentage increase in area,
r the breadth will be decreased by
l 100 x
θ
O

O %
100 + x
B Here, x = 60
352 ✫ Quantitative Aptitude

Then, using the formula


100 × 60
% Area= lb sq. units
100 + 60
100 + 10 110 11
6000 New length = l= l= l
= % 100 100 10
160 100 − 10 90 9
= 37.5%. New breadth = b= b= b
100 100 10
(ii) If the length and breadth of a rectangle
11 9
are increased by x% and y% respectively, the New area = l × b sq. units
10 10

TS
area of the rectangle will increase by
xy 99 lb
x+y+ %. = sq. units
100 100
99 lb

H
● The length and breadth of a rectangle Area decreased = lb –
are increased by 25% and 4% 100
respectively. Find the percentage increase 100 lb − 99 lb

G
=
in its area. 100
Increase in the area of rectangle lb
= x+y+
xy
100
%
U =
100
sq. units
lb 1
× × 100
O
where x and y are the respective Percentage decrease =
100 lb
percentage increases in length and = 1%.
H
breadth. (iv) If any defining dimension or side of
Here x = 25 and y = 4 any plane figure (triangle, quadrilateral, circle,
So, percentage increase in area of the
_T

etc.) is changed by x%, its area changes by


given rectangle x
x 2+ %.
25 × 4 100
= 25 + 4 + %
100 ● If the radius of circle is decreased by
C

= (25 + 4 + 1)% 20%, what is the percentage decrease in


its area?
PS

= 30%.
(iii) Suppose there is an x% increase in length Let x be the percentage change in the
and a y% decrease in breadth, the change in area radius.
x
xy ∴ Change in area = x 2 + %
U

will be x − y + %. Here, x = –20 100


100 (-ve sign indicates decrease)
● The length of a rectangle is increased
@

by 10% and its breadth is decreased by 20


–20 2 − %
10%. Find the area of the new rectangle. 100
Here x = 10%, y = –10% 1
So, the change in area = –20 2 − %
5
xy
= x−y+ % 10 − 1
100 = –20 %
5
10 × 10
10 − 10 + % 9
100 = –20 %
100 5
= = 1%. = –36%
100
[The longer method will be as follows: Area of the circle decreases by 36%.
Let the length and breadth be l and b ● The length and the two diagonals of a
units respectively. rectangle are decreased by 4% each.
Unit Five : Fundamentals / Mensuration ✫ 353

What is the percentage decrease in its (xi) If the area of a square is A sq. units,
breadth? the area of a circle of the same perimeter as
Since the length and the two diagonals 4A
that of the square is sq. units.
decrease by 4% each, the breadth also π
must decrease by 4%. ● If the area of a square is 42 sq. cm, find
(v) If the sides of any plane figure are the area of the circle formed by the
changed by x%, its perimeter also changes in same perimeter.
the same way (i.e., increases or decreases) by Let the area of the square be a.
x%. Required area of the circle

TS
(vi) If each side of a quadrilateral is
4A
increased or decreased by x%, its diagonals =
also increase or decrease by x%. π
(vii) If the diagonal of a square increases Here A = 42

H
by x times, the area of the square becomes x2 4 × 42 × 7
times. 22

G
1176
● The diagonal of a square is doubled. = = 53.45 sq cm.
22
How will the area of the new square be The area of the circle is 53.45 cm2.
affected?
The area will increase by the square of U (xii) The area of the largest circle that can
πa2
O
the increase in diagonal length. The be inscribed in a square of side a is .
4
diagonal has increased 2 times. So the (xiii) The area of a square inscribed in a
H
new area of the square will increase 22 circle of radius r will be 2r2.
times or 4 times.
_T

(viii) If the ratio of the area of two squares ● Find the side of a square inscribed in a
be a : b, the ratio of their sides, diagonals, circle whose circumference is 352 cm.
perimeters will be in the ratio a : b . The circumference of the circle
(2πr) = 352
C

● The ratio of the areas of two squares is


352 352 × 7
25 : 16. Find the ratio of their diagonals. ∴ r = 2 π = 2 × 22
PS

We have, a = 25 and b = 16 = 56 cm.


Then, using the formula, the ratio of If the area of the inscribed square is 2r2,
their diagonals the side of the inscribed square
U

= a : b
= 2r 2 = 2 r
We have 25 : 16
@

So, the side of the given square


= 5 : 4.
= 56 2 cm.
The ratio of their diagonals is 5 : 4.
(xiv) A circular wheel of radius r travelling
(ix) The ratio of the area of a circle
d
circumscribing a square to the area a distance d will make revolutions.
2πr
of a circle inscribed in the square ● The diameter of a wheel is 4 cm. If it
r
2 rolls forward covering 14 revolutions,
is .
1 find the distance travelled by it.
(x) The ratio of the area of a 4
Radius of the wheel (r) = = 2 cm
square circumscribing a circle to 2
the area of a square inscribed in We have, diameter or 2r = 4 cm
2 distance (d )
the circle is . Revolutions =
1 2 πr
354 ✫ Quantitative Aptitude

d 800 × 540
14 =
2 πr 16
d = 14 × 2πr
432000
22 =
= 14 × 2 × × 2 16
7 = 27000 cm
= 176 cm.
(xv) If three circles of equal radius r touch = 270 m
one another, then the area included between (b) The amount required for carpeting
4 2 the floor = 270 × 18 = Rs 4860.

TS
them is nearly r. (Note that the cost is given for metre
25
unit while earlier the length and
breadth are given in cm.)
r
(xviii) If the length and breadth of a

H
r r rectangular area are l and b, the least number
of square tiles required to cover the area will

G
l×b
be .
4 2 (H.C.F. of l , b)2
The area of the shaded portion is nearly r.
25
(xvi) If there are four circles, of equal radius U The side of the largest tile if the tiles are
to fit exactly will be
H.C.F. of l and b.
O
r, each circle touching two others, the area
6 2 ● A hall of length 16 m and breadth 10 m
included by the four circles is nearly r.
H
7 is to be paved with square tiles of equal
size. What will be the size of the largest
tile so that the tiles exactly fit? Also find
_T

r r the number of tiles required.

r r We have, l = 16 and b = 10
Side of the largest possible square tile
C

= H.C.F. (l, b)
6 2 H.C.F. (16, 10)
PS

The area of the shaded portion is nearly r. =2m


7
Alternatively, the area of the shaded portion Number of tiles required
is (4 – π) r2. l×b
=
U

(xvii) If the length and breadth of a (H.C.F. of l , b)2


rectangular area are l and b, and a carpet of 16 × 10
= 40 tiles.
@

width w is used to cover the area, the carpet 2×2


l×b (xix) (a) If a path of w units surrounds a
length required will be .
w rectangular area of length l units and breadth
● (a) How many metres of a carpet 16 cm of b units, the area of the path
wide will be required to cover the = 2w (l + b + 2w) sq. units
floor of a room which is 800 cm long (Measurements must be in the same unit).
and 540 cm broad?
w path
(b) Calculate the amount required for
carpeting the floor if the cost of the
carpet is Rs 18 per metre. rectangular
b area
(a) We have, l = 800, b = 540, W = 16
l
l×b
Length of the carpet =
W
Unit Five : Fundamentals / Mensuration ✫ 355

● A rectangular park 16 m × 10 m is And the area of the plot minus the paths
surrounded by a path 2 m wide. Find will be (l – w) (b – w) sq. units
the area of the path. [In case of the plot being square, l is taken as
equal to b in all the derivations in (xix) and (xx).]
We have, l = 16, b = 10 and w = 2
The area of the path surrounding the l
park
= 2w (l + b + 2w)
2 × 2 (16 + 10 + 2 × 2)
= 4 (16 + 10 + 4) b w

TS
= 4 (30) = 120 sq m.
(b) Suppose a path of w width is made
inside a given rectangular area of length l and

H
breadth b.
path of width w
(xxi) (a) If a path of width w is made around

G
outside a circular area of radius r, the area of
rectangular the path = π × w (2r + w) sq. units.
b area
U w
O
circular
w area
H
l r
O
Area of path (shaded portion)
_T

= 2w (l + b – 2w) sq. units


(Measurement must be in the same units.) path
(shaded area)
● A park of dimensions 112 m by 78 m has
C

(b) If a circular path of width w is made


a paved walk 2.5 m wide all round it inside a circular area of radius r, the area of the
inside. What is the area of the walk? circular path = π × w (2r – w).
PS

Here length l = 112 m


w
breadth b = 78 m
The path is inside and its width (w) = 2.5 m circular
U

Area of the walk = 2w (l + b – 2w) area


2 × 2.5 (112 + 78 – 2 × 2.5)
r
@

= 5 (190 – 5)
= 925 sq. m.
[Note: In problems involving pathways
around rectangles, it is important to path
determine if the length and breadth given (shaded area)
are those of the outer or inner rectangle (c) If r and R are the respective radii of the
before applying the relevant formula.] inner and outer circles which bound a plane
circular (or ring) path of width w, then
(xx) If two parallel paths, each of width w,
R– r= w
are laid in a rectangular plot, one parallel to the
Area of the circular path = πR2 – πr2
length l and the other parallel to the breadth b
= π (R2 – r2)
of the rectangle, the area of the two paths
= π (R + r) (R – r)
(shaded area in figure) will be
= π w (R + r) (  w = R – r).
w (l + b – w) sq. units
356 ✫ Quantitative Aptitude

Miscellaneous Worked Examples (The ratio of areas of a circumcircle and an


● The area of a circle is increased by 22 cm2 incircle of a square is always 2 : 1. So the radii
when its radius is increased by 1 cm. Find will also be in the same ratio, i.e., 2 : 1).
the original radius of the circle.
● The perimeter of a rhombus is 146 cm and one
Let the original radius be R cm. of its diagonals is 55 cm. What is the other
The area will be πr2 diagonal?
Increased radius = (R + 1) cm. The new circle’s We know that the
area will be π (R +1)2 perimeter of a rhombus = 2 d12 + d2 2
As the difference between the areas of the (where d1 and d2 are the diagonals)

TS
two circles is given to be 22 cm2,
So, 146 = 2 (55) + d2
2 2

22 = π (R + 1)2 – πR2
22 = π [(R + 1)2 – R2] 73 = (55)2 + d2 2
22

H
22 = [(R + 1)2 – R2] Squaring both sides,
7 (73)2 = (55)2 + d2 2
7
22 × = R2 + 2R + 1 – R2 3025 + d2 2

G
22 5329 =
22 × 7 d2 2 = 5329 – 3025
= [(2R + 1) (1)] d2 2 = 2304
22
7 = 2R + 1
2R = 7 – 1 U d2 = 2304
O
d2 = 48 cm
2R = 6 Hence, the other diagonal is 48 cm.
R = 3 cm.
H
● A square is of area 200 sq m. A new square
● The length of the side of a square is 14 is formed in such a way that the length
of its diagonal is 2 times the diagonal
_T

cm. Find out the ratio of the radii of the


inscribed and circumscribed circles of the of the given square. Find the area of the
square. new square.
2 × side We know that
C

We have, side = 14 cm, diagonal =


Area of a square = (side)2
A E
B
PS

Side of the first square = area


= 200
O = 10 2 m
U

Square of the diagonal = 2 × side


= 2 × 10 2
@

D C
= 20 m
Then,
Diagonal = Diagonal of the new square = 2 × side
2 × 14
AB 14 = 2 × 20
Radius of incircle = OE = = = 7 cm
2 2 = 20 2 m
Diagonal BD Area of the new square
Radius of circumcircle =
2 1
2 × 14 = × (diagonal)2
= 2
2 1
= × (20 2 )2
= 7 2 cm 2
∴ Required ratio = 7 : 7 2 1
= × 800
2
=1: 2. = 400 sq. m.
Unit Five : Fundamentals / Mensuration ✫ 357

● The sides of a parallelogram are in the ratio = 1920


5 : 4. Its area is 1000 sq units. Altitude on
the greater side is 20 units. Find the = 64 × 30
altitude on the smaller side.
= 8 30 sq cm.
Let the sides of the parallelogram be 5x
and 4x. II. Volume and Surface Area of
Area of parallelogram = base × height
1000 = 5x × 20 Three-Dimensional Figures
Volume is the extent of space occupied by
1000

TS
x = a solid (a three-dimensional shape or object) or
5 × 20 bounded by a closed surface. It is measured in
x = 10 cubic units of length: cm3, m3, and so on. Surface
One side = 5x = 5 × 10 = 50 units area is the area of the plane surfaces (the faces)

H
The other side = 4x = 4 × 10 = 40 units that bound the solid or given space.
∴ Taking the smaller side,
40 × h = 1000

G
1. Rectangular Solid (Cuboid)
1000 A rectangular solid has six rectangular faces
h =
and equal opposite rectangles. It is also called a
40
h = 25 units.
U rectangular parallelopiped. The figure below is
that of a rectangular solid.
O
● What is the area of a triangle having
perimeter 32 cm, one side 11 cm and the G F
difference of other two sides 5 cm?
H
h
We have, B C
Perimeter = 32 cm and one side = 11 cm H A3
_T

E
Let the sides of the triangle be a, b and
c respectively. A1 b
A
Perimeter (2s) = a + b + c l D
C

32 = 11 + b + c
The diagonals are BE, AF, CH and GD
b + c = 32 – 11
l, b and h are the length, breadth and height
PS

b + c = 21 . . . (i)
of the figure.
Given, b – c = 5 . . . (ii)
A1 = area of the base/top = lb
Adding equation (i) and (ii), we get
A2 = area of one side face = lh
b + c = 21
U

A3 = area of other side face = bh


b – c = 5
(i) Any diagonal = l2 + b 2 + h2
2b = 26
@

b = 13 (ii) Total surface area = 2(lb + bh + lh)


Putting the value of b in equation (i), we or 2(A1 + A2 + A3)
get (iii) Volume = l × b × h
13 + c = 21
or A1 × A2 × A3
c = 8
Now, a = 11, b = 13, c = 8 ● Find the volume and the total surface area
Now, 2s = 32 of a cuboid whose dimensions are 22 m,
∴ s = 16 8 m and 3 m.
Area of triangle We have, l = 22 m, b = 8 m and h = 3 m
= s ( s − a ) ( s − b ) (s − c ) Then, using the formula
Volume of the cuboid = l × b × h
16 (16 − 11) (16 − 13) (16 − 8) = 22 × 8 × 3
= 528 m3
= 16 × 5 × 3 × 8
358 ✫ Quantitative Aptitude

∴ Total surface area of the cuboid Total surface area = 6 (edge)2


= 2 (lb + bh + lh) 6 (6)2
= 2 (22 × 8 + 8 × 3 + 22 × 3) = 216 sq cm.
= 2 (176 + 24 + 66) Volume of cube = (edge)3
= 2 (266) (6)3
= 532 m2.
= 216 cm3.
2. Cube ● A cube of edge 15 cm is immersed
A cube is a three-dimensional figure in completely in a rectangular vessel

TS
which each of the six faces is a square. So all faces containing water. If the dimensions of the
are equal. The length, breadth and height of a base of vessel are 20 cm × 15 cm, find the
cube are equal and are called the edges of the rise in water level.
cube. A cube has 12 edges.

H
The water level will increase in the same
G F volume as the volume of the cube, which
a is

G
(15 × 15 × 15) cm3
B a Volume
C

U Rise in water level =


Area
O
15 × 15 × 15 45
H E = =
20 × 15 10
H
= 11.5 cm
a
_T

A D 3. Prism
The diagonals are BE, AF, CH and GD. A prism is a three-dimensional solid with
The length, breadth and height are all same = a two congruent top and bottom faces, and any
number of other faces that are all rectangles.
C

(i) Diagonal = a 3
(ii) Total surface area = 6 (edge)2 = 6a2
or 2 × (diagonal)2
PS

3
diagonal
(iii) Volume = a3 or
3
3
U

surface area
or
6
@

(iv) For two cubes:


(a) Ratio of volumes = (ratio of sides)3
(b) Ratio of surface areas = (ratio of sides)2
(c) Ratio of volumes = Ratio of surface areas
● The diagonal of a cube is 6 3 cm. Find its
total surface area and volume.
(i) In a prism of n sides, number of vertices
Here, diagonal of the cube = 6 3 cm
= 2n and number of faces = n +2
Diagonal of cube = 3 (edge) and
(ii) Lateral surface area
Diagonal of cube = perimeter of base (or top) × height
Edge of cube =
3 (iii) Total surface area
6 3 = 2 × base area + lateral surface area
= 6 cm
3 (iv) Volume = area of base × height
Unit Five : Fundamentals / Mensuration ✫ 359

● Find the volume and the total surface area (b) Ratio of volumes = Ratio of curved
of a triangular prism whose height is 20 m surface areas
and the sides of whose base are 15 m, 12 m
(c) Ratio of curved surface area
and 9 m, respectively.
= Ratio of heights
Perimeter of base of prism = 15 + 12 + 9 = 36 m —when heights are equal
Height of prism = 20 m
(d) Ratio of volumes = (Ratio of radii)2
In the case of a triangle (here base)
(e) Ratio of volumes = (Ratio of curved
Sum of sides 15 + 12 + 9 surface areas)2
s= =

TS
2 2 (f) Radii of curved surface areas
36 = Ratio of radii
= = 18 m
2 —when volumes are equal

H
Area of base (a triangle) (g) Ratio of radii
= s ( s − a ) ( s − b) ( s − c ) = Inverse ratio of heights

G
18 (18 − 15) (18 − 12) (18 − 9) (h) Ratio of curved surface areas
= Inverse ratio of radii
= 18 (3) (6) (9)
= 2916 = 54 sq. cm. U (i) Ratio of curved surface areas
O
= Ratio of heights
Volume of the prism
= area of base × height —when curved surface areas are equal
H
54 × 20 (j) Ratio of radii
= 1080 m3 = Inverse ratio of heights
_T

Surface area of the prism (k) Ratio of volumes


= 2 × base area + lateral surface area = Inverse ratio of heights
= 2 × base area + (perimeter of base
(l) Ratio of volumes = Ratio of radii.
× height)
C

2 × 54 + (36 × 20) (v) For two right circular cylinders,


= 108 + 720 Ratio of curved surface areas
PS

= 828 sq. m. = (ratio of radii) (ratio of heights)


(vi) For two right circular cylinders,
4. Cylinder Ratio of radii
U

A cylinder (a right circular one) is bound by = (ratio of curved surface areas)


two congruent circular ends with one rectangular × (inverse ratio of heights)
face wrapped around these ends.
@

(i) Area of curved surface (vii) For two right circular cylinders,
= circumference of base × Ratio of heights
height = 2πr × h = (ratio of curved surfaces areas)
(ii) Area of total surface × (inverse ratio of radii)
= area of curved surface
axis

+ area of circular ends (viii) For a right hollow cylinder, where R is


= 2πrh + 2πr2 the outer radius and r, the inner radius,
= 2πr (h + r) sq. units (a) Volume of the material of which the
(iii) Volume = area of base cylinder is made = π (R2 – r2) h
× height (b) Curved surface of cylinder
= πr2h cubic units = 2πh(R + r)
(iv) For two cylinders,
(c) Whole surface of cylinder
—when radii are equal
(a) Ratio of volumes = Ratio of heights = 2πh (R + r) + 2π (R2 – r2)
360 ✫ Quantitative Aptitude

● A cylinder of height 19 cm has a base of and 4 : 7, respectively. Find the ratio of their
radius 3 cm. Find the total surface area of radii.
the cylinder.
The ratio of curved surface areas = 4 : 7 and
We have, h = 19 cm, r = 3 cm ratio of radii = 2 : 3
Total surface area = 2π r (h + r) Ratio of radii = (ratio of curved surface
22 areas) × (inverse ratio of heights)
=2× × 3 × (19 +3)
7 1 1
(4 : 7) :
22 2 3
=2× × 3 × 22

TS
7 = (4 : 7) ( 3 : 2)
2904 6 = 12 : 14 or 6 : 7.
= = 414 sq cm.
7 7 ● The radii of two right circular cylinders are
● Two circular cylinders of equal volume have

H
in the ratio of 4 : 5 and their curved surface
their heights in the ratio 4 : 9. Find the ratio areas are in the ratio of 6 : 7. Find the ratio
of their radii. of their heights.

G
Ratio of radii = Inverse ratio of heights
The ratio of radii = 4 : 5 and the ratio of
9:4 curved surface areas = 6 : 7
= 3 : 2.
U Ratio of heights
= (ratio of curved surface areas)
O
● Two circular cylinders of equal volume have
× (inverse ratio of radii)
their heights in the ratio 9 : 16. Find the ratio
of their curved surface areas. 1 1
= (6 : 7) :
H
4 5
Ratio of heights = 9 : 16 = (6 : 7) (5 : 4)
Ratio of curved surface area
_T

= 30 : 28
= Ratio of heights = 15 : 14.
9 : 16 5. Cone
C

= 3 : 4. A right circular cone is obtained by rotating


a right-angled triangle around its height.
● Two circular cylinders of equal curved surface Given r is the radius of the base, h is the
PS

areas have their heights in the ratio of 3 : 5. height and l is the slant height,
Find the ratio of their volumes.
(i) l = h2 + r 2
Ratio of volumes
(ii) Area of curved surface
U

= Inverse ratio of heights


= πrl sq. units
1 1
: = πr h 2 + r 2 sq. units
@

3 5
= 5 : 3. (iii) Total surface area = area
of base + area of
● The heights and radii of two right circular curved surface
cylinders are in the ratio 3 : 4 and 5 : 6, = πr2 + πrl
respectively. Find the ratio of their curved
surface areas. = πr (r + l) sq. units
1 2
Ratio of curved surface areas (iv) Volume = πr h cubic
3
= (ratio of radii) (ratio of heights) units
(3 : 4) (5 : 6)
= 15 : 24 (v) For two cones
= 5 : 8. (a) When volumes are equal
Ratio of radii
● The heights and curved surface areas of two
right circular cylinders are in the ratio 2 : 3 = Inverse ratio of heights
Unit Five : Fundamentals / Mensuration ✫ 361

(b) When radii are equal 1 2


Ratio of volumes = Ratio of heights Volume of a cone = πr h
3
(c) When heights are equal
1 22
Ratio of volumes = (Ratio of radii)2 1056 = × × (6)2 × h
(d) When curved surface areas are equal 3 7
Ratio of radii 1056 × 3 × 7
h =
= Inverse ratio of slant heights 22 × 6 × 6
(vi) For two right circular cones (or cylinders), h = 28 cm
Ratio of radii Slant height l is given by the relation

TS
= (Ratio of volumes) (Inverse ratio of heights)
l = h2 + r2
(28)2 + (6)2
(vii) For two right circular cones (or cylinders),

H
Ratio of volumes = 784 + 36
= (Ratio of radii)2 × (Ratio of heights)
= 820

G
(viii) For two right circular cones,
= 28.64 cm
Ratio of heights = (Inverse ratio of radii)2
× (Ratio of volumes) Slant height of the cone is 28.64 cm.

(ix) A cone with a portion of its top cut off is


called the frustum of the original cone.

U The height of a cone is 6 cm and the radius
of its base is 8 cm. Find its total surface area.
O
Given that R is radius of the base of the Here, r = 8 cm and h = 6 cm
frustum, r is the radius of the top of the
H
frustum, h is height and l is slant height, Slant height (l) = r 2 + h2
64 + 36
_T

= 100 = 10 cm
Total surface area = πr (l + r)
22
C

× 8 (10 + 8)
7
22
PS

= × 8 × 18
7
3168
= = 452.57 sq. cm.
7
U

● Two right circular cones of equal volumes


have their heights in the ratio of 4 : 9. Find
the ratio of their radii.
@

(a) l = h 2 + ( R − r )2 units Ratio of radii


(b) Area of curved surface of frustum = Inverse ratio of heights
= π (R + r) l sq. units 1 1
(c) Total surface area of frustum = :
4 9
= π [( R2 + r 2 ) + l( R + r)] sq. units
(d) Volume of frustum = 9 : 4 = 3 : 2.
πh 2 2
= (R + r + Rr ) cubic units ● Two right circular cones of equal heights
3 have their radii in the ratio of 2 : 5. Find the
● Find the slant height of a cone whose ratio of their volumes.
volume is 1056 cm3 and base radius is 6 cm.
Ratio of volumes = (Ratio of radii)2
Given, volume of cone = 1056 cm3 = (2 : 5)2
and radius (r) = 6 cm = 4 : 25.
362 ✫ Quantitative Aptitude

● The volumes of two cones are in the ratio 6. Pyramid


4 : 3 and their heights are in the ratio 2 : 6. A pyramid is a three-dimensional solid with
Find the ratio of their radii. a regular polygon (of 3 or more sides) as its base
Given, ratio of volumes = 4 : 3 and ratio of and triangles for its other faces meeting at a
heights = 2 : 6 vertex. In a right pyramid, the foot of the
Ratio of radii perpendicular from the vertex is to the midpoint
of the base.
= (Ratio of volumes) (Inverse ratio of heights)
1 1 O vertex

TS
= (4 : 3) :
2 6
= (4 : 3) (6 : 2) edge slant
height (l)

H
= 24 : 6
= 4:1 side

G
= 2 : 1. face

● The heights of two cones are in the ratio of

U
7 : 6 and their radii are in the ratio 3 : 4. Find
the ratio of their volumes.
O
X
Given, ratio of radii = 3 : 4 and
ratio of heights = 7 : 6
base ( b)
H
Ratio of volumes b (base side)
= (ratio of radii)2 × (ratio of heights)
_T

(3 : 4)2 × (7 : 6) (i) Slant surface area


9 7 1
= × = base perimeter × slant height
16 6 2
C

= 21 : 32. 1
= l × b × n (n = number of sides)
2
PS

● The volumes of two cones are in the ratio


of 2 : 3 and their diameters are in the ratio (ii) Whole surface area = base area + total
of 5 : 6. Find the ratio of their heights. slant surface area (base area depends on
shape of base polygon)
U

Given, ratio of volumes = 2 : 3 and


1
ratio of diameters = 5 : 6. (iii) Volume = base area × h
3
@

1 1
The inverse ratio of diameter = : (OX is the altitude = h)
5 6
Ratio of heights
= (inverse ratio of diameter) 2 7. Sphere
× (ratio of volumes) A sphere is a solid contained by one curved
surface, such that all points on it are equidistant
2
1 1 from a fixed point within it,
: × (2 : 3)
5 6 namely the centre. It is
formed by revolving a
= (6 : 5)2 × (2 : 3)
semicircle on its diameter.
36 2 The midpoint of the
= × r
25 3 diameter is called the centre
24 of the sphere, and the radius
= = 24 : 25. of the semicircle is the radius
25
of the sphere.
Unit Five : Fundamentals / Mensuration ✫ 363

(i) Surface area of sphere = 4πr2 4 22


4 3 × × 7 × 7 × 7
(ii) Volume = πr 3 7
3
4312
(iii) A zone of a sphere is the part between =
3
two parallel planes. If the sphere has = 1437.33 cm3
radius r, and the distance between the
planes is h, the area of the curved ● The radii of two spheres are in the ratio of
3 : 4. What is the ratio of their surface areas?
surface of the zone is 2πrh.

TS
Given, ratio of radii = 3 : 4
Ratio of surface areas = (ratio of radii)2
(3 : 4)2
h
= 9 : 16.

H
● The radii of two spheres are in the ratio of
r
3 : 4. Find the ratio of their volumes.

G
Given, ratio of radii = 3 : 4
Ratio of volumes = (Ratio of radii)3
(iv) For two spheres
(a) (Ratio of radii)2 = Ratio of surface areas U (3 : 4)3
= 27 : 64.
O
(b) (Ratio of radii)3 = Ratio of volumes ● The surface areas of two spheres are in the
(c) (Ratio of surface areas)3 = (Ratio of ratio 4 : 5. Find the ratio of their volumes.
H
volumes)2 Given, ratio of surface areas = 4 : 5
(Ratio of surface areas)3
_T

(v) A hemisphere is a half-sphere: part of a


sphere cut off by a plane through its centre. = (Ratio of volumes)2
(4 : 5)3 = (Ratio of volumes)2
It is thus a zone of one base with an altitude
64 : 125 = (Ratio of volumes)2
C

equal to the sphere’s radius.


Ratio of volumes = 64 : 125
If r is the radius of the sphere,
=8:5 5.
PS

2
(a) Volume of hemisphere = π r3cubic units. ● Find the volume, curved surface area and
3 total surface area of a hemisphere of radius
(b) Area of curved surface = 2πr2 sq. units of
7 cm.
U

hemisphere.
(c) Total surface area of hemisphere Given, radius (r) = 7 cm
@

= 3πr2 sq. units. 2


Volume of hemisphere = πr3
3
● The diameter of a sphere is 14 cm. Find its 2 22
surface area and volume. × ×7×7×7
3 7
Given, diameter of the sphere = 14 cm = 718.67 cm3
14 Curved surface area = 2πr2
∴ Radius of the sphere = = 7 cm
2 22
Surface area = 4πr2 2×
7
×7×7
22 = 308 sq. cm.
4 × × 7 × 7
7 Total surface area = 3πr2
= 616 sq. cm.
22
4 3 3× ×7×7
Volume of sphere = πr 7
3 = 462 sq. cm.
364 ✫ Quantitative Aptitude

Fast-Track Formulae and 224 512


= 28 + + %
Worked Examples 100 10000
= 28 + 2.24 + 0.0512
(i) If all three measuring dimensions of
= 30.2912
a sphere, cuboid, cube, cylinder or cone are
increased or decreased by a%, b% and c% ≈ 30.3%.
respectively, then the volume of the figure The percentage increase in the volume
will increase or decrease by of the cuboid is 30.3 per cent.
● Each side of a cube is decreased by

TS
ab + bc + ca abc 20%. Find the percentage change in
a+b+c+ + %
100 (100)2 the volume of the cube.
For a cuboid, the three measuring We have a= b = c

H
dimensions are length, breadth and height. a = –20 (decrease of 20%)
Then, using the formula
For a cube, all three measuring dimen- percentage change in volume

G
sions are equal,
3a2 a3
that is a = b = c. = 3a + +
100 (100)2 %
For a cylinder or a cone two measuring
dimensions are equal to the radius and the U 3(−20)2 (−20)3
O
third measuring dimension is the height, 3( −20) + + %
100 (100)2
that is, a = b = r and c = h.
H
For sphere all three measuring 1200 8000
= −60 + − %
dimensions are equal and is given by the 100 10000
_T

radius. That is a = b = c = r. = [–60 + 12 – 0.8]%


Note: In case of percentage increase, = –48.8%.
values of a, b and c are positive and in case Negative sign indicates decrease in
volume, that is, there is a decrease in
C

of percentage decrease, values of a, b and c


are negative. volume of the cube by 48.8%.
PS

● The length, breadth and height of a ● The diameter of a sphere is increased


cuboid are increased by 4%, 8% and by 10%. What is the percentage
16%, respectively. Find the percentage increase in its volume?
increase in its volume.
U

We have a= b = c
We have, a = 4, b = 8 and c = 16 a = 10
∴ percentage increase in volume
@

Then, using the formula,


percentage increase in volume 3a2 a3
ab + bc + ca = 3a + +
a+b+c+ +
abc
% 100 (100)2 %
= 100 (100)2
3(10)2 (10)3
3 × 10 + +
(4 × 8) + (8 × 16) + (16 × 4) 100 (100)2
%
4 + 8 + 16 +
100
300 1000
4 × 8 × 16 = 30 ++ %
+ %
100 10000
(100)2 = [30 + 3 + 0.1]
32 + 128 + 64 512 = 33.1%.
= 28 + + % The volume of the sphere increases by
100 (100)2 33.1 per cent.
Unit Five : Fundamentals / Mensuration ✫ 365

● The radius of a right circular cylinder ● The radius of a right circular cone is
is decreased by 10% but its height is increased by 20% and the slant height
increased by 20%. What is the is decreased by 25%. Find the
percentage change in its volume? percentage change in the curved
surface area of the cone.
We have, a = b = –10 and c = 20
Then, using the formula, We have, a = 20 and b = –25
Percentage change in volume Then, using the formula,
ab + bc + ca abc percentage change in curved surface
= a+b+c+ + % area

TS
100 (100)2
ab
(−10 ×−10) + (−10 × 20) + (20 ×−10) = a+b+ %
−10 −10 + 20 + 100
100 20 × −25

H
−10 × −10 × 20 20 − 25 + %
+ 100
%
(100)2 500

G
= −5 − %
100 − 200 − 200 2000 100
= 0+ + % = (–5 – 5)%
100
= [0 – 3 + 0.2]%
= –2.8%
10000

U = –10%.
The curved surface area decreases by
O
Therefore, the volume decreased by 2.8%. 10%.
● The radius and height of a cylinder are
H
(ii) If the two measuring dimensions
which are included in the surface area of increased by 5% and 10%, respectively.
a sphere, cuboid, cube, cylinder or cone Find the percentage increase in its
_T

are increased or decreased by a% and b%, surface area.


then the surface area of the figure will We have, a = 5 and b = 10
increase or decrease by Then, using the formula,
C

ab
a+b+ % percentage increase in surface area
100
ab
PS

Note: In case of percentage increase a+b+


= %
values of a, b and c are positive and in 100
case of percentage decrease, values of a, 5 × 10
b and c are negative. 5 + 10 + %
100
U

● Each edge of a cube is increased by = (15 + 0.5)%


10%. What is the percentage increase = 15.5%.
@

in its surface area? The surface area of the cylinder


increases by 15.5 per cent.
We have, a = b = 10
Then, using the formula, (iii) (a) If a sphere of radius x is melted
percentage increase in surface area to make a cone of height h, then
ab x3
= a+b+ % Radius of the cone = 2
100 h
10 × 10 (b) If a cone of height h is melted to
10 + 10 + % make a sphere of radius x, then
100
= (20 + 1)% x3
= 21%. Radius of the cone = 2
h
The surface area of the cube increases
by 21 per cent. ● A solid cone of copper of height 4 cm
366 ✫ Quantitative Aptitude

is melted and a solid sphere of radius 3 × 3 × 32 × 3


4 cm is made. What is the diameter of =
4
the base of the cone? = 216.
The number of bullets that can be
We have x = 4, and h = 4
made is 216.
Then, using the formula,
radius of the base of the cone (v) If a sphere of radius R is melted
3 to form smaller spheres each of radius r,
x the number of smaller spheres
=2×
h

TS
Volume of the bigger sphere
=
(4)3 Volume of the smaller sphere
2 × 3
4 R

H
64 r
=2×
4 ● Find the number copper balls of radius

G
= 2 × 16 2 cm each that can be made from a
= 2 × 4 = 8 cm sphere of radius 6 cm.
Diameter of the base of the cone
= 2 × 8 = 16 cm.
U We have, R = 6, and r = 2
Then, using the formula,
O
3
(iv) If a cylinder is melted to form R
Number of copper balls =
spheres each of a radius r, r
H
3
the number of small spheres 6
=
Volume of cylinder 2
_T

=
Volume of 1 sphere = (3)3 = 27.
● How many bullets can be made out of
(vi) If by melting x spheres, each of
a loaded cylinder 32 cm high and 6 cm
C

radius r, a big sphere is made, then


diameter, each bullet being 2 cm in
Radius of the big sphere = r 3
diameter? x.
PS

We have ● If by melting 27 spheres, each of radius


height (h) = 32 cm, 4 cm, a big sphere is made, what will
diameter of the cylinder = 6 cm be the radius of the big sphere?
U

6
radius of the cylinder = = 3 cm We have, r = 4 and x = 27
2
@

Then, using the formula


diameter of the sphere = 2 cm
Radius of the big sphere
2
radius of the sphere = = 1 cm = r 3
x
2
Then, using the formula, = 4 3
27
number of bullets
Volume of cylinder = 4 × 3 33
= = 4 × 3 = 12 cm.
Volume of 1 sphere
π r 2h Solids Inscribed in/ Circumscribing
=
4 3 Other Solids
πr
3 (i) The radius and height of the largest
π × 3 × 3 × 32 possible cone inscribed in a cylinder of radius of
4 r cm and height of h cm are also r and h
× π × 1× 1× 1
3 respectively.
Unit Five : Fundamentals / Mensuration ✫ 367

● Find the volume of the largest possible (iv) The radius of the largest possible
cone inscribed in a cylinder of radius 9 sphere inscribed in a cone of radius r cm and
cm and height 21 cm. slant height equal to the diameter of the base
r
As the radius and height of the cone will = .
be the same as those of the cylinder, 3
the radius of the cone (r) = 9 cm ● Find the surface area of the largest
and height of the cone (h) = 21 cm possible sphere inscribed in a cone of
radius 14 cm and slant height equal to
1
Volume of the cone = πr2h the diameter of the base.
3

TS
1 22 The radius r of the cone = 14 cm
× × 9 × 9 × 21 r
3 7 Radius of the sphere =
= 1782 sq. cm. 3

H
14
(ii) The edge of the largest possible cube = cm
2r 3
inscribed in a sphere of radius r cm = . Surface area of the sphere = 4πr2

G
3
22 14 14
● Find the surface area of the largest 4 × × ×
possible cube inscribed in a sphere of
radius 6 cm.
Here, r = 6 cm 2r U 7
= 4 × 22 × 2 ×
3
14
3
O
3
Edge of the cube = = 821.33 sq. cm.
3
2 × 6 12 (v) If the largest possible sphere inscribed
H
= in a cube of edge a cm, the radius of the sphere
3 3 a
Surface area of the cube = 6 (edge)2 = .
_T

2
2
12 (vi) If the largest possible cube is inscribed
6 in a sphere of radius r, the edge of the cube
3 2r
C

144 = .
= 6 × 3
3 (vii) The edge of the largest possible cube
PS

= 288 sq. cm.


inscribed in a hemisphere of radius r cm
(iii) The radius of the largest possible
2
sphere inscribed in a cylinder of radius r and = r .
height h cm 3
U

= r if h > r ● Find the length of the diagonal of the


h largest possible cube inscribed in a
and if r > h hemisphere of radius 5 2 cm.
@

2
● Find the surface area of the largest The radius of the hemiphere = 5 2 cm
possible sphere that can be inscribed in 2
a cylinder of radius 21 cm and height Edge of the cube = r
25 cm. 3

Here the radius r of the cylinder r = 21 cm, 2


5 2×
and its height h = 25 cm 3
∴ Radius of the sphere = 21 cm (  h > r) 10
Surface area of sphere = 4πr2 = cm
3
22
4× × 21 × 21 Diagonal of the cube = 3 (edge)
7
10
= 5544 sq. cm. = 3×
The surface area of the sphere will be 3
5544 cm2. = 10 cm.
368 ✫ Quantitative Aptitude

Clocks ● At what time between 3 and 4 o’clock are


The dial of a clock, which is a circle, has its the hands of a clock together?
circumference divided into 60 equal parts, called We have, T = 3
minute spaces. The clock has two hands. The ∴ Hands of the clock will be together
longer hand is the minute hand and the shorter
60 T
one is the hour hand. The minute hand passes at
over 60 minute spaces while the hour hand goes 11
over 5 minute spaces in 1 hour or 60 minutes. One 60
× 3
day contains 24 hours or 1440 minutes or 86400 11

TS
seconds.
180 4
The angle traced by an hour hand in 12 = = 16
hours is 360° while the same angle is traced by 11 11
the minute hand in 60 minutes. So, the hands of the clock will coincide

H
(i) The minute hand (M.H.) takes 60 minutes 4
at 16 minutes past 3 o’clock.
to gain 55 minute space (M.S.) over the 11

G
hour hand (H.H.). Therefore to gain 1 (ii) The two hands of a clock will be in
60 the same straight line but not together between
M.S. over the H.H., the M.H takes
minutes or
12
11
minutes.
55

U T and (T + 1) o’clock at

(a) (5T – 30)


12
minutes past T, when T > 6
O
(ii) The minute hand moves through 6° in 11
each minute; an hour hand moves through and at
1o 12
H
in each minute. (b) (5T + 30) minutes past T, when T < 6
2 11
(iii) In every hour,
● Find at what time between 8 and 9
_T

(a) the hands coincide once;


(b) the hands are straight (point in o’clock will the hands of a clock be in
opposite directions) once; and the same straight line but not together.
(c) the hands are twice at right angles. We have, T = 8 > 6
C

(iv) Starting from 12 a.m., the hands coincide


11 times in every 12 hours or 22 times in 12 12
∴ (5T – 30) = (5 × 8 – 30) ×
PS

24 hours; between 11 and 1 o’clock there 11 11


is a common position 12 o’clock when the 12
= (40 – 30) ×
hands coincide. 11
(v) The hands point towards each other 11 120 10
U

times in 12 hours or 22 times in 24 hours = = 10


11 11
(between 6 and 7 there is a common position
@

at 6 o’clock when the hands are straight). So, the hands will be in the same straight
(vi) The hands of a clock are at right angles 10
line but not together at 10 minutes
twice every hour. In 12 hours, they are at 11
at past 8 o’clock.
right angles 22 times and thus 44 times in
a day. Two positions common in every 12 [Alternative solution (if you don’t
hours, are at 3 o’clock and 9 o’clock. remember the formula): At 8 o’clock,
when the hour hand is at 8 and the
Fast-Track Formulae and minute hand is at 12, they are 20 minute
Worked Examples divisions apart. For them to be in straight
(i) The two hands of a clock will be line (but not together) they will have to
together between T and (T + 1) o’clock at be 30 minute divisions apart. The minute
60 T hand will have to gain 10 minutes over
minutes past T o’clock.
11 the hour hand.
Unit Five : Fundamentals / Clocks ✫ 369

55 minutes gained in 60 minutes [Alternative solution (if you don’t remember


60 120 the formula): At 5 o’clock, the minute hand
= 10 minutes gained in × 10 =
55 11 will be 25 minute spaces behind the hour
10 hand. (The minute hand will be at 12 and the
= 10 minutes hour hand at 5 on the dial, the space
11
between them being 25 minute spaces.)
∴ The hour hand and the minute hand of (a) When the M.H. is 3 minute spaces behind
a clock will be in a straight line between the H.H., the M.H. has to gain 25–3 or
10 22 minute spaces.
8 o’clock and 9 o’clock after at 10

TS
11 In 60 minutes, the M.H. gains 55 minute
minutes past 8.] spaces
● At what time between 4 and 5 o’clock 22 minutes are gained in
will the hands of a clock be in the same 60

H
× 22 = 24 minutes
straight line but not together? 55
(b) When the M.H. is 3 minute spaces ahead
We have, T = 4 < 6

G
of the H.H., the M.H. has to gain 25 + 3
12 12 or 28 minute spaces.
∴ (5T + 30) = (5 × 4 + 30) ×
11 11 In 60 minutes, the M.H. gains 55 minute
= 50 ×
12
11 U spaces.
28 minute spaces are gained in
O
600 6 60 6
= = 54 × 28 = 30 minutes
11 11 55 11
H
So, the hands will be in the same straight From (a) and (b) it is clear the hands will
6 be 3 minutes apart at 24 minutes and at
line but not together at 54 minutes
_T

11 6
past 4 o’clock. 30 minutes past 5 o’clock.]
11
(iii) Between T and (T + 1) o’clock, the two (iv) The two hands of the clock will be at
C

hands of a clock are M minutes apart at right angles between T and (T + 1) o’clock at
12 12
(5T ± M) minutes past T o’clock. (5T ± 15) minutes past T o’clock.
PS

11 11
[The hands can be apart by M minutes in [The hands can be apart by M minutes in
two cases—before or after the hour hand; hence two cases—before or after the hour hand; hence
(+) as well as (–).] (+) as well as (–).]
U

● At what time between 5 and 6 o’clock ● At what times between 4 and 5 o’clock will
will the two hands of a clock be 3 the hands of a clock be at right angles?
@

minutes apart?
We have, T = 4
We have, T = 5 and M = 3 12 12
∴ (5T ± 15) = (5 × 4 ± 15)
12 12 11 11
∴ (5T ± M) = (5 × 5 ± 3) 12 12
11 11 = 35 × and 5 ×
12 12 11 11
= 28 × and 22 × 420 60
11 11 = and
11 11
6 2 5
= 30 and 24 = 38 and 5
11 11 11
So the hands will be 3 minutes apart at The hands of a clock are at right angles
6 5 2
30 minutes past 5 and at 24 minutes at 5 minutes past 4 and at 38
11 11 11
past 5 o’clock. minutes past 4 o’clock.
370 ✫ Quantitative Aptitude

[Alternative solution (if you don’t remember 38 1440


the formulae): The M.H. and H.H. are at = ×
right angles to each other twice in an hour, 11 62
i.e., they are 15 minute spaces apart twice in 38 720 27360
= × =
an hour. At 4 o’clock, the M.H. is 20 minute 11 31 341
spaces behind the H.H. So the two cases 80
when the M.H. and H.H. are at right angles = 80 minutes.
are (i) when the M.H. is 15 minute spaces 341
behind the H.H.; and (ii) when the M.H. is [Alternative solution: In one hour, the minute
hand gains 55 minutes over the hour hand.

TS
15 minute spaces ahead of the H.H.
In case (i) the M.H. will have to gain In order to coincide, it must gain 60 minutes
20 – 15 or 5 minute spaces. 60 720
i.e., 60 minutes gained in × 60 =
Now 55 minute spaces are gained by the 55 11

H
M.H. in 60 minutes 5
5 minute spaces will be gained in = 65 minutes
11

G
60 5 But here, they are together after 62 minutes
× 5 = 5 minutes
55 11 itself.
So, the two hands are at right angles at 5
∴ Gain in 62 minutes = 65 – 62
5
5
11
minutes after 4 o’clock in one case.
U 720
11

− 62
O
In case (ii) the M.H. will have to gain =
11
20 + 15 or 35 minute spaces.
720 − 682
H
Now 55 minute spaces are gained by the =
M.H. in 60 minutes. 11
35 minutes will be gained in 38 5
_T

60 2 = =3
× 35 = 38 minutes 11 11
55 11 24 × 60 × 38 54720
So the two hands are at right angles at Gain in 24 hours = =
62 × 11 682
C

2
38 minutes past 4 o’clock in this case.] 27360
11 =
PS

341
(v) The minute hand of a clock overtakes 80
the hour hand at intervals of S minutes of = 80 minutes.
341
correct time. The clock gains or loses in a day (vi) Angle between the hands of a clock
60 × 24
U

720 (a) When the minute hand is behind the


by −S minutes.
11 S hour hand, the angle between the two
hands at S minutes past T o’clock
@

● The minute hand of a clock overtakes


the hour hand at intervals of 62 minutes S S
= 30 T − + degrees.
of correct time. How much does the 5 2
clock gain or lose in a day? (b) When the minute hand is ahead of the
We have, S = 62 hour hand, the angle between the two
The clock gains in a day by hands, at S minutes past T o’clock
S S
720 60 × 24 = 30 − T − degree.
= −S minutes 5 2
11 S
● Find the angle between the hour hand
720 60 × 24 and the minute hand of a clock when the
− 62 time is 7 : 30.
11 62
720 − 682 1440 We have, T = 7 and S = 30
= Here, the M.H. (at 6 for 30 minutes) will
11 62
Unit Five : Fundamentals / Calendars ✫ 371

be behind the H.H. (at more than 7 for Miscellaneous Examples


the hour)
● An accurate clock shows 7 o’clock in the
∴ The required angle
morning. Through how many degrees will
S S the hour hand rotate to show 1 o’clock in the
= 30 T − + degrees
5 2 afternoon of the same day?
30 30 Here, the hours are 7 ‘clock to 1 o’clock or
30 7 − +
5 2 6 hours
30 An hour hand moves 360° in 12 hours

TS
= 30 (7 – 6) + ∴ In 6 hours the hour hand would have moved
2
360
= 30 + 15 = 45° × 6 = 180°
[Alternative solution 12
The hour hand would have moved 180° from

H
Angle traced by hour hand in
12 hours = 360° 7 o’clock to 1 o’clock.
30

G
● A watch which gains uniformly is 2 minutes
Angle traced by hour hand in 7 hours
60 slow at noon on Tuesday and is 4 minutes
15 360 15 and 48 seconds fast at 2 p.m. on the following
i.e.
2
hours =
12
= 225°
×
2
U Tuesday. At which point was the clock correct?
O
The time period between 12 p.m. Tuesday to
Angle traced by minute hand in 60 minutes 2 p.m. next Tuesday = 7 days and 2 hours
= 360° = 170 hours
H
Angle traced by minute hand in 30 minutes
4 34
° The watch gains 2 + 4 minutes or
360 5
_T

= × 30 5
60 minutes in 170 hours
= 180° 48 4
∴ Required angle = 225° – 180° [48 seconds = = minutes]
C

60 5
= 45°.]
34
● What is the angle between the two If minutes are gained in 170 hours,
PS

5
hands of a clock when the time is 3 : 25? 2 minutes are gained in
Here T < 6
So the formula to get the angle is 5
170 × × 2 = 50 hours
U

S S 34
30 −T − degrees 50
5 2 = days
@

24
25 25
30 −3 − = 2 days and 2 hours
5 2
25 Two days and 2 hours after Tuesday 12 p.m.
= 30 (5 – 3) – will be Thursday 2 p.m.
2
25 So the watch will be correct on Thursday at
= 60 – 2 p.m.
2
120 − 25
= Calendars
2
95 1° A day is the chief measure of time. A day
= = 47
2 2 consists of 24 hours, which is the average time
The angle between the two hands will in which earth turns round on its axis.
1° An ordinary year has 365 days, i.e., 52
be 47 .
2 weeks and one day. (  365 = 52 × 7 + 1)
372 ✫ Quantitative Aptitude

These extra number of days left in a year/ (x) Cumulative number of days in the
month after the complete number of weeks are months of a year:
called odd days.
January 31 31
Every year which is divisible by 4 is a leap February +28 59 (60 in case of leap year)
year; but any century year (year ending in 00)
March +31 90
which is not divisible by 400 is not a leap year.
April 30 120
For example: 1964, 1988, 1992, 2000, are all May 31 151
leap years; whereas 100, 200, 1500, 2100, 1989, June 30 181
1990... are all examples of ordinary years. July 31 212

TS
A leap year has 366 days. August 31 243
September 30 273
Points to Note October 31 304

H
November 30 334
The following facts should be remembered: December 31 365 days

G
(i) 1 week = 7 days In case of a leap year, from February
(ii) In an ordinary year there are 52 onwards, the cumulative sum increases by one
weeks + 1 day = 365 days. Therefore,
it contains 1 odd day. U day.
Division of a year (Quarterwise)
O
(iii) A leap year contains 2 odd days. (A Period No. of days
leap year has 366 days which is
H
52 × 7 + 2 days.) February 28 (29)
1st quarter (Jan 1 to March 31) 90 (91)
(iv) 100 years = 76 ordinary years + 24 2nd quarter (April 1 to June 30) 91
_T

leap years 3rd quarter (July 1 to Sept. 30) 92


∴ Number of odd days in 100 years 4th quarter (Oct 1 to Dec 31) 92
= 76 odd days + 24 × 2 odd days Jan 1 to Dec 31 365 (366)
C

= 124 odd days = 17 weeks + 5 Number of days in a leap year is given in brackets.
odd days
PS

(xi) A day repeats itself after 7, 14, 21, ...


100 years contain 5 odd days 364 days (including the present day).
(v) As 100 years contain 5 odd days, 200 (xii) An ordinary year contains 1 odd day,
years contain 5 + 5 = 10 odd days therefore in a non-leap year, the weekday
U

and therefore 3 odd days (10 days of a specific date will increase by one
make a week and 3 days). in the next year. But a leap year contains
@

2 odd days. Therefore, for each leap


(vi) In 300 years there are 5 + 5 + 5 =
year, the day of the week advances by
15 odd days and therefore 1 odd day two for a date (after February of that
(15 days make 2 weeks and 1 day). year upto February of the next year).
(vii) 400 years contain 5 + 5 + 5 + 6 (400th
(xiii) To find the day of the week on a
year is a leap year) = 21 days and
particular date when no reference day
therefore 0 odd days. is given:
(viii) The last day of a century cannot be
First count the net number of odd days
a Tuesday, a Thursday or a Saturday. left over on the given date.
(ix) The first day of a century must be Then,
a Monday, a Tuesday, a Thursday or For 0 odd day it is Sunday
a Saturday. For 1 odd day it is Monday
Unit Five : Fundamentals / Calendars ✫ 373

For 2 odd days it is Tuesday 87 years have 21 leap years and 66 ordinary
For 3 odd days it is Wednesday years.
For 4 odd days it is Thursday As a leap year has 2 odd days, the total of
For 5 odd days it is Friday odd days in 1987 years are 21 × 2 + 66 × 1
For 6 odd days it is Saturday = 42 + 66 = 108 days
i.e., 15 weeks and 3 odd days.
Worked Examples Number of days between January 1, 1988 to
August 1, 1989.
● If April 4, 1988 was a Monday, what day was
September 2, 1988? Jan. Feb. Mar. April May June July Aug.

TS
31 + 29 + 31 + 30 + 31 + 30 + 31 + 1
From April 4, 1988, the number of days = 214 days
till September 2, 1988 are as follows. That is, 30 weeks and 4 odd days
= Apr May June July Aug Sept. Total number of odd days = 0 + 1 + 3 + 4

H
26 + 31 + 30 + 31 + 31 + 2 = 8 odd days; or 1 odd day left over
= 151 days = 21 weeks + 4 odd days August 1, 1988 was thus a Monday. The date

G
i.e., September 2, 1988 was a Friday. on Friday would thus be 5, and every seven
● Ashu was born on August 19, 1992. What day days from 5th would be a Friday.
of the week was he born? Hence, Friday falls on 5th, 12th, 19th and
August 19, 1992  1991 years + 7 months
+ 19 days U 26th in August 1988.
O
● In 400 consecutive years, how many 29s will
We begin with 1600 as reference year as it is
occur over the months?
known that it had no odd days.
H
1600 years contain 0 odd days In 400 consecutive years there are 97 leap
300 years contain 1 odd day years. [Dividing 400 by 4 we get 100. But
_T

91 years with 100, 200 and 300 are not leap years. However,
69 ordinary years 69 odd days 400 will be a leap year as it is divisible by
contain 400. So there are 97 leap years in 400
22 leap year s contain 44 odd days consecutive years.]
C

Adding up, 69 + 44 = 113 → 1 odd day ∴ In 400 consecutive years, February has 29
left over days 97 times and 29 occurs in the remaining
PS

From 1st January 1992, the number of days 11 months.


till August 19, 1992 are as follows. 400 × 11 = 4400 times
Jan. Feb. Mar. Apr. May June July Aug. ∴ 29th day of the month occurs 4400 + 97
U

31 29 31 30 31 30 31 19 = 4497 times.
= 232 days
● The first Republic Day of India was
@

= 33 weeks + 1 odd day


celebrated on January 26, 1950. What was
→ 1 odd day the day of the week on that date?
Total number of odd days
= (1+1) = 2 odd days 1950 ⇒ 1949 years + 26 days
 August 19, 1992 was a Tuesday. We begin with 1600 as reference year as
● What dates of August 1988 were Fridays? it is known that it had no odd days.
We first find the day on which August 1 fall. 1600 years contain 0 odd days
August 1, 1988 300 years contain 1 odd day
1987 years + 7 months
49 years have 12 leap
Let’s calculate the number of odd days in years and 37 ordinary 5 odd days
1987 years. years,
1600 years have 0 odd day so (12 × 2) + (37 × 1) days or 61 days or
300 years have 1 odd day 8 weeks and 5 odd days
374 ✫ Quantitative Aptitude

26 days of January 5 odd days = (1 week + 6 days)


Total odd days = 0 + 1 + 5 + 5 ≡ 6 odd days
Odd days in 2011 years = (0 + 0 + 6)
= 11 days or 1 week and
= 6
4 odd days are left over. For the period of the year 2012, total days:
So, the day was Thursday. Jan Feb March April May June
(31 + 29 + 31 + 30 + 31 + 30
● What was the day of the week on 4th
July Aug. Sept. Oct. Nov. Dec.
December, 2012? + 31 + 31 + 30 + 31 + 30 + 4)

TS
4th December 2012 = (2011 years + period Total days = 339 days
from 1.1.2012 to 4.12.2012) = 48 weeks + 3 days
Odd days in 1600 years = 0 ≡ 3 odd days
Total number of odd days

H
Odd days in 400 years = 0 = (6 + 3) = 9
11 years = (2 leap years + 9 ordinary years) = 1 week 2 days

G
= (2 × 2 + 9 × 1) odd days = 2 odd days
= 13 odd days Hence, the required day is Tuesday.

U
O
H
C _T
PS
U
@
Unit Five : Practice Session ✫ 375

PRACTICE SESSION
Practice Exercise 28
1. If the perimeter of an equilateral triangle (a) 16 cm (b) 15 cm
is 36.9 m and the length of an altitude (c) 14 cm (d) 13 cm
is 8 m then the area of the triangle is
8. The sides of a triangle are 3 cm, 4 cm and
(a) 29.2 m2 (b) 39.2 m2
2 5 cm. Its area is
(c) 49.2 m (d) 59.2 m2

TS
(a) 4 cm2 (b) 6 cm2
2. The area of the shaded portion of the (c) 8 cm2 (d) 10 cm2
given figure is
9. The area of an equilateral triangle of side

H
A 4 cm is
(a) 6.42 cm2 (b) 6.93 cm2
2
(d) 6.82 cm2

G
(c) 6.05 cm
D
2 cm
10. The legs of a right triangle are in the ratio

B E
1 cm
C
U of 1 : 2 and the area of the triangle is
36 square units. The hypotenuse of the
triangle is
O
2 cm
4 cm (a) 3 (b) 5
(a) 8 cm2 (b) 3 cm2 (c) 3 (d) 6 5
H
(c) 64 cm2 (d) 12 cm2 11. If the area of an equilateral triangle is
3. If the length of the hypotenuse of a right- 64 3 cm2, the perimeter of the triangle
_T

angled triangle is 5 cm and its area is 6 is


sq cm, then the length of the remaining (a) 42 cm (b) 44 cm
sides are (c) 48 cm (d) 49 cm
C

(a) 1 cm and 3 cm
(b) 3 cm and 2 cm 12. The area of a right-angled triangle is 40
(c) 3 cm and 4 cm times its base. What is its height?
PS

(d) 4 cm and 2 cm (a) 60 cm (b) 120 cm


4. The side of an equilateral triangle whose (c) 80 cm (d) 50 cm
area is 3 cm2 is 13. If the base of a triangle is 15 m and its
U

(a) 1 cm (b) 2 cm height is 12 m, the height of another


(c) 3 cm (d) 4 cm triangle with twice the area and base
@

5. If the lengths of the three sides of a 20 m is


triangle are 5 cm, 5 cm and 6 cm, its area (a) 4.5 m (b) 9 m
in cm2 is (c) 15 m (d) 18 m
(a) 6 (b) 10
(c) 12 (d) 15 14. If every side of a triangle is doubled, then
the increase in the area of the triangle is
6. The hypotenuse of a right-angled triangle
(a) ( 2 × 100) % (b) 200%
is 10 cm and one of its sides is 6 cm. The
area of the triangle is (c) 300% (d) 400%
(a) 24 cm2 (b) 21 cm2 15. The ratio of the areas of two triangles is
(c) 16 cm2 (d) 15 cm2 4 : 3 and the ratio of their heights is 3 : 4. Find
7. The perimeter of an isosceles triangle the ratio of their bases.
whose base is 6 cm and altitude is 4 cm (a) 16 : 9 (b) 9 : 16
is (c) 9 : 12 (d) 16 : 12

375
376 ✫ Quantitative Aptitude

16. If the sum of the diagonals of a rhombus (a) 21 m (b) 26 m


is 10 cm and its area is 12 cm2, then the (c) 13 m (d) 19 m
lengths of its diagonals are
25. The sides of a parallelogram are 12 m and
(a) 5, 5 (b) 9, 1
17 m respectively. If one of the diagonals
(c) 8, 2 (d) 6, 4
is 25 m long, the area of the parallelogram
17. The diagonal of a rhombus is 6 cm. If its is
area is 24 cm2, then the length of each side (a) 90 sq m (b) 100 sq m
of the rhombus is (c) 140 sq m (d) 180 sq m
(a) 5 cm (b) 6 cm

TS
26. The area of a parallelogram is 72 cm2 and
(c) 7 cm (d) 8 cm
its altitude is twice the corresponding
18. The area of a rhombus is 120 m2. If one base. Then the length of the base is
of the diagonals is 24 m, the perimeter (a) 4 cm (b) 5 cm

H
of the rhombus is (c) 6 cm (d) 7 cm
(a) 43 m (b) 44 m
27. If the base of a parallelogram is (x + 4),

G
(c) 48 m (d) 52 m
the altitude to the base is (x – 3) and the
19. The sides of a rhombus are 10 cm and area is (x2 – 4), then the actual area is
one diagonal is 16 cm. The area of the
rhombus is
(a) 95 cm2 (b) 96 cm2 U equal to
(a) 32 (b) 48
O
2
(c) 60 (d) 64
(c) 94 cm (d) 90 cm2
28. If the perimeter of a square is 16 cm, then
20. The area of a rhombus, one side of which
H
the area of the square is
measures 25 cm and the diagonal 30 cm, (a) 8 sq cm (b) 16 sq cm
is
_T

(c) 32 sq cm (d) 64 sq cm
(a) 100 cm2 (b) 200 cm2
(c) 400 cm2
(d) 600 cm2 29. If the side of a square is increased by 8
cm, then its area increases by 120 sq cm.
21. If the diagonals of a rhombus are 112 cm
The side of the square is
C

and 66 cm then the length of each side


(a) 2.5 cm (b) 3.5 cm
is
(c) 4.5 cm (d) 5.5 cm
PS

(a) 65 cm (b) 78 cm
(c) 56 cm (d) 48 cm 30. The ratio of the areas of two squares
whose sides are in the ratio 1 : 2 is
22. If the sum of the diagonals of a rhombus
(a) 1 : 4 (b) 4 : 1
U

is 12 cm, and its perimeter is 8 5 cm,


then the lengths of the diagonals are (c) 1 : 2 (d) 2 : 1
(a) 6 cm and 6 cm 31. If the side of a square is increased by 4
@

(b) 7 cm and 5 cm cm, the area increases by 60 sq cm. The


(c) 8 cm and 4 cm side of the square is
(d) 9 cm and 3 cm (a) 5.5 cm (b) 5.7 cm
23. If the length of a side of a rhombus is (c) 6 cm (d) 6.2 cm
13 cm and one of its diagonals is of length 32. The diagonal of one square is double the
24 cm, then the area of the rhombus is diagonal of another square. What is the
(a) 240 cm2 (b) 156 cm2 ratio of the areas of the two squares?
2
(c) 130 cm (d) 120 cm2 (a) 4 : 1 (b) 1 : 4
24. The altitude of a parallelogram is twice (c) 2 : 1 (d) 1 : 2
as long as the corresponding base. If the 33. A floor area of 72 sq. m. requires 50
area of the parallelogram is 338 m2, the square stone slabs of equal size. The
altitude is length of each stone slab is
Unit Five : Practice Session ✫ 377

(a) 230 cm (b) 12 m (a) 40 2 cm (b) 40 cm


(c) 102 cm (d) 120 cm (c) 10 2 cm (d) Cannot be determined
34. If the length of the diagonal of a square 35. If the side of a square is increased by 50%,
the percentage increase in area is
is 20 cm, what is the perimeter of the
(a) 50 (b) 100
square? (c) 125 (d) 150

Answers with Solutions


1

TS
1. Ans. (c) 6 = × 3 × 4
2
Sol. Perimeter of equilateral triangle = 3a 6 = 6 which is also true.
⇒ 36.9 = 3a 4. Ans. (b)
⇒ a = 12.3 m Sol. Let the length of each side of an

H
Side of an equilateral triangle is 12.3 m equilateral triangle be a cm
Given, altitude of equilateral triangle 3 2
Area of equilateral triangle = a

G
is 8 m 4
3 2
1 3 = a
Area of triangle = ah 4
2
1
= × 12.3 × 8 U ∴ 2
a =4
a = 2 cm.
O
2 5. Ans. (c)
= 49.2 m2. Sol. From the given dimensions the triangle
is isosceles. So if a denotes the equal
H
2. Ans. (b) sides,
Sol. Area of shaded portion b
4 a2 − b2
_T

= Area of ΔABC – Area of ΔDEC Area =


4
1 1 6
= ×2×4 − ×1× 2 = 4 × 52 − 6 2
2 2 4
C

= 4 – 1 6
= 100 − 36
= 3 cm2. 4
PS

3. Ans. (c) 6 6
= 64 = × 8
Sol. A 4 4
= 12 sq cm.
a 5 [Alternative (Fast-track) solution:
U

a = 5, b = 5, c = 6
∴ 2s = a + b + c
B C
@

b = 5 + 5 + 6 = 16
or s = 8
We know by Pythagoras Theorem that
Area of triangle = s (s − a) (s − b) ( s − c)
52 = a2 + b2
25 = a2 + b2 = 8×3×3×2
From the options given, only (c) satisfies = 4 × 3 = 12 sq cm.]
the conditions of the question as 6. Ans. (a)
a2 + b2 = 25 Sol. Using Pythagoras Theorem,
If a = 3 and b = 4, then 102 = a2 + b2
32 + 42 = 52 (if a is the unknown side)
9 + 16 = 52 a = (10)2 − (6)2
25 = 25 which is true.
Now, = 100 − 36 = 64 = 8 cm
1 1
Area of right-angled triangle =
2
ab ∴ Area of the triangle = × 8 × 6 = 24 sq cm.
2
378 ✫ Quantitative Aptitude

7. Ans. (a) x =6
Sol. A So, 2x = 12
∴ Hypotenuse = 62 + 12 2 = 180
l l = 6×6×5= 6 5.
4 cm 11. Ans. (c)
Sol. Area of the equilateral triangle = 64 3
If side is a,
B C
D

TS
3 2
6 cm Area = a = 64 3
4
Let AB = AC = l cm 4
a2 = 64 3 ×
In right ΔADC 3

H
6
2
= 256
l2 = 4 +
2
= 16 + 9 a = 16 cm
2

G
2
l = 25 Perimeter of the equilateral triangle = 3a
l = 5 cm 3 × 16 = 48 cm.
∴ Perimeter of isosceles triangle
= (5 + 5 + 6)
U 12. Ans. (c)
Sol.
A
O
= 16 cm.
8. Ans. (b)
a a
Sol. Given, a = 3, b = 4, c = 5
H
x
a+b+c 3+ 4+ 5
s= = =6
2 2
D C
_T

Area of triangle B a
2
= s ( s − a) (s − b) ( s − c)
1
= 6 (6 − 3) (6 − 4) (6 − 5) Area of a triangle = base × height
C

2
= 6 × 3× 2×1 1
40 × base = base × height
PS

2 2
= 36 = 6 cm . base × height
9. Ans. (b) 40 × 2 =
base
Sol. A side (a) of the triangle = 4 cm Height = 80 cm
U

Area of equilateral triangle


13. Ans. (d) 1
3 2
Sol. Area of the first triangle = bh
@

= a
4 1
2
3 = × 15 × 12 = 90 m2
× (4)2 2
4 Given that the area of the other triangle
= 1.732 × 4 is twice that of the first triangle,
1
= 6.93 cm2. 180 = × 20 × h
10. Ans. (d) 2
Sol. Let the two sides of the right-angled h = 18 m.
triangle be x and 2x 14. Ans. (c)
1 Sol. Let a, b, c be the sides of one triangle
Area of triangle = ab
2 a+b+c
1 Then s1 =
36 = (x) (2x) 2
2
∴ Area A1 = s1 ( s1 − a ) ( s1 − b ) ( s1 − c )
∴ x2 = 36
Unit Five : Practice Session ✫ 379

Let the new sides (after increase) be 2a, Factorising, we get


2b and 2c x2 – 6x – 4x + 24 = 0
x (x – 6) – 4 (x – 6) = 0
2a + 2b + 2c 2( a + b + c )
Then s2 = = (x – 6) (x – 4) = 0
2 2 ∴ x = 6, 4.
⇒ s2 = 2s1 Lengths of the diagonals of the rhombus
⇒ s2 – 2a = 2s1 – 2a = 2 (s1 – a) = 6, 4
Similarly, 17. Ans. (a) 1
s2 – 2b = 2(s1 – b), and Sol. Area of rhombus = d1d2

TS
2
s2 – 2c = 2(s1 – c) [d1 and d2 being the two diagonals]
1
∴ New area A2 = s2 (s2 − 2 a) ( s2 − 2 b) (s2 − 2 c ) ⇒ 24 = × 6 × d2
2

H
48 = 6d2
= 2 s1 × 2 (s1 − a ) × 2( s1 − b) × 2( s1 − c )
d2 = 8

G
= 4 s1 ( s1 − a ) ( s1 − b) ( s1 − c ) The other diagonal of the rhombus is
= 4A1 8 cm.

% increase in area =
4 A1 − A1
A1
× 100
U When d1 and d2 are the two diagonals,
the side of the rhombus
1
O
3 A1 = d12 + d22
= × 100 = 300%. 2
A1 1
H
15. Ans. (a) ⇒ (6)2 + (8)2
2
Sol. Let the base and height of the two 1 a a
_T

= 36 + 64
triangles be b1, h1 and b2, h2 respectively. 2 8 cm
Then 1
= 100
Ratio of the area of the two triangles 2 6 cm
C

1 a a
=4:3 = × 10
2
PS

h1 : h2 = 3 : 4
= 5 cm.
1 18. Ans. (d)
b1h1 4
2 = Sol. Let’s find the length of the other
1
U

b2 h2 3 diagonal
2 1
Area of rhombus = d1d2
b1 4 h 4 4 16 2
@

= × 2 = × = 1
b2 3 h1 3 3 9 120 = × 24 × d2
∴ Ratio of their bases = 16 : 9 2
120
16. Ans. (d) ∴ d2 = ×2
Sol. Let the length of one of the diagonals 24
of the rhombus be x cm. = 10 m
∴ The length of the other is 10 – x
1 a a
Area of rhombus = d1 d2 = 12 (given)
2 5m
1
(10 – x)x = 12 12 m 12 m
2 5m
a a
∴ x2 – 10x + 24 = 0
380 ✫ Quantitative Aptitude

Let the sides of the rhombus be a m. In a rhombus,


We know that diagonals of a rhombus 4 × side2 = d12 + d22
bisect each other at 90° 4 × (2 5 )2 = d12 + d22
∴ a2 (which is the hypotenuse) 4 × 4 × 5 = d12 + d22 = 80 cm
= 52 + 122
= 25 + 144 Plug in the numbers from each option
= 169 and check.
a = 13 m (a) Not likely, as the diagonals of a
∴ Perimeter of rhombus = 4a rhombus are not equal. Even so,
62 + 62 or 72 ≠ 80

TS
= 4 × 13
= 52 m. (b) 7 + 52 or 74 ≠ 80
2

19. Ans. (b) (c) 82 + 42 or 80 = 80


Sol. Side of rhombus (a) (d) 92 + 32 or 90 ≠ 80

H
1 So (c) is the correct answer.
= d12 + d2 2
2 23. Ans. (d)

G
1 Sol. Area of rhombus
10 = 16 2 + d2 2
2 1
= × product of diagonals
202 = 162 + d22
400 – 256 = d22 U =
2
1
× 24 × 10
O
d2 = 144 = 12 cm 2
1 1 [  small diagonal = 2 13 2 − 122 = 10]
∴ Area of rhombus = × d1×d2 = ×16×12
H
2 2 = 120 sq cm.
= 96 cm2. 24. Ans. (b)
_T

20. Ans. (d) Sol. Let the base of the parallelogram be x m.


Sol. In a rhombus, Then, altitude of parallelogram is 2x m
4 (side)2 = d12 + d22 Area of parallelogram = 338 m2
C

4 × 25 × 25 = 302 + d22 Area of parallelogram = base × altitude


2
d2 = 2500 – 900 = 1600 ∴ Base × Altitude = 338
PS

d2 = 1600 = 40 x (2x) = 338


1 2x2 = 338
∴ Area of rhombus = × 30 × 40 338
2
x2 =
U

= 600 cm2. 2
21. Ans. (a) x2 = 169
@

Sol. If a is a side of the rhombus, and d1 and x = 169


d2 the diagonals,
∴ x = 13 m
4a2= d12 + d22 Hence, base = 13 m and
= 1122 + 662 altitude = 2x = 2 × 13
= 12544 + 4356 = 16900 = 26 m.
16900 25. Ans. (d)
∴ a2 = = 4225 Sol.
4
17
∴ a = 4225 = 65 cm. D C

22. Ans. (c) 25


12 12
Sol. If the perimeter of the rhombus is 8 5 cm,
8 5
one side will be = 2 5 cm A B
4 17
Unit Five : Practice Session ✫ 381

In Δ s ADC and ABC a2 + 16a – a2 = 120 – 64


a = 17, b = 12 and c = 25 16a = 56
a+b+c a = 3.5 cm.
∴ s =
2 30. Ans. (a)
17 + 12 + 25
s = Sol. Let the side of the first square be a
2
54 Side of the other square = 2a
= = 27
2 Area (A1 ) of first square a2
Area of parallelogram ABCD Area (A2 ) of second square =
4 a2

TS
= 2 × area of ΔABC A1 : A2 = 1 : 4.
= 2 × s ( s − a) ( s − b ) ( s − c ) 31. Ans. (a)
Sol. Area of square = (side)2
= 2 × 27 (27 − 17) (27 − 12) (27 − 25)

H
Let side of square be x
Then, area of square = x2
= 2 × 27 × 10 × 15 × 2 Side is increased by 4 cm; thus the new

G
= 2 × 3 ×3× 3× 2× 5× 3× 5× 2 area is (x + 4)2
Given that
= 2 × 3 × 3 × 2 × 5
= 180 sq m.
26. Ans. (c) U (x + 4)2 – x2 = 60
x2 + 8x + 16 – x2 = 60
8x = 44
O
Sol. Let base be x cm; then altitude is 2x cm x = 5.5 cm.
Area of parallelogram = base × altitude 32. Ans. (a)
H
∴ (x) (2x) = 72 (given) Sol. Let their diagonals be 2d and
x2 = 36 d respectively.
1
_T

x = 6 cm (2d)2 4
Ratio of their areas = 2 =
Hence, base is 6 cm. 1 2 1
d
27. Ans. (c) 2
C

Sol. Area of a parallelogram = 4 : 1.


= base × height
33. Ans. (d)
PS

2
(x + 4) (x – 3) = x2 – 4 72
Sol. Area of each slab = m = 1.44 m2
x2 + x – 12 – x2 + 4 = 0 50
x– 8 = 0 One side or the length of a slab (which
x = 8
U

is square) = 1.44 = 1.2 m


∴ Actual area = (8 + 4) × (8 – 3)
= 120 cm
= 12 × 5 = 60.
@

34. Ans. (a)


28. Ans. (b)
Sol. The diagonal d of a square of side
Sol. Perimeter of square = 4a
16 = 4a l = 2 × l
∴ a = 4 cm 20
l =
Area of square = a2 = 16 sq cm. 2
29. Ans. (b) 20
The perimeter = 4l = 4 ×
Sol. Let the original side of the square be a. 2
Then, area = a2 Multiplying numerator and denominator
New side = a + 8 by 2 , we get
New area = a2 + 120 (given)
20 2 80 2
New area = (new side)2 4× × = = 40 2
a2 + 120 = (a + 8)2 2 2 2
= a2 + 16a + 64 The perimeter of the square is 40 2 cm.
382 ✫ Quantitative Aptitude

35. Ans. (c) [Alternative solutions (Fast-track):


150 3a Change in area of the square will be
Sol. New side = a =
100 2 x
x 2+ %
where a is the side of the original square. 100
3a 3a 9 a2 (where x is the percentage increase in
Area = × = the side)
2 2 4
9a2
× a2 ⇒ 50 2 +
50
%
% increase in area = 4 × 100 100

TS
a2
1
Increase in area = 50 2 + %
× 100 2
Original area

H
5
5a 2
× 100 = 50 ×
= 2
4 a2

G
= 125%. = 125%.]

U
O
H
C _T
PS
U
@
Unit Five : Practice Session ✫ 383

Practice Exercise 29
1. If half the diagonal of a square is 5 cm, (a) 42 m (b) 45 m
then the area of the square is (c) 48 m (d) 49 m
(a) 20 2 cm2 (b) 25 cm2 9. The ratio of two unequal sides of a
2
(c) 50 cm (d) 100 cm2 rectangle is 1 : 2. If the perimeter is 24 cm,
2. The perimeter of one square is 12 cm and then the length of a diagonal is
that of another is 16 cm. The side of a 2 4
(a) cm (b) cm

TS
square whose area is equal to the sum of 5 5
the areas of the above two squares would (c) 2 5 cm (d) 4 5 cm
be
(a) 4 cm (b) 5 cm 10. The floor of a rectangular hall is 40 m long

H
and 24 m wide. How many carpets of size
(c) 5.5 cm (d) 6 cm
6 m × 4 m are required to cover the floor
3. The number of square-shaped tin sheets

G
of the hall?
of side 20 cm that can be cut off from (a) 40 (b) 30
a square tin sheet of side 1 m is (c) 20 (d) 10
(a) 5
(c) 50
(b) 25
(d) 100 U 11. A square lawn is surrounded by a path
3 m wide. If the area of the path is 180
O
m2, the area of the lawn is
4. The cost of cultivating a square field at
(a) 120 m2 (b) 132 m2
the rate of Rs 160 per hectare is Rs 1440.
H
2
(c) 144 m (d) 148 m2
Find the cost of putting a fence around
it at the rate of 75 paise per metre. 12. The perimeter of a rectangle is 48 m and
_T

(a) Rs 900 (b) Rs 850 its area is 135 m2. The sides of the
(c) Rs 800 (d) Rs 750 rectangle are
(a) 15 m, 9 m (b) 19 m, 5 m
5. The ratio of the area of a square to that
C

(c) 45 m, 3 m (d) 27 m, 5 m
of a square drawn on its diagonal is
(a) 1 : 1 (b) 3 : 2 13. The areas of a rectangle and a square are
PS

(c) 1 : 2 (d) 1 : 4 equal. The side of the square is 5 cm and


the smaller side of the rectangle is half
6. A path 1.5 m in width is laid all around that of the square. The length of the other
a square lawn on the outside. If the area side of the rectangle is
U

of the path is 129 m2, then the area of (a) 5 cm (b) 8 cm


the lawn excluding the path is (c) 10 cm (d) 12.5 cm
@

(a) 361 m2 (b) 225 m2 14. A rectangular plate of size 9 cm by 4 cm


2
(c) 121 m (d) 400 m2 is to be melted and cast into 4 squares
7. If the length of the diagonal of a square of equal area but of the same thickness
and that of the side of another square are as the rectangular plate. The side of each
both 10 cm, the ratio of the area of the such square is
(a) 2 cm (b) 3 cm
first square to that of the second is
(c) 4 cm (d) 4.5 cm
(a) 1 : 2 (b) 1 : 3
(c) 1 : 4 (d) 2 : 3 15. The sides of a rectangular field are in
the ratio 3 : 1 and its area is 7500 sq m.
8. The length and breadth of a rectangular The cost of fencing the field at 25 paise
field are in the ratio 3 : 2. If its area is per metre is
14406 sq m, then the difference between (a) Rs 100 (b) Rs 94.50
the length and the breadth is (c) Rs 80 (d) Rs 57.50
384 ✫ Quantitative Aptitude

16. The diagonal of a rectangle is 41 cm and Inside the field, a path of 12 m width is
made around the field. The area of the
its area is 20 sq. cm. The perimeter of the path is
rectangle must be (a) 3224 m2 (b) 7344 m2
(a) 9 cm (b) 18 cm (c) 2256 m2 (d) 4224 m2
(c) 20 cm (d) 41 cm
23. A lawn of 5 m width is maintained all
17. A rectangular plank 2 m wide is placed along the outside of a rectangular plot
symmetrically on the diagonal of a square measuring 90 m × 40 m. The total area
of side 8 m as shown in the figure. The of the lawn is

TS
area of the plank is (a) 1100 m2 (b) 1200 m2
2
A B (c) 1300 m (d) 1400 m2
24. If the perimeter of a rectangle and a
C

H
square each is equal to 80 cm and the
8m
difference in their areas is 100 cm2, the
2m sides of the rectangle are

G
(a) 15 cm, 35 cm
(b) 30 cm, 10 cm
E D
(a) (16 2 − 3) sq m (b) 7 2 sq m U (c) 15 cm, 25 cm
(d) 28 cm, 12 cm
O
(c) 98 sq m (d) 14 sq m 25. Three squares have perimeters of 16 m,
18. How many tiles 40 cm square will be 20 m and 24 m respectively. If a rectangle
H
is constructed whose area is equal to the
required to pave a footpath 1 m wide
combined areas of the three squares such
carried round the outside of a grass plot
that its length is 11 m, then the breadth
_T

28 m long and 18 m broad?


is
(a) 600 (b) 750
(a) 6 m (b) 8 m
(c) 810 (d) 900 (c) 7 m (d) 12 m
C

19. A room 15 m long requires 7500 tiles, each 26. In a rectangular garden 10 m wide and
15 cm by 12 cm, to cover the entire floor. 20 m long, we wish to pave a walk around
PS

The breadth of the room is the borders of uniform width, so as to


(a) 8 m (b) 9 m have an area of 96 m2 for flowers. How
(c) 10 m (d) 11 m wide should the walk be?
(a) 1 m (b) 2 m
U

20. A room 5 m by 4 m is to be carpeted


leaving a margin of 25 cm from each wall. (c) 3 m (d) 2.5 m
If the cost of the carpet is Rs 80 per m2, 27. The cost of levelling a rectangular ground
@

the cost of carpeting the room will be at Rs 1.25 per square metre is Rs 900. If
(a) Rs 1270 (b) Rs 1260 the length of the ground is 30 m, then
(c) Rs 1250 (d) Rs 1240 the width is
(a) 33 m (b) 30 m
21. The length of a plot of land is four
(c) 24 m (d) 18 m
times its breadth. A playground measuring
1200 m2 occupies one-third of the total 28. If the perimeter of a rectangular field is
area of the plot. The length of the plot 440 m and its breadth is two-thirds its
is length, then the area of the field is
(a) 300 m (b) 210 m (a) 7200 m2 (b) 9600 m2
2
(c) 120 m (d) 90 m (c) 10000 m (d) 11616 m2
22. The length and breadth of a rectangular 29. The areas of a square and a rectangle are
field are 120 m and 80 m respectively. equal. The length of the rectangle is
Unit Five : Practice Session ✫ 385

greater than the length of any side of the 33. What will be the cost of digging up a
square by 5 cm and the breadth is less 1-metre-broad boundary around a
by 3 cm. Find the perimeter of the rectangular plot of perimeter 340 metres,
rectangle. if the cost of digging is Rs 10 per square
(a) 17 cm (b) 26 cm metre?
(c) 30 cm (d) 34 cm (a) Rs 3440
(b) Rs 1700
30. A towel, when bleached, was found to (c) Rs 3400
have lost 20% of its length and 10% of (d) Cannot be determined
its breadth. The percentage of decrease in

TS
34. What would be the cost of laying a carpet
area is
on a floor which has its length and
(a) 10% (b) 10.08%
breadth in the respective ratio of 32 : 21
(c) 20% (d) 28%
and where its perimeter is 212 feet, if the

H
31. What is the least number of square tiles cost per square foot of laying the carpet
required to pave the floor of a room 15 m is Rs 2.5?

G
17 cm long and 9 m 2 cm broad? (a) Rs 6,720
(a) 814 (b) 820 (b) Rs 5,420
(c) 840 (d) 844 (c) Rs 7,390
32. The length and breadth of a courtyard are
20 m and 10 m respectively. Square tiles U (d) Cannot be determined
35. The total area of a circle and a rectangle
O
of 2 m length of different colours are to is equal to 1166 sq.cm. The diameter of
pave the courtyard. Black tiles are laid the circle is 28 cms. What is the sum of
H
along all the sides as the first row. If the circumference of the circle and the
green tiles are laid in one-third of the perimeter of the rectangle if the length of
_T

remaining area and yellow tiles in the rest the rectangle is 25 cm?
of the area, how many yellow tiles will (a) 186 cm
be required? (b) 182 cm
(a) 32 (b) 24 (c) 184 cm
C

(c) 16 (d) None of these (d) Cannot be determined


PS

Answers with Solutions


1. Ans. (c) 2. Ans. (b)
Diagonal Sol. Perimeter of the first square = 12 cm
U

Sol. = 5 cm 4a1 = 12
2
Diagonal = 10 12
a1 = = 3
@

4
1 Hence, the side of the first square is 3 cm.
Side (a) = diagonal
2 Perimeter of the second square = 16 cm
1 4a2 = 16
= × 10 16
2 a2 = = 4
4
Multiplying numerator and denominator Hence, the side of the second square is 4 cm.
by 2 , Area of the new square
10 × 2 = Area of the first square
10 × 2 + Area of the second square
⇒ =
2× 2 2 (a3)2 = (3)2 + (4)2
⇒ 5 2 = 9 + 16
= 25 cm2
∴ Area of the square (a2) = 5 2 × 5 2 a3 = 5 cm
= 50 cm2. ∴ The side of the third square is 5 cm.
386 ✫ Quantitative Aptitude

3. Ans. (b) Let x be the side of the square lawn


Sol. Area of square sheet =12 m ABCD
=1002 cm Each side of PQRS is x + 3
=10000 cm ∴ Area of the path
Area of square piece =202 cm = Area of PQRS – Area of ABCD
=400 sq. cm (x + 3)2 – x2 = 129
10000 x2 + 6x + 9 – x2 = 129
No. of square sheets = 6x = 129 – 9 = 120
400
x = 20

TS
= 25.
4. Ans. (a) ∴ Area of lawn ABCD = 20 × 20 = 400 m2.
Total cost [Alternative solution: The area of pathway
Sol. Area =
Ap = 2w (l + b + 2w)

H
Rate per hectare
1440 where l and b are length and breadth
= hectares of the inner rectangle and w the width

G
160
of the pathway. Here l = b as it is a
= 9 hectares square.
= 90000 m2
Side of square = 90000 U 129 = 2 × 1.5 [(2 × l ) + (2 × 1.5)]
= 3 (2 l + 3)
O
= 300 m =6l+9
Perimeter of the square = 4 × side 129 − 9 120
=
H
l = = 20 m
= 4 × 300 6 6
Now area of the inner square (or the
_T

= 1200 m area of lawn excluding the pathway)


1200 × 75 = 20 × 20 = 400 m2.]
Cost of fencing = Rs
100 7. Ans. (a)
= Rs 900. Sol.
C

5. Ans. (c) 10
Sol. If a is the side of the square, then length
PS

10 10
of the diagonal
diagonal
= 2a [  side = ] A B
2
U

Area (A1) = a2 1
Area of square A = (diagonal)2
A2 = Area of square drawn on the 2
@

diagonal as a side of 2a 1
= × 10 × 10
= 2a × 2a = 2a2 2
∴ A1 : A2 = a2 : 2a2 = 1 : 2. = 50 cm2
6. Ans. (d) Side of square B = 10 cm
∴ Area of square B = (side)2 = (10)2
Sol.
P Q
= 100 cm2
A Area of square A 50 1
B ∴ = =
x Area of square B 100 2
1.5 1.5 x +3 The ratio of the areas = 1 : 2.
D C 8. Ans. (d)
S R
Sol. Let the length and the breadth of the
x+1.5+1.5=x+3 rectangular field be 3x and 2x.
Unit Five : Practice Session ✫ 387

Area of rectangular field Then, area of lawn = a2


= length × breadth EF = (a + 6) m
14406 = 3x × 2x Area of square EFGH = (a + 6)2
14406 Now, area of path = (a + 6)2 – a2
= x2
6 180 = a2 + 12a + 36 – a2
2401 = x2 12a = 144
∴ x = 49 a = 12 m
Hence, the difference between the length So, area of lawn = a2 = (12)2
and the breadth of the rectangular field = 144 m2.

TS
is 3x – 2x = 3 (49) – 2 (49) [Alternative solution: The area of the
= 49 (3 – 2) pathway
= 49 m.
Ap = 2w (l + b + 2w)
9. Ans. (d)

H
Sol. Let the sides of the rectangle be x and 2x. where l and b are length and breadth
Perimeter of the rectangle of the inner rectangle and w the width

G
= 2 (length + breadth) of the pathway. Here l = b as it is a
24 = 2 (x + 2x) square.
12 = 3x 180 = 6 (2 l + 6)
∴ x = 4
Diagonal of the rectangle U = 12 l + 36
180 − 36 144
O
l = = = 12 m
= (Length)2 +(Breadth)2 12 12
Area of the lawn = 12 × 12 = 144 m2.]
H
(x )2 + (2 x )2 12. Ans. (a)
Sol. Perimeter of the rectangle = 48 m
(4)2 + (2 × 4)2
_T

If a and b are the sides,


= 16 + 64 2 (a + b) = 48
= 80 = 4 5 cm. a + b = 24 m … (i)
C

10. Ans. (a) Area of rectangle = 135 m2


Sol. Length of hall = 40 m, breadth = 24 m ab = 135 … (ii)
PS

Area of floor of hall = 40 × 24 = 960 m2 135


∴ b=
Length of carpet = 6 m a
Breadth of carpet = 4 m Putting the value of b in equation (i),
Area of one carpet = 6 × 4 = 24 m2
U

we get
Total floor area 135
Number of carpets = a + = 24
@

Carpet area a
960 a 2 + 135
= = 40. = 24
24 a
11. Ans. (c) E F a2 + 135 = 24a
2
Sol. a a – 24a + 135 = 0
B
a2 – 15a – 9a + 135 = 0
A
3m a(a – 15) –9 (a – 15) = 0
(a – 9) (a – 15) = 0
D C a = 9, 15
135
Hence, (i) b = = 15 (when a = 9)
H G 9
Area of path = 180 m2 135
(ii) b = = 9 (when a = 15)
Let side of lawn be a 15
388 ✫ Quantitative Aptitude

[Alternative solution: Use the shortcut = 2 (3 × 50 + 50)


Perimeter = 2 (150 + 50)
Area = x – x2 = 400 m
2
where the two values of x will give you Cost of fencing is 25 paise per metre
the required length and breadth Cost of fencing 400 m = 400 × 0.25
Here, = Rs 100.
48 16. Ans. (b)
135 = x – x2
2 Sol. Diagonal of a rectangle
⇒ 135 = 24x – x2
⇒ x2 – 24x + 135 = 0 = l 2 + b2

TS
⇒ x2 – 15x – 9x + 135 = 0 ⇒ 41 = l + b
2 2

⇒ (x – 15) (x – 9) = 0 Squaring of both sides,


∴ x = 15, 9 41 = l 2 + b 2

H
The sides of the rectangle are 15 m and Given, area of rectangle
9 m.] = l × b = 20 sq. cm.

G
13. Ans. (c) (l + b)2 = (l 2 + b 2) + 2lb
Sol. Area of rectangle = 41 + 40 = 81
= Area of square (Given) ⇒ (l + b) = 9

5
ab = a2
U ∴ Perimeter of the rectangle
O
× b = 25 = 2 (l + b) = 18 cm.
2 [Alternative solution: We know that
25 × 2
b = Perimeter 2 Diagonal 2
H
5 Area= −
b = 10 cm 8 2
Perimeter 2 ( 41)2
_T

Hence, the other side of the rectangle ⇒ 20 = −


is 10 cm. 8 2
14. Ans. (b) 41
Sol. As thickness is same, only area is ⇒ 20 + × 8 = Perimeter 2
2
C

relevant. Area of rectangular plate 40 + 41


= 4 × area of one square ⇒ × 8 = 324
2
PS

l × b = 4 (a)2
⇒ 324 = Perimeter2
(a being the side of the square)
9 × 4 = 4 (a)2 ⇒ Perimeter = 324 = 18 cm.]
9 = a2
U

∴ a =3 17. Ans. (d)


Hence, the side of each such square is Sol. In ΔABC, AB = BC, AC = 2 , ∠B = 90°
@

3 cm. AB = BC = 1 cm
15. Ans. (a) (In ΔECD use Pythagoras theorem)
Sol. Let the length and breadth of a Length of rectangular plank
rectangular field be 3x and x.
= 49 + 49
Area of field = ab
= 98 = 7 2
7500 = 3x × x
7500 ∴ Area of the plank = 7 2 × 2 = 14 m2.
= x2 18. Ans. (a)
3
x2 = 2500 Sol. Width of the footpath is 1 m
x = 2500 ∴ Outside dimensions of the plot are
30 m = (28 + 2)
∴ x = 50 and 20 m = (18 + 2)
Perimeter of the rectangular field Now outside area of plot = 30 × 20
= 2 (a + b) = 600 m2
Unit Five : Practice Session ✫ 389

Inside area of plot = (30 – 2) × (20 – 2) 21. Ans. (c)


= 28 × 18 = 504 m2 Sol. Let breadth be x m, then length is 4x m
∴ Area of the footpath Area of plot = length × breadth
= outside area – inside area = (4x) × x
= 600 – 504 3 × 1200 = 4x2
= 96 m2 ∴ x2 = 900
40 40
Area of one tile = × x = 30 m
100 100
∴ Length of plot = 4x = 4 × 30
= 0.16 m2 = 120 m.
96

TS
∴ Number of tiles required = = 600. 22. Ans. (d)
0.16
[Fast-track solution: In the given case, the Sol. AB = CD = 120 m
area of the pathway BC = AD = 80 m

H
= 2w (l + b + 2w) sq. units A B
12 12
(w being width of the path, and l and S R
b, the length and breadth respectively

G
80
of the rectangle outside which the
path is made.) P Q

U
Area of path = 2 × 1 [28 + 18 + (2 × 1)] m2 D
Area of path
120
12 12
C
O
2
= 2 × 48 = 96 m ,
Area of one tile = 40 cm × 40 cm = Area of the field – Area of the inside field
= 0.4 m × 0.4 m = 0.16 m2 = (80 × 120) – [(120 – 24) × (80 – 24)]
H
(Note that units must be same.) = 9600 – 96 × 56
Number of tiles required to pave the path = 9600 – 5376 = 4224 m2.
_T

Area of path 96 [Fast-track solution: In this case, the area


= = 600.] of the path
Area of one tile 0.16 = 2w (l + b – 2w)
C

19. Ans. (b) (where w is width of the path, and


15 12 l and b, the length and breadth of the
Sol. Area of one tile = × sq. m
PS

100 100 rectangle inside which the path is


made)
Number of tiles = 7500 Area of path = 2 × 12 [120 + 80 – (2 × 12)]
15 12 = 24 (200 – 24)
×
U

Area of the floor = 7500 × = 24 × 176


100 100
= 135 sq m = 4224 m2.]
@

Length of the room = 15 m (given) 23. Ans. (d)


Area Sol.
∴ Breadth of the room = P Q
Length 90 5m
135 A B
= = 9 m. 40
15
20. Ans. (b) D C
Sol. Length of carpet = 5 – (0.25 + 0.25) S R
= 4.5 m
Length (PQ) = 90 + 5 + 5 = 100
Breadth of carpet = 4 – (0.25 + 0.25)
= 3.5 m Breadth (QR) = 40 + 5 + 5 = 50
∴ Area of carpet = 4.5 × 3.5 Area of lawn = Area of PQRS – Area of ABCD
= 15.75 m2 = 100 × 50 – 90 × 40
Cost of carpet = 80 × 15.75 = 5000 – 3600
= Rs 1260. = 1400 m2.
390 ✫ Quantitative Aptitude

[Fast-track solution: The relevant formula 25. Ans. (c)


here is Sol. Lengths of the sides of the three squares
2w (l + b + 2w)
(where w is width of the lawn around are
the plot, and l and b are the length 16 20
and breadth of the rectangle inside) = 4 m, = 5 m and 24 = 6 m
4 4 4
Area of lawn = 2 × 5 [90 + 40 + (2 × 5)]
Combined areas of the squares
= 10 (130 + 10)
= 10 × 140 = 1400 m2] = (4)2 + (5)2 + (6)2

TS
24. Ans. (b) = 16 + 25 + 36
Sol. Let l and b be the length and breadth = 77 m2
of the rectangle. So, area of the rectangle = 77 m2
Let a be the side of the square. Length of the rectangle = 11 m (given)

H
Perimeter of square is 4a = 80 77
Breadth of the rectangle = = 7 m.
80 11

G
a= = 20 cm 26. Ans. (b)
4
Sol. Area of rectangular garden
Area is a2 = 20 × 20 = 400 cm2
Perimeter of rectangle
= 2 × (l + b) = 80 cm U = 10 × 20 = 200 m2
Let the width of the ‘walk’ be x m
O
l + b = 40 ... (i) 20
Area of rectangle (l × b)
H
= 400 – 100 = 300 cm2 ... (ii) 96 10
Now that the area of the rectangle is
known, a look at the given options will
_T

help you in getting the correct answer.


Then the area of the flower garden will be
15 × 35 ≠ 300; reject (a)
30 × 10 = 300; (b) is correct (20 – 2x) × (10 – 2x) = 96
i.e. 200 – 40x – 20x + 4x2 = 96
C

15 × 25 ≠ 300; reject (c)


28 × 12 ≠ 300; reject (d) 200 – 60x + 4x2 – 96 = 0
PS

Option (b) is correct. 4x2 – 60x + 104 = 0


[However, to confirm— Dividing by 4 throughout, we get
From equation (ii) x2 – 15x + 26 = 0
300
U

b = (x – 13) (x – 2) = 0
l x = 13 or x = 2
Putting the value of b in equation (i),
@

we get x =2
(  x cannot be 13 in the garden of
300
l+ = 40 given dimensions.)
l
2
l + 300 = 40l [Alternatively, you can check out the
l 2 – 40l + 300 = 0 options and get your answer.
l 2 – 10l – 30l + 300 = 0 The length (l) of inner rectangle is the
l (l – 10) – 30 (l – 10) = 0 length of the outer rectangle (L) –2 [width
(l – 30) (l – 10) = 0 (w) of path]
l = 30 cm The breadth of outer rectangle (B) –2
Putting the value of l in equation (i),
[width (w) of path]
30 + b = 40
b = 40 – 30 You have to see which of the options fit
b = 10 cm.] the picture
l = 30 cm and b = 10 cm.] L = 20; B = 10
Unit Five : Practice Session ✫ 391

Area of inner rectangle, l × b = 96 30. Ans. (d)


What should 20 – 2w and 10 – 2w be to Sol. Let original length = x and
get l × b = 96 original breadth = y.
Only option (b) fits. Decrease in area
(a) (20 – 1) × (10 – 1) = 19 × 9 ≠ 96 100 − 20 100 − 10
(b) (20 – 4) × (10 – 4) = 16 × 6 = 96 = xy – x× ×y
(c) (20 – 6) × (10 – 6) = 14 × 4 ≠ 96 100 100
(d) (20 – 5) × (10 – 5) = 15 × 5 ≠ 96] 80

90
y
= xy –
27. Ans. (c) 100 100
900

TS
Sol. Area of the ground = = 720 m2 18 7
1.25 = xy − xy = xy
25 25
Length of the ground = 30 m (given)
720 7 1
∴ Width of the ground = ∴ Decrease% = 25 xy × xy × 100 % = 28% .

H
= 24 m.
30
28. Ans. (d) 31. Ans. (a)

G
Sol. 2 (l + b) = 440 Sol. If length is l and breadth is b, the least
number of square tiles required to
l + b = 220 cover the rectangular area
b= ;
2l 2l
3 3
5l
+ l = = 220
3 U =
l×b
(H.C.F. of l and b)2
O
220 × 3
l= = 132 1517 cm × 902 cm
5
(H.C.F. of 1517 and 902)2
H
2
∴ b = × 132 = 88 (Converting metres into centimetres)
3
_T

∴ Area of the field = l × b The H.C.F. of 1517 and 902 is 41


= 132 × 88 = 11616 m2. 1517 × 902
29. Ans. (d) = 814.
41 × 41
Sol. Let the side of the square be a. Then,
C

32. Ans. (c)


the sides of the rectangle are (a + 5) and
(a – 3). Sol. The area of the inner rectangle has to
PS

Given that be found; it is area of courtyard – area


Area of the square = Area of the rectangle of black-tiled area. The area of the
(side)2 = length (l) × breadth (b) inner rectangle where l = 20, b = 10
and w = 2
U

a2 = (a + 5) (a – 3)
a2 = a2 + 5a – 3a – 15 20 m
2a = 15
@

15
a = 10 m
2
15 black
length (l) = a + 5 = + 5 tiles
2
15
breadth (b) = a – 3 = – 3 (20 × 10) – 2 × 2 [20 + 10 – (2 × 2)]
2 = 200 – 4 (30 – 4) = 200 – 104 = 96 m2
Perimeter of a rectangle = 2 (l + b)
Alternatively, area of the courtyard
15 15
= 2 +5+ −3 after paving with black tiles
2 2 = [(20 – 4) × (10 – 4)] m2
15 + 10 + 15 − 6
= 2 = 16 × 6 = 96 m2]
2 1
Area paved by green tiles = of 96
2 × 34 3
= = 34 cm. = 32 m2
2
392 ✫ Quantitative Aptitude

Area paved by yellow tiles = 96 – 32 21


= 64 m2 = × 106
(32 + 21)
Number of yellow tiles required
21
Number of tiles 64 = × 106 = 42 feet
= = = 16 m2. 53
Area of 1 tile 2×2 ∴ Area of the floor = 64 × 42
33. Ans. (a) = 2688 sq feet
Sol. Perimeter of rectangle = 2 (l + b) Cost of laying the carpet
Perimeter = 340 m (given) = Rs 2688 × 2.5
∴ 2 (l + b) = 340 = Rs 6720.

TS
Here the boundary is to be dug up 35. Ans. (b)
outside the rectangle. Sol. Total area of a circle and a rectangle
⇒ Area of the boundary to be dug = 1166 sq.cm.

H
= 2w (l + b + 2w) ∴ Diameter of the circle = 28 cm
(where w is the width of the boundary, 28
and l and b, the length and breadth r = = 14 cm

G
2
of the rectangle) Area of circle = π r2
⇒ 2 × 1 [l + b + (2 × 1)]
= 2 (l + b) + 4
= 340 + 4 [as 2 (l + b) = 340 m]
U =
22
7
× (14)2
O
= 344 m2 = 616 sq. cm
[Alternatively, area of the boundary ∴ Area of rectangle = 1166 – 616
= [(l + 2) (b + 2) – lb]
H
= 550 sq. cm
= 2 (l + b) + 4 = 344.] ⇒ Width of the rectangle
Cost of digging = 344 × 10 = Rs 3440. Area of rectangle
_T

=
34. Ans. (a) Length of rectangle
Sol. Perimeter of the floor = 2 (l + b) 550
⇒ 212 = 2 (l + b) =
25
C

⇒ l + b = 106 feet = 22 cm
∴ l : b = 32 : 21 ∴ Sum of the circumference of the circle
PS

Length of the floor (l) and perimeter of rectangle


32
= × 106 = 2π r + 2 (l + b)
(32 + 21) 22
=2× × 14 + 2 (25 + 22)
U

32 7
= × 106 = 88 + 2 (47)
53
@

= 64 feet = 88 + 94
and breadth of the floor (b) = 182 cm.
Unit Five : Practice Session ✫ 393

Practice Exercise 30
1. The area of a trapezium is 364 sq cm. If sides is 5 : 3, the length of the longer
its parallel sides are in the ratio 5 : 8 and parallel side is
the perpendicular between them is 14 cm, (a) 45 m (b) 60 m
the larger of the parallel sides is (c) 75 m (d) 120 m
(a) 26 cm (b) 28 cm
10. The length of a rectangle is 20 per cent
(c) 30 cm (d) 32 cm
more than its breadth. What will be the
2. If the area of the trapezium, whose parallel ratio of the area of the rectangle to that

TS
sides are 6 cm and 10 cm, is 32 sq cm, of a square whose side is equal to the
then the distance between the parallel breadth of the rectangle?
sides is (a) 5 : 6 (b) 6 : 5
(a) 2 cm (b) 4 cm

H
(c) 2 : 3 (d) None of these
(c) 5 cm (d) 8 cm
11. A triangle and a parallelogram are
3. The parallel sides of a trapezium are 5 cm

G
constructed on the same base such that
and 9 cm. The slant sides are 4 cm each.
their areas are equal. If the altitude of the
The area (in cm2) is
parallelogram is 100 m, then the altitude
(a) 11 3
(c) 14 3
(b) 12 3
(d) 15 3
U of the triangle is
(a) 10 2 m (b) 100 m
O
4. What is the area of a field in the form (c) 100 2 m (d) 200 m
of a trapezium, whose parallel sides are
10 m and 40 m and the perpendicular 12. A square and an equilateral triangle have
H
distance between them is 6 m? equal perimeters. If the area of the
(a) 150 m2 (b) 140 m2 equilateral triangle is 16 3 cm2, then the
_T

(c) 130 m 2
(d) 120 m2 side of the square is
(a) 4 cm (b) 4 2 cm
5. The area of a regular hexagon (in sq m)
(c) 6 2 cm (d) 6 cm
of side 4 m is
C

(a) 6 3 (b) 9 2 13. The sum of the diameter of two circles


(c) 12 6 (d) 24 3 is 35 cm and the difference in their
PS

6. The floor of a room is of size 4 m × 3 m circumference is 22 cm. The area of the


and its height is 3 m. The walls and ceiling smaller circle is
of the room require painting. The area to (a) 121 sq cm (b) 144 sq cm
be painted is (c) 154 sq cm (d) 308 sq cm
U

(a) 43 m2 (b) 33 m2 14. An athlete runs around a circular park 15


2
(c) 54 m (d) 66 m2 times. If the park’s diameter is 140 m, the
@

7. The area of a regular hexagon of side 10 distance covered by the athlete is


cm is (a) 8800 m (b) 7700 m
(a) 150 3 cm2 (b) 170 3 cm2 (c) 6600 m (d) 5500 m
(c) 175 3 cm2 (d) 190 3 cm2 15. What will be the cost of fencing of a
8. Each interior angle of a regular polygon circular park at the rate of Rs 3 per metre,
is 150°. The number of sides of the if its area is 2464 m2?
polygon is (a) Rs 528 (b) Rs 525
(a) 6 (b) 8 (c) Rs 521 (d) Rs 500
(c) 10 (d) 12 16. A cow is tied to a pole with an eight-m-
9. The area of a field in the shape of a long rope. The area of the land (in m2)
trapezium measures 1440 m 2 . The on which the cow can graze is
perpendicular distance between its parallel (a) 8π (b) 16π
sides is 24 m. If the ratio of the parallel (c) 24π (d) 64π
394 ✫ Quantitative Aptitude

17. A circular area having a radius 20 cm is (a) 8 m (b) 7 m


divided into two equal parts by a (c) 6.5 m (d) 6 m
concentric circle of radius r. The value of 27. The area of a circle is 13.86 hectares. The
r will be cost of fencing it at the rate of 60 paise
(a) 5 cm (b) 10 cm per metre is
(c) 5 2 cm (d) 10 2 cm (a) Rs 784.00 (b) Rs 788.00
18. If the radius of the circle in a plane is (c) Rs 792.00 (d) Rs 796.00
increased three times, then the area of the 28. The number of revolutions made by a
circle will be increased bicycle wheel 56 cm in diameter in covering

TS
(a) 2 times (b) 3 times a distance of 1.1 km is
(c) 6 times (d) 9 times (a) 31.25 (b) 625
19. If the circumference of a circle is 352 cm, (c) 62.5 (d) 312.5
then its area (in m2) is 29. If the diameter of a wheel is 28 cm, then

H
(a) 94.32 (b) 9432 the number of times the wheel of the
(c) 98.56 (d) 0.9856 scooter rotates in a journey of 22 km is

G
20. The area of a sector of radius 5 cm formed (a) 20000 (b) 50000
by an arc of length 3.5 cm is (c) 25000 (d) 40000
(a) 8.48 cm2 (b) 8.52 cm2
(c) 8.59 cm 2
(d) 8.75 cm2
U 30. A wheel makes 100 revolutions in covering
a distance of 88 km. The diameter of the
wheel is
O
21. The length of a rope by which a cow must
(a) 240 m (b) 400 m
be tethered in order that the cow can
(c) 280 m (d) 140 m
graze an area of 9856 cm2 is
H
(a) 52 cm (b) 56 cm 31. The largest circle that can be drawn inside
(c) 58 cm (d) 60 cm a rectangle with sides 18 cm and 14 cm
_T

will have an area of


22. The perimeter of a semicircle of 48 cm
(a) 504 cm2 (b) 252 cm2
diameter will be 2
(c) 49 cm (d) 154 cm2
(a) 24π cm (b) 12π cm
(c) 6π cm (d) π cm 32. If the ratio of the areas of two squares
C

is 225 : 256, what is the ratio of their


23. The circumference of a circle is 88 m. The perimeters?
area of the circle is
PS

(a) 3 : 4 (b) 256 : 225


(a) 616 × 104 cm2 (b) 92π × 104 cm2 (c) 26 : 25 (d) 15 : 16
(c) 64π × 102 cm2 (d) 48π × 102 cm2
33. The area of a triangle is 216 cm2. Its sides
24. A person buys a circular plot of land for are in the ratio 3 : 4 : 5. The perimeter
U

Rs 79,200 at the rate of Rs 700 per sq m. of the triangle


The radius of the plot is (a) is 12 cm (b) is 72 cm
@

(a) 5 m (b) 6 m (c) is 144 cm (d) cannot be determined


(c) 4 m (d) 11 m
34. While measuring the side of a square plot, Anil
25. A copper wire is bent in the form of a makes an error of 2 per cent in excess. What
square with an area of 121 cm2 . If the is the percentage of error in the calculated area
same wire is bent in the form of a circle, what of the square?
will be the radius (in cm) of the circle? (a) 2 per cent (b) 4 per cent
22 (c) 2.05 per cent (d) 4.04 per cent
Take π =
7 35. A circular park has a path of uniform
(a) 10 (b) 11 width around it. The difference between
(c) 7 (d) 14 the outer and inner circumferences of the
26. In a circular plot of radius 105 m, a path path is 132 m. What is the width of the
of uniform width all around its inside path?
border with an area of 4466 sq m is (a) 21 m (b) 22 m
constructed. The width of the path is (c) 25 m (d) Cannot be determined
Unit Five : Practice Session ✫ 395

Answers with Solutions


1. Ans. (d) 9−5
Then, AE = FB = = 2 cm
Sol. Let the parallel sides of the trapezium 2
be 5x and 8x In right ΔAED
a = 5x, b = 8x, h = 14 cm DE = AD2 – AE2 = (4)2 − (2)2
Area of trapezium = 364 sq. cm. (given)
1
= 16 − 4 = 12 = 2 3 cm
Area of trapezium = (a + b) h 1

TS
2 Area of trapezium = (a + b) × h
1 2
364 = (5x + 8x) 14 (where a and b are the parallel sides and
2 h the height)
1 1

H
364 = × 13x × 14 (5 + 9) × 2 3
2 2
364 = 14 3 cm2.

G
13x =
7 [Fast-track method: The height
52
x =
13
=4
U h =
2
x
s ( s − c) ( s − b) (s − x )
where x is the difference in length
O
Hence, the larger side of the trapezium
between the parallel sides and
is 8x = 8 × 4 = 32 cm.
c+d+x
s =
H
2. Ans. (b) 2
Sol. Let the distance between the parallel Here x = 9 – 5 = 4;
sides be x
_T

4 + 4 + 4 12
Parallel sides a = 6 cm and b = 10 cm s = = = 6
1 2 2
Area of trapezium = (sum of parallel
2 h = 2 6(6 − 4) (6 − 4) (6 − 4)
C

sides) × distance between them 4


1 1
= (a + b) × x = 6(2) (2) (2)
PS

2 2
1 1
Area of trapezium = × (6 + 10) × x = 3×2×2×2×2
2 2
U

32 = 8x 2×2
x = 4 cm. = 3
2
Hence, distance between parallel sides
@

= 2 3 cm
is 4 cm.
1
3. Ans. (c) Area of the trapezium = (a + b) h
2
Sol. 1
D 5 cm = (5 + 9) × 2 3
C 2
a 1
4 cm 4 cm = (14) ( 2 3 )
2
c c = 14 3 cm2.]
b 4. Ans. (a) 10 m
A E F B Sol.
9 cm
6m
Here AB = 9 cm, CD = 5 cm
and AD = BC = 4 cm
40 m
396 ✫ Quantitative Aptitude

1 9. Ans. (c)
Area of trapezium = (a + b) × h
2 Sol. Area of trapezium
where a and b are the parallel sides and 1
h is height. = (sum of parallel sides)
2
Here, a = 10 m, b = 40 m, h = 6 m × distance between them
1 1
⇒ Area = (10 + 40) × 6 = (a + b) × h
2 2
= 150 m2. Here, a = 5x, b = 3x, h = 24 m
5. Ans. (d) Area of trapezium-shaped field

TS
Sol. Area of a regular hexagon of side 4 m. 1
Area of a regular hexagon = × (5 x + 3 x ) × 24 m2
2
= 6 × Area of equilateral triangle
⇒ 1440 = (96x) m2

H
3 1440
= 6 × (side)2 ⇒ x =
4
96

G
3
=3× ×4×4 ⇒ x = 15
2
Hence, the length of the longer parallel
= 24 3 m2.
6. Ans. (c) U side = (5x) = 5 × 15 = 75 m.
10. Ans. (b)
O
Sol. We have, length = 4 m, breadth = 3 m Sol. Let the breadth be x metres. Then the
and height = 3 m length is 120% of x.
H
Area of the four walls of the room 120 6
= 2 × height (length + breadth) = x= xm
100 5 6
= 2 × 3 (4 + 3) = 42 sq. metre Area of rectangle = x × x
_T

Area of ceiling = length × breadth 5


Area of square = x × x
= 4 × 3 = 12 sq. m. Ratio of the two areas
Required area = Area of the four walls 6
C

of the room + Area = x × x: x × x


5
of ceiling
6 1
PS

= 42 + 12 = x × x×
= 54 sq metre. 5 x×x
7. Ans. (a) 6 x2 1 6
Sol. Area of regular hexagon = × 2 =
5 x 5
U

= 6 × Area of equilateral triangle


Ratio is 6 : 5.
3
= 6× (side)2
@

11. Ans. (d)


4 Sol. Let the altitude of the triangle be h1
3 and the common base be b.
= 3× × (10)2
2 Then, area of the triangle
= 150 3 cm2. = area of the parallelogram
8. Ans. (d) 1
Sol. Since interior angle is 150°, × b × h1 = b × h2
2
Exterior angle = 180° – interior angle 2 × b × h2
h1 = where h2 = 100 m
= 180° – 150° b
= 30° ⇒ h1 = 2h2 = (2 × 100) m = 200 m.
Number of sides 12. Ans. (d)
360° 360°
= = Sol. Let the side of square be x and that
exterior angle 30° of triangle be y
= 12. Perimeter of the square = 4x
Unit Five : Practice Session ✫ 397

Perimeter of the equilateral triangle = 3y Distance travelled by athlete in one


Then, round = Circumference of circular path
Perimeter of the square 22
= Perimeter of the equilateral triangle = 2 × × 70
7
∴ 4x = 3y = 440 m
3 Distance covered in 15 rounds
x = y
4 = 15 × 440 = 6600 m.
3 2
Area of equilateral triangle = y 15. Ans. (a)
4

TS
Sol. Area of circular park πr2
3 2
16 3 = 4 y 22 2
2464 = r
y = 8 7
2
22r = 2464 × 7

H
3
∴ x = y 2464 × 7
4 r2 =
22

G
3
and x = × 8 = 6 cm. ∴ r2 = 784
4
r = 784
Hence, side of square is 6 cm.
13. Ans. (c)
U r = 28 m
Perimeter of circular park = 2πr
O
Sol. The sum of the diameters of two circles 22
= 35 cm = 2 × × 28
7
H
2R + 2r = 35 = 176 m
35 Rate of fencing is Rs 3 per metre
R+r = … (i) So, cost of fencing the park = 176 × 3
_T

2
Difference between their circumference = Rs 528.
16. Ans. (d)
= 22 Sol. Area of land on which the cow can graze
C

2πR – 2πr = 22 would be a circle round the pole


2π (R – r) = 22 = πr2
PS

22 = π(8)2
R–r = = 64π m2.

22 17. Ans. (d)
R –r = ×7
2 × 22 Sol.
U

7 r
R–r = … (ii)
@

2
By solving (i) and (ii), we get From the given condition, we get,
21 area of inner circle
R = and r = 7
2 1
Hence, the area of the smaller circle = πr2 = × area of outer circle
2
= π(7)2 1
πr2 = × π × (20)2
22 2
= × 7 × 7 r2 = 200
7
= 154 sq cm. ∴ r = 10 2 cm.
14. Ans. (c) 18. Ans. (d)
Sol. Distance = No. of revolutions × 2πr Sol. Let the radius of circle be x
Diameter of park = 140 m Area of circle = πr2 = π (x)2
Radius = 70 m If radius is increased three times,
398 ✫ Quantitative Aptitude

then new area = π (3x)2 Area of the circle = π r2


= 9πx2 22
∴ Hence, the area of circle will be increased = × 14 × 14 = 616 m2
7
9 times.
[  1 m2 = 10000 cm2]
19. Ans. (d)
Hence 616 m2 = 616 × 104 cm2.
Sol. Circumference of circle = 352 cm
24. Ans. (b)
Circumference of circle = 2πr
Sol. Total cost of plot = Rs 79200
2 π r = 352
352 Rate of land per sq m. = Rs 700

TS
r = = 56 cm
2π Area of the plot =
Total cost
22 Rate
Area of circle = πr2 = × 56 × 56
7 79200 792
=

H
= 9856 cm2 = sq m.
700 7
= 0.9856 m2. 792

G
20. Ans. (d) π r2 = sq m.
7
1 2 arc 22 792
r × × r2 =
Sol. Area of sector =
1
2 r
U 7
r2 =
7
792
×
7
O
= × r × arc
2 7 22
1 r2 = 36
= × 5 × 3.5
H
2 r = 6 m.
= 8.75 cm2. 25. Ans. (c)
_T

21. Ans. (b)


Sol. We have, area of square = 121 cm2
Sol. Area of circle = 9856
πr2 = 9856 Side of square = Area of square
22 = 121 = 11 cm
× r2 = 9856
C

7 7 Length of wire = 4 × side of square


r2 = 9856 ×
22 Length of wire = 4 × 11 = 44 cm
PS

2
r = 3136 ∴ 2 π r = 44
r = 3136 22
r = 56 cm. 2 × × r = 44
7
U

22. Ans. (a) 44 × 7


Sol. Perimeter of semicircle = π r r =
2 × 22
@

Here, semicircle of diameter = 48 cm


= 7 cm.
48
r = 26. Ans. (b) Path
2
Perimeter of semicircle of 48-cm diameter
Sol.
48
= π Area of outer circle = πr2 105m
2
= 24π cm. 22
= × 105 × 105
23. Ans. (a) 7
Sol. Circumference of a circle = 88 m = 34650 sq m
Circumference of a circle = 2πr Area of path = 4466 sq m
2 π r = 88 ∴ Area of inner circle = 34650 – 4466
88 × 7 π r2 = 30184 sq m
r = = 14 m 30184
2 × 22 r2 = sq m
π
Unit Five : Practice Session ✫ 399

r2 = 9604 ∴ Wheel can cover the journey of 22 km in


r = 9604 22
rounds
r = 98 m 0.00088
Radius of inner circle is 98 m = 25000 rounds.
Hence, width of path is 105 – 98 = 7 m. 30. Ans. (c)
27. Ans. (c) Sol. Distance travelled in 1 revolution
Sol. Area of circle = πr2 88 × 1000
=
13.86 = π r2 100

TS
22 = 880 m
13.86 = × r2
7 Circumference of the wheel = 2πr
13.86 × 7 880 = 2πr
r2 =

H
22 ∴ 2r = 280 m
4.41 = r2 Hence, diameter of the wheel is 280 m.
r = 2.1

G
31. Ans. (d)
Perimeter of circle = 2πr Sol. The largest circle inside the rectangle
22
can have maximum diameter equal to
= 2 ×
= 13.2
7
× 2.1

U the smaller side of the rectangle. So


the diameter of the circle here will be
O
= 1320 m
14 cm, and radius 7 cm.
Cost of fencing per metre = 60 paise
22
The cost of fencing = 1320 × 0.60 Area = πr2 = × 7 × 7
H
= Rs 792. 7
= 154 cm2.
28. Ans. (b)
_T

Sol. Bicycle wheel of diameter = 56 cm 32. Ans. (d)


56 Sol. If the sides of the two squares are a
Then radius = cm and b, the ratios of their area
2
C

a 2 225 (15)2
Distance covered by wheel in one = =
revolution b2 256 (16)2
PS

a 15
= 2πr =
56 b 16
= 2π ×
2 Perimeters are 4a and 4b
= 176 cm = 0.00176 km
U

4a
The distance of 1.1 km is covered by the Ratio will be
4b
@

wheel in
4 × 15 15
1.1 = =
= 625 revolutions. 4 × 16 16
0.00176
29. Ans. (c) [Fast-track solution: If the ratio of the
28 area of two squares is a : b, the ratio
Sol. Here r = cm of their perimeters (and sides and
2
Perimeter of wheel = 2πr diagonals) will be in the ratio a : b
28 Here a : b = 225 : 256
2π 225 15
2 or = ]
= 28π = 88 cm 256 16
33. Ans. (b)
In one rotation, wheel covers 88 cm
Sol. If the ratio of sides is 3 : 4 : 5, the
= 0.00088 km [ 1 km = 10000 cm] triangle must be right-angled.
400 ✫ Quantitative Aptitude

Let the sides be 3x, 4x and 5x, with Actual area = 100 × 100
5x as hypotenuse. = 10000 sq cm.
1 1 Difference = 10404 – 10000
Area = × 4x × 3x = × 12x2 = 6x2 = 404 sq m.
2 2
Now 6x2 = 216 cm2 404
216 Percentage error = × 100 = 4.04%.
x2 = = 36 cm2 10000
6 [Alternative (fast-track) solution: The
x = 36 = 6 cm problem can be reduced to finding
per cent change in area if there is a

TS
Perimeter = 3x + 4x + 6x
= (3 × 6) + (4 × 6) + (5 × 6) change in any defining dimension of
= 18 + 24 + 30 a plane figure.
= 72 cm. x
The per cent change = x 2 + %

H
[Alternative solution: Use the formula 100
Area = s ( s − a ) ( s − b) ( s − c ) where x is the change in dimension.

G
The change in measurement here due
Here a = 3x, b = 4x, c = 5x
to error is 2%.
3x + 4x + 5x


s =

Area =
2
= 6x

6x (6x − 3x ) (6 x − 4x) (6x − 5x) U Per cent change in area

= 2 2+
2
%
O
100
= 200 + 2
6 x × 3x × 2 x × x = 2
H
100
= 36x 4 202
= 2
_T

= 6x2 100
2
6x = 216 (Given area) 404
216 = = 4.04.]
x2 = = 36 100
6
C

35. Ans. (a)


x = 36 = 6 cm. Sol. We know, if the radius of the outer
circumference is R and the radius of
PS

Perimeter = 3x + 4x + 5x
= 18 + 24 + 30 = 72 cm.] the inner circumference is r,
34. Ans. (d) 2πR – 2πr = 132
Sol. If the side of the sphere is 100 cm, 2π (R – r) = 132
U

an error of 2% means the side is 132


R – r (which is the width) =
measured as 102 cm. 2π
@

Wrongly calculated area 132 7


=102 × 102 = 10404 sq cm = × = 21 m.
2 22
Unit Five : Practice Session ✫ 401

Practice Exercise 31
1. Circle A has a radius of 3 cm. Two circles If a semicircular portion with BC as
B and C have a radius each equal to the diameter is cut off, then the area of the
diameter of circle A. The radius of a circle remaining paper is
D which has an area equal to the total (a) 78.45 sq cm (b) 78.55 sq cm
area of A, B and C is (c) 78.65 sq cm (d) 78.75 sq cm
(a) 9 cm (b) 12 cm 9. Find the area of a square that can be
(c) 15 cm (d) 18 cm inscribed in a circle of radius 8 cm.

TS
2. If Δ ABC is an equilateral triangle of area (a) 127 cm2 (b) 128 cm2
2
36 3 cm2, then the area of the inscribed (c) 129 cm (d) 130 cm2
circle is 10. Semicircular lawns are attached to all
(a) 48π cm2 (b) 36π cm2

H
sides of a rectangular field measuring
(c) 24π cm2 (d) 12π cm2 42 m × 35 m. The area of the total field

G
3. If the difference in the areas of the region is
bounded by the inscribed and the (a) 3379.4 m2 (b) 3641.8 m2
2
circumscribed circles of a square is 9π, (c) 3818.5 m (d) 3987.2 m2
then the area of the square will be (in
square units) U 11. If a square is inscribed in a circle, the ratio
of the areas of the circle and the square
O
(a) 6π (b) 5π is
(c) 25 (d) 36 (a) π : 1 (b) π : 2
H
4. A wire is in the form of a circle of radius (c) 2 : 1 (d) 11 : 7
42 cm. If it is bent into a square, then the 12. If the perimeter of an equilateral triangle
_T

side of the square will be is equal to the circumference of a circle,


(a) 60 cm (b) 62 cm then the ratio of their areas is
(c) 64 cm (d) 66 cm (a) π : 3 (b) π : 3
5. If a circle is inscribed in an equilateral (c) π : 9 (d) π : 3 3
C

triangle of side 5 cm, then the radius of 13. The perimeters of a circular field and
the circle will be
PS

square field are equal. If the area of the


5 3 square field is 12100 m2, the area of the
(a) cm (b) 5 3 cm
6 circular field will be
5 5 3 (a) 15500 m2 (b) 15400 m2
U

(c) cm (d) cm (c) 15200 m 2


(d) 15300 m2
6 2
6. A copper wire when bent in the form of 14. A square is inscribed in a circle of radius
@

a square encloses an area of 121 cm2. If 8 cm. The area of the portion between the
the same wire is bent in the form of a circle and the square is
circle, then the area of the circle will be (a) 48.27 m2 (b) 73.14 m2
(a) 121 cm2 (b) 242 cm2 (c) 169.14 m2 (d) 88.26 m2
2
(c) 154 cm (d) 60 cm2 15. A horse is placed for grazing inside a
7. A wire in the form of a circle of radius square field 12 m long and is tethered to
98 cm is cut and bent in the form of a one corner by a rope 8 m long. The area
square. The side of the square thus formed it can graze is
is (a) 50.08 m2 (b) 50.28 m2
2
(a) 146 cm (b) 152 cm (c) 50.48 m (d) none of the above
(c) 154 cm (d) 156 cm 16. The perimeter of a square whose area is
8. A paper is in the form of a rectangle equal to that of a circle with perimeter
ABCD where AB = 14 cm and BC = 7 cm. 2πx is
402 ✫ Quantitative Aptitude

(a) 2πr (b) π x 23. The area of the shaded portion is


(c) 4 π x (d) 4x π
17. The diameter of a circle circumscribing a 1 cm 2 cm
square is 10 cm. The sides of the square 2cm
will be
(a) 5 cm (b) 5 2 cm
(c) 10 cm (d) 10 2 cm 4 cm
(a) 8 sq cm (b) 6 sq cm
18. A wire is in the form of an equilateral (c) 16 sq cm (d) 4 sq cm

TS
triangle with area 5 m2. If it is changed
into a circle, the radius will be 24. The area of the shaded portion is
3
3 20 4
(a) m (b) 3 20 m 2 cm

H
2π 2π 4 cm
3 × 20 2 cm

G
(c) m (d) none of the above

4 cm
19. A figure consists of a square of side x
metre with semicircles on the outside of
the square. The area (in m2) of the figure U (a) 16 sq cm
(c) 8 sq cm
(b) 12 sq cm
(d) 4 sq cm
O
so formed will be 25. The area of the figure is
(a) x2 (b) x2 + 2πx2
H
90°
πx 2
2
(c) 4πx 2
(d) x + r
2
_T

20. If the area of a circle be equal to that of


a square, then the ratio of the side of the 3 2 1 2
(a) πr (b) πr
square to the radius of the circle is 4 4
C

(a) π :1 (b) 1 : π (c) π r2 (d) 3π r2


(c) 1 : π (d) π : 1 26. The area of the unshaded portion of the
PS

figure is 14 m
21. If a wire is bent into the shape of a square,
then the area of the square is 81 sq. cm.
When the wire is bent into a semi-circular
U

shape, then the area of the semi-circle will 14 m


be
@

(a) 22 cm2 (b) 44 cm2


2
(c) 77 cm (d) 154 cm2
22. The area of the smallest circle containing (a) 128.3 m2 (b) 142.8 m2
a square of side a is (c) 150.5 m2 (d) 157.5 m2
a
27. In the given figure, the area of the shaded
region (in cm2) is
a a

a
14 cm

πa π a2
(a) (b)
2 2 28 cm
2
π (a) 324 (b) 428
(c) 2π a (d) a
2 (c) 462 (d) 500
Unit Five : Practice Session ✫ 403

28. In the figure M, N, P are the midpoints 32. What is the perimeter of the shaded
of the sides of the rectangle ABCD. If the portion?
sides of the rectangles are 14 cm and 8
cm, the area of the shaded portion is
A M
B
N

D C
P
2
(b) 54 cm2

TS
(a) 50 cm
(a) 100 π (b) 50 π
(c) 56 cm2 (d) 58 cm2
(c) 100 (d) 50
29. 33.
L x

H
gravel
Lawn y pathway
90°

G
N A rectangular plot of lawn shown in the
M

U figure has dimensions x and y and is


surrounded by a gravel pathway of width
O
2 m. What is the total area of the pathway?
(a) 2x + 2y + 4 (b) 2x + 2y + 8
If in the figure given above, MN = x, and
H
(c) 4x + 4y + 8 (d) 4x + 4y + 16
each side has a semicircle drawn on it,
what is the area of the shaded region? 34. Four equal circles of diameter 4 m touch
_T

π x2 π x2 at four points as shown in the figure.


(a) (b)
2 4
(c) π x2 (d) 4π x2
C

30. Three circles of radius 3.5 cm each are


What is the area of the shaded portion (in
placed in such a way that each circle
PS

square m.)?
touches the other two. What is the area π
(a) 8 − (b) 16 – 4π
of the portion enclosed by the circles? 4
π
(a) 21 cm2 (c) 8 − 4π (d)
U

(b) 19.67 cm2 4


(c) 1.96 cm2 35. Consider the following figure and answer
the question that follows:
@

(d) Not possible to estimate


31. The length and width of rectangle AEFG 15
are each 2/3 of the corresponding parts
of ABCD. AEB =12; AGD = 6. What is the 48
area of the shaded part?

A square is divided into four rectangles


as shown above. The lengths of the sides
of rectangles are natural numbers. The
areas of two rectangles are indicated in
the figure. What is the length of each side
of the square?
(a) 48 (b) 40 (a) 10 (b) 11 (c) 15
(c) 36 (d) 24 (d) Cannot be determined as the given
data are insufficient
404 ✫ Quantitative Aptitude

Answers with Solutions


1. Ans. (a) [Alternative solution (After calculating
Sol. Area of circle A of radius 3 cm the side of the triangle).
= π(3)2
Now, area of ΔABC = Area of ΔOBA
= 9π sq cm
Area of circle B of radius 6 cm + Area of ΔOBC + Area of ΔOAC
= π(6)2 1 1 1
= 36π sq cm 36 3 = 2 AB × r + 2 BC × r + 2 AC × r
As B and C have equal radii, 1

TS
Area of circle C = 36π sq cm 36 3 = 2 r (AB + BC + AC)
Area of circle D = Total area of A, 1
B and C = r (12 + 12 + 12)
2
πr2 = 9π + 36π + 36π

H
36 3 × 2
πr2 = 81π ∴ r =
36
r2 = 81

G
= 2 3 cm
∴ r = 9 cm.
The radius of circle D is 9 cm. ∴ Area of the circle inscribed = πr2
2. Ans. (d)
Sol. U = π (2 3)
2

= 12π cm2.]
O
A
3. Ans. (d)
Sol. Let the side of the square be a.
H
F r r E Then, its diagonal = 2a
O Radius of circumcircle
_T

r 1
B C = diagonal of square
D 2
3 2
Area of equilateral triangle = a 1
C

4 R = 2a
where a is a side. 2
a
PS

3 2 R =
36 3 = a 2
4 π a2
36 3 × 4 ∴ Area of circumcircle =
a2 = 2
U

3 Radius of inner circle


∴ a2 = 144 1
a = 12 cm = side of square
2
@

Side of the equilateral triangle is 12 cm. 1


Once you know the length of the side r = a
2
of the triangle is 12 cm, apply formula— π a2
radius (r) of incircle of an equilateral ∴ Area of inner circle =
4
a π a2 π a2
triangle of side (a) = Given that, − = 9π
2 3 2 4 a
12 6 2a2 − a2
= π
2 3 3 4 = 9π r
6 a a
If r is cm, area of the incircle R
3 a2
2 = 9
6 4
= πr2 = π 2
a = 36
a
3
36 Hence, area of square is 36 square units.
= π = π 12 cm2.
3
Unit Five : Practice Session ✫ 405

[Alternative (Fast-track) solution: and denominator by 3 , we get


Remember that if x is the radius of
5 3
the inscribed circle and R the radius r = ×
of the circumcircle of a square of side 2 3 3
a, 5 3 5 3
a = =
r = 2× 3 6
2 Option (a) matches this result.
a
and R = 6. Ans. (c)
2 Sol. Area of square = 121

TS
The areas of the two circles will be (Side)2 = (11)2
πr2 and πR2. Side = 11
We are given that Perimeter of square = Perimeter of circle
πR2 – πr2 = 9π

H
4 × side = 2π r
π(R – r2) = 9π
2
22
2 2 4 × 11 = 2 × × r

G
a a 7
π − = 9π
2 2 44 × 7
r =
a2 a2
2

4
=

π U 44
∴ Radius of circle = 7 cm
Area of circle = π r2
O
2a2 − a2 22
= 9 = × 7 × 7
H
4 7
2
a = 4 × 9 = 36 = 154 cm2.
∴ The area of the square is 36 sq. units.] 7. Ans. (c)
_T

4. Ans. (d) Sol. Perimeter of circle = Perimeter of square


2 π r = 4 × side
Sol. Perimeter of circle = 2πr 22
22 2 × × 98 = 4 × side
C

= 2 × × 42 [  r = 42 cm] 7
7 2 × 22 × 98
= 264 cm ∴ Side of square =
7×4
PS

From a circular wire, a square is formed. = 154 cm.


Hence, 8. Ans. (d)
Perimeter of circle = Perimeter of square D C
Sol.
U

264 = 4a
264 7 cm
a =
@

4
a = 66 cm. A 14 cm B

∴ Side of the square = 66 cm. Area of shaded portion


5. Ans. (a) = Area of rectangle – Area of semicircle
1 2
Sol. If a is the side of the equilateral triangle = length × breadth – πr
2
and r the radius of the inscribed circle, Here, length = 14 cm, breadth = 7 cm
a diameter 7
r = r = = cm
2 3 2 2
1 22 7 7
5 = 14 × 7 – × × ×
r = 2 7 2 2
2 3
= 98 – 19.25
None of the options gives this.
However, if we multiply numerator = 78.75 sq cm.
406 ✫ Quantitative Aptitude

9. Ans. (b) 11. Ans. (b)


A r= B
Sol. 8 Sol. Let a be the side of a square.
cm
Then, radius r of circle is given by
O 2 2
a a
r = +
D C 2 2
Let each side of square be a cm. a2 a2 2a2
Then, in ΔADC = + =
4 4 4
AC2 = AD2 + DC2

TS
16)2 = a2 + a2 a 2 a
= =
256 = 2a2 2 2
128 = a2 Hence, ratio of area of circle and area
∴ Area of square = a2 = 128 cm2.

H
of square
[Alternative solution (Fast-track): As 2

π
a π a2
the circle is circumscribed on the
πr 2
π

G
2
square, its radius = 2 = = 22 =
a a2 a 2
a
R = = π : 2.
2
where a is the side of the square. U [Alternative solution (fast-track): The
area of a square inscribed in a circle
O
a of radius r is 2r2.
8 =
2 So, area of circle : area of square
H
a = 8 2 = πr2 : 2r2
Area of the square = a2 πr 2 π
_T

= (8 2 )2 = :
2r 2 2
= 64 × 2 = 128 cm2] The ratio is π : 2.]
10. Ans. (c) 12. Ans. (d)
Sol. Sol. Perimeter of equilateral triangle
C

A B = Circumference of a circle
= 3 × side = 2πr
PS

3a = 2 π r
3a
D C r =

U

∴ Ratio of area of equilateral triangle to


Area of rectangle = length × breadth
area of circle
@

Area of semicircle of radius r


πr 2 3 2 3 2
=
2 = 4a = 4a
Area of the given field πr2 3a
2

π
= Area of rectangle + area of 2 2π
3 2
small semicircles + area of 2 4a
big semicircles =
9a2
2 π× 2
1 22
× ×
35 4π
= 42 × 35 + 2 2 7 2 = π : 3 3.
2
1 22 42
+2 × × 13. Ans. (b)
2 7 2 Sol. Perimeter of circular field
= 1470 + 1386 + 962.5 = Perimeter of square
= 3818.5 m2. 2πr = 4 a
Unit Five : Practice Session ✫ 407

Given that area of square = 12100 π r 2θ 12 m


a2 = 12100 =
360
∴ a = 110 m
Hence, 2πr = 4 × 110 Here, r = 8 m, θ = 90
π × 8 × 8 × 90 8
440 440
r= = = 70 m 360 θ
2π 22
2× 22
7 = 16 × 2
= 50.28 m . 8m
7
Area of circular field = πr2 16. Ans. (d)

TS
22 Sol. Here, perimeter of the circle = 2πx
= × 70 × 70
7 2πr = 2πx
= 15400 m2. r = x
[Alternative (fast-track) solution: If Side of square = a

H
the perimeters of a square and a circle ∴ Area of square = Area of circle
are the same, the area of the circle is (a)2 = π(x)2

G
given by a2 = πx2
4A a = x π
π
sq. units
where A is the area of the square. U Perimeter of the square = 4 × side

17. Ans. (b)


= 4x π .
O
Here the area of the square = 12100 m2 Sol. Diameter of the circle
4 × 12100 2 = Diagonal of the square
H
Area of the circle = m
π If a is the side of the square, 10 = 2a
7 10
= 4 × 12100 × ∴ a = = 5 2 cm.
_T

22 2
= 15400 m2.] The side of the square = 5 2 cm.
14. Ans. (b) 18. Ans. (b)
Sol. We have to find the area of the shaded
C

3 2
portion. Sol. Area of an equilateral triangle = a
3 2 4
PS

a = 5
4
2
5×4
8 cm a =
3
U

20 20
a = = 1
3 34
@

Length of the wire = 3 × side


Diagonal of the given square = 16 cm 3 20
(as the diameter of the circle = diagonal = 3a = 1
of the square) 34
If it is changed into circle.
The area of the shaded portion = area
Then,
of circle – area of square
Perimeter of circle = length of the wire
22 162
× (8)2 −
3
= 2πr =
7 2 34 × 20
3
2
= 73.14 cm . 3 × 20 4
15. Ans. (b) ∴ r = m.
2π 3
Sol. Area which can be grazed is the area
3 4 × 20
of the sector of a circle Radius of the circle = metre.

408 ✫ Quantitative Aptitude

19. Ans. (d) a a


∴ Area of circle = π × ×
Sol. 2 2
A B
a2
x = π .
2 2
23. Ans. (b)
x
2 Sol. D C
x E H
D C 1 cm 2 cm
2cm

TS
Area of the figure F G
A B
= area of square + area of four semicircles 4 cm
1 x2 Area of rectangle ABCD = AB × BC
⇒ x2 + 4 × π ×

H
2 4 = 4 × 2 = 8 sq cm
π x2 Area of rectangle EFGH = EF × FG
= x2 + .
= 1 × 2 = 2 sq cm

G
2
20. Ans. (a) ∴ Area of shaded portion = (8 – 2) sq cm
Sol. Let the radius of the circle be r and side = 6 sq cm.
of the square be a.
Then, U 24. Ans. (b)
Sol. A B
O
Area of the square = Area of the circle
a2 = πr2 2 cm
a = πr 4 cm C D
H
a π 2 cm
=
_T

r 1 G E
F
a : r = π : 1. 4 cm
21. Ans. (c) GE = AB + CD
Sol. Area of the square = 81 cm
C

Length of each side of the square AB = GE – CD


= 81 cm = 9 cm = 4 – 2
PS

Length of wire= perimeter of square = 2 cm


= (4 × 9) cm = 36 cm Area of ABCDEFG
Perimeter of semicircle = Area of ABFG + Area of CDEF
U

= πr + 2r or r (π + 2) = 4 × 2 + 2 × 2
Here perimeter is 36 cm = 8 + 4
36 = r (π + 2)
@

= 12 sq cm.
36
⇒ r = = 7 cm 25. Ans. (a)
22
+2 Sol.
7 1 2
Area of semi-circle = π r 90°
2
1 22 r
= × ×7 ×7
2 7
= 77 cm2.
22. Ans. (b) From the figure it is clear that one-
Sol. Let the side of the square be a. fourth of the circle has been removed.
Then, its diagonal = 2a So we need the area of the remaining
1 a 3
Radius of circle = × 2a = portion which is πr2.
2 2 4
Unit Five : Practice Session ✫ 409

26. Ans. (d) 29. Ans. (b)


14 m
Sol. Sol.
L

14 m 90°

x
M N

Side of the square = 14 m

TS
= 2 × diameter of a semicircles
Diameter of one semicircle = 7 m = 2r
Radius = 3.5 m LM2 + LN2 = MN2 = x2
Area of unshaded region

H
Shaded area
= Area of square
2 2 2
– area of two semicircles 1 LM LN MN
= 2π + +

G
1 22 2 2 2
= 14 × 14 – [ 2 × × × (3.5)2]
2 7
1 π
= 196 – 38.5
= 157.5 m2. U = × (LM2 + LN2 + MN2)
2 4
π
O
27. Ans. (c) = (MN2 + MN2)
8
Sol. Area of the bigger semicircle π
H
= (2MN2)
1 22 8
= × × 14 × 14
π π x2 .
_T

2 7
= 308 cm2
= × 2x 2 =
8 4
Area of the two smaller semicircles
= Area of 1 circle 30. Ans. (c)
C

Sol. With the three circles touching are


22
= × 7 × 7 = 154 cm2 another, we will have a figure:
7
PS

∴ Total area of shaded region


= 308 + 154 3.5 cm
= 462 cm2.
U

3.5 cm 60°
28. Ans. (c) 3.5 cm
A M
Sol. B
@

60° 60°
N 8
D C
P
14
AN = ND = 4 cm, BC = 8 cm —and we have to find the area of the
shaded portion.
AM = DP = PC = 7 cm
Area of ΔMAN = Area of ΔPDN The required area
1 = (Area of equilateral triangle of side
= × 4 × 7 = 14 cm2
2 7 cm) – (3 × area of sector with angle
1
Area of ΔBCP = × 7 × 8 = 28 cm2 = 60° and r = 3.5 cm)
2
∴ Area of shaded portion 3 22 60
= 14 × 8 – (14 + 14 + 28) = ×7 ×7 − 3× × 3.5 × 3.5 × cm2
4 7 360
= 112 – 56 = 56 cm2.
410 ✫ Quantitative Aptitude

It is seen that the area of the shaded


49 3 portion is area of square – area of one
= − 11 × 0.5 × 3.5 cm2
4 circle (four quarters of circles of equal
= (21.217 – 19.25) cm2 = 1.967 cm2. radius).
Thus the area of shaded portion
[Alternative (Fast-track) solution: If three 4
2
circles of equal radius r touch one another, = (4)2 – π
the area included between them may be 2
4 2 = 16 – 4π m2
taken as r. [Alternative method
25

TS
4 Area of shaded portion
Here the shaded portion = × 3.5 × 3.5
25 = (4 – π) (radius)2
= 1.96 cm2.] 2
4
= (4 – π)

H
31. Ans. (b) 2
2 = (4 – π) (4)
Sol. AEB = 12, ∴ AE = 12 × =8 = 16 – 4π m2
3

G
2 (If the answer options did not have
AGD = 6, ∴ AG = 6 × =4 6 2
3
π, the formula to use would be
Area of AEFG = AE × AG = 8 × 4 = 32
Area of ABCD = AB × AD = 12 × 6 = 72
U 35. Ans. (b)
7
r .]
O
Area of shaded part Sol. Now the rectangle of area 15 may
= Area of ABCD – Area of AEFG have sides of 15 and 1, or 5 and 3.
= 72 – 32 = 40. The rectangle with area 48 may have
H
32. Ans. (a) sides of 48 and 1, 24 and 2, 12 and
4, 16 and 3, or 6 and 8. However, we
Sol. From the given figure, the perimeter
_T

of the shaded figure is formed by the need sides for the two rectangles to
curve of four circle quadrants add up to the sides of the square of
Perimeter of four circle quadrants which they are parts; as the sides of
C

2π × 50 a square are equal, the sides of the


= × 4 = 100 π . two given rectangles must add up to
2
PS

33. Ans. (d) give the same sum. This happens only
if we take the sides to be
Sol. Total area of the pathway
5 × 3 = 15
= Area of the plot – Area of the lawn
and 6 × 8 = 48
U

= (x + 2 + 2) × (y + 2 + 2) – xy Now 5 + 6 = 11
= (x + 4) × (y + 4) – xy = 4x + 4y + 16. and 3 + 8 = 11
@

34. Ans. (b) 5 6


Sol. If you join the midpoints of the circles 3 15 3
you get a square as shown below, with
each side = 4 m.

8 48 8

5 6
So, the length of each side of the
square is 11.
Unit Five : Practice Session ✫ 411

Practice Exercise 32
1. A box with a square base of side 50 cm (a) 2 : 1 (b) 1 : 2
has a volume of 15,000 cm3. Its height is (c) 4 : 1 (d) 1 : 4
(a) 25 cm (b) 30 cm 11. The percentage increase in surface area of
(c) 50 cm (d) 75 cm a cube when each side is doubled is
2. A solid wooden box has dimensions 4 m, (a) 100% (b) 200%
3 m and 12 m. Its surface area is (c) 300% (d) 400%

TS
(a) 195 m2 (b) 194 m2 12. A cube of wood was bought for Rs 768.
2
(c) 193 m (d) 192 m2 If the wood costs Rs 1500 per cubic metre,
3. How many cubic cm of wood are there the length of each edge of the cube is

H
in a box which measures 24 cm by 22 cm (a) 80 cm (b) 85 cm
by 17 cm, with the thickness of the wood (c) 90 cm (d) 95 cm
being 1 cm?

G
13. Two cubes each of volume 64 cm3 are
(a) 2376 cm3 (b) 2296 cm3
3 joined end to end. The surface area of the
(c) 2367 cm (d) 2276 cm3
4. The surface area of a cube is 1176 cm2.
The volume of this cube is U resulting cuboid is
(a) 160 cm2
(c) 158 cm 2
(b) 159 cm2
(d) 157 cm2
O
(a) 1331 cm3 (b) 1728 cm3
3 14. An underground water tank is in the form
(c) 2197 cm (d) 2744 cm3
H
of a cuboid of edges 48 m, 36 m and 28
5. The volume of a cube is 15625 cm3. Its m. Find the volume of the tank.
edge is (a) 48384 m3 (b) 47321 m3
_T

3
(a) 15 cm (b) 25 cm (c) 45789 m (d) 44017 m3
(c) 35 cm (d) 45 cm
15. The length, breadth and height of a
6. The diagonal of a cube is 27 3 cm. Its rectangular box are in the ratio 5 : 4 : 2.
C

volume is If its total surface area is 1216 cm2, then


(a) 24389 cm3 (b) 19683 cm3 its length is
PS

3
(c) 4096 cm (d) 2197 cm3 (a) 10 cm (b) 20 cm
7. The length of the longest rod that can fit (c) 30 cm (d) 40 cm
in a cubical room of 4 m side is 16. Six cubes, each with 10 cm edge, are
U

(a) 7 m (b) 6.92 m joined end to end in a row. The surface


(c) 6.51 m (d) 5.99 m area of the resulting cuboid is
@

8. Three metal cubes whose edges are 3 cm, (a) 2600 cm2 (b) 2400 cm2
2
4 cm and 5 cm respectively are melted and (c) 1400 cm (d) 600 cm2
formed into a single cube. Its edge is 17. The volume of a cube of 60 cm side is
(a) 16 cm (b) 12 cm the same as that of a cuboid, one of whose
(c) 6 cm (d) 4 cm sides is 36 cm. If the ratio of the other
9. Three cubes each of edge 10 cm are joined two sides is 15 : 16, then the longest side
end to end. The surface area of the of the cuboid is
resulting cuboid is (a) 60 cm (b) 75 cm
(a) 6000 cm2 (b) 8140 cm2 (c) 80 cm (d) 90 cm
2
(c) 1000 cm (d) 1400 cm2 18. If the total length of diagonals of a cube
10. If the volumes of two cubes are in the is 12 cm, what is the total length of edges
ratio 8 : 1, the ratio of their edges is of the cube?
412 ✫ Quantitative Aptitude

(a) 15 cm (b) 12 cm (a) 72 cm2 (b) 216 cm2


(c) 6 3 cm (d) 12 3 cm (c) 432 cm2 (d) 108 cm2
19. If the diagonal of a cube is 3 3 cm, then 25. The perimeter of one face of a cube is 20
its volume is cm. Its volume must be
(a) 3 c.c. (b) 3 c.c. (a) 8000 cm3 (b) 1000 cm3
3
(d) 400 cm3
( 3) ( ) (c) 125 cm
3 2
(c) c.c. (d) 3 3 c.c.
26. Two cubes each with 8 cm edge are joined
20. If three metal cubes whose edges are 30
end to end. The surface area of the

TS
cm, 40 cm and 50 cm respectively are
resulting cuboid is
melted and formed into a single cube, the
(a) 560 cm2 (b) 640 cm2
total surface area of the single new cube 2
(c) 830 cm (d) 1440 cm2
is

H
(a) 2 m2 (b) 2.15 m2 27. A cube of side 6 cm is cut into a number
2
(c) 2.16 m (d) 2.2 m2 of cubes each of side 2 cm. The number

G
of cubes obtained will be
21. If a 20 × 20 m hall is such that the sum
(a) 6 (b) 7
of the areas of the floor and roof is equal
to that of the four walls, then the volume
of the hall is U (c) 12 (d) 27
28. A wooden box of dimension 8 m × 7 m × 6 m
O
(a) 2400 m3 (b) 3200 m3 is to carry rectangular boxes of dimension
3
(c) 4000 m (d) 4800 m3 8 cm × 7 cm × 6 cm. The maximum number
H
22. A cuboid has a volume of 64000 cm3. If of boxes that can be carried in the wooden
the ratio of its sides is 1 : 2 : 4, then the box is
(a) 1,20,000 (b) 10,00,000
_T

largest side is
(c) 1,00,000 (d) 75,00,000
(a) 40 cm (b) 60 cm
(c) 64 cm (d) 80 cm 29. A cube of edge 6 cm is melted and smaller
cubes of edge 2 cm each are formed; how
C

23. The maximum length of a pencil that can


many such cubes are possible?
be kept in a rectangular box of dimensions
(a) 24 (b) 27
PS

8 cm × 6 cm × 2 cm is
(a) 52 cm (b) 56 cm (c) 18 (d) 29

(c) 104 cm (d) 200 cm 30. A metal sheet 27 cm long, 8 cm broad and
1 cm thick is melted into a cube. What
U

24. The volume of a cuboid is twice the is the difference between the surface area
volume of a cube. If the dimensions of the of the two solids?
@

cuboid are 9 cm, 8 cm and 6 cm, the total (a) 284 cm2 (b) 296 cm2
surface area of the cube is (c) 286 cm 2
(d) 300 cm2

Answers with Solutions


1. Ans. (c) 2. Ans. (d)
Sol. Volume of cuboid = 2 (lb + bh + hl) Sol. Surface area of cuboid = 2 (lb + bh + hl)
Here, l = 50 cm, b = 50 cm = 2 (4 × 3 + 3 × 12 + 12 × 4)
= 2 (12 + 36 + 48) = 2 (96)
15000 = 2 (50 × 50 + 50 × h + h × 50)
= 192 m2.
7500 = 2500 + 100h 3. Ans. (a)
100h = 5000 Sol. Outer volume of box = lbh
h = 50 = 24 × 22 × 17
∴ The height of box is 50 cm. = 8976 cm3
Unit Five : Practice Session ✫ 413

Inner volume = (24 – 2) (22 – 2) (17 – 2) 7. Ans. (b)


= 22 × 20 × 15 Sol. Length of longest rod will be the length
= 6600 cm3 of the diagonal. So its length
Volume of wooden the box = l 2 + b 2 + h2
= Outer volume of the box – Inner volume
= 8976 – 6600 = (4)2 + (4)2 + (4)2
= 2376 cm3. Here l = b = h = 4 m
4. Ans. (d) = 16 + 16 + 16
Sol. Surface area of cube = 6 × (edge)2 = 48 = 6.92 m.

TS
1176
= (edge) 2 [The result will be the same using the
6 formula,
196 = (edge) 2
Diagonal = side 3 4 3 = 6.92 m.]
Edge = 14 cm

H
Volume of cube = (edge)3 8. Ans. (c)
= 14 × 14 × 14 Sol. Volume of big cube

G
= 2744 cm3. = Volume of cubes (I + II + III)
(Edge)3 = 33 + 43 + 53
[Alternative (Fast-track) solution:
= 27 + 64 + 125
Volume of a cube
Surface area
3
U = 216
=
216 3
O
=
6
Edge = 6 cm.
3
1176
( ) 9. Ans. (d)
3
H
= = 196
6 Sol. 30
_T

= (14)3 = 2744 cm3.] 10


5. Ans. (b)
Sol. Volume of cube = (edge)3
15625 = (edge) 3 10
C

Edge = 3 15625
= 3 25 × 25 × 25
PS

The new dimensions are:


Edge = 25 cm.
6. Ans. (b) l = 30 cm, b = 10 cm, h = 10 cm
Sol. Diagonal of cube = 3 × edge Surface area of cuboid = 2 (lb + bh + hl)
U

27 3 = 3 × edge = 2 (30 × 10 + 10 × 10 + 30 × 10)


= 2 (300 + 100 + 300)
@

27 3 = 2 (700)
Edge = = 1400 cm2.
3
10. Ans. (a)
= 27 cm
Volume of cube = (edge)3 Volume of cube 1 (edge1 )3 8
Sol. = =
= (27)3 Volume of cube 2 (edge 2 )3
1
= 19683 cm3. 3
2
[Alternative (Fast-track) Solution: =
3
1
diagonal ∴ Ratio of their edges is 2 : 1.
Volume of a cube =
3
3
11. Ans. (c)
27 3 Sol. Let the edge of the cube = a
= Then, surface area = 6(edge)2 = 6a2
3
= (27)3 = 19683 cm3.] New edge = 2a
414 ✫ Quantitative Aptitude

So, new surface area = 6 (New edge)2 ∴ Length of rectangular box


= 6 × (2a)2 = 5x = 5 × 4 = 20 cm.
= 24a2 16. Ans. (a)
Increase per cent in surface area Sol. Length of cuboid = 6 × edge
18 a2 = 6 × 10 = 60 cm
= × 100 %
6a2 Breadth and height = 10 cm
= 300%. ∴ Surface area of the cuboid
= 2 (lb + bh + lh)
12. Ans. (a)
= 2 (60 × 10 + 10 × 10 + 60 × 10)
Sol. Total cost of cube of wood = Rs 768

TS
= 2 (600 + 100 + 600)
Rate of wood per cubic metre = Rs 1500 = 2600 sq cm.
Total cost 17. Ans. (c)
Volume of cube =
Rate Sol. Volume of cube = Volume of cuboid

H
768 (Edge) 3 = l × b × h
Volume of cube = Let the breadth and height of the cuboid
1500

G
= 0.5120 m3 be 15x and 16x respectively.
Here, edge of the cube = 60 cm and
= 512000 cm3
length (l) of the cuboid = 36 cm
∴ Length of each edge
= (512000)1/3 U (60) 3 = 36 × 15x × 16x
60 × 60 × 60
O
= (80 × 80 × 80)1/3 x2 =
= 80 cm. 36 × 15 × 16
H
13. Ans. (a) 2
x = 25
Sol. Volume of cube = (edge)3 x =5
64 = (edge)3
_T

(4)3 = (edge)3 The longest side of cuboid = 16x


Edge = 4 cm = 16 × 5 = 80 cm.
By joining two cubes, we get a cuboid 18. Ans. (d)
Sol. Total length of diagonals of a cube
C

having l = 8 cm, b = 4 cm and h = 4 cm


∴ Surface area of cuboid = 2 (lb + bh + hl) = 12 cm
A cube has 4 diagonals, and one diagonal
PS

= 2 (8 × 4 + 4 × 4 + 4 × 8)
= 2 (32 + 16 + 32) is equal to 3 × edge.
= 2 (80) Therefore, 4 ( 3 × edge) = 12
= 160 cm2.
U

3 × edge = 3
14. Ans. (a) 3
Sol. Volume of cuboid tank = lbh edge = = 3
@

3
= 48 × 36 × 28
A cube has 12 edges; therefore, the total
= 48384 m3.
length of edges of the cube is 12 3 cm.
15. Ans. (b)
Sol. Let the length, breadth and height of 19. Ans. (d)
rectangular box be 5x, 4x and 2x Sol. Let a be the side of the cube.
respectively. Then, ∴ Diagonal of cube = 3 × side of the cube
Total surface area of cuboid = 3a
= 2 (lb + bh + hl) Given that,
1216 = 2 (5x × 4x + 4x × 2x + 5x × 2x) 3a = 3 3
1216 = 2 (20x2 + 8x2 + 10x2)
or a = 3 cm
608 = 38x2
Hence, volume of cube = 33 cubic cm or c.c.
16 = x2
= 27 c.c.
( )
x =4 2
= 3 3 c.c.
Unit Five : Practice Session ✫ 415

20. Ans. (c) ∴ Volume = (side)3


Sol. Volume of the new cube = 53 = 125 cm3.
= (30)3 + (40)3 + (50)3 26. Ans. (b)
= 27000 + 64000 + 125000 Sol. Two cubes are joined end to end.
= 216000 Then,
= (60)3 Length of the cuboid = 8 + 8 = 16 cm
Edge of new cube Breadth = height = 8 cm
∴ Total surface area of the cuboid
= 3
Volume of new cube = 2 (lb + bh + lh)

TS
= 3 (60)3 = 2 (16 × 8 + 8 × 8 + 16 × 8)
= 60 cm = 2 (128 + 64 + 128)
Total surface area of new cube = 640 cm2.
= 6 × edge2 = 6 × 60 × 60 27. Ans. (d)

H
= 21600 cm2 = 2.16 m2. Sol. Small cubes each of side 2 cm.
21. Ans. (c) Number of cubes
Big cube’s volume

G
Sol. Let h be the height of wall, then =
2 (20 × 20) = 2h (20 + 20) Small cube’s volume
40h = 20 × 20 6×6×6
h = 10 m
∴ Volume of the hall = l × b × h U 28.
=
2×2×2
Ans. (b)
= 27 cubes.
O
∴ = 20 × 20 × 10 Sol. The maximum number of boxes that can
= 4000 m3. be kept within the bigger box
H
22. Ans. (d) Volume of big wooden box
Sol. Let the sides be x, 2x and 4x respectively. Volume of rectangular box
l × b × h = Volume of the cuboid 800 × 700 × 600
_T

∴ x × 2x × 4x = 64000 = [ 1 m = 100 cm]


8×7 ×6
x3 = 8000 = (20)3 = 1000000.
x = 20 29. Ans. (b)
C

∴ Largest side = 4x = 4 × 20 = 80 cm. Sol. We have original length of edge = 6 cm


23. Ans. (c) and new length of edge = 2m.
PS

Sol. The maximum length of the pencil will Number of cubes


3
be the length of the diagonal Original length of edge
= l2 + b 2 + h2 =
New length of edge
U

= 3
82 + 62 + 22 6
= = (3)3 = 27.
= 64 + 36 + 4 2
@

30. Ans. (c)


= 104 cm. Sol. Here, length = 27 cm, breadth = 8 cm,
24. Ans. (b) height = 1 cm
Sol. Twice the volume of cube Total surface area of the metal sheet
= Volume of a cuboid = 2 (lb + bh + lh)
2 × (side of a cube)3 = l × b × h = 2 (27 × 8 + 8 × 1 + 1 × 27)
2a3 = 9 × 8 × 6 = 2 (216 + 8 + 27)
a3 = 216 = (6)3 = 2 × 251 = 502 cm2
Volume of the new cube = 27 × 8 × 1
∴ a (side of a cube) = 6 cm
= 216 = 63 cm3
Surface area = 6a2 = 6 × 6 × 6 So, each side of the new cube = 6 cm = a
= 216 cm2. ∴ Total surface area = 6 × a2
25. Ans. (c) = 6 × 6 × 6 = 216 cm2
Perimeter 20 Difference between the surface area of
Sol. Side of a cube = = = 5 cm
4 4 the two solids = 502 – 216 = 286 cm2.
416 ✫ Quantitative Aptitude

Practice Exercise 33
1. The slant height of a cone is l cm and (a) 65π sq ft (b) 65 sq ft
radius of its base is 7 cm. If the total (c) 120π sq ft (d) 60π sq ft
surface area of the cone is 550 cm2, then
9. If both the radius and the height of a cone
the value of l is
are increased by 50%, then the volume of
(a) 36 cm (b) 18 cm
(c) 10 cm (d) 9 cm the cone will increase by
(a) 100% (b) 200%
2. The volume of a right circular cone is 68

TS
(c) 225% (d) 237.5%
m3. The height of the cone is 4 m. The
area of the base of the cone is 10. A right circular cone is divided into two
(a) 17 cm2 (b) 34 cm2 portions by a plane parallel to the base
2
(c) 51 cm (d) 68 cm2 and passing through a point which is one-

H
3. Fifty circular plates, each of radius 7 cm third of the height from the top. The ratio
and thickness 0.5 cm, are placed one of the volume of the smaller cone to that

G
above the other to form a solid right of the remaining frustum of the cone is
circular cylinder. The volume of the (a) 1 : 3 (b) 1 : 9
cylinder so formed is equal to
(a) 3700 cm3
3
(b) 3750 cm3
(d) 3850 cm3 U (c) 1 : 26 (d) 1 : 27
11. The length of a cloth, 2 m wide, which
O
(c) 3800 cm
is used to make a conical tent 12 m in
4. Two cones have their heights in the ratio diameter with a slant height of 6.3 m is
H
2 : 3 and the radii of their bases in the (a) 54.3 m (b) 59.4 m
ratio 3 : 4. The ratio of their volume is (c) 60 m (d) 62.7 m
(a) 3 : 6 (b) 3 : 8
_T

(c) 3 : 7 (d) 3 : 5 12. A semi-circular thin sheet of paper of


diameter 28 cm is bent and an open
5. If the height of a cone is increased by 50%,
the percentage increase in the volume of conical cup is made. The capacity of the
cone is
C

the cone is
(a) 30 (b) 40 (a) 422.73 cm3 (b) 522.92 cm3
3
(c) 622.36 cm (d) 722.54 cm3
PS

(c) 50 (d) 60
6. The radius of a cone is increased by 50% 13. The base radii of two right circular cones
and its height is decreased by 20%. The of the same height are in the ratio 3 : 5.
volume of the cone The ratio of their volume is
U

(a) decreases by 20% (a) 2 : 9 (b) 9 : 25


(b) remains the same (c) 10 : 21 (d) 7 : 19
@

(c) increases by 50%


(d) increases by 80% 14. The radius and height of a right circular
cone are in the ratio 5 : 12 and its volume
7. The diameter of a right conical tent is 6 m. is 2512 cubic cm. Its radius is
If a pole of length 2 m can be fixed up
(a) 12 cm (b) 10 cm
in the tent at half the distance of the
(c) 9 cm (d) 7 cm
radius from the centre of the base, then
the area of the canvas required is 15. A right circular cone is 3.6 cm high and
(a) 10π (b) 12π the radius of its base is 1.6 cm. It is melted
(c) 15π (d) 16π and recast into a right circular cylinder of
8. The area of canvas cloth needed to erect base radius 1.2 cm. The height of the
a right conical tent of height 12 ft and cylinder is
circular base having circumference 10π ft (a) 2.13 cm (b) 2.14 cm
is (c) 2.15 cm (d) 2.16 cm
Unit Five : Practice Session ✫ 417

16. An edge of a cube measures 10 cm. If the (a) 4 : 1 (b) 6 : 1


largest possible right circular cone is cut (c) 8 : 1 (d) 10 : 1
out of this cube, then the volume of the 23. A right triangle with sides 3 cm, 4 cm and
cone (in cm3) is 5 cm is rotated about the side of 3 cm
(a) 260 (b) 260.9 to form a cone. The volume of the cone
(c) 261.90 (d) 262.7 so formed is
17. The diameters of two cones are equal and (a) 12π cm3 (b) 15π cm3
3
their slant heights are in the ratio 5 : 4. (c) 16π cm (d) 20π cm3
If the curved surface area of the smaller
24. The volume of the largest right circular

TS
cone is 200 cm2, then the curved surface
cone that can be cut out of a cube of edge
area (in cm2) of the bigger cone is
7 cm is
(a) 200 (b) 250
(a) 13.6 cm3 (b) 89.8 cm3
(c) 400 (d) 500

H
3
(c) 121 cm (d) 147.68 cm3
18. The base of a conical tent is 19.2 m in
25. A solid metallic spherical ball of diameter
diameter and the height to its vertex is

G
6 cm is melted and recast into a cone
2.8 m. The area of the canvas required to
22 having a base diameter of 12 cm. What
put up such a tent (in m2), taking π = ,
is nearly
(a) 3017.1 (b) 3170
7
U is the height of the cone?
(a) 9 cm
(c) 3 cm
(b) 6 cm
(d) None of the above
O
(c) 301.7 (d) 30.17 26. A solid piece of iron of dimensions 49, 33
19. A cone of height 7 cm and base radius and 24 cm is moulded into a sphere. The
H
1 cm is carved from a cuboidal block of radius of the sphere is
wood 10 cm × 5 cm × 2 cm. Assuming (a) 21 cm (b) 24 cm
_T

22 (c) 33 cm (d) None of the above


π = , the percentage of wood wasted
7
in the process is 27. A hollow sphere of internal and external
diameters 4 cm and 8 cm respectively is
2 1
C

(a) 92 % (b) 46 % melted into the shape of a cone with base


3 3 diameter of 8 cm. What is the height of
2 1
PS

(c) 53 % (d) 7 % the cone?


3 3 (a) 7 cm (b) 12 cm
20. The radius and slant height of a cone are (c) 14 cm (d) 18 cm
in the ratio 4 : 7. If its curved surface area
U

28. If the volume of a sphere is divided by


is 3168 cm2, then its radius is equal to
its surface area, the result is 27 cm. What
(a) 12 cm (b) 18 cm
is the radius of the sphere?
@

(c) 24 cm (d) 30 cm
(a) 9 cm (b) 18 cm
21. A right circular cone has a base radius (c) 36 cm (d) 1 cm
5 cm. If the radius is increased by 20%, 29. Spheres A and B have radii 40 cm and 10
without any change in the height of the cm respectively. The ratio of the surface
cone, then the percentage increase in the area of A to the surface area of B is
volume is (a) 1 : 4 (b) 1 : 16
(a) 22 (b) 25 (c) 16 : 1 (d) 4 : 1
(c) 40 (d) 44
30. A hemisphere and a cone have equal bases
22. If both the height and radius of a cone and equal heights. What is the ratio of
are doubled, then the ratio of the volume their curved surfaces?
of the bigger cone to that of the smaller (a) 2 : 1 (b) 2 : 1
cone will be (c) 2 : 3 (d) 1 : 2
418 ✫ Quantitative Aptitude

Answers with Solutions


1. Ans. (b) 5. Ans. (c)
Sol. Here, Radius of base (r) = 7cm, Sol. Let the height of the cone be h.
slant height = l cm 1
Then, Volume of cone = π r 2 h = V
3
Total surface area of the cone = πr(r + l) Height of cone is increased by 50%
= πr(r + l) = 550 50 1
22 ∴ New height = h 1 + = h 1+
⇒ × 7 (7 + l) = 550 100 2

TS
7 3
550 = h
⇒ = 7 + l = = 25 2
22 1 3 3
l = 25 – 7 Volume of new cone = πr2 h = V
3 2 2

H
l = 18 cm.
2. Ans. (c) 3V V
1 2 Increase in volume = −V =
Sol.Volume of cone = πr h 2 2

G
3 ∴ Percentage increase in volume
Here, volume of cone
= 68 m3, h = 4 m

68 =
1
× πr2 × 4 U =
2
v
× 100 %
O
3 v
68 × 3 1
⇒ πr2 = = 51 = × 100 % = 50%
H
4 2
∴ The area of the base of the cone 6. Ans. (d) 1 2
= πr2 = 51 cm2. Sol. Volume of the cone (V) = π r h
_T

3
3. Ans. (d) 50 r
New radius = r + r = r+
Sol. 50 Circular plates, each of thickness 0.5 100 2
cm. Then, 3r
C

Height of the cylinder =


2
= 50 × 0.5 = 25 cm
PS

20
∴ Volume of the cylinder = πr2h New height = h− h
22 100
= × 7 × 7 × 25 h 4h
7 = h− =
= 3850 cm3.
U

5 5
2
4. Ans. (b) 1 3 2 4
Sol. Let the height and radius of the cones V’ = π r h
@

3 2 5
be 2h, 3h and 3r, 4r respectively
Volume one of the cone 1 9 4
∴ = 1
V = × πr2h
Volume of another the cone V2
3 4 5
1 2 9
π r1 h1 ∴ V’ = V
= 3 5
1 2 9 4
π r2 h2 ∴ Volume increase = V −V = V
3 5 5
Percentage increase in volume
3 r × 3 r × 2h
= 4
4 r × 4r × 3h V
3 = 5 × 100%
= V
8 4
∴ V1 : V2 = 3 : 8. = × 100 % = 80%.
5
Unit Five : Practice Session ✫ 419

7. Ans. (c) 10. Ans. (c)


Sol. Slant height of the conical tent Sol.
= h2 + r 2 = 4 2 + 32 = 5 m h

3h
r

TS
2
3r
3 Volume of smaller cone
3
Volume of frustum

H
∴ Area of the canvas = πrl
= π × 3 × 5 1 2
πr h
= 15π sq m. 3

G
=
8. Ans. (a) 1 1
π (3r )2 (3h) − π r 2 h
Sol. Radius of tent = r ft 3 3
Circumference of tent = 10π
2πr = 10π U =
1 2
3
πr h
O
∴ r = 5 ft 1 2
π r h (9 × 3 − 1)
∴ Slant height of tent = r 2 + h 2 3
H
1
= 122 + 52 = = 1 : 26.
26
= 169 = 13 ft 11. Ans. (b)
_T

∴ Canvas cloth needed to erect the tent Sol. Let length of cloth = l m
= πrl Width of cloth = 2 m
= π × 5 × 13 Diameter of cone = 12 m
C

= 65π sq ft Radius of cone = 6 m


9. Ans. (d) Slant height = 6.3 m
1
Sol. Volume of cone = πr2h = V Area of cloth
PS

3 = curved surface area of cone


50 r
New radius = r+ r = r+ l × b = π × radius × slant height
100 2 22
3
U

= r l×b = × 6 × 6.3
7
2
l × 2 = 118.8
50 h
@

New height = h+ h = h+ ∴ l = 59.4 m.


100 2 12. Ans. (c)
3 Sol. Circumference of the base of cone =
= h curved circumference of semi-circular
2 2
1 3 3 sheet
New volume of cone = π r2 h 22
3 2 2 πR = × 14 = 44 cm
1 9 × 3 2 27 7
⇒ π r h= V Let the radius of base = r cm
3 8 8
27 19 ∴ 2πr = 44
Increase in volume = V −V = V
8 8 22
2 × × r = 44
Percentage increase in volume 7
19 r = 7 cm
V × 100
= 8 % = 237.5% Slant height (l) = h2 + r 2
V
420 ✫ Quantitative Aptitude

Squaring both sides, Here,


l2 = h2 + r2 Radius of circular cone (r) = 1.6 cm,
h2 = l2 – r2 height of circular cone (h) = 3.6 cm
Height (h) = l 2 − r 2 and radius of cylinder = 1.2 cm
28 1
Here, l = π (1.6)2 × 3.6 = π(1.2)2h
2 3
Because a semi-circular thin sheet of 1.6 × 1.6
⇒ h =
paper of diameter 28 cm is bent. 1.2
= 2.13 cm.

TS
2
28 16. Ans. (c)
= − 72
2 Sol. Edge of the cube = 10 cm
= 196 − 49 = Diameter of cone

H
= 147 = 12.124 cm 10
1 ∴ Radius of cone = = 5 cm
Volume of cone = πr2h 2

G
3 1 22
=
1 22
× × 7 × 7 × 12.124 Volume of cone = × × 5 × 5 × 10
3 7 3 7

13. Ans. (b)


= 622.36 cm3.
U 17. Ans. (b)
= 261.90 cm3.
O
Sol. Let the radii of two cones be 3x and 5x, Sol.
then
H
Volume of one of the cone l1
Volume of another of the cone l2
_T

1
V1 3 π (3x) h 9
2

= = = r
V2 1 π (5x)2 h 25 Let curved surface area of bigger cone
C

3
be A.
∴ = V1 : V2 = 9 : 25.
Curved surface area of bigger cone
PS

14. Ans. (b)


Curved surface area of smaller cone
Sol. Let the radius and height of cone be 5x
and 12x, then A π rl1
1 =
Volume of cone = πr2h 200 π rl2
U

3 A l 5
1 22 = 1 =
2512 = × × 5x × 5x × 12x 200 l2 4
@

3 7
200 × 5
2512 × 3 × 7 A= = 250 cm2.
x3 = 4
22 × 5 × 5 × 12
x3 = 7.99 18. Ans. (c)
Sol. Area of canvas = curved surface area
x3 = 8 (approximate)
of conical tent
∴ x= 2
Radius of the cone is 5x 19.2
Here, r = m, height (l) = 28 m
= 5 × 2 = 10 cm. 2
2
15. Ans. (a) 22 19.2 19.2
= πrl = × × + (2.8)2
Sol. Volume of right circular cone 7 2 2
= Volume of cylinder
 l = r2 + h2
1 2
πr h = πr2h
3 = 301.7
Unit Five : Practice Session ✫ 421

19. Ans. (a) Here, R = 6 cm and r = 5 cm


Sol. Here, cone of height (h) = 7 cm, 1 1
= π (6)2 h − π (5)2 h
cone of base radius = 1 cm 3 3
Cuboid of length = 10 cm, 1
breadth = 5 cm and height = 2 cm = πh (36 – 25)
3
Percentage of wood wasted 11
Volume of cuboid – = πh
3
volume of cone Percentage increase in volume
= × 100
Volume of cuboid

TS
Increase volume of cone
= × 100
1 Volume of cone
lbh − π r 2 h
= 3 × 100 11
lbh π h × 100

H
11
1 22 = 3 = × 100 = 44.
10 × 5 × 2 − × × 12 × 7 25 25
3 7 πh
= × 100 3

G
10 × 5 × 2 22. Ans. (c)
22 Sol. Here, the height and radius of a
100 −
100
3 × 100
U bigger cone be 2h and 2r respectively.
1
π (2r )2 2h
O
Volume of bigger cone 3
22 278 =
= 100 – = Volume of smaller cone 1 2
3 3 πr h
3
H
2
= 92 %. 8 πr h
2

3 =
20. Ans. (c) πr 2 h
_T

Sol. Let the radius and slant height of a 8


= = 8 : 1.
cone be 4x and 7x respectively. 1
Curved surface area of cone = πrl 23. Ans. (a)
C

Sol. We have r = 3 cm and h = 4 cm


22
3168 = × 4x × 7x ∴ Volume of the cone
PS

7 1 2 1
3168 7 = πr h = × π × 32 × 4 cm3
∴ x2 = × 3 3
4 × 7 22
x2 = 36 = 12π cm3.
U

⇒ x =6 24. Ans. (b)


∴ Radius of the cone is 4x = 4 × 6 Sol. Volume of the cone with diameter of
@

= 24 cm. base 7 cm and height 7 cm. Then


21. Ans. (d) 7
r = = 3.5 cm and h = 7 cm
Sol. Radius of the base of the cone = 5 cm 2
When the radius of the base is increased The volume of the largest cone
by 20%, then the radius of base of the 1 2
5 × 20 = πr h
cone = 5 + = 5 + 1 = 6 cm 3
100 1 22
1 = × × 3.5 × 3.5 × 7 cm3
Volume of cone = π (5)2h 3 7
3
25 269.5
= πh = cm3 = 89.8 cm3.
3 3
∴ Increase in volume of cone 25. Ans. (c)
1 2 1 Sol. Volume of the sphere
= πR h − πr 2 h = volume of the cone
3 3
422 ✫ Quantitative Aptitude

If r1 and r2 radius of sphere and cone 28. Ans. (d)


respectively and h the height of the cone, Sol. If r is the radius, the surface area of
4 1 a sphere = 4πr2 and volume of a sphere
πr 3 = πr22 h
3 1 3 4
6 12 = πr3
Here r, = = 3 cm and r2 = 3
2 2
= 6 cm 4 3
πr ÷ 4πr2 = 27 cm
4 1 3
⇒ π × 3 × 3 × 3 = π × 6 × 6 × h 4 3 1
3 3 ⇒ πr × = 27 cm

TS
4 1 3 4 π r2
⇒ π × 27 = π × 36 h r
3 3
⇒ = 27 cm
4π × 9 × 3 3
⇒ h = = 3 cm
π × 36 ⇒ r = 27 × 3 = 81 cm

H
26. Ans. (a)
Sol. The volume of the iron piece 29. Ans. (c)

G
= (49 × 33 × 24) cm3 Sol. If r1 and r2 are the radii of A and B
If the radius of the sphere is r, its respectively.
volume 4

Given,
= πr
3
3

U The ratio of surface areas of A and B

=
4 πr12
=
r12
O
4 πr2 2 r2 2
4 3
= πr = 49 × 33 × 24 40
2
16
3 ⇒ =
H
49 × 33 × 24 × 3 × 7 10 1
⇒ r3 = = 9261
4 × 22 The surface areas of a A and B are
_T

⇒ r = 9261 = 21 cm
3 3
in the ratio 16 : 1
27. Ans. (c) [Fast-track: For two spheres,
Sol. Here the volume of the material in the
(ratio of radii)2 = ratio of surface areas
sphere is volume of external sphere –
C

2
volume of internal sphere Radius of A
⇒ = ratio of surface areas
If r1 and r2 are the two radii, Radius of B
PS

of A and B
4 3 4
⇒ πr1 – πr2 3 gives the volume of the 40
2
16
3 3 = = .]
material. 10 1
U

4 4 30. Ans. (b)


⇒ π (4)3 – π (2)3
3 3 Sol. If the radius of each is r, the height
@

4 h of the hemisphere = r.
= π [(4)3 – (2)3] cm3
3 ∴ Height of the cone is also r.
4
= π (64 – 8) cm3
3 slant height of cone = h2 + r 2
4
= π × 56 cm3 ⇒
3 r 2 + r 2 = 2r 2
Let the height and radius of the base Ratio of curved surface areas of
of the cone be h and r its volume hemisphere and cone
1 2
= πr h 2 πr 2
3 =
1 4 πr 2r 2
⇒ π × (4)2h = π × 56 (Given)
3 3
4 3 2 πr 2
⇒ h = π × 56 × = 14 cm. = =
3 π×4×4 πr × 2 r 2 : 1.
Unit Five : Practice Session ✫ 423

Practice Exercise 34
1. If the radius and the height of a right 9. A right circular cylindrical tunnel of
circular cylinder are 21 cm and 35 cm diameter 2 m and length 40 m is to be
respectively, then the total surface area of constructed from a sheet of iron. The area
the cylinder is of the iron sheet required (in m2) is
(a) 7092 sq cm (b) 7192 sq cm (a) 40π (b) 80π
(c) 7292 sq cm (d) 7392 sq cm (c) 160π (d) 200π
2. A rectangular paper is 44 cm long and 10. Find the weight of a lead pipe 3.5 m long,

TS
20 cm wide. A cylinder is formed by if the external diameter of the pipe is
rolling the paper along its length. The 2.4 cm, the thickness of the lead is 2 mm
volume of the cylinder is and 1 cc of lead weighs 12.2 gm.
(a) 3010 cm3 (b) 3040 cm3

H
(a) 5.9048 kg (b) 9.5048 kg
(c) 3060 cm3 (d) 3080 cm3 (c) 4.5098 kg (d) None of these

G
3. A cylindrical pipe has an inner diameter 11. The curved surface area of a cylindrical
of 7 cm. Water flows through it at the rate pillar is 264 m2 and its volume is 924 m3.
of 192.5 litres per minute. The rate of flow
Taking π = 22 7 , find the ratio of its
in km/h is
(a) 1 km/h (b) 2 km/h U diameter to its height.
(a) 7 : 6 (b) 6 : 7
O
(c) 3 km/h (d) 4 km/h
(c) 3 : 7 (d) 7 : 3
4. The diameter of the base of a right circular
H
cylinder is 42 cm and its height is 10 cm. 12. A water tank is hemispherical at the
The area of the curved surface of the bottom and cylindrical at the top. The
radius is 12m. If the total capacity is
_T

cylinder is
(a) 2789 sq cm (b) 2146 sq cm 3312π m3, then the capacities of the two
(c) 1320 sq cm (d) 1195 sq cm portions are in the ratio
(a) 8 : 9 (b) 8 : 11
5. If the diameter of the base of a cylindrical
C

(c) 8 : 13 (d) 8 : 15
pillar is 4 m and its height is 21 m, then
the cost of construction of the pillars at 13. A cylindrical vessel can hold 154 gm of
PS

Rs 1.50 per cubic metre is water. If the radius of its base is 3.5 cm
(a) Rs 396 (b) Rs 400 and 1 cubic cm of water weighs 1 gm, then
(c) Rs 410 (d) Rs 420 the depth of the water is
U

6. The volume of a right circular cylinder is (a) 3 cm (b) 4 cm


1100 cm3 and the radius of its base in 5 (c) 4.5 cm (d) 5 cm
@

cm. Its curved surface area is 14. If the radius and height of a right circular
(a) 410 sq cm (b) 440 sq cm cylinder are both halved, then the ratio
(c) 460 sq cm (d) 490 sq cm of volume of the original cylinder to the
7. A hollow garden roller 63 cm wide with volume of the smaller cylinder is
a girth of 440 cm is made of iron 4 cm (a) 1 : 8 (b) 1 : 4
thick. Find the volume of the iron. (c) 4 : 1 (d) 8 : 1
(a) 107712 cm3 (b) 58752 cm3 15. The radius of the base of a right circular
3
(c) 54982 cm (d) 57636 cm3 cylinder is halved and the height is
8. The radius of a wire is decreased to one- increased by 50%. The ratio of the volume
third. If volume remains the same, the of the original cylinder to that of the new
length increases cylinder will be
(a) 2 times (b) 3 times (a) 2 : 1 (b) 8 : 3
(c) 6 times (d) 9 times (c) 3 : 1 (d) 4 : 1
424 ✫ Quantitative Aptitude

16. If a rectangular piece of paper of size 21. A solid metallic cylinder of base 3 cm and
44 cm × 20 cm is rolled to form a cylinder height 5 cm is melted to make n solid
of height 20 cm, then the radius of the cones of height 1 cm and base radius 1
base is mm. The value of n is
(a) 7 cm (b) 7π cm (a) 450 (b) 1350
(c) 22 cm (d) 11 cm (c) 4500 (d) 13500
17. Ice cream completely filled in a cylinder 22. A cylinder is circumscribed about a
of diameter 35 cm and height 32 cm is to hemisphere and a cone is inscribed in the
be served by completely filled identical hemisphere such that its base coincides

TS
disposable cones of diameter 4 cm and with the base of the hemisphere. The
height 7 cm. The maximum number of volume of cylinder, volume of hemisphere
persons that can be served this way is and the volume of cone are in the
proportion

H
(a) 950 (b) 1000
(c) 1050 (d) 1100 (a) 1 : 2 : 3 (b) 3 : 2 : 1
(c) 2 : 3 : 1 (d) 4 : 3 : 2

G
18. A solid is composed of a cylinder with
hemispherical ends. If the whole length of 23. A right circular cylinder and a sphere are
of equal volumes and their radii are also
the solid is 108 cm and the diameter of
the hemispherical ends is 36 cm, the cost
of polishing the surface at the rate of 7 U equal. If h is the height of the cylinder and
d is the diameter of the sphere, then
(a) h = d (b) 2h = d
O
paise per sq cm is h d h d
(a) Rs 748.35 (b) Rs 794.52 (c) = (d) =
3 2 2 3
H
(c) Rs 855.36 (d) Rs 884.57
24. 66 cubic centimetres of silver is drawn
19. The material of a solid cone is converted into a wire 1 mm in diameter. The length
_T

into the shape of a solid cylinder of the of the wire in metres will be
same radius. If the height of the cylinder (a) 8 4 (b) 9 0
is 5 cm, the height of the cone is (c) 168 (d) 336
C

(a) 18 cm (b) 15 cm
(c) 13 cm (d) 10 cm 25. A cylindrical vessel of radius 4 cm contains
water. A solid sphere of radius 3 cm is
PS

20. From a cylindrical piece of wood of radius lowered into the water until it is completely
4 cm and height 5 cm, a right circular cone immersed. The water level in the vessel
with the same base radius and height 3 will rise by:
cm is carved out. The surface area (in cm2)
U

2 4
of the remaining wood is (a) cm (b) cm
9 9
(a) 56π (b) 60π 9 9
@

(c) 76π (d) 92π (c) cm (d) cm


4 2
Answers with Solutions
1. Ans. (d) 2. Ans. (d)
Sol. Here, h = 35 cm and r = 21 cm Sol. Length of paper = 44 cm
Total surface area of cylinder Width of paper = 20 cm
= 2πr (h + r) The sheet is rolled along its length, so
2 × 22 a right circular cylinder is formed with
= × 21 (35 + 21) circumference 44 cm and height 20 cm
7
⇒ 2πr = 44
44 22
= × 21 × 56 2 × × 2 = 44
7 7 44 × 7
= 44 × 21 × 8 r =
44
= 7392 sq cm. ⇒ r = 7 cm
Unit Five : Practice Session ✫ 425

Volume of cylinder = πr2h 22


1100 = × 5 × 5 × h
22 7
= × 7 × 7 × 20 1100 7
7 ⇒ h = ×
= 3080 cm3. 5 × 5 22
3. Ans. (c) ⇒ h = 14 cm
Sol. Diameter of cylindrical pipe = 7 cm Curved surface area of cylinder
7 = 2πrh
radius of cylindrical pipe = = 3.5 cm 2 × 22
2 = × 5 × 14
Volume of cylindrical pipe 7

TS
= Volume of water = 440 sq cm.
πr2h = 192.5 × 1000 cubic cm 7. Ans. (a)
192.5 × 1000 7 Sol. Circumference (girth) of the roller
⇒ h = × = 2πr = 440

H
(3.5)(3.5) 22 22
⇒ h = 5000 cm = 2 × × r = 440
7

G
[  100 cm = 1 m, 1000 m = 1 km] ∴ r = 70 cm
= 50m = 0.05 km Let r1 be the inner radius
Distance travelled in one minute
= 0.05 km
Distance travelled in 60 minutes U = 70 – 4 = 66 cm
∴ Volume of the iron = (r2 – r12) × h
22 2
( )
O
= 0.05 × 60 = 70 − 662 × 63
7
= 3 km 22
H
Thus, speed of water flow = 3 km/h. = × [(70 – 66) × (70 + 66)] × 63
7
4. Ans. (c) 22
= × [4 × 136] × 63
_T

Sol. Curved surface area of cylinder 7


= 22 × 544 × 9
= 2πrh
= 107712 cm3.
diameter 42
Here, r = = cm 8. Ans. (d)
C

2 2 Sol. Let the length of the wire = h


and h = 10 cm If r be the radius,
PS

22 42 then volume = πr2h


= 2 × × × 10
7 2 Let the new radius be r1,
= 1320 sq cm. and the new length of the wire be h1
5. Ans. (a) 1
U

r1 = r
Sol. Volume of cylindrical pillar 3 2
r
Total cost Volume = π × × h1
@

= ∴
Rate of per cubic metre 3
π r h1
2
Here, rate of per cubic metre = Rs 1.50, i.e., πr2h =
9
diameter 4
r= = = 2 m and h = 21 m 9 × π r 2h
2 2 h1 =
Total cost = Rate of per cubic metre π r2
i.e., h1 = 9h.
× volume of cylindrical pillar Hence, the length increase = 9 times
22
= 1.50 × × 2 × 2 × 21 9. Ans. (b)
7 2
= Rs 396. Sol. Radius = = 1 m, height = 40 m
2
6. Ans. (b) Curved surface area = 2πrh
Sol. Volume of cylinder = πr2h = 2π × 1 × 40
Here, r = 5 cm, = 80 π m2.
and volume of cylinder = 1100 cm3 Hence, the area of the iron sheet is 80 π m2.
426 ✫ Quantitative Aptitude

10. Ans. (a) and total capacity = 3312π m3


Sol. Height = 3.5 m = 350 cm 2
External radius of the pipe π (12)3 + π (12)2h = 3312π
3
2.4 2
= = 1.2 cm π ( × 1728 + 144h) = 3312π
2 3
Internal radius of the pipe 1152 + 144h = 3312
= 1.2 – 0.2 = 1 cm 144h = 2160
Volume of the lead pipe 2160
= π [External radius – internal radius] × h ⇒ h = = 15 m
144
= π [(1.2)2 – 12] × 350

TS
Hence, the ratio of the capacities of the
22 two portions = Ratio of volumes of
= [1.44 – 1] × 350
7 hemisphere and cylinder
22 2

H
= [0.44] × 350 = π(12)3 : π(12)215
7 3
= 484 cm3 2

G
Weight of the lead = 484 × 12.2 = × π × 1728 : π × 144 × 15
= 5904.8 gm 3
= 1152 : 2169
11. Ans. (d)
= 5.9048 kg.

Sol. Curved surface area of pillar U 13. Ans. (b)


= 8 : 15.
O
Sol. Volume of cylinder = πr2h
= 2πrh Here, r = 3.5 cm
= 264 m2 22
H
Volume of cylindrical pillar 154 = × (3.5)2 × h
7
= πr2h = 924 m3 154 × 7
_T

⇒ h = = 4 cm
Curved surface area 2π rh 3.5 × 3.5 × 22
= Hence, the depth of water is 4 cm.
Volume of cylindrical pillar π r 2h
264 14. Ans. (d)
C

= Sol. When the dimensions of the solid are


924
924 × 2 halved, the corresponding volume
3
PS

r = 1 1
264 becomes , i.e. th of the original
2 8
⇒ r = 7m 1
Curved surface area of pillar ∴ Ratio is 1 : = 8 : 1.
8
U

2πrh = 264 15. Ans. (b)


264 Sol. Volume of original cylinder = πr2h
@

h = 50 3
2π r new height = h + h × = h and
264 100 2
= = 6 r
22 new radius =
2× ×7 2 2
7 r 3
Volume of new cylinder = π h
Diameter (D ) 2r 2 × 7 7 2 2
= = = =
height ( h) h 6 3 Volume of original cylinder π r2h
=
= 7 : 3. Volume of new cylinder r 3
2

12. Ans. (d) π h


2 2
Sol. Let h m be the height of the cylinder. 8 × πr 2 h
Volume of hemisphere + Volume of =
3πr 2 h
cylinder = Total capacity
Here r = 12 m 8
= = 8 : 3.
3
Unit Five : Practice Session ✫ 427

16. Ans. (a) Cost of polishing 3888 π cm2


Sol. Area of rectangular 22 7
= 3888 × ×
= Surface area of cylinder 7 100
l × b = 2πrh = Rs 855.36.
Here, l = 44 cm, b = 20 cm and h = 20 cm
19. Ans. (b)
2 × 22
44 × 20 = × r × 20 Sol. Volume of cone = Volume of cylinder
7 1 2
44 × 20 × 7 πr h1 = πr2h2
∴ r = = 7 cm 3

TS
22 × 20 × 2 h1 = 3h2
17. Ans. (c) Given that, height of cylinder (h2) = 5 cm
Sol. Let n be the number of persons. ∴ h1 = 3 × 5 = 15
∴ n × Volume of the cone Hence, height of cone is 15 cm.

H
= Volume of cylinder 20. Ans. (c)
Here, diameter of cones = 4 cm, Sol. Surface area of the remaining wood =

G
4 curved surface area of the cylindrical
radius of cones = = 2cm,
2 wood + base area + curved surface area
and height of cones = 7 cm
diameter of cylinder = 35 cm
35 U of cone
Here, r = 4 cm, height of cylinder = 5 cm,
height at cone = 3 cm
O
radius of cylinder = cm
2 l = h 2 + r 2 = 32 + 4 2
and height of cylinder = 32 cm
H
= 2πrh + πr2 + πrl
1 35
2 = 2π × 4 × 5 + π × 4 × 4 + π × 4 × 32 + 4 2
n × π (2)2 7 = π
_T

⇒ 32 = 40π + 16π + 20π


3 2 = 76π cm2.
1 1225 21. Ans. (d)
n× π × 28 = π × × 32
3 4 Sol. Volume of cylinder = πr2h
C

1225 × 32 × 3 Here, radius of cylinder = 3 cm and


n= height = 5 cm
4 × 28
PS

∴ n = 1050. =π × 3 × 3 × 5 1
18. Ans. (c) Given that, radius of cone = cm and
10
Sol. Total height of solid = 108 cm height = 1 cm
U

Radius of each hemispherical end Volume of cylinder


Number of cones =
diameter 36 Volume of cone
= = = 18 cm
@

2 2 π ×3×3×5
Height of cylindrical part =
1 1 1
= 108 – (18 + 18) ×π × × ×1
3 10 10
= 72 cm = 13500.
Total surface area of solid
= surface area of cylinder 22. Ans. (b)
+ 2 (surface area of hemisphere) Sol. Let the radius of cylinder, hemisphere
= 2πrh + 2 (2πr2) and cone be r; let their volumes be V1,
= 2π × 18 × 72 + 2 [2π (18)2] V2 and V3 respectively.
= 2592π + 1296π 2 3 1 3
∴ V1 : V2 : V3 = πr3 : πr : πr
= 3888π cm2 3 3
Cost of 1 sq cm of polishing the surface = 3πr : 2πr : πr
3 3 3

7 ( height of cone = height of cylinder


= Rs = radius of hemisphere) = 3 : 2 : 1.
100
428 ✫ Quantitative Aptitude

23. Ans. (d) (Be careful of the units of measurements)


Sol. Given that, Now we have
volume of cylinder = volume of sphere 22 1 1
4 3 × × × h = 66
πr2h = πr 7 20 20
3
66 × 20 × 20 × 7
4 3 ⇒ h =
πr 22
h = 3 2
πr = 8400 cm = 84 m. [  1 m = 100 cm]

TS
4
∴ h = r 25. Ans. (c)
3
Sol. Let the rise in the water level be h cm.
Given that, diameter of the sphere = d
Here, radius of cylindrical vessel = 4 cm,

H
4 d radius of solid sphere = 3 cm
h =
3 2 Volume of cylinder

G
3h = 2d = Volume of solid sphere
4
h d πr2h = πr3

24. Ans. (a)
2
=
3
.

U π×4×4×h = π×3×3×3
3
4
O
Sol. Let the length of the wire be h. 3
1 1 3× 3 9
⇒ h = = cm.
H
Radius = mm = cm 4
2 20 4
C _T
PS
U
@
Unit Five : Practice Session ✫ 429

Practice Exercise 35
1. If the radius of a sphere is increased by (a) 2 : 3 (b) 3 : 2
100%, then the increase in the surface area (c) 4 : 9 (d) 9 : 4
of the sphere increase by 9. The radius of a hemisphere is 3 cm. The
(a) 100% (b) 200% ratio of its volume to its total surface area
(c) 300% (d) 400% is
2. The number of spherical bullets, each (a) 1 : 3 (b) 2 : 1
bullet being 4 cm in diameter, that can be (c) 1 : 1 (d) 2 : 3

TS
made out of a cube of lead whose edge 10. A sphere of radius 5 cm exactly fits into
is 44 cm is a cubical box. The ratio of the surface area
(a) 2541 (b) 2551 of the box to the surface area of the
(c) 2561 (d) 2571

H
sphere is
3. If two spheres have their radii in the ratio (a) 19 : 9 (b) 21 : 11
2 : 3, then the ratio of their volumes is (c) 23 : 13 (d) 25 : 13

G
(a) 8 : 27 (b) 4 : 9 11. A sphere of diameter 10 cm has the same
(c) 6 : 12 (d) 1 : 1.5 volume as that of a cone with a circular
4. If the radius of a sphere
of another sphere, the
is one half that
ratio of their U base of radius 5 cm. The height of the
cone is
O
volumes is (a) 35 cm (b) 30 cm
(a) 1 : 4 (b) 4 : 1 (c) 25 cm (d) 20 cm
H
(c) 1 : 8 (d) 8 : 1 12. The diameter of a copper sphere is 6 cm.
5. A spherical ball of lead 3 cm in diameter The sphere is melted and drawn into a
_T

is melted and recast into three spherical long wire of uniform circular cross-section.
balls. The diameter of two balls is 1 cm If the length of the wire is 36 cm, then
and 1.5 cm respectively. The diameter of the radius of the wire is
(a) 0.5 cm (b) 0.8 cm
C

the third ball is


(a) 2.82 cm (b) 2.41 cm (c) 1.0 cm (d) 1.2 cm
(c) 2.13 cm (d) 2.09 cm
PS

13. The circumference of the edge of a


6. The diameters of the internal and external hemisphere bowl is 264 cm. The capacity
surfaces of a hollow spherical shell are 6 of the bowl is
cm and 10 cm respectively. If it is melted (a) 178596 cm3 (b) 155232 cm3
U

3
and recast into a solid cylinder of diameter (c) 134874 cm (d) 163928 cm3
14 cm, the height of the cylinder is 14. A sphere of diameter 6 cm is dropped into
@

(a) 2.82 cm (b) 2.74 cm a cylindrical vessel partly filled with water.
(c) 2.66 cm (d) 2.51 cm The radius of the vessel is 6 cm. If the
7. If the external and internal radii of a sphere is completely submerged in water,
spherical shell are 5 cm and 3 cm how much will the surface level of water
respectively, the volume of the shell in be raised?
cm3 is (a) 1 cm (b) 2 cm
2 2 (c) 3 cm (d) 4 cm
(a) 405 (b) 400
3 3 15. Six spherical balls of radius r are melted
2 2 and cast into a cylindrical rod of metal
(c) 410 (d) 415
3 3 of same radius. The height of the rod will
8. If the volumes of two spheres are in the be
ratio of 8 : 27, the ratio of their surface (a) 4 r (b) 6 r
areas is (c) 8 r (d) 12 r
430 ✫ Quantitative Aptitude

16. The volume of the largest sphere which 2500


is carved out of a cube of side 5 cm is (a) π cm3 (b) 2500π cm3
3
(a) 487.29 cm3 5000
(b) 65.47 cm3 (c) π cm3 (d) 5000π cm3
3
(c) 351.42 cm3
(d) 214.49 cm3 19. If the height and diameter of a right
circular cylinder are 32 cm and 6 cm
17. A metallic sphere of radius 10.5 cm is respectively, then the radius of a sphere
melted and then recast into small cones, whose volume is equal to the volume of
each of radius 3.5 cm and height 3 cm. the cylinder is

TS
The number of cones so obtained is (a) 3 cm (b) 4 cm
(a) 108 (b) 54 (c) 6 cm (d) none of these
(c) 126 (d) 432
20. If the radius of a sphere is increased by

H
18. A cylindrical box of radius 5 cm contains
2 cm, then its surface area increases by
10 solid spherical balls each of radius 5
352 cm2. The radius of the sphere before
cm. If the top most ball touches the upper

G
the increase was:
cover of the box, then volume of the
(a) 3 cm (b) 4 cm
empty space in the box is

U
Answers with Solutions
(c) 5 cm (d) 6 cm
O
1. Ans. (c) [Alternative method (Fast-track): The
H
Sol. Surface area of sphere = 4πr2 number of spheres each of radius r will
Radius of sphere is increased by 100% be
new radius = 2r
_T

Volume of cube Side3


New surface area = 4π (2r)2 =
Volume of 1 sphere 4 3
= 4π × 4r2 πr
= 16 πr2 3
44 × 44 × 44
C

Increase in surface area


4 22
= (4 – 1) × 100 = 300% × ×2×2×2
PS

[Alternative method (Fast-track): If the 3 7


two dimensions in surface area increase 44 × 44 × 44
= ×3×7
by a% and b%, the surface area increases 4 × 22 × 2 × 2 × 2
ab = 2541.]
by a + b +
U

%. Here a and b are the


100 3. Ans. (a)
radius, so we have
Sol. Let the radii of two spheres be 2x and 3x
@

r2
2r + Then,
100
100 2 Volume of one sphere (V1 )
=> 2 × 100 + = 300%.]
100 Volume of the other sphere (V2 )
2. Ans. (a)
4 3
Sol. Let n be the number of bullets made out π r1
of the cube = 3
∴ n × Volume of one spherical bullet 4 3
π r2
= Volume of the cube 3
4 2x × 2x × 2x
n × π (2)3 = (44)3 =
3 3x × 3x × 3x
44 × 44 × 44 7 8
n= ×3× =
4×2×2×2 22 27
n = 2541 ∴ V1 : V2 = 8 : 27
Unit Five : Practice Session ✫ 431

[Alternative method (Fast-track): For Volume of hollow spherical shell


two spheres, the ratio of volumes 4
= (ratio of radii)3. = π [(5)3 – (3)3]
3
ratio of volumes = ( 2 : 3)3 4
= π [125 – 27]
= 8 : 27.] 3
4. Ans. (c) 4
= π × 98 cm3
4 3 3
π r1 Let the height of cylinder = h cm
Volume of small sphere 3
Sol. = 4

TS
Volume of big sphere 14
π r23 Radius of cylinder = = 7 cm
3 2
3 Volume of cylinder = πr h
2
1
= π (7 × 7) × h

H
2
= Volume of hollow shell
(1)3
= Volume of cylinder

G
1 4
= =1:8 π × 98 = π (7 × 7) × h
8 3
[Alternative method (Fast-track): The
ratio of volumes will be (ratio of radii)3.
1 U h = ×
4 98
3 7 ×7
O
Here if one radius is 1, the other is . = 2.66 cm
3
2 7. Ans. (c)
1
H
Ratio of volumes = :1 Sol. If R and r are the external and internal
2 radii, the volume of the spherical shell
1
: 1 or 8 : 1.]
_T

8 4
= π ( R3 − r 3 )
3
5. Ans. (a) 4
= π (53 − 33 )
Sol. Volume of big ball = Volume of first ball 3
C

+ Volume of second ball 4 22


= × (125 − 27)
+ Volume of third ball
PS

3 7
4 3 4 3 4 3 4 3 4 22
πr = π r1 + π r2 + π r3 = × × 98
3 3 3 3 3 7
r3 = r13 + r23 + r33 1232 2
U

= = 410 cm3.
3 3 3 3
3 3
(1.5) = (0.75) + (0.5) + r3 8. Ans. (c)
@

3 Sol. Let r1 and r2 be the radius of the two


3.375 = 0.422 + 0.125 + r3
spheres V1 and V2 the volumes and S1
r33 = 2.828
and S2 the surface areas,
r3 = 3
2.828 4 3
V1 8 π r1 r
3
r3 = 1.414 (approximate) = = 3 = 1
V2 27 4 3 r2
Diameter = 2.82 cm. π r2
r1 2 3
∴ =
6. Ans. (c) r2 3
2
4π r12
2
10 r 2
Sol. External radius of shell = = 5 cm S1 : S2 = = 1 =
2 4π r22 r2 3
6 4
Internal radius of shell = = 3 cm = =4:9
2 9
432 ✫ Quantitative Aptitude

[Alternative method (Fast-track): In this 12. Ans. (c)


case, if S1 and S2 are the surface areas,
Sol. Volume of sphere = Volume of wire
3
S1 4
(Ratio of volumes)2 = π (3)3 = πr2 × 36
S2 3
2 3 4×3×3× 3
8 S1 r2 =
= 3 × 36
27 S2
3
r2 = 1
64 S1 ∴
= r = 1 cm

TS
729 S2
13. Ans. (b)
2 3
S1
= 3
64 Sol. Volume of hemisphere = π r
3
S2 729 Given that, 2πr = 264

H
4 132
= or 4 : 9.] r =
9 π

G
9. Ans. (d) 2 2 3 2 132 132 132
Sol. Volume of hemisphere = πr3 ∴ πr = × π × × ×
π π π
Surface area = 3 πr2
2
∴ Ratio is πr3 : 3πr2
3

U 3 3
2 7 7
= × 132 × × 132 × × 132
O
3 3 22 22
2
π (3)3 = 155232 cm3
= 3
H
3 π (3)2 14. Ans. (a)
2 Sol. Volume of water that rises in the
_T

= =2:3 cylindrical vessel. If R is the radius of the


3 cylinder and r1 the radius of the sphere,
10. Ans. (b)
Whole surface area of cube = Volume of sphere
Sol. Cruved surface area of sphere
C

4 3
πR2h = πr
6 × (edge)2 3
=
PS

4π r 2
4 3
R2h = r
6 × 10 × 10 × 7 3
= 4
4 × 22 × 5 × 5 (6)2 h = × (3)3
3
U

(as the diameter of the sphere is edge of


h = 1 cm
the cube)
15. Ans. (c)
@

21
= Sol. Let the height of the rod be h. If r is the
11
radius (equal for each sphere and
∴ Ratio is 21 : 11
cyclindrical rod given)
11. Ans. (d) 4
Sol. Volume of sphere = Volume of cone 6 × πr3 = πr2h
3
4 3 1 2 h = 8r
πr = πr h
3 3 16. Ans. (b)
4 1
× π × (5)3 = × π × (5)2 × h Sol. Diameter of the largest sphere
3 3
4 × π × 125 × 3 = Side of cube from which it is carved out
h =
25 × π × 3 5
∴ Radius (r) = cm
h = 20 cm 2
Unit Five : Practice Session ✫ 433

4 22 5 5 5 Volume of the empty space in box


∴ Volume of sphere = × × × ×
3 7 2 2 2 5000
= 2500 π – π
= 65.47 cm3 3
2500
17. Ans. (c) = π cm3
3
Sol. Let n be the number of cones 19. Ans. (c)
∴ n × Volume of cone Sol. Volume of cylinder = πr2h
= Volume of the metallic sphere π × 3 × 3 × 32
1 4 = 288 π cm3

TS
n × πr 2 h = πr 3
3 3 4
1 Volume of the sphere = πr3
n × π 3.5 × 3.5 × 3 3
3 4 3
πr = 288 π

H
4 3
= π × 10.5 × 10.5 × 10.5 288π
3 ×3
r3 =

G
Removing decimals, we have 4π
= 72 × 3
1 35 35 4 105 105 105
n× π × × ×3 = π ×
3 10 10 3
× ×
10 10 10
21 21 21 2 2 1 U = 216
= 3 216
O
n = 4× × × × × × ∴ r = 6 cm
2 2 2 7 7 3
H
= 126 20. Ans. (d)
The number of cones obtained is 126. Sol. If the original radius of the sphere is r,
_T

18. Ans. (a) after increase it is r + 2.


Sol. Height of box So area = 4πr2
= 10 × diameter of spheres Change in area
= 4π (r + 2)2 – 4πr2 = 352 (given)
C

= 10 × 10 = 100 cm
7 1
Volume of box = πr2h (r + 2)2 – r2 = 352 × × = 28
22 4
PS

= π × 5 × 5 × 100
(r + 2 + r) (r + 2 – r) = 28
= 2500 π cm3
2r + 2 = 14
Volume of 10 spherical balls
14
U

4 3 ⇒ r = −1
= 10 × πr 2
3
14 − 2
@

4 =
= 10 × ×π×5×5×5 2
3
5000 12
= π cm3 =
2
= 6 cm.
3
434 ✫ Quantitative Aptitude

Practice Exercise 36
1. A clock strikes once at 1 o’clock, twice at 4
(a) 16 minutes past 9
2 o’clock, thrice at 3 o’clock and so on. 11
The total number of strikes in a day is 4
(b) 22 minutes past 9
(a) 24 (b) 98 11
(c) 124 (d) 156 4
(c) 26 minutes past 9
2. How many times are the hands of a clock 11
(d) None of these
at right angles in a day?

TS
(a) 22 (b) 24 8. If the two hands of a watch coincide every
(c) 44 (d) 48 66 minutes, then the watch
3. How many times do the hands of a clock (a) gains time

H
coincide in a day? (b) loses time
(a) 20 (b) 22 (c) shows correct time
(d) alternatively loses and gains time

G
(c) 21 (d) 24
4. A man goes out between 5 p.m. and 6 p.m. 9. Two clocks are set correct at 10.00 a.m. on
When he comes back between 6 p.m. and
7 p.m., he observes that the two hands
of a clock have interchanged their positions.U Friday. The first clock gains 2 minutes
every hour and gains twice as much as
the second clock. What time will the
O
At what time did the man go out? second clock register when the correct
1 time is 2.00 p.m. on the following Monday?
H
(a) 31 minutes past 4
11 (a) 3.15 pm (b) 3.16 pm
4 (c) 4.32 pm (d) 3.18 pm
(b) 32 minutes past 5
_T

13
(c) 43 minutes past 5 10. When the time is 2 o’clock, the angle
3 between the hour and minute hand is
(d) 24 minutes past 6 (a) 90° (b) 75°
16
C

(c) 60° (d) 30°


5. At what time between 5:30 and 6:00 in
1
the morning will both the hands of a clock 11. In 2 hours, the hour hand of a clock
PS

be at right angles? 2
rotates through an angle of
7
(a) 43 minutes past 5 (a) 75° (b) 900°
11
(c) 180° (d) none of these
U

6
(b) 40 minutes past 5 12. Between 6 o’clock and 7 o’clock, when
11
5 will both the hands of a clock be in
@

(c) 39 minutes past 5 opposite direction?


13
3 (a) 3 minutes past 6
(d) 34 minutes past 5 5
16 (b) 35 minutes past 6
11
6. At what time between 11 o’clock and 12 (c) At no time
o’clock will the hands of a clock coincide? (d) None of these
5
(a) 5 minutes past 11 13. The angle between the minute hand and
11
(b) At 12 o’clock the hour hand of a clock when the time
(c) It never happens is 4.20 is
(d) None of these (a) 5° (b) 10°
(c) 15° (d) 20°
7. In between 9 a.m. and 10 a.m., when will
both hands of a clock be in opposite 14. A clock is started at noon. By 4 minutes past
direction? 4 p.m., the hour hand has turned through
Unit Five : Practice Session ✫ 435

(a) 272° (b) 252° (b) After 3 days 11 hours


(c) 241° (d) 236° (c) After 4 days 4 hours
15. At what time between 4 o’clock and 5 (d) None of these
o’clock will the hands of a clock point in 18. The hands of a clock coincide after every
opposite directions? 64 minutes of correct time. How much is
(a) 40 minutes past 4 the clock fast or slow in a day?
(b) 45 minutes past 4 3 1
4 (a) 21 minutes (b) 28 minutes
(c) 50 minutes past 4 5 9
11 5 8

TS
6 (c) 30 minutes (d) 32 minutes
(d) 54 minutes past 4 13 11
11
19. A clock is set right at 5 a.m. The clock
16. A watch which gains 5 seconds in 3 loses 16 minutes in 24 hours. What is the
minutes was set right at 8 a.m. On the

H
correct time when the clock shows 10 p.m.
afternoon of the same day, when the on the fourth day?
watch indicated quarter past 5 o’clock, the (a) 11 p.m. (b) 12 p.m.

G
actual time is (c) 1 a.m. (d) 2 a.m.
(a) 4 p.m. (b) 5 p.m.
20. A watch which gains uniformly is 2 minutes
(c) 5.30 p.m. (d) none of these
17. A watch which gains uniformly is 4 minutes
U slow at noon on Monday, and is 4 minutes
48 seconds fast at 2 p.m. on the following
O
slow on Friday morning at 6 o’clock, and Monday. When was it correct?
is 4 minutes 36 seconds fast on the (a) Tuesday, 11.30 p.m.
following Friday at 10 o’clock in the
H
(b) Tuesday, 5 p.m.
morning. When was the watch correct? (c) Wednesday, 2 p.m.
(a) After 3 days 4 hours (d) Wednesday, 7 p.m.
_T

Answers with Solutions


C

1. Ans. (d) 3. Ans. (b)


Sol. In every hour, both hands coincide once.
PS

Sol. After every 12 hours, the system is


repeated. In 12 hours, the total number ⇒ In 12 hours, the hands coincide 11 times—
of strikes because between 11 o’clock and 1 o’clock,
= 1 + 2 + 3 + ........ + 11 + 12 the hands coincide only at 12 o’clock
U

⇒ In 24 hours, the hands coincide 22 times.


12 (1 + 12) n (n + 1)
=  Sn = 4. Ans. (b)
2
@

2
Sol. Let the man go out x minutes past 5
12 × 13
= 720 + 780 × 5
2 Then x = minutes past 5
143
= 78
∴ Total number of strikes = 2 × 78 = 156 720 + 3900 4620
= =
2. Ans. (c) 143 143
Sol. In one hour, the hands are at right 44 4
= 32 = 32 minutes past 5
angles twice; so in 12 hours, the hands 143 13
are at right angles 22 times,—the 5. Ans. (a)
exception because 2 positions—3 o’clock
Sol. The minute hand is ahead of 5.30. At
and 9 o’clock—are common. In 24
5 o’clock, the hands are 25 minute
hours, they are at right angles 44
spaces apart. Then for the hands to be
times.
436 ✫ Quantitative Aptitude

at right angles after 5.30, 25 + 15 = ∴ 24 hours of the correct time


40 minute spaces have to be gained = 24 hours + 24 minutes of the second
over the hour hand, i.e., 40 minutes 122
40 × 60 480 clock = hours
gained in = 5
55 11 ∴ 76 hours of the correct time
7
= 43 minutes. 122 76 1159
11 = × =
7 5 24 15
At 43 minutes past 5, the hands will
11 = 77 hours 16 minutes
be at right angles between 5.30 and 6

TS
o’clock. ∴ Required time on second clock
= 77 hours 16 minutes
6. Ans. (b)
On Monday, the second clock will
Sol. At 11 o’clock, the minute hand is 55-

H
show 3.16 p.m. when the correct time is
minute spaces apart. To be coincident,
it must gain 55-minute spaces. 2.00 p.m.

G
55 minutes gained in 60 minutes. 10. Ans. (c)
⇒ The hands coincide at 12 o’clock only. Sol. Angle traced by hour and minute hand

7. Ans. (a)
U at 2 o’clock =
360°
12
×2
O
Sol. At 9 a.m., both the minute hand and
= 60°
hour hand are 15-minute spaces apart.
11. Ans. (a)
To be in a straight line, i.e., in opposite
H
directions, it has to gain 30-minute Sol. The angle traced by the hour hand in
1 1
spaces. 2 hours = 2 × 30° = 75°.
_T

∴ It has to gain 30 – 15 = 15-minute 2 2


spaces over the hour hand. 12. Ans. (c)
i.e., 15-minutes will be gained in Sol. At 6 o’clock, the hands are already 180°
60 × 15 180 4 apart and in opposite directions. After
C

= = 16 minutes
55 11 11 this, both hands will not come in
i.e., the hands will be in opposite opposite directions.
PS

directions at 13. Ans. (b)


4 13
16 minutes past 9 a.m. Sol. Angle traced by hour hand in hours
11 3
360° 13
U

8. Ans. (b) = × = 130°


Sol. 55 minutes are gained in 60 minutes 12 3
60 Angle traced by minute hand in 20
@

60 minutes are gained in × 60 minutes


55
5 360°
= 65 minutes = × 20 = 120°
11 60
5
Loss in 66 minutes = 66 – 65 ∴ Required angle = (130° – 120°) = 10°
11
6
= minutes 14. Ans. (a)
11 Sol. Angle traced by hour hand in 12 hours
9. Ans. (b) = 360°
Sol. Time from 10.00 a.m. on Friday to 2.00 Angle traced by hour hand in 9 hours
p.m. on Monday 4 minutes
= 24 × 3 + 4 = 76 hours 4
The second clock gains 1 minute every i.e., 9 + hours
hour 60
136 360
∴ In 24 hours, the second clock gains 24 = × = 272°
15 12
minutes
Unit Five : Practice Session ✫ 437

15. Ans. (d) 5 5


⇒ 65 – 64 = 1 minutes
Sol. At 4 o’clock, the hands are 20-minute 11 11
16
spaces apart. To be in opposite Gained (i.e., fast) = minutes
11
directions, they must be 30-minute ∴ Time gained in 24 hours
spaces apart. 16 60
⇒ The minute hand has to gain (30 + 20 = × × 24
= 50) 50 minutes spaces 60 11 64
× 50 360 8
50 minutes are gained in = = 32
6 55 11 11
= 54 minutes 8

TS
11 i.e., The clock is fast by 32 minutes.
⇒ The hands of the clock are in opposite 11
6 19. Ans. (a)
directions at 54 minutes past 4 o’clock. Sol. From 1st day at 5 a.m. to 4th day at 10
11

H
16. Ans. (b) p.m., the number of hours = 89 hours
Sol. Number of hours from 8 a.m. to quarter 24 hours of the correct clock
past 5 o’clock = 9 hours 15 minutes

G
= 23 hours 44 minutes of this clock
= 555 minutes
The watch gains 5 seconds in 3 minutes 23 × 60 + 44
= minutes
⇒ 3 minutes 5 seconds of this watch
= 3 minutes on a correct watch
⇒ 3 × 60 + 5 seconds U =
1424
60
60
=
356
15
hours
O
= 3 minutes on correct watch i.e., 24 hours of correct clock
185 356
minutes = 3 minutes on a = hours of this clock
H
60 15
correct watch
∴ 89 hours of this clock
⇒ 555 minutes in this watch
24 × 15 × 89
_T

3 × 60 × 555 = hours of the correct clock


= 356
185 = 90 hours of the correct clock
= 540 minutes = 9 hours
⇒ Correct time = 9 hours after 8 a.m. i.e., true/correct time
C

= 5 p.m. = 11 p.m., on the 4th day


17. Ans. (a)
PS

Sol. Number of hours from Friday 6 a.m. to 20. Ans. (c)


following Friday 10 a.m. = 7 × 24 + 4 Sol. Number of hours from noon on Monday
= 172 hours to 2 p.m. on the next Monday
i.e., the watch gained 4 + 4 minutes 36 = 7 days + 2 hours
U

seconds in 172 hours = 170 hours


4 × 60 + 4 × 60 + 36 The watch gained (2 minutes + 4 minutes
@

i.e., 48 seconds) in 170 hours


60
516 i.e., the watch gained
= minutes in 172 hours
60 2 × 60 + 4 × 60 + 48
∴ The watch gained 4 minutes in minutes in 170 hours
60
172 × 60 × 4 408 34
= 80 hours ⇒ = minutes in 170 hours
516 60 5
= 3 days 8 hours
2 minutes slow on first Monday,
∴ The watch was correct after 3 days 8 hours
18. Ans. (d) 170 × 5 × 2
∴ 2 minutes are gained in
Sol. The hands of a correct clock coincide 34
5 = 50 hours
after every 65 minutes After 2 days and 2 hours
11
If it coincides every 64 minutes, then The watch was correct on Wednesday
the clock is gaining time at 2 p.m.
438 ✫ Quantitative Aptitude

Practice Exercise 37
1. If today is Tuesday, what day of the week 8. What should be the first of January, if the
will it be after 46 days? average usage in gm/brushing should be
(a) Monday (b) Thursday minimum?
(c) Saturday (d) Sunday (a) Monday (b) Tuesday
2. The number of odd days in a leap year is (c) Wednesday (d) Any of these
(a) 1 (b) 2
9. If the fourth Saturday of a month is the
(c) 3 (d) 4

TS
22nd day, what day is the 13th of that
3. Today is Friday. After 62 days, it will be month?
(a) Thursday (b) Saturday (a) Tuesday (b) Wednesday
(c) Sunday (d) Tuesday (c) Thursday (d) Friday

H
4. What day of the week will be 95 days
10. Monday falls on 4th April, 1988. What was
from today, if today is Monday?
the day on 3rd November, 1987?

G
(a) Monday (b) Wednesday
(a) Monday (b) Sunday
(c) Thursday (d) Friday
(c) Tuesday (d) Wednesday
5. How many days are there from April 18
to June 6, 2002?
(a) 45 days (b) 50 days U 11. The first republic day of India was celebrated
on 26th January, 1950. It was a
O
(c) 49 days (d) 51 days (a) Monday (b) Tuesday
(c) Thursday (d) Friday
Directions: Refer to the data below and answer
H
Questions 6 to 8. 12. The year next to 1988 having the same
Manoj uses a toothpaste tube of 400g in calendar as that of 1988 is
_T

a month. He brushes twice a day except (a) 1990 (b) 1992


on Sundays when he brushes only once (c) 1993 (d) 1995
a day. 13. Mahatma Gandhi was born on October 2,
C

6. If the first of January is Monday, then 1869. Find the day of the week.
what is his average usage in gm/brushing (a) Saturday (b) Friday
PS

in January? (c) Tuesday (d) None of these


(a) 6.72 (b) 6.89
14. Find the day of the week on June 6, 2002.
(c) 7.01 (d) 7.19
(a) Thursday (b) Sunday
U

7. If the first of January is a Sunday, then (c) Friday (d) Wednesday


what is his average usage in gm/brushing
in January? 15. July 4, 1964 was a
@

(a) 6.72 (b) 6.89 (a) Sunday (b) Monday


(c) 7.01 (d) 7.19 (c) Saturday (d) Friday

Answers with Solutions


1. Ans. (c) 2. Ans. (b)
Sol. Today is Tuesday, so Sol. A leap year has 52 weeks + 2 days. So,
7th day : Tuesday the number of odd days in a leap year
14th day : Tuesday is 2.
21st day : Tuesday 3. Ans. (a)
28th day : Tuesday Sol. Each day of the week is repeated after
35th day : Tuesday 7 days. So, after 63 days, it will be
42nd day : Tuesday Friday. Hence, after 62 days, it will be
∴ 46th day will be Saturday Thursday.
Unit Five : Practice Session ✫ 439

4. Ans. (d) Nov. Dec. Jan. Feb. March April


Sol. Same day of the week is repeated after Day 27 + 31 + 31 + 29 + 31 + 4
7 days = 153 days containing 6 odd days
∴ After 91 days, it would be Monday i.e., (7–6) = 1 day beyond the day on
∴ 95th day will be Friday. 4th April, 1988
5. Ans. (b) So, the day was Tuesday
Sol. April 18 May June 6 11. Ans. (c)
13 + 31 + 6 = 50 days Sol. 1600 has zero odd days

TS
6. Ans. (b) 300 years have 1 odd day
Sol.  1st January is Monday 49 years contain 12 leap years and
∴ Sunday falls on 7th, 14th, 21st, 28th 37 ordinary years and therefore,

H
∴ 4 Sundays + 27 other days (24 + 37) odd days, i.e., 5 odd days
∴ He brushes 4 × 1 + 27 × 2 i.e., 1949 years contain (0 + 1 + 5)

G
= 58 times in January or 6 odd days
26 days of January contain 5 odd days
400
∴ Average usage =

7. Ans. (c)
58
= 6.89
U Total odd days = 6 + 5 = 11
or 4 odd days
O
So, the day was Thursday.
Sol. 1st January is Sunday
∴ 8th, 15th, 22nd and 29th January are 12. Ans. (c)
H
also Sundays Sol. By counting the number of odd days till
the sum is divisible by 7
_T

So, there are 5 Sundays in January


Years 1988 1989 1990 1991 1992
∴ He brushes 5 × 1 + 26 × 2 = 57 times
400 Odd days 2 + 1 + 1 + 1 + 2
∴ Average usage = = 7.01 = 7, i.e., 0 odd days
57
C

∴ Calendar for 1993 is the same as that of


8. Ans. (d) 1988.
PS

Sol. If first of January is Monday or


13. Ans. (a)
Tuesday or Wednesday, Sunday comes
Sol. October 2, 1869
four times.
Jan Feb Mar Apr May Jun Jul Aug Sept Oct
U

∴ If 1st January is Sunday or Saturday


31 + 28 + 31 + 30 + 31 + 30 + 31 + 31 + 30 + 2
then Sunday comes 5 times.
Total days = 275 2 odd days
@

Average usage will be minimum if


1600 — 0 odd days
Sundays are less.
200 — 3 odd days
9. Ans. (c) Till 1868 (51 ordinary days) — 51 odd days
Sol. Ist Saturday = 22 – 21 = 1 Leap 17 years — 34 odd days
90
So the day on the 13th of the month will Odd days 0 + 3 + 51 + 34 + 2 =
7
be = 6 odd days
1 + 7 = 8 = Saturday October 6, 1869 was a Saturday.
and, 8 + 5 = 13 = Thursday 14. Ans. (a)
10. Ans. (c) Sol. June 6, 2002
Sol. By counting the number of days after 400 × 5 ⇒2000 – 0 odd day
3rd November, 1987 we have 2001 – 1 odd day
440 ✫ Quantitative Aptitude

2002 Jan Feb Mar Apr May June 186


31 28 31 30 31 6 = = 26 + 4
7
157 3 odd days 1600 300 63 years
⇒ 4 odd days There are 48 ordinary 15 leap years
7
⇒ June 6, 2002 was a Thursday 48 30
0 1 6 2 4
7 7
15. Ans. (c)
Sol. July 4th, 1964 = 1 + 6 + 2 + 4

TS
1964 Jan Feb Mar Apr May June July = 6 days
31 29 31 30 31 30 4 ⇒ July 4th, 1964 was a Saturday.

H
G
U
O
H
C _T
PS
U
@
UNIT SIX
✫ SERIES ✫ PERMUTATION AND COMBINATION
✫ PROBABILITY

FUNDAMENTALS AND

TS
FAST-TRACK FORMULAE

H
Series the successive numbers are obtained by
A succession of numbers formed according to dividing the previous number by 2. So the next

G
some definite rule or pattern is known as a number is (6 ÷ 2 =) 3. This is also a geometric
series. In a sequence that shows arithmetical series.
progression, the difference between two
successive terms of the sequence of numbers is U A series may be obtained by squaring or cubing
every successive number.
In the series,
O
constant. If the sequence shows geometric
progression, every term (except the first one) on 3, 9, 81, 6561
every number is squared to get the next number.
H
the sequence bears a constant ratio with its
preceding term. Complex Patterns
In the competitive examinations, however, the
_T

Arithmetic Series
In the series, pattern on which a series of numbers moves is
25, 30, 35, 40, 45, ... not confined to simple arithmetic or geometric
every successive number is obtained by adding progression. The difference between successive
C

5 to the previous number. The next number will terms need not be constant, even though the
be (45 + 5) = 50. difference is always based on a logical pattern.
PS

Take the series, However, in questions involving series, there


209, 200, 191, 182, 173, ... may be a mixture of patterns. There are no limits
Here the numbers diminish by 9 at successive to the number of such patterns, but the tools you
steps. The next number in the series will be need to work with are the basic arithmetical
U

(173 – 9) = 164. functions of addition, subtraction, multiplication


and division. And, since you may be required to
@

Geometric Series do without calculators in these tests, you must


In the series, learn to do these calculations both accurately
3, 9, 27, 81, 243, ... and fast.
every term (except the first one) bears a constant In a series of numbers, if the last term is
ratio of 3 with the preceding term: 3 × 3 = 9, given, it is a finite series. If the last term is not
9 × 3 = 27; and so on. The next number in the given, the series is infinite as it can go on
sequence will be (243 × 3) = 729. indefinitely:
Take the series 2, 4, 6, 8, 10, ... 12 (Finite)
3, –6, 12, –24, 48, ... 2. 4, 6, 8, 10, ... (Infinite)
This too is geometric, the constant ratio being –2:
3 × –2 = –6; –6 × –2 = 12 .... Difference between Consecutive Terms in
The next number in the series is (48 × –2) = –96. Arithmetic or Geometric Progression The
In the series, difference between the terms itself could be one
48, 24, 12, 6, ... of arithmetic progression, as in the series,

441
442 ✫ Quantitative Aptitude

1, 2, 4, 7, 11, 16 Points to Note


+1 +2 +3 +4 +5
● A look at the given series will tell you
1 1 1 1 whether it shows an increasing trend, a
Or, there could be decrease in arithmetic decreasing trend or a mixture of both.
progression. ● A gradually increasing trend indicates a
18, 14, 11, 9, 8 pattern based on addition of some number
at every step: it could be the same number
–4 –3 –2 –1 or a gradually increasing number.

TS
–1 –1 –1 ● A sharply increasing trend indicates a
The differences could follow a pattern of pattern based on multiplication or squaring
geometric progression as in and cubing of the given numbers.
21, 30, 57, 138, 381 ● A trend of increase that is initially sharp

H
Now, but gradually slows down indicates
21, 30, 57, 138, 381 addition of squared/cubed numbers.

G
● A haphazard progression—rising
Difference 9 27 81 243 irregularly or even falling now and then—
= 3 × 3 = 9 × 3 = 27 × 3 = 81 × 3 indicates either (i) a combination of series
= 32 = 33 = 34
The next number in the series will be
= 35
U (alternate terms sharing a pattern) or (ii)
a combination of alternating operations on
O
381 + (243 × 3) or 381 + 36 every term. The irregularity in the
= 381 + 729 = 1110. differences between terms in the case of
The difference could also be successive prime
H
(i) is likely to be much greater than in the
numbers, or squares/cubes of consecutive odd case of (ii).
or even numbers. ● Some of the various patterns and formats
_T

There Could be a mixture of operations in are illustrated below:


establishing the difference—say, addition (i) × 12 + 1; × 22 + 1; × 32 + 1 ...
followed by multiplication; multiplication (ii) +12 + 1; +22 + 2; + 32 + 3 …
followed by addition; addition and squaring/
C

(iii) × 1 + x; × 2 + x; × 3 + x; …
cubing; and so on. (where x is a specific number)
PS

Mixed Series The series could be a mixture of (iv) × 1 + x; × 2 + 2x; × 3 + 3x; …


two series: alternative numbers following their (where x is a specific number)
own patterns. (v) ÷ 2 × 1; ÷ 2 × 2; ÷ 2 × 3; …
Take The numbers may increase consecutively;
U

64, 32, 36, 18, 22 they may increase skipping one number; the
Looking at the first two terms, you note that powers may increase in a certain pattern.
@

dividing the first by 2 will get the second. The The difference between numbers may not
same is not true for the second, as dividing 32 even be according to the basic mathematical
by 2 will not get 36. However, an addition of operations; it could be a matter of prime
4 seems to be the correct rule here. Could these numbers, or the sum/product of the digits of
be the alternate steps? each term. The patterns are practically infinite.
Check.
64 32 36 18 22
÷2 +4 ÷2 +4 Worked Examples
The pattern is clear: the next step is of dividing In the examinations, various formats based on
by 2. The next term in the series will be 11. series have appeared. However, fundamental to
As already mentioned, the patterns are solving any such problem is the determination
inexhaustible. It is only through practice that of the pattern governing the series. Once that
you can determine patterns and solve questions pattern is discovered, the rest of the problem
fast. becomes easy to solve.
Unit Six : Fundamentals / Series ✫ 443

● What should come in place of the question At another level, if you take 54 and 36,
mark in the following number series? 54 – 36 = 18 (the next term); 18 – 12 = 6 (the
1, 4, 27, 256, 3125, ? next term); so 6 – 4 = 2; which is the term
We can see here that the terms show a we need.
sharply increasing trend. There seems to be In other words, patterns can be established
a geometric increase, in all probability in more than one way, though the result is
involving squares and cubes. We see that 4 the same.
is 22. The next term 27 is 33, and the next,
256, is 44. The series pattern is established: ● Find the missing number (or what number
will replace the question mark) in the series

TS
the powers of the numbers are the same as
the numbers, both showing consecutive 15, 25, 40, 130, ?, 2560
increase. The pattern in the series has to be determined.
1 4 27 256 3125 Here the increase is rather sharp but not

H
11 22 33 44 55 regular. Addition alone will not be the pattern;
6
The next term will be 6 = 46656. some multiplication or squaring/cubing is
likely. Since the last digit of all the number

G
● What should come next in the series is 5 or 0, one multiple must be 5 and the
5, 10, 17, 26, 37, 50 number added, if any, will also be having 5
The series is gradually increasing: there
seems to be addition of increasing numbers
at every step. Let’s determine the difference. U or 0 as the last digit.
Let’s break down the terms
O
15 25 40 130
5 10 17 26 37 50 65 Possible? 5 × 3 5 × 5 5 × 8 5 × 26
No pattern is obvious among 3, 5, 8 and 26.
H
Difference 5 7 9 11 13 15
Difference Try again—how is 15 related to 25?
2 2 2 2 2
There must be some multiplication involved
_T

At every step the difference between the going by the trend in the increase. So,
terms increases by 2. The next addition will
25 = 15 + 10 = 15 × 1 + 10
be of 13 + 2 or 15 to the term 50; the next
But it does not work for 40
term will be 65.
40 ≠ 25 × 1 + 10
C

● What comes next in the series Try out 25 × 2 + 10.


PS

108, 54, 36, 18, 12, 6, 4 Again, it is not 40.


The series is decreasing, so the operation However, 25 × 2 – 10 = 40
must involve either division or subtraction. Check out the alternate steps. If the pattern
As it is sharply decreasing, division is more is correct, the next term would be 40
U

likely. But the decrease is not regular, so multiplied by 3 and 10 added to the product.
there could be division by alternate numbers Check it out:
or a combination of division and subtraction
@

40 × 3 + 10 = 130 (✔)
(or even addition) at alternate steps. You
The pattern is multiplication of successive
may have to try out some operations.
terms by consecutively increasing numbers,
Determine the operation in one set:
with the product increased and reduced by
108 54 36 18 12 6 4 10 alternatively:
× 1 + 10; × 2 – 10; × 3 + 10;
÷2 ÷2 ÷2
× 4 – 10; × 5 + 10 …
At this very stage, you can establish the next
term: it will be 4 ÷ 2 = 2. However, just to Now the missing term. The multiplier of the
satisfy yourself about the other series or previous term increases by 1 to 4, and this
alternate operation, start with 6 and 4; time 10 will be subtraced from the product.
6 ÷ 4 = 1.5. So,
So is 18 ÷ 1.5 = 12; and 54 ÷ 36 = 1.5. So the 130 × 4 – 10 = 510
operation at the alternate positions is division The missing term must be 510. However, see
by 1.5. if the next operation leads to the next term.
444 ✫ Quantitative Aptitude

By the required operation, we have Here again, the pattern for the series has to
510 × 5 + 10 = 2560 (the given last term) be determined. We see that the series is not
uniformly rising, but the increase in the later
● In the series given below, one term is wrong.
Pick out the wrong term and give the correct part is steep. So multiplication must be
term: involved. Let’s take the first three terms 8,
4, 11, 25, 53, 108, 221, 445 6 and 9.
In such items, too, you need to determine the 8 × 1 – 2 = 6
pattern for the series, but it is a little more But 6 × 1 – 2 ≠ 9
difficult than just locating the next term or So the multiplier must be increasing:

TS
the missing term. The wrong term may be 6 × 2 – 2 ≠ 9
any one of the given terms, even the first So the added number must also be increasing:
one. So, some bit of trial and error may be 6 × 2 – 3 = 9 (correct)
inevitable. Start at the beginning, however, Let’s try the pattern for the next terms.

H
and determine the difference between terms. 9 × 3 – 4 = 27 – 4 = 23 (Correct)
4 11 25 53 108 221 445 23 × 4 – 5 = 92 – 5 = 87 (Correct)

G
So the series pattern is:
Difference +7 +14 +28 +55 +113 +224 8 × 1 – 2 =6
=7×1 =7×2 =7×4
= 7 × 20 = 7 × 2 1 = 7 × 22
= 7 × 32
= 7 × 25U 6×2–3 =9
9 × 3 – 4 = 23
O
A look at the first three numbers for the 23 × 4 – 5 = 87
difference between terms indicates that they 87 × 5 – 6 = 429
H
are multiplies of 7. The break occurs with Now you are asked to find the term that will
108. But once again the difference between come in place of (c) in the series following
the same pattern. Since you are asked for (c),
_T

221 and 445 is 224, which is a multiple of 7.


So the difference between the terms in all you can stop at the third term:
likelihood must be a multiple of 7. Now take 6 × 1 – 2 = 4 (a)
55 which is the difference between 53 and 4 × 2 – 3 = 5 (b)
C

108. By just adding 1 to the difference (i.e., 5 × 3 – 4 = 11 (c)


55 + 1 = 56) we get a difference that is a So 11 will come in place of (c).
PS

multiple of 7; 53 + 56 = 109. Check with the


● Follow the same directions as in the previous
next term: 221 – 109 = 112, again a multiple
example for the following:
of 7. Clearly, the wrong term is 108 which
2 3 10 39 172 885
U

must be replaced by 109.


1 (a) (b) (c) (d) (e)
[If the difference is reduced to the form of
What will come in place of (b)?
7 × 2 raised to the required power, it becomes
@

even more easy to locate the wrong term. Here, the series is increasing and steeply so;
After 7 × 22, it will be 7 × 23 or 56; so the most probably addition and multiplication/
correct term is 53 + 56 = 109.] squaring are involved. After some trial and
error, the pattern is established:
● In the following series, a number series is 2 × 1 + 12 = 3
given. After the series, below it, a number 3 × 2 + 22 = 6 + 4 = 10
alongwith (a), (b), (c), (d) and (e) is given, 10 × 3 + 32 = 30 + 9 = 39
which together form a series. You have to 39 × 4 + 42 = 156 + 16 = 172
complete the series following the same 172 × 5 + 52 = 860 + 25 = 885
sequence as that of the given series. Then
answer the question that follows. Since you are asked to find the second term
8 6 9 23 87 429 (b), you can stop after that:
6 (a) (b) (c) (d) (e) 1 × 1 + 12 = 2 → (a)
What will come in place of (c)? 2 × 2 + 22 = 4 + 4 = 8 → (b)
Unit Six : Fundamentals / Permutation and Combination ✫ 445

∴ From the above series we can see that Permutation and Combination
the required number in place of (b) = 8 Questions are often asked on the lines, ‘in how
● In the following number-series, only one many ways can seven children be seated at a
number is wrong. If the wrong number is round table’ or ‘in how many ways can 3 balls
corrected, the series gets established be picked up of a group of 10 and kept in a
following a certain logic. Below the series a row’. One can get the answer to such questions
number is given followed by (a), (b), (c), (d), without actually writing down all the different
(e) and (f). You have to complete the series possibilities. Some basic principles and formulae
following the same logic as in the given will have to be understood and remembered.

TS
series after correcting the wrong number. A permutation of objects is the number of
possible arrangements for that set of objects.
2 3 2 15 76 246 1434
If 5 different bottles are to be arranged on a
3 (a) (b) (c) (d) (e) (f)

H
shelf, there are 5 choices for the first spot on
What will come in place of (c)?
the shelf, 4 choices for the second spot, 3
Here, not only has the pattern of the series choices for the third spot, 2 choices for the

G
to be determined; the wrong number in the fourth spot, and only 1 left for the fifth spot.
series has to be located and corrected before The total number of arrangements for those 5
going on to find the required number for the
given term in the next series.
U bottles can be represented by an operation:
5 ! = 5 × 4 × 3 × 2 × 1 = 120. This is known
O
The series shows an increase followed as 5 factorial.
by decrease in the first three terms. The rest The factorial of a whole number n is the
of the terms show increasing trend, sharply
H
product of that number and each of the natural
so. So some squaring and possibly subtracting numbers less than the number.
is required. It is written as n or n !
_T

At a simple level, 2 + 1 = 3; 3 – 1 = 2; however, = n × (n–1) × (n–2) × … × 1


you cannot get 15 in the same way. Try again.
Multiplication Principle of Counting
2 × 1 + 1 = 3; 3 × 1 – 1 = 2. How do you get If an operation can be performed in m different
C

15? Or is 15 the wrong term? Suppose we ways, following which a second operation can
have 3 × 2 = 6. 6 + 9 = 15. But 9 is 32. So can be performed in n different ways, the two
PS

we work out a pattern starting with 2 × 1 + operations in succession can be performed in


(1)2 and 3 × 2 – (2)2 … ? m × n different ways.
The pattern is: ● How many possible outcomes are there
U

2 × 1 + (1)2 = 2 + 1 = 3 if a coin is tossed four times?


3 × 2 – (2)2 = 6 – 4 = 2 2 × 2 × 2 × 2 = 16
@

2 × 3 + (3)2 = 6 + 9 = 15 ● There are 4 entrances and 3 exits to a


15 × 4 – (4)2 = 60 – 16 = 44 hall. In how many ways can one enter
44 × 5 + (5)2 = 220 + 25 = 245 into and exit from the hall?
245 × 6 – (6)2 = 1470 – 36 = 1434 4 × 3 = 12
∴ The wrong number is 76 in whose place
should come 44. Addition Principle of Counting
Similarly the next series can be formed If an operation can be performed in m different
in the following way: ways and another operation, independent of
3 × 1 + (1)2 = 4 → (a) the first one, can be performed in n different
4 × 2 – (2)2 = 4 → (b) ways, then either of the two operations can be
4 × 3 + (3)2 = 21 → (c) performed in (m + n) ways.
∴ The required number (c) = 21 ● There are 8 doors in a room, two on
We can stop here as we have the required one side and 6 on another. If a man
term. has to go out of the room, he can do
446 ✫ Quantitative Aptitude

so in 8 ways. He can go out of 2 doors of one type and q are alike and of one type, and
on one side and out of 6 doors on the rest are different
another, the total being (6+2) = 8 ways. n!
=
Permutation p!q!
The word permutation means arrangement. Each ● There are 4 red, 3 white and 2 blue
of the arrangements that can be made by taking marbles in a bag. They are drawn one
some or all of a number of dissimilar things or by one and arranged in a row. Assuming
objects is called permutation. that all the 9 marbles are drawn,
In other words, the number of permutations determine the number of different

TS
(P) of n objects taken n at a time is arrangements.
P = n!
n n We have n = 9, P1 = 4, P2 = 3 and
(n Pn and nPn mean the same.) P3 = 2

H
(i) At times, all the objects are not considered ∴ The required number of different
for each different arrangement. In such a case, arrangements
n represents the total number of objects from

G
which one has to choose, and r is the number n!
=
actually chosen to be arranged in a particular P1 ! P2 ! P3 !
way. In this case,
P or P(n,r) =
n! U =
9!
4!3!2!
O
n r
(n − r)! 9 × 8 × 7 × 6 × 5 × 4!
● There are 6 runners in a race. How =
4! × 3 × 2 × 1 × 2 × 1
H
many different orders are there for the
9× 8 ×7 × 6× 5
first, second and third places? =
3× 2 × 2
_T

n!
Use the formula nPr = = 1260.
(n − r )!
6! 6! Circular Permutation When the objects are
P = =
C

6 3 (6 − 3)! 3! arranged in a circle, then circular permutation


can be applied.
6× 5× 4× 3× 2×1
PS

= = 120 . (i) The number of circular permutations of


3× 2×1 n
Pr
[There is a faster way. There are 3 n different things taken r at a time is .
different places (first, second and third) r
(ii) Number of arrangements of n different
U

to be considered. There are 6 different


objects in a circle
choices for the first place, 5 for the
second place, and 4 for the third. So = (n – 1) !
@

the number of arrangements are 6 × 5 ● In how many ways can seven people be
× 4 = 120.] seated at a round table?
(ii) The number of permutations of n Required number of ways = (n – 1)!
different things taken r at a time when each = (7 – 1)!
thing may be repeated any number of times is = 6!
nr . = 6 × 5 × 4 × 3 × 2 × 1
● In how many ways can 5 mangoes be = 720.
distributed among 3 boys, there being
no restriction to the number of mangoes (iii) Number of arrangements of n different
each boy may get? things in a circle when clockwise or anticlockwise
arrangements are not different (i.e., when
The required number of ways = 35
observations can be made from both sides)
= 243.
1
(iii) The number of permutations of n things, = (n–1) !
taken all at a time, out of which p are alike and 2
Unit Six : Fundamentals / Permutation and Combination ✫ 447

● Find the number of ways in which 6 (i) The formula for the number of
different beads can be arranged to form combinations (C) of n dissimilar objects taken
a necklace. r at a time is
n
Required number of arrangements n! Pr
C = =
n r r ! (n − r )! r!
1
= (6 − 1)! n( n − 1) (n − 2)...(n − r + 1)
2 =
r (r − 1) (r − 2)...3.2.1
1
= × 5! [nCr may also be written as n
Cr]
2

TS
1
= × 5 × 4 × 3 × 2 Note that nCn will always be 1, as there
2 is only one way to select all the numbers of
= 60. a group.

H
● A question paper has two parts I and II,
Points to Remember each containing 10 questions. If a

G
candidate has to choose 8 from Part I and
(i) The number of permutations of n things 5 from Part II, in how many ways can the
taken together when P are alike and the

rest are different is


n!
. U questions be selected?
Using the formula nCr =
n!
, from
O
P! r ! (n − r )!
(ii) The number of permutations of n Part I, the choice will be
dissimilar things taken not more than
H
10! 10 × 9 × 8 × 7 × 6 × 5 × 4 × 3 × 2 × 1
r at a time when repetition is allowed =
8! (10 − 8)! 8 × 7 × 6 × 5 × 4 × 3 × 2 × 1 × (2 × 1)
n (np − 1)
_T

is . 10 × 9
n −1 =
(iii) When n different things taken r at a 2
time in which ‘p’ particular things do From Part II, the choice will be
C

not occur is (n–p)Pr. 10! 10 × 9 × 8 × 7 × 6 × 5 × 4 × 3 × 2 × 1


=
(iv) n Pr + r.n P(r −1) = ( n+1) Pr 5! (10 − 5)! 5 × 4 × 3 × 2 × 1 × (5 × 4 × 3 × 2 × 1)
PS

(v) n.( n−1) P(r −1) = n Pr 10 × 9 × 8 × 7 × 6


=
(vi) (n − r + 1). n P( r−1) = n Pr 5× 4× 3× 2× 1
Total number of choices will be
(vii) n−1Pr + r n−1P( r−1) = n Pr
U

10 × 9 10 × 9 × 8 × 7 × 6
(viii) The number of permutations of n ×
different things taken r at a time in 2 5× 4× 3× 2×1
@

which ‘p’ particular things are present = 5 × 9 × 3 × 2 × 7 × 6 = 11340.


is (n− p ) P × rP (ii) Number of combinations of n different
( r− p ) p
things taken r at a time
Combination (a) When p particular things are always
Each of the different groups or selections which included
can be made by taking some or all of a number = n–pCr–p
of things (irrespective of order) is called a (b) When p particular things are not
combination. included
In other words, a combination is the total = n–pCr
number of groupings of a set of objects. Order
is not important with combinations. The grouping ● In how many ways can 6 members
of objects changes the number of combinations forming a committee out of 11 be selected
that exist. so that
448 ✫ Quantitative Aptitude

● In how many ways can 5 members


More Formulae to Remember forming a commitee out of 10 be selected
so that two particular members must
(a) C = nCn = 1
n 0 not be together in any selection.
(b) C = n
n 1
When two particular members are not
(c) If nCx = nCy, either x = y together, then,
or y = n – x or x + y = n = nCr – n – pCr – p
(d) C + nCr–1 = C
n r n+1 r = C –
10 5
C
10 – 2 5 – 2

TS
(e) Number of selections of r consecutive
= C – 8C3
things out of n things in a row 10 5
= n–r + 1 10! 8!
(f) Number of selections of r consecutive = –
5! (10 − 5)! 5! (8 − 3)!

H
things out of n things in a circle
= n, when r < n; 10! 8!
= –

G
= 1, when r = n 5! 5! 3! 5!
(g) Number of ways of dividing m + n 10 × 9 × 8 × 7 × 6 × 5! 8 × 7 × 6 × 5!
different things in two groups = –
containing m and n things, respectively,
when m ≠ n U 5! × 5 × 4 × 3 × 2 × 1
= 252 – 56
3 × 2 × 5!
O
(m + n)! = 196.
C =
m+n m
m!n!
H
(i) three particular members must be
Fast-Track Formulae with
included. Worked Examples
_T

(ii) three particular members must not be


included. (i) The number of diagonals which can be
formed by joining the vertices of a polygon of
(i) When three particular members are n(n − 3)
C

included then, we have to select n sides are


2
6 – 3 = 3 members out of 11 – 3 = 8.
PS

∴ The required number of ways ● How many diagonals are there in a


8! octagon?
= C (8, 3) = The required number of diagonals
3! 5!
n(n − 3)
U

8 × 7 × 6 × 5! =
= 3 × 2 × 1 × 5! 2
8(8 − 3)
@

= 56. =
(ii) When three particular members are 2
not included, then, we have to select 8×5
6 members out of 11 – 3 = 8 = = 20.
2
∴ The required number of ways
(ii) The number of triangles which can be
8!
= C(8, 6) = formed by joining the angular points of a polygon
6! 2! n(n − 1)(n − 2)
8 × 7 × 6! of n sides as vertices are
= 6
6! × 2 × 1 ● Find the number of triangles formed by
= 28. joining the vertices of a hexagon.
(c) When p particular things are not The required number of triangles
together in any selection n(n − 1)( n − 2)
= nCr – n-pCr-p =
6
Unit Six : Fundamentals / Permutation and Combination ✫ 449

6(6 − 1)(6 − 2) (v) If there are ‘m’ horizontal lines and ‘n’
= vertical lines then the number of different
6
rectangles formed are given by
6 × 5× 4
= (mC2 × nC2)
6
= 20. ● In a chess board there are 9 vertical and
(iii) When there are n points in a plane and 9 horizontal lines. Find the number of
no points are collinear, then the number of rectangles formed in the chess board.
straight lines that can be drawn using these ‘n’ The required number of rectangles
points are given by n(n − 1) .

TS
= 9C2 × 9C2
2 9! 9!
● How many straight lines can be drawn = ×
2!(9 − 2)! 2!(9 − 2)!
with 15 points on a plane of which no

H
9! 9!
points are collinear? = ×
2!7! 2!7!
The required number of straight lines

G
9 × 8 × 7! 9 × 8 × 7!
n(n − 1) = ×
= 2 × 1 × 7! 2 × 1 × 7!

=
2
15(15 − 1)
U = 36 × 36 = 1296.
(vi) There are ‘n’ points in a plane out of
O
2 which ‘m’ points are collinear. The number of
15 × 14 triangles formed by the points as vertices are
= = 105.
H
2 given by nC3 – mC3.
(iv) In a party every person shakes hands ● There are 12 points in a plane out of
_T

with every other person. If there was a total of which 4 are collinear. Find the number
H handshakes in the party, then the number of of triangles formed by the points as
person ‘n’ who were present in the party can be vertices.
calculated from the equation:
C

n(n − 2) The required number of triangles


= H 12
2 = C3 – 4C3
PS

● In a party every person shakes hands 4!


12!
with every other person. If there was a = –
total of 120 handshakes in the party, 3!(12 − 3)! 3!(4 − 3)!
U

find the number of persons who were 12! 4!


present in the party. = –
3! × 9! 3! × 1!
@

Let ‘n’ be the number of persons present 12 × 11 × 10 × 9! 4 × 3!


in the party. = –
3 × 2 × 9! 3! × 1
We have the equation
= 220 – 4
n(n − 1)
= H = 216.
2 (vii) There are ‘n’ points in a plane out of
n(n − 1)
⇒ = 120 which ‘m’ points are collinear. The number of
2 straight lines formed by joining them are given
n2 – n = 240 by (nC2 – mC2 + 1).
n2 – n – 240 = 0
n2 – 16n + 15n – 240 = 0 ● There are 12 points in a plane out of
n(n – 16) + 15 (n – 16) = 0 which 6 are collinear. Find the number
(n – 16) (n + 15) = 0 of straight lines formed by joining
n = 16. them.
450 ✫ Quantitative Aptitude

The required number of straight lines restriction on the number of toy cars a boy
n m gets?
= C2 – C2 + 1
= 12C2 – 6C2 + 1 The required number of ways = 45
= 1024.
12! 6!
= – +1 ● In an examination paper, there are two parts,
2! (12 − 2)! 2! (6 − 2)! I and II, each containing 4 questions. A
12! 6! candidate has to attempt 5 questions in all
= – +1 but not more than 3 questions from any one
2! 10! 2! 4!
part. In how many ways can the questions

TS
12 × 11 × 10! 6 × 5 × 4! be selected?
= – +1
2 × 1 × 10! 2 × 1 × 4!
In this case, the sum of choices from each
= 66 – 15 + 1
part is 3 Qs from I and 2 Qs from II or 2 Qs
= 52.

H
from I and 3 Qs from II.
(viii) The number of quadrilaterals that can
One set of choices are 4C3 × 4C2
be formed by joining the vertices of a polygon

G
n(n − 1)(n − 1)(n − 3) 4! 4! 4! 4!
of n sides are given by , × = ×
24 3! (4 − 3)! 2!(4 − 2)! 3! 1! 2! 2!
where n > 3.
● Find the number of quadrilaterals that U =
4× 3× 2×1 4× 3× 2×1
×
3 × 2 × 1 × 1 2 × 1× 2 × 1
= 4×6
O
can be formed by joining the vertices of
Another set of choices are 4C2 × 4C3
an octagon. 4! 4!
×
H
The required number of quadrilaterals 2!(4 − 2)! 3!(4 − 3)!
n(n − 1)(n − 2)(n − 3) 4!
×
4!
_T

= =
24 2! 2! 3! 1!
8(8 − 1)(8 − 2)(8 − 3) 4× 3× 2×1 4 × 3× 2×1
= = × =6×4
24 2 ×1× 2 × 1 3 × 2 ×1× 1
C

8×7×6×5 Total number of choices


=
24 = 4C3 × 4C2 + 4C2 × 4C3
PS

= 70. =4×6+6×4
= 24 + 24 = 48.
Worked Examples
U

● How many different arrangements of 5


● There are 5 green, 3 black and 2 white balls students can be made in a row of 3 desks?
in a bag. They are all drawn out one by one
@

at random and arranged in a row. How Since the order is important here, it is a
many possible arrangements are there? permutation problem. Five students in 3
desks can be arranged in 5P3 = 5 × 4 × 3 = 60
Here n = (5 + 3 + 2) = 10, different arrangements.
p = 5, q = 3; r = 2
Different arrangements possible ● How many 3-digit numbers can be formed
from the digits 2, 3, 5, 6, 7 and 9, which are
10 ! divisible by 5 and in which none of the digits
=
5 ! 3 ! 2! is repeated?
10 × 9 × 8 × 7 × 6 × 5 × 4 × 3 × 2 × 1 We need to consider units, tens and hundreds
= = 2520. place for a 3-digit number. If the number is
5× 4× 3× 2×3× 2× 2
to be divisible by 5, the unit place of the
● In how many ways can 5 toy cars be number must be 5. So there is only 1 way of
distributed among 4 boys, there being no getting the unit place right.
Unit Six : Fundamentals / Permutation and Combination ✫ 451

In the tens place, any of the other five numbers 7! 6!


can be placed: so there are 5 different ways = 3! (7 − 3)! × 2! (6 − 2)!
of filling the tens place.
7! 7!
The hundreds place can now be filled with + ×6+
any of four digits: so there are 4 different 4! (7 − 3)! 5! (7 − 5)!
ways of filling the place. 7 × 6 × 5 × 4! 6 × 5 × 4!
∴ The number of 3-digit numbers from the = 3 × 2 × 1 × 4! × 2 × 1 × 4!
given digits will be (1 × 5 × 4) = 20
7 × 6 × 5 × 4! 7 × 6 × 5!
+ ×6+

TS
● In how many different ways can the letters
3 × 2 × 1 × 4! 2 × 1 × 5!
of the word ‘RUMOUR’ be arranged?
7×6×5 6×5
The given word has 6 letters: = ×
R-2, U-2, M-1 and O-1 3× 2×1 2×1

H
So one can arrange the letters in 7×6×5 7×6
+ ×6 +
6! 3× 2×1 2×1

G
(2 !) (2 !) (1 !) (1 !) = (525 + 210 + 21) = 756.
6 × 5 × 4 × 3 × 2 × 1 720
=
2 × 1× 2 × 1× 1× 1
=
4
= 180 ways.
U ● The letters of the word PROMISE are
arranged so that no two vowels come
together. Find the total number of
O
● In how many different ways can the letters arrangements.
of the word ‘JUDGE’ be arranged in such a
The four consonants can be written in 4! ways
H
way that the vowels always come together?
=4×3×2×1
While the word has 5 letters, the given = 24 ways
_T

condition is that vowels always come The three vowels can be written in 3!
together. There are two vowels, U and E, ways = 3 × 2 × 1
and if we always take them together, we = 6 ways
Since no two vowels can come together,
C

may consider them as one entity. So there are


4 letters to arrange: therefore vowels can be inserted in any
JDG (UE), which can be done in three places out of the five places available,
PS

4! = 4 × 3 × 2 × 1 = 24 ways. such as P R M S
However, the vowels, U and E, can be i.e. in 5C3 ways
arranged among themselves in 2 ways. So 5!
=
U

the required number of ways is 3!.2!


24 × 2 = 48. 5 × 4 × 3!
= = 10 ways
@

3! × 2
● From a group of 7 men and 6 women, five ∴ Total number of arrangements required
persons are to be selected to form a = 24 × 6 × 10
committee so that at least 3 men are there on = 1440.
the committee. In how many ways can it be
done? ● A box contains 10 balls out of which 3 are red
and the rest are blue. In how many ways can
There are three ways in which there can be at
a random sample of 6 balls be drawn from
least 3 men on the committee of 5:
the bag so that at the most 2 red balls are
3 men + 2 women; 4 men + 1 woman; or 5 men. included in the sample and no sample has all
There are 7 men and 6 women. the 6 balls of the same colour?
So the number of ways will be The possible ways are as follows:
1 red ball out of the three and 5 blue balls
( 7
C 3 × 6 C2 ) + ( 7
C4 × 6 C1 ) + ( 7
C5 ) out of the seven = 3C1 × 7C5
452 ✫ Quantitative Aptitude

2 red balls out of the three and 4 blue balls ● The different letters of an alphabet are
out of the seven = 3C2 × 7C4 given. Words with five letters are formed
∴ Total number of ways in which a random from these given letters. Find the number of
sample of six balls can be drawn words which have at least one letter repeated.
= 3C1 × 7C5 + 3C2 × 7C4 The total number of words that can be formed
with five letters out of ten given letters = 105
3 7 ×6×5×4×3 3× 2 7 × 6 × 5× 4
= × + × = 100000
1 5 × 4 × 3 × 2 ×1 2 ×1 4 × 3 × 2×1 The total number of words that can be
= 3 × 21 + 3 × 35 formed with five distinct letters

TS
= 63 + 105 = 168. = 10 × 9 × 8 × 7 × 6
= 30240
● A cricket team of 11 players is to be formed ∴ The total number of words in which at least
from 20 players including 6 bowlers and 3 one letter is repeated

H
wicket keepers. Find the number of ways in = 100000 – 30240
which a team can be formed having exactly = 69760.

G
4 bowlers and 2 wicket keepers.
● In how many ways is it possible to choose
There are 6 bowlers, 3 wicket keepers and a white square and a black square on a chess
11 batsmen in all.
The number of ways in which a team of 4
U board so that the squares do not lie in the
same row or column?
O
bowlers, 2 wicket keepers and 5 batsmen can
be chosen. A black square can be chosen in 32 ways.
= 6C4 × 3C2 × 11C5 Once a black square is there, you cannot
H
choose the 8 white squares in its row or
6×5×4×3 3× 2 11 × 10 × 9 × 8 × 7 column, so the number of white squares
= × ×
_T

4 × 3× 2×1 2 ×1 5× 4× 3× 2×1 available = 24


= 15 × 3 × 462 ∴ Number of ways = 32 × 24
= 20790. = 768.
C

● In how many different ways can the letters


of the word TRAINER be arranged so that
PS

the vowels always came together? Probability


Ordinarily speaking the probability of an event
The word ‘TRAINER’ contains 7 different
denotes the likelihood of its happening. If an
letters.
event is certain to happen its probability would
U

When the vowels AIE are always together,


be 1 and if it is certain that the event would not
they can be supposed to form one letter.
take place, then the probability of its happening
@

Thus,
is 0. Therefore, probability is always measured
we have TRNR (AIE).
between 0 to 1.
This has 5 letters of which R occurs 2 times
and the rest are different. Certainty of Event Percentage Probability
Number of ways of arranging these letters
Will occur 100% 1
5! 5× 4× 3× 2×1 Will not occur 0% 0
= =
2! 2 ×1 Chance to occur 25% ¼
Chance to occur 50% ½
= 60.
Since (AIE) can also be arranged in 3! ways, Probability is used to predict the outcomes
required number of ways of random experiments. Random experiments
= 60 × 3! are those experiments in which results are not,
= 60 × 3 × 2 × 1 however, essentially same even though the
= 360. conditions may be nearly identical. A few
Unit Six : Fundamentals / Probability ✫ 453

examples of random experiments are: drawing with a random experiment is ‘p’ (0 ≤ p ≤ 1),
a card from a pack of playing cards; tossing a i.e., P(E) = p, then
coin; selecting an odd number from a given set (i) the chance that event E happens is p
of numbers; throwing a pair of dice; and drawing (ii) the chance that event E fails to happen
a ball of a particular colour from a bag having is q or 1–p
balls of different colours.
(iii) the odds in favour of event E are
A pack of cards has 52 cards. It has 13 cards p : q or p : (1–p)
of each suit, namely spades, clubs, hearts and
(iv) the odds against the event E are q : p
diamonds. Cards of spades and clubs are black

TS
or (1–p) : p
cards. Cards of hearts and diamonds are red
cards. There are four honours of each suit. These Thus, according to this theory, the
are Aces, Kings, Queens and Jacks. These are probability of getting the number 6 in a single
1

H
called face cards and other cards are called throw of a dice (or p) is and the probability that
number cards. 6 5
this event will not take place (or q = 1–p) is .

G
When we throw a coin, then either a Head 6
(H) or a Tail (T) appears.
Sample Space
A dice is a solid cube, having 6 faces, marked
1, 2, 3, 4, 5, 6 respectively. When we throw a dice,
the outcome is the number that appears on its U The set of all possible outcomes of an experiment
is called a sample space.
O
upper face. Examples of sample space:
● In tossing a coin, S = {H, T}
H
Measuring Probability ● If two coins are tossed, then S = {HH, HT,

There are different ways of measuring TH, TT}


_T

probability. ● In rolling a dice, we have, S = {1, 2, 3, 4,

Classical Method When the event (E) occurs 5, 6}


in m different ways out of a total number of n
C

possible ways, all of which being equally likely, Event


m To perform a random experiment is called a trial
then the probability of the event P(E) is .
PS

n and the outcome of the trial is known as an


Frequency Method If an event (E) is repeated event. Event can also be defined as the occurrence
n number of times (n being very large) and it in defined context. For example, getting tails in
occurs only m times, the probability of the the toss of a coin is an event whereas the context
U

m is ‘one coin tossed’. Another term for event is


event, P(E) is .
n outcome.
@

For example, when a dice is rolled once, we Probability of an Event Probability of an


have 6 equally likely sample points, 1, 2, 3, 4, event denotes the likelihood of its happening. If
5 and 6 and hence the probability of each of an event A can happen in m ways and fails to
1 happen in n ways then the probability P(A) of
these 6 outcomes is , i.e.,
6 happening of an event A is given by
1
P(1) = P(2) = P(3) = P(4) = P(5) = P(6) = Total number of
6
Similarly, if E is the event, ‘a number multiple favourable cases to A
P(A) =
Total number of exhaustive
of 3 shows up’, then E = {3, 6} and hence P(E) =
equally likely cases
2 1 m
= =
6 3 m+n
If the probability of an event E associated Since the event fails in n ways, the probability
454 ✫ Quantitative Aptitude

of failing or not happening of the event, denoted


Probability theory Set theory
by P(A ) , is expressed as
n m Sample space S
P( A) = = 1− = 1 − P(A) Outcome or a sample point ω
m+n m+n
Event A A⊂ S
 P(A) + P ( A) = 1
Event A has occurred ω ∈A
The value of probability ranges from 0 to 1. The
maximum value of probability is 1, i.e., the Event A has not occurred ω ∉A
A ⊂ B

TS
event is certain to happen, and its minimum Event A implies event B
value 0, indicates that the event will never take Event ‘not A’ AC
place.
Event ‘A or B’ A ∪B

H
● A coin is tossed once. What are all Event ‘A and B’ A ∩B
possible outcomes? What is the
probability of the coin coming up ‘tails’? Event ‘A but not B’ A – B = A ∩ BC

G
Events A and B are A∩ B = φ
The coin come up either ‘heads’ (H) or
mutually exclusive
‘tails’ (T). Thus, the set S of all possible
outcomes is S = {H, T}
U
O
1 Types of Events Let us get clear about the
∴ P (T) = types of events.
2
H
● What is the probability of getting an 1. Mutually exclusive events Two events
even number in a single throw of a die? A and B are called mutually exclusive if they do
not occur simultaneously, i.e., if A ∩ B = φ . For
_T

Clearly, a die can fall with any of its faces


example, the events A = {1, 2, 4} and B = {3, 5, 6}
uppermost. The number on each of the
associated with the experiment of rolling a dice
faces is, therefore, a possible outcome.
are mutually exclusive.
C

Thus there are total 6 outcomes. Since


there are 3 even numbers on the die, ● If E1 is the event “drawing an ace from
namely, 2, 4 and 6,
PS

a deck of cards and E2 is the event


3 1 4 1
P (even number) = = . “drawing a king”, then P(E1) = =
6 2 52 13
4 1
and P(E2) = = . What is the proba-
U

● What is the probability of drawing a 52 13


‘king’ from a well-shuffled deck of 52 bility of drawing either an ace or a king
cards?
@

in a single draw?
Well-shuffled ensures equally-likely P(E1+E2) = P(E1) + P(E2)
outcomes. There are 4 king in a deck. 1
1
Thus, = +
4 1 13 13
P(a king) = = .
52 13 2
Theoretic Notations of Events Let A and B = .
be two events of a random experiment whose 13
sample space is 5. We have observed that the 2. Complementary events Two events
events and the relations among them can be best where it is certain that one or the other will occur,
described by using concepts of ‘set theory’. The but not both the events together are
following table gives the equivalent forms of complementary events. For example, if a single
probability statements and the corresponding coin is tossed, the event ‘heads’ and the event
statements in the set theory. ‘tails’ are complementary.
Unit Six : Fundamentals / Probability ✫ 455

3. Compatible events Two events when Odd against getting a ‘3’


they happen simultaneously are called Number of unfavourable outcomes
compatible events. =
Number of favourable outcomes
5
‘Odds in Favour of’ and ‘Odds Against’ = or 5 to 1
an Event 1
● If the odds in favour of an event are 3 to
Ratio of probability of success to the probability
of failure is called ‘odds in favour of’ event A. If 5, find the probability that it will occur.
odds in favour are m : n. 3

TS
The odds in favour of the event are .
5
P ( A) m P ( A) 3
then, = Thus, =
P( A ) n 1 − P( A) 5

H
Ratio of probability of failure to the probability 5P(A) = 3 – 3P(A)
of success is called the ‘odds against’ the 5P(A) + 3P(A) = 3
happening of the event A

G
8P(A) = 3
P( A ) 3
i.e., odds against A = P(A) =
P ( A)
If odds against are m : n then odds in
favour of A are n : m and U ∴
8
The probability that it will occur = .
3
8
O
n Probability Theorems
P( A) =
m+n Important theorems of probability are as follows:
H
● Suppose odds in favour of an event A (i) Addition theorems (a) If A and B are two
_T

are 6 : 9, then find the probability of the mutually exclusive events associated with a
event A? random experiment, then
P(A or B) = P(A) + P(B)
m
P ( A) = i.e., P(A ∪ B) = P(A + B) = P(A) + P(B)
m+n
C

(b) If A and B are any two events associated


Here m = 6, n = 9 with a random experiment, then
PS

m 6 6 2 P(A or B) = P(A) + P(B) – P(A and B)


= = = i.e., P(A ∪ B) = P(A) + P(B) – P(A ∩ B)
m + n 6 + 9 15 5
(c) Thus if the probabilities of N mutually
Note : If the event A implies the event exclusive events are P1, P2, P3, Pn then the
U

B, then P(A) < P(B). probability that one of these events will happen
would be
● What are the odds in favour of getting a
@

‘3’ in a throw of a die? What are the odds P1 + P2 + P3 + ... Pn


against getting a ‘3’? The theorem of addition discussed above
will not be applicable when the events are not
There is only one outcome favourable to mutually exclusive (where two or more events
the event ‘getting’ a 3, the other five can take place together). The addition formula
outcomes, namely, 1, 2, 4, 5, 6 are in such cases then takes the following shape:
unfavourable. Thus, odds in favour of P (A or B) = P (A) + P (B) – P (AB)
getting a ‘3’. If there are three such events which overlap,
the probability
Number of favourable outcomes
= P (A or B or C) = P (A) + P (B) + P (C) – P (AB)
Number of unfavourable outcomes
– P (AC) – P (BC) + P (ABC)
1
= or 1 to 5 ● In a large metropolitan area, the
5 probabilities that a family owns a colour
456 ✫ Quantitative Aptitude

television set, a black and white set, or P(A) ≠ 0 and P(B) ≠ 0, then
both kind of sets are 0.86, 0.35 and 0.29. B
What is the probability that a family P(A ∩ B) = P(A) P
A
owns either or both kinds of sets? A
P(A ∩ B) = P(B) P
Let A be the event having a colour set B
and B be having a black and white set (b) If A and B are two independent events,
P(A) = 0.86, P(B) = 0.35, P( A ∩ B) = 0.29 then
P(A ∪ B) = P(A) + P(B) – P(A ∩ B) P(A ∩ B) = P(A) . P(B)
P(A ∪ B) = 0.86 + 0.35 – 0.29 = 0.92 P( A ∩ B) = P( A) . P( B)

TS
P( A ∩ B) = P( A) . P( B)
● If from a pack of cards a single card is
drawn, what is the probability that it is P( A ∩ B) = P( A) . P( B)
either a spade or a king? [Note: P( A) = 1 − P( A) and P(B) = 1 − P( B)]

H
13
The probability of a spade card = ● There are three events A, B, C one of
4 52

G
The probability of a king = which must happen and only one can
52 happen at a time. The odds are 8 to 3
But in the spade card also there will against A, 5 to 2 against B. Find the
be a king. As such if we add the above
two probabilities we will be counting U odds against C.
O
the spade king twice. Therefore the The given events are mutually exclusive
answer would be: and exhaustive.
Probability of a spade or a king ∴ P(A ∪ B ∪ C)= P(A) + P(B) + P(C)
H
= Probability of a spade and P(A ∪ B ∪ C) = 1
+ Probability of a king

_T

P(A) + P(B) + P(C) = 1 ...(i)


– Probability of (spade × king)
Odds against A are 8 : 3
13 4 13 4 3 3
= + − ×
52 52 52 52 ∴ P(A) = =
8+3 11
C

17 1 16 4 Odds against B are 5 : 2.


= − = =
PS

52 52 52 13 2 2
∴ P(B) = =
Note that the probability of a card 5+2 7
being both (A) and (B) is P (A) × P (B). 3 2
1 ∴ + + P (C ) = 1 from (i)
U

In the above case the value is . 11 7


52
3 2
Otherwise also if 13 cards of spade and P(C) = 1 − −
@

4 kings were added, we would have 11 7


subtracted 1 from the total (as the 77 − 21 − 22 34
= =
thirteen cards of spade already contain 77 77
the spade king) and thus the total
∴ Odds against C = P(C ) : P(C)
number of favourable cases would have
been 13 + 4 – 1 or 16. Since the total 34 34
number of ways in which a card can = 1− :
77 77
be drawn is 52, the required probability
of drawing a spade or a king would 43 34
= : = 43 : 34.
16 4 77 77
be or .
52 13 ● A box of fuse contains 20 fuses, of which
(ii) Multiplication theorems (a) If A and 5 are defective. If 3 of the fuses are
B are two events associated with a random selected at random and removed from
experiment such that the box without replacement, what is
Unit Six : Fundamentals / Probability ✫ 457

the probability that all the three fuses 4 2


would be defective? = +
15 15
Suppose A, B and C are the events that 6 2
all the three fuses are defective. = = .
15 5
5 B 4 C 3
P( A) = ,P = ,P = (ii) P (only one of them be selected)
20 A 19 A∩B 18
= P (not A and not B)
P( A ∩ B ∩ C )
= P (not A) × P (not B)
B C  1  1
= P ( A) × P ×P

TS
A A ∩B = 1−  × 1− 
 3  5
5 4 3 1 3−1 5−1
= × × = . = ×
20 19 18 114 3 5

H
Independent Events 2 4
= ×
Suppose A and B are two events and the 3 5

G
occurrence or non-occurrence of A does not 8
= .
effect the probability of occurrence of B, it is 15
called an independent event otherwise they are
called dependent events. U Worked Examples
● If two coins are tossed 4 times, then what
O
P ( A ∩ B) = P(A) . P(B) is the probability that we get heads on both
● Prateek and Durgesh appear for an the coins?
H
interview for two vacancies. The
1 Result can be indicated as
probability of Prateek’s selection is
3 (Head–H, Tail–T)
_T

1
and that of Durgesh’s selection is . Find (TT, HT, TH, HH)
5 So, the probability of having heads on both
the probability that
(i) only one of them will be selected, the coins is only 25%.
C

(ii) none of them will be selected. ● If the probability that A will be alive in 20
Let A: Prateek is selected years is 0.7 and the probability that B will
PS

and B: Durgesh is selected be alive in 20 years is 0.5, then what is the


probability that they both will be alive in 20
Then, years?
1 1
U

P(A) = and P(B) = P(A) = 0.7


3 5
P(B) = 0.5
Clearly, ‘A’ and ‘not B‘ are independent,
@

The probability that they will both be alive


also ‘not A’ and ‘not B’ are independent,
in 20 years is (0.7) (0.5) = 0.35.
‘B’ and ‘not A‘ are independent.
● A consumer research organisation has
(i) P (only one of them will be selected)
studied the services under warranty
= P(A and not B or B and not A)
provided by 50 new car dealers in a certain
= P(A) P (not B) + P(B) P (not A)
city, and its findings are as given below:
1  1  1 1
= 1 −  +  1 −  Time Good service Poor Service
3  5  5 3 under under
warranty warranty
1  5 − 1 1  3 − 1 In business 10 16 4
=   +  
3 5  5  3  years and more
1 4 1  2 In business less 10 20
=   + 5   than 10 years
3 5  3
458 ✫ Quantitative Aptitude

If a person randomly selects one of these new (a) Probability that events A and B are
car dealers, what is the probability that he independent is
will get one who provides good service 1 1 1
under warranty? P ( A ∩ B ) = P ( A) . P ( B ) = × = .
4 2 8
n(G) 16 + 10 (b) Probability that events B and C are
P(G) = = = 0.52
n(S) 50 dependent is
1 3 3
Here P(G) = Probability of good dealer P(B ∩ C ) ≠ P(B) . P(C ) = × =
giving warranty 2 8 16
3
n(G) = Number of elements in G But here P(B ∩ C ) ≠

TS
16
n(S) = Number of elements in whole
sample space ∴ Events B and C are not independent.
● A speaks truth in 75 per cent cases and B in
● Find the probability of randomly drawing 80 per cent of the cases. In what percentage

H
two aces in succession from an ordinary of cases are they likely to contradict each
deck of 52 playing cards, if we sample other, narrating the same incident?

G
(a) without replacement
Let A = Event that A speaks the truth
(b) with replacement
and B = Event that B speaks the truth

second card is drawn, then probability of


U
(a) If the first card is not replaced before the
Then, P (A) =
75 3
= , P (B) =
100 4
80
=
100 5
4
O
getting aces in succession is
3 1
4 3 1 ∴ P ( A) = 1 − =
× = = 0.0045. 4 4
H
52 51 221
(b) If the first card is replaced before the 4 1
and P (B ) = 1 − =
_T

second is drawn, the probability of getting 5 5


aces is P (A and B contradict each other)
4 4 1
× = = 0.0059. = P [(A speaks the truth and B tells a lie)
52 52 169
C

or (A tells a lie and B speaks the truth)]


● Three coins are tossed 8 times (HHH, HHT,
HTH, THH, HTT, THT, TTH and TTT). If = P [(A and B ) or ( A and B)]
PS

A is the event that a head occurs on each of


= P (A and B ) + P ( A and B)
the first two coins, B is the event that a tail
occurs on the third coin, and C is the event = P (A) . P ( B ) + P ( A ) . P (B)
U

that exactly two tails occur in the three


3 1 1 4 3 1
coins, show that = × + × = +
4 5 4 5 20 5
@

(a) events A and B are independent 7 7


(b) events B and C are dependent = = × 100 % = 35%
20 20
A = 2 times {HHH, HHT}
∴ A and B contradict each other in 35% of
B = 4 times {HHT, HTT, THT, TTT}
the cases.
C = 3 times {HTT, THT, TTH}
[As given in result of tosses: HHH, HHT, ● In a simultaneous throw of two dice, what
HTH, THH, HTT, THT, TTH and TTT] is the probability of getting a doublet?
A ∩ B = {HHT} In a simultaneous throw of two dice,
B ∩ C = {HTT, THT}
n(S) = (6 × 6) = 36
According to the above information we get:
1 1 3 A ‘doublet’ means that both the dice show
P(A) = , P(B) = , P(C) = the same number on the uppermost faces.
4 2 8
1 1 Let E = event of getting a doublet
P ( A ∩ B) = and P ( B ∩ C ) = = {(1, 1), (2, 2), (3, 3), (4, 4), (5, 5), (6, 6)}
8 4
Unit Six : Fundamentals / Probability ✫ 459

Thus n(E) = 6 ● A bag contains 2 red, 3 green and 2 blue balls.


n(E) Two balls are drawn at random. What is the
∴ P(E) = probability that none of the balls drawn is
n(S)
6 1 blue?
= = .
36 6 The total number of balls
● Two cards are drawn from a pack of 52 = 2 + 3 + 2 = 7
cards. Find the probability that either both Let S be the sample space. Then,
are red or both are kings. n(S) = Number of ways of drawing 2 balls
out of 7 = 7C2

TS
52
n(S) = C2
7 ×6
52 × 51 = = 21
= = 1326 2× 1
2×1

H
Let E1 = event of getting both red cards, Let E = Event of drawing 2 balls, none of
E2 = event of getting both kings. which is blue.
Then, ∴ n(E) = Number of ways of drawing 2 balls

G
E1 ∩ E2 = event of getting 2 kings of red out of (2 + 3) balls
cards. = 5C 2
∴ n(E1) = 26C2
26 × 25 U =
5×4
2× 1
= 10
O
= = 325
2×1 n(E)
∴ P(E) =
H
n(E2) = 4C2 n(S)
4×3 10
= =6 = .
_T

2×1 21
2 2×1
n(E1 ∩ E2) = C2 = ● A party of n men is to be seated round a
2×1 circular table. Find the probability that two
C

= 1 particular men sit together.


n(E1 ) 325
∴ P (E1) = = Total number of ways in which n men can
PS

n(S) 1326
sit round a circular table = (n – 1)!
n(E2 ) 6
P (E2) = = Now, if two particular men (say A and B) sit
n(S) 1326
1 together, then the number of ways in which
U

P (E1 ∩ E2) = (n – 1) men sit round table = (n – 2)!


1326
∴ P (both red or both kings) = P (E1 ∪ E2) But the two men can interchange their seats
@

= P (E1) + P (E2) – P (E1 ∩ E2) in two ways.


∴ Number of ways that two particular men sit
 325 6 1 
=  + −  together =2 (n – 2)!
 1326 1326 1326  ∴ Probability that two particular men sit
330 2( n − 2)!
= together =
1326 (n − 1)!
55 2
= . = .
221 n −1
460 ✫ Quantitative Aptitude

PRACTICE SESSION
Practice Exercise 38
Directions: What will come in place of the question- 10. 1 7 49 343 (?)
mark (?) in each of the following number series? (1) 16807 (2) 1227
(3) 2058 (4) 2401
1. 4 5 13 40 104 ?
(5) None of these

TS
(1) 229 (2) 308
(3) 315 (4) 241 11. 13 20 39 78 145 (?)
(5) None of these (1) 234 (2) 244
(3) 236 (4) 248

H
2. 8 9 13 22 ? 63 (5) None of these
(1) 31 (2) 41 12. 12 35 81 173 357 (?)

G
(3) 36 (4) 38 (1) 725 (2) 715
(5) None of these (3) 726 (4) 736
3. 7 6
(1) 520
10 27 104
(2) 420
?
U (5) None of these
13. 3 100 297 594 991 (?)
O
(3) 515 (4) 525 (1) 1489 (2) 1479
(5) None of these (3) 1478 (4) 1498
(5) None of these
H
4. 3 7 15 ? 63 127
(1) 35 (2) 37 14. 112 119 140 175 224 (?)
_T

(3) 33 (4) 29 (1) 277 (2) 276


(3) 287 (4) 266
(5) None of these
(5) None of these
5. 2.5 4 ? 10 14.5 20 26.5 15. 8844 8115 7603 7260 7044 ?
C

(1) 8 (2) 7.5 (1) 6911 (2) 6919


(3) 6 (4) 5.5 (3) 6916 (4) 6917
PS

(5) None of these (5) None of these


6. 4 5 12 39 160 805 ? 16. 12 13 28 87 352 ?
(1) 4836 (2) 3224 (1) 1765 (2) 1775
U

(3) 5642 (4) 4030 (3) 1760 (4) 1770


(5) None of these (5) None of these
@

7. 8 108 189 253 302 ? 363 17. 2 1 1 1.5 3 ?


(1) 351 (2) 327 (1) 7 (2) 6.5
(3) 338 (4) 311 (3) 7.5 (4) 6
(5) None of these (5) None of these
8. 248 217 188 165 ? 129 116 18. 11 21 40 77 150 ?
(1) 144 (2) 136 (1) 298 (2) 295
(3) 134 (4) 146 (3) 293 (4) 299
(5) None of these (5) None of these
9. 3 15 39 75 123 183 ? 19. 7 16 34 61 97 ?
(1) 255 (2) 218 (1) 144 (2) 145
(3) 243 (4) 225 (3) 143 (4) 142
(5) None of these (5) None of these
Unit Six : Practice Session ✫ 461

20. 30 46 78 126 190 270 ? 23. 186 94 48 25 ? 7.75


(1) 356 (2) 366 (1) 13.5 (2) 14.8
(3) 382 (4) 398 (3) 12.5 (4) 14
(5) 414 (5) None of these
21. 1250 500 200 80 32 12.8 ? 24. 384 381 372 345 264 ?
(1) 5.12 (2) 6.4 (1) 23 (2) 25
(3) 4.3 (4) 6.02 (3) 43 (4) 24
(5) 5.16 (5) None of these
22. 23 26 24 27 25 28 ? 25. 282 286 302 ? 402 502

TS
(1) 27 (2) 29 (1) 366 (2) 318
(3) 26 (4) 24 (3) 326 (4) 338
(5) 21 (5) None of these

H
Answers with Solutions
1. Ans. (1) 5. Ans. (5)

G
Sol. (0)2 + 4 = 4 Sol. The sequence is
(0 + 1)2 + 4 = 1 + 4 = 5 +1.5, +2.5, +3.5, +4.5, +5.5, +6.5
(1 + 2)2 + 4 = 9 + 4 = 13
(3 + 3)2 + 4 = 36 + 4 = 40 U Then,
2.5 + 1.4 = 4,
O
(6 + 4)2 + 4 = 100 + 4 = 104 ? = 4 + 2.5 = 6.5.
? = (10 + 5)2 + 4 = 225 + 4 6. Ans. (1)
H
? = 229. Sol. The sequence is
2. Ans. (4) ×1 + 1, ×2 + 2, ×3 + 3, ×4 × 4, ×5 × 5, ×6 × 6
_T

Then,
Sol. The sequence is
4 × 1 + 1 = 4 + 1 = 5,
+(1)2, +(2)2, +(3)2, +(4)2, +(5)2 5 × 2 + 2 = 10 + 2 = 12,
Then, 12 × 3 + 3 = 36 + 3 = 39,
C

8 + (1)2 = 8 + 1 = 9, 9 + (2)2 = 9 + 4 = 13, 39 × 4 + 4 = 156 + 4 = 160,


13 + (3)2 = 13 + 9 = 22, 160 × 5 + 5 = 800 + 5 = 805
PS

? = 22 + (4)2 ? = 805 × 6 + 6
? = 22 + 16 = 38. ? = 4830 + 6 = 4836.
3. Ans. (3) 7. Ans. (3)
U

Sol. The sequence is Sol. The sequence is


× 1 – 1, × 2 – 2, × 3 – 3, × 4 – 4, × 5 – 5 +(10)2, +(9)2, +(8)2, +(7)2, +(6)2, +(5)2
Then,
@

Then,
7 × 1 – 1 = 7 – 1 = 6, 6 × 2 – 2 = 12 – 2 = 10, 8 + (10)2 = 8 + 100 = 108,
10 × 3 – 3 = 30 – 3 = 27, 108 + (9)2 = 108 + 81 = 189,
27 × 4 – 4 = 108 – 4 = 104 189 + (8)2 = 189 + 64 = 253,
? = 104 × 5 – 5 253 + (7)2 = 253 + 49 = 302
? = 520 – 5 ? = 302 + (6)2 = 302 + 36 = 338.
? = 515. 8. Ans. (4)
4. Ans. (5) Sol. The sequence decreases by consequent
Sol. The sequence is prime numbers
× 2 + 1, × 2 + 1, × 2 + 1, × 2 + 1, × 2 + 1 –31, –29, –23, –19, –17, –13
Then, Then,
3 × 2 + 1 = 6 + 1 = 7, 7 × 2 + 1 = 14 + 1 = 15 248 – 31 = 217, 217 – 29 = 188,
? = 15 × 2 + 1 188 – 23 = 165
? = 165 – 19 = 146.
? = 30 + 1 = 31.
462 ✫ Quantitative Aptitude

9. Ans. (1) 7603 – (7)3 = 7603 – 343 = 7260,


Sol. The sequence is 7260 – (6)3 = 7260 – 216 = 7044
+12, +24, +36, +48, +60, +72 ? = 7044 – (5)3
Then, ? = 7044 – 125
3 + 12 = 15, 15 + 24 = 39, 39 + 36 = 75, ? = 6919.
75 + 48 = 123, 123 + 60 = 183
16. Ans. (1)
? = 183 + 72 = 255.
Sol. The sequence is
10. Ans. (4) × 1 + 1, × 2 + 2, × 3 + 3, × 4 + 4, × 5 + 5
Sol. 1 7 49 343 ? Then,

TS
Clearly, the given series is: 12 × 1 + 1 = 12 + 1 = 13,
1 × 7 = 7, 7 × 7 = 49, 49 × 7 = 343 13 × 2 + 2 = 26 + 2 = 28,
? = 343 × 7 28 × 3 + 3 = 84 + 3 = 87,
? = 2401. 87 × 4 + 4 = 348 + 4 = 352

H
11. Ans. (4) ? = 352 × 5 + 5
Sol. 13 20 39 78 145 (?) ? = 1760 + 5 = 1765.

G
Clearly, the sequence is: 17. Ans. (3)
+7, +19, +39, +67, +103 Sol. The sequence is

+12 +12+8 +20+8


∴ ? = 145 + 103
28+8
U × 0.5, × 1.0, × 1.5, × 2.0, × 2.5
Then,
O
2 × 0.5 = 1, 1 × 1.0 = 1, 1 × 1.5 = 1.5,
= 248. 1.5 × 2.0 = 3
12. Ans. (1) ? = 3 × 2.5
H
Sol. 12 35 81 173 357 (?) ? = 7.5.
Clearly, the sequence is: 18. Ans. (2)
_T

+23, +46, +92, +184, +368 Sol. The sequence is


× 2 – 1, × 2 – 2, × 2 – 3, × 2 – 4, × 2 – 5
×2 ×2 ×2 ×2 Then,
∴ ? = 357 + 368
C

11 × 2 – 1 = 22 – 1 = 21,
= 725. 21 × 2 – 2 = 42 – 2 = 40,
13. Ans. (5) 40 × 2 – 3 = 80 – 3 = 77,
PS

Sol. 3 100 297 594 991 (?) 77 × 2 – 4 = 154 – 4 = 150


Clearly, the sequence is: ? = 150 × 2 – 5
+97, +197, +297, +397, +497 ? = 300 – 5
U

? = 295.
+100 +100 +100 +100
∴ ? = 991 + 497 19. Ans. (4)
@

Sol. The sequence is


= 1488.
+ 9, + 18, + 27, + 36, + 45
14. Ans. (3) Then,
Sol. 112 119 140 175 224 (?) 7 + 9 = 16, 16 + 18 = 34,
Clearly, the sequence is: 34 + 27 = 61,
+7 × 1, +7 × 3, +7 × 5, +7 × 7, +7 × 9 61 + 36 = 97,
∴ ? = 224 + (7 × 9) ? = 97+ 45
= 224 + 63 ? = 142.
= 287.
15. Ans. (2) 20. Ans. (2)
Sol. The sequence is Sol. 30 46 78 126 190 270 ?
–(9)3, –(8)3, –(7)3, –(6)3, –(5)3 Adding 16, 32, 48, 64, 80, 96 to
Then, consecutive numbers we get the next
8844 – (9)3 = 8844 – 729 = 8115, number
8115 – (8)3 = 8115 – 512 = 7603, ∴ ? = 270 + 96 = 366.
Unit Six : Practice Session ✫ 463

21. Ans. (1) 1


? = 25 × +1
Sol. 1250 500 200 80 32 12.8 ? 2
Multiply each number by 0.4 to get the ? = 12.5 + 1
next number ∴ ? = 13.5.
? = 12.8 × 0.4 = 5.12.
24. Ans. (5)
22. Ans. (3) Sol. 384 381 372 345 264 ?
Sol. 23 26 24 27 25 28 ? The given sequence follows the pattern:
By adding 3, –2, 3, –2 consecutively we –3, –9, –27, –81, –243

TS
get the next number Then,
? = 28 – 2 = 26. 384 – 3 = 381; 381 – 9 = 372;
23. Ans. (1) 372 – 27 = 345;

H
Sol. 186 94 48 25 ? 7.75 345 – 81 = 264;
The given sequence follows the pattern: ∴ ? = 264 – 243 = 21

G
1 1 1 1 1
× + 1, × + 1, × + 1, × + 1, × + 1 25. Ans. (4)
2 2 2 2 2
Then,
1 1 U Sol. 282 286 302 ? 402 502
The given sequence follows the pattern:
O
(186 × ) + 1 = 94; (94 × ) + 1 = 48; +(2)2, +(4)2, +(6)2, +(8)2, +(10)2
2 2
1 282 + (2)2 = 286; 286 + (4)2 = 302;
(48 × ) + 1 = 25
H
2 ? = 302 + (6)2 = 338
C _T
PS
U
@
464 ✫ Quantitative Aptitude

Practice Exercise 39
Directions: In each of the following questions a 10. 4 6 18 49 201 1011
number series is given. Only one number is wrong in (1) 1011 (2) 201
each series. You have to find out the wrong number. (3) 18 (4) 49
(5) None of these
1. 10 15 24 35 54 75 100
(1) 35 (2) 75 11. 48 72 108 162 243 366
(3) 24 (4) 15 (1) 72 (2) 108

TS
(5) 54 (3) 162 (4) 243
(5) None of these
2. 1 3 4 7 11 18 27 47
(1) 4 (2) 11 12. 2 54 300 1220 3674 7350

H
(3) 18 4. 7 (1) 3674 (2) 1220
(5) 27 (3) 300 (4) 54

G
(5) None of these
3. 3 2 3 6 12 37.5 115.5
(1) 37.5 (2) 3 13. 8 27 64 125 218 343
(3) 6
(5) 12
(4) 2
U (1)
(3)
27
125
(2) 218
(4) 343
O
(5) None of these
4. 2 8 32 148 765 4626 32431
14. 4 6 12 30 75 315 1260
H
(1) 765 (2) 148
(1) 315 (2) 75
(3) 8 (4) 32
(3) 12 (4) 6
(5) 4626
_T

(5) 30
5. 2 3 11 38 102 229 443
15. 3 4 13 38 87 166 289
(1) 11 (2) 229
(1) 38 (2) 13
C

(3) 102 (4) 38


(3) 87 (4) 166
(5) 3
(5) 4
PS

6. 439 778 1456 2812 5624 10948


16. 4 5 9 29 111 556 3335
(1) 5624 (2) 1456
(1) 5 (2) 9
(3) 778 (4) 2812
(3) 29 (4) 111
U

(5) None of these (5) 556


7. 156 468 780 1094 1404 1716 17. 2 6 16 38 84 176 368
@

(1) 468 (2) 1094 (1) 6 (2) 16


(3) 1717 (4) 780 (3) 38 (4) 84
(5) None of these (5) 176
8. 113 130 164 215 293 368 18. 7 4 6 9 20 52.5 160.5
(1) 215 (2) 130 (1) 6 (2) 4
(3) 164 (4) 293 (3) 20 (4) 9
(5) None of these (5) 52.5
9. 36 54 135 472.15 2126.25 11694.375 19. 172 37 2530 350 2440
(1) 135 (2) 54 (1) 37 (2) 253
(3) 472.15 (4) 11694.375 (3) 350 (4) 2440
(5) None of these (5) None of these
Unit Six : Practice Session ✫ 465

20. 132 286 693 176 342 23. 2 8 3 27 5 125 9 343


(1) 132 (2) 286 (1) 9 (2) 5
(3) 693 (4) 342 (3) 27 (4) 9
(5) None of these (5) None of these
21. 1 6 13 24 37 55 24. 318 158 86 38 18 8 3
(1) 6 (2) 13 (1) 38 (2) 3
(3) 24 (4) 55 (3) 158 (4) 318
(5) None of these (5) 86
22. 480 96 24 6 4 4 25. 3 10 36 172 885 5346 37471

TS
(1) 92 (2) 24 (1) 10 (2) 5336
(3) 6 (4) 480 (3) 885 (4) 36
(5) None of these (5) 172

H
Answers with Solutions

G
1. Ans. (1) 3. Ans. (5)
Sol. The given series is Sol. The given series is
10 15 24 35 54 75 100
The series can be obtained by adding
U 3 2 3 6 12 37.5 115.5
The pattern is as follows:
O
alternate odd numbers starting from 5 3 × 0.5 + 0.5 = 2
to the respective numbers in the series. 2 × 1+1 = 3
Therefore we get the series as: 3 × 1.5 + 1.5 = 6
H
10 + 5 = 15 6 × 2 + 2 = 14
15 + 9 = 24 14 × 2.5 + 2.5 = 37.5
_T

24 + 13 = 37 37.5 × 3 + 3 = 115.5
37 + 17 = 54 We can see that the wrong number is 12.
54 + 21 = 75 14 is the correct number.
75 + 25 = 100
C

4. Ans. (4)
The number which is wrong is 35. The Sol. The given series is
correct number is 37.
PS

2 8 32 148 765 4626 32431


2. Ans. (5) The series is formed by multiplying the
Sol. The given series is each number by a natural number and
1 3 4 7 11 18 27 47 at the same time adding the square of
U

A number is the difference between the that particular natural number to it.
succeeding number and the preceding We can get the sequence in the following
@

number. manner:
∴ We get the series as 2 × 2 + 22 = 8
4–1 =3 8 × 3 + 32 = 33
7–3 =4 33 × 4 + 42 = 148
11 – 4 = 7 148 × 5 + 52 = 765
18 – 7 = 11 765 × 6 + 62 = 4626
27 – 11 = 16 4626 × 7 +72 = 32431
We can see that here instead of 27 it We can see from the above sequence
should be 29. Then only would we get that the number which is wrong is 32.
the difference as 18. The correct number is 33.
29 – 11 = 18 5. Ans. (2)
47 – 18 = 29 Sol. The given series is
The wrong number is 27; 29 is the correct 2 3 11 38 102 229 443
number. The pattern in the series to add the cube
466 ✫ Quantitative Aptitude

of successive natural numbers starting 10. Ans. (3)


from 1 to the consecutive terms: Sol. 4 6 18 49 201 1011
2 +13 = 3 The given sequence follows the pattern:
3 + 23 = 3 + 8 = 11 × 1 + 2, × 2 + 3, × 3 + 4, × 4 + 5, × 5 + 6
11 + 33 = 11 + 27 = 38 Then,
4 × 1 + 2 = 6; 6 × 2 + 3 = 15;
38 + 43 = 38 + 64 = 102
15 × 3 + 4 = 49; 49 × 4 + 5 = 201;
102 + 53 = 102 + 125 = 227
201 × 5 + 6 = 1011
227 + 63 = 227 + 216 = 443 ∴ The wrong number is 18.
∴ The wrong number is 229.

TS
11. Ans. (5)
6. Ans. (1) Sol. 48 72 108 162 243 366
Sol. 439 778 1456 2812 5624 10948 The given sequence follows the pattern:
Multiply each term by 2 and subtract × 1.5, × 1.5, × 1.5, × 1.5, × 1.5

H
100 to get the next term Then,
439 × 2 – 100 = 778 48 × 1.5 = 72; 72 × 1.5 = 108;

G
778 × 2 – 100 = 1456 108 × 1.5 = 162; 162 × 1.5 = 243;
1456 × 2 – 100 = 2812 243 × 1.5 = 364.5
∴ The wrong number is 364.5.
2812 × 2 – 100 = 5524
∴ 5624 is the wrong number. U 12. Ans. (1)
O
Sol. 2 54 300 1220 3674 7350
7. Ans. (2) The given sequence follows the pattern:
Sol. 156 468 780 1094 1404 1716 × 6 + 6 × 7, × 5 + 5 × 6, × 4 + 4 × 5,
H
Multiply 156 by 3, 5 7, 9, … to get × 3 + 3 × 4, × 2 + 2 × 3
consecutive terms Then,
_T

156 × 3 = 468; 156 × 5 = 780; 2 × 6 + 6 × 7 = 54


156 × 7 = 1092 54 × 5 + 5 × 6 = 300
1094 is the wrong number. 300 × 4 + 4 × 5 = 1220
1220 × 3 + 3 × 4 = 3672
C

8. Ans. (4) 3672 × 2 + 2 × 3 = 7350


Sol. 113 130 164 215 293 368 ∴ The wrong number is 3674.
PS

The given sequence follows the pattern


13. Ans. (2)
of adding consecutive multiples of 17 to Sol. 8 27 64 125 218 343
the consecutive terms: 23 = 8; 33 = 27; 43 = 64; 53 = 125;
U

+17, +34, +51, +68 63 = 216; 73 = 343


Then, ∴ The wrong number is 218.
113 + 17 = 130; 130 + 34 = 164; 164 + 51
@

14. Ans. (2)


= 215; 215 + 68 = 283 Sol. The given sequence is
∴ 293 is the wrong number. 4 6 12 30 75 315 1260
9. Ans. (3) There are two series formed of the
Sol. 36 54 135 472.15 2126.25 11694.375 alternate terms:
Multiply consecutive terms by (i) 4, 12, 75, 1260
3 5 7 (ii) 6, 30, 315 ...
, , , ... 3 5
2 2 2 In (i), multiply the numbers by , ,
7 2 2
3 5 respectively and in (ii) multiply the
36 × = 54; 54 × = 135 2
2 2 numbers by 2, 3, 4 respectively
7 9 ∴ We get,
135 × = 472.5; 472.5 × = 2126.25
2 2 3
∴ 472.15 is the wrong number. 4× = 6, 6 × 2 = 12,
2
Unit Six : Practice Session ✫ 467

5 sequence by 2. And to their products we


12 × = 30, 30 × 3 = 90 add the even natural numbers (starting
2
7 from 2).
90 × = 315, 315 × 4 = 1260 2×2+2=6
2
6 × 2 + 4 = 16
We can see that all but the number 75
16 × 2 + 6 = 38
satisfies the given sequence.
38 × 2 + 8 = 84
∴ 75 is the wrong number.
84 × 2 + 10 = 178
90 is the correct number.
178 × 2 + 12 = 368

TS
15. Ans. (4) We can see that 176 is the wrong
Sol. The given sequence is number. The correct number is 178.
3 4 13 38 87 166 289
The difference between corresponding 18. Ans. (1)

H
numbers in the sequence are Sol. The given sequence is
1 9 25 49 79 123 7 4 6 9 20 52.5 160.5
The given series can be fomed in the

G
The first four numbers given above can
be written as following way:
12 32 52 72 7 × 0.5 + 0.5 = 4
The pattern of the given sequence is
3 + 12 = 4 U 4×1+1=5
5 × 1.5 + 1.5 = 9
O
4 + 32 = 4 + 9 = 13 9 × 2 + 2 = 20
13 + 52 = 13 + 25 = 38 20 × 2.5 + 2.5 = 52.5
H
38 + 72 = 38 + 49 = 87 52.5 × 3 + 3 = 160.5
87 + 92 = 87 + 81 = 168 The wrong number in the sequence is 6.
168 + 112 = 168 + 121 = 289
_T

The correct number is 5.


∴ We can see that the wrong number in
the sequence is 166. The correct number 19. Ans. (3)
should be 168. Sol. There appears to be no logical pattern
C

at first sight. However, the pattern


16. Ans. (3)
appears when you add the digits:
Sol. The given sequence is
PS

172 : 1 + 7 + 2 = 10
4 5 9 29 111 556 3335
In the given sequence, the pattern is: 37 : 3 + 7 = 10
4×1+1=5 2530 : 2 + 5 + 3 + 0 = 10
350 : 3 + 5 + 0 = 8
U

5×2–1=9
9 × 3 + 1 = 28 2440 : 2 + 4 + 4 + 0 = 10
28 × 4 – 1 = 111 The term 350 does not fit the pattern
@

111 × 5 + 1 = 556 as the sum of the digits in the case


556 × 6 – 1 = 3335 of each of the other terms is 10.
Here we are multiplying the numbers in 20. Ans. (4)
the given sequence by natural numbers Sol. Here in each term, the middle digit
and at the same time we are alternately is the sum of the first and third
adding 1 to the product and subtracting digits. The only term that does not
1 from the product. fall into this pattern is 342, as
So we get 29 as the wrong number. The 3 + 2 ≠ 4.
correct number is 28.
21. Ans. (4)
17. Ans. (5) Sol. The pattern is to add consecutive
Sol. The given sequence is prime numbers to successive terms.
2 6 16 38 84 176 368 1 + 5 = 6
Here we multiply the numbers in the 6 + 7 = 13
468 ✫ Quantitative Aptitude

13 + 11 = 24 24. Ans. (5)


24 + 13 = 37 Sol. Given series
37 + 17 = 54 318 158 86 38 18 8 3
So instead of 55, which is wrong, it The given series can be formed by
should be 54. dividing each number by 2 and
subtracting 1 from the quotient at every
22. Ans. (3) step.
Sol. The series is a descending one. The 318 ÷ 2 – 1 = 158
last two terms are 4; 4 ÷ 1 = 4. 158 ÷ 2 – 1 = 78
Now take the first two terms. 78 ÷ 2 – 1 = 38

TS
480 ÷ 5 = 96. Then, 96 ÷ 4 = 24 38 ÷ 2 – 1 = 18
Clearly, consecutive terms are being 18 ÷ 2 – 1 = 8
divided by consecutive numbers in 8÷2–1=3
∴ We can see that the wrong number is 86.

H
descending order starting from 5.
So 24 is to be divided by 3 to get 25. Ans. (4)

G
8. But here it is 6 which is wrong. Sol. Given series
If we get 8, we divide it next by 3 10 36 172 885 5346 37471
2 to get 4, and 4 ÷ 1 is 4. The given series can be formed in the
23. Ans. (4)
Sol. Every alternate term is a prime U following way:
3 × 2 + 22 = 10
O
10 × 3 + 32 = 39
number in consecutive order, so 39 × 4 + 42 = 172
after 5, it should be 7, not 9. [Each
H
172 × 5 + 52 = 885
prime number is cubed to get the 885 × 6 + 62 = 5346
next term; 23 = 8; 33 = 27, and so 5346 × 7 + 72 = 37471
_T

on. 73 = 343.] ∴ We can see that the wrong number is 36.


C
PS
U
@
Unit Six : Practice Session ✫ 469

Practice Exercise 40
Directions (Qs. 1-7): A number series is given in each 7. 2 3 6 15 45
of the following questions. A number is given after the 16 (a) (b) (c) (d) (e)
series and after that (a), (b), (c), (d) and (e) are given. What would come in place of (d)?
First you have to understand how this series is formed (1) 3600 (2) 120
and then starting with that number/letter you have to (3) 1080 (4) 1260
complete the second series. (5) None of these

TS
1. 5 7 10 36 136 690 Directions (Qs. 8-11): In the following number-
2 (a) (b) (c) (d) (e) series only one number is wrong. If the wrong
What will come in place of (e)? number is corrected, the series gets established
(1) 310 (2) 330

H
following a certain logic. Below the series a number
(3) 110 (4) 64 is given followed by (a), (b), (c), (d), (e) and (f).
(5) None of these

G
You have to complete the series following the same
2. 8 4 6 15 52.5 236.25 logic as in the given series after correcting the wrong
4 (a) (b) (c) (d) (e) number.
Which of the following
of (d)?
will come in place
U 8. 1 2 8 33 148 740 4626
O
2 (a) (b) (c) (d) (e) (f)
(1) 36.25 (2) 33.25 What will come in place of (d)?
(3) 26.25 (4) 32.75 (1) 156 (2) 164
H
(5) None of these (3) 168 (4) 172
3. 13 14 5 18 0.5 (5) None of these
_T

19 (a) (b) (c) (d) (e)


9. 2 4.5 11 30 93 312 1136
What would come in place of (e)?
1 (a) (b) (c) (d) (e) (f)
(1) 12.75 (2) 25.5
What will come in place of (b)?
C

(3) 18.75 (4) 15.25


(1) 6 (2) 81
(5) None of these
(3) 16.75 (4) 18.75
PS

4. 17 21.5 30.5 44 62 (5) None of these


21 (a) (b) (c) (d) (e)
10. 2 14 18 46 82 176 338
What would come in place of (e)?
4 (a) (b) (c) (d) (e) (f)
U

(1) 84.5 (2) 80.5


What will come in place of (e)?
(3) 90 (4) 95
(1) 238 (2) 338
(5) None of these
@

(3) 218 (4) 318


5. 1 8 10 35 136 (5) None of these
2 (a) (b) (c) (d) (e)
What would come in place of (c)? 11. 1 3 7 11 21 43 85
(1) 42 (2) 41 4 (a) (b) (c) (d) (e) (f)
(3) 51 (4) 55 What will come in place of (f)?
(5) None of these (1) 275 (2) 277
(3) 279 (4) 273
6. 12 26 11 36 9
(5) None of these
7 (a) (b) (c) (d) (e)
What would come is place of (c) Directions (Qs. 12-15): One number is wrong in
(1) 22 (2) 21 the number series given in each of the following
(3) 23 (4) 24 questions. You have to identify that number and
(5) None of these assuming that a new series starts with that number
470 ✫ Quantitative Aptitude

following the same logic as in the given series, which (3) 183.5 (4) 182
of the numbers given in (1), (2), (3), (4) and (5) (5) None of these
given below each series will be the third number in 14. 2 7 19 43 99 209 431
the new series? (1) 181 (2) 183
12. 3 4 12 45 190 1005 6066 (3) 87 (4) 85
(1) 98 (2) 96 (5) None of these
(3) 384 (4) 191 15. 1 2 6 33 148 765 4626
(5) None of these (1) 6 (2) 124
13. 6 10.5 23 59.5 183 644 2580 (3) 18 (4) 7

TS
(1) 183 (2) 182.5 (5) None of these

Answers with Solutions

H
1. Ans. (2) 7.5 × 3.5 = 26.25 → (d)
Sol. The given series is 26.25 × 4.5 = 118.125 → (e)

G
5 7 10 36 136 690 ∴ The required number in place of (d)
2 (a) (b) (c) (d) (e) is 26.25.
The given series can be formed in the
following way:
5×1+2= 7 U 3. Ans. (3)
Sol. The given series is
O
13 14 5 18 0.5
7 × 2 – 4 = 10 19 (a) (b) (c) (d) (e)
10 × 3 + 6 = 36
H
The series can be formed in the
36 × 4 – 8 = 136 following way:
136 × 5 + 10 = 690 13 × 1 + 1 = 14
_T

The next series needs to be formed in 14 ÷ 2 – 2 = 5


the same way: 5 × 3 + 3 = 18
2×1+2 = 4 → (a) 18 ÷ 4 – 4 = 0.5
4×2–4 = 4 → (b)
C

Similarly, the next series can be formed:


4 × 3 + 6 = 18 → (c) 19 × 1 + 1 = 20 → (a)
18 × 4 – 8 = 64 → (d) 20 ÷ 2 – 2 = 8 → (b)
PS

64 × 5 + 10 = 330 → (e) 8 × 3 + 3 = 27 → (c)


∴ From the above series we can see that 27 ÷ 4 – 4 = 2.75 → (d)
the required number in place of (e) 2.75 × 5 + 5 = 18.75 → (e)
U

is 330. ∴ The required number in place of (e)


2. Ans. (3) is 18.75.
@

Sol. In the given series is 4. Ans. (5)


8 4 6 15 52.5 236.25 Sol. The given series is
4 (a) (b) (c) (d) (e) 17 21.5 30.5 44 62
The given series can be formed in the 21 (a) (b) (c) (d) (e)
following way: The series can be formed in the
8 × 0.5 = 4 following way:
4 × 1.5 = 6 17 + 4.5 × 1 = 21.5
6 × 2.5 = 15 21.5 + 4.5 × 2 = 21.5 + 9 = 30.5
15 × 3.5 = 52.5 30.5 + 4.5 × 3 = 30.5 + 13.5 = 44
52.5 × 4.5 = 236.25 44 + 4.5 × 4 = 44 + 18 = 62
Similarly the next series can be formed: Similarly the next series can be formed:
4 × 0.5 = 2 → (a) 21 + 4.5 × 1 = 25.5 → (a)
2 × 1.5 = 3 → (b) 25.5 + 4.5 × 2 = 25.5 + 9 = 34.5 → (b)
3 × 2.5 = 7.5 → (c) 34.5 + 4.5 × 3 = 34.5 + 13.5 = 48 → (c)
Unit Six : Practice Session ✫ 471

48 + 4.5 × 4 = 48 + 18 = 66 → (d) 48 × 2.5 = 120 → (c)


66 + 4.5 × 5 = 66 + 22.5 = 88.5 → (e) 120 × 3 = 360 → (d)
∴ The required number in place of (e) ∴ The required number in place of (d)
is 88.5. is 360.
5. Ans. (2) 8. Ans. (4)
Sol. The given series is Sol. The given series is
1 8 10 35 136 1 2 8 33 148 740 4626
2 (a) (b) (c) (d) (e) 2 (a) (b) (c) (d) (e) (f)
The series can be formed in the The series can be formed in the

TS
following way: following way:
1×1+7=8 1 × 1 + (1)2 = 2
8 × 2 – 6 = 10 2 × 2 + (2)2 = 8
10 × 3 + 5 = 35 8 × 3 + (3)2 = 33

H
35 × 4 – 4 = 136 33 × 4 + (4)2 = 148
Similarly the next series can be formed: 148 × 5 + (5)2 = 765

G
2×1+7=9 → (a) 765 × 6 + (6)2 = 4626
9 × 2 – 6 = 12 → (b) ∴ The wrong number is 740.
12 × 3 + 5 = 41 →
∴ The required number in place of (c)
(c)
U Similarly the next series can also be
formed:
2 × 1 + (1)2 = 3 → (a)
O
is 41.
3 × 2 + (2)2 = 10 → (b)
6. Ans. (2) 10 × 3 + (3)2 = 39 → (c)
H
Sol. The given series is 39 × 4 + (4)2 = 172 → (d)
12 26 11 36 9 ∴ The required number (d) is 172.
7 (a) (b) (c) (d) (e)
_T

The pattern of the series is 9. Ans. (5)


12 × 2 + 2 = 26 Sol. The given series is
26 ÷ 2 – 2 = 11 2 4.5 11 30 93 312 1136
C

11 × 3 + 3 = 36 1 (a) (b) (c) (d) (e) (f)


36 ÷ 3 – 3 = 9 The series can be formed:
PS

Similarly the next series can be formed: 2 × 2 + 0.5 = 4.5


7 × 2 + 2 = 16 → (a) 4.5 × 2 + (0.5) × 4 = 11
16 ÷ 2 – 2 = 6 → (b) 11 × 2 + (0.5 × 4) × 4 = 30
6 × 3 + 3 = 21 → (c) 30 × 2 + (0.5 × 4 × 4) × 4 = 92
U

∴ The required number in place of (c) 92 × 2 + (0.5 × 4 × 4 × 4) × 4 = 312


is 21. 312 × 2 + (0.5 × 4 × 4 × 4 × 4) × 4 = 1136
@

∴ The wrong number is 93.


7. Ans. (5)
Similarly the next series can be formed
Sol. The given series is
as follows:
2 3 6 15 45
1 × 2 + 0.5 = 2.5 → (a)
16 (a) (b) (c) (d) (e)
2.5 × 2 + (0.5) × 4 = 7 → (b)
The given series can be formed in the
following way: ∴ The required number (b) is 7.
2 × 1.5 = 3 10. Ans. (1)
3×2=6 Sol. The given series is
6 × 2.5 = 15 2 14 18 46 82 176 338
15 × 3 = 45 4 (a) (b) (c) (d) (e) (f)
Similarly the next series can be formed: The given series can be formed:
16 × 1.5 = 24 → (a) 2 × 2 + 10 = 14
24 × 2 = 48 → (b) 14 × 2 – 10 = 18
472 ✫ Quantitative Aptitude

18 × 2 + 10 = 46 We can see that the wrong number is


46 × 2 – 10 = 82 190.
82 × 2 + 10 = 174 ⇒ The new series will start with 190. The
174 × 2 – 10 = 338 new series is as follows:
∴ The wrong number is 176. 190 → 1st No.
The next series can be similarly 190 × 1 + 12 = 190 + 1= 191 → 2nd No.
completed: 191 × 2 + 22 = 382 + 4= 386 → 3rd No.
4 × 2 + 10 = 18 → (a) 386 × 3 + 32 = 1158 + 9= 1167→ 4th No.
18 × 2 – 10 = 26 → (b) ∴ The 3rd number in the new series is 386.

TS
26 × 2 + 10 = 62 → (c)
13. Ans. (3)
62 × 2 – 10 = 114 → (d)
Sol. The given series is
114 × 2 + 10 = 238 → (e)

H
6 10.5 23 59.5 183 644 2580
∴ The required number (e) is 238.
The given series can be formed in the
11. Ans. (2) following way:

G
Sol. Given series 6 × 1.5 + 1.5 = 10.5
1 3 7 11 21 43 85 10.5 × 2 + 2 = 23
4 (a) (b) (c) (d) (e) (f)
Given series can be formed: U 23 × 2.5 + 2.5 = 60
60 × 3 + 3 = 183
O
1×2+1=3
183 × 3.5 + 3.5 = 644
3×2–1=5
644 × 4 + 4 = 2580
H
5 × 2 + 1 = 11
∴ We can see that the wrong number is
11 × 2 – 1 = 21
59.5.
_T

21 × 2 + 1 = 43
⇒ The new series will start with 59.5. The
43 × 2 – 1 = 85
new series is as follows:
∴ The wrong number is 7.
59.5 → 1st No.
Similarly the next series can be
C

59.5 × 1.5 + 1.5 = 90.75 → 2nd No.


completed:
4×2+1 =9 → (a) 90.75 × 2 + 2 = 183.5 → 3rd No.
PS

9 × 2 – 1 = 17 → (b) ∴ The third number in the new series is


17 × 2 + 1 = 35 → (c) 183.5.
35 × 2 – 1 = 69 → (d) 14. Ans. (2)
U

69 × 2 + 1 = 139 → (e) Sol. The given series is


139 × 2 – 1 = 277 → (f) 2 7 19 43 99 209 431
@

∴ The required number (f) is 277. The given series can be formed in the
12. Ans. (5) following way (multiply by 2 and add
Sol. The given series is consecutive odd numbers to successive
3 4 12 45 190 1005 6066 terms):
Given series can be formed in the 2×2+3=4+3= 7
following way: 7 × 2 + 5 = 14 + 5 = 19
3 × 1 + 12 = 3 + 1 = 4 19 × 2 + 7 = 38 + 7 = 45
4 × 2 + 22 = 8 + 4 = 12 45 × 2 + 9 = 90 + 9 = 99
12 × 3 + 32 = 36 + 9 = 45 99 × 2 + 11 = 198 + 11 = 209
45 × 4 + 42 = 180 + 16 = 196 209 × 2 + 13 = 418 + 13 = 431
196 × 5 + 52 = 980 + 25 = 1005 We can see that the wrong number in
1005 × 6 + 62 = 6030 + 36 = 6066 the series is 43.
Unit Six : Practice Session ✫ 473

⇒ The new series will start with 43. The 2 × 2 + 22 = 4 + 4 = 8


new series is as follows: 8 × 3 + 32 = 24 + 9 = 33
43 → 1st No. 33 × 4 + 42 = 132 + 16 = 148
43 × 2 + 3 = 86 + 3 = 89 → 2nd No. 148 × 5 + 52 = 740 + 25 = 765
89 × 2 + 5 = 178 + 5 = 183 → 3rd No. 765 × 6 + 62 = 4590 + 36 = 4626
∴ The third number in the new series is We can see that the wrong number in
183. the series is 6.
15. Ans. (3) ⇒ The new series will start with 6. The
new series is as follows:

TS
Sol. The given series is
1 2 6, 33 148 765 4626 6 → 1st No.
The given series can be formed in the 6 × 1 + 12 = 6 + 1 = 7 → 2nd No.
following way: 7 × 2 + 22 = 14 + 4 = 18 → 3rd No.

H
1 × 1 + 12 = 1 + 1 = 2 ∴ The third number in the new series is 18.

G
U
O
H
C _T
PS
U
@
474 ✫ Quantitative Aptitude

Practice Exercise 41
1. There are 5 letters and 5 directed 9. In how many ways can 5 persons be
envelopes. The number of ways in which seated at a round table, so that all shall
all the letters can be put in wrong not have the same neighbours in any two
envelopes is arrangements?
(a) 119 (b) 44 (a) 10 (b) 12
(c) 59 (d) 40 (c) 17 (d) 5
2. If the letters of the word WOMAN are 10. There are 8 students appearing in an

TS
written in all possible orders and these examination of whom 3 have to appear in
words are written out as in a dictionary, a Mathematics paper and the remaining 5
then the rank of the word WOMAN is in different subjects. In how many ways
(a) 117 (b) 120 can they be made to sit in a row if the

H
(c) 119 (d) 118 candidates in Mathematics cannot sit next
to each other?

G
3. If a polygon has 44 diagonals, then the (a) 14400 (b) 16400
number of its sides are (c) 15400 (d) 17400
(a) 11 (b) 7
(c) 8 (d) none of these
4. Out of 18 points on a plane, no three are U 11. A box contains 10 balls out of which 3 are
red and the rest are blue. In how many
ways can a random sample of 6 balls be
O
in the same straight line except five points drawn from the bag so that at the most
which are collinear. The number of straight 2 red balls are included in the sample and
H
lines that can be formed joining them is no sample has all the 6 balls of the same
(a) 143 (b) 144 colour?
_T

(c) 153 (d) none of these (a) 105 (b) 168


5. Everybody in a room shakes hands with (c) 189 (d) 120
everybody else. The total number of 12. A cricket team of 11 players is to be
handshakes is 66. The total number of formed from 20 players including 6
C

persons in the room is bowlers and 3 wicketkeepers. The number


(a) 11 (b) 12 of ways in which a team can be formed
PS

(c) 8 (d) 14 having exactly 4 bowlers and 2


6. The number of ways in which 5 prizes can wicketkeepers is
(a) 20790 (b) 6930
be distributed among 4 boys, while each
U

(c) 10790 (d) 360


boy is capable of having any number of
prizes is 13. Three boys and three girls are to be
@

(a) 64 (b) 45 seated around a table in a circle. Among


4 them the boy X does not want any girl
(c) 4! 2 (d) 6 . (4!)
neighbour and the girl Y does not want
7. The number of diagonals for n-sided
any boy neighbour. How many such
polygon is
arrangements are possible?
n (n − 1) n ( n − 1) (n − 2) (a) 5 (b) 6
(a) (b) (c) 4 (d) 2
2 6
n (n − 3) 14. A box contain 2 white balls, 3 black balls
(c) n ( n − 1) (d)
2 and 4 red balls. In how many ways can
8. How many words can be made out of the 3 balls be drawn from the box, if at least
letters of the word ASSASSINATION taken one black ball is to be included in the
all together? draw?
(a) 1396421 (b) 37289300 (a) 64 (b) 69
(c) 9112143 (d) 10810800 (c) 75 (d) 80
Unit Six : Practice Session ✫ 475

15. In a group of 6 boys and 4 girls, four 22. In how many ways can 7 boys and 6 girls
children are to be selected. In how many be seated on a round table, so that no
different ways can they be selected such two girls are sitting together?
that at least one boy should be there? (a) 6! (b) 7!
(a) 109 (b) 190 (c) 6! × 7! (d) 5! × 6!
(c) 209 (d) 290 23. In how many ways can six different rings
be worn on four fingers of one hand?
16. In how many different ways can the
(a) 47 (b) 44
letters of the word ‘SOFTWARE’ be 6
(c) 4 (d) 48

TS
arranged in such a way that the vowels
always come together? 24. In how many ways can the letters of the
(a) 4320 (b) 5430 word LEADER be arranged?
(c) 6540 (d) 7650 (a) 240 (b) 270

H
(c) 310 (d) 360
17. There are 12 buses running between Jaipur
25. In how many different ways can the
and Delhi. In how many ways can a man

G
letters of the word ALLAHABAD be
go to Delhi and return by a different bus?
permuted?
(a) 132 (b) 134
(c) 136 (d) 138
18. A department has 8 male and female U (a) 2570
(c) 7460
(b) 7560
(d) 6540
O
26. How many words can be formed from the
employees each. A project team involving letters of the word SIGNATURE so that
3 male and 3 female member needs to be the vowels always come together?
H
chosen from the department employees. (a) 17280 (b) 17000
How many different project teams can be (c) 16280 (d) 16000
_T

chosen? 27. From a group of 7 men and 6 women,


(a) 3112 (b) 3136 five persons are to be selected to form
(c) 3143 (d) 3179 a committee so that at least 3 men are
C

19. In how many ways can a student choose there on the committee. In how many
a programme of 5 courses it 9 courses are ways can it be done?
PS

available and 2 courses are compulsory for (a) 284 (b) 490
(c) 568 (d) 756
every student?
(a) 25 ways (b) 35 ways 28. In how many ways can a group of 5 men
U

(c) 55 ways (d) 65 ways and 6 women be formed from a total of


8 men and 10 women?
20. The number of permutations that can be (a) 12060 (b) 11960
@

had from the letters of the word SIMLA (c) 11860 (d) 11760
taking all the letters together is
29. In how many ways can a group of 5 men
(a) 170 (b) 123
and 2 women be formed from a total of
(c) 120 (d) 195
7 men and 3 women?
21. In an election, there are 5 candidates (a) 21 (b) 63
contesting for 3 vacancies, and a voter is (c) 45 (d) 78
entitled to vote for any number of 30. In how many different ways can the
candidates not greater than the number letters of the word MACHINE be arranged
of vacancies. In how many ways can a so that the vowels may occupy only the
voter choose to vote? odd positions?
(a) 20 (b) 25 (a) 576 (b) 565
(c) 36 (d) 42 (c) 550 (d) 542
476 ✫ Quantitative Aptitude

31. In how many different ways can the 36. In how many ways can 8 people line up
letters of the word BANKING be arranged at a ticket window of a cinema hall?
so that the vowels always come together? (a) 40320 (b) 56980
(a) 520 (b) 620 (c) 643400 (d) 791900
(c) 720 (d) 820
37. Ten students are participating in a race.
32. How many numbers can be made with In how many ways can the first three
digits 0, 7, 8 which are greater than 0 and prizes be won?
less than a million? (a) 920 (b) 820
(a) 496 (b) 486 (c) 720 (d) 620

TS
(c) 1084 (d) 728
38. How many different signals can be
33. The letters of the word ARTICLE are
transmitted by arranging 3 red, 2 yellow
arranged so that no two vowels come
and 2 green flags on a pole?

H
together. Find the total number of
arrangements. (a) 240 (b) 230
(a) 49 (b) 1440 (c) 220 (d) 210

G
(c) 7 (d) 1898 39. How many parallelograms will be formed
34. There are 10 candidates for 4 posts. In if 9 parallel horizontal lines intersect 8
how many ways can the posts be filled?
(a) 3020 (b) 2030 U vertical lines?
(a) 1120 (b) 1008
O
(c) 5040 (d) 4050 (c) 1310 (d) 1022
35. How many three-digit odd numbers can 40. A polygon has 12 sides. How many
H
be formed from the digits 1, 2, 3, 4, 5 triangles can be formed by joining the
when repetition of digits is not allowed? vertices of the polygon?
_T

(a) 36 (b) 40 (a) 12 (b) 144


(c) 42 (d) 48 (c) 220 (d) 48
C

Answers with Solutions


PS

1. Ans. (b) Words starting with N are |4


Sol. Number of ways in which all the five Words starting with O are |4
letters are posted in wrong envelopes Words starting with WA are |3
1 1 1 1 1 Words starting with WM are |3
U

= |5 1 − + − + − Words starting with WN are |3


|1 |2 |3 |4 |5
Words starting with WOA are |2
@

= 5×4×3×2 Next word will be WOMAN


1 1 1 1 ∴ Position of WOMAN
1−1+ − + − = |4 +|4 +|4 +|4 +|3 +|3 +|3 +|2 +1
2 3×2 4× 3×2 5×4×3× 2
= 4×3×2+4×3×2+4×3×2
1 1 1 1
= 120 − + − +4×3×2 +3×2+3×2+3×2
2 6 24 120 +2+1
60 − 20 + 5 − 1 = 24 + 24 + 24 + 24 + 6 + 6 + 6 + 2 + 1
= 120
120 = 117.
= 60 – 20 + 5 – 1 3. Ans. (a)
= 44. Sol. If n is the number of sides, then the
2. Ans. (a) number of diagonals = nC2 – n = 44
Sol. Words starting with A are |4 1
⇒ n (n – 1) – n = 44
Words starting with M are |4 2
Unit Six : Practice Session ✫ 477

⇒ (n – 11) (n + 8) = 0
13 × 12 × 11 × 10 × 9 × 8 × 7 × 6 × 5 × 4!
⇒ n = 11. =
3× 2 × 4 ! ×2 × 2
4. Ans. (b)
Sol. The number of straight lines = 13 × 12 × 11 × 10 × 9 × 2 × 7 × 5
= 18C2 – (5C2 – 1) = 10810800.
18! 5! 9. Ans. (b)
= (18 − 2)! 2! − (5 − 2)! 2! − 1 Sol. Five persons can be seated at a round
table in
18 × 17 × 16! 5 × 4 × 3! (5–1)! = 4! = 4 × 3 × 2 = 24 ways
− −1

TS
= 16! × 2 3! × 2 Required number of arrangements
= 9 × 17 – (10 – 1) 1
= 153 – 9 = × 24 = 12.
2
= 144.

H
10. Ans. (a)
5. Ans. (b) Sol. Total number of candidates = 8
Sol. If n is number of persons in a room, 5 different subjects candidates can be

G
total number of handshakes = nC2 = 66 seated in P (5, 5) = 5! Ways
1 In between 5 candidates there are 6

2
n (n – 1) = 66
⇒ n2 – n = 132 U places for 8 mathematics candidates
∴ The Mathematics candidates can be
O
seated in P (6, 3) ways
⇒ n2 – n – 132 = 0
∴ By fundamental principle of counting:
⇒ n2 – 12n + 11n – 132 = 0
The required number of ways
H
⇒ n (n – 12) + 11 (n – 12) = 0
= 5 ! × P (6, 3)
⇒ (n + 11) (n – 12) = 0 6!
= 5 × 4 × 3 × 2 ×
⇒ n = 12.
_T

3!
6. Ans. (b) = 120 × 6 × 5 × 4
Sol. Every prize can be given by any of = 14400.
the four boys, so number of ways 11. Ans. (b)
C

= 4 × 4 × 4 × 4 × 4 = 45 . Sol. The possible ways are as follows:


7. Ans. (d) (i) 1 red ball out of three and 5 blue
PS

Sol. Total number of lines obtained by balls out of seven.


joining n vertices of polygon is nC2. Out (ii) 2 red balls out of three and 4 blue
of these, n lines are sides and remaining balls out of seven.
∴ Total number of ways in which a
are diagonals.
U

∴ Number of diagonals random sample of six balls can be


drawn
n! n × (n − 1)
@

= nC2 – n = −n = −n = 3C1 × 7C5 + 3C2 × 7C4


2! (n − 2)! 2
n (n − 1) − 2 n n2 − n − 2n 3! 7!
= = = (3 − 1)! 1! × (7 − 5)! 5!
2 2
3! 7!
n 2 − 3n n (n − 3) + ×
= = . (3 − 2)! 2! (7 − 4)! 4!
2 2
8. Ans. (d) 3 × 2! 7 × 6 × 5!
Sol. There are 13 letters in the given word. = ×
2! 2!× 5!
3 are A’s, 4 are S’s, 2 are N’s, 2 are 3 × 2! 7 × 6 × 5 × 4!
I’s and rest 2 are different + ×
∴ Required number of words 1! × 2! 3!× 4!
13 ! = 3 × 21 + 3 × 35
= 3! 4! 2! 2!
= 63 + 105 = 168.
478 ✫ Quantitative Aptitude

12. Ans. (a) 6×5 4×3


Sol. There are 6 bowlers, 3 wicketkeepers = (6 × 4) + ×
2 ×1 2×1
and 11 batsmen in all. 6×5×4 6×5
The number of ways in which a team + ×4 +
3× 2×1 2×1
of 4 bowlers, 2 wicketkeepers and 5
batsmen can be chosen = 6 × 4 + 15 × 6 + 20 × 4 + 15
= 6C4 × 3C2 × 11C5 = 24 + 90 + 80 + 15 = 209.
16. Ans. (a)
6! 3! 11! Sol. The word SOFTWARE has 3 vowels
= × ×
(6 − 4)! 4! (3 − 2)! 2! (11 − 5)! 5! and 5 consonants. We can take 3

TS
6 × 5 × 4! 3 × 2! vowels as one letter.
= 2! × 4! × 1! × 2! ∴ Word can be written in 6! ways = 720
and 3 vowels can be arranged in 3!
11 × 10 × 9 × 8 × 7 × 6! ways = 6

H
× ∴ Required number of words
6! × 5!
= 720 × 6 = 4320.
= 15 × 3 × 462

G
17. Ans. (a)
= 20790. Sol. Required number of ways
= (ways of going to Delhi)
13. Ans. (c)
Sol. 1, 2, X are boys; 3, 4, Y are girls
1, 2 can be interchanged in U × (ways of returning to Jaipur)
= 12 × 11 = 132.
O
2
18. Ans. (b)
P2 = 2 ways Sol. Total employees = 8
3 and 4 can also be 3 male employees out of 8 = 8C3
H
interchanged in 2 ways 3 female employees out of 8 = 8C3
∴ Required number of ways = 2 × 2 = 4. So, the number of project teams
_T

14. Ans. (a) 8! 8!


Sol. We may have 1 black and 2 non-black = 8C3 × 8C3 = ×
(8 − 3)! 3! (8 − 3)! 3!
balls or 2 black and 1 non-black balls
8! 8!
or 3 black balls. = ×
C

5! 3! 5! 3!
∴ Required number of ways
8 × 7 × 6 × 5! 8 × 7 × 6 × 5!
PS

= (3C1 × 6C2) + (3C2 × 6C1) + (3C3) = ×


3! 6! 5! × 3 × 2 × 1 5! × 3 × 2 × 1
= × = 56 × 56
(3 − 1)! 1! (6 − 2)! 2! = 3136.
U

3! 6! 19. Ans. (a)


+ × +1
(3 − 2)! 2! (6 − 1)! 1! Sol. Out of available nine courses, two are
@

6×5 3×2 compulsory. Hence, the student is free


= 3× + × 6 +1 to select 3 courses out of 7 remaining
2×1 2×1
courses. If P is the number of ways
= 45 + 18 + 1 of selecting 3 courses out of 7 courses,
= 64. then
15. Ans. (c) 7!
P = C (7, 3) =
Sol. We may have 1 boy and 3 girls or 2 3! (7 − 3)!
boys and 2 girls or 3 boys and 1 girl 7! 7 × 6 × 5 × 4!
or 4 boys = =
3! 4! 3 × 2 × 1 × 4!
6
C1 × 4C3 + 6C2 × 4C2 + 6C3 × 4C1 + 6C4 = 7 × 5 = 35 ways
6! 4! 6! 4! 20. Ans. (c)
= × + × Sol. There are five letters in the word SIMLA.
5! 1! 1! 3! 4! 2! 2! 2! The number of permutation = 5 !
6! 4! 6! 5 != 5 × 4 × 3 × 2 × 1
+ × +
3! 3! 3! 1! 2! 4! = 120.
Unit Six : Practice Session ✫ 479

21. Ans. (b) letters; 4 vowels and 5 consonants. We


Sol. A voter can vote for 1 candidate, 2 can take 4 vowels as one letter.
candidates or 3 candidates. ∴ We have 6 letters in total
∴ Required number of ways = 6P6
= 6! = 720
= 5C1 + 5C2 + 5C3 4 vowels can be arranged in 4! ways
5! 5! 5! = 24
= + + ∴ Required number of words
(5 − 1)! 1! (5 − 2)! 2! (5 − 3)! 3!
5 × 4! 5 × 4 × 3! 5 × 4 × 3! = 720 × 24
= + + = 17280.
4! × 1! 3! × 2! 2! × 3!

TS
27. Ans. (d)
= 5 + 10 + 10
Sol. We may have 3 men and 2 women or
= 25.
4 men and 1 woman or 5 men
22. Ans. (c) 7
C3 × 6C2 + 7C4 × 6C1 + 7C5

H
Sol. 7 boys can be seated on a round table 7! 6! 7! 6! 7!
in (7 – 1)!, i.e., 6! ways = 4! 3! × 4! 2! + 3! 4! × 5! 1! + 2! 5!

G
6 girls in 7 places can be seated in
= 35 × 15 + 35 × 6 + 21
7
7! = 525 + 210 + 21
P6 ways = = 7!
(7 − 6)!
∴ Required number of ways = 6! × 7!.
23. Ans. (c) U 28.
= 756.
Ans. (d)
O
Sol. (5 men out of 8) and (6 women out
Sol. Each ring can be worn on any of the of 10) are to be chosen.
four fingers and this can be done in
Required number of ways
H
4 ways
Here n = 4 and r = 6 = 8C5 × 10C6
∴ The required number of ways = nr 8! 10!
= 3! 5! × 4! 6!
_T

= 46
24. Ans. (d) = 56 × 210
Sol. The word LEADER contains 1 – L, = 11760.
C

2 – E, 1 – A, 1 – D, 1 – R and a total of 29. Ans. (b)


6 letters Sol. 5 men out of 7 = 7C5 and
∴ Total number of ways
PS

2 women out of 3 = 3C2


6!
= Then,
(1!) (2!) (1!) (1!) (1!) Required number of ways
= 6×5×4×3 = 7C5 × 3C2
U

= 360. 7! 3!
25. Ans. (b) = 2! 5! × 11! 2!
@

Sol. The word ALLAHABAD has 9 letters, in


= 21 × 3
all. The letter A occurs 4 times, the letter
= 63.
L occurs 2 times and the remaining three
30. Ans. (a)
letters H, B, D each occur once.
Sol. The word MACHINE has 7 letters; 3
∴ The require number of permutations vowels and 4 consonants
9! Now, 3 vowels can be placed at any
=
4! 2! 1! 1! 1! of the three places, out of the four
9 × 8 × 7 × 6 × 5 × 4! marked.
= ∴ Number of ways of arranging the
4! × 2! vowels = 4P3 = 4 × 3 × 2 = 24
= 9 × 8 × 7 × 3 × 5 Also, the 4 consonants at the remaining
= 7560. 4 positions may be arranged in
26. Ans. (a) = 4P4 = 4! = 24 ways
Sol. The word SIGNATURE contains 9 ∴ Required number of ways = 24 × 24
= 576.
480 ✫ Quantitative Aptitude

31. Ans. (c) and hundred’s place can be filled in


Sol. The word BANKING has 7 letters; 2 3 ways
vowels and 5 consonants. We can take Hence, required number of ways
2 vowels as one letter = 3 × 4 × 3 = 36.
∴ Number of ways of writing the word is 36. Ans. (a)
6! = 6 × 5 × 4 × 3 × 2 = 720 Sol. Required number of ways = 8!
720 =8×7×6×5×4×3×2×1
Letter N comes twice = 360 = 40320.
2
2-vowels can be arranged in 2! ways 37. Ans. (c)

TS
= 2 Sol. Out of 10 students, three prizes can be
∴ Required number of ways won in 10C3 ways
= 360 × 2 = 720. 10!
=
(10 − 3)!

H
32. Ans. (d) 10! 10 × 9 × 8 × 7!
Sol. Number of ways for single digit = 2 = =
7! 7!

G
2 digits = 2 × 3 = 6
3 digits = 2 × 3 × 3 = 18 = 10 × 9 × 8 = 720.
4 digits = 2 × 3 × 3 × 3 = 54 38. Ans. (d)
5 digits = 2 × 3 × 3 × 3 × 3 = 162
6 digits = 2 × 3 × 3 × 3 × 3 × 3 = 486
U
Total = 2 + 6 + 18 + 54 + 162 + 486 = 728.
Sol. Here, n = 3 + 2 + 2 = 7
P1 = 3, P2 = 2 and P3 = 2
O
∴ Required number of different signals
33. Ans. (b)
Sol. The word ARTICLE has 7 letters; 3 vowels n! 7!
H
= =
and 4 consonants P1 ! P2 ! P3 ! 3! 2! 2!
4 consonants can be written in 4! ways
7 × 6 × 5 × 4 × 3!
_T

= 4 × 3 × 2 = 24
= = 210.
3 vowels can be written in 3! ways 3! × 2 × 1 × 2 × 1
=3×2=6
No two vowels should come together, 39. Ans. (b)
Sol. mC2 × nC2
C

i.e., 5C3 ways


Here m = 9 and n = 8
5! 5×4×3! 5×4
= = = = 10 9! 8!
PS

3! (5 − 3)! 3! × 2! 2×1 9
C2 × 8C2 = 7! 2! × 6! 2!
Total number of arrangements required
9×8 8×7
= 24 × 6 × 10 = 1440. = ×
34. Ans. (c) 2×1 2×1
U

Sol. Required number of ways = 36 × 28


= (ways of filling 1st post) = 1008.
@

× (ways of filling 2nd post) 40. Ans. (c)


× (ways of filling 3rd post) Sol. A triangle will be formed by joining
any 3 vertices of the polygon.
× (ways of filling 4th post) The given polygon has 12 sides; so it
= 10 × 9 × 8 × 7 has 12 vertices.
= 5040. Number of triangles formed = 12C3
35. Ans. (a) 12 !
Sol. We need three-digit odd numbers, =
(12 − 3) ! 3 !
thus the digit at unit’s place must be
1, 3 or 5, that is, unit’s place can be 12 × 11 × 10 × 9 !
=
filled in 3 ways. 9 ! × 3× 2×1
Ten’s place can be filled up by any of 12 × 11 × 10
the remaining digits, i.e., in 4 ways = = 220.
6
Unit Six : Practice Session ✫ 481

Practice Exercise 42
1. Probability of getting three tails in three 8. A committee of 5 persons is to be
random tosses of a balanced coin is constituted from a group of 6 men and
1 1 8 women. If the selection is made randomly,
(a) (b) the probability that there are 3 women
8 3
1 and 2 men in the committee is
(c) (d) 3 60 7
6
(a) (b)
2. Probability of an odd number appearing 143 10

TS
in a single toss of a fair dice is 97 75
1 1 (c) (d)
(a) (b) 130 193
3 2 9. Three balls are drawn successively from
1

H
(c) (d) 2 a box containing 6 red, 4 black and 5 blue
6 balls. Find the probability that they are
3. Probability of getting sum 7 in a single drawn in order of red, blue and black if

G
toss of a pair of fair dice is each ball is replaced.
1 1
(a) (b) 6 7

(c)
7
1
(d)
2
1
U (a)
215
8
(b)
220
9
O
3 6 (c) (d)
4. From 20 tickets of lottery numbered 1 to 225 230
20, two tickets are drawn simultaneously. 10. If we toss a coin 1000 times and find that
H
Probability that both the tickets drawn it comes up tails 703 times, then the
have prime numbers is probability of heads is
_T

20 8 (a) 0.703 (b) 0.382


(a) (b) (c) 0.297 (d) 4.83
8 20
14 1 11. If a man has 24 shirts and 9 ties, then in
(c) (d)
C

95 190 how many ways can he choose a shirt and


5. From the word COMMISSION a single a tie?
(a) 216 (b) 2.66
PS

letter is selected at random. Probability


that it is a vowel is (c) 33 (d) 15
(a) 0.2 (b) 0.3 12. The number of ways in which 4 cards can
(c) 0.4 (d) 0.5
be chosen from 9 cards is
U

6. Two friends, A and B are playing a game. (a) 190 (b) 112
A throws 2 dices first and gets 9.
(c) 110 (d) 126
Probability that B gets a higher score is
@

1 1 13. If the probability of rain on any given day


(a) (b) in Pune city is 50%, then what is the
2 4
1 1 probability that it rains on exactly 3 days
(c) (d) in a 5-day period?
6 8
5 4
7. From a group of 10 persons containing 4 (a) (b)
men, 4 women and 2 children, 4 persons 16 25
are selected at random. Probability of 8 2
selecting 1 man, 1 woman and 2 children (c) (d)
125 25
is 14. If we randomly pick two television tubes
16 18 in succession from a shipment of 240
(a) (b)
105 95 television tubes of which 15 are defective,
17 2 what is the probability that they both will
(c) (d)
80 5 be defective?
482 ✫ Quantitative Aptitude

7 7 21. A bag contains 9 white and 3 black balls.


(a) (b) Another bag contains 6 white and 10 black
1932 1793
7 7 balls. If a ball is transferred from the first
(c) (d) bag to the second bag and a ball is drawn
1912 1780
15. In how many ways can 10 people be from the second, find the probability that
seated on a bench if only 4 seats are the ball drawn is white.
available? 19 41
(a) 5040 (b) 918 (a) (b)
25 60
(c) 1075 (d) 7850
3 27

TS
16. An urn contains 5 red and 7 yellow balls. (c) (d)
28 68
Two balls are drawn at random from the
urn. The number of elements in the space 22. 5 persons enter the lift cabin on the
when one ball is red and other is yellow ground floor of an 8-floor building. Each

H
is of them independently and with equal
(a) 12 (b) 24 probability, can leave the cabin at any

G
(c) 35 (d) 2
floor beginning with the first. Probability
17. Four dice are thrown together. The of all five persons leaving at different
probability of getting a sum less than 8
is
13 13 U floors is
5
P7 7
P5
O
(a) 7 (b)
(a) (b) 5 75
1296 216
57 75
8 8
H
(c) (d) (c) 5 P (d) 7
P5
216 435 7

18. 5 boys and 5 girls sit in a row at random. 23. A card is taken out of a pack of 52 cards
_T

The probability that all the girls sit together numbered 2 to 53. The probability that the
is number on the card is a prime number
1 1 less than 20 is
C

(a) (b) 2 2
32 42 (a) (b)
1 1 13 15
PS

(c) (d) 3 4
52 62 (c) (d)
13 15
19. Letters of the word MATHEMATICS are
placed at random in a row. Probability of 24. There are 9 oranges and 12 guavas in a
U

getting both the M together is box. If the two fruits are chosen at
2 11 random, the probability that one is an
(a) (b) orange and other is a guava is
@

11 2 (a) 0.62 (b) 0.91


10 7 (c) 0.37 (d) 0.51
(c) (d)
7 10
25. A positive integer is chosen at random.
20. For two vacancies for the same post, a The probability that sum of the digits of
husband and a wife appear in an interview. its square is 33 is
4
If probability of husband’s selection is 1 2
2 5
(a) (b)
and wife’s selection is , then the 33 33
5
probability that both are selected is 1
(c) (d) None of these
8 7 11
(a) (b) 26. A basket contains 10 oranges out of which
25 12 4 are rotten. If two oranges are taken out
6 10 together in which one is good then the
(c) (d)
5 6 probability that the other is also good is
Unit Six : Practice Session ✫ 483

17 5 independent trials of the experiment are


(a) (b) performed. The probability that the event
30 18
5 4 A happens at least once is
(c) (d) (a) 0.936 (b) 0.784
9 10
(c) 0.904 (d) 0.888
27. Out of 5 mathematicians and 7 physicists,
a committee consisting of 2 mathemati- 33. Suppose six coins are flipped. Then the
cians and 3 physicists is to be less formed. probability of getting at least one tail is
In how many ways can it be done so that 71 53
one particular physicist must be in the (a) (b)
72 54

TS
committee?
(a) 350 (b) 150 63 1
(c) (d)
(c) 105 (d) 495 64 12
34. A bag, A contains 3 white balls and 2 black

H
28. A man has two purses. One purse contains
5 copper coins and 3 silver coins whereas balls. Another bag, B contains 2 white
the second purse contains 8 copper coins balls and 4 black balls. A bag and a ball

G
and 10 silver coins. The probability of are picked at random. The probability that
taking out a copper coin from any of the the ball will be white is
purses is

(a)
73
(b)
77
U (a)
7
11
5
(b)
7
30
7
O
65 144 (c) (d)
10 18 11 15
(c) (d) 35. A bag contains 6 red and 9 black balls.
H
51 93
29. A non-leap year is selected randomly. Two draws of 4 balls are made. Find the
Probability that the year has 53 Sundays probability that the first draw will give
_T

is 4 red and the second draw 4 black balls,


1 1 if the balls are returned to the bag after
(a) (b) the first draw.
2 5
C

1 1 (a) 0.1 (b) 0.01


(c) (d) (c) 0.001 (d) 0.0001
4 7
PS

30. A secretary wrote 5 letters to 5 different 36. Amar can hit a target with a pistol 3 times
clients. Different addresses on 5 envelopes in 5 shots; Akbar can hit 2 times in 5 shots;
were also written. Without looking at the and Anthony can hit 3 times in 4 shots.
If Amar, Akbar and Anthony fire a volley
U

addresses what is the probability that the


letters go into the right envelopes? of shots, what is the probability that two
shots are hit?
1 1
@

(a) (b) (a) 0.15 (b) 0.32


25 80 (c) 0.39 (d) 0.45
1 1
(c) (d) 37. From a bag containing 5 red, 4 white and
100 120
3 blue marbles, three marbles are drawn,
31. A single letter is selected from the word with replacement. What is the probability
DEVELOPMENT. The probability that it that one is white and two are red?
is a vowel is 2 2
7 3 (a) (b)
(a) (b) 19 17
11 5 2 2
4 1 (c) (d)
(c) (d) 13 11
11 8 38. Hema throws 2 dices simultaneously. Find
32. The probability that an event happens in the probability that the sum of the dots
one trial of an experiment is 0.4. Three on the upper side will be 9.
484 ✫ Quantitative Aptitude

1 1 41. A dice is thrown twice. The probability


(a) (b) that at least one of the two throws comes
9 8
1 1 up with the number 3 is
(c) (d)
7 6 5 11
39. Find the probability of drawing out a king (a) (b)
12 36
or a diamond card, in a single trial, from
a pack of well-shuffled cards. 1 7
(c) (d)
1 2 12 24
(a) (b)
13 13

TS
3 4 42. A bag contains 3 black and 4 red balls.
(c) (d) Two balls are drawn at random one at
13 13
a time without replacement. The
40. The probability of solving a problem by probability that the first ball drawn is

H
three students X, Y and Z is 12 , 13 and black if the second ball is known to be
1
respectively. The probability that the red is

G
4
problem will be solved is 1 1
1 1 (a) (b)
2 4
(a)

(c)
4
3
(b)

(d)
2
1 U (c)
1
(d)
1
O
6 8
4 3
H
Answers with Solutions
1. Ans. (a) Since two tickets are drawn, the total
_T

20
Sol. In each turn, the probability of getting number of cases = C 2
1 20! 20!
tails is . So multiplying the = =
2 2! (20 − 2) ! 2! 18!
C

probabilities of all the three turns is 20 × 19 × 18 !


1 1 1 = = 190
= × × = 1. 2 × 1 × 18 !
PS

2 2 2 2 Number of favourable cases = Number


2. Ans. (b) of combinations of 8 prime numbers
Sol. Odd numbers in a dice are 1, 3, 5 so taken 2 at a time
U

3 1 8!
P = = . = 8 C2 =
6 2 2! (8 − 2) !
@

3. Ans. (d) 8! 8×7×6 !


Sol. There are 6 ways in which we can = = = 28
2! 6! 2 × 1× 6 !
have sum 7 on the pair of dice 28 14
i.e., (1, 6) (2, 5) (3, 4), (4, 3), (5, 2), ∴ P(both are prime) = 190 = 95 .
(6, 1)
6 1 5. Ans. (c)
then P = = Sol. Total number of words = 10
36 6
(One face of a dice is paired with 6 Number of vowels (O, I, I, O) = 4
faces of another dice so the total 4
Required Probability = = 0.4.
number of cases are 6 × 6 = 36). 10
4. Ans. (c) 6. Ans. (c)
Sol. From 1 to 20, there are eight prime Sol. If A gets score 9 then B will have to
numbers, viz., 2, 3, 5, 7, 11, 13, 17, 19. score 10, 11 or 12 for a higher score
Unit Six : Practice Session ✫ 485

Exhaustive number of cases for 9. Ans. (c)


throwing two dices is 6 × 6 = 36 Sol. Total number of balls in a bag is
Number of events for scoring higher 6 + 4 + 5 = 15
than 9 is Probability of drawing balls in order of
= {(4,6), (6,4), (5,5), (6,5), (5,6), (6,6)} red, blue and black is
i.e., 6 6 5 4
n(E) = 6 = × ×
n(E) 6 1 15 15 15
P(E) = = = .
n(S) 36 6 = 2 × 4 = 8 .
7. Ans. (a) 15 15 225

TS
Sol. Exhaustive number of cases
10. Ans. (c)
= 10C4
Sol. Number of tosses = 1000
10 ! 10 !
= (10 − 4) ! 4 ! = 6! 4 ! Number of tails = 703

H
Number of heads = 1000 – 703 = 297
10 × 9 × 8 × 7 × 6 ! 297
= Probability of heads = = 0.297.
6 ! × 4× 3×2 ×1

G
1000
= 10 × 3 × 7 11. Ans. (a)
= 210 Sol. Total shirts = 24
1 man, 1 woman and 2 children can
be selected in U Total ties = 9
Number of ways of choosing a shirt
O
4
C 1 × 4 C 1 ×2 C 1 and a tie is 24 × 9 = 216.
= 4 × 4 × 2 = 32 ways 12. Ans. (d) n!
H
Sol. nCr =
32 16 r ! (n − r )!
Required probability = = .
210 105 Here n = 9 and r = 4
_T

8. Ans. (a) 9! 9 × 8 × 7 × 6 × 5!
Sol. Total number of people = 6 + 8 = 14 = =
4! 5! 4 ! × 5!
There are 14 people and 5 should be 9×8×7 ×6
selected = = 126.
C

4× 3× 2×1
Number of ways of selection is 14 C 5 13. Ans. (a)
14 ! 14 !

PS

14
C5 = =
(14 − 5) ! 5 ! 9! 5 ! Sol. Probability that it rains on the 1st day
14 × 13 × 12 × 11 × 10 × 9 !
= = 1
9 ! × 5 × 4 × 3 × 2 ×1 2
U

14 × 13 × 12 × 11 × 10 Probability that it rains on the 2nd day


=
5 × 4 × 3 × 2 ×1 = 1
@

= 2002 2
Number of ways of selecting 2 men Probability that it rains on the 3rd day
and 3 women = 1
6! 8! 2
 6
C2 × 8 C3 = ×
4! 2! 5! 3! Probability that it rains on the 4th day
6 × 5× 4 ! 8×7× 6× 5 ! =
1
= ×
4 ! ×2 5 ! × 3× 2×1 2
6×5 8×7 ×6 Probability that it rains on the 5th day
= × = 840 1
2×1 3× 2×1 =
n (E) 840 2
Required probability = = Probabilitythat it rains on the any day
n (S) 2002
60 1
= . in a 5-day period =
143 32
486 ✫ Quantitative Aptitude

Probability that it rains on exactly 3 18. Ans. (b)


days in a 5-days period Sol. Total number of arrangements = (5+5)!
1 5! 1 = 10 !
= 5C 3 × = × Considering 6 girls as a group of one
32 (5 − 3) ! 3 ! 32
∴ Number of favourable cases = (5+1)! 5!
5 × 4 × 3! 1 10 5 = 6! 5!
= × = = . 6! 5!
2 × 3! 32 32 16
14. Ans. (c) ∴ P(all girls together) = 10!
Sol. Probability of the first tube being 6! 5!
=

TS
defective is 10 × 9 × 8 × 7 × 6!
15 5× 4×3×2×1
= =
240 10 × 9 × 8 × 7
Probability that the second tube will

H
1
be defective is = .
14 42
19. Ans. (a)

G
=
239 Sol. The word MATHEMATICS contains
Thus, the probability that both the 2 – M, 2 – A, 2 – T, 1 – A, 1 – E, 1 – T,
tubes might be defective
=
15
×
14
=
7
. U 1 –C and 1–S.
∴ Total number of arrangements
11!
O
240 239 1912 = 2! 2! 2! 1! 1!1!1!1! = 4989600
15. Ans. (a)
Sol. The first seat can be filled in 10 ways, Now consider 2 – M as one block
H
second seat in 9 ways, the third in 8 ∴ Number of favourable cases
ways, and the fourth seat in 7 ways. 10!
= 1! 2! 2! 1!1!1!1!1! = 907200
_T

Number of arrangements of 10 people


taken 4 at a time = 10 × 9 × 8 × 7 907200 2
∴ Required probability = = .
= 5040. 4989600 11
C

16. Ans. (c) 20. Ans. (a) 4


Sol. Let S be the event of getting one red Sol. P (Husband) =
5
PS

and one yellow ball. 2


P (Wife) =
∴ Number of elements in S is 5
= 5 C1 × 7C1 P (both are selected) = P(H) P(W)
4 2 8
× =
U

= 5 × 7 = 35. = .
17. Ans. (a) 5 5 25
21. Ans. (d)
Sol. Here S = {(1, 1, 1,1), (1, 1, 1, 2)...
@

...(6, 6, 6, 6)} 9 white 6 white


Sol. 3 black 10 black
∴ Number of possible outcomes
= 6 × 6 × 6 × 6 Let W1 and B1 are the events of
= 1296 transferring a white ball and a black
Let A be the event of getting a sum less ball from bag I to bag II respectively.
than 8 Let W2 be the event of drawing a
A= {(1,1,1,1), (1,1,1,2), (1,1,1,3), (1,1,1,4) white ball from bag II after the
(1,1,2,1), (1,1,3,1) (1,1,4,1), (1,2,1,1) transfer of one ball.
(1,3,1,1), (1,4,1,1) (2,1,1,1), (3,1,1,1) ∴ Required probability = P(W2)
(4,1,1,1)} P(W2) = P(W1W2) or P(B1W2)
P  (sum > 8) = P(W1W2) + P(B1W2)
n( A) 13 W2 W2
P(A) =
n(S)
=
1296
. = P(W1 )P W + P(B1 )P B
1 1
Unit Six : Practice Session ✫ 487

9 6+1 Number of ways that at least one


= ×
9 + 3 (6 + 1) + 10 orange selected is good = 6 × 9 = 54
3 6 15 5
+ + Required probability = = .
9 + 3 6 + (10 + 1) 54 18
9 7 3 6 27 27. Ans. (a)
= × + × = .
12 17 12 17 68 Sol. 2 mathematicians out of 5 can be selected
22. Ans. (b) 5! 5×4×3!
Sol. Number of floors where persons can = 5 C2 = = = 10
leave the cabin = 8 – 1 = 7 (5 − 2) ! 2 ! 3 ! × 2 × 1

TS
Total number of ways for 5 persons 3 physicists out of 7 can be selected
to leave the cabin = 7 × 7 × 7 × 7 × 7 7! 7 × 6 × 5 × 4!
= 75
7
C3 = =
Total number of ways for 5 persons (7 − 3)! 3! 4! × 3 × 2 × 1
= 35

H
to leave the cabin at different floors
7
= 7 × 6 × 5 × 4 × 3 = P5 Total number of possible ways

G
7
P = 5C2 × 7C3 = 10 × 35 = 350.
∴ Required probability = 55 . 28. Ans. (b)
7
23. Ans. (a) 1 5 1 8
Sol. Prime numbers less than 20 are
2, 3, 5, 7, 11, 13, 17, 19 U Sol. Required probability = × + ×
2 8 2 18
5 2
+ =
77
O
 n (A) = 8, n (S) = 52 =
16 9 144
.
n ( A) 8 2 29. Ans. (d)
∴ P (A) = = = .
H
n (S) 52 13 Sol. A non-leap year contains 365 days
24. Ans. (d) which has 52 weeks and one day
Sol. Let A be the event of choosing an
_T

more. This one day may be one of the


orange and a guava seven days of a week in which there
21 ! is only one Sunday.
Then n(S) = 21C2 = Thus n(S) = 7
(21 − 2) ! 2 !
C

Number of favourable cases = 1


21 × 20 × 19 ! Required probability
= = 21 × 10 = 210
PS

19 ! × 2 × 1 No. of favourable cases


n(A) = 12 × 9 = 108 =
total no. of cases
n ( A) 108 1
∴ P(A) = = = 0.51. = .
U

n (S) 210 7
25. Ans. (d) 30. Ans. (d)
Sol. Let x be an integer, whose sum of the Sol. Total number of ways of putting the
@

digits square is 33. letters in the envelope is


∴ 3 divides x2 since 3 is prime. 5! = 5 × 4 × 3 × 2 × 1 = 120 ways
∴ 3 divides x The favourable case = 1
∴ 9 divides x2 The required probability =
1
.
Any number is divisible by 9 if its sum 120
is divisible by 9. But 33 is not divisible 31. Ans. (c)
by 9. Hence, probability cannot be Sol. Sample space S
found. = {D,E,V,E,L,O,P,M,E,N,T}
26. Ans. (b) ∴ n(S) = 11
Sol. Number of ways in which two good
Event A = {E,E,O,E}
oranges can be taken out is 6 C 2
∴ n(E) = 4
6! 6×5×4 ! n(E)
= = = 15 4
(6 − 2) ! 2 ! 4 ! × 2 × 1 The required probability is n(S) =
11
.
488 ✫ Quantitative Aptitude

32. Ans. (b) 6! 9!


Sol. Probability of happening (p) = 0.4
2! 4! 5! 4!
∴ Probability of not happening (q) = 15! × 15!
= 1 – 0.4 = 0.6
Probability (event does not happen) 11! 4! 11! 4!
= q × q × q = (0.6)3 = 0.216 6! × 11! × 4! 9! × 11! × 4!
Probability (event A happens at least = 2! × 4! × 15! × 5! × 4! × 15!
once) = 1 – 0.216 = 0.784.
6
=
33. Ans. (c) 7 × 13 × 5 × 13

TS
Sol. Total number of outcomes that would 6
= = 0.001.
occur by flipping six coins = 26 = 64 5915
1 36. Ans. (d)
Probability that no tail occurs = Sol. The probability of Amar hitting a

H
64
∴ Probability of at least one tail occurring 3
target =
1 63 5

G
= 1 – = . The probability of Akbar hitting a
64 64 2
34. Ans. (d) 1 target =
Sol. Probability that bag A is drawn =
Probability that white ball is drawn
2
U 5
The probability of Anthony hitting a
3
O
from bag 1 3 3 target =
A = × = 4
2 5 10 1 Two shots can be hit in the following
H
Probability that bag B is drawn =
2 ways:
Probability that white ball is drawn Let E1 be the event where Amar and
_T

1 2 1 Akbar hit and Anthony fails


from bag B = × =
2 6 6 E2 be the event where Amar and
Probability that white ball is drawn Anthony hit and Akbar fails, and
E3 be the event where Akbar and
C

either from bag A or from bag B


Anthony hit and Amar fails
3 1 7 3 2 3 3 2 1
+ =
PS

= . × × 1− = × ×
10 6 15 ∴ P(E1) =
5 5 4 5 5 4
35. Ans. (c) 6
Sol. Total number of balls = 15 =
100
U

The probability of drawing 4 red balls 3 3 2 3 3 3


in the first draw is given as P(E1) P(E2) = × × 1− = × ×
5 4 5 5 4 5
@

No. of cases favourable to E1


P(E 1) = 27
Total number of cases =
6
C4 100
= 15 2 3 3 2 3 2
C4 P(E3) = × × 1− = × ×
Probability of drawing 4 black balls in 5 4 5 5 4 5
the second draw is given as P(E2) 12
9 =
C 100
P(E2) = 15 4 6 27 12
C4 Required probability = + +
100 100 100
The required probability = P(E1 E2) 45
= = 0.45 .
= P(E1) . P(E2) 100
6
C4 9
C4 37. Ans. (d)
= 15
× 15 Sol. 3 marbles are taken out of 12 in 12C3
C4 C4 ways.
Unit Six : Practice Session ✫ 489

1 white marble out of 4 can be taken Now, probability that X shall not solve
out in 4C1 ways. the problem
2 red marbles tan to taken out of 5 P(X ) = 1 – P(X)
in 5C2 ways.
1 1
∴ Probability of drawing 1 white and 2 = 1 – =
red marbles 2 2
5! 5! Probability that Y shall not solve the
4× 4× problem
4
C × C2
5
(5 − 2)! 2 ! 3! 2!
= 112 = = P(Y ) = 1 – P(Y)
C3 12! 12 ! 1 2
=

TS
(12 − 3)! 3 ! 9! 3! = 1 –
3 3
5× 4 Probability that Z shall not solve the

2 40 problem
= =
12 × 11 × 10 20 × 11 P( Z) = 1 – P(Z)

H
3×2 1 3
2 = 1 – =
4 4

G
= . The probability that the problem shall
11
38. Ans. (a) not be solved by X, Y, Z together
Sol. Total number of chances in two dices is
6 × 6 = 36.
U = P(X ) × P(Y ) × P(Z )
1 2 3 1
O
Probability of having sum 9 is 4 = × × =
(6 + 3; 5 + 4; 4 + 5; 3 + 6) 2 3 4 4
n (E1 ) 4 1 ∴ The probability that the problem shall
H
P(E) = = = . be solved = 1 – (probability that the
n ( S) 36 9
39. Ans. (d) problem shall not be solved)
_T

Sol. Here, there are 52 cards, i.e., n = 52; 4 1 3


kings, i.e., r = 4; 13 diamonds, i.e., = 1− = .
4 4
r’ = 13; and there is 1 king of diamonds 41. Ans. (b)
C

(common event, both king and Sol. A dice is thrown twice


diamond). Thus, we have:
∴ The number of outcomes = 6 × 6 = 36
4 13 1
PS

P(A) = , P(B) = , P(A ∩ B) = Favourable cases that at least one of the


52 52 52 two throws comes up with number 3 is
Since P(A ∪ B) = P(A) + P(B) – P(A ∩ B) {(1, 3), (2, 3), (3, 1), (3, 2), (3, 3), (3, 4),
4 13 1
U

+ − (3, 5), (3, 6), (4, 3), (5, 3), (6, 3)}
∴ P(A ∪ B) =
52 52 52 i.e., 11 cases
16 4
@

= n(E) 11
=
52 13
. ∴ Probability = = .
n(S) 36
40. Ans. (c) 42. Ans. (a)
Sol. Probability that X solves the problem Sol. The second ball drawn from the bag is
1 red. We had 3 black and 4 red balls in
= P(X) =
2 the bag and now only 3 black and 3 red
Probability that Y solves the problem balls in the bag.
1 So, the probability that a black ball is
= P(Y) =
3 drawn
Probability that Z solves the problem
3 1
1 = = .
= P(Z) = 6 2
4
490 ✫ Quantitative Aptitude

UNIT SEVEN
LOGARITHM ✫ TRIGONOMETRY ✫ HEIGHTS AND DISTANCES

FUNDAMENTALS AND

TS
WORKED EXAMPLES

H
Logarithm Earth, so Saturn is two orders of magnitude more
massive than Earth. Order of magnitude

G
Order of Magnitude and Logarithm differences are called decades when measured
When we make rough estimates, comparisons on a logarithmic scale.
or calculations, we sometimes round off a
number to zero—the nearest power of 10.
A number rounded to the nearest power U Logarithm
The word ‘logarithm’ was coined from two
O
of 10 is called an order of magnitude. If, for Greek words, logos which means a ‘ratio’ and
example, we take the average height of a arithmos, meaning ‘number’.
H
human to be about 1.7 metres, we may round If a is a positive real number such that
off 1.7 metres to the nearest power of 10, which ax = b, then the exponent x is called the logarithm
of b to the base a. Symbolically,
_T

is 100 m (or 1 m). This does not mean that


the average height of a person is a mere 1 ax = b logab = x
metre, but rather the average height is closer and it is read as ‘logarithm of b to the base
to 1 metre (or 100 metres) than it is to 10 metres a is x3.’
C

(or 101 metres). Similarly, rounding off the For example, 20 = 1 ⇔ log21 = 0
height of an ant, which is about 8 × 10–4 metres, 1
PS

to the nearest power of ten results in 10–3 641/3 = 4 ⇔ log644 =


3
metres. There are two bases of logarithms: one is
We can also say that the order of magnitude base ‘e’ (e = 2.71828 approx.) and the other is
of the height of an ant is 10–3 metres. Now, base ‘10’. The logarithms to base e are called
U

if we compare the height of a human being (100 natural logarithms and the logarithms to base
metres) with the height of an ant (10–3 metres), 10 are called common logarithms.
@

we come up with the ratio of human height


Properties of Logarithms
to ant height = 100/10–3 = 100 – (–3) = 103 = 1000.
So we may conclude that a human being is (i) loga(pq) = logap + logaq
about 1000 times (or 103 times) taller than an p
ant. In other words, a human being is 3 orders (ii) log a = logap – logaq
q
of magnitude (3 powers of 10) taller than an
ant. (iii) logaa = 1
To be an order of magnitude greater is to (iv) loga1 = 0
be 10 times as large. The logarithmic scale is
used to measure such differences in order of (v) loga(pq) = qlogap
magnitude. (vi) logap × logpa = 1
An order of magnitude difference between
two values is a factor of 10. For example, the 1
mass of the planet Saturn is 95 times that of log p a
490
Unit Seven : Fundamentals / Logarithms ✫ 491

log b p log p it always lies between 0 and 1. For mantissa,


(vii) logap = log a = log a we look through the log table.
b
(Note that the old base a is converted to the Thus, to obtain log n we find characteristic
newbase b.) and mantissa of log n, and then add them.
(viii) alogap = p Applications of Logarithm
(ix) When the base is not given, it is taken The invention of logarithms shortens
as 10. calculations. It is used in numerical calculations
(x) Negative numbers and zero have no and for calculating compound interest,
logarithms.

TS
population growth, and depreciation of value.
Characteristic and Mantissa of Logarithm
Worked Examples
The characteristic of a number is the integer
which is raised as power of 10 when it is ● What is the value of log10 0.00001?

H
written in standard form. For example, 1
Log10 0.00001 = log 10
32.4 = 3.24 × 101 is in standard form. Any 100000

G
positive decimal number n can be written in = log10 10–5
the form n = m × 10x, where x is an integer = –5 log10 10 [as log1010 = 1]
and 1 ≤ m < 10.
This is called the standard form of the decimal ‘n’.
U = –5 × 1
= –5
O
The characteristic of a logarithm of a
number greater than one is one less than the 12 log 10 10
● Find the value of
number of digits in the integral part of the 2 log10 100
H
number. It is positive. 12 log 10 10
_T

Number Digits in Characteristic 2 log10 100


integral part 12 × 1
= [as log1010 = 1]
243.72 243 (3) 3–1 = 2 2 × log 10 10 2
C

78.345 78 (2) 2–1 = 1 12


4932.751 4932 (4) 4–1 = 3 =
2 × 2 log 10 10
PS

8.04 8 (1) 1–1 = 0


12
= =3
The characteristic of a logarithm of a 4
number less than one is one more than the
● If log 3 = 0.4771, what is the number of digits
U

number of zeros immediately after the decimal.


in 356 ?
It is negative, but instead of –1, –2, ..., we write
356= log 356
@

1 (one bar), 2 (two bar), and so on.


= 56 log 3
Number Number of zeros Characteristic = 56 × 0.4771
after decimal
= 26.7176
0.00379 2 2+1 = 3
● What is the value of log10 4 100 ?
0.043 1 1+1 = 2
0.128 0 0+1 = 1 log10 4 100
1
0.00064 3 3+1 = 4 = log 10 (100 ) 4
Characteristic is always an integer positive, 1
= log 10 100
negative or zero. 4
The mantissa is the decimal part of the
logarithm of a number. It is never negative and =
1
4
(log 10 10 2 )
492 ✫ Quantitative Aptitude

2 log 625
= log 10 10 (as log10 10 = 1) =
4 log 5
1
= log (5 4 )
2 =
log 5
log 8
● What is the value of ? 4 log 5
log 8 = = 4.

log 8
=
log 8 ( )
1
2

log 5
If log102 = 0.3010 and log103 = 0.4771, find the

TS
log 8 log 8 value of log101.5.
1 3
log 8 log10(1.5) = log10
2 1
= = 2

H
log 8 2
= log103 – log102
a b
+ log = log ( a + b) , which of the

G
● If log
b a = 0.4771 – 0.3010
following is true?
(a) a + b = 1
(b) a – b = 1 U ●
= 0.1761.

If log102 = 0.3010, find the value of log105.


O
(c) a = b
(d) a2 – b2 = 1
log105 = log10 10
H
a b 2
log + log = log ( a + b) = log1010 – log102
b a
_T

a b = 1 – 0.3010 [  log1010 = 1]
log ( a + b) = log × = log 1
b a = 0.6990.
∴ a +b = 1
● log55 log49 log32 = ?
C

The correct answer is (a).


● Find the value of log5 125? log 5 log 9 log 2
× ×
PS

Let log5 125 = n log 5 log 4 log 3


Then 5n = 125 log 32 log 2
5 n = (5)3 = 1 × ×
log 22 log 3
U

n = 3.
● Find the value of log31, 2 log 3 log 2
= 2 log 2 × log 3
@

We know that, loga1 = 0


So log31 = 0. = 1.
● Find the value of log10103.
m n
log10103 ● If log + log = log (m + n), find the value
= 3 log1010
n m
of m + n.
= 3 × 1 [  log1010 = 1] m n
=3 log (m + n) = log + log
n m
● Find the value of (log53) × (log3625). m n
= log ×
(log53) × (log3625) n m
log 3 log 625 = log 1
= ×
log 5 log 3 ∴ m + n = 1.
Unit Seven : Fundamentals / Trigonometry ✫ 493

Basic Trigonometry
Trigonometry is a Greek word, having three
words in itself. Trio means thrice, Gonia means
angle and Metron means measure. So the word A r B
meaning of trigonometry simply put is O
“measurement of triangles” (sides and angles). r
r
Measuring Angles
When the two rays (initial and terminal) unite C

TS
together at a point with the rotation in a plane Thus, the length of the arc = radius × angle
to bring the position of one ray into the subtended by the arc at the centre in radians
position of the other ray, an angle is formed. Given in the figure,
arc ACB πr

H
∠AOB = = = π radians
r r
π radians = 180 = 2 right angles.
o
de

G
id
si

ls
al

180
a
in

iti
rm

1 radian = degree
In

π
Te

Po
s iti
ve
N
eg
at
iv
e

U o
= 57 17’44.8’’(approximate)
O
O O Sectorial Area Let ABC be a sector having
(a) Initial side (b) Terminal side central angle θ and radius r.
H
Rotation of ray Angle measure B
Anticlock wise Positive
_T

Clockwise Negative
Measurement of Angles by Sexagesimal System θ
A C
C

(degrees, minutes and seconds) 1 2


1 The area of sector ABC = r θ
1 degree = 1o = right angles 2
PS

90 Trigonometrical Ratios
1 The ratios of the sides of a right-angled
1 minute = 1’ = degree
60 triangle with respect to its angles are called
1 trigonometric ratios.
U

1 second = 1’’ = minute


60
or other way P
(x, y)
@

90o degree = 90o = 1 right angle


60 minutes = 60’ = 1o
60 seconds = 60” = 1’
r y
Measurement of Angles by Circular System
(Radians)
θ
Radian is the angle subtended by an arc at the
O x M
centre of a circle whose length is equal to the
radius of the circle. One circular measure is In a right-angled triangle OPM,
denoted by 1c. The S.I. symbol for a radian
Perpendicular MP y
is rad. ∩ (i) = = = sine θ
In the given figure AO = OC = AC = r. Hypotenuse OP r
So, ∠AOC = 1c Adjacent side OM x
(ii) = = = cosine θ
∠AOC = one radian Hypotenuse OP r
494 ✫ Quantitative Aptitude

Values of Trigonometric Ratios


π π π π 3
Angle θ 0º 300 or 450 or 600 or 900 or 1800 or π 2700 or π 3600 or 2 π
6 4 3 2 2

1 1 3
sin 0 1 0 –1 0
2 2 2
3 1 1
cos 1 0 –1 0 1
2 2 2

TS
1
tan 0
3
1 3 ∞ 0 ∞ 0

H
2
cosec ∞ 2 2 3
1 ∞ -1 ∞

G
2
sec 1
3 2 2 ∞ –1 ∞ 1

cot ∞ 3 1
1
3 U 0 ∞ 0 ∞
O
Perpendicular MP y Heights and Distances
H
(iii) Adjacent side = = = tangent θ
OM x Trigonometry helps to calculate the distance
Hypotenuse OP r between the points and the heights of the
_T

(iv) = = = cosecant θ objects, without measurements. For solving


Perpendicular MP y
such problems, generally angle of elevation and
Hypotenuse OP r angle of depression are used.
(v) Adjacent side = = = secant θ
x
C

OM
Angle of Elevation
Adjacent side OM x
(vi) = = = cotanget θ A
PS

Perpendicular MP y (object)
[Abbreviations normally used are sin, cos,
tan, cosec, sec, cot respectively.] Angle of elevation
Note: The values of the trigonometric rataios (eye)
U

B
of an angle do not vary with the lengths of the O
When an object is higher than the observer’s
sides of the triangle, if the angle remains the
@

eye, the angle between the horizontal line


same. passing through the observer’s eye and the line
Basic Trigonometrical Ratios joining the object and the eye is called the angle
of elevation.
1
cosec θ = i.e., cosec θ . sin θ = 1
sin θ Angle of Depression
1
sec θ = i.e., sec θ . cos θ = 1
cos θ
1
cot θ = i.e., cot θ . tan θ = 1
tan θ
sin θ cos θ
tan θ = and cot θ =
cos θ sin θ When an object is lower than the observer’s
sin (–θ) = –sin θ, cos (–θ) = cos θ eye, the angle between the horizontal line
tan (–θ) = –tan θ passing through the observer’s eye and the line
Unit Seven : Fundamentals / Heights and Distances ✫ 495

joining the object and the eye is called the angle Worked Examples
of depression. ● A kite is flying with a thread 250 m long.
If the thread is assumed to be stretched
Pythagoras Theorem
straight and makes an angle of 55° with the
In a right-angled triangle the square of its
horizontal then find the height of the kite
hypotenuse is equal to the sum of the squares
above the ground.
of perpendicular and base. B
(Hypotenuse)2 = (Perpendicular)2 + (Base)2
P
0m
25

TS
h
se

Perpendicular
(r enu

55°
ot

C A
)
yp

H
(y)
H

AB = h metres, CB = 250 metres


AB
In Δ ABC, we have = sin 55°

G
CB
h
= sin 55° = 0.8192
250
O
Base (x)
(OP)2 = (OM)2 + (MP)2
M

U h = 250 × 0.8192 = 204.8 m


Hence, the height of the kite is 204.8 m.
O
r2 = x2 + y2 ● If a 30m ladder is placed against a 15m
Working Rule wall such that it just reaches the top of
H
Step I : Understand the problem first. the wall, what is the elevation of the wall?
Step II : Make a clear figure of the problem AB = 15 m, BC = 30 m B
_T

and mark all its dimensions. In Δ BCA


Step III : Choose right angle triangle to solve AB 15
the problem. sin θ = =
BC 30
m
Step IV : If the problem comprises of more than
C

1 30
15 m
one triangle, then solve one by one. = sin θ
Step V : Use correct trigonometric ratio 2
PS

unknown side sin 30° = sin θ θ


i.e., θ = 30° C A
known side
= known angle of the triangle. Hence, the angle of elevation is 30°.
U

● An observer measures angle of elevation


The trigonometrical ratios for right-angled of two towers of equal heights from a
triangles discussed earlier are useful in point between the towers. If the angle of
@

determining the height of a distant object or elevation are 60° and 30° and distance of
the distance between two far-off objects. the nearer tower is 100 m, what is the
height of each tower and distance between
For triangles (other than right-angled) the towers, respectively?
a b c B D
We can use = =
sin A sin B sin C
b2 + c2 − a 2
or cos A = h
h
2bc
60º 30º
A few important values to memorise A C
100 m P x
2 = 1.414
Let AB and CD be the two towers
3 = 1.732
In ΔABP
5 = 2.236
496 ✫ Quantitative Aptitude

AB h AB = cliff = 90 m
tan 60° =  3=
AP 100 ∠ADE = 30°
h = 100 3 m ∠ACB = 60°
In ΔCPD CD = tower = h m
CD BC = x m
tan 30° = From ΔABC,
CP AB
1 100 3 tan 60° =
 = BC
3 x
90
x = 100 3 × 3 3 =  tan 60°= 3

TS
x
= 300m
90
∴ AC = Distance between the towers x=
= 100+300 = 400m 3

H
● Two ships leave a port at the same time. x = 30 3 m
One sails at 30km/h in the direction
From ΔADE,

G
N 32°E while other sails at 20km/h in the
direction S 58°E. After 2 hours how far AE
tan 30° =
are the ships from each other? ED
Let A and B be the two ships. After 2 hours
AB = x U 1
3
=
90 − h
30 3
O
∠AOB = 180° – (58°+32°)  1 
= 90°  tan 30° = 3 , BC = ED 
 
H
OA = 2 × 30 = 60 km
OB = 2 × 20 = 40 km N A 30 3
∴ 90 – h =
_T

In ΔOAB 3
AB2 = OA2 + OB2 90 – h = 30
h = 90 – 30
32°
x = (60) 2 + ( 40) 2 h = 60 m
C

58° ∴ Hence, the height of the tower is 60 m.


= 3600 + 1600 W E
PS

O 32° ● P and Q are two points observed from the


= 5200 top of a building 10 3 m high. If the angles
58°
= 4 × 100 × 13 of depression of the points are complementary
and PQ = 20 m, find the distance of P from
U

= 2 × 10 13 the building.
= 20 13 km
@

S B
● From the top of a cliff 90 metre high, the
angles of depression of the top and bottom
of a tower are observed to be 30° and 60°
respectively. Find the height of the tower. 10 3 m

AB = Building = 10 3 m.
PQ = 20 m
Let BQ = x metre
Unit Seven : Fundamentals / Heights and Distances ✫ 497

If ∠ABP = θ then, ∠ADB = 90° – θ


AQB = 90° – θ and ∠ACB = θ
From ΔABP, From ΔABC,
AB AB
tan θ = tan θ =
BP BC
10 3 h
tan θ = ... (i) tan θ = ... (i)
x + 20 y
From ΔABQ, From ΔABD,
AB

TS
AB tan (90° – θ) =
tan (90° – θ) = BD
BQ
h
10 3 cot θ = ... (ii)
cot θ = ... (ii) x
x

H
By multiplying both equations,
[ tan (90° – θ) = cot θ] h h
By multiplying both equations, tan θ. cot θ = ×

G
y x
10 3 10 3
tan θ. cot θ =
2
× h
x + 20 x 1=

1= 2
100 × 3
x + 20 x U h2 = xy
xy
O
x2 + 20x = 300 h= xy m
⇒ x2 + 20x – 300 = 0 Hence, the height of the tower is xy m.
H
x2 + 30x – 10x – 300 = 0
x (x + 30) – 10 (x + 30) = 0 ● A telegraph post is bent at a point above
_T

(x – 10)(x + 30) = 0 the ground due to storm. Its top just


x = 10, –30 meets the ground at a distance of 8 3
metres from its foot and makes an angle
∴ BP = BQ + PQ = x + 20
of 30°. Find the height of the post.
C

= 10 + 20 = 30 m AB = h metre
Hence, the distance of P from the Telegraph post bends at point D.
PS

building is 30 m A
● The angles of elevation of the top of a
tower from two points at a distance x D
U

h metre

and y from the foot of the tower are


complementary. Find the height of the
xm

tower.
@

30°
B
8 3m C
DB = x m
∴ AD = CD = (h – x) metre
BC = 8 3 m
From ΔDBC,
DB
sin 30° =
DC
AB = h metre 1 x
BD = x m, BC = y m =
2 h−x
498 ✫ Quantitative Aptitude

h – x = 2x D and E are the positions of the plane


h = 2x + x ∠DAB = 60°,
h = 3x ∠EAB = 30°
Now,
DB = 1500 3 m
DB
tan 30° = In ΔABD,
BC
1 x tan 60° = BD
= AB
3 8 3
1500 3

TS
3 ×x = 8 3 3 =
AB
8 3 1500 3
x =
3 AB =
3

H
x = 8 m
AB = 1500 m
∴ h = 3x
In ΔACE,
= 3 × 8 [ x = 8 m]

G
CE
= 24 m tan 30° =
Hence, the height of the telegraph post AC


is 24 m.
The angle of elevation of an aeroplane U 1
3
=
1500 3
AC
O
from a point on the ground is 60°. After AC = 1500 3 × 3
15 seconds flight, the elevation changes AC = 4500 m
H
to 30°. If the aeroplane is flying at a DE = BC = AC – AB
height of 1500 3 m, find the speed of = 4500 – 1500
_T

the plane. = 3000 m


Distance
Speed of plane =
Time
C

3000
∴ Speed of plane =
15
PS

= 200 m/sec
Hence, the speed of the plane is 200 m/sec.
U
@
Unit Seven : Practice Session ✫ 499

PRACTICE SESSION
Practice Exercise 43
1
12 log 10 10 10. The value of log 10 125 − 2log10 4 + log10 32
1. The value of is is
3
2 log 10 100
(a) 2 (b) 3 (a) 0 (b) 1
(c) 4 (d) 5 (c) 2 (d) 3

TS
1 log 8
2. If log (11 + 4 7 ) = log (2 + x) , then the 11. is equal to
2 log 8
value of x is 1 1
(a) (b)

H
(a) 7 (b) 11 8 4
(c) 4 (d) 2 1 1
(c) (d)

G
3. log1010 = 1 when expressed in exponential 2 8
a b
form is 12. If log + log = log (a + b) , then
(a) 110 = 1
10
(c) 1 = 10
(b) 1010 = 1
(d) 101 = 10
U b
(a) a + b = 1
(c) a = b
a
(b) a – b = 1
(d) a2 – b2=1
O
4. If log 2 = 0.3010 and log 3 = 0.4771, then
the value of log 1.5 is 13. log53 × log3625 equals
(a) 1 (b) 2
H
(a) 0.0876 (b) 0.1761
(c) 0.7782 (d) 0.7781 (c) 3 (d) 4
_T

16 14. The value of log216 is


5. The value of log 10 2 + 16 log 10
15 (a) 4 (b) 8
25 81
+ 12 log 10 + 7log 10 is (c) 16 (d)
1
24 80 8
C

(a) 0 (b) 1 15. If log103 = 0.477, the number of digits in


(c) log102 (d) log103 350 is
PS

2 (a) 23 (b) 24
6. The value of log 2 6 + log 2 is
3 (c) 50 (d) 150
1
(a) (b) 1 16. The value of log1040,000 – log104 is eqaul
U

2
3 to
(c) (d) 2 (a) 4 (b) 10,000
@

2
1 1 (c) log1039,996 (d) 39,996
7. The value of log 10 1 + log10 1 + ... upto
2 3 16 25 81
199 terms, is equal to 17. 7 log2 + 5 log2 + 3 log2
15 24 80
(a) 0 (b) 2
(c) 100 (d) 10 is equal to
11 14 22 (a) log10 2 (b) log2 2
8. The expression log + log − log is
5 3 15 (c) log10 5 (d) log5 5
equal to
(a) log 2 (b) log 3
18. If log 8 = 0.9031 and log 9 = 0.9542 then
(c) log 5 (d) log 7
value of log 6 is
9. log10(101 × 102 × 103 × ... × 109) is
(a) 0.7781 (b) 0.9221
(a) 10 (b) 20
(c) 45 (d) 55 (c) 0.3279 (d) 0.4213
500 ✫ Quantitative Aptitude

19. If log3 = 0.4774 then the number of digits 25. Value of log 8 + log 1 is
in 950 is 8
(a) 47 (b) 77 (a) 0 (b) 1
(c) 48 (d) none of these (c) 4 (d) log 64
26. Value of log2 3x + log2 2x + log2 x = 11
20. Value of log5 125 × 625 is then value of x is
25 (a) 2 (b) 4
(a) 6 (b) 5 (c) 20 (d) 128
(c) 725 (d) 3125

TS
27. (log55) (log49) (log32) is equal to
21. If log10 98 + x2 − 12x + 36 = 2 find x (a) 1 (b)
3
2
(a) 8 or 4 (b) 9 or 6 (c) 2 (d) 5

H
(c) 4 or 6 (d) 17 or 9  1 
28. If log107 = a, then log10   is equal to
22. Value of logb a logc b logd c loga d is  70 

G
(a) 2 (b) –2 (a) –(1 + a) (b) (1 + a)–1
(c) 1 (d) –1 a 1

23. If log10 m = b – log10 n find the value of m. U (c)


10
(d)
10a
O
29. What will be the value of
10b
(a) (b) 10 log5 {log5 25 + log5 125}?
n
H
(a) 2 (b) 1
10n
(c) (d) None of these (c) 5 (d) None of the above
b
_T

1 30. If log 3 = 0.477 and (1000)x = 3, then x


24. If log x = log 5 + 2 log 3 – log 25,
2 equals
find the value of x.
(a) 0.159 (b) 10
(a) 7 (b) 0
C

(c) –1 (d) 9 (c) 0.477 (d) 0.0477


PS

Answers with Solutions


1. Ans. (b) 3. Ans. (d)
12 × log10 10 12 × 1 Sol. log1010 = 1
U

Sol. = In exponential form it is written as 101 = 10.


2 × log 10 100 2 × log 10 10 2
@

12 × 1 4. Ans. (b)
=
2 × 2 log10 10 Sol. log 1.5 = log (3 ÷ 2)
[  log1010 = 1] = log 3 – log2
12 12 = (0.4771 – 0.3010)
= = = 3. = 0.1761.
2×2 4
2. Ans. (a) 5. Ans. (b)
1 16 25 81
Sol. log (11 + 4 7 ) = log (2 + x) Sol. log10 2 + 16 log 10 + 12log + 7 log10
2 1
15 24 80
log (11 + 4 7 ) 2 1 = log (2 + x) = log102+16 log1016–16 log1015
log ( 7 ) + (2) + 2 × 2 × 7 =
2 2 2
log (2 + x) +12 log1025–12 log1024
log ( 7 + 2) = log (2 + x) + 7 log1081 – 7 log1080
∴ ( 7 + 2) = 2+x
= log102 + 16 log10(24) – 16 log10(3 × 5)
or x = 7.
Unit Seven : Practice Session ✫ 501

+ 12log10(52) – 12log10(23 × 3) = –log102 + log102 + 2log1010


+ 7 log10(34) – 7 log10(24 × 5) = 2log1010
= log102 + 64 log 102 – 16 log103 = 2. [ log1010 = 1]
– 16log105 + 24log105 – 36log102 8. Ans. (d)
– 12log103 + 28log103 – 28log102 11 14 22
Sol. log + log − log
– 7log105 5 3 15
= log102 + 64log102 – 36log102 = log 11 – log 5 + log 14 – log 3 – log 22
+ log 15

TS
– 28log102 – 16 log103 – 12 log103
= log 11 – log 5 + log (2 × 7) – log 3
+ 28log103 –16log105 + 24log105 – log (2×11) + log (5 × 3)
– 7log105 = log 11 – log 5 + log 2 + log 7 – log 3

H
= log102 – 0 + log105 – log 2 – log11 + log 5 + log3
= log102 + log105 = log 7.

G
= log10(2 × 5) 9. Ans. (c)
= log1010 Sol. log10(101 × 102 × 103 × ... × 109)
= 1.
6. Ans. (b)
[  log1010 = 1]
U = log1010 + log10102 + log10103 + ... + log10109
= 1 + 2log1010 + 3log1010 + ... + 9log1010
O
2 = 1+2+3+4+5+6+7+8+9
Sol. log 2 6 + log 2
3 [  log1010 = 1]
H
1
1 2 2 = 45.
= log 2 (6) 2 + log 2
3 10. Ans. (b)
_T

1 1 2 1
= log 2 6 + log 2 Sol. log10 125 − 2log10 4 + log10 32
2 2 3 3
1 1 1
log 2 (2 × 3) + [log 2 2 − log 2 3] log 10 (53 ) − 2log10 (22 ) + log 10 (25 )
C

= =
2 2 3
1
= 3 × log 10 5 − 2 × 2 log 10 2 + 5 log 10 2
PS

1 1 1 1
= log 2 2 + log 2 3 + log 2 2 − log 2 3 3
2 2 2 2 = log105 – 4log102 + 5log102
1 1 = log105 + log102
= log 2 2 + log 2 2
U

2 2 = log10(5 × 2)
= 1 log22 [as logaa = 1] = log1010 [ log1010 = 1]
@

= 1. = 1.
7. Ans. (b) 11. Ans. (c)
1

1 1 1 log 8 log(8) 2
Sol. log10 1 + log 10 1 + log 10 1 + ...199 Sol. =
2 3 4 log 8 log 8
terms. 1
log 8 1
3 4 5
= log10 + log 10 + log 10 + ...log10
200 = 2 = .
2 3 4 199 log 8 2
= log103 – log102 + log104 – log103 12. Ans. (a)
a b
+ l o g 105 – l o g 104 + . . . + l o g 102 0 0 Sol. log + log = log (a + b)
b a
– log10199 a b
log (a + b) = log ×
= –log102 + log10200 b a
= –log102+log10(2 × 102) log (a + b) = log 1
∴ a + b = 1.
502 ✫ Quantitative Aptitude

13. Ans. (d) = 28 log22 – 15 log22 – 12 log22 – 7 log23


Sol. log53 × log3625 – 5 log23 + 12 log23 – 7 log25 + 10 log25
log 3 log 625 – 3 log25
= ×
log 5 log 3 = log2 2.
log 625 log(5)4 18. Ans. (a)
= =
log 5 log 5 Sol. log 8 = 0.9031
4 log 5 log 23 = 0.9031
= = 4.

TS
log 5 3 log 2 = 0.9031
14. Ans. (a) 0.9031
log 2 = = 0.3010
Sol. log216 = log224 3
log 9 = 0.9542

H
= 4log22 = 4. [ log22 = 1]
log 32 = 0.9542
15. Ans. (b)

G
2 log 3 = 0.9542
Sol. Suppose n = 350 0.9542
Taking log on both sides, we get log 3 = = 0.4771
log10n = 50 log103
= 50 × 0.477
U ∴
2
log 6 = log (3 × 2)
= log 3 + log 2
O
= 23.85
∴ Number of digits contained in 350 are = 0.4771 + 0.3010
23 + 1 = 24. = 0.7781
H
16. Ans. (a) 19. Ans. (a)
Sol. Suppose n = 950
_T

40000
Sol. log1040000 – log104 = log10
4 Taking log on both sides, we get
A
 log = log A − log B log10n = 50 log109
B
C

= log1010000 log10n = 50 log1032


= log10(10 × 10 × 10 × 10) log10n = 50 × 2 log103
PS

= log1010 + log1010 + log1010 + log1010 log10n = 100 log103


= 1+1+1+1 log10n = 100 × 0.4771
= 4. = 47.71
U

17. Ans. (b) Number of digits in 950 are 47 + 1 = 48.


20. Ans. (b)
@

Sol. 7 log216 – 7 log215 + 5 log225 – 5 log224


125 × 625
+ 3 log281 – 3 log280 Sol. log5 log5 (5 × 54 )
25
= 7 log22 4 – 7 log2(3×5) + 5 log 25 2
∴ log5 55 = 5 log55
– 5 log2(8×3) + 3 log234 – 3 log2(16×5) ∴ = 5 × 1 = 5. [  log55 = 1]
= 7×4 log22 – 7 log23 – 7 log25 + 5×2 log25 21. Ans. (a)
– 5 log22 3 – 5 log 23 + 3×4 log23 Sol. log10 98 + x 2 − 12x + 36 = log10 100
– 3 log224 – 3 log25
98 + x 2 − 12x + 36 = 100
= 28 log22 – 7 log23 – 7 log25 + 10 log25
– 15 log22 – 5 log23 + 12 log23 – 12 log22 x 2 − 12x + 36 = 100 – 98
– 3 log25 x 2 − 12x + 36 =2
Unit Seven : Practice Session ✫ 503

Squaring both sides, we get 27. Ans. (a)


x2 – 12x + 36 = 4 Sol. (log55) (log49) (log32)
x2 – 12x + 32 = 0 log 9 log 2 log 32 log 2
2
x – 4x – 8x + 32 = 0 = 1× × = ×
log 4 log 3 log 2 2 log 3
x (x – 4) – 8 (x – 4) = 0
2 log 3 log 2 2
(x – 8) (x – 4) = 0 = × = = 1.
2 log 2 log 3 2
x = 8 or 4.
22. Ans. (c) 28. Ans. (a)

TS
Sol. logb a logc b logd c loga d 1
Sol. log10 = log101 – log1070
log a log b log c log d 70
= × × × = – log10 (7 × 10)
log b log c log d log a

H
= 1. = – (log107 + log1010)
[ given log107 = a]
23. Ans. (a)

G
= – (a + 1).
Sol. log10 m = b – log10 n
29. Ans. (b)


log10 m + log10 n = b
log10 (mn) = b
10b = mn
U Sol. Given expression
= log5 (log5 25 + log5 125)
O
log 25 log 125
10b = log5 +
∴ m = . log 5 log 5
H
n
24. Ans. (d) log 5 2 log 5 3
1 = log5 +
log 5 log 5
_T

Sol. log x = log 5 + 2 log 3 – log 25


2 1 2 log 5 3 log 5
log x = log 5 + log 32 – log (25) 2 = log5 +
log 5 log 5
log x = log 5 + log 9 – log 5
C

5 log 5
log x = log 9 = log5
log 5
PS

∴ x = 9. = log5 5
25. Ans. (a) log 5
1 1 = = 1.
Sol. log 8 + log = log 8 × log 5
8 8
U

= log 1
30. Ans. (a)
= 0.
Sol. (1000)x = 3
@

26. Ans. (d)


⇒ log [(1000)x] = log 3
Sol. log2 3x + log2 2x + log2 x = 11 ⇒ x log 1000 = log 3
1 1
⇒ log 2 x + log 2 x + log2 x = 11 ⇒ x log (10)3 = log 3
3 2
⇒ 3x log 10 = log 3
11
⇒ log 2 x = 11 ⇒ 3x = log 3 [ log 10 = 1]
6
6 3x = 0.477
⇒ log 2 x = 11 × =6
11 0.477
⇒ 2 = x = 128
6 x = = 0.159.
3
504 ✫ Quantitative Aptitude

Practice Exercise 44
1. What is the angle of elevation of the sun 7. A person at the top of a hill observes that
when the length of the shadow of a tree the angle of depression of two consecutive
is 3 times the height of the tree? kilometre stones on a road leading to the
(a) 30° (b) 45° foot of the hill are 30° and 60°. The height
(c) 60° (d) 90° of the hill is
2. From a point P on a level ground, the 3 5
(a) km (b) km
angle of elevation of the top of a tower 2 2

TS
is 30°. If the tower is 100 m high, the 6 7
distance of point P from the foot of the (c) km (d) km
2 2
tower is:
8. If from a point 100m above the ground,
(a) 149 m (b) 156 m

H
the angle of depression of two objects due
(c) 173 m (d) 200 m
south on the ground are 60° and 45°, the
3. The angle of elevation of the top of a

G
distance between the objects is
tower at any point on the ground is 30°. 100( 3 − 1)
Moving 20 m towards the tower, it becomes (a) 100( 3 + 1) m (b) m
60°. The height of tower is
(a) 10 m (b) 10 3 m U (c) 100 3
3
(d) None of these
O
9. A person standing on the bank of a river
10 3
(c) m (d) None of these observes that the angle subtended by a
3 tree on the opposite bank is 60°. When
H
4. A kite is flying with a string 150m long. he retires 40 m from the bank he finds
It makes an angle of 60° with the the angle to be 30°. Then the breadth of
_T

horizontal. Find the height of the kite the river is


from the ground. (a) 40 m (b) 60 m
(a) 132 m (b) 139.5 m (c) 20 m (d) 30 m
(c) 129.9 m (d) 175.2 m
C

10. A tree is broken by the wind. The top


5. From the top of a house 32m high the struck the ground at an angle of 30° and
angle of elevation at the top of a tower at a distance of 30 m from the root. Find
PS

is 45° and the angle of depression at the the whole height of the tree.
foot of the tower is 30°. The distance of (a) 47.23 m (b) 51.96 m
the tower from the house is (c) 37.58 m (d) 56.78 m
U

(a) 30 3 m (b) 32 3 m
11. A ladder is placed along a wall of a house
(c) 35 3 m (d) None of these such that its upper end is touching the top
@

6. An observer measures angle of elevation of the wall. The foot of the ladder is 2 m
of two towers of equal heights from a away from the wall and the ladder is
point between the towers. If the angle of making an angle of 60° with the level of
elevation are 60° and 30° and distance of the ground. Determine the height of the
the nearer tower is 100m then the height wall.
of each tower and distance between the 2
(a) 2 3 m (b) m
towers, respectively are 3
3
50 (c) m (d) None of these
(a) m and 300 m 2
3
50 12. A circus artist is climbing from the ground
(b) m and 200 m along a rope stretched from the top of
3
vertical pole and tied at the ground. The
(c) 50 3 m and 200 m height of the pole is 12 m and the angle
(d) 50 3 m and 300 m made by the rope with ground level is
Unit Seven : Practice Session ✫ 505

30°. Calculate the distance covered by the ship during the period of observation.
artist in climbing to the top of the pole. (a) 70 m (b) 84 m
(a) 29 m (b) 17 m (c) 73.2 m (d) 85.3 m
(c) 46 m (d) 24 m 17. A tree stands vertically on the bank of a
13. From a point on the ground 40m away river. From a point on the other bank
from the foot of a tower, the angle of directly opposite the tree, the angle of
elevation of the top of the tower is 30°. elevation of the top of the tree is 60°.
The angle of elevation to the top of a From a point 20 m behind this point on
water tank is 45°. Find the height of the the same bank the angle of elevation of

TS
tower and depth of the tank. the top of the tree is 30°. Find the height
40 20 of the tree and the width of the river.
(a) 3 m, 16.9 m (b) m, 17.4 m (a) 17.3 m, 10 m (b) 18.5 m, 2 m
3 3
12 (c) 16.2 m, 9 m (d) 14.7 m, 7 m

H
(c) m, 19 m (d) None of these 18. The horizontal distance between two
3
towers is 80 m. The angle of depression
14. An aeroplane, when 3000 m high, passes

G
of the top of the first tower when seen
vertically above another aeroplane at an
from the top of the second tower is 30°.
instant when the angle of elevation of the
If the height of the second tower is 160 m,
two aeroplanes from the same point on
the ground are 60° and 45°. Find the
U find the height of the first tower.
(a) 110 m (b) 115.3 m
O
vertical distance between the two planes. (c) 113.8 m (d) 114 m
(a) 984 m (b) 1268 m
(c) 1543 m (d) 873 m 19. The angle of elevation of an aeroplane from
H
a point on the ground is 45°. After a flight
15. The height of a pole is 20 m. It is broken
for 15 seconds, the elevation changes to 30°.
by the wind in such a way that its top
_T

touches the ground and makes an angle If the aeroplane is flying at a height of
of 30º with the ground. Find the height 3000 m, find the speed of the aeroplane.
from the bottom to the point from where (a) 527 km/h (b) 432 km/h
C

the pole was broken. (c) 492 km/h (d) 500 km/h
1 5 20. The upper part of a tree is broken over
(a) 7 m (b) 4 m
PS

9 6 by wind and in the tilted position, it


2 3 makes an angle of 30° with the ground.
(c) 6 m (d) 5 m If the distance from the root to the point
3 2
16. As observed from the top of a light house, where the top of the tree meets the
U

100 m high above sea level, the angle of ground is 20/3 m, then the height of the
depression of a ship, sailing directly tree must be
@

towards it, changes from 30° to 45°. (a) 40 m (b) 60 m


Determine the distance travelled by the (c) 40 3 m (d) 80 m

Answers with Solutions


1. Ans. (a) 2. Ans. (c)
Sol. Let AB be the tree and AC be its Sol. Let AB be the tower. Then,
shadow. B
B ∠APB= 30° and AB = 100 m.
Let angle of elevation
AB 1
∠ACB = θ = tan 30° =
AC AP 3 100 m
Then, = 3 ⇒ AP = AB × 3
AB
⇒ cot θ = 3 = 100 3m 30°
P A
⇒ cot θ = cot 30° C A

⇒ θ = 30°. = (100 × 1.73) m = 173 m.


506 ✫ Quantitative Aptitude

3. Ans. (b) 32
Sol. Let CD be the tower and A and B be In ∠ACB , tan 30° =
x
the points of observation. Then, 1 32
=
CD CD h 3 x
= tan 60° = 3 BC = =
BC 3 3 x = 32 3 m
CD 1 Hence, the distance of the tower from
= tan 30° = the house is 32 3 m.
AC 3
6. Ans. (c)
AC = CD × 3=h 3 Sol.

TS
h D
AB = (AC – BC) = h 3 − B
3 D
h
∴ 20 = h 3 −

H
3 h
3h − h h
20 = h
3 60º

G
30º
A C
2h 30° 60° 100 m P x
20 = A
3 20 B x C

h=
20 3
2
= 10 3 m
U Let AB and CD be the two towers.
Let AP = 100 m and CP = x m
In ΔABP
O
4. Ans. (c) AB
tan 60° =
Sol. Let OP be the kite and OQ is the level AP
H
h
ground.  3=
50
Then ∠POQ = 60° and OP = 150 m.
_T

h = 50 3 m
Let PQ = h metres
In ΔCPD
PQ CD
Now = sin 60° P
OP tan 30° =
C

CP
h 3 1 50 3
=  =
PS

150 2 3 x
3 150 h
 h = 150 × x = 50 3 × 3
2
h = 75 3 = 150 m
O 60°
U

Q ∴ AC = Distance between the towers


= 75 × 1.732
= 50 + 150
= 129.9m. = 200 m
@

7. Ans. (a)
5. Ans. (b) D Sol. Let AB be the hill and C, D be the two
Sol. stones
B
30° 60°
45°
C E
30°
h
House
32m

Tower
30° 60° 30°
A x B A x km C 1 km D
Let AB = x = distance between the In ΔABC
tower and the house AB
AC = 32 m tan 60° =  h = 3x
AC
Unit Seven : Practice Session ✫ 507

In ΔABD Then, ∠TAR = 60°, ∠TBR = 30° and


AB 1 h AB = 40 m.
tan 30° =  = Let TR = h metres and AT = x metres
AD 3 x +1
h Then, BT = (40 + x) metres
3h = x + 1 = +1 TR
3 = tan 30°
1 BT
 3− h = 1
3 h 1
 =
1 1 1 40 + x 3
or h = = =
1 3−1 2 40 + x

TS
3−  h = ... (i)
3 3 3 3
3 TR
= km = tan 60°
2 AT
3

H
∴ Height of the hill is km . h
2  = 3
8. Ans. (b) x

G
Sol. D h = 3x ... (ii)
R

U h
O
100
30° 60°
H
B 40 A x T
45° 60°
A B x C From (i) and (ii) we get
y
40 + x
_T

= 3x
Let BC = x and AC = y 3
Then CD = 100 m 40 + x = 3x
CD 100 2x = 40
C

In Δ BDC, tan 60° = = x = 20 m.


BC x
100 10. Ans. (b)
3=
PS

x Sol. Let AB be the tree broken at a point C


100 and part CB, taking the position CB',
 x =
meet the ground at B’.
3
U

CD 100 In Δ B’AC
In Δ ACD, tan 45° = =
100 AC y AC
1 = = tan 30°
@

y AB'
x
 y = 100 = tan 30°
30
∴ AB = y – x
x 1 1
100 3 − 100 = B  = 0.5774
100 3
 100 − = 30 3
3 3
y
100( 3 − 1)
∴ AB = m.
3
9. Ans. (c) C
Sol. Let TR be the tree and AT be the
river. y x
Let A and B be the two positions of 30°
B' A
the person. 30m
508 ✫ Quantitative Aptitude

x = 30 × 0.5774 13. Ans. (a)


D
= 17.322 m Sol.
B’C tank
Also = sec 30°
AB’ B
y

Tower
sec 30
30 h
2 45°
= = 1.1547 A 30° C
3 40m
y = 30 × 1.1547 = 34.641 m BC

TS
∴ Height of the tree In ΔACB, = tan 30°
AC
= (x + y) m
= (17.322+34.641) m h 1 40 40 3
 = h m
= 51.96 m. 40 3 3 3

H
11. Ans. (a) 40 3
Height of the tower is
Sol. Let AB be the wall and AC be the 3
CD

G
ladder. In ΔACD , = tan 45° = 1
Then, ∠ACB = 60°, AC
AB = h, BC = 2 CD = AC = 40 m
A
U CD = 40 m
O
Depth of tank = CD – BC
h
r

40 3
de

40 −
H
d

=
la

wall 3
= (40 – 23.1) m
_T

60° = 16.9 m approximate.


C B
2m
14. Ans. (b)
In ΔABC Sol. AP = 3000 m, ∠AOQ = 45° and
AB h ∠AOP = 60°
C

= = tan 60°
BC 2 In ΔQAO
PS

h AQ
= 3 = tan 45° = 1
2 AO
h = 2 3m P
U

12. Ans. (d)


3000 m

Sol. In ΔABC Q
@

AB = 12 m, AC = l metre
A
pole

rope 60° 45°


l A O
12m
AQ = AO ...(i)
30° In ΔPAO
B C
AB 12 PA
= = sin 30° = tan 60° = 3
AC l AO
3000
12 1 = 3
= AO
l 2
3000
l = 12 × 2 or AO = ...(ii)
l = 24 m 3
Unit Seven : Practice Session ✫ 509

From (i) and (ii) we get, d+k = 100 3


3000 3 d + 100 = 100 3
AQ = × = 1000 3
3 3  d = 100 3 – 100
= 1000 × 1.732 = 1732 m d = 100( 3 − 1)
or PQ = AP – AQ = (3000 – 1732) m = 100 (1.732 – 1)
= 1268 m = 100 (0.732)
Required distance = 1268 m. = 73.2 m (approximate)
15. Ans. (c) 17. Ans. (a)

TS
Sol. Let AB be the tree bent at the point C Sol. In ΔOMP
so that part CB takes the position CD.
Then, h
= tan 60° = 3
BC= (20 – h) m d

H
CD = BC = (20 – h) m
 h = d 3 P ...(i)
B

G
h (tree)
U
O
C 20 m
h 30° 60°
- A
20 20m O d M
H
30° In ΔAMP
D A
In ΔADC h 1
= tan 30° =
_T

h 1 20 + d 3
20 − h
= sin 30° =
2 20 + d
 h = ...(ii)
2h = 20 – h 3
C

2h + h = 20 From (i) and (ii) we have


20 + d
3h = 20 =d 3
PS

20 2 3
h = = 6 m 3d = 20 + d  d = 10
3 3
2 Thus h = d 3 = 10 3
∴ Required height = 6 m. = 10 × 1.73 = 17.3
U

3 Height of the tree is 17.3 m and width


16. Ans. (c) of the river is 10 m.
@

Sol. O 18. Ans. (c)


30°
45° Sol. In ΔARP
C A
30°
100m

P 30° R
80m
45° 30°
h (tower)

C k B d A 160m
k
In ΔOCB , = cot 45° = 1
100
k = 100
Q B
In ΔOCA
AR
d+k = tan 30°
= cot 30° = 3 PR
100
510 ✫ Quantitative Aptitude

AB – BR 1 3000 3 = 3000 + CD
=
80 3 CD = 3000 ( 3 − 1)
160 − h 1 3 = 3000 (0.732)
= =
80 3 3 = 2196 m
3 i.e., Distance covered by the aeroplane
160 – h = × 80 in 15 seconds = 2196 m
3
1.732 × 80 ∴ Speed of the aeroplane
=
3 2196 60 × 60
= ×

TS
= 46.2 15 1000
= 527.04 km/h
h = 160 – 46.2
= 113.8 m 20. Ans. (b)

H
19. Ans. (a) Sol.
Sol. CA = DB = 3000 m y

G
AC
In ΔACO = tan 45° A
OC
3000 y

OC
=1
A B
U x

30°
O
B C
20 3
3000 m

H
In ΔABC,
x 1
= tan 30° =
_T

30° 20 3 3
45°
O C D ∴ x = 20 m
 OC = 300 m y 2
BD = sec 30° =
20 3 3
C

In ΔBDO = tan 30°


OD y = 40
3000 1
∴ Height of the tree = x + y
PS

= 20 + 40 = 60
OC + CD 3
U
@
Unit Eight : Fundamentals / Data Analysis: Tables, Graphs and Diagrams ✫ 511

UNIT EIGHT
DATA ANALYSIS: TABLES, GRAPHS AND DIAGRAMS ✫ PASSAGES

FUNDAMENTALS AND

TS
WORKED EXAMPLES

H
Basic Concepts and Tools it is possible to express in figures certain
Data may be in the form of tables, charts, factors related to this phenomenon of

G
diagrams or just statements. Analysis and honesty and thus study it in a statistical
light. A study of the number of thefts or
interpreting data calls for careful reading,
understanding, and organising of facts and
figures, doing the required calculations and then U cases of cheating or swindling can indirectly
tell us something of the problem.
(ii) Statistics does not study individuals.
O
reaching meaningful conclusions. Statistics deals with aggregates. This is a
Most questions requiring interpretation of limitation, too. If 100 persons die due to
H
data call for a basic knowledge of statistical starvation in a country with a huge
tools. Principles of calculating percentages, ratios, population, the percentage would work
rates and averages also need to be kept in mind. out to a practically negligible figure:
_T

Statistical Methods statistically we would be justified in


ignoring it; but on an individual level it
Statistics is a body of methods for collecting,
would not lessen the horror of death and
organising, summarising, presenting and
C

its circumstances.
analysing data, as well as drawing valid (iii) Statistical laws are true only on an average.
conclusions and making reasonable deductions. Laws of statistics are not universally true
PS

On the basis of such analysis, statistics is a like the laws of physics or astronomy. If we
method of handling data. Statistical methods say the per capita income of a certain
range from a very simple set of devices to highly country is 500 dollars, we cannot conclude
complicated and complex mathematical
U

that each person in that country gets an


procedures which can be used only by those income of 500 dollars, for it is merely an
who have received adequate training in these average figure. Since a multiplicity of factors
@

areas. For our purpose, the simple set of devices affect a phenomenon studied statistically
is adequate. and since it is not possible to study the
effect of each factor separately (as is done
under experimental methods), the
Points to Note
conclusions arrived at are not accurate. It
Limitations Statistics is not a precise is possible to arrive at slightly different
science and its limitations must be kept in mind conclusions with the same data at different
while assessing the validity of conclusions times. Here we talk in terms of probability
drawn from a study or investigation. and not certainty.
(i) Statistics does not study qualitative (iv) Statistics is only one method of studying
phenomena directly. Honesty, for instance, a problem. It does not tell the complete
cannot be measured in figures and, story. Factors like religion, culture and
therefore, in a study of honesty, statistical mental attitudes which affect human actions
methods cannot be of much use. However, do not come in the field of statistics.

511
512 ✫ Quantitative Aptitude

(v) The data must be sufficient to study the For example, production of wheat in country
salient features of the variable under A is 4,50,600 tons and in B, it is 2,00,000 tons.
investigation. To arrive at the conclusion We can say that the ratio of production in these
that the crime rate has increased merely countries is
because the number of convicts in jails 450600 : 200000
has increased is obviously reaching a The ratio is 4.5 : 2 approximately.
conclusion on inadequate data.
(vi) An alteration in the definition of a key [In most questions, approximate figures will be
term could give a wrong conclusion. If sufficient to get the correct answer.]

TS
two different definitions of ‘wages’ are ● If you have to evaluate a ratio where the
used by different parties, the same figures numerator is very much less than the
can be used to arrive at opposite 4
conclusions to suit each party’s ends. denominator, such as , reverse the
283

H
(vii) Statistics is certainly concerned with operation and divide the denominator by
making comparisons. But some the numerator.

G
comparisons are inaccurate or 283 ÷ 4 = 70 approximately
inappropriate. The statement that the 1
“incidence of death among sick persons So the given ratio is or 1 : 70.
is higher in hospitals than at home” is
likely to lead to the conclusion that U 70
[Don’t forget that 4 ÷ 4 is 1—which is the
first part of the ratio.]
O
hospitals are not the right place for sick
persons—obviously erroneous if we ● If the numerator is a decimal fraction much
less than the denominator, approximate it to
H
remember that it is generally only
seriously ill persons who are hospitalised. the nearest fraction having integers only
Sometimes a certain degree of correlation and then reverse the operation.
_T

may be apparent from a set of figures 12.27 12 12.62


Thus would be and would
when actually no such relationship exists. 432 432 432
For example, if it was found that several 13
be and then apply the reverse division
C

high income persons took size ten in 432


shoes, it would be wrong to conclude a 12.27
to get approximate ratio. would be
PS

correlation between shoe size and income 432


level. 12.62
approximately 1 : 36 and would be
432
Analysis: Statistical Tools approximately 1 : 33.
U

Besides brushing up the ways of adding,


subtracting, multiplying and dividing numbers ● If the number in the numerator and
denominator are more than two, but the
@

quickly, you require to master the use of tools


such as ratios, percentages, averages, dispersion same number of digits, say 4632 , take the
and so on. Figures have to be related to one 2124
another before a meaningful conclusion can be first two digits to form the approximate
derived. 46
fraction as and divide to get the
Ratio 21
approximate ratio 2 : 1.
Ratios make comparisons of one magnitude with
another as multiples or as fractions and thus aid
Percentages
in the interpretation of statistical data. They are
one of the simplest and most commonly used Ratios or proportions with the base 100, that is
basic techniques in statistics. quantity per 100, are more easily understood
In the simplest possible form, a ratio is a and compared. For example, the ratio of
quotient or the numerical quantity obtained by scheduled castes to the total population in
dividing one figure by another. Kerala and Bihar in 1991 can be expressed as:
Unit Eight : Fundamentals / Data Analysis: Tables, Graphs and Diagrams ✫ 513

2549000 Original (or base) value


SC ratio in Kerala = × 100
29011237 Value after x % increase or decrease
=
= 8.78% x

10142000 100
SC ratio in Bihar = × 100
86338853 (+x if increase; –x if decrease)
= 11.75%
Comparison reveals that the SC ratio is
Points to Note
favourable in Bihar.
● Size and Types of Bases The difference

TS
Rates between ratios, rates, percentages and co-
A ratio between two magnitudes usually shown efficients and rate per unit is only in the
over a period of time is called a rate or rate of base on which they are calculated; otherwise
they all give a relative picture of two

H
change. This is a special kind of ratio which
interrelated phenomena.
measures the rate of change over a period of
Various types of bases can be used for
time.

G
the computation of ratios.
For example: If a man driving at a speed of
Total to total If one group of figures as
40 km per hour increases the speed to 50 km per
hour, then clearly he has increased his speed by,
50 − 40
× 100 = 25% U a whole is compared with another group,
the base of the ratio would be the total of
one of the groups. Per capita income, for
O
40 example, is calculated by dividing total
Now if he reduces the speed once more income by total population.
to 40 km per hour he has changed his speed
H
Part to total We may compare a part to
by, its whole, males or females as a proportion
40 − 50 of the total population or workforce, for
_T

× 100 = − 20%
50 example. In such cases the base of the ratio
or reduced the speed by 20%. is usually the value of the universe (or the
(i) Generally we may express percentage given total). If these ratios are expressed as
C

rate of change as percentages, the total of these ratios would


x −x add up to 100 per cent.
Percentage rate of change = 1 0 × 100
PS

x0 Part to part We may compare part to


Where x1 = the changed or new value part within a whole. We may compare the
x0 = the original value. export earnings from jute, coffee,
handicrafts, etc. We may express the value
U

If the result is positive, the rate of change of coffee exports as a ratio of the value of
is positive or the new value is greater than jute exports. However, a better comparison
the original value; if negative, the rate of would be if export earnings in both these
@

change is negative or the new value is less items are expressed as a percentage of total
than the original value. exports.
We could have expressed the new car Standard area and units Sometimes the
50 base of the ratio is a standard area—
speed as × 100 = 125% of the original population per square kilometre; standard
40
speed and after slowing down, the new distance—cost of railway line per kilometre;
40 a standard unit—students per school, rooms
speed as × 100 = 80% of the old speed. per house.
50
It is important to note that the denominator Arbitrary ratios In many enquiries it is
part contains the value with which the possible to use arbitrary units; examples of
comparison is made. such arbitrary units are horse power, light-
(ii) If the original value is to be found when year, ton-kilometre. The most common
the value after its x% increase or decrease arbitrary units are 1, 10, 100, 1000; among
is known, use the formula, these 100 (or per cent) is the most popular.
514 ✫ Quantitative Aptitude

● Comparison in Comparable Units It is Average


necessary that the two figures compared When making comparisons it is obviously useful
should have the same characteristics and to have a single measure that is representative
should be expressed either in the same unit or typical of the distribution. Such measures are
or in comparable units. Production of cotton referred to as measures of central tendency, measures
in bales can be compared to consumption of of location or simply as averages.
cotton in bales but we cannot calculate ratios The basic concepts on ‘averages’ have
between production of wheat in tons and already been discussed in an earlier unit. Here
production of cotton in bales. However, in we will look at the topic from the point of view

TS
many cases comparison has to be done of interpreting data.
between items which are expressed in The arithmetic mean (A.M.) is the average
different units. The common denominator of a set of numbers. Suppose the number of visits
in such cases is a number or quantity. Thus made by ten mothers to a clinic were

H
in comparing total income with total 8 6 5 5 7 4 5 9 7 4
population we really divide the number of If you were asked to calculate the average

G
rupees representing total income by the number of visits, what would you do? Probably
number of persons representing the total you would add together the number of visits,
population. In comparing the number of making a total of 60, and divide by the number
kilometres done with a litre of petrol,
though the units are different, it is the U of mothers, giving 6 as the average number of
visits. This particular type of average is known
O
number of kilometres which is compared as the arithmetic mean. It is calculated by totalling
with the number of litres. the values of the observations and dividing that
H
● Two percentage values cannot be compared total by the number of observations.
unless their base values are known. If The formula is
exports of cotton in 2012 formed 25 per cent x
_T

of the production in that year and in 2013 x =


n
exports of cotton was 30 per cent of the
production in that year, one cannot make The symbol x (pronounced x-bar) stands
for the arithmetic mean or average.
C

any conclusion about which year export


was more (or less), unless the absolute The symbol ∑ stands for summation:
PS

value—amount of production—in 2012 and whenever you see this symbol in a formula it
means ‘the total of’. ( ∑ is a Greek capital letter
2013 is known.
● Two percentage values can, however, be
pronounced sigma.) The letter x refers to the
compared as percentage values though not
U

value of an observation, and n is the number


in absolute values, if given that the absolute
of observations.
values or base is the same (though absolute
@

values are not known): if cotton exports


form 25% of cotton production in 2012 and Points to Note
30% of production in 2013, if it is given that
production is the same in both years, ● If the average of values of any item over
comparison of percentage values of export a period of time has to be found, the sum
is valid. of the values is divided by the number of
Increase in export value in 2013 over 2012 units of time.
30 − 25 For example, if production of cotton is x
is × 100 = 20%.
25 bales in n years, the average annual
Export value in 2012 is different from that x
25 − 30 production is .
in 2013 by × 100 = –16.66%. n
30 If x bales are produced in n months, the
(The minus sign indicates decrease or x
average is per month.
‘less than’.) n
Unit Eight : Fundamentals / Data Analysis: Tables, Graphs and Diagrams ✫ 515

● If the average percentage change in the The formula for the arithmetic mean
value of an item over a given period of calculated from a frequency distribution has
time has to be found, use the formula as to be amended to include the frequency. It
needed: becomes:
Average percentage change
fx
=
Percentage change during the given time x =
Number of years (or given unit of time) f
or number of items In this case,
● Weighted Average Value Suppose we have fx

TS
710
to find the A.M. of a frequency x = = = 7.1
distribution. Consider the following table: f 100
The frequencies are also the weights, and
Numbers of visits made to clinic the average is thus the weighted average.

H
by 100 mothers Some questions indicate the ‘weights’
Number of visits Number of mothers attached to the item (as in the subjects in a

G
(variable) (frequency) particular examination carrying weights of 2,
3 and so on) and these ‘weights’ are to be
4 8
5
6
12
15 U
treated as the frequencies, and the same formula
applied to get the correct average. The weights
O
or frequencies have to be considered in the
7 25
value of each variable or item.
8 17
H
9 13
10 10 Interpretation of Tables
_T

Total 100 Quantitative data is often presented in a tabular


form. In a simple form, just one characteristic of
Each value of the variable occurs more than the data is presented, while in multiple tables,
once. To find the sum of the number of visits a number of characteristics of the data are
C

made by all the mothers we multiply each value presented for example, a table showing the
of the variable by the frequency with which number of students in different classes in a
PS

it occurs. We get: school.


The key to understanding tables is to read
Frequency distribution and sum of visits the title and the stubs and footnotes, if any;
made by 100 mothers to the clinic these will tell you what the numbers in the table
U

mean.
Number of visits Number of mothers Sum of
The title (placed above the table) should
@

(variable) (frequency) visits


indicate the nature of the data presented, the
x f fx
areas covered, and the time period if required.
4 8 32 The stub is the section containing the row
5 12 60 headings (horizontally arranged data) generally
6 15 90 indicating the different time periods, type/name
7 25 175 of the groups, and other things to elucidate the
8 17 136 title. Columns (vertically arranged data) have
9 13 117 headings referring to the time period, name/
10 10 100 type of the groups of data or other elements
Total 100 710 relevant to the title. Footnotes below the table
may be appended to elucidate the data if
The arithmetic mean is found by dividing necessary. A source may be given below the
the total number of visits made by all the footnotes, indicating from where the data has
mothers (710) by the number of mothers (100). been taken.
516 ✫ Quantitative Aptitude

Analysing and interpreting the data calls for Worked Examples


an assessment of how the numbers in the context ● A student obtained the following marks in
of the stub and column headings of the table are a test.
inter-related.
Subject Marks obtained Maximum marks
Points to Note Physics 60 100
Chemistry 80 100
(i) Look at the table as a whole and try to Mathematics 80 100
grasp what it is trying to convey. What are Hindi 40 50

TS
the items, what are the totals, what is the English 40 50
time period?
(ii) Be alert to the units which are used in a What is the aggregate percentage marks
table—which may be different from the obtained by the student?

H
units asked for in the problem. If a table (a) 50 per cent (b) 60 per cent
is listing the production of agricultural (c) 75 per cent (d) 80 per cent

G
crops, units may easily differ—wheat
First total (i) the marks obtained and (ii)
would be in tons, but cotton in bales. So
just going by the numbers may lead you the maximum marks. You have
to an inaccurate answer.
(iii) The questions generally relate to annual U Marks obtained
Maximum marks = 400
= 300
O
rate of increase, maximum/minimum Now if from 400, marks obtained are 300,
percentage increase/decrease, maximum/ from 100, they will be:
300
H
minimum ratio of any two parameters, or × 100 = 75
the trend of movement of a variable. Simple 400
arithmetical computation should help you which is the aggregate percentage.
_T

arrive at the correct answer. So, option (c) is correct.


(iv) It is a good idea to estimate the numbers [The question should be attempted
that you have to use from a table rather mentally, with no written calculations.]
C

than use them in their printed form. ● The following table shows data on the
Usually, the first and second digit are all weight of various organs of different
that is needed. If, for instance, you need
PS

animals on the basis of (1) absolute weight


to find the ratio of 3625 to 1205, just find of the organ and (2) its percentage of total
the ratio of 36 to 12, and you get a close body weight.
approximation to the answer.
U

(v) Generally, the questions ask for Ani- Brain Heart Liver Spleen
approximate figures, and often enough mal (1) (2) (1) (2) (1) (2) (1) (2)
the responses in the multiple choice
@

A. 650 g 1.0 3.4 kg 0.7 5.0 kg 1.5 1.2 kg 0.16


questions are also approximate figures. B. 120 g 0.9 45 g 0.4 2.0 kg 1.7 424 g 0.12
So it is necessary to first consider if a C. 130 g 1.1 2.4 kg 0.4 700 g 1.5 120 g 0.17
question can be answered without actual
D. 450 g 1.2 2.5 kg 0.5 0.5 kg 1.2 1.1 kg 0.17
calculations and just mental calculations
could do.
1. Which animal has the heaviest brain
(vi) All questions, however, cannot be
and heaviest heart?
answered without calculations. But even
(a) D (b) C
here, you need to be alert to possibilities
(c) B (d) A
of fast-tracking, of using shortcuts, and of
knowing when to stop calculating and 2. The heaviest liver and lightest heart
begin to estimate the answer. Avoid as a percentage of body weight is
unnecessary calculations, for time is of the found in
essence. (a) B (b) A
(c) D (d) C
Unit Eight : Fundamentals / Data Analysis: Tables, Graphs and Diagrams ✫ 517

Solutions The data can be easily read to Now you have to find the percentage,
arrive at the answers. Just be careful of which is
the units, and do not let the plethora of 34
numbers confuse you. × 100 = 52.3%
65
1. Look for the animal with the heaviest (b) Number of employees earning less
brain: it is A. Does it also have the than Rs 100.00 but at least Rs 60.00
heaviest heart? Yes, its heart weighs per month
3.4 kg. (Don’t go by numbers alone = 10 + 16 + 14 + 10 = 50.
or you may think it is B—but it is Percentage of employees earning less

TS
45 grams, not kg.) than Rs 100.00 but at least Rs 60.00
You can choose (d). per month
2. Column 2 for each animal gives the 50
percentage of organ to body weight. = = 76.9%.
65

H
You have to look in the columns for
● Study the following table and answer the
liver first—the heaviest liver to body
question below it.

G
weight per cent is that of B. Now
look for the lightest heart to body Admission Figures for a University in Arts
weight per cent in the heart column: and Science Faculties (1986 to 1988)
both B and C have the same
percentage. But in combination with U Student
Arts
1986

Sc. Total Arts


1987

Sc. Total Arts


1988

Sc. Total
O
the heaviest liver to body weight
percentage, it is B. Males 4,805 695 5,500 5,030 870 5,900 5,362 1,128 6,490
H
So (a) is your answer. Females 675 325 1,000 724 396 1,120 759 473 1,232

● Table shows a frequency distribution of Total 5,480 1,020 6,500 5,754 1,266 7,020 6,121 1,601 7,722
_T

the monthly wages in Rs of 65 employees What is the percentage increase of male


in the P and R company. students in 1988 as compared to 1986?
(a) 99 (b) 18
Wages (Rs) Number of Employees
C

(c) 50 (d) 10
50.00 — 59.99 8
Solution A look at the table tells you that
60.00 — 69.99 10
PS

there are several subtotals and totals. You


70.00 — 79.99 16
need to go to the correct year and correct
80.00 — 89.99 14
totals. Note the years—1986 and 1988, so
90.00 — 99.99 10
skip the data for 1987.
U

100.00 — 109.99 5
110.00 — 119.99 2 First find the increase in male students: the
figures you need are total male students in
@

Total 65
1986 and total male students in 1988; these
What is are 5500 and 6490 respectively. Now find
the difference between the two figures:
(a) The percentage of employees earning
less than Rs 80.00 per month. 6490 – 5500 = 990
(b) The percentage of employees earning To find the percentage increase, we will
less than Rs 100.00 but at least Rs use as base the total male students in the
60.00 per month. earlier year, i.e., 1986. So,
Solutions First find the total number of 990
Percentage increase = × 100 = 18%
employees in the required range of wages. 5500
(a) Those earning less than Rs 80 per Answer response (b) is correct.
month will be in the wage classes
Rs 50 to Rs 79.99. This adds up to 16 [You have to be careful in the choice of the
+ 10 + 8 = 34. base while finding the percentage.]
518 ✫ Quantitative Aptitude

● Table showing the number of workers of a 1. In which bank did the number of clerks
factory in the years 2000 and 2005. recruited continuously increase during the
Year 2000 Year 2005 years 2004 to 2009?
Workers Union Non- Total Union Non- Total (a) Only D and E (b) Only B
Members union Members union (c) Only E (d) Only C and D
Members Members (e) None of these
Male 1175 375 1550 1290 180 1470
Female 25 175 200 220 28 248
2. Total number of officers recruited by all the
banks together in the year 2004 was
Total 1200 550 1750 1510 208 1718
approximately what percentage of the total

TS
1. What is the increase in the percentage number of clerks recruited by Bank B in the
of the female workers belonging to the years 2008 and 2009 together?
union in the five-year period? (a) 112 (b) 77
(a) 78% (b) 195% (c) 93 (d) 127

H
(c) 780% (d) 700% (e) 102
2. In 2000, the ratio of male workers to 3. In which bank is the total number of officers

G
female workers in the factory was and clerks recruited together in the year 2008
(a) 21 : 9 (b) 0.88 : 0.11 the second highest?
(c) 31 : 4 (d) 9 : 1
Solution 1. The five-year period is 2000-
U (a) E
(c) B
(b) C
(d) A
O
2005. In 2000, the female workers who (e) D
belonged to the union were 25. In 2005, 4. What was the ratio of the total number of
the figure went up to 220. The increase is
H
officers recruited by Banks C and D together
220 – 25 = 195. in the year 2006 to the number of clerks
195 recruited by Bank A in the year 2009?
_T

Percentage increase is × 100 = 780%


25 (a) 31 : 18 (b) 31 : 17
(The base is the figure of female union (c) 18 : 35 (d) 19 : 11
members in the earlier year.) (e) None of these
C

Answer response (c) is correct.


5. What was the average number of officers
2. The figures involved are the total
recruited by Bank C over all the years?
men workers and total female workers in
PS

2000, which is 1550 and 200 respectively. (a) 3,000 (b) 5,000
Their ratio is (c) 30,000 (d) 5,500
1550 : 200 = 31 : 4 (e) None of these
U

Answer response (c) is correct. Solutions Take a good look at the table
● Study the following table carefully to answer caption and the data. Be careful to take the
@

the questions that follow: figures for the category and year/years
Number of Clerks and Officers recruited (in asked for in each question. Unless the ‘is
thousands) by five different banks during thousands’ in relevant, it may be ignored in
six different years the calculations.
Bank 1. No calculations are required. Run your
A B C D E eye down the column for clerks for each
Year Clerk Offi- Clerk Offi- Clerk Offi- Clerk Offi- Clerk Offi- bank. Only E shows continuous increase.
cer cer cer cer cer So mark (c) as your answer.
2004 4.2 1.6 3.6 2.2 5.3 3.8 6.9 5.6 7.7 6.9 2. First total the number of officers in all
2005 5.2 2.7 5.8 4.6 6.9 4.0 7.0 3.8 8.8 4.4 banks for 2004.
2006 3.3 3.9 4.9 2.8 6.4 4.6 9.5 7.8 9.9 7.9 (1.6 + 2.2 + 3.8 + 5.6 + 6.9) thousand
2007 5.7 4.1 5.8 5.6 7.9 5.7 8.6 6.3 11.5 9.8
= (20.1) thousand
2008 6.9 3.9 6.9 5.7 5.8 6.4 11.7 9.8 14.8 11.3
= 20100
2009 7.2 5.5 8.9 5.1 6.8 5.5 10.6 8.9 15.9 10.7
Now total the number of clerks recruited
Unit Eight : Fundamentals / Data Analysis: Tables, Graphs and Diagrams ✫ 519

by Bank B in the years 2008 and 2009 Hence, the total number of officers and
together clerks recruited together in the year
= (6.9 + 8.9) thousand 2008 is the second highest in Bank D.
= (15.8) thousand But this long method may be avoided.]
= 15800
20100 4. Once again, we may ignore the ‘in
Required percentage = × 100
15800 thousands’ which may be taken for
≈ 127%. granted and just concentrate on the
Mark (d) as the correct option. numbers as given in the columns.

TS
[Fast-track A look at the answer Officers in Banks C and D for 2006
responses shows that the options are not = 4.6 + 7.8 = 12.4
close. So an estimate can be made. Instead Clerks in Bank A for 2009 = 7.2
of adding the actual number of officers Ratio of officers to clerks

H
and clerks, round off decimals and add. 12.4 3.1
Officers in all banks in 2004 = = which is 31 : 18.
7.2 1.8

G
2 + 2 + 4 + 6 + 7 = 21 (if both numerator and denominator are
Clerks in Bank B in 2008 + 2009 multiplied by 10.)
7 + 9 = 16
You get officers to clerks as 20 : 16
20 U Mark (a) as the answer.
5. Here the options are in thousands, so
O
approximately. Now × 100 is 125. that must be kept in mind.
16
The nearest option is (d), so it must be However, the initial calculations need
H
correct.] nor take that into account.
Here the average
3. A look at the data shows that the highest
_T

number of officers and clerks in 2008 is Total number of officers in Bank C


=
in Bank E (14.8 + 11.3) followed by Bank Number of years
D (11.7 + 9.8). We may ignore the units Rounding off the decimal fractions
C

and just approximate the results taking


4+4+5+6+6+6
just the integers : E 14 + 11 = 25;
6
PS

D 11 + 9 = 20.
So Bank D is second highest. 31 30
= ≈
Mark (e) as the correct answer. 6 6
[Long Method Total number of officers or 5.
U

and clerks recruited together Now recall the ‘thousands’: the total
by Bank A in the year 2008 number of officers is 5 × 1000 = 5000.
@

= (6.9 + 3.9) thousand Mark answer response (b).


= 10800 [You get the same answer if you do it
by Bank B, with decimals.
= (6.9 + 5.7) thousand Average number of officers recruited by
= 12600 Bank C over all the years
by Bank C,
= (5.8 + 6.4) thousand (3.8 + 4.0 + 4.6 + 5.7
= 12200 +6.4 + 5.5)
= thousands
by Bank D, 6
= (11.7 + 9.8) thousand 30000
= 21500 = = 5000.
6
by Bank E,
= (14.8 + 11.3) thousand ● Study the table carefully to answer the
= 26100 questions that follow.
520 ✫ Quantitative Aptitude

Number of Cell Phones (in thousands) and years, whether items being manufactured
Manufactured (M) and Sold (S) by Six or sold are referred to in each question.
Companies Over the Years 1. A look at the ‘M’ column for each company
Company
should lead you to the answer without
calculation. Note that Company F shows
Year A B C D E F highest manufacturing figures in 2003,
M S M S M S M S M S M S 2004, 2005 and 2006, and that too by
considerable margins compared to the
2001 34 28 30 21 28 19 25 18 31 27 21 16
other companies. So F is the leading
2002 41 35 40 32 36 27 32 27 42 34 40 32

TS
company, and your answer response is
2003 49 40 46 37 42 31 50 44 45 38 71 65
(d).
2004 55 45 58 49 48 39 73 62 68 51 92 80 However, you may want to be very sure.
2005 70 61 85 70 60 51 98 87 89 78 115 96 You can confirm the correctness of your

H
2006 108 98 120 111 75 63 108 96 115 100 135 118 answer by adding the ‘M’ figures for
each company for the first three years—
1. Which company has manufactured the

G
2001, 2002 and 2003, as from 2003
maximum number of cell phones over the
onwards, F clearly outdoes the others in
years?
manufacturing figures.
(a) A
(c) D
(e) None of these
(b) C
(d) F
U A
2001 2002 2003
34 + 41 + 49 = 124 thousand
O
B 30 + 40 + 46 = 116 thousand
2. What is the respective ratio of total number C 28 + 36 + 42 = 106 thousand
of cell phones manufactured by companies
H
D 25 + 32 + 50 = 107 thousand
A, B and C together in the year 2001 to those E 31 + 42 + 45 = 118 thousand
manufactured by companies D, E and F F 21 + 40 + 71 = 132 thousand
_T

together in the year 2003?


[Long method Number of cell phones
(a) 46 : 83 (b) 43 : 83
manufactured by A
(c) 43 : 86 (d) 46 : 89 = (34 + 41 + 49 + 55 + 70 + 108) × 1000
C

(e) None of these = 357000


3. What is the percentage of number of cell Number of cell phones manufactured
PS

phones sold by Company D in the year 2002 by B


to those manufactured by it in that year? = (30 + 40 + 46 + 58 + 85 + 120) × 1000
(Rounded-off to two digits after decimal) = 379000
(a) 94.48 (b) 84.38 Number of cell phones manufactured
U

(c) 82.58 (d) 76.88 by C


(e) None of these = (28 + 36 + 42 + 48 + 60 + 75) × 1000
@

= 289000
4. What is the total number of cell phones sold Number of cell phones manufactured
by Company B in all the years together? by D
(a) 386000 (b) 357000 = (25 + 32 + 50 + 73 + 98 + 108) × 1000
(c) 320000 (d) 379000 = 386000
(e) None of these Number of cell phones manufactured
5. What is the approximate per cent increase in by E
the number of cell phones sold by Company = (31 + 42 + 45 + 68 + 89 + 115) × 1000
E in the year 2005 from the previous year? = 390000
(a) 62 (b) 48 Number of cell phones manufactured
(c) 39 (d) 23 by F
= (21 + 40 + 71 + 92 + 115 + 135) × 1000
(e) 53
= 474000
Solutions Note the title and the units, and ∴ Company F has manufactured maximum
be careful to use correct data—the number number of cell phones.
Unit Eight : Fundamentals / Data Analysis: Tables, Graphs and Diagrams ✫ 521

2. Carefully note the years. ABC for 2001 Percentage of Income Generated by the Sale
and DEF for 2003. Just add the numbers. of Different Types of Foodgrains Produced
(As all of them have ‘000 attached, you by Six Different Farmers out of their Total
may ignore the ‘000.) Income in a Year
The ratio
Percentage of
A + B + C manufacture in 2001 Farmer Total Income Income Generated
=
D + E + F manufacture in 2003 (Rs in lakhs) Wheat Rice Bajra Jowar Maize

34 + 30 + 28 92 46 J 2.55 28 36 12 15 9
= = = or 46 : 83 K 2.00 27 26 16 13 18

TS
50 + 45 + 71 166 83 L 1.85 19 32 18 18 13
Answer response (a) is correct. M 1.75 23 29 21 17 10
N 2.15 14 19 26 23 18
3. In 2002, Company D made 32000 cell
O 2.40 16 14 27 21 22

H
phones and sold 27000. Once again, you
may ignore the ‘000 and calculate the 1. Which farmer has generated the maximum
percentage of ‘S’ to ‘M’. Just be careful

G
amount of income from the sale of rice?
to put ‘M’ value in the base as the (a) K (b) L
question asks for percentage of ‘sold’ to (c) M (d) N
‘manufactured’.
27 U (e) None of these
2. What is the total income generated by the
O
× 100 = 84.375 ≈ 84.38% sale of maize by all farmers together?
32
(You are asked to find the percentage (a) Rs 1,54,000 (b) Rs 1,92,000
H
rounded-off to two decimal digits.) (c) Rs 1,80,600 (d) Rs 1,67,500
(e) None of these
Answer response (b) is correct.
_T

3. What is the income generated by Farmer


4. Total number of cell phones sold by K from the sale of wheat and rice together?
Company B in all the years together (a) Rs 98,900 (b) Rs 1,15,000
= (21 + 32 + 37 + 49 + 70 + 111) × 1000 (c) Rs 1,06,000 (d) Rs 1,10,500
C

= 320000 (e) None of these


PS

Answer response (c) is correct. 4. What is the respective ratio of income


generated from the sale of jowar by Farmer
5. The previous year here is 2004 and the
N to that generated from the sale of jowar
Company is E. (Ignore the ‘000 as they
by Farmer O?
U

cancel out.) Just be careful to use the


(a) 989 : 1008 (b) 873 : 967
correct value in the base.
(c) 967 : 873 (d) 1008 : 989
Percentage increase in cell phones sold in
@

(e) None of these


2005 over 2004
Solutions Read the table title and column
‘S’ in 2005 – ‘S’ in 2004 captions carefully. Note the unit, but use it
= × 100
‘S’ in 2004 in your calculations only when you need to
78 − 51 do so.
= × 100
51 1. If a farmer has a total income of x and
27 y% of this income comes from the sale
= × 100 = 52.94% of a particular item, in absolute terms his
51 income from that item
This is approximately 53%. y
Answer response (e) is correct. = × x
100
Since the base 100 is common in all the
● Study the table carefully to answer the cases, it may be ignored and just y × x
questions that follow. will give you the required amount.
522 ✫ Quantitative Aptitude

So converting percentage into absolute [Long/Conventional method Total income


numbers in this case involves multiplying generated by the sale of maize by the
the percentage of income figure with the farmers
total income figure. Now, just a look at 9 18 13
the numbers will tell you that Farmer J = × 2.55 + × 2.00 + × 1.85
100 100 100
has the maximum percentage of income
10 18 22
generated from rice as well as highest + × 1.75 + × 2.15 + × 2.40 lakh
total income. 100 100 100
So multiplying the two figures we will = (0.2295 + 0.36 + 0.2405 + 0.175
+ 0.387 + 0.528) lakh

TS
get the maximum amount for Farmer J.
As J is not included among the options, = 1.92 lakh
your answer is (e). = Rs 192000.]
[Long method: To confirm try out the 3. Income generated by Farmer K by selling

H
long way. wheat and rice together
Income of Farmer J from the sale of rice 27 26
× 2.00 + × 2.00 lakh

G
36 = Rs
= × 2.55 = Rs 0.918 lakh 100 100
100 = Rs (0.54 + 0.52) lakh
Income of Farmer K from the sale of rice
=
26
× 2.00 = Rs 0.52 lakh U = Rs (1.06) lakh
= Rs 106000.
O
100 Answer response (c) is correct.
Income of Farmer L from the sale of rice
4. As only ratio is to be found, ignore 100
H
32
= × 1.85 = Rs 0.592 lakh base and lakh unit. Multiply the
100 corresponding values for percentage and
_T

Income of Farmer M from the sale of rice total income for N and O from jowar.
29
= × 1.75 = Rs 0.5075 lakh N 23 × 2.15
100 Ratio =
O 21 × 2.40
C

Income of Farmer N from the sale of rice


49.45 9.89
19 = =
= × 2.15 = Rs 0.4085 lakh 50.40 10.08
PS

100
This corresponds to answer response
Income of Farmer O from the sale of rice
(a).
14
= × 2.40 = Rs 0.336 lakh
U

100 Interpretation of Graphs


∴ Farmer J has generated the maximum of and Diagrams
@

income from the sale of rice.] Line Graphs


2. Multiply the numbers in the maize Graphs may be geometrical or statistical.
column with their corresponding Statistical graphs illustrate comparisons and
numbers in the total income column trends in given data. The fundamental idea
(ignoring lakhs and the base of 100) and about graphs is that they all use some distance
add the products. to represent value.
Total = (2.55 × 9) + (2 × 18) + (1.85 × 13) + A line graph is a pictorial representation of
(1.75 × 10) + (2.15 × 18) + (2.40 × 22) data and shows trends, generally over a period
= 22.95 + 36 + 24.05 + 17.5 + 38.7 + 52.8 of time. There may be more than one line, each
= 192 or 1.92 lakh. one representing a different item. A line that
Now remember the ‘Rs in lakhs’; the goes up shows increasing trend, one that goes
amount will be Rs 1.92 lakhs or Rs 192000. downwards indicates decline and one that is
Answer response (b) is correct. horizontal indicates ‘no change’. The line may
Unit Eight : Fundamentals / Data Analysis: Tables, Graphs and Diagrams ✫ 523

move up and down or remain horizontal or From the graph we can see that the value of
parallel to the X-axis, indicating the trends of imports has grown much higher than that of
increase, decrease or stagnancy. The highest exports in the period 1974-75 to 1980-81. It can
point on the line indicates the maximum of the be seen from the graph that imports registered
variable. a maximum growth in value in 1980-81 over the
preceding period, as the slope for that period is
The steepest slope indicates the portion/
maximum. It is also seen that value of exports
time period of maximum percentage change, shows a consistent though not uniform rising
either in the growth or in the fall of the value, trend—as there is no dip in the line graph.
depending on whether the slope is rising or

TS
declining. Diagrams (or Charts)
Two or more variables can be plotted on 1. Bar Diagrams Bar diagrams (also called
‘bar charts’ or ‘bar graphs’) are one-dimensional
a graph provided the unit of measurement is
diagrams in the shape of vertical or horizontal

H
the same. (A ‘variable’ is any characteristic that bars.
can vary for the population of individuals or The thickness of the bars is not relevant to

G
objects being analysed. For example, gender the data. (But in any one chart, the bars have the
and height are variables among people; value same width.) The simple bar diagram is used to
and quantity are variables among manufactured
goods or crops or trade.)
U represent only one variable. The values of the
variables are shown in the shape of bars which
are of equal width but of varying heights. The
O
One should read the labels, margins and
notes given carefully. height of the bar represents the value of a
The line graph below shows the movement variable, therefore it becomes easy to compare
H
of imports and exports of India in value over a the values of a variable. The bars are arranged
in time sequence or according to the size of the
certain period of time. The X-axis contains the
_T

variable.
time parameter and the Y-axis represents the The base may be on the vertical line also,
variable of value of exports in rupees. in which case the bars will be horizontally
placed.
C

Exports and Imports of India from


1974-75 to 1980-81 (Rs crores) A bar diagram can be adapted to compare
changes in more than one variable in which
PS

case it is termed a multiple or compound bar


diagram.
80
U
Value in Rupees Crore

70
Average daily earnings (Rs.)

Men 21+
@

60 Women 18+ (full time)


S

50
RT
PO

TS
OR
IM

P 40
EX
30

20

10

0
2000 2001 2002 2003 2004
Year
524 ✫ Quantitative Aptitude

Sub-divided Bar Diagram are also called 2. Pie Diagrams (or Charts) Two
component bar diagrams (or charts); these are dimensional diagrams are in the shape of figures
used to show the breakdown of a total into with two dimensions, that is, figures like
its component parts. rectangles, squares or circles. The areas of
In many cases, it is meaningless to compare
rectangles, square or circles are in proportion
absolute values, and it is better to employ
relative values. For this purpose, percentage to the size of the items they represent.
bar diagrams are employed to depict the Though a rectangle would appear similar
relative change in the values of a variable. to a bar diagram, in the latter case only the
height is taken into consideration, while in the
Component bar diagram showing

TS
former it is the area—length × width—which
families above poverty line
is taken into account.
(2001-2005)
The pie diagram is the most common of
Key two-dimensional diagrams.
Number of Families (thousands)

H
6 or more The term pie diagram has been derived
70 children
5 children from the word ‘pie’—the diagram looks like
60

G
4 children a pie and its parts, like the slices cut from a
3 children pie. A circle is sub-divided into various sectors
50 2 children
and the diagram shows the relationship of the
40

30
1 child

U Pie Diagrams Showing Expenditure on


Various Heads by Family A and Family B
O
20 Misc.
H
20°
10
Education
0 40°
_T

2001 2002 2003 2004 2005


Year Food
Clothing 160°
The percentage bar diagram tells us nothing
60°
about the absolute values—the total number of
C

families for each year—but shows the


breakdown into components as a proportion of Rent
PS

the whole. 80°


Percentage component bar diagram showing
families above poverty line in 2001-2005
U

100 Key
90 360° = Rs 10,000
@

80
Misc.
70 30%
Percentage of families

60 Education Food
50 50% 120%
40
Clothing
30 60%
Rent
20
100%
10

0
2001 2002 2003 2004 2005
Year
Unit Eight : Fundamentals / Data Analysis: Tables, Graphs and Diagrams ✫ 525

parts to the whole. A pie diagram may be Number of Trees planted by three different
defined as a circle divided into sectors by radii, NGOs in five different States
each sector corresponding in area, arc and
NGO-A NGO-B NGO-C
angle formed by the radii to the number of 200
frequencies or elements represented. 180
The angle at the centre of the circle is 360° 160

NUMBER OF TREES
which represents the total (or 100%). This angle 140
is sub-divided depending upon the nature of 120

TS
data to be represented; 1 per cent is equal to 100
1 80
3.6° and 1° = %.
3.6 60
This formula is necessary to remember, for 40

H
in some pie diagrams, values of different elements 20
are given in terms of degrees instead of per cent. 0

G
Bihar Punjab Haryana Assam Tamil Nadu
However, unless it is specifically asked in the
STATE
problem, do not convert from degree to per cent
or vice versa.
The pie chart below represents the animal U 1. In which state is the total number of trees
planted by NGO–A and NGO–B together
second lowest?
O
agricultural production of a state. (a) Bihar (b) Punjab
(c) Haryana (d) Assam
H
(e) Tamil Nadu
Sugar 2. What is the difference between the number
_T

Wheat of trees planted by NGO–A in Haryana and


130° the number of trees planted by NGO–C in
Tamil Nadu?
110° (a) 90 (b) 60
C

(c) 120 (d) 100


Rice (e) None of these
PS

Millet
3. What was the average number of trees
planted in Haryana by all the NGOs together?
(a) 420 (b) 140
If you are told that sugar production is 3000
U

(c) 120 (d) 390


tons, you can find out the total production of all
(e) None of these
the components.
@

4. Total number of trees planted by NGO–A and


Value of sector NGO–B together in Bihar is approximately
Total = × 360
Angle of sector what per cent of the total number of trees
planted by NGO–B and NGO–C together in
3000 Punjab?
= × 360 = 12000 tons
90 (a) 85 (b) 90
You can also find out the production of any (c) 105 (d) 110
of the other components. (e) 95
5. What is the ratio between the number of
Worked Examples trees planted by NGO–B in Tamil Nadu,
number of trees planted by NGO–C in Assam
● Study the following graph carefully to an- and the number of trees planted by NGO–A
swer the questions that follow: in Assam?
526 ✫ Quantitative Aptitude

(a) 5 : 3 : 6 (b) 5 : 6 : 3 Total number of trees planted by NGO–


(c) 6 : 4 : 5 (d) 6 : 5 : 3 B and NGO–C together in Punjab
(e) None of these = 80 + 88 = 168
Solutions Note carefully the different types ∴ Required percentage
of lines representing each NGO. In each 160
question you must carefully locate the line = × 100%
representing the NGO as demanded by the 168
question. Otherwise your calculations and = 95.23%
answer will be wrong. Look at the graph and ≈ 95% (approximate)

TS
grasp the basic essentials. Answer response (e) is correct.
1. A look at the graph ought to tell you that 5. Ratio between number of trees planted by
the lowest number is in Bihar, as other NGO–B in Tamil Nadu, number of trees

H
points on the graph for A and B are against planted by NGO–C in Assam and the
higher numbers on the Y-axis for both A number of trees planted by NGO–A in
Assam

G
and B. But you are required to find the
‘second’ lowest. In Bihar A + B = 100 + 60 = 180 : 120 : 150
= 160. In Punjab A + B = 120 + 80 = 200. = 18 : 12 : 15
As the line for A dips for Haryana, check
for Haryana as well: A + B = 80 + 140 = U =6:4:5
Answer response (c) is correct.
O
220. For Assam and Tamil Nadu, the
points are much higher for both A and B. ● Study the graph carefully to answer the
H
So Punjab is the second lowest in trees questions that follow.
planted by NGO–A and NGO–B together. Per cent Increase in Number of Students
(All this should be done mentally.)
_T

Studying in Colleges A and B over the Years


Answer response (b) is correct.
College A College B
2. NGO–A 80 in Haryana 70
NGO–C 160 in Tamil Nadu 60
C
PER CENT INCREASE

Difference 160 – 80 = 80
[Should be calculated mentally.] 50
PS

As 80 is not among the answer responses, 40


mark (e) as correct.
[The question can be answered by looking 30
at the graph and calculating the difference
U

20
graphically. The difference is 4 divisions
on the Y-axis; each division = 20 units. So 10
@

the total difference is 4 × 20 = 80.] 0


2000

2003

2004
2001

2002

2005

3. Average number of trees planted in


Haryana by all the NGOs
Years
(80 + 140 + 170)
= 1. If the number of students studying in College
3 B in the year 2002 was 30,375, what was the
390 number of students studying in that college
= = 130.
3 in the year 2001?
Answer response (e) is correct as 130 is (a) 22,500
not among the answer responses. (b) 30,800
4. Total number of trees planted by (c) 17,500
NGO–A and NGO–B together in Bihar (d) Cannot be determined
= 100 + 60 = 160 (e) 12%
Unit Eight : Fundamentals / Data Analysis: Tables, Graphs and Diagrams ✫ 527

2. What is the average per cent increase in 100


number of students in College B over the = 30375 ×
100 + 35
years? (Rounded off to two digits after
100
decimal) = 30375 ×
(a) 38.27 (b) 51.04 135
(c) 35.76 (d) 46.67 = 225 × 100 = 22500.
(e) None of these (Refer to the formulae given earlier in the
unit under General Concepts.)
3. If the number of students studying in College Answer response (a) is correct.
A in the year 2001 was 2,520, what was the

TS
number of students studying in that college 2. Average percentage increase
in the year 2002? (30 + 40 + 35 + 50 + 60 + 65)
(a) 2,481 (b) 3,276 =
6
(c) 3,940 (d) 2,965 280

H
(e) None of these =
6
4. Which of the following statements is true = 46.666%

G
based on the graph? = 46.67%
(a) The number of students studying in Answer response (d) is correct.
College B in the year 2002 was less than
the number of students in 2001
U 3. Number of students studying in college
A in the year 2002
O
(b) Number of students studying in College 100 + 30
B was more than the number of students = 2520 ×
studying in College A in the year 2005 100
H
(c) The number of students studying in 130
= 2520 ×
College A in the years 2003 and 2005 100
_T

were the same = 252 × 13 = 3276.


(d) The increase in the number of students (Refer to the formulae given earlier in
of College A in the year 2004 from the this unit under General Concepts.)
Answer response (b) is correct.
C

previous year is 5%
(e) None of these 4. As this is a graph showing percentage
increase, there can be no ‘decrease’ in
PS

5. If the number of students studying in colleges number of students. Don’t be misled by


A and B in the year 2003 was 4,000 each, what the declining curve. Option (a) is not
was the respective ratio of the number of true. As the percentage graph does not
students studying in colleges A and B in the
U

indicate any absolute numbers, we can


year 2004? make no comparisons about ‘number of
(a) 31 : 30 (b) 30 : 31 students’ in the different colleges. Option
@

(c) 33 : 32 (d) 32 : 33 (b) is not true. By the same reasoning


(e) None of these option (c) is not true. Option (d) is true,
as there is a 65 per cent increase in 2004
Solutions Note that the graph presents ‘per in College A as against 60 per cent increase
cent increase’ and do not make the mistake of in 2003, showing a 5 per cent increase in
thinking that is the ‘percentage’ of students number of students.
in the college in a particular year. Answer response (d) is correct.
5. Ratio of number of students in College A
1. In 2002 there was a 35 per cent increase,
and B in 2004
meaning there was 35 per cent fewer
165 160
students in College B in 2001. = 4000 × : 4000 ×
100 100
So, the number of students studying in = 40 × 165 : 40 × 160
College B in the year 2001 = 6600 : 6400
528 ✫ Quantitative Aptitude

= 66 : 64 5. The number of girls from IT department is


= 33 : 32 approximately what per cent of the total
Answer response (c) is correct. number of girls from all the departments
[Be careful not to mark (d).] together?
(a) 47 (b) 38
● Study the following graph carefully to answer (c) 23 (d) 13
the questions that follow. (e) 30
Total Number of Boys and Girls in Five
Solutions:
Different Departments 1. Read the differences between number of

TS
Boys Girls boys and girls in each department from
5000 the graph itself and add the numbers to
get the total. Each division on the Y-axis
4500

H
is of 500.
4000
IB – difference is 500
NUMBER OF STUDENTS

3500 Marketing – difference is 1250

G
3000 Finance – difference is 500
2500 HR – difference is 500
2000
1500 U IT – difference is 1000
Total difference 3750
O
1000 Answer response (c) is correct.
500
[Alternative method: Difference between
H
0 the total number of boys and the total
IB Marketing Finance HR IT
number of girls from all the Departments
_T

DEPARTMENTS
= (4250 + 3500 + 3750 + 3000 + 4500)
1. What is the difference between the total – (3750 + 2250 + 3250 + 2500 + 3500)
number of boys and the total number of girls = 19000 – 15250
from all the departments together?
C

= 3750.
(a) 1,000 (b) 3,150 However, this is a lengthier method.]
(c) 3,750 (d) 500
PS

(e) None of these 2. Average number of boys from all the


Departments
2. What is the average number of boys from all
the departments together? 4250 + 3500 + 3750 + 3000 + 4500
=
U

(a) 3,600 (b) 3,800 5


(c) 3,300 (d) 3,000
19000
= = 3800.
5
@

(e) None of these Answer response (b) is correct.


3. What is the respective ratio of number of girls
3. Number of girls from Finance Department
from finance department to the number of = 3250
girls from IB department? Number of girls from IB Department
(a) 15 : 14 (b) 15 : 16 = 3750
(c) 5 : 7 (d) 13 : 15 ∴ Ratio is 3250 : 3750
(e) None of these = 13 : 15.
4. The total number of boys from HR and Answer response (d) is correct.
marketing departments together are what per 4. Total number of boys from HR and
cent of the total number of boys from IT, marketing departments together
finance and IB departments together? = 3000 + 3500
(a) 52 (b) 83 = 6500
(c) 69 (d) 25 Total number of boys from IT, finance and
(e) 99 IB departments together
Unit Eight : Fundamentals / Data Analysis: Tables, Graphs and Diagrams ✫ 529

= 4500 + 3750 + 4250 2. In which year is the total number of flights


= 12500 (both national and international) cancelled the
6500 second lowest?
Required percentage = × 100 (a) 2003 (b) 2004
12500
(c) 2006 (d) 2007
= 52%
(e) None of these
Answer response (a) is correct.
3. What is the ratio of the number of
5. Number of girls from IT departments international flights cancelled in the year 2007
= 3500 to the total number of national flights
Total number of girls from all the

TS
cancelled in the years 2004 and 2005
departments together?
= 3750 + 2250 + 3250 + 2500 + 3500 (a) 9 : 17 (b) 9 : 8
= 15250 (c) 16 : 9 (d) 3 : 1
∴ Required percentage

H
(e) None of these
3500 4. What is the approximate average number of
= × 100

G
15250 national flights cancelled over all the years?
(a) 780 (b) 867
= 22.95 (c) 898 (d) 824
= 23 (approximate)
Answer response (c) is correct.
U (e) 765
5. Total number of international flights
O
[In items like 4 and 5, you must get the
cancelled in the years 2006 and 2007 together
base correct.]
is approximately what percentage of
H
● Study the following graph and answer the international flights cancelled in the years
questions that follow. 2003, 2004 and 2008 together?
(a) 81 (b) 85
_T

Number of National and International


(c) 96 (d) 91
flights (in hundreds) cancelled in (e) 99
six different years
Solutions
C

National flights International flights


1. One way of finding the answer is this:
16 Total number of national flights cancelled
PS

over all the years


Number of flights (in hundreds)

14
= (6 + 9 + 8 + 7 + 12 + 10) hundred
12 = 5200
Total number of international flights
U

10
cancelled over all the years
8 = (4 + 5 + 3 + 12 + 9 + 14) hundred
= 4700
@

6 ∴ Difference = 5200 – 4700


4 = 500.
Answer response (a) is correct.
2 Alternatively, observe the difference in
0
the diagram (calculate mentally).
2003 2004 2005 2006 2007 2008 National – International
Years in 2003, 600 – 400 = 200
1. What is the difference between the total in 2004, 900 – 500 = 400
in 2005, 800 – 300 = 500
number of national flights and the total
in 2006, 700 – 1200 = (–) 500
number of international flights cancelled over in 2007, 1200 – 900 = 300
all the years? in 2008, 1000 – 1400 = (–) 400
(a) 500 (b) 250
Total difference 500
(c) 700 (d) 450
[The calculations need to be done mentally:
(e) None of these
200 + 400 + 500 – 500 + 300 – 400 = 500.]
530 ✫ Quantitative Aptitude

2. From a look at the heights of the bars in 2100


the chart, it is obvious that the lowest ∴ Required percentage = × 100
2300
number of cancellations is in 2003 and the
= 91.30%
second lowest in 2005.
Answer response (b) is correct. ≈ 91%
Answer response (d) is correct.
[Cancellations in
● The figure gives details of cereals production
2003 600 + 400 = 1000
in India over a period of years. Answer the
2004 900 + 500 = 1400
questions 1 and 2 based on it.
2005 800 + 300 = 1100

TS
The other three years may safely be
ignored as the bars are much higher.]
3. Note the years and which type of flight
is mentioned in each case, and what ratio

H
is asked for.
Number of international flights cancelled

G
in the year 2007 = 900
Total number of national flights cancelled
in the years 2004 and 2005 together
= 900 + 800
= 1700 U
O
∴ Ratio of number of international flights
cancelled in 2007 to number of national
H
flights cancelled in 2004 and 2005
= 900 : 1700
Period of years
_T

= 9 : 17.
Answer response (a) is correct. 1. Which of the following statements can be
concluded from the above diagram?
4. Add up (the correct flight) and divide I. The only cereal to show steady increase
C

by 6. is rice.
Average number of national flights II. The trend in cereal crops is one of
cancelled over all the years
PS

growth in production.
600 + 900 + 800 + 700 + 1200 + 1000 III. Wheat and other cereals show
=
6 fluctuations in production.
5200 (a) I and III (b) I only
U

= = 866.67 (c) I, II and III (d) II only


6
≈ 867. 2. In which year did all three crops register a
@

Answer response (b) is correct. rise in production over the preceding years?
(a) 1972-73 (b) 1973-74
5. Again be careful about choosing the
correct base for calculating the (c) 1974-75 (d) 1975-76
percentage. Solutions
Total number of international flights 1. A careful observation of the given multiple
cancelled in the years 2006 and 2007 bar diagram will help you reach the
together
conclusion. You must be careful to identify
= 1200 + 900
= 2100 the components correctly to compare
Total number of international flights accurately. The trend is the average
cancelled in the years 2003, 2004 and 2008 tendency over a long period of time and
together it is seen to be an increasing one.
= 400 + 500 + 1400 Take each statement one by one.
= 2300 I is true; the bars for ‘wheat’ and ‘other
Unit Eight : Fundamentals / Data Analysis: Tables, Graphs and Diagrams ✫ 531

cereals’ show a decline in at least one (a) 37 (b) 57


of the years. (c) 52 (d) 47
II is also true. The general movement (e) 42
of the line of growth is positive on the 5. In which match the total runs scored by all
whole. The decline is never too large. the teams together is second highest?
III is true; in fact, we saw it when (a) Match-2 only
considering I. (b) Match-6 only
So (c) is the correct answer. (c) Match-4 only
2. Be careful that the increase is to be in all (d) Both Match-2 and Match-6

TS
three items—which is there only in 1974- (e) Both Match-2 and Match-4
75 over 1973-74. So (c) is the correct
Solutions You have to be careful in matching
answer.
the team and bar correctly while answering

H
● Study the following graph carefully to answer the questions. Use a paper edge to measure
these questions. the height of the bars against the units on the
Y-axis.

G
Number of runs scored by three different 1. The previous match is Match 3.
teams in six different cricket matches Number of runs scored by Team-B in
400
350
Team-A Team-B Team-C
U Match-4 = 300
Number of runs scored by Team-B in
O
the previous match (Match-3) = 250
300
Required percentage increase
N o. of Ru ns

250
H
200 300 − 250
= × 100
150 250
_T

100 50
50 = × 100
250
0 = 20.
Match-1 Match-2 Match-3 Match-4 Match-5 Match-6
Answer response (c) is correct.
C

1. What was the percentage increase in number


of runs scored by Team-B in Match-4 as 2. Number of runs scored by Team-A in
PS

compared to that of the previous match? Match-2 = 200


(a) 40 (b) 30 Number of runs scored by Team-C in
(c) 20 (d) 25 Match-6 = 300
Required ratio = Team A : Team C
(e) None of these
U

= 200 : 300
2. What is the ratio between the number of runs = 2 : 3.
scored by Team-A in Match-2 and the number
@

Answer response (c) is correct.


of runs scored by Team-C in Match-6?
(a) 5 : 4 (b) 2 : 5 3. Average number of runs scored by
Team-B
(c) 2 : 3 (d) 3 : 4
(e) None of these (200 + 300 + 250 + 300 + 250 + 200)
=
3. What was the average number of runs scored 1500 6
= = 250.
by Team-B in all the matches together? 6
(a) 250 (b) 275 Answer response (a) is correct.
(c) 200 (d) 300 4. Add up all the bar heights in Match 3
(e) 225 only and then the bar heights of Team C
4. Number of runs scored by all the teams in all matches.
together in Match-3 is approximately what Number of runs scored by all the teams
percentage of the total runs scored by Team- together in Match-3
C in all the matches together? = 100 + 250 + 200 = 550
532 ✫ Quantitative Aptitude

Number of runs scored by Team-C in all State Expenditure Per Capita


the matches together
= 100 + 250 + 200 + 350 + 100 + 300

Per capita expenditure (in Rs.)


= 1300
Required percentage
550
= × 100
1300
= 42.30

TS
 42%
(Again be careful of which number to use
as base.)

H
Answer response (e) is correct.
5. One can estimate in which match all

G
teams together made the highest total
runs by looking at the bars. In Match-4,
all bars are high compared to the other
matches. As for second highest, one can U 1. What was the approximate state expenditure
per capita in cities having populations of
O
leave out Match-1 and Match-3; as for 2,00,000 to 2,99,000?
Match-5, the dark bar is too low compared (a) Rs 100 (b) Rs 150
H
to the corresponding bars Match-2 and (c) Rs 180 (d) Rs 200
Match-6, so it too can be ignored. Now 2. Of the three categories of expenditures which
_T

as the answer options actually mention was least dependent on city size?
two matches also, check out for Match- (a) Health (b) Utilities
2 and Match-6. (c) Administration (d) None
C

Total number of runs in Match-2 Solutions


= 200 + 300 + 250 = 750 1. The diagram is a subdivided bar diagram.
PS

Total number of runs in Match-6 The middle bar of the seven bars shown
= 250 + 200 + 300 = 750 represents the cities with population of
∴ Total runs scored by all the teams together 200000 and 299000. It reaches about
is second highest in both Match-2 and
U

halfway between 100 and 200, so the


Match-6. approximate estimate is Rs 150 per capita.
Answer response (d) is correct.
@

[Checking out for the other matches, we Answer response (b) is correct.
find: 2. Note that the dark part of each bar (the
Total number of runs in Match-1 part that represents utilities) varies least
= 150 + 200 + 100 = 450 as city size increases. It shows that even
Total number of runs in Match-3 as city size increases, utilities have not
= 100 + 250 + 200 = 550 shown much change in expenditure per
Total number of runs in Match-4 capita.
= 250 + 300 + 350 = 900 So, (b) is the correct answer.
Total number of runs in Match-5
● The pie chart given here shows the break-
= 300 + 250 + 100 = 650]
up of the cost of construction of a house
● Answer the questions that follow the on various heads. Study the chart and
diagram given below. answer the questions based on it.
Unit Eight : Fundamentals / Data Analysis: Tables, Graphs and Diagrams ✫ 533

Break-up of the Cost of Cement 72°


Construction of a House Required percentage
54 + 54 + 72
= × 100
360
Timber 180
Labour = × 100
36° 360
54° 90° = 50%.
Supervision Answer response (a) is correct.

TS
54° 3. Timber 36°
72°
Steel Cement 72°
54° Cement Get the base right. As 360° is common to

H
Bricks both, the percentage required is
36
× 100 = 50%.

G
72
Answer response (b) is correct.
1. If the total cost of construction of the house is
Rs 15,00,000, how much amount of money
was spent on labour? U 4. Labour
Supervision
90°
54°
O
(a) Rs 90,000 (b) Rs 2,50,000 Together 144°
(c) Rs 3,60,000 (d) Rs 3,75,000 Amount spent on labour and supervision
is
H
2. The total expenditure incurred on bricks, steel
and cement is what per cent of the total cost 144
= × 1500000
_T

of construction? 360
(a) 50 (b) 54 = Rs 600000.
(c) 72 (d) 75 Answer response (c) is correct.
3. Expenditure incurred on timber is what per
C

● Study the pie-chart carefully to answer the


cent of the expenditure on cement? questions that follow.
(a) 36 (b) 50
PS

(c) 72 (d) 18 Percentage of Students Enrolled in


Different Hobby Classes in a School
4. Out of the total cost (Rs 15,00,000) of
Total Number of Students = 3600
construction, how much amount of money
U

was spent on labour and supervision Singing


combined together? Cooking
Classes
Classes
@

(a) Rs 1,44,000 (b) Rs 3,00,000


(c) Rs 6,00,000 (d) Rs 7,50,000 22%
18%
Solutions Recall how to convert degree into
amount
13% 21%
Angle degree Drama Dancing
× amount Classes Classes
360 11%
1. Here, money spent on labour 15%
90
= × 1500000 Stitching
360
Classes Painting
= Rs 375000.
Classes
Answer response (d) is correct.
2. Bricks 54° 1. What is the total number of students enrolled
Steel 54° in stitching and drama classes together?
534 ✫ Quantitative Aptitude

(a) 684 (b) 846 4. Let x be the required percentage.


(c) 648 (d) 864 22 x 21
(e) None of these So, = ×
100 100 100
2. How many students are enrolled in painting 22 × 100
classes? x = = 104.76%
21
(a) 550 (b) 480 Answer response (b) is correct.
(c) 450 (d) 520 ● The pie charts below give the characteristics
(e) None of these of foreign tourists in one year. Study them
3. What is the ratio of number of students and answer the question that follows.

TS
enrolled in singing and dancing classes
together to those enrolled in drama classes?
(a) 3 : 1 (b) 4 : 7 American Below 20 years
60%

H
(c) 7 : 5 (d) 3 : 5 60%
(e) None of these
ee n

G
4. The number of students enrolled in cooking si an Be tw 4 0
Rus 0% British -
Above
classes is what per cent of those enrolled in 1 20 40 years
Others 15% a r s
dancing classes? (rounded off to two digits ye
after decimal.)
(a) 101.45 (b) 104.76 U 15%
Tourists Countrywise
20% 20%

American Tourists
O
(c) 113.84 (d) 110.28 100,000 = 100% in Age Groups
(e) None of these From this chart, the number of American
H
tourists in the age group 20-40 who visited
Solutions Study the diagram and the note during the year, is
on the total number of students.
_T

(a) 12,000 (b) 20,000


The pie chart represents 100 per cent. (c) 40,000 (d) 60,000
In total numbers we are given it is 3600,
3600 Solution Both pie diagrams must be
C

so 1% = = 36. considered together. 20% American tourists


100
are between 20-40 years and total American
1. Number of students in stitching and tourists are 60% of 100000 = 60000.
PS

drama = 11 + 13 = 24% Therefore, American tourists between 20-40


Now 1% 36 years are 20% of 60000 = 12000.
∴ 24% 24 × 36 = 864 Answer response (a) is correct.
Answer response (d) is correct.
U

● The expenditure on the various items by


2. Students in painting classes = 15% two families A and B are below.
@

As 1% equates with 36, Study them and answer the question that
15% equates with 36 × 15 = 540 follows.
Answer response (e) is correct, as 540 Family A Family B
is not among the options. Total Expenditure Total Expenditure
3. Here a ratio is to be found. There is Rs 20,000 p.a. Rs 1,00,000 p.a.
no need to convert percentages into
absolute values, or even use 100 as a
base.
The ratio of students in singing and
dancing classes to those in drama classes
= (18 + 21) : 13
39
= 3 : 1
13
Answer response (a) is correct.
Unit Eight : Fundamentals / Data Analysis: Tables, Graphs and Diagrams ✫ 535

From these diagrams we can conclude that 1. What is the total number of male lecturers in
(a) Family A spent more money on food the university?
than Family B (a) 696 (b) 702
(b) Family B spent more money on food (c) 712 (d) 668
than Family A (e) None of these
(c) Family A and Family B spent the same
amount of money on food 2. What is the ratio of the number of female
(d) The expenditure on food by Family A lecturers in Physics to the number of male
and Family B cannot be compared lecturers in Mathematics?
Solution These are percentage bar diagrams. (a) 5 : 9 (b) 2 : 9

TS
For Family A, the break-up of total (c) 3 : 7 (d) 5 : 3
expenditure is as follows: (e) None of these
Food 50% of 20000 = Rs 10000
3. Total number of lecturers (both male and
Other Items 30% of 20000 = Rs 6000

H
female) in Hindi is approximately what per
Education 20% of 20000 = Rs 4000
cent of the total number of female lecturers
For Family B, the break-up is as follows: in Mathematics and Chemistry together?

G
Food 10% of 100000 = Rs 10000 (a) 58 (b) 43
Other items 60% of 100000 = Rs 60000 (c) 47 (d) 51
Education 30% of 100000 = Rs 30000
The answer is now clear: (c) is correct.
U (e) 40
4. What is the difference between the total
O
● Study the following pie-chart and the table number of lecturers (both male and female)
given below it carefully to answer the in Zoology and the total number of male
H
questions. lecturers in Chemistry and Education
Percentage-wise distribution of together?
Lecturers in six different (a) 192 (b) 182
_T

subjects in a University (c) 146 (d) 136


(Total Number of Lecturers: 1600) (e) None of these
5. What is the difference between the number
C

Chemistry of female lecturers in Zoology and the


13% number of male lecturers in Hindi?
PS

Zoology
(a) 156 (b) 160
22%
(c) 150 (d) 153
Education (e) None of these
U

Physics 18%
14% Solutions Note that the total number of
lecturers is 1600 while the pie chart totals
@

Hindi
100%; 100 = 1600, so 1% = 16. In each
Mathematics 12%
department the number will be percentage
21% × 16. The break-up of male–female is given
in the form of ratios in the table. The total
number of lecturers in a department may
Ratio of male to Female Lecturers in be calculated with the help of the pie chart
the University alone (percentage × 16). If male/female
Lecturers Males Females numbers are required, the table of ratios
Mathematics 3 4 must also be used.
Education 5 3
Hindi 1 3 1. The ratio is given, so calculate from that
Chemistry 1 7 the number of males in each department
Physics 9 5 (as deduced from the pie chart) and add
Zoology 7 9 up to get the total.
536 ✫ Quantitative Aptitude

3 numbers, especially, as there is a 51


Mathematics × 21 × 16 = 144 among the options.)
7
Answer response (d) is correct.
5
Education × 18 × 16 = 180 4. From the pie chart we get the total
8
1 number of lecturers in Zoology
Hindi × 12 × 16 = 48 22 × 16 = 352
4
1 From the pie chart and ratio table, male
Chemistry × 13 × 16 = 26 lecturers in Chemistry and Education
8

TS
may be calculated. As you have already
9
Physics × 14 × 16 = 144 done so for Question 1, take the numbers
14
from there.
7 Total male lecturers in Chemistry plus

H
Zoology × 22 × 16 = 154
16 Education are 26 + 180 = 206.
Total male lecturers = 696 The difference between the total number

G
Answer response (a) is correct.
of Zoology lecturers and male lecturers
2. In Physics, the number of female in Chemistry and Education
lecturers =
5
14
× 14 × 16 = 80
U = 352 – 206 = 146.
Answer response (c) is correct.
O
From solving Question 1 above, we
know the number of male lecturers in 5. Number of female lecturers in Zoology
Mathematics to be 144. So the required
H
9
ratio = × 22 × 16 = 198
16
80 5 Number of male lecturers in Hindi is 48
_T

= = or 5 : 9.
144 9 (Recall from calculations for Question 1.)
[In such questions you need to note what The difference is 198 – 48 = 150
is asked and not hastily calculate the Answer response (c) is correct.
C

numbers for females in both cases or for


the same department in both cases. Also, ● Study the following pie-chart and bar chart
PS

if the ratio asked is for females to males, and answer the questions.
do not turn it into a ratio of males to
females; 5 : 9 is not the same as 9 : 5.] Percentagewise Distribution of
Students in Six Different Schools
U

Answer response (a) is correct.


3. Here the total number of lecturers in (Total number of Students = 6000)
@

Hindi = 12 × 16 = 192
Schoo

Number of female lecturers in School A


Mathematics and Chemistry 12%
lF

4 7
6%

= × 21 × 16 + × 13 × 16 School B
7 8 School E 9%
= 192 + 182 = 374. 29%
Percentage of total lecturers in Hindi in
terms of total of female lecturers in
School C
Mathematics and Chemistry 26%
192 School D
= × 100 = 51 per cent approximately. 18%
374
(As the answer options do not have
decimals, you may stop with whole
Unit Eight : Fundamentals / Data Analysis: Tables, Graphs and Diagrams ✫ 537

Number of Boys out of the 6000 Solutions Here a pie chart and bar diagram
Students in each School have to be studied to gather the necessary
Separately information. We can see that 1 per cent is
equivalent in absolute numbers to 60
Boys students. Have a careful look at the bars
School-F and note how the units are distributed: for
100, though markings are at a gap of 200
School-E each division is for 200.
1. The bar diagram gives the number of
School-D
boys. So we have to calculate the number

TS
School-C of girls with the help of the pie chart
as well.
School-B In School-C the number of girls is the

H
total number of students minus the
School-A
number of boys
0 200 400 600 800 1000 1200 1400 (26 × 60) – 900

G
= 1560 – 900 = 660
Number of Boys Similarly, in School-E, the number of
1. What is the total number of girls in School-C,
number of girls in School-E and the number
U girls
(29 × 60) – 1200
= 1740 – 1200 = 540
O
of boys in School-D together?
(a) 1700 (b) 1900 Together in Schools-C and E the girls
(c) 1600 (d) 1800 number (660 + 540 =) 1200 to which add
H
(e) None of these 600 (number of boys in School-D as read
from the diagram) to get the total 1800.
2. What is the respective ratio between the
_T

number of boys in School-C, number of girls Answer response (d) is correct.


in School-B and total number of students in 2. Number of boys in School-C = 900
School-E? Number of girls in School-B
= 9 × 60 – 400
C

(a) 45:7:97 (b) 43:9:97


(c) 45:7:87 (d) 43:9:87 = 540 – 400 = 140
Total number of students in School-E
PS

(e) None of these


= 29 × 60 = 1740
3. What is the difference between the total Required ratio= 900 : 140 : 1740
number of students in School-F and the = 45 : 7 : 87
number of boys in School-E?
U

Answer response (c) is correct.


(a) 820 (b) 860
(c) 880 (d) 900 3. Total number of students in School-F
@

(e) None of these = 6 × 60 = 360


Number of boys in School-E = 1200
4. In which school is the total number of Difference = 1200 – 360
students (both boys and girls) together equal = 840.
to the number of girls in School-E? Answer response (e) is correct as 840
(a) A (b) B is not among the given options.
(c) C (d) D 4. You would recall from the calculations
(e) F for Question 1 that the number of girls
5. Number of girls in School-A is approximately in School-E = 540
You have now to just see which
what per cent of total number of students in percentage number on the pie chart
School-B? multiplied by 60 equals 540. Clearly
(a) 55 (b) 50 School-B with 9 per cent is the correct
(c) 35 (d) 45 choice as 9 × 60 = 540.
(e) 40 Answer response (b) is correct.
538 ✫ Quantitative Aptitude

5. Number of girls in School-A 1. What is the ratio of the number of pen drives
= 12 × 60 – 500 produced by Company A to the number of
= 720 – 500 = 220 computers produced by Company B?
Where total number of students in School- (a) 8 : 9 (b) 9 : 7
B is concerned, we just found in the (c) 7 : 9 (d) 4 : 9
previous question that it is 540 (e) None of these
∴ Number of girls in School-A as a 2. What is the total number of phones produced
percentage of number of students in by both the companies together?
School-B (a) 430 (b) 420

TS
220 (c) 390 (d) 530
= × 100 = 40.74
540 (e) None of these
As the answer responses do not have
3. The number of phones produced by Company
41 but do have 40 among them, this is

H
A is what percentage of the total number of
the approximate figure.
products produced by Company B?
Answer response (e) is correct. (a) 25 (b) 29

G
(c) 33 (d) 37
Passage-based Questions (e) None of these
Tables and diagrams are two means of presenting
data for analysis and interpretation. However, U 4. What is the average of the number of pen
drives, CDs and computers produced by
O
data may be presented in a passage. The facts/ Company B?
data in these passages appear to be heaped (a) 840 (b) 280
H
together in a confusing manner, but careful (c) 270 (d) 860
reading helps one to understand the data so as (e) None of these
to answer the questions based on the passages.
_T

5. What is the difference between the number


The data in the passage may need to be of CDs produced by Company B and the
arranged in a table/chart or diagram and some number of computers produced by Company
calculations may be required to plug in to A?
C

complete the picture more numbers. (a) 200 (b) 250


The worked examples will illustrate the (c) 300 (d) 350
PS

method of tackling these problems. (e) None of these


● Study the information carefully to answer Solutions Before going on to answer the
the questions that follow: question, make some basic calculations and
U

There are two companies A and B. Both a chart of the data that enables easy
companies produce all the four different understanding.
@

products, viz., computers, phones, pen drives Company A produces 800 products
and compact discs (CDs). Company A produces 5
a total of 800 products. The ratio of the total Company B produces of A’s products
4
products produced by Company A to that by 5
i.e., of 800 = 1000 products.
Company B is 4 : 5. 20% of the total products 4
4
produced by Company B are pen drives and 40% [We are told that A’s total products is
of them are CDs. Two-fifths of the remaining 5
of B’s total products, so B’s total products
products produced by Company B are phones.
The total number of computers produced by 5
is of A’s.]
both the companies together is 340. 20% of the 4
total products produced by Company A are CDs. Take the data as it comes and make the
Company A produces equal number of pen chart to show the products made by the
drives and phones. . two companies.
Unit Eight : Fundamentals / Passage-based Questions ✫ 539

Data for Company B is given first. (Again calculations to be done mentally.)


For Company B Answer response (b) is correct.
20 5. The difference between number of CDs
Pen Drives = × 1000 = 200 produced by B and number of computers
100
40 produced by A
CDs = × 1000 = 400 = 400 – 100 = 300
100 (Again, should be mentally calculated.)
2 Answer response (c) is correct.
Phones = of (1000 – 200 – 400)
5 [You may wonder at such ridiculously

TS
2 simple problems. However, it is easy to
= of 400 = 160
5 pick the wrong product or number—say
Computers = 1000 – 200 – 400 – 160 = 240 computers of both A and B when CDs are
asked for, or interchange the values of

H
For Company A
Computers 340 – 240 = 100 products—when attempting the problems
(as total computers produced is 340 and A in a hurry, and thus get the wrong answer.

G
produces 240) So you have to be alert.]
20
CDs =
100
× 800 = 160
Pen Drives + phones = 800 – 100 – 160 = 540 U ● Study the information carefully to answer
the questions that follow.
O
540 In a ship there are 1200 passengers. 18 per
Pen Drives = = 270
2 cent of the total number of passengers is from
H
540 Britain. Two-fifths of the total number of
Phones = = 270 passengers is from South Africa. 6 per cent of the
2
_T

Now take up the questions. total number of passengers is from Madagascar.


1. Ratio of pen drives by A to computers Remaining number of passengers is from India.
by B 25 per cent of the number of passengers from
Britain is female. Half the number of passengers
C

= 270 : 240 = 9 : 8
As none of the options gives 9 : 8, from South Africa is male. There is no female
answer response (e) is correct. passenger from Madagascar. Two-third of the
PS

(Note that 9 : 8 is not the same as number of passengers from India are females.
8 : 9; get the ratio right—it is A to B,
1. What is the respective ratio between the
not B to A)
number of passengers from Madagascar,
U

2. To be mentally added : B → 160; A → 270. number of female passengers from South


Total = 160 + 270 = 430 Africa and the total number of passengers
@

Answer response (a) is correct. from India?


3. B’s total products number 1000. So the (a) 2 : 5 : 18 (b) 3 : 10 : 18
phones produced by A (270) as a (c) 3 : 11 : 18 (d) 2 : 18 : 5
percentage of B’s total products (e) None of these
270 2. Number of male passengers from South
= × 100 = 27 per cent. Africa is approximately what percentage
1000
(Again, should be mentally calculated.) of the total number of passengers from
Answer response (e) is correct, as 27 Britain?
per cent is not among the options. (a) 111 (b) 115
4. The average of pen drives, CDs and (c) 120 (d) 125
computers produced by B (e) 131
200 + 400 + 240 3. What is the average number of male
= = 280 passengers from all the four countries?
3
540 ✫ Quantitative Aptitude

(a) 154.5 (b) 164.5 5. Total number of male passengers from


(c) 145 (d) 164 Britain and female passengers from India
(e) None of these = 162 + 288
4. What is the difference between the number = 450.
of male passengers from Madagascar and the As none of the options is 450, the
number of male passengers from India? correct answer response is (e).
(a) 64 (b) 82
(c) 74 (d) 72 ● Study the following information carefully
(e) None of these and answer the questions given below it.

TS
5. What is the total number of male passengers In a colony of 5500 members, 18% of the
from Britain and female passengers from people manage their business and 10% of those
India together? who manage their business are females. 65% of
(a) 340 (b) 420
(c) 350 (d) 460 the total number of people in the colony serve

H
(e) None of these various organisations. 40% of the number of
people serving various organisations are females.

G
Solutions 12% of the total number of people in the colony
1. Ratio between the total number of are unemployed. 30% of the number of people
passengers from Madagascar, number of unemployed are females. The remaining people
female passengers from South Africa and
total number of passengers from India. U in the colony are children, 60% of whom are
females.
O
= 72 : 240 : 432
= 3 : 10 : 18 1. The number of children (both males and
females) is approximately what per cent of
H
Answer response (b) is correct.
people who manage their business (both
2. Number of male passengers from South males and females)?
_T

Africa as a percentage of total number of (a) 28 (b) 24


passengers from Britain (c) 32 (d) 21
240 (e) 35
= × 100
C

216 2. What is the difference between the male and


3000 female children in the colony?
=
PS

27 (a) 65 (b) 55
≈ 111%. (c) 45 (d) 35
(As the answer responses include only (e) None of these
111 and no decimal places, you may
U

3. What is the total number of adult males in


stop dividing when you get 111.) the colony (excluding the children)?
Answer response (a) is correct. (a) 3496 (b) 3490
@

3. Average number of male passengers from (c) 3500 (d) 3504


all the countries (e) None of these
162 + 240 + 72 + 144 4. What is the total number of females in the
=
4 colony?
618 (a) 1884 (b) 1896
= = 154.5.
4 (c) 1888 (d) 1892
Answer response (a) is correct. (e) None of these
4. Difference between the number of male 5. What is the ratio of the unemployed males
passengers from Madagascar and number to the number of males working for various
of male passengers from India organisations?
= 144 – 72 (a) 3 : 13 (b) 12 : 65
= 72 (c) 4 : 13 (d) 16 : 65
Answer response (d) is correct. (e) None of these
Unit Eight : Fundamentals / Passage-based Questions ✫ 541

Solutions Organise the information in the 5. Unemployed males = 462


form of a chart. You have the total numbers Males in various organisations = 2145
and percentages of various categories as Ratio = 462 : 2145
well as division on gender basis. None of the answer responses gives the
(In such problems, you need to note which correct ratio. Response (a) comes close:
category the percentage figures refer to.) the ratio is 3 : 13.9 which is more
Total numbers = 5500 approximately 3 : 14.
Total Females Males
So answer response (e) is correct.
18 10 [In such cases as 462 : 2145 where finding

TS
Business × 5500 × 990 990 – 99 a common factor and hence the ratio seems
100 100
= 990 = 99 = 891 difficult, try out the options. In this case
(or 0.1 × 990) taking (a), divide 462 by 3 to get 154. Now
65 40 as 13 is the other proportion, if you divide

H
Various × 5500 × 3575 3575 – 1430
100 100 2145 by 154 you should get 13; but you
Organisations = 3575 = 1430 = 2145 get 13.92. The other options get nowhere

G
(or 0.4 × 3575) near.]
30
Unemployed 12 × 55* × 660 660 – 198 ● Study the following information carefully
= 660
100
= 198
60
= 462
U and answer the questions given below it.
There are 2500 residents in a village. 1,375
O
Children (5500 – 990 × 275 275 – 165
100 residents from this village speak only their local
– 3575 – 660) = 165 = 110
language. 200 residents of the village speak the
H
= 275
local language as well as English. The number of
* Since we note that the 00’s cancel out. residents in the village who speak the local
_T

language as well as Hindi is 625. 300 residents


Now go on to the questions. of the village speak all the three languages i.e.,
1. Number of children = 275; English, Hindi and the local language.
Number in business = 990
C

Percentage asked for 1. The number of residents who speak English


275 as one of the languages forms what per cent
= × 100 = 27.77
PS

of the total residents in the village?


990
(a) 12 (b) 8
28 per cent approximately. (c) 20 (d) 18
Answer response (a) is correct. (e) None of these
U

2. Difference between male and female 2. The number of residents who speak only the
children = 165 – 110 = 55 local language forms what per cent of the
@

(Calculate mentally.) total number of residents in the village?


Answer response (b) is correct. (a) 45 (b) 55
3. Total adult males = 891 + 2145 + 462 (c) 58 (d) 40
= 3498 (e) None of these
(Calculate mentally.) 3. The number of residents who speak Hindi
As the answer responses do not include as one of the languages is approximately what
this number, so (e) is correct. per cent of the number of residents who speak
4. Note that total number of females is only the local language?
asked for, so the number of female (a) 67 (b) 70
children will be included: (c) 61 (d) 59
99 + 1430 + 198 + 165 = 1892 (e) 63
(Calculate mentally.) 4. What is the ratio of the number of residents
Answer response (d) is correct. who speak all the three languages to the
542 ✫ Quantitative Aptitude

number of residents who speak the local 1. Those who speak English as one of the
language as well as Hindi? languages is 200 + 300 = 500
(a) 12 : 55 (b) 10 : 25 As percentage of total residents
(c) 14 : 55 (d) 12 : 25 500
(e) None of these × 100 = 20%
2500
5. If 25 more people who can speak all the three Answer response (c) is correct.
languages come to reside in the village and [Note: If the question referred to those
45 more people who can speak the local who spoke only English, it would be
language and Hindi come to reside in the different.]

TS
village, what would be the difference
2. Those who speak only local language
between the number of residents who can
= 1375
speak all the three languages and the number
The percentage they form of the total
of residents who can speak the local language

H
residents
and Hindi?
(a) 325 (b) 330 1375
= × 100 = 55

G
(c) 340 (d) 355 2500
(e) None of these Answer response (b) is correct.

Solutions In such passages, we find that


numbers overlap, with elements being U 3. Again, the question refers to speakers
of Hindi as one of the languages.
O
The required number is
common. Here, it is best to draw circle 625 + 300 = 925
diagrams (or Venn diagrams) to represent As a percentage of those who speak
H
the data before going on to answer the only the local language it is
questions. We find the main categories to 925
_T

be speakers of local language (LL), English × 100 = 67 approximately.


1375
(E) and Hindi (H). So one circle represents Answer response (a) is correct.
each language. As some speakers speak
more than one language, the circles need 4. The ratio of those who speak all the
C

to overlap, and the common area among three languages to those who speak the
the three will represent those who speak local language and Hindi
PS

all the three languages. So we have: = 300 : 625


= 12 : 25
Answer response (d) is correct.
U

5. Just add the numbers to the correct


E H
all three categories and be alert to the categories
asked for.
@

languages
300 New number for speakers of all three
200 625 languages
= 300 + 25 = 325
LL and E LL and H New number for speakers of Hindi and
1375 the local language
LL = 625 + 45 = 670
The difference between the categories
= 670 – 325 = 345
Answer response (e) is correct as the
Total residents = 2500
responses do not include 345.
Unit Eight : Practice Session ✫ 543

PRACTICE SESSION
Practice Exercise 45
Directions (Qs. 1–5): Read the following table and (a) 9,90,000 (b) 5,49,000
answer the questions. (c) 54,900 (d) 99,000
(e) None of these
Internet owners in our country
(Approximate) Directions (Qs. 6–10): Read the following table

TS
carefully and answer the questions given below it.
Years Govt. Private
1995-96 3,900 — Details of leading openers’ performance
in 20 one-day cricket matches
1996-97 29,400 —

H
1997-98 90,000 — Openers Total Highest Runs Scored in
scores number of matches

G
1998-99 2,30,000 12,000
100 or more 50-99 0’s
1999-2000 5,20,000 1,20,000
A 994 141 5 3 1
2000-2001
2001-2002
10,60,000
15,50,000
4,50,000
9,50,000
U B
C
751
414
130
52
1

8
2
2
3
O
(Expected) D 653 94 — 4 1
E 772 85 — 7 —
1. What is the approximate percentage increase
H
expected in the private Internet owners in 6. What is the difference between the average
the period 2001-2002 over that in the period runs of the top two openers in terms of
_T

1998-99? highest scores, if matches having 0’s were


(a) 9,000 (b) 6,000 ignored?
(c) 7,000 (d) 4,000 (a) 10.59 (b) 13.7
(e) 8,000
(c) 11.1 (d) 16.62
C

2. What is the proportion of government (e) None of these


Internet owners to the private Internet
PS

7. If matches having zero runs and the highest


owners in the period 1999-2000?
runs are ignored, what will be the average
(a) 13 : 4 (b) 13 : 3
(c) 3 : 13 (d) 4 : 13 runs for opener C?
(a) 21.29 (b) 21.79
U

(e) None of these


(c) 20.7 (d) 21.17
3. What is the approximate percentage of (e) 20.19
@

private Internet owners to the total Internet


owners in 1998-99? 8. By how much does the difference between
(a) 20 (b) 50 the two highest total runs differ from
(c) 10 (d) 15 the difference between the two lowest total
(e) 5 runs?
4. In which period is the percentage increase (a) Lower by 18 (b) More by 17
in the total Internet owners least over the (c) Lower by 4 (d) More by 4
earlier period? (e) Lower by 17
(a) 1997-98 (b) 1998-99 9. Which of the given pairs of openers have a
(c) 1999-2000 (d) 2000-2001 ratio of 3 : 2 in their highest runs?
(e) None of these (a) B and D (b) B and C
5. What is the total number of fresh Internet (c) A and D (d) D and C
owners expected in the period 2001-2002? (e) None of these
544 ✫ Quantitative Aptitude

10. Excluding the matches with 50-99 runs, what Directions (Qs. 16–20): Study the following table
will be the approximate average runs for carefully and answer the questions given below it.
opener B? Number of students from various schools
(a) 25 (b) 15 playing various games
(c) 50 (d) 60
(e) 63 Schools
Directions (Qs. 11–15): Study the following table Games A B C D E
carefully and answer the questions given below it.
Football 125 250 100 175 250
Basketball 175 200 195 245 225

TS
Percentage of marks obtained by
seven students in six subjects Cricket 250 200 225 215 200
Tennis 240 210 200 130 165
Subject and Maximum Marks Badminton 75 125 55 45 100
Student History Geography Economics Maths Science English

H
(75) (75) (50) (100) (150) (100) 16. If 20% of the students playing football from
School A also play badminton, what would
A 76 84 82 89 72 71

G
be the total number of students playing
B 82 72 69 77 74 65
badminton from School A?
C 56 64 75 83 84 66
(a) 110 (b) 120
D
E
F
66
84
75
68
92
78
58
87
86
73
88
84
75
82
78
55
75
69 U (c) 95
(e) None of these
(d) 100
O
G 78 69 65 76 75 58 17. The number of students playing basketball
from School C is approximately what per
H
11. What are the total marks obtained by student cent of the students playing basketball from
C in all the subjects together? School E?
(a) 408 (b) 402 (a) 75 (b) 87
_T

(c) 408.50 (d) 412 (c) 94 (d) 70


(e) None of these (e) 81
12. What is the difference in the total marks 18. What is the difference between the average
C

obtained by students G and F in all the number of students playing cricket from all
subjects together? the schools and the average number of
PS

(a) 40.5 (b) 43.5 students playing tennis from all the schools?
(c) 37.5 (d) 38.5 (a) 31 (b) 26
(e) None of these (c) 29 (d) 33
13. What are the average marks obtained (e) None of these
U

(rounded-off to two decimals) by all the 19. The number of students playing football
students in Science? from School D is what per cent of the total
@

(a) 115.71 (b) 201.11 number of students playing all the given
(c) 105.74 (d) 116.21 games from that school? (Rounded off to 2
(e) None of these digits after decimal)
14. What is the percentage of marks (rounded- (a) 20.61 (b) 21.59
off to two decimals) obtained by student E (c) 22.60 (d) 20.59
in all the subjects together? (e) 21.60
(a) 84.15 (b) 83.90 20. What is the difference between the average
(c) 81.57 (d) 73.15 number of students playing all the given
(e) None of these games from School B and the number of
15. What are the total marks obtained by student students playing badminton from the
B in History and Economics together? school?
(a) 113.25 (b) 65 (a) 72 (b) 65
(c) 96 (d) 151 (c) 78 (d) 69
(e) None of these (e) None of these
Unit Eight : Practice Session ✫ 545

Directions (Qs. 21–25): Study the following table Directions (Qs. 26–30): Study the following table
carefully and answer the questions given below it. carefully and answer the questions given below it.
Annual income of various individuals Price per kilogram (in Rs) at which 5 different
over the years farmers sell 6 different products
(Income in lakh rupees)
Farmers
Years Individuals
Products P Q R S T
A B C D E F
Rice 20 15 20.5 24 22

TS
2002 5.50 3.20 4.80 6.35 6.15 3.50 Wheat 18 16 15 15.5 20
2003 5.65 3.25 4.85 6.55 6.25 3.65 Jowar 16.5 15 18 20 15
2004 5.7 3.725 4.95 7.155 6.425 3.75 Bajra 15 14 14.5 13.5 12
2005 6.20 4.25 5.10 7.35 7.15 4.0 Maize 13 14 12.5 12 14.5

H
2006 6.50 4.50 5.20 7.40 7.25 4.25 Sugarcane 10 8 11.5 10.51 8
2007 6.75 5.0 5.25 7.48 7.285 4.80

G
2008 7.0 5.35 5.30 8.0 7.30 5.10 26. What is the difference between the average
price per kg of maize sold by all the farmers
21. What is the approximate average monthly
income of A over the years?
(a) Rs 44,762 (b) Rs 48,348 U and the average price per kg of sugarcane
sold by all the farmers?
(a) Rs 3 (b) Rs 4.5
O
(c) Rs 53,216 (d) Rs 51,547 (c) Rs 4.2 (d) Rs 3.6
(e) Rs 42,102 (e) None of these
H
22. What is the ratio of the annual income of B 27. What is the ratio of the total price (per kg) of
to that of C in the year 2005? all the products sold by Q to that sold
_T

(a) 6 : 5 (b) 6 : 7 by R?
(c) 5 : 6 (d) 7 : 6 (a) 41 : 43 (b) 43 : 48
(e) None of these (c) 43 : 46 (d) 41 : 48
C

(e) 41 : 46
23. What is the difference between the total
PS

annual income of all the individuals in the 28. If farmer Q sells 14 kg of wheat, 21 kg of
year 2003 and the total annual income of all jowar and 30 kg of sugarcane, what will be
the individuals in the year 2007? his total earnings?
(a) Rs 7,36,500 (b) Rs 6,45,400 (a) Rs 699 (b) Rs 779
U

(c) Rs 6,36,500 (d) Rs 7,43,300 (c) Rs 650 (d) Rs 786


(e) None of these (e) None of these
@

24. The annual income of E in the year 2004 is 29. If farmer P sells 50 kg of rice and farmer S
sells 60 kg of wheat, what is the difference
approximately what per cent of the average
in the amount earned by the farmers in
annual income earned by E over the years?
selling their products?
(a) 98 (b) 86
(a) Rs 70 (b) Rs 65
(c) 88 (d) 94
(c) Rs 80 (d) Rs 85
(e) 91
(e) None of these
25. What is the average annual income of F over 30. If each farmer sells 100 kg of jowar and 50
the years? kg of bajra, which farmer will earn the most?
(a) Rs 4,16,000 (b) Rs 4,25,000 (a) P (b) R
(c) Rs 4,01,500 (d) Rs 4,15,500 (c) S (d) Q
(e) Rs 4,15,000 (e) T
546 ✫ Quantitative Aptitude

Directions (Qs. 31–34): Study the following table Directions (Qs. 35–39): Study the following table
carefully and answer the questions given below it: to answer the given questions.
Percentage population below poverty line Centre and post-wise number of candidates
in six different towns and proportion
of males and females Post Officer Clerk Field Supervisor Specialist
Centre Officer Officer
Percentage Proportion of Males
and Females Bengaluru 2,000 5,000 50 2,050 750

Town Below Below Above Delhi 15,000 17,000 160 11,000 750

TS
Poverty Line Poverty Line Poverty Line Mumbai 17,000 19,500 70 7,000 900
M F M F Hyderabad 3,500 20,000 300 9,000 1,150
Kolkata 14,900 17,650 70 1,300 1,200
A 12 3 : 2 4 : 3

H
Lucknow 11,360 15,300 30 1,500 650
B 15 5 : 7 3 : 4 Chennai 9,000 11,000 95 1,650 500

G
C 25 4 : 5 2 : 3
D 26 1 : 2 5 : 6
35. In Kolkata, number of candidates for post of
specialist officers is approximately what per
E
F
10
32
6
2
:
:
5
3
3
4
:
:
2
5
U cent of the number of candidates for post of
officers?
O
(a) 8.7 (b) 9
31. If the total population of town A is 3,000, (c) 6.5 (d) 8
H
what is the approximate number of females (e) None of these
above poverty line in the town?
36. What is the difference between the total
(a) 1,700 (b) 2,112
_T

number of candidates for the posts of officers


(c) 1,100 (d) 1,950
and clerks?
(e) None of these
(a) 31,690 (b) 34,180
C

32. If the population of towns C and D together (c) 32,690 (d) 28,680
is 18,000, what is the total number of female (e) None of these
PS

members below poverty line in the two


37. In Chennai, the number of candidates for
towns?
post of clerk is approximately how much per
(a) 5,000 (b) 5,500
cent more than that for the post of officers?
U

(c) 4,800 (d) Data inadequate


(a) 18 (b) 22
(e) None of these (c) 20 (d) 21
@

33. If the number of male members below (e) None of these


poverty line for town B is 500, what is the 38. Which centre has 300% more number of
total population of that town? candidates for the post of clerk as compared
(a) 14,400 (b) 5,400 to Bengaluru?
(c) 8,000 (d) 9,800 (a) Lucknow (b) Hyderabad
(e) None of these (c) Delhi (d) Chennai
34. If the number of females above poverty line (e) None of these
for town E is 19,800, what will be the number 39. Which centre has the highest number of
of male members below poverty line? candidates?
(a) 5,500 (b) 3,000 (a) Chennai (b) Kolkata
(c) 2,870 (d) Data inadequate (c) Hyderabad (d) Mumbai
(e) None of these (e) None of these
Unit Eight : Practice Session ✫ 547

Directions (Qs. 40–44): Study the table carefully to Directions (Qs. 45–48): Study the table carefully to
answer the questions that follow. answer the questions that follow.
Percentage of marks obtained by six students Number of students (in lakhs)
in six different subjects studying in various standards
in different states
Subjects
Students Maths English Science Hindi Social Studies Marathi Standard
(out (out (out (out (out (out State
of 150) of 75) of 125) of 50) of 100) of 25) V VI VII VIII IX X

TS
A 74 68 62 68 81 74 P 2.5 2.8 4.3 2.9 4.1 4.3
B 64 72 82 68 63 66 Q 3.2 2.9 4.1 3.0 3.6 2.7
C 72 84 78 66 77 70 R 2.3 3.2 3.8 3.5 2.9 3.7

H
D 78 82 64 70 69 84 S 4.8 2.4 3.6 3.4 3.4 4.3
E 82 64 84 72 65 60 T 3.1 3.9 4.7 4.0 3.4 4.0

G
F 68 72 74 74 83 80 U 3.3 4.2 2.7 4.1 3.6 3.8

40. If to pass in the exam, a minimum of 54


marks in English and a minimum of 93
U
45. Which standard has the highest total number
of students from all states together?
O
marks in Science are required, how many (a) VII (b) VIII
students passed the exam? (c) X (d) IX
(a) One (b) Four
H
(e) None of these
(c) Two (d) Three
46. What is the ratio of the number of students
(e) None of these
_T

studying in Standard VI of State P to those


41. Which student scored the highest marks in studying in Standard X of State U?
all subjects together?
7 14
(a) F (b) E (a) (b)
C

8 19
(c) B (d) C
(e) None of these 6 12
PS

(c) (d)
7 13
42. What are the average marks obtained by all
students together in Hindi? (Rounded off to (e) None of these
two digits after decimal)
U

47. Number of students studying in Standard IX


(a) 35.02 (b) 32.68 of State T forms what per cent of the total
(c) 31.33 (d) 30.83
@

number of students from that state?


(e) None of these
(Rounded off to two digits after decimal)
43. What is the overall percentage of marks (a) 16.84 (b) 12.95
obtained by F in all subjects together? (c) 14.72 (d) 15.34
(a) 74 (b) 72
(e) None of these
(c) 75 (d) 78
(e) None of these 48. What is the average number of students
studying in Standard V from all states
44. What are the total marks obtained by B in
Maths and Social Studies together? together ?
(a) 153 (b) 159 (a) 18,00,000 (b) 32,00,000
(c) 146 (d) 149 (c) 1,80,000 (d) 3,20,000
(e) None of these (e) None of these
548 ✫ Quantitative Aptitude

Answers with Solutions


1. Ans. (e) This can be calculated in the following way:
Sol. Percentage increase expected in private Years Total Required percentage
Internet owners in the period 2001- Internet
2002 over that in the period 1998-99 owners
Owners in 2001 - 02 1995-96 3900
– Owners in 1998 - 99
= × 100 29400 – 3900
Owners in 1998 - 99 1996-97 29400 × 100 = 654%
3900

TS
950000 – 12000
= × 100 90000 – 29400
12000 1997-98 90000 × 100 = 206%
29400
= 7816.67 242000 – 90000
× 100 = 169%

H
or 8,000 (approximate). 1998-99 242000
90000
2. Ans. (b) 640000 – 242000
1999-00 640000 × 100 = 164%

G
Sol. Proportion of government Internet 242000
owners to private Internet owners in 1510000 – 640000
1999-2000 2000-01 1510000 × 100 = 136%

=
520000
120000 U 2001-02 2500000
640000
2500000 – 1510000
× 100 = 66%
O
1510000
52 13
= = or 13 : 3. From the above table, we can see that
12 3
H
percentage increase is least for the period
3. Ans. (e) 2001-2002.
Sol. Private Internet owners in 1998-99
_T

[Fast-track: A good look at the numbers


= 12000 will help you estimate that the difference
Total Internet owners in 1998-99 between the figures of one year over the
= private Internet owners previous years (government only till 1997-
C

+ government Internet owners 98, and government + private from then on)
= 12000 + 230000 declines over the period. It is least for 2001-
PS

= 242000 2002. Consequently, the percentage increase


Percentage of private Internet owners will also be least for 2001-2002. And when
to total Internet owners in 1998-99 you add and subtract the figures, use the
first digits only (i.e., leave out the
U

12000
= ×100 thousands)—as you need just a rough
242000
12 figure.
@

= × 100
242 29 − 3 26
1996-97 → =
= 4.95% or 5% (approximate). 3 3
90 − 29 61
4. Ans. (e) 1997-98 → =
Sol. Percentage increase in total Internet 29 29
owners to number of Internet owners (230 + 12) − 90
1998-99 →
in earlier period 90
Total Internet owners in a 242 − 90 152
= =
90 90
period – Total Internet
(520 + 120) − 242
owners in previous period 1999-2000 →
= × 100 242
Total Internet owners in 640 − 242 398
previous period = =
242 242
Unit Eight : Practice Session ✫ 549

(1060 + 450) − 640 No. of matches


2000-2001 → = Total No. of matches
640
1510 − 640 870 – matches having 0’s
= = = 20 – 2 = 18 matches
640 640
∴ Average runs of Opener B
(1550 + 950) − 1510
2001-2002 → 751
1510 =
18
2500 − 1510 990
= = = 41.72 runs
1510 1510 ∴ Difference between average runs of the

TS
Only the 2001-2002 figure shows a two top openers
numerator lower than the denominator, = (52.32 – 41.72)
indicating it is the least.] = 10.59 runs.

H
5. Ans. (a) 7. Ans. (a)
Sol. Total number of Internet owners in Sol. Average runs for C
2000-2001 = 1060000 + 450000 =

G
Total runs – highest runs
1510000 =
Number of fresh Internet owners Total number of matches
expected in the period 2001-2002
(1550000 + 950000) – 1510000
U =
– matches with 0’s
414 – 52
O
= 2500000 – 1510000 20 – 3
= 990000 362
=
[Fast-track Calculate without 000’s and
H
17
add the 000 later. = 21.29.
The required number is
_T

8. Ans. (e)
(1550 – 1060) + (950 – 450)
Sol. Two highest total runs are 994 and 772.
= 990
Difference between two highest total
Or (1550 + 950) – (1060 + 450)
runs
C

= 990
= 994 – 772 = 222
Now add the 000 and get 990000.]
Two lowest total runs are 653 and 414.
PS

6. Ans. (a) Difference between the two lowest total


Sol. Two top openers are A and B (in terms runs
of highest runs) = 653 – 414 = 239
Opener A: Difference between the two differences.
U

Total Runs (lowest – highest)


Average runs =
Number of matches = 239 – 222 = 17.
@

Total Runs of A= 994 The difference of the two highest total


Number of matches runs is thus lower by 17 runs from the
= Total number of matches difference between the two lowest runs.
– matches having 0’s 9. Ans. (c)
= 20 – 1 = 19 matches Sol. Try out the options.
∴ Average runs of Opener A Option (a): B and D
994 Ratio between their highest runs
=
19 130
= 52.32 runs =
94
Opener B:
Total Runs 65
Average runs = = or 65 : 47
No. of matches 47
Total Runs = 751 ∴ Option (a) is not the answer.
550 ✫ Quantitative Aptitude

Option (b): B and C 12. Ans (d)


Ratio between their highest runs Sol.
130 Subjects Marks obtained Marks obtained
=
52 by G by F
65
= or 65 : 26 78 75
26 History ×75 = 58.50 ×75 = 56.25
100 100
∴ Option (b) is not the answer
Option (c): A and D 69 78
Required ratio Geography ×75 = 51.75 ×75 = 58.50
100 100

TS
141
= 65 86
94 Economics ×50 = 32.50 ×50 = 43.00
100 100
3

H
= or 3 : 2
2 76 84
Maths ×100 = 76 ×100 = 84
∴ Option (c) is the required answer. 100 100

G
There is no need of further calculations
because we have ascertained that the 75 78
Science ×150 = 112.50 ×150 = 117
answer is option (c). 100 100
10. Ans (e)
Sol. Average runs for Opener B U English
58
× 100 = 58
69
×100 = 69
O
100 100
Total runs
=
Number of matches Total Marks 389.20 427.75
H
– matches with 50 - 99 runs
751 Difference in total marks obtained by F and G
=
_T

20 – 8 = 427.75 – 389.25
751 = 38.5 marks.
= 13. Ans (a)
12
C

Sol.
= 62.58 or 63 (approximate).
Students Marks obtained in Science
11. Ans (e)
PS

Sol. Note that the table gives percentage 72


of marks, but you need to calculate A ×150 = 108
100
actual marks from the given maximum
marks. 74
B ×150 = 111
U

Marks obtained = % of marks in the 100


subject × Maximum Marks by C 84
C ×150 = 126
@

56 100
History ×75 = 42 75
100 D ×150 = 112.50
64 100
Geography ×75 = 48
100 82
75 E ×150 = 123
Economics ×50 = 37.50 100
100 78
83 F ×150 = 117
Maths ×100 = 83 100
100
84 75
Science ×150 = 126 G ×150 = 112.50
100 100
66 Total marks 810
English × 100 = 66
100 Average marks obtained by all students
Total marks 402.50
in Science
Unit Eight : Practice Session ✫ 551

Total marks obtained by all students 16. Ans (d)


= Sol. Students from School A playing football
No. of students
810 and badminton
= = 115.71. 20
7 = × 125 = 25
100
14. Ans (b) ∴ Total number of students playing
Sol. First the actual marks have to be badminton from School A is
obtained and then the percentage found = 25 + 75 = 100.
out.
Subjects Marks obtained by E 17. Ans (b)

TS
84 Sol. Let x be the required percentage
History ×75 = 63 x
100 195 = × 225
92 100

H
Geography ×75 = 69.00 195
100 × 100 = 86.66
x=
87 225

G
Economics ×50 = 43.50
100 = 87% (approximately).
88
18. Ans (c)
Maths

Science
100
82
×100 = 88

×150 = 123 U Sol. Average number of students playing


cricket is
O
100 250 + 200 + 225 + 215 + 200
75 =
5
English ×100 = 75
H
100 1090
= = 218
Total marks obtained 461.50 5
_T

Average number of students playing


Sum of maximum marks in all subjects tennis is
= 75 + 75 + 50 + 100 + 150 + 100 240 + 210 + 200 + 130 + 165
=
= 550 5
C

945
Required percentage of marks = = 189
5
PS

Total marks obtained by Difference = 218 – 189 = 29.


E in all subjects 19. Ans (e)
= 100 ×
Sum of maximum marks
Sol. If x is the required percentage, then by
U

in all subjects
the given facts,
461.50 x
= ×100 = 83.90%.
@

550 175 = × (175 + 245 + 215 + 130 + 45)


100
17500
15. Ans (c) ∴ x= = 21.60%.
810
Sol. Marks obtained by B in History
20. Ans (a)
82
= × 75 = 61.5 Sol. Average number of students playing all
100
games from school B
Marks obtained by B in Economics 250 + 200 + 200 + 210 + 125
69 =
5
= × 50 = 34.5
100 985
= = 197
∴ Total marks obtained by B in History and 5
Economics together No. of students playing badminton from
= 61.5 + 34.5 the school = 125
= 96 marks. Difference = 197 – 125 = 72.
552 ✫ Quantitative Aptitude

21. Ans (d) 25. Ans (e)


Sol. Average monthly income of A over the Sol. Average annual income of F over the
years is years
5.50 + 5.65 + 5.7 + 6.20 3.50 + 3.65 + 3.75 + 4.0 + 4.25 + 4.8 + 5.1
=
+ 6.50 + 6.75 + 7 7
= lakh 29.05
7 × 12 = = 4.15 lakh
43.3 7
= lakh = Rs 415000.
7 × 12 26. Ans (d)

TS
= 0.515476 lakh Sol. Average price per kg of maize
= Rs 51547.6 13 + 14 + 12.5 + 12 + 14.5
=
≈ Rs 51547 (approximately). 5
66

H
22. Ans (c) = = Rs 13.20
Sol. Required ratio 5
Average price per kg of sugarcane
Annual income of B in the year 2005

G
= 10 + 8 + 11.5 + 10.5 + 8
Annual income of C in the year 2005 =
5
=
4.25 5
=
5.10 6
U =
48
5
= Rs 9.60
Required difference = 13.20 – 9.60
O
[4.25 ÷ 5.10 = 0.8333.
In such question, one can try out the = Rs 3.60.
options: (a) 6 ÷ 5 = 1.2, so it is not correct;
H
27. Ans (e)
(b) 6 ÷ 7 = 0.857; (c) 5 ÷ 6 = 0.833. So (c) Sol. Ratio of the total price (per kg) of
is correct.] all the products sold by Q to those
_T

23. Ans (c) sold by R


Sol. Annual income of all in 2003 15 + 16 + 15 + 14 + 14 + 8
= 5.65 + 3.25 + 4.85 + 6.55 + 6.25 + 3.65 =
20.5 + 15 + 18 + 14.5 + 12.5 + 11.5
C

= 30.20 lakh 82
Annual income of all in 2007 = = 41 : 46.
92
PS

= 36.565 lakh
28. Ans (b)
Difference = 36.565 – 30.20
Sol. Total earnings of farmer Q
= 6.365 lakh
= (14 × 16) + (21 × 15) + (30 × 8)
= Rs 636500.
U

= 224 + 315 + 240


24. Ans (d) = Rs 779.
@

Sol. Annual income of E in 2004 29. Ans (a)


= 6.425 lakh Sol. Difference in amount
Average annual income of E = (50 × 20) – (60 × 15.5)
6.15 + 6.25 + 6.425 + 7.15 = 1000 – 930
+ 7.25 + 7.285 + 7.30 = Rs 70.
=
7 30. Ans (c)
47.810
= = 6.83 lakh Sol. P’s earnings = (100 × 16.5) + (50 × 15)
7 = Rs 2400
Required percentage Q’s earnings= (100 × 15) + (50 × 14)
6.425 = Rs 2200
= × 100
6.83 R’s earnings = (100 × 18) + (50 × 14.5)
= Rs 2525
= 94.07
S’s earnings = (100 × 20) + (50 × 13.5)
= 94% (approximately).
= Rs 2675
Unit Eight : Practice Session ✫ 553

T’s earnings = (100 × 15) + (50 × 12) 33. Ans. (c)


= Rs 2100 Sol. Let x be the population of town B
∴ S earns the most. No. of people below poverty line
[Fast-track: A look at the figures will = 15% of x
tell you farmer S gets the most for 15
= x
jowar as his price is highest—100 × 20 = 100
Rs 2000. Selling 50 kg of bajra at No. of male members below poverty line
Rs 13.5/kg, he gets Rs 675; so he makes 5 15
a total of Rs 2675. His nearest competitor = × ×x
12 100

TS
is R with Rs 18/kg for jowar and
Given that number of male members
Rs 14.5/kg for bajra. He gets a total of
below poverty line for town B = 500
1800 + 725 = Rs 2525. So S earns the
5 15
most.] ⇒ 500 = × ×x

H
12 100
31. Ans. (c) 1 1
Sol. Total population of town A = 3000 ⇒ 500 = × ×x

G
4 4
Percentage population below poverty
⇒ x = 500 × 4 × 4
line =12

Population below poverty line
= 12% × 3000 U ∴
x = 8000
Total population of town B = 8000.
O
= 360 [Fast-track: Proportion of males below
∴ Population above poverty line 5
= 3000 – 360 poverty line is
H
12
= 2640 If 15% is below poverty line, males form
5
_T

[Another way to get this figure: If 12% is 25


× 15 = %.
12 4
below poverty line, 100 – 12, i.e. 88% is
25
88 If males are below poverty line, total
C

above poverty line. So × 3000 = 2640 4


100
people are above poverty line.] population is 100.
PS

Number of females above poverty line If 500 males are below poverty line, total
3 population is
= × 2640 100 × 4
4 +3 × 500 = 8000.]
U

3 25
= × 2640 34. Ans. (b)
7
Sol. No. of females above poverty line for
@

= 1131.43.
town E = 19800 ... (i)
Or 1100 (approximately).
Let x be the total population of town E
32. Ans. (d) No. of people below poverty line
Sol. Total population of C and D
= 10%x
= 18000
10
Let population of towns C and D be a = x = 0.1x
and b respectively 100
∴ No. of people above poverty line
⇒ a + b = 18000
= x – 0.1x
⇒ a = 18000 – b
∴ People below poverty line in town = 0.9x
C = 25% (18000 – b) No. of females above poverty line
2
But as the value of neither a nor b is = ×0.9 x
given, data is inadequate. 5
554 ✫ Quantitative Aptitude

2 1200
But ×0.9 x = 19800 [from (i)] = × 100 = 8.05%
5
14900
1.8 x or 8% (approximately).
⇒ = 19800
5
36. Ans. (c)
19800×5 Ans. Total of all officers and all clerks.
⇒ x=
1.8
Officers Clerks
⇒ x = 55000
2000 5000
∴ Total population of town E = 55000 15000 17000

TS
∴ No. of people below poverty line 17000 19500
= 0.1 × 55000 3500 20000
= 5500 14900 17650
11360 15300

H
∴ No. of males below poverty line
9000 11000
6
= ×5500

G
11 Total 72760 105450
= 3000. Difference between total number of
[Fast-track: Find the total population of
town E. U clerks and officers
= 105450 – 72760
O
Proportion of females above poverty line = 32690.
2 37. Ans. (b)
H
in E is Sol. The percentage of no. of clerks more
5
Percentage of population above poverty than that of officers in Chennai
_T

line in E = 100 – 10 = 90. No. of clerks – No. of officers


2 = × 100
180 No. of officers
∴ of 90 or % females are above
5 5 11000 – 9000
poverty line. = × 100
9000
C

We know number of females above


poverty line is 19800. 2000
= × 100
PS

180 9000
Now, if are females above poverty = 22.22% or 22% (approximately).
5
line, total population is 100. 38. Ans. (b)
If 19800 are above poverty line, total Sol. No. of clerks in Bengaluru = 5000
U

5 300% more number of clerks


population = 100 × × 19800 = 55000
180 = 5000 + 300% of 5000
@

Now, if 100 is total population, males = 5000 + 15000


6 = 20000
below poverty line = × 10
11 From the table, we can see that
If 55000 is total population, males below Hyderabad is the centre which has 20000
6 1 clerks, i.e., 300% more than that of
poverty line = × 10 × × 55000
11 100 Bengaluru.
= 3000.] 39. Ans. (b)
35. Ans. (d) Sol. In fact, looking at the table, you can
Ans. In Kolkata, number of specialist officers conclude that only Delhi or Mumbai
expressed as percentage of number of could be the answer, as these centres
officers have larger numbers than the others.
Number of specialist officers Delhi is not mentioned in the option.
= × 100 But as the question does not say ‘of the
Number of officers
Unit Eight : Practice Session ✫ 555

following’ the question does not limit = 108 + 63 + 97.5 + 33 + 77 + 17.5


itself to the cities mentioned in the = 396
options. So you can get the answer just D’s marks =150 ×0.78 + 75 × 0.82+ 125 ×0.64 +
by counting up for Mumbai and Delhi. 50 × 0.70 + 100 × 0.69 + 25 × 0.84
Mumbai : 17000 + 19500 + 70 + 7000 + 900 = 117 + 61.5 + 80 + 35 + 69 + 21
= 44470 = 383.5
Delhi: 15000 + 17000 + 160 + 11000 + 750
E’s marks =150 ×0.82 + 75 × 0.64 + 125 × 0.84 +
= 43910
So, Mumbai is the correct option. 50 × 0.72 + 100 × 0.65 + 25 × 0.60
= 123 + 48 + 105 + 36 + 65 + 15

TS
(If Delhi had a higher number, the correct
option would have been ‘None of these’.) = 392
F’s marks = 150 × 0.68 + 75 × 0.72+ 125 ×0.74 +
[Just to check:
50 × 0.74 + 100 × 0.83 + 25 × 0.80
Hyderabad : 3500 + 20000 + 300 + 9000 + 1150

H
= 33950 = 102 + 54 + 92.5 + 37 + 83 + 20
Kolkata : 14900 + 17650 + 70 + 1300 + 1200 = 388.5

G
= 35120]. ∴ C scored the highest marks in all the
subjects together.
40. Ans. (c)
Sol. 54 marks in English =
54
75
× 100 = 72%
U 42. Ans. (e)
Sol. Average marks in Hindi (which you can
O
93 get from the calculations for Q. 41)
93 marks in Science = × 100 = 74.4% 34 + 34 + 33 + 35 + 36 + 37
125 =
6
H
To pass the exam a student should get a
209
minimum of 72% in English and 74.4% = = 34.83
in Science. From the table, only B and C 6
_T

Alternatively add the percentage and


have passed.
divide by 600 to get average percentage
41. Ans. (d) 68 + 68 + 66 + 70 + 72 + 74
Sol. The table gives percentages, so they have
C

600
to be converted into marks. 418
=
74 68 62 600
PS

A’s marks =150 × + 75 × + 125 × 418


100 100 100 Average marks = × 50
+ 600
(50 being the maximum marks)
68 81 74
50 × + 100 × + 25 ×
U

100 100 100 = 34.83 .


43. Ans. (a)
Alternatively, each percentage can be
@

Sol. Overall percentage of marks obtained by


converted into decimals and then multiplied F in all subjects (which you can get from
with the maximum marks; e.g., 74% is 0.74. the calculations for Q. 41).
So,
388.5
0.74 × 150 + 0.68 × 75 + 0.62 × 125 = × 100
150 + 75 + 125 + 50 + 100 + 25
+ 0.68 × 50 + 0.81 × 100 + 0.74 × 25
= 111 + 51 + 77.5 + 34 + 81 + 18.5 = 373 38850
= = 74%.
B’s marks =150 × 0.64 + 75 × 0.72 + 125 × 0.82 + 525
50 × 0.68 + 100 × 0.63 + 25 × 0.66 44. Ans. (b)
= 96 + 54 + 102.5 + 34 + 63 + 16.5 Sol. B’s marks in Maths and Social Studies
= 96 + 63 = 159.
= 366
[Refer to calculations in answer to Q. 41
C’s marks =150 × 0.72 + 75× 0.84 + 125 ×0.78 + to get B’s marks in Maths and Social
50 × 0.66 + 100 × 0.77 + 25× 0.70 Studies.]
556 ✫ Quantitative Aptitude

45. Ans. (a) 2.8 14


= = or 14 : 19.
Sol. Adding columns we get the number of 3.8 19
students in
47. Ans. (c)
Standard V = 19.2 lakh Sol. Let x be the per cent.
Standard VI = 19.4 lakh x
Standard VII = 23.2 lakh 3.4 = × (3.1 + 3.9 + 4.7 + 4 + 3.4 + 4)
100
Standard VIII = 20.9 lakh 340
Standard IX = 21 lakh ∴ x= = 14.72%.
23.1
Standard X = 22.8 lakh

TS
48. Ans. (d)
∴ Highest number of students are in Sol. Average number of students studying in
Standard VII. Standard V from all states
46. Ans. (b) 19.2

H
=
Sol. Ratio of number of students studying in 6
Standard VI of State P to those studying = 3.2 lakh

G
in Standard X of State U ⇒ 320000.

U
O
H
C _T
PS
U
@
Unit Eight : Practice Session ✫ 557

Practice Exercise 46
Directions (Qs. 1–5): Study the given table carefully Marks obtained by seven students in different
and answer the questions given below it. subjects in their annual school examination
Student
The number of candidates who appeared and Subject
qualified in an examination during A B C D E F G
1991 to 1996 from six states
Physics out of 100 65 58 73 75 68 56 51
Year 1991 1992 1993 1994 1995 1996 Chemistry out of 100 59 63 69 52 71 49 72

TS
State app. qual. app. qual. app. qual. app. qual. app. qual app. qual. Maths out of 150 132 105 141 128 119 120 135
Social Studies out of 200 138 126 162 144 150 118 132
A 5600 840 7250 925 8250 876 7856 824 8349 932 7964 853
English out of 50 32 27 33 41 29 35 37
B 7200 864 8100 840 7865 792 8425 896 7658 878 8107 940
Biology out of 150 105 98 112 106 119 107 96

H
C 4850 588 6450 650 7120 685 7763 735 6984 792 7058 827
D 6325 745 7185 795 8545 842 6987 898 5896 685 6754 746 Total out of 750 531 477 590 546 556 485 523
E 5200 640 6225 685 7962 934 7645 888 7389 843 7766 812 6. Approximately what is the maximum

G
F 6500 820 7380 860 6895 788 7844 762 8105 798 8934 911
difference between the percentage of total
Note: app. = appeared, qual. = qualified marks obtained by any two students?
1. In which of the given years did the number
of canditates who appeared from State D
have the maximum percentage of qualified U (a) 15
(c) 13
(e) None of these
(b) 10
(d) 17
O
candidates? 7. Approximately what is the average of marks
(a) 1992 (b) 1995 obtained by the seven students in Biology?
H
(c) 1993 (d) 1994 (a) 126 (b) 106
(e) None of these (c) 116 (d) 96
2. What is the difference between total (e) None of these
_T

candidates who appeared in six states in the 8. What is the difference between the
years 1991 and 1993? percentage of marks obtained by Student E
(a) 6915 (b) 10962 in Chemistry and Maths together and the
C

(c) 10692 (d) 9615 percentage of marks obtained by Student A


(e) None of these in Social Studies and English together?
(a) 6 (b) 8
PS

3. The percentage increase/decrease in the


number of the appreared to the qualified (c) 12 (d) 14
candidates in State B in 1991 is (e) None of these
(a) 1.25 (b) 2.25
Directions (Qs. 9–13): Study the table and answer
U

(c) 2.50 (d) 1.00


(e) 2.7 the questions below it.
@

4. What is the difference between total Year UNIT


qualified candidates in the six states in the A B C D E
year 1993 and 1996? 1994 156 132 98 76 125
(a) 217 (b) 172 (Initial Left Joined Left Joined Left Joined Left Joined Left Joined
(c) 168 (d) 182 Strength)
(e) None of these 1995 12 15 23 32 12 36 06 26 11 13
1996 17 18 16 14 08 19 17 28 11 15
5. What is the percentage of total qualified 1997 09 20 12 12 17 14 09 16 19 16
candidates to the total number of appeared 1998 32 40 14 17 23 35 12 23 23 14
candidates among all the six states in 1996? 1999 22 35 11 15 18 25 14 24 32 38
(a) 15 (b) 13 2000 26 32 17 21 13 18 11 19 21 36
(c) 19 (d) 11 Note: A factory was opened in 1994 with certain initial
(e) None of these strengths in different units as shown in the table. In the
beginning of the subsequent years some of the workers left
Directions (Qs. 6–8): Study the following table and some new workers were deployed. No worker left or
carefully and answer the questions given below it. joined in-between.
558 ✫ Quantitative Aptitude

9. What was the strength of Unit B in 1998? 12. What was the total strength of workers in all
(a) 142 (b) 125 the five units in 1996?
(c) 135 (d) 124 (a) 570
(e) None of these (b) 607
10. In 1999 the strength of workers was (c) 670
maximum in which unit? (d) 690
(a) C (b) D (e) None of these
(c) A (d) B 13. What was the increase/decrease in the

TS
(e) None of these strength of the workers in Unit D in 1998 with
11. The strength of workers in Unit C in 1996 was respect to its initial strength?
approximately what per cent of the strength (a) 57.37% increase

H
in Unit E in 1997? (b) 64.47% increase
(a) 98 (b) 109 (c) 64.47% decrease

G
(c) 104 (d) 110 (d) 57.37% decrease
(e) None of these (e) None of these

U
Directions (Qs. 14–18): Study the following table carefully and answer the questions given below it.
O
The number of candidates who appeared, passed and were selected in a
competitive examination from five states over the years
H
State A B C D E
Year A P S A P S A P S A P S A P S
_T

1995 850 215 25 1050 245 35 990 195 28 1080 300 36 1120 240 40
1996 880 240 20 980 230 30 650 150 28 1150 320 38 960 180 26
1997 750 180 22 1120 210 28 840 180 25 995 280 42 885 177 32
C

1998 920 290 36 890 190 32 780 160 32 975 260 39 1040 220 30
1999 960 300 32 950 225 40 1020 220 36 888 240 32 980 280 34
2000 820 250 28 1180 200 38 930 215 35 864 216 30 900 228 24
PS

A = Appeared, P = Passed, S = Selected

14. In the year 1995, which state had the lowest (a) 81 (b) 88 (c) 80
U

percentage of candidates selected over the (d) 75 (e) 85


candidates who appeared? 17. During which of the following years the
@

(a) A (b) B (c) C percentage of candidates selected over


(d) D (e) E passed is the lowest for State B?
15. During which of the following years the (a) 1995 (b) 1997
percentage of pass to appeared is the highest (c) 1998 (d) 1999
from State D? (e) None of these
(a) 1998 (b) 1999 (c) 2000 18. What is the ratio between number of
(d) 1997 (e) None of these candidates who passed from State A in 1996
16. Total number of candidates selected from to those who passed from State E in 1999?
State A is approximately what percentage of (a) 14 : 15 (b) 6 : 7
the total number of candidates selected from (c) 13 : 16 (d) 12 : 16
State B? (e) None of these
Unit Eight : Practice Session ✫ 559

Directions (Qs. 19–23): Study the following table Directions (Qs. 24–28): Study the following table
carefully to answer the questions given below it. carefully and answer the questions given below:

Number of candidates who appeared in the Number of students who passed and failed in
examination and percentage of students who five classes of a school over the years
passed from various institutes over the years Classes
Institute Year VI VII VIII IX X
Year A B C D E F Pass Fail Pass Fail Pass Fail Pass Fail Pass Fail

App. % App. % App. % App. % App. % App. % 2001 50 42 76 14 58 18 65 17 48 23

TS
Pass Pass Pass Pass Pass Pass 2002 50 19 95 22 71 30 75 12 76 28
2001 450 60 540 40 300 65 640 50 600 45 680 60 2003 45 13 61 19 49 15 48 08 74 20
2002 520 50 430 70 350 60 620 40 580 70 560 70 2004 58 21 75 25 80 28 60 11 84 14
2003 430 60 490 70 380 50 580 50 680 70 700 66
2005 55 18 66 29 59 26 70 13 65 17

H
2004 400 65 600 75 450 70 600 75 720 80 780 70
2005 480 50 570 50 400 75 700 65 700 48 560 50 2006 68 31 54 38 77 34 82 20 55 14
2006 550 40 450 60 500 68 750 60 450 50 650 60
24. What is the average number of failed

G
2007 500 58 470 60 470 60 720 70 560 60 720 50
students from class VII for the given years?
19. What is the total number of students who (a) 29.5 (b) 27
passed from all institutes together in the year
2006? U (c) 26.5
(e) 24.5
(d) 25
O
(a) 1895 (b) 1985
(c) 1295 (d) 1465 25. What is the ratio between total number of
passed students and total number of failed
H
(5 None of these
students for the year 2006?
20. Approximately, what is the overall
(a) 2 : 1 (b) 56 : 23
_T

percentage of students who passed from


institute C for all the years? (c) 69 : 13 (d) 69 : 35
(a) 60 (b) 70 (e) 336 : 137
(c) 75 (d) 55 26. Which of the following classes has the
C

(e) 65 maximum number of passed students, as


21. What is the ratio between the number of compared to the total number of students
PS

students who passed from institute F in 2003 of that class, over the years?
and the number of students who passed (a) VI (b) VII
from institute B in 2005 respectively? (c) VIII (d) IX
U

(a) 95 : 154 (b) 154 : 95 (e) X


(c) 94 : 155 (d) 155 : 94
(e) None of these 27. What is the number of passed students, for
@

all the classes together, in the year 2003?


22. What is the ratio between the average
number of students who appeared from (a) 277 (b) 297
institute A for all the years and those who (c) 315 (d) 357
appeared from institute D respectively? (e) 377
(a) 463 : 353 (b) 353 : 463 28. What is the total percentage of passed
(c) 461 : 333 (d) 333 : 461 students of class VI from all the years
(e) None of these
together?
23. What is the overall percentage of students (a) 50 (b) 71.4
who passed from all institutes together in (c) 70 (d) 79.4
2004? (rounded off to the nearest integer) (e) 90
(a) 68 (b) 70
(c) 69 (d) 71 Directions (Qs. 29–33): Study the table carefully to
(e) None of these answer the questions that follow:
560 ✫ Quantitative Aptitude

Number of items manufactured (M) and 32. Which company manufactured the highest
sold (S) (in millions) by six different number of items over all the years?
companies over the years (a) C (b) E
(c) F (d) B
Company
Year (e) None of these
A B C D E F 33. What is the number of items not sold by
M S M S M S M S M S M S Company C in the year 2003?
2003 8.5 5.3 7.3 6.6 8.0 6.0 7.6 5.2 7.5 6.1 7.8 4.5 (a) 2000 (b) 2000000
2004 8.3 6.2 7.9 6.2 8.1 5.8 8.3 5.7 8.0 6.6 7.8 5.0 (c) 200000 (d) 20000
(e) None of these

TS
2005 6.5 3.1 6.9 4.8 7.8 4.3 7.8 4.5 8.5 6.8 8.4 5.4
2006 7.2 5.2 8.3 5.3 7.9 4.6 7.9 4.8 6.7 5.4 8.2 6.2 Directions (Qs. 34–35): Study the following table
2007 7.1 5.8 8.0 5.9 7.9 4.9 6.8 5.0 7.7 4.9 8.7 6.0 and answer the next two questions.
2008 8.0 6.2 8.2 6.1 7.6 6.0 7.5 6.1 7.9 4.9 6.5 4.2
Percentage of Proportion of Males and Females

H
State Population below Below Poverty Above Poverty
29. What is the respective ratio of total number Poverty Line Line Line
of items sold by Company A over all the

G
M : F M : F
years to those sold by Company D over all
P 35 5 : 6 6 : 7
the years?
(a) 351 : 323
(c) 289 : 296
(b) 313 : 318
(d) 291 : 263 UQ
R
S
25
24
19
3
1
3
:
:
:
5
2
2
4
2
4
:
:
:
5
3
3
O
(e) None of these T 15 5 : 3 3 : 2
30. Total number of items not sold by Company 34. What will be the number of females above
H
B over all the years together is poverty line in State S if it is known that the
approximately what per cent of total number population of State S is 7 million?
_T

of items manufactured by it over all the (a) 3 million


years together? (b) 2.43 million
(a) 25 (b) 38 (c) 1.33 million
(c) 12 (d) 42 (d) 5.7 million
C

(e) 6 (e) None of these


31. Number of items sold by Company E in the 35. If the male population above poverty line
PS

years 2006 and 2007 together is what per for State R is 1.9 million, then the total
cent of the number of items manufactured population of State R is
by it in these years? (rounded off to the (a) 4.5 million
U

nearest integer) (b) 4.85 million


(a) 61 (b) 35 (c) 5.35 million
(c) 56 (d) 72 (d) 6.25 million
@

(e) None of these (e) None of these


Answers with Solutions
1. Ans. (d) 842
Sol. Here the percentage is being asked for. In 1993 ⇒ × 100 = 9.85%
8545
Percentage of qualified candidates 898
Total qualified candidates in a year In 1994 ⇒ × 100 = 12.85%
= × 100 6987
Total candidates who appeared 685
in that year In 1995 ⇒ × 100 = 11.62%
5896
745
In 1991 ⇒ × 100 = 11.77% 746
6325 In 1996 ⇒ × 100 = 11.04%
6754
795
In 1992 ⇒ × 100 = 11.06% [Fast-track Since the question does not
7185
Unit Eight : Practice Session ✫ 561

ask for the actual percentages, rough = 5600 + 7200 + 4850 + 6325 + 5200 + 6500
estimates will get you the answer. = 35675
745 Total candidates who appeared in 1993
In 1991 the ratio is = 8250 + 7865 + 7120 + 8545 + 7962 + 6895
6325
Leaving out the unit’s and ten’s digits, = 46637
7 Difference between the number of candidates
approximate the ratio ⇒ in the two years
63
Multiply the numerator by 100 and get = 46637 – 35675 = 10962.
700 3. Ans. (e)
the resultant fraction ⇒

TS
= 11 Sol. Percentage increase/decrease in the
63
(Stop with the integer; decimal values are number of appeared to qualified
not required.) candidates in State B in 1991
Now, go on Increase / decrease in those

H
who qualified
1992: 8 (79 rounded off) = × 100
Increase / decrease in those

G
71 who appeared
800
⇒ = 11 =
840 – 864
71 ×100

1993:
8
85
800
85
=9
U =
8100 – 7200
24 (decrease)
× 100 = 2.67%
O
900(increase)
1994: 9 → (89 rounded off) or 2.7% (approximately).
70 → (69 rounded off)
H
4. Ans. (b)
900
⇒ = 12 Sol. Total qualified candidates in 6 states:
70
_T

In year 1993
1995: 7 → (68 rounded off) = 876 + 792 + 685 + 842 + 934 + 788
58 = 4917
700 In year 1996
C

⇒ = 11
58 = 853 + 940 + 827 + 746 + 812 + 911
7 700 = 5089
PS

1996: = 10 Required difference


67 67
= 5089 – 4917
Clearly the ratio is highest for 1994; the
percentage will consequently also be the = 172.
U

5. Ans. (d)
highest for 1994. Sol. Percentage of total qualified candidates
(Use this method: Wherever you have a to those appeared in all the six states
@

numerator smaller than the denominator in 1996


and the actual results are not asked or Total qualified candidates
= × 100
only approximate results are required, Total candidates appeared
simplify the fraction by leaving out the Total qualified candidates in 1996
same number of digits from numerator as = 853 + 940 + 827 + 746 + 812 + 911
well as denominator starting from the = 5089
units. Then add zeros to the numerator Total appeared candidates in 1996
= 7964 + 8107 + 7058 + 6754 + 7766 + 8934
to make it just one digit more than those
= 46583
in the denominator, and divide.)]
5089
2. Ans. (b) = ×100
46583
Sol. Total candidates who appeared in six
= 10.92%
states in 1991 or 11% (approximately).
562 ✫ Quantitative Aptitude

6. Ans. (a) 9. Ans. (a)


Sol. Maximum marks = 590 Sol. Strength of Unit B in 1998
Minimum marks = 477 = Initial strength + Total number of
Difference = 590 – 477 = 113 workers who joined till 1998 – Total
The difference on total marks of 750 number of workers who left till 1998
is 113 Initial strength = 132
On 100, the difference is Total number of workers who joined till
113 1998
× 100 = 15.06 = 32 + 14 + 12 + 17 = 75
750

TS
Total number of workers who left till
As percentage the difference is
1998
approximately 15.
= 23 +16 + 12 + 14 = 65
7. Ans. (b) ∴ Strength of Unit B in 1998

H
Sol. Average marks obtained by seven = 132 + 75 – 65 = 142.
students in Biology [Fast-track: Just go down the column for B

G
Total marks obtained in Biology till 1998. Calculate mentally the difference
= between ‘left’ and ‘joined’, putting (+) if
Number of students
joined is more and (–) if left is more before
=
105 +98+112+106 +119+107 +96
7
U the difference.
1995: –23 + 32 = +9
O
743
= = 106.14 1996: –16 + 14 = –2
7 1997: –12 + 12 = 0
≈ 106.
H
1998: –14 + 17 = +3
8. Ans. (b) +10
Sol. Percentage of marks
_T

In 1998, there were 132 + 10 = 142 workers.]


Total of marks in subjects
concerned 10. Ans. (c)
= × 100 Sol.
Total maximum marks for
C

Unit Initial (+) Joined (–) Left Total


subjects concerned
Strength till 1999 till 1999 Strength
Student E (15+18+20 (12+17+9
PS

Total marks obtained in Chemistry and +40+35) +32+22)


Maths together= 71 + 119 A 156 + 128 – 92 = 192
= 190 B 132 + 90 – 76 = 146
U

Percentage of marks obtained (32+14+12 (23+16+


190
= × 100 +17+15) 12+14+11)
100 + 150
@

C 98 + 129 – 78 = 149
190
= × 100 = 76% (36+19+ (12+8+17
250
14+35+25) +23+18)
Student A
D 76 + 117 – 58 = 135
Total marks obtained in Social Studies
(26+28+16 (6+17+9
and English together = 138 + 32 = 170
Percentage of marks +23+24) +12+14)
170 E 125 + 96 – 96 = 125
= ×100
200 + 50 (13+15+16 (11+11+
170 +14+38) 19+23+32)
= ×100 = 68%
250 From above table, we can see that in 1999
∴ Required difference = (76 – 68)% strength of workers was maximum in
= 8%. Unit A.
Unit Eight : Practice Session ✫ 563

[Fast-track: Use the same shortcut method Strength in 1996


as given in solution 9 for all the units. Count A : 156 + 3 + 1 = 160
down the columns till 1999. B : 132 + 9 – 2 = 139
A B C D E
C : 98 + 24 + 11 = 133
D : 76 + 20 + 11 = 107
1995 –12+15 –23 + 32 –12 + 36 –6 + 26 –11 + 13 E : 125 + 2 + 4 = 131
= +3 = +9 = +24 = +20 = +2
1996 +1 –2 +11 +11 +4
Total strength 670
1997 +11 0 –3 +7 –3 13. Ans. (b)
1998 +8 +3 +12 +11 –9 Sol. Increase/decrease in the strength of

TS
1999 +13 +4 +7 +10 +6 workers in Unit D in 1998
Total +36 +14 +51 +59 0 (26 + 28 + 16 + 23) – (6 + 17 + 9 + 12)
+1994 156 132 98 76 125 = 93 – 44

H
= 49 (i.e., increase)
Just a look will tell you Unit A has the
Percentage of net increase in workers in
maximum strength.]
Unit D to that of its initial strength

G
11. Ans. (c) 49
Sol. Strength of workers in Unit C in 1996 = × 100
76
= Initial strength + Number of workers who
joined till 1996 – Number of workers left
U = 64.47% increase.
[Fast-track: Use the chart for solution 10. In
O
till 1996 1998 there is a net increase of
= 98 + (36 + 19) – (12 + 8) 20 + 11 + 7 + 11 = 49
= 98 + 55 – 20 = 133 49
H
Similarly, strength of workers in Unit E in Percentage increase = × 100
76
1997 = 64.47.]
_T

= 125 + (13 + 15 + 16) – (11 + 11 + 19)


= 125 + 44 – 41 14. Ans. (c)
= 128 Sol. Percentage of candidates selected over
∴ The required percentage candidates appeared
C

Strength of workers in C in 1996 Candidates selected


= × 100
= Strength of workers in E in 1997 × 100 Candidates appeared
PS

133 This can be calculated in the following


= ×100 way—
128
= 103.9% State Percentage of candidates selected
U

or 104%(approximately). over candidates appeared in 1995


[Fast-track: You can use the chart you made 25
A × 100 = 2.94%
@

for solution to Questions 10 Fast-track and 850


calculate. 35
B × 100 = 3.33%
1996; Unit C: 98 + (24 + 11) = 98 + 35 = 133 1050
1997; Unit E: 125 + (2 + 4 – 3) = 125 + 3 = 128 28
C × 100 = 2.83%
Now the required percentage is worked out: 990
36
Unit C 133 D × 100 = 3.33%
: = 1.039 1080
Unit E 128 40
1.39 × 100 = 103.9 ⇒ 104% approximately.] E × 100 = 3.57%
1120
12. Ans. (c) ∴ We can see from the above table that the
required percentage was lowest for State C.
Sol. Use fast-track method, taking figures
for all units from the chart for fast-track [Fast-track: As the actual percentage is not
solution to Question 10. asked for, stop when you get the integer on
564 ✫ Quantitative Aptitude

dividing. All the candidates (B, D and E) with 17. Ans. (e)
3 may be left aside as the ‘lowest’ is asked Sol.
for. As A and C both have 2, you need to Year Percentage of candidates selected
calculate decimal places, but you can stop over those who passed from State B
when you get the first decimal place at which
point you know the answer.] 35
1995 × 100 = 14.29%
245
15. Ans. (d)
Sol. Percentage of candidates passing over 30
1996 × 100 = 13.04%
those appeared 230

TS
28
Candidates passing 1997 × 100 = 13.33%
= × 100 210
Candidates appeared
32
1998 × 100 = 16.84%
This can be calculated in the following

H
190
way— 40
1999 × 100 = 17.78%
Years Passing percentage over

G
225
appeared from State D 38
2000 × 100 = 19%
1995
300
1080
× 100 = 27.78%
U 200
∴ The lowest percentage was in 1996.
O
320 [Calculate decimal places only for 1996 and
1996 × 100 = 27.83%
1150 1997 because the integers are the same, and
H
280 stop at one decimal place.]
1997 × 100 = 28.14%
995
18. Ans. (b)
260
_T

1998 × 100 = 26.67% Sol. Ratio between number of condidates who


975 passed from State A in 1996 to those who
240 passed from State E in 1999
1999 × 100 = 27.03%
888
C

No. of candidates passed from State A in 1996


216 =
2000 × 100 = 25% No.of candidates passed from State E in 1999
PS

864 240 24
∴ The required percentage is highest in the year = =
280 28
1997. 6
= or 6 : 7.
7
U

[Fast-track: As in the previous solution, here


too stop your calculations with the integer and 19. Ans. (a)
do not waste time over getting the decimal
@

Sol. Number of students successful in 2006


places.]
16. Ans. (c) 40 60 68
= ´ 550 + ´ 450 + ´ 500
Sol. Percentage of total number of candidates 100 100 100
selected from State A to that of State B 60 50 60
+ ´750 + ´ 450 + ´ 650
Total number of candidates 100 100 100
selected from State A = 220 + 270 + 340 + 450 + 225 + 390
= × 100
Total number candidates = 1895.
selected from State B
25 + 20 + 22 + 36 + 32 + 28 20. Ans. (e)
= × 100
35 + 30 + 28 + 32 + 40 + 38 Sol. Number of students who passed from
163 institute C
= × 100
203 65 60 50 70
= ´ 300 + ´ 350 + ´ 380 + ´ 450
= 80.29 or 80% (approximately). 100 100 100 100
Unit Eight : Practice Session ✫ 565

= 195 + 210 + 190 + 315 + 300 + 340 + 282 24. Ans. (e)
= 1832 Sol. Average number of failed students
Total number of students from class VII
= 300 + 350 + 380 + 450 + 400 + 500 + 470 14 + 22 + 19 + 25 + 29 + 38
= 2850. =
6
1832 147
Required percentage = × 100 = = 24.5.
2850 6
= 64.2 ≈ 65%. 25. Ans. (e)
21. Ans. (b) Sol. Ratio between total number of students

TS
Sol. Number of students who passed from F who passed and those who failed in 2006
in 2003 68 + 54 + 77 + 82 + 55
=
66 31 + 38 + 34 + 20 + 14
= × 700 = 462. 336

H
100 = , i.e., 336 : 137.
Number of students who passed from B 137
in 2005 26. Ans. (d)

G
50 Sol. Adding columns, we observe that in
= × 570 = 285
100 class IX 400 students have passed and
∴ Required ratio =
462
285
=
154
95
U 81 students have failed.
27. Ans. (a)
O
= 154 : 95.
22. Ans. (d) Sol. In 2003, students passed
Sol. Ratio between the average number of = 45 + 61 + 49 + 48 + 74
H
students who appeared from A and those = 277.
from D
_T

28. Ans. (c)


(450 + 520 + 430 + 400 + 480 + 550 + 500) ¸ 7 Sol. Percentage of passed students of
=
(640 + 620 + 580 + 600 + 700 + 750 + 720) ¸ 7 class VI
3330 336
C

= = 333 : 461. = × 100 = 70%.


4610 480
29. Ans. (e)
PS

23. Ans. (e)


Sol. Required ratio
Sol. Students who passed in 2004
5.3 + 6.2 + 3.1 + 5.2 + 5.8 + 6.2
65 75 70 = 5.2 + 5.7 + 4.5 + 4.8 + 5 + 6.1
= ´ 400 + ´ 600 + ´ 450
U

100 100 100


31.8
75 80 70 = = 318 : 313.
+ ´ 600 + ´ 720 + ´780 31.3
100 100 100
@

= 260 + 450 + 315 + 450 + 576 + 546 [Option (b) gives the opposite ratio.]
30. Ans. (a)
= 2597
Sol. Required percentage
Total number of students in 2004 = [(7.3 – 6.6) + (7.9 – 6.2) + (6.9 – 4.8)
= 400 + 600 + 450 + 600 + 720 + 780 + (8.3 – 5.3) + (8 – 5.9) + (8.2 – 6.1)]
100
= 3550 × (7.3 + 7.9 + 6.9 + 8.3 + 8 + 8.2)
Required percentage
100
2597 = (0.7 + 1.7 + 2.1 + 3.0 + 2.1 + 2.1) ×
= × 100 46.6
3550 Items not sold
11.7 × 100  × 100
= 73.1 or 73% = Total manufactured
46.6
(rounded off to the nearest integer). = 25.1% ≈ 25%.
566 ✫ Quantitative Aptitude

31. Ans. (d) 34. Ans. (b)


Sol. Number of items sold by E expressed as Sol. Total population of State S is 7 million
percentage of items manufactured by it Population per cent above poverty line
in 2006 and 2007 = 100 – 19 = 81
Percentage of female population above
Items sold by E in 2006 and 2007
= × 100 poverty line in State S
Items manufactured in the same years
3
100 = of 81%
= (5.4 + 4.9) × 7
(6.7 + 7.7) 3 81 243
×

TS
103 =
= × 100 = 71.52% 7 100 700
144 The number of female population above
≈ 72%. poverty line
32. Ans. (c) æ243 ö

H
= ç of 7 ÷ million = 2.43 million.
Sol. Total number of items manufactured in è 700 ø
millions by

G
35. Ans. (d)
Company A Sol. Percentage of male population above
= 8.5 + 8.3 + 6.5 + 7.2 + 7.1 + 8.0 = 45.6 poverty line in State R
Company B
U
= 7.3 + 7.9 + 6.9 + 8.3 + 8.0 + 8.2 = 46.6
æ2
= çè of 100 – 24 ÷
ö
ø%
O
5
Company C
æ2 ö
= 8.0 + 8.1 + 7.8 + 7.9 + 7.9 + 7.6 = 47.3 = çè of 76 ÷ ø%
H
5
Company D
= 7.6 + 8.3 + 7.8 + 7.9 + 6.8 + 7.5 = 45.9 Let population of State R = x million
_T

Male population of State R above poverty


Company E
line
= 7.5 + 8.0 + 8.5 + 6.7 + 7.7 + 7.9 = 46.3
2 æ 76 ö
Company F of ç x million
è 100 ÷
=
ø
C

5
= 7.8 + 7.8 + 8.4 + 8.2 + 8.7 + 6.5 = 47.4
It is given that male population above
F manufactured the hightest number of
PS

poverty line in State R is 1.9 million. So


items. we can form the equation
33. Ans. (b) 2 76
Sol. Number of items not sold by C in 2003 × × x = 1.9
5 100
U

= 8 – 6 = 2 million 5 × 100 × 1.9


= 2000000. Þ x = = 6.25 million.
76 × 2
@
Unit Eight : Practice Session ✫ 567

Practice Exercise 47
Directions (Qs. 1–4): Study the following graph 4. If the profit earned by the company in the
carefully and snwer the questions given below it. year 2008 was Rs 90,000, what was the
Profit earned by a company over the years income of the company in that year?
(a) Rs 2,90,000
80 (b) Rs 2,00,000
70 (c) Rs 2,50,000
(d) Cannot be determined

TS
60
(e) None of these
Per cent of Profit

50
Directions (Qs. 5–8): Study the following graph
40 carefully and answer the questions given below it.

H
30 Percentage profit earned by two companies
20 A and B over the given years

G
10 Company A
Company B
0
2005 2006 2007
Years
2008 2009
U Percentage of Profit 70
65
60
O
Income – Expenditure 55
Per cent Profit = × 100 50
Expenditure 45
H
1. If the expenditure of the company in the year 40
2006 was Rs 75,000, what was the ratio of 35
30
_T

income to expenditure of the company in that 25


year? 20
(a) 3 : 2 15
10
(b) 5 : 4
C

5
(c) 4 : 3 0
(d) Cannot be determined 2007 2008 2009 2010 2011 2012
PS

(e) None of these Year


2. The per cent profit earned by the company Percentage profit/loss
in the year 2009 was what per cent of the per
U

Income – Expenditure
cent profit earned by the company in the year = × 100
2005? Expenditure
@

(a) 200 5. Expenditures of Company B in 2008 and 2009


(b) 240 are Rs 12 lakh and Rs 14.5 lakh respectively.
(c) 260 What was the total income of Company B in
(d) 255
2008 and 2009 together (in lakh rupees)?
(e) None of these
(a) 39.75 (b) 37.95
3. If the income of the company in the year 2005 (c) 38.75 (d) 38.5
and the year 2007 was Rs 6,80,000 each, then (e) None of these
what was the difference in expenditures of
the company in the year 2005 and 2007? 6. Ratio of expenditures of Companies A and
(a) Rs 1,24,000 B in 2011 was 3 : 4 respectively. What was
(b) Rs 1,44,000 the respective ratio of their incomes in 2011?
(c) Rs 1,50,000 (a) 21 : 26 (b) 13 : 14
(d) Cannot be determined (c) 14 : 13 (d) 11 : 26
(e) None of these (e) None of these

567
568 ✫ Quantitative Aptitude

7. If the expenditures of Companies A and B in 11. What is the difference in the total production
2012 were equal and the total income of the of the two companies for the given years?
two companies was Rs 5.7 lakh, what was (a) 27,00,000 (b) 31,00,000
the total expenditure of the two companies (c) 2,70,000 (d) 3,90,000
in 2012? (e) None of these
(a) 4 lakh
(b) 2 lakh 12. Which of the following is the closest average
(c) 4.2 lakh production in lakh units of Company B for
(d) Can’t be determined the given years?
(e) None of these (a) 4.1 (b) 3.9

TS
(c) 4.0 (d) 3.75
8. If the incomes of Company B in 2009 and (e) None of these
2010 were in the ratio of 2 : 3 respectively,
what was the respective ratio of Directions (Qs. 13–15): Study the following graph

H
expenditures of that company in these two carefully and answer the questions given below it:
years? Per cent profit earned by two companies A

G
(a) 20 : 29 (b) 9 : 10 and B over the years 2007 to 2007
(c) 29 : 45 (d) 10 : 29
(e) None of these 100 Company A

Directions (Qs. 9–12): Study the following graph


U 90
80
Company B
OPer cent Pr ofit

to answer the given questions. 70


60
Production of two companies A and B
H
50
over the years (Production in lakh units)
40
30
_T

Company A
Company B 20
Prod uction in Lakh Un its

10
0
C

2001 2002 2003 2004 2005 2006 2007


Years
PS

13. Investment of Company B in 2007 is more


by 40% than that in the previous year. Income
in 2007 was what per cent of the investment
U

in 2006?
(a) 252% (b) 280%
2004 2005 2006 2007 2008 2009 2010 2011
(c) 242% (d) 52%
@

Years
(e) None of these
9. For Company A, what is the per cent
decrease in production from 2004 to 2005? 14. Income of Company A in 2005 was Rs 21.7
(a) 25 (b) 50 lakh. What was the investment?
(c) 75 (d) 10 (a) Rs 14 lakh (b) Rs 15.4 lakh
(e) None of these (c) Rs 15.8 lakh (d) Rs 14.6 lakh
(e) None of these
10. For Company A, in which year is the
percentage increase or decrease in the 15. Investment of Company B in 2003 was
production from the previous year the Rs 15,40,000. What was its income in that
highest? year?
(a) 2011 (b) 2005 (a) Rs 22.33 lakh (b) Rs 22 lakh
(c) 2008 (d) 2006 (c) Rs 22.23 lakh (d) Rs 23.33 lakh
(e) None of these (e) None of these
Unit Eight : Practice Session ✫ 569

Directions (Qs. 16–20): Study the following graph 20. Expenditure of Company B in 2008 was
carefully and answer the questions given below it. Rs 6.5 lakh which was equal to the income
of Company C in 2006. What was the ratio of
Per cent profit earned by three income of Company B in 2008 to the
companies A, B and C over the expenditure of Company C in 2006?
years 2004 to 2010 (a) 1 : 42 (b) 81 : 25
(Profit = Income – Expenditure) (c) 25 : 81 (d) 18 : 25
A B C (e) None of these
100
90
Directions (Qs. 21–23): Study the following graph

TS
80
carefully and answer the questions given below:
Per cent Profit

70 Percentage rise in population of six different


60 States A, B, C, D, E and F from the year 2004 to
50

H
the year 2005 and the year 2005 to the year 2006
40
30 2004 to 2005

G
20 80 2005 to 2006
10 70
70 70
Per cent Rise in Pop u lation
0
2004 2005 2006 2007
Years
2008 2009 2010

U 60
50
55 55
60
50
O
16. Income of Company C in 2009 was Rs 8.5
40
lakh. What was its income in the year 2010? 40 25
H
(a) Rs 9.35 lakh 30 30 35
(b) Rs 9.5 lakh 20
20
_T

(c) Rs 9.15 lakh 15


10
(d) Cannot be determined
(e) None of these 0
A B C D E F
C

17. Income of Company C in 2009 was States


Rs 1,68,000. What was the expenditure in 21. If the population of states C and D in 2005
that year?
PS

are in the ratio 2 : 3 respectively and the


(a) Rs 2,68,000 (b) Rs 1,50,000 population of State C in 2004 was 2.5 lakh,
(c) Rs 1,05,000 (d) Rs 1,00,800 what was the population of State D in 2005?
(e) None of these (a) 5.25 lakh (b) 4.75 lakh
U

18. Expenditure of the three companies in the (c) 3.5 lakh (d) 6 lakh
year 2010 were equal. What was the ratio of (e) None of these
@

incomes of companies A, B and C in that 22. Population of State E in 2004 was what
year? fraction of its population in 2006?
(a) 25 : 27 : 24 (b) 35 : 37 : 34 4 3
(c) 15 : 17 : 14 (d) 32 : 35 : 34 (a) (b)
5 2
(e) None of these 5 3
(c) (d)
19. Expenditure of Company B in 2006 is equal 8 4
to the income of company A in 2008. What is 2
(e)
the respective ratio of the expenditures of 3
companies B and A in 2006 and 2008 23. Population of State F in 2005 was approxi-
mately what per cent of its population in
respectively?
2006?
(a) 15 : 8 (b) 3 : 2 (a) 65 (b) 67
(c) 33 : 20 (d) 7 : 5 (b) 75 (d) 70
(e) None of these (e) 68
570 ✫ Quantitative Aptitude

Directions (Qs. 24–28): Study the following graph cent of the number of female students
carefully and answer the questions below it. passing out from colleges C and D together?
(a) 45 (b) 40
Number of Students (Males and Females) who (c) 35 (d) 50
Passed Out from Various Colleges in a Year (e) None of these
40
Directions (Qs. 29–33): Study the graph carefully to
35 × answer the questions that follow.
Number of Students

30 × Per cent Increase in Profit of Three


(in thousands)

TS
25 Companies Over the Years
×
20 × 70
× Company X

Per cent in crease in p rofit


15 60 Company Y

H
Company Z
10 50 ×
× ×
5 40 × ×

G
×
0 30 ×
A B C D E
Colleges
Males
U 20
10
O
Females 0
2003 2004 2005 2006 2007 2008
24. What is the average number of students
Years
H
(male and female) who passed out from all
the colleges together? 29. What was the per cent increase in profit of
(a) 38,000 (b) 48,000 Company Y in the year 2008 from the
_T

(c) 42,000 (d) 51,000 previous year?


(e) None of these (a) 2 (b) 10
25. The number of female students who passed (c) 20 (d) 15
(e) 14
C

out from College C is approximately what


per cent of the total number of female 30. What was the approximate per cent increase
students who passed out from all the in profit of Company Z in the year 2005 from
PS

colleges together? the previous year?


(a) 28 (b) 30 (a) 14 (b) 21
(c) 36 (d) 25 (c) 8 (d) 26
(e) 40 (e) 19
U

26. What is the difference between the total 31. If the profit earned by Company X in the
number of students passing out from year 2004 was Rs. 2,65,000, what was its
@

College A and the total number of students profit in the year 2006?
passing out from College E? (a) Rs 6,21,560 (b) Rs 4,68,290
(a) 20,500 (b) 21,000 (c) Rs 7,05,211 (d) Rs 5,00,850
(c) 10,500 (d) 10,000 (e) None of these
(e) None of these
32. What is the average per cent increase in
27. What is the respective ratio of the total profit of Company Z over the years?
number of male students to the total number 5 2
of female students who passed out from all (a) 41 (b) 41
6 3
the colleges together?
(a) 19 : 23 (b) 18 : 25 1 1
(c) 28 (d) 23
(c) 23 : 19 (d) 25 : 18 6 3
(e) None of these (e) None of these
28. The number of male students passing out 33. Which of the following statements is true
from colleges A and B together is what per with respect to the graph?
Unit Eight : Practice Session ✫ 571

(a) Company X incurred a loss in the year 37. Total exports of Company A for all the years
2004 are approximately what per cent of the total
(b) The amount of profit earned by exports of Company B for all the years?
Company Y in the years 2006 and 2007 (a) 75 (b) 128
is the same (c) 139 (d) 68
(c) Company Z earned the highest rise in (e) 72
profit in the year 2008 as compared to 38. What is the per cent increase in exports of
the other years Company C from 2004 to 2008?
(d) Profit earned by Company X in the (a) 50 (b) 33.33

TS
year 2004 is lesser than the profit (c) 150 (d) 133.33
earned by Company Z in that year (e) None of these
(e) None of these
Directions (Qs. 39–40): Study the following graph
Directions (Qs. 34–38): Study the following graph

H
carefully and answer the questions given below it:
carefully to answer these questions.
Quantity of Wheat (in thousand tonnes) Number of employees working in various

G
exported by three companies over the years departments of two different companies
Quantity of Wheat (in thousand tonnes)

1000 Company A Company B


900
800 U 9
No. of Employees (in hundreds)

8
O
700
7
600 ×
×
H
500 × × 6
400 × × 5
×
_T

300
4
200
3
100
C

0 2
2002 2003 2004 2005 2006 2007 2008
1
PS

Years
× Company A 0
Company B IT HR Marketing Finance Admin
Company C Departments
34. Per cent rise in exports from the previous
U

year was the maximum during which year 39. If the number of employees working in the
for Company B? IT departments of Company B is increased
@

(a) 2005 (b) 2004 by 20%, what would be the difference


(c) 2006 (d) 2008 between the number of employees working
(e) None of these in the IT department of Company B and the
35. What is the ratio between total exports of the Admin. department of Company A?
three companies in 2003 and 2006 (a) 500 (b) 350
respectively? (c) 700 (d) 400
(a) 41 : 29 (b) 51 : 29
(c) 29 : 51 (d) 29 : 41 (e) None of these
(e) None of these 40. What is the average number of employees
36. What are the average exports of Company B working in all the departments together in
for all the years? (in thousand tonnes Company A?
rounded off to two digits after decimal) (a) 600 (b) 585
(a) 766.67 (b) 667.14 (c) 620 (d) 615
(c) 657.14 (d) 756.57 (e) None of these
(e) None of these
572 ✫ Quantitative Aptitude

Answers with Solutions


1. Ans. (a) P + 100 I
Sol. Given. ⇒ =
100 E
Per cent profit (P)
100 × I
Income (I) – Expenditure (E) ⇒ E =
= × 100 P + 100
Expenditure (E) Substituting values for 2005 profit %
P I E and income,
⇒ = −
100 E E 100 × 680000

TS
E =
P I 25 + 100
⇒ = −1
100 E 100 × 680000
P I =
125

H
⇒ +1 =
100 E Now for 2007, in the same way,
(which is the ratio asked for)
100 × 680000

G
50 + 100 I E =
⇒ = 70 + 100
100 E
100 × 680000


(as profit per cent in 2006 is 50)
150
= =
I 3
U =
170
Difference between expenditures of the
O
100 E 2 company in 2005 and 2007
Ratio of income to expenditure in 2006
is 3 : 2. 100 × 680000 100 × 680000
H
= −
[Note that the actual value of 125 170
expenditure need not be used to get the = 544000 – 400000 = Rs 144000.
_T

answer, as only the ratio is asked for.]


4. Ans. (a)
2. Ans. (c) Sol. Given,
Sol. Let the per cent profit earned in 2009 be
C

Income – Expenditure
x per cent of profit earned in 2005. Profit % = × 100
Expenditure
x
PS

⇒ 65 = of 25 Profit (P)
100 = × 100
Expenditure (E)
(65 being profit per cent in 2009) (  Income – Expenditure is Profit)
65 × 100 90000
U

⇒ =x ⇒ 45 = × 100
25 E
6500 (Substituting the values)
@

⇒ =x 90000
25
⇒ E = × 100 = Rs 200000
⇒ 260 = x 45
The required per cent profit is 260. Now,
Income = Expenditure + Profit
3. Ans. (b) = 200000 + 90000
Sol. Given, = Rs 290000.
Profit (P) 5. Ans. (b)
Income (I) – Expenditure (E) Sol. Given expenditure of Company B
= × 100 in 2008 = Rs 12 lakh
Expenditure (E) in 2009 = Rs 14.5 lakh
P I E I Profit earned by Company B (from the
⇒ = – = −1
100 E E E graph)
P I in 2008 = 35%
⇒ +1 = in 2009 = 50%
100 E
Unit Eight : Practice Session ✫ 573

It is given, [Alternative Method


Percentage profit We are given Profit %
Income – Expenditure Income – Expenditure
= ×100 ... (i) = × 100
Expenditure Expenditure
∴ Income
For 2008,
putting the amount of expenditure and Profit × Expenditure
= + Expenditure
percentage of profit in (i), we get 100
Profit × Expenditure + 100 Expenditure
Income – 12 =
35 = ×100 100

TS
12
⇒ Income
35 ×12
Income – 12 = Expenditure (100 + per cent profit)
100 =
(Remember this 100formula.)
420

H
Income = +12 Required Income
100
= 4.2 + 12 = Income of 2008 + Income of 2009

G
= Rs 16.2 lakh 12×(100 + 35) 14.5(100 + 50)
⇒ + lakh
For 2009, 100 100
putting the amount of expenditure and
percentage profit in (i), we get
Income – 14.5 U =
1620 + 2175
100
lakh
= Rs 37.95 lakh.]
O
50 = ×100
14.5 6. Ans. (e)
50×14.5 Sol. Ratio of expenditures of companies
H
Income – 14.5 = A and B in 2011 = 3 : 4
100
Let x be the common variable
725 ∴ Expenditure of A in 2011 = 3x
_T

Income = + 14.5
100 Expenditure of B in 2011 = 4x
= 7.25 + 14.5 Profit in 2011 (from the graph)
= Rs 21.75 lakh Company A = 30%
C

∴ Total Income = (Rs 16.2 + Rs 21.75) lakh Company B = 40%


=Rs 37.95 lakh It is given,
PS

However, you do not have to write all Percentage profit


this down. Here it has been done to Income – Expenditure
explain. What you note would be = ×100 ... (i)
Expenditure
something like Putting the values for Company A in (i),
U

B 2008 Income – 3x
30 = ×100
I − 12 3x
× 100 90 x
@

35 =
12 ⇒ = Income – 3x
100
35 × 12 90 x
⇒ + 12 = I ⇒ Income = + 3x
100 100
420 90 x + 300 x
⇒ + 12 = I ⇒ Income =
100 100
⇒ 4.2 + 12 or 16.2 lakh = Incone is 2008 390 x
⇒ Income of A =
B 2009 100
I − 14.5 Putting the values for Company B in (i),
50 = × 100
14.5 Income – 4 x
50 × 14.5 ⇒ 40 = ×100
⇒ + 14.5 = I 4x
100 160 x
⇒ 7.25 + 14.5 = I ⇒ = Income – 4x
100
⇒ 21.75 = Income in 2009 160 x
⇒ Income = + 4x
Together I = 16.2 + 21.75 = Rs 37.95 lakh. 100
574 ✫ Quantitative Aptitude

160 x + 400 x If the total income of A and B is


⇒ Income of B = Rs 57000, their total expenditure
100
560 x 200
= = × 57000
100 285
∴ Ratio of incomes of A and B in 2011 = 200 × 2000 = Rs 400000
390 x or Rs 4 lakh.
100 8. Ans. (c)
= Sol. The ratio of incomes of Company B in
560 x
2009 and 2010 = 2 : 3
100

TS
390 x Let expenditure for 2009 be E1 and for
⇒ ⇒ 39 : 56 2010 be E2
560 x
Let x be the common variable
[Alternative Method ∴ Income of B in 2009 = Rs 2x
Expenditure (100 + Per cent Profit)

H
Income = and Income of B in 2010 = Rs 3x
100 Percentage profit of B:
Given ratio of expenditures of A and B in 2009 = 50

G
=3:4 in 2010 = 45
Let x be the common variable It is given
∴ Expenditure of A = Rs 3x
Expenditure of B = Rs 4x
∴ Required ratio is U Percentage profit/loss
=
Income – Expenditure
×100 ... (i)
O
3 x(100 + 30) Expenditure
Income of A For 2009,
100 putting the respective values in (i), we
H
=
Income of B 4 x (100 + 40)
get
100 2 x – Expenditure (E1 )
_T

3×130 50 = × 100
Expenditure (E1 )
= 4×140
2 x – E1
50 = ×100
39 E1
C

= 50 2x
56
= E –1
∴ Required ratio = 39 : 56.] 100 1
PS

7. Ans. (a) 2x 50 +100


Sol. Assume expenditures of A and B to be E1 = 100
Rs 100 each. By the given formula, 2x 150
E1 = 100 = 1.5
U

For A, if income for 2012 is IA and profit


for 2012 as read from the graph is 40%, 2x = 1.5E1 ... (ii)
I A − 100 For 2010,
@

40 = × 100 putting the respective values in (i), we


100
40 × 100 get
⇒ + 100 = IA
100 3x – E2
⇒ 140 = IA 45 = ×100
For B, if income for 2012 is IB and profit as E2
read from the graph is 45% 45 3x – E2
=
I − 100 100 E2
45 = B × 100
100 45 3x
45 × 100 = –1
⇒ + 100 = IB 100 E2
100 3x
⇒ 145 = IB 45+100
E2 =
Now, if IA and IB together, i.e., total 100
income of A and B, is 140 + 145 = 3x 145
Rs 285, total expenditure of A and B =
E2 100
is Rs 200. 3x = 1.45E2 ... (iii)
Unit Eight : Practice Session ✫ 575

Dividing (ii) by (iii), we get production in 2005 with the production


in 2006 is the longest or steepest in slope
2x 1.5 E1 among the lines on the graph.
=
3x 1.45 E2 ∴ The correct answer is 2006.
2 1.5 E1 11. Ans. (a)
⇒ =
3 1.45 E2 Sol. We must take the readings from the graph,
E1 2(1.45) though exact figures will not be available.
⇒ E = Difference in total production of the
2 3(1.5)
E1 two companies for the given years
2.9 29

TS
⇒ E 2 = 4.5 = 45
= Total production of Company A for the
given years – Total production of
or E1 : E2 = 29 : 45
Company B for the given years
[As only ratio is asked for, you may use
= (4 + 3 + 8 + 8 + 8 + 7 + 8 + 12) lakh units
100 in place of x. Let B’s income be Rs

H
– (1 + 1 + 1 + 2 + 3 + 5 + 7 + 11) lakh units
200 in 2009 and Rs 300 in 2010, and carry = 58 lakh units – 31 lakh units
on from there.] = 27 lakh units

G
[Alternative Method or 2700000 units.
Ratio of Incomes of B in 2009 and 2010 [Fast-track: Read the difference (the
=2:3
Income =
Expenditure (100 + per cent profit)
U gap between the two points) for each
year from the graph and add as you
O
100 go on, mentally:
Let Expenditure in 2009 be E1 and 3 + 2 + 7 + 6 + 5 + 2 + 1+ 1 = 27 (lakh)
Expenditure in 2010 be E2 ⇒ 2700000.]
H
E1 (100+ 50) 12. Ans. (d)
∴ Income in 2009 =
100 Sol. Average production of Company B for
_T

= 1.5 E1 given years


E 2 (100 + 45) Total production for given years
Income in 2010 = =
100 No. of years
= 1.45 E2 1+ 1+ 1 + 2 + 3 + 5 + 7 + 11
C

1.5 E1 2 = lakh units


⇒ = = 8
1.45 E 2 3 31 lakh units
PS

E1 2×1.45 2.9 29 = = 3.875 lakh units


8
∴ = = = or 3.9 lakh units (approximately)
E2 3×1.5 4.5 45
⇒ E1 : E2 = 29 : 45.] which is 390000.
U

9. Ans. (a)
13. Ans. (a)
Sol. Percentage decrease in production from
Sol. We know that
2004 to 2005 of Company A
@

Production in 2005 Income


– Production in 2004 = Investment + Profit on Investment
= × 100 For Company B,
Production in 2004 the per cent profit earned in 2006
4–3
= × 100 = per cent profit earned in 2007
4
1 = 80%
= × 100 Let Investment in 2006 be Rs x
4
= 25%. ⇒ Income in 2006 = x + 80% x
10. Ans. (d) 80
Sol. We can see from the given graph that in = x+ x
100
2006 the percentage increase in (100 +80)x
production from the previous year is the =
100
highest. This inference can be drawn 180 x
from the fact that the line joining the =
100
576 ✫ Quantitative Aptitude

∴ Income in 2006 = 1.8x Income


Given that investment in 2007 is more = Investment + Profit on Investment
than that in 2006 by 40% = 1540000 + 45% × 1540000
⇒ Investment in 2007 = x + 40%x 45
= 1540000 + × 1540000
40 100
= x+ x
100 = 1540000 + 693000
140 = Rs 2233000
= x or Rs 22.33 lakh
100
= 1.4x 16. Ans. (d)

TS
∴ Income in 2007 = 1.4x + (80%) × 1.4x Sol. The question gives information
regarding income of 2009. But there is
= 1.4x + 1.12x
no information about expenditure of
= 2.52x 2010. So it is not possible to determine

H
Income in 2007 expressed as per cent of income of 2010 from the given
the investment in 2006 information. The relationship Profit =

G
Income in 2007 Income – Expenditure is valid for each
= × 100 year and cannot be used for the next
Investment in 2006

=
2.52x
x
× 100
U year without adequate information.
17. Ans. (c)
Sol. Given, Income of Company C in 2009
O
= 252% = Rs 168000
Let Expenditure in 2009 be E
H
14. Ans. (a)
We know that
Sol. Given that income of Company A in 2005 Profit per cent
_T

= Rs 21.7 lakh
Let investment be Rs x lakh Income – Expenditure
= × 100
Profit per cent in 2005 = 55% Expenditure
We know that
C

168000 – E
Income ⇒ 60 = ×100
= Investment + Profit on Investment E
PS

21.7 = x + 55%x 168000 E 6


⇒ – =
55 E E 10
⇒ 21.7 = x + x 168000 6
100 ⇒ = +1
10
U

100 x + 55 x E
⇒ 21.7 = 168000 16
100 ⇒ =
E 10
@

155 x
⇒ 21.7 = ⇒ 16E = 168000 × 10
100
21.7 ×100 1680000
⇒ x= ⇒ E = = 105000.
155 16
⇒ x = 14
[Fast-track: We know that
∴ Investment of Company A in 2005
Income×100
= Rs 14 lakh. Expenditure =
100+Profit per cent
15. Ans. (a) 168000×100
=
Sol. Given that investment of Company B in 100 + 60
2003 16800000
= Rs 1540000 =
160
Per cent profit in 2003 = 45% = Rs 105000.]
We know that
Unit Eight : Practice Session ✫ 577

18. Ans. (b) 117


Sol. We know that IB
∴ Required ratio = = 10
EC 65
Income×100
Expenditure = 18
100 + Profit per cent 81
117 18
Let Income of the three companies in = × =
10 65 25
2010 be IA, IB, IC respectively Required ratio = 81 : 25.
Let Expenditure 21. Ans. (a)
= E for the three companies
Sol. Let x lakh be the common variable
I B × 100 I C × 100

TS
I A × 100 Given that population of states C and D
⇒ = =
100+75 100+85 100 + 70 in 2005 are in the ratio 2 : 3
Cancelling the common numerator 100, ∴ In 2005, population of State C = 2x lakh
we get In 2005, population of State D = 3x lakh

H
IA I I Given population of State C in 2004
= B = C = 175 : 185 : 170
175 185 170 = 2.5 lakh

G
= 35 : 37 : 34. We know that
19. Ans. (c) Percentage rise in population from 2004 to
Sol. Let Expenditure of Company B in 2006
= Income of Company A in 2008
Let Income of Company B in 2006 = IB U 2005
Population in 2005
– Population in 2004
O
and Expenditure of Company A in 2008 = EA = × 100
Population in 2004
To find: Ratio of x : EA in 2006 ⇒ For State C
H
For 2008, we know that
2 x – 2.5
Expenditure (100 + Per cent profit) 40 = × 100
Income = 2.5
_T

100 40 2x 2.5
x=
EA (100 +65) ⇒ = –
100 2.5 2.5
100
x 165 33 2x
⇒ 0.4 = –1
C

= = 2.5
EA 100 20
2x
x : EA = 33 : 20. ⇒ 0.4 + 1 =
PS

2.5
20. Ans. (b) 2x
Sol. Consider year 2008 ⇒ = 1.4
2.5
We know that ⇒ 2x = 1.4 × 2.5
U

Expenditure (100+Per cent Profit) ⇒ 2x = 3.5


Income =
100 ⇒ x = 1.75
∴ Income of Company B
@

∴ Population of State D in 2005


6.5 (100 + 80) = 3x lakh
IB =
100 = 3 × 1.75 lakh
1170 117 = 5.25 lakh.
= = [Fast-track: The per cent rise in population
100 10
Consider year 2006. for C in 2004-05 was 40; i.e., on a population
We know that in 2004 of 100, there was a rise of 40. If
Income ×100 the population was 2.5 lakh in 2004 (given),
Expenditure =
100 + Per cent profit 40
∴ Expenditure of Company C rise was × 2.5 = 1 lakh. Population
100
6.5×100 in C in 2005 was 2.5 + 1 = 3.5 lakh.
EC = 100 +80
Given ratio of population of C and D is 2 : 3,
650 65 C 2 3.5
= = ⇒ = =
180 18 D 3 D
578 ✫ Quantitative Aptitude

⇒ 2D = 10.5 The percentage of population is 2005 to


10.5 170
⇒ D = = 5.25 lakh.] population in 2006 is × 100 = 66.66
2 255
22. Ans. (e) or 67% approximately.]
Sol. Fast-track: Take population of E in 2004 24. Ans. (c)
to be 100. With a per cent rise of 20, Sol. Average number of students who passed
population in 2005 will be 100 + 20 = 120. out from all the colleges
Now there is a rise of 25 per cent from (15 + 17.5 + 27.5 + 25 + 10)
2005-06. So population in 2006 is + (22.5 + 20 + 35 + 30 + 7.5)

TS
25 = thousand
× 120 = 150. 5
100 = 42000.
Population of E in 2004 as a fraction 25. Ans. (b)
of population of E in 2006 ?
Sol. 35000 = 100 × (22.5 + 20 + 35 + 30 + 7.5)

H
100 2 thousand
= = .
150 3 35000 × 100

G
23. Ans. (b) ? =
Sol. Let population of State F be x in 2005 115000
Population of State F in 2006 ? = 30.43%
= Population of State F in 2005 + Per cent
increase from 2005 to 2006
U ∴ Required percentage ≈ 30%.
26. Ans. (e)
O
= x + 50% of x Sol. Difference between total number of
50 x students passing out from College A and
=x+
100 College E
H
100 x + 50 x = [(15 + 22.5) – (7.5 + 10)] thousand
= = 20000.
100
_T

150 x 27. Ans. (a)


=
100 Sol. Ratio of total number of male students to
15 x total number of female students
=
C

100 15 + 17.5 + 27.5 + 25 + 10


∴ Population of State F in 2005 expressed =
22.5 + 20 + 35 + 30 + 7.5
PS

as percentage of its population in 2006


95 19
Population in 2005 = =
= × 100 115 23
Population in 2006 = 19 : 23.
x
U

= ×100 28. Ans. (d) (15 + 17.5) × 100


15 Sol. Required percentage =
x (35 + 30)
10
@

32.5 × 100
10 x =
= × × 100 65
15 x = 50%.
10
= ×100 29. Ans. (e)
15 Sol. From the graph it is clear that Increase in
= 66.67% or 67% (approximate). profit of Company Y in 2008 over 2007 is
[Fast-track: As only the percentage is to 5% over 35.
be found, you may assume the population 5
So percentage increase = × 100
of F in 2004 to be 100. As there is a 70 35
per cent rise from 2004 to 2005, in 2005 = 14.285 or 14% approximately.
the population is 170. On a population of 30. Ans. (a)
170, a rise of 50 per cent (as shown in the Sol. Percentage increase in profit of
graph) means population in 2006 is Company Z
40 − 35
50 = × 100 ≈ 14%.
× 170 + 170 = 85 + 170 = 255. 35
100
Unit Eight : Practice Session ✫ 579

31. Ans. (d) 36. Ans. (b)


Sol. Profit earned in 2004 Sol. Average exports of Company B for all
= Rs 265000 the years
Profit earned in 2005 500 + 400 + 600 + 800 + 900 + 700 + 700
135 =
= × 265000 7
100 4600
= = 657.14.
Profit earned in 2006 7
140 135 37. Ans. (e)
= × × 265000
100 100 Sol. Total exports of Company A

TS
= Rs 500850. = 350 + 500 + 400 + 600 + 550 + 400 + 500
32. Ans. (b) = 3300
Sol. Total per cent increase in profit of Total exports of B = 4600

H
Company Z 3300 ´ 100
∴ Required percentage =
= 20 + 35 + 40 + 45 + 50 + 60 4600
= 250 = 71.7 » 72%.

G
250 2
∴ Average per cent increase = = 41 .
6 3 38. Ans. (a)
33. Ans. (c)
Sol. The graph presents ‘increase’ in profit
by percentage, so one cannot deduce U Sol. Percentage increase in exports of
Company C from 2004 to 2008
O
‘loss’ from the figure just because of 750 - 500
= × 100
the downward slope of the curve; all 500
= 50%.
H
one can say is the profit percentage
was less for X in 2004 compared to 39. Ans. (a)
_T

2003. So option (a) is not correct. As the Sol. An increase of 20% employees in
graph represents percentages, we Company B in the IT department would
cannot estimate the actual amount of make the number
profit or compare the profits made by
20
C

the companies. So options (b) and (d) 7.5 + 7.5 × hundred


are not correct. Company Z had a rise 100
PS

of 50 per cent in profit in 2008 which 7.5


= 7.5 + hundred
is higher than its figures for the other 5
years. So option (c) is correct. = 7.5 + 1.5 = 9 hundred
U

34. Ans. (b) The difference in number from the


Sol. In 2004, the percentage rise was number of employees in Admin
department of Company A
@

600 - 400
= × 100 = 50%. = 900 – 400 = 500.
400
35. Ans. (d) 40. Ans. (e)
Sol. Total exports of the three companies in Sol. The average number of employees in all
2003 departments together in Company A
= 500 + 400 + 550
5.5 + 8 + 8.5 + 4.5 + 4
= 1450 = hundred
Total exports of the three companies in 5
2006 30.5
= 550 + 900 + 600 = hundred
5
= 2050
1450 29 = 610
∴ Ratio = = . None of the answer responses has 610.
2050 41
580 ✫ Quantitative Aptitude

Practice Exercise 48
Directions (Qs. 1–5): Study the following graph and (a) August and April only
answer the questions given below. (b) April only
(c) June only

7020
(d) May only
8000
(e) None of these

6250
7000

5730
Directions (Qs. 6–10): Study the following graph
6000
4970
4940
4800

carefully and answer the questions given below it.

TS
Price (Rs)

5000
3980
3210

4000

Number of Tourists
600
3000

(in thousands)

H
500
2000
400
1000 300

G
200
Feb. Mar. April May June July Aug. Sep.
2012 2012 100

Months
U
O
1. In how many months was the increase in
the price of crude oil more than 10% with Districts
respect to the previous month?
H
(a) 2 (b) 3 6. How many districts in Himachal Pradesh
(c) 4 (d) 6 were visited by more than 10% of the total
_T

(e) None of these Indian tourists?


(a) 5 (b) 3
2. If in April the crude oil price had been lesser (c) 4 (d) 2
by Rs 223 per metric tonne then how much (e) None of these
C

would have been the percentage increase in


price over the earlier month? 7. By what percentage were the Indian tourists
(a) 10 (b) 12 visiting Chamba less than those visiting
PS

(c) 14 (d) 15 Shimla?


(e) None of these (a) 50 (b) 55
(c) 60 (d) 75
3. Which month/s experienced more than 10 (e) 79
U

per cent but less than 20 per cent increase in


the price of crude oil over the earlier month? 8. Approximately what percentage did foreign
tourists form of all the tourists visiting
@

(a) July and September


(b) June and September Himachal Pradesh?
(c) April and July (a) 7 (b) 2
(d) March and July (c) 4 (d) 5
(e) None of these (e) None of these
4. What is the approximate percentage increase 9. What was the ratio between Indian tourists
in the price of crude oil from February to to foreign tourists visiting Kullu?
September? (a) 105 : 3 (b) 170 : 3
(a) 100 (b) 140 (c) 107 : 3 (d) 35 : 13
(c) 130 (d) 120 (e) None of these
(e) None of these 10. Which of the following districts in Himachal
5. Which month/s had less than one per cent Pradesh were visited by less than 10% of the
increase in crude oil price over the earlier total foreign tourists who visited Himachal
month? Pradesh?
Unit Eight : Practice Session ✫ 581

(a) Chamba, Kullu 13. What is the average number of students in


(b) Solan, Kangra College A for all the years together?
(c) Solan, Chamba, Shimla (a) 45,000 (b) 50,000
(d) Solan, Chamba (c) 52,000 (d) 48,000
(e) None of these (e) None of these
Directions (Qs. 11–15): Study the following graph 14. What is the approximate percentage rise in
carefully to answer the questions. the number of students of College B from
2005 to 2006?
Number of students in College A and (a) 8 (b) 12

TS
College B over the years (c) 4 (d) 15
(e) 20
College A College B
90 15. The number of students of College B in year

H
2008 is what per cent of the total students of
80 College B in all the years together? (Round
No. of students (in thousands)

off to two digits after decimal)

G
70
(a) 20.61 (b) 23.79
60 (c) 21.79 (d) 17.29
50

40 U (e) None of these


Directions (Qs. 16–19): Study the following graph
O
carefully and answer the questions given below it.
30
Production of wheat in 6 states
H
20
in 2010, 2011 and 2012
10
_T

2010
Production of Wheat (in lakh tonnes)

70
0 2011
2003 2004 2005 2006 2007 2008 60 2012
Years
C

50
11. For which college(s) and in which year was
the per cent rise in number of students from 40
PS

the previous year the highest?


30
(a) College A in year 2004 and College B
in year 2005 20
U

(b) Only College B in year 2004


10
(c) College A in year 2004 and College B
@

in year 2004 0
A B C D E F
(d) College A in year 2007 and College B
in year 2004 States
(e) None of these 16. The total production in three years in State
12. What is the ratio of the total number of A is what per cent of the total production in
students of College A in years 2004, 2006 and State F in three years?
2007 together and the total number of (a) 120% (b) 125%
students of College B in years 2003, 2004 and (c) 90% (d) 80%
2008? (e) None of these
(a) 35 : 32 (b) 33 : 37 17. What is the difference in the average
(c) 34 : 31 (d) 32 : 35 production for all states in 2011 and that in
(e) None of these 2012?
582 ✫ Quantitative Aptitude

(a) 4.17 (b) 2.17 (a) 5% (b) 20%


(c) 6.14 (d) 4.14 (c) 17.5% (d) 12.5%
(e) None of these (e) None of these
18. From 2011 to 2012 the per cent increase/ 22. From the following companies in which one
decrease is minimum for which state? was the per cent rise in production from 2011
(a) B (b) D to 2012 the minimum?
(c) A (d) C (a) B (b) A
(e) None of these (c) C (d) D
(e) F

TS
19. The total production in all states in 2012 was
approximately what per cent of total 23. Production of Company C in 2012 and the
production of the states in 2011? production of Company F in 2011 together
(a) 90% (b) 110% was what percentage of production of

H
(c) 120% (d) 75% Company B in 2013?
(e) None of these (a) 80% (b) 125%
(c) 133% (d) 120%

G
Directions (Qs. 20–24): Study the following graph (e) None of these
carefully and answer the questions given below it. 24. The difference between the average
Production of steel (in lakh tonnes)
by different companies for three U production of three years was maximum for
which of the following pairs of companies?
O
consecutive years 2011, 2012, 2013 (a) E and F (b) D and C
(c) E and C (d) A and E
H
2011 2012 2013 (e) None of these
90 Directions (Qs. 25–29): Study the following graph
Production of Steel (in lakh tonnes)

_T

80 carefully and answer the questions given below it.


70 Number of Students Studying in Various
Colleges from Various Faculties
60
C

80
Number of Students (in thousands)

50
70
PS

40 65
60
60 56
30 51.2
50
20
50 44
U

40
10 40 36.5
33
30 30
0 30
@

25
A B C D E F
20
Companies
10
20. What was the difference between the
average production of six companies in 2011 0
and the average production of six companies H I J K
in 2012? Colleges
(a) 7,05,000 tonnes Arts Science Commerce
(b) 7,50,000 tonnes
25. What is the difference between the total
(c) 75,000 tonnes
number of students studying in College H
(d) 75,00,000 tonnes
and those studying in College K?
(e) None of these
(a) 16,100 (b) 15,800
21. What was the per cent fall in production of (c) 16,300 (d) 16,700
steel from 2012 to 2013 for Company C? (e) None of these
Unit Eight : Practice Session ✫ 583

26. What is the total number of students 30. What is the average number of people
studying in all the colleges together? using mobile service M for all the years
(a) 5,20,900 (b) 5,20,700 together?
(c) 6,10,200 (d) 5,10,800 2 1
(e) None of these (a) 16 (b) 14444
3 6
27. What is the ratio of the students from the 2 1
faculty of Science from Colleges H and I (c) 16666 (d) 14
3 6
together to the students from the same faculty (e) None of these
from Colleges J and K together?

TS
31. The total number of people using all the
(a) 43 : 45 (b) 41 : 43
three mobile services in the year 2010 is what
(c) 45 : 43 (d) 43 : 41
(e) None of these per cent of the total number of people using

H
all the three mobile services in the year 2011?
28. The number of students from the faculty of
science from College I is approximately what (rounded off to two digits after decimal)

G
per cent of the total number of students (a) 89.72
studying in that college? (b) 93.46
(a) 34
(c) 29
(e) 39
(b) 37
(d) 31
U (c) 88.18
(d) 91.67
O
(e) None of these
29. What is the average number of students from 32. The number of people using mobile service
H
the faculty of Commerce from all the colleges
N in the year 2009 forms approximately
together?
(a) 36825 (b) 38655 what per cent of the total number of people
_T

(c) 35625 (d) 36585 using all the three mobile services in that
(e) None of these year?
Directions (Qs. 30–34): Study the given graph (a) 18 (b) 26
C

carefully to answer the questions that follow. (c) 11 (d) 23


Number of people using three different (e) 29
PS

types of mobile services over the years


33. What is the ratio of the number of people
L M N using mobile service L in the year 2008 to
70
U

that using the same service in the year 2007?


No. of People (in thousands)

60 (a) 8 : 7 (b) 3 : 2
@

50 (c) 19 : 13 (d) 15 : 11
(e) None of these
40
34. What is the total number of people using
30
mobile service M in the years 2011 and 2012
20
together?
10 (s) 35,000 (b) 30,000
0 (c) 45,000 (d) 25,000
2007 2008 2009 2010 2011 2012 (e) None of these
Years
584 ✫ Quantitative Aptitude

Answers with Solutions


1. Ans. (c) (iii) April: 10% of 4800 = 480
Sol. Percentage increase in price of crude oil May – April = 4940 – 4800 = 140
Price in current month 140 < 480, so May does not qualify (×)
– Price in previous month (iv) May: 10% of 4940 = 494
= 100 ×
Price in previous month June – May = 4970 – 4940 = 30
30 < 494, June (×)
The % increase in price of crude oil for
(v) June: 10% of 4970 = 497
various months can be calculated as
July – June = 5730 – 4970 = 760

TS
follows:
760 > 497, July qualifies (✓)
Months Percentage increase in price (vi) July: 10% of 5730 = 573
August – July = 6250 – 5730 = 520
520 < 573, August (×)

H
3980 – 3210
March × 100 = 23.99%
3210 (vii) August: 10% of 6250 = 625
September – August = 7020 – 6250 = 770

G
4800 – 3980
April × 100 = 20.60% 770 > 625, September (✓)
3980
Count the (✓); there are four.]
May
4940 – 4800
4800
× 100 = 2.91%
U 2. Ans. (d)
Sol. Crude oil price in April = Rs 4800
O
4970 – 4940 Had it been lesser by Rs 223, then the
June × 100 = 0.61%
4940 price would have been
H
5730 – 4970 Rs (4800 – 223) = Rs 4577
July × 100 = 15.29%
4970 % increase in crude oil price over earlier
_T

6250 – 5730 month


August × 100 = 9.08%
5730 Price new in April – Price in March
7020 – 6250 = × 100
Price in March
C

September × 100 = 12.32%


6250
4577 – 3980
= × 100
PS

∴ The increase in price of crude oil was 3980


more than 10% in 4 months, i.e., March, 597
= × 100
April, July and September. 3980
U

[Fast-track: You may avoid such lengthy = 15%.


calculations. Just find what a 10 per cent 3. Ans. (a)
@

increase on each month’s price would be, Sol. From the table in solution to Question
and note in how many cases the next 1, we can see that July and September
month’s price is higher than the 10 per are the months which experienced more
cent increase figure for the month in than 10% but less than 20% increase in
consideration. the price of crude oil over the earlier
(i) February: 10% of 3210 = 321 month.
Difference between prices in March [You may easily get the solution from
and February = 3980 – 3210 = 770, the fast-track method for Sol. 1, too. As
which is greater than 321. So March April and June do not even show a 10%
qualifies as showing more than 10% increase, you may eliminate options
increase over the previous month. (✓) with either of these months; so eliminate
(ii) March: 10% of 3980 = 398 options (b) and (c). Now for those
April – March = 4800 – 3980 = 820 months that show 10% increase, a 20%
820 > 398, so April qualifies (✓) increase means double the 10% increase.
Unit Eight : Practice Session ✫ 585

You have to check whether the actual 6. Ans. (b)


increases reach 20% or not as the Sol. Total Indian tourists
question asks for increases between = (540 + 220 + 130 + 535 + 140)
10% and 20%. = 1565 thousand
Take option (a) 10% of total Indian tourists
For July, we have 760 > 497 10
but 760 < 994 [497 × 2 = 20%] = 1565 ×
100
For September we have 770 > 625 = 156.5 thousand
but 770 < 1250 [625 × 2 = 20%] ∴ The number of districts in Himachal

TS
You may stop right here and mark Pradesh which were visited by more
option (a). than 10% of total Indian tourists = 3 (i.e.
However, you may want to check out Shimla, Solan, Kullu).
option (d). You know July qualifies. 7. Ans. (d)

H
For March the figures are 820 > 398.
Sol. No. of Indian tourists visiting Chamba
Now you can see that 820 is more than
= 130000

G
double of 398, so the increase is more
No. of Indian tourists visiting Shimla
than 20%.
So option (d) is not correct.] = 540000
4. Ans. (d)
Sol. % increase in price of crude oil from U ∴ Required percentage
(Tourists in Shimla – Tourists
O
February to September in Chamba)
= × 100
Price in September – Price in February Tourists in Shimla
H
= × 100 540000 – 130000
Price in February = ×100
7020 – 3210 540000
= × 100
_T

3210 410000
= ×100
3810 540000
= × 100
3210 = 75.92
or 75% (approximately).
C

= 118.69%
or 120% (approximately). [In such questions that ask for
PS

5. Ans. (c) percentages only, there is no need to


take the ‘thousands’ into consideration.
Sol. From the table in solution to Question
From the graph read 540 – 130 = 410.
1, we can see that the month which had
Required percentage
U

less than one per cent increase in crude


oil price over earlier month was June. 410
= × 100 = 75.92%
540
@

[Once again, you may get the conclusion Closest to option (d).]
from the fast-track method for Sol. 1.
Months that clocked less than 10% 8. Ans. (b)
increase only need to be checked. These Sol. Total foreign tourists visiting Himachal
are April, June and August. At a glance, Pradesh
June with a figure of 30 as difference = (7 + 2 + 0.5 + 15 + 10) thousand
in price appears to be the correct choice.
= 34.5 thousand
However, you may check:
= 34500
Difference in price Total tourists visiting Himachal Pradesh
× 100
Price in previous month = Total Indian tourists
30 + Total foreign tourists
⇒ × 100 = 0.60%
4940 = 1565000 + 34500
which is less than 1%.] = 1599500
586 ✫ Quantitative Aptitude

[Total Indian tourists from solution to From the graph read the differences
Question 6] in number (given by the height of the
∴ Required percentage bar for each college in each year).
Total foreign tourists Check out each answer option.
= ×100 Option (a) College A in 2004:
Total number of tourists
difference = 20
34500 Number in 2003 = 20
= × 100
1599500 20
Ratio = = 1 or 100%
= 2.16% 20

TS
College B in 2005:
or 2% (approximately). difference = 5
[Once again, you may ignore the 000s Number in 2004 = 60
in your calculations, as only 5 1

H
percentage is asked for. Just add up Ratio = =
the numbers as given against the bars.] 60 12
Option (b) College B in 2004:

G
9. Ans. (c) difference = 30
Sol. Foreign tourists visiting Kullu Number in 2003 = 30
= 15000
Indian tourists visiting Kullu = 535000
∴ Required ratio U Ratio =
30
30
= 1 or 100%.
O
Option (c)
Indian tourists visiting Kullu
= College A = 100% (from above)
Foreign tourists visiting Kullu
H
College B = 100% (from above)
535000 107
= = or 107 : 3. This appears to be the correct answer
15000 3
_T

choice. However, check out option (d)


10. Ans. (d) for College A in 2007:
Sol. Total no. of foreign tourists who visited difference in number = 20
Himachal Pradesh = 34500 Number in previous year = 50
C

(From solution to Question 8) 20


Ratio = < 1 or 100%
10% of total foreign tourists who visited 50
PS

Himachal Pradesh If you check for each year for A and


= 10% × 34500 B in this way, you find that option
10 (c) is correct.
= × 34500
U

100 12. Ans. (d)


= 3450 Number of students in A in
@

∴ The districts which were visited by less 2004 + 2006 + 2007


Sol. Ratio =
than 10% of total foreign tourists are Number of students in B in
Solan and Chamba. 2003 + 2004 + 2008
11. Ans. (c) 40 + 50 + 70 160 thousand
Sol. Per cent rise in a year would be = 30 + 60 + 85 = 175 thousand
Difference in numbers from = 160 : 175
one year to the next = 32 : 35.
100 ×
The number in the first
year of referenece 13. Ans. (b)
Sol. Average number of students in
But just the ratio without multiplying
College A for all the years
by 100 would tell you the highest/
lowest rise in number, and this will 20 + 40 + 45 + 50 + 70 + 75
be true for the percentage as well. = thousand
6
Unit Eight : Practice Session ✫ 587

300 255
= thousand = 50 thousand = lakh tonnes
6
6
= 42.5 lakh tonnes
= 50000.
Average production for all states in 2012
14. Ans. (a)
Sol. Rise in students in 2006 over 2005 (45+ 45+ 40 + 55+ 55+ 40)
= lakh tonnes
= 5 thousand 6
Percentage rise 280
= lakh tonnes
Rise in number in 2006 6
= × 100
= 46.67 lakh tonnes

TS
Number in 2005
5 ∴ The average difference
= × 100
65 = (46.67 – 42.5) lakh tonnes
(Ignoring 000, as it is common to both = 4.166

H
numbers)
or 4.17 (approximately).
= 7.69

G
or 8% approximately. 18. Ans. (d)
Sol. In such questions, it is essential to read
15. Ans. (c)
the correct bars for the years concerned.
Sol. In 2008, number of students in B = 85
thousand
U Per cent increase/decrease from 2011 to
2012
O
Total number of students in B over
the years Difference in production of
= (30 + 60 + 65 + 70 + 80 + 85) thousand 2012 and 2011
H
= × 100
= 390 thousand Production in 2011
The required percentage For State A
_T

85 45 – 40
= × 100 (ignoring 000) = ×100 = 12.5% increase
390 40
= 21.79 (rounded off to 2 digits after For State B
C

decimal). 45 – 35
= ×100 = 28.57% increase
16. Ans. (d) 35
PS

Sol. Total production in 3 years in State A For State C


= (35 + 40 + 45) 40 – 45
= ×100 = (–)11.11% or decrease
= 120 lakh tonnes 45
For State D
U

Total production in 3 years in State F


= (60 + 50 + 40) lakh tonnes 55 – 45
= ×100 = 22.22% increase
= 150 lakh tonnes 45
@

Required percentage For State E


Total production of State A 55 – 40
= Total production of State F × 100 = ×100 = 37.5% increase
40
120 For State F
= ×100
150 40 – 50
= ×100 = (–)20% or decrease
= 80%. 50
17. Ans. (a) ∴ The percentage is minimum for State C.
Sol. Average production for all states [You may read the difference from the bar
Total production for all states graph directly. You need to remember the
= base figure—that of the previous year—
Number of states
Average production for all states in 2011 while finding the percentage.
40 + 35 + 45 + 45 + 40 + 50 Since either increase or decrease is asked,
= lakh tonnes just the change is being asked, and the
6
588 ✫ Quantitative Aptitude

difference in the bar levels may be taken 20. Ans. (b)


into account straightaway. Sol. Average production of the 6 companies
Difference in any year
Percentage = × 100
Base year figure Total production in that year
=
However, there is even no need to multiply Number of companies
by 100 to get the correct answer (as
In 2011
whichever lowest fraction you get, when
multiplied by 100 will give you the least (50 + 45+ 30 + 50 +70 + 35)
percentage as well). So, = lakh tonnes

TS
6
5 280
State A : = 0.125 = lakh tonnes
40 6
5 [There is no read find the quotient at this

H
State B : = 0.142 stage.]
35
5 Average production of the 6 companies

G
State C : = 0.111 in 2012
45
(55+ 55+ 40 +70 +60 + 45)
10 = lakh tonnes
State D :
45
15
= 0.222

U =
325
6
6
lakh tonnes
O
State E : = 0.375
40 ∴ Difference between the average
10 production in 2011 and 2012
H
State F : = 0.2
50 325 280 45
10 = − = lakh tonnes
6 6 6
_T

In fact as soon as you see (for D) and


45
15 = 7.5 lakh tonnes
(State E) you need not bother to
40 = 750000 tonnes.
calculate at all, as the fractions are larger
C

5 5 21. Ans. (d)


than (State C) and (State A). You
45 40
PS

Sol. Per cent fall in production from 2012 to


need to know when to leave out 2013
calculations so as to save time.
Clearly State C shows least change.] Production in 2012
U

– Production in 2013
19. Ans. (b) = × 100
Production in 2012
Sol. Total production in all states in 2012
@

= (45 + 45 + 40 + 55 + 55 + 40) lakh tonnes 40 – 35


= × 100
= 280 lakh tonnes 40
Total production in all states in 2011 5
= × 100 = 12.5%.
= (40 + 35 + 45 + 45 + 40 + 50) lakh tonnes 40
= 255 lakh tonnes
22. Ans. (b)
∴ Required percentage
Sol. Per cent rise in production from 2011 to
Total production in all states in 2012 2012
= ×100
Total production in all states in 2011 Production in 2012
– Production in 2011
280 = ×100
= ×100 Production in 2011
255
= 109.8% This can be calculated in the following
or 110% (approximately). way:
Unit Eight : Practice Session ✫ 589

Company Per cent rise Production of B in 2013


55 – 50 = 60 lakh tonnes
A ×100 = 10%
50 ∴ Required percentage
55 – 45 Total production of C in 2012
B ×100 = 22.22% and F in 2011
45 = × 100
40 – 30 Production of B in 2013
C ×100 = 33.33%
30 40 + 35 75
70 – 50 = × 100 = × 100
D ×100 = 40% 60 60
50

TS
60 – 70 = 125%.
E ×100 = (–)14.29% 24. Ans. (c)
70
45 – 35
Sol.We know that difference between
F ×100 = 28.57% production of two companies can be

H
35 maximum if one of the companies has
∴ Percent age rise in production was the highest production and the other
minimum for Company A. one has the lowest production. From the

G
[Fast-track: Read from the diagram the graph it is clear that highest production
difference—which is the difference in is that of Company E and lowest is that
the levels of the bars for 2011 and 2012.
Then place the 2011 figure as base to U of Company C. So, the difference
between average production of both
O
the figure of difference and find the these companies would be maximum.
least value of fractions, which will also 25. Ans. (d)
H
be the minimum percentage rise. Sol. Difference between the total number of
5 students in College H and the total
A = = 0.1 number in College K
_T

10 = [(51.2 + 40 + 36.5) – (30 + 56 + 25)]


10 thousand
B = = 0.22
45 = [127.7 – 111] thousand
= 16.7 thousand
C

10
C = = 0.33 = 16700.
30
PS

20 26. Ans. (b)


D = = 0.4 Sol. Total number of students
50
= [(127.7) + (65 + 50 + 33) + (44 + 30 + 60) + (111)]
−10
E = = –0.01 = 520.7 thousand
U

70 = 520700.
10 27. Ans. (c)
F = = 0.28
Sol. Ratio of Science students from Colleges
@

35
Of all companies, E shows minimum H and I to those from Colleges J and K
rise as it shows a decrease. But as E 40 + 50 90 45
is not mentioned in the options, A is = = = = 45 : 43.
30 + 56 86 43
the correct choice.] 28. Ans. (a)
Sol. Percentage of number of science students
23. Ans. (b)
from College I out of the total number
Sol. Production of Company C in 2012
of students in that college
= 40 lakh tonnes
50
Production of Company F in 2011 = × 100
= 35 lakh tonnes 148
Total production of Company C in 2012 = 33.78% or 34% approximately.
and F in 2011 29. Ans. (e)
= (40 + 35) lakh tonnes Sol. Average number of Commerce
= 75 lakh tonnes students from all the colleges
590 ✫ Quantitative Aptitude

(36.5 + 33 + 60 + 25) 31. Ans. (d)


= thousand Sol. In 2010, total number of people using
4
154.5 all three mobile services is 55 thousand.
= thousand In 2011, total number of people using
4
= 38.625 thousand all three mobile services is 60 thousand
= 38625. Percentage of number in 2010 to
number in 2011
Note: Questions 30 to 34 are based on a 55
subdivided bar diagram, so the divisions marked = × 100
60

TS
off for each item should be read carefully = 91.666
against the numbers on the Y-axis. or 91.67 approximately.
30. Ans. (c) 32. Ans. (a)
Sol. In 2007, the portion marked off for M Sol. In 2009, number of people using N

H
begins at 10 and ends at 15, so is 10 thousand.
measures 5, i.e., 5 thousand people Total mobile users in 2009 = 55

G
using it. Percentage of users of N to number
In 2008, M begins at 15 and ends at of users of all mobiles
25, so measures 10 thousand
In 2009, M measures 20 to 45 U =
10
55
× 100 = 18 approximately.
O
(As percentage is asked, 000 may be
= 25 thousand
ignored.)
In 2010, M measures 15 to 35
H
= 20 thousand 33. Ans. (b)
In 2011, M measures 10 to 35 Sol. Number of people using L in 2008
= 15 thousand
_T

= 25 thousand
Number of people using L in 2007
In 2012, M measures 10 to 25
= 10 thousand
= 15 thousand Ratio of 2008 users to 2007 users
Total number of people using M service
C

15
= (5 + 10 + 25 + 20 + 25 + 15) = or 3 : 2.
10
PS

thousand 34. Ans. (e)


= 100 thousand Sol. M users in 2011 = 25 thousand
100 100000 M users in 2012 = 15 thousand
Average = thousand =
6 6 Together they number (25 + 15)
2
U

= 16666 . thousand or 40000


3 This is not among the options.
@
Unit Eight : Practice Session ✫ 591

Practice Exercise 49
Directions (Qs. 1–5): Study the following pie-chart Directions (Qs. 6–10): Study the following pie-chart
and answer the questions given below. carefully to answer the questions.

Sales of cars in Country X Expenditure of funds by a university


for various purposes
according to their colour Total Expenditure = Rs 60 lakh
Medical
Yellow facility for
Blue 10%

TS
students
12%
Green 7%
Research Purchase of
8% work overhead
Red 8% projectors
Golden 20%

H
Psychology for PhD
10% classes
laboratory
3% 10% 24%
wn %

G
Bro 5
c k
B la White
Silver 25%
Publication
7%

U Computer
laboratory
18%
of journals
15%
O
1. A 3% increase in the production of cars of
Purchase of
which colour along with red cars will make Sports books for
H
them 35% of the total cars? 12% library
(a) Silver (b) Blue 6%
(c) Black (d) Yellow 6. What is the ratio of the expenditure made
_T

(e) None of these by the university on Research work and that


on Purchase of books for library?
2. The cars of which colours together make (a) 4 : 5 (b) 5 : 4
them 50%? (c) 8 : 3 (d) 8 : 5
C

(a) Blue, Black, Red (e) None of these


(b) White, Black, Red
PS

(c) White, Silver, Blue 7. What is the total sum of expenditures on


(d) White, Blue, Green Research work, Purchase of overhead
(e) None of these projectors for PhD classes and Purchase of
books for library together?
U

3. White car is 18% more popular than cars of (a) Rs 22.6 lakh (b) Rs 22.8 lakh
which colour? (c) Rs 23.4 lakh (d) Rs 20.8 lakh
(a) Black (b) Brown
@

(e) None of these


(c) Silver (d) Blue
8. What is the difference between the expendi-
(e) None of these
ture made by the university for Publication
4. If the total production of cars during a certain of journals and for Psychology laboratory?
period was 4,20,000, how many more blue (a) Rs 41 lakh (b) Rs 3 lakh
cars were sold in comparison to golden cars? (c) Rs 4.2 lakh (d) Rs 3.8 lakh
(a) 12580 (b) 8400 (e) None of these
(c) 12850 (d) 840 9. If the expenditure on the Purchase of
(e) None of these overhead projectors for PhD Students is
5. Which colour is 17% less popular than white decreased by 7%, what will be the
colour? expenditure on the same after the decrease?
(a) Blue (b) Golden (a) Rs 1,33,920 (b) Rs 13,39,200
(c) Green (d) Yellow (c) Rs 1,02,000 (d) Rs 1,08,000
(e) None of these (e) None of these

591
592 ✫ Quantitative Aptitude

10. Which of the following is definitely true? (a) 352 (b) 342
(a) The ratio of expenditure of university (c) 643 (d) 653
for the Purchase of library books and (e) None of these
expenditure on Computer laboratory 14. What is the ratio of the number of teachers
is 3 : 1 respectively. who teach mathematics to the number of
(b) Expenditure on Medical facility for teachers who teach hindi?
students is Rs 4.6 lakh. (a) 13 : 7 (b) 7 : 13
(c) The difference between the expenditure (c) 7 : 26 (d) 8 : 15
on Research work and that on Medical (e) None of these

TS
facility for student is Rs 60,000. 15. If the percentage of mathematics teachers is
(d) All are true increased by 50 per cent and the percentage
(e) None of these of hindi teachers decreased by 25 per cent
Directions (Qs. 11–15): Study the following pie-chart then what will be the total number of

H
mathematics and hindi teachers together?
carefully to answer the questions.
(a) 390 (b) 379
Percentage-wise distribution of teachers

G
(c) 459 (d) 480
who teach six different subjects (e) None of these
Total number of teachers = 1800

U Directions (Qs. 16–20): Study the following information


to answer the given questions.
Ma 13%

Biology
O
Percentage of students in various
th e m

12%
courses (A, B, C, D, E, F) and percentage
Hindi
atic

of girls out of these


H
8%
s

Physics Total students: 1200


17% (800 girls + 400 boys)
_T

English Percentage of students in various courses


27%
Chemistry
C

23% F
13% A
20%
E
PS

12%
11. If two-ninths of the teachers who teach B
15%
physics are female, then the number of male D
35%
U

physics teachers is approximately what


5%C

percentage of the total number of teachers


who teach chemistry?
@

(a) 57 (b) 42
Percentage of girls in courses
(c) 63 (d) 69
(e) 51
12. What is the total number of teachers teaching F
chemistry, english and biology? 14% A
(a) 1,226 (b) 1,116 E 30%
(c) 1,176 (d) 998 14%
(e) None of these
13. What is the difference between the total B
D 10%
number of teachers who teach english and 30%
physics together and the total number of
teachers who teach mathematics and biology
2 %C

together?
Unit Eight : Practice Session ✫ 593

16. For course D what is the respective ratio of Percentage break-up of girls
in each discipline
boys and girls ?
Total number of girls = 2000
(a) 3 : 4 (b) 4 : 5
(c) 3 : 5 (d) 5 : 6 Management
Arts
(e) None of these
17. For which pair of courses is the number of 8%
boys the same? 14%
(a) E and F (b) A and D Science 32%
(c) C and F (d) B and D 24% Commerce

TS
(e) None of these
18. For course E, the number of girls is how 12% 10%
much per cent more than the boys for course

H
E?
Engineering Catering
(a) 150 (b) 350

G
(c) 250 (d) 180
21. What is the number of girls studying in
(e) None of these
Engineering discipline?
19. For which course is the number of boys the
minimum?
(a) E (b) F U (a) 180
(c) 320
(e) None of these
(b) 220
(d) 260
O
(c) C (d) A
22. Which discipline has the highest number of
(e) None of these
boys studying in it?
H
20. How many girls are there in course C ? (a) Science (b) Commerce
(a) 18 (b) 16 (c) Arts (d) Management
_T

(c) 80 (d) 160 (e) None of these


(e) None of these
23. The number of boys studying in Commerce
Directions (Qs. 21–25): Study the pie-charts carefully discipline forms approximately what per
C

to answer the questions that follow: cent of the total number of students studying
in that discipline?
Percentage distribution of candidates
PS

(a) 41 (b) 52
studying in various disciplines of a college
(c) 36 (d) 28
and the percentage of girls (e) 68
in each discipline
U

24. What is the respective ratio of number of


Percentage break-up of students boys studying in Management discipline to
in each discipline the number of boys studying in Science
@

Total number of students = 4800 discipline?


(a) 71 : 20 (b) 18 : 67
Management (c) 20 : 71 (d) 67 : 18
Arts (e) None of these
10% 25. The number of girls studying in catering
18%
discipline forms what per cent of the total
37% number of students in the college?
Science 21% Commerce (Rounded off to two digits after decimal)
(a) 3.28 (b) 5.61
6% (c) 3.95 (d) 4.17
8% (e) None of these
Catering Directions (Qs. 26–30): Study the information in
Engineering the following pie charts to answer the questions.
594 ✫ Quantitative Aptitude

Details of 1500 employees working in an of employees in that scale? (rounded off to


organisation in various scales nearest integer)
(a) 35 (b) 46
Break-up of 1500 employees (c) 44 (d) 34
across the scales (e) None of these
VII 30. What is the ratio between female employees
VI 8% in Scale VI and male employees in Scale I
14% respectively?
V I (a) 32 : 23 (b) 22 : 33

TS
12% 22% (c) 12 : 23 (d) 23 : 12
(e) None of these
IV
10%
Directions (Qs. 31–35): Study the following pie-charts
II
carefully and answer the questions given below:

H
III 15%
19%
Discipline-wise break-up of Number of
candidates who appeared in an interview

G
Total Number of Candidates who Appeared
in the Interview = 25780
Break-up of 800 male employees
across the scales
U Percentage Distribution
O
Other
VI VII 12% Commerce
9% 10% 18%
H
V
Agriculture
12%
14%
I
_T

IV 24%
Management Science
11%
12% 28%
III II
C

16% 18% Engineering


16%
PS

Discipline-wise break-up of number of


26. How many females are working in Scale II? candidates selected after interview
U

(a) 144 (b) 81 by the organisation


(c) 96 (d) 138 Total Number of Candidates selected
(e) None of these After Interview = 7390
@

Percentage Distribution
27. What is the ratio between male and female
employees respectively working in Scale V?
(a) 7 : 8 (b) 2 : 1 Other
Commerce
(c) 8 : 7 (d) 1 : 2 Ag 14%
16%
(e) None of these ric
u
7% l t u
28. What is the total number of female re
employees working in scales III and IV
together? Management Science
(a) 219 (b) 157 20% 32%
11% ering

(c) 162 (d) 285


(e) None of these
e
g in

29. Number of male employees working in


En

Scale VI is what per cent of the total number


Unit Eight : Practice Session ✫ 595

31. What was the ratio of the number of 33. Approximately what was the difference
candidates who appeared in the interview between the number of candidates selected
from other disciplines and the number of from Agriculture discipline and number of
candidates selected from Engineering candidates selected from Engineering
discipline (rounded off to the nearest discipline?
integer)? (a) 517 (b) 665
(a) 3609 : 813 (c) 346 (d) 813
(b) 3094 : 813 (e) 296
(c) 3094 : 1035 34. For which of the following disciplines was

TS
(d) 4125 : 1035 the difference in number of candidates
(e) 3981 : 767 selected to number of candidates who
32. The total number of candidates who appeared in interview the maximum?
appeared in the interview from Management (a) Management (b) Engineering

H
and other disciplines was what percentage (c) Science (d) Agriculture
of the number of candidates who appeared (e) Commerce

G
from Engineering discipline? 35. Approximately what was the total number
(a) 50 of candidates selected from Commerce and
(b) 150
(c) 200
(d) Cannot be determined U Agricultural disciplines together?
(a) 1700 (b) 1800
O
(c) 2217 (d) 1996
(e) None of these (e) 1550
H
Answers with Solutions
1. Ans. (b) ∴ White cars are 18% more popular than
_T

Sol. As the pie chart represents 100, we can silver cars.


just go by the given percentages. 4. Ans. (b)
As per given conditions, production of
Sol. Total production of cars = 420000
C

red cars + production of cars of another


Production of blue cars
colour need to total 35%.
= 12% × 420000 = 50400
PS

Red cars stand at 20%. As 35 – 20 = 15,


Production of golden cars
and as the production of car of the other
= 10% × 420000 = 42000
colour needs to increase by 3%, the
present production must be 15 – 3 = 12%. ∴ Blue cars sold more in comparison to
U

Only the blue cars qualify. golden cars by


50400 – 42000 = 8400.
2. Ans. (b)
@

5. Ans. (c)
Sol. We can see from the pie chart that Sol. Percentage of white colour cars = 25%
white + black + red cars together make Percentage of cars of a particular colour
50%. which are 17% less popular than white
White + Black + Red colour cars
= 25% + 5% + 20% = (25 – 17)%
= 50%. = 8%
[In this item, trying out the answer We can see that it is the green-coloured
options will help get the correct answer.] car which has 8% sales.
3. Ans. (c) ∴ Green colour is 17% less popular than
Sol. Per cent of white cars = 25 white colour.
Cars over which white cars are 18% more 6. Ans. (e)
popular will number = 7% Sol. Given, total expenditure is Rs 60 lakh
We can see that silver cars = 7% Ratio of the expenditure on Research
596 ✫ Quantitative Aptitude

work to expenditure on Purchase of 10. Ans. (c)


books for the library Sol. Check out the options.
= 8 × 60 lakh : 6 × 60 lakh (a) The ratio of expenditure on the
8 4 Purchase of library books to
= = or 4 : 3. expenditure on the Computer
6 3
7. Ans. (b) laboratory
Sol. Total expenditure out of 100 on Research = 6 : 18
work, Purchase of overhead projectors =1:3
for PhD classes and Purchase of books So (a) is wrong.

TS
for library (b) The expenditure on Medical facility
= 8 + 24 + 6 = 38 = 7% of 60 lakh
Given total expenditure is Rs 60 lakh, = 4.2 lakh
the expenditure on the above items So (b) is wrong.

H
38 (c) The difference between the
= × 60 lakh expenditures on Research work and
100

G
on Medical facility
= 22.8 lakh.
= (8 – 7)% of Rs 60 lakh
8. Ans. (b) = 1% of Rs 60 lakh
Sol. Difference between the expenditures on
the Publication of journals and the U = Rs 60,000
So (c) is definitely true.
O
Psychology laboratory on 100
11. Ans. (a)
= 15 – 10 = 5
Sol. Total physics teachers
Given total expenditure is Rs 60 lakh, the
H
17
expenditure on the above items is = × 1800 = 17 × 18
5 100
_T

× 60 Lakh Total chemistry teachers


100 23
= Rs 3 lakh. = × 1800 = 23 × 18
100
2
C

9. Ans. (b) Now, if physics teachers are female,


Sol. Given total expenditure is Rs 60 lakh. 9
7
A decrease of 7% in expenditure on
PS

must be male.
Overhead projectors for PhD students 9
∴ Male physics teachers
will mean expenditure of
7
Rs [24% of 60 – 7% of (24% of 60)] lakh = of 17 × 18 = 238
U

= Rs [14.4 – 1.008] lakh 9


Male physics teachers as a percentage of
= Rs 13.392 lakh
total chemistry teachers
@

or Rs 13,39,200.
238
[You have to clearly understand that 7% = × 100
decrease implies 7% decrease on 24. 23 × 18
Another way of reaching the answer: = 57 per cent approximately.
7 12. Ans. (b)
7% decrease on 24 = × 24 = 1.68
100 Sol. Out of 100, the number of teachers
New expenditure 24 – 1.68 = 22.32 teaching chemistry, english and biology
22.32 is the expenditure out of a total total
expenditure of 100. 23 + 27 + 12 = 62
If Rs 60 lakh is the total expenditure, the If the total number of teachers is 1800,
new expenditure will be the number becomes
22.32
× 60 = 13.392 lakh 62
100 × 1800 = 1116.
or Rs 13,39,200.] 100
Unit Eight : Practice Session ✫ 597

13. Ans. (b) Number of boys in course D


Sol. Number of english and physics teachers = Total number of students in course D
= (27 + 17) × 18 = 792 – Number of girls in course D
Number of mathematics and biology = 420 – 240 = 180
teachers ∴ Ratio of boys and girls in course D
= (13 + 12) × 18 = 450
= 180 : 240
Difference = 792 – 450 = 342.
= 18 : 24
14. Ans. (e) = 3 : 4.
Sol. The ratio of teachers of mathematics to [Fast-track:

TS
teachers of Hindi is clearly 13 : 8 Required ratio of boys : girls
This is not among the given options. = (35 × 12) – (30 × 8) : 30 × 8
15. Ans. (c) 420 – 240 : 240

H
Sol. Work the numbers out for 100 and then 180 : 240
multiply the end result by 18 to get the 3 : 4.]
total.

G
17. Ans. (c)
After an increase of 50 per cent, Sol. Number of boys in various courses
mathematics teachers will be = Total number of students in the
50
100
× 13 + 13
U particular course – Number of girls in
the particular course
O
= 6.5 + 13 = 19.5% This can be calculated as follows:
After decrease of 25%, hindi teachers Course Total number Number of Difference
H
will be of students girls
25 A 20 × 12 = 240 30 × 8 = 240 0
8– × 8
_T

100 B 15 × 12 = 180 10 × 8 = 80 100


= 8 – 2 = 6% C 5 × 12 = 60 2 × 8 = 16 44
Together hindi and mathematics teachers D 35 × 12 = 420 30 × 8 = 240 180
now E 12 × 12 = 144 14 × 8 = 112 32
C

= 19.5 + 6 = 25.5% F 13 × 12 = 156 14 × 8 = 112 44


The total number of mathematics and
PS

hindi teachers now will be We can see that for the courses C and F the
25.5 × 18 = 459. number of boys is the same at 44.

For Qs. 16-20, you need to consider both pie 18. Ans. (c)
U

diagrams. Be careful to read the diagrams and Sol. Number of girls for course E
the questions before answering them. = 112 (from the chart for Sol. 17)
@

Note: (i) There are 1200 students, so 1% is Number of boys for course E
equivalent to 12 students; so multi- = 32 boys (from the chart for Sol. 17)
plying the percentage with 12 will give Per cent of number of girls more than
the total number of students in that boys for course E
course. Number of girls – Number of boys
(ii) There are 800 girls, so 1% girls is = × 100
Number of boys
equivalent to 8 girls; so multiplying the
112 − 32
percentage with 8 will give the total = × 100
number of girls in that course. 32
80
16. Ans. (a) = × 100 = 250%.
32
Sol. Total number of students in course D
= 35 × 12 = 420 19. Ans. (d)
Number of girls in course D Sol. We can see from the chart given in
= 30 × 8 = 240 solution to question 17 that for course
598 ✫ Quantitative Aptitude

A, the number of boys is minimum, 25. Ans. (d)


i.e., 0. Sol. Required percentage
20. Ans. (b) Number of girls studying
Sol. Number of girls in course C Catering
= 16 girls (See chart to Sol. 17) = × 100
Total number of students
200
Note: For items 21-25: As per the first pie-chart = × 100
1% = 48 students; as per the second pie-chart 4800
1% = 20 girls. 200

TS
= = 4.17%.
21. Ans. (e) 48
Sol. Number of girls studying in Engineering 26. Ans. (b)
= 12 × 20 = 240. Sol. Females working in Scale II
22. Ans. (a) = 15 × 15 – 18 × 8

H
Sol. Number of boys studying in each = 225 – 144
discipline is as follows: = 81.

G
Management = (10 × 48) – (8 × 20) 27. Ans. (c)
= 480 – 160 = 320 Sol. Employees in Scale V
Arts = (18 × 48) – (14 × 20)
= 864 – 280 = 584
U = 12 × 15 = 180
Males in Scale V
O
Commerce = (21 × 48) – (24 × 20) = 12 × 8 = 96
= 1008 – 480 = 528 Females in Scale V
= 84
H
Catering = (6 × 48) – (10 × 20) 96 8
= 288 – 200 = 88 ∴ Required ratio = =
84 7
Engineering = (8 × 48) – (12 × 20) = 8 : 7.
_T

= 384 – 240 = 144 28. Ans. (a)


Science = (37 × 48) – (32 × 20) Sol. Employees in scales III and IV
= 1776 – 640 = 1136 = 29 × 15 = 435
C

∴ Science has the highest number of boys. Male employees in scales III and IV
= 27 × 8 = 216
23. Ans. (b) ∴ Female employees in scales III and IV
PS

Sol. Required percentage = 435 – 216 = 219.


Number of boys studying 29. Ans. (d)
Commerce Sol. Males in Scale VI
= × 100
U

Total number of students = 9 × 8 = 72


studying Commerce Total employees in Scale VI
= 14 × 15 = 210
@

528
= × 100 ∴ Required percentage
1008 72
= × 100 = 34.2
(See the numbers from Sol. 22) 210 or 34% approximately
= 52.38 or 52% approximately. 30. Ans. (e)
24. Ans. (c) Sol. Female employees in Scale VI
= 14 × 15 – 9 × 8
Sol. Required ratio = 210 – 72 = 138
Number of boys studying Male employees in Scale I
Management = 24 × 8 = 192
= 138 23
Number of boys studying Science Required ratio = =
320 20 × 16 20 192 32
= = = = 23 : 32.
1136 71 × 16 71 (which is not among the given answer
= 20 : 71. responses).
Unit Eight : Practice Session ✫ 599

31. Ans. (b) Total number of appeared candidates


Sol. Ratio of number of candidates who = 25780
appeared for ‘others’ to the number who 1
were selected from Engineering 1 per cent would be × 25780
100
12 = 257.80
× 25780
= 100 or 258 approximately.
11 Multiplying percentages in each
× 7390
100 discipline by 258 will give approximate
3093.6 numbers of appeared candidates.

TS
=
812.9 Similarly for selected candidates:
or 3094 : 813 If total is 7390,
(rounded off to the nearest integer) 1

H
1 per cent = × 7390 = 73.90
32. Ans. (b) 100
Sol. The total number of candidates that or 74 approximately.

G
appeared from Management and ‘others’ Multiplying percentages in each
(12 + 12 = 24) discipline by 74 will give the approximate
= × 25780
100
Number of candidates for interview from
U numbers of selected candidates.
Now, work out the difference between
O
Engineering
appeared and selected for the disciplines
16
= × 25780 mentioned in the answer responses.
H
100
The required percentage (a) Management : (12 × 258) – (20 × 74)
24 = 3096 – 1480 = 1616
× 25780
_T

= 100 × 100 (b) Engineering : (16 × 258) – (11 × 74)


16 = 4128 – 814 = 3314
× 25780
100 (c) Service : (28 × 258) – (32 × 74)
C

24 100
= × 25780 × × 100 = 7224 – 2368 = 4856
100 16 × 25780 (d) Agriculture : (14 × 258) – (7 × 74)
PS

24
= × 100 = 150. = 3612 – 518 = 3094
16
(e) Commerce : (18 × 258) – (16 × 74)
33. Ans. (e) = 4644 – 1184 = 3460
U

Sol. Percentage selected from Agriculture


=7 It is clear the difference is maximum for
Percentage selected from Engineering Science.
@

= 11
Difference = (11 – 7)% of 7390 35. Ans. (a)
4 Sol. The total number of candidates selected
= × 7390 from Commerce and Agriculture
100
= 295.6 16 + 7
= of 7390
or 296 approximately. 100
23
34. Ans. (c) = × 7390
Sol. As you are not asked for actual numbers, 100
you can calculate with approximate = 1699.7
numbers. or 1700 approximately.
600 ✫ Quantitative Aptitude

Practice Exercise 50
Note: In this exercise questions are based on a combination of table and diagram or two different kinds
of graphs/diagrams. While fast calculation is a requisite in all items, it is essential to read the data carefully
and correctly and avoid silly mistakes of taking data for the wrong year or the wrong category.
Directions (Qs. 1–4): Study the following pie-chart 3. What is the percentage of candidates who
and table carefully and answer the questions given qualified from States A and C together,
below them: rounded-off to two decimal points?
Distribution of candidates who appeared (a) 49.93 (b) 50.93

TS
in a competitive examination (c) 50.43 (d) 51.26
from seven states (e) None of these
Total candidates who appeared = 3 lakh 4. What is the total number of candidates who
qualified from States E and D together?

H
G (a) 54460 (b) 45540
7% A (c) 54540 (d) 54450
15% (e) None of these

G
F
19% Directions (Qs. 5–10): The comparative data on the
B
production of two items by the seven companies is
E
12%
18%

U given in the following graph and the table. Study them


carefully and answer the questions given below.
O
C
D 6% Percentage of the total production of
23% two items by seven companies
H
Cost of the total production (both items
together) by seven companies = Rs 25 crore
State-wise percentage and ratio of male
_T

and female qualified candidates


G 12% A 15%
State % Qualified Ratio of F5
over Qualified %
C

Appeared Candidates B 11%


from a state Male : Female
PS

A 49 4 : 5 E 27%
B 61 6 : 4 C 22%
C 54 7 : 8
U

D 45 3 : 2 D 8%
E 65 7 : 6
F 57 11 : 8
@

Ratio of production between items


G 48 9 : 11 I and II and the per cent profit
earned for the two items
1. What is the number of male candidates who
qualified from State G? Ratio Per cent
(a) 4568 (b) 5454 of profit
(c) 5544 (d) 4536 Company Production earned
(e) None of these Item I Item II Item I Item II
A 2 3 25 20
2. What is the total number of female
B 3 2 32 35
candidates who qualified from States A and C 4 1 20 22
B together? D 3 5 15 25
(a) 26526 (b) 26426 E 5 3 28 30
(c) 24526 (d) 25426 F 1 4 35 25
(e) None of these G 1 2 30 24
Unit Eight : Practice Session ✫ 601

5. What is the total cost of the production of Upto XII Std Pass
Item I by companies A and C together in Total Number = 32 Lakh
Rs crore?
(a) 9.25 (b) 4.9
(c) 5.9 (d) 4.1625 F A
(e) None of these 20% 15%
6. What is the amount of profit earned by
Company D on Item II? B
E 16%
(a) Rs 3.125 crore (b) Rs 31.25 crore
19%
(c) Rs 3.125 lakh (d) Rs 31.25 lakh

TS
(e) None of these D C
7. Cost of production of Item I by Company F 12% 18%
is what per cent of the cost of production of
Item II by Company D?

H
(a) 20% (b) 33.33% Male-Female (M : F) Ratio
(c) 66.67% (d) 12.5%

G
(e) None of these Graduate and Above Upto XII Std Pass
8. What is total profit earned by Company G State M : F M : F
for Items I and II together?
(a) Rs 1.62 crore (b) Rs 78 lakh
(c) Rs 7.8 crore (d) Rs 16.2 lakh U A
B
7
5
5
3
7
3
9
5
O
(e) None of these C 5 4 4 5
9. What is the ratio of cost of production of Item D 9 8 5 7
H
I by Company A to the cost of production of E 9 7 9 10
Item I by company D? F 4 3 3 2
_T

(a) 3 : 5 (b) 1 : 2
(c) 2 : 1 (d) 2 : 3 11. What is the difference between the graduate
(e) None of these male population and XII Std male
10. What is the total of the profit earned by population from state A?
C

Company B on production of Item I and the (a) 24,000 (b) 14,000


profit earned by Company A on production (c) 28,000 (d) 36,000
PS

of Item II? (e) None of these


(a) Rs 9.78 crore (b) Rs 52.8 lakh
(c) Rs 97.8 lakh (d) Rs 5.28 crore 12. What is the ratio of the graduate female
(e) None of these population of state E to Std XII female
U

population of state D?
Directions (Qs. 11–15): Study the following pie- (a) 7 : 5 (b) 5 : 7
charts and the table to answer the questions. (c) 16 : 15 (d) 15 : 16
@

State-wise details of the (e) None of these


adult population of a country
13. The graduate female population of state C
Graduate and above is what per cent of the Std XII female
Total Number = 24 Lakh population of that state?
(a) 40 (b) 62.5
F A (c) 50 (d) 52.5
14% 16%
(e) None of these
E B 14. The total graduate population of state F is
20% 18% what per cent of the total Std XII population
of state A?
D C
17% (a) 56 (b) 72
15%
(c) 68 (d) 70
(e) None of these
602 ✫ Quantitative Aptitude

15. What is the ratio of the total graduate 19. In which department are the lowest number
population of state D to the total Std XII of female employees working?
population of that state? (a) Marketing (b) Production
(a) 17 : 16 (b) 16 : 17 (c) HR (d) Finance
(c) 64 : 51 (d) 51 : 64 (e) None of these
(e) None of these 20. What is the total number of employees
Directions (Qs. 16–20): Study the graph and table from all departments together in the
carefully to answer the questions that follow. organisation?
(a) 1500 (b) 1575

TS
Number of Employees Working in (c) 1525 (d) 1625
Different Departments of an Organisation (e) None of these
and the Ratio of Males to Females
400
Directions (Qs. 21–22): Study the following graph

H
and table carefully, and answer the questions.
350
Population: Data of different states in 2010
300

G
No. of employees

(Total population of all states = 25 lakh)


250
200
150
U
O
100
50
H
0
Marketing IT Produc- Merchan-
Departments tion dising
_T

Department Males Females


HR 9 16
Marketing 3 2
C

IT 9 31
Finance 2 3 Sex and Literacy-wise Population Ratio
PS

Production 11 4
States Sex Literacy
Merchandising 4 3
Male : Female Literate : Illiterate
16. What is the total number of male employees
U

Uttar Pradesh 5 : 3 2 : 7
working in all the departments together? Bihar 3 : 1 1 : 4
(a) 755 (b) 925 Andhra Pradesh 2 : 3 2 : 1
@

(c) 836 (d) 784


Karnataka 3 : 5 3 : 2
(e) None of these
Maharashtra 3 : 4 5 : 1
17. What is the number of female employees Tamil Nadu 3 : 2 7 : 2
working in the HR department? Kerala 3 : 4 9 : 4
(a) 158 (b) 128
(c) 136 (d) 144 21. Approximately what is the number of lit-
(e) None of these erate people in Maharashtra and Karnataka
18. What is the respective ratio of total number together?
of employees working in the Production (a) 4.5 lakh (b) 6.5 lakh
department to those working in the (c) 3 lakh (d) 3.5 lakh
Merchandising department? (e) None of these
(a) 15 : 14 (b) 8 : 7
(c) 14 : 15 (d) 7 : 8 22. In Tamil Nadu, if we suppose 70% of the
(e) None of these females and 75% of the males become
Unit Eight : Practice Session ✫ 603

literate, what is the total number of illit- 25. In which district the number of male teachers
erates in the state? is more than the number of female teachers?
(a) 75,000 (b) 85,000 (a) B only (b) D only
(c) 71,000 (d) 81,000 (c) Both B and E (d) Both E and F
(e) None of these (e) None of the above
Directions (Qs. 23 to 27): Study the following 26. What is the difference between the number
pie-chart and bar-chart and answer the questions. of female teachers in District-F and the total
Percentagewise Distribution of number of teachers (both male and female)
Teachers in Six Different Districts in District-E?
Total number of Teachers = 4500 (a) 625 (b) 775

TS
Percentage of Teachers (c) 675 (d) 725
(e) None of these
District-F
6%

District-A 27. What is the ratio between the number of

H
14% male teachers in District-C and the number
District-E of female teachers in District-B?

G
21% District-B (a) 11 : 15 (b) 15 : 11
16% (c) 15 : 8 (d) 30 : 13
District-D (e) None of these
15% District-C
28% U
Directions (Qs. 28–32): Study the graphs carefully to
answer the questions that follow.
O
Total number of children in 6 different
Number of Males out of the 4500 teachers schools and the percentage of girls in them
H
in each district separately Number of Children
Males
_T

3500
District-F 3000

District-E 2500
C

District-D 2000
PS

1500
District-C
1000
District-B
500
U

District-A 0
P Q R S T U
0 100 200 300 400 500 600 700 Schools
@

23. What is the total number of male teachers in Percentage of Girls


District-F, number of female teachers in 50
District-C and the number of female teachers 45
in District-B together? 40
(a) 1080 (b) 1120 35
(c) 1180 (d) 1020 30
(e) None of these
25
24. Number of female teachers in District-D is 20
approximately what percentage of total 15
number of teachers (both male and female) 10
in District-A?
(a) 70 (b) 75 5
(c) 80 (d) 95 0
P Q R S T U
(e) 90 Schools
604 ✫ Quantitative Aptitude

28. What is the total percentage of boys in Total Number of Men, Women and Children
schools R and U together? (rounded off to in the state over the years
two digits after decimal)
Years Men Women Children
(a) 78.55 (b) 72.45
(c) 76.28 (d) 75.83 2008 54,000 38,000 15,000
(e) None of these 2009 75,000 64,000 21,000
29. What is the total number of boys in School 2010 63,000 60,000 12,000
T? 2011 66,000 54,000 16,000
(a) 500 (b) 600 2012 70,000 68,000 20,000

TS
(c) 750 (d) 850 2013 78,000 75,000 45,000
(e) None of these
33. What was the approximate average of obese
30. The total number of students in School R is
men, obese women and obese children in

H
approximately what per cent of the total
number of students in school S? 2011?
(a) 89 (b) 75 (a) 12,683 (b) 12,795

G
(c) 78 (d) 82 (c) 12,867 (d) 12,843
(e) 94 (e) 12,787
31. What is the average number of boys in
Schools P and Q together? U34. The number of obese men in the year 2013
was what per cent of the men not suffering
O
(a) 1425 (b) 1575
from obesity in the same year?
(c) 1450 (d) 1625
(a) 55 (b) 60
H
(e) None of these
(c) 50.5 (d) 65.5
32. What is the ratio of the number of girls in
(e) None of these
_T

School P to the number of girls in School Q?


(a) 27 : 20 (b) 17 : 21 35. What was the ratio of the obese women in
(c) 20 : 27 (d) 21 : 17 the year 2010 to the obese men in the year
(e) None of these 2012?
C

Directions (Qs. (33–37): Study the following graph (a) 6 : 7 (b) 21 : 65


and table carefully and answer the questions given below
PS

(c) 15 : 73 (d) 48 : 77
them. (e) None of these
Obese men Obese women 36. What is the difference between the number
U

Obese children of obese women and obese children together


40
in the year 2010 and the number of obese
Percentage of obese men, obese

35 men in the same year?


women and obese children

30 (a) 5,475 (b) 5,745


(c) 4,530 (d) 31,650
25
(e) None of these
20
37. What was the total number of children not
15
suffering from obesity in the year 2008 and
10 2009 together?
5 (a) 4,350 (b) 31,560
0 (c) 4,530 (d) 31,650
2008 2009 2010 2011 2012 2013 (e) None of theses
Years
Unit Eight : Practice Session ✫ 605

Answers with Solutions


1. Ans. (d) Total number of female candidates who
Sol. From the pie diagram we can calculate qualified from both States A and B
total number of candidates who appeared = 12250 + 13176 = 25426.
from State G 3. Ans. (c)
= 7% × 300000 Sol. Total number of candidates who
7 qualified from State A
= × 300000 = 21000
100 = 49% of (15% of 300000)
Taking data from the table, we get

TS
49
number of candidates who qualified = × 45000
from those who appeared from State G 100
= 22050
= 48% × 21000
48 Total number of candidates who

H
= × 21000 = 10080 qualified from State C
100 = 54% of (6% of 300000)
Again, from the table we know 9 out of

G
54
20 candidates qualified are male. So, the = × 18000
number of male candidates who 100
qualified from State G
=
9
× 10080 = 4536 U = 9720
Total number of candidates who
qualified from States A and C together
O
20
[Fast-track: Total number who qualified = 22050 + 9720
from State G = 31770
H
∴ Percentage of candidates who qualified
48 7
= of of 300000 from States A and C together
100 100
_T

Total candidates qualified


= 48 × 7 × 30 = 10080
from A and C
Male candidates who qualified = × 100
9 Total candidates who appeared
= × 10080 = 4536.]
C

20 from States A and C together


2. Ans. (d) 31770 31770
= × 100 = × 100
PS

Sol. Total number of candidates who (45000 +18000) 63000


qualified from State A
= 50.428%
49 15 or 50.43%.
= × × 300000
U

100 100 4. Ans. (d)


= 22050 Sol. Total number of candidates who
qualified from State E
@

Number of female candidates who


qualified from State A = 65% of 12% of 300000
5 = 23400
= × 22050 = 12250 Total number of candidates who
9
Total number of candidates who qualified from State D
qualified from State B = 45% of 23% of 300000
61 18 = 31050
= × × 300000 ∴ Total number of candidates who
100 100
qualified from States E and D together
= 32940
= 23400 + 31050
Number of female candidates who = 54450.
qualified from State B
5. Ans. (c)
4
= × 32940 = 13176 Sol. Cost of production of Item I = Ratio of
10 production of Item I × Percentage of total
606 ✫ Quantitative Aptitude

production × Cost of total production ∴ Required percentage


Cost of production of Item I by Cost of production
Company A of Item I by F
= × 100
2 15 Cost of production
= × × 25 crore
5 100 of Item II by D
3 Rs 0.25 crore
= crore = Rs 1.5 crore = × 100
2 Rs 1.25 crore
Cost of production of Item I by = 20%.
Company C

TS
8. Ans. (b)
4 22 Sol. Cost of production of both Items I and
= × × 25 crore II for Company G
5 100
22 12
= × 25 crore

H
= crore = Rs 4.4 crore 100
5
∴ Total cost of production of item I by = Rs 3 crore

G
Companies A and C together Cost of production of Item I
= Rs (1.5 + 4.4) 1
= × Rs 3 crore
= Rs 5.9 crore.
6. Ans. (d)
U 3
= Rs 1 crore
Cost of production of Item II
O
Sol. Cost of production of Item II for
2
Company D = × Rs 3 crore
3
5 8
H
= × × Rs 25 crore = Rs 2 crore
8 100 Profit earned
= Rs 1.25 crore = Per cent profit × Cost of production
_T

Amount of profit earned by Company D ∴ Profit earned on Item I


on Item II 30
= × Rs 1 crore = Rs 0.3 crore
= Per cent profit earned on Item II 100
C

Profit earned on Item II


× Cost of production of Item II 24
PS

= 25% × Rs 1.25 crore = × Rs 2 crore = Rs 0.48


100
25 ∴ Total profit earned by G for Items I
= × Rs 1.25 crore
100 and II
= Rs 0.3125 crore = Rs 0.3 crore + Rs 0.48 crore
U

= Rs 0.3125 × 100 lakh = Rs 0.78 crore


= Rs 31.25 lakh. (  I crore = 100 lakh) = Rs 78 lakh [  1 crore = 100 lakh]
@

7. Ans. (a) 9. Ans. (c)


Sol. Cost of production of Item I by Sol. Cost of production of Item I by Company
Company F A = Rs 1.5 crore
1 5 [From Solution to Q. 5]
= × × 25 crore Cost of production of Item I by Company
5 100
D = Cost of production of both Items I
1
= crore and II
4 = Cost of production of Item II by
= Rs 0.25 crore Company D
Cost o f production of Item II by = Rs 2 crore – Rs 1.25 crore
Company D [From Solution to Q. 6]
= Rs 1.25 crore = Rs 0.75 crore
(From Solution to Q. 6) ∴ The ratio of cost of production of Item I
Unit Eight : Practice Session ✫ 607

by Company A to the cost of production 12. Ans. (d)


of Item I by Company D Sol. Graduate female population of state E :
= 1.5 : 0.75 Std XII female population of state D
= 2 : 1. 7 20 7 12
10. Ans. (c) ⇒ × ×24 : × ×32
16 100 12 100
Sol. Cost of production of Item I by B [We can leave out 105 as it is common
3 11 to both sides.]
= × × 25 crore 210000 15
5 100 ⇒ 210000 : 224000 = =
224000 16
= Rs 1.65 crore ⇒ Ratio is 15 : 16.

TS
Profit earned by Company B on
production of Item I 13. Ans. (c)
Sol. Required %
32
= × Rs 1.65 crore Graduate female

H
100 population of state C
= Rs 0.528 crore = × 100
XII Std female population
Cost of production of Item II by

G
Company A = Cost of production of both of state C
Items – Cost of production of Item I 4
× 15 × 24
= Rs 3.75 crore – Rs 1.5 crore
[From solution to question 5]
= Rs 2.25 crore U ⇒ 9
5
9
× 18 × 32
× 100
O
Profit earned by Company A on
1
production of Item II ⇒ × 100 = 50%.
H
2
20
= × Rs 2.25 crore 14. Ans. (d)
100
Sol. Total graduate population of State F
_T

= Rs 0.45 crore
14
∴ Total profit earned by Company B on = × 24 lakh
100
production of Item I and by Company A
Total XII Std population of State A
C

on production of Item II 15
= Rs 0.528 crore + Rs 0.45 crore = × 32 lakh
100
PS

= Rs 0.978 crore 14
× 24
= Rs 97.8 lakh. 100
Required percentage = × 100
15
Note: For Questions 11-15, the two pie-charts as × 32
100
U

well as the table may have to be consulted for


some items. 336
⇒ × 100 = 70%.
@

11. Ans. (b) 480


Sol. Total graduate male population
= 24 lakh 15. Ans. (a) 17
× 24 lakh
or 24 × 105 100 17
Sol. Required ratio = =
Total XII Std pass = 32 × 105 12 16
× 32 lakh
Graduate male population of state A 100
7 16 ⇒ 17 : 16.
= × × 24 × 105 = 224000
12 100 16. Ans. (c)
XII Std male population of state A Sol. Number of male employees working in
7 15 9
= × × 32 × 105 = 210000 HR = × 225
16 100 9 + 16
The difference is 224000 – 210000 9
= × 225 = 81
= 14000. 25
608 ✫ Quantitative Aptitude

3 Population of Maharashtra
Marketing = × 275 = 165 11
5 = × 25 lakh
9 100
IT = × 200 = 45 ∴ Literate people in Maharashtra
40
5 11
2 = × × 25 lakh
Finance = × 175 = 70 6 100
5 = 2.29 lakh (approximate)
11
Production = × 375 = 275 Literate people in Karnataka
15
3 15

TS
4 = × × 25 lakh
Merchandising = × 350 = 200 3 + 2 100
7
= 2.25 lakh
∴ Total number of male employees
∴ The number of literate people in
= 81 + 165 + 45 + 70 + 275 + 200

H
Maharashtra and Karnataka together
= 836. = 2.29 + 2.25 = 4.5 lakh (approximate)
17. Ans. (d)

G
Sol. Female employees working in HR 22. Ans. (d)
Sol. As we need to find the illiterate
department
population, we may take the percentage
=
16
25
× 225 = 144.
U of illiterates.
In Tamil Nadu (100 – 70) = 30% of
O
18. Ans. (a) 2
females are illiterate or 30% of
375 15 5
Required ratio = = females are illiterate.
H
350 14
Number of illiterate females in Tamil
= 15 : 14.
Nadu
19. Ans. (b)
_T

30 2 12
Sol. Number of female employees working = × × × (25 lakh)
in the departments is as follows. 100 5 100
16 = 36000
C

HR = × 225 = 144 As for males, (100 – 75 =) 25% are illiterate


25
2 Number of illiterate males in Tamil Nadu
Marketing = × 275 = 110
PS

25 3 12
5 = × × × (25 lakh)
31 100 5 100
IT = × 200 = 155
40 = 45000
3
U

Finance = × 175 = 100 Total illiterates will be


5 36000 + 45000 = 81000.
4
Production = × 375 = 100
@

15 Note for Items 23-27: As per the pie-chart,


3 100 = 4500. So 1 = 45; the number in each district
Merchandising = × 350 = 150 is the percentage multiplied by 45. As only the
7
∴ The lowest number of female workers is number of male teachers is given in the pie
in the Production department. chart, wherever a question demands, the number
of female teachers needs to be calculated:
20. Ans. (e) Percentage of teachers in a district × 45 –
Sol. Total number of employees number of male teachers in that district.
= 275 + 225 + 200 + 175 + 375 + 350
23. Ans. (a)
= 1600.
Sol. Number of male teachers in District-F
21. Ans. (a) = 100
Sol. Literate people in Maharashtra Number of female teachers in District-C
5 5 = (28 × 45) – 600
= =
5+1 6 = 1260 – 600 = 660
Unit Eight : Practice Session ✫ 609

Number of females teachers in District-B 26. Ans. (b)


= 16 × 45 – 400 Sol. Number of female teachers in District-F
= 720 – 400 = 320 = 170
∴ Total number of male teachers in District- Total number of teachers in District-E
F and female teachers in District-C and = 945
District-B [Both these numbers are available from
= 100 + 660 + 320 solution to previous question.]
= 1080. Difference = 945 – 170
24. Ans. (e) = 775.
Sol. Number of female teachers in District-D 27. Ans. (c)

TS
= 15 × 45 – 100 Sol. Number of male teachers in District-C
= 675 – 100 = 575 = 600
Total number of teachers in District-A Number of female teachers in
District-B

H
= 14 × 45 = 630
∴ Required percentage = 320
[from solution to previous question]
575

G
= × 100 ∴ Required ratio = 600 : 320
630 = 15 : 8.
= 91.27% Note for Questions 28-32:
or 90% approximately.
25. Ans. (c) U Here the line graph gives the percentage of
girls, so if a question requires it, the number
O
Sol. The options mention B, D, E and F, so of boys needs to be calculated. As the bar graph
calculate for these. gives the numbers in ‘hundreds’ and the line
H
District B graph gives percentages, to find out the number
Number of teachers = 16 × 45 = 720 of girls you may ignore two 00s (not all) in the
numbers in the bar graph and multiply with the
_T

Number of male teachers = 400


Number of female teachers percentage numbers. For example, in School P,
= 720 – 400 = 320 the number of girls = 40 × 25 = 1000. For Q,
So, male teachers are more. it will be 45 × 30 = 1350 (leave out just two
C

District D zeros from 3000.)


Number of teachers = 15 × 45 = 675 28. Ans. (d)
PS

Males = 100 Sol. From the line graph work out the
Females at 675 – 100 = 575. percentage of boys.
So female teachers are more. In R, percentage of boys
District E
U

= 100 – 27.5 = 72.5


Number of teachers = 21 × 45 = 945 Number of boys = 72.5 × 20 = 1450
Males = 500 In U, percentage of boys
@

Females 945 – 500 = 445 = 100 – 17.5 = 82.5


So, male teachers are more. Number of boys = 82.5 × 10 = 825
Here itself you have the answer. Both Together the boys number
B and E have more male teachers than = 1450 + 825 = 2275.
female teachers, so (c) is the correct As a percentage of the total that is
answer response. If F also has more
male teachers, then two options would 2275
× 100
be correct which is unlikely. Sum of total children in R and U
Check: 2275
= × 100
District F 2000 + 1000
Number of teachers 6 × 45 = 270 2275
Males = 100 = × 100 = 75.83%
Females = 270 – 100 = 170 3000
So, female teachers are more. (rounded off to two digits after decimal).
610 ✫ Quantitative Aptitude

29. Ans. (c) Average of all three


Sol. Percentage of girls in T 23100 + 13500 + 2000
= 100 – 40 = 60 =
3
Number of boys in T = 12866.6
= Percentage × Number of children in T or 12867 approximately.
60 34. Ans. (b)
⇒ × 1250 = 750. Sol. Obese men in 2013 = 37.5 × 780
100
Percentage of non-obese men in 2013
30. Ans. (a)
= 100 – 37.5 = 62.5
Sol. The required percentage
Number of non-obese men = 62.5 × 780

TS
Number of children in R Percentage of obese to non-obese
= × 100
Number of children in S 37.5 × 780
2000 = × 100
= × 100 62.5 × 780

H
2250 375
= 88.8 or 89 approximately. = × 100 = 60%
[Read the numbers correctly in such 625

G
35. Ans. (d)
cases where the bar is midway between Sol. The ratio of obese women in 2010 to
the divisions indicated on the Y-axis.] obese men in 2012
31. Ans. (b)
Sol. Percentage of boys in P = 100 – 40 = 60 U =
20 × 600
27.2 × 700
=
12
O
Percentage of boys in Q = 100 – 45 = 55 19.25
Average number of boys in P and Q So, now you check with the answer
(60 × 25) + (55 × 30) options if your result can equate with
H
= any of them, before you decide to mark
2 (e).
1500 + 1650 3150
_T

= = = 1575. Multiply or divide the numerator and


2 2 denominator by the same number.
32. Ans. (c)
Option (a) seems unlikely. You may
Sol. The ratio of number of girls in P to the get 6 by dividing 12 by 2, but not 7
C

number of girls in Q by dividing 19.25 by 2.


Number of girls in P Options (b) and (c) may similarly be
= Number of girls in Q
PS

rejected.
40 × 25 20 Option (d) works as 12 × 4 = 48, and
= = 19.25 × 4 = 77.
45 × 30 27 36. Ans. (a)
⇒ 20 : 27
U

Sol. Obese women in 2010


[Be careful not to mark (a).] = 20 × 600 = 12000
@

Obese children in 2010


Note for Questions 33-37: = 25 × 120 = 3000
The number of obese is calculated by multiplying Together they number
percentages in the line graph with the number of 12000 + 3000 = 15000
the category concerned from the table. Since the Obese men in 2010 = 32.5 × 630 = 20475
base of 100 will cancel out two 00s from the Difference is 20475 – 15000 = 5475.
numbers, you may multiply the percentage 37. Ans. (d)
number with the total number without two 00s; Sol. In 2008, the percentage of non-obese
for example, obese men in 2004 number 25 × 540. children = 100 – 15 = 85
The number = 85 × 150 = 12750
33. Ans. (c)
In 2009, the percentage of non-obese
Sol. For the year 2011 children = 100 – 10 = 90
Obese men = 35 × 660 = 23100 The number = 90 × 210 = 18900
Obese women = 25 × 540 = 13500 Together they number 12750 + 18900
Obese children = 12.5 × 160 = 2000 = 31650.
Unit Eight : Practice Session ✫ 611

Practice Exercise 51
Directions (Qs. 1–5): Study the following information Directions (Qs. 6–10): Study the following
carefully and answer the questions that follow. information carefully and answer the questions that
follow.
An office consists of 520 employees working
in different departments, viz., HR, IT, Production A company produces four different products,
and Marketing. The ratio of men to women in viz. mobile phone, television sets, refrigerators
the organisation is 5 : 3 respectively. 20 per cent and computers. Each of the products has two
categories, viz. category-A and category-B. The
of the men work in the IT department. 40 per

TS
total number of all the products is 600. Television
cent of the women work in the HR department.
makes up 20 per cent of the total number of
The total number of employees in the Production
products. 40 per cent of the remaining products
department is 135. Two-fifth of the women work
are refrigerators. 50 per cent of the number of
in the IT department and the remaining work

H
computers as well as the number of mobile
in the marketing department. 40 per cent of the
phones is of category-A. 25 per cent of the
men work in the Production department. Four

G
number of television sets is of category-B. One-
per cent of the men work in the HR department
fourth of the number of refrigerators is of
and the remaining work in the Marketing category-A.
department.
1. Number of men working in the Marketing U 6. What is the ratio of number of mobile
phones of category-A to the number of
O
department forms what per cent of the total computers of category-A?
number of employees in the organisation? (a) 7 : 6 (b) 5 : 9
H
(a) 22.5 (b) 34.5 (c) 7 : 3 (d) 7 : 5
(c) 19.5 (d) 38.5 (e) None of these
(e) None of these
_T

7. What is the approximate average number of


2. What is the ratio of the number of men products of category-A together?
working in the HR department to the number (a) 81 (b) 86
of women working in the same department? (c) 90 (d) 95
C

(a) 1 : 5 (b) 2 : 3 (e) None of these


(c) 4 : 7 (d) 1 : 6
8. What is the total number of television sets
PS

(e) None of these


of category-A, refrigerators of category-B
3. What is the number of women working in and mobile phones of category-A together?
the Marketing department? (a) 318 (b) 328
U

(a) 41 (b) 34 (c) 296 (d) 266


(c) 46 (d) 39 (e) None of these
(e) None of these
@

9. The combination of which of the two


4. Total number of employees working in the products of category-B is exactly equal in
Production department forms number?
approximately what per cent of the total (a) Mobile phone and computer
number of employees working in the (b) Mobile phone and refrigerator
organisation? (c) Television and refrigerator
(a) 12 (b) 17 (d) Television and computer
(c) 21 (d) 26 (e) None of these
(e) None of these 10. What is the difference between the number
5. What is the total number of employees of computers of category-B and the number
working in the IT department? of refrigerators of category-A?
(a) 130 (b) 124 (a) 42 (b) 18
(c) 143 (d) 101 (c) 32 (d) 36
(e) None of these (e) None of these
612 ✫ Quantitative Aptitude

Directions (Qs. 11–12): Study the following and the remaining boys take dancing classes.
information carefully to answer the questions. The girls taking singing classes is twice the
number of boys taking the same. The remaining
An institute having 450 employees has sent girls take painting classes.
all its employees for training in one or more
areas out of HRM, Computer Skills and Financial 13. What is the ratio of boys taking painting
Skills. Employees are classified into two classes to the boys taking singing classes?
categories, officers and clerks, which are in the (a) 4 : 3 (b) 2 : 1
ratio of 4 : 5 respectively. 10% of the officers (c) 6 : 5 (d) 3 : 4
take training only in Computer Skills, 16% of (e) None of these

TS
the clerks take training only in HRM which is 14. The number of girls taking cooking classes
equal to the number of officers taking training is what per cent of the total number of
only in Financial Skills, and 50% of the number children in the school? (Rounded off to the
of officers take training in HRM and Financial nearest integer)

H
Skills both. 6% of the total employees take (a) 14 (b) 20
training in all three of which two-thirds are (c) 6 (d) 26
officers. 10% of the total employees take training

G
(e) None of these
in HRM and Computer Skills both, which is five
times the number of clerks taking training in 15. The number of boys taking cooking classes
Computer Skills and Financial Skills. 10% of the
clerks take training in HRM and Computer
U is what per cent of the total number of
children in the school? (Rounded off to two
digits after decimal)
O
Skills both. Number of officers taking training
only in HRM is 25% of the number of clerks (a) 9.68 (b) 12.61
taking training only in HRM. 20% of the total (c) 10.71 (d) 8.43
H
number of employees take training only in (e) None of these
Computer Skills. Number of clerks taking training 16. What is the total number of children taking
_T

in HRM and Financial Skills both is 20% of the dancing classes?


total number of clerks. (a) 325 (b) 445
11. How many clerks are taking training in (c) 390 (d) 405
Computer Skills but not in HRM? (e) None of these
C

(a) 79 (b) 88 17. What is the number of girls taking painting


(c) 104 (d) 113
PS

classes?
(e) None of these
(a) 116 (b) 15
12. What is the total number of employees (c) 125 (d) 28
taking training in Financial Skills but not in (e) None of these
U

HRM? Directions (Qs. 18–22): Read the following infor-


(a) 108 (b) 126 mation carefully and answer the questions given
(c) 134 (d) 135
@

below it.
(e) None of these
Directions (Qs. 13–17): Study the information Out of the total number of 7800 girls in a
carefully to answer the questions that follow. college, 15% of the total number of girls are
good at dancing and 20% of the number of girls
In a school consisting of 2800 children, the good at dancing are also good in studies. 5%
ratio of girls to boys is 4 : 3 respectively. All of the total number of girls in the college are
the children have taken different hobby classes good at drawing and 10% of the girls good at
viz. singing, dancing, painting and cooking. 20 drawing are also good in studies. 35% of the
per cent of the boys take painting classes. The total number of girls in the college are good at
number of girls taking dancing classes is five- cooking and 20% of the girls good at cooking
fourth of the number of boys taking the same. are also good in studies; also, 30% of the girls
One-fourth of the girls take cooking classes. The good at cooking are good at dancing but not
total number of students taking cooking classes good in studies. 10% of the total number of girls
is 700. Two-fifth of the boys take singing classes in the college are good only at singing. 25% of
Unit Eight : Practice Session ✫ 613

the total number of girls in the college are good (a) 22 (b) 63
at sports and 16% of the number of girls good (c) 36 (d) 58
at sports are also good in studies. Rest of the (e) None of these
girls from the college are only good at studies.
24. What is the total number of girls studying
18. How many girls from the college are good Hindi?
at dancing? (a) 960 (b) 592
(a) 1755 (b) 1989 (c) 992 (d) 432
(c) 1170 (d) 1898 (e) None of these
(e) None of these
25. What is the respective ratio of total number

TS
19. How many girls from the college are good of boys studying English to the total number
only at cooking? of girls studying English?
(a) 2730 (b) 2184 (a) 90 : 101 (b) 101 : 90
(c) 1365 (d) 1545

H
(c) 65 : 101 (d) 101 : 65
(e) None of these (e) None of these
20. How many girls from the college are good

G
26. What is the total number of students
at drawing? studying only English?
(a) 351 (b) 380
(a) 648 (b) 888
(c) 400
(e) None of these
(d) 341
U (c) 726
(e) None of these
(d) 908
O
21. How many girls from the college are good
27. How many girls do not study Marathi?
at studies?
(a) 1851 (b) 780 (a) 968 (b) 960
H
(c) 990 (d) 1911 (c) 808 (d) 1368
(e) None of these (e) None of these
_T

22. How many girls from the college are good Directions (Qs. 28-32): Study the following
only at singing? information carefully to answer the questions that
(a) 780 (b) 750 follow:
C

(c) 700 (d) 800 In a tournament, a total number of 400


(e) None of these players have participated in five different sports,
PS

Directions (Qs. 23–27): Study the information below viz., badminton, hockey, lawn tennis, cricket
and answer the questions that follow. and baseball. 15 per cent of the total players
have participated in badminton. Two-fifths of
In a school of 3,000 students, the ratio of
the total players have participated in hockey. 6
U

girls to boys is 7 : 8 respectively. All the students


per cent of the total players have participated
study three different languages viz., Hindi,
English and Marathi. 30% of the boys study only in lawn tennis. 25 per cent of the total players
@

English. Two-sevenths of the girls study only have participated in cricket. Remaining players
English and Hindi. One-eighths of the boys have participated in baseball. One-fourth of the
study only English and Marathi. Number of hockey players are females. 20 per cent of
girls studying only English is 85% of the boys badminton players are males. Half the players
studying the same. Two-fifths of the boys study who have participated in lawn tennis are males.
only Hindi and Marathi. 40% of the girls study There are 45 female cricket players. No female
only Hindi. The remaining girls study only player has participated in baseball.
Hindi and Marathi. The number of boys studying 28. What is the ratio of the number of male
only English and Hindi is 10% of the girls players participating in badminton to the
studying the same. The remaining boys study number of female players participating in
only Hindi. hockey?
23. Number of boys studying only Hindi form (a) 3 : 11 (b) 3 : 10
approximately what per cent of the number (c) 6 : 11 (d) 11 : 6
of girls studying only Hindi? (e) None of these
614 ✫ Quantitative Aptitude

29. What is the total number of female players (a) 115 (b) 140
participating in the tournament? (c) 135 (d) 130
(a) 130 (b) 120 (e) 120
(c) 145 (d) 155 35. If the cost of one Oven is Rs 8,200, what is
(e) None of these the cost of all the Ovens manufactured by
30. The number of female players participating the company?
in badminton is approximately what (a) Rs 2,16,480 (b) Rs 21,68,400
percentage of the number of players (c) Rs 2,16,48,000 (d) Rs 2,16,840
participating in baseball? (e) None of these

TS
(a) 72 (b) 75 36. What is the difference between the number
(c) 80 (d) 95 of Refrigerators and the number of Mobile
(e) 86 phones?
(a) 160 (b) 140

H
31. What is the difference between the number (c) 120 (d) 130
of male players participating in hockey and (e) None of these
the number of female players participating

G
in lawn tennis? 37. If 25 per cent of the number of Ovens is
(a) 92 (b) 98 defective, what is the number of non-
(c) 102
(e) None of these
(d) 108
U defective Ovens?
(a) 176
(c) 198
(b) 188
(d) 186
O
32. If due to certain reasons the cricket game (e) None of these
was dropped and all the cricket players left Directions (Qs. 38–41): Study the following
H
the tournament, then what would be the total information carefully to answer the questions that follow.
number of male players in the tournament?
(a) 200 (b) 210 A company produces 4 different products,
_T

(c) 190 (d) 220 viz., AC, fans, refrigerators and ovens, each
(e) None of these product being of two different qualities, i.e.,
Quality A and Quality B. The company produces
Directions (Qs. 33 to 37): Study the information a total of 500 products. One-fifth of the total
C

carefully to answer the questions that follow: number of products is fans, out of which 35%
A company produces five different products are of Quality B. Fifteen percent of the total
PS

viz., Television, Refrigerator, Mobile phone, number of products are AC. Two-thirds of the
Oven and Water heater. Total number of all the ACs are of Quality A. Twenty-five per cent of
the total number of products are refrigerators,
five products manufactured is 1200. 15 per cent
out of which 40 are of quality B. Ten per cent
U

of the total number of products is Television. of the number of ovens are of Quality B.
Three-tenth of the total number of products is
38. What is the total number of AC and ovens of
@

Refrigerator. Number of Mobile phones


Quality B and fans and refrigerators of
manufactured is 40 more than the number of
Quality A together made by the company?
Television. 22 per cent of the total number of (a) 165 (b) 205
products is Oven and the remaining number of (c) 155 (d) 185
products is Water heater. (e) 145
33. What is the total number of Refrigerators 39. What is the average number of products of
and Water heaters? Quality A made by the company?
(a) 436 (b) 476 (a) 90 (b) 75
(c) 576 (d) 536 (c) 80 (d) 95
(e) None of these (e) None of these
34. Number of Mobile phones is approximately 40. What is the ratio of the number of ovens of
what percentage of the number of Quality B to the number of fans of Quality
televisions? A?
Unit Eight : Practice Session ✫ 615

(a) 5 : 2 (b) 4 : 13 42. The number of males working in the IT


(c) 5 : 13 (d) 4 : 9 department forms approximately what per
(e) None of these cent of the total number of males in the
41. What is the difference between the number organisation?
of ACs of Quality A and Quality B? (a) 5 (b) 12
(a) 25 (b) 50 (c) 21 (d) 8
(c) 35 (d) 40 (e) 18
(e) None of these
43. The total number of employees working in
Directions (Qs. 42–45): Study the given information the Accounts department forms what per cent

TS
carefully to answer the questions that follow. of the total number of employees in the
An organisation consists of 2400 employees organisation? (rounded off to two digits after
working in different departments, viz HR, decimal)

H
Marketing, IT, Production and Accounts. The (a) 19.34 (b) 16.29
ratio of male to female employees in the (c) 11.47 (d) 23.15
organisation is 5 : 3. Twelve per cent of the (e) None of these

G
males work in the HR department. Twenty-four
per cent of the females work in the Accounts 44. The number of females working in the
Production department forms what per cent
department. The ratio of males to females
working in the HR department is 6 : 11. One-
ninth of the females work in the IT department. U of the total number of females in the
organisation?
O
Forty-two per cent of the males work in the (a) 7 (b) 2
Production department. The number of females (c) 4 (d) 15
H
working in the Production department is ten per (e) None of these
cent of the males working in the same. The
remaining females work in the Marketing 45. What is the total number of females working
_T

department. The total number of employees in the HR and Marketing departments


working in the IT department is 285. Twenty together?
two per cent of the males work in the Marketing (a) 363 (b) 433
C

department and the remaining work in the (c) 545 (d) 521
Accounts department. (e) None of these
PS

Answers with Solutions


For Questions 1 to 5, make some basic 1. Ans. (a)
calculations and a chart. Then it is easy to find Sol. Required percentage
U

the answers.
Number of men in marketing
Total employees = 520 = × 100
@

Total employees
Ratio of men to women = 5 : 3 117
5 = × 100 = 22.5%.
∴ Men = × 520 = 325 520
8
Women = 520 – 325 = 195 2. Ans. (d)
Sol. Ratio of men to women in HR
Department Men Women Total ⇒ 13 : 78
HR 4% of 325 = 13 40% of 195 = 78 91 ⇒ 1 : 6.
2 3. Ans. (b)
IT 20% of 325 = 65 of 195 = 78 143 Sol. Number of women working in the
5
marketing department = 34.
Production 40% of 325 = 130 135–130 = 5 135
4. Ans. (d)
Marketing 325 – (13+65+130) 195–(78+78+5) Sol. Total employees = 520
= 117 = 34 151 Employees in Production = 135
616 ✫ Quantitative Aptitude

Required percentage Make two sets of diagrams, one for officers


135 in various sections and the other for clerks
= × 100 in various sections and put in the numbers
520
as per the data in the passage.
= 25.96% = 26% (approximate).
5. Ans. (c) HRM
Sol. Total number of employees in IT
20
department = 143. 20 Computer
10 (10%)
Make a Chart for Questions 6 to 10 Skills
18

TS
Product Category-A Category-B Total 80 12
Mobile 50% of 168 84 600 – 432 40
phone = 84 = 168

H
1
T.V. 200 – 150 25% of 200 of 600
= 150 = 50
3
= 200
Financial Skills

G
1 I. Officers = 200
Refrigerator of 112 112 – 28 40% of
4 = 28 = 84 280 = 112
HRM
Computer 20% of 120
= 60
60 20% of 600
= 120
U 40
O
Total 322 278 600 25 70 Computer
(16%)
Skills
9
6. Ans. (d)
H
50 9
Sol. Ratio of mobile phones to computers
84 7
in category-A is 84 : 60 or =
_T

60 5 47
⇒ 7 : 5.
7. Ans. (a) 322 Financial Skills
C

Sol. Average =
4 II. Clerks = 250
= 80.5 ⇒ 81 (approximate).
PS

11. Ans. (a)


8. Ans. (a) Sol. The required number is 70 + 9 = 79.
Sol. Required total = 150 + 84 + 84
12. Ans. (a)
= 318.
U

Sol. The number of clerks taking training in


9. Ans. (b) Computer Skills but not in HRM is (40
Sol. Both equal 84. + 12) officers and (47 + 9) clerks; total
@

10. Ans. (c) number 52 + 56 = 108.


Sol. Required difference = No. of computers For Qs. 13-17, a general chart will help in getting
in category-B – No. of refrigerators in all the answers.
category-A Number of children = 2800
3
⇒ 60 – 28 = 32. Number of boys = × 2800 = 1200
7
For items 11 and 12, it is best to make some 4
Number of girls = × 2800 = 1600
basic calculations and a diagram of the given 7 20
Boys taking painting classes = × 1200
data. 100
Total employees = 450 = 240
4 1
Officers = × 450 = 200 Girls taking cooking classes = × 1600
9 4
Clerks = 250 = 400
Unit Eight : Practice Session ✫ 617

∴ Boys taking cooking classes = 700 – 400 Girls good at cooking and dancing but
= 300 not good in studies
2 30
Boys taking singing classes = × 1200 = × 2730 = 819
5 100
= 480 ∴ Number of girls who are only good at
Boys taking dancing classes cooking = 2730 – (546 + 819) = 1365.
= 1200 – (240 + 300 + 480) = 180
20. Ans. (e)
5 Sol. Girls good at drawing
Girls taking dancing classes = × 180 = 225
5

TS
4
Girls taking singing classes = 2 × 480 = 960 = × 7800 = 390.
100
Girls taking painting classes 21. Ans. (d)
= 1600 – (400 + 225 + 960) = 15. Sol. Girls good at drawing = 390

H
(From Sol. to Q. 20)
13. Ans. (e)
Girls good at singing
Sol. The ratio of number of boys taking

G
painting classes to number of boys 10
= × 7800 = 780
taking singing classes is 100

or
240 : 480
1 : 2. U Girls good at sports
25
O
14. Ans. (a) = × 7800 = 1950
100
Sol. Number of girls taking cooking classes
Girls good at cooking = 2730
= Total number of children of x%
H
x (From explanation to Q. 19)
400 = × 2800 Girls good at dancing = 1170
100
_T

(From Sol. to Q. 18)


400 100 2 Girls good at sports and studies
x= = = 14 ≈ 14%.
28 7 7 16
15. Ans. (c) = × 1950 = 312
100
C

300 × 100 Girls good at drawing and studies


Sol. Required per cent = = 10.71% .
2800 10
PS

16. Ans. (d) = × 390 = 39


100
Sol. Total number of children learning dance
Girls good at dancing and studies
=180 + 225 = 405. 20
U

= × 1170 = 234
17. Ans. (b) 100
Sol. Girls taking painting classes Girls good at cooking and studies
@

= 1600 – (400 + 960 + 225) = 546 (From Sol. to Q. 19)


= 1600 – 1585 ∴ Girls who are only good at studies are
= 15. = 7800 – (1170 + 2730 + 390 + 780 + 1950)
18. Ans. (c) = 7800 – 7020
Sol. Girls good at dancing = 780
15
= × 7800 = 1170. ∴ Total number of girls good at studies is
100
19. Ans. (c) = 780 + 312 + 39 + 234 + 546
Sol. Girls good at cooking = 1911.
35 22. Ans. (a)
= × 7800 = 2730 Sol. Girls good at singing
100
Girls good at cooking and studies 10
20
= × 7800
100
= × 2730 = 546
100 = 780.
618 ✫ Quantitative Aptitude

Basic Calculations for Questions 23 to 27: 24. Ans. (c)


Number of boys in the school Sol. Total number of girls studying Hindi
8 = 400 + 560 + 32
= × 3000 = 992.
7+8
8 25. Ans. (a)
= × 3000 = 1600 Sol. Total number of boys studying English
15
Number of girls in the school = 480 + 200 + 40
= 3000 – 1600 = 1400 = 720
Number of boys studying only English Total number of girls studying English

TS
30 = 400 + 408
= 1600 × = 480 = 808
100 ∴ Ratio of number of boys studying English
Number of girls studying English and
to the number of girls studying English
2

H
Hindi = 1400 × = 400 = 720 : 808
7 = 90 : 101.
Number of boys studying only English

G
26. Ans. (b)
1 Sol. Total number of students studying only
and Marathi = 1600 × = 200
8 English
Number of girls studying only English
= 480 ×
85
= 408 U 27. Ans. (d)
= 480 + 408 = 888.
O
100 Sol. Number of girls who do not study
Number of boys studying only Hindi Marathi = 1400 – 32
H
2 = 1368.
and Marathi = 1600 × = 640
5 Chart for Questions 28 to 32
Number of girls studying only Hindi
_T

40 Sport Total Male Female


= 1400 × = 560
100 15 20
Number of remaining girls studying only Badminton 400 × 60 × 60 – 12 = 48
100 100
C

Hindi and Marathi


= 60 = 12
= 1400 – (400 + 408 + 560)
PS

= 1400 – 1368 2 1
= 32 Hockey 400 × 160 – 40 160 × = 40
5 4
Number of boys studying only English = 160 = 120
and Hindi 1
U

6
10 Lawn 400 × 24 × = 12 24 – 12 = 12
= 400 × = 40 100 2
100 Tennis = 24
@

Number of remaining boys studying


only Hindi 25
Cricket 400 × 100 – 45= 55 45
= 1600 – (480 + 200 + 640 + 40) 100
= 1600 – 1360 = 240 = 100

23. Ans. (e) Baseball 400 – (60 + 160 56 Nil


Sol. Number of boys studying only Hindi as + 24 + 100)
a percentage of number of girls studying = 400 – 344
only Hindi = 56
240 × 100 28. Ans. (b)
= % Sol. Ratio of the number of male players
560
300 participating in badminton to the
= % number of female players participating
7
= 42.86% in hockey = 12 : 40
≈ 43% (approximately). = 3 : 10.
Unit Eight : Practice Session ✫ 619

29. Ans. (c) 35. Ans. (e)


Sol. Total number of female players Sol. Cost of one Oven = Rs 8200
participating in the tournament Cost of 264 Ovens = Rs (264 × 8200)
= 48 + 40 + 12 + 45
= 145. = Rs 2164800.
30. Ans. (e) 36. Ans. (b)
48
Sol. Required percentage = × 100% Sol. Difference between number of
56
Refrigerators and number of Mobile
= 85.71%
phones = 360 – 220
≈ 86%. = 140.

TS
31. Ans. (d)
Sol. Difference between the number of male 37. Ans. (c)
players participating in hockey and the Sol. Number of non-defective ovens
number of female players participating 100 − 25

H
in lawn tennis = 120 – 12 = 264 ×
= 108. 100
75

G
32. Ans. (a) = 264 × = 198.
Sol. The required number is the difference 100
between total male players and players
of cricket
= (12 + 120 + 12 + 55 + 56) – 55 U Basic Calculations for Items 38-41
Total products = 500
O
= 255 – 55 Products Total Quality A Quality B
= 200.
1
H
For Questions 33 to 37: Fans of 500 65 35% of 100
5
Total number of products = 100 = 35
_T

= 1200 15 2
Number of Televisions AC of 500 of 75 25
100 3
15 = 75 = 50
= 1200 × = 180
100
C

25
Number of Refrigerators Refrigerators × 500 85 40
3 100
PS

= 1200 × = 360 = 125


10
Number of Mobile phones Ovens 500 – (100 + 75 180 10% of 200
= 180 + 40 = 220 + 125) = 200 = 20
Number of Ovens
U

38. Ans. (e)


22 Sol. Total of AC and ovens (Quality B) and
= 1200 × = 264
100 fans and refrigerators (Quality A)
@

Number of Water heaters = 25 + 20 + 65 + 85 = 195.


= 1200 – (180 + 360 + 220 + 264) 39. Ans. (d)
= 1200 – 1024 Sol. Average number of Quality A products
= 176
33. Ans. (d) 65 + 50 + 85 + 180 380
= = = 95.
Sol. Total no. of Refrigerators and Water 4 4
heaters = 360 + 176 40. Ans. (b)
= 536. Sol. Ratio of ovens of Quality B to fans
34. Ans. (e) of Quality A
Sol. Number of Mobile phones as percentage ⇒ 20 : 65
of number of Televisions ⇒ 4 : 13.
220 41. Ans. (a)
= × 100% Sol. Difference between AC of Quality A
180 and Quality B
= 122.22%
= 50 – 25 – 25.
≈ 120%.
620 ✫ Quantitative Aptitude

Basic Calculations for Items 42-45 42. Ans. (b)


Number of employees = 2400 Sol. As a percentage of total males in the
Ratio of males to females = 5 : 3 organisation, males in IT department
5 are
Number of males = × 2400
8 185
= 1500 × 100 = 12.33
1500
Number of females = 2400 – 1500 = 12 per cent approximately.
= 900
43. Ans. (b)
Department Males Females Total

TS
Sol. As a percentage of total employees in
12 11 the organisation, the total employees in
HR × 1500 of 180 510
100 6 the Accounts department
= 180 = 330*
391

H
22 = × 100 = 16.29.
Marketing × 1500 900 – (330 + 100 521 2400
100
= 330 + 63 + 216) = 191 (rounded off to two digits after decimal.)

G
1 44. Ans. (a)
IT 185 of 900 = 100 285

Production
42
× 1500
9
10
× 630 693 U Sol. As a percentage of the total female
employees in the organisation, the
females in the production department
O
100 100
= 630 = 63 form
24 63
× 100 = 7%.
H
Accounts 1500 – (180 + 330 × 900 391 900
100
+ 185 + 630) = 216 45. Ans. (d)
_T

= 175 Sol. Total number of females in HR and


* Ratio of males to females being 6 : 11, if males are 6, marketing departments together
females are 11; if males are 180, females are 11 × 180. = 330 + 191 = 521.
6
C
PS
U
@
Unit Nine : Fundamentals / Data Sufficiency ✫ 621

UNIT NINE
DATA SUFFICIENCY

FUNDAMENTALS AND

TS
WORKED EXAMPLES

H
Type and Strategies three, you first need to consider each
Data sufficiency questions are not found as such statement and decide whether by itself it

G
in school level examinations; they are, however, gives sufficient information to solve the
set in competitive examinations which have problem. Be careful—you must disregard
the information in the other statement/s
mathematics (or numeracy) as a subject.
Problems of this type require you to assess
whether the given statements or data are U when you consider one statement. If one
statement gives sufficient information, put
O
a tick mark against it and go on to
sufficient to answer a certain question. You are
consider the next statement. You may find
not required to fully solve the question to get
each of the two or three (or more)
H
the correct answer; you may stop as soon as you statements is sufficient to solve the
can tell that the data provided will enable you problem. You need to then choose your
_T

to solve the given question, or not, as the case answer corresponding to your conclusion.
may be.
● If you find no statement by itself gives
In data sufficiency items, there is a question
you enough information to solve the
followed by two or more statements providing
problem, you may need to consider all
C

some data. It is possible that each statement by statements together. They might, in
itself can help to solve the problem; or only one combination, be enough to solve the given
PS

of the statements helps, the others is/are not of problem.


use; or both or all of them are required to get
In case three or more statements are given,
the answer; or, may be, none can on its own or
it is possible that only two in combination
in combination with the others help to get the
U

help to solve the problem. You must pick


answer. In other words, the ‘sufficiency’ or the your answer response carefully to reflect
‘insufficiency’ of the data is to be judged. And, the correct combination of statements.
@

generally, this can be done only if you are well-


grounded in basic mathematical principles and ● You need to seek the answer to the given
problem; the answer statement has to be
formulae.
sufficient to get a single answer. If, for
example, you are asked, ‘what is the value
Points to Remember of x’, and one statement says, ‘x = y + 3’,
● You do not have to solve the problem; so you do not have sufficient information to
do not waste time trying to do so. You get a specific value for x (as ‘y + 3’ could
only need to determine if the data give a wide range of values for x.)
provided is sufficient to solve the given ● If there are diagrams, remember they may
problem—for which you need to be well- not necessarily be drawn to scale; so if two
versed in methods of solving various sides of a quadrilateral look equal, do not
problems. assume they are equal. Just go by the
information provided.
● Whether you are given two statements or

621
622 ✫ Quantitative Aptitude

Worked Examples ● Is ABCD a rectangle?


In most questions relating to Data Sufficiency,
the instructions include a set of answer responses
from which you have to choose the correct one
for each question. The options usually read:
Give answer
(a) if the data in Statement I alone are
sufficient to answer the question, while I. AB = CD
the data in Statement II alone are not II. AC = BD

TS
sufficient to answer the question;
(b) if the data in Statement II alone are Solution: Clearly either statement by itself
sufficient to answer the question, while is not enough to get the answer. Taken
the data in Statement I alone are not together, all we know is that the figure

H
sufficient to answer the question; could be a square or a rectangle. So both
(c) if the data either in Statement I or in statements even if taken together are not

G
Statement II alone art sufficient to answer sufficient to answer the question.
the question; ● What is the age of Zoya?
(d) if the data even in both Statements I and
II together are not sufficient to answer
the question; U I. Aizo is 12 years younger than Zoya.
II. Five years later Aizo will be more than
half of Zoya’s age.
O
(e) if the data in both Statements I and II
together are necessary to answer the Solution: I by itself does not lead us to the
answer. In II an indefinite term such as
H
question.
If an examination paper format allows only ‘more than’ (which does not tell us exactly
four answer options, one of the options by how much) shows us that it by itself does
_T

mentioned above will be omitted. not give the answer. The two together also
In other Data Sufficiency questions, each lead nowhere. [However, if II had said
question may carry its own set of options. “equal to”, the data in both statements
C

In either case, you need to read the options together would have helped us reach the
and be careful both while deciding upon the answer.] As it is, I and II, taken single or
together, do not provide sufficient data to
PS

correct one and when you mark it on the answer


sheet. answer the question.

● How much profit was earned by selling a Question Type I


U

product for Rs 100? Directions: Each of the questions given below consists
I. If the product had been sold for Rs 90, of a statement and/ or a question and two statements
the profit would have been 20%. numbered I and II given below it. You have to decide
@

II. The profit was one-third the cost price. whether the data provided in the statements are
sufficient to answer the question. Read both the
Solution: 100 statements and
I. C.P. = 90 × = Rs 75 Give answer (a) if the data in Statement I alone
120
Now you can find the profit. So I is are sufficient to answer the question, while the data
sufficient. in Statement II alone are not sufficient to answer the
x question;
II. x + = 100, Give answer (b) if the data in Statement II alone
3
100 × 3 are sufficient to answer the question, while the data
or x = = 75
4 in Statement I alone are not sufficient to answer the
Now profit can be found. So II is question;
sufficient. Give answer (c) if the data either in Statement
Thus either statement by itself will I or in Statement II alone are sufficient to answer the
provide the answer. question;
Unit Nine : Fundamentals / Data Sufficiency ✫ 623

Give answer (d) if the data even in both Statements Let’s take I. The integer may be 12, 21
I and II together are not sufficient to answer the or 30. But no single value for x can be
question. determined. So I is not sufficient.
● What per cent of a group of people are Let’s take II. There are several two-digit
women with blue eyes? integers divisible by 3: 12, 15, 18, 24,
I. Of the women in the group, 5 per just to name a few. So II does not help
cent have blue eyes. to get a single value for x, and hence
II. Of the men in the group, 10 per cent is not sufficient. Since you are not given
have blue eyes. the option of I and II together providing

TS
the answer, you may mark (d).
Solution: Unless the total number of
people in the group and the total number ● What is the rate of compound interest
of women with blue eyes are known, earned by Hiren at the end of 2 years?

H
the problem cannot be answered. I. Simple interest at the same rate for
Now take I. It does not give the required one year is Rs 1020 and the rate of

G
information. So it is not sufficient on its interest is 12% per annum.
own. II. The amount invested is Rs 8500.
Take II. The information given is
irrelevant unless totals are given. So II
alone is also insufficient. U Solution: Take I. You have the rate per
cent and simple interest for a year. The
question gives you the time. So you can
O
Now, we have to see if both taken easily find the principal
together can supply the answer. Neither 100 × S.I.
H
statement gives information on the total P =
R×T
numbers of people on women, so the Once you know the principal, you can
_T

question cannot be answered with the find the C.I.


data provided by statements I and II. T
The correct answer response is (d). R
C.I. = P 1+ −1
100
C

● If n is an integer, is n + 1 odd?
I. n + 2 is an even integer. as all elements are available.
PS

II. n – 2 is an odd integer. So I is sufficient.


Now take II. Just with the principal and
Solution: Take I. If n + 2 is an even the time (given by the question) you
integer, n must be even. So n + 1 will cannot find the compound interest
U

be an odd integer. I is sufficient. without more information. So II is not


But you have to take up II as well. If sufficient.
n – 1 is an odd integer, n must be even.
@

Your answer is (a).


So n + 1 would be an odd integer.
So I is sufficient on its own, and II is Question Type I: Variance in Answer Responses
sufficient on its own to answer the The set of answer responses given in the
question. directions may vary: the response (c) in Question
Type I may read:
The correct answer response is (c).
Give answer (c) if the data in both Statements
● What is the value of the two-digit I and II together are necessary to answer the question.
integer x? The other responses will remain as they are.
I. The sum of the two digits is 3.
II. X is divisible by 3. If this response is among the options, and
you find that neither I nor II is sufficient by itself
Solution: The digits will be from among to solve the problem, you should not be in a
the integers from, 0 to 9, inclusive. hurry to mark the response that neither is
624 ✫ Quantitative Aptitude

sufficient. You have to check if both statements Question Type II


together can help you get the answer. In this format, there are three statements below
each question, and you have to check the
Study the example given below.
sufficiency of each statement to answer the
● What is the area of the rectangle? question. The answer options in this kind of
I. The ratio of the length and breadth question usually follows each question item as
is 3 : 2. various combinations of the statements are
II. The area of the rectangle is 3.6 times provided in the options.
its perimeter.
Directions: Each of the questions below consists of

TS
Solution: Take I. Let’s take length l as a question followed by three statements. You have to
3x and breadth b = 2x metres. study the question and the statements and decide
Area = 3x × 2x = 6x2 metres2 which of the statement(s) is/are necessary to answer
the question.

H
However, a specific value of x, and ● What is the area of the rectangular field?
therefore the area, cannot be derived. So I. The perimeter of the field is 110

G
I is not sufficient. metres.
Now take II. Without a specific number II. The length is 5 metres more than the
for at least one dimension, l or b, or for
the perimeter, we cannot get a clear
U width.
III. The ratio between the length and
the width is 6 : 5 respectively.
O
answer with the help of II alone. So II
also is not sufficient by itself. (a) I and II only
(b) Any two of the three
H
But with option (c) asking if both
(c) All I, II and III
statements together can get the answer, (d) I, and either II or III only
we need to check further. Now, you can
_T

combine what I and II say to arrive at Solution: The answer options are not
an answer. simple. You have not only to check each
From I, we have l and b = 3x and 2x. statement for its necessity in answering
C

Interpolating in II, we have the question, but you have also to see
Perimeter = 2 (3x + 2x) m = (10x) m. in which combinations the statements
PS

From I, we have area = 6x2 work.


Now 6x2 = 3.6 times perimeter (from II) From I, you can get the combined length
and breadth of the field
or 6x2 = 3.6 × 10x Perimeter 110 m = 2 (l + b)
U

x2 (3.6 × 10) l + b = 55 m
or = But since it is a rectangle, you need to
x 6
@

know the length and breadth separately.


or x = 6 metres
From II you get
Now as from I we know 3x × 2x to be
l = b + 5
length and breadth, and we know value
From I and II now you can get l and b
for x, the area can be found.
and hence the area of the field.
So I and II together are sufficient to
But don’t be in a hurry to mark (a).
answer the question.
There is option (b) which says, ‘Any two
So you must read the answer responses
of the three’. So you need to check III—
carefully. If an option asks you to
which you should, anyway.
consider the sufficiency of both
III tells us l : b : : 6 : 5
statements together, you cannot stop
with just checking I and II separately. l 6
Or =
You must also check whether both b 5
together can supply the answer. 5l + 6b = 0
Unit Nine : Fundamentals / Data Sufficiency ✫ 625

Now III and I can also solve the problem the option. Once again you have three
and III and II can also solve the problem unknown variables and two equations.
No answer; eliminate.
So the correct answer response is (b).
Take (c) II and III—again two equations
● What is the price of one dozen oranges? but three unknown variables. No answer,
I. Total cost of 2 dozen oranges and 3 eliminate.
dozen bananas is Rs 110. With three unknown variables, and three
II. Total cost of 3 dozen apples and 1 equations, a solution could be possible.
dozen bananas is Rs 170. From I 2x + 3y = 110 … (i)

TS
III. Total cost of 1 dozen each of oranges, From II 3z + y = 170 … (ii)
apples and bananas is Rs 95. From III z + x + y = 95 … (iii)
(a) I and II only or I and III only
Multiplying (iii) by 3 we get

H
(b) I and III only or II and III only
3z + 3x + 3y = 285
(c) II and III only
(d) I, II and III are all required to answer Subtracting (ii)

G
the question 3z + 3x + 3y = 285
3z + y = 170
Solution: In such questions, take up the
options.
U We get
– – –
3x + 2y = 115 … (iv)
O
In (a) you have I and II as part of the We have equations (i) and (iv) for x and
option. y. Now we can work out the equation
Statements I and II do not give you an for two variables and find the price of
H
answer: there are three variables and 2 one dozen oranges.
equations. So (a) cannot be correct; But we have used all three statements
_T

eliminate it. for arriving at the solution.


In (b) you have I and III only as part of So answer response (d) is correct.
C
PS
U
@
626 ✫ Quantitative Aptitude

Practice Exercise 52
Directions (Qs. 1–17): In the following questions, 8. A shop sells only toy cars and toy boats. It
a question is given followed by two statements. You sells each in only two colours—red and blue.
have to consider the statements and decide whether Last year the shop sold 900 toys, half of which
they provide sufficient data to answer the given were red. How many toy boats did the shop
question. Give answer sell?
(a) if Statement I alone is sufficient but I. The shop sold three times as many
statement II alone is not sufficient; blue toy boats as red toy cars last
(b) if each Statement alone is sufficient; year.

TS
(c) if both Statements together are sufficient II. The shop sold half as many blue toy
but neither Statement alone is sufficient; cars as blue toy boats last year.
(d) if Statements I and II are not sufficient 9. If x is a positive integer, is x divisible by 48?
and more data is required to answer the

H
I. x is divisible by 8.
question. II. x is divisible by 6.
1. What is the value of integer x?

G
10. A box contains a number of toys, including
I. x is a multiple of 7. 5 dolls. If two toys are selected from the box
II. 575 < x < 590 at random, what is the probability that both
2. Shyam can chop down 4 trees in an hour.
How long does it take Mohan to chop down
U will be dolls?
I. The probability that the first toy
O
4 trees? 1
I. Mohan takes twice as long as Shyam picked will be a doll is .
to chop down trees. 2
II. The probability that the second toy
H
II. Mohan spends 6 hours per day 4
chopping down trees. picked up will be a doll is .
9
_T

3. A B 11. Rahul, Anurag and Vivek started a business


together. In what proportion would the
r annual profit be distributed among them?
C D
C

p q I. Rahul got one-fourth of the profit.


II. Rahul and Vivek contributed 75 per
PS

E F cent of the total investment.


If both ABDC and CDFE are parallelograms, 12. Ravi, Gagan and Nitin are running a business
what is q + r? firm in partnership. What is Gagan’s share
I. r = 70 in the profit earned by them?
U

II. p = 110 I. Ravi, Gagan and Nitin invested the


4. Is the average (arithmetic mean) of x, y, and amounts in the ratio of 2 : 4 : 7.
@

z equal to 8? II. Nitin’s share in the profit is Rs 8750.


I. Three times the sum of x, y, and z 13. How long will Machine Y, working alone,
is equal to 72. take to produce x candles?
II. The sum of 2x, 2y, and 2z is equal I. Machine X produces x candles in 5
to 48. minutes.
5. What is the value of x? II. Machine X and Machine Y working
I. x2 + 5x + 4 = 0 at the same time produce x candles
II. x is not prime. in 2 minutes.
6. What is the sum of a, b, and c?
I. 2a + b + 3c = 45 14. A and B together can complete a task in 7
II. a + 2b = 30 days. B alone can do it in 20 days. What part
of the work was carried out by A?
7. What is the value of integer x?
I. A completed the job alone after A
I. x
64 = 4
2
and B worked together for 5 days.
II. x = 2x + 8 II. Part of the work done by A could
Unit Nine : Practice Session ✫ 627

have been done by B and C together I. The diagonal of the square is x inches.
in 6 days. II. The side of the square is y inches.
15. How much time did X take to reach the 20. What is the area of the circle?
destination? I. An arc of length 4 cm subtends an
I. The ratio between the speeds of X angle of 60° at the centre.
and Y is 3 : 4. II. A chord of length 5 cm subtends an
II. Y takes 36 minutes to reach the same angle of 90° at the centre.
destination. 21. What is the capacity of a cylindrical tank?
16. Two towns are connected by railway. Can you I. Radius of the base is half of its height

TS
find the distance between them? which is 28 metres.
I. The speed of a mail train is 12 km/hr II. Area of the base is 616 sq. metres and
more than that of an express train. its height is 28 metres.
II. A mail train takes 40 minutes less 22. Is a given rectangular block, a cube?

H
than an express train to cover the I. At least 2 faces of the rectangular
distance. block are squares.

G
17. The towns A, B and C are on a straight line. II. The volume of the block is 64.
Town C is between A and B. The distance Directions (Qs. 23–25): Each of the three items
from A to B is 100 km. How far is A from C?
I. The distance from A to B is 25% more
than the distance from C to B. U below consists of a question and two statements
numbered I and II given below it. You have to decide
whether the data provided in the statements are
O
1
II. The distance from A to C is of the sufficient to answer the question. Read both the
distance from C to B. 4 statements and give answer.
H
(a) if data in statement I alone are
Directions (Qs. 18–22): Each of the questions
sufficient to answer the question, but
given below consists of a statement and/or a
_T

the data in statement II alone are not


question and two statements numbered I and II
given below it. You have to decide whether the sufficient to answer the question;
data provided in the statement(s) is/are sufficient (b) if data in either of the statements alone
to answer the question. Read both the statements are sufficient to answer the question;
C

and give answers (c) if the data in both statements together


(a) if the data in Statement I alone are are required to answer the question;
PS

sufficient to answer the question, while (d) if the data in both statements together
the data in Statement II alone are not are not sufficient to answer the
sufficient to answer the question; question.
(b) if the data in Statement II alone are 23. How many children are there in the group?
U

sufficient to answer the question, while I. Average age of the children in this
the data in Statement I alone are not group is 15 years. The total age of all
@

sufficient to answer the question; the children in this group is 240 years.
(c) if the data either in Statement I or in
II. The total age of all children in the
Statement II alone are sufficient to
group and the teacher is 264 years. The
answer the question;
(d) if the data in both Statements I and II age of the teacher is 8 years more than
together are not sufficient to answer the the average age of the children.
question. 24. What is the per cent profit earned?
18. The area of a playground is 1600 m2. What I. A shopkeeper spent Rs 140 and bought
is its perimeter? some articles.
I. It is a perfect square playground. II. All articles were sold for Rs 150.
II. It costs Rs 3200 to put a fence around 25. What is the number?
the playground at the rate of Rs 20 I. The sum of two digits is 8. The ratio
per metre. of the two digits is 1 : 3.
19. Area of a square is equal to the area of a II. The product of two digits of a number
circle. What is the circumference of the circle? is 12. The quotient of two digits is 3.
628 ✫ Quantitative Aptitude

Answers with Solutions


1. Ans. (d) 4. Ans. (b)
Sol. There are an infinite number of multiples Sol. We are asked only about the average and
of 7, so I is not sufficient to answer the not the individual values of x, y and z. We
question; (a) is eliminated and so is (b). can find the average if we know the sum of
Consider statement II: it narrows the the three numbers. Statement I tells us that
range of possibilities, and with the help 3 (x + y + z) = 72.
of I could give the answer. If x is a 72
So x + y + z = = 24
multiple of 7 and lies between 590 and

TS
3
575, we have to see how many numbers If the sum is known, we can find the
between the two are multiples of 7. 24
Divide 590 by 7; you get 84 and average: = 8
3

H
remainder 2. So 590 – 2 = 588 is a This is the average mentioned by the
multiple of 7. If you subtract 7 from 588, question, so I alone is sufficient. But we
you get another multiple of 7, namely, have to check II before option (b) can be

G
581, which is also between 575 and 590. eliminated.
So there are two multiples of 7 that can Statement II tells us that
be x, and no definite value of x is
possible even with both statements U 2 (x + y + z) = 48
Or x + y + z =
48
= 24
O
together. The answer is (d). 2
24
2. Ans. (a) Again, = 8
3
H
Sol. Statement I tells you that Mohan takes So II alone is also sufficient
twice as long as Shyam to chop down Answer response (b) is correct.
trees. The question tells you the time
_T

5. Ans. (d)
taken by Shyam to cut down 4 trees. You
Sol. In statement I there is a quadratic equation,
can now calculate the time it would take
so we need to factorise and get our options
Mohan to do the same. So I is sufficient.
for x.
C

However, you have to check II before x2 – 5x + 4 = 0


you can eliminate option (b). Statement (x – 4) (x – 1) = 0
PS

II tells you how long. Mohan works ∴ x could be either 4 or 1


every day at chopping down trees, but
Statement I alone is not sufficient, as we
this gives you no data on time taken to
have no single value for x. Options (a)
cut down 4 trees. So II is not sufficient.
U

and (b) are eliminated. But we have to


Answer response (a) is correct. check II to see if (c) can be the answer.
3. Ans. (b) From statement II we can have many
@

Sol. As the given figures are parallelograms, values for x, as the majority of numbers
there are only two angle measures—one are not prime. And, as both 1 and 4
acute and the other obtuse. We know (what we deduced from I) are not prime,
that the two angles together add up to we still have no specific value for x. The
180°. From statement I, we have r = 70. statements together are not sufficient to
Angle q will be the same. So I alone is answer the question. Answer response
sufficient. However, we have to check II (d) is correct.
before we can eliminate option (b). 6. Ans. (c)
Statement II gives us the measure of the Sol. Statement I is insufficient on its own as
obtuse angle which we can subtract from it provides a three-variable equation
180° to get the value of the smaller angle, which cannot be solved. So (a) is
and we can calculate the value of q. So eliminated; so is (b). We have now to
II is also sufficient on its own. The check for (c). Let’s take both statements
correct answer is (b). together. Add the two equations:
Unit Nine : Practice Session ✫ 629

I. 2a + b + 3c = 45
Red Blue Total
II. + a + 2b = 30
3a + 3b + 3c = 75 Toy boats 2x
3 (a + b + c) = 75 Toy cars x
75
a + b + c = = 25 Total 450 450 900
3
So the two statements together can get
you the answer to the question. The Though two unknowns remain here, too,
correct answer response is (c). we can get some useful information.

TS
7. Ans. (a) If 2x + x = 450,
Sol. From I it is clear that x = 3 as 3 64 = 4 450
x = = 150
We have to check II to see if (b) is to 3

H
be eliminated. So the shop sold 2 × 150 = 300 blue toy
x2 – 2x + 8 boats.
x2 – 2x – 8 = 0 Now, we can add the information to

G
Factorising, we get what we have in I and get an answer to
(x – 4) (x + 2) = 0 the question. As the shop sold 3 times
So, x can be either 4 or –2. As we get
no single value for x, statement II is not U as many blue toy boats as red toy cars
(from I), the number of red toy cars sold
O
sufficient. 300
is = 100.
8. Ans. (c) 3
Subtract 100 from 450 to get total of red
H
Sol. Draw up a table to solve the question. toy boats sold. So 350 red toy boats were
The data of the question can be filled in. sold. A total of 350 + 300 (blue toy boats)
_T

Red Blue Total add up to 650 toy boats.


Answer response (c) is correct.
Toy boats 9. Ans. (d)
C

Toy cars Sol. Check statement I. Plug in some numbers


for x divisible by 8 and see if all of them
PS

Total 450 450 900 are divisible by 48. Actually, there are
many numbers divisible by 8 that are not
As half the toys were red, half will be divisible by 48. Take numbers less than
blue. Now, let’s enter the data of 48, say 32, 24, 16 … all of which are
U

statement I in the table as: divisible by 8 but not by 48. So I is not


sufficient. By the same token, statement
@

Red Blue Total II is also insufficient. And taken together,


the statements solve nothing. Don’t be
Toy boats 3x misled by 8 × 6 being 48: there are
Toy cars x several numbers divisible by both 8 and
6, but not by 48, such as 24, 72, 120. So
Total 450 450 900 (d) is the correct answer response.
10. Ans. (b)
There are still two unknowns—red toy Sol. If there are 5 dolls, and the probability
boats and blue toy cars which cannot be of picking one first is 1 2 , it means
related. So I is not sufficient.
1 5
We now fill in data from statement II in = (taking x as total toys)
2 x
the table to see if (c) can be an option.
If 2x toy boats were sold, half that or x So, x = 10. So we can go on to the
toy cars were sold: probability of the second toy being a
630 ✫ Quantitative Aptitude

doll. After picking one, we have 4 dolls 13. Ans. (c)


and 9 toys left. So the probability of x
Sol. I. Gives, Machine X produces
selecting a doll a second time is 5
candles in 1 min.
1 4 2 II. Gives, Machines X and Y produce
× or
2 9 9 x
candles in 1 minute
Statement I is sufficient. 2 x x 3x
From I and II, Y produces – =
But we have to check statement II to see 2 5 10
if (b) could be an option. candles in 1 minute

TS
Statement II tells us the probability of 3x
candles are produced by Y in 1 min.
picking a doll for the second selection 10
4 x candles will be produced by Y in
is . It implies that the probability of 10
9 10

H
5 × x minutes = minutes
the first toy picked being a doll is . 3x 3
10
So statement II also can answer the Thus, I and II both are necessary to get

G
question on its own. the answer.
Answer response (b) is correct. 14. Ans. (a)
11. Ans. (c)
U Sol. B’s 1 day’s work =
1
1
20
(A + B)’s 1 day’s
O
Sol. Let the total investment be Rs x work = .
x 7
Then, R = 5
H
4 I. (A + B)’s 5 days’ work = .
6
75 3x 5 2
×x = Remaining work = 1 – =
_T

R + V = 7 7
100 4
2
3x x x ∴ work was carried out by A.
V= – = 7
4 4 2 II. is irrelevant.
C

x x x
∴ A = x – + = 15. Ans. (c)
4 2 4
PS

Sol. I. If Y takes 4 min., then X takes 3 min.


x x x
R : A : V = : : = 1 : 1 : 2 II. If Y takes 36 min., then X takes
4 4 2
3
Thus, both I and II are needed to get the × 36 min = 27 min.
U

answer. 4
∴ Correct answer is (c). Thus, I and II together give the answer.
@

12. (c) Let Ravi, Gagan and Nitin be R, G and 16. Ans. (d)
N respectively Sol. Let the distance between the two stations
be x km.
I. R : G : N = 2 : 4 : 7
II. N = 8750 I. Let the speed of the express train be
y km/hr.
From I and II, we get:
Then, speed of the mail train
When N = 7, then G = 4.
= (y + 12) km/hr.
When N = 8750, then G
x x 40
4 II. – =
= × 8750 = 5000. y ( y + 12) 60
7
Thus, even I and II together do not
Thus, both I and II are needed to get the
give the value of x.
answer. Correct answer is (c).
Unit Nine : Practice Session ✫ 631

17. Ans. (b) r2 = y2


Sol. Let AC = x km. Then, CB = (100 – x) y
r=
km. π
∴ Circumference of the circle = 2 πr,
A x C (100 – x) B which can be obtained.
I. AB = 125% of CB Thus, II alone gives the answer
∴ Correct answer is (c).
125
⇔ 100 = × (100 – x) 20. Ans. (c)
100
2πRθ

TS
100 × 100 Sol. I. Length of arc =
⇔ 100 – x = = 80 360
125
22
⇔ x = 20 km 2× × R × 60
∴ AC = 20 km. ⇔4= 7

H
Thus, I alone gives the answer. 360
1 1

G
II. AC = CB ⇔ x = (100 – x)
4 4
⇔ 5x = 100 ⇔ x = 20
∴ AC = 20 km.
Thus, II alone gives the answer.
U R
O
o R
O
90
∴, Correct response is (b)
18. Ans. (c) A B
H
2
Sol. Area = 1600 m
_T

I. Side = 1600 m2 = 40 m. This gives R and therefore, area of the


So, perimeter = (40 × 4) m = 160 m. circle = πR2.
∴ I alone gives the answer. Thus, I only gives the answer.
Total cost 3200 II. R2 + R2 = 52
C

II. Perimeter = = 2R2 = 25


Cost per metre 20 25
or R2 =
PS

= 160 m
2
∴ II alone gives the answer.
∴ Area of the circle = πR2
∴ Correct answer is (c)
22 25
19. Ans. (c) ×
U

= sq. cm.
7 2
Sol. I. Area of the circle = Area of the square Thus, II only gives the answer.
1 ∴ Correct answer is (c).
@

= x 2 sq. inches.
2 21. Ans. (c)
1 Sol. I Gives, h = 28 m and r = 14 cm.
r2 = x2
2 ∴ Capacity = πr 2h, which can be obtained.
Thus, I alone gives the answer.
x2 x
r= = II Gives, πr 2 = 616 m2 and h = 28 m
2π 2π
∴ Capacity = (πr 2 × h) = (616 × 28) m3
∴ Circumference of the circle = 2πr, `
Thus, II alone gives the answer.
which can be obtained.
∴ Correct answer is (c).
∴ I alone gives the answer.
22. Ans. (d)
II. Area of the circle = Area of the Sol. I Gives, any two of l, b, h are equal.
square
II Gives, Ibh = 64
= y 2 sq. inches. From I and II, the values of l, b, h
632 ✫ Quantitative Aptitude

may be (1, 1, 64), (2, 2, 16), (4, 4, 4) S.P. – C.P.


Profit per cent = × 100
Thus, the block may be a cube or C.P.
a cuboid. 150 − 140 10
= × 100 = × 100
∴ Correct answer is (d). 140 140
23. Ans. (a) = 7.14%
Sol. To find : No. of children in the group ∴ Both statements I and II are necessary to
Statement I answer the question.
We know that
25. Ans. (b)
No. of children

TS
Sol. Let the two digits be x and y
Total age of all children
= Consider statement I.
Average age Given that x + y = 8 ...(i)
Statement I provides both the required x 1

H
data, so statement I is sufficient. and y = 3 ...(ii)
In the data provided in Statement II, We have to find value of the two

G
there are 3 unknown variables i.e. variables x, y. For this we have two
teacher’s age, total age of children and equations.
no. of children but only two equations.
∴ Required no. of children cannot be
determined from this statement. U ∴ Value of variables can be found from
these two equations.
O
Now consider statement II.
So statement I alone is sufficient to Given that x × y = 12 ...(i)
answer the question. x
H
and y = 3 ...(ii)
24. Ans. (c)
Again we have two equations which are
Sol. To find : Per cent profit
_T

sufficient to find the value of the two


We can see that variables x and y.
Statement I gives C.P. = Rs 140 and ∴ The data either in statement I alone or
statement II says S.P. = Rs 150 in statement II alone are sufficient to
C

And we know that answer the question.


PS
U
@
Unit Nine : Practice Session ✫ 633

Practice Exercise 53
Directions: (Qs. 1–15): Each of the following (a) I only
questions consists of a question followed by three (b) II and III only
statements. You have to study the question and the (c) I and II only
statements and decide which of the statement/s is/are (d) Either only I or both II and III
necessary to answer the question.
5. How many workers are required for
1. What is the area of the hall? completing the construction work in 10
I. Material cost of flooring per sq. metre

TS
days?
is 250.
I. 20% of the work can be completed
II. Labour cost of flooring the hall is
3,500. by 8 workers in 8 days.
III. Total cost of flooring the hall is II. 20 workers can complete the work in

H
14,500. 16 days.
(a) I and II only III. One eighth of the work can be

G
(b) II and III only completed by 8 workers in 5 days.
(c) All I, II and III (a) I and III only
(d) Any two of the three (b) II and III only
2. What was the percentage of discount
offered? U (c) I only
(d) Any one of the three
O
I. Profit earned by selling the article for 6. What is the area of the isosceles triangle?
Rs 252 after giving discount was Rs I. Perimeter of the isosceles triangle is
H
52.
14 metres
II. Had there been no discount, the
II. Base of the triangle is 8 metres.
_T

profit earned would have been Rs 80.


III. Had there been no discount, the III. Height of the triangle is 5 metres.
profit earned would have been 40%. (a) I and II only
(a) I and II only (b) II and III only
C

(b) II and either I or III (c) I and II only or II and III only
(c) I and either II or III (d) I and III only
PS

(d) None of these 7. What is the speed of the train A?


3. What is the speed of the train? I. Train A crosses a 200-metre-long train
I. The train crosses a signal pole in 13 B running in the opposite direction
seconds.
U

in 20 seconds.
II. The train crosses a platform of length II. Speed of train B is 60 kmph.
250 m in 27 seconds. III. Length of train A is twice that of train
@

III. The train crosses another train B.


running in the same direction in 32
(a) I and II only
seconds.
(a) I and II only (b) II and III only
(b) I and III only (c) I and III only
(c) II and III only (d) All I, II and III
(d) Any two of the three statements 8. What is the cost of flooring a rectangular
4. What is the population of State A? hall?
I. When the population of State A is I. Perimeter of the rectangle is 60
increased by 15% it becomes 1.61 metres.
lakh. II. Angle between diagonal and breadth
II. Ratio of population of State A to that is 60°.
of State B is 7 : 8 respectively. III. Cost of flooring per square metre is
III. Population of State B is 1.6 lakh. Rs 125.
634 ✫ Quantitative Aptitude

(a) I and III only I. Total amount invested in the business


(b) II and III only is Rs 22,000.
(c) I and III only or II and III only II. Profit earned at the end of 3 years
(d) The question cannot be answered is 3/8 of the total investment.
even with the data in all the three III. The average amount of profit earned
statements per year is Rs 2750.
(a) I or II or III
9. Who among M, N, P, Q and R earns the
(b) Either III only, or I and II together
maximum?
(c) Any two of the three
I. M earns less than P but not less than
(d) All I, II and III are required

TS
R.
II. Q earns more than M but not as 13. In how many days can 10 women finish a
much as N. work?
III. N earns more than M and R. I. 10 men can complete the work in 6

H
(a) The question cannot be answered days.
even with the data in all the three II. 10 men and 10 women together can
3

G
statements complete the work in 3 days.
(b) I and II only 7
III. If 10 men work for 3 days and
(c) I and II only or I and III only
(d) I and III only
U thereafter 10 women replace them,
the remaining work is completed in
O
10. What is the price of one dozen oranges? 4 days.
I. The total cost of 2 dozen oranges and (a) Any two of the three
3 dozen bananas is Rs 110. (b) I and II only
H
II. The total cost of 3 dozen apples and (c) II and III only
1 dozen bananas is Rs 170. (d) I and III only
_T

III. The total cost of 1 dozen each of 14. In how many days can the work be
oranges, apples and bananas is Rs completed by A, B and C together?
95.
I. A and B together can complete the
(a) I and III only or II and III only
C

work in 6 days.
(b) I and II only or II and III only
(c) II and III only II. B and C together can complete the
PS

(d) All I, II and III are required to answer 3


work in 3 days.
the question. 4
11. What is R’s share of profit in a joint venture? III. A and C together can complete the
U

I. Q started a business investing Rs 1


work in 3 days.
80,000. 3
@

II. R joined him after 3 months. (a) I only


III. P joined after 4 months with a capital (b) II only
of Rs 1,20,000 and got Rs 6,000 as his (c) III only
share of profit. (d) Information in all the three statements
(a) All I, II and III is necessary to answer the question.
(b) I and III only 15. If both the pipes are opened, how many
(c) II and III only hours will it take to fill the tank?
(d) Even with the statements I, II, and III, I. The capacity of the tank is 400 litres.
the answer cannot be arrived at II. The pipe A fills the tank in 4 hours.
12. Three friends P, Q and R started a partnership III. The pipe B fills the tank in 6 hours.
business investing money in the ratio of 5 : 4 (a) Only I and II
: 2 respectively for a period of 3 years. What (b) Only II and III
is the amount received by P as his share in (c) All I, II and III
the total profit? (d) Any two of the three
Unit Nine : Practice Session ✫ 635

Answers with Solutions


1. Ans. (c) 25200
Sol. Let x m2 be the area of the hall C.P.3 = = 180
140
Material cost of flooring per sq metre
Here again this is not possible since
= 250
C.P.3 < C.P.1
∴ For total area it would be 250 × x
So statement III is also not relevant.
Labour cost of flooring the hall = 3500
Total cost of flooring the hall = 14500 3. Ans. (a)
Sol. Consider statement I

TS
Total cost = Material cost + Labour cost
14500 = 250x + 3500 Distance (D)
= Time (T)
Now x can be found. Speed (S)
∴ All the three statements are necessary Length of Train L t

H
to calculate the answer. = 13 seconds ... (i)
Speed S
2. Ans. (d) [ When a train crosses a pole, distance

G
Sol. Given statement (I) covered by train = length of train]
Here, selling price (S.P.) = Rs 252 Consider statement II
Profit earned P1 = Rs 52
Let cost price (after discount) = C.P.1
U Distance covered by a train crossing
a platform = Lt + Length of platform
D = Lt + 250 m
O
We know that
S.P. = C.P. + Profit Time taken = 27 sec
252 = C.P.1 + 52 D
H
S =
∴ C.P.1 = 252 – 52 = Rs 200 T
By statement II, Lt +250
_T

had there been no discount, profit earned ∴ S =


27
would have been Rs 80 Lt +250
Let cost price without discount be C.P.2 27 =
S
C

S.P. = C.P.2 + Profit Lt 250


252 = C.P.2 + 80 27 = +
S S
PS

C.P.2= 252 – 80 = 172 250


This cannot be possible since cost price 27 = 13 + [From (i)]
250 S
before discount should be more than
= 14
cost price after discount, i.e., S
U

C.P.2 should be > C.P.1 250


S = = 17.86 m/s (approximate)
but here C.P.2 < C.P.1 14
@

(172 < 200) Now consider statement III.


∴ Statement (II) is not relevant. We know that when a train crosses
Now consider statement (III). another train running in the same
Let cost price without discount be C.P.3 direction, distance covered =
Here without discount, profit earned difference in lengths of both trains.
= 40% Here length of the other train is not
S.P. = C.P. + Profit given.
So, statement III is not complete.
252 = C.P.3 + C.P.3 × 40% ∴ Statements I and II only are necessary
[ Profit percentage is always calculated to answer the question.
on cost price] 4. Ans. (d)
100 C.P.3 + 40 C.P.3 Sol. Consider statement I.
252 = Let x be the population of State A
100
After increasing the population by
25200 = 140 C.P.3 15%, we get
636 ✫ Quantitative Aptitude

x + 15% x = 1.61 lakh Consider statement I.


100 x + 15 x 20% of work can be completed by 8
= 1.61 lakh workers in 8 days.
100
∴ Remaining days = 10 – 8 = 2 days
115x = 1.61 × 100 lakh 20
161 Remaining work = 1 – 20% = 1 –
x = = 1.4 lakh 100
1
115 = 1 –
5
∴ Population of State A = 1.4 lakh 4
Now from statement II, = work
5

TS
1
Ratio of population of State A to that In 8 days, work is completed by 8
of B = 7 : 8 5
workers 1
From statement III, In 1 day, work is completed by
Population of State B = 1.6 lakh 5

H
8 × 8 workers
From II and III, In 1 day, 1 work is completed by 8 × 8 × 5
Population of State A(P) 7 workers

G
=
Population of State B 8 In 2 days, 1 work is completed by
Population of State A 8×8×5
7 workers
1.6lakh
=
8
7×1.6 lakhU 2
In 2 days,
4
work is completed by
O
Population of State A = 5
8 8×8×5× 4
= 1.4 lakh = 128 workers
2×5
H
∴ Statement I by itself or II and III
∴ Additional workers required to complete
together are necessary for answering
the work in 10 days can be found.
_T

the question.
[Alternative Method [(128 – 8) = 120 workers]
Consider statement I. ∴ Statement I can answer the question.
Let x be the population of State A. Now consider statement II.
C

Then 20 workers can complete the work in 16


x + 15%x = 1.61 lakh days
PS

and x can be found In 16 days, 1 work is completed by 20


So, statement I is necessary to find the workers
answer.
In 1 day, 1 work is completed by 20
Now consider statement II
workers 16
U

Ratio of population of State A : State B


=7:8 1 work is completed by
In 10 days,
@

Population of State B (1.6 lakh) is given 16


in statement III 20
workers
From both statements II and III, we get 10
an equation In 10 days, 1 work is completed by
7 Population of State A 20
= × 16 = 32 workers
8 1.6 lakh 10
∴ Statement II can answer the question.
and the population of State A can be Now consider statement III.
easily found. 1
∴ Either statement I or both II and III are of work can be completed by
8
necessary to answer the question.] 8 workers in 5 days
5. Ans. (d) ∴ Remaining days = 10 – 5 = 5 days
Sol. No. of days in which construction 1 7
should be completed = 10 days Remaining work = 1 – =
8 8
Unit Nine : Practice Session ✫ 637

1 7. Ans. (d)
In 5 days, work is completed by
8 Sol. We know that when two trains are
8 workers running in the opposite directions,
1 their Relative Speed
In 1 day, work is completed by = Speed of first train
8
8 × 5 workers + Speed of second train
In 1 day, 1 work is completed by 8 × 5 × 8 Here relative speed
workers = Speed of train A + Speed of train B
In 5 days, 1 work is completed by Also Relative speed of approach

TS
8 × 5 × 8 workers Distance covered
=
5 Time taken
7 Let speed of train A be x m per hour
In 5 days, work is completed by
8 We also know that when two trains cross

H
8×5×8 7 each other,
× = 56 workers distance covered = Length of train A
5 8

G
+ Length of train B
∴ Additional workers needed to complete
Speed of train A + Speed of train B
the work in 10 days
= 56 – 8 = 48 workers
∴ Statement III can answer the problem.
∴ Any one of the three statements can U =
Length of train A + Length of train B
Time taken
O
Statement I gives us length of train B,
answer the question. i.e., 200 m and time taken i.e., 20 seconds
Statement II gives us speed of train B,
H
6. Ans. (c) i.e., 60 km per hour
Sol. Area of isosceles triangle Statement III says that length of train A
base
_T

= 4(side)2 – (base)2 a a = twice length of train B


4 h Length of train A = 2 × 200 metres
b
= 4(a)2 – (b)2 = 400 metres
4 b Putting these values in the formula,
Statement I:
C

Perimeter = 14 metres we get 60 × 1000


2a + b = 14 .... (i) Speed of train A + metres per
3600
PS

Statement II: second


Base b = 8 metres 400 + 200
Putting b = 8 in (i), we get =
20
2a + 8 = 14
U

50 600
2a = 14 – 8 x + =
3 20
6
@

a = = 3 50
3 x + = 30
3
∴ From statements I and II, area of the Δ 3x + 50 = 90
can be determined. 3x = 90 – 50
Again consider statement II. 3x = 40
base b = 8 metres (Given) 40
Consider statement III. x =
3
Height h = 5 metres (Given) 40
∴ Speed of Train A = metres per second.
We know that 3
1 All statements I, II and III are necessary
Area of a triangle = × base × height
2 to answer the question.
∴ We can find area from statements II and III.
8. Ans. (d)
Either statements I and II
Sol. Cost of flooring a hall
or statements II and III are sufficient to
= Area of the floor
answer the question. × Cost per square metre
638 ✫ Quantitative Aptitude

Consider statement I. 9. Ans. (a)


Given: Perimeter of the rectangle Sol. Statement I
= 60 metres. M earns less than P but not less than R
We know that R ≤ M < P
Perimeter of a rectangle = 2 (l + b) Statement II
Where l is length and b is the breadth Q earns more than M but not as much as N
of the rectangle M < Q < N
60 = 2(l + b) Statement III
N earns more than M and R
60
l + b = N > M

TS
2 N > R
l + b = 30
From the three statements we can say
l = 30 – b that N and P earn more than any one
But we don’t know the value of l or b.
else. But among N and P who earns

H
Consider statement II.
more cannot be determined.
Given: Angle between diagonal and
breadth is 60° ∴ The answer cannot be determined even

G
This information does not provide sufficient with the data in all the three statements.
data to calculate the area of the floor. 10. Ans. (d)
Sol. To find: Price of one dozen oranges
Consider Statement III.
Cost of flooring per square metre
= Rs 125 U Let price of one dozen oranges be Rs C,
price of one dozen apples be Rs A and
O
We know that price of one dozen bananas be Rs B
Area of a rectangle = l × b ... (i) Consider statement I
2C + 3B = 110 ... (i)
H
Putting l = 30 – b in (i) we get
Area = (30 – b) × b Consider statement II
= 30b – b 2 3A + B = 170 ... (ii)
_T

But value of b is not given. Consider statement III


So cost of flooring cannot be found even C + A + B = 95 ... (iii)
with the data in all the three statements. Multiplying equation (iii) by 3, we get
[Alternative Method 3C + 3A + 3B = 285 ... (iv)
C

D C Subtracting equation (ii) from equation


(iv), we get
60°
PS

3C + 2B = 115 ... (v)


Now consider equation (v) and equation
(i)
A B Multiplying equation (v) by 3, we get
We know that
U

9C + 6B = 345 ... (vi)


Cost of flooring rectangular hall and multiplying equation (i) by 2, we get
= Area of hall × Cost per sq. metre 4C + 6B = 220 ... (vii)
@

Area = length × breadth Subtract equation (vii) from equation


= l × b (vi), we get
Statement I says perimeter = 60 metres 5C = 125
We know that 125
Perimeter of a rectangle = 2 (l + b) C =
5
60 = 2 (l + b) C = 25
Statement II gives us the angle between ∴ Cost of one dozen oranges = Rs 25
diagonal and breadth which is not All I, II and III are required to answer
sufficient to find the area. the question
Statement III gives cost of flooring per [Alternative Method
sq. metre. Let price of one dozen oranges be Rs C
But none of the statements gives us the Let price of one dozen apples be Rs A
value of l or b. Let price of one dozen bananas be Rs B
So the question cannot be answered even From statement I, we get
with data in all the three statements.] 2C + 3B = 110 ... (i)
Unit Nine : Practice Session ✫ 639

From statement II, we get So, 10 women can finish the work in
3A + B = 170 ... (ii) 8 days.
From statement III, we get Consider Statement III
C + A + B = 95 ... (iii) (10 men’s work for 3 days) + (10
We can see that there are three unknown
women’s work for 4 days) = 1
variables A, B, C, and three equations (i),
(10 × 3) men’s 1 day’s work + (10 × 4)
(ii), (iii)
women’s 1 day’s work = 1
∴ Value of A, B, C can be determined.
All I, II and III are required to answer 30 men’s 1 day’s work + 40 women’s
the question.] 1 day’s work = 1.
Thus, I and III will give us the answer.

TS
11. Ans. (d)
Sol. From I, II and III, we get P : Q : R And, II and III will give us the answer.
= (120000 × 8) : (80000 × 12) : (x × 9) Correct answer response is (a).
Where x is the investment of R. 14. Ans. (d)

H
Since R’s investment is not given, the Sol. Consider Statement I 1
above ratio cannot be obtained (A + B)’s I day’s work =
Consider Statement II 6
Given data is inadequate.

G
12. Ans. (b) 4
(B + C)’s 1 day’s work =
Sol. I and II give profit after 3 years 15
= Rs 3 × 22000 = Rs 8250
8 U Consider Statement III
(A + C)’s 1 day’s work =
Adding, we get
10
3
O
From III also, profit after 3 years
= Rs (2750 × 3) = Rs 8250 2(A + B + C)’s 1 day’s work
5
H
∴ P’s share = Rs 8250 × = Rs 3750 1 4 3
11 = + +
6 15 10
Thus, either III is redundant or I 5+8+9
_T

32
and II are redundant. = =
13. Ans. (a) 30 30
Sol. Consider Statement I (A + B + C)’s 1 day’s work
1 22 11
(10 × 6) men can complete the work
C

= × =
in 1 day. 2 30 30
1 Thus, A, B and C together can finish the
1 man’s 1 day’s work =
PS

Consider Statement II 60 30
work in days
24 24 11
10 × men + 10 × women can Hence I, II and III are necessary to
7 7
U

complete the work in 1 day. answer the question.


240 15. Ans. (b)
@

men’s 1 day work Sol. Consider Statement II


7 1
240 Part of the tank filled by A in 1 hour =
+ women’s 1 day work = 1 4
7 Consider Statement III
240 1 240 1
× + women’s 1 Part of the tank filled by B in 1 hour =
7 60 7 6
(A + B)’s 1 hour’s work
day’s work = 1.
1 1 5
240 = + =
women’s 1 day’s work 4 6 12
7 ∴ When both A and B are opened
4 3 together, they will fill the tank in
= 1– =
7 7 5
10 women’s 1 day’s work hrs = 2 hrs 24 min.
12
3 7 1 So, statements II and III are needed.
= × × 10 =
7 240 8 ∴ Correct answer is (b).
640 ✫ Quantitative Aptitude

Practice Exercise 54
Directions (Qs. 1–10): Each of the questions below II. Area of the base is 616 sq. metres and
consists of a question and two statements numbered its height is 28 metres.
I and II given below it. You have to decide whether
6. What is the per cent rate of interest per
the data provided in the statements are sufficient to
annum on an investment of Rs 12,500?
answer the questions. Read both the statements and
I. The compound interest for 2 years is
give answer.
(a) If the data in statement I alone are more than the simple interest for the
same period by Rs 500.

TS
sufficient to answer the question, while
the data in statement II alone are not II. The income from simple interest is
sufficient to answer the question. Rs 5000.
(b) If the data in statement II alone are 7. What is the length of the train which crosses

H
sufficient to answer the question, while a signal pole in 20 seconds?
the data in statement I alone are not I. The speed of the train is 54 kmph.

G
sufficient to answer the question. II. The train crosses the 150-metre-long
(c) If the data either in statement I alone platform in 30 seconds.
or in statement II alone are sufficient
to answer the question.
(d) If the data even in both the statements U 8. What is the depth of a cylindrical pipe?
I. The area of the base is 616 cm2.
O
I and II together are not sufficient to II. The perimeter of the base is 88 cm.
answer the question. 9. A class had 48 children. They were asked to
H
(e) If the data in both the statements I and sit in rows and columns. How many children
II together are necessary to answer the are seated in each row?
question.
_T

I. The number of columns is more than


1. By selling a product for Rs 100 how much the number of rows. 3
profit was earned? II. The number of rows is th of the
I. 20% profit would have been earned if 4
number of columns.
C

it had been sold for Rs 90.


II. The profit was one-third of the 10. What is the height of the triangle ABC?
I. AB is the base and the sum of the sides
PS

purchase price.
is 25 cm.
2. A train crosses another train running in the II. The ratio of the sides AB, BC and CA
opposite direction in x seconds. What is the is respectively 2 : 2 : 1.
speed of the train?
U

I. Both the trains are running at the same Directions (Qs. 11–15): Three statements follow the
speed. question asked. How many statement(s) would be
@

II. The first train is y cm long. required to answer the questions? Choose the number
3. The difference between the two digits of a from the options given below.
number is 6. What is the number ? 11. By selling an article what is the profit per
I. The digit at the unit place is bigger cent gained?
than the other digit. A. 5% discount is given on list price.
II. The sum of the two digits is 12. B. If discount is not given, 20% profit is
4. X, Y and Z are integers. Is X an odd number? gained.
I. An odd number is obtained when X C. The cost price of the article is Rs 5000.
is divided by 5. (a) Only A and B
II. (X + Y) is an odd number. (b) Only A and C
5. What is the capacity of a cylindrical tank? (c) Only B and C
I. Radius of the base of the tank is half (d) All A, B and C together
of its height which is 28 metres. (e) None of these
Unit Nine : Practice Session ✫ 641

12. What is the present age of father? (a) A and B only


A. Difference in age of father and son is (b) A only
24 years. 1 (c) B and C only
B. Present age of son is th of present (d) A and C only
5
age of father. (e) None of these
C. After 6 years from now, the age of son Directions (Qs. 16–25): Each of these questions has a
1
will be rd of the father’s age. question followed by information given in three
3
(a) Only A and B statements (I), (II) and (III). You have to study the
(b) Only B and C question along with the information in the three

TS
(c) Only A and C statements and decide as to the information in which of
(d) All A, B and C together the statement(s) is/are necessary to answer the question?
(e) Any two of the statements are required 16. What is the principal amount?

H
to answer the question. I. Difference between simple interest and
13. What is the cost of fencing a rectangular compound interest for 2 years is
Rs 37.50.

G
plot?
A. Length of the plot is twice its breadth. II. Simple interest for 3 years is Rs 2,250.
B. Area of the plot is 288 square metres. III. Simple interest and compound interest
C. Cost of fencing is Rs 4 per metre
square U for the first year are equal.
(a) Only I
O
(b) Only I and III
(a) B and C only
(c) Only I and II
(b) A and C only
(d) Any two of the three
H
(c) A and B only
(e) All I, II and III
(d) All the three statements A, B and C are
_T

required to answer the question 17. What is the area of the rectangular garden?
(e) The question cannot be answered even I. Perimeter of the garden is 220 metres.
with all the three statements together II. Length and the breadth of the garden
are in the respective ratio of 7 : 4.
C

14. What is the length of the train? III. Length is less than twice the breadth
A. The train crosses a signal pole in 18 by 10 metres.
PS

seconds. (a) Only I and II


B. The train crosses a signal pole in 30 (b) Only I and III
seconds (c) I and either II or III only
C. Speed of the train is 60 kmph. (d) Only II and III
U

(a) A and C only (e) The question cannot be answered even


(b) B and C only with the information in all the three
@

(c) C and A or B only statements


(d) A or B only
18. What was the population of State A?
(e) None of these
I. Males and females were in the ratio of
15. What will be the compound interest earned 4 : 5 respectively.
on an amount of Rs 5,000 in two years? II. 70% of the population was literate.
A. The simple interest on the same amount III. 8 lakh males were literate in the state.
at the same rate of interest in five years (a) Only I and II
is Rs 2,000. (b) Only II and III
B. The compound interest and the simple (c) Any two of the three
interest earned in one year is the (d) All I, II and III
same. (e) The question cannot be answered even
C. The amount becomes more than double with the information in all the three
on compound interest in 10 years. statements.
642 ✫ Quantitative Aptitude

19. In how many days can the work be (d) I, II and III
completed by 10 women? (e) None of these
I. 5 men can complete the work in 8 23. What is the rate of interest p.c. per annum?
days. I. An amount doubles itself at simple
II. 6 men and 4 women together can interest in 10 years.
complete the work in 5 days. II. Difference between the compound
III. One man and one woman together can interest and simple interest on an
do thrice the work done by a woman amount of Rs 15,000 in two years is
in one day. Rs 150.

TS
(a) Only I III. The compound interest accrued in 8
(b) Only II and III years is more than the amount
(c) Only I and III (principal).
(d) Any two of the three (a) Only I

H
(e) All I, II and III (b) Only II
20. What is the speed of the train? (c) Only II and III

G
I. The train crosses a signal pole in 14 (d) Only I and III
seconds. (e) Either I or II
II. The train crosses another train in 16
seconds.
III. The train crosses a 200-metre-long U 24. What are the marks scored by Abhijit in
English?
O
I. Marks scored by Abhijit in Maths are
platform in 24 seconds. more than his marks in Science by 20.
(a) Only I and II II. Total marks scored by Abhijit in Maths,
H
(b) Only I and III Science and English are 198.
(c) Only II and III III. Marks scored by Abhijit in Science is
_T

(d) Any two of the three more than his marks in English by 12.
(e) The question cannot be answered even (a) Any two of the these
with the information in all the three (b) Only II and III
statements. (c) I, II and III
C

21. What is the area of the right-angled triangle? (d) The question cannot be answered even
I. Base of the triangle is x cm. with the information in all the three
PS

II. Height of the triangle is y cm. statements


III. Hypotenuse of the triangle is z cm. (e) None of these
(a) Only I and II 25. In how many days will B alone complete the
U

(b) Only II work?


(c) Only II and III I. A and B together can complete the
(d) Any two of these
@

work in 8 days.
(e) None of these II. B and C together can complete the
22. What is the average age of the six members work in 10 days.
A, B, C, D, E and F in the family? III. A and C together can complete the
I. Total age of D and E is 14 years. work in 12 days.
II. Average age of A, B, C and F is 50 (a) Only I and II
years. (b) Only II and III
III. Average age of A, B, D and E is 40 (c) I, II and III
years. (d) The question cannot be answered even
(a) Only I and II with the information in all the three
(b) Only I and III statements
(c) Only II and III (e) None of these
Unit Nine : Practice Session ✫ 643

Answers with Solutions


1. Ans. (c) From Statement I, S.P. × 100
Sol. Consider Statement I. we can find C.P. =
Selling price or S.P. = Rs 100 (given) 100 + Profit per cent
S.P. is given above as Rs 100
If S.P. = Rs 90,
Profit = (S.P. – C.P.)
profit would have been 20% ∴ Profit can be found from Statement I.
We know that From Statement II,
S.P. = C.P. + Profit 1
given Profit = × C.P.

TS
where C.P. is the cost price 3
⇒ 90 = C.P. + 20% C.P. We know that
∴ Profit = S.P. – C.P.
[  Profit is always calculated as a
1
percentage of C.P.] i.e. C.P. = S.P. – C.P.

H
3
20 S.P. is given
⇒ 90 = C.P. + × C.P.
∴ Profit can be found from Statement II.

G
100

100 C.P. + 20 C.P.
90 = ⇒ Either of the Statements is sufficient.]
100


90 × 100 = 120 C.P.
C.P. =
9000
= Rs 75 U 2. Ans. (d)
Sol. We know that when a train crosses
another train running in the opposite
O
120 direction, the distance covered by the
⇒ C.P. = 75, when S.P. = 100 train
H
Actual profit = S.P. – C.P. D = Sum of lengths of both the trains
∴ The profit earned can be calculated from Speed of the train S = Relative Speed of
Statement I alone
_T

both the trains


Now consider Statement II. Time taken T = The time taken to
The profit was one-third of purchase completely clear each other.
price (given) Given, Time taken T = x seconds.
C

1
⇒ Profit = × C.P. Speed of the train =
Distance covered
3
Time taken
PS

If S.P. = 100, we get Consider Statement I.


S.P. = C.P. + Profit Given that both the trains are running at
1 same speed.
⇒ 100 = C.P. + C.P.
U

3 But nothing is mentioned about lengths


3+1 of the train.
⇒ 100 = C.P.
3 So speed of the train cannot be calculated
@

3 × 100 from Statement I.


⇒ C.P. =
4 Now consider Statement II.
⇒ C.P. = Rs 75 The first train is y cm long.
1 But length of second train is not given.
Profit=
3
× 75 ∴ Speed of train cannot be calculated from
∴ The profit earned can be calculated from Statement II.
Statement II alone. ⇒ The data in both the Statements I and II
⇒ The data either in Statement I alone or in together are not sufficient to answer the
Statement II alone are sufficient to answer question.
the question. 3. Ans. (e)
[Alternative Method Sol. Let x be the digit at unit’s place and
Given S.P. = Rs 100 let y be the digit at ten’s place
To find = Profit ∴ Number is 10y + x
644 ✫ Quantitative Aptitude

Given that difference between the two But the value of Y is not mentioned any-
digits is 6. where
Consider Statement I. ∴ Statement II alone is not sufficient to
Given that digit at unit’s place is bigger answer the question.
than the other digit ⇒ The data in Statement I alone is sufficient
⇒ x>y to answer the question.
∴ x–y=6 ... (i) 5. Ans. (c)
Now consider Statement II. Sol. We know that
The sum of the two digits is 12 (given) Capacity of a cylindrical tank or volume

TS
x + y =12 ...(ii) of a cylinder
Adding equations (i) and (ii), we get V = πr2h
2x = 18 where r is the radius of the base and h is
18 the height of the cylinder

H
x = =9
2 Radius of the base is half of its height
Putting the value of x = 9 in equation (i) Height h= 28 metres (given)

G
we get 1
Radius r = × 28 metres
9–y=6 2
∴ y=9–6=3 = 14 metres
∴ The number is 39 (it is given in Statement
I that the digit at unit’s place is bigger U Volume V = π × (14)2 × 28
=
22
× 14 × 14 × 28
O
than the other digit) 7
22
⇒ Data in both Statement I and II together [  We know that π = ]
7
H
are necessary to answer the question. = 17,248 sq. metres
[Alternative Method ∴ Statement I alone is sufficient to
_T

Let the two-digit no. be 10y + x answer the question.


From Statement I Now consider Statement II,
Given that area of base = 616 sq. m. and
x>y
height h = 28 metres
∴ As per data given in the question,
C

We know that area of base of a cylinder


x–7= 6
= πr2 = 616
From Statement II
PS

Volume V= 616 × 28
x + y = 12 = 17248 sq. metres
There are two unknown variables in the ∴ Statement II alone is sufficient to answer
two equations. So we can find the answer. the question.
∴ Both Statements I and II are necessary.]
U

⇒ The data either in Statement I alone or in


4. Ans. (a) Statement II alone are sufficient to answer
the question.
@

Sol. Given that X, Y, Z are integers.


Consider Statement I [Alternative Method
When X is divided by 5, an odd number We know that
is obtained (given). Capacity of a cylindrical tank = πr 2h
We know that this is possible only if X is where r = radius of its base and
an odd number. h = height
⇒ From Statement I, we can say that X is an Consider Statement I
odd number. We have values of r and h
Now consider Statement II, ∴ Capacity can be determined.
Given that (X + Y) is an odd number. Consider Statement II
In this case X shall be an odd number if Area of base = πr2 = 616 sq. metres
and only if Y is either 0 or an even Value of h is given
number. ∴ Capacity can be determined.
[  Sum of one even no. and an odd no. ⇒ Either Statement I alone or Statement II
is always an odd no.] alone is sufficient.]
Unit Nine : Practice Session ✫ 645

6. Ans. (a) ⇒ r2 × 1.25 = 500


Sol.Given Principal P = Rs 12500 500
Consider Statement I ⇒ r2 =
1.25
n = 2 years ⇒ r2 = 400
C.I. – S.I. = Rs 500 ⇒ r = 400 = 20
n
r Pnr ∴ Rate of interest = 20%
⇒ P 1+ –1 – = 500
100 100 ∴ We can find the answer from Statement I.
Now consider Statement II
Here we have the value of P and n.
Given S.I. = 5000
∴ r i.e. rate of interest, can be easily found

TS
out. Pnr
⇒ 5000 =
∴ Statement I is sufficient to answer the 100
⇒ 5000 × 100 = 12500 × n × r
question.
But n is not given.

H
Now consider Statement II
∴ Statement II aone is not sufficient to
We know that
answer the question.

G
Pnr
S.I. = ⇒ The data in Statement I alone are
100 sufficient to answer the question while
Income from S.I. =Rs 5000

⇒ 5000 =
12500 × n × r
100 U data in Statement II alone are not
sufficient to answer the question.]
O
7. Ans. (c)
Here value of n is not given Sol.Given, Time taken T = 20 seconds
∴ Value of r cannot be ascertained Consider Statement I
H
∴ Statement II alone is not sufficient Speed of train S = 54 kmph
⇒ Data in Statement I alone are sufficient We know that
_T

while data in Statement II are not Distance covered D by a train crossing a


sufficient to answer the question. signal pole = Length of the train L
[Alternative Method Also Distance = Speed × Time
Consider Statement I ∴ L = 54 kmph × 20 seconds
C

Principal P = Rs 12500 So the data in Statement I are sufficient


n = 2 yrs to find the answer.
PS

Also C.I. – S.I. = Rs 500 Now consider Statement II.


We know that We know that distance covered by a
n
r train crossing a platform
C.I. = P 1+ −1
U

100 = Length of train + Length of platform


Distance
Pnr Speed S =
@

and S.I. = Time


100
n
The train takes 20 seconds to cross the
r Pnr signal pole
⇒ P 1+ –1 – = 500
100 100 D
S=
2 20
r P ×2× r
⇒ P 1+ –1 – = 500 ⇒ D = 20S
100 100 i.e., length of train = 20 S metres
2 Also
r 2r Length of train + Length of platform
⇒ P 1+ – 1– = 500
100 100 = Speed × Time
r2 2r 2r ⇒ 20S + 150 = S × 30
⇒ 12500 1+ + –1– = 500 From above, we can easily find the length
(100)2 100 100 of the train.
r2 So, Statement II is also sufficient to
⇒ 12500 = 500
10000 answer the question.
646 ✫ Quantitative Aptitude

⇒ The data either in Statement I alone or in Given in Statement I,


Statement II alone are sufficient to Area of base = 616 cm2
answer the question. We know that
[Alternative Method Area of base = πr2
Given, Time taken T by the train to cross ⇒ πr 2 = 616 cm2
a signal pole = 20 seconds 616
Consider Statement I ⇒ r2 =
π
Speed of train S = 54 kmph 616 22
54×1000 ⇒ r2 = π =
22 7

TS
= mps
3600 7
S = 15 mps
616
We know that length of a train crossing a ⇒ r2 = × 7 = 28 × 7 = 196
pole = Distance covered by the train 22

H
And, Distance = Speed × Time ⇒ r = 196 = 14 cm
∴ Length of train = 15 metre per second But value of h is not given.

G
× 20 seconds So depth of cylindrical pipe cannot be
= 300 metres determined from Statement I.
∴ We can determine the answer from the
data in statement I.
Now consider Statement II U Now consider Statement II
Perimeter of base = 88 cm
O
We know that
Let Speed of the train be S metres when
Perimeter of base = 2πr
it crosses a signal pole in 30 seconds
⇒ 2πr = 88 cm
H
Length of train L = Speed × Time
88×7
= S × 20 ⇒ r =
2×22
_T

= 20 S metres
The train crosses the platform in 30 ⇒ r = 14 cm
seconds (given) Again, here also value of h is not given
We know that when a train crosses a and so depth of pipe cannot be
C

platform, determined from Statement II.


Distance covered = Length of train (L) ∴ The data even in both the Statements I and
PS

+ Length of platform II are not sufficient to answer the


⇒ L + Length of platform = Speed × Time question.
⇒ 20 S + 150 = S × 30 [Alternative Method
⇒ 20 S + 150 = 30 S We know that
U

⇒ 30 S – 20 S = 150 Depth of cylindrical pipe


⇒ 10 S = 150 = Volume of cylindrical pipe πr2h
@

150 Consider Statement I


⇒ S = = 15 m/sec.
10 Area of base = 616 cm2
⇒ Length of train = 20 × 15 = 300 metres Area of base = πr 2
∴ The data either in Statement I alone or in ⇒ πr2 = 616
Statement II alone are sufficient to Consider Statement II
answer the question.] Perimeter of base = 2πr = 88 cm
8. Ans. (d) From this, value of r can be found out but
Sol. Consider Statement I value of h is not given in any of the
We know that statements.
Depth or volume of cylindrical pipe ∴ Data even in both the statements are not
V = πr2h sufficient to answer the question.]
Where r is the radius of the base and 9. Ans. (b)
h is the height of the cylinder Sol. Total no. of children = 48
Unit Nine : Practice Session ✫ 647

Consider Statement I ⇒ Statement II alone is sufficient to answer


No. of columns is more than no. of rows the question.]
(given)
10. Ans. (e)
But it is not given by how much or by
what quantity the no. of columns is more Sol. Let ABC be the triangle and CD the
than the no. of rows. attitude on base AB.
So, Statement I is not sufficient Consider Statement I
Now consider Statement II AB is the base and sum of sides = 25 cm
3 ⇒ AB + BC + CA = 25 cm
Given that no. of rows. is th of the no.

TS
4 Consider Statement II
of columns.
Ratio of AB, BC and CA
Let no. of columns be x
=2:2:1
3
∴ No. of row = x Let x be the common variable

H
4 ⇒ Length of AB = 2x
We know that
Length of BC = 2x

G
Total no. of children
= No. of rows × No. of columns Length of CA = x
⇒ 2x + 2x + x = 25 cm
⇒ 48 =
3 2
3
4
x×x
U ⇒

5x = 25
x =
25
= 5 cm
O
⇒ x = 48 5
4
48× 4 ∴ Length of AB = 2 × 5 = 10 cm
⇒ x2 =
H
Length of BC = 2 × 5 = 10 cm
3
⇒ x2 = 64 Length of CA =1 × 5 = 5 cm
We know that
_T

x = 64
Area of a triangle
⇒ x =8
∴ No. of columns = 8 = S(S – a)(S – b) (S – c)
C

3 a+b+c
No. of rows = ×8 = 6 Where S = and a, b, c are the sides
4 2
PS

We know that of the triangle


No. of children in each row ∴ Area = S(S – 10)(S – 10) (S – 5)
= no. of columns
10 +10+ 5 25
U

∴ No. of children in each row = 8 S= = = 12.5 cm


2 2
∴ The data given in Statement II alone is ∴ Area =
@

sufficient to answer the question.


12.5 (12.5 –10) (12.5 − 10) (12.5 – 5)
[Alternative Method
⇒ Area = 12.5×2.5×2.5×7.5
Consider Statement I
Given no. of columns > no. of rows ⇒ Area = 585.9375
But no exact value or relationship is ⇒ Area = 24.2 cm2
given. Also, area of a triangle
So, Statement I alone is not sufficient.
1
Consider Statement II =
× base × height
2
3 2×Area
No. of rows =
4
× No. of columns ∴ Height of triangle =
Base
Here a relation between rows and 2×24.2
columns is given, so the required answer ∴ Height =
10
can be determined. = 4.84 cm
648 ✫ Quantitative Aptitude

∴ Both statements I and II together are ∴ All (A), (B) and (C) together are necessary
necessary to answer the question. to answer the question.
11. Ans. (d) [Alternative Method
Sol. Consider Statement (A) We know that
S.P. − C.P.
Given that 5% discount is given on List Profit per cent = × 100
C.P.
price, L.P. As per Statement C,
⇒ Selling Price S.P. = L.P. – 5% L.P. C.P. = Rs 5000
100 L.P. – 5 L.P. We can see that Statement A gives us the
⇒ S.P. =

TS
100 List price, L.P. and selling price, S.P. can
95 L.P. be found from both Statements (A) and
⇒ S.P. =
100 (B)
⇒ S.P. = 95% L.P.

H
∴ All three statements together are
Now consider Statement (B) necessary for answering the question.]
Given that if discount is not given, 20%

G
profit is gained. 12. Ans. (e)
We know that Sol. Let x years be the present age of father


S.P. = C.P. + Profit
S.P. = C.P. + 20% C.P. U and let y years be the present age of
son.
O
Consider Statement I
(  Profit per cent is always calculated
on cost price) x – y = 24 ... (i)
Consider Statement II
H
20
⇒ S.P. = C.P. + C.P. 1
100 Given that y = x ... (ii)
5
_T

100 C.P. + 20 C.P.


⇒ S.P. = Consider Statement III
100
6 years from now,
120 C.P.
⇒ S.P. = Father’s age = (x + 6) years
C

100
It is given in statement (C) that Son’s age = (y + 6) years
C.P. = Rs 5000 Given that
PS

120 1
⇒ S.P. = × 5000 (y + 6) = (x + 6) ... (iii)
100 3
⇒ S.P. = Rs 6000 Solving equations (i) and (ii),
U

When no discount is given. we get


We know that when no discount is given 1
x– x = 24
@

S.P. = L.P. = Rs 6000 5


If L.P. = Rs 6000 then after 5% discount 5x – x
= 24
S.P. = 95% L.P. 5
= 95% × Rs 6000 4x
= 24
S.P. = Rs 5700 5
⇒ S.P. = Rs 5700, C.P. = Rs 5000 4x = 24 × 5
4x = 120
S.P. – C.P.
∴ Profit per cent = × 100 120
C.P. x= = 30
5700 – 5000 4
=
5000
× 100 ∴ Father’s age = 30 years
700 From (iii), we get
= × 100 1 1
5000 y+6= x+ ×6
= 14% 3 3
Unit Nine : Practice Session ✫ 649

1 Consider Statement (B)


⇒ y+6= x+2 1
3
1 y= x ... (ii)
5
⇒ y– x+6–2=0 Consider Statement (C)
3
1 6 years from now,
⇒ y– x+4= 0 ... (iv)
3 1 Father’s age =(x + 6) years
Putting the value of y = x in the above Son’s age =(y + 6) years
5
equation (iv), we get Given that
1 1 1

TS
x – x + 4 =0 (y + 6) = (x + 6) ... (iii)
5 3 3
3x – 5x Now, we have to find two unknown
⇒ +4= 0
15 variables x and y. For this at least two

H
–2 x linear equations containing these two
⇒ = –4 variables are necessary. Here, from
15
Statements (A), (B) and (C) we have three

G
2x
⇒ = 4 linear equations.
15
⇒ 2x = 4 × 15 So the question can be answered from

⇒ x=
4 × 15
2
= 2 × 15
U any two of the statements.]
13. Ans. (a)
O
x = 30
Sol. Consider Statement (A)
∴ Father’s age = 30 years
Given that length l of the rectangular plot
H
From equation (i),
is twice its breadth b
y = x – 24
⇒ l = 2b
_T

Putting the value of y = x – 24 in equation


Consider Statement (B)
(iv), we get
Area of the plot = 288 sq. meters.
1 Consider Statement (C)
x – 24 – x+4= 0
C

3 Cost of fencing is Rs 4 per sq. metre


1 We know that
⇒ x – x – 20 = 0
PS

3 Cost of fencing a rectangular plot


3x – x = Area of the plot × Cost of fencing per
⇒ – 20 = 0
3 square metre
2x
⇒ = 20 ∴ From Statements (A) and (B)
U

3
20×3 Cost of fencing the plot
⇒ x= = 288 sq m × Rs 4 per sq m.
@

2
⇒ x = 30 = Rs (288 × 4)
∴ Father’s age = 30 years = Rs 1152
∴ We can see that the father ’s present age ⇒ Only Statements B and C are necessary
can be ascertained from any of the two to answer the question.
given statements. [Alternative Method
[Alternative Method We know that
Let x years be the present age of father Cost of fencing a rectangular plot
and let y years be the present age of son. = Area of the plot × Cost of fencing per
Consider Statement (A) metre square
Given that difference in the age of father We can see that Statements (B) and (C)
and son is 24 years provide both the required information.
⇒ Statement (B) and (C) only are necessary.]
⇒ x – y = 24 ... (i)
650 ✫ Quantitative Aptitude

14. Ans. (c) ∴ We can determine the length of the train


Sol. Statements (A) and (B) give the time from Statements C and A or B.]
taken by the train to cross a pole.
15. Ans. (b)
Statement (C) gives the speed of the train
Sol. Principal P = Rs 5000
Speed = 60 kmph
We know that No. of years n = 2 years
Distance = Speed × Time Consider Statement (A)
We also know that when a train crosses Simple Interest S.I. = Rs 2000
a pole, No. of years n1 = 5 years

TS
Length of train = Distance covered Rate of interest = r
From Statements (A) and (C) We know that
Time = 18 seconds Pnr
18 ×1 S.I. =
100

H
= hours
3600 5000×5× r
[  1 hr = 3600 seconds] 2000 =

G
100
1 ⇒ 200000 = 25000r
= hrs
200
∴ Length of train = 60 ×
1
200 U ⇒
r = 8%
r=
200000
25000
O
= 0.3 km
= 0.3 × 1000 metres ∴ Compound Interest
n
H
= 300 metres r
= P 1 + 100 –1
[ 1 km = 1000 metres]
_T

Now from Statements (B) and (C) 2


8
Time = 30 seconds = 5000 1 + 100 –1
1
= 30 × hours
3600
= 5000 (1 + 0.08) – 1
C

[ 1 hr = 3600 seconds]
1
= 5000 (0.08) – 1
2
PS

= hrs
120
1 = 5000 [(a)1664 – 1]
∴ Length of train = 60 ×
120
= 5000 (0.1664)
= 0.5 km
U

= Rs 832
= 0.5 × 1000 metres
[  1 km = 1000] ∴ Compound Interest = Rs 832
@

= 500 metres Now consider Statement (B)


∴ We can see that Statements C and A or B C.I. = S.I. for 1 year
only are necessary to answer the r
n
Pnr
question. ⇒ P 1+ –1 =
100 100
[Alternative Method 1
We know that r 5000 × 1 × r
⇒ 5000 1 + –1 =
When a train crosses a signal pole, 100 100
Distance covered = Length of the train r r
⇒ 1+ –1=
∴ Length of Train = Distance covered 100 100
= Speed × Time r r
⇒ =
Statements A and B provide data 100 100
regarding time and statement C provides From the above data, rate of interest or
data on speed C.I. cannot be calculated.
Unit Nine : Practice Session ✫ 651

Now consider Statement (C). Where P is the Principal, n = no. of years


Given that Amount becomes more than and r is the rate of interest
double on C.I. in 10 years P ×3× r
Here the exact amount or any exact ⇒ 2250 =
100
information about the amount is not
given. It is said that the amount becomes ⇒ 3 Pr = 2250 × 100
more than double which is not sufficient ⇒ 3 Pr = 225000
data to find the C.I.
225000
⇒ Statement A only is necessary to answer ⇒ Pr =

TS
the question. 3
Pr = 75000 ... (i)
[Alternative Method
Here again we do not have the rate of
We know that interest.

H
n
r ∴ Principal cannot be calculated.
C.I. = P 1+ –1
100 Consider Statement III.

G
Here value of P and n is given but value Given that S.I. = C.I. for the first year
of r is not given. Let S.I. = C.I. = Rs x
Consider Statement (A).
Pnr U We know that
r
n
O
S.I. = C.I. = P 1+ –1
100 100
Here S.I., P, n are given
H
1
So r can be determined r
⇒ x= P 1+ –1
And so from this data we can find C.I. 100
_T

Consider Statement (B).


Given that C.I. = S.I. earned in one year Pnr
S.I. = x =
but this is true for C.I. and S.I. on any 100
amount if the rate is same.
C

P ×1× r
So Statement (B) is not sufficient. =
100
Consider Statement (C).
PS

Pr
Here no precise information is given. x=
So only Statement (A) is required to find 100
the answer.] 75000
⇒ x=
100
U

16. Ans (e)


Sol. Consider Statement I. [  From (i), Pr = 75000]

@

Difference between Compound Interest x = 750


C.I. and Simple Interest S.I. = Rs 37.50 for ⇒ S.I. for 1 year = 750
a period of 2 years
Given in statement I that C.I. – S.I. = Rs
⇒ C.I. – S.I. = 37.50
37.50 for a 2-year period. This is because
From this information, we cannot
the C.I. includes interest on the sum of the
calculate principal P. since neither the
first year ’s principal and its interest.
rate of interest is given nor the exact
amount of interest is given. ⇒ Rs 37.50 is the interest on Rs 750 for
Now consider Statement II. 1 year
Given that 100×S.I.
∴ Rate of Interest =
S.I. for 3 years = Rs 2250 P ×n
We know that 100×37.5
Pnr
=
750×1
S.I. =
100 = 5% p.a.
652 ✫ Quantitative Aptitude

From (i) ∴ Area cannot be ascertained only from


Pr = 75000 Statement I.
75000 Now consider Statement II.
P =
r Length and breadth are in the ratio 7 : 4
75000 ⇒ If length l = 7x then
P = breadth b = 4x
r
= 15000 ∴ Area = l × b = 7x × 4x = 28x2
Again Area cannot be found using
∴ All I, II and III are necessary to find the
Statement II alone.

TS
principal (P).
Now in Statement I, given that
[Alternative Method P = 220 metres
To find : Principal = P If l = 7x and b = 4x

H
Consider Statement I. P = 2 (7x + 4x)
We know that ⇒ 220 = 2 (7x + 4x)

G
n
r nr 220
C.I. – S.I. = P 1+ –1– ⇒ 7x + 4x =
100 100 2
Here value of C.I. – S.I. and value of n is
given. U ⇒


11x = 110
x = 10
O
Length = 7 × 10 = 70 metres
Consider Statement II
and Breadth = 4 × 10 = 40 metres
Pnr
∴ Area = 70 × 40
H
S.I. = = 2250
100
Here value of n is given so we can = 2800 m2
Now consider Statement III.
_T

calculate value of P × r
Consider Statement III. Given that length is less than twice the
breadth by 10 metres
Given C.I. = S.I. for 1 year
⇒ 2b – 10 = l ... (ii)
From this statement we can calculate
C

Substituting the value of l in (ii) in


value of r after substituting value of
equation (i), we get
P × r from Statement II and value of
PS

2b – 10 + b = 110
interest from Statement I
⇒ 3b – 10 = 110
∴ When we get the value of r, we can easily
⇒ 3b = 110 + 10
calculate value of P.

U

3b = 120
∴ All the three statements I, II, III are
⇒ b = 40 So, l = 110 – 40 = 70 [From (i)]
necessary to answer the question.]
∴ Area = 40 × 70 = 2800 m2
@

17. Ans (c) ⇒ Statement I and either II or III only are


Sol. Consider Statement I. necessary to answer the question
Let l be the length and b be the breadth [Alternative Method
of the garden To find: Area of rectangular garden
Perimeter = 220 metres (given) We know that
We know that Perimeter of a rectangle P Area of rectangular garden
= 2 (l + b) = length × breadth
⇒ 2 (l + b) = 220 =l×b
220 Consider Statement I
l+b= = 110
2 Perimeter of garden
∴ l + b = 110 ... (i) 2 (l + b) = 220
But, Area A = l × b l + b = 110
Unit Nine : Practice Session ✫ 653

∴ Consider Statement II. Work done by 1 man in 1 day from


Ratio of length and breadth is given 1
So from statements I and II we can easily Statement I = th of the work
40
calculate the value of length and breath ∴ Work done by 6 men in 1 day
and also the required area. 1
Consider Statement III. =6× of the work
40
Given that length is less than twice the Now work done by 6 men in 5 days
breadth by 10 metres
1
⇒ 2b – 10 = l =6× × 5 of the work
40

TS
Again, with the help of the data in
Statements I and III we can easily 6 3
= or of the work
calculate the length and breadth and 8 4
thereby the area can also be calculated. ⇒ When 6 men work for 5 days they can

H
∴ Statement I and either Statement II or III 3
only are necessary.] complete of the work
4
The remaining work to be completed

G
18. Ans (e)
Sol. Consider Statement I 3 1
=1– = of work
According to this statement ratio of
males and females = 4 : 5
But no other information is given. So U 4 4
This remaining work shall be done by 4
women in 5 days
O
population of State A cannot be ∴ Work done by 1 woman in 5 days
determined from Statement I alone. 1
= work
H
Consider Statement II. 4×4
Statement II gives information that 70% Now work done by 1 woman in 1 day
of population was literate. But no. of
_T

1
literate people is not given. = of the work
4×4×5
So here again population cannot be
determined. 1
= work
C

Consider Statement III. 80


This statement gives the no. of literate ∴ Work done by 10 women in 1 day
1 1
PS

males which is not sufficient to calculate = × 10 = work


the population of the state. 80 8 1
⇒ The question cannot be answered even In 1 day, 10 women can complete work
8
with the information in all the three ∴ To complete 1 work by 10 women, no. of
U

statements. days required = 8 days


∴ Statements I and II together are necessary
19. Ans (d)
@

to answer the question


Sol. Consider Statement I. Now consider Statement III
5 men can complete 1 work in 8 days. 1 man and 1 woman can do thrice the
Work completed by 5 men in work done by a woman in 1 day
1 ⇒ 1 man + 1 woman = 3 women
1 day = th of the work
8 ⇒ 1 man = (3 – 1) women
1 man can do the work in one day ⇒ 1 man = 2 women
1 1 Statement I says that 5 men can complete
= × work the work in 8 days
8 5
1 ⇒ 1 × 5 men = 2 × 5 women i.e. 10 women
= th of the work can complete the work in 8 days
40
Consider Statement II. ∴ Statements I and III together are
6 men and 4 women together can sufficient.
complete the work in 5 days. Now consider Statements II and III
654 ✫ Quantitative Aptitude

From III, Distance covered


1 man = 2 women Speed of the train =
Time taken
In Statement II, Also, when a train crosses a signal port
6 men and 4 women together complete Distance covered = Length of train
the work in 5 days
⇒ 6 men = 2 × 6 women When it crosses a platform,
= 12 women Distance covered = Length of train
⇒ (12 + 4) women = 16 women Complete + Length of platform
the work in 5 days 5×16 But the data in none of the statements

TS
∴ 10 women shall take days provides the value for the length of the
10
= 8 days train. So the question cannot be
∴ Statements II and III toether are sufficient answered even with the information in
all the three statements.]

H
to answer the question.
So we can seen that any two of the three 21. Ans (a)
statements are sufficient to answer the

G
Sol. If base and height are known then area
question.
1
20. Ans (e) = × base × height
Sol. Speed of the train = ?
Consider Statement I. U 2
Consider Statement I and II
Given base = x cm
O
The train crosses a signal pole in 14 and height = y cm
seconds. So, Area
H
Distance 1
Speed = = ×x×y
Time 2
_T

Here, Distance of Train = Length of train xy


= sq. cm.
(which is not given) 2
Time = 14 seconds. 22. Ans (a)
Sol. Total age of D and E is 14 years
C

Now consider Statement II.


The train crosses another train in 16 (Statement I)
Average age of A, B, C, and F is 50 years
PS

seconds
(Statement II)
Here, Time = 16 seconds
∴ Total age of A, B, C and F is 4 × 50 = 200
Distance = Length of train years
U

+ Length of the other train ∴ Average age of A, B, C, D, E and F is


Both the above information is not given. 200 + 14
= = 35.6
Now consider Statement III.
@

6
Length of platform = 200 metres 23. Ans (e)
Time = 24 seconds. Sol. Consider Statement I
Here, distance = Length of train
PTR
+ Length of platform we can find R from + P = 2P
100
Again length of train is not given. So 100 100
speed cannot be determined R= = = 10%
T 10
⇒ The question cannot be answered even Consider Statement II.
with the information in all the three C.I. – S.I. = Difference (D)
statements.
D = 150
[Alternative Method P = 15000
2
To find: Speed of the train R
We know that D=P
100
Unit Nine : Practice Session ✫ 655

2
R So he scored approximately 51.33 marks
150 = 15000 in English
100
Information in all the statements is re-
R2 quired to answer the question.
150 = 15000 ×
100 × 100
25. Ans (c)
150 × 100 × 100 1
R2 = In 1 day A and B can do of the work
15000 8
R2 = 100 1
In 1 day B and C can do of the work
R = 10% 10

TS
Consider Statement III 1
In 1 day A and C can do of the work
Data insufficient 12
∴ In 1 day 2 (A + B + C) can do
24. Ans (c)
1 1 1 37

H
Sol. If Abhijit scores x marks in Science, then + + =
he scores (x + 20) in Maths, as per the 8 10 12 120
∴ A, B and C can do the work in

G
information in Statement I, and (x – 12)
in English, according to the information
in Statement III. 120 240
´2 =
Statement II tells us that he scored a total
of 198 marks. U 37 37
days
A and C can do the work in 12 days
O
So, x + x + 20 + x – 12 = 198 æ 240 ö
3x = 198 – 8 ∴ B can do the work in çç - 12÷÷÷ days.
çè 37 ø
H
190 ⇒ We can find the answer only by using the
x = = 63.33
3 data in all the statements.
C _T
PS
U
@
656 ✫ Quantitative Aptitude

APPENDIX
Tables of Metric Units of Measurement
In the metric system of measurement, designations of multiples and sub-divisions of any unit may be arrived
at by combining with the name of the unit the prefixes deca, hecto, and kilo meaning, respectively, 10,
100, and 1000, and deci, centi, and milli, meaning, respectively, one-tenth, one-hundredth, and one-thousandth.
In some of the following metric tables, some such multiples and sub-divisions have not been included for
the reason that these have little, if any currency in actual usage.
In certain cases, particularly in scientific usage, it becomes convenient to provide for multiples larger
than 1000 and for sub-divisions smaller than one-thousandth. Accordingly, the following prefixes have been

TS
introduced and these are now generally recognized:
yotta, (Y), meaning 1024 deci, (d), meaning 10-1
zetta, (Z), meaning 1021 centi, (c), meaning 10-2
exa, (E), meaning 1018 milli, (m), meaning 10-3

H
peta, (P), meaning 1015 micro, (u), meaning 10-6
12
tera, (T), meaning 10 nano, (n), meaning 10-9

G
giga, (G), meaning 109 pico, (p), meaning 10-12
mega, (M), meaning 106 femto, (f), meaning 10-15
kilo, (k), meaning 103 atto, (a), meaning 10-18
hecto,
deka,
(h),
(da),
Units of Length
meaning 102
meaning 101
U zepto,
yocto,
(z),
(y),
meaning 10-21
meaning 10-24
O
10 millimetres (mm) = 1 centimetre (cm)
10 centimetres = 1 decimetre (dm) = 100 millimetres
H
10 decimetres = 1 metre (m) = 1000 millimetres
10 metres = 1 dekametre (dam)
10 dekametres = 1 hectometre (hm) = 100 metres
_T

10 hectometres = 1 kilometre (km) = 1000 metres


Units of Area
100 square millimetres (mm2) = 1 square centimetre (cm2)
100 square centimetres = 1 square decimetre (dm2)
C

100 square decimetres = 1 square metre (m2)


100 square metres = 1 square dekametre (dam2) = 1 are
100 square dekametres = 1 square hectometre (hm2) = 1 hectare (ha)
PS

100 square hectometres = 1 square kilometre (km2)


Units of Liquid Volume
10 millilitres (mL) = 1 centilitre (cL)
10 centilitres = 1 decilitre (dL) = 100 millilitres
U

10 decilitres = 1 litre1 = 1000 millilitres


10 litres = 1 dekalitre (daL)
10 dekalitres = 1 hectolitre (hL) = 100 litres
@

10 hectolitres = 1 kilolitre (kL) = 1000 litres


Units of Volume
1000 cubic millimetres (mm3) = 1 cubic centimetre (cm3)
1000 cubic centimetres = 1 cubic decimetre (dm3)
= 1 000 000 cubic millimetres
1000 cubic decimetres = 1 cubic metre (m3)
= 1 000 000 cubic centimetres
= 1 000 000 000 cubic millimetres
Units of Mass
10 milligrams (mg) = 1 centigram (cg)
10 centigrams = 1 decigram (dg) = 100 milligrams
10 decigrams = 1 gram (g) = 1000 milligrams
10 grams = 1 dekagram (dag)
10 dekagrams = 1 hectogram (hg) = 100 grams
10 hectograms = 1 kilogram (kg) = 1000 grams
1000 kilograms = 1 megagram (Mg) or 1 metric ton(t)
Learning Curve 2

TS
In this section, there are question papers in quantitative
aptitude (or numerical ability) set by various service
boards, such as the IBPS and other banking recruitment

H
establishments as well as the Insurance Corporations.

G
While most of the test papers included here (based on the

U
recall of various candidates) are those set for recruitment
of probationary/specialist officers in banks, some are from
O
examinations held for assistants and other posts such as
H
administrative officers in insurance companies. So
_T

attempting each one in ‘test mode’ will help the reader


in valuable self-assessment in terms of both knowledge and
C

time-management. A set of 50 questions ought to be


answered in 30 minutes. In other words, an effort should
PS

be made not to spend more than 30 seconds on a


question—on an average.
U
@

All test papers are fully solved, with fast-track methods


indicated wherever possible, so that readers may check
how they have fared—and give more attention to those
areas in which they need improvement.
658 ✫ Quantitative Aptitude

TS
H
G
U
O
H
C_T
PS
U
@
Solved Test Papers of Various Examinations ✫ 659

Solved Test Papers of


Various Examinations
Institute of Banking Personnel Selection (IBPS) Specialist
Officers Common Written Examination, March 2013

TS
1. A 320-metre-long train crosses a pole in 50 total number of candidates interviewed by
seconds. What is the speed of the train? all the firms on the same day?
(A) 4.6 m/sec (B) 7.2 m/sec (A) 24 (B) 23 (C) 38
(C) 5.4 m/sec (D) 6.4 m/sec (D) 29 (E) None of these

H
(E) Cannot be determined 5. In which firm the number of candidates
2. In a school some sweets were to be interviewed decreased consistently from

G
distributed among 420 children on the Monday to Saturday?
occasion of Teacher’s Day. But 140 children (A) B (B) None (C) D
remained absent on that particular day and
hence each child got one sweet extra. How
U (D) C (E) A
6. What is the number of candidates
O
many sweets would each child have got interviewed by all the firms on Monday?
originally? (A) 101 (B) 102 (C) 114
(A) Cannot be determined (D) 98 (E) 96
H
(B) 2 (C) 5
7. By what per cent the number of candidates
(D) 4 (E) 1 interviewed by firm E on Tuesday increased
_T

Directions (Qs. 3–7): In the following questions with respect to number of candidates
information about number of candidates interviewed interviewed on the preceding day?
by five commercial firms on different working days has (A) 45 (B) 26 (C) 61
C

been provided. You are required to read the table carefully (D) 56 (E) 39
and answer the questions given below: 8. In how many ways the letters of the word
PS

Number of candidates interviewed VISITING can be re-arranged?


by five firms on different working days (A) 6720 (B) 5720 (C) 720
(D) 7620 (E) None of these
Firms
Directions (Qs. 9–14): In the following pie-chart per
U

Working Day A B C D E
cent of students enrolled in different cultural activities
Monday 17 18 23 25 18 of a school has been shown. You are required to study
@

Tuesday 21 19 14 28 25
the pie-chart carefully and answer the questions given
Wednesday 23 22 23 12 18
Thursday 24 14 12 23 18 below:
Friday 10 10 16 15 22 Number of students = 1800
Saturday 17 26 20 20 24
Karate
3. What is the respective ratio between the 10%
7% ket

number of candidates interviewed by firm


ic
Cr

D on Friday and Saturday together and


Dance
number of candidates interviewed by firm 38%
B on the same days? Painting
(A) 35 : 38 (B) 39 : 40 (C) 43 : 44 32%
5%
8%

(D) 45 : 46 (E) 35 : 36
ts
or

Music

4. The number of candidates interviewed by


Sp

firm C on Wednesday is what per cent of


660 ✫ Quantitative Aptitude

9. What is the number of students who opted 19. ? % of 800 = 293 – 22% of 750
for dancing? (A) 14 (B) 18 (C) 12
(A) 648 (B) 684 (C) 864 (D) 16 (E) 20
(D) 664 (E) None of these Directions (Qs. 20-24): In the following table, the
10. What is the respective ratio between the number of employees working in five companies and the
corresponding ratio of male and female employees have
number of students who opted for painting
been given. You are required to study the table carefully
and those who opted for Karate? and answer the questions:
(A) 3 : 7 (B) 5 : 16 (C) 16 : 5
Company Number of Employees
(D) 5 : 7 (E) None of these

TS
L 400
11. By what per cent is the number of candidates M 600
who opted for music more than those who N 800
opted for sports? O 1000

H
(A) 30 (B) 55 (C) 45 P 1200
(D) 60 (E) None of these Company Male : Female

G
12. How many students have opted for cricket L 12 : 8
and painting together? M 15 : 5
(A) 702
(D) 645
(B) 602 (C) 660
(E) None of these U N
O
4
13
:
:
1
12
O
P 7 : 5
13. By what per cent is the number of students
who opted for cricket and dancing together 20. What is the respective ratio between the
H
number of females in company P and
more than those who opted for painting and
number of females in company L?
music together?
_T

(A) 8 : 25 (B) 25 : 8
(A) 14.5 (B) 12.5 (C) 16.5 (C) 3 : 16 (D) 16 : 3
(D) 21.5 (E) None of these (E) None of these
14. What is the respective ratio between the
C

21. The number of female employees working


number of students who opted for dancing in company O is what per cent of total
and karate to those who opted for painting? employees working in that company?
PS

(A) 2 : 3 (B) 1 : 3 (C) 3 : 1 (A) 24 (B) 12


(D) 3 : 2 (E) None of these (C) 48 (D) 13
(E) None of these
U

Directions (Qs. 15-19): What value should come in


22. What is the average number of employees
place of the question mark (?) in the following questions?
in all companies together?
@

15. 18.5 × 21.4 × ? = 6255.22 (A) 800 (B) 775


(A) 15.8 (B) 14.6 (C) 17.4 (C) 760 (D) 600
(D) 17.2 (E) 16.4 (E) None of these
16. 1.5 × 78 ÷ 0.5 = ? 23. The number of female employees in
(A) 238 (B) 234 (C) 243 company M is
(D) 216 (E) 261 (A) 450 (B) 150
(C) 250 (D) 350
17. 302.46 + 395.72 – 123.47 = ? (E) None of these
(A) 576.77 (B) 547.17 (C) 547.77
24. The total number of male employees
(D) 574.71 (E) 577.71 working in companies N and P together is
18. 3 4096 ÷ 3 64 = 3 ? (A) 1240 (B) 1360
(A) 16 (B) 8 (C) 64 (C) 1340 (D) 1260
(D) 4 (E) 4 (E) None of these
Solved Test Papers of Various Examinations ✫ 661

Directions (Qs. 25-29): What approximate value


(A) 7 : 6 (B) 1 : 2
should come in the following questions at the places
(C) 1 : 3 (D) 3 : 1
indicated by the question mark. (You are not required to
(E) None of these
calculate the exact value)
25. (13.001)3 = ? 32. Which company sold highest percentage of
(A) 1900 (B) 2200 units as compared to units manufactured by
(C) 2000 (D) 1800 it?
(E) 2100 (A) Q (B) R
(C) P (D) T
26. 55.003 × 54.998 + 5.001 = ?
(E) S

TS
(A) 3500 (B) 3630
(C) 2540 (D) 3030 33. What is the average number of units (in
(E) 2750 thousand) sold by all the companies
together?

H
27. 50.001% of 99.99 ÷ 49.999 = ?
(A) 1 (B) 0.1 (A) 25.7 (B) 30
(C) 0.01 (D) 0.02 (C) 27.7 (D) 28.8

G
(E) None of these (E) None of these
28. 999.0001 + 899.999 – 349.88 = ? 34. What is the respective ratio between the
(A) 1549
(C) 1449
(B) 1560
(D) 1460 U units manufactured by company P and
company S?
O
(E) None of these (A) 7 : 8 (B) 8 : 7
29. (2.0001)3 × (1.999)–2 ÷ (3.999)–4 = ? (C) 3 : 5 (D) 5 : 3
H
(A) 32 (B) 16 (E) None of these
(C) 64 (D) 256 35. From a well-shuffled pack of 52 playing
_T

(E) 512 cards, one card is drawn at random. What


Directions (Qs. 30-34): In the following table the is the probability that the card drawn will
number of units manufactured and sold by five be a black king?
C

companies, P, Q, R, S and T has been given. Study the 1 7


given data carefully and answer the questions. (A) (B)
26 13
PS

Company Number of Number of 3 9


(C) (D)
units units 13 13
manufactured sold 1
(E)
U

(In thousand) (In thousand) 13


P 35 20 36. A horse is tethered to a peg with a 14-metre-
@

Q 45 30 long rope at the corner of a 40-metre-long


R 35 25 and 24-metre-wide rectangular grassfield.
S 40 35 What area of the field will the horse graze?
T 50 40
(A) 154 m2 (B) 308 m2
2
30. What is the average number (in thousand) (C) 240 m (D) 480 m2
of units manufactured by all the companies (E) None of these
together? 37. Farah was married 8 years ago. Today her
(A) 41 (B) 40 2
(C) 43 (D) 44 age is 1 times to that at the time of her
7
(E) 46 marriage. At present her daughter’s age is
31. What is the respective ratio between the 1
th of her age. What was her daughter’s age
number of units sold by company S and that 6
sold by company Q? 3 years ago?
662 ✫ Quantitative Aptitude

(A) 6 years Directions (Qs. 44-48): What will Come at the place
(B) 7 years of the question mark (?) in the following number series?
(C) 3 years 44. 3 1731 ? 3243 3459 3523
(D) Cannot be determined (A) 2371 (B) 2731
(E) None of these (C) 3731 (D) 3127
38. Manish bought 25 kg of rice at Rs 32 per kg (E) None of these
and 15 kg of rice at Rs 36 per kg. What profit 45. 10 18 34 ? 130 258
did he get when he mixed the two varieties (A) 32 (B) 60
together and sold the rice at Rs 40.20 per (C) 66 (D) 68

TS
kg? (E) None of these
(A) 25% (B) 40%
46. 4 10 33 136 ? 4116
(C) 30% (D) 20% (A) 822 (B) 534
(E) None of these

H
(C) 685 (D) 745
Directions (Qs. 39-43): In each of the following (E) 548
questions two equations are given. Solve these equations

G
47. 4000 800 ? 80 40 8
and give the answer: (A) 140 (B) 100
(A) if x ≥ y, i.e., x is greater than or equal (C) 400 (D) 160
to y
(B) if x > y, i.e., x is greater than y U (E) 200
48. 3 4 16 75 364 ?
O
(C) if x ≤ y, i.e., x is less than or equal
(A) 783 (B) 828
to y
(C) 1293 (D) 1945
H
(D) if x < y, i.e., x is less than y
(E) None of these
(E) x = y or no relation can be established
between x and y 49. The area of a circle is seven times the
_T

numerical value of its circumference. What


39. I. x2 + 5x + 6 = 0 is the circumference of the circle?
II. y2 + 7y + 12 = 0 (A) 616 units
C

40. I. x2 + 20 = 9x (B) 132 units


II. y1 + 42 = 13y (C) 88 units
(D) Cannot be determined
PS

41. I. 2x + 3y = 14
(E) None of these
II. 4x + 2y = 16
50. The compound interest earned on a sum in
42. I. x = 625 3 years at 15% per annum compounded
U

II. y = annually is ` 6500.52. What is the sum?


676
(A) ` 12480 (B) ` 10500
@

2
43. I. x + 4x + 4 = 0 (C) ` 14800 (D) ` 13620
II. y2 – 8y + 16 = 0. (E) None of these
Answers and Explanatory Notes
1. Ans. (D) 2. Ans. (B)
Sol. Length of train = 320 m, Sol. Let the number of sweets be x.
Time = 50 seconds Total number of children = 420
Then, Number of children absent
Length of train = 420 – 140 = 280
Speed of train =
Time taken Then,
320 x x
= – = 1
50 280 420
= 6.4 m /sec. 3x − 2x
Hence, the speed of the train is 6.4 m = 1
840 x = 840
/sec.
Solved Test Papers of Various Examinations ✫ 663

∴ Number of sweets = 840 8. Ans. (A)


Now, Sol. The given word consists of 8 letters out
Number of sweets each child of which I occurs three times.
840 8!
= = 2. Number of arrangements =
420 3!
3. Ans. (E) 8 × 7 × 6 × 5× 4 × 3 !
=
3!
Sol. Number of candidates interviewed by =8×7×6×5×4
firm D on Friday and Saturday together
= 6720.
= 15 + 20

TS
= 35 9. Ans. (B)
Number of candidates interviewed by Sol. Number of students = 1800
firm B on Friday and Saturday together Per cent of students enrolled in dancing
= 38%

H
= 10 + 26 = 36
∴ Required ratio between firm D and Number of students who opted for
firm B dancing

G
= 35 : 36. 38
= 1800 ×
4. Ans. (B) 100
Sol. Number of candidates interviewed by
firm C on Wednesday = 23
U 10. Ans. (C)
= 684.
O
Total number of candidates inter- Sol. Number of students = 1800
viewed by all the firms on Wednesday Per cent of students enrolled in painting
= 23 + 22 + 23 + 12 + 18
H
= 32%
= 98 Per cent of students enrolled in karate
Required percentage = 10%
_T

23 Then,
= × 100
98 Number of students who opted for
= 23.46% painting
C

= 23% (approximate). 32
= 1800 ×
5. Ans. (B) 100
Sol. It is obvious from the table.
PS

= 576
6. Ans. (A) Number of students who opted for
Sol. Number of candidates interviewed by karate
all the firms on Monday 10
U

= 17 + 18 + 23 + 25 + 18 = 1800 ×
100
= 101. = 180
@

7. Ans. (E) ∴ Required ratio = 576 : 180


Sol. Number of candidates interviewed by = 16 : 5.
firm E on Tuesday
= 25 11. Ans. (D)
Number of candidates interviewed by Sol. Number of students = 1800
firm E on Monday (Preceding day) Per cent of students enrolled in music
= 18 = 8%
∴ Percentage increase Per cent of students enrolled in sports
25 − 18 = 5%
= × 100 Number of students who opted for
18
7 music
= × 100 8
18 = 1800 ×
= 38.88% 100
= 39% (approximate). = 144
664 ✫ Quantitative Aptitude

Number of students who opted for ∴ Required percentage


sports 5 810 − 720
= 1800 × = × 100
100 720
= 90 90
∴ Required percentage = × 100 = 12.5%.
720
144 − 90 14. Ans. (D)
= × 100
90
54 Sol. Number of students = 1800
= × 100 = 60%. Number of students who opted for
90
dancing and karate together

TS
12. Ans. (A)
38 10
Sol. Number of students = 1800 = 1800 × + 1800 ×
100 100
Per cent of students enrolled in cricket = 684 + 180

H
= 7%
= 864
Per cent of students enrolled in painting
= 32% Number of students who opted for

G
Number of students who opted for painting 32
cricket = 1800 ×
7 100
= 1800 ×
= 126 100
U = 576
∴ Required ratio = 864 : 576
= 3 : 2.
O
Number of students who opted for
painting 15. Ans. (A)
32 Sol. 18.5 × 21.4 × ? = 6255.22
= 1800 ×
H
= 576 100 395.90 × ? = 6255.22
∴ Number of students who opted for 6255.22
_T

? =
cricket and painting together 395.90
= 126 + 576 ∴ ? = 15.8.
= 702. 16. Ans. (B)
C

13. Ans. (B) Sol. ? = 1.5 × 78 ÷ 0.5


Sol. Number of students = 1800 1.5 × 78
PS

?=
Per cent of students enrolled in cricket 0.5
= 7% ∴ ? = 3 × 78 = 234.
Per cent of students enrolled in dancing
17. Ans. (D)
= 38%
U

Sol. ? = 302.46 + 395.72 – 123.47


Per cent of students enrolled in painting
? = 698.18 – 123.47
= 32%
@

? = 574.71.
Per cent of students enrolled in music
= 8% 18. Ans. (C)
Number of students who opted for Sol. 3
?= 3
4096 ÷ 3
64
cricket and dancing together
7 38 = 16 × 16 × 16 ÷
3 3 4×4× 4
= 1800 × + 1800 × = 16 ÷ 4
100 100
= 126 + 684 = 810
3
? =1 4
3× 3
Number of students who opted for (?) 3 = (4)
painting and music together ∴ ? = 4 × 4 × 4 = 64.
32 8 19. Ans. (D)
= 1800 × + 1800 ×
100 100 Sol. ?% of 800 = 293 – 22% of 750
= 576 + 144 ? 22
800 × = 293 – 750 ×
= 720 100 100
Solved Test Papers of Various Examinations ✫ 665

8 × ? = 293 – 165 Total number of male employees in


8 × ? = 128 companies N and P together
128 4 7
∴ ?= = 16. = 800 × + 1200 ×
8 4 +1 7+5
20. Ans. (B) 4 7
Sol. Number of females in company P = 800 × + 1200 ×
5 12
5 = 640 + 700 = 1340.
= 1200 ×
7+5 25. Ans. (B)
5
= 1200 × = 500 Sol. ? = (13.001)3

TS
12 ? ≈ (13)3
Number of females in company L
? ≈ 2197
8
= 400 × ∴ ? ≈ 2200.
12 + 8

H
8 26. Ans. (D)
= 400 × = 160 Sol. ? = 55.003 × 54.998 + 5.001
20
? ≈ 55 × 55 + 5

G
∴ Required ratio = 500 : 160
? ≈ 3025 + 5
= 25 : 8.
∴ ? ≈ 3030.
21. Ans. (C)
Sol. Number of female employees working U 27. Ans. (A)
Sol. ? = 50.001% of 99.99 ÷ 49.999
O
in company O
12 ? ≈ 50% of 100 ÷ 50
= 1000 × 50
13 + 12 ? ≈ 100 ×
H
÷ 50
12 100
= 1000 × ? ≈ 50 ÷ 50
25
_T

= 480 ∴ ? ≈ 1.
480 × 100
∴ Required percentage = 28. Ans. (A)
1000 Sol. ? = 999.001 + 899.999 – 349.88
= 48.
C

? ≈ 999 + 900 – 350


22. Ans. (A) ? ≈ 1899 – 350
Sol. Average number of employees in all ∴ ? ≈ 1549.
PS

companies together
4000 29. Ans. (E)
= Sol. ? = (2.0001)3 × (1.999)–2 ÷ (3.999)–4
5 ? ≈ (2)3 × (2)–2 ÷ (4)–4
U

= 800.
? ≈ (2)3–2 ÷ (4)–4
23. Ans. (B) ? ≈ 2 ÷ (4)–4
@

Sol. Number of employees in company M 2


= 600 ∴ ?≈ ≈ 2 × (4)4 ≈ 2 × 256
Number of female employees in (4)−4
company M ? ≈ 512 .
5 30. Ans. (A)
= 600 × Sol. Average number of units manufactured
15 + 5
5 by all the companies together
= 600 × 35 + 45 + 35 + 40 + 50
20 = thousands
= 150. 5
205
24. Ans. (C) = = 41 thousands.
Sol. Number of employees in company 5
N = 800 31. Ans. (A)
Number of employees in company Sol. Number of units sold by company
P = 1200 S = 35 thousands
666 ✫ Quantitative Aptitude

Number of units sold by company n(E)


Q = 30 thousands ∴ Required probability =
n(S)
∴ Required ratio = 35 : 30 2 1
= 7 : 6. = = .
52 26
32. Ans. (E) 36. Ans. (A)
Sol. Percentage sale: Sol. As the horse is pegged at the corner of
Company P = a rectangular field, it will graze the
Numberof units sold area of the field enclosed between two
by company P sides of the rectangle.

TS
× 100
Numberof units 40 m C
D
manufactured by company P
20
= × 100 = 57.14

H
24 m
35
30
× 100 = 66.66

G
Company Q =
45
A 14 m B
25 1 2
× 100 = 71.42
Company R =
35
35
× 100 = 87.5
U Area of the field = πR
Here R = 14 m
4
O
Company S = 1 22
40 = × × 14 × 14
40 4 7
H
Company T = × 100 = 80 = 154 sq. m.
50
∴ Highest percentage of units sold as 37. Ans. (A)
_T

compared to units manufactured by Sol. Let Farah’s age 8 years ago be x years.
Company S. Then, Farah’s present age = (x + 8)
years.
33. Ans. (B) 2
∴ x+8=1 x
C

Sol. Average number of units sold by all 7


the companies together 9
x+8= x
PS

20 + 30 + 25 + 35 + 40 7
= thousands ⇒ 7x + 56 = 9x
5
150 ⇒ 2x = 56
= = 30 thousands. x = 28
U

5
34. Ans. (A) ∴ Farah’s present age = 28 + 8 = 36 years
Sol. Number of units manufactured by
@

Her daughter’s age 3 years ago


company P = 35 thousands
Number of units manufactured by 1
= 36 × = 6 years.
company S = 40 thousands 6
The respective ratio between the units 38. Ans. (D)
manufactured by company P and Sol. Cost price of 25 kg of rice at Rs 32
company S per kg
= 35 : 40 = 25 × 32 = Rs 800
= 7 : 8. Cost price of 15 kg of rice at Rs 36
35. Ans. (A) per kg
Sol. Total possible outcomes n(s) = 15 × 36 = Rs 540
= 52C1 Cost price of (25 kg + 15 kg) 40 kg of
= 52 mixture
Favourable outcomes black king n(E) = Rs 800 + Rs 540
=2 = Rs 1340
Solved Test Papers of Various Examinations ✫ 667

Selling price of 40 kg of mixture Putting the value of y = 3 in equation (I),


= Rs (40 × 40.20) we get
= Rs 1608
2x + 3(3) = 14
Profit = Rs (1608 – 1340)
2x + 9 = 14
= Rs 268
2x = 5
268 5
Profit% = × 100 x=
1340 2
∴ x<y
= 20%.
i.e. x is less than y.
39. Ans. (A)
42. Ans. (E)

TS
Sol. I. x2 + 5x + 6 = 0
⇒ x2 + 2x + 3x + 6 = 0 Sol. I. x= 625
⇒ x(x + 2) + 3 (x + 2) = 0 II. y = 676
⇒ (x + 2) (x + 3) = 0

H
From equation I,
x = – 2 or – 3
II. y2 + 7y + 12 = 0 x = 625

G
⇒ y2 + 3y + 4y + 12 = 0 = ± 25
⇒ y(y + 3) + 4 (y + 3) = 0 From equation II,


(y + 3) (y + 4) = 0
y = – 3 or – 4
x≥y U y = 676
y = ± 26
O
∴ No relation can be established between
i.e, x is greater than or equal to y.
x and y.
H
40. Ans. (D)
43. Ans. (D)
Sol. I. x2 + 20 = 9x
Sol. I. x2 + 4x + 4 = 0
⇒ x2 – 9x + 20 = 0
_T

x2 + 2x + 2x + 4 = 0
⇒ x2 – 4x – 5x + 20 = 0
x(x + 2) + 2(x + 2) = 0
⇒ x(x – 4) – 5 (x – 4) = 0
(x + 2) (x + 2) = 0
⇒ (x – 4) (x – 5) = 0

C

(x + 2)2 = 0
x = 4 or 5
⇒ x=–2
II. y2 + 42 = 13y
II. y2 – 8y + 16 = 0
PS

⇒ y2 – 13y + 42 = 0
⇒ y2 – 4y – 4y + 16 = 0
⇒ y2 – 6y – 7y + 42 = 0
⇒ y(y – 6) – 7 (y – 6) = 0 ⇒ y(y – 4) – 4 (y – 4) = 0
⇒ (y – 4) (y – 4) = 0
⇒ (y – 6) (y – 7) = 0
U

y = 6 or 7 (y – 4)2 = 0
∴ x<y y=4
@

i.e., x is less than y. ∴ x<y


i.e., x is less than y.
41. Ans. (D)
Sol. I. 2x + 3y = 14 44. Ans. (B)
II. 4x + 2y = 16 Sol. 3 1731 ? 3243 3459 3523
Multiply by 2 in equation (i) The given sequence follows the pattern:
4x + 6y = 28 ... III 3523 – 3459 = 64 = (4)3
Subtracting equation (II) from 3459 – 3243 = 216 = (6)3
equation III 3243 – ? = 512 = (8)3
4x + 6y = 28 ? = 3243 – 512
4x + 2y = 16 ? = 2731
4y = 12 2731 – 1731 = 1000 = (10)3
y= 3 1731 – 3 = 1728 = (12)3.
668 ✫ Quantitative Aptitude

45. Ans. (C) 75 × 4 + 43 = 300 + 64 = 364


Sol. 10 18 34 ? 130 258 ? = 364 × 5 + 53
The given sequence follows the pattern: = 1820 + 125
+ 8, + 16, + 32, + 64, + 128 = 1945.
Then, 49. Ans. (C)
10 + 8 = 18, 18 + 16 = 34, Sol. Area of circle = 7 × circumference of
? = 34 + 32 = 66, the circle
66 + 64 = 130, 130 + 128 = 258. πr2 = 7 × 2πr
r = 14
46. Ans. (C) ∴ Circumference of the circle

TS
Sol. 4 10 33 136 ? 4116 = 2πr
The given sequence follows the pattern 22
=2× × 14
: × 2 + 2, × 3 + 3, × 4 + 4, × 5 + 5, × 6 + 6 7

H
Then, = 88 units.
4 × 2 + 2 = 8 + 2 = 10, 50. Ans. (A)

G
10 × 3 + 3 = 30 + 3 = 33, Sol. Let the principal be Rs x,
33 × 4 + 4 = 132 + 4 = 136, r = 15%, t = 3 years, C.I. = Rs 6500.52
? = 136 × 5 + 5 = 680 + 5 = 685,
685 × 6 + 6 = 4110 + 6 = 4116.
U C.I. = P 1 +
r
100
t
–P
O
3
47. Ans. (C) 15
6500.52 = x 1 + –x
Sol. 4000 800 ? 80 40 8 100
H
The given sequence follows the pattern: 650052
3
115
÷ 5, ÷ 2, ÷ 5, ÷ 2, ÷ 5 =x –x
100 100
_T

Then, 3
4000 ÷ 5 = 800, 650052 23
=x –x
? = 800 ÷ 2 = 400, 100 20
400 ÷ 5 = 80, 80 ÷ 2 = 40, 650052 12167
C

=x –x
40 ÷ 5 = 8. 100 8000
PS

48. Ans. (D) 650052 12167 x − 8000 x


=
Sol. 3 4 16 75 364 ? 100 8000
The given sequence follows the pattern: 650052 4167 x
× 1 + 13, × 2 + 23, × 3 + 33, =
U

100 8000
× 4 + 43, × 5 + 53 650052 × 8000
Then, x=
@

4167 × 100
3 × 1 + 13 = 3 + 1 = 4,
4 × 2 + 23 = 8 + 8 = 16, x = 156 × 80
x = Rs 12480
16 × 3 + 33 = 48 + 27 = 75,
Hence, the principal is Rs 12480.
Solved Test Papers of Various Examinations ✫ 669

Institute of Banking Personnel Selection (IBPS)


Probationary Officers Examination, October 2013
Directions (Qs. 1–5): What value will come in place of (1) 450 (2) 460
the question mark(?) in the following questions? (You (3) 475 (4) 375
are not expected to calculate the exact value) (5) None of these
1. 21 + 3.9 × 2.9 + 8.99 = ? 7. What is the ratio of selected employees for
(1) 42 (2) 46 the post of Assistant IT Managers by
(3) 44 (4) 34

TS
Companies A, B and C respectively?
(5) 36 (1) 8 : 10 : 11 (2) 10 : 8 : 11
2. 22.9889 ÷ ? = 23 (3) 11 : 10 : 8 (4) 10 : 11 : 8
(1) 23 (2) 1 (5) None of these

H
(3) 232 (4) 24 8. By what per cent is the number of selected
(5) None of these employees for Finance Manager by

G
3. 1000000.000001 = ? Company C more than that of the selected
(1) 1000 (2) 100 employees by Company B for the same
(3) 1000.001
(5) 999
(4) 10000

U post?
(1) 35%
(3) 25%
(2) 30%
(4) 40%
O
4. 134% of 3894 + 38.94 of 134 = ?
(1) 11452 (2) 10000 (5) None of these
(3) 10452 (4) 1100
H
9. What is the average number of selected
(5) None of these employees for the post of Assistant
5. 103 × 1003 + 999999999 =10?+10? Marketing Managers by all companies taken
_T

(1) 6 (2) 9 together?


(3) 7 (4) 10 (1) 570 (2) 520
(5) 12 (3) 620 (4) 720
C

(5) None of these


Directions (Qs. 6–10): Study the following graph
carefully to answer the questions given below: 10. What is the ratio of selected employees for
PS

Number of selected employees in IT Managers by all Companies A, B and C?


different grades/ranks by three (1) 6 : 4 : 7 (2) 5 : 3 : 7
companies during 2012 (3) 4 : 7 : 9 (4) 8 : 7 : 6
U

Company A Company B Company C (5) None of these


1000
Number of selected employees

900 Directions (Qs. 11–15): Read the following information


@

800 800 carefully to answer the questions given below:


700
700 700 In a college, 150 students of MBA are
600 550 650 enrolled. The ratio of boys to girls is 7 : 8. There
660
500 550 are three disciplines, namely Marketing, HR
0

400
50

400 350
400
and Finance, in the college. In Marketing
300 250 150
350 30
0 discipline, girls are 50% of their total number
200 300
200
200 and boys are 40% of their total number. In HR
100 150
discipline, girls are 30% of their total number
0
Finance Assistant Marketing Assistant Assistant
while boys are 30% of their total number.
IT
Manager Finance Manager Manager IT Marketing Finance discipline has girls 20% of their total
Manager Manager Manager number and boys 30% of their total number. 7
6. What is the average number of selected boys and 9 girls are in HR and Marketing both.
employees by Company A in all grades 6 boys and 7 girls are in HR and Finance both.
taken together? 5 boys and 8 girls are in Marketing and Finance
670 ✫ Quantitative Aptitude

both. 2 boys and 3 girls are enrolled in all three


disciplines. Ratio of male to female employees
11. What percentage of students are enrolled in Company Male : Female
all the three disciplines? A 13 : 6
(1) 3.3% (2) 7.2% B 4 : 3
(3) 8.5% (4) 9.32% C 7 : 8
(5) None of these
D 9 : 11
12. What is the ratio of boys to girls only in E 23 : 12
Marketing discipline?
16. What is the number of male employees,

TS
(1) 13 : 9 (2) 9 : 13
(3) 9 : 11 (4) 11 : 9 taking all the companies together?
(5) None of these (1) 2084 (2) 2048
13. The ratio of the number of boys in Marketing (3) 2064 (4) 2046

H
and Finance both to that of girls only in (5) 2066
Finance is 17. What is the approximate average number of

G
(1) 5 : 3 (2) 3 : 5 female employees, taking all the companies
(3) 5 : 4 (4) 4 : 7 together?
(5) None of these (1) 340 (2) 315
14. By what per cent is the number of boys in
Marketing discipline more than the number U (3) 335
(5) 321
(4) 325
O
of girls in HR discipline? 18. What is the difference between the total
1 1 number of male employees working in
(1) 13 % (2) 33 %
H
3 3 companies A and C together and the total
2 2 number of female employees working in
(3) 14 % (4) 16 %
_T

3 3 companies B and D together?


(5) None of these (1) 164 (2) 146
15. The ratio of boys to girls enrolled only in (3) 144 (4) 154
HR discipline is (5) 184
C

(1) 10 : 11 (2) 9 : 10 19. What is the ratio of female employees


(3) 7 : 5 (4) 5 : 7 working in companies D and E respectively?
PS

(5) None of these (1) 17 : 22 (2) 22 : 17


Directions (Qs. 16–20): Study the following bar (3) 15 : 22 (4) 22 : 15
diagram and table carefully to answer the questions. (5) None of these
U

Number of employees working in five 20. By what per cent is the number of total
different companies. A, B, C, D and E employees of Company C more than that of
@

900
Company D?
840 (1) 12.5% (2) 16.5%
800 760 (3) 21% (4) 20%
720
Number of employees

700
700 640
(5) 16%
600 Directions (Qs. 21–25): Each of the following
questions consists of a question followed by three
500 statements I, II and III. You have to study the question
400 and the statements and decide which of the statement(s)
300 is/are necessary to answer the question.
200 21. What is the speed of a boat in still water?
I. The boat covers 12 km in 2 hours
100
downstream.
0 II. The boat covers the same distance in
A B C D E
4 hours upstream.
Company
Solved Test Papers of Various Examinations ✫ 671

III. The speed of the stream is one-third III. One of the angles of the triangle is 60°.
that of the boat in still water. (1) Only II
(1) Both I and II (2) Only III
(2) I and either II or III (3) Either II or III
(3) All I, II and III (4) Both I and III
(4) The question cannot be answered even (5) The question cannot be answered even
with the information in all three with the information in all the three
statements. statements.
(5) None of these
Directions (Qs. 26–30): The following questions are
22. What is the speed of a train?

TS
based on the pie-charts given below:
I. The length of the train is 240 metres.
Percentage-wise distribution of students studying
II. The train crosses a pole in 24 seconds.
III. The train crosses a platform in 48 in Arts and Commerce in seven different
institutions..

H
seconds.
(1) Both I and III Different institutes—A, B, C, D, E, F and G
(2) Both I and II Total number of students

G
(3) Both II and III studying Arts = 3800
(4) Any two of the three
(5) None of these
23. What is the age of a class teacher? U G
12%
A
15%
O
I. There are 11 students in the class. F B
II. The average age of the students and 13% 8%
H
the teacher is 14 years.
III. The average age of the teacher and the E C
students is 3 years more than that of 14% 17%
_T

D
the students.
21%
(1) Both I and III
(2) Both I and II
C

(3) II and either I or III Total number of students studying


(4) All I, II and III Commerce = 4200
(5) None of these
PS

24. Sri Gupta borrowed a sum at compound G A


interest. What is the amount returned in 2 11% 12%
years? F
U

13% B
I. The rate of interest is 5% per annum.
17%
II. The simple interest incurred on the
@

sum in 1 year is ` 600. E


III. The borrowed sum is ten times the 18% C
D 15%
amount earned at simple interest in
14%
two years.
(1) Only I
(2) Only III
(3) Both II and III 26. What is the total number of students
(4) Either I or III studying Arts in Institutes A and G together?
(5) All I, II and III (1) 1026 (2) 1126
(3) 1226 (4) 1206
25. What is the area of a given right-angled (5) 1306
triangle?
I. The length of the hypotenuse is 5 cm. 27. HOW many students from Institute B study
II. The perimeter of the triangle is four Arts and Commerce?
times that of its base. (1) 1180 (2) 1108
672 ✫ Quantitative Aptitude

(3) 1018 (4) 1208 34. The present age of Romila is one-fourth that
(5) 1408 of her father. After 6 years the father’s age
28. The ratio of the number of students studying will be twice the age of Kapil. If Kapil
Arts to that of students studying Commerce celebrated his fifth birthday 8 years ago,
in Institute E is what is Romila’s present age?
(1) 27 : 14 (2) 19 : 27 (1) 7 years (2) 7.5 years
(3) 19 : 16 (4) 19 : 28 (3) 8 years (4) 8.5 years
(5) None of these (5) None of these

29. The ratio of the number of students studying 35. In an examination, 30% of the total students

TS
Arts in Institute E to that studying commerce failed in Hindi, 45% failed in English and
in Institute D is 20% failed in both the subjects. Find the
(1) 12 : 17 (2) 12 : 7 percentage of those who passed in both the
(3) 19 : 21 (4) 17 : 19 subjects.

H
(5) None of these (1) 35.7% (2) 35%
(3) 40% (4) 45%
30. How many students in institutes B and D

G
(5) 44%
together study commerce?
(1) 1320 (2) 1302 Directions (Qs. 36–40): In each of these questions, two
(3) 1202
(5) None of these
(4) 1220
U equations (I) and (II) are given. You have to solve both
the equations and give answer.
O
31. Three men A, B and C start a business (1) if x < y
together. They invest ` 30000, ` 24000 and (2) if x > y
(3) if x = y
H
` 42000 respectively in the beginning. After
4 months, B takes out ` 6000 and C takes out (4) if x ≥ y
` 10000. They get a profit of ` 11960 at the (5) if x ≤ y or no relationship can be
_T

end of the year. B’s share in the profit is established between x and y.
(1) ` 2700 (2) ` 2803
36. I. x2 – 24x + 144 = 0
(3) ` 2900 (4) ` 2785
II. y2 – 26y + 169 = 0
C

(5) None of these


32. The edge of an ice cube is 14 cm. The volume 37. I. 2x2 + 3x – 20 = 0
PS

of the largest cylindrical ice cube that can be II. 2y2 + 19y + 44 = 0
formed out of it is 38. I. 6x2 + 77x + 121 = 0
(1) 2200 cu cm II. y2 + 9y – 22 = 0
(2) 2000 cu cm
U

(3) 2156 cu cm 39. I. x2 – 6x = 7


(4) 2400 cu cm II. 2y2 + 13y + 15 = 0
@

(5) None of these 40. I. 10x2 – 7x + 1= 0 ,


33. A sum of ` 16800 is divided into two parts. II. 35y2 – 12y + 1 = 0
One part is lent at a simple rate of interest
Directions (Qs. 41–42): Study the following diagram
6% per annum and the other at 8% per
to answer the questions.
annum. After 2 years the total sum received
is ` 19000. The sum lent at the rate of 6%
A B
simple interest is
(1) ` 12200
(2) ` 12000
(3) ` 11000
(4) ` 10000
(5) None of these

D C E
Solved Test Papers of Various Examinations ✫ 673

41. If the diameter of each circle is 14 cm and Directions (Qs. 46-50): Study the following table
DC = CE, the area of Δ BDE is carefully to answer these questions.
(1) 784 sq cm (2) 748 sq cm Percentage of marks obtained by six
(3) 874 sq cm (4) 441 sq cm students in six different subjects
(5) None of these Subject → History Geography Maths Science English Hindi
(out of (out of (out of (out of (out of (out
42. The area of the shaded region of square
Student ↓ 50) 50) 150) 100) 75) of 75)
ABCD is
(1) 186 sq cm (2) 168 sq cm Amit 76 85 69 73 64 88
(3) 188 sq cm (4) 441 sq cm Bharat 84 80 85 78 73 92

TS
(5) None of these Umesh 82 67 92 87 69 76
Nikhil 73 72 78 69 58 83
43. A pump can fill a tank with water in 2 hours.
Pratiksha 68 79 64 91 66 65
1
Because of a leak, it took 2 hours to fill Ritesh 79 87 88 93 82 72

H
3
the tank. The leak can drain all the water in 46. What is the approximate integral percentage
the tank in of marks obtained by Umesh in all the

G
1 subjects?
(1) 4 hours (2) 7 hours
3 (1) 80% (2) 84%
(3) 8 hours
(5) None of these
(4) 14 hours
U (3) 86%
(5) 77%
(4) 78%
O
44. A person travels from P to Q at a speed of 40 47. What is the average percentage of marks
kmph and returns to Q by increasing his obtained by all students in Hindi?
H
speed by 50%. What is his average speed for (approximated to two places of decimal)
both the trips? (1) 77.45% (2) 79.33%
(1) 36 kmph (2) 45 kmph (3) 75.52% (4) 73.52%
_T

(3) 48 kmph (4) 50 kmph ((5) None of these


(5) None of these
48. What is the average marks of all the students
45. A shopkeeper sells two watches for ` 308 in Mathematics?
C

each. On one he gets 12% profit and on the (1) 128 (2) 112
other, 12% loss. His profit or loss in the entire (3) 119 (4) 138
PS

transaction was (5) 144


11 49. What is the average marks obtained by all
(1) 1 % loss the students in Geography?
25
U

(1) 38.26 (2) 37.26


11
(2) 1 % gain (3) 37.16 (4) 39.16
25 (5) None of these
@

2
(3) 3 % loss 50. What is the total marks obtained by Ritesh
25 in all the subjects taken together?
2 (1) 401.75 (2) 410.75
(4) 3 % gain
25 (3) 402.75 (4) 420.75
(5) None of these (5) None of these

Answers and Explanatory Notes


1. Ans. (1) 2. Ans. (2)
Sol. 21 + 3.9 × 2.9 + 8.99 = ? Sol. 22.9889 ÷ ? = 23
23 ÷ ? ≈ 23
? ≈ 21 + 4 × 3 + 9 23
? ≈ 21 + 12 + 9 or ≈ 23
? 23
∴ ? ≈ 42. ∴ ? = = 1.
23
674 ✫ Quantitative Aptitude

3. Ans. (1) ∴ Required per cent


Sol. ? = 1000000.000001 250 − 200
= × 100
? ≈ 1000000 200
≈ 1000 × 1000 50
= × 100 = 25%.
∴ ? ≈ 1000. 200
9. Ans. (4)
4. Ans. (3)
Sol. Average number of selected
Sol. 134% of 3894 + 38.94 of 134 = ?
employees for the post of Assistant
134 Marketing Managers by all companies
? = 3894 × + 134 × 38.94

TS
100 800 + 700 + 660
? = 38.94 × 134 + 134 × 38.94 =
3
? ≈ 2 × (39) × 134 2160
? ≈ 78 × 134 = = 720.
3

H
∴ ? = 10452. 10. Ans. (1)
5. Ans. (2) Sol. Selected employees for IT Managers

G
Sol. 103 × 1003 + 999999999 = 10? + 10? by Company A = 300
103 × (102)3 + 1000000000 ≈ 2 × (10?) Selected employees for IT Managers
by Company B = 200
103 × 106 + 109 ≈ 2 × (10?)
109 + 109 ≈ 2 × (10?)
U Selected employees for IT Managers
by Company C = 350
O
2 × (109) ≈ 2 × 10? ∴ Ratio of selected employees for IT
∴ 10? = 109 Managers by all companies
∴ ? = 9.
H
= 300 : 200 : 350
6. Ans. (1) = 6 : 4 : 7.
For Qs. 11 to 15:
_T

Sol. Average number of selected


employees by Company A Total students of MBA = 150
150 + 300 + 300 + 500 + 650 + 800 Ratio of boys to girls = 7 : 8
= 7
6
C

Boys = 150 ×
2700 7+8
= = 450. 7
6 = 150 × = 70
PS

7. Ans. (5) 15
Sol. Selected employees for the post of 8
Girls = 150 ×
Assistant IT Managers by Company 7+8
8
U

A = 650 = 150 × = 80
Selected employees for the post of 15
Assistant IT Managers by Company
@

Discipline Boys Girls


B = 700 40 50
Selected employees for the post of Marketing, n(M) 70 × = 28 80 × = 40
100 100
Assistant IT Managers by Company 30 30
HR, n(H) 70 × = 21 80 × = 24
C = 550 100 100
Required ratio = 650 : 700 : 550 Finance, n(F) 70 ×
30
= 21 80 ×
20
= 16
= 13 : 14 : 11. 100 100
HR and Marketing, 7 9
8. Ans. (3) n(H∩M)
Sol. The number of selected employees
HR and Finance, n(H∩F) 6 7
for Finance Manager by Company
C = 250 Marketing and Finance, 5 8
The number of selected employees n(M∩F)
for Finance Manager by Company Marketing, Finance and 2 3
B = 200 HR, n(M∩F∩H)
Solved Test Papers of Various Examinations ✫ 675

Boys 15. Ans. (1)


Marketing Sol. Number of boys enrolled only in HR
HR discipline = 10
18 5 10 Number of girls enrolled only in HR
2 discipline = 11
3 4 ∴ Ratio of boys to girls enrolled only
12 in HR discipline = 10 : 11.
16. Ans. (1)
Finance Sol. Number of male employees:
13

TS
Girls In Company A = 760 ×
Marketing 13 + 6
HR 13
26 6 11 = 760 ×
19

H
5
3
4
= 40 × 13 = 520
4
4 In Company B = 840 ×

G
4+3
11. Ans. (1) Finance 4
= 840 ×
Sol. Required percentage
=
2+3
× 100 U 7
= 120 × 4 = 480
O
150 7
In Company C = 720 ×
5 7+8
= × 100 = 3.3%. 7
H
150 = 720 ×
12. Ans. (2) 15
Sol. Ratio of boys to girls only in = 48 × 7 = 336
_T

Marketing discipline 9
In Company D = 640 ×
= number of boys in Marketing 9 + 11
Discipline : number of girls in 9
= 640 ×
C

Marketing discipline 20
= 18 : 26 = 32 × 9 = 288
= 9 : 13.
PS

23
13. Ans. (3) In Company E = 700 ×
23 + 12
Sol. Number of boys in Marketing and
23
Finance both = 5 = 700 ×
U

Number of girls only in Finance = 4 35


Required ratio = 5 : 4. = 20 × 23 = 460
∴ Total number of male employees
@

14. Ans. (4) = 520 + 480 + 336 + 288 + 460


Sol. Number of boys in Marketing = 2084.
discipline = 28 17. Ans. (2)
Number of girls in HR discipline Sol. Number of female employees:
= 24 6
In Company A = 760 ×
28 − 24 13 + 6
∴ Required per cent = × 100 6
24 = 760 ×
4 19
= × 100 = 40 × 6 = 240
24 3
100 50 In Company B = 840 ×
= = 4+3
6 3 3
2 = 840 ×
= 16 %. 7
3 = 120 × 3 = 360
676 ✫ Quantitative Aptitude

8 ∴ Ratio of female employees working


In Company C = 720 × in companies D and E
7+8
8 = 352 : 240
= 720 × = 22 : 15.
15
= 48 × 8 = 384 20. Ans. (1)
Sol. Number of total employees of
11 Company C = 720
In Company D = 640 ×
9 + 11 Number of total employees of
11 Company D = 640
= 640 ×
20
720 − 640

TS
= 32 × 11 = 352 ∴ Required per cent = × 100
12 640
In Company E = 700 × 80
23 + 12 = × 100
12 640

H
= 700 × = 12.5%.
35
21. Ans. (2)
= 20 × 12 = 240

G
Sol. Let the speed of the boat be x and
∴ Average no. of female employees
that of the stream be y.
240 + 360 + 384 + 352 + 240
=

=
1576
= 315.2
5
U Then speed of boat downstream
= x + y
Speed of boat upstream
O
5
= 315 (approximate). = x – y
18. Ans. (3) From statement I,
H
Sol. Total number of male employees 12
x + y =
working in companies A and C 2
_T

x + y = 6 kmph ... (i)


13 7
= 760 × + 720 × From statement II,
13 + 6 7+8
13 7 12
= 760 × + 720 × x – y =
C

19 15 4
= 520 + 336 = 856 x – y = 3 kmph ... (ii)
From statement III,
PS

Total number of female employees


x
working in company B and D y = ... (iii)
3
3 11 From statement I and II,
= 840 × + 640 ×
U

4+3 9 + 11 x + y = 6
3 11 x – y = 3
= 840 × + 640 ×
@

7 20 2x = 9
= 360 + 352 = 712 9
∴ Required difference = 856 – 712 ∴ x = = 4.5 kmph
2
= 144. From statements I and III,
19. Ans. (4) From equations (i) and (iii), we get
Sol. Number of female employees working x
x + = 6
11 3
in Company D = 640 × 3x + x
9 × 11 = 6
11 3
= 640 × = 352 4x = 18
20
Number of female employees working 18
12 ∴ x = = 4.5 kmph
in Company E = 700 × 4
23 + 12 Hence, statements I and either II or
12 III are sufficient to answer the
= 700 × = 240
35 question.
Solved Test Papers of Various Examinations ✫ 677

22. Ans. (2) 24. Ans. (3)


Sol. From statement I, Sol. From statement I,
The length of the train = 240 m. Rate (R) = 5% per annum
But time is not given in the statement. From statement II,
Hence statement I alone is not S.I. for 1 year = Rs 600
sufficient. S.I. for 2 years = 2 × 600 = Rs 1200
From statement II, From statement III,
Time taken by the train to cross a P = 10 × S.I. in two years
pole = 24 sec. = 10 × Rs 1200
But the length (distance) is not given = Rs 12000

TS
in the statement. Hence, statement II From statements II and III, we get
alone is not sufficient. S.I. × 100
From statement III, R=
P ×T

H
Time taken by the train to cross the
1200 × 100
platform = 48 sec. = = 5%
But the length of the train and the 12000 × 2

G
T
platform are not given. R
∴ Amount = P 1 +
Therefore, statement III alone is not 100
sufficient.
Now,
U = 12000 1 +
5
100
2
O
From statements I and II, we get 2
length of the train = 240 m 1
= 12000 1 +
Time taken by the train to cross a 20
H
2
pole = 24 Sec. 21
Length of the train = 12000
Speed of train = 20
_T

time 441
240 = 12000 × = 30 × 441
= = 10 m/sec 400
24 = Rs 13230
Hence, both I and II together are
C

Hence, statements II and III are


needed to answer the question. necessary to answer the question.
23. Ans. (4)
PS

Sol. From statement I, 25. Ans. (4)


Number of students in class = 11 Sol. From statement I,
From statment II, Length of the hypotenuse
AC = b = 5 cm
U

Average age of students and class


teacher = 14 years From statement II,
From statement III, Perimeter of the triangle
@

The average age of class teacher is 3 = 4 × base


years more than that of students.
Now, A
From statements I, II and III
Average age of (students + teacher)
= 14 × 12 = 168 years
c b
Average age of 11 students
= 14 – 3 = 11 years
Total age of 11 students 60°
= 11 × 11 = 121 years B a C
∴ Class teacher’s age
= 168 – 121 = 47 years From statement III,
Hence, all statements (I, II and III) are One of the angles of the triangle, say
needed to answer the question. ∠ C be 60°
678 ✫ Quantitative Aptitude

From statements I and II, Number of students studying


BC Commerce in Institute E
cos 60° =
AC 18
or BC = AC × cos 60° = 4200 ×
100
1 5 1 = 756
=5× =  cos 60° =
2 2 2 ∴ Required ratio = 532 : 756
5 = 19 : 27.
 BC = a =
2 29. Ans. (3)
Now, area of the triangle ABC Sol. Number of students studying Arts in

TS
1 14
= ab sin θ Institute E = 3800 ×
2 100
1 5 = 532
= × × 5 × sin 60°
2 2 Number of students studying

H
Commerce in Institute D
1 5 3
= × × 5 × 14

G
2 2 2 = 4200 ×
100
25 3 = 588
= cm2
∴ Required ratio = 532 : 588
8
Hence, statements I and III are
necessary to answer the question. U 30. Ans. (2)
= 19 : 21.
O
26. Ans. (1) Sol. Number of students studying
Sol. Number of students studying Arts in
Commerce in Institute B
H
15 17
Institute A = 3800 × = 4200 ×
100 100
= 714
_T

= 570
Number of students studying Art in Number of students studying
Commerce in Institute D
12
Institute G = 3800 × = 456 14
= 4200 × = 588
C

100
∴ Total number of students studying 100
Arts in institutes A and G together ∴ Total number of students studying
PS

= 570 + 456 = 1026. commerce in institutes B and D


27. Ans. (3) together
Sol. Number of students studying Arts in = 714 + 588 = 1302.
31. Ans. (2)
U

8
Institute B = 3800 × Sol. A’s share in business
100 = Rs 30000 × 12 – Rs 360000
= 304
@

After 4 months, B took out Rs 6000.


Number of students studying Then, B’s share in business
Commerce in Institute B = Rs [24000 × 4 + (24000 – 6000) × 8]
17 = Rs (96000 + 144000)
= 4200 ×
100 = Rs 240000
= 714 After 4 months, C took out Rs 10000
∴ Total number of students studying in then, C’s share in business
Arts and Commerce in Institute B = Rs [42000 × 4 + (42000 – 10000) × 8]
= 304 + 714 = Rs [168000 + 32000 × 8]
= 1018. = Rs [168000 + 25000]
28. Ans. (2) = Rs 424000
Ratio of capital
Sol. Number of students studying Arts in = 360000 : 240000 : 424000
14
Institute E = 3800 × = 360 : 240 : 424
100 = 45 : 30 : 53
= 532
Solved Test Papers of Various Examinations ✫ 679

Sum of ratio = 45 + 30 + 53 Kapil’s age after 6 years


= 128 = 12 + 6 = 18 years
Now, B’s share Romila’s father’s age
30 = 2 × Kapil’s age
= × 11960 = 2 × 18 = 36 years
128
= Rs 2803.125 Father’s present age
= Rs 2803 (approximate). = 36 – 6 = 30 years
32. Ans. (3) ∴ Romila’s present age
Sol. The edge of an ice cube = 14 cm. 1
= × father’s present age

TS
Radius of the cylinder 4
Edge of an ice cube 1
= = × 30 years
2 4
14 = 7.5 years.

H
= = 7 cm
2 35. Ans. (4)
Height of the cylinder = edge of an Sol. No. of students who failed in Hindi

G
ice cube = 14 cm only
∴ Volume of the largest cylinder = (30 – 20) = 10
= πr2h No. of students who failed in English

=
22
× 7 × 7 × 14 U only
= (45 – 20)
O
7 = 25
= 22 × 7 × 14
Students who failed in both the
= 2156 cm3. subjects = 20
H
33. Ans. (1) ∴ Those who passed in both the subjects
Sol. Let the sum lent at 6% rate of interest = 100 – (10 + 25 + 20)
_T

be Rs x = (100 – 55)%
Then, Rs (16800 – x) is lent at 8% rate = 45%.
of interest [Alternative Method (Fast-Track)
Then, S.I. = Rs (19000 – 16800) Let the number of students be 100.
C

= Rs 2200 Number of students who failed in


(16800 − x ) × 2 × 8 Hindi is 30%.
x×6×2
PS

∴ + = 2200 n(H) = 30
100 100 Number of students who failed in
12 x 268800 − 16 x English is 45%
⇒ + = 2200 n(E) = 45
100 100
U

⇒ 12x + 268800 – 16x = 2200 × 100 Number of students who failed in


⇒ 268800 – 4x = 220000 both the subjects is 20%
n(H∩E) = 20
@

⇒ 4x = 268800 – 220000
∴ n(H∪E) = n(H) + n(E) – n(H∩E)
⇒ 4x = 48800
= 30 + 45 – 20
48800
x = = 55
4 Percentage of students who failed in
x = Rs 12200 both the subjects = 55%
∴ The sum lent at 6% rate of interest is ∴ Number of students who passed in
Rs 12200. both the subjects = 100 – 55
= 45%.]
34. Ans. (2) 36. Ans. (1)
Sol. Kapil celeberated his fifth birthday 8 Sol. I. x2 – 24x + 144 = 0
years ago. ⇒ x2 – 12x – 12x +144 = 0
So, Kapil’s present age ⇒ x (x – 12) – 12 (x – 12) = 0
= (8 + 5 – 1) ⇒ (x – 12) (x – 12) = 0
= 12 years ∴ x = 12
680 ✫ Quantitative Aptitude

II. y2 – 26y + 169 = 0 40. Ans. (4)


⇒ y2 – 13y – 13y + 169 = 0 Sol. I. 10x2 – 7x + 1 = 0
⇒ y (y – 13) – 13 (y – 13) = 0 10x2 – 5x – 2x + 1 = 0
(y – 13) (y – 13) = 0 ⇒ 5x(2x – 1) –1 (2x – 1) = 0
∴ y = 13 ⇒ (5x – 1) (2x – 1) = 0
Hence, x < y. 1 1
∴ x = ,
37. Ans. (4) 5 2
2
Sol. I. 2x2 + 3x – 20 = 0 II. 35y – 12y + 1 = 0
2x2 + 8x – 5x – 20 = 0 35y2 – 7y – 5y + 1 = 0

TS
⇒ 2x(x + 4) –5(x + 4) = 0 7y(5y – 1) – 1 (5y – 1) = 0
⇒ (x + 4) (2x – 5) = 0 ⇒ (7y – 1) (5y – 1) = 0
5 1 1
∴ x = – 4, ∴ y = ,
7 5

H
2
II. 2y2 + 19y + 44 = 0 Hence, x ≥ y.
2y2 + 8y + 11y + 44 = 0 41. Ans. (1)

G
⇒ 2y(y + 4) + 11(y + 4) = 0 Sol.
A B
⇒ (y + 4) (2y + 11) = 0
∴ y = – 4, –
11
2
Hence, x ≥ y. U
O
38. Ans. (5)
Sol. I. 6x2 + 77x + 121 = 0
H
6x2 + 66x + 11x + 121 = 0
⇒ 6x(x + 11) + 11(x + 11) = 0 D C E
_T

⇒ (x + 11) (6x + 11) = 0 In Δ BDE


11 DC = 28 cm (because diameter of each
∴ x = – 11, –
6 circle is 14 cm)
II. y2 + 9y – 22 = 0
C

∴ DE = DC + CE
y2 + 11y – 2y – 22 = 0 = 28 + 28
⇒ y(y + 11) – 2 (y + 11) = 0
PS

[∴ DC = CE = 28 cm]
⇒ (y + 11)(y – 2) = 0 = 56 cm
∴ y = –11, 2 And BC = 28 cm (because diameter
Hence, no relationship can be of each circle is 14 cm)
U

established between x and y. 1


Area of Δ BDE = × DE × BC
39. Ans. (2) 2
1
@

Sol. I. x2 – 6x = 7 = × 56 × 28
2
x2 – 6x – 7 = 0
= 784 sq cm.
⇒ x2 – 7x + x – 7 = 0
42. Ans. (2)
⇒ x(x – 7) +1 (x – 7) = 0 Sol. Area of the square = (28)2
⇒ (x + 1) (x – 7) = 0 = 784 sq. cm
∴ x = – 1, 7 Area of four circles = 4 × πr2
II. 2y2 + 13y + 15 = 0 22
2y2 + 10y + 3y + 15 = 0 = 4 × × 7 × 7
7
⇒ 2y(y + 5) +3(y + 5) = 0 = 616 sq. cm
⇒ (2y + 3)(y + 5) = 0 ∴ Area of shaded parts
3 = Area of the square
∴ y = – , – 5 – Area of four circles
2 = 784 – 616
Hence, x > y.
= 168 sq. cm.
Solved Test Papers of Various Examinations ✫ 681

43. Ans. (4) 1 = 41 + 33.50 + 138 + 87 + 51.75 + 57


Sol. Part filled by the pump in 1 hr = = 408.25
2
Net part filled by the pump and leak ∴ Required per cent
3 408.25
in 1 hr = = × 100
7 (50 + 50 + 150 + 100 + 75 + 75)
Emptying work done by the leak in 408.25
= × 100 = 81.65%
1 3 500
1 hr = − = 80% (approximate).
2 7
7−6

TS
47. Ans. (2)
=
14 Sol. Average percentage marks obtained
1 by all the students in Hindi
= part 88 + 92 + 76 + 83 + 65 + 72
14

H
= × 100
∴ Leak can empty the tank in 14 hours. 6
476
= = 79.33%.

G
44. Ans. (3)
6
Sol. Speed of man from P to Q = 40 kmph
And man returns to Q by increasing 48. Ans. (3)
speed by 50%
Then, speed of man from Q to P U Sol. Average marks obtained by all the
students in Mathematics
O
100 + 50 (69 + 85 + 92 + 78 + 64 + 88)
= 40 × = 150 ×
100 100 × 6
H
476
150 = 150 ×
= 40 × 600
100 476
_T

= 60 kmph = = 119.
2 × 40 × 60 4
∴ Average speed =
40 + 60 49. Ans. (4)
C

= 48 kmph. Sol. Average marks obtained by all the


45. Ans. (1) students in Geography
(85 + 80 + 67 + 72 + 79 + 87 )
2
PS

Common gain or loss


Sol. Loss% = = 50 ×
10 6 × 100
Here, gain = 12%, loss = 12% 470
2
12 = 50 ×
U

Loss% = 600
10 470
144 36 = = 39.16.
12
@

= =
100 25
11 50. Ans. (5)
= 1 %. Sol. Total marks obtained by Ritesh in all
25
46. Ans. (1) the subjects together
Sol. Total marks obtained by Umesh in all 79 87 88
= 50 × + 50 × + 150 ×
subjects together 100 100 100
82 72
82 67 92 + 93 + 75 × + 75 ×
= 50 × + 50 × + 150 × 100 100
100 100 100
87 69 76 = 39.50 + 43.50 + 132 + 93 + 61.50 + 54
+ 100 × + 75 × + 75 × = 423.5.
100 100 100
682 ✫ Quantitative Aptitude

State Bank of India Probationary Officers


Examination, April 2013
Directions (Qs. 1–5): Study the following graph Directions (Qs. 6–10): Study the radar graph carefully
carefully to answer the questions that follow: and answer the questions that follow:
Cost of three different fruits (in rupees Number of Students (in thousand) in two
per kg) in five different cities Different Universities in six Different years
Apple Grapes Guava Number of Students
200
Cost of Fruits (rupees per kg)

TS
University-1
180
160 University-2
140
120 2007
35
100

H
80 30
60 20

G
40
2012 20 2008
20
0 15
Jalandhar Delhi Chandiarh Hoshiarpur Ropar
Cities
U 10
5
O
1. In which city is the difference between the 0
cost of one kg of apple and the cost of one
kg of guava the second lowest?
H
(1) Jalandhar (2) Delhi
(3) Chandigarh (4) Hoshiarpur 2011 2009
_T

(5) Ropar
2. The cost of one kg of guava in Jalandhar is
approximately what per cent of the cost of
two kg of grapes in Chandigarh? 2010
C

(1) 66 (2) 24 6. What was the difference between the number


(3) 28 (4) 34 of students in University-1 in the year 2010
PS

(5) 58 and the number of students in University-2


3. What total amount will Ram pay to the in the year 2012?
shopkeeper for purchasing 3 kg of apples (1) Zero (2) 5,000
U

and 2 kg of guavas in Delhi? (3) 15,000 (4) 10,000


(1) ` 530 (2) ` 450 (5) 1,000
(3) ` 570 (4) ` 620
@

(5) ` 490 7. What is the sum of the number of students


in University-1 in the year 2007 and the
4. Ravinder had to purchase 45 kg of grapes number of students in University-2 in the
from Hoshiarpur. The shopkeeper gave him year 2011 together?
a discount of 4% per kg. What amount did
(1) 50000 (2) 55000
he pay to the shopkeeper after the discount?
(1) ` 8208 (2) ` 8104 (3) 45000 (4) 57000
(3) ` 8340 (4) ` 8550 (5) 40000
(5) ` 8410 8. If 25% of the students in University-2 in the
5. What is the ratio of the cost of one kg of year 2010 were females, what was the
apples from Ropar to the cost of one kg of number of male students in University-2 in
grapes from Chandigarh? the same year?
(1) 3 : 2 (2) 2 : 3 (1) 11250 (2) 12350
(3) 22 : 32 (4) 42 : 92 (3) 12500 (4) 11500
(5) 92 : 42 (5) 11750
Solved Test Papers of Various Examinations ✫ 683

9. What was the per cent increase in the number 14. The number of players participating in Kho-
of students in University-1 in the year 2011 Kho from School-4 is what per cent of the
as compared to the previous year? number of players participating in Hockey
(1) 135 (2) 15 from School-2?
(3) 115 (4) 25 (1) 42 (2) 48
(5) 35 (3) 36 (4) 40
10. In which year was the difference between the (5) 60
number of students in University-1 and the 15. 25% of the number of players participating
number of students in University-2 the in Hockey from School-5 are females. What

TS
highest? is the number of the Hockey players who are
(1) 2008 (2) 2009 males in School-5?
(3) 2010 (4) 2011 (1) 15 (2) 18
(5) 2012 (3) 30 (4) 21

H
Directions (Qs. 11–15): Study the graph carefully to (5) 27
answer the questions that follow. Directions (Qs. 16–20): Study the following bar-graph

G
Number of players participating in three carefully and answer the following questions.
different games in five different schools Earning (in rupees) of three different

90
Hockey Basketball Kho-Kho
U persons on four different days
Rahul Gita Naveen
O
80
70 Thursday
Number of players

H
60
50
_T

40 Wednesday
30
20
10 Tuesday
C

0
School-1 School-2 School-3 School-4 School-5
PS

School
Monday
11. What is the total number of players
participating in Hockey from all the five
0 100 200 300 400 500
schools together?
U

Earning (in Rupees)


(1) 324 (2) 288
(3) 342 (4) 284 16. What is Gita’s average earning over all the
@

(5) 248 days together?


12. What is the ratio of the number of players (1) ` 285 (2) ` 290
participating in Basketball from School-1 to (3) ` 320 (4) ` 310
the number of players participating in Kho- (5) ` 315
Kho from School-3? 17. What is the total amount earned by Rahul
(1) 5 : 7 (2) 7 : 9 and Naveen together on Tuesday and
(3) 7 : 8 (4) 9 : 7 Thursday together?
(5) 5 : 8
(1) ` 1,040 (2) ` 1,020
13. In which school is the number of players (3) ` 980 (4) ` 940
participating in Hockey and Basketball (5) ` 860
together the second highest?
(1) School-1 (2) School-2 18. Gita donated her earnings of Wednesday to
(3) School-3 (4) School-4 Naveen. What was Naveen’s total earning on
(5) School-5 Wednesday after Gita’s donation?
684 ✫ Quantitative Aptitude

(1) ` 520 (2) ` 550 21. What is the difference between the total
(3) ` 540 (4) ` 560 number of employees in teaching and
(5) ` 620 medical profession together and the number
of employees in management profession?
19. What is the difference between Rahul’s (1) 6770 (2) 7700
earning on Monday and Gita’s earning on (3) 6700 (4) 7770
Tuesday? (5) 7670
(1) ` 40 (2) ` 20 22. In management profession three-fourths of
(3) ` 50 (4) ` 30 the number of employees are females. What
(5) ` 10

TS
is the number of male employees in
20. What is the ratio of Naveen’s earning on management profession?
Monday, Wednesday and Thursday? (1) 1239 (2) 1143
(3) 1156 (4) 1289
(1) 7 : 3 : 5 (2) 8 : 6 : 5

H
(5) 1139
(3) 8 : 7 : 4 (4) 9 : 5 : 4
(5) 9 : 3 : 4 23. 25% of employees from film production

G
profession went on a strike. What is the
Directions (Qs. 21–25): Study the pie-chart and number of employees from film production
answer the following questions.
Percentage distribution of employees
U who did not participate in the strike?
(1) 3271
(3) 3948
(2) 3819
(4) 1273
O
in six different professions
Total number of employees = 26800 (5) 1246
24. What is the total number of employees in
H
En

engineering profession and industries


Ma

g
9% ng
in e e

together?
na %

_T
ge m
17

(1) 5698 (2) 5884


ri

Teaching (3) 5687 (4) 5896


e nt

15% (5) 5487


Industries
C

13% 25. In teaching profession if three-fifths of the


Film teachers are not permanent, what is the
PS

Production number of permanent teachers in the


19% teaching profession?
Medical
(1) 1608 (2) 1640
27%
(3) 1764 (4) 1704
U

(5) 1686
Directions (Qs. 26–30): Study the table carefully to answer the questions that follow:
@

Monthly Bill (in rupees) on landline phone, electricity, laundry and


mobile phone paid by three people in five months
Monthly Bills
Month Landline Phone Electricity Laundry Mobile Phone
Ravi Dev Manu Ravi Dev Manu Ravi Dev Manu Ravi Dev Manu
March 234 190 113 145 245 315 93 323 65 144 234 345
April 124 234 321 270 220 135 151 134 35 164 221 325
May 156 432 211 86 150 98 232 442 132 143 532 332
June 87 123 124 124 150 116 213 324 184 245 134 125
July 221 104 156 235 103 131 143 532 143 324 432 543
Solved Test Papers of Various Examinations ✫ 685

26. What is the total amount of bill paid by Dev (3) ` 190 (4) ` 167
in the month of June for all the four (5) ` 196
commodities? 29. In which months respectively did Manu pay
(1) ` 608 (2) ` 763 the second highest mobile phone bill and the
(3) ` 731 (4) ` 683 lowest electricity bill?
(5) ` 674 (1) April and June
27. What is the average electricity bill paid by (2) April and May
Manu over all the five months together? (3) March and June
(1) ` 183 (2) ` 149 (4) March and May

TS
(3) ` 159 (4) ` 178 (5) July and May
(5) ` 164 30. What is the ratio of the electricity bill paid
28. What is the difference between the mobile by Manu in the month of April to the mobile
phone bill paid by Ravi in the month of May phone bill paid by Ravi in the month of June?

H
and the laundry bill paid by Dev in the (1) 27 : 49 (2) 27 : 65
month of March? (3) 34 : 49 (4) 135 : 184

G
(1) ` 180 (2) ` 176 (5) 13 : 24
Directions (Qs. 31–35): Study the table carefully and answer the questions that follow:

Station Arrival time Departure U


Halt time (in
Distance
travelled from
No. of passengers
boarding the train
O
time minutes) origin (in km) at each station
Dadar Starting 12.05 am — 0 km 437
H
Vasai Road 12.53 am 12.56 am 3 minutes 42 km 378
Surat 4.15 am 4.20 am 5 minutes 257 km 458
_T

Vadodara 6.05 am 6.10 am 5 minutes 386 km 239


Anand Jn 6.43 am 6.45 am 2 minutes 422 km 290
Nadiad Jn 7.01 am 7.03 am 2 minutes 440 km 132
Ahmedabad 8.00 am 8.20 am 20 minutes 486 km 306
C

Bhuj 5.40 pm Ending point — 977 km None


PS

31. What is the distance travelled by the train 34. If the halt time (stopping time) of the train at
from Surat to Nadiad Jn? Vadodara is decreased by 2 minutes and
(1) 176 km (2) 188 km increased by 23 minutes at Ahmedabad, at
U

(3) 183 km (4) 193 km what time will the train reach Bhuj?
(5) 159 km (1) 6.10 am (2) 6.01 pm
@

32. How much time does the train take to reach (3) 6.05 am (4) 6.50 pm
Ahmedabad after departing from Anand Jn (5) 6.07 pm
(including the halt time)? 35. The distance between which two stations is
(1) 1 hr 59 min (2) 1 hr 17 min the second lowest?
(3) 1 hr 47 min (4) 1 hr 45 min (1) Nadiad Jn to Ahmedabad
(5) 1 hr 15 min (2) Anand Jn to Nadiad Jn
33. What is the ratio of the number of passengers (3) Dadar to Vasai Road
boarding from Vasai Road to that boarding (4) Anand Jn to Vadodara
from Ahmedabad in the train? (5) Vasai Road to Surat
(1) 21 : 17 (2) 13 : 9
(3) 21 : 19 (4) 15 : 13 Directions (Qs. 36–40): Study the table carefully to
(5) 13 : 15 answer the questions that follow.
686 ✫ Quantitative Aptitude

Maximum and Minimum temperature (in degree Celsius) recorded on


1st day of each month in five different cities
Temperature
Month Bhuj Sydney Ontario Kabul Beijing
Max Min Max Min Max Min Max Min Max Min
1st September 24 14 12 2 5 1 34 23 12 9
1st October 35 21 5 –1 15 6 37 30 9 3
1st November 19 8 11 3 4 0 45 36 15 1

TS
1st December 9 2 –5 –9 –11 –7 31 23 2 –3
1st January –4 –7 –11 –13 –14 –19 20 11 5 –13

H
36. What is the difference between the Directions (Qs. 41–45): Study the following pie-
maximum temperature of Ontario on 1st chart and table carefully to answer the questions that

G
November and the minimum temperature follow.
of Bhuj on 1st January? Total cars = 700
(1) 3°C
(3) 15°C
(5) 11°C
(2) 18°C
(4) 9°C
U Distribution of cars
O
State-1
37. In which months respectively is the 14%
State-4
maximum temperature of Kabul the second
H
26%
highest and the minimum temperature of
Sydney the highest? State-2
_T

(1) 1st October and 1st January 28%


State-3
(2) 1st October and 1st November
32%
(3) 1st December and 1st January
(4) 1st September and 1st January
C

(5) 1st December and 1st September


Table showing the ratio of diesel to petrol
PS

38. In which month (on 1st day) is the difference


between maximum temperature and engine cars which are distributed among
minimum temperature of Bhuj the second four different states
highest? State Diesel Engine Petrol Engine
U

(1) 1st September (2) 1st October Cars Cars


(3) 1st November (4) 1st December
State-1 3 4
@

(5) 1st January


State-2 5 9
39. What is the average maximum temperature State-3 5 3
of Beijing over all the months together? State-4 1 1
(1) 8.4 °C (2) 9.6 °C
(3) 7.6 °C (4) 9.2 °C 41. What is the difference between the number
(5) 8.6 °C of diesel engine cars in State-2 and the
40. What is the ratio of the minimum number of petrol engine cars in State-4?
temperature of Beijing on 1st September to (1) 159 (2) 21
the maximum temperature of Ontario on 1st (3) 28 (4) 34
October? (5) 161
(1) 3 : 4 (2) 3 : 5 42. The number of petrol engine cars in State-3
(3) 4 : 5 (4) 1 : 5 is what per cent more than the number of
(5) 1 : 4 diesel engine cars in State-1?
Solved Test Papers of Various Examinations ✫ 687

(1) 100 (2) 200 5 1


(3) 300 (4) 125 (1) (2)
18 18
(5) 225 1 1
(3) (4)
43. If 25% of diesel engine cars in State-3 are AC 9 6
and the remaining cars are non-AC, what is 5
(5)
the number of diesel engine cars in State-3 36
which are non-AC? Directions (Qs. 48-50): Study the information carefully
(1) 75 (2) 45 to answer these questions.
(3) 95 (4) 105 In a team there are 240 members (males and

TS
(5) 35 females). Two-thirds of them are males. Fifteen
44. What is the difference between the total per cent of males are graduates. Remaining
number of cars in State-3 and the number of males are non-graduates. Three-fourths of the
petrol engine cars in State-2? females are graduates. Remaining females are

H
(1) 96 (2) 106 non-graduates.
(3) 112 (4) 102 48. What is the difference between the number

G
(5) 98
of females who are non-graduates and the
45. What is the average number of petrol engine number of males who are graduates?
cars in all the states together?
(1) 86.75 (2) 89.25
U (1) 2
(3) 4
(2) 24
(4) 116
O
(3) 89.75 (4) 86.25 (5) 36
(5) 88.75
49. What is the sum of the number of females
H
46. A bag contains 7 blue balls and 5 yellow who are graduates and the number of males
balls. If two balls are selected at random, who are non-graduates?
what is the probability that none is yellow?
_T

(1) 184 (2) 96


5 5
(1) (2) (3) 156 (4) 84
33 22 (5) 196
7 7
(3) (4)
C

22 33 50. What is the ratio of the total number of males


7 to the number of females who are non-
PS

(5) graduates?
66
(1) 6 : 1 (2) 8 : 1
47. A die is thrown twice. What is the probability
(3) 8 : 3 (4) 5 : 2
of getting a sum 7 from both the throws?
(5) 7 : 2
U

Answers and Explanatory Notes


@

1. Ans. (2) In Chandigarh,


Sol. According to the graph, Cost of one kg of apple = Rs 180
In Jalandhar, Cost of one kg of guava = Rs 100
Cost of one kg of apple = Rs 160 Difference between the cost of apple
Cost of one kg of guava = Rs 60 and guava
Difference between the cost of apple = Rs (180 – 100) = Rs 80
and guava In Hoshiarpur,
= Rs (160 – 60) = Rs 100 Cost of one kg of apple = Rs 90
In Delhi, Cost of one kg of guava = Rs 30
Cost of one kg of apple = Rs 130 Difference between the cost of apple
Cost of one kg of guava = Rs 90 and guava
Difference between the cost of apple = Rs (90 – 30) = Rs 60
and guava In Ropar,
= Rs (130 – 90) = Rs 40 Cost of one kg of apple = Rs 40
688 ✫ Quantitative Aptitude

Cost of one kg of guava = Rs 10 Number of students in University-2


Difference between the cost of apple in the year 2012 = 20000
and guava Difference between the number of
= Rs (40 – 10) = Rs 30 students in University-1 and
Hence, the second lowest difference University-2
between price of one kg apple and = 20000 – 15000
one kg guava is in Delhi. = 5000.
7. Ans. (5)
2. Ans. (4)
Sol. Number of students in University-1
Sol. Cost of one kg of guava in Jalandhar in the year 2007 = 10000
= Rs 60

TS
Number of students in University-2
Cost of two kg of grapes in Chandigarh in the year 2011 = 30000
= Rs (2 × 90) ∴ Total number of students
= Rs 180 = 10000 + 30000

H
∴ Required percentage = 40000.
60 8. Ans. (1)
= × 100%

G
180 Sol. Total number of students in
100 University-2 in the year 2010 = 15000
= % = 33.33% Number of female students in
3
= 34% (approximate).
U University-2 in the year 2010
= 15000 ×
25
O
3. Ans. (3) 100
Sol. In Delhi, = 3750
Cost of three kg of apples ∴ Number of male students in
H
= Rs (3 × 130) University-2 in the year 2010
= Rs 390 = 15000 – 3750
_T

Cost of two kg of guavas = 11250.


= Rs (2 × 90) [Alternative Method (Fast-track)
= Rs 180 Number of male students
∴ Total amount = Rs (390 + 180) 100 − 25
C

= Rs 570. = 15000 ×
100
4. Ans. (1) 75
PS

Sol. Cost of 45 kg grapes in Hoshiarpur = 15000 ×


= Rs (45 × 190) 100
= Rs 8550 = 11250.]
After 4% discount, cost price of grapes 9. Ans. (4)
U

4 Sol. Number of students in University-1


= 8550 – 8550 ×
100 in the year 2011 = 25000
= 8550 – 342
@

Number of students in University-1


= Rs 8208
in the year 2010 (previous year)
Hence, Ravinder had to pay Rs 8208.
= 20000
5. Ans. (3) ∴ Required percentage
Sol. Cost of one kg of apples in Ropar
25000 − 20000
= Rs 40 = × 100%
Cost of one kg of grapes in 20000
Chandigarh 5000 × 100
= Rs 90 = %
20000
Required ratio = 40 : 90
= 4 : 9 = 22 : 32. = 25%.

6. Ans. (2) 10. Ans. (5)


Sol. Number of students in University-1 Sol. Difference between the number of
in the year 2010 = 15000 students in University-1 and
University-2:
Solved Test Papers of Various Examinations ✫ 689

In 2007 = 20000 – 10000 = 10000 Number of male players players parti-


In 2008 = 25000 – 15000 = 10000 cipating in Hockey from school-5
In 2009 = 35000 – 25000 = 10000 = 36 – 9 = 27.
In 2010 = 20000 – 15000 = 5000 [Alternative Method (Fast-track)
In 2011 = 30000 – 25000 = 5000 Number of male players
In 2012 = 35000 – 20000 = 15000 100 − 25
Hence, there is the highest difference = 36 ×
in the year 2012. 100
75
11. Ans. (2) = 36 × = 27.]
100

TS
Sol. Total number of players participating
16. Ans. (2)
in Hockey from all the five schools
Sol. Gita’s average earning over all the
together
days together
= 68 + 80 + 54 + 50 + 36
140 + 200 + 420 + 400

H
= 288. = Rs
4
12. Ans. (3)

G
Sol. Number of players participating in 1160
= Rs = Rs 290.
Basketball from School-1 = 42 4
Number of players participating in
Kho-Kho from School-3 = 48
Required ratio = 42 : 48 U 17. Ans. (4)
Sol. Rahul’s earning on Tuesday and
Thursday
O
= 7 : 8. = Rs (180 + 340)
13. Ans. (2) = Rs 520
H
Sol. Total number of players participating Naveen’s earning on Tuesday and
in Hockey and Basketball together: Thursday
_T

In School-1 = 68 + 42 = 110 = Rs (260 + 160)


In School-2 = 80 + 34 = 114 = Rs 420
In School-3 = 54 + 28 = 82 Total amount earned by Rahul and
In School-4 = 50 + 60 = 110 Naveen together on Tuesday and
C

In School-5 = 36 + 82 = 118 Thursday


Hence, the number of players = Rs (520 + 420) = Rs 940.
PS

participating in Hockey and Basketball 18. Ans. (3)


together is the second highest in
Sol. Gita donated her earnings of
School-2.
Wednesday to Naveen
U

14. Ans. (4) = Rs 420


Sol. Number of players participating in Naveen’s earning on Wednesday
@

Kho-Kho from School-4 = 32 = Rs 120


Number of players participating in Naveen’s total earning on Wednesday
Hockey from School-2 = 80 after Gita’s donation
32 = Rs (120 + 420)
∴ Required percentage = × 100%
80 = Rs 540.
= 40%. 19. Ans. (1)
15. Ans. (5) Sol. Rahul’s earning on Monday = Rs 240
Sol. Number of players participating in Gita’s earning on Tuesday = Rs 200
Hockey from School-5 = 36 Difference between Rahul’s earning
Number of female players partici- on Monday and Gita’s earning on
pating in Hockey from School-5 Tuesday
25 = Rs (240 – 200)
= 36 × = 9 = Rs 40.
100
690 ✫ Quantitative Aptitude

20. Ans. (5) Number of employees from film


Sol. Ratio of Naveen’s earning on Monday, production who went on strike
25
Wednesday and Thursday = 5092 ×
= 360 : 120 : 160 = 1273 100
= 9 : 3 : 4. ∴ Number of employees who have not
21. Ans. (3) participated in strike = 5092 – 1273
Sol. Total number of employees = 26800 = 3819.
Number of employees in teaching [Alternative Method (Fast-track)
profession Required number of employees who
15 have not participated in strike

TS
= 26800 ×
= 4020 100 19 100 − 25
= 26800 × ×
Number of employees in medical 100 100
profession 27 19 75
= 26800 × ×

H
= 26800 × 100 100
100
= 7236 = 67 × 19 × 3 = 3819.]

G
Total number of employees in
teaching and medical profession 24. Ans. (4)
= 4020 + 7236 Sol. Total number of employees = 26800
= 11256
Number of employees in management U Number of employees in engineering
profession 9
O
= 26800 ×
profession 17 100
= 2412
= 26800 ×
100 Number of employees in industries
H
= 4556
13
Difference between the total number = 26800 ×
of employees in teaching and medical = 3484 100
_T

profession and the number of Total number of employees in


employees in management profession engineering profession and industries
= 11256 – 4556 together
C

= 6700. = 2412 + 3484


22. Ans. (5) = 5896.
PS

Sol. Total number of employees = 26800 25. Ans. (1)


Number of employees in management Sol. Total number of employees = 26800
profession 17 Number of employees in teaching
U

= 26800 × profession
100 15
= 4556 = 26800 ×
Number of female employees in = 4020 100
@

management profession Number of teachers who are not


3 permanent 3
= 4556 ×
= 3417 4 = 4020 ×
5
∴ Number of male employees in = 2412
management profession Number of teachers who are
= 4556 – 3417 permanent
= 1139. = 4020 – 2412
= 1608.
23. Ans. (2)
Sol. Total number of employees = 26800 26. Ans. (3)
Number of employees in film Sol. Total amount of bill paid by Dev in
production the month of June for all the four
19 commodities (Landline phone,
= 26800 ×
= 5092 100 Electricity, laundry and mobile phone)
Solved Test Papers of Various Examinations ✫ 691

= Rs (123 + 150 + 324 + 134) 33. Ans. (1)


= Rs 731. Sol. Number of passengers boarding the
27. Ans. (3) train at Vasai road station = 378
Sol. Average electricity bill paid by Manu Number of passengers boarding the
over all the five months together train at Ahmedabad = 306
(315 + 135 + 98 + 116 + 131) ∴ Required ratio = 378 : 306
= Rs = 21 : 17.
5
795 34. Ans. (2)
= Rs Sol. If the halt time of the train at Vadodara
5

TS
= Rs 159. is decreased by 2 minutes and
increased by 23 minutes at
28. Ans. (1)
Sol. Mobile phone bill paid by Ravi in the Ahmedabad, then,
month of May = Rs 143 Arrival time of the train at Bhuj

H
The laundry bill paid by Dev in the = (5.40 + 0.23 – 0.2)
month of March = Rs 323 = (5.40 + 0.21)

G
∴ Required difference = Rs (323 – 143) = 6.01 p.m.
= Rs 180. 35. Ans. (4)
29. Ans. (4)
Sol. According to the table, Manu paid
second highest mobile bill of Rs 345 U Sol. According to the table,
Distance between Dadar to Vasai Road
= 42 km
O
in the month of March of and the Distance between Vasai Road to Surat
lowest electricity bill of Rs 98 in the = 257 – 42
H
month of May. = 215 km
30. Ans. (2) Distance between Surat to Vadodara
_T

Sol. Electricity bill paid by Manu in the = 386 – 257


month of April = Rs 135 = 129 km
Mobile phone bill paid by Ravi in the Distance between Vadodara to
month of June = Rs 245 Anand Jn. = 422 – 386
C

Required ratio = 135 : 245 = 36 km


= 27 : 49. Distance between Anand Jn. to
PS

31. Ans. (3) Nadiad Jn.= 440 – 422


Sol. Distance from origin to Surat = 18 km
= 257 km Distance between Nadiad Jn. to
Distance from origin to Nadiad Jn. Ahmedabad = 486 – 440
U

= 440 km = 46 km
Then, total distance from Surat to Distance between Ahmedabad to Bhuj
@

Nadiad Jn. = 977 – 486


= 440 – 257 = 491 km
= 183 km. The is second lowest distance is
32. Ans. (5) between Anand Jn. to Vadodara.
Sol. Departure time of the train at Anand 36. Ans. (5)
Jn. (including the halt time) Sol. Maximum temperature of Ontario on
= 6.45 a.m.
Ist November
Arrival time of the train at Ahmedabad
= 4 °C
= 8.00 a.m.
Minimum temperature of Bhuj on Ist
∴ Total time taken by the train from
January
Anand junction to Ahmedabad
= –7 °C
= 8.00 a.m. – 6.45 a.m.
∴ Required difference = [4 – (–7)]°C
= 1 hour 15 minutes.
= 11 °C.
692 ✫ Quantitative Aptitude

37. Ans. (1) Number of petrol cars in State-4


Sol. According to the table, 1
The maximum temperature of Kabul = 182 ×
1+ 1
is the second highest on Ist October 1
= 37 °C = 182 × = 91
2
The minimum temperature of Sydney ∴ Difference between the number of
is the highest on Ist January diesel engine cars in State-2 and
= –13°C. the number of petrol engine cars in
38. Ans. (3) State-4

TS
Sol. Difference of temperature in Bhuj on = 91 – 70
Ist September = (24 – 14)°C = 10°C = 21.
Difference of temperature in Bhuj on 42. Ans. (1)
Ist October = (35 – 21)°C = 14°C Sol. Total cars = 70

H
Difference of temperature in Bhuj on The number of cars in State-3
Ist November = (19 – 8)°C = 11°C 32

G
Difference of temperature in Bhuj on = 700 × = 224
100
Ist December = (9 – 2)°C = 7°C The number of petrol engine cars in
Difference of temperature in Bhuj on
Ist January = [–7 – (–4)]°C
= (–7 + 4)°C = –3°C U State-3
= 224 ×
3
5+3
O
Hence, the second highest difference
in temperature is on Ist November. 3
= 224 × = 84
H
39. Ans. (5) 8
Sol. The average maximum temperature The number of cars in State-1
_T

of Beijing over all the months together 14


= 700 × = 98
12 + 9 + 15 + 2 + 5 100
= °C The number of diesel engine cars in
5
43 State-1
C

= °C = 8.6°C.
5 3
40. Ans. (2) = 98 ×
3+ 4
PS

Sol. The minimum temperature of Beijing 3


on Ist September = 9°C = 98 × = 42
7
The maximum temperature of Ontario
∴ Required percentage
U

on Ist October = 15°C


84 − 42
∴ Required ratio = 9 : 15 = × 100
= 3 : 5. 42
@

42 × 100
41. Ans. (2) = = 100%.
Sol. Total cars = 700 42
Number of cars in State–2 43. Ans. (4)
28 Sol. Total cars = 700
= 700 × = 196 Number of cars in State-3
100 32
Number of diesel cars in State-2 = 700 ×
5 = 224 100
= 196 ×
5+9 Number of diesel engine cars in
= 196 × 5 = 70 State-3
14 5
Number of cars in State-4 = 224 ×
5+3
26 5
= 700 × = 224 × = 140
= 182 100 8
Solved Test Papers of Various Examinations ✫ 693

Number of diesel engine cars which Total number of cars in State-4


26
25 = 700 × = 182
are AC = 140 × = 35 100
100 Number of petrol engine cars in
Number of diesel engine cars which
State-4
are non-AC
1
= 140 – 35 = 105. = 182 ×
1+ 1
44. Ans. (5) 1
Sol. Total cars = 700 = 182 × = 91
2
Total number of cars in State-3 ∴ Average number of petrol engine cars

TS
32 in all the States together
= 700 × = 224
100 56 + 126 + 84 + 91
Total number of cars in State-2 =
28 4
= 700 × = 196

H
357
100 = = 89.25.
Number of petrol engine cars in 4

G
State-2 46. Ans. (3)
9 Sol. Total number of balls = 7 + 5 = 12
= 196 ×
5+9 Let S be the sample space. Then,
= 196 ×
9
14
= 126
U n(S) = Number of ways of drawing
2 balls out of 12
O
∴ Required difference = 224 – 126 = 98. = 12C2
45. Ans. (2) 12
H
=
Sol. Total cars = 700 12 − 2. 2
Total number of cars in State-1
12 × 11 × 10 12 × 11
_T

14 = = = 66
= 700 × = 98
100 10. 2 2×1
Number of petrol engine cars in
Let E = Event of drawing 5 balls,
State-1
none of which is yellow
C

4
= 98 × ∴ n(E) = Number of ways of drawing
3+ 4
5 balls out of 7 balls
PS

4
= 98 × = 56 7
7 = 7C5 =
Total number of cars in State-2 7 − 5. 5
28
= 700 × = 196 7×6× 5
U

100 =
Number of petrol engine cars in 2. 5
State-2
@

7 ×6
9 = = 21
= 196 × 2 ×1
5+9
9 n(E)
= 196 × = 126 ∴ P(E) =
14 n(S)
Total number of cars in State-3 21 7
32 = = .
= 700 × = 224 66 22
100 47. Ans. (5)
Number of petrol engine cars in
Sol. We know that in two throws of a die,
State-3
3 n(S) = 6 × 6 = 36
= 224 × Let E = event of getting a total of 7
5+3 = {(1, 6), (2, 5), (3, 4), (4, 3),
3
= 224 × = 84 (5, 2), (6, 1)}
8
n(E) = 6
694 ✫ Quantitative Aptitude

n(E) 48. Ans. (3)


P(E) = Sol. Number of females who are non-
n(S)
6 1 graduates = 20
= = . Number of males who are graduates
36 6
= 24
For Qs. 48 to 50: ∴ Required difference = 24 – 20 = 4.
Total number of members (males and 49. Ans. (5)
females) = 240 2 Sol. Number of females who are graduates
Number of males = 240 × = 160 = 60
3

TS
Number of males who are graduates Number of males who are non-
15 graduates = 136
= 160 × = 24 ∴ Total Sum = 60 + 136 = 196.
100
Number of males who are non-graduates 50. Ans. (2)

H
= 160 – 24 = 136 Sol. Total number of males = 160
Number of females = 240 – 160 = 80 Number of females who are non-

G
Number of females who are graduates graduates = 20
3 ∴ Ratio of the total number of males to
= 80 × = 60
4
Number of females who are non-
graduates = 80 – 60 = 20. U the number of females who are non-
graduates = 160 : 20
= 8 : 1.
O
H
C _T
PS
U
@
Solved Test Papers of Various Examinations ✫ 695

Reserve Bank of India


Grade B Officers Examination, August 2013
Directions (Qs. 1–5): What value will come in place of Directions (Qs. 8–10): What will come in place of the
the question mark (?) in the following questions. question mark (?) in the following number series?
1 8. 224 576 752 840 884 ?
1. 7072 ÷ (16% of 884) = 30 × 1 of (? ÷ 39)
12 (A) 960 (B) 890
(A) 60 (B) 62 (C) 906 (D) 908
(C) 65 (D) 55 (E) None of these

TS
(E) 67
9. 55 66.15 88.45 121.9 166.5 ?
2. (562.5 × 6)6 ÷ (135 ÷ 9)10 ÷ (37.5 × 6)7 = (3.75 × 4)?–6 (A) 212.25 (B) 322.25
(A) 0 (B) 2 (C) 224.25 (D) 222.25

H
(C) 3 (D) 4 (E) None of these
(E) 5 10. 36 49 75 88 114 (?)

G
3. 1478.4 ÷ 56 + 66.8 × 57 = (? × 3) + (34 × 34.5) (A) 130 (B) 140
(C) 132 (D) 128
(A) 785 (B) 887
(C) 889
(E) 885
(D) 989
U (E) 127
Directions (Qs. 11–15): In the following graph the
O
( ) number of laptops manufactured by six different
2
4. 13 + 2 5 = ? 5 + 189 companies in the years 2010 and 2011 has been given.
(A) 26 (B) 25 Read the graph carefully and answer the questions.
H
(C) 52 (D) 130 Number of laptops (in thousands)
(E) None of these manufactured by 6 different
_T

5. 0.2 × 1.1 + 0.6 × 0.009 = ? – 313.06 companies


(A) 353.2184 2011 2010
Samsung
(B) 353.2854
C

(C) 331.54 40
(D) 313.2854
PS

35
(E) 331.2854 30
Dell Apple
6. Train A crosses a pole and platform in 18 25
seconds and 39 seconds respectively. The 20
U

length of the platform is 157.5 metre. What 15


will be the length of train B if it is equal to 10
@

the sum of half of the length of train A and 0

twice the length of the platform? 0


5
10 5
(A) 382.5 metre (B) 328.5 metre 15
20 10
(C) 238.5 metre (D) 315 metre 25
30 15
(E) None of these 35 HP
Le

20
40
no

7. If 12 boys or 15 girls can do a work in 48 days,


vo

25
in what time will 24 boys and 6 girls do 30
twice the work? 35
(A) 42 days 40
(B) 40 days Abascus
(C) 45 days 11. The respective ratio between the number of
(D) 30 days laptops manufactured by Lenovo in 2010 and
(E) None of these that by Abascus in 2011 is
696 ✫ Quantitative Aptitude

(A) 8 : 7 (B) 7 : 8 at random. What is the probability of their


(C) 3 : 5 (D) 5 : 3 being 2 brown balls, 1 orange ball and 2 black
(E) None of these balls?
12. What is the average number of laptops (in 191 180
(A) (B)
thousands) manufactured by all companies 1547 1547
taken together in 2010? 280 189
(A) 22 (B) 22.5 (C) (D)
1547 1547
(C) 32.5 (D) 23.5
(E) 27.5 (E) None of these
18. The ratio of the salaries of A and B is 8 : 9. If

TS
13. What is the percentage increase in
production of laptops by HP in 2011 in A’s salary is increased by 50% and B’s salary
comparison to that in 2010? is reduced by 25%, their ratio becomes
(A) 125 (B) 100 16 : 9. What is the salary of A?

H
(C) 150 (D) 250 (A) Rs 22000
(E) None of these (B) Rs 28500
(C) Rs 37000

G
14. The difference between the number of
(D) Cannot be determined
laptops manufactured by Apple, Lenovo
(E) None of these
and Samsung in 2010 and that by Dell, HP
and Abascus in 2011 is
(A) 5500 (B) 4550 U 19. If tax on a commodity is reduced by 10%,
total revenue remains unchanged. What is
O
(C) 3550 (D) 4500 the percentage increase in its consumption?
(E) 5000 1
H
(A) 11 % (B) 20%
15. In 2011, which company manufactured the 9
1
maximum number of laptops? (C) 10% (D) 9 %
11
_T

(A) Abascus (B) Lenovo (E) None of these


(C) Dell (D) Samsung
(E) HP 20. The denominators of two fractions are 5 and
7 respectively. The sum of these fractions is
16. Sujata scored 2240 marks in an examination
C

41
that is 128 marks more than the minimum . On interchanging the numerators, their
passing percentage of 64%. What is the 35
PS

43
percentage of marks obtained by Meena if sum becomes . The fractions are
she scores 907 marks less than Sujata? 35
(A) 35 (B) 40 2 4 3 4
(A) and (B) and
U

(C) 45 (D) 36 5 7 5 7
(E) 48 4 2 3 5
(C) and (D) and
@

17. There are 8 brown balls, 4 orange balls and 5 7 5 7


5 black balls in a bag. Five balls are chosen (E) None of these

Directions (Qs. 21–25): Read the following table carefully to answer the given questions.
Number of Students enrolled in different academic faculties of 4 institutions during 2008-2012
College DAV College CT College BKM College VP College
Year B.Ed M.Ed M.Sc B.Ed M.Ed M.Sc B.Ed M.Ed M.Sc B.Ed M.Ed M.Sc
2008 330 156 675 543 457 884 1090 699 574 657 633 674
2009 534 344 1145 234 554 684 126 243 743 478 744 1185
2010 222 429 454 444 332 393 1133 723 1223 434 835 1349
2011 465 434 810 665 443 790 855 621 2374 1332 964 2234
2012 250 242 666 877 332 534 334 503 1235 334 1098 1534
Solved Test Papers of Various Examinations ✫ 697

21. What is the average number of enrolled Marks: obtained by five students in
students in B.Ed course during 2009? different subjects—Maths, English and
(A) 343 Science—in an examination
(B) 334
Maths English Science
(C) 336 100
(D) 338 9088 92

Marks obtained by students


90
(E) 340 80 78 80 78
72 74
22. How many students got admission in M.Sc 70 68 68
64
in DAV college taking all the years together? 60 55

TS
52
(A) 3850 50 44
42
40
(B) 3750
30
(C) 3650

H
20
(D) 3950 10
(E) 4080 0

G Rajeev

Narendra

Anita

Veena

Ramesh
23. The difference between the number of
students enrolled in CT college in 2010 and
that enrolled in DAV college in 2011 is
(A) 565 U Students
O
(B) 560 26. Which student scored the maximum marks in
(C) 540 all the subjects taken together?
H
(D) 545 (A) Rajeev (B) Narendra
(E) 580 (C) Amit (D) Veena
(E) Ramesh
_T

24. By what per cent the number of students


27. The difference between the average marks
enrolled in M.Ed, in BKM College in 2009 is obtained by Narendra and Veena is
more than that enrolled in B.Ed in the same (A) 5 (B) 3
C

college in the same year? (C) 4 (D) 6


(A) 87 (E) 7
PS

(B) 88 28. If the maximum marks of each subject are


(C) 89 100, what is the approximate percentage of
marks got by Ramesh?
(D) 93 (A) 67 (B) 66
U

(E) None of these (C) 64 (D) 69


25. The ratio between the number of students (E) 72
@

enrolled in B.Ed and M.Sc in DAV college in 29. By what percentage, the marks obtained by
Amit in Science is less then his marks in
2011 is
Maths?
(A) 31 : 54 (A) 21 (B) 24
(B) 54 : 31 (C) 22 (D) 15
(C) 52 : 27 (E) 18
(D) 27 : 52 30. The ratio of marks obtained by all students
(E) None of these in Maths and Science is respectively
(A) 89 : 91 (B) 23 : 25
Directions (Qs. 26–30): Read the following bar diagram (C) 25 : 23 (D) 93 : 89
carefully to answer the questions given below. (E) None of these
698 ✫ Quantitative Aptitude

Answers and Explanatory Notes


1. Ans. (A) 6. Ans. (A)
1 Sol. Length of train A = x metre
Sol. 7072 ÷ (16% of 884) = 30 × 1 of (? ÷ 39)
12 x x + 157.5
16 13 ∴ =
7072 ÷ 884 × = 30 × × (? ÷ 39) 18 39
100 20 ⇒ 39x = 18x + 2835
390 ? ⇒ 39x – 18x = 2835
⇒ 7072 ÷ 141.44 = ×
12 39 ⇒ 21x = 2835
10

TS
⇒ 50 = ×? 2835
12 x =
21
⇒ 10 × ? = 50 × 12 x = 135
50 × 12 Length of train A is 135 m.

H
? = Then,
10
∴ ? = 60. Length of train B

G
2. Ans. (A) 135
= + 2 × 157.5 m
Sol. (562.5 × 6)6 ÷ (135 ÷ 9)10 ÷ (37.5 × 6)7 2
= (3.75 × 4)?–6
⇒ (3375) ÷ (15) ÷ (225) = (15)?–6

6 10 7

(153)6 ÷ (15)10 ÷ (152)7 = (15)?-6 U = (67.5 + 315) m


= 382.5 m
O
Hence, length of train B is 382.5 m.
⇒ (15)3×6 ÷ (15)10 ÷ (15)2×7= (15)?-6
7. Ans. (B)
⇒ (15)18 ÷ (15)10 ÷ (15)14 = (15)?-6
H
Sol. 12 boys = 15 girls
18 – 10 – 14 = ? – 6
Then,
–6 =?–6
_T

24 boys = 30 girls
? =6–6
24 boys + 6 girls = 30 girls + 6 girls
∴ ? = 0.
= 36 girls
3. Ans. (B)
Here, M1 = 15 girls, M2 = 36 girls,
C

Sol. 1478.4 ÷ 56 + 66.8 × 57


= (? × 3) + (34 × 34.5) work (W1) = 1, W2 = 2,
PS

⇒ 26.4 + 3807.6 = (? × 3) + 1173 D1 = 48 days, D2 = ?


⇒ 3834 = (? × 3) + 1173
M1D1 M2 D2
⇒ 3 × ? = 3834 – 1173 =
⇒ W1 W2
U

3 × ? = 2661
2661 15 × 48 36 × D2
? = ⇒ =
3 1 2
@

∴ ? = 887. ⇒ 15 × 48 × 2 = 36 × 1 × D2
4. Ans. (C) 15 × 48 × 2
( )
2
Sol. 13 + 2 5 = ? 5 + 189 D2 =
36
169 + 20 + 52 5 = ? 5 + 189 D2 = 5 × 8 = 40 days.
189 + 52 5 = ? 5 + 189 8. Ans. (C)
? 5 = 189 + 52 5 – 189 Sol. 224 576 752 840 884 ?
? 5 = 52 5 Clearly, the given sequence follows the
∴ ? = 52. pattern : + 352, + 176, + 88, + 44, + 22
5. Ans. (D) Then,
Sol. 0.2 × 1.1 + 0.6 × 0.009 = ? – 313.06 224 + 352 = 576, 576 + 176 = 752,
⇒ 0.22 + 0.0054 = ? – 313.06 752 + 88 = 840, 840 + 44 = 884,
⇒ 0.2254 = ? – 313.06 = 884 + 22 = ?
? = 0.2254 + 313.06
∴ ? = 313.2854. ∴ ? = 906.
Solved Test Papers of Various Examinations ✫ 699

9. Ans. (D) 25 − 10 1500


= × 100 =
Sol. 55 66.15 88.45 121.9 166.5 ? 10 10
The given sequence follows the pattern: = 150.
+ 1 × 11.15, + 2 × 11.15, + 3 × 11.15, + 4 14. Ans. (E)
× 11.15, + 5 × 11.15 Sol. Total number of laptops manufactured
Then, by Apple, Lenovo and Samsung in the
55 + 1 × 11.15 = 55 + 11.15 = 66.15, year 2010
66.15 + 2 × 11.15 = 66.15 + 22.3 = 88.45, = (15 + 40 + 25) thousand
88.45 + 3 × 11.15 = 88.45 + 33.45 = 121.9, = 80 thousand

TS
121.9 + 4 × 11.15 = 121.9 + 44.6 = 166.5, Total number of laptops manufactured
? = 166.5 + 5 × 11.15 by Dell, HP and Abascus in the year
? = 166.5 + 55.75
2011
? = 222.25.
= (15 + 25 + 35) thousand

H
10. Ans. (E)
Sol. 36 49 75 88 114 ? = 75 thousand
∴ Required difference

G
The given sequence follows the pattern:
+ 1 × 13, + 2 × 13, + 1 × 13, + 2 × 13, + 1 = (80 – 75) thousand
× 13 = 5 thousand
Then,
36 + 1 × 13 = 36 + 13 = 49, U 15. Ans. (A)
= 5000.
O
49 + 2 × 13 = 49 + 26 = 75, Sol. From the graph, the company Abascus
75 + 1 × 13 = 75 + 13 = 88, manufactured the maximum number
H
88 + 2 × 13 = 88 + 26 = 114, of laptops in the year 2011.
? = 114 + 1 × 13 16. Ans. (B)
? = 114 + 13 = 127.
_T

Sol. Let the total maximum marks be x.


11. Ans. (A)
Sol. Number of laptops manufactured by Then,
Lenovo company in the year 2010 x × 64
= 2240 – 128
C

= 40 thousand 100
Number of laptops manufactured by x × 64
⇒ = 2112
PS

Abascus company in the year 2011 100


= 35 thousand x × 64 = 2112 × 100
∴ Required ratio = 40 : 35
2112 × 100
= 8 : 7. x =
U

12. Ans. (B) 64


Sol. Number of laptops manufactured by x = 3300
Marks obtained by Sujata
@

all companies in the year 2010


= (25 + 15 + 10 + 15 + 40 + 30) thousand = 2240 – 907
= 135 thousand = 1333
135
∴ Required average = thousand ∴ Required percentage
6
1333
= 22.5 thousand. = × 100
3300
13. Ans. (C)
Sol. Number of laptops manufactured by = 40.39%
HP company in the year 2010 = 40% (approximate).
= 10 thousand 17. Ans. (C)
Number of laptops manufactured by Sol. Total no. of balls = 8 + 4 + 5
HP company in the year 2011 = 17 balls
= 25 thousand Number of ways of drawing 5 balls out
∴ Required percentage increase of 17
700 ✫ Quantitative Aptitude

n(S) = 17C5 10
= × 100 %
17 × 16 × 15 × 14 × 13 (100 − 10)
=
1× 2 ×3 × 4 ×5 10
= 6188 = × 100 %
90
Number of ways of drawing 2 brown
100 1
balls out of 8, 1 orange ball out of 4 and = % = 11 % .
2 black balls out of 5. 9 9
20. Ans. (B)
n(E) = 8C2 × 4C1 × 5C2 Sol. Let the numerator of two fractions be x
8 4 5 and y

TS
= × × x y 41
2. 8 − 2 1. 4 − 1 2. 5 − 2 + = ... (i)
8×7× 6 4× 3 5 × 4× 3 5 7 35
= × × Interchanging the numerators,
2 × 1× 6 1× 3 2 × 1× 3 y x 43

H
= 28 × 4 × 10 + = ... (ii)
5 7 35
= 1120 n(E) From equation (i),

G
∴ Required probability P(E) =
n(S) x y 41
+ =
1120 35
=
6188
280 U 5 7
7x + 5y
35
=
41
35
O
= . 7x + 5y = 41 ... (iii)
1547
18. Ans. (D) From equation (ii),
H
Sol. The ratio of the salaries of A and B y x 43
+ =
=8:9 5 7 35
_T

Let A’s salary = Rs 8x and 7 y + 5x 43


=
B’s salary = Rs 9x 35 35
100 + 50 7y + 5x = 43
8x × or 5x + 7y = 43 ... (iv)
C

100 16
= Multiplying by 7 in equation (iii) and
100 − 25 9
9x × by 5 in equation (iv), and then
PS

100
subtracting, we get
150
8x ×
⇒ 100 = 16 49x + 35y = 287
75 9 25x + 35y = 215
U

9x ×
100 24x = 72
8x × 150 16 x=3
@

⇒ =
9x × 75 9 Putting the value of x in equation (iii),
16 x 16 21 + 5y = 41
⇒ =
9x 9 5y = 41 – 21
16 16 5y = 20
⇒ =
9 9 y= 4
Hence, A’s salary cannot be determined. 3 4
∴ The fractions are and .
5 7
19. Ans. (A) 21. Ans. (A)
Sol. Commodity decrease R = 10 Sol. Average number of enrolled students
Increase percentage in consumption in B.Ed course during 2009
534 + 234 + 126 + 478
R =
× 100 % 4
=
(100 − R) 1372
= = 343
4
Solved Test Papers of Various Examinations ✫ 701

22. Ans. (B) ∴ The maximum marks have been scored


Sol. Total number of students who got by Amit.
admission in M.Sc in DAV college in
all the years together 27. Ans. (B)
= 675 + 1145 + 454 + 810 + 666 Sol. Average marks obtained by Narendra
= 3750. 44 + 68 + 80 192
= =
23. Ans. (C) 3 3
Sol. Total number of students enrolled in = 64
CT college in 2010 Average marks obtained by Veena
= 444 + 332 + 393 68 + 55 + 78 201

TS
= 1169 = =
3 3
Total number of students enrolled in = 67
DAV college in 2011 ∴ Required difference = 67 – 64

H
= 465 + 434 + 810 = 3.
= 1709 28. Ans. (D)
∴ Required difference = 1709 – 1169

G
Sol. Percentage of marks obtained by
= 540. Ramesh
24. Ans. (D) 92 + 64 + 52
Sol. Number of students enrolled in M.Ed
in BKM college in 2009 = 243 U =
100 + 100 + 100
208
× 100
O
Number of students enrolled in B.Ed = × 100
in BKM college in 2009 = 126 300
∴ Required percentage
H
= 69.33 = 69 (approximate).
243 − 126 29. Ans. (E)
= × 100
_T

126 Sol. Marks obtained by Amit in Science


117 = 74
= × 100
126 Marks obtained by Amit in Maths
= 92.85 = 93 (approximate). = 90 90 − 74
C

25. Ans. (A) ∴ Required percentage = × 100


90
Sol. Number of students enrolled in B.Ed 16
PS

= × 100
in DAV college in 2011 = 465 90
Number of students enrolled in M.Sc in = 17.77 = 18.
DAV college in 2011 = 810 30. Ans. (D)
U

∴ Requred ratio = 465 : 810 Sol. Marks obtained by all students in Maths
= 31 : 54. = 78 + 44 + 90 + 68 + 92
26. Ans. (C) = 372
@

Sol. Marks obtained in all subjects by Marks obtained by all students in


Rajeev = 78 + 42 + 72 = 192 Science
Narendra = 44 + 68 + 80 = 192 = 72 + 80 + 74 + 78 + 52
Amit = 90 + 88 + 74 = 252 = 356
Veena = 68 + 55 + 78 = 201 ∴ Required ratio = 372 : 356
Ramesh = 92 + 64 + 52 = 208 = 93 : 89.
702 ✫ Quantitative Aptitude

Reserve Bank of India


Assistants Examination, July 2013
1. What will come in place of the question mark 7. (1600 ÷ 1000)6 ÷ (0.256 × 10)7 × (1.024 × 4)3
(?) in the following question? = (1.6)? – 9
11 5 (A) 8 (B) 9
(?) 2 648 × (?) 2 (C) 10 (D) 7
=
72 8 (E) None of these

TS
(A) 18 (B) 24
8. (? ÷ 25) ÷ 0.16 = 1935 ÷ 9
(C) 12 (D) 16
(A) 560 (B) 1860
(E) None of these
(C) 760 (D) 860
Directions (Qs. 2–3): Read the following information (E) None of these

H
to answer the questions.
9. 96 × 2117 ÷ 73 = (? – 198) × 32
There are 650 candidates from five different
(A) 285 (B) 385

G
states to participate in a competition. From
(C) 185 (D) 825
state 1, the number of candidates is 12% of the
(E) None of these
total candidates. From state 2 there are one-fifth
of the total candidates. There are 8% of the total
candidates from state 3. The number of U 10. 57 × 9.5 − (63 × 11.5) + 1408 = ?
O
(A) 25 (B) 35
candidates from state 4 and state 5 is equal. (C) 45 (D) 55
2. How many candidates participated from (E) None of these
H
state 4? 11. 565 – 469.3 ÷ 19 × 10 = ?
(A) 390 (B) 195 (A) 381 (B) 391
_T

(C) 78 (D) 187 (C) 325 (D) 324


(E) None of these (E) None of these
3. What is the ratio between the number of 12. ?% of 5450 – 12% of 750 = 1654
C

candidates from state 2 and state 3? (A) 32 (B) 34


(A) 3 : 5 (B) 2 : 5 (C) 36 (D) 23
PS

(C) 5 : 2 (D) 5 : 3 (E) None of these


(E) None of these
13. (?)2 % of 108 = (17)2 + 386
Directions (Qs. 4–28): What will come in place of the (A) 24 (B) 25
question mark (?) in the following questions?
U

(C) 15 (D) 35
4. 76% of 567 ÷ 4 = ? × 7 (E) None of these
(A) 16.93 (B) 16.39
@

14. 19.8% of 1750 + 6% of 150 = ? + 276.8


(C) 15.93 (D) 15.39 (A) 78.9 (B) 87.7
(E) None of these (C) 78.7 (D) 87.9
5. 12985 – 2378 + 12121 – 6776 = ? + 9988 (E) None of these
(A) 5964 (B) 5694 15. 675.5 + (9.4)2 – 126.8 = (?)2 – 38.94
(C) 6594 (D) 9564 (A) 32 (B) 36
(E) None of these (C) 24 (D) 26
5 5 1
6. 1 × 1 ÷1 = ? (E) None of these
12 34 8
1 4 21 7
16. of of ? = 2432 – 1109
(A) 1 (B) 1 37 15
3 9
5 5 (A) 5995 (B) 3595
(C) 1 (D) 2 (C) 4595 (D) 3995
9 9
(E) None of these (E) 4995
Solved Test Papers of Various Examinations ✫ 703

17. 4326.73 – 2332.52 + 765.91 = ? + 2494.75


(A) 265.37 (B) 265.73
27. (292.41) × 42.25 = ? + 121.98
(C) 256.37 (D) 256.73 (A) 10.38 (B) –10.38
(E) None of these (C) 10.83 (D) –10.83
4 5 7 1 (E) None of these
18. 2 – 1 – 1 =?–1
5 8 10 40 28. 148 ÷ 1369 + 142 = 6 × (?)2 – 70
1 1
(A) (B) (A) 6 (B) 8
5 2
(C) 4 (D) 9
1 11
(E) None of these

TS
(C) (D)
8 40
13 Directions (Qs. 29–33): In each of these questions an
(E) equation is given with a question mark (?) in place of a
40
correct symbol. Based on the values on the right hand

H
19. (2444 ÷ 94) ÷ 2 = ? side and the left hand side of the question mark, you
(A) 225 (B) 144 have to decide which of the following symbols will come

G
(C) 169 (D) 189 in place of the question mark.
(E) None of these Give answer If in place of question
20. 23 – (23)2 + 23 × (23 ÷ 0.23 + 23) = ?
(A) 2442 (B) 2424 U mark (?) the following
will come
O
(C) 3232 (D) 2323 (A) > (greater than)
(E) None of these (B) = (equal to)
H
1
(C) < (lesser than)
21. 2184 ÷ 39 ÷ 4 – 67= (?) 2 – 109
(D) ≥ (either greater than
(A) 3136 (B) 3316
_T

or equal to)
(C) 6316 (D) 3116
(E) None of these (E) ≤ (either lesser than
or equal to)
22. [(27)2 + (6)2 + (11)2] – (17 × 24) = (?)3 + 135
C

(A) 17 (B) 7 29. [(1678 – 654) + (399 ÷ 21)] ?


(C) 9 (D) 8 [(167 – 98) × 8 – 509]
PS

(E) None of these


2 2
30. {546 – (4) 2
}
÷8 ? [(15)2 × 8 – 1734.55)]
23. 7569 ÷ 12 × 104 = (?) + (23) 1
31. ± (56 + 87) + (12) 2 + 2 2 ? (34% of 50)
(A) 25 (B) 8
U

(C) 16 (D) 15 1

(E) None of these 32. ± (15 × 4) + (30)2 +l 2 ? 1764 − 11


@

(3 ) ( )
2 1
24. 5 + 6 = ?+36 5 + 59 33. 5776 − 169 ? ± (5554 − 1585) 2

(A) 22 (B) 24
(C) 26 (D) 12 Directions (Qs. 34–35): What should come in place of
(E) None of these the question mark (?) in the following number series?
25. 873.53 + 532.32 – ? = 535.1 + 232.95 34. 43 55 199 209 309 (?)
(A) 673.8 (B) 637.8 (A) 319 (B) 349
(C) 367.8 (D) 763.8 (C) 317 (D) 320
(E) None of these (E) None of these
26. (58)2 – (4)3 × 486 ÷ 18 = (?)2 + 547 35. 65 141.8 180.2 199.4 209 (?)
(A) 43 (B) 23 (A) 225.8 (B) 213.8
(C) 33 (D) 53 (C) 228.8 (D) 221.8
(E) None of these (E) None of these
704 ✫ Quantitative Aptitude

36. The area of a square is 9604 sq.cm. The Directions (Qs. 39–40): Read the following
diameter of a circle is equal to half of the side information to answer the questions.
of the square. What will be the circumference Two unbiased dice are thrown simul-
of the circle? taneously.
(A) 144 cm (B) 164 cm 39. What will be the probability that sum of the
(C) 308 cm (D) 154 cm numbers on the two faces is a composite
(E) None of these number?
1 7 5
37. What will be the value of 26% of of the (A) (B)
15

TS
12 12
sum of the five times of 45 and 2 times of 60?
(A) 5.98 (B) 6.98 3 1
(C) 3.98 (D) 5.89 (C) (D)
4 2
(E) None of these

H
(E) None of these
38. The sum of the heights of Gita and Rita is
equal to 160 cm. The height of Sita is equal 40. What is the probability of getting a doublet?

G
to three times the height of Rita. Rita’s height 1 1
is 39 cm. What will be the height of Gita, Sita (A) (B)
3 6
and Rita altogether?
(A) 257 cm (B) 277 cm
U (C)
1
(D)
2
O
(C) 267 cm (D) 287 cm 4 3
(E) None of these (E) None of these
H
Answers and Explanatory Notes
1. Ans. (A) From state 4 and state 5, number of
_T

11 5 candidates
(?)
2 648 × (?) 2
= 650 – 130 – 78 – 52
Sol. =
72 8 = 390
5
C

11 Then, from state 4, number of candidates


8 × (?) 2 = 72 × 648 × (?) 2 390
= = 195.
PS

11
72 × 648 2
(?) 2 3. Ans. (C)
5 =
8 Sol. From state 2, number of candidates
(?) 2
11

5 1
U

(?) 2 2 = 9 × 648 = 650 ×


6 5
(?) = 58322 = 130
@

(?)3 = (18)3 From state 3, number of candidates


8
∴ ? = 18. = 650 ×
100
2. Ans. (B) = 52
Sol.From state 1, number of candidates ∴ Ratio between the number of candidates
12 from state 2 and state 3
= 650 ×
= 78 100 = 130 : 52
From state 2, number of candidates = 5 : 2.
1 4. Ans. (D)
= 650 ×
5 Sol. 76% of 567 ÷ 4 = ? × 7
= 130
From state 3, number of candidates 76
567 × ÷4 =?×7
8 100
= 650 × 76 1
= 52
100 567 × × =?×7
100 4
Solved Test Papers of Various Examinations ✫ 705

567 × 19 9. Ans. (A)


=?×7 Sol.96 × 2117 ÷ 73 = (? – 198) × 32
100
567 × 19 96 × 2117
? = = (? – 198) × 32
100 × 7 73
81 × 19 1539 96 × 29 = (? – 198) × 32
? = = 96 × 29
100 100 ? – 198 =
∴ ? = 15.39. 32
? – 198 = 3 × 29
5. Ans. (A)
? – 198 = 87

TS
Sol. 12985 – 2378 + 12121 – 6776 = ? + 9988
? = 87 + 198
? = 12985 – 2378 + 12121 – 6776 – 9988
∴ ? = 285.
= 25106 – 19142
= 5964. 10. Ans. (B)

H
6. Ans. (B) Sol. ? = 57 × 9.5 − (63 × 11.5) + 1408
5 5 1 541.5 − (724.5) + 1408
Sol. ? = 1 × 1 ÷ 1 ? =

G
12 34 8
17 39 9 = 541.5 + 683.5
= × ÷

=
12 34
1 13
× ÷
9
8
U = 1225
∴ ? = 35.
O
4 2 8 11. Ans. (E)
13 8 Sol. ? = 565 – 469.3 ÷ 19 × 10
= ×
H
8 9 = 565 – 24.7 × 10
13 = 565 – 247
= ∴ ? = 318.
_T

9
4 12. Ans. (A)
∴ ? = 1 .
9 Sol.?% of 5450 – 12% of 750 = 1654
7. Ans. (C)
C

? 12
Sol.(1600 ÷ 1000)6 ÷ (0.256 × 10)7 × (1.024 × 4)3 5450 × – 750 × = 1654
100 100
= (1.6)?–9 ?
PS

(1.6)6 ÷ (1.62)7 × (4.096)3 = (1.6)?–9 5450 × – 90 = 1654


100
(1.6)6 ÷ (1.6)2×7 × (1.63)3 = (1.6)?–9 ?
5450 × = 1654 + 90
(1.6)?–9 = (1.6)6 ÷ (1.6)14 × (1.6)3×3 100
U

(1.6)?–9 = (1.6)6 ÷ (1.6)14 × (1.6)9 ?


5450 × = 1744
? – 9 = 6 – 14 + 9 100
1744 × 100
@

?–9 =1 ? =
? =1+9 5450
∴ ? = 32.
∴ ? = 10.
13. Ans. (B)
8. Ans. (D) Sol.(?)2 % of 108 = (17)2 + 386
Sol.(? ÷ 25) ÷ 0.16 = 1935 ÷ 9 (?)2
108 × = 289 + 386
(? ÷ 25) ÷ 0.16 = 215 100
2
? (?)
= 215 108 × = 675
25 × 0.16 100
? 675 × 100
= 215 (?)2 =
4 108
? = 215 × 4 (?)2 = 625
∴ ? = 860. (?)2 = (25)2
∴ ? = 25.
706 ✫ Quantitative Aptitude

14. Ans. (C) 19. Ans. (C)


Sol. 19.8% of 1750 + 6% of 150 = ? + 276.8
Sol. ? = (2444 ÷ 94) ÷ 2
19.8 6
1750 × + 150 × = ? + 276.8 2444
100 100 =
94 × 2
346.50 + 9 = ? + 276.8
? = 13
355.50 = ? + 276.8
Squaring both sides,
? = 355.50 – 276.8
( ?)2 = (13)2
∴ ? = 78.7.
∴ ? = 169.

TS
15. Ans. (D)
20. Ans. (D)
Sol.675.5 + (9.4)2 – 126.8 = (?)2 – 38.94 Sol. ? = 23 – (23)2 + 23 × (23 ÷ 0.23 + 23)
675.5 + 88.36 – 126.8 = (?)2 – 38.94
23
(?)2 = 675.5 + 88.36 – 126.8 + 38.94 = 23 – 529 + 23 × + 23

H
0.23
(?)2 = 802.8 – 126.8
= 23 – 529 + 23 × (100 + 23)
(?)2 = 676
= 23 – 529 + 23 × 123

G
(?)2 = (26)2 = 23 – 529 + 2829
∴ ? = 26. = 2852 – 529
16. Ans. (E)
21 7 U ∴ ? = 2323.
21. Ans. (A) 1
O
Sol. of of ? = 2432 – 1109
37 15 Sol.2184 ÷ 39 ÷ 4 – 67 = (?) 2 – 109
1
2184
7 21 – 67 = (?) 2 – 109
×
H
?× = 2432 – 1109 39 × 4 1
15 37 14 – 67 = (?) 12 – 109
7 21
× 14 – 67 + 109 = (?) 2
_T

?× = 1323
15 37 1

1323 × 15 × 37 (?) 2 = 123 – 67


1
?= (?) 2 = 56
7 × 21
C

? = 9 × 15 × 37 Squaring both 1
sides,
2
∴ ? = 4995. (? 2 )2 = (56)

PS

? = 3136.
17. Ans. (A)
22. Ans. (B)
Sol.4326.73 – 2332.52 + 765.91 = ? + 2494.75
Sol. (?)3 + 135 = [(27)2 + (6)2 + (11)2] – (17 × 24)
? = 4326.73 – 2332.52 + 765.91 – 2494.75 (?)3 + 135 = [729 + 36 + 121] – 408
U

? = 5092.64 – 4827.27 (?)3 = 886 – 408 – 135


∴ ? = 265.37. (?)3 = 886 – 543
@

18. Ans. (B) (?)3 = 343


(?)3 = (7)3
4 5 7 1 ∴ ? = 7.
Sol. 2 − 1 − 1 = ?− 1
5 8 10 40 23. Ans. (D)
14 13 17 41 Sol.(?)2 + (23)2= 7569 ÷ 12 × 104
− − = ?−
5 8 10 40 (?)2 + 529 = 87 × 87 ÷ 12 × 104
14 13 17 41 (?)2 = 87 ÷ 12 × 104 – 529
? = − − +
5 8 10 40 (?)2 = 7.25 × 104 – 529
112 − 65 − 68 + 41 (?)2 = 754 – 529
=
40 (?)2 = 225
20 1 (?)2 = (15)2
∴ ? = = .
40 2 ∴ ? = 15.
Solved Test Papers of Various Examinations ✫ 707

24. Ans. (A) 30. Ans. (A)


Sol.(3 5 + 6)2 = ? + 36 5 + 59 Sol.L.H.S. = [{546 – (4)2 ÷ 8}]
= {546 – 16 ÷ 8}]
45 + 36 + 2 × 3 5 × 6 = ? + 36 5 + 59
= [546 – 2]
81 + 36 5 = ? + 36 5 + 59 = 544
? = 81 + 36 5 – 36 5 – 59 R.H.S. = [(15)2 × 8 – 1734.55]
∴ ? = 22. = [225 × 8 – 1734.55]
= 1800 – 1734.55
25. Ans. (B)
= 65.45
Sol. 873.53 + 532.32 – ? = 535.1 + 232.95 ∴ L.H.S. > R.H.S.

TS
1405.85 – ? = 768.05
? = 1405.85 – 768.05 31. Ans. (E)
Sol. L.H.S. = ±[(56 + 87) + (12)2 + 2)]1/2
∴ ? = 637.8.
= ±[143 + 144 + 2] 1/2

H
26. Ans. (C) = ± [289] 1/2
Sol.(58)2 – (4)3 × 486 ÷ 18 = (?)2 + 547 = ±17

G
⇒ 3364 – 64 × 486 ÷ 18 = (?)2 + 547 R.H.S. = (34% of 50)
⇒ (?)2 + 547 = 3364 – 64 × 27 34
= 50 × = 17


(?)2 = 3364 – 1728 – 547
(?)2 = 3364 – 2275 U ∴ L.H.S. ≤ R.H.S.
100
O
⇒ (?)2 = 1089 32. Ans. (E)
⇒ ? = 1089 Sol.L.H.S. = ± [(15 × 4) + (30)2 + 1]1/2

H
? = 33. = ± [60 + 900 + 1]1/2
27. Ans. (D) = ± [961]1/2
_T

Sol. ? + 121.98 = (292.41) × 42.25 = ± 31


⇒ ? + 121.98 = 17.1 × 6.5 R.H.S. = 1764 − 11
⇒ ? + 121.98 = 111.15 = [42 – 11]
⇒ ? = 111.15 – 121.98 = 31
C

∴ ? = –10.83. ∴ L.H.S. ≤ R.H.S.


PS

28. Ans. (A) 33. Ans. (D)


+ 142 = 6 × (?)2 – 70
Sol.148 ÷ 1369
⇒ 6 × (?)2 – 70 = 148 ÷ 37 + 142
Sol.L.H.S. = ( 5776 − 169 )
⇒ 6 × (?)2 – 70 = 4 + 142 = [76 – 13]
U

⇒ 6 × (?)2 = 146 + 70 = 63
216 R.H.S. = ± [(5554 – 1585)]1/2
@

⇒ (?)2 = = ± [3969]1/2
6
⇒ (?)2 = 36 = ± 63
⇒ ? = 36
∴ L.H.S. ≥ R.H.S.
∴ ? = 6.
29. Ans. (C) 34. Ans. (C)
Sol.L.H.S. = [(1678 – 654) + (399 ÷ 21)] Sol. 43 55 199 209 309 ?
= [(1024) + (19)] The given sequence follows the pattern:
= [1024 + 19] = 1043 + 12, + 122, + 10, + 102, + 8
R.H.S. = [(167 – 98) × 8 – 509] Then,
= [(69) × 8 – 509] 43 + 12 = 55, 55 + (12)2 = 55 + 144 = 199,
= [552 – 509] 199 + 10 = 209, 209 + (10)2 = 209 + 100 = 309,
= 43 ? = 309 + 8
∴ L.H.S. < R.H.S. ∴ ? = 317.
708 ✫ Quantitative Aptitude

35. Ans. (B) 38. Ans. (B)


Sol. 65 141.8 180.2 199.4 209 ? Sol.Sum of heights of Gita and Rita = 160 cm
The given sequence follows the pattern: Rita’s height = 39 cm
76.8 38.4 19.2 9.6 Sita’s height = 3 × Rita’s height
+76.8, + ,+ ,+ ,+
2 2 2 2 = 3 × 39
or + 76.8, + 38.4, + 19.2, + 9.6, + 4.8 = 117 cm
Then, ∴ Height of Gita, Sita and Rita altogether
65 + 76.8 = 141.8, 141.8 + 38.4 = 180.2, = (160 + 117) cm
180.2 + 19.2 = 199.4, 199.4 + 9.6 = 209,
= 277 cm.

TS
? = 209 + 4.8
∴ ? = 213.8. 39. Ans. (A)
36. Ans. (D) Sol.Total possible outcomes n(S) = (6 × 6)
Sol. Area of square = 9604 sq. cm = 36

H
2
 Area of square = (side of square) Let E = Event that the sum is a prime
⇒ 9604 = (side of square) 2
number

G
⇒ Side of square = 9604 = {(1, 1), (1, 2), (1, 4), (1, 6), (2, 1), (2, 3),
Side of square = 98 cm (2, 5), (3, 2), (3, 4), (4, 1), (4, 3), (5, 2),
 Diameter of a circle = Half of the side of
square
U (5, 6), (6, 1), (6, 5)}
∴ n(E) = 15
O
1 ∴ Probability that the sum is a prime
= × 98
2 number
H
= 49 cm n (E) 15 5
∴ Circumference of the circle = π × diameter P(E) = = =
n (S) 36 12
22
_T

= × 49 ∴ Required probability = 1 – P(E)


7 5
= 154 cm. =1–
37. Ans. (A) 12
7
C

26 1 = .
Sol. Expression = × (5 × 45 + 2 × 60) 12
100 15
PS

40. Ans. (B)


26 Sol. Total possible outcomes n(s) = 6 × 6 = 36
= × (225 + 120)
100 × 15 Let E = Event of getting a doublet
26 × 345 = (1, 1), (2, 2), (3, 3), (4, 4), (5, 5), (6, 6)
=
U

100 × 15 =6
= 5.98. 6 1
∴ P(E) = = .
@

36 6
Solved Test Papers of Various Examinations ✫ 709

Management Aptitude Test, February 2013


Directions (Qs. 1–5): Study the graph to answer 3. If, in 2012, the global sales of brand ‘C’
these questions. increases by 12%, while its sales in USA
increases by 34% and in China by 22%, what
The bar graph below shows the sales of six
is the approximate sales increase in the rest
different cell phone manufacturers in 2011 (in
of the countries taken together?
‘000 units). The following pie chart shows the
(A) 1.83% (B) 2.25%
break-up of sales of brand ‘C’ alone in the same (C) 2% (D) 1.5%
year for those countries where it is sold.
4. If ‘A’ alone registers growth in sales in 2012

TS
2011 Global Sales (‘000s) and draws level with the volume of ‘B’, what
60
would be their combined share of the global
54
sales volume?

H
50 47 (A) 29.9% (B) 31.2%
(C) 27% (D) 33%
40
5. If the sale of phones in 2012 registers a

G
32
30 28 uniform 33% growth across all brands and
21 all countries, what would be the combined
20

10
12
U sales volume of phones in India, Japan and
Korea taken together, in that year?
O
(A) 87224 (B) 82136
0 (C) 85146 (D) 78958
A B C D E F
H
Directions (Qs. 6–10): Study the table below to
Brand ‘C’ : Countrywise sales, 2011
answer these questions.
_T

Country
Korea
China 14% Item A B C D E F
21% Iron ore 18 12 25 11 32 5
C

India
Coal 12 38 22 6 29 42
9%
Silver 14 18 26 3 48 23
Copper 23 12 20 2 2 16
PS

Japan
10% Gold 2 4 8 1 3 10
USA
18% The table shows the quantities of minerals
U

Germany exported from India to six different countries A,


12% B, C, D, E and F (in ‘000 tonnes) in the year 2010.
UK
6. If the second-largest importing country
@

16%
increases its off-take of all the minerals, taken
together, from India and is at par with the
1. What is the difference in volume of sales of largest importer, its share of the total exports
brand ‘C’ in UK and India? of minerals from India would be—
(A) 2892 (B) 3290 (A) 26% (B) 19%
(C) 3112 (D) 2918 (C) 23% (D) 24%
2. By what percentage should global sales of 7. If India were to ban the export of Gold
altogether, by what approximate percentage
brand ‘C’ phone increase so that its sales
should the export of the rest of the minerals
volume in Korea becomes 7,000 units, while increase so that the total quantity of minerals
the volume of sales to all other countries exported remains the same?
remains the same? (A) 18 (B) 3
(A) 1.1 (B) 0.9 (C) 32 (D) 6
(C) 2.3 (D) 0.7
8. Eighteen per cent of country E’s total
710 ✫ Quantitative Aptitude

requirement of Iron ore is being met by I. The daily water supply to the village
imports from India. Next year, its total is 700 m gallon.
requirement of iron ore is expected to fall II. The cost to the village for every 24000
by 22%, and only 12% of the revised gallons of water lost is Rs 25.
requirement will be imported from India. 12. A citrus fruit grower receives Rs 980 for each
Assuming all other exports to country ‘E’ by crate of lemon shipped and Rs 1,500 for each
India remain at the same levels, what would crate of oranges shipped. How many crates
be its total percentage drop in quantity of of oranges did the grower ship last week?
exports to ‘E’? I. Last week the number of crates of

TS
(A) 28.4 (B) 11.2 lemon that the grower shipped was 20
(C) 13.4 (D) 18.2 more than twice the number of crates
9. If international demand for Coal from India of orange shipped.
rises by 12%, that of Iron ore by 8% and II. Last week the grower received a total

H
Copper by 15%, by how much per cent will of Rs 3,87,000 from the crates of lemon
total exports from India rise, assuming and oranges shipped.

G
export of Gold and Silver remain at the same 13. If a total of 84 students are enrolled in two
level? sections of a calculus course, how many of
(A) 6.8
(C) 8.1
(B) 11.2
(D) 7.6 U the 84 students are female?
2
O
I. of the students in section 1 are female.
10. If due to rise in prices, volume of exports of 3
Gold decreases by 12% while that of Silver 1
H
decreases by 14%, but due to global rise in II. of the students in section 2 are female.
2
car production levels, however, demand for 14. Company R’s annual profit has increased by
_T

Iron ore increases by 6%, what is the net effect a constant amount each calendar year since
on volume of exports from India? 2005. What was Company R’s annual profit
(A) Increase of 3.2% in 2011?
(B) Decrease of 2.3% I. In 2005 Company R’s annual profit
C

(C) Increase of 2.3% was Rs 2,12,00,000; in 2009 Company


(D) Decrease of 3.2% R’s annual profit was Rs 2,42,00,000.
PS

II. Company R’s annual profit has


Directions (Qs. 11–15): Each of these has a question increased by Rs 7,50,000 each year
followed by two statements numbered I and II. Mark since 2005.
answer as—
U

15. Committee X and Committee Y, which have


(A) If statement I alone is sufficient but
no common members, will combine to from
statement II alone is not sufficient to
@

Committee Z. Does committee X have more


answer the question. members than Committee Y?
(B) If statement II alone is sufficient but I. The average (arithmetic mean) age of
statement I alone is not sufficient to the members of Committee X is 25.7
answer the question. years and the average age of the
(C) If both statements I and II together are members of Committee Y is 29.3 years.
required to answer the question but II. The average (arithmetic mean) age of
neither statement alone is sufficient. the members of Committee Z will be
(D) If statements I and II together are not 26.6 years.
sufficient to answer the question.
Directions (Qs. 16–20): Two quantities A and B
11. If a certain village is losing 12% of its water are given in each of the following questions. Mark
supply each day because of a burst answer as—
water pipe, then what is the loss in rupees (A) If quantity A is greater than quantity
per day? B.
Solved Test Papers of Various Examinations ✫ 711

(B) If quantity B is greater than quantity Break-up of Car Sales in 2010


A.
(C) If quantities A and B are equal. D
(D) If comparison cannot be made. C 16%
22%
16. A. The number of ways you can divide 6 E
different candies to 4 different people. 12%
B. The number of ways you can divide 7 B
different candies to 3 different people. 25%
A F

TS
17. Arun and Rakesh bought the same brand of 21% 4%
car. Arun got a 10% discount from the dealer
after which 7% sales tax was added. For

H
Rakesh 7% tax was added and only then a
Break-up of Car Sales in 2011
10% discount was given.
F

G
A. The amount Arun paid for the car.
E 6%
B. The amount Rakesh paid for the car. 8%
A
18. The average height in the class didn’t change
after Shaan and John left and Aarti and Geeta
U D
16%
27%
O
came.
A. The height of Shaan.
B
H
B. The height of Aarti. C
17%
26%
19. Travis received an allowance. With a fifth of
_T

the money, he bought a ticket to a movie and


with a quarter of what was left he bought a 21. What is the ratio of the number of cars sold
slice of pizza. of model A in 2010 to the number of cars sold
C

A. The price of a movie ticket. of model C in 2011?


B. The price of a slice of pizza. (A) 1.3 : 2 (B) 0.58 : 1
PS

20. Mukesh and Vijay each have some stamps (C) 0.62 : 1 (D) 1 : 1.27
while Sharon has none. They decided to give 22. By how much percentage is the increase in
Sharon half of the number that Mukesh has sales from 2010 to 2011 of model D more/
U

and half of the number that Vijay has. less than that of model C?
A. The average number of stamps that (A) 69% less (B) 60% more
(C) 59% less (D) None of the above
@

Mukesh and Vijay have before giving


Sharon any stamps. 23. How many models have registered higher
B. The number of stamps that Sharon gets percentage increase in sales individually, as
from Mukesh and Vijay. compared to the percentage increase in total
sales?
Directions (Qs. 21–25): Study the graphs to answer (A) 2 (B) 3
these questions. (C) 1 (D) None of these
The combined total sale of six models of 24. If the declining trend in model B sales is
cars A, B, C, D, E and F sold by a manufacturer projected to continue in 2012 as well, what
in 2010 and 2011 was 2,222 and 2,884, respecti- would be the approximate model B sales in
vely. The percentage break-up of the sales of that year?
these six models in the two years is shown in (A) 459 (B) 424
the pie-charts below— (C) 430 (D) 434
712 ✫ Quantitative Aptitude

25. What was the ratio of the combined sale of 70


Arts
IT
the two highest selling models to that of the 62
two lowest selling models in 2011? × × Engg.
60 Science
(A) 2 : 1 (B) 3 : 1 × 58
52
(C) 5 : 1 (D) 1 : 4
50 45
Directions (Qs. 26–30): Study the information below 46
42
and answer these questions.
40 39
The adult population of a town in 1990 was 38 35
20,440 people of which 53% were females while
31 28

TS
60% were illiterates. Of these, the number above 30 28 32
the age of 50 in that year was 17%. In the 2000 24 × 27 ×26
Census, it was found that adult population has 20
24
22 23
reached 30,760, 48% of them being females. Of 17
18

H
the total adult population in 2000, 72% were ×12 12
below the age of 50 and total illiterates were 10
A B C D E F
7,000. Assume uniform pattern of age distribution

G
amongst males and females. 31. The total number of graduates of which two
26. What is the ratio of adult females below the streams from all the states taken together are
age of 50 in the years 1990 and 2000?
(A) 2.50 : 2 (B) 2 : 2.50 U in the ratio 4 : 5?
(A) Science and Engg.
O
(C) 1 : 1.18 (D) 1.18 : 1 (B) Science and Arts
27. If every adult female above the age of 50 was (C) Arts and IT
H
given a special annual allowance of Rs (D) Engg. and IT
24,000 in 1990 which was increased to Rs 32. In which two states is the number of IT
_T

30,000 by the year 2000, by what percentage


graduates, as a percentage of the total
has the quantum of this annual allowance
increased/decreased from 1990 to 2000? number of graduates in that state, most
(A) 108 (B) 280 nearly the same?
C

(C) 208 (D) 180 (A) C and F (B) E and B


(C) A and E (D) D and F
28. What is the ratio of total adult illiterates in
PS

1990 to total adult females below 50 in 2000? 33. By what per cent is the sum total of Engg.
(A) 1 : 0.87 (B) 1.15 : 1 graduates in B, C and F taken together
(C) 0.87 : 1 (D) 1 : 1.15 greater than the sum total of Arts graduates
in D and E taken together?
U

29. What has been the percentage increase in


adult illiterates from 1990 to 2000? (A) 56 (B) 68
(A) 127 (B) 191 (C) 65 (D) 86
@

(C) 132 (D) 275 34. The number of Science graduates in all the
30. If expense incurred on education (per states combined is what per cent of the
literate adult) in 1990 was Rs 1200 and the number of graduates of all other streams
same had increased by 25% in 2000, what is taken together?
the percentage increase in expenditure under (A) 28.5 (B) 27.9
this head? (C) 29.7 (D) 21.4
(A) 236 (B) 263
(C) 250 (D) 178 35. In which states is the difference between the
number of Engg. and IT graduates higher
Directions (Qs. 31–35): Study the graph to answer than the difference between the number of
these questions.
Science and Arts graduates?
The graph below shows the number of
graduates (in ‘000) of different streams, in six (A) A, C and E (B) C, F and B
different states of a country. (C) F, B and D (D) C, D and E
Solved Test Papers of Various Examinations ✫ 713

Directions (Qs. 36–40): Study the graph to answer (C) White, 26%
these questions. (D) Lemon, 28%
The graph below shows sales of different 37. Sale of which colour(s) of refrigerator
colours of Brand-X refrigerator for the years
registered the highest percentage increase
2009, 2010 and 2011.
between 2010 and 2011?
Brand-X : Sales Volume by colour of (A) Blue (B) Lemon
refrigerator (in ‘000s) (C) Red and Blue (D) Grey
2009 2010 2011
38. What is the share of sales volume accounted
50 48 47
45 for by the year 2011?

TS
45
43
(A) 43.26% (B) 46.14%
40 (C) 44.16% (D) 41.64%
34
32 32 32
30 39. What is approximately the ratio of sales of

H
30 27
25 Red, Lemon and White refrigerators, taken
20 together, in 2010 to that of the combined total
20

G
sales of Blue and Grey ones in 2009 and 2011
taken together?
10

0
2

U (A) 9 : 10
(C) 11 : 12
(B) 10 : 9
(D) 1 : 1.2
O
Red Blue Lemon Grey White 40. How many instances are there in the graph
36. Over the given three years which has been when the difference in sale of refrigerators
was greater or lesser by 35% or more as
H
the highest selling colour; and what
percentage of volume did it account for? compared to the previous year?
(A) Blue, 21% (A) 4 (B) 5
_T

(B) White, 25% (C) 6 (D) 7

Answers and Explanatory Notes


C

1. Ans. (B) 420


Sol. Volume of sales of brand C in UK ∴ Percentage increase = × 100
47000
PS

16 42
= 47000 × =
100 47
= 7520
= 0.89
Volume of sales of brand C in India
U

= 0.9% (approximate).
9
= 47000 × 3. Ans. (C)
100 Sol. Global sales of brand C in 2012
@

= 4230
100 + 12
∴ Difference in volume of sales of brand = 47000 ×
C in UK and India 100
112
= 7520 – 4230 = 47000 ×
100
= 3290. = 52640
2. Ans. (B) Sales in USA in 2011
Sol. Sales volumes of brand C in Korea 18
14 = 47000 ×
= 47000 × 100
100 = 8460
= 6580 units Sales in China in 2011
Increase in sales volume 21
= 7000 – 6580 = 47000 ×
100
= 420 units = 9870
714 ✫ Quantitative Aptitude

Sales in the rest of the countries together 5. Ans. (C)


in 2011 Sol. Total sales of six different cellphone
= 47000 – 8460 – 9870 manufactures in 2011
= 28670 = (21 + 32 + 47 + 54 + 28 + 12) thousand
Now, = 194000
Sales in USA in 2012 Combined sales volume of phones in
100 + 34 India, Japan and Korea taken together
= 8460 × in 2011
100
= (9% + 10% + 14%) × 194000
134
= 8460 × 33

TS
100 = × 194000
100
= 11336.4
= 64020
Sales in China in 2012
∴ Required combined sales volume of
100 + 22

H
= 9870 × phones in India, Japan and Korea taken
100 together in 2012
122

G
= 9870 × 100 + 33
100 = 64020 ×
= 12041.4 100
Sales in the rest of the countries in 2012
= 52640 – 11336.4 – 12041.4
= 29262.2 U = 64020 ×
= 85146.
133
100
O
∴ Percentage increase in sales of brand C For Qs. 6-10:
in the rest of the countries taken together Quantities of Minerals Exported from India to six
H
in 2012 countries (in thousand tonnes) in 2010
29262.2 − 28670 Item Country
= × 100
_T

28670
592.2 A B C D E F Total
= × 100
28670 Iron ore 18 12 25 11 32 5 103
Coal 12 38 22 6 29 42 149
C

= 2.06%
= 2% (approximate). Silver 14 18 26 3 48 23 132
Copper 23 12 20 2 2 16 75
PS

4. Ans. (B) Gold 2 4 8 1 3 10 28


Sol. Combined share of (A + B) in the global Total 69 84 101 23 114 96 487
sales volume in 2011 (in %)
6. Ans. (C)
U

(21 + 32) × 100 Sol. According the table, the largest


=
(21 + 32 + 47 + 54 + 28 + 12) importing country = E
@

The second largest importing country


53 × 100
= =C
194 Total exports of minerals from India
= 27.319 = (69 + 84 + 101 + 23 + 114 + 96) × 1000
= 487000 tonnes
= 27.32
∴ Share of total exports of minerals from
Combined share of (A + B) in the global India to second largest importing
sales volume in 2012 (in %) country (in %)
(32 + 32) × 100 (101 + 13) × 1000
= = × 100
(32 + 32 + 47 + 54 + 28 + 12) (487 + 13) × 1000
114
64 × 100 = × 100
= 500
205
= 22.8
= 31.2%. = 23% (approximate).
Solved Test Papers of Various Examinations ✫ 715

7. Ans. (D) 9. Ans. (D)


Sol. Total export of Gold Sol.  Rise in demand for coal from India
= (2 + 4 + 8 + 1 + 3 + 10) × 1000 tonnes 12
= 28000 tonnes = 149 × = 17.88 thousand tonnes
100
Export of the rest of minerals
Rise in demand for Iron ore from India
= (487 – 28) × 1000 tonnes
8
= 459000 tonnes = 103 × = 8.24 thousand tonnes
∴ Per cent increase in the export of rest of 100
minerals Rise in demand for Copper from India
487000 − 459000 15

TS
= × 100 = 75 × = 11.25 thousand tonnes
459000 100
28000 Total rise in total exports from India
= × 100
459000 = 17.88 + 8.24 + 11.25

H
= 6.1 = 37.37 × 1000 tonnes
= 6% (approximate). = 37370 tonnes
8. Ans. (C) ∴ Required % rise in total exports from

G
Sol. 18% of country E’s total requirement India
= Import of Iron ore from India 37370
Total requirement of country E
=
100
× 32000 tonnes U =
487000
= 7.6%.
× 100
O
18 10. Ans. (D)
= 177777.78 tonnes Sol.  Decrease in export of Gold
H
Revised total requirement of Iron ore 12
= 28000 ×
of country E for next year 100
(100 − 22)
_T

= 3360 tonnes
= × 177777.78
100 Decrease in export of Silver
78 14
= × 177777.78 = 132000 ×
100 100
C

= 138666.67 tonnes = 18480 tonnes


Only 12% of revised requirement is Increase in export of Iron ore
PS

imported from India 6


12 = 103000 ×
= × 138666.67 100
100 = 6180 tonnes
U

= 16640 tonnes Total decrease in volume of exports


= 16.64 × 1000 tonnes = (3360 + 18480 – 6180) tones
@

Quantity of total exports to E = (21840 – 6180) tones


= (114 – 32 + 16.64) × 1000 tonnes = 15660 tonnes
= (130.64 – 32) × 1000 tonnes
∴ Required net effect on volume of
= 98.64 × 1000 tonnes
∴ Total per cent drop in the quantity of exports from India (in %)
exports to E Total decrease in
(114 − 98.64) × 1000 volume of exports
= × 100 = × 100%
114 × 1000 Total exports of
minerals from India
15.36 × 1000
= × 100 15660
114 × 1000 = × 100
1536 487000
= = 3.21
114
or Decrease of 3.2%.
= 13.4%.
716 ✫ Quantitative Aptitude

11. Ans. (C) From statement I


Sol. From statement I 2
The daily water supply to the village Female = ×x
3
= 700 m gallon
2
Losts of water supply each day Male = x – x
3
12
= 700 × 1000000 × 3x − 2 x x
100 = =
= 84000000 gallon 3 3
From statement II
From statement II
Loss in rupees per day 1

TS
Female = y
25 2
= × 84000000 1
24000 Male = y – y
= 25 × 3500 2

H
= Rs 87500 2y − y 1
= = y
Hence both statements I and II together 2 2
are required to answer the question,

G
Hence, statement I and II together are
but neither statement alone is sufficient. not sufficient and additional data is
12. Ans. (C) needed to answer the question.
Sol. Let the number of crates of lemons and
oranges be x and y respectively. U 14. Ans. (C)
Sol. From statement I
O
From statement I Profit increased in 4 years (2005 to
x = 20 + 2y 2009)
H
x – 2y = 20 . . . (i) = 24200000 – 21200000
From statement II = Rs 3000000
980x + 1500y = 387000 . . . (ii) Profit increased in each year
_T

Multiplying by 750 in equation (i), we


3000000
get =
750x – 1500y = 15000 . . . (iii) 4
= Rs 750000
C

Adding equation (iii) and (ii), we get


980x + 1500y = 387000 So, Company R’s annual profit in 2011
750x – 1500y = 15000 = 21200000 + 750000 × 6
PS

= 21200000 + 4500000
1730x = 402000 = Rs 25700000
402000 From statement II
x =
U

1730 Company R’s annual profit has


x = 232.36 increased
x = Rs 232 (approximate) = Rs 750000 each year
@

Putting the value of x in equation (i) Then, Company R’s annual profit in
232 – 2y = 20 2011
2y = 232 – 20 = 21200000 + 750000 × 6
2y = 212 = 21200000 + 4500000
y = Rs 106 = Rs 25700000
Hence, both statements taken together Hence, both the given statements taken
are sufficient to answer the question. together are sufficient to answer the
13. Ans. (D) question.
Sol. Total number of students in two 15. Ans. (C)
sections = 84 Sol. Let total members in Committees X and
Let the number of student in section 1 Y be x and y respectively. Then,
and section 2 be x and y respectively. From statement I
Then, Sum of age of the members of
x + y = 84 Committee X = 25.7x
Solved Test Papers of Various Examinations ✫ 717

Sum of age of the members of 90 x 7x


Committee Y = 29.3y = +
100 100
From statement II 97 x
Sum of age of the members of = = 0.97x
100
Committee Z = 26.6 (x + y) Now,
Now, tax = 7% and discount = 10%
25.7x + 29.3y = 26.6 (x + y) B = Amount Rakesh paid for car
25.7x + 29.3y = 26.6x + 26.6y 7x 100 − 10
29.3y – 26.6y = 26.6x – 25.7x = x+ ×
100 100

TS
2.7y = 0.9y 100x + 7 x 90
2.7 = ×
y = x 100 100
0.9 107 x 9
3y = x = ×

H
Or x = 3y 100 10
Hence, Committee X does not have = 0.963x

G
more members than Committee Y. So, ∴ A > B.
both the given statements are sufficient 18. Ans. (D)
to give the answer to the question.
16. Ans. (B) U Sol. A = The height of Shaan
B = The height of Aarti
O
Sol. According to the question, The comparison of heights of Shaan
A = Number of ways 6 different candies and Aarti cannot be made.
can be divided to 4 different people
H
19. Ans. (C)
= 6C4 Sol. Let Travis receive an allowance of
6! Rs x
_T

=
4 ! (6 − 4) ! Then,
6×5×4 ! A = Price of a movie ticket
= = 15
4!× 2 ×1 1 x
×x=
C

=
B = Number of ways dividing 7 different 5 5
candies to 3 different people B = Price of a slice of pizza
PS

= 7C3 1 x
= × x−
7! 4 5
= 1 5x − x
U

3 ! (7 − 3) ! = ×
7! 7×6×5×4 ! 4 5
= = 1 4x
3 ! × 4 ! 3 × 2 × 1× 4 !
@

= ×
= 35 4 5
∴ B > A. =
x
5
17. Ans. (A) ∴ A = B.
Sol. Let the cost of car = Rs x
20. Ans. (C)
Here, discount = 10%, sales tax = 7%
Sol. Let the number of stamps Mukesh and
A = Amount Arun paid for car
Vijay each have be x and y respectively.
100 − discount Then,
= × cost of car A = The average number of stamps
100
+ sales tax × cost of car that Mukesh and Vijay have
before giving Sharon any stamps
100 − 10 7 x+y
= ×x+ × x =
100 100 2
718 ✫ Quantitative Aptitude

B = The number of stamps that Sharon Number of cars sales of model C in


gets from Mukesh and Vijay 2010
x y 22
= + = × 2222
2 2 100
x+y = 488.84
=
2 Number of cars sales of model C in
∴ A = B. 2011
21. Ans. (C) 26
= × 2884
Sol. Total cars manufactured in 2010 = 2222 100

TS
Number of cars sold of model A in = 749.84
2010 Percentage increase in sales from 2010
21 to 2011 of model C
= × 2222
749.84 − 488.84

H
100 = × 100
= 466.62 488.84
Total cars manufactured in 2011 = 2884 26100

G
=
Number of cars sold of model C in 2011 488.84
26 = 53.39%
=
100
× 2884

= 749.84 U ∴ Required % =
29.79 − 53.39
53.39
× 100
O
Ratio of the number of cars sold of 23.6
=– × 100
model A in 2010 to the number of cars 53.39
H
sold of model C in 2011 2360
=–
53.39
_T

466.62 46662
= = = –44.2
749.84 74984 Increase in sales from 2010 to 2011 of
0.622 model D is less than that of model C by
C

= 44.2%.
1
= 0.62 : 1. 23. Ans. (B)
PS

Sol. Total car sales in 2010 = 2222


22. Ans. (D)
Total car sales in 2011 = 2884
Sol. Number of car sales of model D in 2010
Percentage increase in total sales
16
2884 − 2222
U

= × 2222
100 = × 100
2222
= 355.52
@

662
Number of car sales of model D in 2011 = × 100
2222
16 = 29.79%
= × 2884
100 Percentage increase in sales of model A
= 461.44
27% of 2884 – 21% of 2222
Percentage increase in sales from 2010 = × 100
21% of 2222
to 2011 of model D
27 21
461.44 − 355.52 × 2884 − × 2222
= × 100 100 100
355.52 = × 100
21
105.92 × 2222
= × 100 100
355.52 778.68 − 466.62
= × 100
= 29.79% 466.62
Solved Test Papers of Various Examinations ✫ 719

312.06 Percentage increase in sales of model F


= × 100 6 4
466.62 × 2884 − × 2222
= 66.87% = 100 100 × 100
4
Percentage increase in sales of model B × 2222
17 25 100
× 2884 − × 2222 173.04 − 88.88
= 100 100 × 100 = × 100
25 88.88
× 2222
100 84.16
= × 100
490.28 − 555.5

TS
= × 100 88.88
555.5 = 94.68%
−65.22 Hence 3 models have registered higher per
= × 100
555.5 cent increase in sales (A, C and F)

H
= –11.74 individually as compared to % increase in
= 11.74% decrease total sales.

G
Percentage increase in sales of model C 24. Ans. (D)
26 22 Sol. Percentage decline in sales of model B
× 2884 − × 2222
= 100
22
100
× 2222
× 100
U = 11.74
Then,
O
100 Model B sales in the year 2012
749.84 − 488.84 (100 − 11.74)
= × 100 = × 490.28
488.84
H
100
261 88.26
= × 100 = × 490.28
488.84
_T

100
= 53.39% = 432.72
Percentage increase in sales of model D = 434 (approximate).
16 16 25. Ans. (B)
× 2884 − × 2222
C

100 100 Sol. Required ratio


= × 100 Combined sale of two
16
× 2222
PS

100 highest selling models


=
461.44 − 355.52 Combined sale of two
= × 100 lowest selling models
355.52
U

105.92 Combined sale of models A and C


= × 100 =
355.52 Combined sale of models E and F
@

27 26
= 29.79% × 2884 + × 2884
Percentage increase in sales of model E = 100 100
8 6
8 12 × 2884 + × 2884
× 2884 − × 2222 100 100
= 100 100 × 100
12 (778.68 + 749.84)
× 2222 =
100 (230.72 + 173.04)
230.72 − 266.64 1528.52
= × 100 =
266.64 403.76
−35.92 3.78
= × 100 =
266.64 1
= –13.47 = 3.78 : 1
Or 13.47% decrease = 3 : 1 (approximate).
720 ✫ Quantitative Aptitude

For Qs. 26-30: 28. Ans. (A)


Sol. Ratio of total adult illiterates in 1990 to
Census year
total adult females below 50 in 2000
1990 2000 Total adult illiterates in 1990
=
Adult 20440 30760 Total adult females below 50 in 2000
population 12264 2 1
= = =
53 48 10630.6 1.73 0.865
Females × 20440 = 10833.2 × 30760 = 14764.8
100 100 = 1 : 0.87.
60 29. Ans. (B)

TS
Illiterates × 20440 = 12264 7000
100 Sol. Number of adult literates in 1990
= Total adults – Number of adult
72
Below 50 10833 .2 × (100 – 17)% × 14764.8 illiterates
100
= 20440 – 12264

H
83
10833.2 × = 8991.5 = 10630.6
100 = 8176
17 Number of adult literates in 2000

G
Above 50 10833.2 × 14764.8 × (100 – 72)% = Total adults – Number of adult
100
28 illiterates
= 1841.6 14764.8 × = 4134.1

26. Ans. (C)


Sol. Ratio of adult females below the age of
100

U = 30760 – 7000
= 23760
∴ Percentage increase in adult illiterates
O
50 in the years 1990 and 2000 from 1990 to 2000
Adult females below 23760 − 8176
H
= × 100
the age of 50 in 1990 8176
=
Adult females below 15584
_T

the age of 50 in 2000 = × 100


8176
8991.5 = 190.6%
=
10630.6 = 191% (approximate).
C

1 30. Ans. (B)


= = 1 : 1.18. Sol. Number of adult literates in 1990
1.18
PS

27. Ans. (D) = Total adults – Number of adult


Sol. Every adult female above the age of 50 illiterates
got a special allowance in 1990 = 20440 – 12264
= 8176
U

= 1841.6 × Rs 24000
Expense incurred on education in 1990
= Rs 44198400 = 8176 × Rs 1200
@

Every adult female above the age of 50 = Rs 9811200


got a special allowance in 2000 Number of adult literates in 2000
= 4134.1 × Rs 30000 = Total adults – Number of adult
= Rs 124023000 illiterates
∴ Required percentage of quantum = 30760 – 7000
allowance increased = 23760
Expense incurred on education in 2000
124023000 − 44198400
= × 100 = 23760 × Rs 1500
44198400 = Rs 35640000
79824600 Required percentage increase in
= × 100
44198400 expenditure on education
= 180.61% 35640000 − 9811200
= × 100
= 180% (approximate). 9811200
Solved Test Papers of Various Examinations ✫ 721

25828800 State C
= × 100
9811200 Number of IT graduates
= × 100
= 263.2% Total number of all graduates
= 263% (approximate). 23
= × 100
31. Ans. (A) (35 + 27 + 23 + 62)
Sol. Total number of graduates in Science 23
= 22 + 28 + 35 + 32 + 39 + 12 = × 100
147
= 168
= 15.65%
Total number of graduates in Arts
State D

TS
= 46 + 42 + 27 + 45 + 24 + 52
= 236 Number of IT graduates
= × 100
Total number of graduates in IT Total number of all graduates
= 31 + 38 + 23 + 18 + 28 + 17 18

H
= 155 = × 100
(32 + 45 + 18 + 58)
Total number of graduates in
18

G
Engineering = × 100
= 24 + 28 + 62 + 58 + 12 + 26 153
= 210 = 11.76%
Total number of graduates
in Science U State E
Number of IT graduates
O
Ratio = = × 100%
Total number of graduates Total number of all graduates
in Engineering 28
= × 100
H
=
168 4
= (39 + 24 + 28 + 12)
210 5 28
= × 100
_T

Hence the required two streams in 103


which the total number of graduates = 27.18%
from all the states are in the ratio 4 : 5 State F
are Science and Engineering.
C

Number of IT graduates
32. Ans. (A) = × 100
Total number of all graduates
Sol. The percentage of IT graduates to the 17
PS

total number of all graduates in = × 100


(12 + 52 + 17 + 26)
State A
17
Number of IT graduates = × 100
= × 100 107
U

Total number of all graduates = 15.88%


31 ∴ Two states, in which the percentage is most
= × 100
@

(22 + 46 + 31 + 24) nearly the same, are = State C and State F.


31
= × 100 33. Ans. (B)
123 Sol. Sum total of Engineering graduates in
= 25.2% B, C and F taken together
State B = (28 + 62 + 26) × 1000
Number of IT graduates = 116000
= × 100% Sum total of Arts graduates in D and E
Total number of all graduates
38 taken together
= × 100 = (45 + 24) × 1000
(28 + 42 + 38 + 28)
= 69000
38 ∴ Required percentage
= × 100
136
116000 − 69000
= 27.94% = × 100
69000
722 ✫ Quantitative Aptitude

47000 State E = 39000 – 24000 = 15000


= × 100 State F = 12000 ~ 52000
69000
4700 or 52000 – 12000 = 40000
= Hence the difference between the
69
= 68.11% number of Engineering and IT
= 68% (approximate). graduates is higher than the difference
34. Ans. (B) between the number of Science and
Sol. Number of Science graduates in all Arts graduates in following states
states combined = C, D and E.

TS
= (22 + 28 + 35 + 32 + 39 + 12) × 1000 36. Ans. (B)
= 168000 Sol. Total number of red colour
Number of graduates of all other
refrigerators sold in the years 2009,
streams taken together

H
2010 and 2011
= (168 + 236 + 155 + 210) × 100 – 168000
= (20 + 30 + 45) × 1000
= 769000 – 168000

G
= 95000
= 601000
Total number of blue colour
Required percentage
Number of Science graduates
in all states combined U refrigerators sold in the years 2009,
2010 and 2011
O
= × 100% = (25 + 32 + 48) × 1000
Number of graduates of all other
streams taken together = 105000
H
168000 Total number of lemon colour
= × 100 refrigerators sold in the years 2009,
601000
_T

2010 and 2011


16800
= = (47 + 23 + 32) × 1000
601
= 102000
= 27.95%
Total number of grey colour
C

= 27.9% (approximate).
refrigerators sold in the years 2009,
35. Ans. (D) 2010 and 2011
PS

Sol. Difference between IT and Engineering = (02 + 27 + 34) × 1000


graduates in = 63000
State A = 31000 – 24000 = 7000 Total number of white colour
U

State B = 38000 – 28000 = 10000 refrigerators sold in the years 2009,


State C = 23000 ~ 62000 2010 and 2011
@

or 62000 – 23000 = 39000 = (32 + 45 + 43) × 1000


State D = 18000 ~ 58000 = 120000
or 58000 – 18000 = 40000 Highest selling colour over the given
State E = 28000 – 12000 = 16000 three years = white
State F = 17000 ~ 26000 ∴ Required percentage of volume
or 26000 – 17000 = 9000 120000
Difference between Science and Arts = × 100
(95000 + 105000 + 102000
graduates in + 63000 + 12000)
State A = 46000 – 22000 = 24000 120000
State B = 42000 – 28000 = 14000 = × 100
485000
State C = 35000 – 27000 = 8000
State D = 32000 ~ 42000 = 24.74%
or 42000 – 32000 = 10000 = 25% (approximate).
Solved Test Papers of Various Examinations ✫ 723

37. Ans. (C) Total sales volume


Sol. Per cent increase of sale (red) in the year 2011
= × 100
45 − 30 Total sales volume in the
= × 100
30 year 2009, 2010 and 2011
15 (45 + 48 + 32 + 34 + 43) thousand
= × 100 = × 100
30 [(20 + 25 + 47 + 2 + 32)
1 + (30 + 32 + 23 + 27 + 45)
= × 100 = 50%
2 + (45 + 48 + 32 + 34 + 43)] thousand
Per cent increase of sale (blue)

TS
202 thousand
48 − 32 = × 100
= × 100 [(126) + (157) + (202)] thousand
32
16 202
= × 100

H
32 = × 100
485
1
= × 100 = 50% = 41.64%.

G
2 39. Ans. (A)
Per cent increase of sale (lemon) Sol. Sales of (red + lemon + white) taken
=
32 − 23
23
× 100
U together in 2010
= 30000 + 23000 + 45000
O
9 = 98000
= × 100 Sales of (blue + grey) taken together in
23
2009 and 2011
H
900
= = 39.1% = [(25 + 02) + (48 + 34)] × 1000
23 = [27 + 82] × 1000
_T

Per cent increase of sale (grey) = 109000


98000
34 − 27 ∴ Required ratio =
= × 100 109000
27
89.9
C

7 =
= × 100 100
27 90
PS

700 
= = 25.9% 100
27  9 : 10.
Per cent decrease of sale (white) 40. Ans. (D)
U

43 − 45 Sol. Difference in sale of refrigerators as


= × 100 compared to the previous year
45
expressed in % is as follows.
@

−2
= × 100 In the year 2010,
45
(30 − 20) thousand
−200 Red = × 100
= = –4.44% 20 thousand
45
10 thousand
or 4.44% decrease = × 100
20 thousand
∴ Required colours of refrigerator of = 50%
highest % increase between 2010 and
(32 − 25) thousand
2011 = red and blue. Blue = × 100
25 thousand
38. Ans. (D)
7 thousand
Sol. Share of sales volume accounted = × 100
for by the year 2011 expressed in 25 thousand
percentage = 28%
724 ✫ Quantitative Aptitude

(23 − 47) thousand 16 thousand


Lemon = × 100 = × 100
47 thousand 32 thousand
−24 thousand = 50%
= × 100
47 thousand (32 − 23) thousand
= –51.06 (decrease) Lemon = × 100
23 thousand
(27 − 02) thousand
Grey = × 100 9 thousand
2 thousand = × 100
23 thousand
25 thousand
= × 100 = 39.13%

TS
2 thousand
= 1250% (34 − 27) thousand
Grey = × 100
(45 − 32) thousand 27 thousand
White = × 100
7 thousand

H
32 thousand = × 100
13 thousand 27 thousand
= × 100
32 thousand = 25.9%

G
= 40.625%
(43 − 45) thousand
In the year 2011, White = × 100

Red =
(45 − 30) thousand
30 thousand
× 100 U =
45 thousand
−2 thousand
× 100
O
45 thousand
15 thousand
= × 100 = –4.44% (decrease)
30 thousand
H
= 50% ∴ Number of instances when the
difference in sale of refrigerators was
(48 − 32) thousand
_T

Blue = × 100 greater or lesser by 35% or more as


32 thousand compared to the previous year is 7.
C
PS
U
@
Solved Test Papers of Various Examinations ✫ 725

Reserve Bank of India (RBI) Officers, Grade ‘B’


Examination, November 2015
Directions—(Q. 1-5) In this question, a number 8. A vessel contains a mixture of milk and
series is given. Only one number is wrong water in the respective ratio of 5:1.24 litres
which doesn’t fit in the series. Find out the of mixture was taken out and replaced with
wrong number. the same quantity of milk so that the resultant
1. 3 6 10 16 21 28 ratio between the quantities of milk and

TS
(A) 16 (B) 10 (C) 15 water in the mixture was 13 : 2 respectively.
(D) 28 (E) 21 If 15 litres of the mixture is again taken out
from the vessel, what is the resultant quantity
2. 10080 1440 240 48 12 3 2 of milk in the mixture (in litres) ?

H
(A) 240 (B) 3 (C) 1440 (A) 97 (B) 89 (C) 91
(D) 48 (E) 12 (D) 99 (E) 84

G
3. 1 2 6 21 88 505 2676 Directions—(Q. 9-13) Refer to the table and
(A) 21 (B) 6 (C) 2 answer the given question.
(D) 505
4. 18 21 25 35
(E) 88
52 78 115 UData related to Performance of 6 Batsmen
in a Tournament
O
(A) 25 (B) 21 (C) 28
Number of Number of Average runs Total balls Strike
(D) 35 (E) 52 batsman matches played in scored in the faced in the rate
H
the tournament tournament tournament
5. 120 137 178 222 290 375 477
(A) 178 (B) 137 (C) 290 A 8 — — 129.6
_T

(D) 375 (E) 222 B 20 81 — —


C — 38 400 114
6. Area of a rectangle is 150 metre sq. When D — — — 72
the breadth of the same rectangle is increased E 28 55 1280 —
C

F — — — 66
by 2 meter and the length decreased by 5
metre, the area of the rectangle decreases by Note:
PS

(i) Strike rate = (Total runs scored/ Total balls faced)


30 metre square. What is the perimeter of the x 100
square whose sides are equal to the length (ii) All the given batsmen could bat in all the given
of the rectangle? matches played by them.
U

(iii) Few values are missing in the table (indicated by


(A) 76 m (B) 72 m (C) 120 m ...). A candidate is expected to calculate the
(D) 80 m (E) 60 m missing value, if it is required to answer the given
@

question, on the basis of the given data and


7. A, B and C started a business with information.
investments of ` 4200, ` 3600 and ` 2400
respectively. After 4 months from the start of 9. The respective ratio between total number
the business, A invested ` 1000 more. After 6 of balls faced by D and that by/in the
months from the start of the business, B and tournament is 3 : 4. Total number of runs
C invested additional amounts in the scored by F in the tournament is what per
respective ratio of 1 : 2. If at the end of 10 cent more than the total runs scored by D in
months they received a profit of ` 2820 and the tournament ?
A’s share in the profit was ` 1200, what was 2 4 8
(A) 22 (B) 32 (C) 18
the additional amount that B invested ? 9 9 9
(A) ` 800 (B) ` 200 (C) ` 500 4 2
(D) 24 (E) 28
(D) ` 600 (E) ` 400 9 9
726 ✫ Quantitative Aptitude

10. If the runs scored by E in last 3 matches of a tie. How many registered voters are there
the tournament are not considered, his in the village ?
average runs scored in the tournament will (A) 4000 (B) 3500 (C) 3000
decrease by 9. If the runs scored by E in the (D) 3250 (E) 3750
26th and 27th match are below 128 and no 16. Eight years ago, Poorvi’s age was equal to
two scores among these 3 scores are equal, the sum of the present ages of her one son
what are the minimum possible runs scored and one daughter. Five years hence, the
by E in the 28th match? respective ratio between the ages of her
(A) 137 (B) 135 (C) 141 daughter and her son that time will be 7 : 6.
(D) 133 (E) 139

TS
If Poorvi’s husband is 7 years elder to her
11. In the tournament, the total number of balls and his present age is three times the present
faced by batsman A is 74 less than the total age of their son, what is the present age of
number of runs scored by him. What is the the daughter (in years) ?

H
average run scored by batsman A in the (A) 15 years (B) 23 years
tournament? (C) 19 years (D) 27 years

G
(A) 42.5 (B) 39.5 (C) 38 (E) 13 years
(D) 44 (E) 40.5 17. Boat A travels downstream from Point X to
12. Batsman B faced equal number of balls in
first 10 matches he played in the tournament
U Point Y in 3 hours less than the time taken
by Boat B to travel upstream from Point Y to
Point Z. The distance between X and Y is 20
O
and last 10 matches he played in the
tournament. If his strike rate in first 10 km, which is half of the distance between Y
matches and last 10 matches of the and Z. The speed of Boat B in still water is
H
tournament is 120 and 150 respectively, what 10 km/h and the speed of Boat A in still
water is equal to the speed of Boat B
is the total number of balls faced by him in
_T

upstream. What is the speed of Boat A in still


the tournament ?
water ? (Consider the speed of the current to
(A) 1150 (B) 1400 (C) 1200
be the same.) ?
(D) 1000 (E) 1500
(A) 10 km/h (B) 16 km/h
C

13. What is the number of matches played by (C) 12 km/h (D) 8 km/h
batsman C in the tournament ? (E) 15 km/h
PS

(A) 10 (B) 16 (C) 12


(D) 18 (E) 8 Directions—(Q. 18-22) Study the following
information carefully and answer the given
14. 10 men can complete a project in 12 days, 12
questions—
U

children can complete the same project in 16 Data regarding number of applications
days and 8 women can complete the same received for various courses in University A and
project in 20 days. 5 men and 12 children
@

that in University B in the year 2001.


started working on the project. If after 4 days, (Note : Universities A and B offer courses
8 children were replaced by 4 women. In in six courses only, namely— Commerce, Science,
how many days the remaining project was Engineering, Arts, Management and Law.)
completed ? • In University A, applications received
2 1 1 for commerce, science and engineering together
(A) 4 (B) 5 (C) 7
5 2 2 constituted 70% of the total number of
5 2 applications received (for all the given courses
(D) 3 (E) 6
9 3 together). Applications received for arts,
15. In a village, 60% registered voters cast their management and law were 800, 750 and 400
votes in the election. Only two candidates respectively. Applications received for
(A and B) were contesting the election. A won management were 40% less than that for
the election by 600 votes. Had B received engineering. Applications received for commerce
40% more votes, the result would have been were 20% more than that for science.
Solved Test Papers of Various Examinations ✫ 727

• In University B, applications received for (A) 210 (B) 150 (C) 130
science were 20% less than that for science in (D) 80 (E) 170
University A.
• In University B, applications received for Directions—(Q. 23-27) Refer to the pie charts
Arts were 780 and they constituted 15% of the and answer the given questions—
total number of applications received (for all the Distribution of total number of
given courses together). Also the applications members (both male and female)
received for arts were 40% less than that received in 5 health clubs in 2011
for commerce. Total number of applications
received for engineering and management Total number: 6300

TS
together, were double the total number of
applications received for arts and law together. E A
Applications received for engineering were 12% 21%

H
equal to that for management. D
18. What is the respective ratio between the 20%

G
total number of applications received for
B
Engineering and science together in C
University A and that for the same courses 33%
14%
together in University B ?
(A) 50 : 47 (B) 65 : 53 U
O
(C) 52 : 37 (D) 55 : 42 Distribution of total number
(E) 43 : 36 of male members in 5 health
clubs in 2011
H
19. Number of applications received for only
commerce in University B is what per cent Total number: 3600
_T

less than that in University A ?


7 E
(A) 30 (B) 27 (C) 20 8% A
9
5 D 24%
C

(D) 15 (E) 15 20%


8
5
20. th of the number of applications for arts
PS

8 C B
in University A were by female students. If 18% 30%
the number of female applicants for the same
course in University B is less than that in
U

University A by 120, what is the number of


male applicants for arts in University B? 23. Only 25% of the members (both male and
@

(A) 500 (B) 420 (C) 450 female) in health club D have lifetime
membership. If the number of females in
(D) 360 (E) 400
health club D who have lifetime
21. Total number of applicants (for all the given membership is 126, what per cent of male
six courses together) received by University members in health club D have lifetime
A, was what per cent more than that in membership?
University B ? 1 3 3
(A) 25 (B) 10 (C) 20 (A) 30 (B) 28 (C) 20
2 4 4
(D) 25 (E) 30 1 1
22. What is the difference between the total (D) 26 (E) 24
4 2
number of applications received for 24. What is the average number of female
management and law together in University members in health clubs A, B and C?
A and that for same courses together in (A) 564 (B) 572 (C) 568
University B ? (D) 548 (E) 588
728 ✫ Quantitative Aptitude

25. Number of male members in health clubs A (A) 5 kmph (B) 6 kmph
and C increased by equal number from 2011 (C) 6.25 kmph (D) 7.5 kmph
to 2012. If the respective ratio between (E) None of the above
number of male numbers in health club A 29. Ram bought two articles A and B at a total
and that in C in 2012 is 13 : 10, what is the cost of ` 8000. He sold article A at 20% profit
number of male members in health club C and article B at 12% loss. In the whole deal
in 2012 ? he made no gain and no loss. At what price
(A) 690 (B) 750 (C) 720 should Ram have sold article B to make an
(D) 740 (E) 760 overall profit of 25% ?

TS
26. What is the central angle corresponding to (A) ` 5200 (B) ` 5800
number of (both male and female) in health (C) ` 6400 (D) ` 6200
club B ? (E) ` 4260
(A) 118.8° (B) 112.6° (C) 124.8° 30. An equal sum is invested for six years in

H
(D) 116.4° (E) 128.4° scheme A offering simple interest at X% p.a.
27. Number of female members in health club and in Scheme B for two years offering

G
E is what per cent less than number of male compound interest at 10% p.a. (compounded
members in health club B ? annually). The interest earned from Scheme

(A) 56
2
3
(B) 54
1
3
(C) 60
2
3
U A is double the interest earned from Scheme
B. Had the rate of interest been X + 2%
simple interest p.a. in scheme A, the
O
1 2
(D) 64 (E) 48 difference in the interests earned from both
3 3 the schemes would have been ` 3960, what
H
28. In covering a distance of 30 km, Abhay takes was the sum invested in each of the scheme?
2 hours more than Sameer. If Abhay doubles (A) ` 10000 (B) ` 20000
_T

his speed, then he would take 1 hr. less than (C) ` 12000 (D) ` 18000
Sameer. What is the speed of Abhay ? (E) ` 15000
C

Answers and Explanatory Notes


PS

1. Ans. (A) (88 × 5) + 5 = 445


Sol. The given series is (445 × 6) + 6 = 2676
The wrong number is 505
3 6 10 16 21 28
4. Ans. (B)
U

+3 +4 +5 +6 +7 Sol. The given series is


The wrong number is 16. 18 21 25 35 52 78 115
@

2. Ans. (B) 18 + 2 = 20
Sol. The given series is 20 + (2 + 3) = 25
25 + (2 + 3 + 5) = 35
10080 1440 240 48 12 3 2 35 + (2 + 3 + 5 + 7) = 52
÷7 ÷6 ÷5 ÷4 ÷3 ÷2 52 + (2 + 3 + 5 +7 + 9) = 78
The wrong number is 3. 78 + (2 + 3 + 5 + 7 + 9 + 11) = 115
3. Ans. (D) The wrong number is 21
Sol. The given series is 5. Ans. (A)
1 2 6 21 88 505 2676 Sol. The given series is
(1 × 1) + 1 = 2 120 137 178 222 290 375 477
(2 × 2) + 2 = 6
(6 × 3) + 3 = 21 +17 +34 +51 +68 +85 +102
(21 × 4) + 4 = 88 The wrong number is 178
Solved Test Papers of Various Examinations ✫ 729

6. Ans. (E) ∴ C’s investment for 10 months


Sol. Area of rectangle = 150m2 = 2400 × 6 + (2400 + 2x)4
lb = 150 ..... (1) = 14400 + 9600 + 8x
New breadth = (b + 2) = 2400 + 8x ... (3)
New length = (l – 5) Ratio of their investments
New area = 150 – 30 = 120m2 = 48000 : 36000 + 4x : 24000 + 8x
(l – 5)(b + 2) = 120 [from (1), (2), (3)]
lb + 2l – 5b – 10 = 120 = 12000 : 9000 + x : 6000 + 2x
150 + 2l – 5b = 130 (taking 4 common)
2l – 5b = –20 ..... (2) Total profit = Rs 2820

TS
150 Sum of ratios
From (1) b = = 12000 + 9000 + x + 6000 + 2x
l
Substituting in (2) = 3x + 27000

H
12000
150 A’s profit = × 2820
2l – 5 × = – 20 3 x + 27000
l

G
2
2l – 750 = –20l 12000
1200 = × 2820
2l2 + 20l – 750 = 0 3(x + 9000)

l =
l2 + 10l – 375 =
− b ± b 2 − 4 ac
0

U 3(x + 9000) =
12000 × 2820
1200
O
2a 3(x + 9000) = 28200
−10 ± 100 + 1500 x + 9000 = 9400
H
l = x = 9400 – 9000
2 x = Rs 400
−10 ± 40 B invested an additional amount of
_T

l =
2 Rs 400
−10 + 40 −10 − 40 8. Ans. (C)
l = ; (not valid)
2 2 Sol. Ratio of milk and water = 5 : 1

C

l = 15m Let quantity of milk be 5x and water


Side of the square = 15m be x
PS

Perimeter of the square New quantity of milk = 5x + 24


= 4a = 4 × 15 = 60m New quantity of water = x
7. Ans. (E) Resultant ratio of milk : water
Sol. Let the additional amount B invested = 13 : 2
U

be Rs x 5x + 24 : x = 13 : 2
A invested Rs 4200 for 4 months and 5 x + 24 13
=
@

(4200 + 1000) for 6 months. x 2


∴ Total investment for 10 months = 2(5x + 24) = 13x
= 4200 × 4 + 5200 × 6 10x + 48 = 13x
= Rs 48,000 ... (1) 48 = 13x– 10x
B’s investment for 6 months 48 = 3x
= 3600 × 6 = 21600 16 = x
B’s investment for 4 months ∴ Quantity of milk = 5x + 24
= (3600 + x)4 = 5 × 16 + 24
= 14400 + 4x = 104l
Total investment for 10 months Quantity of water = x = 16l
= 21600 + 14400 + 4x Total mixture = 104 + 16 = 120l
= 3600 + 4x ... (2)
 15l of mixture is taken out, quantity
Additional amount invested by
left
C = 2x ( ratio of B : C is 1 : 2)
= 120 – 15 = 105l
730 ✫ Quantitative Aptitude

Resultant quantity of milk – 3) matches = 46 × 25 = 1150


13 ∴ Runs scored by E in the last 3 matches
= × 105 = 1540 – 1150
15
= 91 litres = 390
9. Ans. (A) Runs scored by E in the 26th and
27th match
Total balls faced by D 3 = 126 + 127 (  runs are below
Sol. Total balls faced by F = = 253 128 and no two
4
scores are equal)
Let balls faced by D be 3x and balls Minimum possible runs scored by E

TS
faced by F be 4x in the 28th match = 390 – 253
Total runs scored = 137
Strike rate = × 100
Total balls faced 11. Ans. (E)

H
runs scored by D Sol. Let number of balls faced by A = x
For D → 72 = × 100 Number of balls faced by A = Total
3x
runs – 74 ... (1)

G
3x × 72 216 x Strike rate for A = 129.6
∴ Runs scored by D = =
100 100 Total runs scored
For F → 66 =
runs scored by F
4x
× 100
U Strike rate =
Total balls faced
Total runs scored
× 100
O
4 x × 66 264 x 129.6 = × 100
Runs scored by F = = x
100 100 129.6x
Total runs scored = = 1.296x
H
264 x 216 x 48 x 100
Difference = – = ∴ (1) ⇒ x = 1.296x –74
100 100 100
_T

Required percentage x – 1.296x = –74


difference – 0.296x = –74
= × 100
runs scored by D 74
C

x =
48x 0.296
= 100 × 100
PS

74000
216x x = = 250
296
100 Substituting in (1)
48 x x = Total runs – 74
U

= × 100
216 x Total runs = x + 74
200 = 250 + 74
= 22 2 9 %
@

=
9 = 324
10. Ans. (A) 324
Sol. Average runs scored by E in the ∴ Average runs =
No. of matches
tournament = 55 (given)
324
Number of matches played by = = 40.5
E = 28 (given) 8
Total number of runs made by 12. Ans. (C)
E = 55 × 28 = 1540 Sol. Let number of balls faced by batsman
If runs scored by E in the last 3 B in the first 10 matches = x
matches are not considered, then Then, number of balls faced by B in
average runs will decrease by 9 (ie) the last 10 matches = x
average = 55 – 9 = 46 Total runs scored
∴ Total number of runs scored in 25 (28 Strike rate = × 100
Total balls faced
Solved Test Papers of Various Examinations ✫ 731

For the first 10 matches 1


In 1 day 1 man can do of the
Total runs scored 120
120 = × 100 work ... (1)
x
12 children can do the work in 16
120 x
∴ Total runs scored = ... (1) days
100
For the last 10 matches ∴ 1 child can do the work in 12 × 16
Total runs scored = 192 days
150 = × 100 1
x ∴ In 1 day 1 child can do of the
192

TS
150 x
∴ Total runs scored = ... (2) work ... (2)
100 8 women can do the work in 20 days
Number of matches played by ∴ 1 woman can do the work in 8 × 20
B = 20

H
= 160 days
Average runs scored by B = 81 1
∴ Total runs scored by ∴ In 1 day 1 woman can do of the
160

G
B = 20 × 81 = 1620 ... (3) work ... (3)
From (1), (2) and (3) we get In 1 day 5 men can do
120 x
100
+
150 x
100
= 1620
U 5 ×
1
120
=
1
24
of the work ...(4)
O
[from (1)]
120x + 150x = 16200
In 1 day 12 children can do
270x = 16200
H
12 × 1 1
16200 = = of the work [from (2)]
x = = 600 192 16
270
_T

In 1 day 4 women can do


∴ Total number of balls faced by
1 1
B = 2x = 4 × = of the work
= 2 × 600 = 1200 160 40
... (5)
C

13. Ans. (C) [from (3)]


Runs scored In 1 day 5 men and 12 children can
PS

Sol. Strike rate = × 100 1 1 2+3 5


Balls faced
do + = = of the
Runs scored 24 16 48 48
114 = × 100 work
400
U

400 × 114 In 4 days 5 men and 12 children can do


Runs scored = = 456 4×5 5
100
@

= of the work
Total runs 48 12
Average runs = 5
Number of matches Remaining work = 1 –
12
456 7
38 = =
Number of matches 12
456 In 1 day 5 men, 4 children and 4
∴ Number of matches = = 12 1 4 1
38 women can do = + +
24 192 40
14. Ans. (E) [from (4), (2) and (5)]
Sol. 10 men can complete the work in 12
days 1 1 1
= + +
24 48 40
∴ 1 man can complete the work in 10 + 5 + 6
12 × 10 = 120 days =
240
732 ✫ Quantitative Aptitude

21 7 ∴ 5 years hence let daughters age be


= = 7x and sons age be 6x
240 80
7 Present age of daughter = 7x – 5
of the work can be done in 1 day Present age of son = 6x – 5
80
Poorvi’s age 8 years ago
∴ Whole work will be done in
= 7x – 5 + 6x – 5
1
× 1 = 13x – 10
7
80 Present age of Poorvi
80 = 13x – 10 + 8

TS
= days = 13x – 2
7
7 Age of Poorvi’s husband
∴ of the work will be done in
12 = 13x – 2 + 7 = 13x + 5
7 80 According to the question

H
×
12 7 13x + 5 = 3 (6x – 5)
20 13x + 5 = 18x – 15
= 6 2 3 days

G
=
3 20 = 5x
15. Ans. (B) 4 = x
Sol. Let votes got by B = x
Then votes got by A = x + 600 U Present age of daughter
= 7x – 5 = 7 × 4 – 5 = 23 years
O
Total votes = x + x + 600 = 2x + 600 17. Ans. (D)
If B had got 40% more votes, then Sol. Distance between X and Y = 20 km
H
40 140 x Distance between Y and Z = 40 km
votes got by B = x + ×x=
100 100 Speed of B in still water = 10 km/hr
... (1) Let the speed of the current be x km/hr
_T

Then votes for A Speed of B upstream = (10 – x) km/hr


40 Speed of boat A in still water
= (x + 600) – × x = Speed of B upstream
100
C

60 x + 60000 = (10 – x) km/hr


= ... (2) Speed of A downstream
100
PS

According to the question (1) = (2) = (10 – x) + x = 10 km/hr


140 x 60 x + 60000 Distance
⇒ = For A, time taken =
100 100 Speed
20
U

140x – 60x = 60000 = = 2 hrs ... (1)


80x = 60000 10
For B time taken
@

x = 750
Total votes = 2x + 600 40
= ... (2)
= (2 × 750) + 600 10 − x
According to the question
= 2100
Time taken by B – time taken by
But only 60% cast their votes
A = 3 hrs
60
∴ 2100 = × total voters 40
100 – 2 = 3 [from (1) and (2)]
2100 × 100 10 − x
∴ Total voters = 40 – 20 + 2x = 30 – 3x
60 5x = 10
= 3500
x = 2 km/hr
16. Ans. (B)
Speed of boat A in still water
Sol. 5 years hence, ratio of ages of
= 10 – x = 10 – 2
daughter and son = 7 : 6
= 8 km/hr
Solved Test Papers of Various Examinations ✫ 733

18. Ans. (D) For university B


Sol. In university A applications for 20
commerce (c), science (S) and S = 1500 – × 1500
100
engineering (E) 3 = 1500 – 300
70 3 = 1200 ... (7)
= × total applications
100 But A = C – 40% C
Applications for Arts (A),
60
management (H) and law (L) 780 = × C
= (100 – 70)% of total applications 100

TS
= 30% 780 × 100
30 ∴ C =
∴ 800 + 750 + 400 = × total 60
100
applications C = 1300 ... (8)

H
30 A = 15% of total applications (given)
1950 = × total applications
100 15
780 = × total applications

G
∴ Total applications 100
1950 × 100 ∴ Total applications
= = 6500 ... (I)


30
C + S + E =
70
× 6500 = 4550 U =
780 × 100
15
= 5200 ... (II)
O
100
... (1) ∴ C + S + E + A + L + M = 5200
40 1300 + 1200 + E + A + L + M = 5200
H
M = E – × E (given) [from (6), (7)]
100
60 E + A + L + M = 2700 ... (9)
_T

M = × E But E + M = 2 (A + L) (given)
100
60 E+M
750 = × E A + L =
100 2
750 × 100
C

∴ E = = 1250 ... (2) Substituting in (9)


60
E+M
PS

∴ C + S = 4550 – 1250 E + M + = 2700


[from (1) and (2)[ 2
C + S = 3300 ... (3)
E+E
20 × S E + E + = 2700
U

But C = S + (given) 2
100 [M = E (given)]
@

120S 3E = 2700
C = ... (4)
100 E = 900 ... (10)
Substituing in (3) ∴ Total number of applications for E
120S and S in A
+ S = 3300 Total number of applications for E
100
120S + 100S = 330000 and S in B
220S = 330000 1250 + 1500
= [from (2), (5), (10), (7)]
330000 900 + 1200
S =
220 2750
S = 1500 ... (5) =
2100
120 × 1500
∴ C= = 1800 [from (4)] ... (6) 55
100 = = 55 : 42
42
734 ✫ Quantitative Aptitude

19. Ans. (B) ∴ M + L = 900 + 120 = 1020


Sol. Number of applications received for ∴ Difference = 1150 – 1020 = 130
commerce in B 23. Ans. (D)
= 1300 [from (8)] Sol. Number of males and females having
Number of applications received for lifetime membership in club
commerce in A 25 20
= 1800 [from (6)] D = × × 6300
100 100
Difference = 1800 – 1300 = 500
Required percentage = 315
Number of females who have lifetime

TS
500 membership in D = 126
= × 100 = 27 7 9 %
1800 Difference = 315 – 126 = 189
20. Ans. (E) Required percentage
Sol. Number of female students in A =

H
189
5 = × 100 ... (1)
× (number of applications for arts Male members
8

G
in A) 20
Male members = × 3600 = 720
100
5
=
8
× 800 = 500
Number of female students in B = U ∴ (1) ⇒
189
720
× 100 =
105
4
1
= 26 %
4
O
24. Ans. (A)
500 – 120 = 380
Sol. Number of female students in
∴ Number of male applicants for arts
H
club A
in B
= 780 – 380 21 24
_T

= 400 = × 6300 – × 3600


100 100
21. Ans. (A)
Sol. Total applications for university A = = 1323 – 864
6500 [from (I)] = 459
C

Total applications for university B = Number of female students in


5200 [from (II)] club B
PS

Difference = 6500 – 5200 = 1300


33 30
1300 = × 6300 – × 3600
Required percentage = × 100 100 100
5200
U

= 25% = 2079 – 1080


22. Ans. (C) = 999
@

Sol. Total number of applications for Number of female students in


management and law in club C
A = 750 + 400 (given)
= 1150 14 18
= × 6300 – × 3600
Total number of applications for the 100 100
same courses in B ⇒ = 882 – 648
Number of management applications
= Number of engineering applications = 234
= 900 from (10) ∴ Average number of females in A, B
But E + M = 2(A + L) (given) and C
900 + 900 = 2(780 + L) 459 + 999 + 234
1800 = 1560 + 2L =
3
240 = 2L 1692
120 = L = = 564
3
Solved Test Papers of Various Examinations ✫ 735

25. Ans. (C) 28. Ans. (A)


Sol. Number of male members in club A Sol. Let speed of Abhay be x km/hr
24 Distance = 30 km.
= × 3600 = 864 Time taken by Abhay
100
Number of male members in club C Distance 30
= =
18 Speed x
= × 3600 = 648
100 If Abay doubles his speed, time taken
Let the number of males increase by 30
x in A and C =
2x

TS
According to the question
According to the question,
864 + x 13
=
648 + x 10 30 30
– = 3

H
8640 + 10x = 8424 + 13x x 2x
216 = 3x 60 – 30 = 6x
72 = x

G
5 = x
∴ Number of males in club C in 2012 ∴ Abhay’s speed is 5 km/hr.
= 72 + 648 = 720 29. Ans. (C)
26. Ans. (A)
Sol. Number of males and females in U Sol. Total C.P. of A and B = Rs 8000
Let C.P. of A be Rs x
O
club B Then C.P. of B = (8000 – x) Rs
33 For A:
= × 6300
H
100 C.P. = Rs x
= 2079
20 20 x
_T

2079 Project = 20% = × C.P. =


∴ Central angle = × 360° 100 100
6300 For B
594 C.P. = (8000 – x) Rs
=
C

5 Loss = 12%
= 118.8°
27. Ans. (A) 12
PS

= (8000 – x)
Sol. Number of females in club E 100
= Number of males and females in ∴ There is no gain or loss
E – Number of males in E
20 x 12
U

12 8 = (8000 – x)
= × 6300 – × 3600 100 100
100 100
@

5x = 3(8000 – x)
= 756 – 288
5x = 24000 – 3x
= 468
Number of males in B 8x = 24000
x = 3000
30
= × 3600 = 1080 ∴ C.P. of A = Rs 3000
100
C.P. of B = Rs 5000
Difference = 1080 – 468
For A
= 612
C.P. = Rs 3000 Profit = 20%
612
Required percentage = × 100 120
1080 ∴ S.P. = × 3000 = Rs 3600
100
170 2 Total C.P. = Rs 8000
= = 56 %
3 3 Project = 25%
736 ✫ Quantitative Aptitude

125 21P
C.I. = ... (2)
Total S.P. = × 8000 100
100 According to the question
= Rs 10,000
∴ S.P. of B = 10,000 – S.P. of A 6 xP 21P
= ×2 [from (1) and (2)]
= 10,000 – 3600 100 100
= Rs 6400
3x = 21
30. Ans. (C)
Sol. Let the sum be P x = 7%
If x = x + 2 = 7 + 2 = 9%

TS
PTR P × 6×x 6xP
S.I. = = = ...(1) P ×6 ×9 54P
100 100 100 Then S.I. = = ... (3)
100 100
n
r According to the question

H
C.I. = P 1+ −1
100 54P 21P
– = 3960
100 100

G
2
10
= P 1+ −1 33P = 396000
100

= P
121
100
−1 U P =
396000
33
= Rs 12,000
O
H
C _T
PS
U
@
Solved Test Papers of Various Examinations ✫ 737

Life Insursance Corporation (LIC)


AAO Examination, March 2016
Directions (Q. 1-5): Refer to the graph and passed the exam from the same schools
answer the given questions. together?
Data given are regarding the number of (1) 4 : 3 (2) 16 : 15 (3) 8 : 3
students who had appeared/passed in a (4) 15 : 11 (5) 16 : 13
certificate exam from five schools 6. Out of the total population, 55% were males.

TS
in the year 2008 Out of the total number of females, 60% were
300
literate and the remaining 360 were illiterate.
If 70% of the males were literate, then how
Number of students
(Appeared/Passed)

250
many males were illiterate?

H
200
(1) 320 (2) 330 (3) 240
150
(4) 220 (5) 300

G
100

50 Directions (Q. 7-11): What will come in place


A B C D E
U of question mark (?) in the given number series?
7. 2 3 7 22 89 ?
O
Appeared 200 260 220 160 120
(1) 446 (2) 412 (3) 324
Passed 160 200 190 110 90 (4) 319 (5) 298
H
1. The number of students who appeared in 8. 196 100 52 28 ? 10
the exam from school A was what per cent (1) 14 (2) 18 (3) 12
_T

more than that who appeared from school (4) 16 (5) 15


D?
9. 140 133 119 98 ? 35
(1) 20 (2) 25 (3) 15
(1) 70 (2) 45 (3) 30
C

(4) 30 (5) 10
(4) 72 (5) 61
2. What is the average number of students who
PS

passed the exam from school C, D and E? 10. 300 325 276 357 236 ?
(1) 150 (2) 110 (3) 120 (1) 302 (2) 426 (3) 415
(4) 130 (5) 140 (4) 496 (5) 405
U

3. Out of all the given schools, for which school 11. 14 ? 7 14 56 448
was the number of students taking the exam (1) 9 (2) 12 (3) 25
the maximum?
@

(4) 7 (5) 14
(1) B (2) E (3) O
(4) A (5) C 12. 8 men can finish a piece of work in 21 days.
14 men started working and after 3 days they
4. In the year 2009, the number of students who were replaced by 9 women. These 9 women
appeared in the exam from school E was 15%
finished the remaining work in 24 days. In
more than that from the same school in the
previous year. How many students appeared how many days can 9 women finish the
in the exam from school E in 2009? whole work?
(1) 136 (2) 138 (3) 140 (1) 24 days (2) 26 days
(4) 142 (5) 148 (3) 36 days (4) 32 days
(5) None of these
5. What is the ratio of the total number of
students who appeared in the exam from 13. The radius of a circular field is equal to the
schools B and C together to that of those who side of a square field. If the difference
738 ✫ Quantitative Aptitude

between the area of the circular field and the The data given in the table is for the
area of the square field is 105 m2, what is the month of March 2015
perimeter of the circular field? (in metres) Company Total number Number of male
(1) 132 (2) 80 (3) 44 of employees employees
(4) 176 (5) 112
A 5250 2520
14. There are 5 consecutive odd numbers. If the
B 4800 2140
difference between the square of the average
C 3000 1560
of the first two odd numbers and the square
D 2400 840
of the average of the last two odd numbers
E 2000 1020

TS
is 492, what is the smallest odd number?
F 3280 1800
(1) 37 (2) 42 (3) 41
(4) 35 (5) 39 19. The total number of male employees in
Company B and A together is what per cent
15. A boat can travel 12.8 km downstream in 32

H
1 of the total number of employees in
minutes. If the speed of the current is of Company E?
5
(1) 233 (2) 226 (3) 223

G
the speed of the boat in still water, what
distance (in km) can the boat travel in 27 (4) 234 (5) 222
minutes?
(1) 7.5
(4) 9
(2) 10.2
(5) 6.5
(3) 8.5
U 20. The number of female employees in
company F is approximately what per cent
less than the number of employees in
O
16. A batsman played three matches in a company E?
tournament. The ratio of the score of 1st to (1) 26 (2) 45 (3) 15
H
2nd match was 8 : 9 and that of the score of (4) 35 (5) 50
2nd to 3rd match was 3 : 2. The difference 21. What is the average number of female
_T

between the 1st and the 3rd match was 16 employees in company A, C and E together?
runs. What was the batsman’s average score (1) 1724 (2) 1716 (3) 1712
in all the three matches? (4) 1600 (5) 1680
1
C

(1) 40 (2) 58 (3) 60 22. What is the difference between the number
4 of female employees in company A and that
1 1
PS

(4) 45 (5) 61 in company B?


5 3 (1) 70 (2) 40 (3) 100
17. The ratio of the present age of A to B is 2 : 3. (4) 90 (5) 80
B’s age twenty one years hence will be twice
23. If in April 2015, the number of male
U

A’s age six years hence. What is the


employees in Company C increased by 10%
difference between the present ages of A and
and the number of total employees in the
@

B?
company remained the same, what was the
(1) 15 years (2) 9 years (3) 8 years number of female employees?
(4) 10 years (5) 6 years (1) 1428 (2) 1314 (3) 1296
18. The cost price of article A is ` 100 more than (4) 1284 (5) 1410
the cost price of article B. Article A was sold
Directions (Q. 24-28): In each of these questions,
at 40% profit and article B was sold at 40%
two equations I and II are given. You have to
loss. If the overall profit earned after selling
solve both the equations and give answer.
both the articles is 5%, then what is the cost
(1) if x > y (2) if x ≥ y
price of article B?
(3) if x < y (4) if x ≤ y
(1) ` 300 (2) ` 400 (3) ` 250
(5) if x = y or no relation can be established
(4) ` 350 (5) ` 850
between x and y.
Direction (Q. 19-23): Based on the following 24. (I) x2 = 10
table, answer the given questions. (II) y2 – 9y + 20 = 0
Solved Test Papers of Various Examinations ✫ 739

25. (I) 2x2 – 15x + 27 = 0 x% pa S.I. The compound interest earned


(II) 2y2 – 13y + 20 = 0 from scheme A is twice that earned from
26. (I) 9x2 – 21x + 10 = 0 scheme B. What is the value of x?
(II) y2 – 8y + 15 = 0 (1) 10 (2) 11 (3) 15
(4) 12 (5) 19
27. (I) 2x2 – 13x + 15 = 0
(II) 2y2 – 11y + 12 = 0 30. There are 7 red balls and 8 yellow balls in a
bag. Two balls are simultaneously drawn at
28. (I) 2x2 + 7x + 6 = 0 random. What is the probability that both the
(II) 2y2 + 17y + 30 = 0 balls are of the same colour?

TS
29. A certain sum is invested for 2 years in 3 13 3
scheme A at 20% pa compound interest (1) (2) (3)
18 30 10
compounded annually. The same sum is also 7 9
invested for the same period in scheme B at (4) (5)

H
15 20

Answers and Explanatory Notes

G
1. Ans. (2) = 260 + 220 = 480
Sol. Number of students who appeared
in the exam from school A = 200
Number of students who appeared U Total number of students who passed
the exam from B and C
= 200 + 190 = 390
O
in the exam from school D = 160 Required ratio
Difference = 200 – 160 = 40 480 16
=
H
Required percentage = = 16 : 13.
390 13
Difference 6. Ans. (2)
_T

= × 100 Sol. Population of males = 55%


Students in D
∴ Female population = 100 – 55 = 45%
40
= × 100 Let the total population = x
160 45 9x
C

= 25% Number of females = ×x =


100 20
2. Ans. (4) Literate females = 60%
PS

Sol. Total number of students who passed Illiterate females = 40%


from C, D and E 40 9 x
= 190 + 110 + 90 ∴ × = 360
100 20
= 390 x = 2000
U

390 Literate males = 70%


∴ Average = = 130 ∴ Illiterate males = 30% of 55%
3
@

3. Ans. (1) 30 55
Sol. From the graph, students from school = × × 2000
100 100
B were the maximum. = 330.
4. Ans. (2) 7. Ans. (1)
Sol. Number of students who appeared Sol. The given series is
in the exam from school E in 2008 2 3 7 22 89 ?
= 120 2 × 1 + 1 = 3
Number of students who appeared 3 × 2 + 1 = 7
in the exam from school E in 2009 7 × 3 + 1 = 22
15
= 120 + × 120 = 138 22 × 4 + 1 = 89
100 89 × 5 + 1 = 446
5. Ans. (5)
8. Ans. (4)
Sol. Total number of students who
Sol. The series is
appeared in the exam from B and C
196 100 52 28 ? 10
740 ✫ Quantitative Aptitude

(196 ÷ 2) + 2 = 98 + 2 = 100 13. Ans. (3)


(100 ÷ 2) + 2 = 50 + 2 = 52 Sol. Let r be the radius of the circular
(52 ÷ 2) + 2 = 26 + 2 = 28 field and a the side of the square
(28 ÷ 2) + 2 = 14 + 2 = 16 ∴ r = a
(16 ÷ 2) + 2 = 8 + 2 = 10 Area of circular field = πr2
9. Ans. (1) Area of the square field = a2 = r2
Sol. The series is Difference in area = πr2 – r2
140 133 119 98 ? 35 105 = r2 (π – 1)
140 – 7 = 133
133 – (7 × 2) = 119 105
= r2

TS
119 – (7 × 3) = 98 22
−1
98 – (7 × 4) = 70 7
49 = r2 => r = 7 m
70 – (7 × 5) = 35
Perimeter of the circular field = 2πr

H
10. Ans. (5)
22
Sol. The series is = 2 × × 7 = 44 m.
300 325 276 357 236 ? 7

G
14. Ans. (1)
300 + (5)2 = 325
Sol. Let the 5 consecutive odd numbers
325 – (7)2 = 276
be x, x + 2, x + 4, x + 6, x + 8
276 + (9)2 = 357
357 – (11)2 = 236
U Average of the first two numbers
x+x+2
O
236 + (13)2 = 405 = = x + 1
11. Ans. (4) 2
Average of the last two numbers
Sol. The series is
H
14 ? 7 14 56 448 x+6+x+8
= = x + 7
14 × 0.5 = 7 2
_T

7 × 1 = 7 Difference of their squares


7 × 2 = 14 = (x + 7)2 – (x + 1)2
14 × 4 = 56 = x2 + 14x + 49 – x2 – 2x – 1
56 × 8 = 448 = 12x + 48
C

12. Ans. (4) According to the question


Sol. 8 men can do a piece of work in 21 12x + 48 = 492
PS

days 12x = 492 – 48


∴ 1 man can do the work in 8 × 21 12x = 444
= 168 days x = 37
1 Smallest odd number is 37.
U

∴ In 1 day 1 man can do of the 15. Ans. (4)


168
work Sol. Let the speed of the boat in still
1
@

∴ 14 men can do 14 × of the water be x km/hr


168 x
work in 1 day Speed of the current = km/hr.
14 1 5
In 3 days work done = ×3= Speed of the boat going downstream
168 4
of the work x 6x
1 3 = x + =
Remaining work = 1 – = 5 5
3 4 4 Distance
of the work was completed by 9 Speed =
4 Time
women in 24 days 6x 12.8
∴ The whole work can be completed in =
5 32
4 hrs.
× 24 = 32 days. 60
3 6x 60 × 12.8
=
5 32
Solved Test Papers of Various Examinations ✫ 741

5 × 60 × 12.8 A was sold at 40% profit


x = ∴ S.P. of A = C.P. + profit
6 × 32
x = 20 km/hr. 40
In 60 minutes distance travelled = (x + 100) + (x + 100)
100
= 20 km (140)
In 27 minutes distance travelled = (x + 100)
100
27 7
= × 20 = 9 km. = (x + 100)
60 5
16. Ans. (5) B was sold at 40% loss

TS
Sol. Ratio of 1st match to 2nd match S.P. of B = C.P. – loss
= 8 : 9 40
Ratio of 2nd match to 3rd match = x – × x
= 3 : 2 100
3x

H
Then 1st match : 2nd match : 3rd =
match = 24 : 27 : 18 5
C.P. of A and B = x + x + 100

G
= 8 : 9 : 6 = 2x + 100 … (1)
(If a : b = x : y and b : c = p : q then S.P. of A and B
a : b : c = xp : py : qy)
Let the scores in the first match be
8x, second match be 9x and third U =
7
5
(x + 100) +
10 x + 700
3x
5
O
match be 6x =
Difference between first and third 5
= 2x + 140 … (2)
H
match = 16 runs
Profit = S.P. – C.P.
8x – 6x = 16
= (2x + 140) – (2x + 100)
x = 8
_T

(from (1) and (2)


Average score in all three matches
= Rs 40 … (3)
8 x + 9x + 6 x Overall profit = 5%
=
3 5
C

8 ×8 +9×8 +6×8 = (C.P. of A and B)


= 100
3 1 x + 50
PS

64 + 72 + 48 184 1 = (x + x + 100) = … (4)


= = = 61 20 10
3 3 3 From (3) and (4)
17. Ans. (2) x + 50
U

Sol. Let the present age of A be 2x and = 40


10
B be 3x x = 400 – 50 = Rs 350
A’s age 6 years hence = 2x + 6
@

The C.P. of article B is Rs 350.


B’s age 21 years hence = 3x + 21 19. Ans. (1)
According to the question Sol. Total number of male employees in
3x + 21 = 2(2x + 6) A and B = 2520 + 2140 = 4660
3x + 21 = 4x + 12 Total number of employees in E
21 – 12 = 4x – 3x = 2000
9 = x Required percentage
A’s present age = 2x = 18
4660
B’s present age = 3x = 27 = × 100 = 233%
Difference of their ages 2000
20. Ans. (1)
= 27 – 18 = 9 years. Sol. Number of female employees in F
18. Ans. (4) = 3280 – 1800 = 1480
Sol. Let the C.P. of article B be Rs x Number of employees in E = 2000
Then C.P. of article A = x + 100 Difference = 2000 – 1480 = 520
742 ✫ Quantitative Aptitude

Required percentage (y – 4) (2y – 5) = 0


520 y – 4 = 0; 2y – 5 = 0
= × 100 = 26% less.
2000 5
21. Ans. (2) y = 4; y = = 2.5 … (2)
Sol. Female employees in A, C and E 2
From (1) and (2) no relationship can
= (5250 – 2520) + (3000 – 1560) be established.
+ (2000 – 1020) 26. Ans. (3)
= 2730 + 1440 + 980 Sol. (I) 9x2 – 21x + 10 = 0
= 5150 9x2 – 15x – 6x + 10 = 0

TS
5150 3x (3x – 5) – 2 (3x – 5) = 0
Average =
3 (3x – 5) (3x – 2) = 0
2 3x – 5 = 0; 3x – 2 = 0
= 1716 3 = 1716. 5 2

H
22. Ans. (1) x = ; x =
3 3
Sol. Number of female employees in A x = 1.6; x = 0.6 … (1)

G
= 5250 – 2520 = 2730 (II) y2 – 8y + 15 = 0
Number of female employees in B y2 – 3y – 5y + 15 = 0
= 4800 – 2140 = 2660 y (y – 3) – 5 (y – 3) = 0

23.
Difference = 2730 – 2660 = 70.
Ans. (4) U (y – 3) (y – 5) = 0
y – 3 = 0; y – 5 = 0
O
Sol. In April 2015, number of male y = 3; y = 5 … (2)
employees in C From (1) and (2) x < y.
H
10 27. Ans. (5)
= 1560 + × 1560 Sol. (I) 2x2 – 13x + 15 = 0
100
2x2 – 10x – 3x + 15 = 0
_T

= 1560 + 156
= 1716 2x (x – 5) – 3 (x – 5) = 0
Number of female employees (x – 5) (2x – 3) = 0
= 3000 – 1716 = 1284. x – 5 = 0; 2x – 3 = 0
C

24. Ans. (3) 3


x = 5; x = = 1.5 … (1)
Sol. (I) x2 = 10 2
PS

(II) 2y2 – 11y + 12 = 0


x = ± 10 = ± 3.1 … (1)
2y2 – 8y – 3y + 12 = 0
(II) y2 – 9y + 20 = 0
2y (y – 4) – 3 (y – 4) = 0
y2 – 4y – 5y + 20 = 0
(y – 4) (2y – 3) = 0
U

y (y – 4) – 5 (y – 4) = 0 y – 4 = 0; 2y – 3 = 0
(y – 4) (y – 5) = 0 3
y – 4 = 0; y – 5 = 0 y = 4; y = = 1.5 … (2)
@

y = 4; y = 5 … (2) 2
From (1) and (2) x = y
From 1 and (2) x < y
28. Ans. (1)
25. Ans. (5)
Sol. (I) 2x2 + 7x + 6 = 0
Sol. (I) 2x2 – 15x + 27 = 0 2x2 + 4x + 3x + 6 = 0
2x2 – 6x – 9x + 27 = 0 2x (x + 2) + 3 (x + 2) = 0
2x (x – 3) – 9 (x – 3) = 0
(x + 2) (2x + 3) = 0
(x – 3) (2x – 9) = 0
x + 2 = 0; 2x + 3 = 0
x – 3 = 0; 2x – 9 = 0
3
9 x = –2; x = – = –1.5 … (1)
x = 3; x = = 4.5 … (1) 2
2 2
(II) 2y + 17y + 30 = 0
(II) 2y2 – 13y + 20 = 0 2y2 + 12y + 5y + 30 = 0
2y2 – 8y – 5y + 20 = 0 2y (y + 6) + 5 (y + 6) = 0
2y (y – 4) – 5 (y – 4) = 0 (y + 6) (2y + 5) = 0
Solved Test Papers of Various Examinations ✫ 743

y + 6 = 0; 2y + 5 = 0 30. Ans. (4)


5 Sol. Total number of balls = 7 + 8 = 15
y = –6; y = – = –2.5 … (2)
2 P (both balls are of the same colour)
From (1) and (2) x > y. = P (both balls are red or both balls
29. Ans. (2) are yellow)
Sol. Let the sum invested be P 7
C2 8
C
n 2 = 15 + 15 2
r 20 C2 C2
C.I. = P 1 + −1 = P 1+ −1
Favourable outcomes
100 100 Probability =
Total number of outcomes

TS
36 − 25 11P 7! 8!
= P = +
25 25 (7 − 2)! 2 ! (8 − 2)! 2 !
=
15 !
PTR P × 2 × x Px

H
S.I. = = = (15 − 2)! 2 !
100 100 50 n!
n
Cr =

G
According to the question ( n − r )! r !
11P 2 Px 7×6 8×7
+
25
=
50
2 Px U = 2
15 × 14
2 = 21 + 28
105
O
11P = 2
2
49 7
⇒ x = 11% = =
H
105 15
C _T
PS
U
@
744 ✫ Quantitative Aptitude

National Insurance Company Ltd.


Administrative Officers Examination, April 2015
Directions—(Q. 1-5) Study the table to answer sold (non-fiction and fiction category
the given questions. together) in the year 2013?
(A) 62% (B) 55% (C) 65%
Year Data related to books sold through
Online Mode and Offline Mode in a city
(D) 60% (E) 25%
Number of books Percentage of Respective Ratio of 5. What is the difference between the number
of fiction books sold in the year 2012 and

TS
sold-in thousands books sold number of Non-
(Online mode + through Online Fiction books sold 2013 together and the number of non-fiction
Offline mode) Mode to number of books sold in the year 2011 and 2012
fiction books sold
(Online Mode + together?
(A) 873 (B) 876 (C) 573

H
Offline Mode)
(D) 673 (E) 676
2010 690 40 2:3
2011 720 57.5 4:5 Directions—(Q. 6-10) What will come in place

G
2012 945 60 7:8 of question-mark (?) in the given number series?
2013 1240 75 1:3
6. 16 37 62 96 146 ?
2014 1600 79

1. In the years 2010 and 2011 together, the


3:5

U (A) 229
(D) 213
(B) 217
(E) 232
(C) 221
O
average number of books bought by each 7. 23 15 22 58 224 ?
3 (A) 1168 (B) 984 (C) 1208
buyer through offline mode was 5. If th of
H
5 (D) 1076 (E) 1112
the number of buyers buying books from
offline mode, during the years 2010 and 2011 8. 541 269 133 65 ? 14
(A) 31 (B) 33 (C) 36
_T

together were of the age group more than or (D) 35 (E) 29


equal to 30 years, then how many were less
than 30 years of age? 9. 28 16 28 76 274 ?
(A) 1269 (B) 1211 (C) 1351
(A) 42000 (B) 41000 (C) 57600
C

(D) 1243 (E) 1079


(D) 42500 (E) 45000
10. 4 11 36 129 ? 2755
PS

2. What is the respective ratio of the total (A) 528 (B) 574 (C) 582
number of books sold through online mode (D) 544 (E) 516
in the year 2010 and 2011 together and the
Directions (Q. 11-15) Refer to the pie charts and
total number of books sold through offline
answer the given questions.
U

mode in the same years together?


(A) 25 : 32 (B) 24 : 25 (C) 23 : 24 Total Number of Participants : 500
@

(D) 35 : 36 (E) 25 : 28
Distribution of total number of
3. What is the approximate percentage participants from 6 colleges in
decrease in the number of books sold Youth Festival ‘PQR’:
through offline mode in the year 2014 from
the year 2010?
(A) 15 (B) 19 (C) 21 Z A
(D) 23 (E) 25 18% 20%

4. In the year 2013 out of the total number of Y


4 10% B
books sold in the fiction category, th of 12%
15
the books sold were written by Indian C
X
authors. Number of books written by the 25% 16%
foreign authors in the fiction category form,
what per cent of the total number of books
Solved Test Papers of Various Examinations ✫ 745

participated from college X. What is the


Distribution of total number of respective ratio between the number of
participants in 6 competitions participants from college A in Street Play
held at Youth Festival ‘PQR’ and the number of participants from college
X in the same competition?
Mimicry (A) 11 : 5 (B) 10 : 5 (C) 5 : 2
10% Dance (D) 10 : 7 (E) 23 : 16
18% 3
Gaming 15. of the total participants in calligraphy
5 5
19%

TS
Singing competition are females and of the total
14% 7
participants in singing competition are
Calligraphy females. What is the difference between the
Street play
number of females participating in

H
25% 15%
calligraphy and singing competitions?
(A) 5 (B) 20 (C) 12

G
(D) 10 (E) 15
Note—Youth Festival ‘PQR’ had only 6
competitions and every student could participate
only in one competition.
U 16. Puneet distributed a sum of money among
his wife, two sons and one daughter and kept
some money for himself. 20% of the total
O
11. Out of the total number of participants in
money that he had, he gave to his wife and
dance competition, 20% could qualify for the
kept 22% of it for himself. 60% of the
H
final round of the competition and these
remaining money he distributed among his
participants were from colleges B and Y
two sons and gave the remaining to his
_T

only. The respective ratio between the


daughter. If the daughter got ` 2,940 more
participants who could qualify for the final
than the money he kept for himself, what was
round of the dance competition from college
the total money that he distributed among
B and Y is 2 : 3. If two-seventh of the total
C

his two sons? (in `)


participants from college Y, participated in
(A) ` 87,178 (B) ` 87,108
Dance competition, how many participants
PS

(C) ` 85,260 (D) ` 86,800


of college Y could not qualify for the final
(E) ` 86,786
round of dance competition?
(A) 3 (B) 6 (C) 8 17. If the lateral surface of a cylinder is 94.2 cm2
U

(D) 10 (E) 12 and its height is 5 cm, then find radius of its
base. [Use π = 3.14]
12. What is the central angle corresponding to
@

(A) 5 cm (B) 7 cm (C) 3 cm


the number of participants in gaming
(D) 9 cm (E) 4 cm
competition?
(A) 69.2° (B) 68.8° (C) 64.8° 18. The length of a rectangle is twice the diameter
(D) 58.2° (E) 68.4° of a circle. The circumference of the circle is
13. The number of participants from college X equal to the area of square of side 22 cm.
is what per cent less than the total number What is the breadth of the rectangle if its
of participants from college C and A perimeter is 668 cm?
together? (A) 24 (B) 26 (C) 52
(A) 33.3% (B) 35.5% (C) 22.2% (D) 40 (E) 22
(D) 37.5% (E) 34.1% 19. There are 5 red balls, 4 yellow balls and 3
14. Out of total number of participants in street green balls in a basket. If 3 balls are drawn
2 7 at random, what is the probability that at
play, participated from college A and least 2 of them are green in colour?
11 55
746 ✫ Quantitative Aptitude

1 13 3 For Q. 22-26:
(A) (B) (C)
11 55 11 Given Ratio of Population of three
11 7 villages A : B : C = 4 : 3 : 5 respectively.
(D) (E) Let the total population of three villages
55 55
be 4x, 3x, 5x.
20. A jar has mixture of milk and water in the According to the Question for village A
respective ratio of 4 : 3. From this jar 28 litres
3 7
of mixture (milk and water) was taken out 4x × + 4x × + 3000 = 4x
and after that 4 litres of pure water was 16 16
12x + 28x + 48000 = 64x

TS
added. Now the respective ratio of milk and
water in the jar is 24 : 19. What is the new 24x = 48000
x = 2000
quantity of mixture in the jar? (in litres)
Total population of village A = 8000
(A) 172 (B) 162 (C) 180

H
Total Population of village B = 6000
(D) 184 (E) 168 Total Population of village C = 10000
21. A merchant marks his goods in such a way

G
22. What is the average number of villagers
that the profit on sale of 50 articles is equal across all the villages who have qualified
to the selling price of 25 articles. What is his
profit margin?
(A) 25% (B) 50% (C) 100% U only till class XII?
(A) 3380
(D) 3340
(B) 3630
(E) 3670
(C) 3650
O
(D) 66.67% (E) 75% 23. If 600 villagers have qualified only till class
X from village B and 650 villagers who have
H
Directions—(Q. 22-26) Study the following
information carefully to answer the question. qualified only till class X from village C are
not self-employed (including unemployed
There is a cluster of 3 villages neighbouring
_T

villagers), what is the respective ratio


city ‘XYZ’—village A, village B and village C.
between the number of self-employed
The respective ratio between the population of villagers who have qualified only till class
village A, village B and village C is 4 : 3 : 5. X from village B and the number of self
C

3 employed villagers who have qualified only


Out of the total population of village A,
16 till class X from village C?
PS

of the total villagers have qualified only till (A) 5 : 3 (B) 2 : 1 (C) 5 : 2
7 (D) 3 : 2 (E) 4 : 1
class X, of the total villagers have qualified
16 24. By what per cent approximately, the number
U

only till XII and the remaining 3000 villagers of villagers in village C who have qualified
have qualified only till graduation. only till class XII are more than the total
3
@

Out of the total population of village B, number of villagers in village A and village
10 B together who have qualified only till class
of the total villagers have qualified only till XII?
1 (A) 29 (B) 42 (C) 33
class X, of the total villagers have qualified (D) 38 (E) 46
5
only till class XII and the remaining villagers
25. In village A, the respective ratio between the
have qualified only till graduation. total number of male villagers to the total
1
Out of the total population of village C, number of female villagers is 11 : 9. If 48% of
8 the villagers who have qualified only till
of the total villagers have qualified only till class X are females, what per cent female
5 population in village A has qualified only
class X, of the total villagers have qualified
8 till class X?
only till class XII and the remaining villagers (A) 20 (B) 15 (C) 18
have qualified only till graduation. (D) 24 (E) 22
Solved Test Papers of Various Examinations ✫ 747

26. In village C, out of the total number of Note—(i) Profit = (Income – Expenditure)
villagers who have qualified only till Profit
(ii) Per cent Profit = × 100
graduation, 28% have applied for post- Expenditure
graduation courses and in village B, out of (iii) Loss = (Expenditure – Income)
the total number of villagers who have
qualified only till graduation, 22% have Loss
applied for post-graduation courses. What (iv) Per cent Loss = × 100
Expenditure
is the total number of villagers who have
29. What is the average profit earned by the
applied for post-graduation courses from
businessman during April, May and August?
village B and C together?

TS
(in ` Thousand)
(A) 1360 (B) 1420 (C) 1350
(A) 280 (B) 220 (C) 240
(D) 1240 (E) 1180
(D) 200 (E) 260
1
27. The present age of Sneha is of Lara’s 30. Combining June and July, what is the loss

H
8
present age and the respective ratio between per cent incurred by the businessman in
the Lara’s age four years ago and the Sneha’s these two months together?

G
age after four years is 7 : 2. What will be (A) 30 (B) 50 (C) 40
Rupali’s age after two years, if Rupali’s (D) 45 (E) 36
present age is twice Sneha’s present age? (in
years)
U
31. What is the respective ratio between total
income of the businessman in April and
O
(A) 6 (B) 12 (C) 10 August together and total expenditure of the
(D) 14 (E) 8 businessman in May and July together?
(A) 16 : 11 (B) 24 : 17 (C) 18 : 17
H
28. A, B and C started a business with their
investment in the ratio 1 : 3 : 5. After 4 (D) 16 : 13 (E) 18 : 13
months, A invested the same amount as 32. Profit earned by the businessman in May is
_T

before and B as well as C withdrew half of what per cent more than the profit earned
their investments. What is the ratio of their by the businessman in August?
profits at the end of the year? (A) 44.4% (B) 48.8% (C) 38.4%
C

(A) 4 : 3 : 5 (B) 5 : 6 : 10 (D) 42.2% (E) 46.2%


(C) 6 : 5 : 10 (D) 10 : 5 : 6
33. If income of the businessman increased by
PS

(E) 5 : 4 : 7
40% from September to October and his
Directions—(Q. 29-33) Study the graph carefully expenditure decreased by 30% from
to answer the given questions. September to October, what was his profit
U

per cent in October?


Data related to Income (in ` thousand)
(A) 56.93% (B) 51.53% (C) 45.93%
and Expenditure (in ` thousand) of a
(D) 53.85% (E) 48.73%
@

businessman during six months


800 Directions—(Q. 34-38) In this question two
equations numbered I and II are given. You
Amounts in Rs. Thousand

700
600 have to solve both the equations and mark the
500 appropriate answer.
400 34. I. 40x2 – 47x + 12 = 0
300 II. 5y2 – 51y + 54 = 0
200 (A) x ≤ y
100 (B) x < y
(C) x > y
April May June July August Sept.
(D) Relationship between x and y cannot
Income 660 740 380 340 620 400
be established
380 480 640 560 440 520 (E) x ≥ y
748 ✫ Quantitative Aptitude

35. I. 22x2 – x – 6 = 0 (A) x ≤ y


II. 63y2 – 11y – 40 = 0 (B) x < y
(A) x ≤ y (C) x > y
(B) x < y (D) Relationship between x and y cannot
(C) x > y be established
(D) Relationship between x and y cannot (E) x ≥ y
be established
(E) x ≥ y 39. The average score of 29 students of a class
in a History test is 68 (out of 120). After
36. I. 20x2 – 37x + 8 = 0 removing the top 3 scores, the average score

TS
II. 24y2 + 38y – 7 = 0 drops by 2.5. If the second highest score is
(A) x ≤ y less than 85 and the second highest and the
(B) x < y third highest score are not the same, the
(C) x > y highest score should be at least of what

H
(D) Relationship between x and y cannot value? (Consider all scores are integers)
be established (A) 96 (B) 98 (C) 106

G
(E) x ≥ y (D) 108 (E) 102
37. I. 12x2 – 28x + 15 = 0 40. 18 men can complete a piece of work in 24
II. 4y2 – 20y + 21 = 0
(A) x ≤ y
U days and 12 women can complete the same
piece of work in 32 days. 18 men start
O
(B) x < y working and after a few days, 4 men leave
(C) x > y the job and 8 women join. If the remaining
(D) Relationship between x and y cannot 15
H
work is completed in 15 days, after
be established 23
(E) x ≥ y how many days did the four men leave the
_T

job?
38. I. 14x2 – 89x + 30 = 0 (A) 8 (B) 5 (C) 6
II. 27y2 – 12y + 1 = 0 (D) 4 (E) 2
C

Answers and Explanatory Notes


PS

1. Ans. (C) Buyers of age group more than or


Sol. Number of books sold online and equal to 30 years
offline in 2010 = 690 3
Percentage of books sold online = × 144
5
U

= 40% 432
∴ Books bought offline in 2010 = = 86.4
5
@

40 ∴ Buyers less than 30 years of age


= 690 – × 690
100 = 144 – 86.4 = 57.6 thousands
= 690 – 276 = 414 = 57.6 × 1000 = 57600
Books bought offline in 2011 2. Ans. (C)
57.5 Sol. Books sold online in 2010 and 2011
= 720 – × 720
100 40 57.5
= 720 – 414 = 306 = × 690 + × 720
100 100
Average number of books bought by
= 276 + 414 = 690
each buyer = 5
Books sold offline in 2010 and 2011
Number of buyers of books offline
= 414 + 306 = 720
414 + 306 720 Required ratio = 690 : 720
= = = 144
5 5 = 23 : 24
Solved Test Papers of Various Examinations ✫ 749

3. Ans. (B) 7. Ans. (E)


Sol. Books sold offline in 2010 = 414 Sol. The given series is
Books sold offline in 2014 23 15 22 58 224 ?
79 (23 × 1) – 8 = 15
= 1600 – × 1600 (15 × 2) – 8 = 22
100
= 336 (22 × 3) – 8 = 58
Percentage decrease (58 × 4) – 8 = 224
414 − 336 (224 × 5) – 8 = 1112
= × 100
414 ∴ ? = 1112

TS
≈ 18.84% ≈ 19% 8. Ans. (A)
4. Ans. (B) Sol. The given series is
Sol. Total number of books (fiction and 541 269 133 65 ? 14
non-fiction) sold in 2013 = 1240 (541 ÷ 2) – 1.5 = 270.5 – 1.5 = 269

H
Books sold in the fiction category (269 ÷ 2) – 1.5 = 134.5 – 1.5 = 133
3 (133 ÷ 2) – 1.5 = 66.5 – 1.5 = 65
= × 1240 = 930

G
4 (65 ÷ 2) – 1.5 = 32.5 – 1.5 = 31
In the fiction category, number of (31 ÷ 2) – 1.5 = 15.5 – 1.5 = 14
books written by Indian authors
=
4
× 930 = 248 U ∴ ? = 31
9. Ans. (D)
O
15 Sol. The given series is
∴ Number of books written by foreign 28 16 28 76 274 ?
authors (28 × 0.5) + 2 = 14 + 2 = 16
H
= 930 – 248 = 682 (16 × 1.5) + 4 = 24 + 4 = 28
Required percentage (28 × 2.5) + 6 = 70 + 6 = 76
_T

682 (76 × 3.5) + 8 = 266 + 8 = 274


= × 100 = 55% (274 × 4.5) + 10 = 1233 + 10 = 1243
1240
5. Ans. (D) ∴ ? = 1243
C

Sol. Number of fiction books sold in 2012


10. Ans. (D)
and 2013
Sol. The given series is
PS

8 3 4 11 36 129 ? 2755
= × 945 + × 1240
15 4 (4 × 1) + 7 = 11
= 504 + 930 = 1434 (11 × 2) + (7 × 2) = 22 + 14 = 36
Number of non-fiction books sold in
U

(36 × 3) + (7 × 3) = 108 + 21 = 129


2012 and 2011 (129 × 4) + (7 × 4) = 516 + 28 = 544
7 4 (544 × 5) + (7 × 5) = 2720 + 35 = 2755
@

= × 945 + × 720
15 9 ∴ ? = 544
= 441 + 320 = 761
Required difference 11. Ans. (A)
= 1434 – 761 = 673 Sol. Total number of participants in dance
6. Ans. (C) competition
Sol. The given series is 18
= × 500 = 90
? 100
16 37 62 96 146
Students who qualified for the final
round
+21 +25 +34 +50 +75 20
= × 90 = 18
100
+(2)2 +(3)2 +(4)2 +(5)2 Number of participants from B
2
∴ ? = 221 = × 18 = 7.2 ≈ 7
5
750 ✫ Quantitative Aptitude

Number of participants from Y 3 15


= 18 – 7 = 11 = × × 500
5 100
Total number of participants from Y
= 45
in the dance competition
Number of females in singing
2 10 competition
= × × 500
7 100 5 14
= × × 500
100 7 100
= ≈ 14
7 = 50
∴ Students who could not qualify for Difference = 50 – 45 = 5

TS
the final round = 14 – 11 = 3 16. Ans. (C)
12. Ans. (E) Sol. Let Puneet have Rs 100 initially
Sol. Central angle corresponding to He have his wife Rs 20
number of participants in gaming He kept Rs 22 for himself

H
competition Remaining amount
19 = 100 – 42 = 58

G
= × 360 = 68.4°
100 60
13. Ans. (C) He gave his sons × 58
100
Sol. Number of participants from C and A
=
16
× 500 +
20
× 500 U = Rs 34.80
His daughter got 58 – 34.80
O
100 100
= Rs 23.20
= 80 + 100 = 180 Difference = 23.20 – 22 = Rs 1.20
Number of participants from X
H
When difference is 1.2, the sons got
28 34.8
= × 500 = 140
100 When the difference is 2940 the sons
_T

Difference = 180 – 140 = 40 got


Required percentage 2940
× 34.8 = Rs 85260
40 1 .2
= × 100 = 22.2%
C

180 17. Ans. (C)


14. Ans. (D) Sol. Lateral surface area of cylinder
PS

Sol. Number of participants in street play = 94.2 cm2


from A Height = 5 cm
2 25 2πrh = 94.2
= × × 500
11 100 2 × 3.14 × r × 5 = 94.2
U

250 94.2
= r=
11 2 × 3.14 × 5
@

Number of participants in street play r = 3 cm.


from X 18. Ans. (B)
7 25 Sol. Length of rectangle
= × × 500 = 2 (diameter of circle)
55 100
l = 2 (2r)
175 l = 4r … (1)
=
11 Circumference = area of square
250 175
Required ratio = : 2πr = a2
11 11 2πr = 22 × 22
= 10 : 7 242
15. Ans. (A) πr = 242 ⇒ r = × 7 = 77cm
22
Sol. Number of females in calligraphy Perimeter of rectangle = 668 cm
competition 2 (l + b) = 668
Solved Test Papers of Various Examinations ✫ 751

2 (4r + b) = 668 from (1) 4 x − 112 24


2 (4 × 77 + b) = 668 =
3 x − 56 19
308 + b = 334 76x – 2128 = 72x – 1344
b = 334 – 308 = 26 cm 4x = 784
19. Ans. (E) x = 196
Sol. Total number of balls New quantity of mixture in the jar
= 5 + 4 + 3 = 12 = 196 – 28 = 168 litres
Total number of outcomes = 12C3 21. Ans. (C)
P (at least 2 of them are green)
= P (2 green and 1 non-green or 3 ( 50 − 25)
Sol. Profit percentage = × 100

TS
green balls) 25
3
C2 × 9C1 3
C3 = 100%
= 12
+ 12 22. Ans. (C)
C3 C3
Sol. Ratio of population of villages A, B

H
3! 9! 3! and C
× + = 4 : 3 : 5
1 ! × 2 ! 8 ! 1! 0! 3!

G
= 12 ! Let the population of A, B, C be 4x,
3x, 5x respectively.
9 ! × 3!

n
Cr =
n!
U According to the question, for village
A
O
(n − r !) × r ! 3 7
× 4x + × 4x + 3000 = 4x
( 3 × 9) + 1 16 16
=
H
3x + 7x + 12000 = 16x
12 × 11 × 10
12000 = 6x
6 2000 = x
_T

28
= ∴ Population of village A
220
= 4 × 2,000 = 8,000
7 Population of village B
=
C

55 = 3 × 2,000 = 6,000
20. Ans. (E) Population of village C
Sol. Let the quantity of mixture be x litres = 5 × 2,000 = 10,000
PS

4x Total number of villages who have


Quantity of milk = qualified till class XII
7
3x 7 1 5
U

Quantity of water = = × 8,000 + × 6,000 + × 10,000


7 16 5 8
 28 l of mixture was taken out = 3,500 + 1,200 + 6,250
@

Quantity of milk taken out


= 10,950
4 ∴ Required average
= × 28 = 16 l
7
Quantity of water taken out 10950
= = 3,650
3 3
= × 28 = 121 23. Ans. (B)
7 Sol. Villagers who qualified till class X
When 4 l of water is added the ratio
of milk : water from village B
= 24 : 19 3
= × 6000 = 1800
4x 10
− 16 24 Number of self-employed villagers
∴ 7 =
3x 19 who qualified till class X from B
− 12 + 4 = 1,800 – 600 = 1,200
7
752 ✫ Quantitative Aptitude

Villagers who qualified till class X 26. Ans. (A)


from C Sol. Total population of village C = 10,000
1 Number of villagers qualified till
= × 10,000 = 1,250 class X
8
Self-employed villagers from C who 1
= × 10,000 = 1,250
qualified till class X 8
= 1,250 – 650 = 600 Number of villagers qualified till
Required ratio = 1200 : 600 class XII
= 2 : 1 5
24. Ans. (C) = × 10,000 = 6,250

TS
8
Sol. Total number of villagers in C who Number of graduates in village C
have qualified till class XII = 10,000 – 1,250 – 6,250 = 2,500
5 Number of villagers who applied for

H
= × 10,000 = 6,250 post-graduation
8
Number of villagers in A and B who 28
= 28% = × 2,500 = 700

G
have qualified till class XII 100
7 1 Similarly, number of villagers who
= × 8,000 + × 6,000
16 5
= 3500 + 1200 = 4700
Difference = 6250 – 4700 = 1550 U applied for post-graduation from
village B
= [Total population of B – (total
O
Required percentage number of villagers who qualified
22
1550 till class X and class XII)] ×
H
= × 100 100
4700 3 1 22
= 6000 − × 6000 + × 6000 ×
= 32.98% ≈ 33%
_T

10 5 100
25. Ans. (A) 22
Sol. Let there be 11x males and 9x females = 6000 − (1800 + 1200 ) ×
100
in village A 22
C

Total population = 8,000 = 3000 ×


100
11x + 9x = 8,000 = 660
PS

20x = 8000 ∴ Total number of villagers who have


x = 400 applied for post-graduation from B
Number of males = 11 × x and C
= 11 × 400 = 4,400 = 700 + 660 = 1360
U

Number of females 27. Ans. (C)


= 9 × 400 = 3600 Sol. Let Lara’s present age be x years
@

Villagers who qualified till class X 1


from A Then, present age of Sneha = × x
8
3 Lara’s age 4 years ago = x – 4
= × 8,000 = 1500 Sneha’s age 4 years hence
16
Number of females = 48% x x + 32
= +4=
48 8 8
= × 1,500 = 720 According to the question
100
Required percentage x−4 7
=
=
720
× 100 x + 32 2
3600 8
8 ( x − 4) 7
= 20% =
x + 32 2
Solved Test Papers of Various Examinations ✫ 753

16x – 64 = 7x + 224 31. Ans. (D)


9x = 288 Sol. Total income in April and August
x = 32 = 660 + 620 = 1280
Sneha’s present age Total expenditure in May and July
x 32 = 480 + 560 = 1040
= = = 4 years Required ratio
8 8
Rupali’s present age 1280 16
= =
= 2 × 4 = 8 years 1040 13
2 years hence, Rupali’s age = 16 : 13

TS
= 8 + 2 = 10 years 32. Ans. (A)
28. Ans. (B) Sol. Profit earned in May
Sol. Let A, B, C invest x, 3x, 5x respectively = 740 – 480 = 260
A invests x for 4 months = 4x Profit earned in August

H
For the remaining 8 months he invests = 620 – 440 = 180
= 2x × 8 = 16x Difference = 260 – 180 = 80

G
A’s total investment for the year Required percentage
= 4x + 16x = 20x 80
Similarly, B invests = × 100

= (3x × 4) + 8 ×
3x
2 U 33. Ans. (D)
180
= 44.4%
O
= 24x Sol. Income in October
5x 140
C invests = (5x × 4) + 8 ×
H
2 = × 400 = 560
100
= 40x Expenditure in October
_T

At the end of the year, profit sharing


ratio 70
= × 520 = 364
= 20x : 24x : 40x 100
= 5 : 6 : 10 Profit = 560 – 364 = 196
C

29. Ans. (C) 196


Sol. Profit = Income – Expenditure Profit % = × 100
364
PS

Profit earned in April, May and 700 11


August = = 53 %
13 13
= (660 – 380) + (740 – 480) + (620 – 440)
= 53.85%
= 280 + 260 + 180
U

34. Ans. (B)


= 720
Sol. (I) 40x2 – 47x + 12 = 0
720
∴ 40x2 – 15x – 32x + 12 = 0
@

Average profit = = 240


3 5x (8x – 3) – 4 (8x – 3) = 0
30. Ans. (C)
(8x – 3) (5x – 4) = 0
Sol. Income in June and July
8x – 3 = 0; 5x – 4 = 0
= 380 + 340 = 720
Expenditure in June and July 3 4
x= ;x=
= 640 + 560 = 1200 8 5
∴ Loss = Expenditure – Income x = 0.37; x = 0.8 … (1)
= 1200 – 720 = Rs 480 2
(II) 5y – 51y + 54 = 0
Loss 5y2 – 45y – 6y + 54 = 0
∴ Loss % = × 100
Expenditure 5y (y – 9) – 6 (y – 9) = 0
480 (y – 9) (5y – 6) = 0
= × 100 = 40% y = 9 = 0; 5y – 6 = 0
1200
6
y = 9; y =
5
754 ✫ Quantitative Aptitude

y = 9; y = 1.2 … (2) (II) 24y2 + 38y – 7 = 0


From (1) and (2) x < y −38 ± ( 38)2 + 4 × 24 × 7
35. Ans. (D) y=
2 × 24
Sol. (I) 22x2 – x – 6 = 0
22x2 + 11x – 12x – 6 = 0 y = −38 ± 2116
11x (2x + 1) – 6 (2x + 1) = 0 48
(2x + 1) (11x – 6) = 0 −38 ± 46
2x + 1 = 0; 11x – 6 = 0 y=
48
1 6
x= − ;x= −38 + 46 −38 − 46

TS
2 11 y= ;
x = –0.5; x = 0.5 … (1) 48 48
1 7
2
(II) 63y – 11y – 40 = 0 y= ; −
Using quadratic formula 6 4

H
y = 0.16; y = –1.75 … (2)
− b ± b 2 − 4 ac From (1) and (2) x > y
y=
2a 37. Ans. (A)

G
11 ± 121 + 4 × 63 × 40 Sol. (I) 12x2 – 28x + 15 = 0
y=
2 × 63 − b ± b 2 − 4 ac

y=
11 ± 10201
U x=
2a
28 ± (28 )2 − 4 × 12 × 15
O
126
x=
11 ± 101 2 × 12
y=
H
126 28 ± 64
x=
11 + 101 11 − 101 24
y= ;
_T

126 126 28 + 8 28 − 8
x= ;
112 −90 24 24
y= ;
126 126 3 5
x= ;
C

8 −5 2 6
y= ; x = 1.5; x = 0.8 … (1)
9 7
PS

y = 0.88; y = –0.7 … (2) (II) 4y2 – 20y + 21 = 0


From (1) and (2) no relation can be 4y2 – 6y – 14y + 21 = 0
established. 2y (2y – 3) – 7 (2y – 3) = 0
36. Ans. (C)
U

(2y – 3) (2y – 7) = 0
Sol. (I) 20x2 – 37x + 8 = 0 2y – 3 = 0; 2y – 7 = 0
Using quadratic formula
@

3 7
− b ± b 2 − 4 ac y= ;y=
x= 2 2
20 y = 1.5; y = 3.5 … (2)
37 ± 1369 − 640 From (1) and (2) x ≤ y
x= 38. Ans. (C)
40
Sol. (I) 14x2 – 89x + 30 = 0
37 ± 729
x=
40 − b ± b 2 − 4 ac
x=
37 + 27 37 − 27 2a
x= =
40 40 89 ± (89)2 − 4 × 14 × 30
x=
64 10 2 × 14
x= ;
40 40
89 ± 6241
x = 1.6; x = 0.25 … (1) x=
28
Solved Test Papers of Various Examinations ✫ 755

89 + 79 89 − 79 x
x= ;x= ∴ Work done in x days =
28 28 24
x
5 Remaining work = 1 –
x = 6; x = 24
14 24 − x
x = 6; x ≈ 0.35 … (1) =
24
(II) 27y2 – 12y + 1 = 0 1 man takes (18 × 24) days to
27y2 – 3y – 9y + 1 = 0 complete the work 1
3y (9y – 1) – 1 (9y – 1) = 0 ∴ In 1 day work done by 1
(9y – 1) (3y – 1) = 0 18 × 24

TS
man
9y – 1 = 0; 3y – 1 = 0
14 7
1 1 ∴ 14 men can do = work
y= ;y= 18 × 24 216
9 3

H
Similarly 8 women can do
y = 0.1; y ≈ 0.33 … (2) 8 1
From (1) and (2) x > y =
12 × 32 48 work

G
39. Ans. (E) ∴ 14 men and 8 women can complete
Sol. Average of 29 students = 68
∴ Total marks = 68 × 29 = 1972
When top 3 scores are removed U the work in =
7
1
+
1
days
O
average drops by 2.5 216 48
∴ Total of 26 students 432
H
= 26 × (68 – 2.5) = days
23
= 26 × 65.5
Remaining work will take
= 1703
_T

∴ Value of 3 highest scores 24 − x 432


× days
= 1972 – 1703 = 269 24 23
Consider the second highest score as
C

84 ( second highest score is less According to the question


than 85) 24 − x 432 15
× = 15
PS

Let the third highest score be 83 24 23 23


Then value of the highest score
= 269 – (84 + 83) = 102 (24 − x ) 18 360
=
∴ The highest score should be at least 23 23
U

102 432 – 18x = 360


40. Ans. (D) 432 – 360 = 18x
@

Sol. Let 4 men leave the job after x days


18 men can complete the work in 24 72 = 18x
days x = 4
756 ✫ Quantitative Aptitude

Institute of Banking Personnel Selection (IBPS)


Specialist Officers (I.T.) Examination, February 2016
1. 15 men and 16 women together can complete (A) 4 (B) 10
a piece of work in 6 days. If 12 women can (C) 7 (D) 6
complete the same project in 32 days, in how (E) 8
many days will 10 men complete the same 4. The number of employees in Department D
project? is what percent less than the total number of
(A) 12 (B) 20

TS
employees in Department A and F together?
(C) 16 (D) 8 (A) 50 (B) 80
(E) 14 (C) 75 (D) 25
2. The interest earned when Rs. ‘P’ is invested (E) 85

H
for four years in a scheme offering 9% p. a. 5. In department C, the respective ratio
simple interest is more than the interest between the number of female employees

G
earned when the same sum (Rs. P) is and male employees was 5:6. There were
invested for two years in another scheme equal number of unmarried males and
offering 12% p. a. simple interest, by ` 360.
What is the value of P?
(A) 2000 (B) 3500 U unmarried females in department C. If the
respective ratio between married females
and married males was 4 : 5, what is the
O
(C) 2500 (D) 4000 number of unmarried females?
(E) 3000 (A) 20 (B) 8
H
Directions—(Q. 3-8) Refer to the pie chart and (C) 10 (D) 12
answer the given questions. (E) 16
_T

Percentage of employees in different 6. In 2014, the number of female employees in


departments of branch ‘LMN’ in department E was 5/11 of the total number
the year 2014 of employees in the same department. If the
C

number of female employees in department


Total number of employees : 550 D was 16 less than that in Department E, what
is the number of male employees in
PS

F Department D?
14% A (A) 32 (B) 31
18% (C) 28 (D) 20
U

E (E) 24
16% 7. What is the average number of employees
B
@

20% in Departments C, E andF?


D
(A) 89 (B) 87
8% (C) 97 (D) 109
C
(E) 99
24%
8. What is the central angle corresponding to
the number of employees in Department F?
3. In 2014, there were 30% post graduate (in Degrees)
employees in department B. In 2015, 20 (A) 55.9 (B) 56.5
employees of the same department were (C) 50.4 (D) 52.8
shifted to branch ‘PQR’. If in 2015, the (E) 51.6
percentage of post graduate employees in 9. Four years ago, the respective ratio between
department B were the same, how many the age of Ram and that of Sonu, was 4:9.
post graduate employees were shifted to Tina is ten years older to Ram. Tina is ten
branch ‘PQR’?
Solved Test Papers of Various Examinations ✫ 757

years younger to Sonu. What is Tina’s 13. What is the difference between total number
present age? of tourists in cities A and B together in
(A) 40 years (B) 36 years January and that in February?
(C) 30 years (D) 20 years (A) 2,500 (B) 2,200
(E) 42 years (C) 1,000 (D) 1,500
10. A jar has 60 lts milk. From the jar, 12 lts of (E) 2,000
milk was taken out and replaced by an equal 14. What is the average number of tourists in
amount of water. If 12 lts of the newly formed city A in February and March?
mixture is taken out of the jar, what is the (A) 25,000 (B) 28,000

TS
final quantity of milk left in the jar? (C) 27,000 (D) 26,000
(A) 38.4 lts (B) 40 lts (E) 28,500
(C) 36 lts (D) 28.6 lts 15. What is the respective ratio between the total
(E) 36.5 lts number of tourists in city A in April and May

H
Directions—(Q. 11-16) Refer to the graph and together and that in City B in the same month
answer the given question. together?

G
(A) 7 : 16 (B) 7 : 12
Data regarding number of tourists in
(C) 5 : 12 (D) 8 : 13
two cities-A and B in five different
months in the year 2005. Data are
given in hundreds U (E) 7 : 8
16. The number of tourists in City B in March is
O
what percent less than that in the same city
350 in May?
(A) 25 (B) 20
H
300
Number of tourists

3 1
250 (C) 18 (D) 15
4 2
_T

200 3
(E) 10
150 5
100 Directions—(Q. 17-21) What approximate value
C

50 will come in place of question mark (?) in the


given questions? (You are not expected to
PS

Jan Feb Mar Apr May calculate the exact value)


City A 300 320 220 200 150 17. ? = (1346.92 + 46.94) ÷ 99.9 – 6.98
(A) 121 (B) 441
U

City B 160 120 260 280 320 (C) 1024 (D) 49


(E) 196
11. Number of tourists in City B in April are what
@

percent more than that in City A in the same 18. (32.01)2 × (512)1/3 × (33.99)2 ÷ [29 × 16.972] = 2?
month? (A) 3 (B) 4
(A) 25 (B) 35 (C) 9 (D) 10
(C) 20 (D) 30 (E) 6
(E) 40 19. (14.99% of 4799.995) ÷ ? = (170% of 7.111)2
12. In the month of December of the previous (A) 150 (B) 25
year (2004), the total number of tourists in (C) 100 (D) 50
both the cities together were 25% more than (E) 5
that in January 2005. What was the number 3
of tourists in both the cities together in 20. of 239 = ? ÷ (1.6 × 0.499)
20
December 2004?
(A) 30 (B) 300
(A) 57,500 (B) 52,000
(C) 600 (D) 120
(C) 60,500 (D) 54,250
(E) 80
(E) 60,000
758 ✫ Quantitative Aptitude

25. Number of hard bound editions sold by


21. 1296.002 ÷ 8.996 ÷ 9.98 + 39.4 = ?
shop C is what percent less than that sold by
(A) 80 (B) 8 shop A?
(C) 4 (D) 120 (A) 90 (B) 50
(E) 40
(C) 75 (D) 70
22. In the year 2013, the population of a village (E) 80
A was 30% more than the population of
26. Number of non-fictions sold by shop B is
village B. The population of village A in 2014
what percent of number of non-fictions sold
increased by 20% as compared to the
previous year. If the population of village A by shop A?

TS
in 2014 was 7176, what was the population (A) 52.5 (B) 64.75
of village B in 2013? (C) 50 (D) 40
(A) 4000 (B) 5000 (E) 60
(C) 4800 (D) 4600 27. Total number of books sold by shop B is

H
(E) 5200 what percent more than that sold by shop
23. Tom gave 20% of a certain amount of money D?

G
to Ali. From the money Ali received, he 1
(A) 15 (B) 25
spent 25% on school fees and 35% on buying 2
school uniform. After the mentioned
expenses, Ali had ` 2800 remaining with him.
How much money did Tom have initially? U (C) 30
(E) 11
1
(D) 10
O
9
(A) ` 21,000 (B) ` 35,000 28. What is the average number of fictions sold
(C) ` 30,000 (D) ` 27,000 by shops A and B?
H
(E) ` 28,000
(A) 900 (B) 720
24. A and B started a business with an (C) 880 (D) 1100
_T

investment of ` 3500 and ` 2500 respectively. (E) 990


After 4 months C joined with ` 6000. If the
difference between C’s share and B’s share 29. In March 2015, the number of paperback
in the annual profit was ` 1977, what was editions sold by shop D was 20% more than
C

the total annual profit? the same sold by the same shop in the
(A) ` 15,620 (B) ` 16,240 previous month. The number of paperback
PS

(C) ` 14,690 (D) ` 12,770 editions sold in March 2015 by shop D


(E)? 13,180 constituted 50% of the total number of books
sold by shop D in March 2015. What was the
Directions—(Q. 25-31) Study the table and total number of books sold in March 2015
U

answer the given questions. by shop D?


Data regarding number of books sold in either (A) 1860 (B) 1240
@

hard bounds or paperback editions and also the (C) 1260 (D) 1600
categories of books sold in fiction and non- (E) 1620
fiction category by four different shops, in a
30. What is the respective ratio between the
particular month (February 2015).
number of non-fictions sold by shop C and
Book Respective ratio Number of % of fictions number of non-fictions sold by shop D?
Shops between number of paperbacks (hardbound +
hardbounds sold sold out of paperback)
(A) 7 : 9 (B) 8 : 9
and number of total sold out of total (C) 6 : 7 (D) 8 : 11
paperbacks sold books sold (E) 3 : 5
A 2:3 1200 60 31. The distance between two places A and B is
B 1:5 1000 65
C 1:3 600 70
140 kms. 1st scooter departs from place A to
D 3:5 675 75 B, at a speed of 50 kmph at 10 a.m. 2nd
scooter departs from place B to A at a speed
Note—(i) Total books sold = number of
of 30 kmph at 12 pm. At what time will both
hard bounds + number of paperbacks sold.
Solved Test Papers of Various Examinations ✫ 759

the scooters meet each other? 38. I. x2 + 8x + 15 = 0


(A) 12 : 30 pm (B) 01 : 50 pm II. y2 + 3y + 2 = 0
(C) 1 : 00 pm (D) 12 : 50 pm (A) x > y (B) x ≤ y
(E) 1 : 30 pm (C) x ≥ y (D) x < y
32. Two mobile phones were purchased at the (E) Relationship between x and y cannot
same price. One was sold at a profit of 30% be determined
and the second was sold at a price which was Directions—(Q. 39-44) What will come in place
` 2500 less than the price at which the first of (?) in the given number series?
was sold. If the overall profit earned by
39. 606 201 66 21 6 ?

TS
selling both the mobile phones was 5%, what
(A) 0.5 (B) 2
was the cost price of one mobile phone?
(C) 5 (D) 1
(A) ` 8000 (B) ` 5000
(E) 3
(C) ` 6000 (D) ` 4500

H
(E) ` 5500 40. 19 10 11 18 38 ?
(A) 97.5 (B) 110
Directions—(Q. 33-38) In these questions two (C) 115 (D) 124.5

G
equations numbered I and II are given. You (E) 99.5
have to solve both the equations and mark the
41. 6 4 5 11 ? 189
appropriate option.
33. I. 2x2 – 21x + 52 = 0 U (A)
(C)
44
65
(B) 39
(D) 96
O
II. 2y2 – 11y + 12 = 0 (E) 62
(A) x > y (B) x ≤ y
(C) x ≥ y 42. 24 26 20 32 12 ?
H
(D) x < y
(E) Relationship between x and y cannot (A) 42 (B) 54
be determined (C) 56 (D) 46
_T

(E) 64
34. I. 3x2 – 13x + 14 = 0
II. 2y2 – 5y + 3 = 0 43. 18 19 24 37 66 ?
(A) x > y (B) x ≤ y (A) 158 (B) 224
C

(C) x ≥ y (D) x < y (C) 219 (D) 192


(E) Relationship between x and y cannot (E) 127
PS

be determined 44. 19 19.6 20.8 23.2 28 ?


2
35. I. 4x – 8x + 3 = 0 (A) 35.7 (B) 37.6
II. 4y2 – 15y + 14 = 0 (C) 31.8 (D) 39.8
(E) 38.2
U

(A) x > y (B) x ≤ y


(C) x ≥ y (D) x < y Directions—(Q. 45-49) These questions consist
(E) Relationship between x and y cannot of a question and two statements I and II given
@

be determined below it. You have to decide whether the data


36. I. 2x2 – 9x + 9 = 0 provided in the statements are sufficient to
II. y2 – 7y + 12 = 0 answer the question.
(A) x > y (B) x ≤ y Read both the statements and choose the
(C) x ≥ y (D) x < y appropriate option.
(E) Relationship between x and y cannot (A) if statement I alone is sufficent to
be determined answer the question.
(B) if statement II alone is sufficient to
37. I. 4x2 + 19x + 22 = 0 answer the question.
II. 2y2 + 11y + 15 = 0 (C) if both statements I and II are needed
(A) x > y (B) x ≤ y to answer the question.
(C) x ≥ y (D) x < y (D) If the given data is not sufficient to
(E) Relationship between x and y cannot answer the question.
be determined
760 ✫ Quantitative Aptitude

(E) Either statement I or statement II is II. When 11 litres of mixture is taken out
needed to answer the question. and 6 litres of juice is added, the ratio
45. What is the curved surface area of the right between juice and water becomes
circular cylinder? 10 : 3 respectively.
I. Area of the base of the cylinder is 49. How much money did Mr. Rajesh receive as
154 cm2. retirement funds?
II. Volume of the cylinder is 1232 cm2. I. Out of the total money received Mr.
46. What is the area of the circular field? Rajesh invested 51% in various
I. Area of the largest square that can be schemes, gave 35% of the total money

TS
inscribed in the given square field is received to his wife and remaining
2450 cm2. money he spent on miscellaneous
II. Area of the smallest square in which the items.
given circular field can be inscribed is II. Out of the total money received, Mr.

H
4900 cm2. Rajesh spent 5% on son’s wedding and
47. In how many days can ‘B’ alone complete gave 35% to his wife. Out of the

G
the work? remaining, he invested 28% in
I. A, B and C together can complete the insurance policies, 22% in pension fund
9
work in 3 days, A and B together
13 4 U scheme, 35% in share market and
remaining ` 4,50,000 he spent on
furnishing the house.
O
can complete the work in 4 days, B
5 6 50. The speed of a boat in still water is 15 km/h
and C together can complete it in 6
7 and the speed of the current is 3 km/h. The
H
days. distance travelled by the boat from point A
II. A, B and C together can complete the to point B downstream is 24 km more than
9
_T

piece of work in 3 days, A and C the distance covered by the same boat from
13 1 point B to point C upstream in the same time.
together can complete the work in 5
3 How much time will the boat take to travel
days. from C to B downstream?
C

48. What was the initial quantity of mixture of (A) 2 hrs (B) 2 hrs 30 mins
juice and water? (C) 2 hrs 40 mins (D) 2 hrs 10 mins
PS

I. Juice and water were in the ratio of (E) 3 hrs 20 mins


8 : 3 respectively in the mixture initially.
U

Answers and Explanatory Notes


1. Ans. (A) 15 men and 16 women can do the
@

Sol. 12 women can complete the work in work in 6 days


1
32 days ∴ In 1 day they can do of the work
∴ 16 women can complete the work in 6 … (4)
12 From (2), (3) and (4)
× 32 days = 24 days … (1)
16 1 1 1
+ =
Let the time taken by 15 men to x 24 6
complete the work be x days 1 1 1
In 1 day work done by 15 men = −
x 6 24
1 1 3 1
= … (2) = = ⇒ x = 8 days
x x 24 8
In 1 day work done by 16 women ∴ 15 men can do the work in 8 days
1 Then 10 men will do the work in
= (from 1) … (3) 15
24 × 8 = 12 days.
10
Solved Test Papers of Various Examinations ✫ 761

2. Ans. (E) Let the number of unmarried males


Sol. S.I. on Rs P for 4 years at 9% p.a. and unmarried females = x
PTR P × 4 × 9 Married males = 72 – x
= = Married females = 60 – x
100 100
According to the question,
9P
= 60 − x 4
25 =
S.I. on Rs P for 2 years at 12% p.a. 72 − x 5
P × 2 × 12 6 P 300 – 5x = 288 – 4x
= = 12 = x
100 25

TS
According to the question Number of unmarried females = 12.
9P 6 P 6. Ans. (D)
− = 360
25 25 Sol. Number of employees in E

H
9P – 6P = 360 × 25 16
3P = 360 × 25 = × 550 = 88
100
360 × 25 Number of female employees in E

G
P = = Rs 3000.
3 5
3. Ans. (D) = × 88 = 40
11
Sol. Number of employees in department
B in 2014
U Number of female employees in D
= 40 – 16 = 24
O
20 Number of employees in D
= × 550 = 110
100 8
Post graduate employees in B = × 550 = 44
H
100
30 Number of male employees in D
= × 110 = 33 = 44 – 24 = 20.
100
_T

In 2015, number of employees in B 7. Ans. (E)


= 110 – 20 = 90 Sol. Number of employees in C, E and F
In 2015, number of post graduate 24 16 14
C

employees = 30% = × 550 + × 550 + × 550


100 100 100
30
PS

= × 90 = 27 550
100 = (24 + 16 + 14)
∴ Number of post graduate employees 100
shifted to ‘PQR’ 11
= (54) = 297
U

= 33 – 27 = 6 2
4. Ans. (C) 297
∴ Average = = 99.
Sol. Employees in D = 8% 3
@

8. Ans. (C)
Employees in A and F together
= 18 + 14 = 32% 14
Sol. Central angle in F = × 360 = 50.4°.
Difference = 32 – 8 = 24% 100
9. Ans. (C)
Required percentage
Sol. Let Ram’s and Sonu’s age 4 years ago
24 be 4x and 9x
= × 100 = 75%.
32 Ram’s and Sonu’s present age
5. Ans. (D) = 4x + 4 and 9x + 4
Sol. Total number of employees in C Tina’s age
24 = 4x + 4 + 10 = 4x + 14
= × 550 = 132 Tina’s age is also
100
5 = 9x + 4 – 10 = 9x – 6
Number of females =
11
× 132 = 60 ∴ 4x + 14 = 9x – 6
6 20 = 5x
Number of males = × 132 = 72 4 = x
11
762 ✫ Quantitative Aptitude

∴ Tina’s present age 14. Ans. (C)


= 4x + 14 = 4 (4) + 14 Sol. Number of tourists in A in February
= 30 years. and March
10. Ans. (A) = 32000 + 22000
Sol. Milk Water = 54000
60 l 0 54000
Average = = 27000.
When 12 l of milk is taken out and 2
15. Ans. (B)
12 l water added
Sol. Total number of tourists in A in April
Milk Water and May
(60 – 12) 12 = 20000 + 15000

TS
∴ Ratio of milk : water = 35000
= 48 : 12 = 4 : 1 Total number of tourists in B in April
When 12 l of the mixture is taken out, and May
quantity of milk = 28000 + 32000

H
= 60000
4 48
= × 12 = = 9.6 l 35000 7

G
5 5 Required ratio = =
60000 12
Quantity of milk left = 7 : 12.
11. Ans. (E)
= 48 – 9.6 = 38.4 l.

Sol. Number of tourists in city B in April U 16. Ans. (C)


Sol. Number of tourists in B in March
O
= 26000
= 28000 Number of tourists in B in May
Number of tourists in city A in April = 32000
H
= 20000 Required percentage
Difference = 28000 – 20000 = 8000 32000 − 26000
_T

Required percentage = × 100


32000
8000 6000 75
= × 100 = 40%. = × 100 =
20000 32000 4
12. Ans. (A)
C

Sol. Total number of tourists in January = 18¾%.


2005 in A and B 17. Ans. (D)
PS

= 30000 + 16000 Sol. ? = (1346.92 + 46.94) ÷ 99.9 – 6.98


≈ (1347 + 47) ÷ 100 – 7
= 46000
≈ 1394 ÷ 100 – 7
Number of tourists in December 2004 ≈ 13.94 – 7 (BODMAS)
U

in A and B ≈ 14 – 7
25 ? ≈ 7
= × 46000 + 46000
49 = 7.
@

100
= 57500. 18. Ans. (E)
13. Ans. (E) Sol. (32.01)2 × (512)1/3 × (33.99)2 ÷
Sol. Number of tourists in A and B in [29 × (16.97)2] = 2?
January (32)2 × (83)1/3 × (34)2 ÷ [29 × (17)2] = 2?
= 30000 + 16000 ( 2 5 )2 × 8 × ( 34 )2
= 46000 = 2?
2 9 × (17 )2
Number of tourists in A and B in
210 × 2 3 × (17 × 2)2
February = 2?
= 32000 + 12000 2 9 × (17 )2
= 44000 213 × 17 2 × 2 2
= 2?
Difference = 46000 – 44000 2 9 × 17 2
= 2000 2 6 = 2?
? = 6
Solved Test Papers of Various Examinations ✫ 763

19. Ans. (E)


717600
Sol. (14.99% of 4799.995) ÷ ? = (170% of x =
7.111)2 156
(15% of 4800) ÷ ? = (170% of 7)2 x = 4600
2 Population of village B in 2013 = 4600.
15 170
× 4800 ÷ ? = ×7 23. Ans. (B)
100 100 Sol. Let the amount Tom had initially be
720 ÷ ? = (11.9)2
Rs x
720 ÷ ? = (12)2
20 x
720 Amount given to Ali = × x=
= ? ⇒ ? = 5. 100 5

TS
144 Ali spent 25% on fees and 35% on
20. Ans. (A) uniforms
3 x
Sol. of 239 = ? ÷ (1.6 × 0.499) Total expenditure (25 + 35)% of
20

H
5
3 60 x
× 240 = ? ÷ 1.6 × 0.5 = ×
20

G
100 5
? 3x
36 = 1.6 × 0.5 =
25
36 × 0.8 = ?
28.8 = ? U Amount left =
x 3x 2x
− =
O
∴ ? = 30 5 25 25
According to the question,
21. Ans. (E)
2x
H
Sol. 1296.002 ÷ 8.996 ÷ 9.98 + 39.4 = ? = 2800
25
1296 ÷ 9 ÷ 10 + 39.4 = ? 2800 × 25
_T

x = = Rs 35000
36 ÷ 9 ÷ 10 + 39.4 = ? 2
36 ÷ 90 + 39.4 = ? (BODMAS) ∴ Tom had Rs 35000 initially.
0.4 + 39.4 = ? 24. Ans. (E)
C

39.8 = ? Sol. A’s share : B’s share : C’s share


40 = ?. 3500 × 12 : 2500 × 12 : 6000 × 8
7 × 12 : 5 × 12 : 12 × 8
PS

22. Ans. (D)


Sol. Let the population of village B in 2013 7 : 5 : 8
= x Let A’s share be 7x, B’s share 5x and
Then population of village A C’s share 8x
U

Difference between C’s and B’s share


30
= x + × x = 1977
100
@

8x – 5x = 1977
13x 3x = 1977
=
10 x = 659
Population of village A in 2014 ∴ Total annual profit
13 x 20 13 x = 7x + 5x + 8x = 20x
= + × = 20 × 659
10 100 10
156 x = Rs 13,180.
= 25. Ans. (C)
100 Sol. Number of hard bound editions sold
According to the question,
1
156 x by C = × 600 = 200
= 7176 3
100 Number of hard bound editions sold
156x = 717600 by A
2
= × 1200 = 800
3
764 ✫ Quantitative Aptitude

Difference = 800 – 200 = 600 29. Ans. (E)


Required percentage Sol. Number of paperback editions sold
600 by D in March 2015
= × 100 = 75%. 20
800 = 675 + × 675
26. Ans. (A) 100
Sol. Total number of books sold by A = 810
810 is 50% of the total number of
2 books sold by D in March 2015
= 1200 + × 1200 = 2000
3 ∴ Total number of books sold by D in
Total number of books sold by B March 2015

TS
1 = 810 × 2 = 1620.
= 1000 + × 1000 = 1200 30. Ans. (B)
5
Percentage of non-fictions sold by A Sol. Total number of books sold by C
1
100 − 60

H
= × 2000 = 800 = 600 + × 600
3
100 = 800
Percentage of non-fictions sold by B

G
Total number of books sold by D
100 − 65 3
= × 1200 = 420 = 675 + × 675
100
Required percentage
420 U = 1080
5

Number of non-fiction sold by C


O
= × 100 = 52.5%. (100 − 70)
800
27. Ans. (E) = × 800
100
H
Sol. Total number of books sold by B 30
= × 800
1 100
= 1000 + × 1000 = 1200
_T

5 = 240
Total number of books sold by D Number of non-fiction sold by D
3 (100 − 75)
= 675 + × 675 = 1080 = × 1080
5 100
C

Difference = 1200 – 1080 = 120 = 270


Required percentage ∴ Required ratio = 240 : 270
PS

120 1 = 8 : 9.
= × 100 = 11 %. 31. Ans. (A)
1080 9
28. Ans. (E) Sol. Let the two scooters meet at C
Distance covered by first scooter in 2
U

Sol. Total number of books sold by A


hrs (by 12 pm)
2
= 1200 + × 1200 = 2000 = 50 × 2 = 100 km (AD = 100)
@

3
Total number of books sold by B
1 x
A D C B
= 1000 + × 1000 = 1200
5 12 p.m.
Number of fiction books sold by A 50 km/hr 30 km/hr
60 10 a.m. 12 p.m.
= × 2000 = 1200
100
Number of fiction books sold by B Time taken by first scooter to cover
65 distance DC (say x)
= × 1200 = 780
100 Distance x
= = … (1)
Total = 1200 + 780 = 1980 Speed 50
AD = 100 km
1980
Average = = 990. ∴ DB = 40 km, then BC = BD – DC
2 = 40 – x
Solved Test Papers of Various Examinations ✫ 765

Time taken by second scooter to cover 33. Ans. (C)


distance BC Sol. (I) 2x2 – 21x + 52 = 0
Distance 40 − x 2x2 – 8x – 13x + 52 = 0
(i.e. 40 – x = = … (2) 2x (x – 4) – 13 (x – 4) = 0
Speed 30
(x – 4) (2x – 13) = 0
From (1) and (2)
x – 4 = 0; 2x – 13 = 0
x 40 − x 13
= x = 4; x = = 6.5 … (1)
50 30 2
30x = 2000 – 50x 2
(II) 2y – 11y + 12 = 0
80x = 2000 2y2 – 8y – 3y + 12 = 0

TS
x = 25 2y (y – 4) – 3 (y – 4) = 0
Time taken by first scooter to cover (y – 4) (2y – 3) = 0
25 km y – 4 = 0; 2y – 3 = 0

H
Distance 25 1 3
= = hr. y = 4; y = = 1.5 … (2)
Speed 50 2 2
∴ The two scooters will meet at 12.30

G
From (1) and (2) x ≥ y
pm.
34. Ans. (A)
32. Ans. (B)
Sol. (I) 3x2 – 13x + 14 = 0
Sol. Let the C.P. of each mobile phone
= x
U 3x2 – 6x – 7x + 14 = 0
3x (x – 2) – 7 (x – 2) = 0
O
S.P. of first mobile
(x – 2) (3x – 7) = 0
30 13x x – 2 = 0; 3x – 7 = 0
= x + × x =
H
100 10 7
S.P. of second mobile x = 2; x = = 2.3 … (1)
3
13x
_T

2
= – 2500 (II) 2y – 5y + 3 = 0
10 2y2 – 2y – 3y + 3 = 0
C.P. of 2 mobiles = 2x 2y (y – 1) – 3 (y – 1) = 0
S.P. of 2 mobiles (y – 1) (2y – 3) = 0
C

13x 13 x y – 1 = 0; 2y – 3 = 0
= + – 2500
10 10 3
PS

26 x y = 1; y = = 1.5 … (2)
= – 2500 2
10 From (1) and (2) x > y
∴ Profit = S.P. – C.P. 35. Ans. (D)
U

26 x Sol. (I) 4x2 – 8x + 3 = 0


= – 2500 – 2x 4x2 – 2x – 6x + 3 = 0
10
2x (2x – 1) – 3 (2x – 1) = 0
@

6x
= – 2500 (2x – 1) (2x – 3) = 0
10
Profit % = 5% 2x – 1 = 0; 2x – 3 = 0
Profit 1 3
× 100 = 5 x= ;x=
C.P. 2 2
6x x = 0.5; x = 1.5 … (1)
− 2500 (II) 4y2 – 15y + 14 = 0
10 × 100 = 5 4y2 – 8y – 7y + 14 = 0
2x 4y (y – 2) – 7 (y – 2) = 0
(6 x − 25000) (y – 2) (4y – 7) = 0
× 10 = 5 y – 2 = 0; 4y – 7 = 0
2x
x = 6x – 25000 7
25000 = 5x y = 2; y = = 1.75 … (2)
4
5000 = x From (1) and (2) x < y
C.P. of each mobile phone = Rs 5000
766 ✫ Quantitative Aptitude

36. Ans. (B) (66 ÷ 3) – 1 = 22 – 1 = 21


Sol. (I) 2x2 – 9x + 9 = 0 (21 ÷ 3) – 1 = 7 – 1 = 6
2x2 – 6x – 3x + 9 = 0 (6 ÷ 3) – 1 = 2 – 1 = 1
2x (x – 3) – 3 (x – 3) = 0 40. Ans. (A)
(x – 3) (2x – 3) = 0 Sol. The given series is
x – 3 = 0; 2x – 3 = 0 19 10 11 18 38 ?
3 (19 × 0.5) + 0.5 = 9.5 + 0.5 = 10
x = 3; x = = 1.5 … (1) (10 × 1) + 1 = 11
2
2
(II) y – 7y + 12 = 0 (11 × 1.5) + 1.5 = 18
(18 × 2) + 2 = 38

TS
y2 – 3y – 4y + 12 = 0
y (y – 3) – 4 (y – 3) = 0 (38 × 2.5) + 2.5 = 97.5
(y – 3) (y – 4) = 0 41. Ans. (B)
y – 3 = 0; y – 4 = 0 Sol. The given series is

H
y = 3; y = 4 … (2) 6 4 5 11 ? 189
From (1) and (2) x ≤ y (6 × 1) – 2 = 4
(4 × 2) – 3 = 5

G
37. Ans. (E)
(5 × 3) – 4 = 11
Sol. (I) 4x2 + 19x + 22 = 0
(11 × 4) – 5 = 39
4x2 + 8x + 11x + 22 = 0
4x (x + 2) + 11 (x + 2) = 0
(x + 2) (4x + 11) = 0 U 42.
(39 × 5) – 6 = 189.
Ans. (A)
O
Sol. The given series is
x + 2 = 0; 4x + 11 = 0
24 26 20 32 12 ?
11 24 – 4 = 20
x = –2; x = = –2.75 … (1)
H
4 26 + 6 = 32
2
(II) 2y + 11y + 15 = 0 20 – 8 = 12
_T

2y2 + 6y + 5y + 15 = 0 32 + 10 = 42
2y (y + 3) + 5 (y + 3) = 0 43. Ans. (E)
(y + 3) (2y + 5) = 0 Sol. The given series is
y + 3 = 0; 2y + 5 = 0 18 19 24 37 66 ?
C

5
y = –3; y = – = –2.5 … (2) 18 + 1 = 19
2
From (1) and (2) relationship between x 19 + 5 = 24
PS

and y cannot be determined. 24 + 13 = 37


38. Ans. (D) 37 + 29 = 66
Sol. (I) x2 + 8x + 15 = 0 66 + 61 = 127
U

x2 + 5x + 3x + 15 = 0 Consider 1 5 13 29 61
x (x + 5) + 3 (x + 5) = 0 1+4=5
(x + 5) (x + 3) = 0
@

5 + 8 = 13
x + 5 = 0; x + 3 = 0
x = –5; x = –3 … (1) 13 + 16 = 29
(II) y2 + 3y + 2 = 0 29 + 32 = 61
y2 + y + 2y + 2 = 0
18 19 24 37 66 127
y (y + 1) + 2 (y + 1) = 0
(y + 1) (y + 2) = 0
y + 1 = 0; y + 2 = 0 +1 +5 +13 +29 +61
y = –1; y = –2 … (2)
From (1) and (2) x < y. +4 +8 +32
+16
39. Ans. (D)
Sol. The given series is 44. Ans. (B)
606 201 66 21 6 ? Sol. The given series is
(606 ÷ 3) – 1 = 202 – 1 = 201 19 19.6 20.8 23.2 28 ?
(201 ÷ 3) – 1 = 67 – 1 = 66 19 + 0.6 = 19.6
Solved Test Papers of Various Examinations ✫ 767

19,6 + 1.2 = 20.8 1


A and C can do the work in 5 days
20.8 + 2.4 = 23.2 16 3
23.2 + 4.8 = 28 =
3 3
28 + 9.6 = 37.6
∴ In 1 day A and C can do of the
45. Ans. (C) 16
Sol. C.S.A. of cylinder = 2πrh work
13 3 4 1
Area of base of cylinder = 154 cm2 ∴ In 1 day B can do − = =
48 16 48 12
πr2 = 154 cm2 … (1)
∴ B alone can complete the work in 12
Volume of cylinder = 1232 cm2
days.
πr2h = 1232 … (2)

TS
48. Ans. (C)
We need to find r and h from (1) and
Sol. Both I and II are needed to answer the
(2).
question.
46. Ans. (B)
49. Ans. (B)
Sol. Area of circular field = πr2

H
Sol. The data given in I is not sufficient
Area of the square = 4900 cm2
to answer the question, but II alone

G
is sufficient to answer the question.
4900 cm
2 50. Ans. (C)
Sol. Speed of the boat in still water = 15

U km/hr.
Speed of the current = 3 km/hr
O
∴ Side of the square = 4900 = 70 cm Let the distance from B to C be x km
Then distance from A to B
Diameter of the circle is 70 cm = (24 + x) km
H
∴ radius = 35 cm. Time taken by the boat to go
47. Ans. (E) downstream
_T

Sol. Using statement I


Distance 24 + x 24 + x
A, B and C can complete the work in = = =
9 48 Speed 15 + 3 18
3 days = days ( speed of the boat downstream =
13 13
C

13 Speed of boat + Speed of current)


In 1 day, A, B and C can do of Time taken by the boat to go upstream
48
the work
PS

from B to C
4
A and B can complete the work in 4 Distance x x
5 = = =
24 Speed 15 − 3 12
days =
U

5 5 ( speed of the boat upstream =


∴ In 1 day A and B can do of the work speed of boat – speed of current)
24
@

13 5 3 1 Time taken in both cases is the same


∴ In 1 day C can do − = =
48 24 48 16 24 + x x
of the work ∴ =
6 18 12
B and C can complete the work in 6 12 (24 + x) = 18x
48 7 2 (24 + x) = 3x
days = days 48 + 2x = 3x
7 7
∴ In 1 day B and C can do of the 48 = x
48 ∴ Time taken by the boat to travel from
work 7 1 4 1 C to B downstream
∴ In 1 day B can do − = =
48 16 48 12 Distance x 48
work = = =
∴ B can complete the work in 12 days Speed 15 + 3 18
Using statement II 8 2
13 = = 2 hrs
In 1 day A, B and C can do of 3 3
48 = 2 hours 40 minutes.
the work
768 ✫ Quantitative Aptitude

Institute of Banking Personnel Selection (IBPS)


PO (PT) Examination, October 2016
Directions (Q. 1-5): What approximate value will be 7 : 4. What was Somi’s age three years
should come in place of question mark (?) in ago? (in years)
the following questions? (1) 13 (2) 29 (3) 17
(4) 25 (5) 27
1. 90.05 + 281 ÷ 4 – 151.06 = 3 ?
12. A bag contains 24 eggs, out of which 8 are
(1) 27 (2) 343 (3) 216

TS
rotten. The remaining eggs are not rotten.
(4) 729 (5) 243 Two eggs are selected at random. What is
2. (17.98)2 ÷ (4.05)2 × 90.11 ÷ 4.98 = ? the probability that one of the eggs is rotten?
(1) 396 (2) 336 (3) 242 11 17 13

H
(4) 325 (5) 365 (1) (2) (3)
23 23 23
3. 80.04% of 150.16 + 60.02% of 50.07 = ? 32 62
(4) (5)

G
(1) 150 (2) 125 (3) 210 69 69
(4) 175 (5) 140 13. A, B and C started a business with their
4. 628 × 17.996 ÷ 15.04 = ?
(1) 30 (2) 10 (3) 5 U investment in the ratio of 1 : 3 : 5. After 4
months, A invested the same amount as
O
(4) 20 (5) 15 before but B as well as C withdrew half of
their investments. The ratio of their profits
1 1 at the end of the year is
H
5. × 121 + × 76 – ? = 25
8 5 (1) 1 : 2 : 3 (2) 3 : 4 : 15
(1) 5 (2) 45 (3) 15 (3) 3 : 5 : 10 (4) 5 : 6 : 10
_T

(4) 10 (5) 25 (5) 4 : 5 : 10


Directions (Q. 6-10): What will come in place 14. Each edge of a cube is decreased by 30%.
of question mark(?) in the following number Find the decrease in its volume.
C

series? (1) 62.5 (2) 63.5 (3) 67.7


6. 14 8 7 11.5 22 ? (4) 62.7 (5) 65.7
PS

(1) 54 (2) 64 (3) 62 15. The circumference of the semicircle is 180


(4) 58 (5) 56 cm. If the side of a square is 60% more than
7. 8 14 25 46 82 ? the diameter of the circle, then what is the
perimeter of the square?
U

(1) 132 (2) 130 (3) 138


(4) 168 (5) 148 (1) 368 cm (2) 464 cm (3) 486 cm
(4) 448 cm (5) 344 cm
@

8. 13 14 30 93 ? 1885
(1) 358 (2) 336 (3) 364 16. The cost price of two beds are equal. One
(4) 376 (5) 356 bed is sold at a profit of 30% and the other
one for ` 5504 less than the first one. If the
9. 65 70 63 74 61 ? overall profit earned after selling both the
(1) 78 (2) 58 (3) 72 beds is 14%, what is the cost price of each
(4) 77 (5) 59 bed?
10. 9 11 16 33 98 ? (1) ` 17000 (2) ` 16800 (3) ` 17600
(1) 350 (2) 355 (3) 335 (4) ` 17800 (5) ` 17200
(4) 455 (5) 395 17. In Jar A, 180 litres milk was with 36 litres
1 water. Some of the mixture was taken out
11. Five years ago, Somi’s age was of Amit’s from Jar A and put in jar B. If after adding 6
3
age at that time. The ratio of Amit’s age six litres of water in the mixture, the ratio of milk
years hence to Somi’s age twelve years hence to water in Jar B was 5 : 2, then what was the
Solved Test Papers of Various Examinations ✫ 769

amount of mixture that was taken out from 22. What is the difference between the total
Jar A (in litres)? number of bags sold by stores M and N
(1) 24 (2) 54 (3) 30 together in 2009 and that in 2011?
(4) 36 (5) 42 (1) 110 (2) 130 (3) 100
(4) 120 (5) 140
Directions (Q. 18-22): Refer to the graph and
answer the given questions. 23. The ratio of the monthly salary of Om to that
of Pihu is 7 : 9. Om and Pihu both save 20%
Data related to the number of bags sold and 40% of their monthly salary respectively.
by two stores (M & N) during 5 years 1
Om invests of his savings in PPF and Pihu

TS
400 2
7
350 invests of his savings in PPF. If Om and
9
300
Number of bags sold

Pihu together saved ` 17500 in PPF, what is

H
250 Pihu’s monthly salary?
200 (1) ` 72000 (2) ` 36000 (3) ` 45000
(4) ` 35000 (5) ` 54000

G
150

100 Directions (Q. 24-28): In each question two


equations numbered I and II are given. You
50

2008 2009 2010 2011 2012 Uhave to solve both the equations and mark the
answer
O
(1) if x > y (2) if x ≥ y
M 230 170 300 340 180
(3) if x < y (4) if x ≤ y
H
N 320 310 380 250 260 (5) if x = y or no relation can be established
between x and y.
18. In 2008, 30% of the bags sold by store M and
_T

25% of the bags sold by store N were leather 24. (I) 4x2 – 15x + 14 = 0
bags. What was the total number of leather (II) 6y2 – 10y + 4 = 0
bags sold by store M and N together in 2008? 25. (I) 3x2 + 10x + 3 = 0
(1) 163 (2) 155 (3) 145
C

(II) 2y2 + 15y + 27 = 0


(4) 149 (5) 159
26. (I) 7x2 + 12x + 5 = 0
PS

19. What is the ratio of the total number of bags (II) 3y2 + 7y + 2 = 0
sold by stores M and N together in 2009 to
that in 2010? 27. (I) 16x2 – 14x + 3 = 0
(1) 12 : 17 (2) 11 : 14 (3) 13 : 17 (II) 6y2 – 19y + 15 = 0
U

(4) 17 : 12 (5) 14 : 17 28. (I) x2 + 11x + 18 = 0


20. If the average number of bags sold by Store (II) y2 – 81 = 0
@

M in 2011,2012 and 2013 was 350, what was 29. Ravi invested ` P in a scheme A offering
the number of bags sold by the same store simple interest at 10% pa for two years. He
in 2013? invested the whole amount he received from
(1) 510 (2) 540 (3) 550 scheme A in another scheme B offering
(4) 530 (5) 520 simple interest at 12% pa for five years. If
21. The number of bags sold by store N the difference between the interests earned
decreased by what per cent from 2008 to from schemes A and B was ` 1300, what is
2012? the value of P?
3 3 1 (1) ` 2500 (2) ` 2000 (3) ` 3000
(1) 18 (2) 20 (3) 14 (4) ` 2800 (5) ` 4500
4 4 4
1 Directions (Q. 30-34): Study the table and answer
(4) 16 (5) None of these the given questions.
4
770 ✫ Quantitative Aptitude

Data regarding the number of students females studying in the same streams
studying in various streams in various together in Kelly?
Universities (St Christ, PD and Kelly) (1) 5 : 6 (2) 10 : 13 (3) 1 : 2
in the year 2012 (4) 5 : 12 (5) None of these
University St Christ PD Kelly 33. Total number of males studying in stream D
Streams Total Female Total Female Total Female in all the universities together in 2013 was
↓ students students students students students students 1190 more than that in the year 2012. In 2013,
what was the total number of students
A 3000 1800 2800 1200 2000 1500
B 1800 600 1400 800 1200 900
(Male + Female) studying in stream D in all

TS
C 1200 500 1600 500 900 300 the universities together, if the total number
D 600 250 400 180 500 260 of male 4 students in stream D in 2013
4
Please Note: Total students = Female students + Male students constituted of the total number of
7

H
students?
30. The total number of students studying in
(1) 2500 (2) 3500 (3) 4500
stream C in St Christ and PD together is what
(4) 4000 (5) 3200

G
per cent less than that of those studying in
Stream B in the same universities together? 34. What is the average number of male students
studying in stream A in all the given
(1) 20
(4) 15.25
(2) 12.5 (3) 18
(5) None of these
U universities?
(1) 1100 (2) 1250 (3) 1150
O
31. The number of males studying in Stream B
(4) 1200 (5) None of these
in Kelly is what per cent of that studying in
stream C in St Christ? 35. The distance between two cities (M and N)
H
6 1 is 569 km. A train from City M at 8 am travels
(1) 42 (2) 40 (3) 48 towards City N at 53 kmph. Another train
7 9
_T

1 starts from City N at 9 am and travels towards


(4) 54 (5) None of these City M at 76 kmph. At what time will the
6
trains meet?
32. What is the ratio of the total number of
(1) 12:30pm (2) 1:00pm (3) 2:30pm
C

females studying in stream A and B together


(4) 1:30pm (5) 2:00pm
in PD university to the total number of
PS

Answers and Explanatory Notes


1. Ans. (4) = 120 + 30
Sol. 90.05 + 281 ÷ 4 – 151.06 = 150
U

= 90 + (280 ÷ 4) – 151 (BODMAS) 4. Ans. (1)


= 90 + 70 – 151
Sol. 628 × 17.996 ÷ 15.04
@

= 9
= 3 9×9×9 = 25 × (18 ÷ 15)
6
= 729 = 25 × = 30
2. Ans. (5) 5
5. Ans. (1)
Sol. (17.98)2 ÷ (4.05)2 × 90.11 ÷ 4.98
1 1
= {(18)2 ÷ (4)2} × (90 ÷ 5) (BODMAS) Sol. × 121 + × 76 – ? = 25
81 8 5
= × 18 = 15 + 15 – ? = 25
4 ∴ ? = 5.
= 364.5 6. Ans. (5)
= 365 Sol. 14 8 7 11.5 22 ?
3. Ans. (1) The series is
Sol. 80.04% of 150.16 + 60.02% of 50.07 (14 × .5) + 1 = 8
80 60 [8 × (.5 + .5)] – 1 = 7
= × 150 + × 50
100 100 [7 × (1.5)] + 1 = 11.5
Solved Test Papers of Various Examinations ✫ 771

[11.5 × 2] – 1 = 22 7x
[22 × 2.5 + 1 = 56 44 – 119 = – 4x
3
7. Ans. (3) 5x
Sol. The series is –75 = −
3
8 14 25 46 82 ?
75 × 3
8 + 6 = 14; 14 + 11 = 25; = x ⇒ x = 45.
25 + 21 = 46; 46 + 36 = 82 5
Somi’s age 5 years ago
8 14 25 46 82 138 1
= × 45 = 15 years
3

TS
6 11 21 36
∴ Somi’s age 3 years ago
56
= 15 + 2 = 17 years.
12. Ans. (4)
5 10 15 20 Sol. Total number of eggs = 24

H
(multiples of 5) Number of rotten eggs = 8
8. Ans. (4) Number of good eggs = 24 – 8 = 16

G
Sol. The series is P (one of the eggs is rotten)
13 14 30 93 ? 1885 Favourable outcomes
(13 × 1) + 1 = 14 =
(14 × 2) + 2 = 30
(30 × 3) + 3 = 93 U Total number of outcomes
Selecting one rotten egg from 8 eggs
O
(93 × 4) + 4 = 376 and one good egg from 16 eggs
=
(376 × 5) + 5 = 1885. Selecting two eggs from 24 eggs
H
9. Ans. (1)
Sol. The series is
8
C1 × 16C1
= 24
65 70 63 74 61 ? C2
_T

65 – 63 = 2; 63 – 61 = 2
70 + 4 = 74; 74 + 4 = 78 8 × 16 n
Cr =
n!
=
10. Ans. (2) 24 ! ( n − r) ! r !
( 24 − 2) ! 2!
C

Sol. The series is


9 11 16 33 98 ? 8 × 16 × 2
=
PS

9 + 12 + 1 = 11
24 × 23
11 + 22 + 1 = 16
16 + 42 + 1 = 33 32
=
33 + 82 + 1 = 98 69
U

98 + 162 + 1 = 355 13. Ans. (4)


11. Ans. (3) Sol. Ratio of A, B and C’s investment
@

Sol. Let Amit’s age 5 years ago = x years = 1 : 3 : 5


1 Let A invest x, B invest 3x and C
Then Somi’s age = x
3 invest 5x in the business
Amit’s age 6 years hence
= x + 5 + 6 = x + 11 In 4 months A’s investment = 4x
Somi’s age 12 years hence For the remaining 8 months A’s
1 x investment = 2x × 8 = 16x
= x + 5 + 12 = + 17 In 4 months B’s investment
3 3
According to the question = 3x × 4 = 12x
x + 11 For the remaining 8 months B’s
x 7
3x
+ 17 = 4 investment = × 8 = 12x
3 2
7x In 4 months C’s investment
4x + 44 = + 119 = 5x × 4 = 20x.
3
772 ✫ Quantitative Aptitude

For the remaining 8 months C’s Profit earned after selling both beds
5x = 14%
investment = × 8 = 20x C.P. of 2 beds = 2x
2
The ratio of their profits S.P. of 2 beds = C.P. + Profit
(4x + 16x) : (12x + 12x) : (20x + 20x) 14
= 20x : 24x : 40x = 2x + × 2x
100
= 5 : 6 : 10. 228 x
14. Ans. (5) = … (3)
100
Sol. Let the edge of the cube be x According to the question
Edge of the cube when decreased by

TS
130x 130 x 228 x
30 7x + – 5504 =
30% = x – ×x = 100 100 100
100 10
[from (1), (2), (3)]
Volume of the cube of edge x = x3 260x – 550400 = 228x

H
Volume of the cube of edge 32x = 550400
7x 7x
3 x = 17200

G
= Cost price of each bed is Rs 17200.
10 10
17. Ans. (4)
Decrease in volume

= x3 –
7x
3

U Sol. Ratio of milk and water in jar A


= 180 : 36
= 5 : 1
O
10
657 x 3 Let 6x litres of mixture be taken out
= from jar A and put in Jar B then
H
1000
= .657x3 Milk in jar B = 5x and water in jar
∴ Volume is decreased by 65.7%. B = x
_T

15. Ans. (4) After adding 6 litres of water the


Sol. Circumference of the semicircle ratio is 5 : 2
= 2r + πr = 180 5x 5
=
C

22 x+6 2
2r + r = 180
7 10x = 5x + 30
PS

180 5x = 30 ⇒ x = 6
⇒ r = = 35 cm ∴ The mixture that was taken out from
22
2+ jar A = 6x
7
Side of the square = 6 × 6 = 36 litres.
U

60 18. Ans. (4)


= 2r + × 2r Sol. Total number of leather bags sold by
100
@

320r store M and N together in 2008


= = 3.2 × 35 = 112 cm 30 25
100 = × 230 + × 320
∴ Perimeter of the square = 4 (side) 100 100
= 4 × 112 = 448 cm. = 69 + 80
16. Ans. (5) = 149.
Sol. Let the C.P. of each bed be Rs x 19. Ans. (1)
S.P. of first bed = C.P. + Profit Sol. Total number of bags sold by stores
30 M and N in 2009
= x + × x = 170 + 310 = 480
100
130 x Number of bags sold in 2010
= … (1) = 300 + 380 = 680
100
S.P. of second bed 480 12
Required ratio = =
130 x 680 17
= – 5504 … (2)
100 = 12 : 17.
Solved Test Papers of Various Examinations ✫ 773

20. Ans. (4) 24. Ans. (1)


Sol. Let number of bags sold by M in Sol. (I) 4x2 – 15x + 14 = 0
2013 = x 4 x2 – 8x – 7x + 14 = 0
Total number of bags sold by M in 4x (x – 2) – 7 (x – 2) = 0
2011, 2012 and 2013 (x – 2) (4x – 7) = 0
= 340 + 180 + x x – 2 = 0; 4x – 7 = 0
= 520 + x … (1) 7
Average = 350 x = 2; x =
4
∴ Total = 350 × 3 = 1050 … (2) x = 2; x = 1.75 … (1)
∴ 520 + x = 1050 (from (1) and (2)

TS
2
x = 1050 – 520 (II) 6y – 10y + 4 = 0
x = 530 2 (3y2 – 5y + 2) = 0
Number of bags sold by M is 2013 3y2 – 5y + 2 = 0
3y2 – 3y – 2y + 2 = 0

H
= 530.
21. Ans. (1) 3y (y – 1) – 2 (y – 1) = 0
Sol. Difference in the number of bags (y – 1) (3y – 2) = 0

G
sold from 2008 to 2012 y– 1 = 0; 3y – 2 = 0
= 320 – 260 = 60 2
y= 1; y =
Percentage decrease
=
60 3
× 100 = 18 % U 3
y = 1; y = .66
From (1) and (2) x > y.
… (2)
O
320 4
22. Ans. (1) 25. Ans. (2)
Sol. Total number of bags sold by stores Sol. (I) 3x2 + 10x + 3 = 0
H
M and N in 2009 = 170 + 310 = 480 3x2 + 9x + x + 3 = 0
Total number of bags sold by stores 3x (x + 3) + 1 (x + 3) = 0
_T

M and N in 2011 = 340 + 250 = 590 (x + 3) (3x + 1) = 0


Difference = 590 – 480 = 110. x + 3 = 0; 3x + 1 = 0
23. Ans. (3) 1
x= –3; x = –
Sol. Let the monthly salary of Om be 7x
C

3
and that of Pihu be 9x x = –3; x = –0.33 … (1)
PS

20 7x (II) 2y2 + 15y + 27 = 0


Om’s savings = × 7x = 2y2 + 6y + 9y + 27 = 0
100 5
2y (y + 3) + 9 (y + 3) = 0
40 18 x (y + 3) (2y + 9) = 0
Pihu’s saving = × 9x =
U

100 5 y + 3 = 0; 2y + 9 = 0
Om invests half his savings 9
y = –3; y = –
@

1 7x 7x 2
= × = y = –3; y = –4.5 … (2)
2 5 10
From (1) and (2) x ≥ y.
7 18 x 14 x
Pihu invests = × = 26. Ans. (5)
9 5 5 Sol. (I) 7x2 + 12x + 5 = 0
Total savings = Rs 17500 7x2 + 7x + 5x + 5 = 0
7x 14x 7x (x + 1) + 5 (x +1) = 0
+ = 17500 (x + 1) (7x + 5) = 0
10 5 (x + 1) = 0; 7x + 5 = 0
7x + 28x = 175000
5
35x = 175000 x = –1; x = –
x = 5000 7
x = –1; x = –0.71 … (1)
∴ Pihu’s monthly salary
= 9x = 9 × 5000 = Rs 45000 (II) 3y2 + 7y + 2 = 0
3y2 + 6y + y + 2 = 0
774 ✫ Quantitative Aptitude

3y (y + 2) + 1 (y + 2) = 0 Difference between the interests


(y + 2) (3y + 1) = 0 = Rs 1300
y + 2 = 0; 3y + 1 = 0 18 P P
− = 1300
1 25 5
y= –2; y = – 18P – 5P = 1300 × 25
3
y = –2; y = –0.33 … (2) 13P = 32500
From (1) and (2) no relation can be 32500
P = = Rs 2500.
established. 13
27. Ans. (3) 30. Ans. (2)
Sol. (I) 16x2 – 14x + 3 = 0 Sol. Total number of students in stream

TS
16x2 – 8x – 6x + 3 = 0 C in St. Christ and PD
8x (2x – 1) – 3 (2x – 1) = 0 = 1200 + 1600 = 2800
(2x – 1) (8x – 3) = 0 Total number of students in stream

H
2x – 1 = 0; 8x – 3 = 0 B in the same universities
1 3 = 1800 + 1400 = 3200
x= ; x = Different = 3200 – 2800 = 400

G
2 8
x = 0.5; x = 0.37 … (1) Required percentage
Difference
(II) 6y2 – 19y + 15 = 0
6y2 – 9y – 10y + 15 = 0
3y (2y – 3) – 5 (2y – 3) = 0 U =
Students in stream B
400
× 100
O
(2y – 3) (3y – 5) = 0 = × 100
3200
2y – 3 = 0; 3y – 5 = 0 100
H
3 5 = = 12.5% less.
y= ;y= 8
2 3 31. Ans. (1)
_T

y = 1.5; y = 1.6 … (2) Sol. Number of male students in stream


From (1) and (2) x < y. B in Kelly = 1200 – 900 = 300
28. Ans. (5) Number of students in stream C in
Sol. (I) x2 + 11x + 18 = 0 St. Christ
C

x2 + 9x + 2x + 18 = 0 = (1200 – 500) = 700 males


x (x + 9) + 2 (x + 9) = 0 Required percentage
PS

(x + 9) (x + 2) = 0 Difference
x + 9 = 0; x + 2 = 0 = × 100
Number of males in C in St. Christ
x = –9; x = –2 .. (1) 300 6
U

(II) y2 – 81 = 0 = × 100 = 42 %.
700 7
y2 = 81 ⇒ y2 = ±9 32. Ans. (1)
@

y = ±3 … (2) Sol. Total number of females in A and B


From (1) and (2) no relation can be in PD university
established. = 1200 + 800 = 2000
29. Ans. (1) Total number of females in A and B
Sol. S.I. on Rs P at 10% pa for 2 years in Kelly
PTR P × 2 × 10 P = 1500 + 900 = 2400
= = = 2000 5
100 100 5 =
Required ratio =
P 6P 2400 6
Amount received = P + = = 5 : 6.
5 5
Interest from scheme B 33. Ans. (2)
PTR 6 P 5 × 12 Sol. Total number of males in D in all the
= = × universities in 2012
100 5 100
18 P = (600 – 250) + (400 – 180) + (500 – 260)
= = 350 + 220 + 240
25
= 810
Solved Test Papers of Various Examinations ✫ 775

Total number of males in D in all the 35. Ans. (2)


universities in 2013 Sol. By 9 a.m. the train starting from M
= 1190 + 810 = 2000 has covered 53 km.
Let the total number of students = x
M 569 km N
According to the question
4
× x = 2000 8 a.m. 9 a.m.
7
4x = 2000 × 7 53 km/hr 76 km/hr
2000 × 7
x = Remaining distance

TS
4
x = 3500 = 569 – 53 = 516 km
Total number of students in D
Relative speed
= 3500.

H
34. Ans. (1) = 53 + 76 = 129 km/hr.
Sol. Number of male students in A in all 516
the universities ∴ They meet after hrs.

G
= (3000 – 1800) + (2800 – 1200) 129
+ (2000 – 1500) Distance
Time =
= 1200 + 1600 + 500
= 3300 U = 4 hrs.
Speed
O
3300
∴ Average = = 1100. They meet at 9 a.m. + 4 hrs = 1 p.m.
3
H
C _T
PS
U
@
776 ✫ Quantitative Aptitude

Institute of Banking Personnel Selection (IBPS)


Specialist Officer (Marketing) Examination, February 2016
Directions—(Q. 1-10) Each of the questions 5. What is the minimum passing percentage in
below consists of a question and two statements a test?
numbered I and II given below it. You have to I. Raman scored 25% marks in the test
decide whether the data given in the state-ments and Sunil scored 288 marks which is
are sufficient to answer the questions. 128 more than Raman.
Read both the statements and give answer— II. Raman scored 64 marks less than the

TS
(A) If the data in statement I alone are minimum passing marks.
sufficient to answer the question, while 6. What is the value of x2 + y + z?
the data in statement II alone are not I. 4x + 3y + 5z = 60 and 2x = y, 2y = z
sufficient to answer the question

H
II. 3x + 3y + 2z = 34 and 2x + 5y + 6z = 72
(B) If the data in statement II alone are
sufficient to answer the question, while 7. Whose body weight is second highest

G
the data in statement I alone are not among the five boys Arun, Vinay, Suraj, Raju
sufficient to answer the question and Pratap?
I. Average weight of Arun, Suraj and
(C) If the data either in statement I alone
or statement II alone are sufficient to
answer the question U Vinay is 68 kg and average weight of
Raju and Pratap is 72 kg. Also Suraj
O
(D) If the data given in both statement I is 78 kg, Raju is 68 kg and Vinay is
and II together are not sufficient to 46 kg.
H
answer the question II. Average weight of Arun, Suraj, Vinay
(E) If the data in both statement I and II and Raju is 68 kg and also Suraj is 78
kg, Raju is 68 kg and Vinay is 46 kg.
_T

together are necessary to answer the


question All of them have different weight.
1. What is the area of the circle? 8. What is the circumference of semi circle?
1
I. Perimeter of the circle is 88 cms.
C

I. The radius of semicircle is of the


II. Diameter of the circle is 28 cms. 2
side of square.
PS

2. What is the rate of interest? II. The area of square is 196 square cm.
I. Simple Interest accrued on an amount 9. What is the population of the city A?
of ` 25,000 in two years is less than I. The ratio of the population of males
the compound interest for the same and females in city A is 27 : 23 and
U

period by ` 250. the differ-ence between their popula-


II. Simple interest accrued in 10 years is tion is 100000.
@

equal to the principal. II. The population of city A is 80% of that


of city B. The difference of population
3. What is the number of trees planted in the of city A and city B is 312500.
field in rows and columns?
I. Number of columns is more than the 10. How many students did participate in
number of rows by 4. elocution?
I. The students who participate in
II. Number of columns is 20.
dancing were 150% more than that
4. What is the speed of the current? who partici-pated in elocution.
I. A man can swim a distance of 9 kms II. 150 students participated in dancing.
1
in 1 hrs down-stream. Directions—(Q. 11-15) Study the following graph
2
II. While coming back up-stream it takes carefully and answer the questions given below
him 3 hrs to cover the same distance. it—
Solved Test Papers of Various Examinations ✫ 777

Percentage profit earned by two 15. If the income of company B in 2010 and 2011
companies A and B over the six years were in the ratio of 2 : 3 respectively. What
Company A was the respective ratio of expenditure of
60 Company B that company in these two years?
50
(A) 20 : 29 (B) 9 :10
(C) 29 : 45 (D) 10 : 29
Percentage Profit

40
(E) None of these
30
Directions—(Q. 16-20) Study the pie-charts

TS
20 carefully to answer the given questions—
10 Percentage of students enrolled in
different activities in a school
2008 2009 2010 2011 2012 2013

H
N = 3000
A 35 45 35 50 30 40

G
B 40 35 50 45 40 45
Singing Dancing
11. Expenditure of company B in 2009 and 2010
21%
are ` 12 lakhs and ` 14.5 lakhs respectively.
What was the total income of company B in
2009 and 2010 together (in lakh rupees)? U 24%
O
Swimming
(A) 39.75 (B) 37.95 16%
(C) 38.75 (D) 38.55 Craft
H
(E) None of these 25%
Drawng
12. Ratio of expenditure of company A and B in 14%
_T

2012 was 3 : 4 respectively. What was the


respective ratio of their incomes in 2012?
(A) 21 : 26 (B) 13 : 14 Percentage break-up of girls enrolled
C

(C) 14 : 13 (D) 26 : 21 in these activities out of the total students


(E) None of these
N = 1750
PS

13. Total expenditure of company A in all the


years together was 82.5 lakhs. What was the
total income of the company A in all the Dancing Craft
years together? 20% 22%
U

(A) l.23 crores


(B) 98.75 crores
@

(C) 99.85 crores Drawing


(D) Cannot be determined Singing 16%
(E) None of these 28%

14. If the expenditure of company A and B in Swimming


2013 were equal and the total incomes of the 14%
two companies was ` 5.7 lakhs. What was
the total expenditure of the two companies 16. Number of girls enrolled in dancing form are
in 2013? what per cent of total number of students
(A) 4 lakhs enrolled in the school? (rounded off to two
(B) 2 lakhs digits after decimal)
(C) 4.2 lakhs (A) 12.35 (B) 14.12
(D) Cannot be determined (C) 11.67 (D) 10.08
(E) None of these (E) None of these
778 ✫ Quantitative Aptitude

17. How many boys are enrolled in Singing and 29. (I) x2 – 14x + 48 = 0
Craft together? (II) y2 + 6 = 5y
(A) 505 (B) 610
30. (I) 38x2 – 3x – 11 = 0
(C) 485 (D) 420
(II) 28y2 + 32y + 9 = 0
(E) None of these
31. Two men P and Q start a journey from same
18. What is the respective ratio of number of 1
girls enrolled in swimming to the number place at a speed of 3 km/hr and 3 km/hr
2
of boys enrolled in swimming? respectively. If they move in the same
(A) 47 : 49 (B) 23 : 29 direction then what is the distance between

TS
(C) 29 : 23 (D) 49 : 47 them after 4 hours?
(E) None of these 1
(A) 3 km (B) 2 km
19. What is the total number of girls enrolled in 2
swimming and drawing together? 1

H
(C) 2 km (D) 3 km
(A) 480 (B) 525 2
(C) 505 (D) 495 (E) None of these

G
(E) None of these
Directions—(Q. 32-35) What will come in place
20. What is the appropriate percentage of boys of question mark (?) in the given question?
in the school?
(A) 34 (B) 56
U 32. 14 × 627 ÷ (1089) = (?)3 + 141
(B) (125)3
O
(C) 28 (D) 50 (A) 5 5
(E) None of these (C) 25 (D) 5
(E) None of these
H
Directions—(Q. 21-30) In each question, two
equations numbered I and II are given. You 21.5 21 13.5 (?)1/3 17
33. + − = +
_T

have to solve both the equations and mark an 5 6 15 4 30


appropriate answer, if
(A) 2 (B) 8
(A) x < y (B) x > y
(C) 512 (D) 324
(C) x ≥ y (D) x ≤ y
C

(E) None of these


(E) relationship between x and y cannot be
established. 34. ( 7 + 11)2 = (?)1/3 + 2 (847 ) + 122
PS

21. (I) 6x2 + 5x + 1 = 0 (A) 36 + 44 (7 ) (B) 6


(II) 15y2 + 8y + 1 = 0 (C) 216 (D) 36
(E) None of these
22. (I) x2 + 5x + 6 = 0
U

(II) 4y2 + 24y + 35 = 0 2


18 455 61
2
23. (I) 2x + 5x + 3 = 0 35. × ÷ =1
@

4 19 799
(II) y2 + 9y + 14 = 0
(A) 6320 (B) 6400
24. (I) 88x2 – 19x + 1 = 0 (C) 6350 (D) 6430
(II) 132y2 – 23y + 1 = 0 (E) 6490
25. (I) 6x2 – 7x + 2 = 0 Directions—(Q. 36-40) What will come in place
(II) 20y2 – 31y + 12 = 0
of question-mark (?) in the given number series?
26. (I) 6x2 + 23x + 20 = 0 36. 28 39 63 102 158 ?
(II) 6y2 + 31y + 35 = 0
(A) 232 (B) 242 (C) 233
27. (I) x2 = 81 (D) 244 (E) None of these
(II) y2 – 18y + 81 = 0
37. 7 16 141 190 919 ?
28. (I) 4x2 + 20x + 21 = 0 (A) 1029 (B) 1019 (C) 1020
(II) 2y2 + 17y + 35 = 0 (D) 1030 (E) None of these
Solved Test Papers of Various Examinations ✫ 779

38. 12 17 32 57 92? 45. What is the total number of books


(A) 198 (B) 195 (C) 137 distributed by publishers O and Q?
(D) 205 (E) None of these (A) 26702 (B) 27324 (C) 55026
39. 19 25 45 87 159 ? (D) 54026 (E) None of these
(A) 254 (B) 279 (C) 284 46. Meena Kumari goes to a shop and buys a
(D) 269 (E) None of these saree, costing ` 5,225, including sales tax of
40. 83 124 206 370 698 ? 12%. The shopkeeper gives her a discount,
(A) 1344 (B) 1324 (C) 1364 so that the price is decreased by an amount
(D) 1334 (E) None of these equivalent to sales tax. The price is

TS
decreased by (nearest value).
Directions—(Q. 41-45) Study the table carefully (A) ` 615 (B) ` 650 (C) ` 560
and answer the given questions— (D) ` 580 (E) ` 680
47. Mr. Phanse invests an amount of ` 24,200 at

H
Publishing Number of Ratio of Percentage Number of
Houses books Academic of books distributors the rate of 4 p.c.p.a. for 6 years to obtain a
published and distributed in publishing
Non-academic house
simple interest, later he invests the principal

G
books amount as well as the amount obtained as
simple interest for another 4 years at the
M 28200 7:3 81 17
N
O
P
32200
29700
31200
5 : 9
6 : 5
8 : 5
74
92
86
23
18
24 U same rate of interest. What amount of simple
interest will be obtained at the end of the last
4 years?
O
Q 33800 7 : 6 79 25 (A) ` 4,800 (B) ` 4,850.32
R 35700 11 : 6 82 21 (C) ` 4,801.28 (D) ` 4,700
H
S 37800 5 : 13 89 24 (E) ` 4,870.32
41. What is the difference between the number Directions—(Q. 48-49) The ques-tions are based
_T

of academic books published by publishing on the following information—


house M and P? There are three different cable channels
(A) 450 (B) 640 (C) 540 namely ahead, luck and bang. In a survey, it
C

(D) 504 (E) None of these was found that 85% of viewers, respond to bang,
20% to luck and 30% of ahead 20% of viewers
42. How many books were given to each respond to exactly two channels and 5% to none.
PS

distributor by publisher Q if each publisher


gets equal number of books? 48. What percentage of the viewers responded
(A) 1806 (B) 1068 (C) 1608 to all three?
(A) 10 (B) 12 (C) 14
U

(D) 1308 (E) None of these


(D) 16 (E) 11
43. What is the average number of non-
49. Assuming 20% respond to ahead and bang
@

academic books published by publishers R


and 16% respond to bang and luck. What is
and S?
the percentage of viewers who watch only
(A) 18750 (B) 18850 (C) 19950
luck?
(D) 18950 (E) 19990
(A) 20 (B) 10 (C) 16
44. If the total number of books published by (D) 18 (E) None of these
publishers P, Q and R is increased by 30%
50. A milkman mixes 20 L of water with 80 L of
and the total number of books published by milk. After selling one-fourth of this mixture,
remaining publishers be decreased by 20%, he adds water to replenish the quantity that
what will be the new average of books he had sold. What is the current proportion
published by all the publishers? of water to milk?
(A) 33418 (B) 33318 (C) 32518 (A) 2 : 3 (B) 1 : 2 (C) 1 : 3
(D) 33618 (E) None of these (D) 2 : 1 (E) 3 : 4
780 ✫ Quantitative Aptitude

Answers and Explanatory Notes


1. Ans. (C) 5. Ans. (E)
Sol. Area of the circle = πr2 Sol. Both I and II are necessary to solve
88 the question.
Perimeter = 2πr = 88 ⇒ r =
2π 6. Ans. (A)
Diameter of the circle Sol. In I we have 3 equations in 3
= 2r = 28 ⇒ r = 14 cm. variables x, y, z which can be solved
2. Ans. (C) whereas an II there are only 2
PTR 25000 × 2 × R equations.

TS
Sol. (I) S.I. = =
100 100 7. Ans. (A)
R
n
R
2 Sol. In II Pratap’s weight is not given.
C.I. = P 1 + = 25000 1 +
100 100 8. Ans. (E)

H
C.I. – S.I. = 250 Sol. Circumference of semi-circle = πr
Solving this equation we can find If a is the side of the square then
rate of interest.

G
1
(II) S.I. = P r = a … (1)
2
PTR 2
a = 196 … (2)
100
10 × R
= P
U Solving (1) and (2) we can find πr.
O
= 1 ⇒ R = 10%. 9. Ans. (C)
100 Sol. In (I), let number of males be 27x and
3. Ans. (E) number of females be 23x
H
Sol. Let number of rows be x, then number ∴ 27x – 23x = 100000
of columns is x + 4 Solving we can find the solution
_T

x + 4 = 20 ⇒ x = 16
∴ Number of columns = 20 In (II), Let population of city B be x
Number of trees = 20 × 16 Then population of city A
= 320 80
C

= × x
4. Ans. (E) 100
Sol. Let the man swim at x km/hr and let Difference = 312500
PS

the speed of the current be y km/hr 80


Distance = 9 km x = × x = 312500
100
Speed of the man swimming Solving we can find the population
upstream = (x – y) km/hr
U

of city A.
Speed of the man swimming 10. Ans. (E)
downstream = (x + y) km/hr Sol. Both I and II are necessary to solve
@

Time taken to go upstream the problem.


Distance 9 11. Ans. (B)
= = Sol. Let income of B in 2009 = Rs x
Speed x−y
Profit %
Time taken to go downstream
9 Income – Expenditure
= = × 100
x+y Expenditure
According to the question ( x − 12)
9 3 35 = × 100
= … (1) 12
x+y 2 12 × 35 = 100x – 1200
9 420 + 1200
= 3 … (2) = x
x−y 100
Solving (1) and (2) we can find the 16.2 = x
speed of the current.
Solved Test Papers of Various Examinations ✫ 781

Let income of B in 2010 = y Percentage profit of A = 40%


y − 14.5 Percentage profit of B = 45%
50 = × 100 Profit %
14.5
50 × 14.5 = 100y – 1450 Income – Expenditure
725 + 1450 = 100y = × 100
Expenditure
2175 y−x
= y 40 = × 100
100 x
21.75 = y 40x = 100y – 100x
Total income = x + y = 16.2 + 21.75 140x = 100y

TS
= 37.95 lakh
14 x
12. Ans. (E) = y … (1)
Sol. Let expenditure of company A in 10
2012 be 3x and that of B be 4x z−x

H
Profit % 45 = × 100
x
Income – Expenditure 45x = 100z – 100x

G
= × 100 145x = 100z
Expenditure
For A profit % in 2012 = 30% 29 x

∴ 30 =
Income of A – 3x
× 100 U 20
= z
Total income = y + z
… (2)
O
3x
14 x 29 x
∴ Income of A 5.7 = = from (1) and (2)
10 20
H
90 x + 300 x 390 x 39 x
= = = 57 x
100 100 10 5.7 =
_T

10
For B profit % in 2012 = 40% 5.7 × 20
Income of B – 4x = x
∴ 40 = × 100 57
4x 2= x
C

160 x + 400 x ∴ Total expenditure of A and B


Income of B =
100 = 2x = 2 × 2 = 4 lakh.
PS

56 x 15. Ans. (C)


= Sol. Let income of B in 2010 be 2x and
10
in 2011 be 3x
Income of A 39 x 10 Let the expenditure of B in 2010 and
U

∴ = ×
Income of B 10 56 x 2011 be y + z respectively.
Profit %
@

39
= = 39 : 56. Income – Expenditure
56 = × 100
13. Ans. (D) Expenditure
Sol. We cannot find expenditure of In 2010
company A in the given years 2x − y
50 = × 100
separately. So we cannot find the y
income of the company. 50y = 200x – 100y
14. Ans. (A) 150y = 200x
Sol. Income of A and B in 2013 200 x 4 x
y = =
= Rs 5.7 lakh 150 3
Let expenditure of A In 2011
= x and income be y 3x − z
This expenditure of B = x and income 45 = × 100
z
be z 45z = 300x – 100z
782 ✫ Quantitative Aptitude

145z = 300x (14 + 16)


300 x = × 1750
z = 100
145 30
60 x = × 1750 = 525.
z = 100
29 20. Ans. (E)
4x 29 29 Sol. Percentage of boys
Required ratio = × =
3 60 x 45 3000 − 1750
= 29 : 45 = × 100
3000
16. Ans. (C) = 41.67.

TS
Sol. Required percentage 21. Ans. (D)
20
= × 1750 = 350 Sol. (I) 6x2 + 5x + 1 = 0
100
Percentage w.r.t. total number of 6x2 + 3x + 2x + 1 = 0

H
students in the school 3x (2x + 1) + 1 (2x + 1) = 0
350 (2x + 1) (3x + 1) = 0

G
= × 100 = 11.67% 2x + 1 = 0; 3x + 1 = 0
3000
17. Ans. (A) 1 1
x = – ; x = –
Sol. Boys enrolled in singing and craft
together
21 + 25 28 + 22 U 2 3
x = –0.5; x = –0.33 … (1)
O
= × 3000 – × 1750 (II)15y2 + 8y + 1 = 0
100 100
15y2 + 5y + 3y + 1 = 0
H
= 1380 – 875
5y (3y + 1) + 1 (3y + 1) = 0
= 505.
18. Ans. (D) (3y + 1) (5y + 1) = 0
_T

Sol. Number of girls enrolled in 3y + 1 = 0; 5y + 1 = 0


swimming 1 1
y = – ; y = –
14 3 5
= × 1750
C

100 y = –0.33; y = –0.2 … (2)


Number of boys enrolled in From (1) and (2) x ≤ y
PS

swimming 22. Ans. (E)


16 14 Sol. (I) x2 + 5x + 6 = 0
= × 3000 – × 1750
100 100 x2 + 3x + 2x + 6 = 0
U

Required ratio x (x + 3) + 2 (x + 3) = 0
14 (x + 3) (x + 2) = 0
× 1750
@

= 100 x + 3 = 0; x + 2 = 0
16 14
× 3000 − × 1750 x = –3; x = –2 … (1)
100 100
(II)4y2 + 24y + 35 = 0
14 × 1750
= 16 × 3000 − 14 × 1750 4y2 + 10y + 14y + 35 = 0
2y (2y + 5) + 7 (2y + 5) = 0
1225 245 (2y + 5) (2y + 7) = 0
= =
1175 235 2y + 5 = 0; 2y + 7 = 0
49 5 7
= = 49 : 47. y = – ; y = –
47 2 2
19. Ans. (B) y = –2.5; y = –3.5 … (2)
Sol. Total number of girls enrolled in From (1) and (2) relationship between
swimming and drawing x and y cannot be established.
Solved Test Papers of Various Examinations ✫ 783

23. Ans. (B) (II)20y2 – 31y + 12 = 0


Sol. (I) 2x2 + 5x + 3 = 0 20y2 – 15y – 16y + 12 = 0
2x2 + 2x + 3x + 3 = 0 5y (4y – 3) – 4 (4y – 3) = 0
2x (x + 1) + 3 (x + 1) = 0 (4y – 3) (5y – 4) = 0
(x + 1) (2x + 3) = 0 4y – 3 = 0; 5y – 4 = 0
x + 1 = 0; 2x + 3 = 0 3 4
y = ; y =
3 4 5
x = –1; x = – = –1.5 ... (1) y = 0.75; y = 0.8 … (2)
2

TS
From (1) and (2) x < y
(II)y2 + 9y + 14 = 0
26. Ans. (E)
y2 + 7y + 2y + 14 = 0
Sol. (I) 6x2 + 23x + 20 = 0

H
y (y + 7) + 2 (y + 7) = 0
6x2 + 8x + 15x + 20 = 0
(y + 7) (y + 2) = 0
2x (3x + 4) + 5 (3x + 4) = 0

G
y + 7 = 0; y + 2 = 0
(3x + 4) (2x + 5) = 0
y = –7; y = –2 … (2)
3x + 4 = 0; 2x + 5 = 0
From (1) and (2) x > y
24. Ans. (C)
U x= –
4
; x = –
5
O
Sol. (I) 88x2 – 19x + 1 = 0 3 2
88x2 – 11x – 8x + 1 = 0 x= –1.3; x = –2.5 … (1)
H
11x (8x – 1) – 1 (8x – 1) = 0 2
(II)6y + 31y + 35 = 0
(8x – 1) (11x – 1) = 0
6y2 + 21y + 10y + 35 = 0
_T

8x – 1 = 0; 11x – 1 = 0
1 1 3y (2y + 7) + 5 (2y + 7) = 0
x = ; x = … (1) (2y + 7) (3y + 5) = 0
8 11
2y + 7 = 0; 3y + 5 = 0
C

(II)132y2 – 23y + 1 = 0
132y2 – 11y – 12y + 1 = 0 7 5
y = – ; y = –
PS

11y (12y – 1) – 1 (12y – 1) = 0 2 3


(12y – 1) (11y – 1) = 0 y = –3.5; y = –1.66 … (2)
12y – 1 = 0; 11y – 1 = 0 From (1) and (2), relationship between
x and y cannot be established.
U

1 1
y = ; y = … (2) 27. Ans. (D)
12 11
@

Sol. (I) x2 = 81
From (1) and (2) x ≥ y
x = 81 = ± 9 … (1)
25. Ans. (A) (II)y2 – 18y + 81 = 0
Sol. (I) 6x2 – 7x + 2 = 0 y2 – 9y – 9y + 81 = 0
6x2 – 3x – 4x + 2 = 0 y (y – 9) – 9 (y – 9) = 0
3x (2x – 1) – 2 (2x – 1) = 0 (y – 9) (y – 9) = 0
(2x – 1) (3x – 2) = 0 ⇒ y – 9 = 0 ⇒ y = 9 … (2)
2x – 1 = 0; 3x – 2 = 0 From (1) and (2) x ≤ y

1 2 28. Ans. (C)


x = ; x = Sol. (I) 4x2 + 20x + 21 = 0
2 3
x = 0.5; x = 0.66 … (1) 4x2 + 14x + 6x + 21 = 0
784 ✫ Quantitative Aptitude

2x (2x + 7) + 3 (2x + 7) = 0 14y (2y + 1) + 9 (2y + 1) = 0


(2x + 7) (2x + 3) = 0 (2y + 1) (14y + 9) = 0
2x + 7 = 0; 2x + 3 = 0 2y + 1 = 0; 14y + 9 = 0
7 3 1 9
x = – ; x = – y = – ; y = –
2 2 2 14
x = –3.5; x = –1.5 … (1) y = –0.5; y = –0.6 … (2)
(II)2y2 + 17y + 35 = 0 From (1) and (2) x ≥ y
2y2 + 10y + 7y + 35 = 0 31. Ans. (C)

TS
2y (y + 5) + 7 (y + 5) = 0 Sol. Speed of P = 3 km/hr
(y + 5) (2y + 7) = 0 1
y + 5 = 0; 2y + 7 = 0 Speed of Q = 3 km/hr
2

H
y = –5; y = –7/2 In 4 hrs P covers a distance of
y = –5; y = –3.5 … (2) 3 × 4 = 12 km (D = s × t)

G
From (1) and (2) x ≥ y 7
29. Ans. (B) In 4 hrs Q covers = × 4 = 14 km
2
Sol. (I) x2 – 14x + 48 = 0
x2 – 6x – 8x + 48 = 0 U Distance between them
= 14 – 12 = 2 km.
O
x (x – 6) – 8 (x – 6) = 0 32. Ans. (D)
(x – 6) (x – 8) = 0
Sol. 14 × 627 ÷ 1089 = (?)3 + 141
H
x – 6 = 0; x – 8 = 0
14 × (627 ÷ 33) = (?)3 + 141
x = 6; x = 8 … (1)
_T

14 × 19 = (?)3 + 141
(II)y2 + 6 = 5y
266 = (?)3 + 141
y2 – 5y + 6 = 0 125 = (?)3
y2 – 3y – 2y + 6 = 0
C

(5)3 = (?)3
y (y – 3) – 2 (y – 3) = 0 5 =?
PS

(y – 3) (y – 2) = 0 33. Ans. (E)


y – 3 = 0; y – 2 = 0 21.5 21 13.5 (?)1/3 17
Sol. + − = +
y = 3; y = 2 … (2) 5 6 15 4 30
U

From (1) and (2) x > y


21.5 21 13.5 17 (?)1/3
30. Ans. (C) + − − =
5 6 15 30 4
@

Sol. (I) 38x2 – 3x – 11 = 0


129 + 105 − 27 − 17 (?)1/3
38x2 + 19x – 22x – 11 = 0 =
30 4
19x (2x + 1) – 11 (2x + 1) = 0
190 (?)1/3
(2x + 1) (19x – 11) = 0 =
30 4
2x + 1 = 0; 19x – 11 = 0
190 × 4
1 11 = (?)1/3
x = – ; x = 30
2 19
76
x = –0.5; x = 0.57 … (1) = (?)1/3
3
(II)28y2 + 32y + 9 = 0 3
76
28y2 + 14y + 18y + 9 = 0 = ?
3
Solved Test Papers of Various Examinations ✫ 785

34. Ans. (C) 39. Ans. (D)


Sol. The given series is
Sol. ( 7 + 11)2 = (?)1/3 + 2 847 + 122
7 + 121 + 22 7 = (?)1/3 + 2 × 11 7 + 122 19 25 45 87 159 ?
7 + 121 – 122 + 22 7 – 22 7 = (?)1/3
6 = (?)1/3 +6 +20 +42 +72 +110
216 = ?
35. Ans. (C) +14 +22 +30 +38

TS
2
18 455 61
Sol. × ÷ =? +8 +8 +8
4 19 799
324 455 799 ∴ ? = 159 + 110

H
× × =?
16 19 61 ? = 269
? ≈ 6350 40. Ans. (E)

G
36. Ans. (C) Sol. The given series is
Sol. The given series is

U 83 124 206 370 698 ?


O
+41 +82 +164 +328 +656
H
×2 ×2 ×2 ×2
_T

∴ ? = 1354
? = 158 + 75 41. Ans. (C)
? = 233 Sol. Academic books published by M
C

37. Ans. (E) 7


= × 28200
Sol. The given series is 10
PS

7 16 141 190 919 ? = 19740


7 + 9 = 16 ⇒ 7 + (3)2 = 16 Academic books published by P
16 + 125 = 141 ⇒ 16 + (5)3 = 141
U

8
141 + 49 = 190 ⇒ 141 + (7)2 = 190 = × 31200
13
190 + 729 = 919 ⇒ 190 + (9)3 = 919 = 19200
@

919 + 121 = 1040 ⇒ 919 + (11)2 = 1040 Difference = 19740 – 19200


38. Ans. (C) = 540
Sol. The given series is 42. Ans. (B)
Sol. Number of books distributed by Q
12 17 32 57 92 ? 79
= × 33800
100
+5 +15 +25 +35 +45 = 26702
Number of distributors in Q = 25
+10 +10 +10 +10 ∴ Each distributes gets
∴ ? = 137 26702
= = 1068
25
786 ✫ Quantitative Aptitude

43. Ans. (C) 46. Ans. (C)


Sol. Non-academic books published by R Sol. Let C.P. of the saree excluding sales
6 tax = Rs x
= × 35,700 Then C.P. + sales tax = 5225
17
= 12,600 12
x + × x = 5225
Non-academic books published by S 100
13 112
= × 37,800 x = 5225
18 100

TS
= 27,300 5 , 225 × 100
x =
Required average 112
x = 4,665
12 , 600 + 27 , 300

H
= Price is decreased by
2
= 19,950 5225 – 4665 = Rs 560

G
44. Ans. (B) 47. Ans. (C)
Sol. Total number of books published by PTR 24 , 200 × 6 × 4
P, Q, R
= 31,200 + 33,800 + 35,700 U Sol. S.I. =
100
=

= Rs 5,808
100
O
= 1,00,700
Amount reinvested
Increased number of books
= 42,200 + 5,808
H
130 = Rs 30,008
= × 1,00,700
100
_T

30 , 008 × 4 × 4
= 1,30,910 S.I. =
100
Total number of books published by
= Rs 4,801.28
M, N, O, S
C

48. Ans. (A)


= 28,200 + 32,200 + 29,700 + 37,800 Sol. Since 5% do not respond to any
PS

= 1,27,900 channel, 95% will respond to all


Decreased number of books channels
∴ percentage of viewers who watch all
80
= × 1,27,900 3 channels
U

100 85 + 20 + 30 − 20 − 95
= 1,02,320 =
2
@

Required average 20
= = 10%
1, 30 , 910 + 1, 02 , 320 2
= 49. Ans. (E)
7 Sol.
2 , 33, 320 Luck
= = 33,318 Bang b
7 85% 20%
45. Ans. (D) a
Sol. Total number of books distributed c d
by O and Q
92 79 Ahead
= × 29,700 + × 33,800 30%
100 100
= 27,324 + 26,702
= 54,026
Solved Test Papers of Various Examinations ✫ 787

In the figure, a = viewers watching ∴ Percentage of viewers who watch


all 3 channels only Luck
∴ a = 10 (solved in question 48)… (1) = 20 – b – a – d
Viewers responding to Bang and = 20 – 6 – 10 – 4
Luck = 16% = 0
∴ a + b = 16 50. Ans. (A)
10 + b = 16 ⇒ b = 6 Sol. Initially ratio of milk : water
Viewers responding to Ahead and = 80 l : 20 l
Bang = 20% = 4 : 1

TS
a + c = 20 Total mixture = 80 + 20 = 100 l
10 + c = 20 Quantity of mixture sold
1
c = 10 = × 100 = 25 l

H
4
∴ Viewers watching Luck and Bang Remaining mixture
only = b = 6 … (2)
= 100 – 25 = 75 l

G
Viewers watching Ahead and Bang 1
only = c = 10 Quantity of water = × 75 = 15
5
Viewers watching Luck and Ahead
only = d
U Quantity of milk =
4
5
× 75 = 60
O
d = 20 – a – b ∴ Current proportion of water : milk
d = 20 – 10 – 6 [from (1) and (2)] is 15 + 25 : 60
H
d = 4 40 : 60
2 : 3
C _T
PS
U
@
788 ✫ Quantitative Aptitude

State Bank of India (SBI)


PT Examination, July 2016
Directions (Q. 1-5): The bar-chart shows the total 5. The total number of members enrolled in
number of members enrolled in different years gymnasium B in 1993 and 1994 together is
from 1990 to 1994 in two gymnasiums A and what per cent more than the number of
B. Based on this bar chart, solve the following members enrolled in gymnasium A in 1990
questions. and 1994 together?
(1) 60% (2) 65% (3) 62.5%

TS
Gymnasium A Gymnasium B
(4) 61.5% (5) None of these
300
Directions (6-10): What should come in place of
250 question mark (?) in the following number

H
series?
200
6. 4 3 4 7 15 ?

G
(1) 38.5 (2) 40 (3) 45
150
(4) 37.5 (5) None of these
100

U 7. 7 5
(1) 321
7 17 63
(2) 309
?
(3) 305
O
50 (4) 301 (5) None of these
8. 11 14 19 28 43 ?
H
1990 1991 1992 1993 1994 (1) 60 (2) 63 (3) 66
Year (4) 70 (5) None of these
1. If in the year 1995, there is 30% increase in
_T

9. 2 60 10 120 30 ?
the total number of members enrolled in (1) 222 (2) 216 (3) 208
1994 in both gymnasiums, then find the total (4) 230 (5) None of these
number of members enrolled in 1995.
10. 23 50 108 232 492 ?
C

(1) 282 (2) 296 (3) 292


(1) 1028 (2) 1024 (3) 1020
(4) 286 (5) None of these
(4) 1032 (5) None of these
PS

2. The ratio of the total number of members of


both gymnasiums in 1991 to the total number Directions (Q. 11-5): There are five companies
of members in 1994 of both gymnasiums is and we have been given the number of
employees working in different companies. In
U

(1) 22 : 27 (2) 21 : 11 (3) 11 : 21


(4) 25 : 13 (5) 27 : 22 the table we have also been given the percentage
of male and female employees in HR and
@

3. The number of members of gymnasium A in marketing department.


1991 is what per cent of the number of
members of gymnasium B in 1994?% HR Marketing
(1) 60% (2) 55% (3) 58% Company Employees Male Female Male Female
(4) 62% (5) None of these P 400 12 14 9 7
4. The number of members enrolled in Q 650 19 10 11 13
gymnasium A from 1991 to 1994 together is R 500 28 14 4 7
what per cent more than the number of S 550 31 9 6 4
members enrolled in gymnasium B in 1993 T 300 12 18 3 7
and 1994 together?(Rounded off to two-digit
decimal places) 11. If 60% of the employees of company T in HR
(1) 10.51% (2) 20.51% (3) 15.51% department have MBA degree and 40% of
(4) 17.51% (5) None of these the employees of the same company in the
Solved Test Papers of Various Examinations ✫ 789

marketing department have MBA degree, 18. Two trains, A and B, start from stations X and
then how many employees have MBA degree Y towards each other. They take 4 hours 48
in company T in both departments together? minutes and 3 hours 20 minutes to reach
(1) 98 (2) 108 (3) 106 Y and X respectively after they meet. If train
(4) 92 (5) 66 A is moving at 45 km/hr, then the speed of
12. What is the ratio of the number of female train B is
employees of company Q in HR department (1) 60 km/hr (2) 64.8 km/hr
to the number of male employees of (3) 54 km/hr (4) 37.5 km/hr
company R in marketing department? (5) None of these

TS
(1) 4 : 13 (2) 5 : 22: (3) 22 : 5 19. Out of his total income, Mr Kapoor spends
(4) 13 : 4 (5) None of these 20% on house rent and 70% of the rest on
13. The total number of HR employees of company household expenses. If he saves ` 1,800 what
P is what per cent more than the total number is his total income (in rupees)?

H
of Marketing employees of company T? (1) ` 7,800 (2) ` 7,000
(1) 236.76% (2) 226.67% (3) 276.76% (3) ` 8,000 (4) ` 7,500

G
(4) 246.67% (5) None of these (5) None of these
14. What is the ratio of the male employees in 20. A can do a piece of work in 8 days which B
HR department of company P and R
together to the female employees of
U can destroy in 3 days. A has worked for 6
days, during the last 2 days of which B has
been destroying. How many days must A
O
marketing department in company S and T
together? row work alone to complete the work?
(1) 187 : 27 (2) 43 : 188 (3) 188 : 43 2
H
(1) 7 days (2) 7 days
(4) 27 : 187 (5) None of these 3
1
15. What is the difference between the number (3) 7 days (4) 8 days
_T

3
of female employees of HR department in (5) None of these
all companies together (excluding company
S) and the female employees of marketing Directions (Q. 21-25): What approximate value
C

department in all companies together should come in place of question mark (?) in
(excluding company Q)? the following questions? (You are not expected
to calculate the exact value.)
PS

(1) 139 (2) 129 (3) 135


(4) 141 (5) None of these 21. 57% of 394 – 2.5% of 996 = ?
16. A mixture contains wine and water in the (1) 215 (2) 175 (3) 200
ratio of 3 : 2 and another mixture contains (4) 180 (5) 205
U

them in the ratio of 4 : 5. How many litres of


22. 96.996 × 9.669 + 0.96 = ?
the latter must be mixed with 3 litres of the
(1) 860 (2) 870 (3) 1020
@

former so that the resultant mixture may


(4) 940 (5) 1100
contain equal quantities of wine and water?
12 2 3 3 1125
(1)
3
litres (2)
5
litres (3) 3 litres
4 23. × ×7=?
5 1228
1 (1) 7 (2) 12 (3) 9
(4) 4 litres (5) None of these
2 (4) 4 (5) 15
17. A trader sells two bullocks for ` 8,400 each,
neither losing nor gaining in total. If he sold 24. ( 339 × 25) ÷ 30 = ?
one of the bullocks at a gain of 20%, the other (1) 12 (2) 15 (3) 24
is sold at a loss of (4) 2 (5) 9
2 2 25. (638 + 9709 – 216) ÷ 26 = ?
(1) 20% (2) 18 % (3) 14 %
9 7 (1) 275 (2) 365 (3) 420
(4) 21% (5) None of these (4) 300 (5) 390
790 ✫ Quantitative Aptitude

Directions (Q. 26-30): Solve the equations given 5 5 7


below and give answer (1) (2) (3)
22 23 22
(1) if x < y (2) if x > y 4
(3) if x ≥ y (4) if x ≤ y (4) (5) None of these
9
(5) if x = y or no relation can be established 33. According to a new plan rolled out by HISP
26. (I) 6x2 + 31x + 35 = 0 Bank, the rate of simple interest on the sum
(II) 2y2 + 3y + 1 = 0 of money is 8% pa for the first two years, 10%
pa for the next three years and 6% pa for the
27. (I) 2x2 – (4 + 13 ) x + 2 13 = 0 period beyond the first five years. The simple
(II) 10y2 – (18 + 5 13 ) y + 9 13 = 0 interest accrued on a sum for a period of

TS
eight years is ` 12,800. Find the sum.
28. (I) 2x2 + 9x + 10 = 0
(1) ` 24,000 (2) ` 16,000
(II) 4y2 + 28y + 45 = 0 (3) ` 15,000 (4) ` 13,500
29. (I) 15x2 – 11x – 12 = 0 (5) None of these

H
(II) 20y2 – 49y + 30 = 0 34. Three science classes A, B and C take a Life
Science test. The average score of students

G
30. (I) 2x2 – 15 = 7x
(II) 17y = –7 – 6y2 of class A is 83. The average score of students
of class B is 76. The average score of class C
31. A and B are partners in a business. They
invest in the ratio of 5 : 6; at the end of 8
months A withdraws. If they receive profits U is 85. The average score of class A and B is
79 and the average score of class B and C is
O
81. Then the average score of classes A, B and
in the ratio of 5 : 9, then find how long B’s C is
investment was used. (1) 80 (2) 80.5 (3) 81
H
(1) 12 months (2) 10 months (4) 81.5 (5) None of these
(3) 15 months (4) 14 months
35. A hemispherical bowl of internal diameter
_T

(5) 18 months 54 cm contains a liquid. This liquid is to be


32. There are 3 red balls, 4 blue balls and 5 white filled in cylindrical bottles of radius 3 cm and
balls. 2 balls are chosen randomly. Find the height 9 cm. How many bottles are required
probability that one is red and the other is to empty the bowl?
C

white. (1) 221 (2) 343 (3) 81


(4) 243 (5) None of these
PS

Answers and Explanatory Notes


U

1. Ans. (4) 3. Ans. (5)


Sol. Total number of members enrolled Sol. Required percentage
@

in 1995 Number of members in


30 gymnasium A in 1991
= (150 + 70) + × (150 + 70) = × 100
100 Number of members in
= 220 + 66
gymnasium B in 1994
= 286
2. Ans. (5) 60
= × 100 = 40%.
Number of members in both 150
gymnasiums in 1991 4. Ans. (2)
Sol. Sol. Total number of members enrolled
Number of members in both
gymnasiums in 1994 in gymnasium A from 1991 to 1994
60 + 210 = 60 + 140 + 200 + 70 = 470
= Total number of members enrolled
70 + 150
in gymnasium B in 1993 and 1994
270 27
= = = 27 : 22 = 240 + 150 = 390
220 22
Solved Test Papers of Various Examinations ✫ 791

470 − 390 10. Ans. (1)


Required percentage = × 100 Sol. The series is
390
80 23 50 108 232 492 ?
= × 100 50 – 23 = 27
390
= 20.51%. 108 – 50 = 58
5. Ans. (3) 232 – 108 = 124
Sol. Total number of members enrolled 492 – 232 = 260
in gymnasium B in 1993 and 1994 ? – 492 = … (1)
= 240 + 150 = 390 27 58 124 260
Total number of members enrolled

TS
58 – 27 = 31
in gymnasium A in 1990 and 1994 124 – 58 = 66
= 170 + 70 = 240
260 – 124 = 136
Required percentage
– 260 = x (say) … (2)

H
390 − 240 31 66 136 x
= × 100
240 66 – 31 = 35
136 – 66 = 70 ⇒ (35 × 2)

G
150
= × 100 = 62.5%. x – 136 = 140 ⇒ (70 × 2)
240
6. Ans. (1) ⇒ x = 140 + 136 = 276
Sol. The series is
4 3 4 7 15 ? U ∴ (2) ⇒ – 260 = 276
= 536
O
[4 × 0.5] + 1 = 2 + 1 = 3 ∴ (1) ⇒ ? – 492 = 536
[3 × (.5 + .5)] + 1 = 3 + 1 = 4 ? = 536 + 492
H
[4 × 1.5] + 1 = 6 + 1 = 7 ? = 1028.
[7 × 2] + 1 = 14 + 1 = 15 11. Ans. (5)
[15 × 2.5] + 1 = 38.5 Sol. Number of employees of company T
_T

7. Ans. (2) in HR department


Sol. The series is 12 + 18
7 5 7 17 63 ? = × 300 = 90
100
C

(7 × 1) – 2 = 5 Number of employees of company T


(5 × 2) – 3 = 7 in the marketing department
PS

(7 × 3) – 4 = 17
(17 × 4) – 5 = 68 ( 7 + 3)
= × 300 = 30
(63 × 5) – 6 = 309 100
8. Ans. (3) Number of employees of company T
U

Sol. The series is in HR department having MBA


11 14 19 28 43 ? degree
60
@

11 + 3 = 14 = × 90 = 54
14 + (3 + 2) = 19 100
Number of employees of company T
19 + (5 + 4) = 28
in the marketing department having
28 + (9 + 6) = 43
MBA degree
43 + (15 + 8) = 66
9. Ans. (5) 40
= × 30 = 12
Sol. The series is 100
2 60 10 120 30 ? ∴ Total number of employees in
2 = 13 + 1 company T having MBA degree
60 = 43 – 4 = 54 + 12 = 66.
10 = 23 + 2 12. Ans. (4)
120 = 53 – 5 Number of female employees
30 = 33 + 3 of Company Q in HR
Sol.
The next number is 63 – 6 = 210 Number of male employees
of Company R in Marketing
792 ✫ Quantitative Aptitude

10 7 7
× 650 = × 400 + × 500
13 100 100
= 100 =
4 4 4 7
× 500 ` + × 550 + × 300
100 100 100
= 13 : 4. = 28 + 35 + 22 + 21
13. Ans. (4) = 106
Sol. Total number of employees of Difference = 245 – 106 = 139.
company P in HR department 16. Ans. (5)
Sol. Let first mixture contain 3x litre of
(12 + 14) wine and 2x litre of water. Let the

TS
= × 400 = 104 second mixture contain 4y litres of
100
Total number of marketing wine and 5y l of water. In 3 litres of
employees of company T the first mixture quantity of wine

H
3
(3 + 7 ) =
5
× 3 = 1.8 litres
= × 300 = 30
100 Quantity of water = 1.2 litres

G
Required percentage When 9y litres (4y + 5y) of the second
mixture is added then quantity of
104 − 30)
=
74
30
× 100
U wine = quantity of water
1.8 + 4y = 1.2 + 5y
0.6 = y
O
= × 100 = 246.67%. ∴ Quantity of second mixture
30
14. Ans. (3) = 4y + 5y = 9y
H
Sol. Male employees in HR department = 9 × 0.6
of company P and R = 5.4 litres
17. Ans. (3)
_T

12 28
= × 400 + × 500 Sol. Total S.P. of 2 bullocks
100 100
= 48 + 140 = 188 = 8400 + 8400 = 16800
Female employees of marketing C.P. of first bullock
C

department in company S and T 100


= S.P. ×
4 7 100 + Profit %
PS

= × 550 + × 300 100


100 100 = 8400 ×
= 22 + 21 100 + 20
= Rs 7000
= 43
Since there is no profit or loss, C.P.
U

188 of second bullock


Required ratio =
43 = 16800 – 7000 = Rs 9800
@

= 188 : 43. Loss = C.P. – S.P.


15. Ans. (1) = 9800 – 8400 = Rs 1400
Sol. Number of female employees of HR Percentage Loss
department in company P, Q, R, T
Loss
14 10 = × 100
= × 400 + × 650 C.P.
100 100
1400 2
14 18 = × 100 = 14 %.
+ × 500 + × 300 9800 7
100 100 18. Ans. (3)
= 56 + 65 + 70 + 54 Sol. Let the speed of train B be x km/hr
= 245 Speed of train A is 45 km/hr
Number of female employees in Time taken by train A = 4 hrs 48 mins
marketing department in company P, 48 4 24
= 4 =4 = h
R, S, T 60 5 5
Solved Test Papers of Various Examinations ✫ 793

Time taken by train B = 3 hrs 20 mins 24


20 1 10 In 1 day B can destroy = = 8 units
=3 = h 3
= 3 In 6 days A can do 6 × 3 = 18 units
60 3 3
Speed of train A In 2 days B can destroy
= 2 × 8 = 16 units
Speed of train B
∴ Work completed in 8 days
Time taken by B = 18 – 16 = 2 units
=
Time taken by A Remaining work
= 24 – 2 = 22 units
10 3 units of work is done by A in 1

TS
45 3 day
= 24 22
x ∴ 22 units of work is done by A in
5 3
days

H
45 10 5 1
= × = 7 days.
x 3 24 3

G
21. Ans. (3)
45 25
= 57 2 .5
x 36 Sol. × 394 – × 996
45
=
5
U 100 100
= 224.58 – 24.9
= 199.68
O
x 6
45 × 6 = 200.
⇒ x = = 54 km/hr. 22. Ans. (4)
H
5
Sol. 96.996 × 9.669 + 0.96
∴ Speed of train B is 54 km/hr.
= 97 × 9.7 + 1
19. Ans. (4)
_T

= 940.9 + 1
Sol. Let Kapoor’s total income be Rs x
= 941.9
Expense on house rent
= 940.
20 x
= ×x= 23. Ans. (4)
C

100 5 3 1125
Remaining amount Sol. × ×7
PS

x 4x 5 1228
= x− = 21 × 225
5 5 ≈ 4.
Amount spent on household 1228
24. Ans. (2)
expenses
( )
U

Sol. 339 × 25 ÷ 30
70 4 x 14 x
= × = = 18 × 25 ÷ 30
100 5 25
@

= 450 ÷ 30
4 x 14 x 6 x
Savings = − = = 15.
5 25 25 25. Ans. (5)
6x Sol. (638 + 9709 – 216) ÷ 26
∴ = 1800
25 = 10131 ÷ 26
1800 × 25 = 390.
x = = Rs 7500 26. Ans. (1)
6
Total income is Rs 7500. Sol. (I) 6x2 + 31x + 35 = 0
20. Ans. (3) 6x2 + 10x + 21x + 35 = 0
Sol. A can do a piece of work in 8 days 2x (3x + 5) + 7 (3x + 5) = 0
B can destroy it in 3 days (3x + 5) (2x + 7) = 0
L.C.M. of 8 and 3 is 24 3x + 5 = 0; 2x + 7 = 0
24 5 7
In 1 day A can do = 3 units x=– ;x=– … (1)
8 3 2
794 ✫ Quantitative Aptitude

(II) 2y2 + 3y + 1 = 0 29. Ans. (5)


2y2 + 2y + y + 1 = 0 Sol. (I) 15x2 – 11x – 12 = 0
2y (y + 1) + 1 (y + 1) = 0 15x2 – 20x + 9x – 12 = 0
(y + 1) (2y + 1) = 0 5x (3x – 4) + 3 (3x – 4) = 0
(y + 1) = 0; (2y + 1) = 0 (3x – 4) (5x + 3) = 0
1 3x – 4 = 0; 5x + 3 = 0
y = –1; y = – … (2)
2 4 3
From (1) and (2) x= ;x=– … (1)
x < y. 3 5
2
(II) 20y – 49y + 30 = 0
27. Ans. (3)

TS
20y2 – 25y – 24y + 30 = 0
( )
Sol. (I) 2x2 – 4 + 13 x + 2 13 = 0 5y (4y – 5) – 6 (4y – 5) = 0
2x – 4 – 13 x + 2 13 = 0
2 (4y – 5) (5y – 6) = 0
4y – 5 = 0; 5y – 6 = 0

H
2x2 – 13x − 4 x + 2 13 = 0 5 6
( ) ( )
y= ; y = … (2)
x 2 x − 13 − 2 2 x − 13 =0 4 5

G
( 2 x − 13 ) (x − 2) = 0
From (1) and (2) no relation can be
established.
30. Ans. (5)
2x –

x=
13
13 = 0; x – 2 = 0
;x=2 ... (1) U Sol. (I) 2x2 – 15 = 7x
2x2 – 7x – 15 = 0
O
2 2x2 – 10x + 3x – 15 = 0
2x (x – 5) + 3 (x – 5) = 0
(II) 10y2 – (18 + 5 13 ) y + 9 13 = 0 (x – 5) (2x + 3) = 0
H
10y2 – 18y – 5 13 y + 9 13 = 0 x – 5 = 0; 2x + 3 = 0
2y (5y – 9) – 13 (5y – 9) = 0 3
_T

x= 5; x = – … (1)
(5y – 9) (2y – 13 ) = 0 2
5y – 9 = 0; 2y – 13 = 0 (II) 17y = –7 – 6y2
6y2 + 17y + 7 = 0
C

9 13 6y2 + 3y + 14y + 7 = 0
y= ;y= ... (2)
5 2 3y (2y + 1) + 7 (2y + 1) = 0
PS

From (1) and (2) x ≥ y. (2y + 1) (3y + 7) = 0


28. Ans. (3) 2y + 1 = 0; 3y + 7 = 0
Sol. (I) 2x2 + 9x + 10 = 0 1 7
2x2 + 4x + 5x + 10 = 0 y=– ;y=– … (2)
U

2 3
2x (x + 2) + 5 (x + 2) = 0 From (1) and (2) no relation can be
(x + 2) (2x + 5) = 0 established.
@

x + 2 = 0; 2x + 5 = 0 31. Ans. (1)


5 Sol. Let B’s investment be used for x
x= –2; x = – … (1)
2 months
(II) 4y2 + 28y + 45 = 0 Ratio of profits = 5 : 9
4y2 + 18y + 10y + 45 = 0 Share of A's investment A's profit
= B's profit
2y (2y + 9) + 5 (2y + 9) = 0 Share of B's investment
(2y + 9) (2y + 5) = 0 5×8 5
=
2y + 9 = 0; 2y + 5 = 0 6×x 9
9 5 5×8×9
y = – ; y = – … (2) x = = 12
2 2 5×6
From (1) and (2) x ≥ y. ∴ B’s investment was used for 12
months.
Solved Test Papers of Various Examinations ✫ 795

32. Ans. (1) Average score of B and C is 81


Sol. Total number of balls ∴ 76y + 85z = 81 (y + z)
= 3 + 4 + 5 = 12 76y + 85z = 81y + 81z
Favourable outcomes 76y – 81y = 81z – 85z
= 3C1 × 5C1 –5y = –4z
(Choosing 1 red from 3 red balls and 5y = 4z … (2)
1 white from 5 white balls) Average scores of A, B and C
Total number of outcomes = 12C2
83x + 76 y + 85z
Required probability = x+y+z
Favourable outcomes

TS
=
Total number of outcomes 3y 5y
83 + 76 y + 85
3
C1 × 5C1 = 4 4
= 12
x+y+z
C2

H
3! 5! [From (1) and (2)]
× !
2 1 4 × 1! n!

G
! !

=
n
Cr = 249 y 425 y
12! (n − r)! r! + 76 y +
4 4

=
10! × 2!
3×5×2
=
5
U =
3y
4
+y+
5y
4
O
12 × 11 22
33. Ans. (5) 249y + 304y + 425y
Sol. Let the sum be Rs P =
H
3y + 4 y + 5 y
S.I. for the first 2 years
978 y
P×2×8 PTR = = 81.5
_T

= S.I. = 12 y
100 100 35. Ans. (5)
S.I. for the next 3 years Sol. Volume of hemispherical bowl
P × 3 × 10 2 3
C

= = πr
100 3
S.I. for the next 3 years 2 22
PS

P×3×6 = × × (27)3
3 7
=
100 Volume of cylindrical bottles
S.I. for 8 years = Rs 12,800 = πr2h
U

16 P 30 P 18 P 22
+ + = Rs 12,800 = × (3)2 × 9
100 100 100 7
@

64P = 12,80000 Number of bottles


P = Rs 20,000. Volume of bowl
34. Ans. (4) =
Volume of 1 bottle
Sol. Let there be x, y, z students in classes
A, B, C respectively. 2 22
× × 27 × 27 × 27
According to the question, average
= 3 7
score of A and B is 79 22
×3×3×9
∴ 83x + 76y = 79 (x + y) 7
83x – 79x = 79y – 76y
= 162 bottles.
4x = 3y … (1)
796 ✫ Quantitative Aptitude

IBPS-RRBs Officers (PT)


Examination, 2017
1. A mixture of milk and water in a jar 4. In Club B, the number of female members
comprises 12 litre of milk. If 5 litre of pure who registered in March was three-fifths of
milk and 3 litre of pure water were added to the number of male members who registered
this jar, the percentage of water in the new in the same month. If the number of female
mixture would be 20%. What was the initial members who registered for Club A in March
quantity of water in the jar? (in litre) was 50 more than that registered for Club B

TS
(A) 5 (B) 4 in the same month, what was the number of
(C) 2 (D) 6 male members who registered for Club A in
(E) 1 March?
(A) 140 (B) 130

H
Direction —(Q. 2-6) Refer to the graph and answer
the given questions— (C) 110 (D) 100
(E) 120

G
Number of members who have
registered for two clubs in five 5. What is the respective ratio between the total
different months number of members who registered for both

600
U the clubs together in March and July?
(A) 9 : 13 (B) 6 : 13
Number of members who

O
registered in the month

500 (C) 7 : 17 (D) 9 : 11


(E) 6 : 11
400
H
6. What is the difference between the total
300 number of members who registered for Club
A in May and June together and those who
_T

200
registered for Club B in the same months
100 together?
(A) 240 (B) 180
0
C

March April May June July (C) 220 (D) 160


A 320 360 400 420 380 (E) 300
PS

B 400 320 480 520 500


7. 8 men can finish a piece of work in 25 days.
2. If in August the number of members who 15 women can finish the same piece of work
registered for both the clubs together in 16 days. 4 men and 8 women started
working together and worked for 10 days.
U

increased by 20% as compared to June, what


is the number of members who registered After that 6 more men joined them. How
for both the clubs together in August? many days will they now take to finish the
@

(A) 1152 (B) 816 remaining work?


(C) 1128 (D) 1056 4 3
(E) 1028 (A) 4 (B) 6
5 5
3. The number of members who registered for 2 3
Club A in April was what per cent less than (C) 6 (D) 5
5 5
that registered in June for the same club? 3
2 2 (E) 4
A) 14 (B) 15 5
7 7 Directions—(Q. 8-12) Study the table and answer
3 3 the given questions—
(C) 12 (D) 11
7 7 Number of items sold and percentage of
4 items returned by the customers over the months
(E) 10 in two different shops—
7
Solved Test Papers of Various Examinations ✫ 797

Shops number of defective items sold by that shop


Month A B in that month?
No. of % of items No. of % of items (A) 116 (B) 168
items returned items returned (C) 198 (D) 216
sold by the sold by the
(E) 172
customers customers
13. ?% of (813.97 × 3.08) = 754.08 + 467.06
August 580 15 650 8
(A) 50 (B) 46
September 600 12 540 10
(C) 44 (D) 52
October 550 4 360 5
(E) 45
November 720 20 416 25

TS
14. (1356.07 –?) ÷ 3.09 = 2196.11 ÷ 5.92
8. The total number of items returned by the (A) 367 (B) 335
customers to shop A in August and October (C) 294 (D) 258
together was approximately what per cent (E) 274

H
more than the total number of items returned 15. ?2 +142.04 – 21.97 × 7.08 = 277.03
by the customers to shop B in September and (A) 9 (B) 5

G
October together? (C) 16 (D) 17
(A) 46 (B) 32 (E) 6
(C) 67 (D) 51
(E) 23
9. The number of items returned by the U 16. (? + 5.04) × 71.94 ÷
(A) 29
2
257 = 6.053
(B) 43
O
customers to shop B in August was what per (C) 48 (D) 50
cent of the number of items returned by the (E) 25
H
(8.07)3 − ( 3.93)
3
customers to that shop in November?
(A) 64 (B) 50 17. = ? × 7.02
( 4.06)2
_T

(C) 44 (D) 32
(E) 26 (A) 18 (B) 11
(C) 4 (D) 12
10. What was the average number of items (E) 9
which were not returned by the customers
C

to shop B in September, October and 18. If 20% of a = b, then b% of 20 is the same as—
November? (A) 4% of a (B) 5% of a
PS

(A) 438 (B) 251 (C) 20% of a (D) 25% of a


(C) 255 (D) 380 (E) None of these
(E) 352 Directions—(Q. 19-23) Refer to the pie chart and
U

11. What is the respective ratio between the answer the given question—
number of items not returned by the Distribution of total number
of brownies sold by 5
@

customers to shop A in October to the


number of items returned by the customers different bakeries
to that shop in November? Total Number = 320
(A) 15 : 6 (B) 21 : 5
(C) 19 : 7 (D) 11 : 3 A
(E) 17 : 8 E 15%
30%
12. All the items returned by the customer to B
shop A in September were found to be 20%
defective and the shop received complaints D
from some of the customers who did not 10% C
1 25%
return the items and about th of those
12
items not returned by the customer were 19. What is the central angle corresponding to
found to be defective. What was the total the number of brownies sold by bakery E?
798 ✫ Quantitative Aptitude

(A) 104° (B) 108° breadth of the rectangle are in the ratio 14 :
(C) 106° (D) 102° 11 respectively, what is its perimeter?
(E) 105° (A) 142 cm (B) 140 cm
20. The respective ratio between the number of (C) 132 cm (D) 150 cm
brownies sold by bakery D and that by (E) 155 cm
bakery F is 2 : 3. If bakery F sold each 26. A metallic cuboid measuring 12 cm × 9 cm ×
brownie for ` 50, what was the amount 2 cm is melted and cast into a cube. Find the
earned by bakery F? length of each edge of the cube—
(A) ` 3200 (B) ` 2400 (A) 5 cm (B) 6 cm

TS
(C) ` 2000 (D) ` 2800 (C) 2 cm (D) 8 cm
(E) ` 3000 (E) 10 cm
21. What is the difference between the average 27. A boat can travel 6.4 km downstream in 16
of the number of brownies sold by bakeries minutes and 9.9 km upstream in 33 minutes.

H
A and B and the average of the number of What is the total time taken by the boat to
brownies sold by bakeries C and E? travel 48 km upstream and the same distance

G
(A) 28 (B) 32 downstream together?
(C) 36 (D) 42 (A) 4 hr 40 min
(E) 45
22. The respective ratio between the number of
U (B) 4 hr
(C) 5 hr
(D) 5 hr 20 min
O
chocolate and that of vanilla sold by bakery
B is 5 : 3 and the respective ratio between (E) 6 hr
the number of chocolate and that of vanilla Directions—(Q. 28-32) What will come in place of
H
brownies sold by bakery C is 3 : 1. What is question-mark (?) in the given number series?
the total number of chocolate brownies sold 28. 48 63 43 68 38 ?
_T

by bakeries B and C together? (Bakeries B


(A) 76 (B) 73
and C sell only chocolate and vanilla
(C) 79 (D) 66
brownies)
(E) 75
(A) 130 (B) 120
C

(C) 100 (D) 150 29. 14 16 22 34 54 ?


(E) 90 (A) 76 (B) 84
PS

23. What is the difference between the total (C) 86 (D) 75


number of brownies sold by bakeries A and (E) 95
D together and that sold by bakery E? 30. 852 285 96 33 12 ?
U

(A) 12 (B) 16 (A) 6 (B) 8


(C) 6 (D) 18 (C) 5 (D) 3
@

(E) 9 (E) 1
24. The respective ratio between the present 31. 0.25 4 32 128 256 ?
ages of A and B is 9 : 10. 8 years ago the
1 (A) 264 (B) 256
respective ratio of th of A’s age that time (C) 234 (D) 274
1 7
(E) 284
and th of B’s age that time was 1 : 2. What
4 32. 9 4 3 3 4 ?
will be the respective ratio between A’s age
and B’s age 8 years hence? (A) 11 (B) 7.5
(A) 17 : 18 (B) 11 : 12 (C) 8.5 (D) 9.5
(C) 9 : 11 (D) 9 : 12 (E) 10.5
(E) 8 : 13 Direction—(Q. 33-37) In these questions, two
25. Area of rectangle is equal to area of the circle equations I and II are given. You have to solve
whose radius is 21 cm. If the length and both the equations and answer as—
Solved Test Papers of Various Examinations ✫ 799

(A) x > y (B) x ≥ y (A) ` 1200 (B) ` 1000


(C) x < y (D) x ≤ y (C) ` 1600 (D) ` 1500
(E) relationship cannot be established or (E) ` 1400
x = y 39. Mohan kept 30% of his retirement fund for
33. I. 3x2 + 13x + 12 = 0 himself and distributed the remaining
II. 4y2 + 5y + 1 = 0 amount among his wife, his only son and
his only daughter in the respective ratio of
34. I. x2 = 25 7 : 3 : 4. If the difference between the amount
II. y2 + 10y + 25 = 0 that Mohan kept for himself and the amount

TS
35. I. 5x2 – 11x + 2 = 0 he gave to his son is ` 31500. What was the
II. 3y2 – 5y + 2 = 0 total retirement fund?
36. I. 5x2 – 13x + 6 = 0 (A) ` 240000 (B) ` 270000
II. 2y2 – 7y + 6 = 0 (C) ` 210000 (D) ` 260000

H
(E) ` 300000
37. I. 2x2 – 17x + 35 = 0
II. 3y2 – 10y + 7 = 0 40. A bag contains 2 red balls, 5 yellow balls and

G
‘X’ green balls. One ball is drawn at random
38. Ram invests a certain sum in Scheme A and the probability of ball being yellow is
offering simple interest @ 5% p.a. for 4 years.
He further invests the amount obtained from
Scheme A into Scheme B offering compound U 2
9
. What is the value of X?
(A) 3
O
interest @ 10% p.a. (compounded annually) (B) 12
for 2 years. If the interest obtained from (C) Other than those given as options
H
Scheme B was ` 378, what was the sum (D) 6
invested in scheme A? (in `) (E) 9
_T

Answers and Explanatory Notes

1. Ans. (E) (15 − x )


C

Sol. Volume of mixture of milk and water i.e. × 100 = 20


= 12 litres 20
PS

Let the quantity of milk in the mixture i.e. (15 – x) × 5 = 20


= x litres. i.e. (15 – x) = 4
Quantity of water in the mixture = i.e. x = 11 litres
(12 – x) litres. ∴ Initial Quantity of water in the jar
U

Volume of pure milk added to the = 12 – x = 12 – 11 = 1 litre


mixture = 5 litres 2. Ans. (C)
@

Hence, total quantity of pure milk in Sol. Total number of members who
the mixture registered for both the clubs in the
= (x + 5) litres. (i) month of June
Volume of pure water added to the = 420 + 520
mixture = 3 litres. = 940
Hence, total quantity of water in the Total number for members who
mixture registered for both the clubs in the
= (12 – x) + 3 = (15 – x) litres ... (ii) month of August increased by 20%
Total volume of mixture = (i) + (ii) as compared to June, i.e. Total no. of
= (x + 5) + (15 – x) members who registered for both the
= 20 litres clubs in the month of August
Given that percentage of water in the 20
= 940 + × 940
mixture = 20 100
800 ✫ Quantitative Aptitude

= 940 + (2 × 94) 5. Ans. (D)


= 940 + (188) Sol. Total members who registered for
= 1128 both clubs in the month of March
3. Ans. (A) = 320 + 400
Sol. Number of members who registered = 720
for club A in April = 360 Total members who registered for
Number of members who registered both clubs in the month of July
for club A in June = 420 = 380 + 500
∴ % decrease = 880
720

TS
420 − 360 Ratio =
= × 100 880
420 9
60 =
× 100 11
420

H
= 9 : 11
600 6. Ans. (B)
= Sol. No. of members who registered for

G
42
club A in May and June together
2
= 14 = 400 + 420.
7
4. Ans. (E)
Sol. Let male members who registered U = 820
No. of members who registered for
club B in May and June together
O
for club B in the month of March
= 480 + 520
= x. = 1000
∴ No. of females who registered for
H
∴ Difference = 1000 – 820
club B in the month of March = 180
3 7. Ans. (D)
_T

= x.
5 Sol. 8 men can finish a piece of work in
Total members in club in month of 25 days.
March = 400 In one day, one man can complete
C

3 1
∴ x + x = 400 work
5 8 × 25
8x
PS

= 400 15 women can finish a piece of work


5 in 16 days
400 × 5 In one day, one woman can complete
x = = 250
8 1
U

No. of female members in club B in work


15 × 16
the month of March Work completed by 4 men and 8
@

3 3 women in 10 days
= x = × 250 4 8
5 5 +
= × 10
= 150 8 × 25 15 × 16
No. of female members registered in 1 1
= + × 10
club A in the month of March 50 30
= 50 + 150 3+5
= × 10
= 200 150
Total no. of members in club A in the 8 8
month of March = 320 = × 10 =
150 15
∴ No. of male members in club A 8 15 − 8
Work left = 1 – =
in the month of March 15 15
= 320 – 200 7
=
= 120 15
Solved Test Papers of Various Examinations ✫ 801

7 No. of items returned to shop B in


work is completed by (4 + 6) November
15
men and 8 women. 25
7 = × 416 = 104
work is completed by 10 men 8 100
15 ∴ %
women. 52
In one day, work completed by 10 × 100
104
men and 8 women
5200
10 8 =
= + 104

TS
8 × 25 15 × 16
= 50
1 1
= + 10. Ans. (D)
20 30 Sol. Total no. of items sold by shop B in
3+2 1 September, October and November

H
= = 5 60 =
60 12 = 540 + 360 + 416
1 = 1316 ... (i)

G
work is completed in 1 day
12 Total no. of items returned by
7 7 customers to shop B in September,

15
work is completed in 12 ×

28
15
U October and November

=
10
× 540 +
5
× 360
O
= 100 100
5
3 25
× 416
H
= 5 days +
5 100
8. Ans. (D) = 54 + 18 + 104
_T

Sol. Total number of items returned by = 176 (ii)


the customers to shop A in August No. of items not returned = (i) – (ii)
and October together = 1316 – 176
15 = 1140
× 580 + 4 × 550
C

=
100 100 ∴ Average no. of items no returned
1140
PS

= 87 + 22 = = 380
= 109 items 3
11. Ans. (D)
Total number of items returned by
Sol. No. of items sold by shop A in
the customer to shop B in September
U

October
and October together
= 550 (i)
10 5 No. of items returned by customers
× 540 + × 360
@

=
100 100 to shop A in October
= 54 + 18 4
= × 550
= 72 100
109 − 72 = 22 ... (ii)
∴ % = × 100
72 ∴ No. of items not returned by
37 customers to shop A in October
= × 100 = 550 – 22
72
= 51 = 528 ... (iii)
9. Ans. (B) No. of items returned by the
Sol. No. of items returned to shop B in customers to shop A in November
August 20
= × 720
8 100
= × 650 = 52 = 144 ... (iv)
100
802 ✫ Quantitative Aptitude

Hence of (iii) : (iv) x2 + 142 – 154 = 277


528 x2 – 12 = 277
=
144
x2 = 277 + 12
11
= = 289
3
= 11 : 3 x = 289
12. Ans. (A) = 17
Sol. No. of items not returned by 16. Ans. (B)
customers to shop A in September

TS
Sol. (? + 5.04) × 71.94 ÷ 257 = (6.05)
2 3
12
= 600 – × 600 1
(x + 5) × 72 ÷ (257) 2 = (6)3
100
= 600 – 72 (x + 5) × 72 ÷ 16 = 216
( x + 5) × 72

H
= 528 = 216
Items not returned but found to be 16
216 × 16

G
defective (x + 5) =
72
1 x + 5 = 48
= × 528
12
= 44 U x = 48 – 5
= 43
O
∴ Total no. of defective items sold 17. Ans. (C)
by shop A (8.07)3 – (3.93)3
Sol. =? × 7.02
H
= 44 + 72
(4.06)2
= 116
13. Ans. (A) (8)3 – (4)3
_T

= x × 7
x (4)2
Sol. × (813.97 × 3.08)
100 512 − 64
= 754.08 + 467.06 = x × 7
16
C

x
× (813 × 3) = 754 + 467 488
100 = x × 7
16
PS

x
× 2442 = 1221 488
100 x =
1221 16 × 17
x = × 100
2442 = 4
U

= 50 18. Ans. (A)


14. Ans. (D) Sol. 20% of a = b
@

20
Sol. (1356.07 – x) ÷ 3.09 = 2196.11 ÷ 5.92 × a = b
100
2196 a
(1356 – x) ÷ 3 = = b ... (i)
6 5
1 2196 b b
(1356 – x) × = × 20 = ... (ii)
3 6 100 5
1356 – x = 366 × 3
1356 – x = 1098 From (i) and (ii)
x = 1356 – 1098 b a a
= 258 = =
5 5×5 25
15. Ans. (D) x a
× a =
Sol. x2 + 142.04 – 21.97× 7.08 = 277.03 100 25
x2 + 142 – (22 × 7) = 277 x = 4% of a
Solved Test Papers of Various Examinations ✫ 803

19. Ans. (B) ∴ No. of chocolate brownies sold by


30 bakery B
Sol. × 360
100 = 5 × 8 = 40 ... (i)
(angle around a point = 360°) Let the no. of chocolate and vanilla
= 3 × 36 brownies sold by bakery C = 3x and
= 108° x respectively
20. Ans. (B)
25
Sol. Let brownies sold by bakery D = 2x 3x + x = × 320
and bakery F = 3x 100
1

TS
Cost of 3x brownies sold by F at 4x = × 320
` 50 each 4
= 3x × 50 4x = 80
= 150x x = 20

H
10 ∴ No. of chocolate brownies sold by
Brownies sold by D = × 320 bakery C
100

G
= 32 = 3 × 20 = 60 ... (ii)
32 = 2x Total no. of chocolate brownies sold
x = 16 by bakery B and C
∴ Amount earned by bakery F
= 150 × 16 U = (i) + (ii) = 40 + 60
= 100
O
= ` 2,400 23. Ans. (B)
21. Ans. (B) Sol. Total no. of brownies sold by bakery
H
Sol. Average no. of brownies sold by A and D together
bakeries A and B 15 10
= × 320 + × 320
_T

15 20 100 100
× 320 + × 320
= 100 100 = 48 + 32
2 = 80 ... (i)
(48 + 64) No. of brownies sold by E
C

= 30
2
= × 320
112 100
PS

= = 56 ... (i) = 96 ... (ii)


2
Average no. of brownies sold by ∴ Difference (ii) – (i) 96 – 80
bakeries C and E = 16
U

25 30 24. Ans. (B)


× 320 + × 320 Sol. Let the present ages of A and B be
= 100 100
9x and 10x respectively
@

2 Age of A eight years ago = 9x – 8


(80 + 96)
= Age of B eight years ago = 10x – 8
2 It is given that
176
= = 88 ... (ii) 1
2 (9 x − 8) 1
Difference = 88 – 56 = 32 7 =
1 2
22. Ans. (C) (10 x − 8)
Sol. Let the no. of chocolate and vanilla 7
brownies sold by bakery B = 5x and
9x − 8 4 1
3x respectively =
20 10x − 8 7 2
5x + 3x = × 320
100 (9 x − 8 ) =
7
8x = 64
x = 8
(10x − 8) 8
804 ✫ Quantitative Aptitude

6.4 16
8(9x – 8) = 7(10x – 8) ⇒ = [60 minutes = 1 hr]
x+y 60
72x – 64 = 70x – 56
2x = 8 6.4 8
i.e. =
x = 4 x+y 30
Present age of A = 9 × 4 = 36
i.e. 30(6.4) = 8x + 8y
Present age of B = 10 × 4 = 40
Eight years hence A’s age = 36 + 8 192 = 8x + 8y
= 44 x + y = 24 ... (i)

TS
Eight years hence B’s age = 40 + 8 9.9
= 48 Upstream : = 33
x−y
44 11 9.9 33
Ratio = =
48 12 x−y 60

H
= 11 : 12 9.9 11
25. Ans. (D) =
x−y 20

G
Sol. Area of circle = πr2
0.9 1
22 =
= × 21 × 21 x−y 20
7
= 66 × 21
= 1386 sq cm U x – y = 18
On adding (i) and (ii) we get
... (ii)
O
Area of rectangle = area of circle 2x = 42
Let the length and breadth of rectangle x = 21
H
be 14x and 11x respectively Speed of boat = 21 km /hr
14x × 11x = 1386 Speed of stream 21 + y = 24
y = 3 km /hr
_T

154x2 = 1386
Time taken to travel 48 km upstream
1386
x2 = 48 48 48 8
154 = = = = hr
x2 = 9 x−y 21 − 3 18 3
C

... (iii)
x = 3
Time taken to travel 48 km downstram
Length =14x = 14 × 3 = 42 cm
PS

Breadth = 11x = 11 × 3 = 33 cm 48 48 48
= = = = 2 hr... (iv)
Perimeter of rectangle = 2(l + b) x + y 21 + 3 24
= 2 (42 + 33) Total time = (iii) + (iv)
U

= 2 (75) 8
= 150 cm + 2
3
26. Ans. (B)
@

14
Sol. Volume of metallic cuboid = l × b × h = hr = 280 min.
3
= 12 cm × 9 cm × 2 cm = 4 hr 42 min. ~ 4 hr 40 min.
= 216 cm3 28. Ans. (B)
Volume of cuboid = volume of cube
Let edge of cube = x cm
∴ x3 = 216
Sol.
x = ( 216) 3
1

x = 6
27. Ans. (A) 29. Ans. (B)
Sol. Let speed of boat = x km /hr Sol. 14 → 14
and speed of current = y km /hr 14 + 2 = 16 → 14 + 2 = 16
6.4 16 + 6 = 22 → 16 + (2 + 4) = 22
downstream: = 16
x+y 22 + 12 = 34 → 22 + (6 + 6) = 34
Solved Test Papers of Various Examinations ✫ 805

34 + 20 = 54 → 34 + (12 + 8) = 54 (y + 5) (y + 5) = 0
54 + 30 = 84 → 54 + (20 + 10) = 84 y + 5 = 0, y + 5 = 0
The series is 14, 16, 22, 34, 54, 84 y = –5, y = –5
30. Ans. (C) The values of x are equal to or
Sol. 852 greater than values of y
(852 ÷ 3) + 1 = 285 35. Ans. (E)
(285 ÷ 3) + 1 = 96 Sol. I. 5x2 – 11x + 2 = 0
(96 ÷ 3) + 1 = 33 5x2 – 10x – x + 2 = 0
(33 ÷ 3) + 1 = 12 5x (x – 2) – (x – 2) = 0
(12 ÷ 3) + 1 = 5 (5x – 1) (x – 2) = 0

TS
5x – 1 = 0, x – 2 = 0
The series is 852, 285, 96, 33, 12, 5
31. Ans. (B) 1
x = x = 2
Sol. 0.25 2
5
II. 3y – 5y + 2 = 0

H
0.25 × 16 = 4
3y2 – 3y – 2y + 2 = 0
4 × 8 = 32 3y(y – 1) – 2(y – 1) = 0
32 × 4 = 128

G
(3y – 2) (y – 1) = 0
128 × 2 = 256 3y – 2 = 0, y – 1 = 0
256 × 1 = 256
2
32. Ans. (B)
U
The series is 0.25, 4, 32, 128, 256, 256 y =
3
, y = 1
One of the values of x is greater than
O
Sol. 9 one of the values of y and the other
(9 × 0.5) – 0.5 = 4 value of x is less than the other value
(4 × 1) – 1 = 3 of y.
H
(3 × 1.5) – 1.5 = 3 36. Ans. (D)
(3 × 2) – 2 = 4 Sol. I. 5x2 – 13x + 6 = 0
_T

(4 × 2.5) – 2.5 = 7.5 5x2 – 10x – 3x + 6 = 0


The series is 9, 4, 3, 3, 4, 7.5 5x(x – 2) –3 (x – 2) = 0
33. Ans. (C) (x – 2) (5x – 3) = 0
Sol. I. 3x2 + 13x + 12 = 0 x – 2 = 0, 5x – 3 = 0
C

3x2 + 9x + 4x + 12 = 0 3
x = 2, x =
3x(x + 3) + 4 (x + 3) = 0 5
PS

(x + 3) (3x + 4) = 0 II. 2y2 – 7y + 6 = 0


2
x + 3 = 0, 3x + 4 = 0 2y – 4y – 3y + 6 = 0
4 2y(y – 2) –3 (y – 2) = 0
x = –3, x = – (y – 2) (2y – 3) = 0
U

3
II. 4y2 + 5y + 1 = 0 y – 2 = 0, 2y – 3 = 0
4y2 + 4y + y + 1 = 0 3
@

y = 2, x =
4y(y + 1) + (y + 1) = 0 2
The values of x are less than or equal
(4y + 1)(y + 1) = 0
to values of y.
4y + 1 = 0, y + 1 = 0
1 37. Ans. (A)
5 = – y = –1 Sol. I. 2x2 – 17x + 35 = 0
4
The values of y are greater than 2x2 – 10x – 7x + 35 = 0
values of x. 2x(x – 5) –7 (x – 5) = 0
34. Ans. (B) (2x – 7) (x – 5) = 0
Sol. I. x2 = 25 2x – 7 = 0, x – 5 = 0
x = ± 5 7
x = + 5, x = –5 x = , x = 5
2
II. y2 + 10y + 25 = 0 II. 3y2 – 10y + 7 = 0
y2 + 5y + 5y + 25 = 0 3y2 – 7y – 3y + 7 = 0
y(y + 5) +5 (y + 5) = 0 y(3y – 7) – 1 (3y – 7) = 0
806 ✫ Quantitative Aptitude

(3y – 7) (y – 1) = 0 30
3y – 7 = 0, y – 1 = 0 Remaining amount = x – x
100
7 3 7x
y = , y = 1 x – x =
3 10 10
The values of x are greater than
values of y. 30 x
38. Ans. (D) Given, – Son’s share = 31500
100
P×4×5
Sol. S.I. = 30 x 3 7x
100 – = 31500
100 14 10

TS
20P P
S.I. = =
100 5 3 x 21x
2 – = 31500
P 10 1 0 140
A = + P 1+
42 x − 21x

H
5 100
= 31500
6P 11 × 11 140
A =

G
5 10 10 21x
= 31500
6P 121 140
A =
5 100
C.I. = A – P U 21x = 31500 × 140
31500 × 140
O
6P 121 6P x =
= – 21
5 100 5 = 1500 × 140
H
It is given = 210000
6P 121 6P 40. Ans. (C)
_T

– = 378
5 100 5 Sol. Let no. of green balls = x
726P 6P Total balls = 2 + 5 + x = 7 + x
– = 378 Probability of drawing one ball at
500 5
C

random = (7 + x) c1
726P – 600P
= 378 Probability that the ball drawn is
PS

500 yellowc = 5c1


126P = 378 × 500
5 c1 2
378 × 500 P(E) = =
P = (7 + x )c1 9
U

126
5 2
378 × 250 = =
= = ` 1500 7+x 9
@

63
45 = 14 + 2x
39. Ans. (C)
2x = 31
Sol. Let the total retirement fund = x.
30 31
Amount kept ×x x = = 15.5
100 2
Solved Test Papers of Various Examinations ✫ 807

IBPS Specialist Officer (PT)


Examination, 2017
Directions (Q. 1-5): Refer to the table and answer 5. What is the difference between the total
the given questions. number of books received by 7 distributors
% of printed Number of together of company D and that by 11
Name of the Number of books books distributors together of company B?
Printing books printed distributed received by (1) 330 (2) 350
Company among each (3) 270 (4) 310

TS
distributors distributor (5) 290
A 5600 80% 128 Directions (Q. 6-11): Refer to the graph and answer
B 3200 60% 80 the given questions.

H
C 3800 75% 190 Data regarding number of lockets sold by
two stores A and B on 5 different days
D 3000 68% 170

G
E 4500 70% 150
Note: Books were equally distributed among the
distributors of respective printing companies.
1. What is the average number of books U
O
distributed by companies C, D and E among
their respective distributors?
H
(1) 2720 (2) 2940
(3) 2700 (4) 2640
_T

(5) 2680
2. Company A prints books either in Hindi or
English language. The ratio of Hindi books
C

distributed to English books distributed is


11: 5. What is the number of English books
PS

received by each distributor of company A?


(1) 40 (2) 55
(3) 60 (4) 45 6. On Monday, the total number of lockets sold
(5) 50 by store A and store B together was 6 more
U

3. 6% of the books printed by company B had than that sold by the same stores together
minor printing mistakes. If the ratio of the on Saturday and store A sold 38 lockets. How
@

number of books with minor printing many lockets did store B sell on Monday?
mistakes printed by company B to that by (1) 34 (2) 42
company A is 3 : 7 then what per cent of (3) 38 (4) 32
books printed by company A had minor (5) 36
printing mistakes?
(1) 12 (2) 6 7. What was the difference between the total
(3) 7 (4) 10 number of lockets sold by store B on
(5) 8 Thursday and Saturday together and the
number of lockets sold by store A on
4. The number of distributors of company E is
Saturday?
what per cent more than that of company C?
(1) 36 (2) 18
(1) 45 (2) 35
(3) 30 (4) 40 (3) 32 (4) 28
(5) 50 (5) 38
808 ✫ Quantitative Aptitude

8. The total number of lockets sold by stores A the ratio of the MRIs done for males by lab
and B together on Tuesday was what per cent R to that done for males by Lab S?
of that sold by the same stores together on (1) 1 : 6 (2) 7 : 9
Friday? (3) 5 : 6 (4) 4 : 9
(1) 120 (2) 136 (5) 5 : 8
1
(3) 128 (4) 113 13. In lab T, the ratio of the number of females
3
(5) 132 who came for MRI to that of males who came
for MRI was 7 : 5. What was the number of
9. On Thursday, if the ratio of the number of
females who came for MRI in labT?
lockets sold by store B to that sold by store

TS
(1) 14 (2) 15
C was 6 : 7 then how many lockets were sold (3) 21 (4) 7
by store C? (5) 10
(1) 49 (2) 56
14. As compared to week I, in week II the

H
(3) 35 (4) 63
number of MRIs done by labs P and R
(5) 42
decreased by 10% each respectively. What

G
10. On Sunday, if the number of lockets sold by is the total number of MRIs done by lab P
store A was 16 less than that sold by store A and R together in week II?
on Wednesday, then what was the average (1) 64 (2) 76
number of lockets sold by store A on
Wednesday, Thursday and Sunday? U (3) 84
(5) 72
(4) 80
O
(1) 31 (2) 30 15. What is the difference between the total
(3) 32 (4) 33 number of MRIs done by lab P and Q
H
(5) 34 together and that done by lab S and T
11. What was the ratio of the total number of together?
_T

lockets sold by store B on Wednesday and (1) 20 (2) 26


Friday together to the number of lockets sold (3) 28 (4) 22
by store A on Friday? (5) 24
16. The total number of MRIs done by all the
C

(1) 27 : 10 (2) 17 : 8
(3) 14 : 5 (4) 19 : 4 given labs together in week II of April 2017
(5) 19 : 10 was 25% more than that in week I of March
PS

2017. What is the total number of MRIs done


Directions (Q. 12-17): Refer to the pie chart and by all the given labs together in week I of
answer the given questions. March 2017?
U

Distribution of the total number of MRI (1) 150 (2) 120


done by five labs in week I of April 2017 (3) 180 (4) 140
Total number = 200 (5) Data inadequate
@

17. What is the central angle corresponding to


T P the number of MRIs done by lab S?
12% 15% (1) 118.6° (2) 115.2°
Q (3) 112.8° (4) 114.9°
S 16% (5) 119.2°
32%
Directions (Q. 18-22): Study the following
R information carefully and answer the given
25% questions.
Data regarding distribution of a certain
12. Out of the total number of MRIs done by lab number of employees (male + female) in four
R, two-fifths were done for males and out of departments—HR, Marketing, Finance and IT—
the total number of MRIs done by lab S, in 2015. The number of males (in all departments)
three-eighths were done for males. What is is 80.
Solved Test Papers of Various Examinations ✫ 809

HR and Marketing: The ratio of the 23. A boat takes T hours less to travel upstream
number of employees (male + female) in from A to B than the time it takes to travel
HR, Marketing and Finance is 3 : 4 : 5. upstream from B to C. The distance between
There are total 21 females in HR and B and C is 15 km more than the distance
Marketing together, which is 35% of the between A and B. If the speed of the boat
number of females (in all four departments). upstream is 15 km/h, then how much
The number of females in Marketing is 3 distance upstream will the boat cover in ‘T
more than that in HR. + 3’ hours? (in km)
IT: There are 44 employees (male + (1) 45 (2) 80
female) in this department. 35% of males (3) 60 (4) 105

TS
(in all four departments) are in this
(5) 75
department.
Directions (Q. 24-29): What approximate value
18. What is the difference between the number
of employees (male + female) in IT and the will come in place of the question mark(?) in the

H
number of employees (male + female) in given questions? (You are not expected to
HR? calculate exact value).

G
(1) 16 (2) 24 24. 4.01 × (18.03 – 11.98) × 14.86 = ?
(3) 22 (4) 18 (1) 350 (2) 360
(5) 20
19. If the total monthly salary of employees
(male + female) in Marketing is ` 12,16,000 U (3) 370
(5) 330
(4) 340
O
25. 15.08 × 18.02 – ? = 142
then what is the average monthly salary of (1) 82 (2) 58
the employees (male + female) in Marketing?
(3) 88 (4) 65
H
(in `)
(5) 74
(1) 34000 (2) 32000
_T

(3) 38000 (4) 35000 26. 179.92 – 218.08 – ? = 19.97


(5) 36000 (1) 129 (2) 137
20. The total number of males in IT and in (3) 144 (4) 121
Marketing together is what per cent more (5) 152
C

than the number of females in Marketing? 27. 640.93 – ? + 874.95 = 590.92


(1) 250 (2) 400 (1) 885 (2) 905
PS

(3) 350 (4) 300 (3) 915 (4) 925


(5) 500
(5) 895
21. In 2016, in Finance, if 3 males joined and 8
28. 1224 – ? =16.03 – 8.97
U

females left, then what was the new ratio of


the number of males to that of females? (No (1) 576 (2) 676
male left and no new female joined in (3) 784 (4) 841
@

Finance in 2016.) (5) 900


(1) 3 : 1 (2) 3 : 2 29. ?% of 259.96 – 28.02 = 153.96
(3) 4 : 3 (4) 5 : 3 (1) 50 (2) 55
(5) 5 : 4 (3) 80 (4) 60
22. The number of employees (male + female) (5) 70
in Finance is what per cent of the total
number of employees (male + female) in all 30. Every month, Sunaina invests 30% of her
four departments? monthly income in a gold scheme, and pays
2 5 15% as house rent. Out of the remaining
(1) 22 (2) 25 monthly incomes, she spends on groceries
7 7
6 1 and paying bills in the ratio of 5 : 6. If the
(3) 26 (4) 29 amount spent on groceries in a year is
7 7
4 ` 96000, then what is the amount that she
(5) 28 invests in gold scheme every month?
7
810 ✫ Quantitative Aptitude

(1) ` 7,500 (2) ` 8,400 (5) if x = y no relationship can be


(3) ` 7,200 (4) ` 10,800 established.
(5) ` 9,600 35. I. x2 – 22x + 117 = 0
31. The ratio of the present age of A to B is 4 : 5 II. y2 + 4y – 96 = 0
and that of the present age of C to D is 7: 5. 36. I. x2 = 144
The average of the present ages of A, B, C II. (y + 12)2 = 0
and D is 27 years and D’s age 3 years hence
will be 14 years less than A’s present age. 37. I. 6x2 – 7x + 2 = 0
What is B’s present age? (in years) II. 2y2 – 5y + 3 = 0

TS
(1) 30 (2) 25 38. I. x2 – 14x + 48 = 0
(3) 40 (4) 50 II. y2 – 16y + 63 = 0
(5) 20
39. I. 3x2 – 7x + 2 = 0
32. In a bag there are 4 red balls, 2 green balls II. y2 – 7y + 10 = 0

H
and 3 yellow balls. If two balls are drawn at
40. I. x2 + 8x + 15 = 0
random, what is the probability that at least
II y2 + 13y + 40 = 0

G
one of the balls is green in colour?
7 1 Directions (Q. 141-146): What will come in place
of question mark(?) in the given number series
(1)
12
5
(2)
4
1 U questions?
41. 14 30 66 130 230 ?
O
(3) (4)
12 6 (1) 374 (2) 378
1 (3) 382 (4) 364
H
(5)
12 (5) 368
33. A trader marks up his good such that he can 42. 65 74 56 92 ? 164
_T

gain 5% profit after giving 20% discount to (1) 25 (2) 18


his customers. One particular customer (3) 24 (4) 20
availed a discount of 10% only. If the profit
(5) 15
C

earned by the trader from this customer is


` 58, then, what is the cost price of the good? 43. 7 14 40 103 ? 442
(1) ` 480 (2) ` 400 (1) 225 (2) 235
PS

(3) ` 240 (4) ` 160 (3) 218 (4) 227


(5) ` 320 (5) 221
34. The average number of pages typed by a 44. 8 18 57 232 ? 6996
U

typist on Monday, Tuesday and Wednesday (1) 1085 (2) 1165


is 39. The average number of pages typed (3) 1195 (4) 1125
@

by a typist on Tuesday, Wednesday and (5) 1215


Thursday is 26. If the typist typed 36 pages
on Thursday, the number of pages typed by 45. 7 2.5 1.5 2 7 ?
him on Thursday is what per cent less than (1) 55 (2) 21
that on Monday? (3) 57 (4) 31
(1) 48 (2) 54 (5) 65
(3) 52 (4) 44 46. 8 11 18 33 64 ?
(5) 60
(1) 127 (2) 113
Directions (Q. 35-40): In this question two (3) 123 (4) 119
equations numbered I and II are given. You have (5) 131
to solve both the equations and mark the
47. ` x was invested in scheme A, which offers
appropriate option, Give answer
simple interest at 10 pcpa for 2 years. The
(1) if x > y (2) if x < y
amount received from scheme A was
(3) if x ≤ y (4) if x ≥ y
Solved Test Papers of Various Examinations ✫ 811

invested into scheme B, which offers the next x days and complete one-third of it.
compound interest (compounded annually) If after ‘x + 4’ days, 24 girls and 30 boys work
at 20 pcpa for 2 years. If the amount received
together and complete the remaining work
from scheme B is ` 5356.80, what is the value 2
of X? in 6 days, how many days will 30 boys
3
(1) ` 3100 (2) ` 2700 take to complete the whole work?
(3) ` 2800 (4) ` 3700 (1) 21 (2) 20
(5) ` 3900 (3) 18 (4) 27
48. A solid right circular metal cylinder with (5) 24

TS
radius 12m and height 15m is melted down 50. Vessel A contains x litres of milk and vessel
and all the metal is used to recast a new solid B contains y litres of water. From vessels A
cylinder with radius 10m. What is the curved and B, 60% milk and 12% water was taken
surface area of the new cylinder? (in m2) and mixed in vessel C. The ratio of the

H
(1) 442 (2) 416 quantity of milk to water in vessel C was
(3) 454 (4) 448 25 : 4. If x was 20 litres more than y then what

G
(5) None of these is the quantity of milk in vessel C? (in litres)
(1) 60 (2) 80
49. 24 girls work for 4 days on a project and
complete one-ninth of it. 30 boys work for
U (3) 90
(5) 75
(4) 50
O
Answers and Explanatory Notes
H
1. Ans. (5) 2. Ans. (1)
Sol. No. of printed books distributed by Sol. The ratio of Hindi books distributed
_T

company C to English books distributed is 11 : 5.


Total no. of books received by each
75
= × 3800 distributor = 128
100 ∴ No. of English books received by
C

= 2850 ... (i) each distributor of company A


No. of printed books distributed by 5
PS

company D is × 128 = 40
16
68 3. Ans. (5)
= × 3000 Sol. No. of books printed by company B
100
U

= 2040 ... (ii) that had minor printing mistakes


No. of printed books distributed by 6
= × 3200
@

company E 100
70 = 192
= × 4500 No. of books printed by company A
100 that had minor printing mistakes = x.
= 3150 ... (iii)
3 192
Total printed books distributed by i.e. ==
company C, company D and company 7 x
E is (i) + (ii) + (iii) 192 × 7
x =
i.e. 2850 + 2040 + 3150 3
= 8040 = 448
Total no. of books printed by
∴ Average number of books company A = 5600
distributed Required percentage
8040 448
= = 2680 = × 100 = 8
3 5600
812 ✫ Quantitative Aptitude

4. Ans. (4) No. of lockets sold by store A on


Sol. No. of books distributed by company Monday = 38
∴ No. of lockets sold by store B on
70
E = × 4500 Monday
100 = 74 – 38
= 3150 = 36
No. of books received by each 7. Ans. (4)
distributor from company E = 150 Sol. Total number of lockets sold by store
3150 B on Thursday and Saturday
Total no. of distributors = = 36 + 30
150

TS
= 21 = 66
Number of books distributed by No. of lockets sold by store A on
75 Saturday = 38
company C = × 3800

H
100 Difference = 66 – 38
= 75 × 38 = 28
8. Ans. (2)

G
= 2850
No. of books received by each Sol. The total no. of lockets sold by stores
distributor from company C = 190 A and B together on Tuesday
Total no. of distributors
2850 U = 40 + 28
= 68
O
= The total no. of lockets sold by stores
190 A and B together on Friday
= 15 = 20 + 30
H
Difference = 21 – 15 = 50
= 6 Required percentage
_T

∴ Required percentage
68 6800
6 = × 100 = = 136
= × 100 50 50
15
= 40 9. Ans. (5)
C

5. Ans. (4) Sol. No. of lockets sold by store B on


Sol. Total no. of books received by 7 Thursday = 36
PS

distributors together of company Let no. of lockets sold by store C on


D = 170 × 7 Thursday = x
= 1190 ... (i) 36 6
=
U

Total no. of books received by 11 x 7


distributors together of company B 6x = 252
= 80 × 11
@

252
= 880 ... (ii) x =
6
Difference = (i) – (ii) = 42
= 1190 – 880 10. Ans. (3)
= 310 Sol. No. of lockets sold by store A on
6. Ans. (5) Wednesday = 42
Sol. No. of lockets sold by store A and No. of lockets sold by store A on
store B on Saturday Sunday = 42 – 16 = 26
= 30 + 38 Average of lockets sold by store A
= 68 on Wednesday, Thursday and
No. of lockets sold by store A and Sunday = 42 + 28 + 26
store B on Monday = 96
= 6 + 68 96
= 74 Average = = 32
3
Solved Test Papers of Various Examinations ✫ 813

11. Ans. (1) No. of MRIs done by lab R in week I


Sol. Total no. of lockets sold by store B 25
on Wednesday and Friday = × 200 = 50
100
24 + 30 = 54 No. of MRIs done by lab R in week II
No. of lockets sold by store A on 10
Friday = 20 = 50 – × 50
100
∴ Required ratio
= 50 – 5
54 27 = 45 ... (ii)
=
20 10 ∴ Total no. of MRIs done by labs P

TS
i.e. 27 : 10 and R in week II
12. Ans. (3) = (i) + (ii)
Sol. Total number of MRIs done by lab R = 27 + 45
= 72

H
25
= × 200 = 50 15. Ans. (2)
100 Sol. Total no. of MRIs done by lab P and

G
Total no. of MRIs done for males by Q together
2 15 16
lab R = × 50 = 20
5
Total number of MRIs done by lab S
U =
100
= 30 + 32
× 200 +
100
× 200
O
32 = 62 (i)
= × 200 = 64
100 Total no. of MRIs done by labs S and
H
Total number of MRIs done for males T together
32 12
3 = × 200 + × 200
by lab S = × 64 = 24
_T

100 100
8
= 64 + 24
20 5
∴ Required Ratio = = = 88 ... (ii)
24 6 ∴ Required difference
C

i.e., 5 : 6 = 88 – 62
13. Ans. (1) = 26
PS

Sol. Total number of MRIs done by lab 16. Ans. (5)


12 Sol. The no. of MRIs done by each lab in
T = × 200 = 24 the first week of March 2017 is not
100
Let the no. of females = 7x and no. given and also no. of MRIs done by
U

of males = 5x each lab in week II of April 2017 is


7x + 5x = 24 not given. The data is inadquate to
@

12x = 24 find the reqd. solution.


x = 2 17. Ans. (2)
No. of females who came for MRI in 32
Sol. × 360
lab T = 7x = 7 × 2 = 14 100
14. Ans. (5) 1152
=
Sol. No. of MRIs done by lab P in week I 10
= 115.2°
15
= × 200 = 30 Total no. of males in all the 4
100
departments
No. of MRIs done by lab P in week II = 80 (Given) ... (i)
10 Let the no. of employees in HR,
= 30 – × 30
100 Marketing and Finance be 3x, 4x, 5x
= 30 – 3 = 27 ... (i) respectively
814 ✫ Quantitative Aptitude

No. of employees in IT = 44 (given) Total no. of employees in HR


No. of females in HR and Marketing = 3 × 8 = 24
together = 21 Total no. of employees in Marketing
35 = 4 × 8 = 32
21 = × x (x = Total no. of
100 Total no. of employees in Finance
females in all four departments) = 5 × 8 = 40
2100 No. of male employees in HR
= x = 24 – 9 = 15
35
No. of male employees in Marketing
x = 60

TS
= 32 – 12 = 20
Total no. of females in all four
No. of male employees in Finance
departments = 60 ... (ii)
= 40 – 23 = 17
∴ Total no. of employees in all four

H
departments Department Total Males Females
= 80 + 60
= 140 ... (iii) 1. IT 44 28 16

G
Let the no. of females in HR = y 2. Finance 40 17 23
Females in marketing = 3 + y 3. Marketing 32 20 12
Total no. of females in marketing and
HR = 21 U 4. HR 24 15 9
O
i.e. 21 = y + 3 + y Total 140 80 60
21 = 2y + 3 18. Ans. (5)
H
2y = 18 Sol. No. of employees in IT = 44
y = 9 No. of employees in HR = 24
∴ Required difference
_T

No. of females in HR = 9 = 44 – 24
No. of females in marketing = 20.
= 3 + y = 3 + 9 = 12 19. Ans. (3)
No. of males in IT department
C

Sol. Total monthly salary of employees in


35 marketing
= × 80
100 = ` 1216000
PS

= 28 No. of employees in marketing = 32


No. of females in IT department 1216000
= 44 – 28 ∴ Average salary =
U

32
= 16
= ` 38000
No. of females in Finance department
@

= 60 – (9 + 12 + 16) 20. Ans. (4)


= 60 – 37 Sol. Total no. of males in IT and in
= 23 marketing together = 28 + 20 = 48
No. of males in IT = 44 – 16 No. of females in marketing = 12
Difference = 48 – 12
= 28
= 36
3x + 4x + 5x + 44 = 80 + 60
∴ Required percentage
12x + 44 = 140
36
12x = 140 – 44 = × 100
= 96 12
= 300
96
x = 21. Ans. (3)
12
Sol. Total no. of males in 2016 in finance
= 8
= 17 + 3 = 20 (3 joined)
Solved Test Papers of Various Examinations ✫ 815

Total no. of females in 2016 in Finance 270 – x = 196


23 – 8 = 15 (8 left) x = 270 – 196
= 74
20 4
∴ Required ratio = = 26. Ans. (3)
15 3
i.e., 4 : 3 Sol. 179.92 – 218.08 – ? = 19.97
22. Ans. (5) 180 – 218 – x = 20
Sol. No. of employees in finance = 40 180 – 14.7x = 20
Total no. of employees in all 180 – 15 – x = 20

TS
departments = 140 x = 165 – 20
∴ Required percentage = 145 ~ 144
40 27. Ans. (4)
= × 100
140

H
Sol. 640.93 – ? + 874.95 = 590.92
400 641 – x + 875 = 591
=
14 641 + 875 – 591 = x

G
200 x = 925
=
7

= 28
4
7 U 28. Ans. (3)
Sol. 1224 – ? = 16.03 – 8.97
O
35 – x = 16 – 9
23. Ans. (3)
35 – x = 7
H
Sol. Let the distance between A and B
= x km x = 35 – 7
Distance between B and C = 15 + x x = 28
_T

Speed upstream = 15 km/h x = (28)2


x = 784
Time taken to cover x km =
15
29. Ans. (5)
C

Time taken to cover (x + 15) km


Sol. % of 259.96 – 28.02 = 153.96
x + 15
= x
PS

15 × 260 – 28 = 154
Difference 100
x + 15 x 26 x
– =T = 154 + 28
15 15 10
U

x + 15 − x 26 x
= 182
= T
15 10
@

26x = 1820
T = 1 hr
T + 3 = 1 + 3 = 4 hr 1820
x =
∴ Distance = 4 × 15 26
= 60 km = 70
24. Ans. (2) 30. Ans. (5)
Sol. 4.01 × (18.03 – 11.98) × 14.86 = ? Sol. Let the monthly income = ` x
4 × (18 – 12) × 15 = x Amount invested in gold scheme
4 × (6) × 15 = x
30 x 3x
x = 24 × 15 = =
= 360 100 10
25. Ans. (5) 15 x 3x
Sol. 15.08 × 18.02 – ? = 142 Rent = =
100 20
15 × 18 – x = 196
816 ✫ Quantitative Aptitude

Remaining amount left D’s age after 3 years = 5y + 3


3x 3x i.e. 5y + 3 = 4x – 14
= x – + 4x – 5y = 17 ... (ii)
20 10
Multiplying eq. (i) by 4 and (ii) by
3x + 6x
= x – 3 and subtracting we get
20 4{3x + 4y = 36}
20 x − 9 x 11x 3{4x – 5y = 17}
= =
20 20 i.e. 12x + 16y = 144
Amount spent on grocery
12 x − 15 y = 51
− + −

TS
5 11x
= ×
11 20 31y = 93
5x 93
= y =
20 31

H
Amount spent on paying bills
= 3
6 11x
= × Substituting y = 3 in eq (ii)

G
11 20 4x – 5 × 3 = 17
6x 3x
= = 4x – 15 = 17
20 10
Amount spent on groceries in a year
= 96000 U 4x = 32
x = 8
∴ B’s present age 5x = 5 × 8 = 40
O
96000
Amount spent per month = 32. Ans. (3)
12
H
Sol. No. of green balls = 2
= 8000 No. of red balls = 4
5x No. of yellow balls = 3
_T

= 8000 Total no. of balls = 2 + 4 + 3


20
5x = 8000 × 20 = 9
8000 × 20 Two balls drawn at random
C

x = n(S) = 9C2
5
= 8000 × 4 9×8 72
PS

= 32000 = = = 36
1× 2 2
Amount invested in gold Out of two balls at least one is green
3x 3
= = × 32000 in colour
U

10 10 n(E) = (2C1 × 7C1) + (2C2 + 7C0)


= 3 × 3200
2 7 2×1
× ×0
@

= 9600 = +
1 1 1×2
31. Ans. (3) = 14 + 1
Sol. Let the present age of A and B be = 15
4x and 5x respectively n(E) 15
Let the present age of C and D be P(E) = =
n(S) 36
7y and 5y respectively 5
Average of present ages of A, B, C =
and D 12
4 x + 5x + 7 y + 5y 33. Ans. (5)
= = 27 Sol. Let CP = 100, SP = 105 (Profit = 5%
4
9x + 12y = 27 × 4 given)
9x + 12y = 108 Discount = 20% on marked price
3x + 4y = 36 ... (i) SP = 80
SP = 80, marked price = 100
Solved Test Papers of Various Examinations ✫ 817

SP = 105, marked price


9 21x 189 x
100 5 = × =
= × 105 = 105 × 10 16 160
80 4
SP = 90, marked price = 100 SP – CP = Profit
[discount of 10% only is availed by 189 x
– x = 58
a customer] 160
marked price is 100, SP – 90 189x – 160x = 58 × 160
29x = 9280
5
∴ marked price is 105 × 9280
4 x =

TS
90 5 29
SP = × 105 ×
100 4 = ` 320
9 5
= × 105 × 34. Ans. (3)

H
10 4 Sol. Average number of pages typed by
9 × 21 5 a typist on Monday, Tuesday and

G
= × Wednesday = 39
2 4
∴ Total no. of pages typed
9 × 105
=
945
8
U = 39 × 3 = 117
Average no. of pages typed by the
typist on Tuesday, Wednesday and
O
=
8 Thursday = 26
94 ∴ Total no. of pages typed
H
Profit = – 100 [Profit = SP – CP] = 26 × 3 = 78
8
945 − 800 14 No. of pages typed on Thursday
_T

= = = 36 pages
8 8
∴ No. of pages typed on Tuesday
145
When profit is , CP – 100 and Wednesday
8 = 78 – 36
C

100 × 8 = 42
Profit is 58, CP – × 58
145 ∴ No. of pages typed on Monday
PS

800 × 58 = 117 – 42
= = 75
145
= ` 320 Difference = 75 – 36
= 39
U

Alternate method ∴ Required percentage


Let CP = x
@

39
105 x × 100
SP = (5% profit) 75
100
= 52
80 is SP – marked price = 100
105 x 35. Ans. (1)
is SP – marked price Sol. I. x2 – 22x + 117 = 0
100
100 105 x 21x x2 – 13x – 9x + 117 = 0
= × = x(x – 13) – 9 (x – 13) = 0
80 100 16
(x – 13) (x – 9) = 0
90 is SP – marked price = 100 x – 13 = 0 x – 9 = 0
When 100 is MP – SP = 90 x = 13, x = 9
21x 90 21x II. y2 + 4y – 96 = 0
When is MP – × y2 + 12y – 8y – 96 = 0
16 100 16
y(y + 12) – 8 (y + 12) = 0
818 ✫ Quantitative Aptitude

(y + 12) (y – 8) = 0 (3x – 1) (x – 2) = 0
y + 12 = 0 y – 8 = 0 (3x – 1) = 0 (x – 2) = 0
y = –12y = 8 1
x = , x = 2
The values of x are greater than 3
values of y. II. y2 – 7y + 10 = 0
y2 – 5y – 2y + 10 = 0
36. Ans. (4) y(y – 5) –2 (y – 5) = 0
Sol. I. x2 = 144 (y – 2) (y – 5) = 0
x = 12, –12 (y – 2) = 0, (y – 5) = 0
II. (y + 12)2 = 0 y = 2, y = 5

TS
(y + 12) (y + 12) = 0 The values of x are equal to or less
y = –12 than the values of y.
The values of x are greater than or
equal to values of y. 40. Ans. (4)

H
Sol. I. x2 + 8x + 15 = 0
37. Ans. (2) x2 + 5x + 3x + 15 = 0
Sol. I. 6x2 – 7x + 2 = 0

G
x (x + 5) + 3 (x + 5) = 0
6x2 – 4x – 3x + 2 = 0 (x + 5) (x + 3) = 0
2x(3x – 2) – (3x – 2) = 0 x + 5 = 0, x + 3 = 0
(3x – 2) (2x – 1) = 0
3x – 2 = 0, 2x – 1 = 0
U x = –5, x = –3
II. y2 + 13y + 40 = 0
O
2 1 y2 + 8y + 5y + 40 = 0
x = , x =
2
3 2 y(y + 8) + 5 (y + 8) = 0
II. 2y – 5y + 3 = 0
H
(y + 5) (y + 8) = 0
2y2 – 3y – 2y + 3 = 0
(y + 5) = 0, (y + 8) = 0
y(2y – 3) – (2y – 3) = 0
y = – 5, y = – 8
_T

(y – 1) (2y – 3) = 0
The values of x are equal to or
y – 1 = 0, 2y – 3 = 0
greater than the values of y.
3
y = 1, y =
2
C

The values of y are greater than 41. Ans. (1)


values of x Sol. 14, 30, 66, 130, 230, ?
PS

38. Ans. (5) 14


Sol. I. x2 – 14x + 48 = 0 14 + 42 = 14 + 16 = 30
x2 – 8x – 6x + 48 = 0 30 + 62 = 30 + 36 = 66
U

x(x – 8) – 6 (x – 8) = 0 66 + 82 = 66 + 64 = 130
(x – 8) (x – 6) = 0 130 + 102 = 130 + 100 = 230
230 + 122 = 230 + 144 = 374
@

x – 8 = 0 x – 6 = 0
x = 8, x = 6 The series is 14, 30, 66, 130, 230, 374
II. y2 – 16y + 63 = 0 42. Ans. (4)
y2 – 9y – 7y + 63 = 0 Sol. 65, 74, 56, 92, ?, 164
y(y – 9) – 7 (y – 9) = 0 65
(y – 7) (y – 9) = 0 65 + 9 = 74
y – 7 = 0 y – 9 = 0 74 – 18 = 56
y = 7 y = 9 56 + 36 = 92
The values of x are less than or 92 – 72 = 20
greater than values of y. 20 + 144 = 164
The series is 65, 74, 56, 92, 20, 164
39. Ans. (3)
Sol. I. 3x2 – 7x + 2 = 0 43. Ans. (4)
3x2 – 6x – x + 2 = 0 Sol. 7, 14, 40, 103, ?, 442
3x (x – 2) – (x – 2) = 0 7
Solved Test Papers of Various Examinations ✫ 819

2
7 + (23 – 1) = 7 + (8 – 1) = 7 + (7) = (14) 6x 20
14 + (33 – 1) = 14 + (27 – 1) = 14 + A= 1+
5 100
(26) = 40 2
40 + (43 – 1) = 40 + (64 – 1) = 40 + 6x 6
=
63 = 103 5 5
103 + (53 – 1) = 103 + (125 – 1) = 103
6x 36
+ 124 = 227 = ×
227 + (63 –1) = 227 + (216 – 1) = 227 5 25
216 x
+ 215 = 442 =
The series is 125

TS
7, 14, 40, 103, 227, 442 216 x
i.e. 5,356.80 =
125
44. Ans. (2)
5356.80 × 125
Sol. 8, 18, 57, 232, ? 6996 x =

H
8 216
669600
8 × 2 + 2 = 16 + 2 = 18 = = ` 3100
216

G
18 × 3 + 3 = 54 + 3 = 57
57 × 4 + 4 = 228 + 4 = 232 48. Ans. (5)
232 × 5 + 5 = 1160 + 5 = 1165
1165 × 6 + 6 = 6990 + 6 = 6996
The series is U Sol. Volume of cyclinder = πr2h
= π × 12 × 12 × 15
O
= 144 × 15π
8, 18, 57, 232, 1165, 6996
= 2160π
45. Ans. (1) Radius of new cyclinder = 10m
H
Sol. 7, 2.5, 1.5, 2, 7, ? Volume of the new cyclinder = 2160π
7 ∴ πr2h = 2160π
_T

7 × 0.5 – 1 = 2.5 π × 10 × 10 × h = 2160π


2.5 × 1 – 1 = 1.5
1.5 × 2 – 1 = 2 100h = 2160
2 × 4 – 1 = 7 216
C

h =
7 × 8 – 1 = 55 10
The series is 7, 2.5, 1.5, 2, 7, 55 108
PS

= m
46. Ans. (1) 5
Sol. 8, 11, 18, 33, 64, ? Curved surface area = 2πrh
8
U

22 108
8 + 3 = 11 = 2 × × 10 ×
7 5
11 + 3 + 4 = 18 2 × 22 × 2 × 108
@

18 + 7 + 8 = 33 =
7
33 + 15 + 16 = 64 88 × 108
64 + 31 + 32 = 127 =
7
The series is 8, 11, 18, 33, 64, 127 = 1357.71 sq m
47. Ans. (1) 49. Ans. (3)
x × 10 × 2
Sol. S.I. = Sol. Remaining project to be completed
x 100
= 1 1 5
5 = 1 – + =
x 9 3 9
Amount = x + 5
5 30 boys and 24 girls complete of
6x 9
= 20
5 project in days
3
820 ✫ Quantitative Aptitude

One complete project is completed 50. Ans. (1)


by 30 boys and 24 girls in Sol. Let vessel A contain x litres of milk
20 9 Vessel B contain y litres of water
× = 12 days Quantity of milk and water in vessel C
3 5
In one day 30 boys and 24 girls can 60 x 12 y 3x 3y
= , = ,
1 100 100 5 25
complete of the project (i)
12 Ratio of milk to water is
1 3x
24 girls can complete of the project
9 25
in 4 days = 5 =

TS
3y 4
∴ 24 girls can complete a project in
25
9 × 4 25
3x 25
= 36 days × =

H
5 3y 4
1
In one day 24 girls can complete 4x = 5y ... (i)
36

G
of the project ... (ii) x = 20 + y (Given)
Substituting x = 20 + y in equation (i)
In one day 30 boys can complete
4(20 + y) = 5y

1 1
(i) – (ii)
3−1 2 U 80 + 4y = 5y
80 = y
O
– = = of the project Water in vessel B = 80 litres
12 36 36 36
Milk in vessel A = 80 + 20 = 100 litres
∴ One complete project is completed
H
Milk in vessel C
by 30 boys in 60
= × 100
36 100
_T

= 18 days.
2 = 60 litres
C
PS
U
@
Solved Test Papers of Various Examinations ✫ 821

IBPS Specialist Officers Marketing


(PT) Examination, 2017
Direction—(Q. 1 to 10) The question consisting 5. What is the minimum passing percentage in
of a question and two statements I and II given a test?
below. You have to decide whether the data I. Raman scored 25% marks in the test
given in the statements are sufficient to answer and Sunil scored 288 marks which is
the question. Read both the statements and 128 more than Raman.
choose the most appropriate option. II. Raman scored 64 marks less than the

TS
(A) The data in statement I alone are minimum passing marks.
sufficient to answer the question, while 6. What is the value of x2 + y + z?
the data in statement II alone are not I. 4x + 3y + 5z = 60 and 2x = y, 2y = z
sufficient to answer the question.

H
II. 3x + 3y + 2z = 34 and 2x + 5y + 6z = 72
(B) The data in statement II alone are
sufficient to answer the question, while 7. Whose body weight is second highest

G
the data in statement I alone are not among the five boys Arun, Vinay, Suraj, Raju
sufficient to answer the question. and Pratap?
(C) The data either in statement I alone or
statement II alone are sufficient to
U I. Average weight of Arun, Suraj and
Vinay is 68 kg and average weight of
O
answer the question. Raju and Pratap is 72 kg. Also Suraj
(D) The data in both statements I and II is 78 kg. Raju is 68 kg and Vinay is
together are not sufficient to answer 46 kg.
H
the question. II. Average weight of Arun, Suraj, Vinay
(E) The data in both statements I and II and Raju is 68 kg and also Suraj is 78
_T

together are necessary to answer the kg. Raju is 68 kg and Vinay is 46 kg.
question. All of them have different weight.
1. What is the area of the circle? 8. How many marks did Subodh obtain in
C

I. Perimeter of the circle is 88 cm. Physics?


II. Diameter of the circle is 28 cm. I. The average marks of Subodh in
PS

History, Geography and Chemistry are


2. What is the rate of interest?
75.
I. Simple interest accrued on an amount
II. His average marks in History,
of ` 25,000 in two years is less than
Geography and Physics are 78.
U

the compound interest for the same


period by ` 250. 9. What is the population of the city A?
@

II. Simple interest accrued in 10 years is I. The ratio of the population of males
equal to the principal. and females in city A is 27 : 23 and
3. What is the number of trees planted in the the difference between their population
field in rows and columns? is 100000.
I. Number of columns is more than the II. The population of city A is 80% of that
number of rows by 4. of city B. The difference of population
II. Number of column is 20. of city A and city B is 312500.

4. What is the speed of the current? 10. How many students did participate in
I. A man can swim a distance of 9 km elocuation?
1 I. The students who participated in
in 1 hr downstream. dancing were 150% more than those
2
II. While coming back upstream, it takes who participated in elocution.
him 3 hr to cover the same distance. II. 150 students participated in dancing.
822 ✫ Quantitative Aptitude

Directions—(Q. 11 to 15) Study the following 15. If the income of company B in 2010 and 2011
graph carefully and answer the questions given were in the ratio of 2 : 3 respectively, what
below it. was the respective ratio of expenditure of
that company in these two years?
(A) 20 : 29
(B) 9 : 10
(C) 29 : 45
(D) 10 : 29
(E) None of these

TS
Direction—(Q. 16 to 20) Study the pie-charts
carefully to answer the given questions.
Percentage of students enrolled in different

H
activities in a school
N = 3000

G
Expenditure × Profit%
Income =
100
+ Expenditure
11. Expenditures of company B in 2009 and 2010
U
were ` 12 lakhs and ` 14.5 lakhs respectively.
O
What was the total income of company B in
2009 and 2010 together (in lakh rupees)?
H
(A) 39.75 (B) 37.95
(C) 38.75 (D) 38.55
_T

(E) None of these Percentage break-up of girls enrolled in


12. Ratio of expenditures of company A and B these activities out of the total students
in 2012 was 3 : 4 respectively. What was the N = 1750
respective ratio of their incomes in 2012?
C

(A) 21 : 26 (B) 13 : 14
(C) 14 : 13 (D) 26 : 21
PS

(E) None of these


13. Total expenditure of company A in all the
years together was 82.5 lakhs. What was the
U

total income of the company A in all the


years together?
(A) 1.23 crore
@

(B) 98.75 crore 16. Number of girls enrolled in dancing form are
(C) 99.85 crore
(D) Cannot be determined what per cent of total number of students
(E) None of these enrolled in the school?
(rounded off to two digits after decimal)
14. If the expenditure of company A and B in
2013 were equal and the total incomes of the (A) 12.35 (B) 14.12
two companies was ` 5.7 lakhs. What was (C) 11.67 (D) 10.08
the total expenditure of the two companies (E) None of these
in 2013?
17. How many boys are enrolled in singing and
(A) 4 lakhs
(B) 2 lakhs craft together?
(C) 4.2 lakhs (A) 505 (B) 610
(D) Cannot be determined (C) 485 (D) 420
(E) None of these (E) None of these
Solved Test Papers of Various Examinations ✫ 823

18. What is the respective ratio of number of 31. Two men P and Q start a journey from same
girls enrolled in swimming to the number 1
place at a speed of 3 km/hr and 3 km/h
of boys enrolled in swimming? 2
(A) 47 : 49 (B) 23 : 29 respectively. If they move in the same
(C) 29 : 23 (D) 49 : 47 direction then what is the distance between
(E) None of these them after 4 hours?
1
19. What is the total number of girls enrolled in (A) 3 km (B) 2 km
2
swimming and drawing together? 1
(A) 480 (B) 525 (C) 2 km (D) 3 km.
2

TS
(C) 505 (D) 495 (E) None of these
(E) None of these
Directions—(Q. 32 to 35) What will come in place
20. What is the approximate percentage of boys of question mark (?) in the given question?
in the school?

H
(A) 34 (B) 56 32. 14 × 627 ÷ (1089) = (?)3 + 141
(C) 28 (D) 50 (A) 5 5 (B) (125)3

G
(E) None of these
(C) 25 (D) 5
Directions—(Q. 21 to 30) In each question, two (E) None of these
equation numbered I and II are given. You have
to solve both the equations and mark an
U 33.
21.5 21 13.5
+ − =
(?) 3
+
17
1
O
appropriate answer. 5 6 15 4 30
(A) x < y (B) x > y (A) 2 (B) 8
(C) x ≥ y (D) x ≤ y
H
(C) 512 (D) 324
(E) relationship between x and y cannot be (E) None of these
established
( )
_T

2
21. I. 6x2 + 5x + 1 = 0 34. (7) + 11 = (?)1/3 + 2. 847 + 122
II. 15y2 + 8y + 1 = 0 (A) 36 + 44 (7) (B) 6
2
22. I. x + 5x + 6 = 0 (C) 216 (D) 36
C

II. 4y2 + 24y + 35 = 0 (E) None of these


2
61
PS

23. I. 2x2 + 5x + 3 = 0 18 455


35. × =?
II. y2 + 9y + 14 = 0 4 19 799
(A) 6320 (B) 6400
24. I. 88x2 – 19x + 1 = 0
(C) 6351.82 (D) 6431.82
U

II. 132y2 – 23y + 1 = 0 (E) 6491.82


25. I. 6x2 – 7x + 2 = 0 Directions—(Q. 36 to 40) What will come in place
@

II. 20y2 – 31y + 12 = 0 of question mark(?) in the given number series?


26. I. 6x2 + 23x + 20 = 0 36. 28 39 63 102 158 ?
II. 6y2 + 31y + 35 = 0 (A) 232 (B) 242
(C) 233 (D) 244
27. I. x2 = 81 (E) None of these
II. y2 – 18y + 81 = 0
37. 7 16 141 190 919 ?
28. I. 4x2 + 20x + 21 = 0 (A) 1029 (B) 1019
II. 2y2 + 17y + 35 = 0 (C) 1020 (D) 1030
(E) None of these
29. I. x2 - 14x + 48 = 0
II. y2 + 6 = 5y 38. 12 17 32 57 92 ?
(A) 198 (B) 195
30. I. 38x2 – 3x – 11 = 0 (C) 137 (D) 205
II. 28y2 + 32y + 9 = 0 (E) None of these
824 ✫ Quantitative Aptitude

39. 19 25 45 87 159? 45. What is the total number of books


(A) 254 (B) 279 distributed by publishers O and Q?
(C) 284 (D) 269 (A) 26702 (B) 27324
(E) None of these (C) 55028 (D) 54026
(E) None of these
40. 83 124 206 370 698?
46. Meena Kumar goes to a shop and buys a
(A) 1344 (B) 1324 saree, costing ` 5,225, including sales tax of
(C) 1364 (D) 1334 12%. The shopkeeper gives her a discount,
(E) None of these so that the price is decreased by an amount
Directions—(Q. 41 to 47) Study the table carefully equivalent to sales tax. The price is

TS
and answer the given question. decreased by (nearest value)—
(A) ` 615 (B) ` 650
Publishing Number of Ratio of Percentage Number of (C) ` 560 (D) ` 580
House books Academic and of books distributors in (E) ` 680
published Non-academic distributed publishing

H
books house 47. Mr. Phanse invests an amount of ` 24,200 at
M 28200 7 : 3 81 17 the rate of 4 p.c.p.a for 6 years to obtain a

G
simple interest later he invests the principal
N 32200 5 : 9 74 23 amount as well as the amount obtained as
O 29700 6 : 5 92 18 simple interest for another 4 years at the
P
Q
31200
33800
8 : 5
7 : 6
86
79
24
25 U same rate of interest. What amount of simple
interest will be obtained at the end of the last
O
4 years?
R 35700 11 : 6 82 21 (A) ` 4800 (B) ` 4850.32
S 37800 5 : 13 89 24 (C) ` 4801.28 (D) ` 4700
H
(E) ` 4870.32
41. What is the difference between the number Directions—(Q. 48 to 50) The questions are based
_T

of academic books published by publishing on the following information—


house M and P? There are three different cable channels namely
(A) 450 (B) 640 ahead, luck and bang. In a survey, it was found
(C) 540 (D) 504 that 85% of viewers respond to bang, 20% to luck
C

(E) None of these and 30% to ahead. 20% of viewers respond to


42. How many books were given to each exactly two channels and 5% to none.
PS

distributor by publisher Q if each distributor 48. What percentage of the viewers responded
gets equal number of books? to all three?
(A) 1806 (B) 1068 (A) 10 (B) 12
(C) 1608 (D) 1308
U

(C) 14 (D) 16
(E) None of these (E) 11
43. What is the average number of non- 49. Assuming 20% respond to ahead and bang,
@

academic books published by publishers R and 16% respond to bang and luck. What is
and S? the percentage of viewers who watch only
(A) 18750 (B) 18850 luck?
(C) 19950 (D) 18950 (A) 20 (B) 0
(E) 19990 (C) 16 (D) 18
44. If the total number of books published by P, (E) 14
Q and R is increased by 30% and the total 50. A milkman mixes 20 L of water with 80 L of
number of books published by remaining milk. After selling one-fourth of this mixture,
publishers be decreased by 20%, what will he adds water to replenish the quantity that
be the new average of books published by he has sold. What is the current proportion
all the publishers? of water to milk?
(A) 33418 (B) 33318 (A) 2 : 3 (B) 1 : 2
(C) 32518 (D) 33618 (C) 1 : 3 (D) 2 : 1
(E) None of these
(E) 3 : 4
Solved Test Papers of Various Examinations ✫ 825

Answers and Explanatory Notes


1. Ans. (C) Data in statement I alone or statement
Sol. I. Let the radius of circle = r II alone is sufficient to find the rate
Perimeter (Circumference) of a of interest.
circle = 2πr 3. Ans. (E)
2πr = 88 Sol. I. No. of rows = x
88 No. of columns = n + 4
r = II. No. of columns = 20

No. of rows = 20 – 4

TS
44
r = = 16
π No. of trees planted in the field
44 × 7 = area of the rectangular field
=
= 20 × 16

H
22
= 2 × 7 = 14 cm = 320
Area of circle = πr2 4. Ans. (E)

G
22 Sol. I. Let the speed of man = x km/hr
= × 14 × 14
7 Speed of current = y km/hr
= 616 sq cm.
U x + y = 1
12
9
O
x + y = 6
II. d (2r) = 28 cm
9
II. x – y =
H
28 3
r = x – y = 3
2
From I and II
_T

r = 14 cm
2x = 9
Area of circle = πr2
22 9
× 14 × 14 x =
= 2
7 9
C

= 616 sq cm. – y = 3
2
Hence, statement I alone or statement 9
PS

y = – 3
II alone is sufficient to find the area 2
of the circle. 3
2. Ans. (C) y =
2
U

n
R y = 1.5 km/hr
Sol. I. C.I. – S.I. = P
100 Both I and II are required to find the
@

2
R speed of the current.
250 = 25000 5. Ans. (E)
100
Sol. I. Let total marks = x
25R 2 25
250 = Marks scored by Raman = x
10 100
2500 = 25R2 Sunil scored 288 marks which is
R2 = 100 128 more than Raman
R = 10% 25
Hence 288 – x = 128
P×N×R 100
II. S.I. = x
100 288 – = 128
4
P × 10 × R 1152 – x = 512
P =
100 x = 1152 – 512
R = 10% x = 640
826 ✫ Quantitative Aptitude

25 i.e., Vinay < Raju < Pratap < Suraj


Marks scored by Raman = × 640
100 < Arun
= 160 Hence, second highest weight is of
Suraj.
II. Raman scored 64 marks less than II. The given information is
the minimum passing marks incomplete to find the weight of
i.e. Passing marks = 160 + 64 Pratap.
= 224 8. Ans. (D)
Minimum pass percentage Sol. From I. total marks of Subodh in
224 History, Geography and Chemistry
× 100 = 75 × 3

TS
=
640 = 225 .... (i)
2240 From II total marks in History,
= Geography and Physics
64
= 35 = 78 × 3

H
Both I and II and required to find the = 234 ... (ii)
Subtracting (i) from (ii)
pass percentage.

G
Physics – Chemistry = 234 – 225 = 9
6. Ans. (A) Information is insufficient to find the
Sol. I. 4x + 3y + 5z = 60 ... (i) marks obtained by Subodh in
2x = y and 2y = z
Substituting the values of x and z in
U Physics.
9. Ans. (C)
O
equation (i) Sol. Let the males = 27x
2y + 3y + 10y = 60 Females = 23x
15y = 60
H
27x – 23x = 100000
y = 4 4x = 100000
x = 2 and z = 8 100000
_T

x2 + y + z x =
4
= 22 + 4 + 8
= 25000
= 16
No. of males = 27 × 25000
II. 3x + 3y + 2z = 34
C

No. of Females = 23 × 25000


2x + 5y + 6z = 72 Total population
Data in statement II alone is not
PS

= 27 × 25000 + 23 × 25000
sufficient to answer the question. = 25000 (27 + 23)
Three equations are required to find = 25000 × 50
the values of x, y and z. = 1250000
U

7. Ans. (A) Statement I is sufficient to find the


Sol. I. Total weight of Arun, Suraj and population of city A.
Vinay II. Let the Population of city B = x
@

= 68 × 3 80
= 204 kg Population of city A = x ×
100
Total weight of Raju and Pratap 80 x
= 72 × 2 =
100
= 144 kg Difference in population of city A
Suraj = 78 kg and city B
Vinay = 46 kg
∴ Arun = 204 – (78 + 46) 80 x
x – = 312500
= 204 – (124) 100
= 80 kg 20x = 31250000
Pratap = 144 – 68 3125000
= 76 kg x =
2
Increasing order of weights = 1562500
46 < 68 < 76 < 78 < 80
Solved Test Papers of Various Examinations ✫ 827

80 × 1562500 Income of A in 2012


Population of city A =
100 3 x × 30 + 3 x
= 80 × 15625 =
100
= 12,50,000
Statement II alone is also sufficient to Income of B in 2012
find the population of city A. 4x × 40 + 4x
=
10. Ans. (E) 100
Sol. I. Let Students who participated in 3 x × 30
elocution = x Income of A in 2012 + 3x
= 100

TS
Students who participated in dancing Income of B in 2012 4 x × 40 + 4 x
150 x 100
= x+
100
90 x
II. No. of students who participated + 3x

H
in dancing = 150. = 100
160 x
150 x + 4x

G
i.e. x+ = 150 100
100 90 x + 300 x
100x + 150x = 15000
250x = 15000
15000 U = 100
160 x + 400 x
100
O
x =
250 390 x 39
x = 60 = =
H
560 x 56
Both statements I and II are required
to find the no. of students in 13. Ans. (D)
_T

elocution. Sol. Income for each year or expenditure


11. Ans. (B) for each year must be known in order
Sol. I. Income of company B in 2009 to calculate the total income of the
C

12 × 35 company A in all the years together.


= + 12 14. Ans. (B)
100 Sol. Let income of company A be xAI and
PS

Income of company B in 2010 income of company B be xBI.


14.5 × 50 Expenditure of company A be xAE
= + 14.5 and expenditure of company B be xBE
100
U

Total income of company B in 2009 x


x
A E × 40 x x
B E × 45
AI + xBI = + AE +
and 2010 100 100
@

× xBE
12 × 35 14.5 × 50
= + 12 + + 14.5 140 x A E + 145 x B E
100 100 5.7 =
100
420 725
= + 12 + + 14.5 570 = 285xAE [xAE = xBE]
100 100 [Expenditure of company A in 2013
= 4.2 + 12 + 7.25 + 14.5 = Expenditure of company B in 2013]
= 16.2 + 21.75 x
570
AE = = 2 lakhs
= 37.95 Lakhs. 285
15. Ans. (C)
12. Ans. (E) Sol. Ix = Income of company B in 2010
Sol. Let the ratio of expenditures of Iy = Income of company B in 2011
company A and B in 2012 be 3x and Ex = Expenditure of company B in
4x respectively 2010
828 ✫ Quantitative Aptitude

Ey = Expenditure of company B in ∴ No. of boys enrolled in singing


2011 = 630 – 490
In 2010 = 140 ... (i)
Ex × 50 No of students enrolled in craft
Ix = + Ex
100 25
In 2011 = × 3000
100
Ey × 45 = 750
Iy = + Ey
100 No. of girls enrolled in craft
Ex × 50 + 100Ex 22

TS
= × 1750
Ix 100 100
=
Iy Ey + 45 + 100Ey = 385
100 ∴ No. of boys enrolled in craft

H
Ix 150Ex = 750 – 385
i.e. = = 365 ... (ii)
Iy 145Ey

G
Total no. of boys enrolled in singing
2 150 Ex and craft
i.e. = = (i) + (ii)
3 145 Ey

i.e.
Ex
=
2 × 145
U = 140 + 365
= 505
O
Ey 3 × 150 18. Ans. (D)
Ex 290 Sol. No. of students enrolled in swimming
i.e. =
H
Ey 450 16
29 = × 3000
Ex 100
i.e. = = 480
_T

Ey 45
No. of girls enrolled in swimming
i.e. Ex : Ey = 29 : 45
16. Ans. (C) 14
× 1750
Sol. Total no. of girls = 1750
C

100
No. of girls in dancing = 20% = 245
∴ No. of boys enrolled in swimming
PS

20
= × 1750
100 = 480 – 245
= 350 = 235
% of girls enrolled in dancing to the Ratio of no. of girls enrolled in
U

total number of students enrolled in swimming to the no. of boys enrolled


school is— in swimming
@

350 35 245
× 100 = = 11.67 =
3000 3 235
17. Ans. (C)
Sol. No. of students enrolled in singing 49
=
47
21
= × 3000 i.e. 49 : 47
100 19. Ans. (B)
= 630 Sol. No. of girls enrolled in swimming
No of girls enrolled in singing 14
= × 1750 = 245 ... (i)
100
28 No. of girls enrolled in drawing
= × 1750
100
16
= 490 × 1750 = 280 ... (ii)
100
Solved Test Papers of Various Examinations ✫ 829

∴ Total no. of girls enrolled in x(2x + 3) + (2x + 3) = 0


swimming and drawing (2x + 3) (x + 1) = 0
245 + 280 = 525 2x + 3 = 0, (x + 1) = 0
20. Ans. (E) 3
x = − , x = –1
Sol. Total no. of students = 3000 2
No. of girls = 1750 (II) y2 + 9y + 14 = 0
No. of boys = 3000 – 1750 y2 + 7y + 2y + 14 = 0
= 1250 y(y + 7) +2 (y + 7) = 0
(y + 7)(y + 2) = 0
1250
% of boys in school = × 100 y + 7 = 0, y + 2 = 0

TS
3000 y = –7, y = –2
125 Values of x are greater than values
=
3 of y.
= 41.67 ~ 42% 24. Ans. (C)

H
21. Ans. (D) Sol. I. 88x2 – 19x + 1 = 0
Sol. I. 6x2 + 5x + 1 = 0 ... (i) 88x2 – 11x – 8x + 1 = 0

G
II. 15y2 + 8y + 1 = 0 ... (ii) 11x(8x – 1) – (8x – 1) = 0
(I) 6x2 + 5x + 1 = 0 (8x – 1) (11x – 1) = 0
8x – 1 = 0, 11x – 1 = 0
6x2 + 3x + 2x + 1 = 0
3x(2x + 1) + (2x + 1) = 0
(2x + 1) (3x + 1) = 0 U x =
1
8
, x =
1
11
O
2
2x + 1 = 0, 3x + 1 = 0 (II) 132y – 23y + 1 = 0
1 1 132y2 – 12y – 11y + 1 = 0
x = − x = −
H
2 3 12y(11y – 1) – (11y – 1) = 0
2
(II) 15y + 8y + 1 = 0 (11y – 1) (12y – 1) = 0
15y2 + 5y + 3y + 1 = 0 11y – 1 = 0, 12y – 1 = 0
_T

5y(3y + 1) + (3y + 1) = 0 1 1
y = , y =
(5y + 1)(3y + 1) = 0 11 12
(5y + 1) = 0, (3y + 1) = 0 Values of x are equal to or greater
C

1 1 than values of y.
y = − , y = − 25. Ans. (A)
5 3
Sol. I. 6x2 – 7x + 2 = 0
PS

The values of y are greater and equal


to the values of x. 6x2 – 4x – 3x + 2 = 0
22. Ans. (E) 2x(3x – 2) – (3x – 2) = 0
Sol. I. x2 + 5x + 6 = 0 (3x – 2) (2x – 1) = 0
U

x2 + 3x + 2x + 6 = 0 3x – 2 = 0, 2x – 1 = 0
x(x + 3) + 2 (x + 3) = 0 2 1
x = , x =
@

(x + 2) (x + 3) = 0 3 2
(x + 2) = 0, (x + 3) = 0 (II) 20y2 – 31y + 12 = 0
x = –2 x = –3 20y2 – 16y – 15y + 12 = 0
(II) 4y2 + 24y + 35 = 0 4y(5y – 4) –3 (5y – 4) = 0
4y2 + 14y + 10y + 35 = 0 (4y – 3) (5y – 4) = 0
2y(2y + 7) + 5 (2y + 7) = 0 4y – 3 = 0, 5y – 4 = 0
(2y + 5) (2y + 7) = 0 3 4
y = + , y =
2y + 5 = 0, 2y + 7 = 0 4 5
5 7 The values of x are less than values
y = − , y = − of y.
2 2
Values of x are greater or smaller 26. Ans. (E)
than values of y Sol. I. 6x2 + 23x + 20 = 0
23. Ans. (B) 6x2 + 15x + 8x + 20 = 0
Sol. I. 2x2 + 5x + 3 = 0 3x (2x + 5) +4 (2x + 5) = 0
2x2 + 3x + 2x + 3 = 0 (2x + 5) (3x + 4) = 0
830 ✫ Quantitative Aptitude

2x + 5 = 0, 3x + 4 = 0 y (y – 3) –2(y – 3) = 0
5 4 (y – 2) (y – 3) = 0
x = − , x = −
2 3 y – 2 = 0, y – 3 = 0
(II) 6y2 + 31y + 35 = 0 y = 2, y = 3
6y2 + 21y + 10y + 35 = 0 The values of x are greater than
3y (2y + 7) +5 (2y + 7) = 0 values of y.
(3y + 5) (2y + 7) = 0 30. Ans. (C)
3y + 5 = 0, 2y + 7 = 0 Sol. I. 38x2 – 3x – 11 = 0
5 7 38x2 – 22x + 19x – 11 = 0
y = − , y = −
3 2 2x (19x – 11) + (19x – 11) = 0

TS
One of the values of y is less than (19x – 11) (2x + 1) = 0
and more than the values of x. 19x – 11 = 0, 2x + 1 = 0
27. Ans. (D) 11 1
Sol. I. x2 = 81 x = − , x = −

H
19 2
x = 81 (II) 28y2 + 32y + 9 = 0
x = + 9, –9 28y2 + 18y + 14y + 9 = 0

G
(II) y2 – 18y + 81 = 0 2y(14y + 9) + (14y + 9) = 0
y2 – 9y – 9y + 81 = 0 (14y + 9) (2y + 1) = 0
y(y – 9) –9 (y – 9) = 0
(y – 9) (y – 9) = 0
U 14y + 9 = 0, 2y + 1 = 0
y = −
9
, y = −
1
O
y – 9 = 0, y – 9 = 0
14 2
y = 9 Values of x are greater than or equal
Values of y are more than or equal to values of y.
H
to values of x. 31. Ans. (C)
28. Ans. (C) Sol. Speed of P = 3 km/hr
_T

Sol. I. 4x2 + 20x + 21 = 0 Time = 4 hr


4x2 + 14x + 6x + 21 = 0 Distance = 4 × 3 = 12 km
2x(2x + 7) +2 (2x + 7) = 0
1
(2x + 7) (2x + 2) = 0
C

Speed of Q = 3 km/hr
(2x + 7) = 0, (2x + 2) = 0 2
7 7
PS

x = − x = –1 = km/hr
2 2
(II) 2y2 + 17y + 35 = 0 Time = 4 hr
2y2 + 10y + 7y + 35 = 0 7
Distance = 4 × = 14 km
U

2y(y + 5) +7 (y + 5) = 0
2
(y + 5) (2y + 7) = 0
Distance between them after 4 hr
y + 5 = 0, 2y + 7 = 0
@

7 = 14 – 12 = 2 km
y = –5, y = − 32. Ans. (D)
2
Values of x are more than or equal Sol. 14 × 627 ÷ 1089 = (?)3 + 141
to values of y.
29. Ans. (B) 14 × 627 ÷ 33 = (?)3 + 141
14 × 19 = (?)3 + 141
Sol. I. x2 – 14x + 48 = 0
266 = (?)3 + 141
x2 – 8x – 6x + 48 = 0
(?)3 = 266 – 141
x(x – 8) –6 (x – 8) = 0 (?)3 = 125 1
(x – 6) (x – 8) = 0 (?) = (125) 3
x – 6 = 0, x – 8 = 0
x = 6, x = 8 (?) = 5
(II) y2 + 6 = 5y 33. Ans. (E)
1
y2 – 5y + 6 = 0 21.5 21 13.5 (?) 3 17
Sol. + – = +
y2 – 3y – 2y + 6 = 0 5 6 15 4 30
Solved Test Papers of Various Examinations ✫ 831

37. Ans. (E)


1
129 + 105 − 27 (?) 3 17 Sol. 7
= + 7 + 32 = 16
30 4 30
16 + 53 = 141
1
207 17 (?) 3 141 + 72 = 190
– = 190 + 93 = 919
30 30 4
919 + 112 = 1040
1
190 (?) 3 The series is
= 16, 141, 190, 919, 1040
30 4
38. Ans. (C)

TS
1
19 (?) 3 Sol. 12
= 12 + 5 = 17
3 4
17 + 15 = 32
19 × 4

H
1 32 + 25 = 57
(?) 3 =
3 57 + 35 = 92
76 92 + 45 = 137

G
1
(?) 3 = The series is
3
3 12, 17, 32, 57, 92, 137
(?) =
76
3
U 39. Ans. (D)
Sol. 19
O
(?) = (25.33)3 19 + 22 + 2 = 25
34. Ans. (C) 25 + 42 + 4 = 45

( ) 45 + 62 + 6 = 87
2
H
Sol. 7 + 11 1
= (?) 3 + 2 847 + 122 87 + 82 + 8 = 159
7 + 22 7 + 121 – (?) 3 + 2 847 + 122
1 159 + 102 + 10 = 269
_T

The series is
1
128 + 22 7 = (?) 3 + 2 847 + 122 19, 25, 45, 87,159, 269
1 40. Ans. (E)
6 + 22 7 = (?) 3 + 2 847
Sol. 83
C

1
6 + 22 7 = (?) 3 + 2 × 11 7 124 = 83 + 41
1 206 = 124 + 82
PS

6 + 22 7 = (?) 3 + 22 7
1 370 = 206 + 164
6 = (?) 3
1 698 = 370 + 328
(?) 3 = 6 1354 = 698 + 656
(?) = (6)3
U

41. Ans. (C)


(?) = 216 Sol. Let no. of academic and non-academic
35. Ans. (C) books published by M = 7x and 3x
@

2
18 455 61 respectively
Sol. × ÷ =? Total books published by M = 28200
4 19 799
2
799 ∴ 7x + 3x = 28200
18 455
× × 10x = 28200
4 19 61 x = 2820
324 455 799 No. of academic books published by
× ×
16 19 61 M = 2820 × 7 = 19740 ...(i)
20.25 × 23.95 × 13.098 Let no. of academic and non-academic
6351.82 books published by P = 8x and 5x
36. Ans. (C) respectively
Sol. 28, 28 + (10 × 1) + 12, 39 + (10 × 2)+22, Total books published by P = 31200
63 + (10 × 3) + 32 ∴ 8x + 5x = 31200
102 + (10 × 4) + 42, 158 + (10 × 5) + 52 13x = 31200
i.e., 28, 39, 63, 102, 158, 233 x = 2400
832 ✫ Quantitative Aptitude

No. of academic books published by ∴ No. of non-academic books


P = 8 × 2400 = 19200 ... (ii) published by S
Difference of academic books = 13x
published by M and P = 13 × 2100
= (i) – (ii) = 27300 ... (ii)
= 19740 – 19200
Average no. of non-academic books
= 540
42. Ans. (B) published by R and S
Sol. Percentage of books distributed by (i) + (ii)
=
Q = 79 2

TS
Total books published by Q =33800 (12600 + 27300)
=
No. of books distributed by Q 2
39900
79 = = 19950
2

H
= × 33800
100 44. Ans. (B)
= 79 × 338 Sol. Total no. of books published by P,

G
= 26702 Q and R is increased by 30%
No. of distributors in publishing 30
i.e. (31200 + 33800 + 35700) ×

house Q = 25
No. of books given to each distributor
U =
100700 × 30
100
O
26702 100
= = 1007 × 30
25
= 30210
H
= 1068
43. Ans. (C) Hence, the new total no. of books
Sol. Let the no. of academic and non- published by P, Q and R
_T

academic books published by R = = 100700 + 30210


11x and 6x respectively. = 130910 (i)
Total no. of books published by Total no. of books published by M,
C

R = 35700 N, O, S is decreased by 20%


∴ 11x + 6x = 35700 i.e. (28200 + 32200 + 29700
PS

17x = 35700 20
+ 37800) ×
35700 100
x = 20
17 = 127900 ×
100
U

= 2100 = 12790 × 2
∴ No. of non-academic books = 25580
@

published by R Hence, the new total no. of books


= 6x = 6 × 2100 published by M, N, O and S is
= 12600 ... (i) 127900 – 25580
Let the no. of academic and non- = 102320 ... (ii)
academic books published by S = 5x Hence, the new total no. of books
and 13x respectively. published by
Total no. of books published by M, N, O, P, Q, R and S
S = 37800 = (i) + (ii)
5x + 13x = 37800 = 130910 + 102320
18x = 37800 = 233230
37800 Hence, new average of books
x = published by all the publishers
18
= 2100 233230
= = 33318
7
Solved Test Papers of Various Examinations ✫ 833

45. Ans. (D) (4) + (5) + (6) + (7) = 30 ... (iii)


Sol. Total no. of books distributed by (1) + (2) + (3) + (4) + (5)
publisher + (6) + (7) = 100 – 5 = 95 ... (iv)
92 (2) + (4) + (6) = 20 ... (v)
= × 29700
100
= 92 × 297 = 27324 ...(i)
Total no. of books distributed by 1 2 3
publisher Q
79 5

TS
= × 33800
100 4 6
= 79 × 338 = 26702 ... (ii)
Hence, total no. of books distributed 7
by publishers O and Q

H
= (i) + (ii) = 27324 + 26702
= 54026 (i) + (ii) + (iii) – (iv) – (v) gives

G
46. Ans. (C) 2(5) = 135 – 95 – 20 = 20
Sol. Let CP of sari = x (5) = 10

x +
12 x
100
= 5225
U 49. Ans. (B)
Sol. Given (4) + (5) = 20
O
112x = 522500 (2) + (5) = 16
522500 (5) = 10
x =
H
112 Thus (4) = 20 – 10 = 10
= 4665 (2) = 16 – 10 = 6
_T

Sales tax = discount = 5225 – 4665 Using equation (v) we get


= 560 (6) = 4 (20 – 10 – 6 = 4)
47. Ans. (C)
Using equation (ii)
24200 × 6 × 4
C

Sol. S.I. = (3) = 20 – (6 + 4 + 10) = 0


100 50. Ans. (A)
= 242 × 24
PS

Sol.
= 5808
MILK WATER TOTAL
Total Amount = 24200 + 5808
(i) 80 L 20 L 100L
= 30008
U

80 20 1
30008 × 4 × 4 (ii)
100
× 25 = 20
100
× 25 = 5 25L SOLD: × 100 = 25L
S.I. = 4
100
@

30008 × 16 (iii)
80
× 75 = 60
20
× 75 = 15 75L Remaining:
3
× 100 = 75L
= 100 100 4
100
(iv) 60 25 L+15 L= 40L 100L
= ` 4801.28
48. Ans. (A) Ratio of water to milk = 40 : 60
Sol. (1) + (2) + (4) + (5) = 85 ... (i) = 2 : 3
(2) + (3) + (5) + (6) = 20 ... (ii)
834 ✫ Quantitative Aptitude

IBPS PO (PT)
Examination, 2018
Directions (Q. 1-5): Study the data carefully and (1) 2 : 1 (2) 1:2
answer the fallowing questions. (3) 35 : 66 (4) 11 : 10
The chart shows the number of labourers (Men (5) None of the given options
and Women) working in six different years. 5. The total number of men working in all six
years is how much more/less than the total
number of women working in all six years

TS
together?
(1) None of the given options
(2) 140

H
(3) 160
(4) 180
(5) 200

G
Directions (Q. 6-15) In each of the following
number series only one number is wrong. Find

U out the wrong number in the given number series.


6. 4 5.1 7.3 10.6 15 20 27.1
O
(1) 5.1 (2) 4
(3) 7.3 (4) 20
H
1. The total number of men working in 2012
(5) 27.1
and 2013 together is what per cent of the total
number of labourers (men + women) 7. 2 3 8 31 154 924 6460
_T

working in 2014? (1) 924 (2) 6460


(1) 60% (2) 70% (3) 154 (4) 8
(3) 80% (4) 90% (5) 31
C

(5) 40% 8. 251 252 254 227 243 118 154


2. The average number of women working in (1) 251 (2) 252
PS

2014, 2015 and 2016 together is how much (3) 227 (4) 243
more/less than the average number of men (5) 154
working in 2011, 2014 and 2016 together? 9. 141 156 147 162 153 165 159
(1) 100 (2) 80
U

(1) 156 (2) 153


(3) 90 (4) 70
(3) 147 (4) 165
(5) None of the given options
@

(5) 159
3. The number of men working in 2017 is 15%
10. 2 6 10 19 36 69 134
more than that in 2015 while the number of
(1) 134 (2) 69
women working in 2017 is 40% less than that
in 2014. Find the total number of labourers (3) 6 (4) 2
(men + women) working in 2017. (5) 10
(1) 561 (2) 456 11. 0.5 2 14 32 512 16384
(3) 489 (4) 594 (1) 1 (2) 2
(5) 630 (3) 4 (4) 32
4. Find the ratio of the total number of (5) 512
labourers (men % women) working in 2012 12. The ratio of the present age of A to that of B
and 2013 together to the total number of is 16 : 7. After 12 years, A’s age is twice of B’s
labourers (men + women) working in 2015 age. Then find the present ages of A and B
and 2016 together. respectively.
Solved Test Papers of Various Examinations ✫ 835

(1) 6 years, 28 years (3) None of those given as options


(2) 80 years, 35 years (4) 0%
(3) None of those given as options (5) 150%
(4) 96 years, 42 years 17. Find the ratio between the total number of
(5) 102 years, 49 years persons who use their coupons in pedicure
13. A man invested a certain sum in scheme A at to the total number of persons who use their
15% pa for 2 years and earned ` 1950 as coupons in hair cutting?
simple interest. He increased his sum by ` X (1) 52 : 23
and invested in another scheme B at 10% pa (2) None of those given as options
(3) 8 : 9

TS
CI for 2 years and received ` 1680 as
compound interest. Find the value of ‘X’. (4) 8 : 7
(1) ` 1750 (2) ` 1500 (5) 7 : 8
(3) ` 1250 (4) None of these 18. Females who use their coupon in hair cutting

H
(5) ` 1850 are how much per cent more than females
14. In a class, there are 30 girls and 15 boys. The who use their coupon in pedicure?

G
7 (1) 15
total average weight of the class is 47 kg. (2) 45
15 (3) 30
The total average weight of boys is 58 kg.
Find the approximate average weight of the
girls. U (4) None of those given as options
(5) 60
O
(1) 32 kg (2) 42 kg 19. Out of the males who use their coupons in
(3) 52 kg (4) 35 kg hair cutting, 25% belong to city A. Then find
H
(5) 50 kg the number of males who use their coupons
in hair cutting and do not belong to city A.
15. Ram bought a bike at 20% discount on MRP. (1) None of those given as options
_T

After one year, Ram sold the bike to Ramesh (2) 108
at 10% loss. After one more year, Ramesh (3) 126
sold the bike at 20% profit to Ranjan. If Ranjan (4) 117
paid ` 1,29,600, then find the MRP of the bike.
C

(5) 135
(1) ` 1,50,000 (2) ` 2,25,000
(3) ` 1,40,000 (4) ` 2,00,000 20. The ratio of males who use their coupon in
PS

(5) ` 1,80,000 pedicure to those who use it in spa is 4 : 5,


while the ratio of females who use their
Directions (Q. 16-20): Study the following coupon in hair cutting to those who use it in
information carefully and answer the questions spa is 6 : 11. Find the total number of people
U

given below: who use their coupons in spa.


There are 450 coupons which can be used (1) 349
@

in pedicure and hair cutting. The ratio of males (2) 481


to females who use their coupons in hair cutting (3) 300
is 13 : 7. The number of males who use their (4) 440
coupons in pedicure is 72 more than the number (5) None of those given as options
of females who use their coupon in hair cutting. Directions (Q. 21-30): In each of these questions,
Total number of males who use their coupon two equations (I) and (II) are given. You have to
in pedicure and hair cutting together is 174 solve both the equations and give answer
more than the total number of females who use accordingly.
their coupon in pedicure and hair cutting (1) x > y
together. (2) x ≥ y
16. Males who use their coupon in pedicure are (3) x ≤ y
what percent of the males who use their (4) x = y or relationship can’t be
coupons in hair cutting? established
(1) 200% (2) 100% (5) x < y
836 ✫ Quantitative Aptitude

21. I. 2x2 + 9x + 9 = 0 The bar graph shows the number of plain


II. 15y2 + 16y + 4 = 0 books and lined books (in hundreds), available
at three different stores.
22. I. 2x3 = 256
II. 2y2 – 9y+ 10 = 0 Plain Book Lined Book

23. I. 2
6x – 11x + 4 = 0 35

Books (in hundreds)


II. 3y2 – 5y + 2 = 0 30
24. I. 2
3x + 11x + 10 = 0 25
II. 2y2 + 11y + 14 = 0 20

TS
2 15
25. I. 12x + 11x + 2 = 0
10
II. 12y2 + 7y + 1 = 0
5
26. I. 21x2 + 10x + 1 = 0
0
24y2 + 26y + 5 = 0

H
II. Store A Store B Store C

27. ‘A’ can complete a work in 20 days while B Plain Book 20 30 25

G
is 25% more efficient than A. B worked for 6 Lined Book 15 20 20
days and left. The remaining work is The table shows the percentage of the total
completed by C in 15 days. Find out in how
many days C can complete the whole work
alone. U books (Plain + Lined) that was sold by different
stores.
Store Percentage of sold books
O
(1) 27 days (2) 21 days
A 20%
(3) 18 days (4) 24 days
B 40%
H
(5) 30 days
28. A man travels from Point P to Q at 90 km/hr C 30%
_T

and from Q to R at 60 km/hr. The total 31. The number of plain books sold by Store A
distance between P to R is 200 km. If his and Store B was 30% and 40% respectively.
average speed is 75 km/hr then find the Then the number of lined books sold by store
distance between P and Q. A and store B together is what per cent of
C

(1) 80 km (2) 120 km the total books available at store A?


(3) 100 km (4) 150 km 6 4
PS

(5) None of those given as options (1) 22 % (2) 23 %


7 7
29. A mixture contains wine and water in the 5
ratio 5 : 1. On adding 5 litres of water, the (3) 25 % (4) 25%
7
U

ratio of wine to water becomes 5 : 2. The (5) None of these


quantity of wine in the mixture is:
(1) 20 litres (2) 22 litres 32. The average of the total books sold by stores
@

(3) 24 litres (4) 26 litres B and C together is how much more than the
total unsold books of store A?
(5) None of these
(1) 1125 (2) 1075
30. The average salary of the entire staff in an (3) 1055 (4) 1175
office is ` 3200 per month. The average salary (5) 1225
of the officers is ` 6800 and that of the non- 33. The ratio of plain to lined books sold for
officers is ` 2000. If the number of officers is store C is 5 : 4 and for store B is 3 : 2. Then
5, then find out the number of non-officers find out the total plain books sold by these
in the office. two stores together.
(1) 8 (2) 12 (1) 1750 (2) 1825
(3) 15 (4) 5 (3) 1850 (4) 1950
(5) None of these (5) 1975
Directions (Q. 31-35): Study the following bar 34. The unsold books of store A is
graph and table and answer the given questions. approximately what per cent more or less
Solved Test Papers of Various Examinations ✫ 837

than the total unsold books of stores B and respectively. Then, find out the total amount
C together? earned by store B on selling these books if
(1) 48% (2) 54% out of the total sold books 60% are lined
(3) 59% (4) 52% books.
(5) 57% (1) ` 2.5 lakh (2) ` 3.6 lakh
35. Selling price of each Plain book and Lined (3) ` 3.5 lakh (4) ` 3.8 lakh
book sold by store B is ` 250 and ` 175 (5) ` 4.1 1akh

Answers and Explanatory Notes

TS
1. Ans. (4) Number of men working in 2011
Sol. Number of men working in 2012 = 80
= 120 Number of men working in 2014

H
Number of men working in 2013 = 160
= 240 Number of men working in 2016
Total number of men working in

G
= 360
2012 and 2013
Total number of men working in
= 120 + 240 = 360
2011, 2014 and 2016
Number of men working in 2014
= 160
Number of women working in 2014 U = 80 + 160 + 360 = 600
The average number of men working
O
= 240 in 2011, 2014 and 2016
Total number of labourers (men + Total number of men
H
women) working in 2014
= 160 + 240 = 400 = working in 2011, 2014 and 2016
% of total number of men working Total number of years
_T

in 2012 and 2013 to the total number 600


= = 200
of labourers (men + women) working 3
in 2014 Difference between average number
C

of women working in 2014, 2015 and


360
= × 100 2016 and average number of men
400 working in 2011, 2014 and 2016
PS

360 = 300 – 200 = 100


= = 90
4 The average number of women
2. Ans. (1) working in 2014, 2015 and 2016 is 100
U

Sol. Number of women working in 2014 more than the average number of
= 240 men working in 2011, 2014 and 2016.
Number of women working in 2015
@

3. Ans. (3)
= 360
Sol. Number of men working in 2015
Number of women working in 2016
= 300
= 300
Total number of women working in Number of men working in 2017 is
2014, 2015 and 2016 15% more than that in 2015
= 240 + 360 + 300 = 900 15
The average number of women = 300 + × 300
100
working in 2014, 2015 and 2016
= 300 + (15 × 3)
Total number of women working
= 300 + 45 = 345
in 2014, 2015 and 2016 Number of women working in 2014
=
Total number of years = 240
900 Number of women working in 2017
= = 300
3 is 40% less than that in 2014
838 ✫ Quantitative Aptitude

40 number of labourers working in 2015


= 240 – × 240 and 2016
100
= 240 – (4 × 24) Total number of labourers
= 240 – (96) = 144 working in 2012 and 2013
=
Total number of labourers (men + Total number of labourers
women) working in 2017 = Number working in 2015 and 2016
of men working in 2017 + Number
660 1
of woman working in 2017 = = i.e. (1 : 2)
= 345 + 144 = 489 1320 2
5. Ans. (5)

TS
4. Ans. (2)
Sol. Number of men working in 2011
Sol. Number of men working in 2012
= 80
= 120
Number of men working in 2012
Number of men working in 2013
= 120

H
= 240
Total number of men working in Number of men working in 2013
= 240

G
2012 and 2013
= 120 + 240 = 360 Number of men working in 2014
Number of women working in 2012 = 160
= 180
Number of women working in 2013
U Number of men working in 2015
= 300
Number of men working in 2016
O
= 120
Total number of women working in = 360
2012 and 2013 Total number of men working in all
H
= 180 + 120 = 300 six years
Total number of labourers working = 80 + 120 + 240 + 160 + 300 + 360
_T

in 2012 and 2013 = Total number of = 1260


men working in 2012 and 2013. Number of women working in 2011
Total number of women working in = 260
2012 and 2013 Number of women working in 2012
C

= 360 + 300 = 660 = 180


Number of men working in 2015 Number of women working in 2013
PS

= 300 = 120
Number of men working in 2016 Number of women working in 2014
= 360 = 240
Total number of men working in Number of women working in 2015
U

2015 and 2016 = 360


= 300 + 360 = 660 Number of women working in 2016
@

Number of women working in 2015 = 300


= 360 Total number of women working in
Number of women working in 2016 all six years
= 300 = 260 + 180 + 120 + 240 + 360 + 300
Total number of women working in = 1460
2015 and 2016 Difference between the total number
= 360 + 300 = 660 of men working in all six years and
Total number of labourers working the total number of women working
in 2015 and 2016
in all six years
= Total number of men working in
= 1460 – 1260 = 200
2015 and 2016 + Total number of
The total number of men working in
women working in 2015 and 2016
all six years is 200 less than the total
= 660 + 660 = 1320
number of women working in all six
Ratio of the total number of labourers
years.
working in 2012 and 2013 to the total
Solved Test Papers of Various Examinations ✫ 839

6. Ans. (4) Present age of A = 16x = 16 × 6


Sol. The series is 4, 4 + 1.1, 5.1 + 2.2, 7.3 = 96 year
+ 3.3, 10.6 + 4.4, 15 + 5.5, 20.5 + 6.6 Present age of B = 7x = 7 × 6
i.e. 4, 5.1, 7.3, 10.6, 15, 20.5, 27.1 = 42 years
(correct answer is 20.5 instead of 20) Alternate method
7. Ans. (1) Let present age of A = x
Sol. The series is 2, (2 × 2) – 1, (3 × 3) – 1, Present age of B = y
(8 × 4) – 1, (31 × 5) – 1, (154 × 6) – 1, Ratio of the present age of A to that
(923 × 7) – 1 of B is 16 : 7
i.e., 2, 4 – 1, 9 – 1, 32 – 1, 155 – 1, 924 – 1, x 16

TS
6461 – 1 i.e. =
y 7
i.e., 2, 3, 8, 31, 152, 923, 6460
(correct answer is 923 instead of 924) 7x = 16y
8. Ans. (2) 7x – 16y = 0 (i)

H
Sol. The series is 251, 251–13, 250 + 22, After 12 years age of A = x + 12
254 – 33, 227 + 42, 243 – 53, 118 + 62 After 12 years age of B = y + 12
i.e., 251, 251 – 1, 250 + 4, 254 – 27, After 12 years, A’s age is twice of B’s

G
227 + 16, 243 – 125, 118 + 36 age
i.e., 251, 250, 254, 227, 243, 118, 154 i.e. x + 12 = 2 (y + 12)
(correct answer is 250 instead of 252)
9. Ans. (4)
U x + 12 = 2y + 24
x – 2y = 12 ... (ii)
O
Sol. The series is Multiplying equation (ii) by 7 and
141, 141 + 15, 156 – 9, 147 + 15, substracting (ii) from (i)
162 – 9, 153 + 15, 168 – 9
H
{7x – 16y = 0} – {7(x – 2y = 12)}
i.e. 141, 156, 147, 162, 153, 165, 159 {7x – 16y = 0} – {7x – 14y – 84} = 0
(correct answer is 162 instead of 165) 7x – 16y – 7x + 14y + 84 = 0
_T

10. Ans. (3)


–2y + 84 = 0
Sol. The series is
2, 2+(21 + 1), 5+(22 + 1), 10+(23 + 1), –2y = –84
19+(24 + 1), 36+(25+1), 69+(26 + 1) 84
C

y =
i.e. 2, 2+(2 + 1), 5+(4 + 1), 10+(8 + 1), 2
19+(16 + 1), 36 + (32 + 1), 69 + (64 + 1) y = 42
PS

i.e. 2, 2+(3), 5+(5), 10+(9), 19+(17), 7x = 16y


36+(33), 69+(65) 7x = 16 × 42
i.e., 2, 5, 10, 19, 36, 69, 134
(correct answer is 5 instead of 6). 16 × 42
x =
U

11. Ans. (2) 7


Sol. The series is = 96
13. Ans. (2)
@

0.5, 0.5 × 1, 0.5 × 2, 1 × 4, 4 × 8, 32


× 16, 512 × 32 P×R×T
i.e., 0.5, 0.5, 1, 4, 32, 512, 16384 Sol. S.I. =
100
(correct answer is 0.5 instead of 2).
12. Ans. (4) P × 15 × 2
1950 =
Sol. Let present age of A = 16x 100
Present age of B = 7x P × 15 × 2 = 195000
After 12 years A’s age = 16x + 12 195000
After 12 years B’s age = 7x + 12 P = = 6500
30
After 12 years, A’s age is twice of B’s
P = ` 6500
age
i.e., 16x + 12 = 2(7x + 12) A – P = C.I.
2
10
16x + 12 = 14x + 24 (6500 + x) 1 + – (6500 + x)
2x = 12 100
x = 6 = 1680
840 ✫ Quantitative Aptitude

11 × 11 120 6 × 72
(16500 + x) 10 × 10 – (6500 + x) Ramesh SP = × 72 = `
100 5
= 1680
121 Ramesh SP = Ranjan’s CP
(6500 + x) 100 – (6500 + x ) = 1680 6 × 72
When Ranjan’s CP is ,
121 5
(6500 + x) 100 − 1 = 1680 MRP = 100
When Ranjan’s CP is 129600,
21 100 × 5
(6500 + x) = 1680 MRP = × 129600
6 × 72

TS
100
136500 + 21x = 168000 = ` 150000
Alternate method
31500 Ranjan’s CP = 129600
21x = 31500 → x = = 1500

H
21 Ranjan’s CP = Ramesh’s SP
14. Ans. (2) 100
Ramesh’s CP = × 129600

G
Sol. Total weight of class = Average 120
weight of class × No. of students Ram’s SP = Ramesh’s CP
7
= 47
15
× 45 (30 girls + 15 boys)
Total weight of boys = Average U Ram’s CP =
100
90
×
100
120
When Ram’s CP is 80 MRP = 100
× 129600
O
weight of boys × No. of boys
When Ram’s CP is
= 58 × 15
100 100
H
Total weight of girls = Total weight × × 129600
of class – Total weight of boys 90 120
7 100 100 100
_T

= 47 × 45 – 58 × 15 MRP × × × 129600
15 80 90 120
712 = ` 150000
= × 45 – 58 × 15
15
C

= 712 × 3 – 58 × 15 Q. (16-20) Total number of coupons = 450


= 3[712 – 58 × 5] Let Number of males who use their
PS

= 3[712 – 290] coupons in hair cutting = 13x


= 3[422] Number of females who use their
carpons in hair cutting = 7x
Average weight of girls
Given that no. of males who use their
U

Total weight of girls coupon in pedicure = 72 + 7x


=
No. of girls Let number of females who use their
@

3 × 422 422 coupon in pedicure be y


= = = 42.2 ≈ 42 kg It is given that
30 10
15. Ans. (1) 13x + 72 + 7x = 174 + 7x + y
Sol. Let MRP = `100 13x – y = 102 ... (i)
Ram bought a bike at 20% discount Also
on MRP 13x + 7x + 72 + 7x + y = 450
Ram’s CP = `80 27x + y = 378 ... (ii)
Ram sold the bike to Ramesh at 10% Adding equations (i) and (ii)
loss 40x = 480
90 x = 12
Ram’s SP = × 80 = ` 72 Substituting x = 12 in equation (i)
100
Ram’s SP = Ramesh’s CP. 13 × 12 – y = 102
Ramesh sold the bike to Rajan at 20% 156 – y = 102
profit y = 54
Solved Test Papers of Various Examinations ✫ 841

No. of males who use their coupons 1


in hair cutting = × 156 = 39
4
= 13 × 12 = 156
∴ No. of males who use their
No. of females who use their coupons
coupons in hair cutting and do
in hair cutting
not belong to city A
= 7 × 12 = 84
= 156 – 39
No. of males who use their coupons
= 117
in pedicure = 72 + (7 × 12)
20. Ans. (1)
= 156
Sol. No. of males who use their coupons
No. of females who use their coupons

TS
in spa is:
in pedicure = 54
16. Ans. (2) 5
× 156
4
Males who use their coupon = 5 × 39 =195

H
in pedicure No. of females who use their coupons
Sol. × 100
Males who use their coupon in spa is:

G
in hair cutting
11
156 = × 84
= × 100 = 100% 6
17. Ans. (5)
156
U = 11 × 14 = 154
∴ Total no. of people in spa
O
Total no. of persons who use = 195 + 154 = 349
their coupons in pedicure 21. Ans. (5)
Sol.
H
Total no. of persons who use Sol. I. 2x2 + 9x + 9 = 0
their coupons in hair cutting 2x2 + 6x + 3x + 9 = 0
_T

156 + 54 210 2x(x + 3) +3(x + 3) = 0


= = (2x + 3)(x + 3) = 0
156 + 84 240
2x + 3 = 0, x + 3 = 0
21 3
= = 7 : 8 x = − , x = –3
C

24 2
18. Ans. (4) x = –1.5, x = –3
PS

Sol. No. of females who use their coupon II. 15y2 + 16y + 4 = 0
in hair cutting— 15y2 + 10y + 6y + 4 = 0
5y (3y + 2) +2(3y + 2) = 0
No. of females who use their (5y + 2) (3y + 2) = 0
U

coupon in pedicure 5y + 2 = 0, 3y + 2 = 0
× 100
No of females who use their 5y = –2, 3y = –2
@

coupon in pedicure
2 2
84 − 54 y = − , y = −
= × 100 5 3
54
The values of x are less than values
30 of y.
= × 100
54 22. Ans. (3)
500 5 Sol. I. 2x3 =
= = 55 % 256
9 9
19. Ans. (4) 256
x3 =
Sol. No. of males who use their coupons 2
in hair cutting and belong to city A 16
= = 8
25 2
= × 156 1
100 x = (8) 3 = 2
842 ✫ Quantitative Aptitude

II. 2y2 – 9y + 10 = 0 II. 12y2 + 7y + 1 = 0


2y2 – 5y – 4y + 10 = 0 12y2 + 4y + 3y + 1 = 0
y(2y – 5) –2(2y – 5) = 0 4y(3y + 1) + (3y + 1) = 0
(y – 2)(2y – 5) = 0 (4y + 1)(3y + 1) = 0
(y – 2) = 0, (2y – 5) = 0 4y + 1 = 0, (3y + 1) = 0
5 1 1
y = 2, y = y = − , y = −
2 4 3
The value of x is equal to and less Values of x are equal to or less than
than y. values of y.
23. Ans. (4) 26. Ans. (4)

TS
Sol. I. 6x2 – 11x + 4 = 0 Sol. I. 21x2 + 10x + 1 = 0
6x2 – 8x – 3x + 4 = 0 21x2 + 7x + 3x + 1 = 0
7x(3x + 1) + (3x + 1) = 0
2x(3x – 4) – (3x – 4) = 0
(7x + 1)(3x + 1) = 0

H
(3x – 4)(2x – 1) = 0 (7x + 1) = 0, (3x + 1) = 0
3x – 4 = 0, 2x – 1 = 0
1 1
4 1 x = − , x = −

G
x = , x = 7 3
3 2 II. 24y2 + 26y + 5 = 0
II. 3y2 – 5y + 2 = 0 24y2 + 20y + 6y + 5 = 0
3y2 – 3y – 2y + 2
3y(y – 1) –2 (y – 1)
U 4y (6y + 5) + (6y + 5) = 0
(4y + 1) (6y + 5) = 0
O
(y – 1)(3y – 2) = 0 (4y + 1) = 0, (6y + 5) = 0
y – 1 = 0, 3y – 2 = 0 1 5
2 y = − , y = −
H
y = 1, y = 4 6
3 No relationship can be established.
No relationship can be established 1 5
_T

between x and y. x = − , y = −
7 6
24. Ans. (2) 1 1
Sol. I. 3x2 + 11x + 10 = 0 x = − , y = −
3 4
3x2 + 6x + 5x + 10 = 0
C

27. Ans. (4)


3x(x + 2) +5 (x + 2) = 0 Sol. I. B is 25% more efficient than A
(x + 2)(3x + 5) = 0 who can complete the work in 20
PS

(x + 2) = 0, (3x + 5) = 0 days.
5 In one day, B can complete
x = –2, x = –
3 1 1 25
×
U

II. 2y2 + 11y + 14 = 0 +


20 20 100
2y2 + 7y + 4y + 14 = 0 5
y(2y + 7) +2 (2y + 7) = 0 i.e. of the work
@

80
(y + 2)(2y + 7) = 0
(y + 2) = 0, (2y + 7) = 0 In 6 days, B completes
7 5 3
y = –2, y = − 6 × = of the work
2 80 8 3 5
The values of x is equal to or greater Work left = 1 − =
than the values of y. 5 8 8
25. Ans. (3) is completed by C in 15 days
8
Sol. I. 12x2 + 11x + 2 = 0
12x2 + 8x + 3x + 2 = 0 ∴ 1 work is completed in
4x(3x + 2) + (3x + 2) = 0 15 × 8
(3x + 2)(4x + 1) = 0 = 24 days
5
(3x + 2) = 0, (4x + 1) = 0 28. Ans. (2)
2 1 Sol. Let PQ = x km
x = − , x = −
3 4 QR = (200 – x) km
Solved Test Papers of Various Examinations ✫ 843

Speed from point P to Q 31. Ans. (3)


= 90 km/hr Sol. Number of plain books sold by store A
Speed from point Q to R 30
= 60 km/hr = × 20 = 6
x 100
Time taken to travel distance PQ = Number of plain books sold by store B
90 40
Time taken to travel distance QR = × 30 = 12
(200 − x ) 100
= Total number of books sold by store A
60 20
Total distance = × 35 = 7

TS
Average speed = 100
Total time Total number of books sold by store B
200 40
i.e. 75 = = (30 + 20)
x (200 − x ) 100
+

H
90 60 40
= × 50 = 20
x (200 − x ) 100
75 + = 200 No. of lined books sold by store A

G
90 60 = 7 – 6 = 1
2 x + 3 (200 − x ) No. of lined books sold by store B
75
180
= 200

200 × 180 U = 20 – 12 = 8
Total no. of lined books sold by A
and B =1 + 8 = 9
O
{2x + 3(200 – x)} =
75 Total books available at store A = 35
2x + 600 – 3x = 480 900 180 5
9
H
x = 120 km ∴ × 100 = = = 25
29. Ans. (5) 35 35 7 7
Sol. Let wine = 5x litres 32. Ans. (1)
_T

water = x litres Sol. Total books sold by store B


5x 40
Ratio = = (50) = 20
x 100
C

Adding 5 litres of water ratio becomes Total books sold by store C


5x 5 30 27
= = = (45) =
PS

x+5 2 100 2
10x = 5x + 25 Average of total books sold by store
5x = 25 B and store C
x = 5 27 1
U

∴ Quantity of wine in the mixture = 20 + ×


= 5 × 5 = 25 litres. 2 2
30. Ans. (3) 40 + 27 67
@

Sol. Total salary of officers = Average = =


4 4
salary of officers × number of officers Total books in store A = 35
= 6800 × 5 = 34000
Let number of non-officers = x 20
Total salary of non-officers = x × 2000 No. sold in store A = × 35 = 7
100
Total salary of entire staff No. unsold = 35 – 7 = 28
= 34000 + 2000x Difference between average of total
34000 + 2000x = 3200 × (x + 5) books sold by store B and store C to
34000 + 2000x = 3200x + 16000 total unsold books of store A
1200x = 34000 – 16000
1200x = 18000 67 112 − 67
= 28 – =
18000 4 4
x = 45 45
1200 = = × 100
x = 15 4 4
= 45 × 25 = 1125 books
844 ✫ Quantitative Aptitude

[The average of the total books sold Unsold books of store B


by store B and store C together is less 40
= 50 – × 50
than total unsold books of store A by 100
1125] = 50 – 20 = 30
33. Ans. (4) Unsold books of store C
Sol. Let ratio of plain to lined books sold
for store C 30 27
= 45 – × 45 = 45 –
= 5x and 4x respectively 100 2
30 90 − 27 63
5x + 4x = × 45 = =
100 2 2

TS
27 Total no. of books sold by store B
9x = and store C
2
27 63 60 + 63
x = = 30 + =

H
18 2 2
3 123
x = =
2

G
2
∴ Total no. of plain books sold by Difference between unsold books of
store C store B and store C to store A

= 5x = 5 ×
3
2
=
15
2 U =
123
2
− 28 =
67
2
O
Let ratio of plain to lined books sold 67
for store B = 3x and 2x respectively.
40 = 2 × 100
H
3x + 2x = (50) 123
100 2
5x = 20
_T

67 2 6700
x = 4 = × × 100 =
2 123 123
∴ Total no. of plain books sold by
store B =3 × 4 = 12 = 54.47% ~ 54%
Books sold by store A is 54% less
C

∴ Total no. of plain books sold by


than books sold by store B and
store C and store B store C taken together.
PS

15 35. Ans. (5)


= + 12
2 Sol. Total books sold by store B
15 40
= + 12 × 100 = × 50 = 20
U

2 100
Lined books sold by store B
15 + 24 60
= × 100 × 20 = 12
@

2 =
100
= 39 × 50 = 1950 ∴ No. of plain books sold by store B
34. Ans. (2) = 20 – 12 = 8
Sol. Unsold books of store A Total amount earned by store B
= 12 × 175 + 8 × 250
20 = 2100 + 2000
= 35 – × 35
100 = 4100 × (100)
= 35 – 7 = 28 = 410000 = 4.1 lakh
Solved Test Papers of Various Examinations ✫ 845

IBPS PO (PT)
Examination, 2018
1. A boat takes 630 minutes to travel from Point (3) if either statement I or II is sufficient.
A to B and then comes back to Point A. The (4) if both statements I and II together are
distance between Point A and B is 72 km. If not sufficient.
the ratio of speed of the boat in still water to (5) If both statements I and II together are
the speed of water current is 7 : 1, what is sufficient, but neither of them alone is
the speed of the boat downstream? (in kmph) sufficient.

TS
(1) 23 (2) 12 7. A vessel contains ‘X’ litres of mixture of milk
(3) 24 (4) 16 and water. The ratio of milk to water in the
(5) 8 vessel is 9 : 4. What is the numerical value

H
Directions (Q. 2-6): What approximate value will of ‘X’?
come in place of question mark (?) in the given I. If 40% of the mixture is taken out and
question? (You are not expected to calculate the 5 litres of water is added, the

G
exact value.) percentage of water in the resultant
25 64 16 mixture will be 40%.
2. 9
(1) 4.5

81 ÷ 324 = ?
(2) 2.5 U II. If 20% of the mixture is taken out and
this action is repeated once more (ie
O
20% of the remaining mixture is taken
(3) 1.5 (4) 3.5 out) the remaining quantity of milk in
(5) 4 the mixture will be 28.8 litres.
H
3. 800.11 × ? – 99 = 3995 8. In how many days can A, B and C together
finish a piece of work?
_T

(1) 9 (2) 1
(3) 25 (4) 36 I. A and B together can finish the same
(5) 64 2
piece of work in 8 days. B and C
3
4. 749.82 ÷ 14.89 × 4.99 = ?2 × 10.002
C

together can finish the same piece of


(1) 5 (2) 2 1
(3) 10 (4) 3 work in 13 days. C and A together
PS

(5) 1 3
can finish the same piece of work in
5. 125% of 260 + ?% of 700 = 500 3
(1) 32 (2) 56 11 days.
7
U

(3) 23 (4) 46 II. The time taken by A alone to finish the


(5) 25 same piece of work is 24 days less
@

than the time taken by C alone to


6. 1728 ÷ 3 262144 × ? – 288 = 4491
finish the same piece of work.
(1) 148 (2) 156
9. What is the height of a cylinder? (in m)
(3) 173 (4) 177
I. The volume of the cylinder is 4158 m3
(5) 185
II. The total surface area of the cylinder
Directions (Q. 7-12): The questions given below is 1485 m3.
contain two statements numbered I and II giving a
certain data. You have to decide whether the data 10. What is the value of ?
b
given in the statements are sufficient to answer I. The value of a – b is 3. The value of
the question. Give answer 2 2
a – b is 45.
(1) if statement I alone is sufficient but II. The value of a + b is 15.
statement II alone is not sufficient.
(2) if statement II alone is sufficient but 11. In a 160-litre mixture of milk and water, the
percentage of water is only 35%. The
statement I alone is not sufficient.
846 ✫ Quantitative Aptitude

milkman gave 40 litres of this mixture to a governmental projects handled by the same
customer and then added ‘X’ litres of pure company in 2003 and 2004 together?
milk to the remaining mixture. As a result, (1) 204 (2) 188
the percentage of water in the final mixture (3) 192 (4) 196
becomes 30%. What is the value of ’X’? (5) 212
(1) 20 (2) 25 15. The number of projects handled by company
(3) 30 (4) 15 B decreased by what per cent from 2004 to
(3) 10 2006?
12. Two stations, A and B, are 677 km apart from 5 7
(1) 35 (2) 30

TS
each other. One train starts from Station A at 8 8
2 pm and travels towards station B at 59 3 3
kmph. Another train starts from station B at (3) 50 (4) 45
8 8
3.30 pm and travel towards station A at 48
5

H
kmph. How far from station B will the two (5) 40
trains meet? (in km) 8

G
(1) 264 (2) 256 16. If the number of projects handled by
(3) 272 (4) 286 company A increased by 20% from 2007 to
(5) 252 2008 and by 5% from 2008 to 2009, then what
Directions (Q. 13-18): Refer to the graph and
answer the given question: U was the number of projects handled by
Company A in 2009?
O
(1) 378 (2) 372
Company
(3) 384 (4) 396
A B
H
(5) 368
Number of projects handled

400
350
360 350 17. What is the difference between the total
_T

320 number of projects handled by company A


300 300
250 in 2003 and 2004 together and the total
250 210
180 230 number of projects handled by company B
200 190 in 2005 and 2007 together?
C

150 (1) 120 (2) 150


130
100 (3) 130 (4) 180
PS

50 (5) 170
0
18. There are two bags A and B. Bag A contains
2003 2004 2005 2006 2007 6 red balls and 10 green balls and bag B
U

Year contains 4 red balls and 6 green balls. One


13. Out of the total number of projects handled bag is selected at random. From the selected
by company A in 2005 and 2006 together, 20% bag, one ball is drawn at random. What is
@

were governmental projects. What was the the probability that the ball drawn is red?
total number of governmental projects 27 31
handled by company A in 2005 and 2006 (1) (2)
80 80
together? 39 29
(1) 108 (2) 132 (3) (4)
(3) 128 (4) 116 80 80
(5) 122 33
(5)
80
14. The project handled by a company can be
broadly classified into types— governmental Directions (Q. 19-24): Read the information given
projects and non-governmental projects. If in the passage and answer the given questions.
the average number of non-governmental There are ‘X’ number of students in a
projects handled by company B in 2003 and college. Each of them likes either one or more
2004 is 127 then what is the total number of of the following types of movies—Hollywood,
Solved Test Papers of Various Examinations ✫ 847

Bollywood and regional. The ratio of male to (1) 9 : 14 (2) 3 : 4


female students is 9 : 7. 16% of the male (3) 5 : 8 (4) 5 : 6
students like only Hollywood movies. 22% like (5) 7 : 8
only Bollywood movies. 12% like only regional 23. What is the value of ‘X’ (mentioned in the
movies. 30% of the male students like only passage)?
Hollywood and Bollywood movies. 10% like (1) 960 (2) 800
only Bollywood and regional movies and 6% (3) 640 (4) 720
like only regional and Hollywood movies. The (5) 880
remaining 18 male students like all the given
24. The ratio of the present ages of P and Q is

TS
type of movies.
5 : 7. The ratio of P’s age 4 years ago to Q’s
14% of the female students like only
age 10 years hence is 1 : 2. What will be the
Hollywood movies. 20% like only Bollywood ratio of P’s age 2 years hence to Q’s age 6
movies. 8% like only regional movies, 26% of years hence?

H
the female students like only Hollywood and (1) 2 : 3 (2) 1 : 3
Bollywood movies. 18% like only Bollywood (3) 7 : 8 (4) 5 : 6

G
and Regional movies and 10% like only regional (5) 3 : 4
and Hollywood movies. The remaining female
students like all the given type of movies. Directions (Q. 25-30): What will come in place of

19. What is the difference between the number


of male students who like Bollywood U question mark (?) in the given number series.
25. 150 29 372 347 374 ?
O
(1) 378 (2) 290
movies and the number of female students
(3) 370 (4) 280
who like the same?
H
(5) 372
(1) 69 (2) 59
(3) 63 (4) 65 26. 10 12 17 28 57 ?
_T

(5) 57 (1) 142 (2) 140


(3) 130 (4) 158
20. The number of students (both male and (5) 136
female) who like all the given types of
C

movies is approximately what per cent of the 27. 4 14 24 39 69 ?


number of female students who like only one (1) 152 (2) 164
(3) 184 (4) 144
PS

of the given type of movies?


(1) 12 (2) 18 (5) 122
(3) 32 (4) 28 28. 158 78 38 18 8 ?
(5) 22 (1) 3 (2) 5
U

(3) 2 (4) 7
21. The number of male students who like only
(5) 6
two of the given types of movies is what per
@

cent more than the number of female 29. 16 9 24 33 50 ?


students who like only two of the given (1) 83 (2) 66
types of movies? (3) 99 (4) 74
1 1 (5) 102
(1) 11 (2) 16 30. A bank offers 5% compound interest
21 7
11 1 calculated on a half-yearly basis. A customer
(3) 9 (4) 8 deposits ` 1600 each on Ist January and Ist
21 7
July of a year. At the end of a year, the
11
(5) 13 amount he would have gained by way of
14 interest is how much?
22. What is the ratio of the number of female (1) ` 120 (2) ` 121
students who like Hollywood movies to the (3) ` 123 (4) ` 122
number of male students who like the same? (5) ` 119
848 ✫ Quantitative Aptitude

Directions (Q. 31-35): Based on the following table, (1) 1750 (2) 1720
answer the given questions. (3) 1710 (4) 1730
The data given in the table is for the month (5) 1760
of March in 2015. 33. What is the difference between the number
of male employees in Company A and that
Total number Number of female in Company F?
Company
of employees employees (1) 840 (2) 810
A 5550 2410 (3) 820 (4) 740
(5) 790
B 3200 1860
34. If in April 2015, the number of female

TS
C 2000 1600 employees in Company E increased by 10%
D 2500 1220 and the number of total employees in the
company remained the same, what was the
E 4240 2600
number of male employees?

H
F 3560 1240 (1) 1430 (2) 1420
31. The number of male emplyees in company (3) 1410 (4) 1360
(5) 1380

G
D is what per cent less than the number of
female employees in company C? 35. The total number of female employees in
Company B and F together is what per cent
(1) 12
(3) 20
(5) 18
(2) 15
(4) 22
U of the total number of employees in
Company D?
O
(1) 124 (2) 122
32. What is the average number of female (3) 125 (4) 134
employees in companies A, C and F? (5) 135
H

Answers and Explanatory Notes


_T

1. Ans. (4) Speed downstream = 8x = 8 × 2 =


Sol. Let the speed of boat = 7x 16 km /hr
Speed of stream = x 2. Ans. (4)
C

Distance between point A and B = 72


km 25 64 16
Sol. − ÷ = ?
Total Time taken to go downstream 9 81 324
PS

and then upstream = 630 minutes 5 8 4


630 21 = − ÷
= = hr 3 9 18
60 2 5 8 18
U

72 72 21 = − ×
+ = 3 9 4
7x + x 7x − x 2 15 − 8 18
@

= ×
72 72 21 9 4
+ =
8x 6 x 2 7 18 7×2
= × =
1 1 21 9 4 4
72 8x + 6 x = 14
2 = = 3.5
72 1 + 1 4
21
= 3. Ans. (3)
2 4 x 3x 2
3+4 21 Sol. 800.11 × ? – 99 = 3995
36 =
12x 2 800 × x – 99 = 3995
(7) 21 800 x = 3995 + 99
3 =
x 2 3995 + 10
x = 2 km /hr x=
800
Solved Test Papers of Various Examinations ✫ 849

4005 Let, the amount of water in the


x= = 5 remaining mixture = y
800
1 i.e.
( x) 2 = 5 4x
x = (5)2 13 = y
= 25 x 3x
5
4. Ans. (1)
Sol. 749.82 ÷ 14.89 × 4.99 = ?2 × 10.002 12 x
y =
750 ÷ 15 × 5 = ?2 × 10 65

TS
50 × 5 = ?2 × 10 5 litres of water is added to the
50 × 5 remaining mixture
?2 = 3x
10 Volume of mixture = + 5
?2 = 25 5

H
?2 = 25 12 x
Volume of water = + 5
= 5 65

G
5. Ans. (5) Amount of water in the resultant
mixture = 40%
Sol. 125% of 260 + x% of 700 = 500
125
× 260 +
x
× 700 = 500 U i.e.
12x + 5
65 × 100 = 40
O
100 100 3x + 5
325 + 7x = 500 5
7x = 500 – 325
H
12x + 325
7x = 175 40
i.e. 65 =
x = 25
3x + 25
_T

100
6. Ans. (4) 5
12 x + 325 5 2
Sol. 1728 ÷ 3 262144 × ? – 288 = 4491 × =
+
C

65 (3 x 25) 5
1728 ÷ 64 × x = 4491 + 288
1728 (12 x + 325) 1 2
PS

× x = 4779 × =
64 (3 x + 25) 13 5
27x = 4779
(12 x + 325) 26
4779 =
(3 x + 25) 5
U

x =
27 5(12x + 325) = 26(3x + 25)
= 177 60x + 1625 = 78x + 650
@

7. Ans. (3) 78x – 60x = 1625 – 650


Sol. Statement I. 18x = 975
Amount of mixture = x litres
975
9x x = ≈ 54.18 litres
Milk = litres 18
13 Hence, statement I is sufficient to
4x find the value of x (amount of
Water = litres
13 mixture)
Out of mixture, 40% is taken out Statement II
Remaining mixture Mixture = x litres
40 x 9x 4x
= x – Milk = Water =
100 13 13
3x 20% of mixture is taken out, i.e.
= mixture remaining
5
850 ✫ Quantitative Aptitude

20 x (C + A) can complete the work in


x – 3 80
100 11 days = days
80 x 4x 7 7
= = ∴ (A + B) + (B + C) + (C + A) can
100 5 complete the work in
Let the amount of milk in the
remaining mixture = y litres 26 40 80
+ + days
y 9x 3 3 7
=
4 13 234
x i.e days
5 x 7

TS
9x 4x i.e. 2A + 2B + 2C can complete the
× = xy
13 5 234
work in days
36 x 7
= y 2(A + B + C) can complete the work

H
65
Again 20% of the remaining mixture 234
is taken out, i.e. remaining mixture in days
7

G
4x 20 4x (A + B + C) can complete the work
= – ×
5 100 5 234 1
in ×
=
4x
5

4x
25 U i.e.
7
234
2
days
O
14
20 x − 4 x
= 5
25 = 16 days
H
16 x 7
= Hence, statement I is sufficient to
25
find the answer.
_T

Let the amount of milk in the


remaining mixture = y Statement II is not sufficient to find
16 x x the answer because nothing has been
= said about B’s capacity to work.
25 9x
C

9. Ans. (4)
y 13
Sol. From statement I
PS

16 x 9x V = πr2h = 4158m3 ... (i)


× = xy
25 13 From statement II
144 x 2πr2 + 2πrh = 1485m2
= y i.e. 2πr (r + h) = 1485m2 ... (ii)
U

325
y is given as 28.8 litre Dividing (ii) by (i)
2 πr ( r + h ) 1485
@

144 x 288 =
∴ =
325 10 πr 2 h 4158
288 × 325 πr ( r + h ) 1485
x = = 65 litres =
144 × 10 πr h 2
4158 ×2
Hence, statement II is sufficient to
r+h 1485
find the value of x. =
8. Ans. (1) hr 8316
Sol. Statement I r h 1485
2 or + =
(A + B) can complete the work in 8 hr hr 8316
26 3
days = days 1 1 1485
3 + =
(B + C) can complete the work in h r 8316
Two equations are required to find
1 40 two unknown quantities.
13 days = days
3 3
Solved Test Papers of Various Examinations ✫ 851

10. Ans. (1) 42


Sol. Statement I i.e. × 100 = 30
120 + x
a – b =3 ... (i)
4200 = 30 (120 + x)
a2 – b2 =45
4200 = 3600 + 30x
(a + b) (a – b) =
45
30x = 600
(a + b) (3) = 45
x = 20 litres.
45 12. Ans. (1)
a + b =
3 Sol.
a + b = 15 (ii)

TS
Adding (i) and (ii) we get
2a = 18
a = 9
From (i) we get

H
a – b = 3
One train starts at 2 pm and travels
b = 6
to station B at 59 km /hr

G
a 9 3 Another train starts at 3.30 pm and
∴ = =
b 6 2 travels to station A at 48 km /hr
II Value of a + b = 15
U Distance travelled by train A before
train B starts
O
This is not sufficient to find two 1
unknown quantities a and b. Single = 59 × 1
2
a
H
value of cannot be found out. 3 117
b = 59 × = = 58.5 km
11. Ans. (1) 2 2
_T

Sol. Total quantity of a mixture of milk Remaining distance to be covered by


+ water = 160 litres A = 677 – 58.5
Water = 35% when it can meet train B = 588.5 km
Let the two trains meet after time t.
C

35
i.e. × 160 = 56 litres x = distance from station B where
100 two trains meet after time t.
PS

Quantity of mixture remaining after x


the milkman gave 40 litres of the t =
48
mixture to a customer 588.5 − x
= 160 – 40 = 120 litres t =
U

Let the quantity of water in 120 litres 59


of mixture = x x 588.5 − x
=
@

Then 48 59
56 x 59x = (588.5 – x) 48
= 59x + 48x = (588.5)(48)
160 120
107x = 28248
160x = 56 × 120
56 × 120 x = 264 km.
x = 13. Ans. (5)
160
= 42 litres Sol. Total no. of projects handled by
Quantity of milk added to the new company A in 2005 and 2006
= 360 + 250
mixture = x litres.
= 610
Total quantity of new mixture
Governmental project = 20%
= (120 + x) litres
20
Percentage of water in final mixture i.e. × 610 = 122
= 30% 100
852 ✫ Quantitative Aptitude

14. Ans. (4) 18. Ans. (2)


Sol. Total no. of projects handled by Sol. Probability of selecting a red ball
company B in 2003 and 2004 from bag A
= 130 + 320 = 450 6 3
Total no. of non-governmental = =
16 8
projects handled by company B in Probability of selecting a red ball
2003 and 2004 from bag B
= 127 × 2 4 2
= 254 = =
10 5
∴ No. of governmental projects

TS
One bag is selected at random
handled by company B in 2003
and 2004 ∴ Probability of selecting bag A = 1 and
= 450 – 254 21
probability of selecting bag B =

H
= 196 2
15. Ans. (5) ∴ Probability of selecting red ball
Sol. The total project handled by company from bag A

G
in 2004 = 320 1 3 3
The total projects handled by = × =
2 8 16
company in 2006 = 190
Decrease = 320 – 190
U Prohability of selecting red ball from
bag B
O
= 130 1 2 1
% Decrease = × =
2 5 5
130 Both the events are mutually
H
= × 100 exclusive.
320
1300 5 3 1
_T

= = 40 % Hence required probability = +


32 8 16 5
16. Ans. (1) 15 + 16 31
= =
Sol. Projects handled by company A in 80 80
C

2007 = 300 19. Ans. (2)


Projects handled in 2008 Sol. Total no. of males = 100
∴ 100 – (16 + 22 + 12 + 30 + 10 + 6)
PS

20 = 4%
= 300 + × 300
100 4% like all type of movies.
= 300 + 60 Let, the total no. of males = x
U

= 360 4
× x = 18
Projects handled in 2009 100
5 1800
@

= 360 + × 360 x = = 450


100 4
= 360 + 18 Total no. of females
= 378 450 x
=
17. Ans. (2) 9 7
Sol. Total no. of projects handled by 450 × 7
company A in 2003 and 2004 x =
9
= 180 + 230 = 410 = 50 × 7 = 350
Total no. of projects handled by No. of males who like Bollywood
company B in 2005 and 2007 movies
= 210 + 350 = 560 22 30 10
= × 450 + × 450 +
Difference = 560 – 410 100 100 100
4
= 150 × 450 + × 450
100
Solved Test Papers of Various Examinations ✫ 853

450 21. Ans. (3)


[22 + 30 + 10 + 4]
100 Sol. No. of male students who like only
45 two of the given types of movies
= [66] 30 10 6
10 = × 450 + × 450 + × 450
100 100 100
9 450
= [66] = (30 + 10 + 6)
2 100
= 9 × 33 45
= 297 = (46)
10

TS
No. of females who like Bollywood
movies = 9 × 23 = 207
No. of female students who like only
20 18 26
= × 350 + × 350 + two of the given types of movies

H
100 100 100 10
26 18
4 = × 350 + × 350 + × 350
× 350 + × 350 100 100 100
100

G
350
[i.e. 100 – (20 + 18 + 26 + 8 + 10 + 14] = (26 + 18 + 10)
100 – (96) = 4% 100

=
350
100
(20 + 18 + 26 + 4)
U =
35
10
(54) = 7 × 27
O
= 189
35
= (68) ∴ Required percentage
10
H
7 207 − 189
= (68) = × 100
2 189
_T

= 7 × 34 18
= × 100
= 238 189
∴ Difference between the no. of 1800
males and females who like =
189
C

Bollywood movies are 11


200
297 – 238 = 59 = = 9 %
PS

21 21
20. Ans. (5)
22. Ans. (2)
Sol. No. of males who like all given types
Sol. No. of males who like Hollywood
of movies = 18
movies
U

No. of females who like all given


types of movies 16 30 6
= × 450 + × 450 +
4 100 100 100
@

= × 350 =14
100 4
× 450 + × 450
∴ Total no. of students who like all 100
given types of movies 450
= (16 + 30 + 6 + 4)
= 18 + 14 = 32 100
Total no. of female students who like 9
= (56)
only one of the given types of movies 2
= 9 × 28 = 252
14 20 8 No. of females who like Hollywood
= × 350 + × 350 + × 350
100 100 100 movies
= 49 + 70 + 28 = 147 14 26 10
= × 350 + × 350 +
Required percentage 100 100 100
32 4
∴ × 100 = 21.76% ≈ 22% × 350 + × 350
147 100
854 ✫ Quantitative Aptitude

27. Ans. (4)


350
= (14 + 26 + 10 + 4) Sol. 4, 14, 24, 39, 69 ?
100 4
7 4 + 10 = 14
= (54) 14 + 10 × 1 = 24
2
= 7 × 27 24 + (10 × 1.5) = 39
= 189 39 + (15 × 2) = 69
189 9 69 + (30 × 2.5) = 144
∴ Required ratio = = 3 : 4 The series is 4, 14, 24, 39, 69, 144
252 12 28. Ans. (1)
23. Ans. (2)

TS
Sol. 158, 78, 38, 18, 8, ?
Sol. Value of X = 350 + 450 = 800
158
(X = no. of students) (158 ÷ 2) – 1 = 79 – 1 = 78
24. Ans. (1) (78 ÷ 2) – 1 = 39 – 1 = 38
Sol. Let the present age of P and Q = 5x (38 ÷ 2) – 1 = 19 – 1 = 18

H
and 7x respectively (18 ÷ 2) – 1 = 9 – 1 = 8
P’s age 4 years ago = 5x – 4 (8 ÷ 2) – 1 = 4 – 1 = 3

G
Q’s age 10 years hence = 7x + 10 The series is 158, 78, 38, 18, 8, 3
5x − 4 1 29. Ans. (1)
Ratio = Sol. 16, 19, 24, 33, 50, ?
7 x + 10 2
2(5x – 4) = 7x + 10
U 16
16 + 3 + 0 = 19
O
10x – 8 = 7x + 10 19 + 3 + 2 = 24
3x = 18 24 + 5 + 4 = 33
x = 6 33 + 9 + 8 = 50
H
P’s age = 5 × 6 = 30 50 + 17 + 16 = 83
Q’s age = 7 × 6 = 42 The series is 16, 19, 24, 33 50, 83
_T

P’s age 2 years hence = 30 + 2 = 32 30. Ans. (2)


2T
Q’s age 6 years hence = 42 + 6 = 48 R/2
Sol. A = P 1+
32 100
Reqd Ratio = 2 ×1
C

48 5/ 2
2 A = 1600 1 +
= 100
PS

3 Amount after 1 year on Rs 1600


i.e. 2 : 3 deposited on Ist Jan when interest is
25. Ans. (3) calculated half yearly
Sol. 150 5
2
U

150 – 112 = 150 – 121 = 29 A = 1600 1 +


29 + 73 = 29 + 343 = 372 200
2
@

372 – 52 = 372 – 25 = 347 1


347 + 33 = 347 + 27 = 374 A = 1600 1 +
40
374 – 22 = 374 – 4 = 370 2
The series is 41
A = 1600 ... (i)
150, 29, 372, 347, 374, 370 40
1

26. Ans. (4) 5/ 2 2
Sol. 10, 12, 17, 28, 57, ? A = 1600 1 +
10 100
10 + 2 = 12 1
12 + 2 + 3 = 17 [Amount after year on ` 1600
2
17 + 5 + 6 = 28 deposited on Ist July when interest
28 + 11 + 18 = 57
57 + 29 + 72 = 158 is calculated half yearly]
41
The series is = 1600 ... (ii)
10, 12, 17, 28, 57, 158 40
Solved Test Papers of Various Examinations ✫ 855

Total amount = (i) + (ii) Required percentage


2
41 41 320
1600 + 1600 40 = × 100 = 20
40 1600
32. Ans. (1)
41 41 Sol. Average no. of female employees in
(1600) +1
40 40 companies A, C and F
41 + 40 2410 + 1600 + 1240
(41 × 40) =
40 3
= 41 × 81 5250
=

TS
= ` 3321 3
Total principal = 1600 + 1600 = 1750
= 3200 33. Ans. (3)
C.I. = 3321 – 3200 Sol. No. of male employees in company

H
= ` 121 A = 5550 – 2410 = 3140
Alternate method Q. 30 No. of male employees in company

G
(i) Interest on ` 1600 deposited on F =3560 – 1240 = 2320
Ist January Difference = 3140 – 2320 = 820
(C.I. = 5% on half-yearly basis)

1600 ×
2.5
= 40 U
... (i)
34. Ans. (5)
Sol. Total no. of female employees in
O
100 company E after an increment of 10%
(ii) Interest on ` 1600 + 40 10
= ` 1640 = 2600 + 2600 ×
H
100
2.5 = 2600 + 260
1640 × = 41 ... (ii) = 2860
100
_T

(iii) Interest on ` 1600 deposited from Total no. of male employees in


July to December company E = 4240 – 2860 = 1380
2.5 35. Ans. (1)
C

1600 × = 40 ... (iii)


100 Sol. Total no. of female employees in
Total Interest = (i) + (ii) + (iii) company B and company F
PS

= 40 + 41 + 40 = 1860 + 1240 = 3100


= 121 No. of employees in company D
= ` 121 = 2500
U

31. Ans. (3) Required percentage


Sol. No. of male employees in company 3100
D = 2500 – 1220 = 1280 = × 100
@

2500
No. of female employees in company 3100
C = 1600 = = 124
25
Difference = 1600 – 1280 = 320
856 ✫ Quantitative Aptitude

State Bank of India (SBI)


PO (PT) Examination, July 2018
Direction—(Q. 1 to 5) Bar graph given below 5. Out of total pens sold on Tuesday ratio
shows pens sold by a retailer on five different between total defective pens sold to total
days. Study the data carefully and answer the pens sold is 7 : 15. Find total number of non-
following questions— defective pens sold on Tuesday by retailer?
Pen sold on different days (A) 20 (B) 25
(C) 30 (D) 35

TS
80
70
(E) 40
60 Direction—(Q. 6 to 10)—In each of given questions
50 two quantities numbered I and II are given. You

H
40
have to solve both the quantities and mark the
30
appropriate answer.
20

G
10 6. Quantity:
0
Mon Tues Wed Thrus Fri I. ‘x’: x2 + x – 6 = 0
1. Find the difference between total number of
pens sold on Monday and Tuesday together U II. ‘y’ : y2 + 7y
(A) Quantity I >
+ 12 = 0
Quantity II
O
(B) Quantity I < Quantity II
to total number of pens sold on Thursday (C) Quantity I ≥ Quantity II
and Friday together ? (D) Quantity I ≤ Quantity II
H
(A) 15 (B) 10 (E) Quantity I = Quantity II or
(C) 5 (D) 20 No relation
_T

(E) 0
7. A’s efficiency is 25% more than B’s
2. Total number of pens sold on Saturday is Quantity:
40% more than total number of pens sold on I. ‘x’ : A can do 5/6th of total work in
C

Wednesday. Find total number of pens sold ‘x’ days.


on Friday and Saturday together? II. ‘y’ : B can do 4/5th of total work in
PS

(A) 92 (B) 110 ‘y’ days


(C) 72 (D) 108 (A) Quantity I > Quantity II
(E) 85 (B) Quantity I < Quantity II
(C) Quantity I ≥ Quantity II
U

3. Total number of pens sold on Tuesday are


25% more than total number of pens sold on (D) Quantity I ≤ Quantity II
(E) Quantity I = Quantity II or
@

Sunday. Find total number of pens sold on


No relation
Sunday?
(A) 64 (B) 50 8. Sum of 8 consecutive even number is S1.
(C) 94 (D) 60 Quantity:
(E) 55 I. Sum of second number and eight
number in S1
4. Out of total pens sold on Thursday, 20% are
II. Sum of third number and sixth number
blue ink pen. Out of remaining 25% are red
in S1
ink pen and remaining are black in pen. Find
(A) Quantity I > Quantity II
total number of blue and black ink pen sold (B) Quantity I < Quantity II
on Thursday? (C) Quantity I ≥ Quantity II
(A) 27 (B) 36 (D) Quantity I ≤ Quantity II
(C) 45 (D) 39 (E) Quantity I = Quantity II or
(E) 30 No relation
Solved Test Papers of Various Examinations ✫ 857

9. An article is sold at ` 1,500 after allowing cylinder and cone is same i.e., 10 cm. If radius
discount of 12.5% on Marked price. of cone is 15 cm then find the ratio of radius
Quantity: of S to radius of C?
I. ` 550 (A) 1 : 2 (B) 3 : 4
II. Mark price of article. (C) 2 : 5 (D) 4 : 5
(A) Quantity I > Quantity II (E) 3 : 5
(B) Quantity I < Quantity II
17. In a box, there are 6 blue balls, X red balls &
(C) Quantity I ≥ Quantity II
10 green balls. Probability of choosing one
(D) Quantity I ≤ Quantity II
(E) Quantity I = Quantity II or 1
red ball from the given box is . Then find

TS
No relation 3
the sum of red and blue balls in the box?
10. If the speed of a boat is 500% more than the (A) 20 (B) 12
speed of a current. (C) 14 (D) 18

H
Quantity: (E) 16
I. ‘x’ : If boat can travel a distance of 63
18. Sum of A’s and B’s age 6 years ago is 88. A’s
km in 3 hr., in downstream then ‘x’ is

G
age 18 years ago is equal to B’s age 6 years
the speed of the boat in upstream
ago. Find the age of A two year hence?
(km/hr.)
(A) 58 years (B) 64 years
II. 15 km/hr
(A) Quantity I > Quantity II
(B) Quantity I < Quantity II U (C) 42 years
(E) 48 years
(D) 52 years
O
(C) Quantity I ≥ Quantity II 19. Train A of length 120 m can cross a platform
(D) Quantity I ≤ Quantity II of length 240 m in 18 second. The ratio of
H
(E) Quantity I = Quantity II or speed of train A and train B is 4 : 5. Then find
No relation the length of train B if train B can cross a pole
_T

in 12 seconds—
Direction—(Q. 11 to 15) What number is wrong
(A) 280 m (B) 300 m
according to given number series pattern— (C) 320 m (D) 350 m
11. 1, 3, 9, 31, 128, 651, 3913 (E) 240 m
C

(A) 9 (B) 1 20. What is the probability of forming word from


(C) 128 (D) 31 the letters of word ‘IMPEACH’ such that all
PS

(E) 3913
vowels come together?
12. 291, 147, 75, 39, 22, 12, 7.5 8 1
(A) 22 (B) 291 (A) (B)
35 7
(C) 147 (D) 75
U

3 17
(E) 7.5 (C) (D)
35 35
13. 26, 27, 34, 58, 106,186, 306
@

2
(A) 26 (B) 34 (E)
(C) 58 (D) 106 7
(E) 27 Direction—(Q. 21 to 25) Find the value of (?) in
following approximation questions—
14. 5.9, 6, 6.1, 6.4, 7.9, 18.5, 112.9
(A) 6 (B) 5.9 21. 2? = 32.01 ÷ 128.01 × 1023.99 ÷ 7.99
(C) 6.1 (D) 18.5 (A) 7 (B) 3
(E) 112.9 (C) 4 (D) 5
(E) 8
15. 330, 80, 280, 120, 250, 130, 240
(A) 330 (B) 130 339.99
(C) 280 (D) 240 22. = 143.99 + 64.01
(E) 80 ?
(A) 17 (B) 20
16. Sum of volume of cylinder (S) and volume (C) 10 (D) 34
of cone (C) is 2190π cm2 & height of both (E) 40
858 ✫ Quantitative Aptitude

23. 34.02% of 550.09 ÷ ? = 297.07 ÷ 28. Total number of officers in company ‘A’ is
728.95
how much less than total number of officers
(A) 14 (B) 21 in company ‘B’?
(C) 8 (D) 27 (A) 4 (B) 2
(E) 17 (C) 0 (D) 6
24. (? ÷ 9.97) × 12.08 = 20.12% of 1319.97 (E) 8
(A) 220 (B) 240 29. Total number of officers and workers in
(C) 260 (D) 280 company D is 50% and 25% more than total
(E) 200 number of officers and workers in company
‘C’ respectively. Find total number of

TS
25. ?% of 179.99 = ( 24.02)2 + (17.98)2 + 60.01% employees in company ‘D’ ?
of 659.98 (A) 279 (B) 297
(A) 80 (B) 60 (C) 342 (D) 324
(C) 40 (D) 20 (E) 306

H
(E) 10 30. Find the difference between total number of
Direction—(Q. 26 to 30) Pie chart given below workers in company ‘A’ and total number of

G
workers in company ‘B’ and ‘C’ together ?
shows total number of workers in three different
(A) 432 (B) 396
companies. Table given below shows ratio (C) 360 (D) 324
between officers and workers working in these
companies. Study the data carefully and answer
U (E) 288
Direction—(Q. 31 to 35) There are three persons
O
the following questions—
A, B and C who each invested in two different
Total Workers = 900 scheme S1 and S2. A invested ` 80,000 for 2 year in
H
scheme S1 and ` 30,000 for 4 years in scheme S2. B
invested ` 30,000 for 3 years in S1 and he did not
_T

invest in scheme S2. B also obtained a profit of


` 10,000 by selling his car. C invested ` 50,000 for
5 years in scheme S1 and ` 10,000 for 3 years in
scheme S2. Total profit obtained from scheme S1 is
C

2 lakh and scheme S2 is ` 90,000.


31. What is the ratio of total profit obtained by
PS

Company Officers : Workers B and profit obtained by C from scheme S1


A 1 : 16 (A) 23 : 47 (B) 54 : 47
B 1 : 18 (C) 36 : 43 (D) 23 : 50
U

C 1 : 12 (E) 27 : 50
Note: Total employees = Officers + Workers 32. Profit obtained by A from scheme S1 is what
@

26. Find the ratio between total number of per cent of profit obtained by C from scheme
workers in company A and C together to total S2 ?
number of officers in company A and C 7 8
(A) 346 % (B) 347 %
together? 9 9
(A) 16 : 1 (B) 12 : 1 7 4
(C) 14 : 1 (D) 18 : 1 (C) 356 % (D) 345 %
9 9
(E) 20 : 1 5
(E) 355 %
27. Total number of employees in company ‘B’ 9
is how much more than total number of 33. If sum of investment of A in both schemes
employees in company ‘C’— and total profit obtained by A from both
(A) 174 (B) 194 schemes is invested at compound interest at
(C) 204 (D) 214 the rate of 20% p.a. then find the total
(E) 184 compound interest obtained in 2 years?
Solved Test Papers of Various Examinations ✫ 859

(A) ` 1,08,240 (B) ` 1,04,206 35. If A had invested his sum at simple interest
(C) ` 1,05,208 (D) ` 1,09,280 for 3 years at the rate of R% p.a. instead in
(E) ` 1,06,220 scheme S1 and B has invested his sum at
34. What is the average of profit obtained by A compound interest at (R + 5%) p.a. for 1 year
from scheme S1 and profit of C obtained from and difference in interest obtained is ` 30,000
scheme S2? then find value of R%—
(A) ` 41,000 (B) ` 42,000 (A) 10% (B) 9%
(C) ` 44,000 (D) ` 55,000 (C) 15% (D) 18%
(E) ` 40,000 (E) 12%

TS
Answers and Explanatory Notes
1. Ans. (C) 20
Total no. of blue ink pen = × 45
Sol. Total no. of pens sold on Monday

H
100
and Tuesday together = 25 + 75 = 100 = 9
Total no. of pens sold on Thrusday Remaining pens = 45 – 9

G
and Friday together = 45 + 50 = 95 = 36
Difference = 100 – 95 = 5 25
× 36
2. Ans. (A)
Sol. Total no. of pens sold on Saturday
40 U No. of red ink pens =
100
= 9
O
× 30 Remaining pens which are black
= 30 +
100 = 36 – 9
= 30 + 12 = 27
H
= 42 ∴ Total number of blue and black ink
Total no. of pens sold on Friday and pen sold on Thursday
_T

Saturday together = 9 + 27
= 50 + 42 = 36
= 92 5. Ans. (E)
Sol. Let no. of defective pens sold = 7x
C

3. Ans. (D)
Sol. Total no. of pens sold on Sunday Total pens sold on Tuesday = 15x
= x 15x = 75
PS

Total no. of pens sold on Tuesday x = 5


25 ∴ Total no. of defective pens sold on
x + ×x Tuesday = 7x = 7 × 5 = 35
100
U

∴ Total no. of non-defective pens


x = 75 – 35 = 40
= x +
4 6. Ans. (C)
@

4x + x Sol. Quantity I ≥ Quantity II


=
4 I. ‘x’: x2+ x – 6 = 0
5x II. ‘y’ : y2 + 7y + 12 = 0
=
4 I. x2 + x – 6 = 0
5x x2 + 3x – 2x – 6 = 0
= 75
4 x(x + 3) – 2 (x + 3) = 0
5x = 4 × 75 (x + 3) (x – 2) = 0
4 × 75 x + 3 = 0, x – 2 = 0
x = x = –3, x = 2
5
= 60 II. y2 + 7y + 12 = 0
4. Ans. (B) y2 + 4y + 3y + 12 = 0
Sol. Total no. of pens sold on Thursday y(y + 4) + 3 (y + 4) = 0
= 45 (y + 4) (y + 3) = 0
860 ✫ Quantitative Aptitude

y + 4 = 0, y + 3 = 0 87.5 x
y = – 4, y = – 3 Given that = 1500
Values of x are equal to or greater 100
than the values of y. 1500000
x = = 1714.285
7. Ans. (B) 875
Quantity I = ` 550
Sol. Quantity I ≤ Quantity II
Let total work = 1 Quantity II = ` 1714.255
Hence Quanity I < Quantity II
5
of work is done by A in x day 10. Ans. (E)
6 5 Sol. Quantity I = Quantity II

TS
In 1 day, A can do of the work Let speed of current = y
6x
4 500
In 1 day, B can do of the work Speed of boat = y + y
5y 100
A’s efficiency is 25% more than B’s

H
= 5y + y
4 25 4 5
i.e. + × = = 6y
5y 100 5y 6x

G
I. Speed downstream = 6y + y
4 1 5 = 7y
+ =
5y 5y 6x
5
5y
=
5
6x U 63
7y
= 3
O
21y = 63
⇒ 6x = 5y y = 3 km/hr
x = 5, y = 6 Quantity I = 5, 10... = 6 × y = 6 × 3 = 21 km/hr
H
x = 10, y = 12 Quantity II = 6, 12... Speed upstream
Quantity I < Quantity II
= 6y – y = 5y = 5 × 3
_T

i.e. x < y
= 15 km/hr
8. Ans. (A)
Sol. Quantity I > Quantity II II. 15 km/hr
Let 8 consecutive even nos. be 2x, 11. Ans. (C)
C

2x + 2, 2x + 4, 2x + 6, 2x + 8, Sol. 1
2x + 10, 2x + 12, 2x + 14 1 × 1 + 2 = 3
PS

Sum of second number and eight 3 × 2 + 3 = 9


number is 9 × 3 + 4 = 31
2x + 2 + 2x + 14 31 × 4 + 5 = 129
= 4x + 16 = Quantity I 129 × 5 + 6 = 651
U

Sum of third number and sixth 651 × 6 + 7 = 3913


number 12. Ans. (A)
@

= 2x + 4 + 2x + 10 Sol. 291
= 4x + 14 = Quantity II (291 + 3) ÷ 2 = 147
4x + 16 > 4x + 14 (147 + 3) ÷ 2 = 75
i.e. Quantity I > Quantity II (75 + 3) ÷ 2 = 39
9. Ans. (B) (39 + 3) ÷ 2 = 21
Sol. Quantity I < Quantity II (21 + 3) ÷ 2 = 12
Let the marked price = x (12 + 3) ÷ 2 = 7.5
12.5 x 13. Ans. (E)
Selling Price = x – Sol. 26
100
26 + 0 = 26
100x − 12.5 x
= 26 + (8 × 1) = 26 + 8 = 34
100 34 + (8 × 3) = 34 + 24 = 58
87.5 x 58 + (8 + 6) = 58 + 48 = 106
= 106 + (8 × 10) = 106 + 80 = 186
100
Solved Test Papers of Various Examinations ✫ 861

186 + (8 × 15) = 186 + 120 = 306 = 12 : 15


The series is 26, 26, 34, 58, 106, 186, = 4 : 5
306 17. Ans. (C)
14. Ans. (D) Sol. Blue balls = 6, green balls = 10,
Sol. 5.9 red balls = X
5.9 + 0.1 = 6 Total balls = 6 + 10 + X
6 + (0.1 × 1) = 6 + (0.1) = 6.1 Probability of choosing 1 red ball
6.1 + (0.1 × 3) = 6.1 + (0.3) = 6.4 1
6.4 + (0.3 × 5) = 6.4 + (1.5) = 7.9 from the given box is given as
3
X 1

TS
7.9 + (1.5 × 7) = 7.9 + (10.5) = 18.4 i.e. =
18.4 + (10.5 × 9) = 18.4 + (94.5) = 112.9 6 + 10 + X 3
The series is X 1
=
5.9, 6, 6.1, 6.4, 7.9, 18.4 112.9 16 + X 3

H
15. Ans. (B) 3X = 16 + X
+40 +20 2X = 16

G
X = 8
Sol. 330 80 280 120 250 130 240 ∴ Total no. of red balls + blue balls in
–50 –30 –10 the box = 8 + 6 = 14
The series is
380, 80, 280, 120, 250, 140, 240 U 18. Ans. (B)
Sol. Let A’s age = x
O
16. Ans. (D) B’s age = y
Sol. Volume of cyclinder + volume of (x – 6) + (y – 6) = 88
cone = 2190 πcm3
H
x + y – 12 = 88
1 x + y = 100 ... (i)
π r12 h1 + π × r22 h2 = 2190π
3 Also
_T

1 x – 18 = y – 6
π r12 h1 + π × r22 ×h1 = 2190π x – y = 24 ... (ii)
3
(h1 = h2 = 10 cm given) Adding (i) and (ii)
C

1 2x = 124
i.e. π r12 × 10 + × π × r22 × 10 x = 62
3
PS

= 2190π A’s age two years hence


1 = 62 + 2 = 64
π r1 × 10 +
2
× π × 15 × 15 × 10
3 19. Ans. (B)
= 2190π [Radius of cone = 15 cm] Sol. Distance = length of Train A + length
U

2250 π of platform
10π r12 + = 2190π
3 = 120 + 240
@

2
30π r1 + 2250π = 6570π = 360
3π r12 + 225π = 657π Time = 18 seconds
( 2
)
3π r1 + 75 = 657π
Speed =
360
657π 18
r12 + 75 = = 20 m/sec

657 Ratio of speed of train A to Train B
r12 + 75 = 4
3 =
2 5
r1 + 75 = 219 4 20
r12 = 219 – 75 = 144 i.e. =
5 x
r1 = 144 = 12 20 × 5
x =
∴ Ratio of radius of cyclinder to radius 4
of cone = 25m/sec
862 ✫ Quantitative Aptitude

Time taken by train B to cross a pole x


= 12 seconds × 12 = 2 × 132
10
length of train = 25 × 12 12 x
= 300 m = 264
10
20. Ans. (B) 12x = 2640
3! × 5!
Sol. Probability = 2640
7! x =
12
3! × 5!
= 7 × 6 × 5! x = 220
25. Ans. (D)
3×2

TS
= x
7×6 Sol. × 180 =
1 100
=
60
7 ( 24 )2 + (18)2 + × 660

H
21. Ans. (D) 100
Sol. 2x = 32 ÷ 128 × 1024 + 8 18 x
576 + 324 + 396

G
=
32 100
= × 1024 ÷ 8 9x
128
=
1
4
× 1024 ÷ 8 U 5
9x
= 1296
O
= 36
5
= 256 ÷ 8 9x = 36 × 5
H
= 32 36 × 5
2x = 32 x =
9
2x = 25 = 4 × 5
_T

x = 5 = 20
22. Ans. (A) 26. Ans. (C)
340 Sol. Total no. of workers in A and C
Sol. = 144 + 64
C

? 32 24
340 = × 900 + × 900
100 100
PS

= 12 + 8 = 288 + 216
x
340 = 504
= 20 No. of officers in company A and C
x
U

20x = 340 together


x = 17 1 x 1 y
23. Ans. (E) = = + =
@

16 288 12 216
34
Sol. × 550 ÷ x = 297 + 729 = {16x = 288 + 12y = 216}
100
34 288 216
× 55 ÷ x = 297 – 27 x= +y=
10 16 12
34 55 {x = 18 + y = 18}
× = 11
10 x {18 + 18}
34 × 55 = 110x
34 × 55 = 36
x = Ratio of total no. of workers to total
110
x = 17 no. of officers
24. Ans. (A) 504 14
= =
20 36 1
Sol. (x ÷ 10) × 12 = × 1320 Ratio = 14 : 1
100
Solved Test Papers of Various Examinations ✫ 863

27. Ans. (E) No. of officers in company C = x


Sol. Total no. of workers in company C 1 x
= =
24 12 216
= × 900 = 216
100 12x = 216
No. of officers in company C 216
1 x x = = 18
= 12
12 216 Total no. of officers and workers in
12x = 216 company
x = 18 216 + 18 = 234

TS
∴ Total no. of employees in company C No. of workers in company D
= 216 + 18 = 234 25
Total no. of workers in company B 216 + × 216
100

H
44 1
= × 900 = 396 216 + × 216
100 4

G
No. of officers in company B 216 + 54 =270
1 x No. of officers in company D
= = 50
18
18x = 396
396
U = 18 +
100
= 18 + 9 = 27
× 18
O
x = 22 = 36000
Total no. of employees in company Total profit share of B
H
B =396 + 22 = 418 =36000 + 10000 = 46000 ... (i)
Difference between employees in Profit share obtained by C from
_T

company B and company C scheme S1


= 418 – 234 = 184 250000
28. Ans. (A) = × 200000
500000
Sol. No. of workers in company ‘A’
C

25
= × 200000 = 100000
32 50
× 900 = 288
PS

100 Ratio = 46000 : 100000


23 : 50
No. of officers in company A = x 30. Ans. (D)
1 x Sol. Total no. of workers in A
i.e. =
U

16 288
16x = 288 32
= × 900 = 288 ... (i)
100
@

288
x = = 18 Total no. of workers in B and C
16
together
No. of workers in company B
44 24 44
= × 900 = 396 = × 900 + × 900
100 100 100
Total no. of employees in company = 216 + 396 = 612 ... (ii)
D Difference = (ii) — (i)
= officers + workers = 612 – 288 = 324
= 27 + 270 = 297 31. Ans. (D)
29. Ans. (B) Sol. Total amount invested by A, B and
Sol. No. of workers in company C C in scheme S1
24 = 80000 × 2 + 30000 × 3
= × 900 = 216
100 + 50000 × 5
864 ✫ Quantitative Aptitude

= 160000 + 90000 + 250000 Total amount invested by A


= 500000 = 110000 + 64000 + 72000
Profit from scheme S1 = 200000 = 246000
2
Profit obtained by B from scheme S1 20
A = 246000 1 +
90000 9 100
= × 200000 = × 200000 2
500000 50 120
No. of officers in company B = x = 246000
100
1 x
i.e. = 6 6
18 396 = 346000 × ×

TS
5 5
18x = 396 = 354240
x = 22 Required interest = 354240 – 246000
∴ Difference between officers in B and C = ` 108240
= 22 – 18 = 4

H
34. Ans. (A)
32. Ans. (E)
Sol. Profit obtained by A from scheme S1 Sol. Profit attained by A from scheme S1

G
= ` 64000
160000
= × 200000 Profit attained by C from scheme S2
500000
=
16
50
× 200000 = 64000
U = ` 18000
Total profit = 64000 + 18000 = 82000
O
Total amount invested in scheme S2 82000
= 30000 × 4 + 10000 × 3 Average profit =
2
H
= 120000 + 30000 = 150000 = 41000
Total profit in scheme S2 = 90000
35. Ans. (C)
Profit of C
_T

30000 80000 × 3 × R
= × 90000 Sol. S.I. =
150000 100
1
C

= × 90000 = 18000 = 800 × 3 × R


5 = 2400R
PS

64000 1
Required % = × 100 R+5
18000 A = 30000 1 +
100
6400 5
= = 355 100 + R + 5
18 9
U

= 30000
33. Ans. (A) 100
Sol. Investment of A in scheme A and = 300(100 + R + 5)
@

scheme B C.I. = 300(100 + R + 5) – 30000


= 80000 + 30000 = 30000 + 300R + 1500 – 30000
= 110000 = 300R + 1500
Profit earned by A in scheme S.I.-C.I. = 30000
A = 64000
∴ 2400R – (300R + 1500) = 30000
Profit earned by A in scheme
B = 72000 2400R – 300R – 1500 = 30000
120000 2100R = 30000 + 1500
i.e. × 90000
150000 2100R = 31500
12 21R = 315
= × 90000
15 315
R = = 15%
= 72000] 21

You might also like